Вы находитесь на странице: 1из 772

EBD_7327

(ii)
• Corporate Office : 45, 2nd Floor, Maharishi Dayanand Marg, Corner Market,
Malviya Nagar, New Delhi-110017
Tel. : 011-49842349 / 49842350

By
Preetima Bajpai
Shristy Khandelwal

Typeset by Disha DTP Team

DISHA PUBLICATION
All Rights Reserved

© Copyright Publisher
No part of this publication may be reproduced in any form without prior permission of the publisher. The author and
the publisher do not take any legal responsibility for any errors or misrepresentations that might have crept in. We have
tried and made our best efforts to provide accurate up-to-date information in this book.

For further information about the books from DISHA PUBLICATION,


Log on to www.dishapublication.com or www.aiets.co.in or email to info@aiets.co.in
(iii)

Contents
NEET Solved Paper 2018 2018-1-2018-8

1. Some Basic Concepts of Chemistry 1-22

2. Structure of Atom 23-47

3. Classification of Elements and Periodicity in Properties 48-63

4. Chemical Bonding and Molecular Structure 64-100

5. States of Matter 101-123

6. Thermodynamics 124-149

7. Equilibrium 150-186

8. Redox Reactions 187-203

9. Hydrogen 204-218

10. The s-Block Elements 219-239

11. The p-Block Elements (Group 13 and 14) 240-260

12. Organic Chemistry : Some Basic Principles and Techniques 261-300

13. Hydrocarbons 301-337

14. Environmental Chemistry 338-352

15. The Solid State 353-373

16. Solutions 374-401

17. Electrochemistry 402-430

18. Chemical Kinetics 431-463


EBD_7327
(iv)

19. Surface Chemistry 464-482

20. General Principles and Processes of Isolation of Elements 483-495

21. The p-Block Elements (Group 15, 16, 17 and 18) 496-525

22. The d and f-Block Elements 526-547

23. Coordination Compounds 548-576

24. Haloalkanes and Haloarenes 577-603

25. Alcohols, Phenols and Ethers 604-634

26. Aldehydes, Ketones and Carboxylic Acids 635-672

27. Amines 673-707

28. Biomolecules 708-729

29. Polymers 730-748

30. Chemistry in Everyday Life 749-760


NEET Solved Paper 2018

1. The correct order of N-compounds in its decreasing order 9. A mixture of 2.3 g formic acid and 4.5 g oxalic acid is treated
of oxidation states is with conc. H2SO4. The evolved gaseous mixture is passed
(1) HNO3, NO, N2, NH4Cl through KOH pellets. Weight (in g) of the remaining product
(2) HNO3, NO, NH4Cl, N2 at STP will be
(3) NH4Cl, N2, NO, HNO3 (1) 1.4 (2) 3.0
(4) HNO3, NH4Cl, NO, N2 (3) 4.4 (4) 2.8
2. The correct order of atomic radii in group 13 elements is 10. Which of the following oxides is most acidic in nature?
(1) B < Al < In < Ga < Tl (1) MgO (2) BeO
(2) B < Al < Ga < In < Tl (3) CaO (4) BaO
(3) B < Ga < Al < In < Tl 11. Nitration of aniline in strong acidic medium also gives
(4) B < Ga < Al < Tl < In
m-nitroaniline because
3. Considering Ellingham diagram, which of the following
(1) Inspite of substituents nitro group always goes to only
metals can be used to reduce alumina?
m-position.
(1) Fe (2) Zn
(3) Cu (4) Mg (2) In electrophilic substitution reactions, amino group is
4. Which one of the following elements is unable to form meta directive.
MF63– ion? (3) In acidic (strong) medium aniline is present as anilinium
(1) Ga (2) Al ion.
(3) In (4) B (4) In absence of substituents, nitro group always goes
5. Which of the following statements is not true for halogens? to m-position.
(1) All form monobasic oxyacids 12. The compound A on treatment with Na gives B, and with
(2) All are oxidizing agents PCl5 gives C. B and C react together to give diethyl ether. A,
(3) Chlorine has the highest electron-gain enthalpy B and C are in the order
(4) All but fluorine shows positive oxidation states (1) C2H5OH, C2H6, C2H5Cl
6. In the structure of ClF3, the number of lone pair of electrons
(2) C2H5OH, C2H5Cl, C2H5ONa
on central atom ‘Cl’ is
(3) C2H5OH, C2H5ONa, C2H5Cl
(1) One (2) Two
(4) C2H5Cl, C2H6, C2H5OH
(3) Three (4) Four
7. The difference between amylose and amylopectin is 13. Hydrocarbon (A) reacts with bromine by substitution to
(1) Amylopectin has 1 ® 4 a-linkage and 1 ® 6 form an alkyl bromide which by Wurtz reaction is converted
a-linkage to gaseous hydrocarbon containing less than four carbon
(2) Amylose has 1 ® 4 a-linkage and 1® 6 b-linkage atoms. (A) is
(3) Amylose is made up of glucose and galactose (1) CH º CH (2) CH2 = CH2
(4) Amylopectin has 1 ® 4 a-linkage and 1® 6 (3) CH4 (4) CH3 – CH3
b-linkage 14. The compound C7H8 undergoes the following reactions:
8. Regarding cross-linked or network polymers, which of the
3Cl / D Br /Fe Zn /HCl
following statements is incorrect? C7 H8 ¾¾¾¾
2 ® A ¾¾¾¾
2 ® B ¾¾¾¾® C
(1) They contain covalent bonds between various linear
The product 'C' is
polymer chains.
(1) m-bromotoluene
(2) They are formed from bi- and tri-functional monomers.
(2) o-bromotoluene
(3) They contain strong covalents bonds in their polymer
chains. (3) p-bromotoluene
(4) Examples are bakelite and melamine. (4) 3-bromo-2,4,6-trichlorotoluene
EBD_7327
2018-2 CHEMISTRY

15. Which oxide of nitrogen is not a common pollutant (1)Formation of intramolecular H-bonding
introduced into the atmosphere both due to natural and (2)Formation of carboxylate ion
human activity? (3)Formation of intermolecular H-bonding
(1) N2O5 (2) NO2 (4)More extensive association of carboxylic acid via van
der Waals force of attraction
(3) NO (4) N2O
21. Compound A, C 8H10O, is found to react with NaOI
16. Which of the following molecules represents the order of
(produced by reacting Y with NaOH) and yields a yellow
hybridisation sp2, sp2, sp, sp from left to right atoms?
precipitate with characteristic smell.
(1) HC º C – C º CH A and Y are respectively
(2) CH2 = CH – C º CH
(3) CH3 – CH = CH – CH3 (1) H3C CH2 – OH and I2
(4) CH2 = CH – CH = CH2
17. Which of the following carbocations is expected to be most
(2) CH2 – CH2 – OH and I2
stable?
CH3
NO2 NO2
Å
(3) CH3 OH and I2
(1) Å
(2)
Y H Y H (4) CH – CH3 and I2

OH
NO2 NO2
22. Identify the major products P, Q and R in the following
H Å
(3) (4) H sequence of reactions:
Y
Y Å Anhydrous
AlCl3
+ CH3CH2CH2Cl ¾¾¾¾®
18. Which of the following is correct with respect to – I effect
(i) O
of the substituents? (R = alkyl) P ¾¾¾¾®
2
+ Q+R
(ii) H3O /D
(1) – NH2 < – OR < – F
(2) – NR2 < – OR < – F P Q R
(3) – NR2 > – OR > – F
CH2CH2CH3 CHO
(4) – NH2 > – OR > – F
19. In the reaction (1) , , CH3CH2 – OH

OH O– Na+

CHO CH2CH2CH3 CHO COOH


+ CHCl3+ NaOH ¾®
(2) , ,
The electrophile involved is

(
(1) Dichloromethyl cation C HCl2
Å
) OH
CH(CH3)2
(Å )
(2) Formyl cation C HO (3) , , CH3 – CO – CH3

(3) Dichlorocarbene (: CCl 2 )


CH(CH3)2 OH
(
(4) Dichloromethyl anion CHCl 2
!
) (4) , , CH3CH(OH)CH3

20. Carboxylic acids have higher boiling points than aldehydes,


23. Which of the following compounds can form a zwitterion?
ketones and even alcohols of comparable molecular mass.
(1) Aniline (2) Acetanilide
It is due to their
(3) Glycine (4) Benzoic acid
NEET Solved Paper 2018 2018-3

24. For the redox reaction 31. Consider the following species :
CN+, CN–, NO and CN
MnO 4– + C 2 O 42– + H + ¾¾
® Mn 2 + + CO 2 + H 2 O
Which one of these will have the highest bond order?
The correct coefficients of the reactants for the balanced (1) NO (2) CN–
equation are (3) CN (4) CN+
MnO4– C2 O2– H+ 32. Which one is a wrong statement?
4
(1) Total orbital angular momentum of electron in 's' orbital
(1) 16 5 2
is equal to zero
(2) 2 5 16 (2) An orbital is designated by three quantum numbers
(3) 5 16 2 while an electron in an atom is designated by four
(4) 2 16 5 quantum numbers
25. Which one of the following conditions will favour maximum (3) The value of m for dz2 is zero
formation of the product in the reaction, (4) The electronic configuration of N atom is
ˆˆ† X 2 (g) D r H = –X kJ :
A 2 (g) + B2 (g) ‡ˆˆ
1s2 2s2 2p1x 2p1y 2p1z
(1) Low temperature and high pressure
(2) Low temperature and low pressure
(3) High temperature and low pressure
33. The correct difference between first and second order
(4) High temperature and high pressure reactions is that
26. When initial concentration of the reactant is doubled, the (1) The rate of a first-order reaction does not depend on
half-life period of a zero order reaction reactant concentrations, the rate of a second-order
(1) is halved reaction does depend on reactant concentrations
(2) is doubled (2) The half-life of a first-order reaction does not depend
(3) remains unchanged on [A]0, the half-life of a second-order reaction does
(4) is tripled depend on [A]0
27. The correction factor ‘a’ to the ideal gas equation corresponds (3) The rate of a first-order reaction does depend on
to reactant concentrations, the rate of a second-order
(1) Density of the gas molecules reaction does not depend on reactant concentrations
(2) Volume of the gas molecules (4) A first-order reaction can be catalyzed, a second-order
(3) Forces of attraction between the gas molecules reaction cannot be catalyzed
(4) Electric field present between the gas molecules 34. In which case is number of molecules of water maximum?
28. The bond dissociation energies of X2, Y2 and XY are in the (1) 18 mL of water
ratio of 1 : 0.5 : 1. DH for the formation of XY is –200 kJ mol–1. (2) 0.18 g of water
The bond dissociation energy of X2 will be (3) 10–3 mol of water
(1) 200 kJ mol–1 (2) 100 kJ mol–1 (4) 0.00224 L of water vapours at 1 atm and 273 K
(3) 400 kJ mol –1 (4) 800 kJ mol–1 35. Among CaH2, BeH2, BaH2, the order of ionic character is
29. Magnesium reacts with an element (X) to form an ionic (1) BeH2 < CaH2 < BaH2
compound. If the ground state electronic configuration of (2) CaH2 < BeH2 < BaH2
(X) is 1s2 2s2 2p3, the simplest formula for this compound is (3) BaH2 < BeH2 < CaH2
(1) Mg2X3 (2) MgX2
(4) BeH2 < BaH2 < CaH2
(3) Mg3X2 (4) Mg2X
36. Consider the change in oxidation state of bromine
30. Iron exhibits bcc structure at room temperature. Above 900°C,
corresponding to different emf values as shown in the
it transforms to fcc structure. The ratio of density of iron at
diagram below :
room temperature to that at 900°C (assuming molar mass
1.82 V 1.5 V
and atomic radii of iron remains constant with temperature) BrO4– ¾¾¾® BrO3– ¾¾¾® HBrO
¾¾

is
Br –¬¾¾¾ Br2 ¬¾¾¾
3 4 3 1.0652 V 1.595 V
(1) (2)
2 3 2 Then the species undergoing disproportionation is
(1) BrO3– (2) BrO 4–
1 3 3
(3) (4) (3) HBrO (4) Br2
2 4 2
EBD_7327
2018-4 CHEMISTRY

37. The solubility of BaSO4 in water is 2.42 × 10–3 gL–1 at 41. Iron carbonyl, Fe(CO)5 is
298 K. The value of its solubility product (Ksp) will be (1) Tetranuclear (2) Mononuclear
(Given molar mass of BaSO4 = 233 g mol–1) (3) Dinuclear (4) Trinuclear
(1) 1.08 × 10–10 mol2L–2 42. The type of isomerism shown by the complex [CoCl2(en)2]
(2) 1.08 × 10–12 mol2L–2 is
(3) 1.08 × 10–8 mol2L–2 (1) Geometrical isomerism
(2) Coordination isomerism
(4) 1.08 × 10–14 mol2L–2
(3) Linkage isomerism
38. Following solutions were prepared by mixing different (4) Ionization isomerism
volumes of NaOH and HCl of different concentrations : 43. Which one of the following ions exhibits d-d transition and
M M paramagnetism as well?
a. 60 mL HCl + 40 mL NaOH (2) Cr2 O72–
10 10 (1) CrO2–
4

(3) MnO 2–
4 (4) MnO4
M M
b. 55 mL HCl + 45 mL NaOH 44. The geometry and magnetic behaviour of the complex
10 10
[Ni(CO)4] are
M M (1) Square planar geometry and diamagnetic
c. 75 mL HCl + 25 mL NaOH (2) Tetrahedral geometry and diamagnetic
5 5
(3) Tetrahedral geometry and paramagnetic
M M (4) Square planar geometry and paramagnetic
d. 100 mL HCl + 100 mL NaOH 45. Match the metal ions given in Column I with the spin
10 10
magnetic moments of the ions given in Column II and assign
pH of which one of them will be equal to 1? the correct code :
(1) b (2) a Column I Column II
(3) c (4) d a. Co3+ i. 8 BM
39. On which of the following properties does the coagulating
b. Cr3+ ii. 35 BM
power of an ion depend?
(1) The magnitude of the charge on the ion alone c. Fe3+ iii. 3 BM
(2) Size of the ion alone d. Ni2+ iv. 24 BM
(3) The sign of charge on the ion alone
v. 15 BM
(4) Both magnitude and sign of the charge on the ion
a b c d
40. Given van der Waals constants for NH3, H2, O2 and CO2 are
(1) iv v ii i
respectively 4.17, 0.244, 1.36 and 3.59, which one of the (2) i ii iii iv
following gases is most easily liquefied? (3) iii v i ii
(1) NH3 (2) H2 (4) iv i ii iii
(3) CO2 (4) O2
NEET Solved Paper 2018 2018-5

Hints & Solutions


+5 +2 0 -3
CO2 is absorbed by KOH.
1. (1) HNO3 , NO, N2 , NH4Cl So the remaning product is only CO.
Moles of CO formed from both reactions
2. (3) 1 1 1
= + =
3. (4) Mg has more – DG value than alumina. So it will be in 20 20 10
the lower part of Ellingham diagram. Metals which
have more – DG value can reduce those metal oxides Left mass of CO = moles × molar mass
which have less – DG value. 1
4. (4) MF63- =
´ 28 = 2.8 g
10
Boron belongs to 2nd period and it does not have
10. (2) In metals moving down the group metallic character
vacant d-orbital. increases, so basic nature increases hence most acidic
5. (4) Due to high electronegativity and small size, F forms will be BeO.
only one oxoacid, HOF known as fluoric (I) acid.
BeO < MgO < CaO < BaO
Oxidation number of F is +1 in HOF. ¾¾¾¾¾¾¾¾¾¾
®
increasingbasic character
6. (2) The structure of ClF3 is
+ +
NH2 NH3 NH3
F
+
+ NO2
11. (3) ¾¾H ¾¾® ¾¾ ¾®
F Cl nitrating
mixture NO2
F
In acidic medium aniline is protonated to form anilinium
ion which is meta-directing.
The number of lone pair of electrons on central Cl
is 2.
Na –
7. (1) Amylose and amylopectin ar e polymers of 12. (3) C2H5OH ¾¾® C2H5O Na+
(A) (B)
a-D-glucose, so b-link is not possible. Amylose is linear
with 1 ® 4 a-linkage whereas amylopectin is branched PCl5
and has both 1 ® 4 and 1 ® 6 a-linkages.
So option (1) should be the correct option. C2H5Cl
8. (3) Cross-linked or network polymers are usually formed (C)
– S 2
from bi-functional and tri-functional monomers and C2H5O Na+ + C2H5Cl ¾¾®
N
C2H5OC2H5
(B) (C)
contain strong covalent bonds between various linear
polymer chains like melamine, bakelite etc.
H SO
So the correct option is (3)
HCOOH ¾¾¾¾®
2 4
9. (4) Dehydrating CO + H2O Br Na
agent [H 2O absorbed 13. (3) CH 4 ¾¾¾
2 ® CH – Br ¾¾¾
3 ® CH3 – CH3
hv ether
by H 2SO 4] (less than four 'C')
2.3 1
At start (moles) = = 0 0 14. (1)
46 20
1 1 CH3 CCl3 CCl3 CH3
Final moles 0
20 20
3Cl Br Zn
¾¾
D ®
2
¾ Fe
¾2 ® ¾HCl
¾ ®
H2SO4 Br Br
H2C2O4 ¾¾¾® CO + CO2 + H2O
m-Bromotoluene
[H 2O absorbed
by H 2SO 4] 15. (1) Nitrous oxide (N2O) occurs naturally in environment.
4.5 1
At start (moles) = = 0 0 0 In automobile engine, when fuel is burnt dinitrogen
90 20
1 1 1 and dioxygen combine to yield NO and NO2.
Final moles 0
20 20 20
EBD_7327
2018-6 CHEMISTRY

sp2 sp2 sp sp CH3


16. (2) CH2 = CH – C CH CH3 – CH – CH3 CH3–C– O– O–H
17. (4) –NO2 group is meta-directing group
O +

NO2 NO2 ¾¾®


D
2 H3O
D
+ Cumene Cumene
¬¾® (P) hydroperoxide
+ OH
Y H Y H O
(Less stable due to more e– withdrawing effect of –NO 2) + CH3 – C– CH3

NO2 NO2 NO2 Phenol Acetone


(Q) (R)
+ + +
¬¾® ¬¾® 23. (3) ˆˆ† OOC – CH 2 – NH 3
HOOC – CH 2 – NH 2 ‡ˆˆ
H H H Glycine Zwitter ion
Y + Y Y
(More stable due to less e– withdrawing effect of –NO2) greater +7 +2
no. of resonating structures. 24. (2) ® Mn2+; 5e– gain
Mn O 4– ¾¾ ...(i)
18. (1) – I effect increases on increasing electronegativity of +3 +4
atom. So, correct order of – I effect is ® CO2 ; 2e - loss
C2O 42– ¾¾ ...(ii)
–NH2 < – OR < – F. Multiplying (i) by 2 and (ii) by 5 to balance e–
*Most appropriate answer is option (1), however
option (2) may also be correct answer. 2 MnO 4– + 5 C 2 O 2– ® 2 Mn 2 + + 10 CO 2
4 ¾¾

– On balancing charge;
19. (3) CHCl3 + NaOH ¾® CCl3 + H2O
+
2 MnO 4– + 5 C2 O 2–
¾®

– Cl (a-elimination) 4 + 16 H ¾¾
®
: CCl2 dichlorocarbene 2 Mn 2 + + 10 CO2 + 8 H 2O
(electrophile)

20. (3) Carboxylic acids have higher boiling points than 25. (1) ˆˆ† X (g); DH = - x kJ
A2 (g) + B2(g) ‡ˆˆ 2
aldehydes, ketones and even alcohols of comparable
On increasing pressure equilibrium shifts in a
molecular mass.
direction where number of moles decreases i.e. forward
This is due to more extensive association through
direction.
intermolecular H-bonding.
On decreasing temperature, equilibrium shifts in
O ¼¼¼H O exothermic direction i.e., forward direction.
R C C R So, high pressure and low temperature favours
O H ¼¼¼O maximum formation of product.

21. (4) Haloform reaction is shown by compound having [A]o


26. (2) (t1/2 )zero =
CH3 – C – or CH3 – CH – 2k
|| | Group \ If [A]o = doubled, t1/2 = doubled
O OH
27. (3) In real gas equation,
– +
CH –
NaOI
CH 3 ¾¾¾¾® C – ONa + CHI3 van der waal constant (a) µ forces of attraction.
or
| NaOH + I2 || Yellow 28. (4) Let B.E of x2, y2 and xy are x kJ mol–1,
OH O ppt.
0.5 x kJ mol–1 and x kJ mol–1 respectively
22. (3) Mechanism :
1 1
x 2 + y 2 ® xy; DH = –200 kJ mol –1
+ – 2 2
AlCl
CH3 – CH 2 – CH 2 – Cl ¾¾¾®
3 CH3 – CH 2 – CH 2 + AlCl4
1° Carbocation
DH = –200 = S (B.E)Reactants – S(B.E)Product

CH3 – CH – CH3 é1 1 ù
+
= ê ´ (x) + ´ (0.5x)ú – [1 ´ (x)]
ESR H
– ë2 2 û
¬¾¾¾ CH3 – CH – CH3 ¬¾¾¾
shift
2° Carbocation On solving, x = 800 kJ mol–1
(P)
NEET Solved Paper 2018 2018-7

29. (3) Element (X) electronic configuration 34. (1)


1s2 2s2 2p3 (1) Mass of water = 18 × 1 = 18 g
So, valency of X will be 3.
18
Valency of Mg is 2. Molecules of water = mole × NA = N = NA
18 A
Formula of compound formed by Mg and X will be
Mg3X2. 0.18
(2) Molecules of water = mole × NA = N
4r 18 A
30. (4) For bcc lattice : Z = 2, a =
3 = 10–2 NA
(3) Molecules of water = mole × NA = 10–3 NA
For fcc lattice : Z = 4, a = 2 2r
0.00224
(4) Moles of water = = 10–4
22.4
æ ZM ö Molecules of water = mole × NA = 10–4 NA
çç 3 ÷÷
d25 °C è NA a øbcc 35. (1) BeH2 < CaH2 < BaH2
\ =
d900 °C æ ZM ö Smaller the size of cation, more will be its polarising
çç 3 ÷÷ power. Hence BeH2 will be least ionic.
è NA a øfcc
36. (3) Calculate E°cell corresponding to each compound
undergoing disproportionation reaction. The reaction
3 for which E°cell comes out + ve is spontaneous.
2 æ 2 2r ö 3 3 HBrO ¾® Br2 E° = 1.595 V, SRP (cathode)
= =
4 ç 4r ÷ 4 2
ç ÷ HBrO ¾® BrO 3– E° = –1.5 V, SOP (anode)
è 3 ø
2HBrO ¾® Br2 + BrO 3–
31. (2) NO : (s1s)2, (s*1s)2, (s2s)2,(s*2s)2,(s2pz)2, E°cell = SRP (cathode) – SRP (anode)
(p2px)2 = (p2py)2,(p*2px)1 = (p*2py)0 = 1.595 – 1.5
= 0.095 V
10 - 5
B.O. = = 2.5 E°cell > 0 Þ DG° < 0 [spontaneous]
2
CN– : (s1s)2, (s*1s)2, (s2s)2,(s*2s)2, 37. (1) Solubility of BaSO4 = 2.42 × 10–3 gL–1
(p2px)2 = (p2py)2, (s2pz)2 2.42 ´ 10-3
\s= = 1.038 ´ 10 -5 mol L-1
10 - 4 233
B.O. = =3
2 Ksp = s2 = (1.038 × 10–5)2
CN : (s1s)2, (s*1s)2, (s2s)2,(s*2s)2, = 1.08 × 10–10 mol2 L–2
(p2px)2 = (p2py)2,(s2pz)1 1
38. (3) Meq. of HCl = 75 ´ ´ 1 = 15
9-4 5
B.O. = = 2.5
2 1
Meq. of NaOH = 25 ´ ´ 1= 5
CN+ : (s1s)2, (s*1s)2, (s2s)2,(s*2s)2, 5
(p2px)2 = (p2py)2 Meq. of HCl in resulting solution = 10
Molarity of [H+] in resulting mixture
8-4
B.O. = =2
2 10 1
= =
Hence, option (2) should be the right answer. 100 10
32. (4) The correct configuration of 'N' is é1ù
pH = –log[H+] = –log ê ú = 1.0
ë 10 û
1s2 2s2 2p1x 2p1y 2p1z
39. (4) According to Hardy Schulze rule, coagulating power
of an ion depends on both magnitude and sign of the
charge on the ion.
33. (2) (t1/2)1st order = Independent of concentration 40. (1) van der waal constant ‘a’, signifies intermolecular
forces of attraction.
1
(t1/2)2nd order µ Higher is the value of ‘a’, easier will be the liquefaction of
[A]o gas.
EBD_7327
2018-8 CHEMISTRY

41. (2) Fe(CO)5 44. (2) Ni(28) : [Ar]3d8 4s2


EAN = Z – O.N. + 2(C.N.) Q CO is a strong field ligand, so unpaired electrons get
= 26 – 0 + 2(5) paired. Hence, configuration would be:
= 26 + 10
sp3–hybridisation
= 36
Only one central metal atom/ion is present and it
follows EAN rule, so it is mononuclear.
42. (1) In the given complex, the CN of Co is 6, and the
complex has octahedral geometry. CO CO CO CO

Cl en For, four ‘CO’ligands hybridisation would be sp3 and


thus the complex would be diamagnetic and of
en Co en Co en tetrahedral geometry.
Cl 45. (1) Co3+ = [Ar] 3d 6 , unpaired e–(n) = 4
Cl Cl Spin magnetic moment = 4(4 + 2) = 24 B.M.
trans (optically inactive) cis (optically active)
Cr3+ = [Ar] 3d 3 , unpaired e–(n) = 3
43. (3) CrO2–
4 Cr6+ diamagnetic Spin magnetic moment = 3(3 + 2) = 15 B.M.
Cr2O72– Cr6+ diamagnetic Fe3+ = [Ar] 3d 5 , unpaired e–(n) = 5
MnO4– Mn7+ diamagnetic Spin magnetic moment = 5(5 + 2) = 35 B.M.
MnO 2–
4 Mn6+ paramagnetic Ni2+ = [Ar] 3d 8 , unpaired e–(n) = 2
Thus unpaired electron is present, so d–d transition is
Spin magnetic moment = 2(2 + 2) = 8 B.M.
possible.

Before After
transition transition
Some Basic Concepts
1 of Chemistry
NATURE OF MATTER
Anything which has mass and occupies space is called matter.
Matter can exist in three physical states: solid, liquid and gas. These three states are interconvertible by changing the conditions of
temperature and pressure.
heat heat
Solid Liquid Gas
cool cool
At macroscopic level, matter is classified as

Matter

Mixtures Pure substances


They contains two or more substances in any ratio. They contains two or more substances in a fixed ratio.
Their components can be separated by Their the components cannot be separated by
simple physical methods. They are further physical methods. They are further classified as
classified as

Homogeneous Heterogeneous Elements Compounds


Their components mix Components of the It consists of only It is formed by the
completely and they mixture remain separate. The one type of particles, combination of
have uniform composition composition is not uniform throughout. which may be atoms 2 or more elements
Example : mixture of salt and sugar or molecules in a fixed ratio.
throughout.
Example : sugar solution,
air, etc.
such as speed, volume, density, etc. can be derived from these
PROPERTIES OF MATTER AND THEIR
units. These base units are listed as follows:
MEASUREMENT
Physical Symbol SI unit Symbol
Every substance has characteristic properties which can be
Quantity
classified as physical properties and chemical properties.
Physical properties are those which can be measured or observed Length l metre m
without changing the identity or composition of the substance. Mass m kilogram kg
Ex: colour, odour, m.pt, b.pt, etc. Time t second s
Chemical properties are those which require a chemical change for Electric current I ampere A
their measurement.
Many properties of matter are quantitative in nature which can be Temperature T kelvin K
measured under the following system of units. Amount of
substance n mole mol
The International System of Units (SI)
The SI system has seven base units which pertain to seven Luminous
fundamental scientific quantities. The other physical quantities Iv candela cd
intensity
EBD_7327
2 CHEMISTRY

UNCERTAINTY IN MEASUREMENT AND (iv) Gay Lussac’s law of Gaseous Volumes


SIGNIFICANT FIGURES According to this law, when gases combine or are produced
Precision and Accuracy in a chemical reaction, they do so in a simple ratio by volume
Very large or very small numbers, having many zeros can be provided all gases are at same temperature and pressure.
expressed by using scientific notation for such numbers i.e.,
exponential notation in which any number can be represented in (v) Avogadro Law
the form N × 10n where n is an exponent having +ve or –ve value It states that equal volumes of gases at same temperature
and N can vary between 1 to 10. and pressure should contain equal number of molecules.
Every experimental measurement has some amount of uncertainty
DALTON’S ATOMIC THEORY
associated with it. However, one would always like the results to
be precise and accurate. Precision refers to the closeness of various In 1808, Dalton published ‘A new system of chemical philosophy’
measurements for the same quantity while accuracy is the in which he proposed the following :
agreement of particular value to the true value of the result. (i) Matter consists of indivisible atoms.
Significant Figures (ii) All the atoms of a given element have identical properties
The uncertainty in experimental or calculated values is indicated including identical mass. Atoms of different elements differ
by mentioning the number of significant figures. Significant figures in mass.
are meaningful digits which are known with certainty. The (iii) Compounds are formed when atoms of different elements
uncertainty is indicated by writing the certain digits and the last combine in a fixed ratio.
imcertain digit. (iv) Atoms are neither created nor destroyed in a chemical
The rules for determining the number of significant figures are: reaction. Dalton’s theory could explain the laws of chemical
(i) All non-zero digits are significant. For ex: in 285 cm, there are
combination.
3 significant figures.
(ii) Zeros preceding to first non-zero digit are not significant. ATOMIC AND MOLECULAR WEIGHT
Such zero indicates the position of decimal point. For ex: 0.03
has one significant figure.
Atomic Mass :
(iii) Zeros between two non-zero digits are significant. For ex: Atomic mass is the number of times an atom of an element is
2.005 has four significant figures. heavier than 1/12 th of an atom of C–12.
(iv) Zeros at the end or right are significant provided they are on
the right side of the decimal point. For ex: 0.200 g has 3 Weight of 1 atom of element
Atomic weight of an element =
significant figure. 1/12 × weight of 1 atom of C-12
(v) If a number ends in zeros that are not to right of a decimal the Determination of atomic weight :
zeros may or may not be significant. For e.g., 3500 may have
two, three or five significant figures. Atomic weight × specific heat = 6.4 (app.)
(vi) Counting no. of objects have infinite significant figures. Molecular weight : It is the number of times a molecule of
(vii) In numbers written in scientific notation, all digits are any compound is heavier than 1/12 th of an atom of C–12
significant. Weight of one molecule
LAWS OF CHEMICAL COMBINATION Molecular weight =
1/12 × weight of one C-12 atom
The combination of elements to form compounds is governed by Determination of molecular weight :
following five basic laws : (i) Vapour density method :
(i) Laws of Conservation of Mass
It states that matter can neither be created nor destroyed, Wt. of a certain vol. of a gas or vapour
under certain temperature and pressure
(ii) Law of Definite Proportion/Composition Vapour density =
Wt. of the same volume of H 2 under
It states that a given compound always contains exactly the same temperature and pressure
same proportion of elements by weight.
(iii) Law of Multiple Proportions Molecular weight = 2 × vapour density
It states that if two elements can combine to form more than (ii) Diffusion method :
one compound, the masses of one element that combine (a) It is based on Graham's law of diffusion.
with a fixed mass of the other element, are in the ratio of small (b) Graham's law states that : The rate of diffusion of
whole numbers. different gases, under similar conditions of temperature
For ex : and pressure are inversely proportional to the square
1 roots of their density (or molecular weights).
H2 + O 2 ¾¾
® H 2O H 2 + O 2 ¾¾
® H2 O2
2g 2 18g 2g 32g 34g
16g r1 d M2
Here, masses of oxygen (i.e., 16 g and 32 g) which combine = 2 =
r2 d1 M1
with a fixed mass of H (2g) bear a simple ratio, 16 : 32 i.e. 1:2.
Some Basic Concepts of Chemistry 3

MOLE CONCEPT
Mole : Mole is a unit which represents 6.023 × 1023 particles of same nature.

22.4 L at NTP/STP One gram mole of a compound


Volume (L) Weight of compound (gm)
Mole = 1 Mole Mole =
22.4 L GMM
23
NA = 6.023 × 10 Particles One gram atom of an element
No. of particles Weight of element
Mole = Mole =
NA GAM

1 Mole = 6.023 × 1023 particles. For ex: Percentage composition of water is:
1 Mole of atoms = 6.023 × 1023 Atoms. Molar mass of water = 18.02 g
1 Mole of molecules = 6.023 × 1023 molecules 2 ´ 1.008
Mass % of H = ´ 100 = 11.18%
1 Mole of electrons = 6.023 × 1023 electrons. 18.02
The number 6.023 × 1023 is called Avogadro number (NA)
16.00
EQUIVALENT WEIGHT Mass % of O = ´ 100 = 88.79%
18.02
Equivalent weight of a substance (element or compound) is defined
Chemical Formulae
as "The number of parts by weight of it, that will combine with or
It is of two types :
displace directly or indirectly 1.008 parts by weight of hydrogen,
8 parts by weight of oxygen, 35.5 parts by weight of chlorine or (i) Molecular formulae : Chemical formulae that indicate the
the equivalent parts by weight of another element". actual number and type of atoms in a molecule are called
molecular formulae.
Molecular mass Example : Molecular formula of benzene is C6H6.
Eq. wt of elements =
Basicity of acid (ii) Empirical formulae : Chemical formulae that indicate only
the relative number of atoms of each type in a molecule are
Molecular mass
Eq. wt of an acid = called empirical formulae.
Basicity of acid Example : Empirical formula of benzene is "CH".
Molecular mass Determination of Chemical Formulae :
Eq. wt of a base =
Acidity of base (a) Determination of empirical formulae :
Step (I) : Determination of percentage of each element
Equivalent mass for salts
Step (II) : Determination of mole ratio
Formula mass Step (III) : Making it whole number ratio
=
(Valency of cation ) ( No. of cations) Step (IV) : Simplest whole ratio
Equivalent mass for oxidising agents
(b) Determination of molecular formulae :–
Formula mass Step (I) : First of all find empirical formulae
= Step (II) : Calculate the empirical weight
No. of electrons gained per molecule
Step (III) : Molecular formulae = n (Empirical formulae)
Equivalent mass for reducing agents
Molecular weight
n=
Formula mass Empirical weight
=
No. of electrons lost per molecule
STOICHIOMETRY
PERCENTAGE COMPOSITION AND CHEMICAL FOR- It deals with the calculation of masses of reactant and products
MULAE involved in a chemical reaction.
Percentage Composition For ex: the balanced equation for combustion of CH4 is:
The percentage composition of an element in a compound is given CH 4 ( g ) + 2O 2 ( g ) ¾¾
® CO2 ( g ) + 2H 2 O ( g )
by: The coefficients of 2 for O2 and H2O are called stoichiometric
Mass % of an element coefficients. The coefficient for CH4 and CO2 is one in each case.
According to the above chemical reaction,
mass of the element in compound
= ´100 (i) One mole of CH4 (g) reacts with 2 moles of O2(g) to give
molar mass of compound 1 mole of CO2(g) and 2 moles of H2O (g).
EBD_7327
4 CHEMISTRY

(ii) One molecule of CH4 (g) reacts with 2 molecules of O2(g) to (iii) Weight – volume percentage (w/V) :
give 1 molecule of CO2(g) and 2 molecules of H2O (g). Weight – volume percentage
(iii) 22.4 L of CH4(g) reacts with 44.8 L of O2(g) to give 22.4 L of
Weight of solute (gm)
CO2(g) and 44.8 L of H2O(g) = × 100
(iv) 16 g of CH4(g) reacts with 2 × 32 g of O2(g) to give 44 g of Volume of solution (ml)
CO2(g) and 2 × 18g of H2O(g) 2. Normality :
The given data can be interconverted as: The number of gram equivalents of the solute dissolved per
Mass Moles No. of molecules litre of the solution. It is denoted by 'N' :
The calculations based on the knowledge of chemical equations Number of gram equivalents of solute
are also called stoichiometry calculations. The following steps Normality =
are generally followed for carrying out such calculations : Volume of solution (lit.)
(i) Write the balanced chemical equation. Q Gram equivalents of solute
(ii) Write the molar relationship from the equation between the Weight of solute (gm)
=
given and the required species. Equivalent weight of solute
(iii) Convert these moles into the desired parameters such as 3. Mole Fraction :
mass, volume, etc.
(iv) Apply unitary method to calculate the result. If a substance A dissolves in substance B and their number
of moles are nA and nB, then their mole fractions (x) are given
Limiting Reagent by
The reactant which gets consumed and limits the amount of nA nB
product formed is called limiting reagent. The moles of product xA = and xB =
nA + nB nA + nB
are always determined by the starting moles of limiting reactant.
Also, xA + xB = 1
EXPRESSION OF STRENGTH/CONCENTRATION OF
SOLUTION 4. Molarity :
The concentration of the solution or the amount of substance It is defined as the number of moles of solute in 1 litre of
present in its given volume can be expressed in any of the following solution. Thus,
ways: No. of moles of solute
Molarity ( M ) =
1. Mass Percent or Weight Percent (w/W%) Volume of solution in litres

Mass of solute 5. Molality :


Mass percent = ´ 100 It is defined as the number of moles of solute present in 1 kg
Mass of solution
of solvent. Thus,
(i) Weight-weight percent (w/W) : No. of moles of solute
Molality ( m ) =
Weight of solute (gm) Mass of solvent in kg
Weight percent = ´ 100
Weight of solution (gm) 6. ppm. (Parts per million) :
(ii) Volume-volume percent (v/V) : The parts of the component per million parts (106) of the
solution.
Volume – volume percentage
w
Volume of solute (ml.) ppm = ´106
= × 100 w +W
Volume of solution (ml.) where, w = weight of solute, W = weight of solvent
Some Basic Concepts of Chemistry 5

CONCEPT MAP
EBD_7327
6 CHEMISTRY

1. The oxide of an element contains 67.67% oxygen and the (a) 1.2 × 10–20 g (b) 5.025 × 1023 g
vapour density of its volatile chloride is 79. Equivalent weight (c) 1.4 × 10 g–21 (d) 6.023 × 10–20 g
of the element is: 14. Among the following pairs of compounds, the one that
(a) 2.46 (b) 3.82 (c) 4.36 (d) 4.96 illustrates the law of multiple proportions is
2. The empirical formula of a compound is CH2O. Its molecular (a) NH3 and NCl3 (b) H2S and SO2
weight is 180. The molecular formula of compound is : (c) CS2 and FeSO4 (d) CuO and Cu2O
(a) C 4 HO4 (b) C 3 H 6 O 3 15. Irrespective of the source, pure sample, of water always yields
88.89% mass of oxygen and 11.11% mass of hydrogen. This is
(c) C 6 H12O 6 (d) C 5 H10 O 5
explained by the law of
3. 0.4 moles of HCl and 0.2 moles of CaCl 2 were dissolved in (a) conservation of mass (b) multiple proportions
water to have 500 mL of solution, the molarity of Cl – ion is: (c) constant composition (d) constant volume
(a) 0.8 M (b) 1.6 M (c) 1.2 M (d) 10.0 M 16. The volume of 20 volume H2O2 required to get 5 litres of O2 at
4. 1021 molecules are removed from 200 mg of CO2. The moles of STP is
CO2 left are : (a) 250 ml (b) 125 ml (c) 100 ml (d) 50 ml.
17. Given P = 0.0030m, Q = 2.40m, R = 3000m, Significant figures
(a) 2.88 ´ 10 - 3 (b) 28.8 ´10 -3
in P, Q and R are respectively
(c) 288 ´ 10 -3 (d) 28.8 ´ 10 3 (a) 2, 2, 1 (b) 2, 3, 4 (c) 4, 2, 1 (d) 4, 2, 3
5. The weight of NaCl decomposed by 4.9g of H2SO4, if 6 g of 18. The prefix zepto stands for (in m)
sodium hydrogen sulphate and 1.825 g of HCl, were produced (a) 109 (b) 10–12
in the reaction is: (c) 10–15 (d) 10–21
(a) 6.921 g (b) 4.65 g (c) 2.925 g (d) 1.4 g 19. Two samples of lead oxide were separately reduced to metallic
6. Which one of the following pairs of compounds illustrate the lead by heating in a current of hydrogen. The weight of lead
law of multiple proportions ? from one oxide was half the weight of lead obtained from the
(a) H2O and Na2O (b) MgO and Na2O other oxide. The data illustrates
(c) Na2O and BaO (d) SnCl2 and SnCl4 (a) law of reciprocal proportions
7. The molecular weight of O2 and SO2 are 32 and 64 respectively. (b) law of constant proportions
At 15°C and 150 mm Hg pressure, one litre of O2 contains ‘N’ (c) law of multiple proportions
molecules. The number of molecules in two litres of SO2 under (d) law of equivalent proportions
the same conditions of temperature and pressure will be : 20. Number of valency electrons in 4.2 gram of N3– ion is
(a) N/2 (b) 1N (c) 2N (d) 4N (a) 4.2 NA (b) 0.1 NA (c) 1.6 NA (d) 3.2 NA
8. In the final answer of the expression 21. 100 ml of solution of H2O2 on decomposition gives 1500 ml of
O2 at N.T.P. The H2O2 has the volume strength
( 29.2 - 20.2) (1.79 ´ 10 5 ) (a) 8.6 volume (b) 10 volume
1.37 (c) 15 volume (d) 25 volume
the number of significant figures is : 22. Which of the following is the best example of law of
(a) 1 (b) 2 (c) 3 (d) 4 conservation of mass?
9. The number of significant figures for the three numbers (a) 12 g of carbon combines with 32 g of oxygen to form 44 g
161 cm, 0.161 cm, 0.0161 cm are of CO2
(a) 3,4 and 5 respectively (b) 3,4 and 4 respectively (b) When 12 g of carbon is heated in a vacuum there is no
(c) 3,3 and 4 respectively (d) 3,3 and 3 respectively change in mass
10. A gas occupies a volume of 300 cc at 27°C and 620 mm (c) A sample of air increases in volume when heated at
pressure. The volume of gas at 47°C and 640 mm pressure is: constant pressure but its mass remains unaltered
(a) 260 cc (b) 310 cc (c) 390 cc (d) 450 cc (d) The weight of a piece of platinum is the same before and
11. The prefix 1018 is after heating in air
(a) giga (b) kilo (c) exa (d) nano 23. With increase of temperature, which of these changes?
12. A sample was weighted using two different balances. The (a) Molality (b) Weight fraction of solute
results were (c) Molarity (d) Mole fraction
(i) 3.929 g (ii) 4.0 g 24. A gas is found to have formula (CO)n. If its vapour density is
How would the weight of the sample be reported? 56, the value of n will be:
(a) 3.93 g (b) 3g (c) 3.9 g (d) 3.929 g (a) 7 (b) 5 (c) 4 (d) 3
25. The least count of an instrument is 0.01 cm. Taking all precautions,
13. The weight of one molecule of a compound of molecular
the most possible error in the measurement can be :
formula C60H122 is (a) 0.005 cm (b) 0.01 cm (c) 0.0001 cm (d) 0.1 cm
Some Basic Concepts of Chemistry 7

26. A metallic chloride contain 47.22% metal. Calculate the 37. What is the molarity of 0.2N Na 2 CO 3 solution?
equivalent weight of metal.
(a) 0.1 M (b) 0 M (c) 0.4 M (d) 0.2 M
(a) 39.68 (b) 31.76 (c) 36.35 (d) 33.46
2+
27. One litre hard water contains 12.00 mg Mg . Milli-equivalents 38. The molar solution of H 2SO 4 is equal to :
of washing soda required to remove its hardness is : (a) N/2 solution (b) N solution
(a) 1 (b) 12. 16 (c) 2N solution (d) 3N solution
(c) 1 × 10 –3 (d) 12. 16 × 10–3 39. The equivalent weight of a solid element is found to be 9. If
the specific heat of this element is 1.05 Jg–1 K–1, then its
28. The percentage weight of Zn in white vitriol [ZnSO4.7H2O] is
atomic weight is :
approximately equal to ( Zn = 65, S = 32, O = 16 and H = 1) (a) 17 (b) 21 (c) 25 (d) 27
(a) 33.65 % (b) 32.56 % (c) 23.65 % (d) 22.65 % 40. The maximum number of molecules are present in
29. 25ml of a solution of barium hydroxide on titration with a 0.1 (a) 15 L of H2 gas at STP
molar solution of hydrochloric acid gave a litre value of 35ml. (b) 5 L of N2 gas at STP
The molarity of barium hydroxide solution was (c) 0.5 g of H2 gas
(a) 0.14 (b) 0.28 (c) 0.35 (d) 0.07 (d) 10 g of O2 gas
30. 20
6.02 × 10 molecules of urea are present in 100 ml of its 41. The vapour density of a gas is 11.2, then 11.2 g of this gas at
solution. The concentration of urea solution is N.T.P. will occupy a volume-
(a) 0.02 M (b) 0.01 M (c) 0.001 M (d) 0.1 M (a) 11.2 L (b) 22.4 L (c) 11.2 mL (d) 22.4 mL
42. What is the mass of 1 molecule of CO.
(Avogadro constant, NA = 6.02 × 1023 mol–1)
(a) 2.325 × 10–23 (b) 4.65 × 10–23
31. Two solutions of a substance (non electrolyte) are mixed in –23
(c) 3.732 × 10 (d) 2.895 × 10–23
the following manner. 480 ml of 1.5 M first solution + 520 ml of
43. Calculate the volume at STP occupied by 240 gm of SO2.
1.2 M second solution. What is the molarity of the final
mixture? (a) 64 (b) 84 (c) 59 (d) 73
44. The number of gram molecules of oxygen in 6.02 × 1024
(a) 2.70 M (b) 1.344 M (c) 1.50 M (d) 1.20 M
CO molecules is
32. What volume of hydrogen will be liberated at NTP by the
(a) 10 gm molecules (b) 5 gm molecules
reaction of Zn on 50 ml dilute H2SO4 of specific gravity 1.3
(c) 1 gm molecules (d) 0.5 gm molelcules
and having purity 40%?
45. Which has maximum number of molecules?
(a) 3.5 litre (b) 8.25 litre (c) 6.74 litre (d) 5.94 litre
(a) 7 gm N2 (b) 2 gm H2
33. Following is the composition of a washing soda sample :
(c) 16 gm NO2 (d) 16 gm O2
46. Number of atoms in 558.5 gram Fe (at. wt. of Fe = 55.85 g mol–1)
Substance Molecular Wt. Mass percent
is
Na2CO3 106.0 84.8 (a) twice that in 60 g carbon (b) 6.023 ´ 1022
NaHCO3 84.0 8.4
(c) half that in 8 g He (d) 558.5 ´ 6.023 ´ 1023
NaCl 58.5 6.8
47. How many moles of magnesium phosphate, Mg3(PO4)2 will
On complete reaction with excess HCl, one kilogram of the contain 0.25 mole of oxygen atoms?
washing soda will evolve:
(a) 1.25 × 10–2 (b) 2.5 × 10–2
(a) 9 mol of CO2 (b) 16 mol of CO2
(c) 0.02 (d) 3.125 × 10–2
(c) 17 mol of CO2 (d) 18 mol of CO2
48. 7.5 grams of a gas occupy 5.6 litres of volume at STP. The gas
34. To neutralise completely 20 mL of 0.1 M aqueous solution of is
phosphorous acid (H3PO3), the value of 0.1 M aqueous KOH
(a) N2O (b) NO (c) CO (d) CO2
solution required is
49. 3 g of an oxide of a metal is converted to chloride completely
(a) 40 mL (b) 20 mL (c) 10 mL (d) 60 mL and it yielded 5 g of chloride. The equivalent weight of the
35. Density of a 2.05M solution of acetic acid in water is metal is
1.02 g/mL. The molality of the solution is (a) 3.325 (b) 33.25 (c) 12 (d) 20
(a) 2.28 mol kg–1 (b) 0.44 mol kg–1 50. The number of molecules in 16 g of methane is
(c) 1.14 mol kg–1 (d) 3.28 mol kg–1
16
36. The equivalent weight of MnSO4 is half of its molecular weight (a) 3.0 × 1023 (b) ´ 10 23
when it is converted to : 6.02

(d) MnO42-
-
(a) Mn 2O3 (b) MnO2 (c) MnO4 16
(c) 6.023 × 1023 (d) ´ 10 23
3.0
EBD_7327
8 CHEMISTRY

51. Number of g of oxygen in 32.2 g Na2SO4.10 H2O is (c) all the oxygen will be consumed
(a) 20.8 (b) 2.24 (d) all the ammonia will be consumed
(c) 22.4 (d) 2.08 65. Assuming fully decomposed, the volume of CO2 released at
52. The number of water molecules present in a drop of water STP on heating 9.85 g of BaCO3 (Atomic mass, Ba = 137) will
(volume 0.0018 ml) at room temperature is be
(a) 1.084 × 1018 (b) 6.023 × 1019 (a) 2.24 L (b) 4.96 L (c) 1.12 L (d) 0.84 L
(c) 4.84 × 10 17 (d) 6.023 × 1023 66. In a compound C, H and N atoms are present in 9 : 1 : 3.5 by
53. The number of moles of oxygen in one litre of air containing weight. Molecular weight of compound is 108. Molecular
21% oxygen by volume, under standard conditions are formula of compound is
(a) 0.0093 mole (b) 0.21 mole (a) C2H6N2 (b) C3H4N
(c) 2.10 mole (d) 0.186 mole (c) C6H8N2 (d) C9H12N3.
54. The number of molecules in 8.96 litre of a gas at 0ºC and 1 atm. 67. The simplest formula of a compound containing 50% of
pressure is approximately element X (atomic mass 10) and 50% of element Y (atomic
(a) 6.023 × 1023 (b) 12.04 × 1023 mass 20) is
(c) 18.06 × 1023 (d) 24.08 × 1022 (a) XY (b) XY3 (c) X2Y (d) X2Y3
55. The mass of a molecule of water is 68. The empirical formula of an acid is CH2O2, the probable
(a) 3 × 10–25 kg (b) 3 × 10–26 kg molecular formula of acid may be :
–26
(c) 1.5 × 10 kg (d) 2.5 × 10–26 kg (a) C3H6O4 (b) CH2O (c) CH2O2 (d) C2H4O2
56. How many atoms are contained in one mole of sucrose 69. An organic compound contains 49.3% carbon, 6.84%
(C12 H22O11)? hydrogen and its vapour density is 73. Molecular formula of
(a) 20 × 6.02 × 1023 atoms/mol the compound is :
(b) 45 × 6.02 × 1023 atoms/mol (a) C 3 H 5 O 2 (b) C 4 H10 O 2
(c) 5 × 6.02 × 1023 atoms/mol
(c) C 6 H10 O 4 (d) C 3 H10 O 2
(d) None of these
70. The number of atoms in 4.25 g of NH3 is approximately
57. How many moles of helium gas occupy 22.4 litre at 0°C and 1
atm pressure ? (a) 6 × 1023 (b) 2 × 1023 (c) 4 × 1023 (d) 1 × 1023
(a) 0.11 (b) 1.11 (c) 0.90 (d) 1.0 71. 30 g of magnesium and 30 g of oxygen are reacted, then the
residual mixture contains
58. Number of moles of NaOH present in 2 litre of 0.5 M NaOH is :
(a) 50 g of Magnesium oxide and 10 g of oxygen
(a) 1.5 (b) 2.0 (c) 1.0 (d) 2.5
(b) 40 g of Magnesium oxide and 20 g of oxygen
59. O2, N2 are present in the ratio of 1 : 4 by weight. The ratio of
number of molecules is : (c) 45 g of Magnesium oxide and 15 g of oxygen
(a) 7 : 32 (b) 1 : 4 (c) 2 : 1 (d) 4 : 1 (d) 60 g of Magnesium oxide only
60. The hydrogen phosphate of certain metal has formula 72. The mass of BaCO3 produced when excess CO2 is bubbled
MHPO4. The formula of metal chloride would be through a solution of 0.205 mol Ba(OH)2 is :
(a) MCl (b) M2Cl2 (c) MCl2 (d) MCl3 (a) 81 g (b) 40.5 g (c) 20.25 g (d) 162 g
61. Number of moles of KMnO4 required to oxidize one mole of 73. A compound contains 54.55 % carbon, 9.09% hydrogen ,
Fe(C2O4) in acidic medium is 36.36% oxygen. The empirical formula of this compound is :
(a) 0.167 (b) 0.6 (c) 0.2 (d) 0.4 (a) C3H 5O (b) C 4 H 8O 2
62. 10 g CaCO3 gives on strong heating CO2. It gives quicklime
(c) C 2 H 4O 2 (d) C 2 H 4O
(in grams)
(a) 5g (b) 4.4 g (c) 5.6 g (d) 4 g 74. In the reaction
63. What is the weight of oxygen required for the complete ® 4NO(g) + 6H 2 O(l ) ,when 1 mole
4NH 3 (g) + 5O 2 (g) ¾¾
combustion of 2.8 kg of ethylene ? of ammonia and 1 mole of O2 are made to react to completion
(a) 2.8 kg (b) 6.4 kg (c) 9.6 kg (d) 96 kg (a) 1.0 mole of H2O is produced
64. In the reaction (b) 1.0 mole of NO will be produced
4 NH3 ( g ) + 5O2 ( g ) ® 4 NO( g ) + 6H2 O(l ) (c) all the ammonia will be consumed
When 1 mole of ammonia and 1 mole of O2 are made to react (d) all the oxygen will be consumed
to completion, 75. A gas is found to have a formula [CO]x. If its vapour density
(a) 1.0 mole of H2O is produced is 70, the value of x is :
(b) 1.0 mole of NO will be produced (a) 2.5 (b) 3.0 (c) 5.0 (d) 6.0
Some Basic Concepts of Chemistry 9

1. 0.5400 g of a metal X yields 1.020 g of its oxide X 2O3. The 11. The specific heat of a metal is 0.16 cal g–1. The equivalent
mass of the metal is 20.04, the correct atomic mass of the metal
number of moles of X is : is :
(a) 0.01 (b) 0.02 (c) 0.04 (d) 0.05 (a) 40 (b) 20.04 (c) 40.08 g (d) 80.16 g
2. 12 L of H2 and 11.2 L of Cl2 are mixed and exploded. Find the 12. A metal oxide has the formula Z2O3. It can be reduced by
composition by volume of mixture. hydrogen to give free metal and water. 0.1596 g of the metal
(a) 11.2, 11.2, 22.4 (b) 0.8, 0, 22.4 oxide requires 6 mg of hydrogen for complete reduction. The
(c) 0.8, 0.8, 22.4 (d) 0.8, 11.2, 22.4 atomic weight of the metal is
(a) 27.9 (b) 159.6 (c) 79.8 (d) 55.8
3. The hydrated salt Na 2 CO3 .x H 2 O undergoes 63% loss in 13. Ratio of Cp and Cv of a gas ‘X’ is 1.4. The number of atoms of
mass on heating and becomes anhydrous. The value of x is the gas ‘X’ present in 11.2 litres of it at NTP will be
(a) 3 (b) 5 (c) 7 (d) 10 (a) 6.02 ×1023 (b) 1.2 × 1023
(c) 3.01 × 10 23 (d) 2.01 × 1023
4. On adding excess of CaCl2 to a solution containing Na2CO3
14. Percent by mass of a solute (molar mass = 28 g) in its aqueous
and NaHCO3, x g of precipitate was obtained. On adding in
solution is 28. Calculate the mole fraction (X) and molality (m)
drops to the filtrate, a further y g of precipitate was obtained.
of the solute in the solution.
In another experiment to the same amount of solution excess (a) X = 0.2, m = 10 (b) X = 0.2, m = 125/9
of CaCl2 was added, boiled and filtered. The amount of the (c) X = 0.8, m = 125/9 (d) X = 0.8, m = 10
precipitate in the second experiment would be 15. The density of 0.5 M glucose solution is 1.0900g ml–1. The
y molality of the solution is
(a) x + y (b) x + (a) 0.1000 (b) 0.2000 (c) 0.2500 (d) 0.5000
2
16. Haemoglobin contains 0.334% of iron by weight. The
x+ y molecular weight of haemoglobin is approximately 67200. The
(c) (d) none of these number of iron atoms (at. wt. of Fe is 56) present in one
2
molecule of haemoglobin are
5. 10 moles SO2 and 15 moles O2 were allowed to react over a (a) 1 (b) 6
suitable catalyst. 8 moles of SO3 were formed. The remaining (c) 4 (d) 2
moles of SO2 and O2 respectively are - 17. Specific volume of cylindrical virus particle is 6.02 × 10–2 cc/gm.
(a) 2 moles, 11 moles (b) 2 moles, 8 moles whose radius and length 7 Å & 10 Å respectively.
(c) 4 moles, 5 moles (d) 8 moles, 2 moles If NA = 6.02 × 1023, find molecular weight of virus
(a) 3.08 × 103 kg/mol (b) 3.08 × 104 kg/mol
6. If 0.5 mol of BaCl2 is mixed with 0.2 mole of Na3PO4, find the 4
(c) 1.54 × 10 kg/mol (d) 15.4 kg/mol
maximum amount of Ba3(PO4)2 that can be formed.
18. Percentage of Se in peroxidase anhydrase enzyme is 0.5% by
(a) 1 mole (b) 0.5 mole weight (at. wt. of Se = 78.4) then minimum molecular weight
(c) 0.1 mole (d) 0.01 mole of peroxidase anhydrase enzyme is
7. On reduction 1.644 gm of hot iron oxide give 1.15 gm of iron. (a) 1.568 × 103 (b) 15.68
Evaluate the equivalent weight of iron. (c) 2.136 × 104 (d) 1.568 × 104
(a) 18.62 (b) 19.13 19. In Haber process 30 litres of dihydrogen and 30 litres of
dinitrogen were taken for reaction which yielded only 50% of
(c) 18.95 (d) 12.95
the expected product. What will be the composition of
8. The volume of chlorine at STP required to liberate all the gaseous mixture under the aforesaid condition in the end?
bromine and iodine in 100 ml of 0.1 M each of KI and MBr2 will (a) 20 litres ammonia, 25 litres nitrogen, 15 litres hydrogen
be: (b) 20 litres ammonia, 20 litres nitrogen, 20 litres hydrogen
(a) 0.224 L (b) 0.336 L (c) 0.448 L (d) 0.560 L (c) 10 litres ammonia, 25 litres nitrogen, 15 litres hydrogen
9. 6.8 gm H2O2 present in 100 ml of its solution. What is the (d) 20 litres ammonia, 10 litres nitrogen, 30 litres hydrogen
molarity of solution? 20. Malachite has the formula Cu2CO3(OH)2. What percentage
(a) 1 M (b) 2 M (c) 3 M (d) 0.5 M by mass of malachite is copper?
(a) 25% (b) 50.9% (c) 57.5% (d) 63.5%
10. 1 c.c. N2O at NTP contains :
21. What volume of hydrogen gas, at 273 K and 1 atm. pressure
6.02
(a) 1.8 ´ 10 22 atoms (b) ´ 10 23 molecules will be consumed in obtaining 21.6 g of elemental boron
224 22400 (atomic mass = 10.8) from the reduction of boron trichloride
by hydrogen ?
(c) 1.32 ´ 10 23 electrons (d) All of these
224 (a) 67.2 L (b) 44.8 L (c) 22.4 L (d) 89.6 L
EBD_7327
10 CHEMISTRY

22. The number of atoms of Cr and O are 4.8 × 1010 and 9.6 × 1010 (a) decrease twice
respectively. Its empirical formula is (b) increase two fold
(a) Cr2O3 (b) CrO2 (c) remain unchanged
(c) Cr2O4 (d) None of these (d) be a function of the molecular mass of the substance
23. The unit J Pa–1 is equivalent to 36. The density (in g mL–1) of a 3.60 M sulphuric acid solution
(a) m3 (b) cm3 that is 29% H2SO4 (molar mass = 98 g mol–1) by mass will be
(c) dm 3 (d) None of these (a) 1.45 (b) 1.64 (c) 1.88 (d) 1.22
24. Sulphur forms the chlorides S2Cl2 and SCl2. The equivalent 37. The molality of a urea solution in which 0.0100 g of urea,
mass of sulphur in SCl2 is [(NH2)2CO] is added to 0.3000 dm3 of water at STP is :
(a) 8 g/mol (b) 16 g/mol
(a) 5.55 ´ 10 -4 m (b) 33.3 m
(c) 64.8 g/mol (d) 32 g/mol
(c) 3.33 × 10–2 m (d) 0.555 m
25. How many moles of KI are required to produce 0.4 moles of
38. Consider a titration of potassium dichromate solution with
K2HgI4 ?
acidified Mohr's salt solution using diphenylamine as
(a) 0.4 (b) 0.8
indicator. The number of moles of Mohr's salt required per
(c) 3.2 (d) 1.6
mole of dichromate is
26. 100 ml O2 and H2 kept at same temperature and pressure.
(a) 3 (b) 4 (c) 5 (d) 6
What is true about their number of molecules
39. A gaseous hydrocarbon gives upon combustion 0.72 g of
(a) NO > NH (b) NO < NH
2 2 2 2 water and 3.08 g. of CO2. The empirical formula of the
(c) NO = NH (d) NO + NH = 1 mole
2 2 2 2 hydrocarbon is :
27. The percentage of P2O5 in diammonium hydrogen phosphate
(a) C2H4 (b) C3H4 (c) C6H5 (d) C7H8
(NH4)2HPO4 is
40. Experimentally it was found that a metal oxide has formula
(a) 23.48 (b) 46.96 (c) 53.78 (d) 71.00
M0.98O. Metal M, present as M2+ and M3+ in its oxide. Frac-
28. Under similar conditions of pressure and temperature, 40 ml
tion of the metal which exists as M3+ would be :
of slightly moist hydrogen chloride gas is mixed with 20 ml of
(a) 7.01% (b) 4.08% (c) 6.05% (d) 5.08%
ammonia gas, the final volume of gas at the same temperature
41. Liquid benzene (C6H6) burns in oxygen according to the
and pressure will be
(a) 100 ml (b) 20 ml equation 2C 6 H 6 (l ) + 15O 2 ( g ) ¾¾® 12CO 2 ( g ) + 6 H 2 O( g )
(c) 40 ml (d) 60 ml How many litres of O2 at STP are needed to complete the
29. How many gram of sulphur can be obtained by the reaction of combustion of 39 g of liquid benzene?(Mol. wt. of O2 = 32,
1 mol of SO 2 with 22.4 L of H 2S at STP? C6H6 = 78)
(a) 96 g (b) 48 g (a) 74 L (b) 11.2 L (c) 22.4 L (d) 84 L
(c) 32 g (d) None of these 42. An organic compound whose empirical and molecular formula
30. 3 g of Mg is burnt in a closed vessel containing 3 g of oxygen. are same, contains 20% carbon, 6.7% hydrogen, 46.7%
The weight of excess reactant left is nitrogen and the rest oxygen. On heating it yields ammonia,
(a) 0.5 g of oxygen (b) 1.0 g of oxygen leaving a solid residue. The solid residue gives a violet colour
(c) 1.0 g of Mg (d) 0.5 g of Mg with dilute solution of alkaline copper sulphate. The organic
31. The mass of carbon an ode consumed (giving only compound is
carbondioxide) in the production of 270 kg of aluminium (a) NH2COONH4 (b) HCOONH4
metal from bauxite by the Hall process is (Atomic mass: (c) NH2NHCHO (d) NH2CONH2
Al = 27) 43. In order to prepare one litre normal solution of KMnO4, how
(a) 270 kg (b) 540 kg (c) 90 kg (d) 180 kg many grams of KMnO4 are required if the solution is to be
32. Volume occupied by one molecule of water used in acid medium for oxidation?
(density = 1 g cm–3) is : (a) 158 g (b) 62.0 g (c) 31.6 g (d) 790 g
(a) 9.0 × 10–23 cm3 (b) 6.023 × 10– 23 cm3 1
44. If 1 moles of oxygen combine with Al to form Al2O3 the
(c) 3.0 × 10–23 cm3 (d) 5.5 × 10– 23 cm3 2
33. How many moles of lead (II) chloride will be formed from a weight of Al used in the reaction is (Al = 27)
reaction between 6.5 g of PbO and 3.2 g of HCl ? (a) 27 g (b) 54 g (c) 49.5 g (d) 31 g
(a) 0.044 (b) 0.333 (c) 0.011 (d) 0.029 45. Number of moles of MnO-4 required to oxidize one mole of
34. Which has the maximum number of molecules among the ferrous oxalate completely in acidic medium will be :
following ? (a) 0.6 moles (b) 0.4 moles
(a) 44 g CO2 (b) 48 g O3 (c) 7.5 moles (d) 0.2 moles
(c) 8 g H2 (d) 64 g SO2 46. 10 g of hydrogen and 64 g of oxygen were filled in a steel
35. If we consider that 1/6, in place of 1/12, mass of carbon atom vessel and exploded. Amount of water produced in this
is taken to be the relative atomic mass unit, the mass of one reaction will be:
mole of a substance will (a) 3 mol (b) 4 mol (c) 1 mol (d) 2 mol
Some Basic Concepts of Chemistry 11

47. In the reaction, (b) Statement -1 is True, Statement -2 is True ; Statement-2 is


2Al( s ) + 6HCl( aq ) ® 2Al3+ (aq) + 6Cl- (aq) + 3H2 ( g ) NOT a correct explanation for Statement - 1
(a) 11.2 L H2(g) at STP is produced for every mole HCl(aq) (c) Statement - 1 is True, Statement- 2 is False
(d) Statement -1 is False, Statement -2 is True
consumed
48. Statement-1 : Volume of a gas is inversely proportional to
(b) 6 L HCl(aq) is consumed for every 3 L H2(g) produced
(c) 33.6 L H2(g) is produced regardless of temperature and the number of moles of gas.
Statement-2 : The ratio by volume of gaseous reactants and
pressure for every mole Al that reacts
products is in agreement with their mole ratio.
(d) 67.2 H2(g) at STP is produced for every mole Al that reacts.
49. Statement-1 : One mole of SO2 contains double the number
DIRECTIONS for Qs. 48 to 50 : These are Assertion-Reason of molecules present in one mole of O2.
type questions. Each of these question contains two statements: Statement-2 : Molecular weight of SO2 is double to that of
Statement-1 (Assertion) and Statement-2 (Reason). Answer these O2.
questions from the following four options. 50. Statement-1 : 1.231 has three significant figures.
(a) Statement- 1 is True, Statement-2 is True, Statement-2 is a Statement-2 : All numbers right to the decimal point are
correct explanation for Statement -1 significant.

Exemplar Questions 6. If the concentration of glucose (C 6H12 O6) in blood is


1. Two students performed the same experiment separately and 0.9 g L–1, what will be the molarity of glucose in blood?
each one of them recorded two readings of mass which are (a) 5 M (b) 50 M
given below. Correct reading of mass is 3.0 g. On the basis (c) 0.005 M (d) 0.5 M
of given data, mark the correct option out of the following 7. What will be the molality of the solution containing 18.25 g
statements. of HCl gas in 500 g of water?
(a) 0.1 m (b) 1 M
Students Readings (c) 0.5 m (d) 1 m
8. One mole of any substance contains 6.022 × 10 23
(i) (ii)
atoms/molecules. Number of molecules of H2SO4 present in
A 3.01 2.99 100 mL of 0.02 M H2SO4 solution is........
(a) 12.044 × 1020 molecules (b) 6.022 × 1023 molecules
B 3.05 2.95
(c) 1 × 1023 molecules (d) 12.044 × 1023 molecules
(a) Results of both the students are neither accurate nor 9. What is the mass percent of carbon in carbon dioxide?
precise. (a) 0.034% (b) 27.27%
(b) Results of student A are both precise and accurate. (c) 3.4% (d) 28.7%
(c) Results of student B are neither precise nor accurate. 10. The empirical formula and molecular mass of a compound
(d) Results of student B are both precise and accurate. are CH2O and 180 g respectively .What will be the molecular
2. A measured temperature on Fahrenheit scale is 200°F. What formula of the compound?
will this reading be on celsius scale? (a) C9H18O9 (b) CH2O
(a) 40 °C (b) 94 °C (c) C6H12O6 (d) C2H4O2
(d) 93.3 °C (d) 30 °C 11. If the density of a solution is 3.12 g mL–1, the mass of
3. What will be the molarity of a solution, which contains 5.85 g 1.5 mL solution in significant figures is......
of NaCl(s) per 500 mL? (a) 4.7 g (b ) 4680 ×10–3 g
(a) 4 mol L–1 (b) 20 mol L–1 (c) 4.680 g (d) 46.80 g
(c) 0.2 mol L –1 (d) 2 mol L–1 12. Which of the following statements about a compound is
4. If 500 mL of a 5 M solution is diluted to 1500 mL, what will be incorrect?
the molarity of the solution obtained? (a) A molecule of a compound has atoms of different
(a) 1.5 M (b ) 1.66 M elements.
(c) 0.017 M (d) 1.59 M (b) A compound cannot be separated into its constituent
5. The number of atoms present in one mole of an element is elements by physical methods of separation.
equal to Avogadro number. Which of the following element (c) A compound retains the physical properties of its
contains the greatest number of atoms? constituent elements.
(a) 4 g He (b) 46 g Na (d) The ratio of atoms of different elements in a compound
(c) 0.40 g Ca (d) 12 g He is fixed.
EBD_7327
12 CHEMISTRY

13. Which of the following statements is correct about the 17. 6.02 × 1020 molecules of urea are present in 100 mL of its
reaction given below? solution. The concentration of solution is : [2013]
(a) 0.01 M (b) 0.001 M
4Fe (s) + 3O2 (g) ¾¾
® 2Fe2O3 (g) (c) 0.1 M (d) 0.02 M
(a) Total mass of iron and oxygen in reactants = total mass 18. When 22.4 litres of H2(g) is mixed with 11.2 litres of Cl2(g),
of iron and oxygen in product therefore it follows law each at S.T.P., the moles of HCl(g) formed is equal to : [2014]
of conservation of mass. (a) 1 mole of HCl(g) (b) 2 moles of HCl(g)
(b) Total mass of reactants = total mass of product, (c) 0.5 moles of HCl(g) (d) 1.5 moles of HCl(g)
therefore, law of multiple proportions is followed. 19. 1.0 g of magnesium is burnt with 0.56 g O2 in a closed vessel.
(c) Amount of Fe2O3 can be increased by taking any one Which reactant is left in excess and how much ? [2014]
of the reactants (iron or oxygen) in excess. (At. wt. Mg = 24 ; O = 16)
(d) Amount of Fe2O3 produced will decrease if the amount (a) Mg, 0.16 g (b) O2, 0.16 g
of any one of the reactants (iron or oxygen) is taken in (c) Mg, 0.44 g (d) O2, 0.28 g
excess. 20. IfAvogadro number NA, is changed from 6.022 × 1023 mol –1 to
14. Which of the following reactions is not correct according to 6.022 × 1020 mol–1 this would change : [2015 RS]
the law of conservation of mass? (a) the definition of mass in units of grams
(a) 2Mg (s) + O2 (g) ¾¾ ® 2MgO (s) (b) the mass of one mole of carbon
(b) C3H8 (g) + O2 (g) ¾¾ ® CO2 (g) + H2O (g) (c) the ratio of chemical species to each other in a balanced
(c) P4 (s) + 5O2 (g) ¾¾ ® P4O10 (s) equation.
(d) CH4 (g) + 2O2 (g) ¾¾ ® CO2 (g) + 2H2O (g) (d) the ratio of elements to each other in a compound
15. Which of the following statements indicates that law of 21. What is the mass of precipitate formed when 50 mL of
multiple proportion is being followed? 16.9% solution of AgNO3 is mixed with 50 mL of 5.8% NaCl
(a) Sample of carbon dioxide taken from any source will solution ? [2015 RS]
always have carbon and oxygen in the ratio 1 : 2. (Ag = 107.8, N = 14, O = 16, Na = 23, Cl = 35.5)
(b) Carbon forms two oxides namely CO2 and CO, where (a) 28 g (b) 3.5 g
masses of oxygen which combine with fixed mass of (c) 7 g (d) 14 g
carbon are in the simple ratio 2 : 1. 22. A mixture of gases contains H2 and O2 gases in the ratio of 1
(c) When magnesium burns in oxygen, the amount of : 4 (w/w). What is the molar ratio of the two gases in the
magnesium taken for the reaction is equal to the mixture ? [2015]
amount of magnesium in magnesium oxide formed. (a) 4 : 1 (b) 16 : 1
(c) 2 : 1 (d) 1 : 4
(d) At constant temperature and pressure 200 mL of
23. 20.0 g of a magnesium carbonate sample decomposes on
hydrogen will combine with 100 mL oxygen to produce
heating to give carbon dioxide and 8.0 g magnesium oxide.
200 mL of water vapour.
What will be the percentage purity of magnesium carbonate
NEET/AIPMT (2013-2017) Questions in the sample ? [2015 RS]
16. In an experiment it showed that 10 mL of 0.05 M solution of (a) 75 (b) 96
chloride required 10 mL of 0.1 M solution of AgNO3, which (c) 60 (d) 84
of the following will be the formula of the chloride (X stands 24. The number of water molecules is maximum in : [2015 RS]
for the symbol of the element other than chlorine): (a) 18 molecules of water
[NEET Kar. 2013] (b) 1.8 gram of water
(a) X2Cl (b) X2Cl2 (c) 18 gram of water
(c) XCl2 (d) XCl4 (d) 18 moles of water
Some Basic Concepts of Chemistry 13

Hints & Solutions


EXERCISE - 1 8. (c) On calculation we find

1. (b) Equivalent weight of an element is its weight which reacts (29.2 – 20.2)(1.79≥105 )
= 1.17×106
with 8 gm of oxygen to form oxide. 1.37
Thus eq. weight of the given element As the least precise number contains 3 significant figures
32.33 therefore, answers should also contains 3 significant
= ´ 8 = 3.82 figures.
67.67
2. (c) Empirical formula weight = 12 + 2 + 16 = 30 9. (d) We know that all non-zero digits are significant and the
zeros at the beginning of a number are not significant.
180 Therefore number 161 cm, 0.161 cm and 0.0161cm have
n= =6
30 3, 3 and 3 significant figures respectively.
Molecular formula = (CH2O)6 = C6H12O6. PV PV
1 1
10. (b) From = 2 2
3. (b) ˆˆ† H + Cl + - T1 T2
HCl ‡ˆˆ
0.4moles 0.4moles
V1 ´ 640 620 ´ 300
=
ˆˆ† Ca 2+ +
CaCl2 ‡ˆˆ 2Cl -
(273 + 47) (273 + 27)
0.2moles 2´0.2 = 0.4moles

Total Cl– moles = 0.4 + 0.4 = 0.8 moles 620 ´ 300 ´ 320
V1 = = 310 cc
Moles 640 ´ 300
Molarity = 11. (c) Exa = 1018
Vol.in L
12. (a) Out of two 3.929 g is more accurate and will be reported
0.8
\ Molarity of Cl– = = 1.6 M. as 3.93 after rounding off.
0.5 13. (c) M.W. = 60 × 12 + 122 = 842
Wt. in g
4. (a) No. of moles = 842
Mol. wt Weight of one molecule = gm
6.02 ´1023
200 = 140 × 10–23 gm = 1.4 × 10–21 gm
No. of moles in 200 mg =
1000 ´ 44 14. (d) In CuO and Cu2O the O : Cu is 1 : 1 and 1 : 2 respectively.
= 4.5 × 10–3 moles This is law of multiple proportion.
No. of moles in 1021 molecules 15. (c) The H : O ratio in water is fixed, irrespective of its source.
10 21 Hence it is law of constant composition.
= = 1.67 × 10–3 moles 16. (a) 20 volume H2O2 means that 1mL of this H2O2 solutions
6.02 ´ 1023 produces 20 mL of O2 at N.T.P. on decomposition by
No. of moles left = (4.5 – 1.67) × 10–3 = 2.88 × 10–3 heat.
\ For 20 mL of O2, the volume of 20 volume H2O2
5. (c) NaCl + H 2SO 4 ¾¾
® NaHSO4 + HCl
xg 4.9g 6g 1.825g required = 1mL
For 1 mL of O2, the volume of 20 volume
According to law of conservation of mass "mass is
neither created nor destroyed during a chemical 1
H2O2 required = mL
change" 20
\ Mass of the reactants = Mass of products For 5000 mL or 5L of O2, the volume of 20
x + 4.9 = 6 + 1.825
or x = 2.925 g 1
volume H2O2 required = ´ 5000 mL = 250 mL
20
6. (d) SnCl2 SnCl4 17. (b) Given P = 0.0030m, Q = 2.40m & R = 3000m. In P(0.0030)
119 : 2 × 35.5 119 : 4 × 35.5 initial zeros after the decimal point are not significant.
Chlorine ratio in both compounds is Therefore, significant figures in P(0.0030) are 2. Similarly
= 2 × 35.5 : 4 × 35.5 = 1 : 2 in Q (2.40) significant figures are 3 as in this case final
7. (c) According to Avogadro's law "equal volumes of all zero is significant. In R = (3000) all the zeros are
gases contain equal number of molecules under similar significant hence, in R significant figures are 4 because
conditions of temperature and pressure". Thus if 1 L of they come from a measurement.
one gas contains N molecules, 2 L of any other gas 18. (d) 1 zepto = 10–21
under the same conditions of temperature and pressure 19. (c)
will contain 2N molecules.
EBD_7327
14 CHEMISTRY

- 31. (b) From the molarity equation.


20. (b) Number of valence electrons in a N 3 ion = 1 M1V1 + M2V2 = MV
- Let M be the molarity of final mixture,
Now, 1 mol or 42 g of N 3 has = 6.023 × 1023 ions
M1V1 +M V2
- 2
So, 42 g of N 3 has 6.023 × 4 × 1023 valency e– M= where V = V1 + V2
V
23 480 ´ 1.5 + 520 ´ 1.2
1 g of N 3 has 6.023 ´ 1´10 valency e–
-
M= = 1.344 M
42 480 + 520
23 32. (d) Zn + H2SO4 ¾¾ ® ZnSO4 + H2
4.2 g of N 3 has 4.2 ´ 6.023 ´ 1´ 10 valency e– i.e., 0.1
-
(50 mL)
42 Normality of H2SO4,
NA valency e–.
21. (c) Given 100 mL of H2O2 gives 1500 mL of O2 at NTP. purity % ´ sp.gravity ´ 10
N =
Þ 1 mL of H2O2 gives 15 mL of O2 at NTP. equ wt of H 2SO4
As we know that when 1 mL of H2O2 gives 10 mL of O2 40 ´ 1.3 ´ 10
at N.T.P., the solution is called 10 volume H2O2 i.e., the = = 10.61 N
98
volume strength of H2O2 is 10 volume. i.e. 1 L of H2SO4 contains = 10.61 gm H2SO4
So, when 1 mL of H2O2 gives 15 mL of O2 at N.T.P., the 10.61
volume strength of H2O2 is 15 voume. 50 mL of H2SO4 contains = ´ 50 g H2SO4
1000
22. (a) = 0.5305 g H2SO4
23. (c) Among all the given options molarity is correct because According to the reaction,
the term molarity involve volume which increases on 1 gm equivalent of H2SO4 will liberate
increasing temperature. = 1 gm equivalent of H2
24. (c) As we know that, So, 0.5305 of H2 SO4 will liberate
Molecular mass = 2 × Vapour density = 0.5305 gm equivalent of H2
112 0.5305
Þ (12 + 16)n = 2 × 56 Þ n = =4 = ´ 22.4 L at NTP = 5.9416 L of H2 at NTP
28 2
25. (a) In case of instrumental error, most possible error is equal 33. (a) Na 2CO 3 + 2HCl ¾¾
® 2NaCl + H 2 O + CO 2
to the least count of the instrument. So, most possible 1mol 1mol
instrumental error can be 0.01 cm for the instrument which NaHCO3 + HCl ¾¾
® NaCl + H 2O + CO 2
has a least count 0.01 cm. 1mol 1mol
26. (b) Suppose weight of metallic chloride = 100 gm
Na2CO3 + NaHCO3 + NaCl + HCl ¾¾
®
Then weight of metal = 47.22 gm
Weight of chlorine = 100 – 47.22 = 52.78 gm 848g 84g 68g (excess)
8mol 1mol
47.22 E55555555555555555F
\ Equivalent weight of metal = ´ 35.5 = 31.76 1kg washingsoda
52.78
27. (a) Mg ++ + Na 2CO3 ¾¾ ® MgCO3 + 2Na + ¾¾
® 8CO2 + CO 2
fromNa 2CO3 from NaHCO 3
1 g eq. 1g eq.
1 g eq. of Mg2+ = 12 g of Mg2+ = 12000 mg Thus, on complete reaction with HCl, 1kg of washing
soda will evolve 9 mol of CO2.
= 1000 milli eq. of Na 2CO3
34. (a) N1V1 = N 2 V2 (Note : H3PO3 is dibasic \ M = 2N)
\ 12 mg Mg++ = 1 milli eq. Na2CO3
20 ´ 0.2 = 0.1´ V (Thus. 0.1 M = 0.2 N)
28. (d) Molecular weight of ZnSO 4 .7H 2O \ V = 40 ml
= 65 + 32 + (4 × 16) + 7(2 × 1 + 16) = 287.
35. (a) Apply the formula d = M æç + 2 ö÷
1 M
65
\ percentage mass of zinc (Zn) = ´ 100 = 22.65% è m 1000 ø
287 where M = molarity, M2 = molecular weight of CH3COOH
29. (d) 25 × N = 0.1 × 35 ; N = 0.14 d = density, m = molality.
Ba(OH)2 is diacid base
N æ1 60 ö
hence N = M × 2 or M = \ 1.02 = 2.05 ç + ÷
2 è m 1000 ø
M = 0.07 M On solving we get, m = 2.28 mol/kg
36. (b) For equivalent weight of MnSO 4 to be half of its
6.02 ´ 1020
30. (b) Moles of urea present in 100 ml of sol.= molecular weight, change in oxidation state must be equal
6.02 ´ 10 23 to 2. It is possible only when oxidation state of Mn in
6.02 ´ 10 20 ´ 1000 product is + 4. Since oxidation state of Mn in MnSO4 is
\M = = 0.01M + 2. So, MnO2 is correct answer.
6.02 ´ 10 23 ´ 100 In MnO2, O.S. of Mn = +4
[ Q M = Moles of solute present in 1L of solution] \ Change in O.S. of Mn = +4 – (+2) = +2
Some Basic Concepts of Chemistry 15

Equivalent mass
44. (b) 6.02 × 1023 molecules of CO =1mole of CO
37. (a) Molarity = Normality × 6.02 × 1024 CO molecules = 10 moles CO
Molecular mass = 10 g atoms of O = 5 g molecules of O2
M 45. (b) 2g of H2 means one mole of H2, hence contains
= 0.2 ´ = 0.1 M 6.023 × 1023 molecules. Others have less than one mole,
2´ M
so have less no. of molecules.
38. (a) Normality 558.5
Molarity =
Replaceable hydrogen atom 46. (a) Fe (no. of moles) = = 10 moles = 10NA atoms.
55.85
Q H2SO4 is dibasic acid. No. of moles in 60 g of C = 60/12 = 5 moles = 5NA atoms.
\ Molar solution of H2SO4 = N/2 H2SO4 47. (d) 1 Mole of Mg3(PO4)2 contains 8 mole of oxygen atoms
39. (d) Using Dulong and Petit's law, \ 8 mole of oxygen atoms º 1 mole of Mg3(PO4)2
At. weight × Specific heat = 6.4 approx 1
0.25 mole of oxygen atom º ´ 0.25 mole of Mg3(PO4)2
6.4 8
Approx . Atomic weight =
Specific heat = 3.125 ´ 10 -2 mole of Mg3(PO4)2
6.4 7.5
= = 25.4780 48. (b) PV = nRT \ 5.6 × 1 = ´ 0.0821´ 273
-1 M. Wt.
(1/ 4.18) ´ 1.05Jg
App.weight 25.4780 M. Wt = 30.12 Hence gas NO.
Valency = = = 2.83 » 3
Equ.weight 9 Wt. of metal oxide
49. (b)
\ Atomic weight = valency × Equ. wt. Wt. of metal chloride
= 3 × 9 = 27
Eq. wt of metal + Eq. wt of oxygen
40. (a) No. of molecules in different cases =
(a) Q 22.4 litre at STP contains Eq. wt of metal + Eq. wt of chlorine
= 6.023 × 1023 molecules of H2
3 E+8
15 = \ E = 33.25
\ 15 litre at STP contains = ´ 6.023 ´ 10 23 5 E + 35.5
22.4
= 4.03 × 1023 molecules of H2 50. (c) 16 g CH4 is 1 mol. Hence number of molecules
(b) Q 22.4 litre at STP contains = Avogadro number = 6.023 × 1023.
= 6.023×1023 molecules of N2 51. (c) M. Wt of Na2SO4.10H2O is 322 g which contains 224 g
oxygen.
5 \ 32.2 g will contain 22.4 g oxygen.
Q 5 litre at STP contains = ´ 6.023 ´ 10 23
22.4 52. (b) 0.0018 ml = 0.0018 g = 0.0001 mole of water = 10-4 mole
= 1.344 × 1023 molecules of N2
(c) Q 2 gm of H2= 6.023×1023 molecules of H2 \ number of water molecules = 6.023 ´ 10 23 ´ 10 -4
= 6.023 × 1019
0.5 53. (a) 21% of 1 litre is 0.21 litre.
Q 0.5 gm of H2= ´ 6.023 ´10 23
2 22.4 litres = 1 mole at STP
= 1.505 × 1023 molecules of H2 0.21
(d) Similarly 10 g of O2 gas \ 0.21 litre = = 0.0093 mol
22.4
10 54. (d) At S.T.P. 22.4 litre of gas contains 6.023 × 1023 molecules
= ´ 6.023 ´ 10 23 molecules of O2 \ molecules in 8.96 litre of gas
32
= 1.88 × 1023 molecules of O2 6.023 ´10 23 ´ 8.96
Thus (a) will have maximum number of molecules = = 24.08 ´10 22
22.4
41. (a) Molecular mass of any gas occupies 22.4 L at N.T.P.
55 (b) Mass of one molecule of Water
Molecular mass
Vapour density = 18
2 = = 3 ´10 - 23 g = 3 ´10 - 26 Kg
Vapour density of any gas occupies a volume of 6.023 ´10 23
11.2 litres at N.T.P. 56. (b) Total atoms in 1 molecule of C12H22O11
42. (b) Gram molecular weight of CO = 12 + 16 = 28 g = 12 + 22 + 11 = 45
6.023 × 1023 molecules of CO weight 28 g \ Total atoms in 1 mole of C12H22O11
= 45 × 6.02 × 1023 atoms/mol.
28 = 4.65 ´10-23 g
1 molecule of CO weighs = 57. (d) 22.4 L of He at STP = 1 mole.
6.02´1023 58. (c) Given V = 2 L, Molarity = 0.5M, Moles = ?
43. (b) Molecular weight of SO2 = 32 + 2 × 16 = 64
64 g of SO2 occupies 22.4 litre at STP Molarity = No. of moles of solute or 0.5 = Moles
V of solution in L 2
22.4
240 g of SO2 occupies = ´ 240 = 84 litre at STP \ Moles = 2 × 0.5 = 1.0
64 59. (a) Let mass of O2 = 1 g
EBD_7327
16 CHEMISTRY

\ Mass of N2 = 4g
3.5 1
1 N 3.5 = 1
No. of molecules of O2 = 14 4
32
Empirical formula = C3H4N
4
No. of molecules of N2 = (C3H4N)n = 108
28 (12 × 3 + 4 × 1 + 14)n = 108
1 4 1 1 (54)n = 108
Ratio of no. of molecules = : = : = 7 : 32
32 28 32 7 108
60. (c) Formula of metal phosphate is M++H+PO4– – – . n= =2
54
Valency of metal + 2. Hence metal chloride is MCl2. \ molecular formula = C6 H8N2
61. (b) The required equation is 67. (c) 50% of X (Atomic mass 10), 50% of Y (Atomic mass 20).
2KMnO 4 + 3H 2SO 4 ¾
¾® Relative number of atoms of X = 50 = 5 and than
K 2 SO 4 + 2MnSO 4 + 3H 2 O + 5 [O ] 10
50
Y= = 2 . 5
nascent oxygen 20
Simple Ratio 2 : 1. Formula X2Y
2Fe(C2 O 4 ) + 3H 2 SO 4 + 3 [ O ] ¾¾
®
68. (c) The acid with empirical formula CH2O2 is formic acid,
Fe 2 (SO 4 )3 + 2CO 2 + 3H 2O H—COOH.
O required for 1 mol. of Fe(C2O4) is 1.5, 5O are obtained 69. (c) Element % Relative no. Simplest ratio of
from 2 moles of KMnO4 of atoms atoms
C 49.3 49.3/12 = 4.1 4.1/2.74 = 1.5
2 1.5 × 2 = 3
\ 1.5 [O] will be obtained from = ´ 1.5 = 0.6 moles of
5 H 6.84 6.84/1 = 6.84 6.84/2.74 = 2.5
KMnO4. = 2.5 × 2 = 5
62. (c) CaCO3 CaO + CO2 O 43.86 43.86/16 = 2.74 2.74/2.74 = 1
1×2=2
100 g 56 g
10 g CaCo3 will give 5.6 gCaO \ Empirical formula = C3H5O2
Empirical formula mass
63. (c) C2H4 + 3 O2 ¾ ¾® 2CO2 + 2H2O = (3 × 12) + (5 × 1) + (2 × 16) = 36 + 5 + 32 = 73
28 g 96 g Molecular mass = 2 × Vapour density
Q 28 g of C2H4 undergo complete combustion by = 2 × 73 = 146
= 96 g of O2
\ 2.8 kg of C2H4 undergo complete combustion by molecular mass
n= = 146/73 = 2
= 9.6 kg of O2. empirical formula mass
64. (c) According to Stoichiometry they should react as follow Molecular formula = Empirical formula × 2
4NH 3 + 5O 2 ¾¾ ® 4NO + 6H 2 O = (C3H5O2) × 2 = C6H10O4
4 mole of NH3 requires 5 mole of O2.
4.25 ´ 6.023 ´10 23 ´ 4
5 70. (a) Number of atoms = = 6 ´ 10 23
1 mole of NH3 requires = = 1.25 mole of O2. 17
4
Hence O2 is consumed completely. (One molecule of NH3 contains 4 atoms 1 N and 3H)
65. (c) BaCO3 ® BaO + CO 2 71. (a) 2Mg + O 2 ¾¾
® 2MgO
197 gm
2 × 24 2×16 2 × 40
Q 197 gm of BaCO3 released carbon dioxide
48 g 32 g 80 g
= 22.4 litre at STP
given 30 g 30 g
22.4 Actually
\ 1 gm of BaCO3 released carbon dioxide = litre
197 Reacting 30 g 20 g 50g (formed)
\ 9.85 gm of BaCO3 released carbon dioxide O2 left (30 – 20) = 10 g MgO formed 50 g.
22.4 72. (b) Ba(OH)2 + CO2 ¾¾ ® BaCO3 + H2 O
= ´ 9.85 = 1.12 litre n mol n mol
197
n mol Ba(OH)2 = n mol BaCO3
66. (c) Percentage R.N.A Simplest ratio \ 0.205 mol Ba (OH) 2 º 0.205 mol BaCO3
9 3 Wt. of substance = No. of moles × Molecular mass
C 9 = 3 = 0.205 × 197.3 = 40.5 g
12 4
73. (d) C 54.55 54.55/12 = 4.5 4.5/2.27 = 2
1 H 9.09 9.09/1= 9.09 9.09/2.27 = 4
H 1 =1 4
1 O 36.36 36.36/16 = 2.27 2.27/2.27= 1
Hence empirical formula of the compound = C2H4O
Some Basic Concepts of Chemistry 17

74. (d) 1.15


4 NH 3 (g ) + 5O 2 (g ) ¾
¾® 4 NO(g ) + 6H 2 O (l) \ Equivalent weight of iron = ´ 8 = 18.62
0.494
4 moles 5 moles 4 moles 6 moles
Thus equivalent weight of metal is = 18.62.
given 1 Mole 1 Mole
Reacting 0.8 1 ® 0.8 1.2
(formed) 8. (b) 2KI (aq ) + Cl 2 (g ) ® 2KCl(aq ) + I 2 ;
All, O2 consumed being limiting.
75. (c) As we know that, MBr2(aq ) + Cl 2( g ) ® MCl 2 (aq ) + Br2
Molecular mass of [CO]x = 2 × V.D. Mol of Cl2 required for liberating iodine from KI
Þ (12 + 16)x = 2 × 70 Þ 28x = 140 Þ x = 5
1 1
EXERCISE - 2 ´ mol of KI = ´ 100 ´ 10 -3 ´ 0.1 = 0.005
=
2 2
Mol of Cl2 required for liberating bromine from MBr2
1. (b) Mass of oxygen combined with 0.5400 g of X = 1.0200 = mol of MBr2 = 0.1 × 100 × 10–3 = 0.01
– 0.5400 = 0.4800 g Hence, volume of Cl2 (STP) required = (0.005 + 0.01) ×
2 ´ 0.48 22.4 = 0.336 L
Mol of X = = 0.02 9. (b) Q Weight of H2O2 in 100 ml of H2O2 solution = 6.8 gm
48
Q Weight of H2O2 in 1000 ml of its solution = 6.8×10 = 68g
2. (b) H2 + Cl2 ® 2HCl Molecular weight of H2O2 = 34
1L 1L 2L 68
11.2 L 11.2 L 22.4L Then, Molarity = = 2M
34
Volume of H2 = [12 – 11.2] = 0.8 L, 10. (d) At NTP 22400 cc of N2O = 6.02 × 1023 molecules
Volume of Cl2 = Zero, Volume of HCl = 22.4 L
3. (d) The loss in mass is due to elimination of water of 6.02 ´ 1023
\ 1 cc N2O = molecules
22400
crystallisation of Na 2 CO 3.xH 2 O.
3 ´ 6.02 ´ 10 23 1.8
18 x ´ 100 = atoms = ´ 1022 atoms
Hence , = 63 Þ x = 10 22400 224
106 + 18 x No. of electrons in a molecule of N 2O = 7 + 7 + 8 = 22
4. (b) On adding CaCl2, only CaCO3 will be precipitated Hence no. of electrons
whereas Ca(HCO3)2 is soluble.
6.02 ´1023 1.32 ≥1023
Na 2CO 3( aq) + CaCl 2(aq) ® Na 2 CO3 + H 2 O = ´ 22 electrons <
22400 224
On boiling, Ca (HCO 3 ) 2 changes into sparingly 11. (c) Following Dulong-Pettit law, approx. atomic mass
soluble CaCO3 as : 6. 4 6.4
= = = 40
Ca (HCO 3 ) 2(aq) ® CaCO 3(s ) + CO 2( g) + H 2O ( l) Specific heat 0.16
Hence, total mass of precipitate in second case 40 40
Valency of the metal = = =2
y Equiv.mass 20.04
= x+
2 Correct atomic mass = valency × eq.mass = 2 × 20.04 =
5. (a) 2SO2 + O2 ¾® 2SO3 40.08
10 15 0 12. (d) The reaction may given as
10 – 2x 15 – x 2x Z2 O3 + 3H2 ¾ ¾® 2Z + 3H2O
Q 2x = 8 x= 4 0.1596 g of Z2O3 react with H2 = 6 mg = 0.006 g
Hence, remaining, SO2 = 10 – 8 = 2 moles, 0.1596
O2 = 15 – 4 = 11 moles \ 1 g of H2 react with = = 26 .6 g of Z 2O 3
0.006
6. (c) 3 BaCl2 + 2 Na3PO4 ® 6 NaCl + Ba3(PO4)2
\ Eq. wt. of Z2O3 = 26.6 (from the definition of eq. wt.)
Molar ratio 3 2 6 1
Eq. wt. of Z + Eq. wt. of O = E + 8 = 26.6
Initial moles 0.5 0.2 0 0
Limiting reagent is Na 3 PO 4 hence it would be Þ Eq. wt. of Z = 26.6 – 8 = 18.6
consumed, and the yield would be decided by it inital Valency of metal in Z2O3 = 3
moles. Atomic wt.
2 moles of Na3PO4 give 1 mole of Ba3 (PO4)2 , Eq. wt.of metal <
valency
\ 0.2 moles of Na3PO4 would give 0.1 mole of Ba3(PO4)2 \ At. wt. of Z = 18.6 × 3 = 55.8
7. (a) Weight of iron oxide = 1.644 gm 13. (a) Cp / Cv = 1.4 shows that the gas is diatomic.
Weight of iron after reduction = 1.15 gm
22.4 litre at NTP º 6.02 × 1023 molecules
weight of displaced oxygen = 1.644 – 1.15 = 0.494 gm
11.2 L at NTP = 3.01 × 1023 molecules
EBD_7327
18 CHEMISTRY

= 3.01 × 1023 × 2 atoms = 6.02 × 1023 atoms 21. (a) 2BCl3 + 3H 2 ® 2B + 6HCl
28
14. (b) Mol. of solute in 100 g solution = =1 or BCl3 + 3 H 2 ® B + 3HCl
28 2
Now, since 10.8 gm boron requires hydrogen
Mol. of water in 100 g solution = 100 - 28 = 4 3
18 ≥ 22.4L at N.T.P
=
2
1 hence 21.6 gm boron requires hydrogen
Mol. fraction of solute = = 0. 2 ;
1+ 4 3 22.4
≥ ≥ 21.6 < 67.2L at N.T. P..
2 10.8
Molality = 1´ 1000 = 125 22. (b) Ratio of atoms of Cr and O = 4.8 × 1010 : 9.6 × 1010 = 1 : 2
72 9 Hence, empirical formula = CrO2
15. (d) Mass of 1 L (= 1000 ml) solution = 1000 × 1.090 = 1090g 23. (a) Joule is the unit of work and Pascal is unit of pressure.
Mass of glucose in 1L = 0.5 × 180 = 90 g. J Work Nm
JPa -1 = = = = m3
Mass of water = 1090.0 g – 90.0 g = 1000 g Pa Pressure Nm -2
0.5 ´ 1000 24. (b) The atomic weight of sulphur =32
Hence, molality = = 0.5 In SCl2 valency of sulphur = 2
1000
32
16. (c) Mass of iron in 100 g haemoglobin = 0.334 g So equivalent mass of sulphur = = 16
2
\ In 67200 g haemoglobin, mass of iron
25. (b) 2KI + HgI 2 ¾¾ ® K 2 HgI 4
67200≥ 0.334
< < 672≥ 0.33g Moles of KI required to produce 0.4 moles of K2HgI4
100 = 2 × 0.4 = 0.8
\ The number of Fe atoms in one molecule of 26. (c) This is Avogadro’s hypothesis.
haemoglobin According to this, equal volume of all gases contain
672 ´ 0.334 equal no. of molecules under similar condition of
= =4 temperature and pressure.
56
17. (d) Specific volume (volume of 1 gm) of cylindrical virus 1
27. (c) 1 mole of (NH4)2HPO4 would give mole of P2O5
particle = 6.02 × 10–2 cc/gm 2
Radius of virus (r) = 7 Å = 7 × 10–8 cm
2(NH 4 ) 2 HPO 4 º P2 O5
Length of virus = 10 × 10–8 cm 2(36 +1+ 31+ 64) = 264 62 +80 =142
Volume of virus
wt. of P2O 5
22 % of P2O5 = ´ 100
pr 2 l = ´ (7 ´ 10 -8 ) 2 ´ 10 ´ 10 -8 = 154 × 10–23 cc wt. of salt
7
142
volume = ´100 = 53.78%
Wt. of one virus particle = 264
specific volume
\Mol. wt. of virus = Wt. of NA particle 28. (b) NH 3 (g) + HCl(g) ® NH 4 Cl(g)
t=0 20ml 40ml 0
t=t 0 20ml solid
-23
154 ´ 10
= ´ 6.02 ´ 10 23 = 15400 g/mol = 15.4 kg/mole Final volume = 20 ml
6.02 ´ 10 - 2
18. (d) 0.5 g of Se = 100 g enzyme 29. (b) SO 2 + 2 H 2S ¾
¾® 2 H 2 O + 3S

100 22.4 L (STP) of H 2S = 1 mol


78.4 g of Se = ´ 78.4 = 1.568 × 104 g enzyme
0.5 3´ 32
Mass of S produced = g = 48 g
N 2 + 3H 2 ® 2NH 3 2
19. (c)
1 vol. 3 vol. 2 vol. 1
10 litre 30 litre 20 litre 30. (b) Mg + O2 ¾
¾® MgO
2
It is given that only 50% of the expected product is 16´ 3
formed hence only 10 litre of NH3 is formed Mass of oxygen required for 3 g of Mg =
= 2g
24
N2 used = 5 litres, left = 30 – 5 = 25 litres Hence, excess reactant = 3 – 2 = 1g oxygen
H2 used = 15 litres, left = 30 – 15 = 15 litres
31. (c) 2Al 2O3 ∗ 3C ¾¾ ↑ Al ∗ 3CO 2
20. (c) Percentage by mass of copper in malachite
Gram equivalent of Al2O3 º gm equivalent of C
2 ´ 63.5 27
= = 57.5%
221 Now equivalent weight of Al = =9
3
Some Basic Concepts of Chemistry 19

12 0 +4 35. (a) Relative atomic mass


Equivalent weight of C = = 3 (C ® CO 2 )
4 Mass of one atom of the element
= th
270 ´ 10 3 1/12 part of the mass of one atom of Carbon - 12
No. of gram equivalent of Al =
9 Mass of one atom of the element
or ´ 12
= 30 × 103 mass of one atom of the C - 12
Hence,
Now if we use 1/6 in place of 1/12 the formula becomes
No. of gram equivalent of C = 30 × 103
Again, Mass of one atom of element
Relative atomic mass = ´6
No. of gram equivalent of C Mass of one atom of carbon
mass in gram \ Relative atomic mass decrease twice
= 36. (d) Since molarity of solution is 3.60 M. It means 3.6 moles
gram equivalent weight
mass of H2SO4 is present in its 1 litre solution.
Þ 30 × 103 = Mass of 3.6 moles of H2SO4
3
Þ mass = 90 × 103 g = 90 kg = Moles × Molecular mass
Mass = 3.6 × 98 g = 352.8 g
32. (c) Density = \ 1000 ml solution has 352.8 g of H2SO4
Volume
1gram 29% H2SO4 by mass means 29 g of H2SO4 is present in
1 gram cm–3 =
cm 3 100 g of solution
Mass 1gram \ 352.8 g of H2SO4 is present in
Volume = = = 1cm 3
Density 1gram cm -3 100
= ´ 352.8 g of solution = 1216 g of solution
\ Volume occupied by 1 gram water = 1 cm3 29
or Volume occupied by Mass 1216
Density = = = 1.216 g/ml = 1.22 g/ml
Volume 1000
6.023 ´ 1023
molecules of water = 1 cm3 37. (a) Molality = Moles of solute / Mass of solvent in kg
18 0.01/ 60 0.01
Molality = = ;
1 0.3 60 ´ 0.3
[\ 1g water = moles of water]
18 d = 1 g/ml = 5.55 ´ 10-4 m
Thus volume occupied by 1 molecule of water 38. (d) The following reaction occurs:
1 ´ 18
cm 3 = 3.0×10–23 cm3. 6Fe 2 + + Cr2 O 27 - + 14H + ¾¾
® 6Fe3+ + 2Cr 3+ + 7H2O
= 23
6.023 ´ 10 From the above equation, we find that Mohr's salt
i.e. the correct answer is option (c). (FeSO4.(NH4)2SO4.6H2O) and dichromate reacts in 6 : 1
33. (d) Writing the equation for the reaction, we get molar ratio.
PbO + 2HCl ¾¾ ® PbCl2 + H2O 39. (d) Q 18 gm, H2O contains = 2 gm H
207 + 16 2 × 36.5 207 + 71 2
= 223 g = 73g = 278g \ 0.72 gm H2O contains = ´ 0.72 gm = 0.08 gm H
18
6.5
No. of moles of PbO = = 0.029 Q 44 gm CO2 contains = 12 gm C
223
12
3.2 \ 3.08 gm CO2 contains = ´ 3.08 = 0.84 gm C
No. of moles of HCl = = 0.0877 44
36.5 0.84 0.08
Thus PbO is the limiting reactant 1 mole of PbO produce \C:H= : = 0.07 : 0.08 = 7 : 8
12 1
1 mole PbCl2. \ Empirical formula = C7H8
0.029 mole PbO produces 0.029 mole PbCl2. 40. (b) For one mole of the oxide
34. (c) No. of molecules Moles of M = 0.98
44 Moles of O2– = 1
Moles of CO2 = =1 NA
44 Let moles of M3+ = x
48 \ Moles of M2+ = 0.98– x
Moles of O3 = =1 NA
48 On balancing charge
8 (0.98 – x) × 2 + 3x – 2 = 0
Moles of H2 = = 4 4NA
2 x = 0.04
64 0.04
Moles of SO2 = =1 NA \ % of M3+ = ≥100 < 4.08%
64 0.98
EBD_7327
20 CHEMISTRY

41. (d) 2C 6 H 6 + 15O 2 (g ) ® 12 CO 2 (g ) + 6 H 2 O(g ) Q 0.5 mole of O2 gives H2O = 1 mol


2 (78) 15(32) \ 2 mole of O2 gives H2O = 4 mol
Q156 gm of benzene required oxygen = 15 × 22.4 litre 47. (a) 2Al(s) + 6HCl(aq) ® 2Al3+(aq) + 6Cl– (aq) + 3H2(g)
15´ 22.4 Q 6 moles of HCl produces = 3 moles of H2
\1 gm of benzene required oxygen = litre = 3 × 22.4 L of H2 at S.T.P
156
\ 39 gm of Benzene required oxygen 3 ´ 22.4
\ 1 mole of HCl produces = L
15 ´ 22.4 ´ 39 6
= = 84.0 litre of H2 at S.T.P
156 = 11.2 L of H2 at STP
42. (d)
48. (d) We know that from the reaction H2 + Cl2 ® 2HCl that the
Atomic Relative no. Simplest ratio of the volume of gaseous reactants and products is
Element Percentage
mass of atoms ratio in agreement with their molar ratio.The ratio of H2 : Cl2 :
C 20% 12 1.66 1 HCl volume is 1: 1: 2 which is the same as their molar ratio.
H 6.7% 1 6.7 4 Thus volume of gas is directly related to the number of
N 46.7% 14 3.33 2
moles. Therefore, the assertion is false but reason is true.
O 26.6% 16 1.66 1
49. (d) One mole of any substance corresponding to
Empirical formula = Molecular formula 6.023 × 1023 entities in respective of its weight.
= CH4N2O or NH2CONH2
Molecular weight of SO2 = 32 + 2 ´ 16 = 64gm.
D
H 2 NCONH 2 + H 2 NCONH 2 ¾¾® Molecular weight of O2 = 16 ´ 2 = 32gm.
H2 NCONHCONH 2 + NH3 \ Molecular weight of SO2 is double to that of O2
biuret, 50. (d) 1.231 has four significant figures all no. from left to right
When an aqueous solution of biuret is treated with dilute are counted, starting with the first digit that is not zero for
sodium hydroxide and a drop of copper sulphate, a violet calculating the no. of significant figure.
colour is produced. This test is known as biuret test, is
characteristic of compounds having the group –CONH. EXERCISE - 3
43. (c) Eq. wt of KMnO4 in acid medium is 31.6 g. Hence this
much amount must be dissolved in 1 litre to prepare normal Exemplar Questions
solution. 1. (b) Average of readings of student A
3 3.01 + 2.99
44. (b) 2Al + O 2 ® Al2O3 = = 3.00
2
2
3 Average of readings of student B
According to equation mole of O2 combines with 2
2
mole Al. 3.05 + 2.95
= = 3.00
2 mole Al = 54 gm 2
45. (a) Correct reading = 3.00
Since, average value in both the cases is close to the
2KMnO4 + 3H2SO4 K2SO4 + 2MnSO4 + 3H2O + 5(O)] × 3
correct value. Hence, readings of both are accurate.
COO
2 Fe + 3(O) Fe2O3 + 2CO2 ]×5 Readings of student A differ only by 0.02 and also
COO close to the correct reading hence, readings are precise
6KMnO4 + 9H2SO4 +10 COO too. But readings of student B differ by 0.1 and hence
Fe 3K2SO4 + 6MnSO4 + 9H2O
(6 moles) COO + 5Fe2O3 + 10CO2 are not precise.
(10 moles) 2. (c) The relation between the temperatures on two scales
From above equation 6 moles MnO4– required to oxidise is given by the following relationship :
10 moles of oxalate. 9
Thus number of moles of MnO4– required to oxidise one °F = T °C + 32
5
6 5 200 - 32
mole of oxalate = = 0.6 moles
10 \ T °C = ( °F - 32) ´ = ´5
1 9 9
46. (b) H2 + O ¾¾® H2 O 168 ´ 5
2 2 Þ T°C = = 93.3 °C
10g 64g 9
æ 10 ö æ 64 ö weight ´ 1000
ç =5 mol ÷ ç = 2 mol ÷ Molarity =
è2 ø è 32 ø 3. (c) molecular weight ´ volume ( mL)
In this reaction oxygen is the limiting agent. Hence
amount of H2O produced depends on the amount of O2 5.85 ´ 1000
taken = = 0.2 mol L–1
58.5 ´ 500
Some Basic Concepts of Chemistry 21

4. (b) For dilution, a general formula is = 6 × CH2O


M1 V1 = M2 V2 = C6H12O6
(Before dilution) (After dilution) 11. (a) For a solution, Mass = volume × density
500 × 5M = 1500 × M2 = 1.5 mL × 3.12 g mL– 1= 4.68 g
The digit 1.5 has only two significant figures, so the
5
M2 = = 1.66 M answer must also be limited to two significant figure.
3 So it is rounded off to 4.7 g.
5. (d) number of atoms = No. of moles × NA 12. (c) The properties of a compound are quite different from
4 the properties of constituent elements. e.g., ammonia
Moles of 4 g He = = 1 mol Þ NA atoms is a compound containing hydrogen and nitrogen
4
combined together in a fixed proportion. But the
46 properties of ammonia are completely different from its
46 g Na = = 2 mol Þ 2 NA atoms
23 constituents, hydrogen and nitrogen.
13. (a) According to the law of conservation of mass,
0.40
0.40 g Ca = = 0.1 mol Þ 0.1 NA atoms Total mass of reactants = Total mass of products
40 14. (b) In this equation
12 C3H8 (g) + O 2 (g) ¾¾
® CO 2 (g) + H 2O (g)
12 g He = = 3 mol Þ 3 NA atoms 44g 32g 44g 18g
4
i.e.12 g He contains greatest number of atoms. i.e. mass of reactants ¹ mass of products
6. (c) In the given question, 0.9 g L–1 means that 1000 mL Hence, law of conservation of mass is not obeyed.
(or 1L) solution contains 0.9 g of glucose. 15. (b) In CO2, 12 parts by mass of carbon combine with 32
Molecular mass of glucose (C6H12O6) = 180 u parts by mass of oxygen while in CO, 12 parts by mass
of carbon combine with 16 parts by mass of oxygen.
0.9
\ Number of moles of glucose = =0.005 M Therefore, the masses of oxygen combine with a fixed
180 mass of carbon (12 parts) in CO2 and CO are 32 and 16
= 5×10–3 mol glucose respectively. These masses of oxygen bear a simple ratio
Hence, 1000 mL or 1L solution contains 0.005 mole of 32 : 16 or 2 : 1 to each other.
glucose or the molarity of glucose is 0.005 M. This is an example of law of multiple proportion.
Moles of solute NEET/AIPMT (2013-2017) Questions
7. (d) Molality (m) =
Mass of solvent ( in kg ) 16. (c) Millimoles of solution of chloride
Q Molecular weight of HCl = 36.5 g = 0.05 × 10 = 0.5
18.25 Millimoles of AgNO3 solution = 10 × 0.1 = 1
\ Moles of HCl = = 0.5
36.5 So, the millimoles of AgNO3 are double than the
chloride solution.
0.5 ´ 1000
m= =1m \ XCl2 + 2AgNO3 ¾® 2AgCl + X (NO3)2
500
8. (a) Number of millimoles of H2SO4. 6.02 ´ 10 20 ´ 1000 6.02 ´ 1021
= molarity × volume in mL 17. (a) M = 23 =
100 ´ 6.02 ´ 10 6.02 ´ 1023
= 0.02 × 100 = 2 millimoles
= 2 × 10–3 mol = 0.01 M
Number of molecules = Number of moles × NA. 18. (a) H2 + Cl2 ¾® 2HCl
= 2 × 10–3 × 6.022 × 1023 t = 0 22.4 lit 11.2 lit 0
= 12.044 × 1020 molecules t = 0 or 1 mole 0.5 mole 0
at time t (1 – 0.5)
9. (b) Molecular mass of CO2 = 44 g
0.5 × 2 = 0.5 = 1 mole
Q 44 g of CO2 contain C = 12 g atoms of carbon
1
12 19. (a) Initially Mg + O2 ¾® MgO
\ % of C in CO2 = ´ 100 = 27.27% 1g 2
44 0.56
i.e. mass percent of carbon in CO2 is 27.27%. 1 0.56
10. (c) Empirical formula mass of CH2O = 30 or mole mole
24 32
Molecular mass = 180 (Given)
0.0416 mole 0.0175 mole
Molecular mass 180
n= = =6 (0.0416 – 2 × 0.0175) (2 × 0.0175) mole
Empirical formula mass 30
= 0.0066 mole
\ Molecular formula = n × empirical formula
EBD_7327
22 CHEMISTRY

\ Mass of Mg = 0.0066 × 24 = (0.049) × (107.8 + 35.5)


= 0.158 » 0.16g = 7.02 g
20. (b) If 6.022 × 1023 changes to 6.022 × 1020/mol than this 22. (a) Ratio of weight of gases = wH : wO = 1 : 4
2 2
would change mass of one mole of carbon.
1 4
21. (c) 50 ml of 16.9% solution of AgNO3 Ratio of moles of gases = nH : nO = :
2 2 2 32
æ 16.9 ö
ç 100 ´ 50 ÷ = 8.45 g of Ag NO3 1 32
è ø \ Molar Ratio = ´ = 4 :1
2 4
8.45g
n mole =
(107.8 + 14 + 16 ´ 3) g / mol 23. (d) MgCO 3 ¾¾
® MgO + CO 2
æ 8.45 g ö 84 g of MgCO3 form 40 g of MgO
=ç ÷ = 0.0497 moles
è 169.8g / mol ø 40 ´ 20
\ 20g of MgCO3 form g of MgO
50 ml of 5.8% solution of NaCl contain 84
æ 5.8 ö = 9.52 g of MgO
NaCl = ç ´ 50 ÷ = 2.9 g
è 100 ø Since 8.0 g of MgO is formed
2.9g 8
nNaCl = = 0.0495 moles Purity of sample = ´ 100 = 84.0%
(23 + 35.5) g/ mol 9.52
AgNO3 + NaCl ® AgCl ¯ + Na + Cl

24. (d) No. of moles of water
1 mole 1 mole 1 mole In 1.8 g of H2O = 0.1 moles
\ 0.049 mole 0.049 mole 0.049 mole of AgCl In 18 g of H2O = 1 moles
1 mole contain 6.022 × 1023 molecules of water
w
n= ® w = (nAgCl) × Molecular Mass therefore maximum number of molecules is in 18 moles
M
of water.
Structure of Atom 23

2 Structure of Atom

SUB-ATOMIC PARTICLES Proton


Many different kinds of sub-atomic particles were discovered in The smallest and lightest positive ion is proton. These positively
the twentieth century. Three of these are electron, proton and charged particles are called canal rays. The credit for the discovery
neutron. of proton goes to Goldstein. Thomson and Wein estimated the
value of e/m as 9.578 × 104 coulomb per gram for the positively
Electron
charged particle proton. Proton is obtained when the only one
Electrons are the basic constituent of all atoms. They were electron present in hydrogen atom is removed. Hydrogen atom
discovered by cathode ray discharge tube experiment. Cathode consists of one electron, one proton and no neutron.
rays consist of negatively charged particles called electrons.
Neutron
The charge to mass ratio of electron is given by
The electrically neutral particles present in the atom are neutrons.It
e was discovered by James Chadwick in 1932. Neutron is relatively
me
= 1.758820 × 1011 C kg–1 heavier out of the three fundamental particles of atom.It is assumed
that a neutron is a result of joining together of an electron and a
where me ® mass of e–
proton. A neutron, being unstable, decays as follows :
e ® magnitude of charge on electron.
Charge on electron is – 1.6 × 10–19 C and the mass of electron is 1n
0
1 0
+1 H + - 1 e + 00q (antineutrino)
9.1 × 10–31 kg.
Its half-life is 20 minutes.
Charge and Mass of Fundamental Subatomic Particles

Name Symbol Absolute charge/c Relative charge Mass ( kg ) Mass ( u ) Approx. mass ( u )
-19 -31
Electron e –1.6 ´ 10 -1 9.1 ´ 10 0.00054 0
Pr oton p +1.6 ´ 10-19 +1 1.67 ´ 10 -27 1.00727 1
Neutron n 0 0 1.67 ´ 10 -27 1.00867 1
ATOMIC MODELS (iii) A few a-particles bounced back, i.e., were deflected by nearly
180°.
Thomson Model of Atom a-Particles deviated from
It is also called plum pudding, raisin pudding or water melon model. their path
It can be visualised as a pudding or watermelon of positive charge
with plums or seeds (e–1s) embedded into it. An important feature
of this model is that the mass of atom is assumed to be uniformly
Undeviated a-particles

distributed over the atom. The model was able to explain the overall
neutrality of atom but was not consistent with results of later
a-Particles Nucleus
experiments.
Rutherford’s Model of Atom
Rutherford performed the a-particle scattering experiment. A stream
of high energy a-particles from a radioactive source was directed
at a thin foil of gold metal. It was observed that:
a-Particle rebounded by the nucleus
(i) Most of a-particles passed through gold foil undeflected
Deviated a-particles
(ii) A small fraction of a-particles was deflected by small angles.
EBD_7327
24 CHEMISTRY

On the basis of the observations, following conclusions were Dual Nature of Electromagnetic Radiation
drawn. It means that the radiations possess both wave-like and particle–
(i) Most of the space in the atom is empty. like properties, i.e.,
(ii) The deflection of particles was due to the positive charge (a) Wave nature : Electromagnetic radiations are waves which
which is concentrated in a very small volume are transmitted when electrically charged particle moves under
(iii) The volume occupied by the nucleus is small as compared to acceleration, producing alternate electric and magnetic fields.
the total volume of the atom. Electromagnetic radiations are of many types, differing from
Thus, according to this model, one another in wavelength (or frequency). They constitute
(i) The positive charge and most of the mass of the atom was the electromagnetic spectrum.
densely concentrated in a small region called the nucleus of These radiations are characterised by frequency (n) and
atom. wavelength (l). SI unit of n is Hertz (Hz) or s–1
(ii) The nucleus is surrounded by e–1s that move around the SI unit of l is m.
nucleus with a very high speed in circular paths called orbits. n, l and c (speed of light) are related as c = nl
(iii) e–1s and nucleus are held together by electrostatic forces of 1
attraction. n = wave number =
l
Drawback of Rutherford model: (i) It cannot explain the stability Electromagnetic spectrum: It is defined as the arrangement
of atom. (ii) It does not explain the electronic structure of atom. of various types of electromagnetic radiation in terms of
increasing (or decreasing) wave lengths (or frequency). The
Atomic Number and Mass Number complete range of electromagnetic waves is called
(i) Atomic number (Z) = number of protons in nucleus of atom electromagnetic spectrum. The wavelength of various waves
= number of e–1s in a neutral atom increases in the following order
(ii) Mass number (A) = number of protons (Z) + number of Cosmic rays < g-rays < X-rays < UV rays < Visible < IR rays
neutrons (n)
< Micro waves < Radio waves
For example, 16 14 23
8 O 7 N 11 Na 24 0
10 u (Hz)
10 IR MW Radio Long
Protons 8 7 11 g-rays X- UV
–16 rays waves radio 108 l (m)
Neutrons 8 7 12 10 waves
¾¾¾¾¾¾¾¾¾¾¾¾¾¾¾¾¾¾¾¾¾¾¾ Visible
spectrum
Mass number 16 14 23 VIBGYOR
¾¾¾¾¾¾¾¾¾¾¾¾¾¾¾¾¾¾¾¾¾¾¾ 400
l(nm) 750
Isobars and Isotopes
(b) Particle nature
(i) Isobars are the atoms with same mass number but different
According to Planck, atoms and molecules emit energy only
atomic number. For ex: in discrete quantities and gave the name quantum to the
40 40 smallest quantity of energy that can be emitted or absorbed
18 Ar and 19 K
(ii) Isotopes are atoms of the same element with same atomic The energy of quantum of radiation is given by:
number but different mass number. For ex: Hydrogen has 3 E = hn
isotopes where n ® frequency
h ® Planck’s constant (6.6 × 10–34 Js)
1 , 2 ,3
1H 1 H 1 H Photoelectric effect: when a beam of light strikes the surface
of a metal, e–1s are ejected from it and the phenomenon is
Isosters called photoelectric effect. For each metal, there is a minimum
They are the molecules which have the same number of atoms frequency (n0) called threshold frequency below which
and electrons. photoelectric effect is not observed.
Ex. CO2 N2O Work function : A part of the photons energy that is absorbed
Atoms = 1+ 2 2+ 1 by the metal surface to release the e–1s known as work
= 3 =3 function of the surface. The remaining part of energy goes
Electrons = 6+8×2 7×2+8 into K.E.
= 22 = 22 At n > n0, ejected e–1s come out with some K.E.
1 2
\ h n = h n0 + mv i.e. E = E 0 + KE
2
DEVELOPMENTS LEADING TO BOHR’S MODEL OF Where, E : energy of radiation
ATOM E0 : minimum energy
Two developments played a major role in the formulation of Bohr’s Electromagnetic wave theory could explain phenomenon like
model of atom. These are: interference, diffraction, etc., but it could not explain some
(i) Dual character of electromagnetic radiation other phenomenon like black body radiation, photoelectric
(ii) Atomic spectra which is explained by assuming quantized effect, etc. These phenomenon could be explained only if em
electronic energy levels in atoms. waves are supposed to have particle nature. It was explained
by Max Planck and is called the Planck’s Quantum Theory.

Structure of Atom 25

ATOMIC SPECTRA (ii) The energy of e– in the orbit does not change. The energy
Spectrum is a series of coloured bands which are formed when change takes place only when an e– jump from lower energy
white light passes through a prism. A spectrum is of mainly three orbit to higher energy orbit or vice versa.
types : (iii) The frequency of radiation absorbed or emitted is given by:
(i) Emission spectrum DE E 2 - E1
(ii) Absorption spectrum n= =
h h
(iii) Molecular spectrum Where, E1 = energy of lower state
Emission Spectra E2 = energy of higher state
The spectrum of radiation emitted by a substance that has (iv) The angular momentum of an electron in a given stationary
absorbed energy is called emission spectrum. Emission spectra
nh
are of the following two types state is mvr =
(i) Continuous spectrum and 2p
(ii) line spectrum or atomic spectrum (v) Frequency (n) associated with the absorption and emission
of photon is given as :
(i) Continuous Spectrum : When sunlight or a glowing heat
fluorescent substance like tungsten wire present in an elec- DE R H æ 1 1ö
tric bulb is analysed with the help of a spectroscope, the n= = ç 2 - 2÷
h h è ni n f ø
spectrum obtained on a screen is observed as divided into
bands of seven colours, which are in a continuous sequence.
Such a spectrum is called a continuous spectrum. v RH æ 1 1ö
n= = ç 2 - 2÷
(ii) Line Spectrum : In the gas phase, the emission spectra of C hc è n i n f ø
atoms is not continuous, rather they emit light only at specific
wavelengths with dark spaces between them. Such spectra æ 1 1ö
are called line spectra or atomic spectra. = 1.09677 ´ 107 ç 2 - 2 ÷ m -1
è ni nf ø
Absorption Spectra
It is observed when a continuum of radiation is passed through a Features of Bohr’s Model
sample which absorbs radiation of certain wavelengths. The According to Bohr’s theory of H-atom:
missing wavelength leaves dark spaces in the bright continuum (i) The stationary states for e– are numbered as 1, 2, 3, .... and
spectrum. are known as principal quantum numbers.
Molecular Spectrum (ii) The radii of the stationary states are given by:
Molecular spectrum is given by molecules and it is also known as r n = n 2 a0 where a0 = 52.9 pm
band spectrum. (iii) The energy of the stationary state is given by:

LINE SPECTRUM OF HYDROGEN æ 1 ö


E n = -R H ç ÷ where, RH ® Rydberg constant
The spectrum of hydrogen consists of several lines named after è n2 ø
their discover. (iv) Bohr’s theory can be applied to ions containing only one e–
The visible lines of the H-spectrum obey the following formula: like He+, Li2+, Be3+ and so on. The energy of stationary
states of these ions is given by:
æ 1 1ö
n = 109,677 -
ç n 2 n2 ÷ æ Z2 ö
è 1 2ø
E n = -2.18 ´10-18 ç ÷J
ç n2 ÷
where n = wave number,, è ø
109,677 cm–1 = Rydberg’s const (RH) and, radii by the expression

Series n1 n2 Spectral region


rn =
( ) pm
52.9 n 2

Lyman 1 2,3... UV Z
Balmer 2 3, 4,... Visible Limitations of the Bohr’s Model
Paschen 3 4,5,... IR Bohr’s model ,
Brackett 4 5, 6,... IR (i) was unable to explain the spectrum of atoms other than
hydrogen.
Pfund 5 6, 7,... IR
(ii) could not explain the ability of atoms to form molecules by
chemical bonds.
BOHR’S MODEL FOR H-ATOM (iii) was unable to explain splitting of spectral lines in magnetic
This model is based on following postulates : field (Zeeman effect).
(i) The electron in H-atom moves around the nucleus in circular (iv) was unable to explain the splitting of lines in electrical field
paths called orbits, stationary states or energy states. (Stark effect)
EBD_7327
26 CHEMISTRY

DUAL BEHAVIOUR OF MATTER orbital. There are (2l + 1) orbitals of each type in a subshell,
DeBroglie’s concept was experimentally verified by Davisson and i.e., one s-orbital (l = 0); three p-orbitals (l = 1) and five
Germer by observing diffraction effect (a property shown by d-orbitals (l = 2) per sub-shell.
(iii) ml designates the orientation of the orbital. For a given value
waves) with an electron beam.
of l, ml has (2l + 1) values.
On the basis of dual nature of matter, de Broglie gave
(iv) ms refers to orientation of the spin of the e–. ms can have two
the following relation betweem wavelength (l) and momentum (p)
1 1
of material particle, i.e. values + or – .
2 2
h h
l= = where, h = Planck’s constant SHAPES OF ATOMIC ORBITALS
mv p
(i) s-orbital:
HEISENBERG’S UNCERTAINTY PRINCIPLE For s-orbitials l= 0, so there is only one value of m i.e. m = 0.
According to the principle, it is impossible to determine Thus, s-orbital can have only one orientation i.e. the
simultaneously, the exact position and exact momentum of an probability of finding electron is same in all directions at a
given distance from the nucleus. Hence s-orbital is
electron. Mathematically, symmetrical around the nucleus and thus has spherical shape.
h
Dx.Dp ³
4p

h
Dx.D ( mv x ) ³
4p

h
Dx.Dv x ³
4 pm
Here
Dx is uncertainty of position, (ii) p-orbital:
Dp is uncertainty of momentum and For p-orbitals l = l, the permissible values of m are +1, 0 and –
h is Planck’s constant 1. Thus, there are three p-orbitals, designated as px, py and
pz, in each p-subshell.
QUANTUM MECHANICAL MODEL OF ATOM x x x
The branch of science that takes into account dual behaviour of y y y
matter is called quantum mechanics.
z z z
Schrodinger Wave Equation
Schrodinger (1927) gave a mathematical expression known as a
Schrodinger wave equation. His theory is based on quantum px pz
mechanical model of atom in which the concept of probability of (iii) d-orbital
finding the electron at any position around the nucleus at any For d-orbitals l = 2 i.e. the permissible values of m are –2,
instant of time is considered. –1,0, +1, +2. This indicates for the five d-orbitals designated
Significance of wave function : An atomic orbital is the wave
as d xy , d yz , d zx , d 2 2 and d 2 .
x -y
function y for an e– in an atom. y2 is known as probability density. z
y2 gives the probability of finding the e– around the nucleus. x y x
QUANTUM NUMBERS
The position of an electron in any atom can be ascertained with y z z
the help of quantum numbers. In an atom, a shell consists of sub-
shells and the sub-shell consists of orbitals. Each orbital can
accommodate only two electrons, which have opposite spins. dxy dyz dxz
Quantum numbers are designated as principal quantum number z
(n), azimuthal quantum number (l), magnetic quantum number x
(m) and spin quantum number(s).The four quantum numbers
provide the following information
(i) n defines the shell, determines the size of the orbital and the y
energy of the orbital. An atom has K, L, M, N, O, P, Q, etc.
shell.
(ii) l identifies the subshell and determines the shape of the dx 2 – y 2 dz 2
Structure of Atom 27

NODES OF ORBITALS (ii) Pauli Exclusion principle: It states that no two e–1s in an
The region where probability density function reduces to atom can have same set of four quantum numbers.
zero is called nodes. (iii) Hund’s rule of maximum multiplicity: It states that pairing
Generally ns – orbital has (n – 1) nodes of e–1s in orbitals belonging to same subshell (p, d or f ) does
Number of spherical/radial nodes in any orbital = n – l – 1 not take place until each orbital belonging to that subshell
Number of planer nodes in any orbital = l has got one e– each, i.e., it is singly occupied.
\ Total number of nodes in any orbital = n – 1 Writing The Configuration of Atoms/ Ions
ENERGIES OF ORBITALS First of all, the configuration of the atom is written. Then,
The energy of orbitals for H-atom depends only on n and increases appropriate number of electrons are deducted from the outermost
as follows: shell for the configuration of the cation. Similarly, appropriate
1s < 2s = 2p < 3s = 3p = 3d < 4s = 4p = 4d = 4f number of electrons are added to the outermost shell for the
The orbitals having same energy are called degenerate. For multi configuration of the anion.
e– atoms, energy of an e– depends on n and l. The lower the value Stability of Completely Filled or Half-filled Sub-
of (n + l) for an orbital, the lower is its energy. If two orbitals have shells
same value of (n + l), the orbital with lower value of n will have The completely filled or exactly half-filled sub-shells are stable
lower energy. due to following reasons.
Filling of Orbitals in Atom (i) Orbitals which are either half filled or fully filled are more
(i) Aufbau principle: It states that in the ground state of atoms, symmetrical and therefore possess lower energy i.e. extra
orbitals are filled in order of their increasing energies. stability.
The order in which energies of orbitals increase is: (ii) When orbitals are half filled or fully filled, the exchange of
1s < 2s < 2p < 3s < 3p < 4s < 3d < 4p < 5s < 4d < 5p < 4f < 5d < electrons between the orbitals is maximum. Such exchanges
6p < 7s. leads to greater stability of electrons in the orbitals, because
low exchange energy results in to higher stabilization.
28

CONCEPT MAP
CHEMISTRY

EBD_7327
Structure of Atom 29

1. The ion that is isoelectronic with CO is 13. The compound in which cation is isoelectronic with anion is :
(a) CN– (b) O2+ (c) O2 – (d) N2+ (a) NaCl (b) CsF (c) NaI (d) K2S
2. Set of isoelectronic species is 14. What is the ratio of mass of an electron to the mass of a
(a) N 2 , CO 2 , CN- , O 2 (b) N 2 , H 2S, CO proton?
(a) 1 : 2 (b) 1 : 1 (c) 1 : 1837 (d) 1 : 3
(c) N 2 , CO , CN - , O +2 2 (d) Ca , Mg, Cl
15. The increasing order for the values of e/m (charge/mass) is
3. The wave-function (y ) for a 2s– electron as a function of its (a) e, p, n, a (b) n, p, e, a (c) n, p, a, e (d) n, a, p, e
distance from the nucleus (r) can be sketched as: 16. If the mass number of an element is W and its atomic number
is N, then
y y (a) Number of e–1 = W – N (b) Number of 1H1 = W – N
(a) (b) (c) Number of 0n1 = W – N (d) Number of 0n1 = N
r r 17. According to Bohr’s theory the energy required for an electron
in the Li2+ ion to be emitted from n = 2 state is (given that the
y y ground state ionization energy of hydrogen atom is 13.6 eV)
(c) (d) (a) 61.2 eV (b) 13.6 eV (c) 30.6 eV (d) 10.2 eV
r r
18. Which one is correct for the de-Broglie equation?
4. The combination of three 2p orbitals of one atom and three 2p (a) E = l/p (b) p = h/l
orbitals of another atom can give rise to : (c) E = h/p (d) none of these
(a) three molecular orbitals 19. Which of the following pairs is correctly
(b) six (p 2p) molecular orbitals matched ?
(c) four (p 2p) and two (s 2p) molecular orbitals (a) Isoelectronics N3–, O2–, Cr 3–
(d) two molecular orbital which are symmetrical around the
(b) Isotones 14Si30, 15P31, 16S32
bond axis
5. The species isoelectronic with C2H4 is (c) Isotopes 20Ca40, 19K40
(d) Isobars 8O16, 8O17, 8O18
(a) O+2 (b) CN –
20. The spectrum of He is expected to be similar to that of
(c) N2+ (d) O2
(a) H (b) Li+ (c) Na (d) He+
6. Cathode rays are deflected by
21. Which of the following statements do not form a part of Bohr’s
(a) an electric field only (b) magnetic field only
model of hydrogen atom ?
(c) by both (d) by none
(a) Energy of the electrons in the orbits are quantized
7. The number of neutrons in dipositive zinc ion with mass
(b) The electron(s) in the orbit nearest to the nucleus has the
number 70 is
lowest energy
(a) 34 (b) 36 (c) 38 (d) 40
(c) Electrons revolve in different orbits around the nucleus
8. Which has the highest e/m ratio ?
(d) The position and velocity of the electrons in the orbit
(a) He2+ (b) H+ (c) He+ (d) D+
cannot be determined simultaneously.
9. One would expect proton to have very large :
22. The radius of hydrogen atom in the ground state is 0.53 Å.
(a) ionization potential (b) radius The radius of Li2+ ion (atomic number = 3) in a similar state is
(c) charge (d) hydration energy (a) 0.17 Å (b) 0.265 Å (c) 0.53 Å (d) 1.06 Å
10. Which is correct statement about proton ? 23. The Bohr orbit radius for the hydrogen atom (n = 1) is
(a) Proton is nucleus of deuterium approximately 0.530 Å. The radius for the first excited state
(b) Proton is a-particle (n = 2) orbit is (in Å)
(c) Proton is ionized hydrogen molecule (a) 0.13 (b) 1.06 (c) 4.77 (d) 2.12
(d) Proton is ionized hydrogen atom 24. MO configuration of He2 is –
11. Which one of the following is expected to have largest size? (a) (s 1s)2, (s* 1s)2, (s 2s)1 (b) (s 1s)2, (s* 1s)2, (s* 2s)1
(a) F - (b) O 2 - (c) Al 3+ (d) N 3- (c) (s 1s)2, (s* 1s)1, (s 2s)2 (d) (s 1s)2, (s* 1s)1, (s* 2s)2
12. The maximum number of electrons in p-orbital with n = 6; 25. The quantum number which is responsible for the size of
m = 0 is electron cloud is
(a) 6 (b) 2 (c) 14 (d) 10 (a) spin (b) azimuthal
(c) principal (d) magnetic
EBD_7327
30 CHEMISTRY

26. "Electrons are filled in energy orbitals, in increasing order of 37. Correct set of four quantum numbers for valence electron of
energy." This statement is related to rubidium (Z = 37)is
(a) Planck's rule (b) Hund's rule
(c) Pauli's rule (d) Aufbau principle 1 1
(a) 5, 0, 0, + (b) 5,1, 0, +
27. The energy of second Bohr orbit of the hydrogen atom is 2 2
-328 kJ mol-1; hence the energy of fourth Bohr orbit would
1
be: (c) 5,1,1, + (d) 6, 0, 0, + 1
(a) -41 kJ mol-1 (b) -82 kJ mol-1 2 2
(c) -164 kJ mol-1 (d) -1312 kJ mol-1 38. Zeeman effect refers to the :
28. The orbital angular momentum for an electron revolving in an (a) splitting up of the lines in an emission spectrum in the
h presence of an external electrostatic field
orbit is given by l (l + 1) . . This momentum for an
2p (b) random scattering of light by colloidal particles
s-electron will be given by (c) splitting up of the lines in an emission spectrum in a
h h 1 h magnetic field
(a) zero (b) (c) 2. (d) + .
2p 2p 2 2p (d) emission of electrons from metals when light falls upon
29. The wavelength of the radiation emitted, when in a hydrogen them
atom electron falls from infinity to stationary state 1, would
39. The energy of electron in first energy level is - 21.79 ´ 10 -12
be (Rydberg constant = 1.097×107 m–1)
(a) 406 nm (b) 192 nm erg per atom. The energy of electron in second energy level
(c) 91 nm (d) 9.1×10–8 nm is :
30. The frequency of radiation emitted when the electron falls (a) - 54.47 ´10-12 erg atom-1
from n = 4 to n = 1 in a hydrogen atom will be (Given : ionization
energy of H=2.18 ×10–18J atom–1and h = 6.625 × 10–34 J s ) (b) - 5.447 ´ 10-12 erg atom-1
(a) 1.54 ´1015 s -1 (b) 1.03 ´1015 s -1 (c) - 0.5447´10-12 erg atom-1
-12 -1
(c) 3.08 ´1015 s -1 (d) 2.00 ´1015 s -1 (d) - 0.05447´10 erg atom
31. According to Bohr's theory, the angular momentum of an 40. Which of the following statements about the electron is
electron in 5th orbit is incorrect?
(a) It is negatively charged particle
(a) 10 h / p (b) 2.5 h / p (c) 25 h / p (d) 1.0 h / p
(b) The mass of electron is equal to the mass of neutron.
32. Which of the following transitions of electrons in the hydrogen
atom will emit maximum energy ? (c) It is a basic constituent of all atoms.
(a) n5 ® n4 (b) n4 ® n3 (d) It is a constituent of cathode rays.
(c) n3 ® n2 (d) all will emit same energy 41. In a Bohr model of an atom, when an electron jumps from n =
33. The allowed values of m for l = 2 is 3 to n = 1, how much energy will be emitted?
(a) 0, ± 1 (b) ± 1, ± 2 (a) 2.15 × 10–11 ergs (b) 2.389 × 10–12 ergs
(c) 0, ± 1, ± 2 (d) 0, ± 2 (c) 0.239 × 10–10 ergs (d) 0.1936 × 10–10 ergs
34. Pick out the isoelectronic structure from the following:
+ +
42. When atoms are bombarded with alpha particles, only, a
(i) C H3 (ii) H3 O few in million suffer deflection, others pass out undeflected.
- This is because
(iii) NH3 (iv) C H3
(a) the force of repulsion on the moving alpha particle is
(a) i and ii (b) iii and iv small
(c) ii, iii and iv (d) iii, iv and i
(b) the force of attraction between alpha particle and
35. The first emission line of hydrogen atomic spectrum in the
oppositely charged electrons is very small
Balmer series appears is (R = Rydberg constant)
(c) there is only one nucleus and large number of electrons
5 3
(a) R cm -1 (b) R cm -1 (d) the nucleus occupies much smaller volume compared to
36 4
the volume of the atom
7 9
(c) R cm -1 (d) R cm-1 43. The ionisation potential of a hydrogen atom is –13.6 eV. What
144 400 will be the energy of the atom corresponding to n = 2.
36. Identify the correct statement:
(a) – 3.4 eV (b) – 6.8 eV (c) – 1.7 eV (d) –2.7 eV
(a) s and p orbitals are degenerate
(b) sp and sp2 orbitals are degenerate 44. If the energy of a photon is given as : = 3.03 × 10–19 J then, the
(c) p and sp3 orbitals are degenerate wavelength (l) of the photon is :
(d) All sp3 orbitals are degenerate (a) 6.56 nm (b) 65.6 nm (c) 656 nm (d) 0.656 nm
Structure of Atom 31

45. In the photo-electron emission, the energy of the emitted 56. The mass of a photon with a wavelength equal to
electron is 1.54 × 10–8 cm is
(a) greater than the incident photon (a) 0.8268 × 10–34 kg (b) 1.2876 × 10–33 kg
(b) same as than of the incident photon
(c) 1.4285 × 10–32 kg (d) 1.8884 × 10–32 kg
(c) smaller than the incident photon
(d) proportional to the intensity of incident photon 57. When a metal surface is exposed to solar radiations
46. Uncertainty in position of a n electron (mass = 9.1 × 10–28 g) (a) The emitted electrons have energy less than a maximum
moving with a velocity of 3 × 104 cm/s accurate upto 0.001% value of energy depending upon frequency of incident
will be (use h/4p) in uncertainty expression where radiations
h = 6.626 ×10–27 erg-second). (b) The emitted electrons have energy less than maximum
(a) 1.93 cm (b) 3.84 cm (c) 5.76 cm (d) 7.68 cm value of energy depending upon intensity of incident
47. The configuration 1s 2 , 2s 2 2 p5 , 3s1 shows : radiation
(c) The emitted electrons have zero energy
(a) excited state of O -2 (d) The emitted electrons have energy equal to energy of
(b) excited state of neon atom photons of incident light
(c) excited state of fluorine atom 58. The wavelength of a moving electron having 4.55 × 10–25 J of
(d) ground state of fluorine atom kinetic energy is :
48. Positron is : (a) 7.27 × 10–7 metre (b) 72.7 × 10–7 metre
(a) electron with positive charge –9
(c) 7.27 × 10 metre (d) 72.7 × 10–9 metre
(b) a nucleus with one neutron and one proton 59. Which one of the following represents noble gas
(c) a nucleus with two protons configuration :
(d) a helium nucleus (a) 1s2, 2s2 2p6, 3s2 3p6 3d10, 4s2 4p6 4d10, 5s2 5p6 5d6, 6s2
49. The position of both, an electron and a helium atom is (b) 1s2, 2s2 2p6, 3s2 3p6 3d10, 4s2 4p6 4d10, 5s2 5p6 5d1, 6s2
known within 1.0 nm. Further the momentum of the electron (c) 1s2, 2s2 2p6, 3s2 3p6 3d10, 4s2 4p6 4d10, 5s2 5p6
is known within 5.0 × 10–26 kg ms–1 . The minimum (d) 1s2, 2s2 2p6, 3s2 3p6 3d10, 4s2 4f14, 5s2 5p6 5d1
uncertainty in the measurement of the momentum of the 60. Number of unpaired electrons in N2+ is
helium atom is (a) 2 (b) 0 (c) 1 (d) 3
(a) 50 kg ms–1 (b) 80 kg ms–1 61. The total number of electrons that can be accommodated in
–26
(c) 8.0 × 10 kg ms –1 (d) 5.0 × 10–26 kg ms–1 all orbitals having principal quantum number 2 and azimuthal
50. If electron, hydrogen, helium and neon nuclei are all moving
quantum number 1 is
with the velocity of light, then the wavelength associated
with these particles are in the order (a) 2 (b) 4 (c) 6 (d) 8
(a) Electron > hydrogen > helium > neon 62. An ion has 18 electrons in the outermost shell, it is
(b) Electron > helium > hydrogen > neon (a) Cu+ (b) Th4+ (c) Cs+ (d) K+
(c) Electron < hydrogen < helium < neon 63. For azimuthal quantum number l = 3, the maximum number of
(d) Neon < hydrogen < helium < electron electrons will be
51. The de Broglie wavelength of a tennis ball of mass 60 g moving
(a) 2 (b) 6 (c) 0 (d) 14
with a velocity of 10 metres per second is approximately
(a) 10–31 metres (b) 10–16 metres 64. If magnetic quantum number of a given atom represented by-
(c) 10–25 metres (d) 10–33 metres 3, then what will be its principal quantum number?
Planck’s constant, h = 6.63 × 10–34 Js (a) 2 (b) 3 (c) 4 (d) 5
52. Which of the following is related with both wave nature and 65. For which one of the following sets of four quantum numbers,
particle nature ? an electron will have the heighest energy?
(a) Interference (b) E = mc2 n l m s
(c) Diffraction (d) E = h n (a) 3 2 1 1/2
53. If the radius of first Bohr orbit be a 0, then the radius of the (b) 4 2 –1 1/2
third orbit would be (c) 4 1 0 –1/2
(a) 3 × a0 (b) 6 × a0 (d) 5 0 0 –1/2
(c) 9 × a0 (d) 1/9 × a0
66. Which of the following should be the possible sub-shells, for
54. The ratio of the radius of the first Bohr orbit for the electron
n+l=7?
orbiting the hydrogen nucleus to that of the electron orbiting
the deuterium nucleus (mass nearly twice that of the hydrogen (a) 7s, 6p, 5d, 4f (b) 4f, 5p, 6s, 4d
nucleus) is approximately (c) 7s, 6p, 5d, 6d (d) 4s, 5d, 6p, 7s
(a) 2 : 1 (b) 1 : 1 67. Wavelength associated with electron motion
(c) 1 : 2 (d) 4 : 1 (a) increases with increase in speed of electron
55. The de-Broglie wavelength of an electron in the ground state (b) remains same irrespective of speed of electron
of hydrogen atom is : [K.E. = 13.6 eV; 1eV = 1.602 ´ 10 -19 J] (c) decreases with increase of speed of e– (electron)
(a) 33.28 nm (b) 3.328 nm (c) 0.3328 nm(d) 0.0332 nm
(d) is zero.
EBD_7327
32 CHEMISTRY

68. Which of the following is not correct for electronic distribution 80. The electronic configuration of element with atomic number
in the ground state ? 24 is :
(a) 1s 2 , 2s 2 2 p 6 ,3s 2 3 p 6 3d 4 , 4 s 2
(a) Co [Ar]
(b) 1s 2 , 2s 2 2 p6 , 3s 2 3 p6 3d 10
(b) Ni [Ar]
(c) 1s 2 , 2s 2 2 p6 , 3s 2 3 p 6 3d 6

(c) Cu [Ar] (d) 1s 2 , 2s 2 2 p6 , 3s 2 3 p6 3d 5 4s1


81. Which of the following represents correct set of the four
(d) All of the above
quantum numbers for an electron in a 4d subshell ?
69. The electronic configuration of gadolinium (Atomic number
(a) 4, 2, 1, 0 (b) 4, 2, 1, – 1/2
64) is
(c) 4, 3, 2, + 1/2 (d) 4, 3, – 2, – 1/2
(a) [Xe] 4f 8 5d 0 6s2 (b) [Xe] 4f 3 5d 5 6s2
6 2 2
82. The de Broglie wavelength associated with a ball of mass
(c) [Xe] 4f 5d 6s (d) [Xe] 4f 7 5d 1 6s2 200 g and moving at a speed of 5 m/h, is of the order of
70. The following quantum numbers are possible for how many (h = 6.625 × 10–34 Js)
orbital(s) n = 3, l = 2, m = +2 ?
(a) 10–15 m (b) 10–20 m (c) 10–25 m (d) 10–30m
(a) 1 (b) 3 (c) 2 (d) 4 83. An electron has principal quantum number 3. The number of
71. The number of spherical nodes in 3p orbitals are its (i) subshells and (ii) orbitals would be respectively
(a) one (b) three (a) 3 and 5 (b) 3 and 7 (c) 3 and 9 (d) 2 and 5
(c) two (d) None of these 84. The number of nodal planes ‘d’ orbital has
72. The order of filling of electrons in the orbitals of an atom will be (a) 1 (b) 2 (c) 3 (d) 0
(a) 3d, 4s, 4p, 4d, 5s (b) 4s, 3d, 4p, 5s, 4d
85. An element M has an atomic mass 19 and atomic number 9, its
(c) 5s, 4p, 3d, 4d, 5s (d) 3d, 4p, 4s, 4d, 5s
ion is represented by
73. The orbitals are called degenerate when
(a) they have the same wave functions (a) M + (b) M 2 + (c) M - (d) M 2 -
(b) they have the same wave functions but different energies 86. The measurement of the electron position is associated with
(c) they have different wave functions but same energy an uncertainty in momentum, which is equal to 1×10–18 g cm s–1.
(d) they have the same energy The uncertainty in electron velocity is, (mass of an electron is
74. The uncertainties in the velocities of two particles, A and B 9 × 10– 28 g)
are 0.05 and 0.02 ms–1 respectively. The mass of B is five times (a) 1 × 109 cm s–1 (b) 1 × 106 cm s–1
5
(c) 1 × 10 cm s –1 (d) 1 × 1011 cm s–1
to that of the mass of A. What is the ratio of uncertainties
D xA 87. Maximum number of electrons in a subshell of an atom is
in their positions ? determined by the following:
D xB
(a) 2 l + 1 (b) 4 l – 2 (c) 2 n2 (d) 4 l + 2
(a) 2 (b) 0.25 (c) 4 (d) 1 88. Which of the following is not permissible arrangement of
75. The orientation of an atomic orbital is governed by electrons in an atom?
(a) Spin quantum number (a) n = 5, l = 3, m = 0, s = + 1/2
(b) Magnetic quantum number (b) n = 3, l = 2, m = – 3, s = – 1/2
(c) Principal quantum number (c) n = 3, l = 2, m = – 2, s = – 1/2
(d) Azimuthal quantum number (d) n = 4, l = 0, m = 0, s = – 1/2
76. The number of d-electrons retained in Fe2+ 89. The total number of atomic orbitals in fourth energy level of
(At. no. of Fe = 26) ion is an atom is :
(a) 4 (b) 5 (c) 6 (d) 3 (a) 8 (b) 16 (c) 32 (d) 4
th
77. The angular speed of the electron in n orbit of Bohr hydrogen 90. Calculate the wavelength (in nanometer) associated with a
atom is proton moving at 1.0 × 103 ms –1.
(a) Directly proportional to n (Mass of proton = 1.67 × 10–27 kg and h = 6.63 × 10–34 Js)
(b) Inversely proportional of n (a) 0.40 nm (b) 2.5 nm (c) 14.0 nm (d) 0.32 nm
(c) Inversely proportional to n 2 91. The frequency of light emitted for the transition n = 4 to n = 2
(d) Inversely proportional to n 3 of the He+ is equal to the transition in H atom corresponding
78. The energy ratio of a photon of wavelength 3000Å and 6000Å to which of the following ?
is : (a) n = 2 to n = 1 (b) n = 3 to n = 2
(a) 1 : 1 (b) 2 : 1 (c) 1 : 2 (d) 1 : 4 (c) n = 4 to n = 3 (d) n = 3 to n = 1
79. What is the correct orbital designation of an electron with the 92. The increasing order of the ionic radii of the given
quantum number, n = 4, l = 3, m = – 2, s = 1/2 ? isoelectronic species is :
(a) 3s (b) 4 f (c) 5p (d) 6s (a) Cl–, Ca2+ , K+, S2– (b) S2–, Cl–, Ca2+ , K+
2+ +
(c) Ca , K , Cl , S– 2– (d) K+, S2–, Ca2+, Cl–
Structure of Atom 33

93. The velocity of particle A is 0.1 ms–1 and that of particle B is 97. In which one of the following pairs the two species are both
0.05 ms–1. If the mass of particle B is five times that of particle isoelectronic and isotopic?
A, then the ratio of de-Broglie wavelengths associated with (Atomic numbers : Ca = 20, Ar = 18, K = 19, Mg = 12,
the particles A and B is Fe = 26, Na = 11)
(a) 2 : 5 (b) 3 : 4 (a) 40Ca2+ and 40Ar (b) 39K+ and 40K+
(c) 6 : 4 (d) 5 : 2 (c) 24Mg2+ and 25Mg (d) 23Na and 24Na+
94. In hydrogen atomic spectrum, a series limit is found at 12186.3 98. Which set of quantum numbers are not possible?
cm–1. Then it belong to
n l m s
(a) Lyman series (b) Balmer series
(a) 3 2 0 +1/2
(c) Paschen series (d) Brackett series
(b) 2 2 1 +1/2
95. The atom/ion that has the highest number of unpaired
electrons is (c) 1 0 0 –1/2
(a) Mg2+ (b) F (d) 3 2 –2 +1/2
(c) N (d) S2– 99. Two fast moving particles X and Y are associated with
96. For Balmer series in the spectrum of atomic hydrogen, the de Broglie wavelengths 1 nm and 4 nm respectively. If mass
æ 1 of X in nine times the mass of Y, the ratio of kinetic energies
1 ö of X and Y would be
wave number of each line is given by v = R H ç – ÷
2
è n1 n 22 ø (a) 3 : 1 (b) 9 : 1
where RH is a constant and n 1 and n2 are integers. Which of (c) 5 : 12 (d) 16 : 9
the following statement(s) is (are) correct? 100. The wavelength (in cm) of second line in the Lyman series
I. As wavelength decreases, the lines in the series of hydrogen atomic spectrum is (Rydberg constant
converge. = R cm–1)
II. The integer n 1 is equal to 2.
æ 8R ö æ 9 ö
III. The ionization energy of hydrogen can be calculated (a) ç ÷ (b) ç ÷
from the wave number of these lines. è 9 ø è 8R ø
IV. The line of longest wavelength corresponds to n 2 = 3. æ 4 ö æ 3R ö
(c) ç ÷ (d) ç ÷
(a) I, II and III (b) II, III and IV only è 3R ø è 4 ø
(c) I, II and IV (d) II and IV only

1. If the shortest wavelength of the spectral line of H-atom in 4. The size of isoelectronic species C4– , N3– and Mg2+ is
the Lyman series is X, then the longest wavelength of the affected by
(a) nuclear charge (Z)
line in Balmer series of Li 2+ is
(b) principle quantum number
x (c) interelectronic repulsion
(a) 9x (b)
9 (d) None of these
5. The ratio of magnetic moments of Fe(III) and Co(II) is
5x 4x
(c) (d) (a) 7 : 3 (b) 3 : 7
4 5
2. Given : The mass of electron is 9.11 × 10–31 kg (c) 7 : 3 (d) 3 : 7
Plank constant is 6.626 × 10–34 Js, 6. If the nitrogen atom has electronic configuration 1s7, it would
the uncertainty involved in the measurement of velocity have energy lower than that of the normal ground state
within a distance of 0.1 Å is configuration 1s22s22p3, because the electrons would be
(a) 5.79 × 107 ms–1 (b) 5.79 × 108 ms–1 closer to the nucleus. Yet 1s7 is not observed because it
5
(c) 5.79 × 10 ms –1 (d) 5.79 × 106 ms–1 violates.
3. The number of nodal planes in a px orbital is (a) Heisenberg uncertainty principle
(a) one (b) two (b) Hund's rule
(c) three (d) zero (c) Pauli exclusion principle
(d) Bohr postulate of stationary orbits
EBD_7327
34 CHEMISTRY

7. The orbital diagram in which the Aufbau principle is violated 15. Li and a proton are accelerated by the same potential, their
is :
de Broglie wavelengths lLi and l p have the ratio (assume
2s 2p
mLi = 9m p )
(a) ­¯ ­¯ ­
(a) 1 : 2 (b) 1 : 4
(b) ­ ­¯ ­ ­ (c) 1 : 1 (d) 1 : 3 3
16. Which two orbitals are both located between the axes of
(c) ­¯ ­ ­ ­ coordinate system, and not along the axes?
(a) d xy , d 2 (b) d yz , px
(d) ­¯ ­¯ ­ ­ z
(c) d , pz (d) none of these
8. If a proton and a -particle are accelerated through the same x2 - y 2
17. Uncertainty in the position of an electron (mass = 9.1 × 10–31 kg)
potential difference, the ratio of de-Broglie wavelengths l p moving with a velocity 300 ms–1, accurate upto 0.001% will be
and l a is (h = 6.63 × 10–34 Js)
(a) 1.92 × 10–2 m (b) 3.84 × 10–2 m
–2
(c) 19.2 × 10 m (d) 5.76 × 10–2 m
(a) 3 (b) 2 2
(c) 1 (d) 2 18. The number of radial nodes of 3s and 2p orbitals are
9. In hydrogen atom, energy of first excited state is –3.4 eV. Find respectively
out KE of the same orbit of Hydrogen atom (a) 2, 0 (b) 0, 2 (c) 1, 2 (d) 2, 1
(a) + 3.4 eV (b) + 6.8 eV (c) – 13.6 eV (d) + 13.6 eV 19. The energy of an electron in the first Bohr orbit of H atom is
10. The ions O2–, F–, Na+, Mg2+ and Al3+ are isoelectronic. Their –13.6 eV. The possible energy value(s) of the excited state(s)
ionic radii show for electrons in Bohr orbits of hydrogen is (are)
(a) a decrease from O2– to F– and then increase from Na + to (a) –3.4 eV (b) –4.2 eV
Al3+ (c) –6.8 eV (d) Both (a) and (c)
(b) a significant increase from O2– to Al3+ 20. Energy levels, A, B, C, of a certain atom correspond to
(c) a significant decrease from O2– to Al3+ increasing values of energy i.e., EA < EB < EC. If l1, l2, l3
(d) an increase from O2– to F– and then decrease from Na+ to are the wave lengths of radiations corresponding to the
Al3+ transition from C to B, B to A and C to A respectively,
11. Kinetic energy of an electron in hydrogen atom increases which of the following statements is correct ?
after transition from an orbit n1 to another orbit n2. Then C
l1
(a) n1 < n2
B
(b) n1 > n2
l2 l3
(c) it is not possible to predict
(d) none of these A
12. In Bohr series of lines of hydrogen spectrum, the third line
l1l 2
from the red end corresponds to which one of the following (a) l3 = l1 + l2 (b) l3 =
inter-orbit jumps of the electron for Bohr orbits in an atom of l1 + l 2
hydrogen (c) l1 + l2 + l3 = 0 (d) l32 = l12 + l 22
(a) 5 ® 2 (b) 4 ® 1 (c) 2 ® 5 (d) 3 ® 2 21. Number of protons, neutrons and electrons in the element
13. Consider the ground state of Cr atom (Z= 24). The number of 231
89Y is
electrons with the azimuthal quantum numbers, l = 1 and 2
are, respectively (a) 89, 231, 89 (b) 89, 89, 242
(a) 16 and 4 (b) 12 and 5 (c) 12 and 4 (d) 16 and 5 (c) 89, 142, 89 (d) 89, 71, 89
14. In a multi-electron atom, which of the following orbitals 22. Which one of the following is not the characteristic of Planck’s
described by the three quantum members will have the same quantum theory of radiation ?
energy in the absence of magnetic and electric fields? (a) The energy is not absorbed or emitted in whole number
or multiple of quantum
(A) n = 1, l = 0, m = 0 (B) n = 2, l = 0, m = 0
(b) Radiation is associated with energy
(C) n = 2, l = 1, m = 1 (D) n = 3, l = 2, m = 1 (c) Radiation energy is not emitted or absorbed continuously
(E) n = 3, l = 2, m = 0 but in the form of small packets called quanta
(a) (D) and (E) (b) (C) and (D) (d) This magnitude of energy associated with a quantum is
(c) (B) and (C) (d) (A) and (B) proportional to the frequency.
Structure of Atom 35

31. A 0.66 kg ball is moving with a speed of 100 m/s. The associated
23. The magnetic moment of a particular ion is 2 6 B.M. The
wavelength will be ( h = 6.6 ´ 10 -34 Js) :
ion is
(a) 1.0 ´ 10–32m (b) 6.6 ´ 10–32m
(a) Mn 2+ (b) Fe3+ –34
(c) 6.6 ´ 10 m (d) 1.0 ´ 10–35m
(c) Co 2+ (d) Co3+ 32. The energies E1 and E2 of two radiations are 25 eV and 50 eV,
24. If the de-Broglie wavelength of a particle of mass m is 100 respectively. The relation between their wavelengths i.e., l1
times its velocity, then its value in terms of its mass (m) and and l2 will be :
Planck’s constant (h) is (a) l1 = l2 (b) l1 = 2l2
1
1 m h 1 h m (c) l1 = 4l2 (d) l1 = l 2
(a) (b) 10 (c) (d) 10 2
10 h m 10 m h
33. If n = 6, the correct sequence for filling of electrons will be :
25. Which of the following graph correspond to one node (a) ns ® (n – 2) f ® (n – 1) d ® np
(b) ns ® (n – 1) d ® (n – 2) f ® np
(c) ns ® (n – 2) f ® np ® (n – 1) d
Y Y
(d) ns ® np ® (n – 1) d ® (n – 2) f
(a) (b) 34. According to the Bohr Theory, which of the following
transitions in the hydrogen atom will give rise to the least
a0 a0 energetic photon ?
(a) n = 6 to n = 1 (b) n = 5 to n = 4
(c) n = 6 to n = 5 (d) n = 5 to n = 3
Y Y 35. Which of the following sets of quantum numbers represents
the highest energy of an atom?
(c) (d)
(a) n = 3, l = 0, m = 0, s = +1/2 (b) n = 3, l = 1, m = 1, s = +1/2
a0 a0 (c) n = 3, l = 2, m = 1, s = +1/2 (d) n = 4, l = 0, m = 0, s = +1/2
26. The five d-orbitals are designated as d xy , d yz , d xz , d 36. Which one of the following constitutes a group of the
x2 - y2 isoelectronic species?
and d 2. Choose the correct statement.
z (a) C2– –
2 , O 2 , CO, NO (b) NO+ ,C2– –
2 , CN , N 2
(a) The shapes of the first three orbitals are similar but that
of the fourth and fifth orbitals are different (c) CN – , N 2 ,O2–
2 , C2
2– (d) N 2 , O2– , NO+ , CO
(b) The shapes of all five d-orbitals are similar 37. The ionization enthalpy of hydrogen atom is 1.312 × 106 J mol–1.
(c) The shapes of the first four orbitals are similar but that of The energy required to excite the electron in the atom from
the fifth orbital is different n = 1 to n = 2 is
(d) Ths shapes of all five d-orbitals are different (a) 8.51 × 105 J mol–1 (b) 6.56 × 105 J mol–1
27. The magnetic moment of Mx+ (atomic number of M = 25) is (c) 7.56 × 105 J mol–1 (d) 9.84 × 105 J mol–1
15 B.M. The number of unpaired electrons and the value 38. In an atom, an electron is moving with a speed of 600 m/s with
an accuracy of 0.005%. Certainity with which the position of
of x respectively are the electron can be located is ( h = 6.6 × 10–34 kg m2s–1, mass
(a) 4, 3 (b) 3, 4 of electron, em = 9.1 × 10–31 kg)
(c) 3, 2 (d) 5, 2 (a) 5.10 × 10 –3 m (b) 1.92 × 10 –3 m
28. The magnetic moments of Cu (Z = 29), Ti (Z = 22), and –3
(c) 3.84 × 10 m (d) 1.52 × 10 –4 m
Cr (Z = 24) are in the ratio of
39. The energy required to break one mole of Cl – Cl bonds in Cl 2
(a) 1: 5 : 4 (b) 4 : 5 :1 is 242 kJ mol–1. The longest wavelength of light capable of
breaking a single Cl – Cl bond is
(c) 1 :2:6 (d) 1 : 3 : 21 (c = 3 × 108 ms–1 and NA = 6.02 × 1023 mol–1).
29. If uncertainty in position and momentum are equal, then (a) 594 nm (b) 640 nm (c) 700 nm (d) 494 nm
uncertainty in velocity is : 40. Ionisation energy of He+ is 19.6 × 10–18 J atom–1. The energy
1 h h 1 h h of the first stationary state (n = 1) of Li2+ is
(a) (b) (c) (d)
2m p 2p m p p (a) 4.41 × 10–16 J atom–1 (b) –4.41 × 10–17 J atom–1
30. The energy absorbed by each molecule (A2) of a substance –15
(c) –2.2 × 10 J atom –1 (d) 8.82 × 10–17 J atom–1
is 4.4 × 10–19 J and bond energy per molecule is 4.0 × 10–19 J.
41. The electrons identified by quantum numbers n and l :
The kinetic energy of the molecule per atom will be:
(a) 2.2 × 10–19 J (b) 2.0 × 10–19 J (A) n = 4, l = 1 (B) n = 4, l = 0
(c) 4.0 × 10–20 J (d) 2.0 × 10–20 J (C) n = 3, l = 2 (D) n = 3, l = 1
EBD_7327
36 CHEMISTRY

can be placed in order of increasing energy as : DIRECTIONS for Qs. 47 to 49 : These are Assertion-Reason
(a) (C) < (D) < (B) < (A) (b) (D) < (B) < (C) < (A) type questions. Each of these question contains two statements:
Statement-1 (Assertion) and Statement-2 (Reason). Answer these
(c) (B) < (D) < (A) < (C) (d) (A) < (C) < (B) < (D)
questions from the following four options.
42. The kinetic energy of an electron in the second Bohr orbit of (a) Statement- 1 is True, Statement-2 is True, Statement-2 is a
a hydrogen atom is [a0 is Bohr radius] : correct explanation for Statement -1
h2 h2 (b) Statement -1 is True, Statement -2 is True ; Statement-2 is
h2 h2
(a) (b) (c) (d) NOT a correct explanation for Statement - 1
4p 2 ma02 16p 2 ma02 32p 2 ma02 64p2 ma02 (c) Statement - 1 is True, Statement- 2 is False
43. Given that the abundances of isotopes Fe, Fe and 57 Fe
54 56
(d) Statement -1 is False, Statement -2 is True
are 5%, 90% and 5%, respectively, the atomic mass of Fe is 47. Statement-1 : The position of an electron can be determined
(a) 55.85 (b) 55.95 (c) 55.75 (d) 56.05 exactly with the help of an electron microscope.
æ Z2 ö Statement-2 : The product of uncertainty in the measurement
44. Energy of an electron is given by E = – 2.178 × 10-18 J ç 2 ÷ . of its momentum and the uncertainty in the measurement of
èn ø
the position cannot be less than a finite limit.
Wavelength of light required to excite an electron in an
48. Statement-1 : The radius of the first orbit of hydrogen atom is
hydrogen atom from level n = 1 to n = 2 will be : 0.529Å.
(h = 6.62 × 10 –34 Js and c = 3.0 × 108 ms–1) Statement-2 : Radius of each circular orbit (rn) - 0.529Å
(a) 1.214 × 10–7 m (b) 2.816 × 10.–7 m (n2/Z), where n = 1, 2, 3 and Z = atomic number.
(c) 6.500 × 10–7 m (d) 8.500 × 10–7 m 49. Statement-1 : Nuclide 30 Al13 is less stable than 40 Ca20
45. A 600 W mercury lamp emits monochromatic radiation of Statement-2 : Nuclide having odd number of protons and
wavelength 331.3 nm. How many photons are emitted from neutrons are generally unstable.
the lamp per second? 50. Statement-1 : Angular momentum of an electron in any orbit
h = 6.62 × 10–34 Js velocity of light = 3 × 108 ms–1 n.h
is given by angular momentum = , where n is the principal
(a) 1.0 × 1019 (b) 1.0 × 1023 2p
(c) 1.0 × 1021 (d) 2.0 × 1023 quantum number.
46. The de-Broglie wavelength of a particle with mass 1 kg and Statement-2 : The principal quantum number, n, can have any
velocity 100 m/s is integral value.
(a) 6.6 × 10 –33 m (b) 6.6 × 10–36 m
+33
(c) 3.3 × 10 m (d) 3.3 × 10–36 m

Exemplar Questions The density of dots in a region represents the probability


1. Which of the following conclusions could not be derived density of finding electrons in the region.
from Rutherford's a-particle scattering experiment? On the basis of above diagram which of the following
(a) Most of the space in the atom is empty. statements is incorrect?
(b) The radius of the atom is about 10–10 m while that of (a) 1s and 2s orbitals are spherical in shape.
nucleus is 10–15 m. (b) The probability of finding the electron is maximum near
(c) Electrons move in a circular path of fixed energy called the nucleus.
orbits. (c) The probability of finding the electron at a given
(d) Electrons and the nucleus are held together by distance is equal in all directions.
electrostatic forces of attraction. (d) The probability density of electrons for 2s orbital
2. Which of the following options does not represent ground decreases uniformly as distance from the nucleus
state electronic configuration of an atom? increases.
(a) 1s22s22p63s23p63d84s2 4. Which of the following statement is not correct about the
(b) 1s22s22p63s23p63d94s2 characteristics of cathode rays?
(c) 1s22s22p63s23p63d104s1 (a) They start from the cathode and move towards the
(d) 1s22s22p63s23p63d54s1 anode.
3. The probability density plots of 1s and 2s orbitals are given (b) They travel in straight line in the absence of an external
in figure. electrical or magnetic field.
(c) Characteristics of cathode rays do not depend upon
the material of electrodes in cathode ray tube.
(d) Characteristics of cathode rays depend upon the nature
1s of gas present in the cathode ray tube.
2s
Structure of Atom 37

5. Which of the following statements about the electron is NEET/AIPMT (2013-2017) Questions
incorrect?
(a) It is a negatively charged particle. 17. According to law of photochemical equivalence the energy
(b) The mass of electron is equal to the mass of neutron. absorbed (in ergs/mole) is given as (h = 6.62 × 10–27 ergs, c
(c) It is a basic constituent of all the atoms. = 3 × 1010 cm s–1, NA = 6.02 × 1023 mol–1)
(d) It is a constituent of cathode rays. [NEET Kar. 2013]
6. Which of the following properties of atom could be explained 1.196 ´ 1016 1.196 ´ 108
correctly by Thomson model of atom? (a) (b)
(a) Overall neutrality of atom l l
(b) Spectra of hydrogen atom 2.859 ´ 105 2.859 ´ 1016
(c) Position of electrons, protons and neutrons in atom (c) (d)
l l
(d) Stability of atom
7. Two atoms are said to be isobars if æ Z2 ö
(a) they have same atomic number but different mass 18. Based on equation E = – 2.178 × 10-18 J çç 2 ÷÷ , certain
number èn ø
(b) they have same number of electrons but different conclusions are written. Which of them is not correct ?
number of neutrons [2013]
(c) they have same number of neutrons but different (a) Larger the value of n, the larger is the orbit radius.
number of electrons (b) Equation can be used to calculate the change in energy
(d) sum of the number of protons and neutrons is same when the electron changes orbit.
but the number of protons is different (c) For n = 1, the electron has a more negative energy than
8. The number of radial nodes for 3p orbital is ......... . it does for n = 6 which mean that the electron is more
(a) 3 (b) 4 loosely bound in the smallest allowed orbit.
(c) 2 (d) 1 (d) The negative sign in equation simply means that the
9. Number of angular nodes for 4d orbital is ........... . energy or electron bound to the nucleus is lower than
(a) 4 (b) 3 it would be if the electrons were at the infinite distance
(c) 2 (d) 1 from the nucleus.
10. Which of the following is responsible to rule out the existence 19. What is the maximum numbers of electrons that can be
of definite paths or trajectories of electrons? associated with the following set of quantum numbers?
(a) Pauli's exclusion principle n = 3, l = 1 and m = –1 [2013]
(b) Heisenberg's uncertainty principle (a) 6 (b) 4 (c) 2 (d) 10
(c) Hund's rule of maximum multiplicity 20. The value of Planck’s constant is 6.63 × 10–34 Js. The speed
(d) Aufbau principle of light is 3 × 1017 nm s–1.. Which value is closest to the
11. Total number of orbitals associated with third shell will wavelength in nanometer of a quantum of light with
be ............... . frequency of 6 × 1015 s–1? [2013]
(a) 2 (b) 4 (a) 25 (b) 50 (c) 75 (d) 10
(c) 9 (d) 3 21. What is the maximum number of orbitals that can be identified
12. Orbital angular momentum depends on ............ . with the following quantum numbers? [2014]
(a) l (b) n and l n = 3, l = 1, ml = 0
(c) n and m (d) m and s (a) 1 (b) 2 (c) 3 (d) 4
13. Chlorine exists in two isotopic forms Cl-37 and Cl-35, but its 22. Calculate the energy in joule corresponding to light of
atomic mass is 35.5. This indicates the ratio of Cl-37 and wavelength 45 nm :
Cl-35 is approximately (Planck’s constant h = 6.63 × 10–34 Js; speed of light
(a) 1 : 2 (b) 1 : 1 c = 3 × 108 ms–1) [2014]
(c) 1 : 3 (d) 3 : 1 (a) 6.67 × 1015 (b) 6.67 × 1011
14. The pair of ions having same electronic configuration (c) 4.42 × 10–15 (d) 4.42 × 10–18
is .............. . 23. Two electrons occupying the same orbital are distinguished by
(a) Cr3+, Fe3+ (b) Fe3+, Mn2+ (a) Principal quantum number [2016]
(c) Fe3+, Co3+ (d) Sc3+, Cr 3+ (b) Magnetic quantum number
15. For the electrons of oxygen atom, which of the following (c) Azimuthal quantum number
statements is correct? (d) Spin quantum number
(a) Zeff for an electron in a 2s orbital is the same as Zeff for 24. Which one is the wrong statement ? [2017]
an electron in a 2p orbital. (a) The uncertainty principle is DE ´ Dt ³ h / 4 p
(b) An electron in the 2s orbital has the same energy as an (b) Half filled and fully filled orbitals have greater stability
electron in the 2p orbital. due to greater exchange energy, greater symmetry and
(c) Zeff for an electron in 1s orbital is the same as Zeff for more balanced arrangement.
an electron in a 2s orbital. (c) The energy of 2s orbital is less than the energy of 2p
(d) The two electrons present in the 2s orbital have spin orbital in case of Hydrogen like atoms
quantum numbers ms but of opposite sign.
h
16. If travelling at same speeds, which of the following matter (d) de-Broglies's wavelength is given by l = ,
waves have the shortest wavelength? mn
(a) Electron (b) Alpha particle (He2+) where m = mass of the particle, n = group velocity of
(c) Neutron (d) Proton the particle
EBD_7327
38 CHEMISTRY

Hints & Solutions


EXERCISE - 1 12. (b) For p orbital with n = 6 and m = 0 indicates 6p z orbital.
It contains maximum of 2 electrons with spins opposite
1. (a) We know that ions which have the same number of
to each other.
electrons are called isoelectronic. We also know that both
13. (d) NaCl : No. of e– in Na+ = At. No. of Na–1
CO and CN– have 14 electrons, therefore these are
= 11 – 1 = 10
isoelectronic. No. e– in Cl– = At. No. of Cl + 1
2. (c) The molecule which contains same number of electrons = 17 + 1 = 18
are called isoelectronic. eg. N2 = CO = CN - = O +2 2 CsF : No. of e– in Cs+ = 55 – 1 = 54
No. of e– in F– = 9 + 1 = 10
= 14e
NaI : No. of e– in Na+ = 11 – 1 = 10
3. (b) The plot given in option (b) is the correct sketch of wave No. of e– in I– = 53 + 1 = 54
function (y ) for a 2s- electron as a function of its distance K2S : No. of e– in K+ = 19 – 1 = 18
from the nucleus (r). As y has no significate, so the plot No. of e– in S2– = 16 + 2 = 18
14. (c)
of y versus r also has no significance.
4. (c) Three 2p orbitals of one atom and three 2p orbitals e 0
15. (d) for (i) neutron = = 0
of another atom give rise to four (p 2p) and two (s m 1
2p) molecular orbitals. 2
Among 2px, 2py and 2pz orbitals of two atoms 2pz (ii) a-particle = = 0.5
4
combines with 2pz along z-aixs give rise to two s 2p
1
molecular orbitals, while 2px combines with 2px and (iii) proton = = 1
2py combines with 2py give rise to two p 2px and two p 1
2py molecular orbitals. 1
(iv) electron = = 1837
5. (d) Those species which contain same no. of electrons are 1 1837
called isoelectronic species. C2H4 has 16 electrons. The 16. (c) No. of neutrons = Mass number – Atomic number
no. of electrons in other given species are = W – N.
+ + 2
O 2 = 15e– ; CN– = 14e– ; N 2 = 13e– ; 17. (c) Energy of electron in 2nd orbit of Li+2 = -13.6 z
O2 = 16e– n2
So, O2 is isoelectronic with C2 H4. - 13.6 ´ (3) 2
6. (c) Cathode rays are made up of negatively charged = = –30.6 eV
particles (electrons) which are deflected by both the (2) 2
electric and magnetic fields. Energy required = 0 – (–30.6) = 30.6 eV
18. (b) E = mc2 (Einstien relation) ....... (1)
7. (d) Number of neutrons = Mass number – Atomic number
= 70 – 30 = 40.
E = hv ....... (2)
8. (b) e/m for He+ = 1/4 e/m for He2+ = 2/4
+ From (1) and (2),
e/m for D = 1/2 e/m for H+ = 1/1 mc2 = hv
\ Value of e/m is highest for H+.
9. (d) Size of H+ (proton) is very small, so its hydration energy mc2 =
hc æ cö
Þ
l çQv = ÷
is very large. è l ø
1 Þ mcl=h
Hydration energy µ
Size h
Þ l=
10. (d) Proton is the nucleus of H-atom (H-atom devoid of its mc
electron) h
11. (d) F– O2– Al3+ N3– Þ l= (Q mc = p)

p
No. of e s 10 10 10 10
Nuclear charge 9 8 13 7 h
Þ p=
(Number of protons) l
All the four given species are isoelectronic and size of 19. (b) · No. of electrons in N 3– , O 2– and Cr 3– are
isoelectronic species decreases with increase in nuclear respectively 10, 10 and 27 so these species are not
charge. Among the four concerned atoms, N has lowest isoelectronic with each other.
atomic number (nuclear charge), hence N3– ion will be · No. of neutrons in 14Si 30 , 15 P31 and 16 S32 are
largest in size. respectively 16, 16 and 16. So, 14S30, 15P31 and 16S32
are isotones.
Structure of Atom 39

· Atoms of the same element having same atomic nh


no. and different mass no. are called isotopes. So mvr =
40 40 2p
20Ca and 19K are not isotopes.
· Isobars are the atoms of different elements having For n = 5, we have
same mass no. and different atomic no. So, 8O16, 5h 2.5h
17 18 Angular momentum of electron = =
8O and 8O are isobars. 2p p
20. (b) Both He and Li+ contain 2 electrons each
therefore their spectrum will be similar. æ 1 1 ö
32. (c) D E for two energy levels = 21.79 ç - ÷ J/atom
21. (d) This statement is known as uncertainty principle which ç n2 n2 ÷
è 1 2 ø
was given by Heisenberg it is not a Bohr’s postulate.
22. (a) For hydrogen atom (n) = 1 (due to ground state) 33. (c) For every l , quantum number 'm' can take values from
Radius of hydrogen atom (r) = 0.53 Å. – l to + l . So for l = 2, m = – 2, – 1, 0, + 1, + 2
Atomic number of Li (Z) = 3.
+
n2 (1) 2 34. (c) (i) C H3 8 electrons
Radius of Li2+ ion = r1 ´ = 0.53 ´ = 0.17
Z 3
+
23. (d) Given : Radius of hydrogen atom = 0.530 Å, Number of (ii) H3 O 10 electrons
excited state (n) = 2 and atomic number of hydrogen atom
(Z) = 1. We know that the Bohr radius. (iii) NH3 10 electrons
n2 (2)2 (iv) CH3 10 electrons
(r ) = ´ Radius of atom = ´ 0.530
Z 1 So, (ii), (iii) and (iv) are isoelectronic species with 10
= 4 ´ 0.530 = 2.12 Å electrons each.
24. (a) He2 – contains 5 electrons. So, the molecular orbital
electronic configuration is é1 1ù
35. (a) D E = R ê 2 - 2 ú;
s 1s2, s* 1s2, s 2s1
25. (c) Principal quantum number (n) gives the information about ëê n1 n2 ûú
the size of electron cloud i.e., the approximate distance of First line in Balmer series results in the transition :
electron cloud from the nucleus. n2 = 3 to n1 = 2.
26. (d) The given statement is related to Aufbau principle. 36. (d) Those orbitals which are equal in energies are called
According to this principle the electrons first occupy the degenerate orbitals. All sp3 orbitals are degenerate as
orbital with lowest energy available to them and then these are equal in energies.
enter into higher energy orbitals only when the lower 37. (a) Rubidium is an alkali metal. It has one valence electron
energy orbitals are filled. in 5s orbital. So, principal quantum number, n = 5
-1312 azimuthal quantum number l = 0
27. (b) We know that En = kJ mol-1
n2 Note :
n = 4 (Fourth Bohr orbit)
orbital : s p q f
-1312
Given E4 = = –82 kJ mol–1 value of l : 0 1 2 3
42
28. (a) For s-electron, l = 0 magnetic quantum number,
h m = – l to l = 0
\ Orbital angular momentum = 0(0 + 1) =0
2p 1
spin quantum number, s = +
1 æ 1 1 ö 2
29. (c) = Rç - ÷
l 2 2
è n1 n2 ø 1
So, correct set of quantum number is 5, 0, 0, +
2
1 æ1 1 ö
= 1.097 ´ 107 ç - ÷ = 1.097 ´107 m -1 38. (c) Zeeman effect refers to splitting of lines of an emission
l è1 ¥ ø spectrum in magnetic field.
l = 91.15 ´ 10 -9 m » 91nm 39. (b) If we assume the atom to be hydrogen like, energy of
nth energy level
1 é1 1ù
30. (c) n = ´ IE ´ ê - ú E
h 2 2 En = 21 where E1 is energy of first energy level
ëê n1 n2 úû n
2.18 ´ 10-18 é1 1 ù E1 E1 -21.79 ´ 10 -12
= ´ê - ú 15 -1 E2 = - =- =
6.625 ´ 10 -34
ë1 16 û = 3.08 ´ 10 s 2 2 4 4
31. (b) Angular momentum of an electron in n th orbit is given = – 5.447 × 10–12 erg per atom.
by
EBD_7327
40 CHEMISTRY

40. (b) 41. (d)


42. (d) The nucleus occupies much smaller volume compared (iii) E = hn = hc is de-Broglie equation and it supports
l
to the volume of the atom. both wave nature and particle nature of electron.
-E a0
43. (a) Energy of an electron E = 0 53. (c) Radius of nth orbit rn = ´ n 2 ; \ r3 = 9a 0
n2 Z
For energy level (n = 2) 2
54. (b) rn = a 0 n , r1 = 0 ( ) for hydrogen
2 a 1
13.6 -13.6 Z 1
E=- 2
= = -3.4 eV.
( 2) 4 2
a (1) r1 1
44. (c) The energy of photon, r2 = 0 for deuterium \ = = 1:1
1 r2 1
hc
E = = 3.03 ´ 10-19 55. (b) For electron in the ground state,
l h h
mvr = Þ mv =
6.626 ´ 10 -34 ´ 3 ´ 108 2p 2pr
or l =
3.03 ´ 10 -19 Now, mv =
h
l
19.878
or l = ´ 10 -7 = 6.56 × 10–7m= 656 nm h h
3.03 So, = Þ l = 2pr
l 2pr
45. (c) K.E. of emitted electron
l = 2 ´ 3.14 ´ 0.53Å = 3.328Å
= hv - hv0 (i.e. smaller than hv ).
= 3.328 ´ 10 -10 m
46. (a) Given mass of an electron(m) = 9.1´10 -28 g; = 0.3328 ´ 10 -9 m = 0.3328 nm
Velocity of electron (v ) = 3 ´104 cm/s; h h
56. (c) We know that l = ; \m =
mu ul
Accuracy in velocity = 0.001% = 0.001 ; The velocity of photon (u) = 3 × 108 m sec–1
100 l = 1.54 ´10-8 cm = 1.54 ´10 -10 meter
Actual velocity of the electron
6.626 ´ 10-34 Js
0.001 \m =
( Dv ) = 3 ´ 10 4 ´ = 0.3 cm/s . 1.54 ´ 10 -10 m ´ 3 ´108 m sec -1
100
Planck’s constant (h) = 6.626×10–27erg-sec. = 1.4285 ´ 10 -32 kg
57. (a)
\ Uncertainty in the position of the electron h h
58. (a) l= =
,27 mv
h 6.626´10 ´7 2mE
( Dx) = <
,
4pmDv 4´ 22 ´(9.1´10 )´ 0.3
28
6.6 ´ 10 -34
l=
=1.93 cm 2 ´ 9.1 ´ 10 -31 ´ 4.55 ´ 10 -25
47. (b) Atomic number of the given element = 10
Electronic configuration = 1s2, 2s2p6 6.6 ´ 10-34
= -28
= 0.727 ´ 10 -6
1s22s22p6 is electronic configuration of Ne. 9 ´ 10
1s22s22p53s1 is excited oxidation state.
= 7.27 ´ 10-7 metres
48. (a) Positron is electron with positive charge, +1e0
59. (c) Noble gas must have 8 electrons in its outer most shell.
h Among the given configuration only option (c) has 8
49. (d) By Heisenberg uncertainty Principle Dx ´ Dp = (which
4p electron in its outermost orbital.
is constant) i.e. the configuration is 2, 8, 18, 18, 8. So option (c) is
As Dx for electron and helium atom is same thus the right answer.
momentum of electron and helium will also be same 60. (c) N(7) = 1s2 2s2 2p3
therefore the momentum of helium atom is equal to N 2 + = 1s 2 , 2 s 2 2 p1x
5 × 10–26 kg. m.s–1. Unpaired electrons = 1.
50. (a) l = h/ mv ; for the same velocity, l varies inversely with 61. (c) n = 2, l = 1 means 2p–orbital. Electrons that can be
the mass of the particle. accommodated = 6 as p sub-shell has 3 orbital and each
h 6.6 ´ 10 -34 orbital contains 2 electrons.
51. (d) l = = = 10 -33 m 62. (a) Cu+ = 29 – 1 = 28 e–
mv 60 ´ 10 -3 ´10
52. (d) (i) Interference and diffraction support the wave nature thus the electronic confingration of Cu+ is
2 2 6
of electron. s 2 3 p 6 3d 10
Cu+ (28) = 1s 2s 2 p 314 4244 3
(ii) E = mc2 supports the particle nature of electron.
18e-
Structure of Atom 41

63. (d) l = 3 means f-subshell. Maximum no. of electrons = 4l + 2 76. (c) Fe++ (26 – 2 = 24) = 1s2 2s2 2p6 3s2 3p6 4s0 3d 6 hence no.
= 4 × 3 + 2 = 14 of d electrons retained is 6. [Two 4s electron are removed]
64. (c) If m = – 3; l = 3 , V 1
[m ranges from -l to +l ] 77. (d) Angular speed is . Vn µ and rn µ n 2 .
r n
So n = 4 as nature of l ranges from \ Angular speed is inversely proportional to n
0 to (n – 1).
So option (c) is the answer. c
78. (b) E = hv = h
65. (b) The sub-shell are 3d, 4d, 4p and 4s, 4d has highest energy l
as n + l value is maximum for this.
1
66. (a) n + l = 7 So, E µ
7 + 0 = 7s ; 6 + 1 = 6p ; 5 + 2 = 5d ; 4 + 3 = 4f l
l 1 : l 2 = 3000 A : 6000 A = 1 : 2
67. (c) l = h ; \ l µ 1 hence answer (c). Hence, E1 : E2 = 2 : 1
mv v
79. (b) n = 4 represents 4th orbit
68. (d) According to Hund’s rule electron pairing in P, d and f
l = 3 represents f subshell
orbitals cannot occur until each orbital of a given subshell
m = – 2 represents orientation of f-orbital
contains one electron each or is singly occupied.
s = 1/2 represents direction of spin of electron.
69. (d) We know that atomic number of gadolinium is 64.
\ The orbital is 4f.
Therefore the electronic configuration of gadolinium is
80. (d) Electronic configuration of element with atomic no 24 is
[Xe] 4f 7 5d1 6s2. Because the half filled and fully filled
orbitals are more stable. 1s 2 , 2s 2 , 2 p 6 ,3s 2 , 3 p 6 , 4s1 ,3d 5
70. (a) Quantum number n = 3, l = 2, m = +2 represent an orbital Exactly half filled d-orbital has extra stability.
with 81. (b) For 4d orbitals, n = 4, l = 2

s =±
1
2 (3d xy or 3d 2 2
x -y ) é For s orbital l = 0 ù
ê For p orbital l = 1 ú
which is possible only for one electron. ê ú
71. (a) No. of radial nodes in 3p-orbital = (n – l – 1) ëê For d orbital l = 2 ûú
[for p ortbital l = 1] m = –2, –1, 0, +1 or + 2
=3–1–1=1 1 1
s = + and -
72. (b) The sub-shell with lowest value of (n + l) is filled up first. 2 2
When two or more sub-shells have same (n + l) value the 1
subshell with lowest value of 'n' is filled up first therefore Thus choice b having n = 4, l = 2, d = 1 and s = is
2
the correct order is correct.
orbital 4s 3d 4p 5s 4d
´ -34
n + l 4+ 0 3 + 2 4 +1 5+ 0 4+ 2 82. (c) l = h = 6.63 10 = 2.4 ´ 10 -30 m
value = 4 =5 =5 =5 =6 mv 5
0.200 ´
73. (d) The orbitals which have same energy are called 3600
degenerate orbitals eg. px , p y and pz . 83. (c)
74. (a) Using the relation, 84. (b) Number of nodal planes in d orbitals is 2.
85. (c) Atomic number 9 is for F and ion is F–.
h 86. (a) Dp = mDv
Dx. Dv = [ Heisenberg' uncertainty principle]
4pm Substituting the given values of Dx and m, we get
1×10–18 g cm s–1 = 9×10–28 g × Dv
h
or Dx = 1´ 10 -18
4pm.Dv or Dv =
9 ´ 10-28
h = 1.1 × 109 cm s–1 ; 1×109 cm s–1
Thus, Dx A = … (i)
4p ´ 0.05 ´ m i.e. option (a) is correct.
87. (d) The number of sub shell is (2 l + 1). The maximum number
h
DxB = … (ii) of electrons in the sub shell is 2 (2 l + 1) = (4 l + 2)
4p ´ 0.02 ´ 5m 88. (b) m = – l to +l, through zero thus for l = 2, values of m will
Dividing (i) by (ii), we get be – 2, –1, 0, + 1, + 2.
Dx A 0.02 ´ 5 10 Therefore for l = 2, m cannot have the value –3.
= = or 2 89. (b) Total no. of atomic orbitals in a shell = n2.
DxB 0.05 5
Given n = 4; Hence number of atomic orbitals in 4th shell
75. (b) Magnetic quantum no. represents the orientation of will be 16.
atomic orbitals in an atom. For example px, py & pz have
orientation along X-axis, Y-axis & Z-axis
EBD_7327
42 CHEMISTRY

96. (c) (1) Beyond a certain wavelength the line spectrum


h 6.63 ´ 10-34
90. (a) l= = becomes band spectrum.
mv 1.67 ´ 10-27 ´ 1 ´103 (2) For Balmer series n1 = 2
= 3.97 × 10–10 meter = 0.397 nanometer » 0.40 nm (3) For calculation of longest wavelength use nearest
91. (a) For He+ value of n2. Hence for longest wavelength in
1 æ 1 1 ö Balmer series of hydrogen spectrum,
v= = RH Z 2 ç 2 - 2 ÷ n1 = 2 & n2 = 3.
l è2 4 ø
97. (b) 39K+ and 40K+ contains same number of electrons so
æ 1 1 ö æ 1 1 ö they are isoelectronic. They have same atomic numbers
= RH (2)2 ç 2 - 2 ÷ = RH ç 2 - 2 ÷
è2 4 ø è (1) (2) ø but different mass numbers so they are also isotopes.
For H 98. (b) Value of l = 0 ..........(n – 1)
l cannot be equal to n.
æ ö
1 ç 1 1÷ h
v = = RH - 99. (d) de Broglie wavelength l =
l ç n12 n2 ÷ mv
è 2ø
l1 m 2 v 2 1 1 v 2
= ; = ´
For same frequency, l 2 m1v1 4 9 v1

æ 1 1 ö æ ö v2 9
1 1 =
2 ÷ = RH ç 2 - 2 ÷
RH ç 2
- v1 4
è (1) (2 ) ø çè n1 n2 ÷ø
v1 4
=
1 1 1 1 v2 9
\ - = -
n12 n22 12 22 1
KE = mv 2
\ n1 = 1 & n2 = 2 2
92. (c) Among isoelectronic species ionic radii increases as 2
KE1 m1 v12 9 æ 4 ö 16
the charge increases. = ´ = ´ç ÷ =
KE2 m2 v 22 1 è 9 ø 9
Order of ionic radii Ca2+ < K+ < Cl– < S2–
The number of electrons remains the same but nuclear
100. (a) 1 é1 1 ù
charge increases with increase in the atomic number u= = Rê 2 – 2ú
l n
ë 1 n 2û
causing decrease in size.
93. (d) Given, vA = 0.1 ms–1 and vB = 0.05 ms–1 also, mB = 5mA For second line in lyman series
h n2 = 3
de-Broglie wavelength, l =
mv
l
1 é 1 ù 1 é1 1 ù 8R
\ A =
h / mA v A m Bv B
= \ l = R ê 2 - 2 ú = R ê1 – 9 ú =
l B h / mB v B m A v A ë1 3 û ë û 9
5mA ´ 0.05
= = 5 ´ 0.5 = 2.5 = 5 / 2
mA ´ 0.1 EXERCISE - 2
\ lA : lB = 5 : 2 1 æ ö
94. (c) Series limit is the last line of the series, i.e. n2 = ¥. 1. (d) = RHz2 ç 1 - 1 ÷
l çn 2 n 2 ÷
é 1 è 1 2 ø
1 1 ù é 1 1 ù R
\u = =Rê 2 - 2ú=Rê 2 - 2ú= 2
l ëê n1 n 2 úû ëê n1 ¥ úû n1 To calculate shorst wavelength take n 2 = ¥ and longest
1 09677.76 wavelength take nearest value of n 2 .
Q u = 12186.3 =
n12 For H-atom,
109677.76
Þ n12 = = 9 Þ n1 = 3 1
12186.3 n 2 = ¥ , Z = 1, n1 = 1
\ The line belongs to Paschen series. l shortest
95. (c) The electronic configuration of the given species is
Mg2+ : 1s2 2s2 2p6 1
\ = RH (Lyman series)
F : 1s22s2 2p5 x
N : 1s2 2s2 2p3
1
S2– : 1s2 2s2 2p6 3s2 3p6
For l for Li 2+ , Z = 3, n1 = 2, n 2 = 3 (Balmer
Ti 3+ : 1s2 2s2 2p6 3s2 3p6 3d1 longest
Therefore, N has the highest number of unpaired series)
electrons.
Structure of Atom 43

9. (a) Total energy (En) = K.E + P.E


1 1 æ 1 1 ö 5
= × 3 2 çç 2 - 2 ÷÷ = in first excited state
l longest x è2 3 ø 4x
1 é ze2 ù 1 ze 2 ze 2
4x E = mv 2 + ê - ú =+ -
\ l longest = 2 ëê r ûú 2 r r
5
h 1 ze 2
2. (d) We know that Dx . Dp ³ -3.4 eV = -
4p 2 r
h h 1 ze 2
Dx . mDv > Dv > \ K.E = = +3.4 eV
4p 4pDxm 2 r
6.626 ´ 10 -34 10. (c) Amongst isoelectronic species, ionic radii of anion is
Dv = more than that of cations. Further size of anion increase
4p ´ 0.1´ 10-10 ´ 9.11 ´10 -31
with increase in –ve change and size of cation decrease
66 with increase in +ve charge. Hence ionic radii decreases
= ´ 10 7 = 5.79 ´ 10 6 m / sec
4p ´ 9 from O2– to Al+++.
3. (a) px orbital being dumbell shaped, have number of nodal 11. (b) K.E. of an electron in a Bohr orbit is equal to the
planes = 1 magnitude of the total energy but of opposite sign. So
it varies inversly to the square of principal quantum
z
number.
12. (a) The lines falling in the visible region comprise Balmer
– + x
series. Hence the third line from red would be n1 = 2,
n2 = 5 i.e, 5 ® 2.
y 13. (b) Electronic configuration of Cr atom ( Z = 24)
2px
= 1s 2 , 2 s 2 2 p 6 ,3s 2 3 p6 3d 5 , 4s1
4. (a) Because number of protons (nuclear charge) is different when l = 1, p-subshell,
while number of electrons is same in isoelectronic Numbers of electrons = 12
species.
when l = 2, d-subshell,
5. (c) Fe (III) = [Ar] 3d 5 unpaired electrons = 5; Numbers of electrons = 5
14. (a) The energy of an orbital is given by (n + l) in (D) and (E).
Magnetic moment = 5(5 + 2) ; (n + l) value is (3 + 2) = 5 hence they will have same
Ratio = energy, since there n values are also same.
7: 3
(h = 6.63 × 10–34 Js)
Co(II) = [Ar] 3d 7 unpaired electrons = 3;
h h
15. (d) lp = ; l Li =
Magnetic moment = 3(3 + 2) 2eVm p 2 ´ 3eVmLi

Ratio = 7 : 3 h
6. (c) As per Pauli exclusion principle "no two electrons in =
2 ´ 3eV ´ 9m p
the same atom can have all the four quantum numbers
equal or an orbital cannot contain more than two
electrons and it can accommodate two electrons only l Li3+ 2eVm p 1
Hence, = =
when their directions of spin are opposite". lp 2 ´ 3eV ´ 9m p 3 3
7. (b) According to Aufbau principle, the orbital of lower
energy (2s) should be fully filled before the filling of 16. (d)
orbital of higher energy starts. 17. (a) Given m = 9.1 × 10–31kg
h = 6.6 × 10–34Js
h
8. (b) lp = ;
2eVmp 300 ´ .001
Dv = = 0.003ms–1
100
h h
l = = From Heisenberg's uncertainity principle
He 2 + 2 ´ 2eV ´ 4m p
2 ´ 2eVm 2+
He
6.62 ´ 10 -34
Dx = = 1.92 ´ 10-2 m
lp 4 ´ 3.14 ´ 0.003 ´ 9.1´ 10 -31
\ = 2 2
l
He 2 +
EBD_7327
44 CHEMISTRY

18. (a) Number of radial nodes = (n - l - 1) h


For 3s: n = 3, l = 0 (Number of radial node = 2) 29. (a) We know Dp.Dx ³
4p
For 2p: n =2, l = 1 (Number of radial node = 0) since Dp = Dx (given)
19. (a) The energy of an electron on Bohr orbits of hydrogen
h
atoms is given by the expression \ Dp.Dp =
4p
Constant
En = - or mDv mDv. =
h
[\ Dp= mDv]
n2 4p
Where n takes only integral values. For the first Bohr 2 h
orbit, n = 1 and it is given that E1 = –13.6 eV or ( Dv ) =
4 pm 2
13.6eV
Hence En = - h 1 h
n2 or Dv = =
4pm2 2m p
Now out of given values of energy, only – 3.4 eV can be Thus option (a) is the correct option.
obtained by substituting n =2 in the above expression. 30. (d) K.E per atom
20. (b) From the given data, we have
(EC – EB) + (EB – EA) = (EC – EA)
=
( 4.4 ×10 ) – ( 4.0 ×10 )
–19 –19

æ hc hc ö hc é 1 1 1 ù 2
or ç + ÷= ê or + = ú
è l1 l 2 ø l3 ë l1 l 2 l3 û
0.4×10 –19
= = 2.0 ´ 10 –20
2
l1l 2 é l1 + l 2 1 ù
or l3 = êQ = ú
l1 + l 2 ë l .l l 3û h 6.6 ´10-34
1 2 31. (d) l = = = 1´ 10-35 m
mv 0.66 ´100
21. (c) In Y89231 number of protons and electrons is 89 and number
32. (b) Given E1 = 25eV E2 = 50 eV
of neutrons = A – Z = 231 – 89 = 142 hc hc E1 l 2
22. (a) Energy is always absorbed or emitted in whole number or E1 = E = \ =
l1 2 l 2 E2 l1
multiples of quantum.
23. (d) Magnetic moment l 25 1
\ 2 = = \ l1 = 2l 2
l1 50 2
= 2 6 = 24 B.M. = n(n + 2) B.M.
33. (a) ns ® (n - 2) f ® (n - 1)d ® np [n = 6]
Hence, n = 4 (unpaired electrons)
34. (c) Energy of photon obtained from the transition n = 6 to
Co3+ – [Ar] 3d6, 4 unpaired electrons.
n = 5 will have least energy.
24. (b) Let wavelength of particle be x
æ 1 1 ö
x DE = 13.6Z 2 -
So, velocity = çè n 2 n 2 ÷ø
100 1 2
35. (c) (a) n = 3, l = 0 means 3s-orbital and n + l = 3
h h ´ 100
l= ; x= (b) n = 3, l = 1 means 3p-orbital n + l = 4
mv m´x
(c) n = 3, l = 2 means 3d-orbital n + l = 5
h h (d) n = 4, l = 0 means 4s-orbital n + l = 4
x 2 = 100 or x = 10
m m Increasing order of energy among these orbitals is
25. (b) 3s < 3p < 4s < 3d
26. (c) First four orbitals contain four lobes, while fifth orbital \ 3d has highest energy..
consists of only two lobes. The lobes of dxy orbital lie 36. (b) Species having same number of electrons are
between x and y axis. Similarly in the case of dyz and dzx. isoelectronic. On calculating the number of electrons
their lobes lie between yz and zx axis respectively. Four in each given species, we get.
lobes of d 2 2 orbital are lying along x and y axis while CN– (6 + 7 + 1 = 14); N2 (7 + 7 = 14);
x -y O22–(8 + 8 +2 = 18) ; C22– (6 + 6 + 2 = 14);
two lobes of d
z2
orbital are lying along z-axis. O2– (8 + 8 + 1 = 17) ; NO+ (7 + 8 – 1 = 14)
CO (6 + 8 = 14) ; NO (7 + 8 = 15)
27. (b) n(n + 2) = 15, n = 3 (n = number of unpaired From the above calculation we find that all the species
electrons) listed in choice (b) have 14 electrons each so it is the
correct answer.
Therefore, x = 4 Q M 4 + = [Ar]3d 3
28. (a)
Structure of Atom 45

37. (d) (DE), The energy required to excite an electron in atom


nh
of hydrogen from n = 1 to n = 2 is DE (difference in So, v =
energy E2 and E1) 2pmr
Values of E2 and E1 are, 1 2
KE = mv
-1.312 ´ 106 ´ (1) 2 2
E2 = = –3.28 × 105 J mol–1
(2) 2
1 æ nh ö2
E1 = – 1.312 × 106 J mol–1 So, KE = mç ÷
\ DE = E2 – E1 = [–3.28 × 105]– [–1.312 × 106 ] J mol–1 2 è 2pmr ø
= (–3.28 × 105 + 1.312 × 106) J mol–1
= 9.84 × 105 J mol–1 ao ´ n2
Since, r =
Thus the correct answer is (d) z
38. (b) According to Heisenberg uncertainty principle.
So, for 2nd Bohr orbit
h h
Dx.mDv = Dx =
4p 4pmDv ao ´ 22
r= = 4ao
600 ´ 0.005 1
Here Dv = = 0.03
100
1 æ 22 h 2 ö
6.6 ´ 10 -34 KE = m ç ÷
So, Dx = 2 çè 4p m ´ (4ao ) ÷ø
2 2 2
4 ´ 3.14 ´ 9.1´ 10-31 ´ 0.03
= 1.92 × 10–3 meter
39. (d) Energy required to break one mole of Cl – Cl bonds in h2
KE =
Cl2 32p 2 mao2
242 ´ 103 hc 43. (b) Average atomic mass of Fe
= =
6.023 ´ 10 23 l
(54 ´ 5) + (56 ´ 90) + (57 ´ 5)
= = 55.95
6.626 ´ 10-34 ´ 3 ´ 108 100
=
l
æ 1 1 ö hc
6.626 ´ 10 -34 ´ 3 ´108 ´ 6.023 ´10 23 44. (a) DE = 2.178 ´ 10-18 ç 2 - 2 ÷ =
\l= è1 2 ø l
242 ´ 108
= 0.4947 × 10–6 m = 494.7 nm 3 hc
Þ 2.178 ´ 10-18 ´ =
Z 2 4 l
40. (b) I. E = ´ 13.6 eV ...(i)
n2
6.62 ´ 10-34 ´ 3 ´ 108
I1 Z12 n22 =
l
or I = 2 ´ 2 ...(ii)
2 n1 Z 2
Given I1 = – 19.6 × 10–18 , Z1 = 2, 6.62 ´ 10-34 3 ´ 108 ´ 4
l=
n1 = 1 , Z2 = 3 and n2 = 1 2.178 ´ 10-18 ´3
Substituting these values in equation (ii).
= 1.214 × 10–7m
19.6 ´10 -18 4 1 45. (c) Energy emitted by the bulb = 600 W = 600 Js–1
– = ´
I2 1 9 (1W = 1 Js–1)
9 l = 331.3 × 10–9 m
or I 2 = -19.6 ´ 10 -18 ´
4 hc 6.62 ´ 10 -34 ´ 3 ´ 108
= – 4.41 × 10–17 J/atom Energy of one photon = hv = =
l 331.3 ´ 10 -9
41. (b) (A) 4 p (B) 4 s
(C) 3 d (D) 3 p = 0.059 ´ 10–17 » 0.06 ´ 10-17 J
According to Bohr Bury's (n + l) rule, increasing order No. of photon emitted from the lamp per second
of energy will be (D) < (B) < (C) < (A).
600
Note : If the two orbitals have same value of (n + l) = = 1.0 ´ 1021
then the orbital with lower value of n will be filled first. 0.06 ´ 10 -17
42. (c) As per Bohr’s postulate,
h 6.6 ´ 10 –34
46. (b) de Broglie wavelength l = =
nh mv 1´ 100
mvr =
2p = 6.6 × 10–36m
EBD_7327
46 CHEMISTRY

47. (d) The statement-1 is false but the statement-2 is true exact over the atom. This model was able to explain the overall
position and exact momentum of an electron can never neutrality of the atom.
be determined according to Heisenberg’s uncertainty 7. (d) Isobars have the same mass number (i.e., sum of
principle. Even not with the help of electron microscope protons and neutrons) but different atomic number (i.e.,
because when electron beam of electron microscope number of protons) e.g., 26Fe58 and 27Ni58 are isobars.
strikes the target electron of atom, the impact causes the 8. (d) For hydrogen atom (1s) = number of radial nodes
change in velocity and position of electron . =n–l–1
Thus the product of uncertainty in position and Number of radial nodes for 3p orbital = 3 – 1 – 1 = 1
momentum is 9. (c) Number of angular nodes = l
h l = 2 for d-orbital
Dx.Dp ³ » 0.57 ergs sec/gram
4p \ Number of angular nodes = 2
48. (a) Both statement-1 and statement-2 are true and Statement- 10. (b) The important implications of Heisenberg uncertainty
2 is the correct explanation of Statement-1. principle is that it rules out existence of definite paths
or trajectories of electrons and other similar particles.
n2 h2 n2 11. (c) Total number of orbitals associated with n th shell = n2
Radius, rn = = ´ 0.529Å.rn
4pe 2 mZ Z \ Total number of orbitals associated with third shell
For first orbit of H-atom = (3)2 = 9
n=1
h
12. (a) Orbital angular momentum, mvr = l ( l + 1)
2
(1) 2p
r1 = ´ 0.529Å = 0.529Å
1 Hence, it depends only on 'l', l can have values ranging
49. (a) It is observed that a nucleus which is made up of even from 0 to (n – 1).
number of nucleons (No. of n & p) is more stable than 13. (c) The fractional atomic mass (35.5) of chlorine is due to
nuclie which consist of odd number of nucleons. If the fact that in ordinary chlorine atom, Cl-37 and Cl-35
number of neutron or proton is equal to some numbers are present in the ratio of 1 : 3.
i.e., 2, 8, 20, 50, 82, or 126 (which are called magic numbers), \ Average atomic mass of Cl
then these posses extra stability.
50. (b) Both Statement-1 and Statement-2 are correct. Statement- 3 ´ 35 + 1´ 37
= = 35.5 amu
2 is not the correct explanation of Statement-1. 4
14. (b) 5 1 Cr3+ = [Ar]3d3
24Cr = [Ar]3d , 4s
EXERCISE - 3 6 2 Fe3+ = [Ar]3d5
26Fe = [Ar]3d , 4s
5 2 Mn2+ = [Ar]3d5
Exemplar Questions 25Mn = [Ar]3d , 4s
7 2 Co3+ = [Ar]3d6
1. (c) The concept of electrons move in a circular path of 27Co = [Ar]3d , 4s
1 2 Sc3+ = [Ar]
fixed energy called orbits was given by Bohr and not 21Sc = [Ar]3d , 4s
derived from Rutherford's scattering experiment. Thus, Fe and Mn 2+
3+ have the same electronic
2. (b) The correct configuration for copper (z = 29) should be configuration.
1s22s22p63s23p63d104s1. Due to extra stability of fully 15. (d) For the two electrons of 2s orbital, the value of ms is
filled orbital of d-subshell, the last electron enter into 1 1
d-orbital instead of s-orbital. between + and -
2 2
3. (d) The probability density of electrons in 2s orbital first
16. (b) From de-Broglie equation wavelength,
increases then decreases and after that it increases
again as distance increases from nucleus. h
l=
4. (d) The characteristics of cathode rays do not depend mv
upon the material of electrodes and the nature of the
gas present in the cathode ray tube. 1
For same speed of different particles, l µ
5. (b) The mass of electron is very small as compared to the m
mass of the neutron. As h is constant, greater the mass of matter waves,
Mass of electron = 9.1 × 10–31 kg lesser is wavelength and vice-versa. Among these
Mass of neutron = 1.67 × 10–27 kg
matter waves, alpha particle (He2+ ) has higher mass,
6. (a) J. J. Thomson, in 1898, proposed plum pudding model
therefore, shortest wavelength.
of atom. An important feature of this model is that the
mass of the atom is assumed to be uniformly distributed
Structure of Atom 47

NEET/AIPMT (2013-2017) Questions 20. (b) c = nl

hc c 3 ´1017
17. (b) E = ´ NA l= = = 50 nm
l n 6 ´ 1015
21. (a) Given: n = 3, l = 1, m = 0
6.62 ´ 10-27 ´ 3 ´ 1010 ´ 6.02 ´ 10 23 Hence orbital is 3p
=
l 0 +1
–1
8
1.19 ´ 10
= ergs mol–1
l
hence the number of orbital identified by m = 0 can be
18. (c) Energy of an electron at infinite distance from the
nucleus is zero. As an electron approaches the nucleus, one only.
the electron attraction increases and hence the energy hc 6.63 ´ 10 - 34 ´ 3 ´ 108
of electron decreases and thus becomes negative. Thus 22. (d) E = =
l 45 ´ 10 - 9
as the value of n decreases, i.e. lower the orbit is, more – 18
= 4.42 × 10 J
negative is the energy of the electron in it.
23. (d) Two electrons occupying the same orbital should have
19. (c) n = 3 ® 3rd shell
l = 1 ® p sub shell. opposite spins i.e. they differ in spin quantum number.
m = – 1 is possible for two electrons present in an 24. (c) For hydrogen like atoms energy of 2s-orbital and 2p-
orbital. orbital is equal.
EBD_7327
48 CHEMISTRY

Classification of

3 Elements and
Periodicity in Properties
DEVELOPMENT OF PERIODIC TABLE Notation for IUPAC Nomenclature of Elements
The elements have been classified into groups for a systematic Digit Name Abbreviation
study of their properties. Various attempts have been made by 0 nil n
scientists from the early 1800’s. The first classification was made 1 un u
by Dobereiner who formulated ‘Triads’. It was followed by 2 bi b
Newland’s ‘Law of octaves’. The next development that came was 3 tri t
Mendeleev’s periodic table which classified elements on the basis 4 quad q
of their atomic masses. 5 pent p
6 hex h
Moseley showed that atomic number is a more fundamental
7 sept s
property of an element than its atomic mass. The Mendeleev’s
8 oct o
periodic law was then modified to a new law called Modern Periodic 9 enn e
law, according to which ‘The physical and chemical properties of
the elements are periodic functions of their atomic numbers’. ELECTRONIC CONFIGURATION OF ELEMENTS AND
Long form of periodic table is based upon the Modern Periodic PERIODIC TABLE
law. This is also known as Bohr’s table as it is based on Bohr’s An element’s location in the periodic table reflects the quantum
scheme for the arrangement of various electrons around the numbers of the last orbital filled.
nucleus. Electronic Configuration in Periods
The horizontal rows of the periodic table are called ‘Periods’ while (i) The period indicates the value of n for the outermost or
the vertical columns are called ‘Groups’. There are 7 periods and valence shell
18 groups in the periodic table. (ii) The number of elements in each period is twice the number of
atomic orbitals available in the energy level that is being
Merits of Long Form of Periodic Table
filled.
(i) Positions of Isotopes and Isobars - Modern periodic table is (iii) There are 2 elements in 1st period; 8 in the 2nd; 8 in the 3rd;
based on atomic numbers. Therefore, various isotopes of the 18 in the 4th; 18 in the 5th; 32 in 6th and 7th period is
same element will occupy the same position in the periodic incomplete.
table. Isobars have to be placed at different positions. Groupwise Electronic Configuration
(ii) The positions of actinoids and lanthanoids is more clear now Elements in same vertical column or group have similar valence
because these have been placed in group 3 and due to paucity shell electronic configurations, the same number of
of space, these are written at the bottom of the periodic table. e–1s in outer orbitals and similar properties.
For ex: all the group 1 elements have ns1 valence shell electronic
NOMENCLATURE OF ELEMENTS WITH ATOMIC configuration.
NUMBER > 100
s-, p-, d- AND f- BLOCK ELEMENTS
A systematic nomenclature has been derived to directly name the
Elements can be classified into four blocks: s-block, p-block, d-
element from its atomic number using numerical roots for 0 and block and f-block depending upon the type of atomic orbitals that
numbers 1-9. The roots are put together in order of digits which being filled with electrons.
make up the atomic number and ‘ium’ is added at the end.
The s-block Elements
Ex : Name of element with atomic number 101 is Unnilunium, 102 is
(i) General electronic configuration is ns1–2
unnilbium, 103 is unniltrium, etc. and their symbols are Unu, Unb, (ii) Group 1 and Group 2 elements are s-block elements because
Unt. they have ns1 and ns2 outermost electronic configuration.
Classification of Elements and Periodicity in Properties 49

(iii) They are all reactive metals with low ionisation energy. They Metalloids
lose the outermost e–1s readily to form +1 ion (grp 1) or +2 The elements which lie on the borderline between metals and non-
ion (grp 2). metals show properties that are characteristic of both metals and
Their compounds are predominently ionic (except Li and Bi). non-metals. They are called semi-metals or metalloids.
(iv) Group 1 elements are known as alkali metals because they PERIODIC TRENDS IN PROPERTIES OF ELEMENTS
react with water to form alkali. Group 2 elements are known Atomic Radius
as alkaline earth metals because their oxides react with water It is defined as half the distance between the nuclei of two bonded
to form alkali and these are found in the soil or earth. The atoms. It refers to both covalent and metallic radius depending on
total number of s-block elements are 14. whether the element is metal or non-metal.
The p-block Elements Atomic radii decreases across a period because e–1s are being
(i) General electronic configuration is ns2 np1– 6 added into same valence shell so that the effective nuclear charge
(ii) They comprise of elements from group 13 to 18. increases as the atomic number increases resulting in increased
(iii) Group 16 elements are called chalcogens while group 17 attraction of e–1s to the nucleus.
elements are called halogens. In a group, atomic radius increases. This is because down the
(iv) Group 18 elements are the noble gases due to completely group, principal quantum number (n) increases and e– is being
filled valence shell. As a result, they are less reactive, added into new shell. As a result valence e–1s are farther from the
(v) The non-metallic character increases as we move from left to nucleus. Thus, nuclear attraction decreases and therefore size
right across a period. Down the group metallic character increases.
increases. Ionic Radius
(vi) The p-block elements together with s-block elements are The removal of an e– from an atom results in the formation of
called Representative elements. cation whereas gain of an e– leads to an anion.
The d-block Elements Ionic radii of elements exhibit the same trend as atomic radii.
(i) General electronic configuration is (n – 1)d1–10 ns0-2 A cation is smaller than its parent atom because it has fewer e–1s
(ii) They comprise of group 3 to 12. They are all metals. while its nuclear charge remains the same.
(iii) They mostly form coloured ions, exhibit variable oxidation The size of anion is larger than parent atom because addition of
states paramagnetism and are used as catalysts. one or more e –1 s results in increased repulsion among
(iv) They form a bridge between chemically active metals of e–1s and a decrease in effective nuclear charge.
s-block and less active metals of group 13 and 14 and thus Isoelectronic species have different radii due to their different
are called ‘Transition Elements’. nuclear charges. Cation with greater positive charge has smaller
(v) Zn, Cd and Hg though are d-block elements but do not known radius due to greater effective nuclear charge. Anion with greater
as transition elements because in these elements negative charge will have larger radius because the net repulsion
d-orbitals are fully filled. of the e–1s will outweigh the nuclear charge and the ion expands
The f-block Elements in size.
(i) General electronic configuration is (n – 2)f 1–14 (n – 1) d0–1 Ionization Enthalpy (IE)
ns 2 It is the amount of energy required to convert gaseous neutral
(ii) They comprise of the two rows of elements at the bottom of atom into cation, i.e. X( g ) ¾¾ ® X+ ( g ) + e-
periodic Table, called the Lanthanoids and Actinoids. Ionization energy is always positive because energy is always
(iii) These two series of elements are called Inner transition required to remove e–1s from an atom.
elements. IE3 > IE2 > IE1 This is because it is more difficult to remove an e–
(iv) They are all metals. The chemistry of early actinoids is more from a positively charged species than from a neutral atom. Down
complicated than corresponding lanthanoids due to larger the group, atomic size increases and IE decreases. While across a
number of oxidation state possible for actinoid elements period, atomic size decreases and IE increases.
(v) The elements after uranium are called Transuranium elements. Factors governing the Ionization energy
Metals, Non-Metals and Metalloids (i) Nuclear charge: IE increases with increases in nuclear charge.
The elements can be divided into Metals and Non-metals. (ii) Atomic size: IE decreases as atomic radius decreases.
Metals (iii) Penetrating effect of e–1s: IE increases as penetration effect
(i) They appear on the left hand side of periodic table. of e–1s increases. Within the same shell, penetration effect
(ii) They are usually solids at room temperature (except Hg which decreases in the order : s > p > d > f. Thus, IE to knock out
is a liquid at room temperature) s – e– will be higher than p – e– of the same shell.
(iii) They have high m.pts and b.pts, are good conductors of (iv) Shielding or screening effect of inner shell e –1 s: As
heat and electricity and are malleable and ductile. shielding or screening effect of inner e–1s. increases, IE
Non-metals decreases.
(i) They are located at the top right hand side of the periodic (v) Effect of exactly half-filled or completely filled orbitals: More
table. stable the electronic configuration, greater is the IE. This is
(ii) They are usually solids or gases at room temperature with because of extra stability associated with exactly half-filled
low m.pts and b.pts. or completely filled orbitals due to which more energy is
(iii) They are poor conductors of heat and electricity. required to remove the e–. This is the reason why IE of N is
(iv) They are brittle and are neither malleable nor ductile. more than that of O.
EBD_7327
50 CHEMISTRY

(vi) Noble gases, being stable with completely filled orbital have SUMMARY OF TRENDS IN PERIODIC PROPERTIES
the highest IE in their respective periods. OF ELEMENTS.
Electron Gain Enthalpy (EGE)
It is defined as the energy released when a neutral isolated gaseous
atom accepts an extra e– to form gaseous negative ion, i.e., anion,
i.e. X( g ) + e - ¾¾® X- (g)
After the addition of one e–, the atom becomes negatively charged
and 2nd e– is to be added to a negatively charged ion. But the
addition of 2nd e– is opposed by electrostatic repulsion and hence
energy has to be supplied for addition of 2nd e–. Thus, 2nd electron
gain enthalpy of an element is positive.
Factors on which EGE depends
(i) Atomic size: As size increases EGE becomes less negative.
PERIODIC TRENDS AND CHEMICAL REACTIVITY
(ii) Nuclear charge: As nuclear charge increases, EGE becomes
Chemical reactivity is highest at the two extremes of a period and
more negative. is lowest in the centre.
(iii) Electronic configuration: Elements having exactly half-filled Nature of Oxides
or completely filled orbitals are very stable and have large If difference of the two electronegativities (XO–XA) is 2.3 or more
positive electron gain enthalpy because they do not accept then oxide will be basic in nature. Similarly if value of XO–XA is
additional e– easily. slightly lower than 2.3 then oxide will be amphoteric and if value of
Variation of EGE in periodic table XO–XA is highly lower than 2.3 then oxide will be of acidic nature.
(i) Down the group, atomic size increases, EGE becomes less Nature of Hydroxides
According to Gallis, if electronegativity of A in a hydroxide (AOH)
negative. Across a period, atomic size decreases, nuclear
is more than 1.7 then it will be acidic in nature whereas it will be
charge increases and EGE becomes more negative. basic in nature if electronegativity is less than 1.7
(ii) Halogens have very high negative EGE because they attain Note : Compounds formed from two nonmetals are called binary
stable noble gas electronic configuration by accepting an e–. compounds. Name of more electronegative element is written at
(iii) Noble gases have large positive EGE because the e– has to the end and ‘ide’ is suffixed to it. The name of less electronegative
enter the next higher shell leading to a very unstable electronic element is written before the name of more electronegative element
configuration. of the formula.
(iv) EGE of O or F is less negative than the succeeding element S Periodicity of Valence or Oxidation State
or Cl. This is because when an e– is added to O or F, the O.S. of an element in a particular compound is defined as the
added e– goes to smaller n = 2 quantum level and suffers charge acquired by its atom on the basis of electronegativity of
other atoms in the molecule. The valence of representative
significant repulsion from other e–1s in this level. For n = 3 elements is usually equal to no. of e–1s in outermost shell.
level (S or Cl), added e– occupies a larger region of space and Variation of oxidation state in periodic table
e– – e– repulsion is much less. Across a period, no. of valence e–1s increases from 1 to 8. The
Electronegativity valence of elements first increases from 1 to 4 and then decreases
It is the tendency of an atom of the element to attract the shared to zero.
Down the group, no. of valence e–1s remain the same, and therefore,
pair of e–1s towards itself in a covalent bond. It is represented by
all elements in a group exhibit the same valence.
X. Noble gases are zerovalent, i.e., their valence is zero because they
The electronegativity of any given element is not constant but are chemically inert.
depends on the following factors: ANOMALOUS PROPERTIES OF SECOND PERIOD
(i) State of hybridization: sp-hybridized carbon is more ELEMENTS
electronegative than sp2 hybridized which in turn is more The first element of each of the groups 1 (Li) and 2(Be) and groups
electronegative than sp3 hybridized carbon. 13-17 (B to F) differs in many respects from other members of its
(iii) O.S. of the element: As O.S. of the element increases, group. Moreover, the behaviour of Li and Be is more similar with
electronegativity increases. the 2nd element of following group i.e. Mg and Al. This sort of
(iv) Nature of substituents attached to the atom: For ex: C-atom similarity is referred to as diagonal relationship in periodic
properties.
in CF3I is more electronegative than in CH3I.
The anomalous behaviour of these elements is attributed to their
Variation of electronegativity in periodic table (i) small size (ii) large charge / radius ratio (iii) high electronegativity
(i) Down the group, atomic radius increases, electronegativity (iv) non-availability of orbitals due to which they cannot expand
decreases their covalence beyond 4.
Across a period, atomic radius and nuclear charge increases, For example : Because of smaller size and higher electronegativity
electronegativity increases. first member of p-block elements displays greater ability to form
(ii) F is the most electronegative element and caesium is the pp-pp multiple bonds to itself (C = C, C º C, N º N) and to other 2nd
period elements (C = O, C = N, C º N, N = O) compared to
least electronegative element.
subsequent members of same group.
Classification of Elements and Periodicity in Properties 51

CONCEPT MAP
EBD_7327
52 CHEMISTRY

1. Which group of periodic table contains no metal: 14. The statement that is not true for the long form of the periodic
(a) IA (b) IIIA table is
(c) VIIA (d) VIII (a) it reflects the sequence of filling electrons in the order
2. Which of the following is the atomic number of metal? of sub-energy levels s, p, d and f.
(a) 32 (b) 34 (c) 36 (d) 38 (b) it helps to predict the stable valence states of the
3. Which one of these is basic ? elements
(a) SiO2 (b) SO2 (c) it reflects trends in physical and chemical properties of
(c) CO2 (d) Na2O
the elements
4. Most acidic oxide is :
(d) it helps to predict the relative ionicity of the bond
(a) Na2O (b) ZnO
(c) MgO (d) P2O5 between any two elements.
5. Which of the following metals shows allotropy? 15. Among the following elements, the one having the highest
(a) Ca (b) Pb ionisation energy is.
(c) Sn (d) K (a) [Ar]3d10, 4s2 4p3 (b) [Ne]3s2 3p1
2
(c) [Ne]3s 3p 3 (d) [Ne]3s2 3p2
6. The electronic configuration of an element is
16. Polarisation power of a cation increases when
1s 2 2s 2 2 p 6 3s 2 3 p3 . What is the atomic number of the (a) size of the cation increases
element, which is just below the above element in the periodic (b) charge of the cation increases
table? (c) charge of the cation decreases
(a) 33 (b) 34 (d) it has no relation with its charge or size
(c) 36 (d) 49 17. Which one of the following is not a transition metal ?
7. An atom has electronic configuration 1s2 2s2 2p6 3s2 3p6
(a) Mn (b) Cr
3d3 4s2, you will place it in which group?
(c) Cu (d) Cd
(a) Fifth (b) Fifteenth
(c) Second (d) Third 18. Which is chemically most active non-metal ?
8. Which one of the following is an amphoteric oxide ? (a) S (b) O
(a) Na2O (b) SO2 (c) F (d) N
(c) B2O3 (d) ZnO 19. Which of the following is non-metallic ?
9. The screening effect of ‘d’ electrons is (a) B (b) Be
(a) Much less than s- electrons (c) Mg (d) Al
(b) Much more than s- electrons 20. The only non-metal which is liquid at ordinary temperature is
(c) Equal to s- electrons (a) Hg (b) Br2
(d) Equal to p- electrons (c) NH3 (d) None of the above
10. The statement that is not correct for the periodic classification 21. Amphoteric-oxide combinations are in
of element is (a) ZnO, K2O, SO3 (b) (b) ZnO, P2O5, Cl2O7
(a) the properties of elements are the periodic functions of (c) SnO2, Al2O3, ZnO (d) PbO2, SnO2, SO3
their atomic numbers. 22. The correct order of ionization energies is
(b) non-metallic elements are lesser in number than metallic (a) Zn < Cd < Hg (b) Hg < Cd < Zn
elements. (c) Ar > Ne > He (d) Cs < Rb < Na
(c) the first ionisation energies of elements along a period
23. Which one of the following has largest size?
do not vary in a regular manner with increase in atomic
(a) Al (b) Al3+
number. +
(d) for transition elements the d-subshells are filled with (c) Al (d) Al2+
electrons monotonically with increase in atomic number. 24. The ionisation potential order for which set is correct?
11. The elements in which 4f orbitals are progressively filled up (a) Cs < Li < K (b) Cs > Li > B
are called as (c) Li > K > Cs (d) B > Li > K
(a) Actinoids (b) Transition elements 25. Which one has least ionisation potential?
(c) Lanthanoids (d) Halogens (a) Ne (b) N
12. Who developed long form of the periodic table? (c) O (d) F
(a) Lothar Meyer (b) Neils Bohr 26. Which one of the following is smallest in size ?
(c) Mendeleev (d) Moseley
13. An element X occurs in short period having configuration
(a) N 3- (b) O2-
ns2 np1. The formula and nature of its oxide is (c) Na + (d) F-
(a) XO3, basic (b) XO3 acidic
(c) X2O3, amphoteric (d) X2O3 basic
Classification of Elements and Periodicity in Properties 53

27. The ionization energy of nitrogen atom is more than that of 42. The correct order of ionization energy for carbon, nitrogen
oxygen atom because of and oxygen atoms is:
(a) greater attraction of electrons by the nucleus. (a) C > N > O (b) C > N < O
(b) smaller size of nitrogen atom. (c) C < N > O (d) C < N < O
(c) more penetrating effect 43. Which of the following pairs of atomic numbers represents
(d) due to half filled p orbital elements belonging to the same group?
28. Sequence of acidic character is (a) 11 and 20 (b) 12 and 30
(a) N2O5 > SO2 > CO > CO2 (b) N2O5 > SO2 > CO2 > CO (c) 13 and 31 (d) 14 and 33
(c) SO2 > CO2 > CO > N2O5 (d) SO2 > N2O5 > CO > CO2 44. The correct order according to size is
29. Which is a metalloid ? (a) O > O– > O2– (b) O– > O2– > O
(a) Manganese (b) Phosphorus 2–
(c) O > O > O – (d) O > O2– > O–
(c) Oxygen (d) Arsenic 45. Which of the following element has maximum, first ionisation
+ 2+ 3+ 4+ potential ?
30. Na , Mg , Al and Si ions are isoelectronic. The value
(a) V (b) Ti (c) Cr (d) Mn
of ionic radii of these ions would be in the order : 46. The outer electronic configuration of transition elements is
(a) +
Na > Mg 2+
> Al 3+
> Si 4+ (a) (n – 1) s2 nd1–5
(b) (n + 1) s2 nd1–5
(b) Na + < Mg 2+ < Al3+ < Si 4+ (c) (n–1) s2 p6 (n–1) d1–10, ns0–2
(d) ns2 (n + 1) d1–10
(c) Na + > Mg 2+ > Al3+ < Si 4+ 47. If 19 is the atomic number of an element, then this element
(d) Na + < Mg 2+ > Al3+ > Si 4+ will be
(a) a metal with + 3 oxidation state
31. Maximum ionisation potential is of : (b) a metal with + 1 oxidation state
(a) Ca (b) Na (c) Be (d) Mg (c) an inert gas
32. Correct order of first IP among following elements Be, B, C, (d) a metal with – 3 oxidation state
N, O is 48. Highest energy will be absorbed to eject out the electron in
(a) B < Be < C < O < N (b) B < Be < C < N < O the configuration
(c) Be < B < C < N < O (d) Be < B < C < O < N (a) 1s2 2s2 2p1 (b) 1s2 2s2 2p3
33. Which one of the following ions has the highest value of 2
(c) 1s 2s 2p 2 2 (d) 1s2 2s2 2p4
ionic radius ? 49. In which of the following process highest energy is
(a) O2– (b) B3+ (c) Li+ (d) F– absorbed ?
34. An element having electronic configuration 1s2, 2s2 2p6, 3s2
(a) Cu ® Cu + (b) Br ® Br -
3p1 will form
(a) neutral oxide (b) acidic oxide (c) I ® I - (d) Li ® Li +
(c) basic oxide (d) amphoteric oxide 50. Which of the following gaseous atoms has highest value of
35. The first ionisation potential in electron volts of nitrogen IE ?
and oxygen atoms are respectively given by (a) P (b) Si
(a) 14.6, 13.6 (b) 13.6, 14.6 (c) Mg (d) Al
(c) 13.6, 13.6 (d) 14.6, 14.6 51. Which ionisation potential (IP) in the following equations
36. The correct order of radii is involves the greatest amount of energy ?
(a) N < Be < B (b) F - < O 2 - < N 3- (a) Na ® Na + + e - (b) K + ® K 2+ + e -
(c) N < Li < K (d) Fe 3+ < Fe 2 + < Fe 4+ (c) C 2 + ® C 3+ + e- (d) Ca + ® Ca 2 + + e -
37. First ionization potential will be maximum for 52. Arrange S, P, As in order of increasing ionisation energy
(a) uranium (b) hydrogen (a) S < P < As (b) P < S < As
(c) lithium (d) iron (c) As < S < P (d) As < P < S
38. Chloride ion and potassium ion are isoelectronic. Then 53. Among the following options, the sequence of increasing
(a) their sizes are same first ionisation potential will be
(b) Cl– ion is bigger than K+ ion (a) B < C < N (b) B > C > N
(c) K+ ion is relatively bigger (c) C < B < N (d) N > C > B
(d) their sizes depend on either cation or anion 54. As per the modern periodic law, the physical and chemical
39. Ionic radii of properties of elements are periodic functions of their
(a) Atomic volume
(a) Ti 4+ < Mn 2+ (b) 35
Cl - < 37
Cl - (b) Electronic configuration
(c) Atomic weight
(c) K + > Cl -1 (d) P 3+ > P 5+
(d) Atomic size
40. When an electron is removed from an atom, its energy 55. Eka-aluminium and EKa-silicon are known as
(a) increases (b) decreases (a) Gallium and Germanium
(c) remains the same (d) none of these (b) Aluminium and Silicon
41. Which of the following has minimum melting point? (c) Iron and Sulphur
(a) CsF (b) HCl (c) HF (d) LiF (d) Neutron and Magnesium
EBD_7327
54 CHEMISTRY

56. The correct order of reactivity of halogens is 66. Fluorine, chlorine, bromine and iodine are placed in the same
(a) F > Cl > Br > I (b) F < Cl > Br < I group 17 of the periodic table because :
(c) F < Cl < Br < I (d) F < Cl < Br > I (a) they are nonmetals
57. Which one of the following represents the electronic (b) they are electronegative
configuration of the most electropositive element ? (c) their atoms are generally univalent
(a) [He] 2s1 (b) [Xe] 6s1 (d) they have 7 electrons in the outer-most shell of their
(c) [He] 2s 2 (d) [Xe] 6s2 atom
58. A group 16 element exists in the monoatomic state in the 67. Which is the correct order of electronegativity ?
metallic state. It also exists in two crystalline forms. The (a) F > N < O > C (b) F > N > O > C
metal is (c) F > N > O < C (d) F < N < O =C
(a) S (b) Po (c) Se (d) Te 68. The electron affinity for the inert gases is
59. Electron affinity is positive when : (a) zero (b) high
(c) negative (d) positive
(a) O changes into O–
69. Which of the following species has the highest electron
(b) O– changes into O2–
affinity ?
(c) O changes into O+ (a) F (b) O (c) O– (d) Na+
(d) electron affinity is always negative 70. An atom with high electronegativity has
60. Electron affinity is maximum for (a) large size
(a) Cl (b) F (c) Br (d) I (b) high ionisation potential
61. Pauling’s electronegativity values for elements are useful in (c) low electron affinity
predicting (d) low ionisation potential
(a) polarity of the molecules 71. The largest size of the ion is :
(b) position in the E.M.F. series (a) Cl– (b) Ca++ (c) K+ (d) S– –
(c) coordination number 72. Which of the following is the most electronegative?
(d) dipole moments. (a) F (b) He (c) Ne (d) Na
62. Which of the following is most electronegative? 73. The outermost electronic configuration of the most
(a) Lead (b) Silicon electronegative element is
(c) Carbon (d) Tin (a) ns2 np3 (b) ns2 np4 (c) ns2 np5 (d) ns2 np6
63. Variable valency is generally exhibited by 74. Which of the following sequence correctly represents the
(a) representative elements (b) transition elements decreasing acidic nature of oxides ?
(c) non-metallic elements (d) metallic elements (a) Li2O > BeO > B2O3 > CO2 > N2O3
64. On going from right to left in a period in the periodic table, (b) N2O3 > CO2 > B2O3 > BeO > Li2O
the electronegativity of the elements (c) CO2 > N2O3 > B2O3 > BeO > Li2O
(a) increases (d) B2O3 > CO2 > N2O3 > Li2O > BeO
(b) decreases 75. Which transition involves maximum amount of energy?
(c) remains unchanged (a) M - (g) ¾
¾® M (g ) + e
(d) decreases first then increases
65. Which one of the following has the highest (b) M - (g) ¾
¾® M + (g ) + 2e
electronegativity?
(a) Br (b) Cl (c) P (d) Si (c) M + (g ) ¾
¾® M 2+ (g ) + e
(d) M 2 + (g ) ¾
¾® M 3+ (g ) + e

1. Which of the following can not be isoelectronic? (c) ionization energy of X – with sign reversed
(a) two different cations (b) two different anions (d) none of these
(c) cation and anion (d) two different atoms 5. Which group is called buffer group of the periodic table ?
2. The species with a radius less than that of Ne is (a) I (b) VII
(a) Mg2+ (b) F– (c) O2– (d) K+ (c) VIII (d) Zero
3. The correct order of acidic strength : 6. The pair of elements having approximately equal ionisation
potential is
(a) Cl 2 O 7 > SO 2 > P4 O10 (b) K 2O > CaO > MgO (a) Al, Ga (b) Al, Si
(c) Al, Mg (d) Al, B
(c) CO 2 > N 2 O 5 > SO 3 (d) Na 2 O > MgO > Al 2 O 3
7. Which electronic configuration of an element has abnormally
4. Electron affinity of X would be equal to high difference between second and third ionization energy?
(a) electron affinity of X – (a) 1s2, 2s2, 2p6, 3s1 (b) 1s2, 2s2, 2p6, 3s1 3p1
(b) ionization energy of X 2 2 6 2
(c) 1s , 2s , 2p , 3s 3p2 (d) 1s2, 2s2, 2p6, 3s2
Classification of Elements and Periodicity in Properties 55

8. Which of the following order is wrong? 15. Consider the following changes
(a) NH3 < PH3 < AsH3 – Acidic
A ® A+ + e- : E1 and A+ ® A2 + + e- : E2
(b) Li < Be < B < C – First IP
(c) Al2O3 < MgO < Na2O < K2O – Basic The energy required to pull out the two electrons are E1 and
E2 respectively. The correct relationship between two
(d) Li+ < Na+ < K+ < Cs+ – Ionic radius
energies would be
9. For which of the following processes, enthalpy change is
(a) E1 < E2 (b) E1 = E2
positive (c) E1 > E2 (d) E1 ³ E2
(a) F(g) + e - ® F(-g ) (b) Cl (g) + e - ® Cl (-g) 16. The incorrect statement among the following is
(a) The first ionization potential of Al is less than the first
ionization potential of Mg
(c) O (g) + 2e - ® O 2(g-) (d) H (g) + e - ® H (-g )
(b) The second ionization potential of Mg is greater than
10. Arrange the elements with the following electronic the second ionization potential of Na
configurations in increasing order of electron affinity (c) The first ionization potential of Na is less than the first
ionization potential of Mg
(i) 1s 2 s 2 2 p5 (ii) 1s 2 2s 2 2 p 4 (d) The third ionization potential of Mg is greater than the
third ionization potential of Al.
(iii) 1s 2 2s 2 2 p6 3s 2 3 p4 (iv) 1s 2 2s 2 2 p6 3s 2 3 p5
17. Successive addition of electronic shells in case of elements
(a) (ii) < (iii) < (i) < (iv) (b) (iii) < (ii) < (iv) < (i ) of 17th group causes a increase in
(a) electronegativity
(c) (iii) < (ii) < (i) < (iv) (d) (ii) < (iii) < (iv) < (i ) (b) ionization energy
11. Among Al2O3, SiO2, P2O3 and SO2 the correct order of acid (c) ease of formation of unipositive ion
strength is (d) oxidizing power
(a) Al2O3 < SiO2< SO2 < P2O3 18. The second ionization potential of an element M is the energy
(b) SiO2< SO2 < Al2O3 < P2O3 required to
(a) remove one mole of electrons from one mole of gaseous
(c) SO2< P2O3 < SiO2 < Al2O3
cations of the element
(d) Al2O3 < SiO2< P2O3 < SO2 (b) remove one mole of electrons from one mole of gaseous
12. The formation of the oxide ion O (2g-) requires first an anions
(c) remove one mole of electrons from one mole of
exothermic and then an endothermic step as shown below : monovalent gaseous cations of the element
(d) remove 2 moles of electrons from one mole of gaseous
O(g) + e- = O - (g) DHº = -142 kJmol-1 atoms
19. Which of the following ions has the most negative value of
O- (g) + e - = O2 - (g) DHº = 844 kJmol-1 enthalpy of interaction with water?
This is because
(a) O– ion will tend to resist the addition of another (a) NH +4 (b) OH - (c) H + (d) F–
electron 20. s-electrons of the valence shell of some elements show
(b) Oxygen has high electron affinity reluctance in bond formation. Such elements are --- and
(c) Oxygen is more elecronegative belong to --- :
(d) O– ion has comparatively larger size than oxygen atom (a) lighter , s-block (b) heavier , d-block
(c) heavier , f-block (d) heavier , p-block
13. In which of the following arrangements, the order is NOT
21. Which of the following cations acts as an oxidizing agent?
according to the property indicated against it? (a) Ga3+ (b) In3+ (c) Tl1+ (d) Tl3+
(a) Li < Na < K < Rb : 22. The first ionization potential of Na, Mg, Al and Si are in the
Increasing metallic radius order
(b) I < Br < F < Cl : (a) Na < Mg > Al < Si (b) Na > Mg > Al > Si
Increasing electron gain enthalpy (c) Na < Mg < Al > Si (d) Na > Mg > Al < Si
(with negative sign) 23. Correct order of polarising power is
(c) B < C < N < O (a) Cs + < K+ < Mg2+ < Al3+ (b) K+ < Cs+ < Mg2+ < Al3+
Increasing first ionization enthalpy (c) Cs+ < K+ < Al3+ < Mg2+ (d) K+ < Cs+ < Al3+ < Mg2+
24. In general, the ionization potentials of elements decreases
(d) Al3+ < Mg 2+ < Na + < F - as one proceeds in the periodic table
Increasing ionic size (a) bottom ® top and right ® left
14. Quite a large jump between the values of second and third (b) top ® bottom and right ® left
ionization potentials of an atom would correspond to the (c) bottom ® top and left ® right
electronic configuration (d) top ® bottom and left ® right
25. Which of the following properties of elements does not
(a) 1s 2 2s 2 2 p 6 (b) 1s 2 2s 2 2 p6 3s 2 exhibit the periodicity ?
(a) Ionization potential (b) Electronegativity
(c) 1s 2 2s 2 2 p 6 3s 2 3 p1 (d) 1s 2 2s 2 2 p6 3s 2 3 p2 (c) Electronic configuration(d) Neutron to proton ratio
EBD_7327
56 CHEMISTRY

26. Identify the correct order of the size of the following: 36. Which of the following represents the correct order of
(a) Ca2+ < K+ < Ar < Cl– < S2– increasing first ionization enthalpy for Ca, Ba, S, Se and Ar ?
(b) Ar < Ca2+ < K+ < Cl– < S2– (a) Ca < S < Ba < Se < Ar (b) S < Se < Ca < Ba < Ar
(c) Ca2+ < Ar < K+ < Cl– < S2– (c) Ba < Ca < Se < S < Ar (d) Ca < Ba < S < Se < Ar
(d) Ca2+ < K+ < Ar < S2– < Cl– 37. An element of atomic weight 40 has 2, 8, 8, 2 as the electronic
27. Which one of the following ionic species has the greatest configuration. Which one of the following statements
proton affinity to form stable compound? regarding this element is not correct
(a) NH -2 (b) F – (a) it belongs to II group of the periodic table
(c) I– (d) HS– (b) it has 20 neutrons
28. The stability of + 1 oxidation state increases in the sequence: (c) the formula of its oxide is MO2
(a) Tl < In < Ga < Al (b) In < Tl < Ga < Al
(d) it belongs to 4th period of the periodic table
(c) Ga < In < Al < Tl (d) Al < Ga < In < Tl
29. Amongst the elements with following electronic 38. Which of the following is not the correct order for the stated
configurations, which one of them may have the highest property ?
ionization energy? (a) Ba > Sr > Mg; atomic radius
(a) Ne[3s23p2] (b) Ar [3d104s24p3 ] (b) F > O > N : first ionization enthalpy
(c) Ne [3s23p1] (d) Ne [3s23p3] (c) Cl > F > I; electron affinity
30. Among the elements Ca, Mg, P and Cl, the order of increasing (d) O > Se > Te; electronegativity
atomic radii is : 39. Which of the following sets has strongest tendency to form
(a) Ca < Mg < P < Cl (b) Mg < Ca < Cl < P anions ?
(c) Cl < P < Mg < Ca (d) P < Cl < Ca < Mg (a) Ga, In, Tl (b) Na, Mg, Al
31. What is the value of electron gain enthalpy of Na+ if IE1 of (c) N, O, F (d) V, Cr, Mn
Na = 5.1 eV ? 40. One of the characteristic properties of non-metals is that
(a) –5.1 eV (b) –10.2 eV they
(c) +2.55 eV (d) +10.2 eV (a) Are reducing agents
32. Following statements regarding the periodic trends of (b) Form basic oxides
chemical reactivity of the alkali metals and the halogens are (c) Form cations by electron gain
given. Which of these statements gives the correct picture? (d) Are electronegative
(a) Chemical reactivity increases with increase in atomic 41. In which of the following electronic configuration an atom
number down the group in both the alkali metals and has the lowest ionisation enthalpy?
halogens (a) 1s2 2s2 2p3 (b) 1s2 2s2 2p5 3s1
(b) In alkali metals the reactivity increases but in the 2
(c) 1s 2s 2p2 6 (d) 1s2 2s2 2p5
halogens it decreases with increase in atomic number
42. Which of the following represents the correct order of
down the group increasing electron gain enthalpy with negative sign for the
(c) The reactivity decreases in the alkali metals but elements O, S, F and Cl ?
increases in the halogens with increase in atomic (a) Cl < F < O < S (b) O < S < F < Cl
number down the group
(c) F < S < O < Cl (d) S < O < CI < F
(d) In both the alkali metals and the halogens the chemical
43. Which is the correct order of ionic sizes (At. No. : Ce = 58,
reactivity decreases with increase in atomic number
Sn = 50, Yb = 70 and Lu = 71) ?
down the group
(a) Ce > Sn > Yb > Lu (b) Sn > Ce > Yb > Lu
33. In which of the following arrangements, the sequence is not
strictly according to the property written against it? (c) Lu > Yb > Sn > Ce (d) Sn > Yb > Ce > Lu
(a) HF < HCl < HBr , HI : increasing acid strength 44. The increasing order of the ionic radii of the given
isoelectronic species is :
(b) NH3 < PH3 < AsH3 <SbH3 : increasing basic strength
(a) Cl–, Ca2+ , K+, S2– (b) S2–, Cl–, Ca2+ , K+
(c) B < C < O < N : increasing first ionization enthalpy 2+ + – 2–
(d) CO2 < SiO2 < SnO2 < PbO2 : increasing oxidising power (c) Ca , K , Cl , S (d) K+, S2–, Ca2+, Cl–
34. The correct sequence which shows decreasing order of the 45. Atom of which of the following elements has the greatest
ionic radii of the elements is ability to attract electrons?
(a) Silicon (b) Sulphur
(a) Al3+ > Mg 2+ > Na + > F- > O 2 -
(c) Sodium (d) Nitrogen
(b) Na + > Mg 2+ > Al3+ > O2- > F- 46. The correct order of decreasing electronegativity values
(c) Na + > F - > Mg 2 + > O 2 - > Al3+ among the elements I-beryllium, II-oxygen, III-nitrogen and
(d) O2- > F- > Na + > Mg 2+ > Al3+ IV-magnesium is
(a) II > III > I > IV (b) III > IV > II > I
35. The correct order of electron gain enthalpy with negative
sign of F, Cl, Br and I, having atomic number 9, 17, 35 and 53 (c) I > II > III > IV (d) II > III > IV > I
respectively, is : 47. The element with positive electron gain enthalpy is
(a) F > Cl > Br > I (b) Cl > F > Br > I (a) hydrogen (b) sodium
(c) Br > Cl > I > F (d) I > Br > Cl > F (c) oxygen (d) neon
Classification of Elements and Periodicity in Properties 57

48. Consider the following statements 49. The element with atomic number 117 has not been discovered
I. The radius of an anion is larger than that of the parent yet. In which family would you place this element if
atom. discovered?
II. The ionization energy generally increases with (a) Alkali metals (b) Alkaline earth metals
increasing atomic number in a period.
(c) Halogens (d) Noble gases
III. The electronegativity of an element is the tendency of
an isolated atom to attract an electron. 50. The set representing the correct order for first ionisation
Which of the above statements is/are correct? potential is
(a) I alone (b) II alone (a) K > Na > Li (b) Be > Mg > Ca
(c) I and II (d) II and III (c) B > C > N (d) Ge > Si > C

Exemplar Questions 8. The period number in the long form of the periodic table is
equal to
1. Consider the isoelectronic species, Na + , Mg2+, F– and (a) magnetic quantum number of any element of the period
O2–. The correct order of increasing length of their radii is (b) atomic number of any element of the period
(a) F– < O2– < Mg2+ < Na+ (c) maximum principal quantum number of any element of
(b) Mg2+ < Na+ < F– < O2– the period
(c) O2– < F– < Na+ < Mg2+ (d) maximum azimuthal quantum number of any element of
(d) O2– < F– < Mg2+ < Na+ the period
2. Which of the following is not an actinoid? 9. The elements in which electrons are progessively filled in 4f-
orbital are called
(a) Curium (Z = 96) (b) Californium (Z = 98)
(a) actinoids (b) transition elements
(c) Uranium (Z = 92) (d) Terbium (Z = 65) (c) lanthanoids (d) halogens
3. The order of screening effect of electrons of s, p, d and f 10. Which of the following is the correct order of size of the
orbitals of a given shell of an atom on its outer shell electrons given species
is (a) I > I– > I+ (b) I+ > I– > I
(a) s > p > d > f (b) f > d > p > s +
(c) I > I > I – (d) I– > I > I+
(c) p < d < s > f (d) f > p > s > d 11. The formation of oxide ion O2–(g), from oxygen atom requires
first an exothermic and then an endothermic step as shown
4. The first ionisation enthalpies of Na, Mg, Al and Si are in the
below
order
O (g) + e– ® O– (g); DH = – 141 kJ mol–1

(a) Na < Mg > Al < Si (b) Na > Mg > Al > Si O– (g) + e– ® O2– (g); DH = + 780 kJ mol–1

(c) Na < Mg < Al < Si (d) Na > Mg > Al < Si Thus, process of formation of O 2– in gas ph ase is
5. The electronic configuration of gadolinium (Atomic unfavourable even though O2– is isoelectronic with neon. It
number 64) is is due to the fact that
(a) [Xe] 4f 3 5d5 6s2 (b) [Xe] 4f 7 5d2 6s1 (a) oxygen is more electronegative
(c) [Xe] 4f 7 5d1 6s2 (d) [Xe] 4f 8 5d6 6s2 (b) addition of electron in oxygen results in larger size of
the ion
6. The statement that is not correct for periodic classification
(c) electron repulsion outweighs the stability gained by
of elements is
achieving noble gas configuration
(a) the properties of elements are periodic function of their (d) O– ion has comparatively smaller size than oxygen atom
atomic numbers. 12. Comprehension given below is followed by some multiple
(b) non-metallic elements are less in number than metallic choice questions. Each question has one correct option.
elements. Choose the correct option.
(c) for transition elements, the 3d-orbitals are filled with In the modern periodic table, elements are arranged in order
electrons after 3p-orbitals and before 4s-orbitals. of increasing atomic numbers which is related to the
(d) the first ionisation enthalpies of elements generally electronic configuration. Depending upon the type of orbitals
increase with increase in atomic number as we go along receiving the last electron, the elements in the periodic table
a period. have been divided into four blocks, viz s, p, d and f.
The modern periodic table consists of 7 periods and 18
7. Among halogens, the correct order of amount of energy groups. Each period begins with the filling of a new energy
released in electron gain (electron gain enthalpy) is shell. In accordance with the Aufbau principle, the seven
(a) F > Cl > Br > I (b) F < Cl < Br < I periods (1 to 7) have 2, 8, 8, 18, 18, 32 and 32 elements
(c) F < Cl > Br > I (d) F < Cl < Br < I respectively.
EBD_7327
58 CHEMISTRY

The seventh period is still incomplete. To avoid the periodic 15. Which of the following orders of ionic radii is correctly
table being too long, the two series of f-block elements, called represented ? [2014]
lanthanoids and actinoids are placed at the bottom of the (a) H– > H+ > H (b) Na+ > F– > O2–
main body of the periodic table.
(i) The element with atomic number 57 belongs to (c) F– > O2– > Na+ (d) Al3+> Mg2+> N3–
(a) s-block (b) p-block 16. The species Ar, K+ and Ca2+ contain the same number of
(c) d-block (d) f-block electrons. In which order do their radii increase ? [2015]
(ii) The last element of the p-block in 6th period is (a) (b) Ca 2+ < K + < Ar
represented by the outermost electronic configuration. Ca 2+ < Ar < K +
(a) 7s2 7p6 (b) 5f 14 6d10 7s2 7p0 (c) K + < Ar < Ca 2+ (d) Ar < K + < Ca 2+
(c) 4f 14 5d10 6s2 6p6 (d) 4f 14 5d10 6s2 6p4
17. The formation of the oxide ion O2–(g), from oxygen atom
(iii) Which of the elements whose atomic numbers are given
requires first an exothermic and then an endothermic step as
below, cannot be accommodated in the present set up
of the long form of the periodic table? shown below :
(a) 107 (b) 118 O(g) + e– ® O–(g); Df H = –141 kJ mol–1
(c) 126 (d) 102
(iv) The electronic configuration of the element which is O– (g) + e– ® O2– (g); Df H = +780 kJ mol–1
just above the element with atomic number 43 in the Thus process of formation of O 2– in gas phase is
same group is ........... . unfavourable even though O2– is isoelectronic with neon. It
(a) 1s2 2s2 2p6 3s2 3p6 3d5 4s2 is due to the fact that [2015 RS]
(b) 1s2 2s2 2p6 3s2 3p6 3d5 4s3 4p6 (a) Electron repulsion outweighs the stability gained by
(c) 1s2 2s2 2p6 3s2 3p6 3d 6 4s2
achieving noble gas configuration
(d) 1s2 2s2 2p6 3s2 3p6 3d7 4s2
(v) The elements with atomic numbers 35, 53 and 85 are (b) O– ion has comparatively smaller size than oxygen
all ................ . atom
(a) noble gases (b) halogens (c) Oxygen is more electronegative
(c) heavy metals (d) light metals (d) Addition of electron in oxygen results in larger size of
13. Electronic configuration of four elements A, B, C and D are the ion.
given below 18. In which of the following options the order of arrangement
A. 1s2 2s2 2p6 B. 1s2 2s2 2p4 does not agree with the variation of property indicated
2
C. 1s 2s 2p 3s 2 6 1 D. 1s2 2s2 2p5
against it ? [2016]
Which of the following is the correct order of increasing 3+ 2+ + –
(a) Al < Mg < Na < F (increasing ionic size)
tendency to gain electron?
(a) A < C < B < D (b) A < B < C < D (b) B < C < N < O (increasing first ionisation enthalpy)
(c) D < B < C < A (d) D < A < B < C (c) I < Br < Cl < F (increasing electron gain enthalpy)
(d) Li < Na < K < Rb (increasing metallic radius)
NEET/AIPMT (2013-2017) Questions
19. The element Z = 114 has been discovered recently.
14. Which one of the following arrangements represents the It will belong to which of the following family/group and
correct order of least negative to most negative electron gain electronic configuration ? [2017]
enthalpy for C, Ca, Al, F and O? [NEET Kar. 2013] (a) Carbon family, [Rn] 5f 14 6d10 7s2 7p2
(a) Ca < Al < C < O < F (b) Oxygen family, [Rn] 5f 14 6d10 7s2 7p4
(b) Al < Ca < O < C < F (c) Nitrogen family, [Rn] 5f 14 6d10 7s2 7p6
(c) Al < O < C < Ca < F (d) Halogen family, [Rn] 5f 14 6d10 7s2 7p5
(d) C < F < O < Al < Ca
Classification of Elements and Periodicity in Properties 59

Hints & Solutions


EXERCISE - 1 I.E. increases
grp III A grp IV A grp V A
1. (c) Group IA and III A contain mostly metals. Group VIII

I.E.decreases
2 1 2 2 2 3
contains transition elements which are metals. Group period 3 [Ne]3s 3p [Ne]3s 3p [Ne]3s 3p
VII A contains mostly non-metals (F, Cl, Br). period 4
1 2
[Ar]3d 4s 4p
3

2. (d) Elements having 1, 2 or 3 electrons in its last shell act


as metals. 16. (b) The tendency of a cation to distort the electron cloud
32 = [Ar] 3 d10 4s2 p2 of an anion when it is approaching the anion, is called
34 = [Ar] 3 d10 4s2p4 polarisation power of cation. As polarisation power of
36 = [Ar] 3d10 4s2p6 cation increases, the covalent character increases.
38 = [Ar] 3d10, 4s2p6, 5s2 According to Fajan's rule high charge and small size of
cation will favour covalency. So, polarisation power of
3. (d) Basicity of oxides decreases in a period from left to
a cation increases with charge of the cation.
right. Na2O is basic oxide, CO2, SiO2 and SO2 are acidic
oxides. 17. (d) Cd is not a transition metal among the given options
because it do not have incomplete d-subshell either in
Alternatively, oxides of metals (e.g., Na2O) are basic,
its atomic state [Cd = 5d10 4s2] or in its common
while oxides of non-metals (SO2, SiO2 and CO2) are
oxidation state (Cd2+ = 5d10 4s0).
acidic.
18. (c) F2 has highest electronegativity, so it is chemically most
4. (d) Oxides of non metals are acidic in nature. P is a non-
active non metal.
metal and its oxides are acidic Rest of the oxides are
19. (a) Metallic character decreases down group and increases
basic because they are oxides of metals.
along a period.
5. (c) Allotropy is characteristic property of group 14
20. (b)
elements. All elements of group 14, except Pb, show
allotropy. 21. (c) Basicity of oxides decreases in a period and increases
in a group.
Sn has three allotropic forms grey tin, white tin and
\ SnO2, Al2O3 and ZnO are amphoteric oxides.
rhombic.
22. (d)
6. (a) Atomic number of the given element is 15 and it belongs
to 5th group. Therefore atomic number of the element 23. (a) A cation is always smaller in size as compared to
below the above element = 15 + 18 = 33. corresponding neutral atom. Greater the magnitude of
charge, smaller will be size of ion. Following is the correct
7. (a) The electronic configuration clearly suggest that it
order of decreasing size Al3+ < Al2+ < Al+ < Al.
is a d-block element (having configuration (n – 1)
d 1– 10 ns 0 – 2) which starts from III B and goes till II B. \ Al has largest size.
Hence with d 3 configuration it would be classified in the 24. (c) While moving down in a group, effective nuclear
fifth group. attraction decreases due to addition of new orbits. As
8. (d) Na 2O (basic), SO2 and B2O3 (acidic) and ZnO is a result ionisation potential decreases. Hence, the
amphoteric. correct order is Li > K > Cs.
25. (c) Ionisation potential increases while moving in a period.
9. (a) The screening effect follows the order s > p > d > f .
Group V VI VII VIII
10. (d) The electrons are not filled in d-subshell monotonically
with increase in atomic number among transition Element N O F Ne
elements. Oxygen (group 6) has low ionisation potential than N
(group 5) because of stable configuration of nitrogen
11. (c) 12. (b)
(half filled p-orbital)
13. (c) ns2 p1 is the electronic configuration of III A period.
26. (c) They are isoelectronic species.
Al2O3 is amphoteric oxide.
N3– O2– Na+ F–
14. (b)
No. of electrons 10 10 10 10
15. (c) I. E. increases across a period and decreases down in a No. of protons 7 8 11 9
group. So, element with electronic configuration [Ne]
\ Attractive forces are highest in Na+.
3s2 3p3 will have the highest I.E. among the given
\ Na+ is smallest in size.
choices.
EBD_7327
60 CHEMISTRY

27. (d) According to the general trend of I.E. in a period, it is 40. (a) Energy is supplied in order to remove electron from
expected that oxygen atom has higher I.E. than nitrogen atoms. So energy of atom increases when electron is
atom but nitrogen atom has more stable half filled p- removed from atom.
orbitals due to which it has higher I.E. than oxygen 41. (b) Ionic compounds have high melting point. Greater the
atom. ionic character, more is melting point.
28. (b) The acidic character of non metal oxides increases across HCl has least ionic character because of maximum
a period from left to right and decreases down a group. electronegativity difference between the two
So, acidic character will follow the order: constituent elements, H and Cl among CsF, HCl, HF
oxide of nitrogen > oxides of sulfur > oxides of carbon. and LiF
Among oxides of carbon acidic character increases with \ HCl has minimum melting point.
+4 42. (c) The ionization energy increases with decrease in size.
the oxidation number of carbon. So, CO2 is more
Further the element having stable configuration has
acidic than CO. Hence the sequence of acidic character
higher ionisation energy than expected. Hence the
is N2O5 > SO2 > CO2 > CO
ionization energy of nitrogen (Z = 7) is more than oxygen
29. (d) Arsenic is the only metalloid among the given options.
(Z = 8) and carbon (Z = 6) because it has half-filled
Its small amounts are even very harmful for humans.
p-orbitals.
30. (a) Species Na+ Mg2+ Al3+ Si4+
6
Protons 11 12 13 14 C = 1s 2 2s 2 2 p 2 ; 7 N = 1s 2 2s 2 2 p3 ; 8 O = 1s 2 2s 2 2 p 4
Electrons 10 10 10 10 Hence the correct order should be C < N > O
Size of isoelectronic cations decreases with increase in
43. (c) 44. (c)
magnitude of nuclear charge
(i) The anion is always larger in size as compared to
\ Order of decreasing size is Na+ > Mg2+ > Al3+ > Si4+
corresponding neutral atom.
31. (c) Ionisation potential is amount of energy required to
take outermost loosly bonded electron from isolated (ii) Greater the magnitude of negative charge, larger will be
gaseous atom. Its value decreases in a group and the size.
increases along a period. Thus, here Be has highest Therefore, the correct order of size is O2– > O– > O
ionisation potential. 45. (d)
32. (a) Be – 1s22s2; B – 1s22s22p1; C – 1s22s22p2; N – 1s22s22p3; 46. (c) (n – 1) s2p6(n – 1)d1–10 ns0–2 represents the correct
O – 1s22s22p4. IP increases along the period. But IP of electronic configuration of transition elements among
Be > B. Further IP of O < N because atoms with fully or the given choices.
partly filled orbitals are most stable and hence have 47. (b) As atomic number, number 19 falls within group I of
high ionisation energy. modern periodic table so it is an alkali metal with + 1
33. (a) O– – and F– are isoelectronic. Hence have same number oxidation state.
of shells, therefore greater the nuclear charge smaller 48. (b) Due to high stability of half-filled orbitals.
will be the size i.e., O– – > F– 49. (a) In Cu it has completely filled d-orbital so highest energy
further Li+ and B3+ are isoelectronic. therefore is absorbed when it convert in Cu+ ion.
Li+ > B3+ 50. (a) Since, stable half filled configuration.
Hence the correct order of atomic size is.

O-- > F– > Li+ > B3+ 51. (b) K + ® K 2 + e - . Since e is to be removed from stable
34. (d) The given electronic configuration represents that it configuration.
has 3 valency electrons or it can shows a maximum 52. (c)
oxidation state of +3 and element with intermediate 53. (a) 1st l.P. increases from left to right in a period.
oxidation states form amphoteric oxides. 54. (b) 55. (a)
35. (a) Ionisation potential of nitrogen is more than that of
56. (a) We know that atomic no. of fluorine (F), chlorine (Cl)
oxygen. This is because nitrogen has more stable half-
Bromine (Br) and Iodine (I) are 9, 17, 35 and 53
filled p-orbitals. (N = 1s2, 2s2, 2p3, O = 1s2, 2s2, 2p4)
respectively. Therefore, correct order of reactivity of
36. (b)
halogens is F > Cl > Br > I
37. (b) Due to presence of most penetrating s-electron,
57. (b) Electropositive nature increases down the group and
hydrogen (1s) shows maximum IP out of list.
decreases across the period.
38. (b)
39. (d) P5+ has more effective nuclear charge and smaller size than 58. (b) Pollonium is only true metal in group 16. It has two
P3+. crystalline forms a-form which is cubic and b-form which
is rhombohedral.
Classification of Elements and Periodicity in Properties 61

59. (a) Electron affinity is said to be positive when an atom 6. (a) In case of Ga there are 10d electrons in the penultimate
has spontaneous tendency to accept an electron. When energy shell which shield the nuclear charge less
O changes to O–, energy is released. So, this change effectively, the outer electron is held firmly by nucleus.
has positive electron affinity while all other given As a result, the ionisation energy remains nearly the
changes required to be forced i.e., these require energy same as that of aluminium inspite of the fact that atomic
to occur. size increases.
60. (a) Electron affinity is energy released when electron is 7. (d) Abnormally high difference between 2nd and 3rd
added to isolated gaseous atom. Its value decreases ionization energy means that the element has two
down the group. So electron affinity of F should be valence electrons, i.e., configuration (d)
highest among halogens but due to its smaller size 8. (b) Along the period, I.P. generally increases but not
electron affinity of Cl is more than F. regularly. Be and B are exceptions. First I.P. increases
\ Cl has highest electron affinity.. in moving from left to right in a period, but I.P. of B is
61. (a) Pauling scale of electronegativity was helpful in lower than Be.
predicting 9. (a) Gaining of an electron by a gaseous atom is usually an
(i) Nature of bond between two atoms exothermic process. Gain of second electron by
(ii) Stability of bond negatively charged species feels a strong repulsion
by calculating the difference in electronegativities and the energy of the system increases.
10. (a)
polarity of bond can be calculated.
11. (d) As the size increases the basic nature of oxides changes
62. (c) 63. (b) 64. (b)
to acidic nature i.e., acidic nature increases.
65. (b) Electronegativity decreases down the group and
increases along a period. Cl lies in 17th group hence SO 2 > P 2 O 3 > SiO 2 > Al 2 O 3
more electronegative than P and Si; further it lies above Acidic Weak Amphoteric
acidic
Br, hence more electronegative than Br.
66. (d) Fluorine, chlorine, bromine and iodine are placed in the SO2 and P2O3 are acidic as their corresponding acids
same group 17 because they have 7 electrons in the H2SO3 and H3PO3 are strong acids.
outermost shell. 12. (a) O– ion exerts a force of repulsion on the incoming
67. (a) electron. The energy is required to overcome it.
68. (a) Zero, because of the stable electronic configuration 13. (c) In a period the value of ionisation potential increases
the noble gases do not show any force of attraction from left to right with breaks where the atoms have
towards the incoming electron. some what stable configuration. In this case N has half
69. (a) Halogens have the highest e– affinity. filled stable orbitals. Hence has highest ionisation
70. (b) An atom with high electronegativity has high IP. energy. Thus the correct order is B < C < O < N
and not as given in option (c)
71. (c) Chlorine and sulphur are in period three. Potassium
14. (b) 3rd ionization involves removal of electron from inert
and calcium are in period four. As K has radius more
gas configuration 1s2 2s2 2p6, hence there would be a
than calcium, K+ ion will have largest size.
large jump between 2nd and 3rd ionization energies.
72. (a) F, because of its smallest size.
15. (a) IE1 is always less than IE2.
73. (c) Halogens are most electronegative.
16. (b) IE2 of Mg is lower than that of Na because in case of
74. (b) On passing from left to right in a period acidic character Mg+, 3s-electron has to be removed whereas in case of
of the normal oxides of the elements increases with Na+, an electron is removed from the stable inert gas
increase in electronegativity. configuration which is difficult.
75. (d) The energy involved is ionisation energy (I.E.). Further 17. (c) 18. (c)
the 3rd ionisation energy will be greater than the 2nd 19. (c) Amongst the ions carrying same charge, the smallest
and 1st. one will have the greatest hydration energy (most
EXERCISE - 2 negative).
1. (d) 2. (a) 20. (d)
21. (d) Tl shows the inert pair effect. Hence Tl+ oxidation state
3. (a) Acidic character of oxide µ Non-metallic nature of is more stable than Tl3+.
element. 22. (a) 23. (a) 24. (b) 25. (d)
Non-metallic character increases along the period. 26. (a) For isoelectronic species, size of anion increases as
Hence order of acidic character is negative charge increases whereas size of cation
Cl2O7 > SO2 > P4O10. decreases with increase in positive charge. Further
ionic radii of anions is more than that of cations. Thus
4. (c) X(g) + e– ® X–(g) + x kJ .......(i)
the correct order is Ca++ < K+ < Ar < Cl– < S– –
X–(g) ® X(g) – x kJ .......(ii) 27. (a) Proton affinity decreases in moving across the period
5. (d) Zero group is also called as buffer group because it is from left to right due to increase in charge, within a
placed between highly electropositive metals (group group the proton affinities decreases from top to
1) and highly electronegative non-metals (group 17). bottom. Nitrogen family > Oxygen family > Halogens
EBD_7327
62 CHEMISTRY

28. (d) The stability of +1 oxidation state increases from 39. (c) N, O and F (p-block elements) are highly electronegative
aluminium to thallium i.e. Al < Ga < In < Tl non metals and will have the strongest tendency to
29. (d) The smaller the atomic size, larger is the value of form anions by gaining electrons from metal atoms.
ionisation potential. Further the atoms having half filled 40. (a) Non metals form oxides with oxygen and thus reduce
or fully filled orbitals are comparitively more stable, oxides of metals behaving as reducing agents.
hence more energy is required to remove the electron 41. (b)
from such atoms. 42. (b) O < S < F < Cl
30. (c) 12 Mg 15 P 17Cl 20Ca Electron gain enthalpy – 141, –200, – 333, – 349 kJ mol–1
160p 110 99 197 (pm) 43. (b) Correct order of ionic size is Sn > Ce > Yb > Lu.
Cl < P < Mg < Ca 44. (c)
31. (a) IE1 of Na = – Electron gain enthalpy of Na + = – 5.1 eV. 45. (d) Halogens have very high values of electron gain
32. (b) The alkali metals are highly reactive because their first enthalpies.
ionisation potential is very low and hence they have 46. (a) Electronegativity values of given elements are as
great tendency to loses electron to form unipositive follows:
ion. Be – 1.5 (I) Mg – 1.2 (IV)
O – 3.5 (II) N – 3.0 (III)
On moving down group- I from Li to Cs ionisation
i.e. II > III > I > IV
enthalpy decreases hence the reactivity increases. The
47. (d) Noble gases have positive values of electron gain
halogens are most reactive elements due to their low enthalpy because the anion is higher in energy than
bond dissociation energy, high electron affinity and the isolated atom and electron.
high enthalpy of hydration of halide ion. However their 48. (c) The tendency of an atom in a compound to attract a
reactivity decreases with increase in atomic number pair of bonded electrons towards itself is known as
33. (b) In hydrides of 15th group elements, basic character electronegativity of the atom.
decreases on descending the group i.e. 49. (c)
NH3 > PH3 > AsH3 > SbH3. 50. (b) The correct order of first ionisation energy is
34. (d) All the given species contains 10 e – each i.e. represented by
isoelectronic. Be > Mg > Ca
For isoelectronic species anion having high negative Since on moving down a group atomic size increases
due to addition of one extra shell, hence I.E decreases.
charge is largest in size and the cation having high
positive charge is smallest. EXERCISE - 3
35. (b) As we move down in a group electron gain enthalpy
Exemplar Questions
becomes less negative because the size of the atom
increases and the distance of added electron from the 1. (b) In case of isoelectronic species
nucleus increases. Negative electron gain enthalpy of 1
F is less than Cl. This is due to the fact that when an ionic radii µ
atomic number
electron is added to F, the added electron goes to the
\ The correct order of increasing ionic radii will be :
smaller n = 2 energy level and experiences significant
repulsion from the other electrons present in this level. Ionic radii Mg 2+ < Na + < F- < O 2-
In Cl, the electron goes to the larger n = 3 energy level Atomic number (12) (11) (9) (8)
and consequently occupies a larger region of space
2. (d) Elements with atomic number, Z = 90 to 103 are called
leading to much less electron–electron repulsion. So
actinoids. Terbium belongs to lanthanoids.
the correct order is Cl > F > Br > I. 3. (a) For a given shell, screening effect decreases in the order
36. (c) On moving along a period from left to right I.E. increases : s > p > d > f.
and on moving down a group I.E. decreases. 4. (a) Electronic configuration for the given elements will be :
hence correct order is : Ba < Ca < Se < S < Ar Na = [Ne]3s1, Mg = [Ne]3s2, Al = [Ne]3s23p1,
37. (c) Its valency is 2. So it will form MO type compound. Si = [Ne] 3s23p2
38. (b) On moving along the period, ionization enthalpy Ionisation enthalpy increases along a period but I.E of
increases. Mg is higher than Al because of completely filled 3s
In second period, the order of ionization enthalpy should orbital in Mg.
be as follows : F > O > N 5. (c) The electronic configuration of Gd (Z = 64) is
But N has half-filled structure, therefore, it is more [Xe] 4f 7 5d1 6s2.
6. (c) In case of transition element, the order of filling of
stable than O. That’s why its ionization enthalpy is
electrons in various orbital is 3p < 4s < 3d. Thus, 3d
higher than O. Thus, the correct order of IE is
orbital is filled only when 4s orbital gets completely
F > N > O. filled.
Classification of Elements and Periodicity in Properties 63

7. (c) As we move in a group from Cl to I, the electron gain The electronic configuration of the element with
enthalpy (i.e., energy released in electron gain) become Z = 25 is
less and less negative due to corresponding increase 1s2 2s2 2p6 3s2 3p6 3d 5 4s2 (i.e., Mn).
in the atomic size. (v) (b) The elements with atomic numbers 35(36 – 1),
However, the electron gain enthalpy of F is less 53( 54 – 1) and 85(86 – 1), lie in a group before noble
negative than that of Cl due to its small size. Thus, the gases, i.e., belongs to halogens (group 17).
negative electron gain enthalpy among halogens 13. (a) Electronic configuration of given elements indicate that
follows the order : A is a noble gas (i.e., Ne), B is oxygen, C is sodium
F < Cl > Br > I metal and D is fluorine.
8. (c) As each period starts with the filling of electrons in a (i) Noble gases have no tendency to gain electrons
new principal quantum number, so, the period number since all their orbitals are completely filled.
in the long form of the periodic table refers to the \ element A will have the least electron gain
maximum principal quantum number (n) of any element enthalpy.
in the period. (ii) Element D has one electron less and element B has
9. (c) The elements in which electrons are filled in 4f-orbital two electrons less than the corresponding noble
are called lanthanoids. Lanthanoids consist of elements gas configuration, hence, element D will have the
from Z = 58 (cerium) to 71 (lutetium). highest electron gain enthalpy in comparison to
10. (d) Generally, cations are smaller in size while anions are element B.
bigger in size than the neutral atom. (iii) Since, element C has one electron in the s-orbital
11. (c) O2– has noble gas configuration and isoelectronic with and need one more electron to complete its
neon but its formation is unfavourable due to strong configuration, therefore, electron gain enthalpy of
electronic repulsion between the negatively charged C is less than that of element B. So, we can conclude
O– ion and the electron being added. that the electron gain enthalpies of the four elements
Thus, the electron repulsion will be more than the increases in the order : A < C < B < D.
stability gained by achieving noble gas configuration. NEET/AIPMT (2013-2017) Questions
12. (i) (c) The element with atomic number 57 belongs to
14. (a) As the nuclear charge increases, the force of attraction
d-block element as the last electron enters into the between the nucleus and the incoming electron
5d-orbital against the aufbau principle. This increases and hence the elecron gain enthalpy becomes
anomalous behaviour can be explained on the basis more negative, hence the correct order is
of greater stability of the xenon (inert gas) core. Ca < Al < C < O < F
After barium (Z = 56), the addition of the next 15. (N) All answers are incorrect.
electron should occur in 4f-orbital in accordance 16. (b) In isoelectronic species the radius decrease with increase
with aufbau principle. This will however, tend to in nuclear charge hence increasing order of radius is
destabilize the xenon core (Z = 54), [Kr] (4d10 4f 0
Ca+2 < K+ < Ar
5s2 5p6 5d0) since the 4f-orbitals lie inside the core.
Therefore, the 57th electron prefers to enter 5d- 17. (a) Incoming electrons occupy the smaller n = 2 shell, also
orbital which lies outside the xenon core and whose negative charge on oxygen (O–) is another factor due to
energy is only slightly higher than that of 4f-orbital. which incoming electron feel repulsion.
Thus, the outer electronic configuration of Hence electron repulsion outweigh the stability gained
La(Z = 57) is 5d1 6s2 rather than the expected 4f 1 6s2. by achieving noble gas configuration.
(ii) (c) Each period starts with the filling of electrons in a 18. (b&c) The correct order is B < C < O < N
new principal energy shell. Therefore, 6th period Generaly Ionisation energy increases across a period.
starts with the filling of 6s-orbital and ends when But here first I.E. of O is less than the first I.E. of N. This
6p-orbitals are completely filled.
is due to the half-filled 2p orbital in N(1s2, 2s2, 2p3)
Thus, the outermost electronic configuration of the
which is more stable than the 2p orbital in O (1s2, 2s2,
last element of the p-block in the 6th period is
represented by 6s2 4f 14 5d10 6p6 or 4f 14 5d10 6s2 2p4)
6p6. (c) The correct order of electron affinity is
(iii) (c) The long form of the periodic table contain element I < Br < F < Cl
with atomic number 1 to 118. Halogens have high electron affinities which decreases
(iv) (a) The fifth period begins with Rb (Z = 37) and ends at on moving down the group. However, fluorine has lower
Xe (Z = 54). Thus, the element with Z = 43 lies in the value than chlorine which is due to its small size and
5th period. Since, the 4th period has 18 elements, repulsion between the electron added and electrons already
thus, the atomic number of the element which lies
present.
immediately above the element with atomic number
19. (a) Z = 114 belong to Group 14, carbon family
43 will be 43 – 18 = 25.
Electronic configuration = [Rn]5f 146d107s27p2
EBD_7327
64 CHEMISTRY

Chemical Bonding
4 and Molecular
Structure
The attraction force which holds various constitutents (atoms, FORMAL CHARGE AND RESONANCE
ions, etc) together in different chemical species is called a chemical
bond.
Formal Charge
It is the difference between the number of valence electrons in an
KOSSEL - LEWIS APPROACH TO CHEMICAL isolated atom and no of electrons assigned to that atom in Lewis
BONDING structure.
(i) Lewis postulated that atoms achieve stable octet when they Formal change ( F.C ) æ Total no. of valenceö
are linked by chemical bonds. on an atom in = ç electrons in the free ÷
(ii) The outer shell electrons are called valence electrons and it Lewis structure çè atom ÷ø
is these electrons which take part in chemical bonding.
æ Total no. of ö 1 æ Total no. of ö
(iii) The valence electrons are represented by putting dots
- ç non - bonding ÷ - ç bonding ( shared )÷
around the symbol of the element. These are called Lewis çè electrons (lone pairs)÷ø 2 ç electrons ÷ø
g g è
symbols. For ex: Lig, Be,
g
gBg , etc.
Note : Among the valence electrons, those pair of electrons which
(iv) Kossel provided the basis for ion formation by electron are involved in bonding are called bond pair of electrons while
transfer and formation of ionic crystalline compounds.
those which are not involved in bonding are called lone pair of
IONIC AND COVALENT BOND electrons.
Ionic or Electrovalent Bond Resonance
The bond formed as a result of electrostatic attraction between It has been found that the observed properties of certain molecules
positive and negative ions is called ionic bond. Ionic bonds are cannot be satisfactorily explained by one structure. The molecule
formed readily between elements with low IE and elements with is then supposed to have many structures, each of which can
high –ve value of electron gain enthalpy. explain most of the properties but not all. This phenomenon is
Factors favouring the formation of ionic bond : called resonance.
(a) Ionisation energy (IE) BOND PARAMETERS
(b) Electron affinity (i) Bond length : It is defined as the equilibrium distance between
(c) Lattice energy the nuclei of two bonded atoms in a molecule. The covalent
Note : A qualitative measure of the stability of an ionic compound radius is half of the distance from the centre of nucleus of
is provided by its lattice enthalpy which is defined as the energy one atom to the centre of nucleus of the other atom provided
required to completely separate one mole of a solid ionic compound bonded atoms will be of same element in a molecule.
into gaseous constituent ions The van der Waal’s radius represents the overall size of the
Covalent Bond and Lewis Structures atom which includes its valence shell in a non-bonded
The bond formed as a result of sharing of electrons between situation.
elements is called covalent bond. When electrons shared by Van der Waal’s radius > Covalent radius
bonded atoms are contributed entirely by one of the bonded atoms (ii) Bond angle : It is defined as the angle between the orbitals
bond formed is known as a coordinate bond. Lewis dot structures containing bonding electron pairs around the central atom in
provide a picture of bonding in molecules and ions in terms of a molecule / complex ion.
shared pair of electrons and the octet rule. (iii) Bond Enthalpy : It is defined as the amount of energy required
Covalency : It is defined as the number of electrons contributed to break one mole of bonds of a particular type between two
by an atom of the element for sharing with other atoms so as to atoms in a gaseous state. Its unit is kJ mol–1. Larger the bond
achieve noble gas configuration. dissociation enthalpy, stronger is the bond.
Chemical Bonding and Molecular Structure 65

(iv) Bond order : It is given by the number of bonds between Applications of Dipole Moment
two atoms in a molecule. Isoelectronic molecules and ions (i) In determining the polarity of bonds :
have identical bond order. For ex: F2 and O 22 - have B.O.= 1. As m = q × d,
(ii) In the calculation of ionic character :
In general,
i.e., % ionic character
1 Observed dipole moment
Bond order µ Bond Enthalpy µ
Bond length = × 100
Dipole moment considering 100% ionic character

POLARITY OF BONDS AND DIPOLE MOMENT (iii) Determination of shape or symmetry of molecules:
When covalent bond is formed between two similar atoms like H2, Note : In A— B molecule
O2, Cl2, N2 etc, the shared pair of electrons is equally attracted by Ionic nature µ XA – XB
two atoms. As a result, electrons pair is exactly situated betwen % Ionic character = 0 when XA = XB (100% covalent bond)
the two atoms. Such a bond is called non-polar covalent bond. % Ionic character = 0 – 15% Non polar covalent bond
In heteronuclear molecules like HF, the shared pair of % Ionic character = 15 – 50% Polar covalent bond
electrons gets shifted towards the more electronegative element % Ionic character = > 50% Ionic bond
(here F), resulting in polarity in bond. Such a bond is called polar
THE VALENCE SHELL ELECTRON PAIR REPULSION
covalent bond. This polarisation of the bond gives rise to a dipole
(VSEPR) THEORY
moment given by:
The main postulates of this theory are:
Dipole moment (m) = charge (q) × distance of separation (d). It is
uuuuuur (i) The geometry of a molecule depends upon the number of
represented as H - F . Dipole moment is a vector quantity valence shell electron pairs (bonded or non-bonded) around
the central atom
Covalent Character in Ionic Compounds
(ii) Pairs of electrons in valence shell repel one another since
Just as covalent bonds have partial ionic character, ionic
their electron clouds are negatively charged.
bonds have partial covalent character. The partial covalent
(iii) These pairs of electrons tend to occupy such positions in
character of ionic bonds was discussed by Fajan in terms of
space that minimise repulsion and thus maximise distance
following rules:
between them.
(i) Smaller the cation and larger the anion, greater is the covalent
(iv) The valence shell is taken as a sphere with electron pairs
character of the ionic bond.
localising on the spherical surface at maximum distance from
(ii) Greater the charge on cation, greater is the covalent character one another.
of the ionic bond.
(v) A multiple bond is treated as if it is a single electron pair
(iii) For cations of same size and charge, the one with electronic
(vi) Where two or more resonance structures can represent a
configuration ns2np6nd10 is more polarising than the one molecule, VSEPR model is applicable to any such structure.
with noble gas configuration ns2 np6.
The decreasing order of repulsive interaction of electron pairs
The covalent character of an ionic bond is explained on the is
basis of polarisation. The cation polarises the anion, pulling
lp – lp > lp – bp > bp – bp
the electronic charge towards itself and thereby increasing
Where lp ® lone pair and bp ® bond pair
electronic charge between the two. This is what happens in
For prediction of geometrical shape of molecules, we divided
a covalent bond, i.e. build up of electron charge density
molecules in two categories :
between the nuclei. The polarising power of the cation, the
(i) molecules in which central atom has no lone pair(s)
polarisability of the anion and the extent of polarisation of
(ii) molecules in which central atom has one or more lone
the anion determine the percent covalent character of the
pair(s).
ionic bond.
EBD_7327
66 CHEMISTRY

Geometry of molecules when central atom has no lone pair of electrons.


No. of Arrangement of electron Molecular Geometry
electron Examples
pairs
pairs
2 180°
B— A— B BeF2, HgCl 2
Linear
A
Linear

3 B
BF 3, AlCl 3
120°
A A
B B
Trigonal planar Trigonal planar

4
B CH 4,SiF4
109.5°

A A
B B

B
Tetrahedral Tetrahedral
5
B PCl5, AsF5
90° B
120° A
A B

B
Trigonal B
Trigonal
bipyramidal
bipyramidal

6
SF6
90° B
B B

A A

B B B
Octahedral
Octahedral
Chemical Bonding and Molecular Structure 67
Geometry of molecules when central atom has one or more lone pair of electrons

Molecule No. of No. of Total Arrangement of Shape Example


type bonding pairs lone pairs electron pairs electron pairs (geometry)

A
AB2L 2 1 3 Bent or Vshaped SO2, O3, SnCl2
B B
Trigonal planar

A B
AB3L 3 1 4 Pyramidal NH3
B B
Tetrahedral

A
AB2L2 2 2 4 V shaped H2O
B B
Tetrahedral

B
B

A
AB4L 4 1 5 See saw SF4
B
B
Trigonal bipyramidal

B A
AB3L2 3 2 5 T shaped CIF3
B
Trigonal bipyramidal

A
AB2L3 2 3 5 Linear XeF2
B
Trigonal bipyramidal

B
B B
A
AB5L 5 1 6 Square pyramidal BrF5
B B
Octahedral

B B
A
AB4L2 4 2 6 Square planar XeF4
B B
Octahedral
EBD_7327
68 CHEMISTRY

VALENCE BOND THEORY (VBT) (iii) Co–axial overlapping ® extent of overlapping more
® high bond strength
(i) This theory is based on the knowledge of atomic orbitals, Collateral overlapping ® extent of overlapping less
electronic configurations of elements, the overlap criteria of ® low bond strength
atomic orbitals, the hybridisation of atomic orbitals and the In Co–axial overlapping, extent of overlapping is
principles of variation and superposition. p–p<s–p<s–s
(iv) VBT explains the formation and directional properties of
(ii) According to orbital overlap concept, the formation of a
bonds in polyatomic molecules like CH4, NH3, etc. in terms
covalent bond between two atoms results by pairing of e–1 s of overlap and hybridisation of atomic orbitals.
present in the valence shell having opposite spins greater is (v) Two types of bonds are formed on account of overlapping.
the overlap, stronger is the bond formed between the atoms. (a) Sigma (s) bond (b) Pi (p) bond

Sigma bond (s ) Pi bond (p )

(i) It is formed by end to end (head on) overlap of (i) It is formed by sideways overlapping,
bonding orbitals along internuclear axis. perpendicular to internuclear axis.
(ii) This bond is formed by overlapping of s with s, s (ii) This bond is formed by overlap of p-p orbitals
with p(along axis) and p with p orbitals. above and below the plane of participating atoms.
(iii) Stronger and less reactive. (iii) Less strong and more reactive.
(iv) Can be independently exist (iv) Always exist along with a s bond
(v) The groups or atoms can undergo bond rotation about (v) Due to resistance to rotation around the p bond
single sigma (s ) bonds the groups attached to it are not free to rotate.

HYBRIDISATION (iv) It is not necessary that only half filled orbitals participate in
According to Pauling, the atomic orbitals combine to form a new hybridisation. In some cases, even filled orbitals of valence
set of equivalent orbitals known as hybrid orbitals. It is these shell take part in hybridisation.
hybrid orbitals which are used in bond formation. The phenomenon Types of Hybridisation
is known as hybridisation. Depending upon the type and number of orbitals involved in
Features of Hybridisation intermixing, the hybridization can be of various types namely sp,
sp2, sp3, sp3d, dsp2, sp3d2, sp3d3.
(i) The number of hybrid orbitals is equal to the number of atomic
orbitals that get hybridised. Method for Finding the Type of Hybridisation
(ii) The hybridised orbitals are always equal in energy and shape. The structure of any molecule can be predicted on the basis of
(iii) The hybrid orbitals are more effective in forming stable bonds hybridization which in turn can be known by the following general
than pure atomic orbitals. formulation :
(iv) The type of hybridisation indicates the geometry of 1
molecules. H = (V + M - C + A)
2
Conditions of Hybridisation where H = Number of orbitals involved in hybridization viz. 2, 3, 4,
(i) The orbitals present in valence shell of the atom are 5, 6 and 7, hence nature of hybridization will be sp, sp2, sp3, sp3d,
hybridised. sp3d2, sp3d3 respectively.
(ii) The orbitals undergoing hybridisation should have almost V = Number of electrons in valence shell of the central atom,
equal energy. M = Number of monovalent atoms,
(iii) Promotion of electron is not essential condition prior to C = Charge on cation,
hybridisation. A = Charge on anion.

Shape of Hybridisation Atomic


Examples
molecule ion Type orbitals
Linear sp s+ p BeCl2 , C 2 H 2 , CO2 , HgCl2 ,
Trigonal planar sp 2
s + p ( 2) SnCl2 , CO32 -
Tetrahedral sp3 s + p ( 3) CH 4 , H 2 O, NH 3
2- 2-
Square planar dsp 2 d + s + p ( 2) é Ni ( CN ) ù
ë 4û
, [ PtCl2 ]

Trigonal bipyramidal sp3 d s + p ( 3) + d PF5 , PCl5


Square pyramidal sp3 d 2 s + p ( 3) + d ( 2 ) BrF5
Octahedral 3 2
sp d , d sp 2 3 s + p ( 3) + d ( 2) , d ( 2) + s + p ( 3) SF6 [ CrF6 ]
3-

Pentagonal bipyramidal sp3 d 3 s + p ( 3) + d ( 3) IF7


Chemical Bonding and Molecular Structure 69

MOLECULAR ORBITAL THEORY (ii) Bond order (B. O.)


1
Important Features of MOT B. O. = ( N b - N a )
2
(i) The electrons in a molecule are present in various molecular
\ + ve B.O. (i.e. Nb > Na) means stable molecule whereas
orbitals (MO’ s) as the electrons of atoms are present in – ve B.O. (i.e. Na > Nb) means unstable molecule molecule.
various atomic orbitals (AO’s). (iii) Nature of bond : B. O. values of 1, 2 or 3 correspond to single,
(ii) The atomic orbitals of comparable energies and proper double and triple bonds. According to MOT even a fractional
symmetry combine to form MO’s. bond order is possible.
(iii) An electron in a MO is influenced by two or more nuclei (iv) Bond length (BL) : BL µ 1/B.O.
depending upon no. of atoms in molecule. Thus, a MO is Bonding in Few Molecules/Ions
polycentric. (i) He2 : EC is s 1 s2 s* 1s2.
(iv) The no. of MO’s formed is equal to the number of combining 1
AO’s. When two AO’s combine, two MO’s are formed. One \ B.O. = (2 - 2) = 0
2
is bonding MO (BMO) and the other is antibonding MO He2 does not exist.
(ABMO).
(ii) C2 : EC is s 1s2 s* 1s2 s 2 s2 s* 2s2.
(v) BMO has lower energy and higher stability than (p2px2 = p2py2)
corresponding ABMO. 1
(vi) The MO’s are filled in accordance with Aufbau’s principle, \ B. O. = ( 8 - 4) = 2
2
obeying the Pauli’s exclusion principle and Hund’s rule. It is diamagnetic due to absence of unpaired electron(s)
Conditions for Atomic Orbitals to Form M.O. (iii) O2 : EC is s 1 s2 s* 1s2. s 2 s2 s* 2s2 s 2pz2
(i) The combining AO’s must have same or nearly same energy. æ p 2p x 2 ö æ p * 2p x1 ö
(ii) The combining AO’s must have same symmetry about the ç ÷ç ÷
ç p 2p y 2 ÷ ç p * 2p y1 ÷
molecular axis. è øè ø
(iii) The combining AO’s must overlap to the maximum extent.
1
Types of MO \ B. O. = (10 - 6) = 2
2
s - MO’s of diatomic molecules are designated as s (sigma), p (pi), It is paramagnetic due to unpaired electrons in p*2px and
d (delta) etc. s - MO’s are symmetrical around bond-axis while p- p*2py.
MO’s are not symmetrical. (iv) H2– :
M.O. configuration – (s 1s)2 (s*1s)1
Filling of Electrons in MO’s
Paramagnetic
(i) For O2 and F2, the increasing order of energies of various Bond order = ½[2 – 1] = ½
MO’s is : (v) N2+ :
s 1s < s* 1s < s 2s < s * 2s < s 2pz < (p 2px = p 2py) M.O. configuration
< (p* 2px = p* 2py) < s* 2pz. = (s1s)2(s*1s)2(s2s)2 (s*2s)2(p2px)2(p2py)2(s2pz)1
(ii) For Li2, Be2, B2, C2, N2 the increasing order of energies of Paramagnetic
various MO’s is : 1
Bond order = [ 9 - 4] = 2.5
s 1s < s* 1s < s 2s < s* 2s < (p2px = p2py) < s 2pz 2
< (p* 2px = p* 2py) < s* 2pz. (vi) O2+:
The main difference between the two types of sequences in M.O. configuration
energy level is that for molecules O2, F2, the s2pz M.O.is
= KK (s 2s)2(s* 2s)2(s 2pz)2(p 2px)2(p 2py)2(p* 2px)1
lower in energy than p2px and p2py.
Electronic Configuration and Molecular Behaviour 1
Bond order = [8 - 3] = 2.5
The distribution of electrons among various MO’s is called 2
electronic configuration of the molecule. It gives important (vii) O2– :
information about the molecule as discussed below:
M.O. configuration
(i) Stability of molecules :
If Nb > Na, molecule is stable = KK (s2s)2(s*2s)2(s2pz)2(p2px)2(p2py)2(p*2px)2 (p*2py)1
If Nb < Na, molecule is unstable 1
Where Nb ® No. of electrons in BMO Paramagnetic, Bond order = [8 – 5] = 1.5
2
Where Na ® No. of electrons in ABMO
EBD_7327
70 CHEMISTRY

HYDROGEN BONDING
O
It is defined as an attractive force which binds H-atom of one
molecule with the electronegative atom (F, O or N) of another N
O
molecule. The magnitude of H-bonding depends on the physical
state of the compound, it is maximum in solid state and minimum in O H
gaseous state.
Types of H–bonding : Effect of H–bond on Physical Properties:
(i) Intermolecular H-bond : H-bond formed between two (i) Melting point and boiling point :– Due to intermolecular
different molecules of same or different compounds. H-bonding, M.P. & B.P. of compounds increases.
These are of two types : (ii) Molecular weight : Increases due to H–bonding
(a) Homo intermolecular : H–bond between molecules of (iii) Solubility :
same compounds (a) Intermolecular H–bonding –Few organic compounds
Example : H—F........H—F........H—F (Non–polar) are soluble in water (Polar solvent) due to
(b) Hetero intermolecular : H–bond between molecules of H–bonding.
different compounds (b) Intramolecular H–bonding : It decreases solubility as it
Example : Solution of alcohol and water. forms chelate by H–bonding, so H–atom is not free for
(ii) Intramolecular H-bond : Formed when H-atom is in between other molecule.
two highly electronegative (F, O, N) atoms present within the (iv) Viscosity and surface tension – H–bond associates
same molecule. For example in o-nitrophenol i.e. molecules together so viscosity and surface tension increases.
Chemical Bonding and Molecular Structure 71
CONCEPT MAP
EBD_7327
72 CHEMISTRY

1. The following group does not contain a dative bond 14. In PO43– ion, the formal charge on each oxygen atom and
(a) – NO2 (b) – N2Cl P—O bond order respectively are
(c) – NC (d) all of these (a) –0.75, 0.6 (b) – 0.75, 1.0
2. Which of the following pairs will form the most stable ionic (c) – 0.75, 1.25 (d) –3, 1.25
bond ? 15. Which of the following hydrogen bonds are strongest in
(a) Na and Cl (b) Mg and F vapour phase?
(c) Li and F (d) Na and F (a) HF---HF (b) HF---HCl
3. Which of the following conceivable structures for CCl4 will (c) HCl---HCl (d) HF---HI
have a zero dipole moment ? 16. As the s-character of hybridised orbital increases, the bond
(a) Irregular tetrahedron angle
(b) Square planar (a) increase (b) decrease
(c) Square pyramid (carbon at apex) (c) becomes zero (d) does not change
(d) None of these 17. Among the following the electron deficient compound is
4. Among the following which compound will show the highest
(a) BCl3 (b) CCl4 (c) PCl5 (d) BeCl2
lattice energy ?
18. How many sigma bonds are there in P4O10?
(a) KF (b) NaF (c) CsF (d) RbF
5. The compound which contains both ionic and covalent (a) 4 (b) 8
bond : (c) 12 (d) 16
(a) KCl (b) KCN (c) CH4 (d) H2 19. The number of possible resonance structures for CO32 - is
6. Which one of the following contains a co-ordinate covalent (a) 2 (b) 3 (c) 6 (d) 9
bond ? 20. The strongest hydrogen bond is :
+ (a) O – H...........S (b) S – H.............O
(a) H 2 O (b) HCl (c) BaCl 2 (d) N 2 H5
(c) F – H............F (d) F – H.............O
7. The strength of bonds formed by s–s and p–p, s–p overlap 21. Which of the following has the highest dipole moment?
in the order of
(a) s–p > s–s > p–p (b) p–p > s–s > s–p H CH3 H
| |
(c) s–s > p–p > s–p (d) s–s > s–p > p–p (a) C=O (b) C = C
8. The types of bonds present in CuSO4. 5H2O are only | |
(a) electrovalent, covalent and co-ordinate covalent bonds H CH H
3
(b) electrovalent, covalent and hydrogen bond
(c) electrovalent and covalent bonds H CH3 Cl CH3
(d) electrovalent and co-ordinate covalent bonds | | | |
9. Which of the following combination will form an electrovalent (c) C = C (d) C = C
| | | |
bond ? CH3 H CH3 Cl
(a) P and Cl (b) NH3 and BF3
(c) H and Ca (d) H and S 22. In the cyanide ion, the formal negative charge is on
10. Which one of the following will dissolve in water most (a) C (b) N
readily? (c) Both C and N (d) Resonate between C and N
(a) I2 (b) BaCO3 (c) KF (d) PbI2 23. Of the following hydrides which one has the lowest boiling
11. When a metal atom combines with non-metal atom, the point ?
non-metal atom will (a) AsH 3 (b) SbH 3 (c) PH 3 (d) NH 3
(a) lose electrons and decrease in size 24. Resonance structures can be written for
(b) lose electrons and increase in size (a) O3 (b) NH3 (c) CH4 (d) H2O
(c) gain electrons and decrease in size 25. Which one of the following is the correct order of
(d) gain electrons and increase in size interactions?
12. Indicate the nature of bonding in CCl4 and CaH2
(a) covalent < hydrogen bonding < van der Waals < dipole-
(a) Covalent in CCl4 and electrovalent in CaH2
dipole
(b) Electrovalent in both CCl4 and CaH2
(c) Covalent in both CCl4 and CaH2 (b) van der Waals < hydrogen bonding < dipole-dipole
(d) Electrovalent in CCl4 and covalent in CaH2 < covalent
13. What is the correct mode of hybridisation of the central (c) van der Waals < dipole-dipole < hydrogen bonding
atom in the following compounds? < covalent
NO–2 SF4 PF6– (d) dipole-dipole < van der Waals < hydrogen bonding
(a) sp sp 2 sp 3 < covalent
(b) sp 2 sp3 d sp3d 2 26. Which one of the following molecules will form a linear
(c) sp 2 sp 3 d2sp 3 polymeric structure due to hydrogen bonding?
(d) sp 3 sp 3 sp3d 2 (a) NH3 (b) H2O (c) HCl (d) HF
Chemical Bonding and Molecular Structure 73

27. Among the following, the species having square planar 40. Which of the following salt shows maximum covalent
geometry for central atom are character?
(i) XeF4 (ii) SF4 (a) AlCl3 (b) MgCl 2 (c) CsCl (d) LaCl3
(iii) [NiCl4]2– (iv) [PtCl4]2–
41. Pauling’s electronegativity values for elements are useful in
(a) (i) and (iv) (b) (i) and (ii) .
predicting :
(c) (ii) and (iii) (d) (iii) and (iv)
(a) polarity of bonds in molecules
28. In [Ag (CN2)]–, the number of p bonds is :
(b) ionic and covalent nature of bonds
(a) 2 (b) 3
(c) coordination number
(c) 4 (d) 6
(d) Both (a) and (b)
29. Which of the following does not contain any coordinate
42. Identify the non polar molecule in the following compounds:
bond?
(a) H2 (b) HCl
(a) H3O+ (b) BF4- (c) HF and HBr (d) HBr
43. Which of the following substances has the greatest ionic
(c) HF2- (d) SO 2 - character ?
30. Hydrogen bonding could not affect the boiling point of : (a) Cl2O (b) NCl3 (c) PbCl2 (d) BaCl2
(a) CH3OH (b) HI (c) H2O (d) NH3 44. Methanol and ethanol are miscible in water due to
31. The low density of ice compared to water is due to : (a) Covalent character
(a) induced dipole-induced dipole interactions (b) Hydrogen bonding character
(b) dipole-induced dipole interactions (c) Oxygen bonding character
(c) hydrogen bonding interactions (d) None of the above
(d) dipole-dipole interactions 45. As compared to covalent compounds, electrovalent
32. Intramolecular hydrogen bonding is found in : compounds generally have
(a) p-nitrophenol (b) m-nitrophenol (a) Low melting points and low boiling points
(c) o-nitrophenol (d) phenol (b) Low melting points and high boiling points
OH (c) High melting points and low boiling points
(d) High melting points and high boiling points
33. The vapour pressure of is higher than 46. Which bond angle q would result in the maximum dipole
NO 2 moment for the triatomic molecule YXY
OH (a) q = 90° (b) q = 120°
(c) q = 150° (d) q = 180°
due to 47. Which does not show resonance :
O 2N (a) Benzene (b) Aniline
(a) dipole moment (b) dipole-dipole interaction (c) Ethylamine (d) Toluene
(c) H-bonding (d) lattice structure 48. Polarisibility of halide ions increases in the order
34. The bond length between C–C bond in sp2 hybridised (a) F –, I – , Br–, Cl– (b) Cl –, Br – , I–, F–
molecule is : (c) I –, Br – , Cl–, F– (d) F –, Cl – , Br–, l–
(a) 1.2 Å (b) 1.39 Å 49. For AB bond if percent ionic character is plotted against
(c) 1.33 Å (d) 1.54 Å electronegativity difference (XA – XB), the shape of the curve
35. Which one of the following pairs of molecules will have would look like
permanent dipole moments for both members ? C
(a) NO2 and CO2 (b) NO2 and O3 100
Percent ionic character

B
(c) SiF4 and CO2 (d) SiF4 and NO2
A D
36. The correct sequence of increasing covalent character is
represented by 50
(a) LiCl < NaCl < BeCl2 (b) BeCl2 < LiCl < NaCl
(c) NaCl < LiCl < BeCl2 (d) BeCl2 < NaCl < LiCl
37. Which of the following would have a permanent dipole 0
moment? 1 2 3
(a) SiF4 (b) SF 4 (c) XeF4 (d) BF3 (XA – XB)
38. Dipole moment is shown by : The correct curve is
(a) cis-1, 2-dichloroethene (a) (A) (b) (B) (c) (C) (d) (D)
(b) trans-1, 2-dichloroethene 50. In which of the following compound electrovalent, covalent
(c) trans-2, 3-dichloro-2 pentene and co-ordinate bonds are present?
(d) Both (a) and (c)
39. The correct order of hybridisation of the central atom in the (a) NH 4 Cl (b) CCl 4 (c) CaCl 2 (d) H 2 O
following species : 51. Following are the molecules or ions of the type AH3 where A
NH3, [PtCl4]2–, PCl5 and BCl3 is is the central atom. In which of them does the central atom A
(a) dsp2, dsp3, sp2, sp3 (b) sp3, dsp 2, dsp3, sp2 use its sp2 hybrid orbitals to form A-H bonds ?
2 2 3
(c) dsp , sp , sp , dsp 3 (d) dsp2, sp3, sp2, sp3 (a) PH3 (b) NH3 (c) CH3+ (d) CH3–
EBD_7327
74 CHEMISTRY

52. Equilateral shape has 66. Hybridization present in CIF3 is :


(a) sp hybridisation (b) sp2 hybridisation (a) sp 2 (b) sp 3 (c) dsp2 (d) sp3 d
3
(c) sp hybridisation (d) None of these 67. Which one of the following has the pyramidal shape?
53. Which one of the following has the shortest carbon carbon (a) CO32– (b) SO3 (c) BF3 (d) PF3
bond length ? 68. Fluorine molecule is formed by
(a) Benzene (b) Ethene (c) Ethyne (d) Ethane (a) the axial p - p orbital overlap
54. Which of the following does not have a tetrahedral structure? (b) the sideways p - p orbital overlap
(c) the s - s orbital overlap
(a) BH–4 (b) BH3 (c) NH +4 (d) CH4 (d) the s - p orbital overlap
55. The bond length in LiF will be 69. In NO3– ion number of bond pair and lone pair of electrons
(a) less than that of NaF (b) more than that of KF on nitrogen atom respectively are
(c) equal to that of KF (d) equal to that of NaF (a) 2, 2 (b) 3, 1 (c) 1, 3 (d) 4, 0
56. Which of the following molecules does not have a linear 70. Among the following the pair in which the two species are
arrangement of atoms ? not isostructural is
(a) H2S (b) C2H2 (c) BeH2 (d) CO2 (a) SiF4 and SF4 (b) IO 3- and XeO 3
57. In which one of the following molecules the central atom
said to adopt sp2 hybridization? (c) BH -4 and NH +4 (d) PF6- and SF6
(a) BeF2 (b) BF3 (c) C2H2 (d) NH3
71. N 2 and O 2 are converted into monoanions, N 2– and O -2
58. In an octahedral structure, the pair of d orbitals involved in
respectively. Which of the following statements is wrong ?
d 2 sp3 hybridization is
(a) In N 2– , N – N bond weakens
(a) d 2 2 d 2
x -y , z
(b) d xz, d x 2 - y 2
(b) In O –2 , O - O bond order increases
(c) d z 2 , d xz (d) d xy, d yz
59. Which of the following shows linear structure? (c) In O –2 , O - O bond order decreases
(a) Ethane (b) Ethene (c) Acetylene (d) CCl 4 (d) N 2– becomes paramagnetic
60. The element X [1s2, 2s22p6, 3s23p 5] reacts with Y (atomic 72. In a regular octahedral molecule, MX6 the number of
no. = 1) to form X - M - X bonds at 180° is
(a) XY2 (b) X2Y (c) XY (d) None (a) three (b) two (c) six (d) four
61. Which one of the following does not follow octate rule? 73. Which of the following molecules has trigonal planar
(a) PF3 (b) BF3 (c) CO2 (d) CCl4 geometry?
62. In BrF3 molecule, the lone pairs occupy equatorial positions (a) BF3 (b) NH3 (c) PCl3 (d) IF3
to minimize 74. The pair of species having identical shapes for molecules of
(a) lone pair - bond pair repulsion only both species is
(b) bond pair - bond pair repulsion only (a) XeF2, CO2 (b) BF3, PCl3
(c) lone pair - lone pair repulsion and lone pair - bond pair (c) PF5, IF5 (d) CF4, SF4
repulsion 75. Which contains both polar and non - polar bonds?
(d) lone pair - lone pair repulsion only (a) NH4Cl (b) HCN
63. Match List I and List II and pick out correct matching codes (c) H2O2 (d) CH4
from the given choices : 76. The values of electronegativity of atoms A and B are 1.20
List I List II and 4.0 respectively. The percentage of ionic character of A
Compound Structure - B bond is
A. ClF3 1. Square planar (a) 50% (b) 72.24% (c) 55.3% (d) 43%
B: PCl5 2. Tetrahedral 77. The electronegativities of F, Cl, Br and l are 4.0, 3.0, 2.8, 2.5
C. IF5 3. Trigonal bipyramidal respectively. The hydrogen halide with a high percentage of
D. CCl4 4. Square pyramidal ionic character is
E. XeF4 5. T-shaped (a) HF (b) HCl (c) HBr (d) HI
Codes 78. Which one of the following has the regular tetrahedral
(a) A-5, B-4, C-3, D-2, E-1 (b) A-5, B-3, C-4, D-2, E-1 structure ?
(c) A-5, B-3, C-4, D-1, E-2 (d) A-4, B-3, C-5, D-2, E-1 (a) BF4- (b) SF4
64. Bond order in benzene is :
(a) 1 (b) 2 (c) XeF4 (d) [ Ni(CN ) 4 ]2-
(c) 1.5 (d) none of these (Atomic nos. : B = 5, S = 16, Ni =28, Xe = 54)
65. The decreasing values of bond angles from NH3 (106º) to 79. The species in which the central atom uses sp2 hybrid orbitals
SbH3 (101º) down group-15 of the periodic table is due to in its bonding is
(a) decreasing lp-bp repulsion +
(b) decreasing electronegativity (a) PH3 (b) NH3 (c) CH 3 (d) SbH3
(c) increasing bp-bp repulsion 80. Which molecule is planar ?
(d) increasing lp-bp repulsion (a) SF4 (b) XeF4 (c) NF3 (d) SiF4
Chemical Bonding and Molecular Structure 75

81. Amongst the following, the molecule/ion that is linear is : (c) sp - hybridized
(d) sp2 - hybridized but linked with one co-ordinate bond
(a) SO2 (b) CO2 (c) ClO-2 (d) NO2- 97. Which statement is NOT correct ?
82. The trigonal bipyramidal geometry is obtained from the (a) A sigma bond is weaker than a p -bond.
hybridisation : (b) A sigma bond is stronger than a p -bond.
(a) dsp 3 or sp 3d (b) dsp 2 or sp 2d (c) A double bond is stronger than a single bond.
(c) d 2sp3 or sp 3d 2 (d) None of these (d) A double bond is shorter than a single bond.
83. The true statements from the following are 98. Linear combination of two hybridized orbitals belonging to
1. PH5 and BiCl5 do not exist two atoms and each having one electron leads to a
2. pp - dp bond is present in SO2 (a) sigma bond
3. Electrons travel at the speed of light (b) double bond
4. SeF4 and CH4 have same shape (c) co-ordinate covalent bond
5. I3+ has bent geometry (d) Pi bond.
99. Main axis of a diatomic molecule is z, molecular orbital px
(a) 1, 3 (b) 1, 2, 5 and py overlap to form which of the following orbital?
(c) 1, 3, 5 (d) 1, 2, 4 (a) p - molecular orbital (b) s - molecular orbital
84. The hybrid state of S in SO3 is similar to that of
(c) d - molecular orbital (d) No bond will be formed
(a) C in C2H2 (b) C in C2H4
(c) C in CH4 (d) C in CO2 100. The number of anti-bonding electron pairs in O -2 2 molecular
85. Which of the following set contains species having same ion on the basis of molecular orbital theory is, (Atomic
angle around the central atom? number of O is 8)
(a) SF4, CH4, NH3 (b) NF3, BCl3, NH3 (a) 5 (b) 2 (c) 3 (d) 4
(c) BF3, NF3, AlCl3 (d) BF3, BCl3, BBr3 101. In which of the following species is the underlined carbon
86. The compound MX4 is tetrahedral. The number of Ð XMX having sp3 - hybridisation ?
formed in the compound are (a) CH 3 – COOH (b) CH 3 CH 2 OH
(a) Three (b) Four
(c) Five (d) Six (c) CH 3 COCH 3 (d) CH 2 = CH - CH 3
87. The bond angle between two hybrid orbitals is 105°. The 102. Which of the following statements is not correct for sigma
percentage of s-character of hybrid orbital is between and pi-bonds formed between two carbon atoms?
(a) 50 - 55% (b) 9 - 12% (a) Sigma-bond determines the direction between carbon
(c) 22 - 23% (d) 11 - 12% atoms but a pi-bond has no primary effect in this regard
88. Which of the following has the shortest C – C bond length? (b) Sigma-bond is stronger than a pi-bond
(a) C2H5OH (b) C2H6 (c) Bond energies of sigma- and pi-bonds are of the order
(c) C2H2 (d) C2H4 of 264 kJ/mol and 347 kJ/mol, respectively
(d) Free rotation of atoms about a sigma-bond is allowed
89. In piperidine , the hybrid state assumed by N is but not in case of a pi-bond
N 103. The bond order in N2+ is
H (a) 1.5 (b) 3.0 (c) 2.5 (d) 2.0
(a) sp (b) sp 2 (c) sp 3 (d) dsp 2 104. How many s- and p-bonds are there in?
90. The compound 1, 2 - butadiene has
(a) only sp hybridized carbon atoms
(b) only sp2 hybridized carbon atoms
(c) both sp and sp2 hybridized carbon atoms
(d) sp, sp2 and sp3 hybridized carbon atoms (a) 14s, 8p (b) 18s, 8p (c) 19s, 4p (d) 14s, 2p
91. In the following molecule, the two carbon atoms marked by 105. Which of the following has p π – d π bonding ?
asterisk (*) possess the following type of hybridized orbitals (a) NO 3– (b) SO 23- (c) BO 33- (d) CO 23-
H3C - C*º C*- CH3 106. How many s and p bonds are present in toluene?
(a) sp3 orbital (b) sp2 orbital (a) 3p + 8s (b) 3p + 10s
(c) sp orbital (d) sp3 and sp respectively (c) 3p + 15s (d) 6p + 3s
92. Which has the least bond angle 107. The calculated bond order in O2– ion is
(a) NH3 (b) BeF2 (c) H2O (d) CH4 (a) 1 (b) 1.5 (c) 2 (d) 2.5
93. The shape of SO 24- ion is 108. The molecular electronic configuration of H-2 ion is?
(a) square planar (b) tetrahedral
( s 1s ) 2 ( s*1s )
2
(c) trigonal bipyramidal (d) hexagonal (a) ( s 1s) 2 (b)
94. The largest bond angle is in
( s 1s ) 2 ( s *1s )
1
(a) AsH3 (b) NH3 (c) H2O (d) PH3 (c) (d) ( s 1s )3
95. Which one has a pyramidal structure
(a) CH4 (b) NH3 (c) H2O (d) CO2 109. In pyrophosphoric acid, H2P2O7, number of s and dp – pp
96. Each carbon in carbon suboxide (C3O2) is bonds are respectively
(a) sp2 - hybridized (a) 8 and 2 (b) 6 and 2
(b) sp3 - hybridized (c) 12 and zero (d) 12 and 2
EBD_7327
76 CHEMISTRY

110. Arrange the following ions in the order of decreasing X – O (a) NH3, NH3 (b) NH3,CH4
bond length, where X is the central atom in (c) H2O, CH3OCH3 (d) CH3OH,CH3OCH3
118. Dipole moment of CO2 is zero which implies that
(a) ClO -4 ,SO 24 - , PO34- ,SiO -4
(a) Carbon and oxygen have equal electronegativities
(b) SiO44 - , PO34- ,SO24 - , ClO-4 (b) Carbon has no polar bond
(c) CO2 is a linear molecule
(c) SiO 44 - , PO34- ,ClO4- ,SO42 - (d) Carbon has bond moments of zero value
119. In which of the following pairs, the two species are iso-
(d) SiO44 - ,SO42- , PO34- , ClO-4 structure?
111. In the change of NO+ to NO, the electron is added to (a) SO32– and NO3– (b) BF3 an NF3
(a) s - orbital (b) p - orbital (c) BrO3– and XeO3 (d) SF4 and XeF4
(c) s* - orbital (d) p* - orbital 120. Four diatomic species are listed below in different sequences.
112. Which of the following represents the given mode of Which of these presents the correct order of their increasing
hybridisation sp2 –sp2 – sp - sp from left to right ? bond order ?
(a) H 2 C = CH - C º N (b) HC º C – C º CH (a) O-2 < NO < C22- < He+2 (b) NO < C22 - < O 2- < He +2
CH2 (c) C 22 - < He +2 < NO < O -2 (d) He+2 < O 2- < NO < C 22 -
(c) H 2 C = C = C = CH 2 (d) H C 121. What is the dominant intermolecular force or bond that must
2
be overcome in converting liquid CH3OH to a gas?
113. In O-2 , O2 and O -2 2 molecular species, the total number of (a) Dipole-dipole interactions
antibonding electrons respectively are (b) Covalent bonds
(a) 7, 6, 8 (b) 1, 0, 2 (c) 6, 6, 6 (d) 8, 6, 8 (c) London dispersion forces
114. If Nx is the number of bonding orbitals of an atom and Ny is (d) Hydrogen bonding
the number of antibonding orbitals, then the molecule/atom 122. In which one of the following species the central atom has
will be stable if the type of hybridization which is not the same as that present
(a) Nx > Ny (b) Nx = Ny (c) Nx < Ny (d) Nx £ Ny in the other three?
115. Which of the following is the correct electron dot structure (a) SF4 (b) I3– (c) SbCl52– (d) PCl5
of N2O molecule? 123. Some of the properties of the two species, NO3- and H3O+
.. + .. are described below. Which one of them is correct?
(a) : N = N = O : (b) : N = N = O -
.. : (a) Similar in hybridization for the central atom with different
.. .. .. .. structures.
(c) N = N = O : (d) : N = N = O : (b) Dissimilar in hybridization for the central atom with
.. ..
different structures.
116. The correct statement with regard to H+2 and H-2 is (c) Isostructural with same hybridization for the central
atom.
(a) Both H+2 and H-2 do not exist (d) Isostructural with different hybridization for the central
(b) H-2 is more stable than H+2 atom.
124. In which of the following molecules the central atom does
(c) H+2 is more stable than H-2 not have sp3 hybridization?
(a) NH +4 (b) CH4 (c) SF4 (d) BF4–
(d) Both H+2 and H-2 are equally stable 125. Which one of the following species does not exist under
117. In which of the following pairs hydrogen-bonding is not normal conditions?
possible ? (a) Be +2 (b) Be2 (c) B2 (d) Li 2
Chemical Bonding and Molecular Structure 77

1. In the anion HCOO– the two carbon - oxygen bonds are found 9. The enolic form of a acetone contains
to be of equal length. What is the reason for it ? (a) 9 sigma bonds, 1 pi bond and 2 lone pairs
(a) Electronic orbitals of carbon atom are hybridised (b) 8 sigma bonds, 2 pi bonds and 2 lone pairs
(b) The C = O bond is weaker than the C –O bond (c) 10 sigma bonds, 1 pi bond and 1 lone pair
(c) The anion HCOO– has two resonating structures (d) 9 sigma bonds, 2 pi bonds and 1 lone pair
(d) The anion is obtained by removal of a proton from the 10. In which case hydrogen bond will not be observed
acid molecule (a) H3 O2 – (b) H2O
2. In compounds of type ECl3, where E = B, P, As or Bi, the (c) H5O2 + (d) H3O+
angles Cl - E- Cl for different E are in the order. 11. Which one of the following formulae does not correctly
(a) B > P = As = Bi (b) B > P > As > Bi represent the bonding capacities of the two atoms involved?
(c) B < P = As = Bi (d) B < P < As < Bi +
3. Molecular shapes of SF4, CF4 and XeF4 are é H ù
ê | ú
(a) the same, with 2, 0 and 1 lone pairs of electrons êH — P — H ú
(a) | (b) F F
respectively ê ú
ë H û O
(b) the same, with 1, 1 and 1 lone pairs of electrons
respectively
H
(c) different, with 0, 1 and 2 lone pairs of electrons O | O
respectively (c) O¬N (d) H – C = C
(d) different, with 1, 0 and 2 lone pairs of electrons O–H O–H
respectively N
12. Trimethylamine is a pyramidal molecule CH 3
4. The molecules BF3 and NF3 are both covalent compounds,
H 3C CH 3
but BF3 is non polar whereas NF3 is polar. The reason for
O
this is
||
(a) atomic size of Boron is larger than nitrogen C H
(b) Boron is metal while nitrogen is gas and formamide is a planar molecule , The
H N
(c) B – F bonds are non-polar while N – F bonds are polar |
(d) BF3 is planar but NF3 is pyramidal H
5. The common features among the species CN–, CO and NO+ hybridisation of Nitrogen in both is
are
(a) bond order three and isoelectronic (a) sp 2 , sp 2 (b) sp 3 , sp 2
(b) bond order three and weak field ligands (c) sp 3 , sp 3 (d) sp 2 , sp
(c) isoelectronic and weak field ligands
(d) None of these 13. If climbing of water droplets is made to occur on a coated
6. The correct order of bond energies in NO, NO+ and NO– is: microscope slide, the slide would have to be coated in which
of the following way
(a) NO - > NO > NO + (b) NO > NO - > NO +
(c) NO + > NO > NO - (d) NO + > NO - > NO
7. A s bonded molecule MX 3 is T-shaped. The number of
non bonded pair of electrons is A B C D
(a) 0 (a) A (b) B
(b) 2 (c) C (d) D
(c) 1 14. Mark the incorrect statement in the following
(d) can be predicted only if atomic number is known (a) The bond order in the species O2 , O2 + and O2 –
8. Experiment shows that H2O has a dipole moment while CO2
has not. Point out the structures which best illustrate these decreases as O +2 > O 2 > O -2
facts (b) The bond energy in a diatomic molecule always
O increases when an electron is lost
(c) Electrons in antibonding M.O. contribute to repulsion
(a) O = C = O ; H H (b) O = C = O ; H – O – H
between two atoms.
C O H (d) With increase in bond order, bond length decreases
(c) ;H – H –O (d)
O O C = O; O–H and bond strength increases.
EBD_7327
78 CHEMISTRY

15. Which of the following is not expected to be resonance (b) two sigma C – C and one 'p' C – C bonds
structure ? (c) three 'p' C – C bonds
– + + .–. (d) one sigma C– C and two 'p' C – C bonds
(a) :CH –N = .O:. (b) CH2 =N – O:
.. 22. N2 and O2 are converted into monocations, N2+ and O2+
2
CH3 CH3 respectively. Which of the following statements is wrong ?
+ .–. (a) In N2+, the N—N bond weakens
(c) CH2 =N – O: .. (d) CH2 =N = O (b) In O2+, the O—O bond order increases
CH3 CH3 (c) In O2+, paramagnetism decreases
(d) N2+ becomes diamagnetic
16. The boiling point of p-nitrophenol is higher than that of
23. The AsF5 molecule is trigonal bipyramidal. The hybrid orbitals
o-nitrophenol because
used by the As atom for bonding are
(a) NO2 group at p-position behave in a different way from
that at o-position. (a) d , d z2 , s, px , p y (b) dxy, s, px, py, pz
x2 - y 2
(b) intramolecular hydrogen bonding exists in p-
nitrophenol (c) s, px, py, pz, dz2 d 2 2 , s, px, py, pz
(d)
x -y
(c) there is intermolecular hydrogen bonding in 24. The dipole moments of diatomic molecules AB and CD are
p-nitrophenol 10.41D and 10.27 D, respectively while their bond distances
(d) p-nitrophenol has a higher molecular weight than are 2.82 and 2.67 Å, respectively. This indicates that
o-nitrophenol. (a) bonding is 100% ionic in both the molecules
17. Which is the correct order of dipole moments of the (b) AB has more ionic bond character than CD
compounds : (c) AB has lesser ionic bond character than CD
NO2 (d) bonding is nearly covalent in both the molecules
Cl
25. Consider the two molecules :
(I) (II)
H H H
OH Cl
H3C C H3C C CH3
CH3
(I) C C CH3 (II) C C .
(III) H
O O O O
CH3
H H
(a) I > II > III (b) II > I > III
(c) III > I > II (d) III > II > I Which of the following statements about them is true ?
(a) II is capable of forming intramolecular hydrogen
X bonding whereas I is not
(b) Both are capable of forming equally stable
18. The dipole moment of is 1.5 D. The dipole moment of intramolecular hydrogen bonding
(c) I is likely to form more stable intramolecular hydrogen
bonding than II
X (d) Both of them cannot form intramolecular hydrogen
bonding
is :
26. The hybridization of P is PO 34- is the same as of
X X (a) S in SO3 (b) N in NO–3
X +
(c) I in ICl 2 (d) I in ICl4–
(a) 1 D (b) 1.5 D 27. The relationship between the dissociation energy of N2 and
(c) 2.25 D (d) 3 D N2+ is :
19. The BCl3 is a planar molecule whereas NCl3 is pyramidal (a) Dissociation energy of N2+ > dissociation energy of
because N2
(a) B-Cl bond is more polar than N-Cl bond (b) Dissociation energy of N2 = dissociation energy of
(b) N-Cl bond is more covalent than B-Cl bond N2+
(c) nitrogen atom is smaller than boron atom (c) Dissociation energy of N2 > dissociation energy of
(d) BCl3 has no lone pair but NCl3 has a lone pair of N2+
electrons (d) Dissociation energy of N2 can either be lower or higher
20. Dipole moment of p-nitroaniline, when compared to than the dissociation energy of N2+
nitrobenzene (X) and aniline (Y) will be : 28. In X — H --- Y, X and Y both are electronegative elements
(a) smaller than both (X) and (Y) (a) thus electron density on X will increase and on H will
(b) greater than both (X) and (Y) decrease
(c) greater than (Y) but smaller than (X) (b) in both electron density will decrease
(d) equal to zero (c) in both electron density will increase and on H will
21. The cylindrical shape of an alkyne is due to the fact that it decrease
has (d) thus electron density will decrease on X and will
(a) three sigma C – C bonds increase on H
Chemical Bonding and Molecular Structure 79

29. The electronegativity difference between N and F is greater


(a) ClO4– = ClO3– = ClO 2– = ClO –
than that between N and H yet the dipole moment of
NH3 (1.5 D) is larger than that of NF3 (0.2D). This is because (b) ClO – < ClO2– < ClO3– < ClO4–
(a) in NH3 the atomic dipole and bond dipole are in the
same direction whereas in NF3 these are in opposite (c) ClO4– < ClO3– < ClO2– < ClO –
directions
(b) in NH3 as well as NF3 the atomic dipole and bond dipole (d) ClO3– < ClO4– < ClO2– < ClO –
are in opposite directions 39. The correct order of increasing bond angles in the following
(c) in NH3 the atomic dipole and bond dipole are in the triatomic species is :
opposite directions whereas in NF3 these are in the (a) NO -2 < NO +2 < NO 2 (b) NO-2 < NO2 < NO2+
same direction (c) NO+2 < NO 2 < NO2- (d) NO+2 < NO-2 < NO2
(d) in NH3 as well as in NF3 the atomic dipole and bond 40. In which of the following molecules / ions BF 3, NO2- , NH 2-
dipole are in the same direction
and H2O , the central atom is sp2 hybridized ?
30. In the process : XH 3 + H + ¾ ¾® XH +4 , (X being N, P, As (a) NH 2- and H2O (b) NO 2- and H2O
and Sb), the maximum increase in the bond angle will be in (c) BF3 and NO2 - (d) NO2- and NH 2-
case of
41. According to MO theory which of the following lists ranks
(a) NH3 (b) PH3 the nitrogen species in terms of increasing bond order?
(c) AsH3 (d) SbH3
31. On changing N2 to N2+, the dissociation energy of N–N bond (a) N 2– –
2 < N2 < N2 (b) N 2 < N 22– < N 2–
..... and on changing O2 to O+2 the dissociation energy of O– (c) N 2– < N 2– (d) N 2– < N 2 < N 2–
2 < N2 2
O bond....
42. Which of the two ions from the list given below have the
(a) increases, decreases (b) decreases, increases geometry that is explained by the same hybridization of
(c) decreases in both cases(d) increases in both cases orbitals, NO2–, NO3–, NH2–, NH4+, SCN– ?
32. H2O is dipolar, whereas BeF2 is not. It is because (a) NO2– and NO3– (b) NH2– and NO3–
(a) the electronegativity of F is greater than that of O (c) SCN– and NH2– (d) NO2– and NH2–
(b) H2O involves hydrogen bonding whereas BeF2 is a 43. Which of the following has the minimum bond length ?
discrete molecule (a) O2+ (b) O2 – (c) O2 2– (d) O2
(c) H2O is linear and BeF2 is angular 44. The pairs of species of oxygen and their magnetic behaviours
(d) H2O is angular and BeF2 is linear are noted below. Which of the following presents the correct
33. The bond order in NO is 2.5 while that in NO+ is 3. Which of description ?
the following statements is true for these two species ? (a) O -2 , O 22 - – Both diamagnetic
(a) Bond length in NO+ is equal to that in NO
(b) Bond length in NO is greater than in NO+ (b) O +2 ,O22 - – Both paramagnetic
(c) Bond length in NO+ is greater than in NO +
(c) O2 ,O2 – Both paramagnetic
(d) Bond length is unpredictable (d) None of these
34. Which is correct about the directional orientation of the 45. Which one of the following pairs is isostructural (i.e., having
orbital? the same shape and hybridization) ?
(a) p > sp (b) p < sp é BCl3 and BrCl3- ù é NH3 and NO3- ù
(a) ë û (b) ë û
(c) p = sp (d) none of these
35. Pb4+ is less stable than Sn4+. It is because of (c) [ NF3 and BF3 ] (d)
- +
é BF4 and NH 4 ù
(a) Higher value of IE1 + IE2 + IE3 + IE4 for Pb than Sn ë û
(b) Lesser polarising power of Pb4+ than Sn4+ 46. Bond order of 1.5 is shown by :
(c) Inert pair effect in Pb (a) O +2 (b) O 2-
(d) None of these (c) O 22 - (d) O2
36. According to molecular orbital theory which of the following
47. Which of the following hydrogen bonds is the strongest?
statement about the magnetic character and bond order is (a) O – H - - - F (b) O – H - - - H
correct regarding O +2 (c) F – H - - - F (d) O – H - - - O
(a) Paramagnetic and Bond order < O2 48. The bond dissociation energy of B – F in BF3 is 646 kJ mol–1
(b) Paramagnetic and Bond order > O2 whereas that of C – F in CF4 is 515 kJ mol–1. The correct
(c) Diamagnetic and Bond order < O2 reason for higher B – F bond dissociation energy as compared
(d) Diamagnetic and Bond order > O2 to that of C – F is
37. Select the most stable cation (a) stronger s bond between B and F in BF3 as compared
to that between C and F in CF4.
(a) NH +4 (b) CH 5+ (b) significant pp – pp interaction between B and F in BF3
whereas there is no possibility of such interaction
(c) OH 3+ (d) FH +2 between C and F in CF4.
38. Which of the following represents the correct order of Cl–O (c) lower degree of pp – pp interaction between B and F in
BF3 than that between C and F in CF4.
bond lengths in ClO – , ClO 2– , ClO3– , ClO 4– ? (d) smaller size of B– atom as compared to that of C– atom.
EBD_7327
80 CHEMISTRY

49. The number of types of bonds between two carbon atoms in 62. Which of the following species contains three bond pairs
calcium carbide is and one lone pair around the central atom ?
(a) One sigma, One pi (b) Two sigma, one pi (a) H2O (b) BF3 (c) NH -2 (d) PCl3
(c) Two sigma, two pi (d) One sigma, two pi
63. The pair of species with the same bond order is :
50. In which of the following pairs the two species are not
isostructural ? (a) O 2–
2 , B2 (b) O +2 , NO+
(c) NO, CO (d) N2, O2
(a) -
CO32 - and NO3 (b) PCl+4 and SiCl4 64. Which of the following species exhibits the diamagnetic
(c) PF5 and BrF5 (d) AlF63 - and SF6 behaviour ?
51. Among the following, the paramagnetic compound is (a) NO (b) O2 2– (c) O2+ (d) O2
65. The charge/size ratio of a cation determines its polarizing
(a) Na2O2 (b) O3 power. Which one of the following sequences represents
(c) N2O (d) KO2 the increasing order of the polarizing power of the cationic
52. The species having bond order different from that in CO is species, K+, Ca2+, Mg2+, Be2+ ?
(a) NO– (b) NO+ (a) Ca2+ < Mg2+ < Be2+ < K+
(c) CN– (d) N2 (b) Mg2+ < Be2+ < K+ < Ca2+
53. Assuming that Hund’s rule is violated, the bond order and (c) Be2+ < K+ < Ca2+ < Mg2+
magnetic nature of the diatomic molecule B2 is : (d) K+ < Ca2+ < Mg2+ < Be2+
(a) 1 and diamagnetic (b) 0 and dimagnetic 66. In which of the following ionization processes, the bond
(c) 1 and paramagnetic (d) 0 and paramagnetic order has increased and the magnetic behaviour has
54. The species having pyramidal shape is : changed?
(a) SO3 (b) BrF3 (a) N2 ® N2+ (b) C 2 ® C2 +
(c) SiO32– (d) OsF2 (c) NO ® NO + (d) O 2 ® O 2 +
67. Which one of the following pairs of species have the same
55. Geometrical shapes of the complexes formed by the reaction bond order?
of Ni2+ with Cl– , CN– and H2O, respectively, are (a) CN– and NO+ (b) CN– and CN+
(a) octahedral, tetrahedral and square planar
(c) O-2 and CN– (d) NO+ and CN+
(b) tetrahedral, square planar and octahedral
(c) square planar, tetrahedral and octahedral 68. The molecule having smallest bond angle is :
(d) octahedral, square planar and octahedral (a) NCl3 (b) AsCl3
(c) SbCl3 (d) PCl3
56. In allene (C3H4), the type(s) of hybridisation of the carbon
atoms is (are) : 69. The electronegativities of four atoms labeled as D, E, F and
(a) sp and sp3 (b) sp and sp2 G are as follows. D = 3.8, E = 3.3, F = 2.8 and G = 1.3. If the
(c) only sp 3 (d) sp2 and sp3 atoms form the molecules DE, DG, EG and DF, the order of
57. Which one of the following molecules is expected to exhibit arrangement of these molecules in the increasing order of
diamagnetic behaviour ? covalent bond character is
(a) C2 (b) N2 (a) DG < EG < DF < DE (b) DF < DG < DE < EG
(c) O2 (d) S2 (c) DG < DF < EG < DE (d) DE < EG < DG < DF
58. Which of the following is the wrong statement ? 70. Which one of the following is the correct statement?
(a) ONCl and ONO– are not isoelectronic. (a) O2 molecule has bond order 2 and is diamagnetic
(b) O3 molecule is bent (b) N2 molecule has bond order 3 and is paramagnetic
(c) Ozone is violet-black in solid state (c) H2 molecule has bond order zero and is diamagnetic
(d) Ozone is diamagnetic gas. (d) C2 molecule has bond order 2 and is diamagnetic
59. In which of the following pairs of molecules/ions, both the 71. Among the following molecules : SO2, SF4, CIF3, BrF5 and
species are not likely to exist ? XeF4, which of the following shapes does not describe any
(a) H +2 , He 22- (b) H -2 , He 22 - of the molecules mentioned?
(a) Bent (b) Trigonal bipyramidal
(c) H 22 + , He2 (d) H -2 , He 22 + (c) See-saw (d) T-shape
- + 72. The type of hybridization in xenon atom and the number of
60. Stability of the species Li2, Li 2 and Li 2 increases in the lone pairs present on xenon atom in xenon hexafluoride
order of : molecule are respectively
(a) Li 2 < Li 2+ < Li 2- (b) Li -2 < Li +2 < Li 2 (a) sp3d3 , one (b) sp3d3 , two
3 3
(c) sp d , two (d) sp3d2 , zero
(c) Li 2 < Li 2- < Li 2+ (d) Li -2 < Li 2 < Li 2+
61. Considering the state of hybridization of carbon atoms, find DIRECTIONS for Qs. 73 to 75: These are Assertion-Reason
out the molecule among the following which is linear ? type questions. Each of these question contains two statements:
(a) CH3– CH = CH–CH3 Statement-1 (Assertion) and Statement-2 (Reason). Answer these
(b) CH3 – C º C – CH3 questions from the following four options.
(c) CH2 = CH – CH2 – C º CH (a) Statement- 1 is True, Statement-2 is True, Statement-2 is a
(d) CH3 – CH2 – CH2 – CH3 correct explanation for Statement -1
Chemical Bonding and Molecular Structure 81

(b) Statement -1 is True, Statement -2 is True ; Statement-2 is 74. Statement-1 : A resonance hybrid is always more stable than
NOT a correct explanation for Statement - 1 any of its canonical structures.
(c) Statement - 1 is True, Statement- 2 is False Statement-2 : This stability is due to delocalization of
(d) Statement -1 is False, Statement -2 is True electrons.
75. Statement-1 : o and p-nitrophenols can be separated by steam
73. Statement-1 : Ice is less dense than liquid water.
distillation.
Statement-2 : There are vacant spaces between hydrogen Statement-2 : o-nitrophenol have intramolecular hydrogen
bonded water molecules in ice. bonding while p-nitrophenol exists as associated molecules.

Exemplar Questions (a) 6, 19 (b) 4, 20


(c) 5, 19 (d) 5, 20
1. Isostructural species are those which have the same shape
and hybridisation. Among the given species identify the 9. Which molecule/ion out of the following does not contain
isostructural pairs. unpaired electrons?
(a) N +2 (b) O2
(a) [NF3 and BF3] (b) [BF4- and NH +4 ]
(c) O 22 - (d) B2
(c) [BCl3 and BrCl3] (d) [NH3 and NO3- ] 10. In which of the following molecule/ion all the bonds are not
2. Polarity in a molecule and hence the dipole moment depends equal?
primarily on electronegativity of the constituent atoms and (a) XeF4 (b) BF4-
shape of a molecule. Which of the following has the highest
(c) C2H4 (d) SiF4
dipole moment?
(a) CO2 (b) HI 11. In which of the following substances will hydrogen bond be
(c) H2O (d) SO2 strongest?
(a) HCl (b) H2O
3. The types of hybrid orbitals of nitrogen in NO+2 , NO3- and
(c) HI (d) H2S
NH +4 respectively are expected to be
12. If the electronic configuration of an element is
(a) sp, sp3 and sp2 (b) sp, sp2 and sp3
1s2 2s2 2p6 3s2 3p6 3d2 4s2, the four electrons involved in
(c) sp2, sp and sp3 (d) sp2, sp3 and sp
chemical bond formation will be
4. Hydrogen bonds are formed in many compounds e.g., H2O,
HF, NH3. The boiling point of such compounds depends to (a) 3p6 (b) 3p6, 4s2
6
(c) 3p , 3d 2 (d) 3d2, 4s2
an extent on the strength of hydrogen bond and the number
of hydrogen bonds. The correct decreasing order of the 13. Wh ich of the following angle corresponds to sp 2
boiling points above compounds is hybridisation?
(a) HF > H2O > NH3 (b) H2O > HF > NH3 (a) 90° (b) 120°
(c) NH3 > HF > H2O (d) NH3 > H2O > HF (c) 180° (d) 109°
5. In PO34- ion the formal charge on the oxygen atom of Direction (Q. no. 14-17) The electronic configurations of the
P – O bond is elements A, B and C are given below. Answer the questions from
(a) + 1 (b) – 1 14 to 17 on the basis of these configurations.
(c) – 0.75 (d) + 0.75 A 1s2 2s2 2p6
6. In NO -3 ion, the number of bond pairs and lone pairs of B 1s 2 2s2 2p6 3s2 3p3
C 1s 2 2s2 2p 6 3s2 3p5
electrons on nitrogen atom are
(a) 2, 2 (b) 3, 1 14. Stable form of A may be represented by the formula
(c) 1, 3 (d) 4, 0 (a) A (b) A 2
7. Which of the following species has tetrahedral geometry? (c) A 3 (d) A 4
(a) BH -4 (b) NH -2 15. Stable form of C may be represented by the formula
(a) C (b) C2
(c) CO32 - (d) H3 O+
(c) C3 (d) C4
8. Number of p bonds and s bonds in the following structure is
H H 16. The molecular formula of the compound formed from B and
C will be
H H (a) BC (b) B2C
(c) BC2 (d) BC3
17. The bond between B and C will be
H H (a) ionic (b) covalent
H H (c) hydrogen (d) coordinate
EBD_7327
82 CHEMISTRY

18. Which of the following order of energies of molecular orbitals 27. In which of the following pair both the species have sp3
of N2 is correct? hybridization? [NEET Kar. 2013]
(a) (p2py ) < (s2pz ) < (p*2px ) » (p*2py ) (a) H2S, BF3 (b) SiF4, BeH2
(b) (p2py ) > (s2pz ) > (p*2px ) » (p*2py )
(c) NF3, H2O (d) NF3, BF3
(c) (p2py ) < (s2pz) < (p*2px) » (p*2py ) 28. XeF2 is isostructural with [2013]
(d) (p2py ) > (s2pz) < (p*2px) » (p*2py ) (a) ICl2– (b) SbCl3
19. Which of the following statement is not correct from the (c) BaCl2 (d) TeF2
view point of molecular orbital theory?
29. Which of the following is paramagnetic ? [2013]
(a) Be2 is not a stable molecule
(a) O -2 (b) CN–
(b) He2 is not stable but He +2 is expected to exist
(c) NO+ (d) CO
(c) Bond strength of N 2 is maximum amongst the 30. Which one of the following molecules contains no p bond?
homonuclear diatomic molecules belonging to the [2013]
second period (a) H2O (b) SO2
(d) The order of energies of molecular orbitals in N2
molecule is (c) NO2 (d) CO2
s2s < s*2s < s2pz < (p2px ; p2py) < (p*2px ; p*2py ) 31. Which of the following is electron - deficient ? [2013]
< s*2pz (a) (SiH3)2 (b) (BH3)2
20. Which of the following options represents the correct bond (c) PH3 (d) (CH3)2
order?
(a) O -2 > O 2 > O 2+ (b) O -2 < O 2 < O2+ 32. Be2+ is isoelectronic with which of the following ions?
[2014]
(c) O -2 > O 2 < O 2+ (d) O -2 < O 2 > O 2+ (a) H + (b) Li+
21. The electronic configuration of the outer most shell of the (c) Na+ (d) Mg2+
most electronegative element is 33. Which of the following molecules has the maximum dipole
(a) 2s22p5 (b) 3s23p5 moment ? [2014]
2
(c) 4s 4p 5 (d) 5s25p5 (a) CO2 (b) CH4
22. Amongst the following elements whose electronic (c) NH3 (d) NF3
configuration are given below, the one having the highest 34. Which one of the following species has plane triangular
ionisation enthalpy is shape ? [2014]
(a) [Ne]3s23p1 (b) [Ne]3s23p3 (a) N3 – (b) NO3–
(c) NO2– (d) CO2
(c) [Ne]3s23p2 (d) [Ar]3d104s24p3
35. The correct bond order in the following species is: [2015]
NEET/AIPMT (2013-2017) Questions
(a) O 22+ < O2– < O 2+ (b) O +2 < O 2– < O 22+
23. Which of the following is a polar molecule ? [2013]
(c) O 2– < O 2+ < O 22+ (d) O 22+ < O 2+ < O 2–
(a) SF4 (b) SiF4
36. Which of the following pairs of ions are isoelectronic and
(c) XeF4 (d) BF3 isostructural ? [2015]
24. The outer orbitals of C in ethene molecule can be considered
to be hybridized to give three equivalent sp2 orbitals. The
(a) ClO3– , CO32– (b) SO32– , NO3–
total number of sigma (s) and pi (p) bonds in ethene molecule (c) ClO3– , SO32– (d) CO32– , SO32–
is [NEET Kar. 2013]
(a) 1 sigma (s) and 2 pi (p) bonds 37. Which of the following options represents the correct bond
(b) 3 sigma (s) and 2 pi (p) bonds order ? [2015]
(c) 4 sigma (s) and 1 pi (p) bonds (a) O 2– < O 2 < O 2+ (b) O –2 > O2 < O 2+
(d) 5 sigma (s) and 1 pi (p) bonds
(c) O –2 < O2 > O 2+ (d) O –2 > O2 > O2+
25. In which of the following ionisation processes the bond
energy increases and the magnetic behaviour changes from 38. Maximum bond angle at nitrogen is present in which of the
paramagnetic to diamagnetic? [NEET Kar. 2013] following ? [2015]
(a) N2 ® N2 + (b) O2 ® O2 + (a) NO 2– (b) NO +2
(c) C2 ® C2 + (d) NO ® NO+ (c) NO3– (d) NO2
26. The pair of species that has the same bond order in the 39. Which of the following species contains equal number of s-
following is: [NEET Kar. 2013] and p-bonds : [2015]
(a) O2, B2 (b) CO, NO+ (a) XeO4 (b) (CN)2
(c) NO–, CN– (d) O2, N2 (c) CH2(CN)2 (d) HCO3–
Chemical Bonding and Molecular Structure 83

40. Decreasing order of stability of O2, O, ∗ 2,


2 ,O 2 and O2 is :
(c) The H–O–H bond angle in H2O is smaller than the H–
N–H bond angle in NH3.
(a) O∗ , 2,
2 = O2 = O2 = O2 [2015 RS] (d) The H–C–H bond angle in CH4 is larger than the H–
N–H bond angle in NH3.
(b) O 22, = O, ∗
2 = O2 = O2
44. Predict the correct order among the following : [2016]
(c) O2 = O∗ 2, ,
2 = O2 = O2 (a) lone pair- lone pair > lone pair - bond pair > bond pair -
bond pair
(d) O, 2, ∗
2 = O2 = O2 = O2 (b) lone pair - lone pair > bond pair - bond pair > lone pair
41. In which of the following pairs, both the species are not - bond pair
isostructural ? [2015 RS] (c) bond pair - bond pair > lone pair - bond pair > lone pair
- lone pair
(a) SiCl 4 , PCl∗
4 (d) lone pair - bond pair > bond pair - bond pair > lone pair
(b) diamond, silicon carbide - lone pair
(c) NH3, PH3 45. Which of the following pairs of compounds is isoelectronic
(d) XeF4, XeO4 and isostructural ? [2017]
(a) TeI2,XeF2 (b) IBr2- , XeF2
42. The hybridization involved in complex [Ni(CN)4]2– is
(At. No. Ni = 28) [2015 RS] (c) IF3, XeF2 (d) BeCl2,XeF2
(a) dsp 2 (b) sp3 46. The species, having bond angles of 120° is :- [2017]
(c) d2 sp 2 (d) d2 sp 3 (a) CIF3 (b) NCl3
43. Consider the molecules CH4, NH3 and H2O. Which of the (c) BCl3 (d) PH3
given statements is false? [2016] 47. Which of the following pairs of species have the same bond
(a) The H–C–H bond angle in CH4, the H–N–H bond angle order ? [2017]
in NH3, and the H–O–H bond angle in H2O are all (a) O2, NO+ (b) CN–, CO
greater than 90°
(c) N 2 , O 2- (d) CO, NO
(b) The H–O–H bond angle in H2O is larger than the H–
C–H bond angle in CH4.
EBD_7327
84 CHEMISTRY

Hints & Solutions


EXERCISE - 1 So, in CuSO 4 .5H2 O one electrovalent bond, 5
1. (b) Dative bond is formed between two atoms when the coordinate bonds and covalent bonds are present.
electrons of shared pair are contributed solely by one 9. (c) Higher the difference in electronegativity between the
of the two atoms and it is directed toward that atom two atoms, more will be electrovalent character of the
which do not contribute the electrons. It is represented bond. Among given choices, calcium and hydrogen
by an arrow on the bond which is directed towards the have maximum difference in their electronegativities.
electron recipient atom. 10. (c) KF being highly ionic compound will dissolve most
readily in water. (Like disolves like).
+
So, - N º NCl– is the only group among the given 11. (d) When a metal for example Na combines with a non
metal e.g., Cl2. Following reaction occurs
options which does not contain a dative bond.
2Na + Cl 2 ¾¾ ® 2NaCl
O In this process Na loses one electron to form Na+ and
+
(a) N ; (b) - N º N - Cl - ; Cl accepts one electron to form Cl–
O Na ¾¾ ® Na + + e -
(c) –N C
2. (b) The stability of the ionic bond depends upon the lattice Cl + e - ¾¾® Cl -
energy which is expected to be more between Mg and Therefore, in this process Cl gain electrons and hence
F due to +2 charge on Mg atom. its size increases.
3. (d) In CCl4, each C – Cl bond is polar but due to the very 12. (a)
symmetrical tetrahedral arrangement they exactly cancel 13. (b) Hybridisation of the central atom in compound is given
each other and hence CCl4 have zero dipole moment. by

Cl 1
H= [V + M - C + A]
2
Cl where V = no. of valency electrons in central metal
C
atom,
Cl Cl M = no. of monovalent atoms surrounding the central
So, none on these i.e. option (d) is correct choice. atom,
C = charge on cation and A = charge on anion
4. (b) For compounds containing cations of same charge,
1
lattice energy increases as the size of the cation · For NO -2 , H = [5 + 0 - 0 + 1] = 3
decrease. Thus, NaF has highest lattice energy. The 2
size of cations is in the order Na+ < K+ < Rb+ < Cs+ sp2 hybridisation
5. (b) In KCN, K+ and CN– have ionic bond 1
and C º N has covalent bond. · For SF4, H = [6 + 4 - 0 + 0] = 5
..
2
6. (d) H–O H – Cl Cl – Ba – Cl H 2 N– NH 2 sp3d hybridisation
| ¯ 1
H H+ · For PF6–, H =[5 + 6 - 0 + 1] = 6
(a) (b) (c) (d) 2
7. (d) The strength of a bond depends upon the extent of sp3d2 hybridisation.
overlapping. s-s and s-p overlapping results in the So, option (a) is correct choice.
formation of s bond but extent of overlapping along 14. (c) Bond order between P – O
internuclear axis is more in case of s-s overlapping than no. of bonds in all possible direction 5
in s-p. p-p overlapping may result in s bond if = = = 1.25
total no. of resonating structures 4
overlapping takes place along internuclear axis or may
result in p–bond if sideways overlapping takes place. O O–
In any case the extent of overlapping is lesser in p - p – – –
O P O O P O
than that of the other two, s - s and s-p. Hence the
correct order is O

O

s - s > s - p > p - p. O– O–
8. (d) CuSO4.5H2O can be represented as –
O P

O

O P O
OH2 2+ O O

H2O OH2
2–
SO 4 3
Cu Formal charge on oxygen = - = - 0.75
H2O OH2 4
Chemical Bonding and Molecular Structure 85

15. (a) A compound having element with highest 23. (c) NH3 undergoes H-bonding and hence has the highest
electronegativity will form strongest hydrogen bond. b.p. Among the remaining hydrides i.e. PH3, AsH3 and
16. (a) Bond angle increases with increase in s-character of SbH3 as we move from PH3 to BiH3, the molecular mass
hybridised orbital. The table given below shows the increases. As a result the van der waal’s forces of
hybridised orbitals, their % s-chatracter and bond attraction increases and the boiling point increases
angles. regularly from PH3 to BiH3.
Hybridised % s-character Bond 24. (a) Resonating structures can be written only for such
orbitals angle molecules in which multiple bonds are present, eg, O3
sp 3 25 109.5°
sp 2 33 120°
sp 50 180°
17. (a) Boron in BCl3 has 6 electrons in outermost shell. Hence 25. (c)
BCl3 is a electron deficient compound. 26. (d) HF form linear polymeric structure due to hydrogen
18. (d) The structure of P4O10 is bonding.
O 27. (a) Square Plannar Structure : XeF4 and [PtCl4]2–
|| 28. (c) In one – C º N, No. of p bonds = 2
P
So in [Ag (CN 2 )] – , No. of p bonds
O O
= 2 ×2 = 4
O 29. (c) HF2 is the only compound among the given options
O=P P=O which does not contain any coordinate bond because
O it has hydrogen bonding.
O P O [H – F•••H]–
|| 30. (b) The compounds having O or F or N along with
O hydrogen show hydrogen bonding and have high
The number of s bonds in it = 16
[Note: Single bonds are s-bonds. Double bond consists boiling points. So HI does not form hydrogen bonds
of 1s and 1p bond]. and its boiling point is not affected.
31. (c) Ice has many hydrogen bonds which give rise to cage
19. (b) There are three resonance structures of CO32 - ion. like structure of water molecules. This, structure possess
larger volume and thus makes the density of ice low.
O O– O–
32. (c) Intramolecular hydrogen bonding is formed (within
C C C molecule) when the two groups capable of forming
– – – H-bonding are very near to each other.
O O O O O O–
H
(I) (II) (III) O
O
20. (c) Higher the electronegativity of the other atom, greater N
is the strength of hydrogen bond. Strongest hydrogen
bond is between H and F. O
F – H ------- F.
21. (a) Greater the difference in electronegativity between the o-Nitrophenol
two atoms, larger will be polarity and hence dipole 33. (c) ortho-Nitrophenol has intramolecular H-bonding
moment. Thus (a) has maximum dipole moment. .
.
. OH
. O and para-nitrophenol has
H
||

H3C H . N
H–C=O C=C O
H3C H intermolecular H-bonding.
(a) (b)
(very less polar) NO 2 NO 2 NO 2
(C–O bond is
more polar)
Cl CH3 ,
H CH3
| | |
C C
O– H O– H O– H
C C hence o-Nitrophenol is more volatile than
H3C H H3C Cl
p-Nitrophenol intermolecular H-bonds are stronger then
(c) (d) intramolecular H-bonds.
Symmetrical molecules (µ = 0) 34. (c) The bond length between carbon & carbon in sp2
hybridisation (double bond) is 1.33 Å.
22. (b) In CN– ion formal negative charge is on nitrogen atom 35. (b) Both NO2 and O3 have angular shape and hence will
due to lone pair of electrons. have net dipole moment.
EBD_7327
86 CHEMISTRY

36. (c) As difference of electronegativity increases % ionic 44. (b) A substance is said to be soluble in water if it is capable
character increases and covalent character decreases of forming hydrogen bonds with water molecules as in
i.e., electronegativity difference decreases covalent methanol and ethanol.
character increases. O H O H O H
Further greater the charge on the cation and smaller
the size more will be its polarising power. Hence R H R
covalent character increases. Hydrogen bonding between alcohol and water.
37. (b) SF4 has permanent dipole moment. However, this hydrogen bonding is restricted when
SF4 has sp3d hybridization and see saw shape (irregular alkyl group of alcohol has four or more carbon atoms.
geometry). 45. (d) Ionic compounds generally have high melting and
boiling points because of the strong electrostatic force
F of attraction between oppositely charged ions.
F
S Consequently, a considerable amount of energy is
F required to overcome strong attractive inter-ionic forces
F and to break down the crystal lattice.
m¹0 46. (a) The dipole moment of two dipoles inclined at an angle
Whereas XeF4 shows squre planar geometry SiF4 has
tetrahedral shape and BF3 has triangular planar shape. q is given by the equation m = X 2 + Y 2 + 2XY cos q
All these are symmetric molecules. Hence m = 0. cos 90° = 0. Since the angle increases from 90° – 180°,
38. (d) Dipole moment is a vector quantity, hence dipole the value of cos q becomes more and more – ve and
moment of a symmetrical molecule is zero. hence resultant decreases. Thus, dipole moment is
maximum when q = 90°.
Cl Cl H Cl 47. (c) Benzene and the compounds having benzene ring show
resonance. Hence, ethylamine does not show
C=C C=C resonance.
48. (d) In case of anions having same charge as the size of
H H Cl H
anion increases, polarisibility of anion also increases.
cis-1, 2-dichloroethene trans-1, 2-dichloroethene
(a) (b) 49. (c) Percent ionic character is given by following equation.
% of ionic character = 16(XA – XB) + 3.5(XA – XB)2
Symetrical hence From the above relation, it is clear that as soon as
dipole moment = 0 (XA – XB) increases, % ionic character will also increase.
Cl C2H5 Therefore, curve C show a correct path.
50. (a) Covalent bonds are formed by sharing of electrons. N
C=C forms 3 covalent bonds with 3 hydrogens.
H
CH3 Cl ••
trans-2, 3-dichloro-2-pentene H •• N ••
(c) ••
H
\ (a) and (c), both show dipole moment.
This leaves one lone pair of electron free which can be
39. (b) In NH3 sp3 hybridisation is found. donated to H+ to form ammonium ion NH4+. This is
40. (a) According to Fajan’s rule, as the charge on the cation called coordinate bond.
increases, and size decreases, its tendency to polarise H +
H é ù
the anion increases. This brings more and more ê | ú
covalent nature to electrovalent compounds. Hence •• H – N ® Hú
• • ê
AlCl3 shows maximum covalent character. H • N • + H ¾¾+ ®ê |
ú
•• ë H û
41. (d) H
42. (a) In H2, both atoms are identical, so the molecule is non Covalent bonds are shown by straight line & coordinate
polar. by an arrow.
43. (d) According to Fajan's rule : This NH4+ cation can gain electrons from Cl– to form a
1 electrovalent bond.
Covalent character µ
size of cation +
é H ù –
ú é•
| •• ù
µ size of anion ê
ê H – N ® H ú ê • Cl• ú ¾¾ ® NH4Cl
Among the given species order of size of cations | êë • • úû
N3+ < O2+ < Pb2+ < Ba2+ ê H ú
ë û
order of size of anions O2– > Cl–.
Hence the order of covalent character is 51. (c) CH 3+ has sp 2 hybridization because S orbital has one
NCl3 > Cl2O > PbCl2 > BaCl2 electron and p orbitals of central atom C have only two
BaCl2 is most ionic in nature. electrons. Therefore s and 2p orbitals participate in
hybridization.
Chemical Bonding and Molecular Structure 87

52. (b) Equilateral or triangular planar shape involves sp2 63. (b)
hybridization. List I List II
53. (c) The bond length decreases in the order sp3 > sp2 > sp. Compound Structure
Because of the triple bond, the carbon-carbon bond (A) ClF3 T-shaped
distance in ethyne is shortest. (B) PCl5 Trigonal bipyramidal
54. (b) BH3 has sp2 hybridization and hence does not have
(C) IF5 Square pyramidal
tetrahedral structure while all others have tetrahedral
structures. (D) CCl4 Tetrahedral
55. (a) We know that size of Na+ ion is greater than Li+ ion. (E) XeF4 Square planar
Since bond length is directly proportional to atomic 64. (c) Benzene has the following resonance structures–
radius. Therefore, the bond length in LiF is less than
that of NaF.
¬
¾®
Option (a) is correct.
56. (a) For linear arrangement of atoms the hybridisation
Hence, its bond order is
should be sp (linear shape, 180° angle). Only H2S has
sp3-hybridization and two lone pair of electrons on no of possible resonating structures
= 1.5.
sulphur atom hence has angular shape while C2H2, 2
BeH2 and CO2 all involve sp-hybridization and hence 65. (b) The bond angle decreases on moving down the group
have linear arrangement of atoms. due to decrease in bond pair-bond pair repulsion.
57. (b) BF3 involves sp2-hybridization. NH3 PH3 AsH3 SbH3 BiH3
58. (a) Only those d orbitals whose lobes are directed along 107º 94º 92º 91º 90º
X, Y and Z directions hybridise with s and p orbitals. This can also be explained by the fact that as the size of
In other three d orbitals namely dxy, dyz and dxz, the central atom increases and its electronegativity
lobes are at an angle of 45° from both axis, hence the decreases. Thus distance between bond pairs of
extent of their overlap with s and p orbitals is much electron increases and bp-bp repulsion decreases. As
lesser than d 2 2 and d 2 orbitals. a result bond angle decrease from NH3 to BiH3.
x -y z
66. (d) Hybridisation present in a molecule can be findout by
59. (c) Acetylene has sp hybridization and linear structure the following formula.
CH º CH
Hence, option (c) is correct. 1
H= (V + M - C + A)
60. (c) X is deficient of e–. Y has 1e– only. Therefore, Y is 2
hydrogen. It forms covalent bond with X (halogen) by Where V = no. of electrons in valence shell of central
sharing 2 valence electrons atom
X Y M = no. of singly charged atoms
C = charge on cation
61. (b) BF3 does not follow octate rule because central atom,
A = charge on anion
boron lacks an electron pair. Thus, it also acts as Lewis
acid. 1
So, H(in ClF3) = [7 + 3 - 0 + 0] = 5
2
×× ×× Þ sp3d Hybridisation
×F ×
× ××
B × F×
× ×× 67. (d) PF3 has pyramidal shape. Phosphorus exist in sp3
hybridization state hence it exist in tetrahedral shape.
×

But due to presence of lone pair its shape is pyramidal.


× ×
× F× 68. (a)
× ×
69. (d) N :
62. (c) F
: 1s2 2s2 2p 3

To form NO 3- , nitrogen uses one p-electron for


Br F p-bond formation two p-electrons for s-bond formation.
2s electrons are used for coordinate bond formation.
Thus there is no lone pair on nitrogen but four bond
: pairs
F
In BrF3, both bond pairs as well as lone pairs of O
electrons are present. Due to the presence of lone pairs
of electrons (lp) in the valence shell, the bond angle is
contracted and the molecule takes the T-shape. This is
N
due to greater repulsion between two lone pairs or
between a lone pair and a bond pair than between the
O O
two bond pairs.
EBD_7327
88 CHEMISTRY

70. (a) Shape (structure ) of a species can be predicted on the


basis of hybridisation of its central atom, which in turn
can be known by determining the number of hybrid
1
bonds (H). Thus H < (V ∗ X , C ∗ A)
2
1 SF4 (sp3d see saw shaped)
For SiF4 : H = (4 + 4 - 0 + 0) = 4; sp 3
2
(Tetrahedral structure) F F

1
For SF4 : H= (6 + 4 - 0 + 0) = 5; sp 3d Xe
2
F F
(See saw structure due to presence of one lp of electrons
on central atom)
Hence the two compounds have different structures XeF4 (sp3d 2 square planar),
71. (c) Species N2 O2 N 2– O –2 79. (c) From amongst given species PH3, NH3 and SbH3 are
Bond order 3 2 2.5 1.5 all sp3 hybridised. Their central atom has both bond
pair as well as lone pair of electrons. The lone pair
The O-O bond is O –2 decreases occupy the fourth orbital. CH3+ has only three pairs of
electrons so it is sp2 hybridised.
72. (a) X5
80. (b) Geometry of compounds can be determined by
X4 X1 hybridisation, which in turn can be known by the
following relation.
M 1
Hybridisation (H) = [No. of electrons in valence shell
2
of central atom + No. of monovalent atoms –charge on
cation + charge on anion]
X3 X2
X6 1
Thus here bond angles between
(i) SF4 : H = ( 6 + 4 - 0 + 0) = 5
2
X 4 - M - X 2 = 180° sp3d (trigonal bipyramidal but actual geometry is see
saw due to presence of one lone pair of electrons.
X1 - M - X 3 = 180°
1
X 5 - M - X 6 = 180° (ii) XeF4 : H = ( 8 + 4 - 0 + 0) = 6
2
73. (a) BF3 is sp2 hybridised. So, it is trigonal planer. NH3, sp3d2 (Octahedral but actually square planar due to
PCl3 has sp3 hybridisation hence has trigonal pyramidal presence of two lone pairs of electrons)
shape. IF3, has sp3d hydridisation and has T-shape.
74. (a) Both XeF2 and CO2 have a linear structure. (iii) NF3 : H = 1 ( 5 + 3 - 0 + 0 ) = 4
F — Xe — F O=C=O 2
75. (c) sp3 (tetrahedral but distorted due to one lone pair of
76. (b) Electronegativity difference is 4.0 - 1.20 = 2.8 percentage electrons)
ionic character is 72.24% when the electronegativity 1
difference is 1.7, the % ionic character is approx 51%. (iv) SiF4 : H = ( 4 + 4 - 0 + 0) = 4
77. (a) Ionic character follows the order HF > HCl > HBr > HI 2
sp3 (tetrahedral)
81. (b) Molecule or ion having sp hybridisation and no lone
78. (a) pair of electrons is linear.
1
Hybridisation = [No. of e– in valence shell of central
2
atom + No. monovalent atoms + Charge on anion –
[ Ni(CN) 4 ]2 - (dsp2 square planar ), Charge on cation]
(a) SO2
1
H= (6 + 0 + 0 – 0)
2
= 3 sp2 (trigonal planar shape)
(b) CO2

BF4- (sp3 tetrahedral),


Chemical Bonding and Molecular Structure 89

1 93. (b) SO24- ion is tetrahedral since hybridization of S is sp3.


H= (4 + 0 + 0 – 0) = 2 sp (linear shape)
2
.O. . N.
(c) ClO2– 94. (b)
1 H 105° H H
107°
H
H = (7 + 0 + 1 – 0) = 4 sp3 (tetrahedral)
2 H
(d) NO2– .P. . .
As
1 H H H H
H= (5 + 0 + 1 – 0) = 3 sp2 (trigonal planar) H H
2 Less then 107°
\ CO2 has linear shape. 95. (b) NH3 has pyramidal structure, as nitrogen is sp 3
82. (a) According to VSEPR theory, trigonal bipyramidal hybridised. This is due to the presence of lone pair of
geometry results from sp3d or dsp3 hybridisation. electrons on N atom.
dsp2 hybridisation results in square planar geometry, 96. (c) O = C = C = C = O is sp hybridised
while d2sp3 leads to octahedral shape. 97. (a) A s - bond is stronger than a p -bond hence option
83. (b) SeF4 has distorted tetrahedral geometry while, CH4 has (a) is not correct. Sigma (s) bonds are formed by head
tetrahedral geometry. on overlap of unhybridised s–s, p–p or s–p orbitals
and hybridised orbitals (sp, sp2, sp3, sp3d and sp3d2)
84. (b) Molecule Hybridization hence s bonds are strong bonds. Whereas Pi (p)-bonds
SO3 sp 2 are formed by sideways overlap of unhybridised
C2 H2 sp p- and d-orbitals hence p bonds are weak bonds.
C2 H4 sp 2 98. (a) Linear combination of two hybridized orbitals leads to
3 the formation of sigma bond.
CH4 sp 99. (a) For p-overlap the lobes of the atomic orbitals are
CO2 sp perpendicular to the line joining the nuclei.
85. (d) 100. (d) Total no. of electrons in O 2 2 - = 16 + 2 = 18
X Distribution of electrons in molecular orbital
86. (d) M three angle below M and three above M hence s1s 2 , s* 1s 2 , s2 s 2 , s* 2s 2 , s2 p z2 ,
X X X p 2Px2 = p 2 Py2 , p* 2 p2x = p* 2 p 2y
Antibonding electrons = 8 (4 pairs)
=6
101. (b) In CH3 CH2OH underlined C is forming 4 σ bonds,
87. (c) s-character µ bond angle
hence sp3 hybridisation. In others it is sp2 hybridised
For 25% s character (as in sp3 hybrid orbital), bond
(due to 3 σ bonds).
angle is 109.5°, for 33.3% s character (as in sp2 hybrid
102. (c) As sigma bond is stronger than the p (pi) bond, so it
orbital), bond angle is 120° and for 50% s character (as
must be having higher bond energy than p (pi) bond.
in sp hybrid orbital), bond angle is 180°.
Similarly, when the bond angle decreases below 109.5°, 103. (c) N 2 + = 7 + 7 – 1 = 13 electrons
the s-character will decrease accordingly. Configuration is
Decreasing in angle = 120° – 109.5° = 10.5° s1s 2 , s *1s 2 , s 2s 2 , s *2 s 2 , p2 px 2 = p2 p y 2 , s2 p1z
Decrease in s-character = 33.3 – 25 = 8.3 Bond order =
Actual decrease in bond angle = 109.5° – 105° = 4.5° 1 æ No. of e - s in bonding - No. of e - s in antibonding ö
Expected decrease in s-character 2 çè molecular orbital molecular orbital ÷ø
8.3 1 1
< ´ 4.5 < 3.56% = (9 - 4) = ´ 5 = 2.5
10.5 2 2
Thus, the s-character should decrease by about 3.56% 104. (b) Only d of S and p of O can be involved in π bonding.
i.e., s-character = 25 – 3.56 = 21.44% 105. (c) The first bond between any two C atoms having
88. (c) Because of the triple bond the carbon - carbon bond multiple bonds is s and rest are p bonds. Draw detailed
distance in ethyne is shortest. structure of compound and find the total bonds in it
89. (c) Hybridisation of N = ½ [5 + 3 + 0 – 0] = 4 hence sp 3
H H H
sp 2 sp sp 2 sp3 s s s
90. (d) CH 2 = C = CH - CH3 1, 2 -butadiene. s C s s C s s Cs H
s p p
91. (c) H C C C s
92. (c) H2O, NH3 and CH4 all are sp3 hybridized due to lp – lp s p s s H
repulsions, bond angle in H2O (104.5°) is lower than in s s
H C p C H H
NH3 (107°) which have one lp and CH4 (109° 28’) which s C s
have no lp BeF2 on the other hand, has sp hybridization s
and hence has a bond angle of 180°. H
\ 19 s bonds and 4p bonds.
EBD_7327
90 CHEMISTRY

106. (c) H
s 116. (c) H +2 : (s1s1 )
s s
H Cs H
s 1 1
\ B.O. = (1 - 0) =
s C s 2 2
s p s
H C C H
H -2 : (s1s 2 )(s *1s1 )
s p s
s s 1 1
H C p C H \ B.O. = (2 - 1) =
s C s 2 2
s
Even though the bond order of H +2 and H -2 are equal
H
15s + 3p but H +2 is more stable than H-2 as in the latter, an
electron is present in the antibonding (s * 1s) orbital of
107. (b) O2- = 16 + 1 = 17 Molecular orbital configuration of higher energy.
O2– (superoxide ion). 117. (b) In H - bonding atoms involved are F, O and N.
KK s 2 s 2 s * 2s 2 s 2 pz2 p 2 Px2 = p 2 p 2y , p * 2 Px2 = p * 2 Py1 118. (c) O d – = C d + = O d – linear molecule hence m = 0 .
8-5 3
Bond order = = = 1.5 119. (c) Both BrO3- and XeO3 have sp3 hybridization but
2 2
108. (c) due to presence of one lp of electrons they have
109. (d) Bond structure of H2P2O7 is following trigonal pyramidal geometry.
O O 120. (d) Calculating the bond order of various species.
s p
s
s p
O-2 : KK s 2s 2 , s* 2s 2 , s2 pz2 ,
H –s O –s P – s
s O – P s– O – H
s s
O s– H O s– H p2 px2 = p2 p 2y , p* 2 px2 = p* 2 p1y
12s, 2dp – pp bonds. 1
110. (b) More will be the electronegativity of X lesser will be B.O. = (Nb - N a )
2
the bond length of X-O bond.
111. (d) M.O. configuration of NO+ is : 8-5
= or 1.5
(s1s)2 (s * 1s)2 (s2s)2 (s * 2s)2 (s2pz)2 (p2px)2 (p2py)2 2
and M.O. configuration of NO is :
NO = K K s 2 s 2 , s* 2s 2 ,62 p 2z , p 2 p x2 = p2 p 2y , p* 2 p1x
(s1s)2(s * 1s)2 (s2s)2 (s * 2s)2 (s2pz)2 (p2px)2
(p2py)2 (p * 2px)1. Nb - N a 8 - 3
B.O. = = or 2.5
112. (a) CH 2 = C H - C º N 2 2
1 2 3 4
2- 2 * 2 2
3 σ bonds (sp2 hybridisation); 2 σ bonds = C2 : KK s2s , s 2 s , p2 px = p2 p 2y , s 2 p z2
(sp - hybridisation)
Nb - N a 8 - 2
C1 = 3 σ bonds, C2 = 3 σ bonds, B.O. = = or 3
C3 = 2 σ bonds 2 2
113. (a) Molecular orbital electronic configuration of these He 22+ = s1s 2 s*1s1
species are :
Nb - N a 2 - 1
O-2 (17e- ) = s1s 2 , s *1s 2 , s2s 2 , s *2s 2 , s2 pz2 , B. O. = = or 0.5
2 2
p2 px2 = p2 p2y , p *2 px2 = p2* p1y From these values we conclude that the correct order
of increasing bond order is
O2 (16e - ) = s1s 2 , s *1s 2 , s2 s 2 , s * 2 s 2 , s2 p z2 , He22+ < O -2 < NO < C22-
p2 px2 = p2 p 2y , p *2 p1x = p * 2 p1y 121. (d) Due to intermolecular hydrogen bonding in methanol,
it exist as assosiated molecule.
O22 - (18e - ) = s1s 2 , s *1s 2 , s2 s 2 , s * 2 s 2 , s 2 p z2 ,
5+5+2
122. (c) For SbCl52– , H = = 6 means sp3d 2 hybridization
p2 px2 = p2 p 2y , p *2 px2 = p *2 p 2y 2
Hence number of antibonding electrons are 7, 6 and 8
I3– , SF4 , and PCl5 ; all have sp3 d hybridization.
respectively.
114. (a) 123. (b) In NO3- , nitrogen have sp2 hybridisation, thus planar

N
..
N in shape. In H3 O+ , oxygen is in sp3 hybridisation,
115. (b) thus tetrahedral geometry is expected but due to
F F F H H H presence of one lp of electrons on central oxygen atom
it is pyramidal in shape.
N –– N– O octet of each atom is complete.
Chemical Bonding and Molecular Structure 91

d-
124. (c) NH +4 : sp3 hybridisation F d + d-
CH4: sp3 hybridisation 4. (d) The shape of BF3 is trigonal planar B – F and
d- F
SF4: sp3d hybridisation
m = 0 hence it is non polar. The shape of NF3 is
BF4– : sp3 hybridisation
+

:
Nd
125. (b) Bond order of Be2 = 0, hence Be2 cannot exist. pyramidal d- F F d - and m ¹ 0 hence it is polar..
F d-
EXERCISE - 2
5. (a) Number of electrons in each species are
s
O O CN - = 6 + 7 + 1 = 14
1. (c) H–C H–C or Resonance CO = 6 + 8 = 14
O
O NO + = 7 + 8 - 1 = 14
s Each of the species has 14 electrons which are
distributed in MOs as below
O For CO and CN–
hybrid H – C due to resonance C – O bond
s1s2 , s *1s 2 , s2 s2 , s *2s 2 ,{p 2 px2 = p 2 p 2y , s 2 pz2
O
10 - 4
length is the same. Bond order = =3
2
Cl For NO+
2. (b) BiCl3: Cl Bi ; sp2 - Hybridisation (Trigonal
Cl
geometry); Bond angle = 120º s1s2 , s *1s2 , s2s2 , s *2s2 , s2 pz2 , p2 px2 = p2 p 2y
sp3 10 - 4
P Bond order = =3
(Pyramidal 2
In PCl3
Cl Cl Cl geometry) 6. (c) Bond energy a Bond order
Bond angle =
o
below 109 28’
Bond order can be determined by MO configuration.
sp3 NO : No. of electrons = 7 + 8 = 15
As and decreases from
In AsCl3 (Pyramidal PCl3 to BiCl3
Cl Cl Cl geometry) s1s2, s* 1s2, s2s2, s* 2s2, s2Pz2 , p2 px2 = p2 p2y , p *2 p1x
sp3
In BiCl3 Bi (Pyramidal 10 - 5
\ Bond order = = 2.5
Cl Cl Cl geometry) 2
In these, order of bond angle : NO+ : No. of electrons = 15 – 1 = 14
BCl3 > PCl3 > AsCl3 > BiCl3 Delete p* 2 p1x from NO configuration
é No. of electrons No. of mono- charge charge ù
1ê 10 - 4
3. (d) H = ê in valence shell + valent atoms + on - on úú \ Bond order = =3
2 2
ëê of central atom anion cation ûú
NO– : No. of electrons = 15 + 1 = 16
For SF4 :
s1s2 , s *1s 2 , s2s 2 , s *2s 2 , s2 p2z , p2 px2 = p2 p2y ,
1
= (6 + 4 + 0 - 0) = 5
1
2 p *2 p1x = p *2 p y
S is sp3d hybridised in SF4. Thus SF4 has 5 hybrid
orbitals out of which only four are used to form bonds 10 - 6
\ Bond order = =2
with four F atoms leaving one lone pair of electrons on 2
sulphur. \ Bond order; hence bond strength
1 NO+ > NO > NO–
For CF4 : H = [4 + 4 + 0 - 0] = 4 \ sp3 hybridisation 7. (b) For T-shape geometry the molecule must have 3 bonded
2
Since all the four orbitals of carbon are involved in pair and 2 lone pair of electrons.
bond formation, no lone pair is presen t on 8. (a) Structure of CO2 is linear O = C = O while that of
c atom O
1 H2O is H H i.e ., bent structure so in CO2 resultant
For XeF 4 : H = (8 + 4 + 0 - 0) = 6 , \ sp 3d 2
2 dipole moment is zero while that of H2O has significant
hybridization of the six hybrid orbitals, out of which value.
only four form bonds with F atoms, leaving behind two
O–H
lone pairs of electrons on Xe.
9. (a) CH3 - C = CH 2 has 9s,1p and 2lone pairs.
EBD_7327
92 CHEMISTRY

10. (d) H3O2– species H bonding in H2O 23. (c) The electronic configuration of As is
- [ Ar ]3d 10 4s1 4 p1x 4 p1y 4 p1z 4d 1
(i) é O - H.....O - H ù (ii) H H
ê| ú | | As = 144444244444 3
êë H úû ¯ sp 3d hybridisation
O - H.....O - H
Thus the hybridisation involved in the AsF5 molecule
(iii) H5O2+ (H-bonding) (iv) H3O+ (no H-bonding)
is trigonal bipyramidal. So, the hybrid orbitals used
+ +
H H H are s, px, py, pz, dz2.
O..... H – O O H 24. (c) As dipole moment = electric charge × bond length
H H H D. M. of AB molecule
H O = 4.8 ´ 10 -10 ´ 2.82 ´ 10 -8 = 13.53D
| P D.M. of CD molecule
11. (d) H , C < C*, O , H
< 4.8´10,10 ´ 2.67 ´10,8 < 12.81D
The star marked carbon has a valency of 5 and hence
Now % ionic character
this formula is not correct.
Actual dipole moment of the bond
12. (b) In amine the nitrogen is sp3 hybridised and in amide the = ´100
nitrogen is sp2 hybridised. Dipole moment of pure ionic compound
13. (c) Since water is polar in nature and like dissolves like, then % ionic character in
the coating must be nonpolar to polar manner. 10.41
14. (b) The removal of an electron from a diatomic molecule AB = ´ 100 = 76 .94 %
13.53
may increase the bond order as in the conversion % ionic character in
+
O2 (2) ¾¾® O 2 ( 2.5) or decrease the bond order as in 10.27
CD < ´100 < 80.17%
the conversion, N2(3.0) ¾ ¾® N +2 ( 2.5), As a result, 12.81
the bond energy may increase or decrease. Thus, 25. (c) Structure I has two resonance forms, which make the
statement (b) is incorrect. hydrogen bonding more stable. The II molecule has
15. (d) Nitrogen can not be pentavalent as it has no d-orbitals. no such resonance stabilization.
16. (c) The b.p. of p-nitrophenol is higher than that of 26. (c) Number of hybrid orbitals of P in PO 34- = ½ [5 + 0 + 3]
o-nitrophenol because in p-nitrophenol there is = 4 (sp3)
intermolecular H-bonding but in o-nitrophenol it is No. of hybrid orbitals of N in NO–3 = ½ [5 + 0 + 1] = 3
intramolecular H-bonding. (sp2)
17. (a) The group moments of –NO2, –OH, –Cl and –CH3 No. of hybrid orbitals of I+ (5s2 5p2x 5p1y 5p1z) : (sp3)2
respectively are 3.98, 1.6, 1.55 and 0.4D. (sp3)2 (sp3)1 (sp3)1
18. (b) Resultant of two C–X dipoles in 1, 4 positions is zero. 1 0
I-atom in excited state : 5s2 5p4 5d 2 5d 2
The resultant of other two C–X dipoles in 3, 5 positions z x - y2
I-atom is sp3d2 hybridised state in ICl–4 : (sp3d2) (sp3d2)
= 1.52 + 1.52 + 2 ´ 1.5 ´1.5cos120º = 1.5 D (sp3d2)1 (sp3d2)1 (sp3d2)1 (sp3d2)0 – Two lps occupy
19. (d) As there is no lone pair on boron in BCl3 therefore no the axial positions of octahedron and the shape of
repulsion takes place. But there is a lone pair on ICl–4 becomes square planar.
nitrogen in NCl3. Therefore repulsion takes place. Thus 27. (c) Dissociation energy of any molecule depends upon
BCl 3 is planar molecule but NCl 3 is a pyramidal its bond order. Bond order in N2 molecule is 3 while
molecule. bond order in N +2 is 2.5. Further we know that more
20. (b) Group moment of –NO2 is –3.98 D (acting towards the the Bond order, more is the stability and more is the
benzene ring) and that of –NH2 is + 1.5D (acting away bond dissociation energy.
from the group). Hence moments of both the groups in 28. (c) In X — H - - - Y, X and Y both are electronegative
p-positions will act in the same direction. elements (i.e attracts the electron pair) then electron
21. (d) In alkynes the hybridisation is sp i.e, each carbon atom density on X and Y will increase and on H will decrease.
undergoes sp hybridisation of one s and one p orbital 29. (a) In NH3 the atomic dipole (orbital dipole due to lone
to form two sp-hybrid orbitals. Out of which one pair) and bond dipole are in the same direction whereas
overlaps to form C – C s bond and other overlaps with in NF3 these are in opposite direction so in the former
case they are added up whereas in the latter case net
half filled orbital of H forming s-bond. The other two
result is reduction of dipole moment. It has been shown
2p-orbitals remain unhybridised and form two p-bonds in the following figure :
via sidewise overlaping.
Hence two p bond and one sigma bond between C — .. ..
C lead to cylindrical shape. N N
22. (d) N2+ is paramagnetic F
s1s2, s*1s2,s*2s2,p2p2x = p2p2y ,s2p1z H H F
H F
Chemical Bonding and Molecular Structure 93

30. (d) The bond angles HXH vary in the order : NH3 > PH3
AsH3 > SbH3. In the formation of XH+4, the bond angle
. .N. +

..N
.... .. N
HXH increases to 109º 28'. Hence, maximum increase ..O.. .... .. ..-O ....
O ..O

|
||
O

..
|
|

||
||
O

..
in the bond angle will be for SbH3. (NO )
2 NO 2 +
NO 2
31. (b) On changing N2 to N+2, B.O. decreases from 3 to 2.5
whereas on changing O2 to O+2, B.O. increases from 2 We know that higher the number of lone pair electron(s)
to 2.5. In former case, the bond dissociation energy on central atom, greater is the lp – lp repulsion. Thus
decreases and in the latter case, it increases. smaller is bond angle.
The correct order of bond angle is
32. (d) In a linear symmetrical molecule like BeF2, the bond
+
angle between three atoms is 180°, hence the polarity NO -2 < NO 2 < NO 2 i.e., option (b) is correct.
due to one bond is cancelled by the equal polarity due 40. (c) On determining hybridisation from H = 1/2 (V + M – C + A).
to other bond, while it is not so in angular molecules, The hybridisation of BF3, NO2–, NH2– and H2O are
like H2O. sp2, sp2, sp3 and sp3 respectively.
33. (b) Now since bond order of NO+ (3) is higher than that of 41. (a) Molecular orbital configuration of
NO (2.5). Thus bond length of NO+ will be shorter.
N 2– 2 2 2 2
2 = s1s s *1s s2 s s * 2 s –
34. (b) The p-orbital has equal sized lobes whereas an sp
hybrid orbital has more probability density on one side. ìï p 2 px2 ìï p * 2 p1x
Hence, sp orbital is more directional in character. í s2 p2z í
2 1
35. (c) ns2 electrons in the havier elements, like lead, remain ïî p 2 p y ïîp * 2 p y
paired and do not take part in bonding. Thus, Pb2+ is 10 – 6
more stable than Pb4+. Bond order = =2
2
36. (b) O2 : s1s 2 , s*1s 2 , s 2s 2 , s* 2 s 2 , s 2 p z2 , ìp2 px2 ìï p * 2 p1x
N 2– = s1s 2 s *1s 2 s2s 2s * 2s 2 ï í s2 pz2 í
ìïp 2 p 2x ìïp* 2 p1x 2 0
ïîp2 p y ïî p *2 p y
í , í * 1
2 10 - 5
îïp 2 p y îïp 2 p y Bond order = = 2.5
2
10 - 6 ìï p2 px2
Bond order = =2 N 2 = s1s 2 s *1s 2 s 2 s 2 s * 2 s 2 í , s 2 pz2
2 2
îï p 2 p y
(two unpaired electrons in antibonding molecular orbital)
10 – 4
ìïp 2 p x2 , ìï p* 2 p1x Bond order = =3
2
O +2 : s1s 2 , s*1s 2 , s 2s 2 , s* 2s 2 , s 2 p 2z , í 2 í *
ïîp 2 p y , ïî p 2 p y
0 \ The correct order is = N 2– –
2 < N2 < N2
1
10 - 5 42. (a) Hybridisation = [No. of valence electrons of central
Bond order = = 2. 5 2
2
atom + No. of monovalent atoms attached to it +
(One unpaired electron in antibonding molecular orbital) Negative charge if any – Positive charge if any]
Hence O2 as well as O +2 both are paramagnetic, and 1
NO2– H = [5 + 0 + 1 - 0] = 3 = sp 2
2
bond order of O +2 is greater than that of O2. 1
NO3– H = [5 + 0 + 1 - 0] = 3 = sp 2
37. (a) Its conjugate base (i.e. NH3) is most stable. 2
38. (c) The b.o in Cl – O– is 1 1
The b.o in O = Cl – O– is 1.5 NH2– H = [5 + 2 + 1 + 0] = 4 = sp3
2
5 1
The b.o. in O = Cl = O is = 1.66 NH4+, H = [5 + 4 + 0 - 1] = 4 = sp3
3 2
O– SCN– = sp
i.e., NO2– and NO3– have same hybridisation.
O
7
= 1.75
43. (a) O2 (16) = s1s 2 , s *1s 2 , s 2s 2 , s *2 s 2 , s 2 p 2z ,
The b.o. in O = Cl – O – is
4
O p2 px2 = p2 p 2y , p * 2 p1x = p * 2 p1y
The bond length increases as b.o. decreases. B.O. = ½ (Nb – Na) = ½ (10 – 6) = 2
39. (b) From the structure of three species we can determine O2 + (15) = s1s 2 , s *1s 2 , s2s 2 , s * 2s 2 , s 2 p z2 ,
the number of lone pair electron(s) on central atom (i.e.
N atom) and thus the bond angle. p2 px2 = p2 p 2y , p * 2 p1x = p * 2 p0y
EBD_7327
94 CHEMISTRY

B.O. = ½ (Nb – Na) = ½ (10 – 5) = 2.5


O2 - (17) = s1s 2 , s *1s 2 , s 2s 2 , s * 2s 2 , s2 p z2 ,
F
p2 px2 = p2 p 2y , p * 2 px2 = p * 2 p1y B F
B.O. = ½ (Nb – Na) = ½ (10 – 7) = 1.5 F
O2 2 - (18) = s1s 2 , s *1s 2 , s 2s 2 , s * 2s 2 , s2 p z2 ,
Vacant Filled
p2 px2 = p2 p 2y , p * 2 px2 = p * 2 p 2y 2p-orbital 2p -orbital

B.O. = ½ (Nb – Na) = ½ (10 – 8) = 1 F F


+
Since, the bond length decreases as the bond order B=F B–F
increases, hence, O2+ have least bond length. F F
+

44. (c) MOT configurations of O2 and O2+ :


+ +1/3
O2+ : (s1s)2 (s*1s)2 (s2s)2 (s*2s)2 (s2pz)2 F F
+1/3
B–F B F
(p2p2x = p2p2y) (p*2px1 = p*2p0y) +1/3
F F
Number of unpaired electrons = 1, so paramagnetic.
49. (d) Calcium carbide exists as Ca2+ and C22–. According to
O2 : (s1s)2 (s*1s)2 (s2s)2 (s*2s)2 (s2pz)2
the molecular orbital model, C22– should have molecular
(p2p2x = p2p2y) (p*2px1 = p*2p1y) orbital configuration :
Number of unpaired electrons = 2, so paramagnetic. s1 s 2 , s *1 s 2 , s 2 s 2 , s *2 s 2 ,
45. (d) BF4- hybridisation sp3, tetrahedral structure.
{p 2 px2 = p 2 p 2y }, s 2 pz2
NH +4 hybridisation sp3, tetrahedral structure.
Thus M.O. configuration suggests that it contains one
46. (b) (O2) = s1s 2 , s* 1s 2 , s 2s 2 , s* 2s 2 , s 2 p z2 , s & 2p bonds.
50. (c) PF5 trigonal bipyramidal
p 2 p x2 = p 2 p 2y , p* 2 p1x = p * 2 Py1
F
N b - N a 10 - 6 4 F
Bond order = = = =2 F P
2 2 2
F
( O+2 ion ) = s1s2 , s*1s2 , s 2s2 , s* 2s2 , s2 pz2 , F
2 * 1 BrF5 square pyramidal (distorted due to presence of
p 2 px2 = p2 p y , p 2 px
one lp of electrons on central atom)
Nb - N a 10 - 5 5 1 F
Bond order = = =2
2 2 2 2 F F
Br
( O -2 ) = s1s 2 , s*1s2 , s 2s2 , s* 2s 2 , s2 Pz2 , F F
p2 px2 = p2 p y2 , p* 2 px2 = p* 2 p1y
51. (d) (i) In Na2O2, we have O 22 - ion. Number of valence
Bond order = ( N b - N a ) = 10 - 7 = 3 = 1 1 electrons of the two oxygen in O 22 - ion = 8 × 2 + 2
2 2 2 2
(O ) = s1s , s 1s , s2s , s 2s , s2 p ,
2-
2
2 * 2 2 * 2 2
z
=18 which are present as follows
2
s1s2, s*1s2, s2s2, s*2s2, s 2 p z2 , { p 2 px2 = p 2 p y ,
p2 px2 = p2 p y2 , p* 2 px2 =p *
2 p 2y
{ p * 2 p x2 = p * 2 p 2y
Nb - Na 10 - 8 2
Bond order = = =1
2 2 2 \ Number of unpaired electrons = 0, hence, O 22 - is
47. (c) Greater the difference between electro-negativity of diamagnetic.
bonded atoms, stronger will be the bond. Since F is (ii) No. of valence electrons of all atoms in
most electronegative hence F – H ...... F is the strongest O3 = 6 × 3 = 18.
bond. Thus, it also, does not have any unpaired electron,
48. (b) The delocalised pp – pp bonding between filled hence it is diamagnetic.
p-orbital of F and vacant p-orbital of B leads to (iii) No. of valence electrons of all atoms in N2O
shortening of B–F bond length which results in higher = 2 × 5 + 6 = 16. Hence, here also all electrons are
bond dissociation energy of the B–F bond. paired. So it is diamagnetic.
Chemical Bonding and Molecular Structure 95

(iv) In KO2, we have O2- . No. of valence electrons of [NiCl4]2– . = 3d8 configuration with nickel in + 2
oxidation state, Cl– being weak field ligand does not
all atoms in O -2 = 2 × 8 + 1 = 17, compel for pairing of electrons.
So,
s1s 2 , s *1s 2 , s 2s 2 , s * 2s 2 , s 2 p z2
4s 4p
3d
p2 px2 = p2 p 2y , p * 2 px2 = p * 2 p1y
[NiCl4 ]2 -
Thus it has one unpaired electron, hence it is
paramagnetic. 3
sp hybridisation
52. (a) Molecular electronic configuration of
Hence, complex has tetrahedral geometry
CO:s1s 2 , s *1s 2 , s 2 s 2 , s * 2 s 2 ,{p2 px 2 = p2 p y 2 , s 2 pz 2
Cl 2–
Nb - N a 10 - 4
Therefore, bond order = = =3
2 2
Ni
NO+ :s1s2 , s *1s2 , s 2 s 2 , s * 2 s 2 , s 2 pz 2 ,{p2 px 2 = p2 p y 2
Cl Cl
10 - 4 Cl
Bond order = =3
2
Ni +2 + 4CN - ¾¾ ®[Ni(CN) 4 ]2 -
CN - = s1s 2 , s *1s 2 , s2 s 2 , s * 2s 2 ,
[Ni(CN)4]2– = 3d 8 configuration with nickel in + 2
{p 2 px 2 = p 2 p y 2 , s 2 pz 2 oxidation state, CN– being strong field ligand compels
for pairing of electrons.
10 - 4 So,
Bond order = =3
2
4s 4p
N2 : s1s 2 , s *1s 2 , s 2s2 , s *2s 2 ,{p2 p2x = p2 p2y , s 2 pz2 3d
10 - 4 [NiCN4]–2
Bond order = =3
2 2
dsp hybridisation
NO – : s1s 2 , s *1s 2 , s 2 s 2 , s * 2 s 2 , s 2 p z2 ,
Hence, complex has square planar geometry.
{p 2 p x2 = p 2 p 2y , {p * 2 p1x = p * 2 p1y
2–
10 - 6 NC CN
Bond order = =2
2
\ NO– has different bond order from that in CO. Ni
53. (a) Molecular orbital configuration of B2(10) as per the
NC CN
condition given in question will be
s1s2, s* 1s2, s2s2, s* 2s2, p 2 px2
6-4 Ni +2 + 6H 2 O ¾¾ ®[Ni(H 2 O) 6 ]2 +
Bond order of B2 = =1. [Ni(H2O)6]2+ = 3d 8 configuration with nickel in + 2
2
B2 will be diamagnetic. oxidation state. As with 3d 8 configuration two
d-orbitals are not available for d 2sp3 hybridisation. So,
6+2 hybridisation of Ni (II) is sp3d 2 and Ni (II) with six
54. (d) OSF2 : H = =4.
co-ordination will have octahedral geometry.
2
It has 1 lone pair. 4s 4p 4d
3d
[Ni(H2O)6]2+
:

S (Shape is trigonal pyramidal)


3 2
sp d hybridisation
O F
F H2O 2+
The shapes of SO3, BrF3 and SiO32- are triangular planar H2O OH2

respectively. Ni

55. (b) Ni+2 + 4Cl - ¾¾


® [NiCl4 ]2 - H2O OH2
sp 3 H2O
EBD_7327
96 CHEMISTRY

sp 2 sp sp 2 6-4 2
= = =1
56. (b) Allene (C3H4) is H 2 C = C = CH 2 2 2
57. (a) & (b) The molecular orbital structures of C2 and N2 are B2 is known in the gas phase
O22 - = (s1s )2 (s *1s)2 ( s2 s ) 2 (s * 2 s ) 2 ( s2 p z )
2
N 2 = s1s 2 s *1s 2 s 2s 2 s * 2s 2 s 2 px2 p 2 py2 p2 pz2

( p 2 p x )2 = ( p 2 p y ) (p* 2 px ) = (p* 2 p y )
2 2 2
C2 = s1s 2 s *1s 2 s 2s 2 s * 2s 2 p2 py 2 p 2 Pz2
Both N2 and C2 have paired electrons, hence they are
diamagnetic. Bond order = 1 (10 - 8) = 1
2
58. All options are correct, 64. (b) Diamagnetic species have no unpaired electrons
(a) ONCl = 8 + 7 + 17 = 32e - ü not isoelectronic
O 2 2- Þ s1s2, s*1s2, s2s2, s*2s2, s 2 p z ,
2
ý
ONO- = 8 + 7 + 8 + 1 = 24e - þ
{p2px2 = p2py2, {p*2px2 = p*2py2
Å. . Whereas paramagnetic species has one or more
. 2 78 O 1.278A° The central atom is sp2
(b) 1 unpaired electrons as in
O 116.8° O hybridized with one lone pair.
O 2 ® s1s 2 , s*1s 2 , s 2 s 2 , s* 2s 2 , s 2 p 2z , {p 2 p 2x = p 2 p 2y ,
(c) It is a pale blue gas. At – 249.7°, it forms violet black
crystals. * 1
{ p* 2 p1x = p 2 p y = 2 unpaired electrons
(d) It is diamagnetic in nature due to absence of unpaired
electrons. O+2 ® s1s 2 , s*1s 2 , s 2 s 2 , s* 2 s 2 , s2 p z2 ,
59. (c) H2+
2 = s1s s*1s
0 0
2 * 1 0
p 2 px2 = p 2 p y p 2 px = p * 2 p y = 1 unpaired
unpaired electron
1
Bond order for H2+
2 =
(0 - 0) = 0
2 NO ® s1s 2 , s*1s 2 , s2 s 2 , s* 2s 2 , s2 p 2z ,
He2 = s1s2s*1s2
p2 px2 = p2 p2y , p* 2 p1y = p* 2 pz0 = 1 unpaired electron
1
Bond order for He2 = (2 - 2) = 0 65. (d) Smaller the size and higher the charge more will be the
2 polarising power of cation. Since the order of the size
so both H22+ and He2 does not exist. of cation is K + > Ca ++ > Mg ++ > Be++ . So the
60. (b) Li2 = s1s2 s*1s2 s2s2
correct order of polarising power is
1 K+ < Ca2+ < Mg2+ < Be2+
\ Bond order = (4 , 2) < 1
2 66. (c) (i) N2 : bond order = 3, diamagnetic
Li2+ = s1s2 s*1s2 s2s1 N2– : bond order = 2.5, paramagnetic
(ii) C2 : bond order = 2, diamagnetic
1
B.O. < (3 , 2) < 0.5 C2+ : bond order = 1.5, paramagnetic
2 (iii) NO : bond order = 2.5, paramagnetic
Li2– = s1s2 s*1s2s2s2s*2s1 NO+ : bond order = 3, diamagnetic
1 (iv) O2 : bond order = 2, paramagnetic
B.O. = (4 , 3) < 0.5 O2+ : bond order = 2.5, paramagnetic
2
67. (a) For any species to have same bond order we can expect
The bond order of Li2+ and Li2– is same but Li2+ is more
them to have same number of electrons. Calculating
stable than Li2– because Li2+ is smaller in size and has 2
the number of electrons in various species.
electrons in antibonding orbitals whereas Li2– has 3
electrons in antibonding orbitals. Hence Li2+ is more O2– (8 + 8 + 1 = 17) ; CN - (6 + 7 + 1 = 14)
stable than Li2–. NO+(7 + 8 – 1= 14); CN+ (6 + 7 –1 = 12)
We find CN– and NO+ both have 14 electrons and bond
sp 3 sp sp sp 3
61. (b) H 3C — C º C — CH 3 order 3.
68. (c) All the members form volatile halides of the type
linear due to sp hybridized C atom.
AX3. All halides are pyramidal in shape. The bond
62. (d) PCl3 angle decreases on moving down the group due to
P decrease in bond pair-bond pair repulsion.
Cl 69. (a) Compound having maximum electronegativity
Cl Cl difference is more ionic. Hence the correct order is
63. (a) Both O2– DG < EG < DF < DE
2 and B2 has bond order equal to 1.
70. (d)
B2 (10) = s 1s s 1s s 2s s 2s
2 * 2 2 * 2 71. (b) SO2 – bent
p2 p1x = p2 p y1 SF4 – see-saw
ClF3 – T-shape
Nb - N a BrF5 – square pyramidal
Bond order =
2 XeF4 – square planar.
Chemical Bonding and Molecular Structure 97

72. (a) XeF4 having one lone pair of electron show distorted 6. (d) In NO3– ion,
pentagonal bipyramidal shape and sp3d3 hybridisation number of bond pairs (or shared pairs) = 4
73. (a) It is correct that during formation of ice from water number of lone pairs = 0
there are vacant spaces between hydrogen bonded
molecules of ice. Ice has a cage like structure. Due to 7. (a) BH -4 Þ 4 bond pairs and 0 lone pair ® sp3 hybridised.
this reason ice is less dense than liquid water. Hence \ Tetrahedral geometry
both statement-1 and statement-2 are true and 8. (c) The correct structure of the given compound will be as
statement-2 is correct explanation of statement-1. follows :
74. (a) H H
75. (a) o and p-nitrophenols can be separated by steam
distillation because o-nitrophenol is steam volatile due H H
to presence of weak intramolecular hydrogen bonding
whereas p-introphenol contains intermolecular hydrogen
bonding. Here, both statement-1 and statement-2 are H H
correct and statement-2 is correct explanation of
statement-1. H H
Now, there are 5 p-bonds and 19 s-bonds are present in
EXERCISE - 3 the above molecule.
9. (c) The electronic configuration of the given molecules
Exemplar Questions
are :
1. (b) BF4- and NH +4 both the species are tetrahedral and
sp3 hybridised. N +2 = s1s 2 , s *1s 2 , s2 s 2 , s * 2s 2 , p 2 px2 = pp 2y , s 2 p1z ;
2. (c) CO2 being symmetrical has zero dipole moment. 1 unpaired e–
Amongst HI, SO2 and H2O. H2O exhibit the highest O2 = s1s 2 , s *1s 2 , s2s2 , s * 2s 2 , s2 p2z , p2 px2 » p2 p2y ,
dipole moment as the central atom in H2O contains 2
lone pairs. p * 2 p1x » p * 2 p1y ; 2 unpaired e–s
..
O
..
..

S O22- = s1s 2 , s *1s2 , s2s 2 , s * 2s 2 , s2 pz2 , p2 p2x » p2 p2y ,


H—I H H O O
m = 0.38D m = 1.84 D m = 1.62 D p*2 px2 » p*2 p y2 ; no unpaired e–s
3. (b) Applying the formula to find the hybridisation of B2 = s1s 2 , s *1s 2 , s2s 2 , s * 2s 2 , p 2 p1x » p 2 p1y ;
central atom (nitrogen) :
2 unpaired e–s
1 10. (c) XeF4 Þ square planar, all bonds are equal.
[No. of valence e – of central atom +
2
BF4- Þ tetrahedral, all bonds are equal.
no. of monovalent atoms attached to it +
H
H
(–ve) charge if any – (+ve) charge if any] C2H4 Þ , C = C bond is not equal to
C=C
H
H
1
For, NO2+ = [5 + 0 + 0 – 1] = 2 Þ sp-hybridised C – H bond.
2
SiF4 Þ tetrahedral, all bonds are equal.
1 11. (b) HCl, HI and H2S do not from H-bonds. Only H2O forms
NO3– = [5 + 0 + 1 – 0] = 3 Þ sp2-hybridised
2 hydrogen bonds and each H2O molecule is linked with
1 four H2O molecules.
NH4+ = [5 + 4 + 0 – 1] = 4 Þ sp3-hybridised
2 H
H
O
O
4. (b) Strength of H-bonding is in the order : H
HF > H2O > NH3 H H H
O
Also, each H2O molecule is linked to four other H2O
molecules through H-bonds while each HF molecule is H H H H
linked only to two other HF molecules. O O
Hence, decreasing order of b.p. will be : H2O > HF >
NH3 12. (d) The given electronic configuration shows that the
5. (c) In PO34- ion, formal charge on each O-atom of P – O element belongs to d-block of the periodic table and
known to be a transition element. In transition elements,
Total charge 3
bond = = - = - 0.75 electrons of ns and (n – 1)d subshell take part in bond
Number of O atom 4 formation.
EBD_7327
98 CHEMISTRY

13. (b) For sp2 hybridisation, generally the geometry is usually s1s < s*1s < s2s < s*2s < (p2px ; p2py ) < s2pz <
taken to be triangular planar. p*2px » p*2py < s*2pz
20. (b) E.C of O2 :
Y
s1s2, s*1s2, s2s2, s*2s2 , s 2 p 2z , p 2 px2 » p 2 p2y ,
120° p *2 p1x » p *2 p1y
X
1 1
B.O = ( N b - Na ) = (10 - 6 ) = 2
Y Y 2 2
Thus, bond angle should be 120°. E.C of O+2 :
14. (a) The given electronic configuration of A shows that it
s1s 2 , s *1s 2 , s2 s 2 , s * 2s 2 , s 2 p z2 , p 2 px2 » p2 p 2y ,
is a noble gas because the octet is complete and hence
it will be the stable form. p *2 p1x » p *2 p0y
15. (b) The electronic configuration of C represent chlorine.
Its stable form is Cl2 i.e., C2. 1
B.O = (10 - 5) = 2.5
16. (d) The electronic configuration show that B represents 2
phosphorus and C represents chlorine. The stable
compound formed will be PCl3 i.e., BC3. E.C of O-2 :
17. (b) The bond between B and C will be covalent as both B s1s 2 , s *1s 2 , s2 s 2 , s * 2s 2 , s 2 p z2 , p 2 px2 » p2 p 2y ,
and C are non-metal atoms.
18. (a) The correct increasing order of energies of molecular p *2 px2 » p * 2 p1y
orbitals of N2 is :
s1s < s*1s < s2s < s*2s < (p2px » p2py ) < s2pz < (p*2px 1
Bond order = (10 - 7 ) = 1.5
; p*2py ) < s*2pz 2
19. (d) (a) Be2 = s1s2, s*1s2, s1s2, s* 2s2 Hence, the correct order of bond order will be :
1 O -2 < O 2 < O2+ .
(B.O) = [Number of bonding electrons (Nb ) –
2
21. (a) Out of the given electronic configuration 2s2 2p5
Number of anti-bonding electrons Na ] represents fluorine which is the most electronegative
4-4 element while 3s23p5, 4s24p5 and 5s25p5 represents
= =0 chlorine, bromine and iodine respectively.
2
Bond order of Be2 is zero thus, it does not exist. 22. (b) The electronic configuration of (b) and (d) have exactly
(b) He2 = s1s2, s*1s2 half-filled p-orbitals but (b) being smaller in size than
(d) will have the highest ionisation enthalpy.
2-2
B.O = =0 NEET/AIPMT (2013-2017) Questions
2
23. (a) SF4 has 4 bond pairs and 1 lone pair of electrons, sp3d
Now, He+2 = s1s 2 , s *1s1
F
2 -1 | F
B.O = = 0.5 hybridisation leads to irregular shape S and
2 |
F
As the bond order is not zero, He2+ molecule is resultant m ¹ 0. F
expected to exist. s
s C = C —H
s
24. (d) H—
(c) N2 (7 + 7 = 14) = s1s2, s*1s2 , s2s2 , s*2s2 , p
s| |s
p 2 px2 » p 2 p2y , s 2 pz2 H H
25. (d) (a) ® N 2+
N 2 ¾¾
10 - 4
B.O = =3 B.O. 3 2.5
2
Bond energy decreases
Thus, dinitrogen (N2 ) molecule contain triple bond
Magnetic behaviour changes from diamagnetic to
and as bond order = 3 while no molecule of second
period have more than double bond. Thus, bond paramagnetic
strength of N 2 its maximum amongst th e O 2 ¾¾® O 2+
(b)
homonuclear diatomic molecules belonging to the B.O. 2 2.5
second period. Bond energy increases
(d) The correct order of energies of molecular orbitals Magnetic behaviour does not change.
in N2 molecule is
Chemical Bonding and Molecular Structure 99

two outer atoms, the molecular geometry is bent with


® C 2+
C 2 ¾¾
Ð120º bond angles.
(c) B.O. 2 2.5
35. (c) O +2 ion - Total number of electrons (16 – 1) = 15.
Bond energy decreases
Electronic configuration
Magnetic behaviour changes from diamagnetic to
paramagnetic σ1s 2 < σ*1s 2 < σ2 s 2 < σ* 2s 2 < σ2 px2
NO ¾¾ ® NO+ < π2 p2y = π2 pz2 < π* 2 p1y
(d)
B.O. 2 2.5 N b - N a 10 - 5 5 1
bond energy increases Bond order = = = =2
2 2 2 2
Magnetic behaviour changes from paramagnetic to O 2–(Super oxide ion): Total number of electrons
diamagnetic (16 + 1) = 17.
26. (b) No. of electrons in CO = 6 + 8 = 14 Electronic configuration
No. of electrons in NO+ = 7 + 8 – 1 = 14
σ1s 2 < σ*1s 2 < σ2s 2 < σ* 2s 2 < σ2 p2x
\ CO and NO+ are isoelectronic species.
< π2 p2y = π2 pz2 < π* 2 p2y = π* 2 p1z
Isoelectronic species have identical bond order.
27. (c) Applying VSEPR theory, both NF3 and H2O are sp3 (N - Na ) 10 - 7 3 1
Bond order = b = = =1
hybridized. 2 2 2 2
28. (a) F ¾ Xe ¾ F sp3d and Linear O+22 ion: Total number of electrons

= (16 – 2) = 14 Electronic configuration
Cl ¾ I ¾ Cl sp3d and Linear s1s2 < s*1s2 < s2s2 < s*2s2 < s2px2 < p2py2 = p2pz2
29. (a) Molecular orbital configuration of O-2 is (N b – Na ) 10 – 4 6
Bond order = = = =3
O-2 (17) = s1s2, s*1s2, s2s2, s*2s2, 2 2 2

So bond order: O2 < O2 < O2+ 2+
s2pz2, p2px2 = p2py2, p*2px2 = p*2py1
36. (c) ClO3– and SO3–2 both have same number of electrons
s O s s s s (42) and central atom in each being sp3 hybridised. Both
30. (a) s
O ¬ S=O O ¬ N =O O = C = O are having one lone pair on central atom hence they are
H H p p p p
pyramidal.
31. (b) (BH3)2 or (B2H6) 37. (a) Oxygen molecule (O2) – Total number of electrons = 16
H H H and electronic configuration is
B B σ1s 2 < σ*1s 2 < σ2 s 2 < σ* 2 s 2 < σ2 p x2
H H H
< π2 p2y = π2 pz2 < π* 2 p1y = π* 2 p1z
It contains two 3 centre - 2 electron bonds and present
above and below the plane of molecules compounds N b - N a 10 - 6 4
Bond order = = = =2
which do not have sufficient number of electrons to 2 2 2
form normal covalent bonds are called electron defi- O +2 ion - Total number of electrons (16 – 1) = 15.
cient molecules. Electronic configuration
32. (b) Be2+ = (4 – 2) = 2
σ1s 2 < σ*1s 2 < σ2 s 2 < σ* 2s 2 < σ2 px2
is isoelectronic with Li+ (3 – 1 = 2)
Since both have same number of electrons in their < π2 p2y = π2 pz2 < π* 2 p1y
outermost shell. N b - Na 10 - 5 5 1
33. (c) Dipole moment of NH3 > NF3 Bond order = = = =2
2 2 2 2
d+
d –.
.
N N O2– (Super oxide ion) Total number of electrons
(16 + 1) = 17 . Electronic configuration
–F F
d+H H (d ) F –
(d )
H –
(d ) σ1s 2 < σ*1s 2 < σ2 s 2 < σ* 2s 2 < σ2 px2
m = 1.4D m = 0.23D
(F is more electronegative than N) < π2 p2y = π2 pz2 < π* 2 p2y = π* 2 p1z
1 (Nb - Na ) 10 - 7 3 1
34. (b) Hybridization of NO3– = (5 + 0 + 1 - 0) Bond order = = = =1
2 2 2 2 2
6 2 38. (b) NO2+ has sp hybridisation so it is linear with bond angle
= = 3 = sp hence geometry is trigonal planner.. = 180°.
2
NO2– (nitrite ion) also has sp2 hybridization and gives
a trigonal planner geometry but because there are only
EBD_7327
100 CHEMISTRY

O (In presence of ligand, pairing of electron occurs)


\ Square planar.
39. (a) Xe 43. (b) CH4 NH3 H2O
O O O
H
Number of s bonds = 4
Number of p bonds = 4 C 109°28' N
40. (a) According to molecular orbital theory as bond order H H H 107°H
decreases stability of the molecule decreases H H
1
Bond order = (N b – Na)
2 O
1 H 104.5° H
Bond order for O+2 = (10- 5) = 2.5
2 Tetrahedral; Trigonal Bent
pyramidal
1
Bond order for O 2 = (10 - 6) = 2 Note: The geometry of H2 O should have been
2 tetrahedral if there are all bond pairs. But due to presence
- 1 of two lone pairs the shape is distorted tetrahedral.
Bond order for O 2 = (10 - 7) = 1.5
2 Hence bond angle reduced to 104.5° from 109.5°.
2- 1 44. (a) According to VSEPR theory order of repulsion in
Bond order for O2 = (10 - 8) = 1.0 between lp – lp, lp – bp and bp – bp is as under
2
hence the correct order is lp – lp > lp – bp > bp – bp
O+2 > O 2 > O2– > O 2–
2
45. (b) IBr2- , XeF2
41. (d) XeF4, XeO4 Total number of valence electrons are equal in both the
O species and both the species exhibit linear shape.
46. (c) BCl3 is trigonal planar and hence the bond angle is
F F Xe
Xe 120°.
O O Cl
F F O
(Square planar) [Tetrahedral] 120°
B
42. (a) Ni2Å = [Ar]18 4s0 3d8
Cl Cl
Valence bond theory can be used to predict shape.

47. (b) CN and CO have same no. of electrons and have same
bond order equal to 3.
3d 4s 4p
2
dsp hybridization
5 States of Matter

INTERMOLECULAR FORCES AND THERMAL ENERGY Intermolecular Forces Vs Thermal Energy


Intermolecular Forces: Intermolecular forces tend to keep molecules together while thermal
These are the forces of attraction and repulsion between energy tends to keep them apart. The three states of matter are the
interacting particles. (The term does not include electrostatic forces result of balance between intermolecular forces and thermal energy
or forces between covalent bonds). of molecules. The predominance of thermal energy and molecular
interaction energy of a substance in the three states is depicted as
Attractive intermolecular forces are called van der Waals forces.
follows:
The different types of van der Waals forces are discussed below:
(i) Dispersion forces or London forces: The forces of attraction Gas Liquid Solid
Intermolecular interaction increases ®
between induced momentary dipoles are called London forces. ¾¾¾¾¾¾¾¾¾¾¾¾¾¾¾¾¾¾¾¾¾¾
These forces arise when the electron cloud of the molecule is
Thermal energy decreases
distorted momentarily, thus generating a momentary dipole ¾¾¾¾¾¾¾¾¾®
which in turn induces dipole in the neighbouring molecules. GASEOUS STATE
These are then attracted to each other. They are important
The gaseous state is characterized by the following physical
only at short distances and their magnitude depends on the
properties:
polarisability of the particle.
· Gases are highly compressible.
(ii) Dipole-Dipole forces : These forces act between molecules
· They exert pressure equally in all the directions.
possessing permanent dipole. The positive pole of one
molecule is attracted towards the negative pole of the other · Gases have much lower density than solids and liquids.
molecule. · The volume and shape of gases are not fixed. They assume
volume and shape of the container.
The magnitude of these forces depends on the polarity of
molecules which, in turn, depends on the electronegativities · Gases mix evenly and completely in all proportions.
of the atoms present in the molecule. The attractive forces Gas Laws
decreases with increase of distance between dipoles. Boyle’s law: According to this law, at constant temperature, the
(iii) Dipole-Induced dipole forces : They operate between pressure of a fixed amount of gas varies inversely with its volume.
molecules having permanent dipole and molecules lacking Mathematically.
permanent dipole. Permanent dipole of the polar molecule 1
induces dipole on electrically neutral molecule by deforming Pµ (at constant T and n)
V
its electronic cloud.
(iv) Hydrogen bond : It is a special case of dipole - dipole k
ÞP= (k ® constant of proportionality)
interactions. It arises in molecules in which highly polar N-H, V
O-H or H-F bonds are present. Þ PV = k
Strength of the H-bond depends on the coulombic interaction If a fixed amount of gas at constant temperature T occupying
between lone pair of electrons of electronegative atom of volume V1 at pressure P1 undergoes expansion, so that
one molecule and H-atom of the other molecule. volume becomes V2 and pressure becomes P2, then according
to Boyle’s law :
Thermal Energy
P1V1 = P2V2 = constant
It is the energy of a body arising from motion of its atoms or
molecules. It is directly proportional to temperature of substance. P1 V2
Þ =
It is a measure of average KE of particles. P2 V1
EBD_7327
102 CHEMISTRY

Graphically, Boyle’s law can be represented as: P1


P1 < P2
P2

V
P
T2
T1 – ve 0 +ve
T
V – 273.15
Gay lussac’s law : It states that at constant volume, pressure of a
fixed amount of a gas varies directly with the temperature, i.e.
PµT
P P
or, = constant = k
T
Hence, if the pressure of a gas P1 at temperature T1 changes
to P2 at T2, volume remaining constant, we have
P1 P2
PV = = constant
T1 T2
or, log P – log T = constant
æ t ö
T2 Pt = P0 ç1 + ÷
è 273.15 ø
T1 Graphically,
P
V1
V2

1/V V1 < V2
P
Charle’s law : According to this law, pressure remaining constant,
the volume of a fixed mass of a gas is directly proportional to its
absolute temperature, i.,e.
V µ T or V = kT T
Other way of representing it is following: Avogadro law : It states that equal volumes of all gases under the
same conditions of temperature and pressure contain equal number
For each degree change of temperature, the volume of sample of a
of molecules.
gas changes by the fraction of 1/273.15 of its volume at 0ºC.
i.e. V µ n, where n ® Number of molecules of gas
t V = kn
Vt = V0 + V0
273.15 Thus, one mole of each gas at standard temperature and pressure
will have same volume. Standard temperature and pressure means
æ t ö æ 273.15 + t ö 273.5 K (0°C) temperature and 1 bar (105 Pa) pressure.
Þ Vt = V0 ç1 + ÷ = V0 ç 273.15 ÷
è 273.15 ø è ø m
As n = where, m ® given mass, M ® molar mass
M
æT ö V T
Þ Vt = V0 ç t ÷ Þ t = t m m
T
è 0ø V0 T0 \ V =k Þ M =k =k d
M V
Where T0 = 273.15 and is called Kelvin temperature or Þ Vµd where, d ® density of gas
Absolute temperature
Ideal gas equation : A gas that follows Boyle’s law, Charle’s law
\ 0°C = 273.15K and Avogadro law strictly is called an ideal gas. Combining the
three laws, we can get the ideal gas equation as discussed below:
At – 273.15°C, volume of a gas will be zero. This means that
the gas will not exist. This temperature at which gases are 1
Boyle’s law : Vµ at constant T and n
supposed to occupy zero volume is called Absolute zero. P
Graphically, it is represented as: Charle’s law : V µ T at constant P and n
States of Matter 103

Avogadro law : V µ n at constant P and T P


Rate of diffusion (r) µ
nT d
\ Vµ
P
r1 p1 d2
=
RnT r2 p2 d1
ÞV=
P As pressure is constant
or, PV = nRT Ideal Gas equation or Equation of State. r1 d2
=
R=Universal gas constant r2 d1

R= =
5
( )(
PV 10 Pa 22.71´10 m
-3 3
) Where r1 & r2 are rates of diffusion of two gases and d1 & d2
are densities.
nT (1 mol ) ( 273.15 K ) \ 2 × vapour density = Molecular mass
= 8.314 Pa m3 K–1 mol–1. \
r1 M2
=
= 8.314 × 10–2 bar L K–1 mol–1. r2 M1
= 8.314 J K–1 mol–1.
Where M1 & M2 are the molecular masses of two gases.
If T, P and V of a fixed amount of gas vary from T1, V1 and P1
to T2, V2 and P2, then Volume of gas diffused
Rate of diffusion =
Time taken for diffusion
P1 V1 P V
= nR and 2 2 = nR
T1 T2 KINETIC MOLECULAR THEORY OF GASES
This theory attempts to elucidate the behaviour of gases. The
P1 V1 P2 V2 assumptions or postulates of this theory are:
or = combined gas Law
T1 T2 (i) Gases consist of large number of identical particles which are
very small and far apart so that the actual volume of molecules
Density and Molar Mass of Gaseous Substance from Ideal Gas is negligible in comparison to empty space between them.
Equation, This explains the compressibility of gases.
n P (ii) There is no force of attraction between the particles of a gas
= at ordinary T and P. This is why gases can expand and occupy
V RT
all the space available to them.
m P æ mö (iii) Particles of a gas are always in constant and random motion.
= çQ n = ÷ If the particles were at rest and occupied fixed position, a gas
MV RT è Mø
would have had a fixed shape, which is not so.
d P (iv) Particles of a gas move in all possible directions in straight
=
M RT lines. During their random motion, they collide with each
other and with the walls of the container. Pressure is exerted
dRT by the gas as a result of collision of the particles with the
M= walls of the container.
P
(v) Collisions of gas molecules are perfectly elastic. This means
Dalton’s Law of Partial Pressures : According to this law, the that total energy of molecules before and after collision remains
total pressure exerted by a mixture of non-reactive gases is equal the same. There may be exchange of energy between colliding
to the sum of partial pressures of individual gases, i.e. molecules, their individual energies may change, but the sum
PTotal = P1 + P2 + P3 + ------------- (at constant T and V) of their energies remains constant.
Pressure exerted by saturated water vapour is called aqueous (vi) At any particular time, the different particles of the gas have
tension. different speeds and hence different kinetic energies.
\ Pdry gas = PTotal – Aq. tension (vii) The average KE of gas molecules is directly proportional to
the absolute temperature.
Partial pressure in terms of mole fraction
P1 = x1 PTotal. MAXWELL’S DISTRIBUTION OF MOLECULAR
P2 = x2 PTotal. SPEEDS/ENERGIES
In general, P = x PTotal. Maxwell and Boltzmann, however, showed that as a result of
where x ® mole fraction; P ® partial pressure collisions, though some molecules are speeded up, some others
are slowed down and hence the fractions of molecules possessing
Graham’s Law of Diffusion or Effusion : According to this law, at particular speed remains constant at constant temperature. This
constant temperature and pressure, the rate of diffusion or effusion is called Maxwell-Boltzmann distributions.
of a gas is inversely proportional to the square root of its density Maxwell gave distribution curves of molecular speeds for CO2 at
and is directly proportional to its pressure. different temperatures.
EBD_7327
104 CHEMISTRY

Special features of the curve are : DEVIATION FROM IDEAL GAS BEHAVIOUR : REAL
(i) Fraction of molecules with too high or too low speeds is very GASES
small. A gas that follows Boyle’s law, Charle’s law and Avogadro law
(ii) No molecules has zero velocity. strictly is called an ideal gas. Such a gas is hypothetical. Real
(iii) Initially the fraction of molecules increases with velocity till gases follow these laws only under certain specific conditions
the peak of the curve which pertains to most probable velocity when forces of interaction are practically negligible. This is
and thereafter it falls with increase in velocity. discussed as follows:
Two questions arise :
(i) Why do gases deviate from ideality?
Ump
(ii) Under what conditions gases deviate from ideality?
300 K ( T1) T1< T2< T3 Gases deviate from ideality because two assumptions of kinetic
Ump theory do not hold good:
Fraction of molecules

Uav (i) There is no force of attraction between molecules of a gas.


Urms
Ump (ii) Volume of molecules of a gas is negligible compared to volume
of the gas.
1500 K(T 2)
1800 K(T 3)
Equation of State for Real Gases (Van der Waals
Equation)
Molecular speed
Correction for pressure : Real gases show deviations from ideal
gas law because molecules interact with each other. At high
pressures, molecules of gas are very close to each other. Molecular
Speed Related to Gaseous State interactions start operating. As a result, pressure exerted by the
gas is lower than the pressure exerted by the ideal gas.
(a) Root mean square speed (RMS speed) -
Hence, corrected pressure :
u12 + u22 + u32 + ......un2
u= an 2
n Pideal = Preal + a ® constant
V2
3PV 3RT 3P Volume correction : The volume occupied by the molecules also
C= = =
M M d becomes significant at high pressure when the molecules are quite
close together because instead of moving in volume V, these are
(b) Average speed (AVS) :
now restricted to volume (V – nb) where nb is approximately the
8RT volume occupied by molecules themselves.
Average speed = i.e., corrected volume = (V – nb)
pm
Thus, taking into account the corrections for P and V, we have
Average speed = .9213 × RMS speed
RMS speed = 1.085 × Average speed
æ an 2 ö
(c) Most probable speed (MPS) : çP + 2 ÷ ( V - nb ) = nRT
ç V ÷
è ø
2RT
Most probable speed = van der Waals equation a, b ® van der Waals constants
M
units of ‘a’ ® atm. litre2 mole–2
MPS = .816 × RMS ; RMS = 1.224 MPS
units of ‘b’ ® litres mole–1
Relation between the three types of velocities
Value of ‘a’ is a measure of magnitude of intermolecular attractive
a :v :u forces within the gas and is independent of temperature and
2 RT 8RT 3RT pressure.
: : Also, at very low temperature, intermolecular forces become
M pM M
significant. Thus, real gases show ideal behaviour when conditions
8
of temperature and pressure are such that the intermolecular forces
2: : 3 are practically negligible.
p
As b is the effective volume of the gas molecules, the constant
1.414 : 1.595 : 1.732 value of b for any gas over a wide range of temperature and pressure
1 : 1.128 : 1.234 i.e., a < v < u. indicates that the gas molecules are incompressible.
States of Matter 105

Study of Deviation Their molecules are held together by attractive intermolecular


The deviation from ideal behaviour can be measured in terms of forces. They have definite volume and the molecules can move
compressibility factor Z which is given by past one another freely and thus liquids can flow, can be poured
and can assume the shape of the container.
PV
Z= Some physical properties of liquids are:
nRT
(i) Vapour Pressure
For ideal gas, Z = 1 at all temperatures and pressures.
It is the pressure exerted by the vapours in equilibrium with
At very low pressure, all gases have Z = 1 and behave ideally. At
the liquid. The temperature at which vapour pressure of liquid
high pressure, all gases have Z > 1. They are difficult to compress.
is equal to external pressure is called boiling point of the
At intermediate pressure, most gases have Z < 1.
liquid.
The temperature at which a real gas obeys ideal gas law over an
At 1 atm, it is the normal boiling point and at 1 bar, it is
appreciable range of pressure is called Boyle temperature or Boyle
standard boiling point. At high altitudes, atmospheric
point. At low pressure and high temperature, gases show ideal
pressure is low. Therefore, liquids at high altitudes boil at
behaviour.
lower temperatures in comparison to that at sea level. Since
The temperature at which a real gas obeys ideal gas laws over an water boils at lower temperature on hills, pressure cooker is
appreciable range of pressure is called Boyle temperature or Boyle used for cooking food.
point TB. TB and two van der Waal’s constants are related as
below (ii) Surface Tension
a A molecule in the bulk of the liquid experiences equal
TB = intermolecular forces from all sides but the molecule on the
bR
surface experiences a net attractive force towards the interior
Liquifaction of Gases of the liquid. This phenomena is called surface tension.
Critical temperature (Tc) of a gas is the highest temperature at It is defined as the force acting at right angles to the surface
which liquifaction of the gas first occurs. along 1 cm length of the surface. Thus, the units of surface
Liquifaction of gas requires cooling as well as considerable tension are dynes per cm (or Nm–1 in SI system).
compression. Compression brings the molecules in close vicinity Due to net downward force at the surface, liquids tend to
and cooling slows down the movement of molecules. Therefore, have minimum number of molecules at their surface. If surface
intermolecular interactions may hold the closely and slowly moving of liquid is increased by pulling a molecule from the bulk,
molecules together and the gas liquifies. attractive forces will have to be overcome. The energy required
Critical Constants to increase the surface area of the liquid by one unit is called
(i) Critical temperature (Tc) : The temperature above which surface energy.
gas cannot be liquefied whatever pressure is applied is called Magnitude of surface tension of a liquid depends on
8a attractive forces between molecules. As temperature
critical temperature Tc = increases, attractive forces decreases and thus, surface
27bR
tension also decreases.
(ii) Critical pressure (Pc) : The minimum pressure required to
liquefy a gas at its critical temperature is called critical Since a sphere has minimum surface area and lowest energy
a state of a liquid is when surface area is minimum, therefore,
pressure. Pc = mercury drops are spherical in shape. Liquid tends to rise (or
27b 2 fall) in capillary because of surface tension.
(iii) Critical volume (Vc) : The volume occupied by one mole of
a gas at the critical temperature and critical pressure is called (iii) Viscosity
critical volume. It is a measure of resistance to flow which arises due to internal
Vc = 3b friction between layers of fluid as they slip past one another
while liquid flows.
There is continuity between gaseous and liquid state. The term
fluid is used for either a liquid or a gas to recognise this continuity. A force is required to maintain the flow of layers. It is given
A liquid can be viewed as a very dense gas. Liquid and gas can be du du
distinguished only when the fluid is below its critical temperature, by: F = h A where A ® area of contact ® velocity
dz dz
since in that case liquid and gas are in equilibrium and a surface gradient h ® coefficient of viscosity.
separating the two phases is visible. At critical temperature, liquid
SI unit of h is 1 N sm–2 and its cgs unit is poise
passes into gaseous state imperceptibly and continuously, the
surface separating two phases disappears. A gas below T c can be Viscosity of liquids decreases as temperature increases
liquified by applying pressure, and is called vapour of the because at high temperature, molecules have high KE and
substance. can overcome intermolecular forces to slip past one another
between the layers.
LIQUID STATE Greater viscosity, more slowly the liquid flows. H-bonding
Molecules in liquids are so close that there is very little empty and van der Waals forces are strong enough to cause high
space between them and under normal conditions, liquids are viscosity. Glass is an extremely viscous liquid. It is so viscous
denser than gases. that many of its properties resemble solids.
106

CONCEPT MAP
CHEMISTRY

EBD_7327
States of Matter 107

1. Correct gas equation is : 10. Select one correct statement. In the gas equation, PV = nRT
PV (a) n is the number of molecules of a gas
V1T2 V2T1 1 1 = T1
(a) = (b) (b) V denotes volume of one mole of the gas
P1 P2 P2V2 T2
(c) n moles of the gas have a volume V
PT
1 2 = P2V2 V1V2
(c) (d) = P1P2 (d) P is the pressure of the gas when only one mole of gas
V1 T2 T1T2 is present.
2. A gas of volume of 15 ml at 300 K and 740 mm of Hg. Find 11. 500 ml of nitrogen at 27°C is cooled to –5°C at the same
the temperature if volume becomes 10 ml at 760 mm pressure. The new volume becomes
pressure of Hg. (a) 326.32 ml (b) 446.66 ml
(a) 209 K (b) 200 K (c) 205 K (d) 275 K
(c) 546.32 ml (d) 771.56 ml
3. The K.E. for 14 g of nitrogen gas at 127°C is nearly
(molecular mass of nitrogen is 28 g/mole) and gas constant 12. 600 c.c. of a gas at a pressure of 750 mm of Hg is compressed
is 8.31 J/mol K) to 500 c.c. Taking the temperature to remain constant, the
(a) 8.3 kJ (b) 4.15 kJ (c) 2.5 kJ (d) 3.3 kJ increase in pressure, is
4. At constant temperature, for a given mass of an ideal gas (a) 150 mm of Hg (b) 250 mm of Hg
(a) the ratio of pressure and volume always remains (c) 350 mm of Hg (d) 450 mm of Hg
constant. 13. From a heated mixture of nitrogen, oxygen and carbon, two
(b) volume always remains constant. compounds (out of the many obtained) are isolated. The
(c) pressure always remains constant. rates of diffusion of the two isolated compounds are almost
(d) the product of pressure and volume always remains identical. The two compounds are
constant. (a) N2O and CO2 (b) CO and NO
5. The molecular weight of two gases are 100 and 81 (c) CO2 and NO2 (d) N2O and CO
respectively. Their rates of diffusions are in the ratio: 14. The ratio of the rate of diffusion of helium and methane
(a) 81 : 100 (b) 100 : 81 under identical condition of pressure and temperature will
(c) 10 : 9 (d) 9 : 10 be
6. Which is not true in case of an ideal gas ? (a) 4 (b) 2 (c) 1 (d) 0.5
(a) It cannot be converted into a liquid 15. Gas equation PV = nRT is obeyed by
(b) There is no interaction between the molecules (a) Only isothermal process (b) Only adiabatic process
(c) All molecules of the gas move with same speed (c) Both (a) and (b) (d) None of these
(d) At a given temperature, PV is proportional to the 16. Densities of two gases are in the ratio 1:2 and their
amount of the gas temperatures are in the ratio 2:1 then the ratio of their
7. The correct value of the gas constant ‘R’ is close to : respective pressures is
(a) 0.082 litre-atmosphere K (a) 1:1 (b) 1:2 (c) 2:1 (d) 4:1
(b) 0.082 litre-atmosphere K–1 mol–1 17. A cylinder of 5 L capacity, filled with air at NTP is connected
(c) 0.082 litre – atmosphere–1 K mol–1 with another evacuated cylinder of 30 L of capacity. The
(d) 0.082 litre –1 atmosphere – 1 K mol resultant air pressure in both the cylinders will
8. In two separate bulbs containing ideal gases A and B (a) 10.8 cm of Hg (b) 14.9 cm of Hg
respectively, the density of gas A is twice of that of gas B, (c) 21.8 cm of Hg (d) 38.8 cm of Hg
while mol wt. of gas A is half of that of gas B at the same 18. Volume occupied by a gas at one atmospheric pressure and
temperature, pressure. PA/PB will be: 0°C is V mL. Its volume at 273 K will be
(a) 1 (b) 4 (a) Vml (b) V/2 ml
(c) 1/4 (d) 1/2 (c) 2 V (d) None of these
9. Under what conditions will a pure sample of ideal gas not
19. Van der Waal’s equation
only exhibit a pressure of 1 atm, but also a concentration of
2 moles per litre? é a ù
(a) At STP ê P + 2 ú (V - b ) = nRT is applicable for :
(b) When volume is 22.41litre ë V û
(c) At 6.1 K (a) Ideal gas (b) Non-ideal gas
(d) When R has no unit (c) Both (a) and (b) (d) None of these
EBD_7327
108 CHEMISTRY

20. Which one of the following indicates the value of the gas 30. Equal weights of methane and hydrogen are mixed in an
constant R? empty container at 25ºC. The fraction of the total pressure
(a) 1.987 cal K–1 mol–1 (b) 8.3 cal K–1 mol–1 exerted by hydrogen is :
–1
(c) 0.0821lit K mol –1 (d) 1.987 Joules K–1 mol–1 1 8 1 16
21. 16 g of oxygen and 3 g of hydrogen are mixed and kept at 760 (a) (b) (c) (d)
2 9 9 17
mm of Hg pressure and 0° C. The total volume occupied by
31. The rate of diffusion of methane at a given temperature is
the mixture will be nearly
twice that of a gas X. The molecular weight of X is
(a) 22.4 litres (b) 33.6 litres (c) 448 litres (d) 44800 ml
(a) 64.0 (b) 32.0 (c) 4.0 (d) 8.0
22. If P is pressure and r is density of a gas, then P and r are
32. X mL of H2 gas effuses through a hole in a container in 5
related as
seconds. The time taken for the effusion of the same volume
(a) P µ r (b) P µ (1/r)
of the gas specified below under identical conditions is :
(c) P µ r2 (d) P µ (1/r2)
(a) 10 seconds : He (b) 20 seconds : O2
23. Pure hydrogen sulphide is stored in a tank of 100 litre capacity
at 20° C and 2 atm pressure. The mass of the gas will be (c) 25 seconds : CO (d) 55 seconds : CO2
(a) 34 g (b) 340 g (c) 282.68 g (d) 28.24 g 33. The inversion temperature Ti (K) of hydrogen is (given van
der Waal’s constants a and b are 0.244 atm L2 mol–2 and
24. If three unreactive gases having partial pressures PA, PB
0.027 L mol–1 respectively)
and PC and their moles are 1, 2 and 3 respectively then their
(a) 440 (b) 220 (c) 110 (d) 330
total pressure will be
34. The ratio between most probable velocity, mean velocity
P + PB + PC and r.m.s velocity is :
(a) P = PA + PB + PC (b) P= A
6
(a) 2 : 8/p : 3 (b) 2 : 3 : 8/ p
PA + PB + PC
(c) P= (d) None of these (c) 1 : 2 : 3 (d) 1 : 2 : 3
3
25. When a substance is dissolved in a solvent, the vapour 35. Root mean square velocity of a gas molecule is proportional
pressure of solvent decreases. It brings: to
(a) a decrease in boiling point of solution (a) m1/2 (b) m0 (c) m–1/2 (d) m
(b) an increase in boiling point of the solution 36. Which of the following is not a property of liquid state?
(c) a decrease in freezing point of the solution (a) Intermolecular force of attraction in a liquid is quite
(d) an increase in freezing point of the solution large
26. Dalton’s law of partial pressure will not apply to which of (b) All liquids accompanied by cooling on evaporation
the following mixture of gases (c) Lower the boiling point of a liquid, greater is its vapour
(a) H2 and SO2 (b) H2 and Cl2 pressure at room temperature
(c) H2 and CO2 (d) CO2 and Cl2 (d) A liquid boils at high temperature at the top of a mountain
than at the sea level
27. A gas diffuse1/5 times as fast as hydrogen. Its molecular
37. In a closed flask of 5 litres, 1.0 g of H2 is heated from 300 to
weight is
600 K. Which statement is not correct?
(a) 50 (b) 25 (c) 25 2 (d) 50 2 (a) Pressure of the gas increases
28. Dominance of strong repulsive forces among the molecules (b) The rate of collision increases
of the gas (Z = compressibility factor): (c) The number of moles of gas increases
(a) Depends on Z and indicated by Z = 1 (d) The energy of gaseous molecules increases
(b) Depends on Z and indicated by Z > 1 38. Internal energy and pressure of a gas per unit volume are
(c) Depends on Z and indicated by Z < 1 related as :
(d) Is independent of Z.
29. The volume-temperature graphs of a given mass of an ideal (a) P = 2 E (b) P = 3 E (c) P = 1 E (d) P = 2 E
gas at constant pressure are shown below. 3 2 2
39. Which of the following gases will have highest rate of
p2 p diffusion?
3
V p1 (a) CO2 (b) NH3
(c) O2 (d) N2
40. The temperature of the gas is raised from 27°C to 927°C, the
root mean square velocity is

O 273 T(K) (a) 927 / 27 time the earlier value

What is the correct order of pressures ? (b) same as before


(a) p1 > p3 > p2 (b) p1 > p2 > p3 (c) halved
(c) p2 > p3 > p1 (d) p2 > p1 > p3 (d) doubled
States of Matter 109

41. Which of the following expressions correctly represents the 51. The ratio between the root mean square speed of H2 at 50 K
relationship between the average molar kinetic energy, KE , and that of O2 at 800 K is,
of CO and N2 molecules at the same temperature ? (a) 4 (b) 2
(c) 1 (d) 1/4
(a) KE CO < KE N 52. The root mean square velocity of an ideal gas at constant
2
pressure varies with density (d) as
(b) KE CO > KE N
2
(a) d 2 (b) d (c) d (d) 1/ d
(c) KE CO = KE N 53. The root mean square velocity of one mole of a monoatomic
2
(d) cannot be predicted unless volumes of the gases are gas having molar mass M is ur.m.s.. The relation between the
given average kinetic energy (E) of the gas and ur.m.s. is
42. The inversion temperature for van der Waal's gas is :
3E 2E
a æ 2a ö (a) ur.m.s. = (b) ur.m.s. =
(a) Ti = (b) Ti = ç ÷ 2M 3M
(Rb) è Rb ø
2E E
a (c) ur.m.s. = (d) ur.m.s. =
(c) Ti = 0.5 T (d) Ti = M 3M
2( R / b)
54. The total pressure of a mixture of two gases is :
43. Root mean square velocity of a molecule is 1000 m/s. The (a) The sum of the partial pressures
average velocity of the molecule is : (b) The difference between the partial pressures
(a) 455.55 m/s (b) 675.55 m/s
(c) The product of the partial pressures
(c) 921.58 m/s (d) 1221.58 m/s
(d) The ratio of the partial pressures.
44. According to the kinetic theory of gases, in an ideal gas,
between two successive collisions a gas molecule travels 55. An ideal gas can’t be liquefied because
(a) in a wavy path (a) its critical temperature is always above 0°C
(b) in a straight line path (b) Its molecules are relatively smaller in size
(c) with an accelerated velocity (c) it solidifies before becoming a liquid
(d) in a circular path (d) forces between its molecules are negligible
45. As the temperature is raised from 20ºC to 40ºC, the average
56. When is deviation more in the behaviour of a gas from the
kinetic energy of neon atoms changes by a factor of which
ideal gas equation PV = nRT ?
of the following ?
(a) At high temperature and low pressure
(a) 313 293 (b) (313 / 293) (b) At low temperature and high pressure
(c) 1 2 (d) 2 (c) At high temperature and high pressure
(d) At low temperature and low pressure
46. When universal gas constant (R) is divided by Avogadro
no. (N0), then the value of R / N0 is equivalent to 57. Some moles of O2 diffuse through a small opening in 18 s.
(a) Rydberg's constant Same number of moles of an unknown gas diffuse through
(b) Boltzmann's constant the same opening in 45 s. molecular weight of the unknown
(c) Planck's constant gas is :
(d) Van der waal's constant
47. The ratio of root mean square velocity to average velocity 452 18 2
(a) 32 ´ (b) 32 ´
of a gas molecule at a particular temperature is 18 2 452
(a) 1.086 : 1 (b) 1 : 1.086 (c) 2 : 1.086 (d) 1.086 : 2
48. The rms speed at NTP of a gas can be calculated from the 45 18
(c) 32 ´ (d) 32 2 ´
expression : 18 45
(a) 3P / d (b) 3PV / M 58. At which one of the following temperature-pressure
conditions the deviation of a gas from ideal behaviour is
(c) 3RT / M (d) All of these
expected to be minimum?
4 (a) 350 K and 3 atm. (b) 550 K and 1 atm.
49. Ratio of molecular weights of A and B is then ratio of
25 (c) 250 K and 4 atm. (d) 450 K and 2 atm.
rates of diffusion will be : 59. In van der Waal’s equation of state of the gas law, the
(a) 5 : 1 (b) 5 : 2 constant ‘b’ is a measure of
(c) 25 : 3 (d) 25 : 4 (a) volume occupied by the molecules
50. When the temperature of an ideal gas is increased from 27ºC
(b) intermolecular attraction
to 927ºC, the kinetic energy will be :
(a) same (b) eight times (c) intermolecular repulsions
(c) four times (d) twice (d) intermolecular collisions per unit volume
EBD_7327
110 CHEMISTRY

60. At what centigrade temperature will the volume of gas 68. Gas deviates from ideal gas nature because molecules
becomes 2x, if the volume of this gas is ‘x’ at 0° C at constant (a) are colouress
pressure? (b) attaract each other
(a) 0°C (b) 237°C
(c) contain covalent bond
(c) 273°C (d) 546 ºC
(d) show Brownian movement.
61. Value of universal gas constant (R) depends upon
(a) Number of moles of gas 69. The van der Waal’s constant ‘a’ for four gases P, Q, R and S
(b) Volume of gas are 4.17, 3.59, 6.71 and 3.8 atm L2 mol–2 respectively.
(c) Temperature of gas Therefore, the ascending order of their liquefaction is
(d) None of these (a) R < P < S < Q (b) Q < S < R < P
62. The temperature at which a real gas obeys the ideal gas laws (c) Q < S < P < R (d) R < P < Q < S
over a wide range of pressure is 70. At low pressure the van der Waal’s equation is reduced to
(a) critical temperature (b) Boyle temperature
pVm ap pVm b
(c) inversion temperature (d) reduced temperature (a) Z= =1- (b) Z= = 1+ p
RT RT RT RT
63. In van der Waal’s equation of state for a non-ideal gas, the
term that accounts for intermolecular forces is pVm a
(c) pVm = RT (d) Z= =1-
(a) (V – b) (b) RT RT RT
æ a ö 71. Same mass of CH4 and H2 is taken in container, The partial
(c) çè P + 2 ÷ø (d) (RT)–1
V pressure caused by H2 is
64. The compressibility factor for an ideal gas is 8 1
(a) 1.5 (b) 1.0 (c) 2.0 (d) ¥ (a) (b)
9 9
65. Graph between P and V at constant temperature is
1
(a) straight (c) (d) 1
(b) curved increasing 2
(c) straight line with slope 72. In which molecule the van der Waal’s force is likely to be the
(d) none of these most important in determining the m.pt. and b.pt.?
(a) H2S (b) Br2
66. A gas described by van der Waal’s equation
(c) HCl (d) CO
(i) behaves similar to an ideal gas in the limit of large molar 73. Soap helps in cleaning clothes, because
volume (a) chemical of soap change
(ii) behaves similar to an ideal gas in the limit of large (b) it increases the surface tension of the solution
pressure (c) it absorbs the dirt
(iii) is characterised by van der Waal's coefficients that are (d) it lowers the surface tension of the solution
dependent on the identity of the gas but are
74. The r.m.s Velocity of hydrogen is 7 times the r.m.s velocity
independent of the temperature
of nitrogen. If T is the temperature of the gas , than
(iv) has the pressure that is lower than the pressure exerted
by the same gas behaving ideally (a) T (H 2 ) = T ( N 2 ) (b) T (H 2 ) > T ( N 2 )
(a) (i) and (ii) (b) (i) and (iii) (c) T (H 2 ) < T ( N 2 ) (d)T (H 2 ) = 7 T ( N 2 )
(c) (i), (ii) and (iii) (d) (ii) and (iv)
75. If temperature increases, the surface tension of a liquid
67. Rate of diffusion of NH3 is twice that of X. What is the (a) increases
molecular mass of X? (b) decreases
(a) 68 (b) 48 (c) remains the same
(c) 12 (d) 8 (d) increases then decreases

1. The molecular velocities of two gases at the same temperature 2. Reducing the pressure from 1.0 atm to 0.5 atm would change
are u1 and u2. Their masses are m1 and m2 respectively . the number of molecules in one mole of ammonia to
Which of the following expression is correct. (a) 25% of its initial value (b) 50% of its initial value
(a) m1 / u12 = m 2 / u 2 2 (b) m1u1 = m 2 u 2 (c) 75% of its initial value (d) None of the above
3. Under what conditions will a pure sample of an ideal gas not
(c) m1 / u1 = m 2 / u 2 (d) m1u12 = m 2u 2 2 only exhibit a pressure of 1 atm but also a concentration of
1 mole litre–1 ? (R = 0.082 litre atm mol–1deg–1)
States of Matter 111

(a) At STP 14. Calculate the total pressure in a 10.0 L cylinder which contains
(b) When V = 22.4 litres 0.4g helium, 1.6 g oxygen and 1.4 g nitrogen at 27°C.
(c) When T = 12 K (a) 0.492 atm (b) 49.2 atm
(d) Impossible under any conditions (c) 4.52 atm (d) 0.0492 atm
4. The average kinetic energy of 28 g CO at, 300 K is E kcal. The 15. When helium is allowed to expand into vacuum, heating effect
average kinetic energy of 2 g H2 at the same temperature is observed. Its reason is that
would be..... k cal. (a) helium is an ideal gas
(a) E (b) 14 E (b) helium is an inert gas
(c) 1/14 E (c) 28 E (c) the inversion temperature of helium is very low
5. Cyclopropane and oxygen at partial pressures 170 torr and (d) the boiling point of helium is the lowest amongst the
570 torr respectively are mixed in a gas cylinder. What is the
elements
ratio of the number of moles of cyclopropane to the number
of moles of oxygen (nC3H6/nO2)? 16. Diffusion of helium gas is four times faster than
(a) CO2 (b) SO2 (c) NO2 (d) O2
170 ´ 42 170 æ 170 570 ö 17. A bottle of dry ammonia and a bottle of dry hydrogen chloride
(a) = 0.39 (b) ç + ÷ » 0.19
570 ´ 32 42 è 42 32 ø connected through a long tube are opened simultaneously
170 170 at both ends the white ammonium chloride ring first formed
(c) = 0.23 (d) = 0.30
740 570 will be
6. 14 g of N2 and 36 g of ozone are at the same pressure and (a) at the centre of the tube.
temperature . Their volumes will be related as (b) near the hydrogen chloride bottle.
(a) 2VN = 3VO3 (b) 3VN = 2VO3 (c) near the ammonia bottle.
2 2
(d) throughout the length of the tube.
(c) 3VN = 4VO (d) 4VN = 3VO3
2 3 2 a
7. If 500 ml of gas A at 400 torr and 666.6 ml of B at 600 torr are 18. The ratio (a and b being the van der Waal’s constants of
b
placed in a 3 litre flask, the pressure of the system will be
(a) 200 torr (b) 100 torr (c) 550 torr (d) 366 torr real gases) has the dimensions of
8. The beans are cooked earlier in pressure cooker, because (a) atm mol -1 (b) L mol -1
(a) boiling point increases with increasing pressure
(c) atm L mol -1 (d) atm L mol - 2
(b) boiling point decreases with increasing pressure
19. At 100°C and 1 atm, if the density of liquid water is 1.0 g cm–3
(c) internal energy is not lost while cooking in pressure
and that of water vapour is 0.0006 g cm–3, then the volume
cooker
(d) extra pressure of pressure cooker, softens the beans occupied by water molecules in 1 litre of steam at that
9. When an ideal gas undergoes unrestrained expansion, no temperature is
cooling occurs because the molecules (a) 6 cm3 (b) 60 cm3 (c) 0.6 cm3 (d) 0.06 cm3
(a) are above the inversion temperature 20. It V is the volume of one molecule of gas under given
(b) exert no attractive forces on each other conditions, the van der Waal’s constant b is
(c) do work equal to loss in kinetic energy 4V
(d) collide without loss of energy (a) 4 V (b)
N0
10. The reciprocal of compressibility factor of a real gas in the
critical state is N0
(c) (d) 4VN 0
3 3 8 1 4V
(a) (b) (c) (d) 21. Which of the following volume (V) - temperature (T) plots
8 4 3 3
11. Use of hot air balloons in sports and meteorological represents the behaviour of one mole of an ideal gas at one
observations is an application of atmospheric pressure ?
(a) Boyle’s law (b) Charle’s law
(c) Kelvin’s law (d) Gay-Lussac’s law V(L) (38.8L, 373K)
12. Air at sea level is dense. This is a practical application of
(a) (22.4L,
(a) Boyle’s law (b) Charle’s law 273K)
(c) Kelvin’s law (d) Brown’s law
T(K)
13. Consider a real gas placed in a container. If the intermolecular
attractions are supposed to disappear suddenly which of the
following would happen? V(L)
(28.6L, 373K)
(a) The pressure decreases
(b) The pressure increases (b) (22.4L,
273K)
(c) The pressure remains unchanged
(d) The gas collapses T(K)
EBD_7327
112 CHEMISTRY

(ii) The gas A has minimum departure from the ideal behavior
V(L) (iii) The gas C has largest molecular volume
(30.6L, 373K)
(a) (i) (b) (i) and (ii)
(c) (22.4L, (c) (ii) and (iii) (d) all the three
273K)
28. The pressure of 11gm of a gas which is placed in a 4 litres
T(K) container at 127°C is 2 atm, then the gas would be
(Take : R = 0.08 litre atm K–1 mol–1)
V(L) (a) N2O (b) CO2
(c) NO2 (d) Both (a) and (b) possible
(d) (22.4L, 29. A mixture of two gases A and B in the mole ratio 2 : 3 is kept
273K) in a 2 litre vessel. A second 3L vessel has the same two gases
(14.2L, 373K) in the mole ratio 3 : 5. Both gas mixtures have the same tem-
T(K)
perature and same total pressure. They are allowed to inter-
22. If for two gases of molecular masses MA and MB at mix and the final temperature and the total pressure are the
temperatures TA and TB, TA MB = TB MA, then which of the same as the initial values, the final volume being 5 litres.
Given that the molar masses are MA and MB. What is the
following properties has the same magnitude for both the
mean molar mass of the final mixture?
gases
5M A + 8M B 77 M A + 123M B
(a) Pressure (b) Density (a) (b)
(c) Molar K.E. (d) rms velocity 13 200
23. Positive deviation from ideal behaviour takes place because of 123M A + 77 M B 123M A + 77 M B
(c) (d)
(a) molecular interaction between atoms and PV/nRT > 1 250 150
(b) molecular interaction between atoms and PV/nRT < 1 30. Two flasks A and B of equal volumes maintained at
(c) finite size of atoms and PV/nRT > 1 temperatures 300K and 600K contain equal mass of H2 and
(d) finite size of atoms and PV/nRT < 1
CH4 respectively. The ratio of total translational kinetic
24. At what temperature the average speed of helium molecule
will be the same as that of oxygen molecule at 527°C energy of gas in flask A to that in flask B is
(a) 100 K (b) 200 K (c) 273 K (d) 400 K (a) unity (b) 2 (c) 4 (c) 0.25
25. There are three closed containers in which equal amount of 31. By what factor does the average velocity of a gaseous
the gas are filled. molecule increase when the temperature (in Kelvin) is
doubled ?
(a) 2.0 (b) 2.8 (c) 4.0 (d) 1.4
32. A bubble of air is underwater at temperature 15°C and the
10cm R=10cm pressure 1.5 bar. If the bubble rises to the surface where the
temperature is 25°C and the pressure is 1.0 bar, what will
Sphere happen to the volume of the bubble ?
10cm (hollow) (a) Volume will become greater by a factor of 1.6.
(Cube) 10cm
(I) (II) (III) (b) Volume will become greater by a factor of 1.1.
If all the containers are placed at the same temperatures, then (c ) Volume will become smaller by a factor of 0.70.
find the incorrect options – (d) Volume will become greater by a factor of 2.5.
(a) Pressure of the gas is minimum in (III) container 33. 50 mL of each gas A and of gas B takes 150 and 200 seconds
(b) Pressure of the gas is equal in I and II container respectively for effusing through a pin hole under the similar
(c) Pressure of the gas is maximum in (I) condition. If molecular mass of gas B is 36, the molecular
(d) The ratio of pressure in II and III container is 4 : 3 mass of gas A will be :
(a) 96 (b) 128 (c) 20.25 (d) 64
PV 34. When r, P and M represent rate of diffusion, pressure and
26. The compressibility factor Z = for 1 mol of a real gas is
RT molecular mass, respectively, then the ratio of the rates of
greater than unity at a pressure of 1 atm and 273 .15 K. The
molar volume of the gas at STP will be diffusion (rA / rB ) of two gases A and B, is given as :
(a) less than 22.4 L (b) greater than 22.4 L
(a) ( PA / PB ) ( M B / M A )1/ 2 (b) ( PA / PB )1/ 2 ( M B / M A )
(c) equal to 22.4 L (d) none of these
27. The van der Waals’s constants for gases A, B and C are as follows (c) ( PA / PB ) ( M A / M B )1/ 2 (d) ( PA / PB )1/ 2 ( M A / M B )
Gas a (L2 atm mol–2) b (L mol–1)
35. The compressibility factor for a real gas at high pressure is :
A 0.024 0.027
B 4.17 0.037 RT
(a) 1 + (b) 1
C 3.59 0.043 pb
Based upon the above data, which of the following statements pb pb
is correct? (c) 1 + (d) 1 –
(i) The gas B has the highest critical temperature RT RT
States of Matter 113

36. At constant volume and temperature conditions, the rate of 41. For gaseous state, if most probable speed is denoted by C*,
diffusion DA and DB of gases A and B having densities rA average speed by C and mean square speed by C, then for a
and rB are related by the expression. large number of molecules the ratios of these speeds are :
12 12 (a) C* : C : C = 1.225 : 1.128 : 1
é r ù é r ù (b) C* : C : C = 1.128 : 1.225 : 1
(a) DA = ê DB A ú (b) DA = ê DB B ú
ë rB û ë rA û (c) C* : C : C = 1 : 1.128 : 1.225
12 12
æ rA ö æ rB ö (d) C* : C : C = 1 : 1.225 : 1.128
(c) DA = D B (d) D A = D B ç ÷
çè r ÷ø è rA ø 42. Small droplets of a liquid are usually more spherical in shape
B
37. The compression factor (compressibility factor) for 1 mole of than larger drops of the same liquid because
a van der Waal’s gas at 0°C and 100 atm pressure if found to (a) force of surface tension is equal and opposite to the
be 0.5. Assuming that the volume of gas molecules is force of gravity
negligible, calculate the van der Waal’s constant 'a'. (b) force of surface tension predominates the force of gravity
(a) 0.253 L2 mol–2 atm (b) 0.53 L2 mol–2 atm (c) force of gravity predominates the force of surface tension
2 –2
(c) 1.853 L mol atm (d) 1.253 L2 mol–2 atm (d) force of gravity and force of surface tension act in the
38. The given graph represents the variation of Z
same direction and are equal
PV
(compressibility factor = ) versus P, for three real gases 43. A weather ballon filled with hydrogen at 1 atm and 27°C has
nRT volume equal to 12000 litres. On ascending it reaches a place
A, B and C. Identify the only incorrect statement
C where the temperature is –23°C and pressure is 0.5 atm. The
volume of the balloon is
A
(a) 24000 litres (b) 20000 litres
(c) 10000 litres (d) 12000 litres
1 44. The pressure exerted by 6.0g of methane gas in a 0.03 m3
vessel at 129°C is (Atomic masses : C = 12.01, H = 1.01 and
R = 8.314 Kpa dm3K–1 mol –1)
Z

B
0 (a) 31684 Pa (b) 215216 Pa
P (atm) (c) 13409 Pa (d) 41777 Pa
(a) For the gas A, a = 0 and its dependence on P is linear at 45. Two gases A and B having the same volume diffuse through
all pressure. a porous partition in 20 and 10 seconds respectively. The
(b) For the gas B, b = 0 and its dependence on P is linear at molecular mass of A is 49 u. Molecular mass of B will be :
all pressure (a) 50.00 u (b) 12.25 u (c) 6.50 u (d) 25.00 u
(c) For the gas C, which is typical real gas for which neither 46. A gaseous mixture was prepared by taking equal mole of CO
a nor b = 0. By knowing the minima and the point of and N2. If the total pressure of the mixture was found 1
intersection, with Z = 1, a and b can be calculated atmosphere, the partial pressure of the nitrogen (N2) in the
(d) At high pressure, the slope is positive for all real gases mixture is :
39. The term that corrects for the attractive forces present in a (a) 0.5 atm (b) 0.8 atm (c) 0.9 atm (d) 1 atm
real gas in the van der Waals equation is
47. Which one of the following statements is NOT true about
an2 an2 the effect of an increase in temperature on the distribution
(a) nb (b)
(c) –
(d) – nb of molecular speeds in a gas?
V2 V2
40. For one mole of a van der Waals gas when b = 0 and (a) The area under the distribution curve remains the same
T = 300 K, the PV vs, 1/V plot is shown below. The value of the as under the lower temperature
van der Waals constant a (atm. liter2 mol–2) is : (b) The distribution becomes broader
(c) The fraction of the molecules with the most probable
24.6
speed increases
PV(litre atm mol )
–1

23.1
(d) The most probable speed increases
21.6
DIRECTIONS for Qs. 48 to 50 : These are Assertion-Reason
20.1 type questions. Each of these question contains two statements:
Statement-1 (Assertion) and Statement-2 (Reason). Answer these
questions from the following four options.
(a) Statement-1 is true, Statement-2 is true, Statement-2 is a
correct explanation for Statement -1
(b) Statement-1 is True, Statement-2 is True ; Statement-2 is NOT
0 2.0 3.0 a correct explanation for Statement-1
–1
1/V(mol litre ) (c) Statement-1 is True, Statement-2 is False
(a) 1.0 (b) 4.5 (c) 1.5 (d) 3.0
(d) Statement-1 is False, Statement-2 is True
EBD_7327
114 CHEMISTRY

48. Statement-1 : Jet aeroplane flying at high altitude need Statement-2 : Molar volume for all gases at the same
pressurization of the cabin. temperature and pressure has the same volume.
Statement-2 : Oxygen is not present at higher altitude. 50. Statement-1 : Greater the value of van der Waal’s constant
49. Statement-1 : 1 mol of H2 and O2 each occupy 22.4 L of ‘a’ greater is the liquefaction of gas.
volume at 0°C and 1 bar pressure. Statement-2 : ‘a’ indirectly measures the magnitude of
attractive forces between the molecules.

Exemplar Questions 7. As the temperature increases average kinetic energy of


1. A person living in shimla observed that cooking food without molecules increases. What would be the effect of increase
using pressure cooker takes more time. The reason for this of temperature on pressure provided the volume is constant?
observation is that at high altitude (a) Increases (b) Decreases
(a) pressure increases (b) temperature decreases (c) Remains same (d) Becomes half
(c) pressure decreases (d) temperature increases 8. Gases possess characteristic critical temperature which
2. Which of the following property of water can be used to depends upon the magnitude of intermolecular forces
explain the spherical shape of rain droplets? between the particles. Following are the critical temperatures
(a) Viscosity (b) Surface tension of some gases.
(c) Critical phenomena (d) Pressure
Gases H 2 He O 2 N2
3. A plot of volume (V) versus temperature (T) for a gas at
constant pressure is a straight line passing through the Critical temperature in Kelvin 33.2 5.3 154.3 126
origin. The plots at different values of pressure are shown in From the above data what would be the order of liquefaction
figure. Which of the following order of pressure is correct of these gases? Start writing the order from the gas liquefying
for this gas? first
(a) H2, He, O2, N2 (b) He, O2, H2, N2
P1
P2
Volume (ml) ®

P3 (c) N2, O2, He, H2 (d) O2, N2, H2, He


P4 9. What is SI unit of viscosity coefficient (h)?
(a) Pascal (b) Nsm–2
(c) km–2s (d) Nm–2
10. Atmospheric pressures recorded in different cities are as
follows
Temperature (K) ®
(a) P1 > P2 > P3 > P4 (b) P1 = P2 = P3 = P4 Cities Shimla Bengaluru Delhi Mumbai
(c) P1 < P2 < P3 < P4 (d) P1 < P2 = P3 < P4 2 5 5 5
4. The interaction energy of London force is inversely p in N/m 1.01 ´ 10 1.2 ´ 10 1.02 ´ 10 1.21 ´ 105
proportional to sixth power of the distance between two Consider the above data and mark the place at which liquid
interacting particles but their magnitude depends upon will boil first.
(a) charge of interacting particles (a) Shimla (b) Bengaluru
(b) mass of interacting particles (c) Delhi (d) Mumbai
(c) polarisability of interacting particles 11. Which curve in figure represents the curve of ideal gas?
(d) strength of permanent dipoles in the particles
5. Dipole-dipole forces act between the molecules possessing
F
permanent dipole. Ends of dipoles possess 'partial charges'. E D
The partial charge is
C A
(a) more than unit electronic charge
PV ®

(b) equal to unit electronic charge B


(c) less than unit electronic charge
(d) double the unit electronic charge
6. The pressure of a 1 : 4 mixture of dihydrogen and dioxygen
enclosed in a vessel is one atmosphere. What would be the 0 P®
partial pressure of dioxygen? (a) Only B (b) C and D
(a) 0.8 × 105 atm (b) 0.008 Nm–2 (c) E and F (d) A and B
4
(c) 8 × 10 Nm –2 (d) 0.25 atm
States of Matter 115

12. Increase in kinetic energy can overcome intermolecular forces 16. What is the density of N2 gas at 227°C and 5.00 atm pressure?
of attraction. How will the viscosity of liquid be affected by (R = 0.0821 atm K–1 mol–1) [NEET Kar. 2013]
the increase in temperature? (a) 0.29 g/ml (b) 1.40 g/ml
(a) Increase (c) 2.81 g/ml (d) 3.41 g/ml
(b) No effect 17. Equal masses of H2,O2 and methane have been taken in a
(c) Decrease container of volume V at temperature 27°C in identical
(d) No regular pattern will be followed
conditions. The ratio of the volumes of gases H2 : O2 :
13. How does the surface tension of a liquid vary with increase
methane would be : [2014]
in temperature?
(a) Remains same (a) 8 : 16 : 1 (b) 16 : 8 : 1
(b) Decreases (c) 16 : 1 : 2 (d) 8 : 1 : 2
(c) Increases 18. A gas such as carbon monoxide would be most likely to
(d) No regular pattern is followed obey the ideal gas law at : [2015 RS]
NEET/AIPMT (2013-2017) Questions (a) high temperatures and low pressures.
14. Maximum deviation from ideal gas is expected from : (b) low temperatures and high pressures.
(a) N2(g) (b) CH4(g) [2013] (c) high temperatures and low pressures.
(d) low temperatures and low pressures.
(c) NH3 (g) (d) H2(g)
19. Equal moles of hydrogen and oxygen gases are placed in a
15. Dipole-induced dipole interactions are present in which of
container with a pin-hole through which both can escape.
the following pairs : [2013]
What fraction of the oxygen escapes in the time required for
(a) Cl2 and CCl4 one-half of the hydrogen to escape ? [2016]
(b) HCl and He atoms (a) 1/8 (b) 1/4
(c) SiF4 and He atoms (c) 3/8 (d) 1/2
(d) H2O and alcohol
EBD_7327
116 CHEMISTRY

Hints & Solutions


EXERCISE - 1 Let at T K the given sample will exhibit pressure of 1 atm
and a concentration of 2M.
PV P V
1. (b) PV = constant or 1 1 = 2 2 For an ideal gas, PV = nRT
T T1 T2 Þ (1 atm) (1 L)
PV = (2 mol) (0.0821 mol–1 L atm K–1)T
1 1 = T1
Þ
P2V2 T2 1
Þ T= K = 6.0901 K » 6.1 K
2. (c) Given: P1 = 740 mm of Hg; V1 = 15 mL; T1 = 300 K and P2 2(0.0821)
= 760 mm of Hg; V2 = 10 mL; T2 = ? So, T = 6.1 K is the required condition.
Using gas equation, 10. (c) In the equation PV = nRT, n moles of the gas have
P1V1 P2 V2 volume V.
= 11. (b) Given initial volume (V1) = 500 ml ; Initial temperature
T1 T2
(T1) = 27ºC = 300 K and final temperature (T2) = –5ºC
740 ´ 15 760 ´10 = 268 K.
Þ =
300 T2 V V 500 V2
From Charle’s law : 1 = 2 or =
T1 T2 300 268
760 ´10 ´ 300
Þ T2 = = 205.405 » 205 K Where V2 = New volume of gas
740 ´15
500
3 V2 = ´ 268 = 446.66ml.
3. (c) K. E of n moles of N2 gas = nRT 300
2 12. (a) Given initial volume (V1) = 600 c.c.; Initial pressure
(P1) = 750 mm of Hg and final volume (V2) = 500 c.c.
14 1
Here n = = moles according to Boyle’s law,
28 2 P1V1 = P2V2
R = 8.31 J/mol/K
T = 127°C = 400K or 750 × 600 = P2 × 500
3 1
[ KE= ´ ´ 8.31´ 400J
or P2 = 750 ´ 600 = 900 mm of Hg
2 2
= 2493.0 J = 2.493 kJ » 2.5 kJ 500
Therefore increase of pressure = (900 – 750) = 150 mm of
4. (d) According to Boyle’s law at constant temperature, Hg
1 13. (a) Rate of diffusion depend upon molecular weight
Vµ or PV = constant
P r1 M2
5. (d) According to Graham’s law of diffusion, = Þ r1 = r2 if M1 = M2
r2 M1
1 Hence, compounds are N2O and CO2 as both have same

M molar mass.
where r is rate of diffusion of gas and M is its molcular 14. (b) Use Grahams’ law of diffusion
weight M CH 4
rHe 16
= = =2
r M2 r 81 9 rCH 4 M He 4
So, 1 = Þ 1 = =
r2 M1 r2 100 10 15. (c) PV = nRT is for an ideal gas which follows both isothermal
Þ r1 : r2 = 9 : 10 and adiabatic processes.
6. (c) Molecules in an ideal gas move with different speeds. P1 d1 T1 1 2
16. (a) P µ d and T, = = ´ Þ P1 : P2 = 1:1
7. (b) R = 0.082 litre atm K–1 mole–1 . P2 d2 T2 2 1
8. (b) For an ideal gas, PV = nRT
17. (a) The gas inside the cylinder of 5L capacity has pressure
n mRT dRT æ m ö 76 cm of Hg (NTP)
Þ P = RT = = ç = density(say d) ÷ The new volume for the gas = (30 + 5)L = 35 L.
V MV M èV ø
According to Boyle's law: P1V1 = P2V2
PA dA M B (2d B ) ´ M B 76 × 5 = P2 × 35
So, = = =4
PB dB M A d B (M B / 2) 76 ´ 5
\ P2 = = 10.85 » 10.8cm of Hg
9. (c) Given P = 1atm, n = 2 mole, V = 1L 35
States of Matter 117

18. (a) 0°C is equivalent to 273° K i.e., conditions are same so


r1 M2
volume will be V ml. 32. (b) Under identical conditions, =
19. (b) Vandar Waal’s equation is applicable for real (non-ideal) r2 M1
gases. As rate of diffusion is also inversely proportional to time,
20. (a) 8.31 J.K–1 mol–1 t2 M2
1 cal = 4.2 J. we will have, =
t1 M1
8.31
\ cal.K –1mol –1 = 1.987calK –1mol –1 4
4.2 (a) Thus, For He, t2 = (5 s) = 5 2 s ¹ 10 s ;
2
16 1
21. (d) n of O2 = = 32
32 2 (b) For O2, t 2 = (5 s ) = 20 s
2
3
n of H2 = 28
2 (c) For CO, t 2 = (5s) ¹ 25s ;
2
3 1
Total no. of moles = + =2 44
2 2 (d) For CO2, t2 = (5s ) ¹ 55s
2
nRT 2 ´ .082 ´ 273 33. (b) Gases become cooler during Joule Thomson’s expansion only
V= = = 44.8lit = 44800 ml if they are below a certain temperature known as inversion
P 1
temperature (Ti). The inversion temperature is characteristic of
1 m 1 r each gas and is given by
22. (a) P µ and = r; =
V V V m 2a
Ti = , where R is gas constant
r bR
So, P µ i.e. Pressure µ r Given a = 0.244 atm L2 mol–2
m b = 0.027 L mol–1
PV m R = 0.0821 L atm deg–1 mol–1
23. (c) n = =
RT M 2 ´ 0.244
\ Ti = = 220 K
MPV 34 ´ 2 ´ 100 0.027 ´ 0.0821
m= = = 282.68gm
RT 0.082 ´ 293 2RT
24. (a) 34. (a) Most probable velocity =
M
25. (a) When vapour pressure decreases, boiling point increases
correspondingly. 8RT
Average velocity =
26. (b) Because H2 & Cl2 gases may react with each other to pM
produce HCl gas hence Dalton’s law is not applicable.
Root mean square velocity = 3RT
1 M
27. (a) rg = .rH 2
5 \ Most probable : Average : Root mean square
2
velocity velocity velocity
Mg é rH ù 2
= ê 2 ú = ( 5 ) = 25; M g = 2 ´ 25 = 50 2RT 8RT 3RT 8
M H 2 ëê rg ûú = : : = 2: : 3
M pM M p
28. (b) Repulsive force will decrease the compressibility factor 35. (c) According to kinetic gas equation
i.e. so, value of Z > 1as 1
PV = mNu 2, u = root mean square velocity
PV 3
Z=
RT 1
Due to repulsion value of PV will be greater than RT 23PV 1 -
so Z > 1. Þ u = or u µ i.e u µ m 2
mN m
29. (a) From the graph we can see the correct order of pressures 36. (d) A liquid boils at lower temperature at the top of a
p1 > p3 > p2 mountain.
30. (b) Pressure exerted by hydrogen will be proportional to its 37. (c) Here volume is constant. Again the mass of H2 is fixed so the
mole fraction. number of moles of the gas do not change. As temperature
w increases the pressure also increases. The rate of collision
2 8 among the gas molecules and their energy also increases.
Mole fraction of H2 = =
w w 9
+ 1 2 1 2
16 2 38. (a) PV = mnu = Mu
3 3
rCH 4 Mx Mx 2 1 2 2
31. (a) =2= = , or Mx = 64 = . M u 2 = E or P = E per unit vol.
rx M CH 4 16 3 2 3 3
EBD_7327
118 CHEMISTRY

39. (b) Rate of diffusion By substituting T in eq. (i) we get


1 1 3RPM 3P
µ µ . = =
density molality dRM d
NH3 has the highest rate of diffusion as it is the lightest 1
molecule. 49. (b) Rate of diffusion µ
molecular weight
40. (d) u a T or u1 / u2 = T1 / T2
25 5
27 + 273 300 1 So, ratio of rate of diffusion = =
= = = 4 2
927 + 273 1200 2
50. (c) K.E. µ T [K.E. of one molecule of monoatomic gas =
u2 = 2u1
3
3 RT ]
41. (c) Average molar kinetic energy = kT 2
2
As temperature increases to
As temeprature is same hence average kinetic energy of
CO and N2 is same. 927 + 273 1200
= = 4 times
42. (b) The inversion temp is the temp below which the gas 27 + 273 300
warms up on expansion therefore, Kinetic engergy will be increased to 4 times.
æ 2a ö 51. (c) The expression of root mean square speed is
for, Vander Waal’s gas, Ti = ç ÷.
è Rb ø 3RT
u rms =
M
3RT Hence,
43. (c) R.M.S. velocity =
M 1/ 2
Average velocity u rms (H 2 ) é 3R(50K) /(2g mol -1) ù
=ê ú =1
u rms (O2 ) ëê 3R(800K) /(32g mol -1) ûú
8RT 8 3RT
= = ´
pM 3p M 3RT
52. (d) urms = Using ideal gas equation,
8 M
= ´ R.M.S. velocity
3p w RT PV p
PV = nRT = RTT; = = where d is the
= 0.921 ´ 1000 = 921m / sec M M w d
44. (b) According to kinetic theory the gas molecules are in a density of the gas
state of constant rapid motion in all possible directions 3P 1
colloiding in a random manner with one another and with \ urms = at constant pressure, u rms µ
d d
the walls of the container and between two successive
collisions molecules travel in a straight line path but show 1 2
53. (c) Average KE = E = M u rms
haphazard motion due to collisions. 2
3
K ´ 313 2 2E 2E
K .E of neon at 40 °C 313 \ u rms = or urms =
45. (a) = 2 = M M
K.E of neon at 20°C 3 K ´ 293 293
2 54. (a) By Dalton’s law of partial pressures, the total pressure
of a mixture of two gases is the sum of the partial
R pressures.
46. (b) is also known as Boltz mann’s constant. 55. (d) In the ideal gas, the intermolecular forces of attraction
No
are negligible and hence it cannot be liquefied.
3RT 8RT 8 56. (b) At low temperature and high pressure.
47. (a) urms : uav = : or 3 : = 1.086 : 1
M pM p V V V V
57. (a) r1 = = , r2 = =
3RT t1 18 t 2 45
48. (d) urms = (As PV = RT ) .......... (i)
M V
By putting this value in eq. (i) we get r1 M2 18 x
= Þ =
3PV r2 M1 V 32
= 45
M
PVM PM x 45 452
Now, = RT = PM = dRT = T = Þ = Þ x = 32 ´ 2
M dR 32 18 18
States of Matter 119

58. (b) At low pressure and high temperature real gas nearly 68. (b) Due to intermolecular interactions appreciable at high P
behave like ideal gas. Hence deviation is minimum from and low T, the ideal gas deviates from ideal behaviour.
ideal behaviour. 69. (c) Easily liquefiable gases have greater intermolecular forces
59. (a) In van der waals equation ‘b’ is for volume correction which is represented by high value of 'a'. The greater the
60. (c) By ideal gas equation value of 'a' more will be liquefiability.
P1V1 P V So, the order is Q < S < P < R.
= 2 2 70. (a) When pressure is low ‘b’ can be neglected, thus
T1 T2
Given that, æ a ö
ç P + 2 ÷V = RT
P1 = P2 , V1 = x, V2 = 2x, è V ø
T1 = 273 K , T2 = ? a
PV + = RT
On putting value V
P2 x P 2x a
= 2 PV = RT -
273 T2 V
T2 = 2 × 273 = 546 K or 273ºC PV RT a
= -
Hence, option (d) is correct. RT RT VRT
61. (d) Value of gas constant depends only upon units of
measurement. PV a
Z= = 1-
62. (b) The temperature at which a real gas behaves like an ideal RT VRT
gas is called Boyle’s temperature or Boyle’s point.
x x
æ a ö æ a ö 71. (a) Let the mass be x, then moles of CH4 and H2 are & ;
63. (c) ç P + 16 2
è ÷ (V – b) = RT
T; Here çè P + 2 ÷ø represents the
V2ø V
9x
intermolecular forces. Total moles =
64. (b) The compressibility factor of a gas is defined as 16

pVm x/2 8
Z= pH 2 = mole Fraction ´ P = .P = P
RT 9 x / 16 9
For an ideal gas, pVm = RT. Hence Z = 1 72. (b)
1 73. (d) Soap helps to lower the surface tension of solution, thus
65. (d) P µ (at constant T) soap get stick to the dust particles and grease and these
V
are removed by action of water.
\ PV = constant.
66. (b) U H2 TH 2 28 TH2
(i) At very large molar volume 1
74. (c) = = 7 Þ =
U N2 TN 2 ´ 2 TN 2 2
a
P+ » P and Vm - b = Vm
Vm2 Þ TN 2 = 2TH 2
(iii) According to van der Waals equation 'a' and 'b' are
independent of temperature. \ TN 2 > TH 2
67. (a) r1 = rate of diffusion of NH3 = 2R 75. (b)
r2 = rate of diffusion of X = R
M1 = Molecular mass of NH3 EXERCISE - 2
= 14 + 1 × 3 = 17
M2 = ? 1 1 1 u12 m 2
1. (d) u rms a M : u1a m ,u 2a m \ 2 = m
1 2 u2 1
r1 M2
= 2. (d) One mole of a substance contains the number of
r2 M1 molecules which is independent of pressure.
n
2R M2 3. (c) PV = nRT or P = RT T = CRT.
or = V
R 17 1
Hence 1 = 1 × 0.082 × T is T = = 12K
0.082
M2
or 2 = 28 2
17 4. (a) 28g of CO = = 1 mole; 2g H2 = = 1 mole
28 2
\ M 2 = 17 ´ 4 = 68
EBD_7327
120 CHEMISTRY

Since the temperature and moles are the same, the is observed.
average kinetic energy will also be the same. 1
16. (b) rate of diffusion µ
molecular mass
5. (d) By Ideal gas equation
Let the molecular mass of the gas X = Mx
1 = n1RT
PV
rHe Mx
n1 µ P1 and n2 µ P2 Thus =4=
rx M He
n1 P1 n 170
= Þ 1 = = 0.30 Mx M
n2 P2 n2 570
or 4= or 16 = x
6. (b) At the same conditions of T and P, V a n 4 4
or Mx = 64
14 1 1 36 3 3 The molecular masses of given gases are 44, 64, 46 and
nN 2 = = ; VN 2 a nO 3 = = ; VO 3 a
28 2 2 48 4 4 32 respectively. Thus (b) is correct option.
1
Hence , VN / VO = 2 , 3VN = 2VO 17. (b) Rate of diffusion µ
2 3 3 2 3 Molecular mass
7. (a) Applying Boyle's law P1V1 = P2V2 for both gases Q Molecular mass of HCl > Molecular mass of NH3
500 200 \ HCl diffuses at slower rate and white ammonium
´ 400 = P ´ 3 Þ P = chloride is first formed near HCl bottle.
1000 3
2
666.6 400 PV
600 ´ = P¢ ´ 3 Þ P¢ = 18. (c) a = ( atm L2 mol - 2 ) ; b = L mol -1 ;
1000 3 2
n
200 400 600
Þ PT = P + P¢ = + = = 200 torr a æ atm L2 mol - 2 ö
3 3 3 Hence, =ç ÷ -1
8. (a) The beans are cooked earlier in pressure cooker because b ç L mol -1 ÷ = atm L mol
è ø
boiling point increases with increasing pressure. 19. (c) Mass of 1 L of vapour = volume × density
9. (b) No work is required to tear apart the molecules due to = 1000 × 0.0006 = 0.6 g
the absence of attractive forces in an ideal gas. mass 0.6
PcVc V of liquid water = = = 0.6 cm3
density 1
10. (c) Critical compressibility factor Z =
RTc 20. (d) van der Waals’s constant b = 4 times the actual volume
of 1 mole molecules = 4 VN 0
a 2 8a
Pc = b ; Vc = 3b, Tc = 21. (c) Find the volume by either
27 Rb V = RT/P (PV = RT) or P1V1 = P2V2 and and match it with
3 1 8 the values given in graph to find correct answer.
Hence, Z = or = Volume of 1 mole of an ideal gas at 273 K and 1 atm
8 Z 3
pressure is 22.4 L and that at 373 K and 1 atm pressure is
11. (b) Hot air is lighter due to less density (Charle’s law)
calculated as ;
æ MP ö
çd = ÷
è RT ø RT 0.082 ´ 373
V= = = 30.58L ; 30.6 L
æ MP ö P 1
12. (a) d µ P, Boyle’s law, ç d = ÷ . At sea level pressure is
è RT ø 22. (d)
more, hence density of air is more. 23. (c) For positive deviation: PV = nRT + nPb
13. (b) PV Pb
Þ = 1+
14. (a) Given T = 27°C = 27 + 273 = 300 K nRT RT
V = 10.0 L Thus, the factor nPb is responsible for increasing the
Mass of He = 0.4 g PV value, above ideal value. b is actually the effective
Mass of oxygen = 1.6 g volume of molecules. So, it is the finite size of molecules
Mass of nitrogen = 1.4 g that leads to the origin of b and hence positive deviation
n He = 0.4/4 = 0.1 at high pressure.
n O2 = 1.6/32 = 0.05 24. (a) For equal average speeds of two of gases,
n N2 = 1.4/28 = 0.05
T1 T2
n total = n He + n O2 + n N2 = 0.1 + 0.05 + 0.05 = 0.2 =
M1 M2
n RT 0.2 ´ 0.082 ´ 300
P= = = 0.492 atm
V 10 T1 = ?; M1 = 4; T2 = 273 + 527 = 800K; M 2 = 32
15. (c) Since the inversion temperature of helium is very low, 1
hence during the expansion into vacuum, heating effect 25. (b) n, T same hence P µ ,
V
V1 = 1000 cm3
States of Matter 121

V2 = p (10)2 × 10 = 1000 p cm3


200 36 4 36
4 4 Þ = Þ =
V3 = p (10)3 = p 1000 cm3 150 MA 3 MA
3 3
\ Pressure of the gas is minimum in (III) container, pres- 16 36 81
Þ = Þ MA = = 20.25
sure of the gas is maximum in (I), 9 MA 4
The ratio of pressure in II and III container is 4 : 3 p
34. (a) r µ
26. (b) The gas is less compressible than ideal gas. Hence, m
Vm > 22.4L
27. (d) (i) Greater is the van der Waals’s constant a, higher rA p A MB
=
would be Tc (easier liquefaction of the gas). rB PB MA
(ii) Smaller the constants a and b, lesser departure from
ideal behaviour. æ a ö a
35. (c) ç P + 2 ÷ø
(V - b) = RT at high pressure can be
(iii) Greater the constant b, larger is the molecular è V V2
volume. neglected
28. (d) Calculate molar mass M, PV – Pb = RT and PV = RT + Pb
mRT 11 ´ 0.08 ´ 400 PV Pb
M= = = 44 = 1+
PV 2´ 4 RT RT
Pb
29. (b) 2L 3L Z = 1+ ; Z > 1 at high pressure
RT
12 12
DA rB é r B ù ær ö
36. (d) = =ê ú ; \ DA = DB ç B ÷
DB rA ër A û è rA ø
nA 2 n'A 3 37. (d) We know that
= =
nB 3 n'B 5
PV 100 ´ V
Z= Þ 0.5 = Þ V = 0.112 lit.
(n A + n¢A ) M A + (nB + nB¢ ) M B nRT 1´ 0.0821´ 273
Mean molar mass = According to van der Waals equation
nA + n¢A + nB + nB¢
æ an2 ö
3 3 w ç P + 2 ÷ (V - nb) = nRT
30. (c) K.E. = nRT ; K.E(H 2 ) = ´ ´ R ´ 300; ç V ÷ø
2 2 2 è
3 w æ a ö
K.E.(CH 4 ) = ´ ´ R ´ 600; çç 100 + (0.112 - 0) = 0.0821´ 273
2 16 2 ÷÷
è (0.112) ø
K.E.(H 2 ) a = 1.253 L2 mol–2 atm
Hence =4 38. (b)
K.E.(CH 4 )
39. (b) Correction factor for attractive force for n moles of real
gas is given by the term mentioned in (b).
8RT
31. (d) Average velocity = 40. (c)
pM
24.6
i.e., v µ T
v2 2T
\ = = 1.41 21.6
v1 T PV
32. (a) Given 20.1
P1 = 1.5 bar T1 = 273 + 15 = 288 K V1 = V
P2 = 1.0 bar T1 = 273 + 25 = 298K V2 = ?
PV
1 1 = P2V2 3.0
T1 T2 2.0 1/V
0
1.5 ´V 1´V2
=
288 298 æ a ö
V2 = 1.55 V i.e., volume of bubble will be almost 1.6 time ç P + 2 ÷ (V ) = RT
è V ø
to initial volume of bubble. PV + a/V = RT
VA VB MB a
33. (c) = PV = RT -
tA tB MA V
y = RT – a(x)
EBD_7327
122 CHEMISTRY

So, slope = – a =
21.6 - 20.1
= – a = – 1.5
EXERCISE - 3
2-3 Exemplar Questions
a = 1.5
2RT 1. (c) At high altitude pressure is low hence boiling point is
41. (c) Most probable speed (C*) = low due to which things take more time to cook.
M
However, in a pressure cooker, pressure is increased
8RT and hence, boiling point increases. Thus, in a pressure
Average Speed (C) = cooker food get cook faster i.e. a less period of time.
pM
2. (b) The property of surface tension explain the spherical
3RT shape of rain droplets. Surface tension tries to decrease
Root mean square velocity (C) = the surface area of the liquid to the minimum. The rain
M
droplets are spherical because for a given volume, a
2RT 8RT 3RT sphere has minimum surface area.
C*: C : C = : :
M pM M 3. (c) According to Boyle’s law at a particular temperature,
PV = constant
4 3 Thus, P1V1 = P2V2 = P3V3 = P4V4
= 1: : = 1:1.128 :1.225
p 2 As V1 > V2 > V3 > V4
42. (b) Therefore, P1 < P2 < P3 < P4
P T 1 250 4. (c) London dispersion force is a temporary attractive force
43. (b) V2 = 1 2 .V1 = ´ ´12000 lit. = 20000 lit. that results when the electrons in two adjacent atom
P2 T1 0.5 300
occupy position that makes the atoms form temporary
nRT 6 ´ 8.314´ 402 dipoles. The energy of interaction varies as
44. (d) P = < ; 41777 Pa
V 16 0.03´103 1
.
rA MB ( distance between two interacting particles ) 6
45. (b) =
rB MA Larger or more complex are the molecules, greater is
the magnitude of London forces. This is obviously due
V
MB to the fact that the large electron clouds are easily
20 = M B Þ 1 = distorted or polarised. Hence, greater the polarisability
V 49 2 49
of the interacting particles, greater is the magnitude of
10 the interaction energy.
1 5. (c) Dipole-dipole forces act between the molecules
MB = ´ 49 = 12.25
4 possessing permanent dipole and ends of dipoles
possess 'partial charges'. Partial charges present on
46. (a) Given n CO = n N2
ends of a dipole are always less than the unit electronic
PCO + PN2 = 1 atm charge.
Partial pressure of a gas 6. (c) Let the number of moles of dihydrogen and dioxygen
= mole fraction of gas × total pressure be 1 and 4.

n N2 n N2 4
1 Mole fraction of O2 =
\ PN2 = ´ 1 = 2n ´ 1 = = 0.5 atm. 5
n CO + n N2 N2 2
Partial pressure of O2 = Mole fraction of O2 × total
47. (c) As temperature rises the most probable speed increases pressure of mixture
and the fraction of molecules possessing most probable
speed decreases. 4
= ´ 1 atm
48. (c) The air pressure decreases with increase in altitude. So 5
the partial pressure of oxygen is not sufficient for = 0.8 atm = 0.8 × 105 Nm–2 = 8 × 104 Nm–2
breathing at higher altitude and thus pressurization is 7. (a) From Gay-Lussac's law, at constant volume, as the
needed to increase the concentration of oxygen. temperature is increased, pressure increases.
49. (a) At STP one mole of each gas occupies 22.4L 8. (d) Higher the critical temperature, more easily is the gas
50. (a) Considering the attractive forces between gas molecules get liquified. Hence, order of liquefaction starting with
pressure in ideal gas equation (PV = nRT) is corrected by the gas liquefying first will be : O2, N2, H2, He.
an 2 9. (b) The SI unit of viscosity coefficient (h) in Nm–2 s or
introducing a factor of 2 where ‘a’ is a van der Waal’ss Nsm–2.
V
constant. dv
As we know that, f = hA
dx
States of Matter 123

where, f = force 15. (b) This type of attractive force operates between the polar
h = viscosity coefficient molecules having permanent dipole and the molecules
lacking permanent dipole.
dv HCl is polar (m ¹ 0) and He is non polar (m = 0), thus
= velocity gradient
dx gives dipole-induced dipole interaction.
Substitute SI units of f = N, dx = m, A = m2 and v = ms–1 PM 5 ´ 28
in above equation, we get, 16. (d) Density = = = 3.41g/ml
RT 0.0821´ 500
N´m 17. (c) According to Avogadro’s law "At same temperature
h= = Nm -2s
m ´ ms -1
2
and pressure
Hence, the SI unit of h is Nsm–2 Volume µ no. of moles"
10. (a) Among all the four cities Shimla has the lowest w w w
n H2 = ; n O 2 = ; n CH 4 =
atmospheric pressure. Since lower the atmospheric 2 32 16
pressure, lower is the boiling point. Thus, at Shimla Q VH : VO : VCH = n H 2 : n O 2 : n CH 4
liquid will boil first. 2 2 4

11. (a) For ideal gas PV = constant at all pressures. Therefore, w w w


= : : = 16 : 1 : 2
only B represents ideal gas. 2 32 16
12. (c) Kinetic energy of the molecules of liquid increases with 18. (a) At high temperature and low pressure.
increase of temperature, which can overcome the 19. (a) Given, n H 2 = n O 2 and t H 2 = t O2
intermolecular forces. Hence, the liquid starts flowing. According to Graham's law of diffusion for two different
In other words the viscosity of a liquid decrease with gases.
increase in temperature.
rH 2 v /t MO 2 32
13. (b) The surface tension of liquids generally decreases with = 1 1 Þ =
increase of temperature and becomes zero at the critical rO2 v 2 / t 2 MH 2 2
temperature. This is due to the fact that with increase 1/ 2
of temperature, the kinetic energy of the molecules = 16 = 4
1/ x
increases and therefore, the intermolecular attraction
decreases. x
=4
2
NEET/AIPMT (2013-2017) Questions \x=8
14. (c) Higher the critical temperature more easily will be the \ Fraction of O2 = 1/8
gas liquify. Now since most easily liquifiable gas show
larger deviation, NH3 will show maximum deviation from
ideal behaviour.
EBD_7327
124 CHEMISTRY

6
SOME IMPORTANT THERMODYNAMIC TERMS
Thermodynamics
(iii) Isochoric process occurs when volume of the system is kept
constant.
The System and The Surroundings
(iv) Isobaric process occurs when pressure of the system is kept
System in thermodynamics refers to that part of universe in which constant.
observations are made and remaining universe constitute the (v) Cyclic Process – The process in which a system proceeds
surroundings. via many intermediate steps and returns to the initial state.
Universe = system + surroundings Change in internal energy (dE) = 0, change in enthalpy (dH)=0
Types of System
(i) Open System : In this system there is exchange of energy Thermodynamic Quantities
and matter between system and its surroundings Internal energy
(ii) Closed System : In this system there is no exchange of matter, The energy stored within a substance is called its internal energy.
but exchange of energy is possible between system and its It is represented by ‘U’ or ‘E’. It is the sum of different types of
surroundings energies associated with atoms and molecules.
(iii) Isolated System : In this system there is no exchange of The change in internal energy is given by:
matter and energy between system and surroundings. DU = U2 – U1.
State of the System and State Variables where,
The state of a thermodynamic system is described by its U2 : final state
measurable or macroscopic properties. Variables like pressure (P), U1 : initial state
volume (V) and temperature (T) are called state variables or state The internal energy of a system changes when
functions because their values depend only on state of system (i) heat passes in or out of the system.
and not on how it is reached. (ii) work is done on or by the system.
The state of a system is specified by state functions or state (iii) matter enters or leaves the system.
variables. It is a state function and is an extensive property.
Macroscopic System and Macroscopic Properties DU is negative if energy is evolved andDU is positive if energy is
absorbed.
If a system contains a large no. of chemical species, it is called a
Internal energy of one mole of a monoatomic gas at ‘T’ Kelvin is
macroscopic system. The properties of macroscopic system like
temperature, pressure, volume, density, melting point, boiling point, 3
RT . Internal energy of ideal gas is a function of temperature
etc are called macroscopic properties. 2
They are further divided into two types : only.
(i) Extensive properties In isothermal processes (T is constant), DU = 0.
They depend on quantity of matter contained in system Work
Example : mass, volume, heat capacity etc. There are two types of work in Thermodynamics :
(ii) Intensive properties depend only on nature of substance (i) Electrical work = EMF × quantity of electricity
and are independent of amount of substance present in (ii) Pressure – volume work
system W = – Pext DV
Example : temperature, pressure, density, etc. (DV = V2 – V1 = Vfinal – Vinitial)
Thermodynamic Processes Work done on the system is positive while work done by the
These are said to occur when a system changes from one state to system is negative
another. For free expansion of an ideal gas, Pext = 0, \ W = 0
(i) Isothermal process occurs when temperature remains Heat
constant throughout the process. It is a mode of energy exchanged between the system and the
(ii) Adiabatic process occurs when no heat can flow from system surroundings as a result of difference of temperature between
to its surroundings and vice-versa. them. It is represented by ‘q’, when heat is given by the system, it
Thermodynamics 125

is given negative sign. Enthalpy is a state function and an extensive property.


When heat is absorbed by the system, it is given positive sign. 5
Work and heat are not state functions. Enthalpy of monoatomic gas = RT per mole.
2
FIRST LAW OF THERMODYNAMICS Change in enthalpy of the products and reactants at 298 K and 1
atmospheric pressure is called standard molar enthalpy change.
It is the law of conservation of energy.
In a cyclic process, i.e. when the system returns to the original
According to this law, energy can neither be created nor
state after a number of changes, DU or DH = 0.
destroyed although it may be converted from one form to another
Relationship between heat of reaction at constant pressure and
or
at constant volume
The energy of an isolated system is constant. We know qp = DH and qv = DU,
Mathematically, DU = q + W
where, q = energy absorbed by the system DH = DU + PDV = DU + P ( V2 - V1 ) = DU + ( PV2 - PV1 )
W = workdone on the system. Putting PV = n1RT, PV2 = n2RT,
SOME IMPORTANT RESULTS DH = DU + ( n 2 RT - n1RT )
(i) For isothermal irreversible expansion,
q = –W = Pext (V2 – V1) DH = DU + Dn g RT
(ii) For isothermal reversible expansion or compression from or, qp = qv + Dng RT
volume V1 to V2
HEAT CAPACITY
V
q = - W = nRT ln 2 It is the amount of heat required to raise the temperature of a
V1 system through 1°C. It is given by:
(iii) For isothermal expansion of an ideal gas against vacuum i.e. q q
for free expansion, C= =
T2 - T1 DT
DU = 0, W = 0, q = 0
(iv) Since internal energy of an ideal gas is a function of \ q = C × DT
temperature, for all isothermal processes involving ideal gas, Specific Heat Capacity
DU =0, whether the process is reversible or irreversible. It is the amount of heat required to raise the temperature of one gm
(v) For adiabatic change, q = 0, therefore DU = Wad. of substance through 1°C.
(vi) For isochoric process , W = 0 q = m × c × DT where m : mass of sample, c : specific heat
\ DU = qv + 0 i.e., heat given to system at constant volume Molar Heat Capacity
changes internal energy.
It is amount of heat required to raise the temperature of 1 mole of
ENTHALPY, H a substance through 1°C.
The total heat content of a system at constant pressure is known C
as its enthalpy i.e., Cm =
n
From 1st law of thermodynamics,
Types of heat capacities or molar heat capacities
DU = q + W
(i) Heat capacity at constant volume (CV)
As W = – P D V
At constant volume, qv = Cv DT = DU
\ q = DU + P D V
(ii) Heat capacity at constant pressure (C p)
At constant volume, D V = 0, we have
At constant pressure, qp = CpDT = DH
qV = DU
Relation between Cp and Cv
At constant pressure, we have
qp = DU + PDV As DH = DU + D ( PV ) = DU + D ( RT )
or, qp = (U2 – U1) + P (V2 – V1) DH = DU + RDT
qp = (U2 + PV2) – (U1 + PV1) .... (1) \ CP DT = Cv DT + RDT
The quantity (U + PV) is called heat content or enthalpy of the Þ CP = Cv + R
system.
or, CP - Cv = R
\ H = U + PV
Relation between ratio Cp/Cv and atomicity of a gas
Also, H 2 = U 2 + PV2 and H1 = U1 + PV1 (i) For monoatomic gases Cp/CV = 1.66
Putting these values in eq (1), (ii) For Diatomic gases Cp/Cv = 1.40
q p = H 2 - H1 (iii) For Triatomic gases Cp/CV = 1.33
EXOTHERMIC AND ENDOTHERMIC REACTIONS
or, q p = DH
Exothermic reactions are those which are accompanied by evolution
as q p = DU + PDV of heat. DH is negative for exothermic reactions
Endothermic reactions are those in which heat is absorbed. DH is
\ D H = DU + PDV positive for endothermic reactions.
EBD_7327
126 CHEMISTRY

THERMOCHEMICAL EQUATION (v) Heat of hydration : The amount of heat released on complete
When a balanced chemical equation not only indicates the hydration of one mole of an anhydrous substance is called
quantities of different reactants and products but also indicates heat of hydration.
the amount of heat evolved or absorbed, it is called thermochemical (vi) Lattice enthalpy and its calculation
equation. The lattice enthalpy of an ionic compound is the enthalpy
HEAT OF REACTION OR ENTHALPY OF REACTION change which occurs when one mole of an ionic compound
dissociates into its ions in gaseous state.
OR ENTHALPY CHANGE OF REACTION
Lattice energy is calculated using the Born-Haber Cycle. It is
The amount of heat evolved or absorbed in a chemical reaction explained by taking the examples of NaCl.
when the number of moles of reactants as represented by the
chemical equation have completely reacted, is called heat of (a) Na(s) ¾¾
® Na (g) i.e., sublimation
reaction or enthalpy of reaction or enthalpy change of reaction
(DrH). Dsub H° = 108.4kJ mol-1
Change in total heat of reaction at 25°C and 1 atm pressure is + –
(b) Na(g) ¾¾
® Na (g) + e i.e., ionization of
called standard heat of reaction
Different types of heats/ enthalpies of reaction Di H° = 496kJ mol-1
(i) Enthalpy of combustion :
It is the heat evolved when 1 mole of substance is completely 1
(c) Cl2 ( g ) ¾¾
® Cl ( g ) i.e., dissociation of Cl
burnt or oxidised in oxygen. It is represented as DcH. 2
ex: CH 4 ( g ) + 2O2 ( g ) ¾¾
® CO2 ( g ) + 2H 2O ( g ) 1
D bond H° = 121 kJ mol-1
DcH° is standard enthalpy of combustion i.e. combustion 2
taking place under standard conditions, i.e., 298 K and 1 bar
pressure. (d) Cl ( g ) + e - ¾¾
® Cl - ( g ) i.e. gain of e– by Cl.
(ii) Enthalpy of formation :
It is the heat change i.e. heat evolved or absorbed when 1 D EG H° = -348.6 k J mol-1
mole of the substance is formed from its elements under given
conditions of T and P. It is represented by Df H.
(e) Na + ( g ) + Cl- ( g ) ¾¾
® Na + Cl- ( s )
Standard enthalpy of formation arises when the substance is and
formed in the standard state from its elements, which is also
1
taken in the standard state (i.e. 298 K and 1 bar pressure). It is Na(s) + Cl2 ( g ) ® NaCl (s) Δf H =–411.2 kJ mol -1
represented by Df H° 2
The standard enthalpy change of the reaction is: This is the Born – Haber Cycle .
Applying Hess’s law, we get
Dr H° = å Df H° ( products ) - å D f H° ( reactants )
1
For elementary substances in standard state, D f Ho is taken DfH° = DsubH° + D bond H° + I.E. + E. A. + Dlattice H°
2
as zero.
(iii) Enthalpy of Neutralization D lattice H° = 411.2 +108.4+121+496 – 348.6
The enthalpy of neutralization of an acid by a base is defined = + 788 k J
as the heat change when one gram equivalent of the acid is
Now, Dsol H° = Dlattice H° + Dhyd H°
neutralized by a base, the reaction being carried out in dilute
aqueous solution. = + 788 kJ mol–1 + (– 784 kJ mol–1)
The enthalpy of neutralization of any strong acid with a strong = + 4 kJ mol–1.
base or vice-versa, is always the same, i.e. 57.1kJ. (vii) Enthalpy of Atomization
(iv) Enthalpy of solution When one mole of a given substance dissociates into gaseous
It is defined as the enthalpy change when 1 mole of the atoms, the enthalpy change accompanying the process is
substance is dissolved in a specified amount of the solvent. called enthalpy of atomization. It is represented by DaH°.
For ionic compounds, enthalpy of solution depends upon
e.g., H 2 ( g ) ® 2H(g), D a H o = 435.0 kJ mol -1
lattice enthalpy and hydration enthalpy, i.e.,
(viii) Bond Enthalpy or Bond energy
Dsol H° = Dlattice H° + D hyd H°
It is the amount of energy released when one mole of bonds
For most ionic compounds, Dsol H° is positive and are formed from the isolated atom in gaseous state or the
dissociation process is endothermic. Thus, solubility of most amount of energy required to dissociate one mole of bonds
salts in water increases with increase in T. If lattice enthalpy present between the atoms in gaseous molecules. It is
of a salt is very high, the dissolution of the compound may represented by DbH or DbondH. It is expressed in kcal/mole
not take place at all. For this reason fluorides are less soluble DrH = å B.E. (Reactants) – å B.E. (Products)
than chlorides.
Thermodynamics 127

Enthalpy Changes During Phase Transitions SECOND LAW OF THERMODYNAMICS


(i) Heat of sublimation : The amount of heat required to change It states that the energy of universe is constant whereas the
one mole of a solid substance into vapour state is called heat entropy of universe is continuously increasing and tends to a
of sublimation. maximum.
I2(s) ® I2(g) DH = +62.07 kJ/mole
GIBBS FREE ENERGY AND SPONTANEITY
(ii) Heat of fusion : The amount of heat required to completely
change one mole of a solid substance into liquid at its melting The available amount of energy to the system during the process
point is called heat of fusion. which can be changed into useful work, is called free energy of
H2O(s) ––––® H2O(l) DH = 6.0 kJ the system.
(iii) Heat of vapourisation : The amount of heat required to Gibbs energy helps in predicting the spontaneity of a process.
completely change one mole of a liquid into vapours at its It is denoted by ‘G’ and is given by
boiling point is called heat of vapourisation. G = H – TS
H2O(l) ––––® H2O(g) DH = +44 k cal The change in Gibb’s energy is given by:
DG = DH – TDS
HESS’S LAW OF CONSTANT HEAT SUMMATION The decrease in the value of Gibbs free energy during a process is
It states that the total amount of heat evolved or absorbed in a equal to the maximum possible useful work that can be obtained
reaction is the same whether the reaction takes place in one step from the system.
or in a number of steps. In other words, the total amount of heat According to Gibbs energy equation :
change in a reaction depends only upon the nature of initial DG = DH – TDS
reactants and nature of final products and is independent of the It combines both the factors of spontaneity, namely, energy factor
path or manner by which this change is brought about. DH and entropy factor TDS
A q2 B Spontaneity in Terms of Free Energy Change
q1 q3 (i) If DG is negative, the process is spontaneous
Reactant R Product, P (ii) If DG = 0, the process does not occur or system is in
equilibrium.
Q
(iii) If DG = positive, process does not occur in forward direction.
Q = q1 + q 2 + q3 It may occur in backward direction.
SPONTANEITY At equilibrium, DrG° = DrH° – T DrS°
= – RT ln K = – 2.303 RT logK
A spontaneous process is an irreversible process and may only
(K ® equilibrium constant)
be reversed by some external agency.
The tendency for a process to occur depends upon two factors: Relation Between DG and emf of Cell
(i) tendency for minimum energy DG = – nFEcell
(ii) tendency for maximum randomness where, Ecell = emf of the cell
The resultant of the above two tendencies which gives the overall n = number of moles of electrons involved
tendency for a process to occur is called driving force of the F = Faraday’s constant i.e., 96500 coulomb
process. If reactants and products are in their standard state,
DG° = – nFE°
ENTROPY
where E° = standard cell potential
It is a measure of randomness or disorder of the system.
Unit of entropy is JK–1mol–1 Effect of Temperature on Spontaneity of Reactions
Greater the randomness, higher is the entropy.
Entropy change during a process is given by: D r H° D r S° Dr G° Description of reaction
DS = S2 – S1 = å Sproduct – å Sreactant - + - Spontaneous at all T
DS is related to q and T for a reversible process as follows:
- - - ( lowT ) spontaneous at low T
q rev
DS = - - + ( high T ) non-spontaneous at high T
T
+ + + ( low T ) spontaneous at low T
In any process :
+ + - ( high T ) spontaneous at high T
D S Total = D Ssystem + D Ssurrounding
+ - + ( at all T ) non-spontaneous at all T
At equilibrium, entropy of the system is maximum and DS = 0
For irreversible process,
THIRD LAW OF THERMODYNAMICS
DSTotal = DSsystem + DSsurrounding > O
At absolute zero temperature, the entropy of a perfectly crystalline
substance is taken as zero. This law was formulated by Nernst in
1906.
CONCEPT MAP
128
CHEMISTRY

EBD_7327
Thermodynamics 129

1. For precipitation reaction of Ag+ ions with NaCl, which of 11. One mole of an ideal gas at 300 K is expanded isothermally
the following statements is correct ? from an initial volume of 1 litre to 10 litres. The DE for this
(a) D H for the reaction is zero process is (R =2 cal. mol–1 K–1)
(a) 163.7 cal (b) zero
(b) D G for the reaction is zero
(c) 1381.1 cal (d) 9 lit. atm
(c) D G for the reaction is negative 12. In a closed insulated container, a liquid is stirred with a
(d) | D G | = | D H | paddle to increase the temperature, which of the following
2. Calorie is equivalent to : is true?
(a) 0.4184 Joule (b) 4.184 Joule (a) DE = W ¹ 0, q = 0 (b) DE = W = q ¹ 0
(c) 41.84 Joule (d) 418.4 Joule
3. In the isochoric process, DH for a system is equal to (c) DE = 0, W = q ¹ 0 (d) W = 0, DE = q ¹ 0
(a) P. DV (b) PV 13. The heat of combustion of methane at 298° K is expressed
(c) E + P. DV (d) DE by
4. If DH is the change in enthalpy and DE is change in internal CH 4(g ) + 2O 2(g) ¾
¾® CO 2(g) + 2H 2 O and DH = 890.2 kJ.
energy accompanying a gaseous reaction, then
Magnitude of DE of reaction at this temperature is
(a) DH is always greater than DE
(a) infinity (b) equal to DH
(b) DH < D E only if the number of moles of the products
is greater than the number of moles of the reactants (c) less than DH (d) greater than DH
14. For the reaction
(c) D H is always less than D E
(d) D H < D E only if the number of moles of products is C3H8 ( g ) + 5O2 ( g ) ® 3CO2 ( g ) + 4H2O(l )
less than the number of moles of the reactants. at constant temperature, DH – DE is
5. Which of the following is correct order ? (a) – RT (b) + RT
(a) 1 erg > 1 Joule > 1 calorie (c) – 3 RT (d) + 3 RT
(b) 1 erg > 1 calorie > 1 joule 15. The work done during the expansion of a gas from a volume
(c) 1 calorie > 1 Joule > 1 erg of 4 dm3 to 6 dm3 against a constant external pressure of
(d) 1 Joule > 1 calorie > 1 erg 3 atm is (1 L atm = 101.32 J)
6. For the reaction (a) – 6 J (b) – 608 J
(c) + 304 J (d) – 304 J
N 2 ( g ) + 3H 2 ( g ) 2NH3(g), DH = ? 16. The molar heat capacity of water at constant pressure is 75
(a) DE + 2RT (b) DE –2RT JK–1 mol–1. When 1kJ of heat is supplied to 100 g of water,
(c) DE – RT (d) None of these which is free to expand, the increase in temperature of water
7. The following is an endothermic reaction : is
(a) decomposition of water (a) 6.6 K (b) 1.2 K
(b) conversion of graphite to diamond (c) 2.4 K (d) 4.8 K
(c) dehydrogenation of ethane to ethylene 17. Assume each reaction is carried out in an open container.
(d) All of these For which reaction will DH = DE ?
8. Which process takes place on chemical bond formation : (a) C(s) + 2H2O (g) ® 2H2 (g) + CO2 (g)
(a) Decrease is energy (b) PCl5 (g) ® PCl3 (g) + Cl2 (g)
(b) Increase in energy (c) 2CO (g) + O2 (g) ® 2CO2 (g)
(c) No change in energy (d) H2 (g) + Br 2 (g) ® 2 HBr (g)
(d) Attractive and repulsive forces are equal 18. Which of the following is combustion reaction?
9. Gibbs free energy, for the reaction at equilibrium, is (a) C + O 2 ® CO2
(a) zero (b) – ve (b) CH 4 + O 2 ® CO 2 + H 2O
(c) + ve (d) either (a) or (c)
10. For which of the following changes, DH¹ DU (c) Mg + O2 ® MgO
(a) N 2 ( g ) + O 2 ( g ) ® 2NO( g ) (d) all of these.
19. An ideal gas expands in volume from 1×10–3 to 1 × 10–2 m3
(b) H 2 ( g ) + Cl2 ( g ) ¾¾
® 2HCl( g ) at 300 K against a constant pressure of 1×105 Nm–2. The
(c) C( s) + O2 ( g ) ¾¾® CO2 ( g ) work done is
(d) (a) 270 kJ (b) – 900 kJ
CaCO3 ( s ) ¾¾ ® CaO( s ) + CO 2 ( g )
(c) – 900 J (d) 900 kJ
EBD_7327
130 CHEMISTRY

20. Consider the reaction : N 2 + 3H 2 ® 2 NH 3 carried out at 30. What is the enthalpy change for,
2H 2O2 (l ) ® 2 H 2O(l ) + O 2 ( g ) if heat of formation of
constant temperature and pressure. If DH and DU are the
H2O2 (l) and H2O (l) are –188 and –286 kJ/mol respectively?
enthalpy and internal energy changes for the reaction, which
(a) –196 kJ/mol (b) + 948 kJ/mol
of the following expressions is true ?
(c) + 196 kJ/mol (d) –948 kJ/mol
(a) DH > DU (b) DH < DU
31. C (dia ) + O 2 ¾
¾® CO 2 ; DH = –395.4 kJ/mole
(c) DH = DU (d) DH = 0 ¾® CO 2 ; DH = –393.5 kJ/mole
C (gr ) + O 2 ¾
21. (DH – DU) for the formation of carbon monoxide (CO) from
its elements at 298 K is (R = 8.314 J K–1 mol–1) ¾® C ( dia ) ; DH = ?
C (gr ) ¾
(a) –2477.57 J mol –1 (b) 2477.57 J mol–1 (a) –3.8 (b) –1.9
(c) –1238.78 J mol –1 (d) 1238.78 J mol–1 (c) +3.8 (d) +1.9
22. Two moles of an ideal gas is expanded isothermally and 32. The D f H ° of O3, CO2, NH3 and HI are 142.2, –393.3, –46.2
reversibly from 1 litre to 10 litre at 300 K. The enthalpy change
and + 25.9 kJ per mol respectively. The order of their
(in kJ) for the process is
increasing stabilities will be
(a) 11.4 kJ (b) – 11.4 kJ
(a) O3, CO2, NH3, HI (b) CO2, NH3, HI, O3
(c) 0 kJ (d) 4.8 kJ
(c) O3, HI, NH3, CO2 (d) NH3, HI, CO2, O3
23. For a reaction Ag 2 O (s ) ¾¾® 2Ag + O , the value of DH =
33. If the bond energies of H - H, Br - Br, and H-Br are 433,
132.6 kJ, DS = 66 JK–1 mol–1
The free energy change for the reaction will be zero at which 192 and 364 kJ mol–1 respectively, the DH° for the reaction
of the temperature : H 2 ( g ) + Br2 ( g ) ® 2HBr( g ) is
(a) 2000 K (b) 2009 K (a) – 261 kJ (b) + 103 kJ
(c) 2023 K (d) 2029 K (c) + 261kJ (d) – 103 kJ
24. For the reaction 34. The absolute enthalpy of neutralisation of the reaction:
MgO (s) + 2HCl (aq) ¾® MgCl2 (aq) + H2O (l) will be:
A( g ) + 2B( g ) ¾¾
® 2C( g ) + 3D( g )
(a) -57.33 kJ mol-1
the change of enthalpy at 27°C is 19 kcal. The value of DE is (b) greater than -57.33 kJ mol- 1
(a) 21.2 kcal (b) 17.8 kcal (c) less than -57.33 kJ mol-1
(c) 18.4 kcal (d) 20.6 kcal (d) 57.33 kJ mol-1
25. For a chemical reaction the enthalpy and entropy change
35. A2(g) + B2(g) 2AB(g); DH = +ve, It :
are –2.5 × 103 cals and 7.4 cals deg–1 respectively. At 25ºC
(a) increases by pressure
the reaction is :
(b) it occurs at 1000 atm pressure
(a) spontaneous (b) non-spontaneous
(c) it occurs at high temperature
(c) reversible (d) irreversible
(d) it occurs at high pressure and temperature
26. A well stoppered thermos flask contains some ice cubes.
36. The enthalpy of hydrogenation of cyclohexene is – 119.5
This is an example of a
kJ mol–1. If resonance energy of benzene is –150.4 kJ mol–1,
(a) closed system
its enthalpy of hydrogenation would be
(b) open system
(a) – 208.1 kg mol–1 (b) – 269.9 kg mol–1
(c) isolated system –1
(c) – 358.5 kg mol (d) – 508.9 kg mol–1
(d) non-thermodynamic system
37. Which of the following equations correctly represents the
27. Given that C + O 2 ® CO 2 : DH º = - x kJ
standard heat of formation (D f H° ) of methane ?
2 CO + O 2 ® 2CO 2 : DH º = - y kJ (a) C (diamond) + 4H( g ) ® CH 4 ( g )
the enthalpy of formation of carbon monoxide will be
(b) C (diamond) + 2H 2 ( g ) ® CH 4 ( g )
2x - y y - 2x
(a) (b) (c) C (graphite) + 2H 2 ( g ) ® CH 4 ( g )
2 2
(c) 2x – y (d) y – 2x (d) C (graphite) + 4H( g ) ® CH 4 ( g )
28. The values of DH for the combustion of ethane and acetylene 38. Enthalpy of solution of NaOH (solid) in water is – 41.6
are –341.1 and –310.0 kcal, respectively. The better fuel is : kJ mol–1. When NaOH is dissolved in water, the temperature
(a) ethane (b) acetylene of water :
(c) both (d) none of these (a) increases (b) decreases
29. The values of heat of formation of SO2 and SO3 are –298.2 (c) does not change (d) fluctuates indefinitely
kJ and –98.2 kJ. The heat of formation of the reaction 39. For a Carnot engine, the source is at 500 K and the sink at
SO 2 + (1 / 2)O 2 ® SO 3 will be 300 K. What is efficiency of this engine :
(a) –200 kJ (b) –356.2 kJ (a) 0.2 (b) 0.4
(c) + 200 kJ (d) – 396.2 kJ (c) 0.6 (d) 0.3
Thermodynamics 131

40. The D f H° for CO2(g) CO(g) and H2O(g) are –393.5, –110.5 51. The latent heat of vaporisation of water at 100° C is 540 cal
g–1. Calculate the entropy increase when one mole of water
and –241.8 kJ/mol respectively, the standard enthalpy change at 100°C is evaporated?
(in kJ) for the reaction CO2(g) + H2(g) ® CO(g) + H2O(g) is : (a) 1.82 cal K–1 mol–1 (b) 540 × 18 cal K–1 mol–1
(a) 524.1 (b) 41.2 –1
(c) 26 cal K mol –1 (d) 367 cal K–1 mol–1
(c) – 262.5 (d) – 41.2 52. The entropy change in the fusion of one mole of a solid
41. If enthalpies of formation of C 2 H 4 ( g ) , CO 2 (g) and melting at 27ºC (Latent heat of fusion, 2930 J mol–1) is :
H 2O(l ) at 25°C and 1atm pressure are 52, – 394 and – 286 (a) 9.77 J K–1 mol–1 (b) 10.73 J K–1 mol–1
kJ/mol respectively, the change in enthalpy is equal to (c) 2930 J K–1 mol–1 (d) 108.5 J K–1 mol–1
(a) – 141.2 kJ/mol (b) – 1412 kJ/mol 53. 2 mole of an ideal gas at 27ºC temperature is expanded
(c) + 14.2 kJ/mol (d) + 1412 kJ/mol reversibly from 2 lit to 20 lit. Find the entropy change
42. How many joules of heat are absorbed when 70.0 grams of (R = 2 cal/mol K)
water is completely vaporised at its boiling point? (a) 92.1 (b) 0
(a) 23,352 (b) 7,000 (c) 4 (d) 9.2
(c) 15,813 (d) 158, 200 54. Considering entropy (S) as a thermodynamic parameter, the
43. Heat of neutralization of strong acid against strong base is criterion for the spontaneity of any process is
constant and is equal to (a) DSsystem + DS surroundin gs > 0
(a) 13.7 kcal (b) 57.3 kJ (b) DSsystem - DSsurroundings > 0
(c) 5.73 × 104 J (d) All of the above (c) DSsystem > 0 only
44. Work done during isothermal expansion of one mole of an
ideal gas from 10 atm. to 1 atm. at 300 K is : (d) DSsurroundin gs > 0 only
(a) 4938.8 joule (b) 4138.8 joule 55. Entropy of vaporisation of water at 100°C, if molar heat of
(c) 5744.1 joule (d) 6257.2 joule vaporisation is 9710 cals mol–1, will be
45. Given the bond energies N º N, H – H and N – H bonds are (a) 28 cal mol–1 K–1 (b) 26 cal mol–1 K–1
(c) 24 cal mol–1 K–1 (d) 20 cal mol–1 K–1
945,436 and 391 kJ mole–1 respectively, the enthalpy of the
56. For conversion C (graphite) ® C (diamond) the DS is
following reaction N 2 ( g ) + 3H 2 ( g ) ® 2NH 3 ( g ) is (a) zero (b) positive
(a) –93 kJ (b) 102 kJ (c) negative (d) cannot be predicted
(c) 90 kJ (d) 105 kJ 57. The following thermochemical reactions are given :
46. The H2O(g) molecule dissociates as
1
(i) H 2O ( g ) ® H ( g ) + OH ( g ) ; DH = 490 kJ M + O 2 ® MO + 351.4kJ
2
(ii) OH ( g ) ® H ( g ) + O ( g ) ; DH = 424 kJ 1
X + O 2 ® XO + 90.8 kJ
The average bond energy (in kJ) for water is 2
(a) 490 (b) 424 It follows that the heat of reaction for the following process
(c) 914 (d) 914/2 M + XO ® MO + X
47. DS° will be highest for the reaction is given by :
1 (a) 442.2kJ (b) 260.7 kJ
(a) Ca( s) + O 2 ( g ) ¾¾ ® CaO( s )
(c) – 442.2 kJ (d) 260.6 kJ
2
(b) CaCO3 ( s ) ¾¾ ® CaO( s ) + CO 2 ( g ) 58. According to second law of thermodynamics heat is partly
(c) C( s) + O 2 ( s) ¾¾ ® CO 2 ( g ) converted into useful work and part of it :
(a) becomes electrical energy
(d) N 2 ( g ) + O 2 ( g ) ¾¾
® 2NO( g ) (b) always wasted
48. The maximum work which a system can perform at constant (c) increases the weight of the body
temperature and pressure equals (d) becomes K.E.
(a) DH – DE (b) (– DG)
(c) q – DE (d) P.DV – DG 59. If C(s) + O2(g) ¾¾
® CO2(g); DH = R and
49. For which of the following process, DS is negative? 1
(a) H2 (g) ¾¾ ® 2 H(g) CO(g) + O2 (g) ¾¾ ® CO 2 (g); DH = S,
(b) 2SO3 (g) ¾¾ 2
® 2SO2(g) + O2 (g)
compressed
then heat of formation of CO is:
(c) N 2 (4l) ¾¾¾¾¾ ® N 2 (2l) (a) R + S (b) R – S
(d) C(diamond) ¾¾ ® C (graphite) (c) R × S (d) S – R
50. Which one of the following is correct ? 60. Which of the following pairs of processes is certain to occur
in a spontaneous chemical reaction?
1
(a) DH = DG – TDS (b) DS = [DG – DH] (a) Exothermic and increasing disorder
T (b) Exothermic and decreasing disorder
1 (c) Endothermic and increasing disorder
(c) DS = [DH – DG] (d) – DG = DH – T D S (d) Endothermic and decreasing disorder
T
EBD_7327
132 CHEMISTRY

61. The H–H bond energy is 430 kJ mol–1 and Cl–Cl bond (c) Both statements (A) and (B) are true
energy is 240 kJ mol–1. DH for HCl is – 90 kJ/mol. The H–Cl (d) Both statements (A) and (B) are false
bond energy is about : 71. DE of combustion of isobutylene is – X kJ mol–1. The value of
(a) 180 kJ mol–1 (b) 360 kJ mol–1 DH is :
(c) 213 kJ mol–1 (d) 425 kJ mol–1 (a) = DE (b) > DE
62. A chemical reaction will be spontaneous if it is accompanied (c) = 0 (d) < DE
by a decrease of
(a) entropy of the system 1
72. For the reaction CO( g ) + O2 ( g ) ¾¾ ® CO2 ( g ), DH, and
(b) enthalpy of the system 2
(c) internal energy of the system DS are –283 kJ and –87 JK–1, respectively. It was intended to
(d) free energy of the system carry out this reaction at 1000, 1500, 3000 and 3500 K. At
63. For a reaction in which all reactants and products are liquids, which of these temperatures would this reaction be
which one of the following equations is most applicable ? thermodynamically spontaneous?
(a) DH < DE (b) DH = DS (a) 1500 and 3500 K (b) 3000 and 3500 K
(c) DH = DE (d) DH = DG (c) 1500, 1500 and 3000 K (d) 1500, 3000 and 3500 K
64. Highest entropy is in :
(d) Energy due to gravitational pull
(a) hydrogen (b) water
(c) graphite (d) mercury 73. The enthalpy of combustion of C 6 H 6 is – 3250 kJ, when
65. If the value of Cp for nitrogen gas is 7 JK–1mol–1, then the 0.39 gm of C 6 H 6 is burnt in excess of oxygen in an open
value of DH on heating 28 g of nitrogen gas from 0°C to 100°C
vessel, the amount of heat evolved is
at constant pressure will be :
(a) 1200 J (b) 1300 J (a) 8.32 kJ (b) 12.36 kJ
(c) 1400 J (d) 1500 J (c) 16.25 kJ (d) 20.74 kJ
66. The enthalpy and entropy change for the reaction 74. The free energy change for the following reactions are given
Br2(l) + Cl2 (g) ® 2 BrCl (g) below,
are 30kJ mol –1 and 105 JK–1 mol –1 respectively. The 5
temperature at which the reaction will be in equilibrium is C2 H2 (g) + O2 (g) ® 2CO 2 (g) + H 2O(l); DG° = -1234kJ
2
(a) 273 K (b) 450 K
(c) 300 K (d) 285.7 K C(s) + O 2 (g) ® CO 2 (g); DG° = -394kJ
67. Given the value of DH= 177.9 kJ and
1
-1 -1
H 2 (g) + O 2 (g) ® H 2 O(l); DG ° = -237kJ
DS= 160.4 JK mol . For the reaction 2
What is the standard free energy change for the reaction
CaCO 3 ( s ) ¾¾
® CaO( s) + CO 2 ( g ) H2(g) + 2C (s) ¾¾ ® C2H2(g)
at 298 K, the free energy change per mole would be : (a) – 209 kJ (b) –2259 kJ
(a) + 130.1 kJ (b) – 130.1 kJ (c) + 2259 kJ (d) 209 kJ
(c) – 4.90 kJ (d) + 4.90 kJ 75. One mole of an ideal gas is allowed to expand reversibly and
68. For spontaneous cooling of a hot metal rod (system) in air : adiabatically from a temperature of 27°C. If the work done
(a) DS system > 0, DS universe < 0 during the process is 3kJ, then final temperature of the gas is
(b) DS system > 0, DS universe > 0 (Cv = 20 J/K):
(c) DS system < 0, DS universe > 0 (a) 100 K (b) 150 K
(d) DS system < 0, DS universe < 0 (c) 195 K (d) 255 K
69. Following reaction occurrs in an automobile
76. The heat of atomization of PH3(g) is 228 kcal mol–1 and that
2C8 H18 ( g ) + 25O 2 ( g ) ® 16CO 2 ( g ) + 18H 2 O ( g ) . The of P2H4(g) is 335 kcal mol–1. The energy of the P–P bond is
sign of DH, DS and DG would be (a) 102 kcal mol–1 (b) 51 kcal mol–1
(c) 26 kcal mol –1 (d) 204 kcal mol–1
(a) +, –, + (b) –, +, –
(c) –, +, + (d) +, +, – 77. The value DH – DE for the following reaction at 27°C will be
70. Consider the following reactions at 1000°C 2NH3(g) ¾¾ ® N2 + 2H2(g)
1 (a) 8.314 × 273 × (–2) (b) 8.314 × 300 × (–2)
-1
(A) Zn( s) + O2 ( g ) ® ZnO( s); DG° = -360kJmol . (c) 8.314 × 27 × (–2) (d) 8.314 × 300 × (2)
2
78. Given that bond energies of H – H and Cl – Cl are
1 -1 430 kJ mol– 1 and 240 kJ mol–1 respectively and DfH for HCl is
(B) C ( gr ) + O2 ( g ) ® CO( g ); DG° = -460kJmol .
2 – 90 kJ mol– 1, bond enthalpy of HCl is
Choose the correct statement at 1000°C (a) 380 kJ mol–1 (b) 425 kJ mol–1
(a) Zinc can be oxidized by carbon monoxide (c) 245 kJ mol –1 (d) 290 kJ mol–1
(b) Zinc oxide can be reduced by graphite
Thermodynamics 133

79. Which of the following are not state functions ? 88. The standard enthalpy of formation of NH3 is – 46.0 kJ mol–1.
(I) q + w (II) q If the enthalpy of formation of H2 from its atoms is
(III) w (IV) H - TS – 436 kJ mol–1 and that of N2 is – 712 kJ mol–1, the average
(a) (I) and (IV) (b) (II), (III) and (IV) bond enthalpy of N – H bond in NH3 is
(c) (I), (II) and (III) (d) (II) and (III) (a) – 964 kJ mol–1 (b) + 352 kJ mol–1
(c) + 1056 kJ mol –1 (d) – 1102 kJ mol–1
80. For vaporization of water at 1 atmospheric pressure, the values
of DH and DS are 40.63 kJmol–1 and 108.8 JK–1 mol–1, 89. The value of enthalpy change (DH) for the reaction
respectively. The temperature when Gibbs energy change C2H5 OH(l) + 3O2 ( g ) ® 2CO2 ( g ) + 3H 2O(l)
( DG ) for this transformation will be zero, is: at 27° C is – 1366.5 kJ mol–1. The value of internal energy
(a) 293.4 K (b) 273.4 K change for the above reaction at this temperature will be :
(c) 393.4 K (d) 373.4 K. (a) – 1369.0 kJ (b) – 1364.0 kJ
81. The following two reactions are known : (c) – 1361.5 kJ (d) – 1371.5 kJ
90. The heats of atomization of PH3(g) and P2H4(g) are 954 kJ
® 2Fe(s) + 3CO2(g); DH = –26.8 kJ
Fe2O3(s) + 3CO (g) ¾¾
mol–1 and 1485 kJ mol–1 respectively. The P–P bond energy
FeO(s) + CO(g) ¾¾
® Fe(s) + CO2(g); DH = –16.5 kJ in kJ mol–1 is
The value of D H for the following reaction (a) 213 (b) 426
Fe2O3(s) + CO(g) ¾¾ ® 2FeO(s) + CO2(g) is; (c) 318 (d) 1272
(a) + 6.2 kJ (b) + 10.3 kJ 91. Pick out the wrong statement
(c) – 43.3 kJ (d) – 10.3 kJ (a) The standard free energy of formation of all elements is
82. Three moles of an ideal gas expanded spontaneously into zero
vacuum. The work done will be : (b) A process accompanied by decrease in entropy is
(a) Zero (b) Infinite spontaneous under certain conditions
(c) 3 Joules (d) 9 Joules (c) The entropy of a perfectly crystalline substance at
absolute zero is zero
83. If the enthalpy change for the transition of liquid water to
(d) A process that leads to increase in free energy will be
steam is 30 kJ mol–1 at 27ºC, the entropy change for the
spontaneous
process would be :
92. Which one of the following demonstrates a decrease in
(a) 10 J mol –1 K–1 (b) 1.0 J mol–1 K–1
–1 –1
entropy?
(c) 0.1 J mol K (d) 100 J mol–1 K–1
(a) Dissolving a solid into solution
84. Enthalpy change for the reaction, (b) An expanding universe
4H( g ) ¾¾
® 2H 2 ( g ) is – 869.6 kJ. (c) Burning a log in a fireplace
The dissociation energy of H–H bond is : (d) Raking up leaves into a trash bag
(a) – 434.8 kJ (b) – 869.6 kJ 93. A certain reaction is at equilibrium at 82° C and the
(c) + 434.8 kJ (d) + 217.4 kJ enthalpy change for this reaction is 21.3 kJ. The value of
85. The enthalpy of fusion of water is 1.435 kcal/mol. DS (in JK mol–1) for the reaction is
The molar entropy change for the melting of ice at 0°C is : (a) 55.0 (b) 60.0
(a) 10.52 cal / (mol K) (b) 21.04 cal / (mol K) (c) 68.5 (d) 120.0
(c) 5.260 cal / (mol K) (d) 0.526 cal / (mol K) 94. The bond enthalpy is the highest for
86. Equal volumes of two monoatomic gases, A and B, at same (a) F2 (b) Cl2
temperature and pressure are mixed. The ratio of specific (c) Br2 (d) H2
heats (Cp/Cv) of the mixture will be : 95. At the sublimation temperature, for the process
(a) 0.83 (b) 1.50 ˆˆ† CO2(g)
CO2(s) ‡ˆˆ
(c) 3.3 (d) 1.67 (a) DH, DS and DG are all positive
87. Identify the correct statement regarding a spontaneous (b) DH > 0, DS > 0 and DG < 0
process (c) DH < 0, DS > 0 and DG < 0
(a) Lowering of energy in the process is the only criterion (d) DH > 0, DS > 0 and DG = 0
for spontaneity.
96. The enthalpy of solution of sodium chloride is
(b) For a spontaneous process in an isolated system, the 4 kJ mol–1 and its enthalpy of hydration of ions is –784 kJ
change in entropy is positive. mol–1. Then the lattice enthalpy of NaCl (in kJ mol –1) is
(c) Endothermic processes are never spontaneous. (a) + 788 (b) + 4
(d) Exothermic processes are always spontaneous. (c) + 398 (d) + 780
EBD_7327
134 CHEMISTRY

97. Calculate the work done (in Joules) when 0.2 mole of an ideal 99. A chemical reaction is spontaneous at 298 K but non-
gas at 300 K expands isothermally and reversibly from an spontaneous at 350 K. Which one of the following is true for
initial volume of 2.5 litres to the final volume of the reaction?
25 litres. DG DH DS
(a) – – +
(a) 996 (b) 1148 (b) + + +
(c) 11.48 (d) 897 (c) – + –
98. Choose the reaction with negative DS value. (d) – – –
(a) 2NaHCO3(s) ® Na2CO3(s) + CO2(g) + H2O(g) 100. A reaction cannot take place spontaneously at any
temperature when
(b) Cl2(g) ® 2Cl(g) (a) both DH and DS are positive
(c) 2SO2(g) + O2(g) ® 2SO3(g) (b) both DH and DS are negative
(d) 2KClO3(s) ® 2KCl(s) + 3O2(g) (c) DH is negative and DS is positive
(d) DH is positive and DS is negative

1. In which of the following equations, the enthalpy change 6. For the reactions,
will represent DH ºf of N2H4(l) with sign reversed? C + O2 ® CO2 ; DH = –393 J
2Zn + O2 ® 2ZnO ; DH = –412 J
(a) N 2 ( g ) + 2 H 2( g ) ® N 2 H 4 ( g ) (a) carbon can oxidise Zn
(b) 2N (g ) + 4H (g ) ® N 2 H 4(l ) (b) oxidation of carbon is not feasible
(c) oxidation of Zn is not feasible
(c) N 2 H 4(l ) ® 2N (g ) + 4H (g ) (d) Zn can reduce carbon dioxide.
7. The internal energy change when a system goes from state A
(d) N 2 H 4( l) ® N 2(g ) + 2H 2(g)
to B is 40 kJ/mole. If the system goes from A to B by a reversible
2. Equal volumes of molar hydrochloric acid and sulphuric acid path and returns to state A by an irreversible path what would
are neutralized by dil. NaOH solution and x kcal and y kcal of be the net change in internal energy ?
heat are liberated respectively. Which of the following is true? (a) > 40 kJ (b) < 40 kJ (c) Zero (d) 40 kJ
1 8. In an irreversible process taking place at constant T and P and in
(a) x = y (b) x = y which only pressure-volume work is being done, the change in
2
(c) x = 2y (d) None of these Gibbs free energy (dG) and change in entropy (dS), satisfy the
3. Hydrogen has an ionisation energy of 1311 kJ mol–1 and for criteria
chlorine it is 1256 kJ mol–1. Hydrogen forms H+ (aq) ions but (a) (dS)V, E > 0, (dG)T, P < 0 (b) (dS)V, E = 0, (dG)T, P = 0
chlorine does not form Cl+ (aq) ions because (c) (dS)V, E = 0, (dG)T, P > 0 (d) (dS)V, E < 0, (dG)T, P < 0
(a) H+ has lower hydration enthalpy
9. If the bond dissociation energies of XY, X 2 and Y2 (all
(b) Cl+ has lower hydration enthalpy
(c) Cl has high electron affinity diatomic molecules) are in the ratio of 1 : 1 : 0.5 and D f H for
(d) Cl has high electronegativity the formation of XY is – 200 kJ mole–1 . The bond dissociation
4. The factor of DG values is important in metallurgy. The DG energy of X 2 will be
values for the following reactions at 800ºC are given as :
-1
S2 ( s ) + 2O 2 ( g ) ¾¾
® 2SO 2 ( g ) ; DG = – 544 kJ (a) 400 kJ mol (b) 300 kJ mol -1
-1 -1
® 2ZnS( s ) ; DG = – 293 kJ
2Zn( s ) + S2 ( s ) ¾¾ (c) 200 kJ mol (d) 800 kJ mol
® 2ZnO( s ) ; DG = – 480 kJ
2Zn( s ) + O 2 ( g ) ¾¾ 10. The standard enthalpy of formation (DfHº) at 298 K for
Then DG for the reaction : methane, CH4 (g) is –74.8 kJ mol–1. The additional information
required to determin e th e average energy for
2ZnS( s ) + 3O 2 ( g ) ¾ ¾
® 2ZnO( s ) + 2SO 2 ( g ) C – H bond formation would be
will be : (a) the first four ionization energies of carbon and electron
(a) –357 kJ (b) –731 kJ (c) –773 kJ (d) –229 kJ gain enthalpy of hydrogen
5. For the reaction A ® B; DH = + 24 kJ/mol and B ® C;
(b) the dissociation energy of hydrogen molecule, H2
DH = – 18 kJ/mol, the decreasing order of enthalpy of A, B
and C follows the order (c) the dissociation energy of H2 and enthalpy of sublimation
(a) A, B, C (b) B, C, A of carbon
(c) C, B, A (d) C, A, B (d) latent heat of vapourization of methane
Thermodynamics 135

11. The enthalpy changes for the following processes are listed 19. If DH and DE are the change in enthalpy and change in
below : internal energy respectively for a gaseous reaction, then
Cl2(g) ® 2Cl(g), 242.3 kJ mol–1 (a) DH > DE, always
I2(g) ® 2I(g), 151.0 kJ mol–1 (b) DH < DE, onlyif nproduct > nreactants
ICl(g) ® I(g) + Cl(g), 211.3 kJ mol–1
(c) DH < DE ,always
I2(s) ® I2(g), 62.76 kJ mol–1
Given that the standard states for iodine and chlorine are I2(s) (d) DH < DE, only if nproduct < nreactants
and Cl2(g), the standard enthalpy of formation for ICl(g) is : 20. Consider the following reactions:
(a) +16.8 kJ mol–1 (b) +244.8 kJ mol–1 (i) H +(aq) + OH–(aq) ¾¾ ® H2O(l),
(c) –14.6 kJ mol –1 (d) –16.8 kJ mol–1 DH = – X1 kJ mol–1
12. The heat of neutralisation of strong base and strong acid is 1
57.0 kJ. The heat released when 0.5 mole of HNO3 is added to (ii) H2(g) + O (g) ¾¾ ® H2O(l),
2 2
0.20 mole of NaOH solution is : DH = – X2 kJ mol–1
(a) 34.9 kJ (b) 57.0 kJ (c) 11.4 kJ (d) 28.5 kJ (iii) CO2(g) + H2(g) ¾¾ ® CO(g) + H2O,
13. Energy required to dissociate 4g of gaseous hydrogen into DH = – X3 kJ mol–1
free gaseous atoms is 208 kcal at 25°C. The bond energy of
5
H–H will be : (iv) C 2 H 2 ( g ) + O2 ( g ) ¾¾
® 2CO2(g) + H2O(l),
(a) 104 kcal (b) 52 kcal (c) 10.4 kcal (d) 1040 kcal 2
14. Given that heat of neutralisation of strong acid and strong DH = + 4X4 kJ mol –1

base is – 57.1 kJ. The heat produced when 0.25 mole of HCl is Enthalpy of formation of H2O (l) is
neutralised with 0.25 mole of NaOH in aqueous solution is : (a) + X3 kJ mol– 1 (b) – X4 kJ mol– 1
(a) 14.275 kJ (b) 57.1 kJ (c) 22.5 kJ (d) 28.6 kJ (c) + X1 kJ mol – 1 (d) – X2 kJ mol– 1
15. The heat of neutralisation of HCl by NaOH is 21. For the gas phase reaction,
–55.9 kJ/mole.The energy of dissociation of HCN is : ˆˆ† PCl3(g) + Cl2(g)
PCl5(g) ‡ˆˆ
(HCN + OH - ® CN - + H 2 O; DH = -12.1 kJ)
(a) 43.8 kJ (b) – 43.8 kJ (c) – 68 kJ (d) 68 kJ which of the following conditions are correct ?
16. Calculate enthalpy change for the change 8S( g ) ¾¾ (a) DH = 0 and DS < 0 (b) DH > 0 and DS > 0
® S8 ( g ),
(c) DH < 0 and DS < 0 (d) DH > 0 and DS < 0
given that
22. Bond dissociation enthalpy of H2, Cl2 and HCl are 434 , 242 and
® 2H( g ) + 2S( g ), DH = 239.0 k cal mol-1 ;
H 2S2 ( g ) ¾¾ 431 kJ mol–1 respectively. Enthalpy of formation of HCl is :
H 2S( g ) ¾¾® 2H( g ) + S( g ), DH = 175.0 k cal mol-1 (a) 93 kJ mol–1 (b) – 245 kJmol–1
(a) + 512.0 k cal (b) – 512.0 k cal (c) – 93 kJmol–1 (d) 245 kJmol–1
(c) 508.0 k cal (d) – 508.0 k cal 23. The values of DH and DS for the reaction,
17. If x1, x2 and x3 are enthalpies of H–H, O=O and O–H bonds C(graphite) + CO2 (g) ® 2CO(g) are 170 kJ and 170 JK–1,
repectively, and x4 is the enthalpy of vaporisation of water, respectively. This reaction will be spontaneous at
estimate the standard enthalpy of combustion of hydrogen: (a) 910 K (b) 1110 K (c) 510 K (d) 710 K
24. From the following bond energies:
x x H – H bond energy: 431.37 kJ mol–1
(a) x1 + 2 - 2 x3 + x4 (b) x1 + 2 - 2 x3 - x 4
2 2 C = C bond energy: 606.10 kJ mol–1
x2 x C – C bond energy: 336.49 kJ mol–1
(c) x1 + - x3 + x 4 (d) 2 x3 - x1 - 2 - x4 C – H bond energy: 410.50 kJ mol–1
2 2 Enthalpy for the reaction,
18. If for a given substance melting point is TB and freezing
point is TA, then correct variation shown by, graph between H H H H
entropy change and temperature is | | | |
C = C + H - H ¾¾ ® H - C- C - H
TB | | | |
DS TB DS
H H H H
TA will be:
(a) TA (b) (a) – 243.6 kJ mol–1 (b) –120.0 kJ mol–1
T T (c) 553.0 kJ mol–1 (d) 1523.6 kJ mol–1
25. Standard entropies of X2 , Y2 and XY3 are 60, 40 and
TB
TB TA 50 JK–1mol–1 respectively. For the reaction
DS TA DS 1 3
X 2 + Y2 ƒ XY3 , DH = – 30 kJ
(c) (d) 2 2
to be at equilibrium, the temperature should be:
T T (a) 750 K (b) 1000 K (c) 1250 K (d) 500 K
EBD_7327
136 CHEMISTRY

26. Match List -I (Equations) with List-II (Type of processes) 32. Oxidising power of chlorine in aqueous solution can be
and select the correct option. determined by the parameters indicated below:
List I List II 1 Q
Equations Type of processes 1 D H
2 diss D eg HQ
Cl2 ( g ) ¾¾¾¾¾ ® Cl( g ) ¾¾¾¾ ® Cl– ( g )
(1) Kp > Q (i) Non spontaneous 2
(2) DG ° < RT lnQ (ii) Equilibrium D HQ
(3) Kp = Q (iii) Spontaneous and
Hyd
¾¾¾¾ ® Cl – (aq)
–1
endothermic (using the data, D diss H Cl = 240 kJ mol ,
2
DH
(4) T > (iv) Spontaneous –1
D eg H Cl = –349 kJ mol , D hyd H Cl – = –381 kJ mol ),
–1
DS
Options: will be
(1) (2) (3) (4) (a) + 152 kJ mol–1 (b) – 610 kJ mol–1
(a) (ii) (i) (iv) (iii) (c) – 850 kJ mol –1 (d) + 120 kJ mol–1
(b) (i) (ii) (iii) (iv) 33. On the basis of the following thermochemical data :
(c) (iii) (iv) (ii) (i) +
[Δ f H°(H (aq) = 0)]
(d) (iv) (i) (ii) (iii)
27. Consider the following processes : H 2O(l ) ® H + ( aq ) + OH – ( aq ); DH = 57.32kJ
DH (kJ/mol)
1
1/2 A® B +150 H2 (g )+ O2 (g ) ¾¾ ® H2O(l); ΔH= –286.20kJ
3B ® 2C + D –125 2
The value of enthalpy of formation of OH– ion at 25° C is:
E + A ® 2D +350
(a) –228.88 kJ (b) +228.88 kJ
For B + D ® E + 2C, DH will be :
(a) 525 kJ/mol (b) – 175 kJ/mol (c) –343.52 kJ (d) –22.88 kJ
(c) – 325 kJ/mol (d) 325 kJ/mol 34. For a particular reversible reaction at temperature T, DH and
28. In which of the following reactions, standard entropy change DS were found to be both +ve. If Te is the temperature at
(DS°) is positive and standard Gibb’s energy change (DG°) equilibrium, the reaction would be spontaneous when
decreases sharply with increase in temperature ?
(a) Te > T (b) T > Te
1
(a) C (graphite) + O2(g) ® CO(g) (c) Te is 5 times T (d) T = Te
2
1 35. Consider the reaction
(b) CO(g) + O2(g) ® CO2(g)
2 4NO2 ( g ) + O 2 ( g ) ® 2N 2O5 ( g ),
1 DrH = – 111 kJ.
(c) Mg(s) + O2(g) ® MgO(s)
2
If N2O5(s) is formed instead of N2O5(g) in the above reaction,
1 1 1 the DrH value will be :
(d) C (graphite) + O2(g) ® CO2(g)
2 2 2 (given, DH of sublimation for N2O5 is –54 kJ mol–1)
29. Standard enthalpy of vapourisation Dvap H° for water at (a) + 54 kJ (b) + 219 kJ
100°C is 40.66 kJ mol–1. The internal energy of vaporisation
of water at 100°C (in kJ mol–1) is : (c) – 219 J (d) – 165 kJ
(a) + 37.56 (b) – 43.76 ˆˆ† B is
36. The value of log10 K for a reaction A ‡ˆˆ
(c) + 43.76 (d) + 40.66
(Assume water vapour to behave like an ideal gas). (Given : D r H °298K = -54.07 kJ mol- 1 ,
30. In conversion of lime-stone to lime,
CaCO3 ( s ) ® CaO( s ) + CO 2 ( g ) the values of DH° and DS° D r S°298K = 10 JK–1 mol–1 and R = 8.314 JK–1 mol–1;
are + 179.1 kJ mol-1 and 160.2 J/K respectively at 298 K and 2.303 × 8.314 × 298 = 5705)
1 bar. Assuming that DH° and DS° do not change with (a) 5 (b) 10
temperature, temperature above which conversion of (c) 95 (d) 100
limestone to lime will be spontaneous is
(a) 1118 K (b) 1008 K 37. For the process H2O(l) (1 bar, 373 K) ® H2O(g) (1 bar, 373
(c) 1200 K (d) 845 K. K), the correct set of thermodynamic parameters is
31. Assuming that water vapour is an ideal gas, the internal (a) DG = 0, DS = +ve (b) DG = 0, DS = –ve
energy change (DU) when 1 mol of water is vapourised at (c) DG = +ve, DS = 0 (d) DG = –ve, DS = +ve
1 bar pressure and 100°C, (given : molar enthalpy of 38. The species which by definition has ZERO standard molar
vapourisation of water at 1 bar and 373 K = 41 kJ mol –1 and enthalpy of formation at 298 K is
R = 8.3 J mol–1 K–1) will be
(a) 41.00 kJ mol–1 (b) 4.100 kJ mol–1 (a) Br2 (g) (b) Cl2 (g)
(c) 3.7904 kJ mol –1
(d) 37.904 kJ mol–1 (c) H2O (g) (d) CH4 (g)
Thermodynamics 137

39. Using the data provided, calculate the multiple bond energy 45. The enthalpies of the following reactions are shown below.
(kJ mol–1) of a C º C bond in C2H2. That energy is (take the [AFMC 2011]
bond energy of a C – H bond as 350 kJ mol–1) 1 1
H (g) + O2(g) ® OH(g) ; DH = 42.09 kJ mol–1
2C(s) + H2(g) ¾¾ ® HC º CH(g); DH = 225 kJ mol–1 2 2 2
H2(g) ® 2H(g) ; DH = 435.89 kJ mol–1
2C(s) ¾¾ ® 2C(g) ; DH = 1410 kJ mol–1
O2(g) ® 2O(g) ; DH = 495.05 kJ mol–1
H2(g) ¾¾ ® 2H(g) ; DH = 330 kJ mol–1
Calculate the O –– H bond energy for the hydroxyl radical.
(a) 1165 (b) 837 (a) 223.18 kJ mol–1 (b) 423.38 kJ mol–1
(c) 865 (d) 815 (c) 513.28 kJ mol –1 (d) 113.38 kJ mol–1
40. A piston filled with 0.04 mol of an ideal gas expands DIRECTIONS for Qs. 46 to 50 : These are Assertion-Reason
reversibly from 50.0 mL to 375 mL at a constant temperature type questions. Each of these question contains two statements:
of 37.0ºC. As it does so, it absorbs 208 J of heat. The values Statement-1 (Assertion) and Statement-2 (Reason). Answer these
of q and w for the process will be: questions from the following four options.
(R = 8.314 J/mol K) (ln 7.5 = 2.01) (a) Statement-1 is True, Statement-2 is True, Statement-2 is a
(a) q = + 208 J, w = – 208 J (b) q = – 208 J, w = – 208 J correct explanation for Statement-1
(c) q = – 208 J, w = + 208 J (d) q = + 208 J, w = + 208 J (b) Statement-1 is True, Statement-2 is True Statement-2 is
41. Four grams of graphite is burnt in a bomb calorimeter of heat NOT a correct explanation for Statement-1
capacity 30 kJ K–1 in excess of oxygen at 1 atmospheric
(c) Statement - 1 is True, Statement-2 is False
pressure.The temperature rises from 300 to 304 K. What is
the enthalpy of combustion of graphite (in kJ mol–1)? (d) Statement -1 is False, Statement-2 is True
(a) 360 (b) 1440 [Kerala PMT 2011] 46. Statement 1 : Many endothermic reactions that are not
(c) –360 (d) –1440 spontaneous at room temperature become spontaneous at
42. The enthalpy change for reaction high temperature.
Statement 2 : Entropy of the system increases with increase
1 3
X 2 ( g ) + Y2 ( g ) ® XY3 ( g ) is – 50 kJ mol–1. If the in temperature. [AIIMS 2013]
2 2 47. Statement 1 : For an isothermal reversible process Q = –W
bond enthalpies of X–X and X–Y are respectively 380 and i.e. work done by the system equals the heat absorbed by
150 kJ mol–1 the bond enthalpy of Y–Y in kJ mol–1 is the system.
(a) 35 (b) 210 [Kerala PMT 2013] Statement 2 : Enthalpy change (DH) is zero for isothermal
(c) 280 (d) 140 process. [AIIMS 2013]
43. Consider the following spontaneous reaction 48. Statement 1 : The mass and volume of a substance are the
2C8 H18 ( g ) + 25 O 2 ( g ) ® 16CO 2 ( g ) + 18H 2O ( g ) extensive properties and are proportional to each other.
The sign of DH, DS, DG would be respectively Statement 2 : The ratio of mass of a sample to its volume is
(a) +, –, + (b) –, + , – [Kerala PMT 2013] an intensive property.
(c) +, +, – (d) –, –, – 49. Statement 1 : Internal energy is an extensive property.
44. One mole of solid iron was vaporized in an oven at its boiling Statement 2 : Internal energy depends upon the amount of
point of 3433 K and enthalpy of vaporization of iron the system.
is 344.3 kJ mol–1 . The value of entropy vaporization 50. Statement 1 : Absolute value of internal energy of a
(in J mol–1) of iron is [Kerala PMT 2013] substance cannot be determined.
(a) 100 (b) 10
Statement 2 : It is impossible to determine exact values of
(c) – 100 (d) 110
constitutent energies of the substances.

Exemplar Questions (b) There is an exchange of energy as well as matter


between the system and the surroundings in a closed
1. Thermodynamics is not concerned about
system.
(a) energy changes involved in a chemical reaction
(b) the extent to which a chemical reaction proceeds (c) The presence of reactants in a closed vessel made up
(c) the rate at which a reaction proceeds of copper is an example of a closed system.
(d) the feasibility of a chemical reaction
2. Which of the following statement is correct? (d) The presence of reactants in a thermos flask or any
(a) The presence of reacting species in a covered beaker other closed insulated vessel is an example of a closed
is an example of open system. system.
EBD_7327
138 CHEMISTRY

3. The state of a gas can be described by quoting the (a) DS (system) decreases but DS (surroundings) remains
relationship between the same
(a) pressure, volume, temperature (b) DS (system) increases but DS (surroundings) decreases
(b) temperature, amount, pressure (c) DS (system) decreases but DS (surroundings) increases
(c) amount, volume, temperature (d) DS (system) decreases but DS (surroundings) also
(d) pressure, volume, temperature, amount decreases
4. The volume of gas is reduced to half from its original volume. 10. On the basis of theromochemical equations (1), (2) and (3),
The specific heat will be find out which of the algebraic relationships given in options
(a) reduce to half (b) be doubled (a) to (d) is correct
(c) remain constant (d) increase four times 1. C (graphite) + O2 (g) ® CO2 (g); DrH = x kJ mol–1
5. During complete combustion of one mole of butane,
1
2658 kJ of heat is released. The thermochemical reaction for 2. C (graphite) + O (g) ® CO (g); DrH = y kJ mol–1
above change is 2 2

(a) 2C4 H10 ( g ) + 13O 2 ( g ) ® 8CO 2 ( g ) + 10H 2O ( l ) ; 3. CO (g) +


1
O (g) ® CO2 (g); DrH = z kJ mol–1
2 2
D c H = -2658.0 kJ mol -1 (a) z = x + y (b) x = y – z
(c) x = y + z (d) y = 2z – x
13
(b) C4 H10 ( g ) + O 2 ( g ) ® 4CO2 ( g ) + 5H 2 O ( l ) ; 11. Consider the reactions given below. On the basis of these
2 reactions find out which of the algebraic relationship given
D c H = -1329.0 kJ mol -1 in options (a) to (d) is correct?
1. C (g) + 4 H (g) ® CH4 (g); DrH = x kJ mol–1
13 2. C (graphite) + 2H2 (g) ® CH4 (g); DrH = y kJ mol–1
(c) C4 H10 ( g ) + O 2 ( g ) ® 4CO2 ( g ) + 5H 2 O ( l ) ;
2 (a) x = y (b) x = 2y
(c) x > y (d) x < y
D c H = -2658.0 kJ mol -1 12. The enthalpies of elements in their standard states are taken
as zero. The enthalpy of formation of a compound
13
(d) C4 H10 ( g ) + O 2 ( g ) ® 4CO2 ( g ) + 5H 2 O ( l ) ; (a) is always negative
2 (b) is always positive
D c H = +2658.0 kJ mol -1 (c) may be positive or negative
(d) is never negative
6. DfU° of formation of CH4 (g) at certain temperature is
13. Enthalpy of sublimation of a substance is equal to
–393 kJ mol–1. The value of DfH° is
(a) enthalpy of fusion + enthalpy of vaporisation
(a) zero (b) < DfU°
(b) enthalpy of fusion
(c) > DfU° (d) equal to DfU°
(c) enthalpy of vaporisation
7. In an adiabatic process, no transfer of heat takes place
(d) twice the enthalpy of vaporisation
between system and surroundings. Choose the correct
14. Which of the following is not correct?
option for free expansion of an ideal gas under adiabatic
(a) DG is zero for a reversible reaction.
condition from the following.
(b) DG is positive for a spontaneous reaction.
(a) q = 0, DT ¹ 0, W = 0 (b) q ¹ 0, DT = 0, W = 0
(c) DG is negative for a spontaneous reaction.
(c) q = 0, DT = 0, W = 0 (d) q = 0, DT < 0, W ¹ 0
(d) DG is positive for a non-spontaneous reaction.
8. The pressure-volume work for an ideal gas can be calculated
Vf
NEET/AIPMT (2013-2017) Questions
by using the expression W = - ò pex dV. The work can also 15. When 5 litres of a gas mixture of methane and propane is
Vi
perfectly combusted at 0°C and 1 atmosphere, 16 litre
of oxygen at the same temperature and pressure is
be calculated from the pV-plot by using the area under the consumed. The amount of heat released from
curve within the specified limits. When an ideal gas is this combustion in kJ (DHcomb (CH4) = 890 kJ mol –1,
compressed (a) reversibly or (b) irreversibly from volume Vi DHcomb (C3H8) = 2220 kJ mol–1) is
to Vf . Choose the correct option. [NEET Kar. 2013]
(a) W (reversible) = W (irreversible) (a) 32 (b) 38
(b) W (reversible) < W (irreversible) (c) 317 (d) 477
(c) W (reversible) > W (irreversible) 16. Three thermochemical equations are given below:
(d) W (reversible) = W (irreversible) + pex.DV (i) C(graphite) + O2(g) ® CO2(g); DrH° = x kJ mol–1
9. The entropy change can be calculated by using the 1
(ii) C(graphite) + O (g) ® CO(g); DrH° = y kJ mol–1
q 2 2
expression DS = rev . When water freezes in a glass beaker,,
T 1
choose the correct statement amongst the following. (iii) CO(g) + O (g) ® CO2(g); DrH° = z kJ mol–1
2 2
Thermodynamics 139

Based on the above equations, find out which of the 19. The correct thermodynamic conditions for the spontaneous
relationship given below is correct? [NEET Kar. 2013] reaction at all temperatures is [2016]
(a) x=y–z (b) z = x + y (a) DH < 0 and DS = 0 (b) DH > 0 and DS < 0
(c) x=y+z (d) y = 2z – x (c) DH < 0 and DS > 0 (d) DH < 0 and DS < 0
17. For the reaction : [2014] 20. For a given reaction, DH = 35.5 kJ mol-1 and DS = 83.6 JK-1
X2O4(l) ® 2XO2(g) mol-1. The reaction is spontaneous at : (Assume that DH and
DS do not vary with tempearature) [2017]
DU = 2.1 k cal, DS = 20 cal K–1 at 300 K
(a) T > 425 K (b) All temperatures
Hence DG is:-
(c) T > 298 K (d) T < 425 K
(a) 2.7 k cal (b) – 2.7 k cal
21. A gas is allowed to expand in a well insulated container
(c) 9.3 k cal (d) – 9.3 k cal
against a constant external pressure of 2.5atm from an initial
18. The heat of combustion of car bon to CO 2 is volume of 2.50 L to a final volume of 4.50L. The change in
–393.5 kJ/mol. The heat released upon formation of 35.2 g of internal energy DU of the gas in joules will be:- [2017]
CO2 from carbon and oxygen gas is [2015 RS] (a) – 500J (b) – 505J
(a) –315 kJ (b) +315kJ (c) + 505J (d) 1136.25J
(c) –630 kJ (d) –3.15 kJ
EBD_7327
140 CHEMISTRY

Hints & Solutions


EXERCISE - 1 = D H – (1 – 3)RTT
= D H + 2RT = 890 + 2RT T
1. (c) Ag + (aq) + NaCl(aq) ¾¾
® AgCl(s) + Na + (aq)
D
Hence E > H D
The precipitation reaction is spontaneous and hence Option (d) is correct.
DG P,T < 0 . 14. (c) DH = DE + nRT
2. (b) 1 calorie = 4.184 joule Dn = 3 – (1 + 5) = 3 – 6 = –3
3. (d) DH = DE + P DV DH - DE = (-3RT )
For isochoric process, DV = 0 15. (b) W = – pDV = -3(6 - 4) = -6 litre atmosphere
\ DH = DE = -6 ´101.32 = -608 J
4. (d) As DH = DE + DngRT
16. (c) Given Cp = 75 JK–1 mol–1.
if np < nr; D ng = np – n r = – ve
100
Hence D H < D E n= mole , Q = 1000 J DT = ?
18
5. (c) 107 erg = 1 Joule, 1 cal = 4.183 J
Hence calorie is the largest amount of energy.
Q = nCpDT Þ DT = 1000 ´ 18 = 2.4 K
6. (b) D ng = 2 – 4 = – 2, D H = D E – 2RT.T. 100 ´ 75
7. (d) Heat energy is absorbed during endothermic reaction. In 17. (d) We know that DH = DE + DngRT
all the given reactions, heat is absorbed during reaction. In the reaction, H2(g) + Br2(g) ® 2HBr(g)
\ All of them are endothermic reactions. Dn = np – nr = 2 – 2 = 0
8. (b) When a chemical bond is formed between the two free So, DH = DE for this reaction
atoms in a gaseous state to form a molecular product in a 18. (d) Combustion is heating of substance in excess of oxygen
gaseous state, some heat is always evolved which is so as to form carbon bioxide and water. Sometimes water
known as bond energy. Hence, there is increase in energy. is not formed during combustion.
9. (a) The Gibb’s free energy is zero at equilibrium because the
rate of backward reaction is equal to the rate of forward 19. (c) W = -PDV = -105 (1 ´ 10 -2 - 1 ´ 10-3 ) = -900J
reaction. 20. (b) DH = DU + DnRT for N 2 + 3H 2 ¾
¾® 2 NH 3
Hence, option (a) is correct. Dng = 2 – 4 = – 2
10. (d) DH = DU + Dn(g) RT ; Dn(g) = 1 in (d); Dn(g) = 0 in \ D H = D U - 2 RT or D U = D H+ 2R T \ D U > D H
other cases. 1
21. (d) C( s) + O 2 ( g ) ¾¾® CO (g)
11. (b) For an isothermal process DE = 0 2
12. (a) Internal energy is dependent upon temperature and 1 1
according to first law of thermodynamics total energy of [ Dn = 1 - = ]
2 2
an isolated system remains same, i.e., in a system of
constant mass, energy can neither be created nor 1 -1
DH – DU = DnRT = ´ 8.314 ´ 298 = 1238.78 J mol
destroyed by any physical or chemical change but can 2
be transformed from one form to another 22. (c) DH = nCp DT solution; since DT = 0 so, DH = 0
DE = q + W 23. (a) DG = DH - T DS
For closed insulated container, q = 0, so, DE = + W, as 0 = 132.6 × 1000 – T × 66
work is done by the system 132.6 ´ 1000
T= = 2000K
13. (d) CH 4 (g) + 2O 2 (g) ¾¾® CO 2 (g) + 2H 2 O 66
(1) (2) (1) 24. (b) A( g ) + 2B( g ) ¾¾
® 2C( g ) + 3D( g )
D H = 890 kJ (given) Dn = 5 - 3 = 2
By Ist law of thermodynamics
DH = DE + nRT or DE = DH - nRT
D H = D E + D nRT T = 19 – 2 × 2 × 10–3 × 300 = 17.8 kcal
Where,
D H = change in enthalpy of reaction 25. (a) DH = -2.5 ´ 103 Cals ; DS = 7.4Cals deg -1
D E = change in internal energy of reaction Now, DG = DH - TDS
D n = change in number of gaseous moles. -
= -2.5 ´ 10 3 - 298 ´ 7.4 = -4705.2 = - ve
= ( nP - n R )
As, DG is negative so, reaction is spontaneous.
According to question,
26. (c) In isolated system neither exchange of matter nor
Þ D E = D H – D nRT T exchange of energy is possible with surroundings.
Thermodynamics 141

27. (b) Given C + O2 ® CO 2 , DH º = -x kJ ....(i)


2CO2 ® 2CO + O2 DH º = + y kJ … (ii) 36. (a) + H2 ¾
¾® ; DH = – 119.5 kJ
or CO2 ® CO + 1/ 2O2 , DH º = + y / 2 kJ ...(iii)
By adding no. (i) and (iii) eq. + 3 H2 ¾ ; DH = 3(– 119.5)
¾®
1
C + O2 + CO 2 ¾¾
® CO 2 + CO + O 2 = – 348.5 kJ
2
The resonance energy provides extra stability to the
1 y - 2x benzene molecule so it h as to be overcome for
C + O2 ¾¾ ® CO , DH º = y / 2 - x = kJ
2 2 hydrogenation to take place. So DH = – 358.5 – (–150.4)
28. (b) M.W. of ethane (C2H6) = 30 gm. = –208.1 kJ
M.W. of acetylene (C2H2) = 26 gm. 37. (c) Standard heat of formation of a substance is amount of
Heat evolved per gm of ethane heat evolved or absorbed when one mole of substance is
341 formed from its elements in their standard states. Since
= = 11.36 cal/gm. standard state of carbon and hydrogen are graphite and
30
Heat evolved per gm of acetylene gaseous H2 respectively. Hence standard heat of
formation of methane is given by
310
= = 11.92 cal/gm C( graphite) + 2H 2 ( g ) ® CH 4 ( g )
26
So, acetylene is better fuel. 38. (a) Dissolution of NaOH in water is an exothermic reaction
as indicated by negative value of enthalpy leading to
1 increase in temperature.
29. (c) SO 2 + O2 ¾
¾® SO 3
2 T1 - T2 200
39. (b) Efficiency of engine = = = 0.4
DH = DH of (SO ) - DH fo (SO ) T1 500
3 2
= –98.2 + 298.2 = 200 kJ/mole é ù é ù
40. (b) DH = S ê DH f° products ú -S ê DH f° reactants ú
30. (a) 2H 2 O 2 (l ) ¾¾
® 2H 2O(l ) + O 2 ( g ) DH = ? ë û ë û

DH = [2´DH f of H 2O(l ) + (DHf of O2 ) DH° = [DH f° (CO)(g) + DH f° (H 2O)(g)] –


-(2´DHf of H 2O2 (l ))]
[DH f° (CO 2 )(g) + DH f° (H 2 )(g)]
= [(2 ´ -286) + (0) - (2 ´ -188)]
= [– 110.5 + ( – 241.8)]– [– 393.5 + 0] = 41.2
= [-572 + 376] = -196 kJ / mole 41. (b) C 2 H 4 + 3O 2 ® 2CO 2 + 2H 2O.
31. (d) C(dia ) + O 2 ¾
¾® CO 2 .....(i) DH = -395.4 kJ/mole Change in enthalpy,
DH = DH products - DH reactants
C(gr ) + O 2 ¾
¾® CO 2 .....(2) DH = -393.5 kJ/mole
= 2 ´ (-394) + 2 ´ (-286) - (52 + 0) = – 1412 kJ/ mol.
(ii) – (i), C( gr ) - C(dia ) ¾
¾® O
42. (d) Q = mL
C(gr ) + ¾
¾® C(dia ) where L = latent heat of vapourisation of water
= 2260 kJ/kg
DH = -393.5 + 395.4 kJ/mole = + 1.9 kJ/mole. = 2260 × 103 J/kg
1 Q = 70 × 10–3 × 2260 × 103 = 158155 Joule
32. (c) Energy absorbed µ 43. (d) 13.7 kcal = 57 kJ = 5.7 × 104 J.
stability of compound
Energy released µ stability of compound p1
Thus, the order of stability is 44. (c) Work done by a gas = 2.303RT log
p2
142.2 > 25.9 > – 46.2 > – 393.2
i.e.. O3 > HI > NH3 > CO2 10
= 2.303 ´ 8.314 ´ 300 ´ log
33. (d) H 2 ( g ) + Br2 ( g ) ® 2HBr( g ) 1
DHº = (BE)reactant – (BE) product = 2.303 ´ 8.314 ´ 300 = 5744.1Joule
= (433 + 192) - (2 ´ 364) = 625 – 728 = – 103 kJ
H
34. (c) As MgO is a weak base hence some energy got
consumed to break MgO (s). Hence enthalpy is less than 45. (a) N º N + 3H – H 2N – H
–57.33 kJ mol–1. 945 + 3 × 436
H
ˆˆ† 2AB
35. (c) A 2 + B2 ‡ˆˆ absorbed energy 2 × (3× 391) = 2346
There is no change in volume so pressure will have no
Energy released
effect on it. As the reaction is endothermic ( DH is
positive), high temperature favours the forward reaction. DH = 2253 – 2346 = – 93 kJ
EBD_7327
142 CHEMISTRY

46. (d) eq. (i) + eq. (ii) find the required result and divide by 2. DH v 9710
47. (b) Eq. (b) shows largest phase change i.e., gas ¾¾ ® solid Using DS = = cal mol–1 K –1
48. (b) Maximum work done by a system at constant pressure T 373
and constant temperature is –DG. If can be derived as = 26.032 cal mol –1 K–1
follows : 56. (c) Conversion of graphite into diamond is an endothermic
From first law of thermodynamics, reaction. But DS would be negative for the conversion
q = DE + wexp. + wnon exp. of graphite into diamond. Diamond has more compact
At constant pressure, wexp = PDV structure so the volume is smaller. However, the atoms in
q = DE + PDV + wnon-exp. graphite are able to move with in their lattice so the entropy
q = DH + wnon exp. ....... (1) is higher.
For the reversible change at const temp., 57. (d) Given reactions are

q rev 1
DS = or qrev = TDS M + O2 ® MO + 351.4 kJ ...(i)
T 2
so from equ. (1), TDS = DH + wnon exp. 1
X + O 2 ® XO + 90.8 kJ ...(ii)
Þ DH – TDS = –wnon exp. ...... (2) 2
For a change taking place under constant pressure and Subtracting equation (ii) from (i), we get
constant temperature, M + XO ® MO + X + (351.4 – 90.8) k J
DH – TDS = DG or DH = (351.4 – 90.8) k J
so from equ (2) = 260.6 k J
DG = –wnon exp. 58. (b) Follow II law of thermodynamics.
(it means wnon exp. or wuseful)
or wmax = –DG 59. (b) Let, C(s) + O2 (g) ¾¾
® CO2(g);
49. (c) When gas is compressed its entropy decreases so, DS is DH = R .......... (i)
negative. 1
50. (c) According to Gibb's – Helmholtz equation and CO(g) + O 2 (g) ¾¾ ® CO2(g);
2
DG = DH – TDS DH = S .......... (ii)
Þ TDS = DH – DG Subtracting equ. (i) from equ (ii), we get
1 1
Þ DS = [DH – DG] C(s) + O2(g) – CO(g) – O 2 (g) ¾¾
®
T 2
51. (c) Latent heat of vaporisation of water CO2(g) – CO2(g); DH = R – S
= 540 cal g–1
\ latent heat of vaporisation for 1 mol of water = 540 × 1
or C(s) + O 2 (g) ¾¾® CO (g); DH = R – S
18 cal = 9720 cal 2
Thus DHvap. = 9720 cal mol–1 Hence, heat of formation of CO = R – S
and T = 100°C = 373 K 60. (a) Measure of disorder of a system is nothing but entropy.
For a spontaneous reaction, DG < 0. As per Gibbs
DH vap 9720 Helmholtz equation,
and DS = = cal mol–1 K–1
T 373 DG = DH – TDS
» 26.6 cal mol–1 K–1 Thus DG is –ve only
When DH = –ve (exothermic)
Latent heat of fusion DH and DS = +ve (increasing disorder)
52. (a) DS = =
Melting point T 61. (d) For the reaction,
H2 + Cl2 ¾ ¾® 2HCl
2930 DH for HCl = – 90 kJ
= J K–1 mol–1 = 9.77 J K–1 mol–1
300 for 2 HCl = – 180 kJ
53. (d) For isothermal reversible expansion DH = S(bond enersies of reactants)
– S(bond energies of product)
V2
w = q = nRT´2.303 log Þ –180 kJ = (B.E.H–H + B.ECl–Cl) – 2BEH–Cl
V1 Þ – 180 kJ = (430 kJ + 240 kJ) – 2 B.E.HCl
20 Þ 2 B.E.HCl = 670 kJ + 180 kJ = 850 kJ
= 2 RT ´ 2.303 log
2 850
Þ B.EH-Cl = kJ = 425 kJ
= 2 × 2 × T × 2.303 × 1 = 9.2 T 2
So, the H – Cl bond energy is 425 kJ mol–1
q 9.2T
Entropy change, DS =
= = 9.2 cal. 62. (d) D G is negative for a spontaneous process.
T T
63. (c) As all reactants and products are liquid D n (g ) = 0
54. (a) For a spontaneous process, DS total is always positive
DH = DE - DnRT
55. (b) Given DHv = 9710 cals mol–1
T = 100°C = 373 K DH = DE (Q Dn = 0)
Thermodynamics 143

64. (a) Entropy is a measure of randomness of the system which 3000


is highest in gases. So, entropy of hydrogen is highest Change in temperature = K = 150 K
among the given options. 20
65. (c) As we know that, Initial temperature = 300 K
DH = nCpDT (where n is no. of moles) Since the gas expands so the temperature decreases and
thus, final temperature is
28 150 K (300 – 150 = 150)
= ´ 7 ´ (373K - 273K)
14 76. (b) Bond dissociation energy of PH3(g)
= 2 × 7 × 100 J = 1400 J = 228 kcal mol–1
66. (d) We know that, DG = DH – TDS 228
When the reaction is in equilibrium, DG = 0 P – H bond energy = = 76 kcal mol -1
3
DH H H
0 = DH - T DS Þ T =
DS P—P
30 ´ 1000 H H
T= = 285.7 K Bond energy of 4 (P – H) + (P –P)
105
67. (a) DG = DH – TDS = 355 kcal mol–1
or 4 × 76 + (P – P) = 355 kcal mol–1
= 177.9 – (298 × 160.4/1000) = 177.9 – 47.79 = + 130.1kJ
68. (c) For spontaneous cooling of hot metal rod in air DSsystem P–P bond energy = 51 kcal mol–1
decreases (as metal rod cools down ) and DSair increases 77. (d) Dn = DE + PDV
(as surrounding air becomes hot). So, DSsystem < 0 and Now PV = nRT
DSuniverse > 0. where n = no of gaseous moles.
69. (b) For combustion reaction, DH is negative, From this we derive
Dn = (16 + 18) – ( 25 + 2 ) = +7, so DS is + ve, reaction is DH = DE + Dn g RT
spontaneous, hence DG is –ve. Dng = n2 – n 1
70. (b) where n1 = no of moles of gaseous reactant
71. (d) Isobutylene on combustion gives CO2 and H2O as shown n2 - no of moles of gaseous moles product
below : For the reaction,
CH3 2NH3(g) ¾¾
® N2(g) + 3H2 (g)
|
Dn g = 4 - 2 = 2.
CH3 - C = CH 2 (g) + 6O2 (g) ¾¾®
isobutylene Hence,
D H - D E = D n g R T = 2 × 8.314 × (273 + 27)
4CO2 (g) + 4H 2O(l)
Here Dng = 4 – 7 = – 3 < 0 = 2 × 8.314 × 300
Using IInd law of thermodynamics, 1 1
78. (b) H 2 + Cl 2 ¾¾ ® HCl
DH = DE + DngRT 2 2
DH = (DE) + (–ve term)
Þ DH < DE
D f H HCl = å B.E. of reactants
72. (c) Q DG = DH - T × DS - å B.E. of products
For a spontaneous reaction DG should be negative
DH = – 238 kJ, DS = – 87 JK –1 1 1
-90 = ´ 430 + ´ 240 - B.E. of HCl
Hence, reaction will be spontaneous when DH > T × DS . 2 2
Therefore, at 1000, 1500 and 3000 K the reaction would be \ B.E. of HCl = 215 + 120 + 90
spontaneous. = 425 kJ mol–1
15 79. (d) We know that heat (q) and work (w) are not state functions
73. (c) C6 H 6 + O2 ¾¾ ® 6CO2 + 3H 2O but (q + w) is a state function. H – TS (i.e. G) is also a state
2
function. Thus II and III are not state functions so the
1 mole º 3250 KJ correct answer is option (d).
0.39 0.39
80. (d) H 2O ( l) ‡ˆˆˆ
mole º 3250 ´ = 16.25 kJ 1atm
ˆˆˆˆ†
ˆ H2O(g)
78 78
74. (d) By 2× (ii) – (i) + (iii) DH = 40630 J mol –1
H2(g) + 2C(s) ® C2H2(g), DG° = 209 kJ DS = 108.8 JK–1 mol –1
75. (b) Since the gas expands adiabatically (i.e., no change in DG = DH - T DS When DG = 0, DH - T DS = 0
enthalpy) so the heat is totally converted into work.
For the gas CV = 20 J/K. Thus, 20 J of heat is required for DH 40630 J mol-1
1° change in temperature of the gas. T = DS = 108.8 Jk -1 mol-1 = 373.4 K.
Heat change involved during the process (i.e., work done)
= 3 kJ = 3000 J \ Correct choice : (d)
EBD_7327
144 CHEMISTRY

81. (a) Fe2O3(s) + 3CO(g) ¾¾ ® 2Fe(s) + 3CO2(g) ; 954


DH = –26.8 KJ ...(i) = = 318 kJ mol -1
3
FeO(s) + CO(g) ¾¾ ® Fe(s) + CO2(g) ;
In P2H4(g), energy of 1 P – P bond + 4 P – H bonds
DH = –16.5 KJ ...(ii)
eq. (i) – 2 × eq. (ii), we get = 1485 kJ mol–1
Q Energy of 1 P – H bond = 318 kJ mol–1
Fe2O3(s) + CO(g) ¾¾ ® 2FeO(s)+ CO2(g)
\ Energy of 4 P – H bond = 318 × 4
D H = –26.8 + 33.0 = + 6.2 kJ = 1272 kJ mol–1
82. (a) Ideal gas during spontaneous expansion into vacuum Thus, the P – P bond energy = 1485 – 1272
does not do any external work.
= 213 kJ mol–1
83. (d) Given DH = 30 kJ mol–1 T = 273 + 27 = 300 K
91. (d) A process is spontaneous only when there is decrease in
DH T 3 ´ 10 4 the value of free energy, i.e., DG is –ve.
DST = = J mol–1
T 300 92. (d) Raking up leaves into a thrash bag results in decrease
= 100 J mol–1 K–1 in randomness i.e. decrease in entropy.
84. (c) Given 93. (b) According to Gibb's equation,
4H( g ) ¾¾ ® 2H 2 ( g ); DH = -869.6 kJ DG = DH – TDS
or 2H 2 ( g ) ¾¾ ® 4H( g ); DH = 869.6 kJ Since reaction is at equilibrium
\ DG = 0, \ DH = TDS
869.6 DH = 21.3 kJ = 21300 J.
H 2 ( g ) ¾¾
® 2H( g ); DH = = 434.8 kJ
2 T = 82ºC = (82 + 273) K = 355 K.
On substituting these values,
DH 1.435 ´ 103
85. (c) DS = = = 5.260 cal / molK DH 21300 –1
T 273 DS = = = 60.0JK –1 mol .
T 355
5 94. (d) Lesser the bond length higher is the bond energy.
R
CP 2 5 H—H bond energy is the highest.
86. (d) = = = 1.67
CV 3 R 3 Bond energy
2 Bond Bond length
(kJ) mol -1
87. (b) Spontaneity of reaction depends on tendency to acquire H—H 0.74 435.1
minimum energy state and maximum randomness. For a F—F 1.42 154.8
spontaneous process in an isolated system the change Cl—Cl 1.99 242.7
in entropy is positive. Br—Br 2.28 192.5
I—I 2.26 149.0
88. (b) N 2 + 3H 2 ¾¾ ® 2NH 3 DH = 2 ´ -46.0 kJ mol–1
95. (d) Since the process is at equilibrium DG = 0 for DG = 0,
Let x be the bond enthalpy of N – H bond then
they should be DH > 0, DS > 0.
[Note : Enthalpy of formation or bond formation enthalpy
96. (a) DHsol = DHlattice + DHhydration
is given which is negative but the given reaction involves
bond breaking hence values should be taken as positive.] DHlattice = DHsol – DHhydration
DH = S Bond energies of reactants – S Bond energies 4 – (– 784) = DHlattice Þ DHlattice = + 788
of products 97. (b) Work done in reversible isothermal expansion
2 × – 46 = 712 + 3 × (436) – 6x V2
W = –2.303nRT log
– 92 = 2020 – 6x V1
6x = 2020 + 92 25
6x = 2112 = –2.303 × 0.2 × 8.314 × 300 log
2.5
x = + 352 kJ/mol = –2.303 × 0.2 × 8.314 × 300 = – 1148
89. (b) C2H5 OH(l) + 3O 2 ( g ) ® 2CO 2 ( g ) + 3H 2O(l) 98. (c) DS has negative value if number of gaseous moles
decreases during a reaction, Dng = –ve
Dng = 2 - 3 = -1 For the reaction
DU = DH - Dng RT 2SO2 + O2 ® 2SO3
Dng = 2 – 3 = –1
8.314 99. (d) DG = DH – TDS
= -1366.5 - ( -1) ´ ´ 300
103 For a reaction to be spontaneous,
DH = –ve, DS = +ve
8.314
= -1366.5 + (1) ´ ´ 300 at all temperatures.
103 but at high temperature, DG = D { H-T{DS
= – 1366.5 + 0.8314 × 3 = – 1364 .006 kJ - ve + ve
90. (a) In PH3(g), energy required to break 3 P–H bonds Thus the second term will have high positive value
= 954 kJ mol–1 and reaction will be non-spontaneous.
\ Energy required to break 1 P – H bond 100. (d) Gibb’s-Helmholtz equation is
DG = DH – TDS
Thermodynamics 145

For a reaction to be non-spontaneous at all C( s ) + 2H 2 ( g ) ¾ ¾


® CH 4 ( g )
temperatures, DH should be +ve and DS should be –ve Hence, dissociation energy of hydrogen and enthalpy
\ DG = +ve – T × (–ve); the value of DG is always of sublimation of carbon is required.
positive for such a reaction and hence it will be non- 11. (a) I 2 ( s ) + Cl2 ( g ) ¾¾ ® 2ICl( g )
spontaneous at all temperatures. DrH = [DH(I2(s) ®I2(g)) + DHI–I + DHCl–Cl] – [DHI – Cl]
EXERCISE - 2 = 151.0 + 242.3 + 62.76 –2 × 211.3 = 33.46
1. (d) 33.46
2. (b) 1 M H2SO4 = 2g eq. of H2SO4. Df H°(ICl) = = 16.73 kJ / mol
2
1
Hence y = 2x or x = y.. 12. (c) Given NaOH + HNO3 ® NaNO3 + H 2O
2 1 mol 1 mol DH = 57.0 kJ
3. (b) Hydration energy of Cl+ is very less than H+ hence it 0.2 mol 0.5 mol DH = ?
doesn’t form Cl+ (aq) ion. Given heat of neutralisation of strong acid by strong base
4. (b) For the reaction = 57.0 kJ
2 ZnS ® 2 Zn + S2 ; DG1º = 293 kJ ..........(1) Q 0.2 mole NaOH is limiting reagent.
2 Zn + O2 ®2 ZnO ; DG2º = –480 kJ ..........(2) \ Heat of neutralization = 0.2 × 57 = 11.4 kJ
S2 + 2 O2 ®2 SO2 ; DG3º = –544 kJ .........(3) 13. (a) No. of moles of hydrogen
DGº for the reaction
2 ZnS + 3 O2 ®2 ZnO + 2 SO2 Mass 4
= = = 2 moles
can be obtained by adding eqn. (1), (2) and (3) Molecular mass 2
Þ DGº = 293 – 480 – 544 = – 731 kJ Given 2H 2 ( g ) ® 4H( g ), DH = 208 kcal
5. (b) A ¾¾ 4g
® B, DH = + 24 kJ / mol
Þ HB – HA = + 24 ...(i) \ H ( g ) ® 2H( g ) DH = 104 kcal
2
B ¾¾ ® C, DH = – 18 kJ/mol 1 mol
Þ HC – HB = – 18 \ Bond energy of H–H bond is 104 kcal mol–1
or HB – HC = + 18 14. (a) Conc. of HCl = 0.25 mole
From Eqs. (i) and (ii), we have Conc. of NaOH = 0.25 mole
HC – HA = 6 ...(ii) Heat of neutralization of strong acid by strong base
\ HB > HC > HA = – 57.1 kJ
6. (d) CO 2 ¾¾
® C + O 2 , DH = 393J ...(i) HCl + NaOH ¾ ¾® NaCl + H2O – 57.1kJ
1 mole of HCl neutralise 1 mole of NaOH, heat evolved
2Zn + O 2 ¾¾ ® 2ZnO, DH = -412J ...(ii) = 57.1 kJ
By adding eq. (i) and (ii) we get \ 0.25 mole of HCl neutralise 0.25 mole of NaOH
\ Heat evolved = 57.1 × 0.25 = 14.275 kJ
2Zn + CO 2 ¾¾
® 2ZnO + C, DH = -19J
reversible path 15. (a) NaOH + HCl ¾¾ ® NaCl + H2O, DH = – 55.9 kJ
7. (c) Actually, heat of neutralisation of a strong acid by a strong
base means heat of formation of water.
A B Thus,
H + + OH - ® H 2O; DH = -55.9 kJ
irreversible path
or H 2 O ® H + + OH - ; DH = 55.9 kJ ..(i)
For a cyclic process the net change in the internal energy
is zero because the change in internal energy does not Given, HCN + OH - ® CN - + H 2 O; DH = -12.1kJ
depend on the path. …(ii)
8. (a) For spontaneous reaction, dS > 0 and dG should be Adding equations (i) and (ii), we get
negative i.e., < 0.
HCN ® H + + CN - ; DH = 55.9 - 12.1 kJ = 43.8 kJ
9. (d) X 2 + Y2 ¾ ¾® 2 XY , DH = 2(–200) kJmol–1
16. (b) DHS-S + 2DH H -S = 239 2DH H -S = 175
Let x be the bond dissociation energy of X2. Then
DH = -400 = Sx-x +S y-y - 2Sx-y Hence, DHS-S = 239 - 175 = 64 kcal mol -1
Then, DH for 8S(g ) ® S8(g ) is 8 × (–64) = –512k cal
= x + 0.5x – 2x = -0.5x
17. (b)
400 18. (a) For a pure substance TA and TB represent the same
or x = = 800 kJ mol -1
0.5 temperature. Hence A is a correct choice.
10. (c) The standard enthalpy of formation of CH4 is given by
the equation :
EBD_7327
146 CHEMISTRY

19. (d) If n p < nr then Dn( g ) = n p - nr = ‘–’ve 27. (b) Given DH


1
Since DH = DE + DnRT A ¾¾ ®B + 150 ...(1)
2
\ DH = DE – some quantity [Q Dn is - tive] 3B ¾¾ ® 2C + D –125 ...(2)
or DH < DE E + A ¾¾ ® 2D +350 ...(3)
20. (d) This reaction shows the formation of H2O, and the – X2 To calculate DH operate
represents the enthalpy of formation of H2O because as 2 × eq. (1) + eq. (2) – eq. (3)
the definition suggests that the enthalpy of formation is DH = 300 – 125 – 350 = – 175
the heat evolved or absorbed when one mole of substance 28. (a) Since, in the first reaction value of Dn is positive.
is formed from its constituent elements. 1 1
21. (b) For the reaction Dn = 1 - = hence entropy will increase i.e.,
2 2
PCl5 ( g ) ƒ PCl3 ( g) + Cl2 ( g) Ds = +ve.
The reaction given is an example of decomposition
reaction and we know that decomposition reactions are C (gr.) + 1 O2(g) ® CO(g); DS° = + ve
endothermic in nature, i.e., DH > 0. 2
Since, DG° = DH° – TDS hence the value of DG decreases
Further
on increasing temperature.
Dn = (1+1) – 1= +1
Hence more number of gaseous molecules are present in 29. (d) H 2O (l) H 2O (g) DH = 40.66 KJmol–1
products which shows more randomness i.e., DS > 0 T = 373 K
(DS is positive) DE = DH – DnRT
22. (c) The reaction for formation of HCl can be written as = 40.66×103 Jmol–1–1× 8.314JK–1mol–1× 373 K
H2 + Cl2 ® 2HCI D E = 37558 J / mol
H – H + Cl – Cl ® 2 (H – Cl) D E = 37.56 kJ mol–1
Substituting the given values, we get enthalpy of 30. (a) DG° = DH° – TDS°
formation of For a spontaneous reaction DG° < 0
2HCl = ( 676 – 862) = –186 kJ. DH °
or DH° – TDS° < 0 Þ T >
-186 DS°
\ Enthalpy of formation of HCl = kJ = –93 kJ.
2 3
Þ T > 179.1 ´ 10 > 1117.9K » 1118K
23. (b) DG = DH – T D S 160.2
At equilibrium, DG = 0 31. (d) Given DH = 41 kJ mol–1 = 41000 J mol–1
Þ 0 = (170 × 103 J) – T (170 JK– 1) T = 100°C = 273 + 100 = 373 K
Þ T = 1000 K n=1
For spontaneity, DG is – ve, which is possible only if DU = DH – DnRT = 41000 – (1 × 8.314 × 373)
T > 1000 K. = 37898.88 J mol–1 ; 37.9 kJmol–1
24. (b) Enthalpy of reaction 32. (b)
= B.E(Reactant)– B.E(Product) 33. (a) Given, for reaction
® H + (aq.) + OH – ( aq.);
(i)H2O (l) ¾¾
= éë B.E (C= C) + 4 B.E.(C–H) + B.E.(H - H) ùû
DH r = 57.32 kJ
- éë B.E.(C - C) + 6 B.E.(C - H) ùû 1
(ii) H 2 ( g ) + O 2 ( g ) ¾¾ ® H 2 O(l );
= [606.1 + (4 × 410.5) + 431.37)] – [336.49 + (6 × 410.5)] 2
= –120.0 kJ mol–1 DH r = –286.20 kJ
1 3 For reaction (i)
25. (a) ΔS for the reaction X 2 + Y2 ƒ XY3
2 2
DH r = DH°f (H + .aq)+DH°f (OH – .aq)–
ΔS = 50 – (30 + 60) = – 40 J
DH°f (H2 O, l)
For equilibrium DG = 0 = DH – T DS
DH -30000 57.32 = 0 + DH°f (OH – , aq) – DH°f (H 2 O, l) …(iii)
T= = = 750 K
DS -40 For reaction (ii)
26. (d) DH r = DH°f (H 2 O, l) –
(1) Kp > 0 (iv) Spontaneous 1
(2) D G < R T ln Q (i) Non spontaneous DH°f (H 2 , g) – DH°f (O 2 , g )
2
(3) Kp = 0 (ii) Equilibrium
–286.20 = DH°f (H 2 O, l)
DH
(4) T> (iii) Spontaneous On replacing this value in eq. (iii) we have
DS
and endothermic 57.32 = DH°f (OH – , aq) – (–286.20)
Thermodynamics 147

42. (d) The bond enthalpy of reaction =


DH°f (OH - , aq) = –286.20 + 57.32 = –228.88 kJ [Sum of bond enthalpies of reactants] –
34. (b) At equilibrium DG = 0 [Sum of bond enthalpies of products]
Hence, DG = DH – TeDS = 0 é1 3 ù 3
–50 = ê ´ 380 + ´ Y - Y ú - 3 ´ 150 = (Y - Y) - 260
DH ë2 2 û 2
\ DH = TeDS or Te < 210 ´ 2
DS Y-Y= = 140
3
For a spontaneous reaction 43. (b) 2C8 H18(g) + 25O 2(g) ¾¾® 16CO 2(g) + 18H 2O (g)
DG must be negative which is possible only if DH – TDS < 0
The given reaction is a combustion reaction. Hence
\ DH < TDS
this will be an exothermic reaction.
DH i.e. DH = –ve
or T > ; Te ; T
DS Further Dn = + ve i.e. DS = +ve
DG = DH – TDS = –DH – TDS = –ve
35. (d) 4NO 2 (g) + O 2 (g) ® 2N 2 O5 (g), D r H = – 111 kJ
D H 344.3 ´ 103
44. (c) DS = = = 100 J mol- 1
– 54 kJ T 3443
DH' 45. (b) We have to calculate the enthalpy of the reaction
2N 2 O5 (s) OH (g) ® O(g) + H(g)
From the given reactions, this can be obtained as follows.
– 111 – 54 = D H'
é1 1 ù
D H' = – 165 kJ – ê H 2 (g) + O2 (g) ® OH(g) ú ; DH = – 42.09 kJ mol–1
ë2 2 û
36. ˆˆ† B
(b) A ‡ˆˆ 1 1
DG° = D H° – TDS° + [H2(g) ® 2H(g)] ; DH = × 435.89 kJ mol–1
2 2
DG° = – 2.303 RTlog10 K 1 1
– 2.303 RT log10 K = DH° – TDS° + [O2(g) ® 2O(g)]; DH = × 495.05 kJ mol–1
2.303 RT log10 K = TDS° – DH° 2 2
Add
T DS ° - DH ° 298×10 +54.07×1000
log10 K = = OH(g) ® H(g) + O(g) ; DH = 423.38 kJ mol-1
2.303RT 2.303×8.314×298 46. (b) The factor TDS increases with increase in temperature.
= 9.998 » 10 47. (b) In an isothermal process change in internal energy (DE)
37. (a) Since, liquid is passing into gaseous phase so is zero (as it is a function of temperature).
entropy will increase and at 373 K during the phase \ According to first law of thermodynamics
transformation it remains at equilibrium. So, DG = 0. Q Q + W = DE. Hence Q = –W (if DE = 0)
38. (b) The species in its elemental form has zero standard If a system undergoes a change in which internal energy
molar enthalpy of formation at 298 K. At 298K, Cl2 is of the system remains constant (i.e. DE = 0) then
gas while Br2 is liquid. –W = Q. This means that work done by the system
39. (d) (i) 2C(s) + H2(g) ¾¾
® H – C º C – H(g) equals the heat absorbed by the system.
DH = 225 kJ mol–1 48. (b) The mass and volume depend upon the quantity of
matter so these are extensive properties while ratio of
(ii) 2C(s) ¾¾
® 2C(g) DH = 1410 kJ mol–1 mass to its volume does not depend upon the quantity
1410 of matter so this ratio is an extensive property.
C( s) ¾¾
® C( g )DH = = 705kJmol -1 49. (a) The properties whose magnitude depends upon the
2
quantity of matter present in the system are called
extensive properties eg, internal energy.
(iii) H2(g) ¾¾® 2H(g) DH = 330 kJ mol–1
50. (a) It is fact that absolute values of internal energy of
From equation (i) : substances cannot be determined. It is also true that it
is not possible to determine exact values of constitutent
225 = éë 2 ´ DHC(s) ¾¾
® C( g )
+ 1 ´ BE H – H ù
û energies of a substance.
- [2 ´ BE C – H + 1 ´ BECº C ] EXERCISE - 3
225 = [1410 + 1 × 330] – [2 × 350 + 1 × BEC º C]
225 = [1410 + 330] – [700 + BEC º C] Exemplar Questions
225 = 1740 – 700 – BEC º C 1. (c) Thermodynamics deals with the energy change,
BEC º C = 815 kJ mol–1 feasibility and extent of a reaction, but not with the
40. (a) Process is isothermal reversible expansion, hence rate and mechanism of a process.
DU = 0, therefore q = – W. 2. (c) For a closed vessel made of copper, there will be no
Since q = + 208 J, W = – 208 J exchange of matter between the system and the
M 12 surroundings but energy exchange can occur through
41. (c) DE = C × Dt × = 30 × 4 × = 360
m 4 its walls.
DE = – 360 kJ mol–1
EBD_7327
148 CHEMISTRY

3. (d) The state of a gas can be described by quoting the 10. (c)
relationship between pressure, volume, temperature and (a) C (graphite) + O2 (g) ® CO2 (g); DrH = x kJ mol–1 …
amount. The ideal gas equation is (i)
PV = nRT
1
4. (c) Specific heat is an intensive property which depends (b) C (graphite) + O 2 (g) ® CO (g); D r H = y kJ mol –1
only on the nature of the gas. Hence, if the volume of 2
gas is reduced to half from its original volume the … (ii)
specific heat will remain constant. On subtracting eqn (i) & (ii) we get
5. (c) The complete combustion of one mole of butane is 1
represented by CO ( g ) + O 2 ( g ) ® CO 2 ( g ) ; Dr H = z kJ mol -1
2
13
C 4 H10 ( g ) + O 2 ( g ) ® 4CO2 ( g ) + 5H 2 O ( l ) Hence, x – y = z or x = y + z
2 11. (c) x > y because same bonds are formed in reaction (1)
Dc H should be negative and have a value of 2658 kJ and (2) but no bonds are broken in reaction (1) whereas
mol–1. in reaction (2) bonds in the reactant molecules are
6. (b) DfH° = DfU° + DngRT broken. As energy is absorbed when bonds are broken,
For the reaction, DfH° energy released in reaction (1) is greater than that in
C(s) + 2H2(g) ® CH4(g) reaction (2).
DfH° = 1 – 2 = –1 12. (c) Enthalpy of formation can be positive or negative.
\ DfH° = DU – 1 × RT For example :
\ DfH° < DfU° C+ O2 ® CO2 is exothermic whereas, C+ 2S ® CS2 is
7. (c) For free expansion, W = 0 ; and endothermic.
For Adiabatic process, q = 0 13. (a) Enthalpy of sublimation of a substance is equal to
According to first law of thermodynamics, enthalpy of fusion + enthalpy of vaporisation.
DU = q + W = 0 Sublimation is, direct conversion of solid to vapour.
Since there is no change in DU hence, temperature Writing in two steps, we have solid ® liquid ® vapour
change will be zero i.e., DT = 0 solid ® liquid requires enthalpy of fusion
8. (b) Area under the curve is always greater in irreversible liquid ® vapour requires enthalpy of vaporisation
compression than that in reversible compression which
14. (b) DG gives a criterion for spontaneity at constant
can be seen from given figure.
pressure and temperature.
(i) If DG is negative (< 0), the process is spontaneous.
(ii) If DG is positive (> 0), the process is non-
spontaneous.
P
(iii) If DG is zero then reaction is at equilibrium.

NEET/AIPMT (2013-2017) Questions

Vf Volume (V) Vi 15. (c) CH 4 + 2O 2 ¾® CO2 + 2H2O


x 2x
Irreversible compression
C3 H8 + 5O 2 ¾® 3CO2 + 4H2O
(5- x ) 5(5- x )
2x + 5(5– x) = 16
P Þ x = 3L
\ Heat released
3 2
= ´ 890 + ´ 2220 = 317
22.4 22.4
Vf Volume (V) Vi 16. (c) Applying Hess’s law, equation (i) can be obtained by
Reversible compression adding equations (ii) and (iii).
9. (c) During the process of freezing, energy is released which \ x =y+ z
is absorbed by the surroundings. 17. (b) Given DU = 2.1 k cal., DS = 20 cal. K– 1
- q rev T = 300 K
\ DSsys =
T Q DH = DU + DngRT
q rev Putting the values given in the equation
DSsurr = i.e., on freezing, entropy of the system
T
decreases and of surrounding increases.
Thermodynamics 149

which is possible only if


2
DH = 2.1 + 2 ´ ´ 300 DH < 0 and DS > 0
1000 \ spontaneous at all temperatures.
= 2.1 + 1.2 = 3.3 k cal. 20. (a) Given DH 35.5 kJ mol–1
Now, DG = DH – TDS DS = 83.6 JK–1 mol–1
20 Q DG = DH – TDS
= 3.3 - 300 ´ = - 2.7 k cal For a reaction to be spontaneous, DG = –ve
1000
18. (a) C + O2 ® CO2 + 393.5 kJ/mol i.e., DH < TDS
12g 44g DH 35.5 ´ 103 Jmol -1
\ T> =
44g is formed from 12g of carbon DS 83.6 JK -1
So, th e given reaction will be spontan eous at
12 ´ 35.2
35.2g is formed from g of C T > 425 K
44 21. (b) The system is in isolated state.
= 9.6 g of C = 9.6/12 = 0.8 mole Q For an adiabatic process, q = 0
1 mole release heat 393.5 kJ DU = q + w
0.8 mole release heat = 393.5 × 0.8 \ DU = w
= 314.8 kJ » 315 kg = –pDV
19. (c) DG = DH – T · DS = –2.5 atm × (4.5 – 2.5) L
For a spontaneous reaction DG = –ve (always) = –2.5 × 2 L-atm
= –5 × 101.3 J
= –506.5 J » –505J
EBD_7327
150 CHEMISTRY

7 Equilibrium
Equilibrium state arises when the rate of forward reaction becomes Example: Soda water bottle in which CO2 is dissolved in solution.
equal to the rate of backward reaction Gas (g) Gas (solution).
EQUILIBRIUM IN PHYSICAL PROCESSES CO2 (gas) CO2 (in solution).
The most familiar examples are phase transformation processes, This equilibrium is governed by Henry's law which states that
eg : solid - liquid; liquid - gas and gas - solid the mass of a gas dissolved in a given mass of a solvent at any
(i) Solid-Liquid Equilibrium temperature is proportional to the pressure of the gas above the
Ice and water in an isolated thermos flask. The equilibrium that solvent.
exists is: EQUILIBRIUM IN CHEMICAL PROCES SES –
Ice(s) ƒ water(l) DYNAMIC EQUILIBRIUM
For any pure substance at atmospheric pressure, the temperature Chemical reactions also attain a state of equilibrium. These
at which the solid and liquid phases are at equilibrium is called reactions occur both in forward and backward directions. When
normal melting point or normal freezing point of the substance.
the rates of forward and reverse reactions become equal,
(ii) Liquid - Vapour Equilibrium concentrations of reactants and products remain constant. This
At equilibrium, rate of evaporation = rate of condensation and, is the stage of chemical equilibrium. It is dynamic in nature i.e., at
H 2O ( l ) ƒ H 2O ( vap ) equilibrium stage, the reaction does not stop. Instead, the reactants
At equilibrium, the pressure exerted by water molecules at a given give product(s) in forward reaction and product(s) give the original
temperature remains constant and is called equilibrium vapour reactants in backward reaction; the rates of the forward and
pressure of water. Vapour pressure of a liquid increases with backward reaction become equal and there is no net change in
temperature. composition.
For any liquid at 1 atm pressure, the temperature at which liquid
Law of Chemical Equilibrium and Equilibrium
and vapours are at equilibrium is called normal boiling point of the
liquid. Constant
Boiling point depends on atmospheric pressure and on altitude of According to equilibrium law or law of chemical equilibrium, at a
a place. At high altitude boiling point decreases. given temperature, the product of concentrations of the reaction
(iii) Solid - Vapour Equilibrium products raised to the respective stoichiometric coefficient in
At equilibrium, balanced chemical equation divided by product of concentrations
Rate of sublimation = Rate of condensation of the reactants raised to their individual stoichiometric
I2 (solid ) ƒ I2 ( vapour ) coefficients in a balanced chemical equation has a constant value.
For a general reversible reaction,
Camphor ( solid ) ƒ Camphor ( vapour )
aA + bB ƒ cC + dD
NH4 Cl ( solid ) ƒ NH4 Cl ( vapour )
The concentrations in an equilibrium mixture are related by the
(iv) Equilibrium Involving Dissolution of Solids or equilibrium equation:
Gases in Liquids
(a) Solids in Liquids : Example : Dissolution of sugar in water. At K C (constant) =
[ C] c [ D ] d
equilibrium, in saturated solution. [ A ] a [ B] b
Sugar (solution) Sugar (solid) and, rate of dissolution where KC is the equilibrium constant. The subscript ‘C’ indicates
of sugar = rate of crystallization of sugar. that KC is expressed in concentrations of mol L–1.
(b) Gases in liquids : The concentration of a gas in liquid is According to law of mass action,
proportional to the pressure (concentration) of the gas over Rf = Kf [A]a [B]b
the liquid. There is equilibrium between molecules in gaseous Rb = Kb [C]c [D]d
state and molecules dissolved in liquid under pressure
Equilibrium 151

At equilibrium, if a substance ‘X’ is involved, then [X(s)] and [X (l)] are constant.
Rf = Rb However,
Kf [A]a [B]b = Kb [C]c [D]d [X (g)] and [X (aq)] will vary.
K f [C]c [D]d Ex: CaCO3(s) D
= CaO(s) + CO2(g)
K b [A]a [B]b
é CaO ( s ) ùû éëCO 2 ( g ) ùû
K KC = ë
\ Kc = f éëCaCO3 ( s ) ùû
Kb
Relations Between Equilibrium Constants for a As, [CaO(s)] and [CaCO3(s)] are constant therefore
General Reaction and its Multiples KC' = [CO2(g)]
or KP = PCO
2
Chemical Equation Equilibrium Constant Some other examples are,
aA + bB ƒ cC + dD KC MgCO3 ( s) ƒ MgO( s ) + CO2 ( g )
cC + dD ƒ aA + bB ¢ =1/ KC
KC 3Fe( s) + 4H 2O(l ) ƒ Fe3O 4 (s ) + 4H 2 ( g )
n
n aA + nbB ƒ ncC + ndD ¢¢ = (KC )
KC Predicting the Extent and Direction of Reaction
Equilibrium Constant in Gaseous Systems Extent of a Reaction : The numerical value of KC for a reaction
indicates the extent of reaction. The magnitude of KC or KP is
In gaseous system, equilibrium constant is denoted by KP and it
is related to KC by the expression : directly proportional to conc. of products and inversely
proportional to conc. of reactants. This implies that high value of
Dn
K P = K C ( RT ) KC is suggestive of high conc. of products. Dependence of extent
Where Dn = (no. of moles of products) – (no. of moles of reactants) of reaction on KC is shown as :
KP = equilibrium constant when conc. of gaseous reactants and
products are expressed in terms of pressure. KC
Negligible Extremely
Unit of Equilibrium Constant : –3 3 large
Unit of Kc = (Concentration)Dn 10 1 10
Unit of Kp = (atm)Dn Reaction Both reactants Reaction proceeds
Significance of Equilibrium Constant hardly and products almost to completion
proceeds are at equilibrium
(i) High value of Kc :
Kc >>> 1 Direction of reaction
Concentration of product > concentration of reactant For a general reaction :
(ii) Low value of Kc :
Kc <<< 1 aA + bB ƒ cC + dD
Concentration of product < concentration of reactant
(iii) Kc or Kp is the measurement of the tendency of the reaction [ C]c [ D]d
to proceed forward direction The reaction quotient, Qc =
[ A ]a [ B]b
Relation Between Kp and Kc
Kp = Kc (RT)Dn Then,
When Dn = 0 (i) if Qc > Kc, reaction will proceed in the backward direction,
Kp = Kc (RT)0 \ Kp = Kc i.e. in direction of reactants
When Dn = + ve i.e., Dn > 0 (ii) if Qc < Kc, reaction will proceed in the forward direction,
Kp > Kc i.e. in direction of products.
When Dn = – ve i.e., Dn < 0 (iii) if Qc = Kc, the reaction mixture is at equilibrium.
Kp < Kc Standard Free Energy Change of a Reaction and
Types of Chemical Equilibrium Its Equilibrium Constant
Homogeneous Equilibrium : In a homogeneous system, all the DG° = –2.303 RT log Kc
reactants and products are in the same phase. For example: and DG° = –2.303 RT log Kp
N 2 ( g ) + 3H 2 ( g ) ƒ 2NH3 ( g ) where DG° = free energy change.
Kc and Kp = equilibrium constant.
H 2 ( g ) + I2 ( g ) ƒ 2HI( g ) If DG < 0, log Kc > 1
Heterogeneous Equilibrium : Equilibrium in a system having more \ Spontaneous reaction proceeds in forward direction
than one phase is called heterogeneous equilibrium. The If DG > 0, log Kc < 1
equilibrium expression for the heterogeneous equilibria involving \ Non-spontaneous reaction proceeds in backward direction
a pure liquid or a pure solid can be simplified, as the molar • If DG° = 0, KC = 1
concentration of a pure solid or liquid is constant. In other words, Reaction will be at equilibrium.
EBD_7327
152 CHEMISTRY

Le Chatelier’s Principle and Factors Affecting reactions to same extent. It lowers the activation energy for
Equilibria forward and backward reactions by same amount thus catalyst
A change in temperature, pressure and concentration alters the helps in attaing equilibrium quickly and it does not affect the
equilibrium. In order to decide what course the reaction adopts equilibrium composition of the reaction mixture.
and make a qualitative prediction about the effect of a change in IONIC EQUILIBRIUM
conditions on equilibrium, we use the Le-Chatelier’s principle. Substances can be divided into two categories based on their
According to this principle, "If a system at equilibrium is subjected ability to conduct electricity. One category of substances conduct
to a change of concentration, pressure or temperature, the electricity in their aqueous solutions and are called electrolytes
equilibrium shift in a direction that tends to undo the effect of the while the other do not and are called non-electrolytes.
change". Types of Electrolytes
(i) Effect of change in concentration : (a) Strong Electrolytes : Compounds which completely ionise
When the concentration of any of the reactants or products in aqueous solution and are good conductors of electricity
in a reaction at equilibrium is changed, the composition of e.g. HCl, H2SO4, HNO3, NaOH, KOH, CH3COONa, NH4Cl
the equilibrium mixture changes so as to minimize the effect etc. (a = 100%)
of concentration changes. (b) Weak Electrolytes : Compounds which ionise partially in
For a general reaction, aqueous solution and are poor conductors of electricity e.g.
aA +bB ƒ cC + dD CH3COOH, HCN, NH4OH, Al(OH)3 etc. (a < 100%)
In weak electrolytes, equilibrium is established between ions
Qc =
[ C]c [ D]d and unionized molecules. This type of equilibrium involving
ions in aqueous solution is called ionic equilibrium.
[ A]a [ B]b
Acids, Bases and Salts
[C]c [ D]d (i) Arrhenius concept of acids and bases:
at equilibrium, K c = Qc = According to this concept, acids are substances that
[ A ]a [ B]b dissociate in water to give H+ ions and bases are substances
If conc. of reactants are increased, Qc < Kc. Thus, to attain that produce OH– ions. Example :
equilibrium again, reaction proceeds in forward direction.
If conc. of products are increased, Qc > Kc. Thus, to attain HX ( aq ) ¾¾
® H + (aq) + X - ( aq )
acid
equilibrium again, reaction proceeds in backward direction.
(ii) Effect of pressure change MOH ( aq ) ¾¾
® M + (aq) + OH - ( aq )
base
If the pressure is increased, then according to Le-Chatelier’s
principle, then the reaction proceeds in the direction where (ii) Bronsted - Lowry acids and bases
the no. of moles (or pressure) is decreased (pressure is According to this concept, acid is a substance that is capable
proportional to moles of gas). of donating H+ and base is a substance which is capable of
(iii) Effect of Inert Gas addition accepting H+. In other words, acids are proton donors while
If the inert gas is added at constant volume, the equilibrium bases are proton acceptors.
remains undisturbed. This is because the addition of an inert The species B accepts a proton and thus behaves as a base,
gas at constant volume does not change the partial pressures while the species BH+ gives up a proton and thus acts as an
or molar concentrations of the substance involved in the acid
reaction. B + H+ ƒ BH+
When an inert gas is added to the equilibrium system at (Base) (Proton) (acid)
constant pressure, then the volume will increase. As a result, Such related pair of an acid and a base is said to be conjugate
the number of moles per unit volume of various reactants to one another so B is a conjugate base of the acid BH+ and
and products will decrease. hence, the equilibrium will shift BH+ is the conjugate acid of the base B.
in a direction in which there is increase in number of moles of eg. :
gases.
(iv) Effect of temperature change
The temperature dependence of equilibrium constant +
H2SO4 + H2 O H3O + HSO4 –
depends on the sign of DH for the reaction.
The equilibrium constant for an exothermic reaction (negative Acid Base Conjugate acid Conjugate base
DH) decreases as temperature increases. The reaction then (iii) Lewis acids and Bases
proceeds in backward direction. Lewis acid is a species which accepts an electron pair while
The equilibrium constant for an endothermic reaction (+ve Lewis base is a species which denotes an electron pair.
DH) increases as temperature increases. The reaction then Electron deficient species like AlCl3, Co3+, Mg2+, ZnCl2,
proceeds in forward direction. SnCl2, BF3, Ag+, Cu2+ etc. can act as Lewis acids while
(v) Effect of a catalyst ..
electron rich species like H2O, NH3, OH–, R – O – R, CN–,
A catalyst increases the rate of forward and backward Cl– etc., can act as Lewis bases.
Equilibrium 153

Ionic Product of Water Ionization of weak bases :


In pure water, the following equilibrium exists : The equation for ionization of weak base is:

H 2 O ( l ) + H 2 O ( l ) ƒ H 3O + ( aq ) + OH - ( aq ) MOH ( aq ) ƒ M + ( aq ) + OH - ( aq )
base conj.acid conj.base Initial conc. C 0 0
Equilibrium conc C(1 – a) Ca Ca
é H3O + ù éOH - ù
[B+ ][OH- ]
K=ë ûë û
Kb =
[ 2 ]
H O [BOH]

K [ H 2 O] = é H3O+ ù é OH - ù ca ´ ca ca 2
ë ûë û = =
c(1 - a ) 1 - a
K w = é H3O+ ù é OH- ù In the case of weak base, (1–a) is equal to 1.
ë ûë û
(Kw ® ionic product of water) \ Kb = ca2

[H 3O + ] = [OH - ] = 1´ 10 -7 M (For pure water) a=


Kb
\
\ Kw = (1 × 10–7)2 = 1 × 10–14 M2. c
Value of Kw depends on temperature. Value of Kw increases with pKb = – log (Kb)
increase in temperature and decreases with decrease in Relation Between Ka and Kb
temperature.
Ka × Kb = Kw.
+ -
if éë H3O ùû > éëOH ùû , aqueous solution is acidic pKa + pKb = pKw = 14 (at 298 K)
Polybasic Acids and Polyacidic Bases
+ -
if éë H3O ùû < éëOH ùû , aqueous solution is basic The acids which have more than one ionizable proton per molecule
of the acid are called polybasic or polyprotic acids.
+ -
if éë H3O ùû = éëOH ùû , aqueous solution is neutral The bases which have more than one ionizable OH– ion per
molecule of the base are called polyacidic bases.
The pH Scale Polybasic acids :
The term pH was given by Sorenson. pH is defined as the negative These acids undergo complete ionisation in several steps; the
logarithm to the base 10 of the hydrogen ion concentration i.e., number of steps is equal to the number of replaceable hydrogen
pH = –log [H+] atoms in the molecule of the acid
Acidic solution has pH < 7
Basic solution has pH > 7 For example, orthophosphoric acid (H3PO4) is completely ionised
Neutral solution has pH = 7 in three steps :
pH value decreases on heating the solution. ˆˆˆ
K
1† + -
H 3PO 4 ‡ˆˆˆ H + H 2 PO 4
Ionisation of Weak Acids and Bases
K
Ionization constants of weak acids H 2 PO- ˆˆˆ
2† +
4 ‡ˆˆˆ H + HPO42-
Consider a weak acid HX that is partially ionized in aqueous K
solution. The equilibrium can be expressed by: ˆˆˆ
3† +
HPO 24- ‡ˆˆˆ H + PO34-
HX(aq) + H2O(l) ƒ H3O+ (aq) + X - (aq) The relative value of these constants is as below :
Initial conc. C 0 0 K1 > K2 > K3
(M) The overall dissociation constant (K) is given by the following
If a is the extent of ionization, then relation.
At, equilibrium K = K1 × K2 × K3
conc. (M) C – Ca Ca Ca The higher order ionization constants are smaller than lower order
C a . Ca C2 a 2 ionization constants. This is because it is difficult to remove
\ Ka = = positively charged proton from a negative ion due to electrostatic
C - Ca C(1 - a) forces.
Factors affecting Acid Strength
Ca 2 The extent of dissociation depends on the strength and polarity
Þ Ka = and (pKa = – log Ka)
1- a of H–A bond.
\ Larger the value of Ka, stronger is the acid. In general, when strength of H–A bond decreases, i.e., energy
In case of weak acids, the degree of dissociation (a) is very small, required to break the bond also decreases, HA becomes a stronger
therefore (1–a) may be taken as 1. acid. On the other hand, when H–A bond becomes more polar, i.e.
\ Ka = ca2 electronegativity difference between atoms H and A increases,
charge separation occurs, cleavage of the bond becomes easier
Ka thereby increasing the acidity.
or a=
c While comparing elements of the same group of the periodic table,
H–A strength is more important factor in determining acidity than
EBD_7327
154 CHEMISTRY

its polar nature. As the size of A increases down the group, H–A (ii) For the salt of weak acid and strong base, Example :
bond strength decreases and so acid strength increases. For ex: CH3COONa,
HF << HCl << HBr << HI
¾¾¾¾¾¾¾¾¾® CH3COONa ( aq ) ¾¾® CH3COO- ( aq ) + Na + ( aq )
Size increases
Acid strength increases CH3COO– ions undergo hydrolysis to give :
While comparing the elements of the same period, bond polarity CH 3COO - ( aq ) + H 2O ( l ) ƒ CH 3COOH ( aq ) + OH - ( aq )
dominates. As the electronegativity of A increases, acid strength Thus, the solution is alkaline with pH > 7.
increases.
Kw
CH < NH < H O< HF Kh =
¾¾¾¾¾¾¾¾¾¾¾®
4 3 2
Ka
Electronegativity of A increases
Acid strength increases
Note : Kh
Þ Kh = Ch2 Þ h =
(a) Strength of all strong acids in water is same and become c
equal to the strength of H3O+. This is called levelling effect.
(b) Relative strength of acids in glacial acetic acid is in the K a .K w
following order : [H + ] = , pH = [7 + ½pK a + ½ log C]
c
HClO4 > HI > HBr > H2SO4 > HCl > HNO3 > H3O+
H3PO4 > HF > CH3COOH > H2CO3 > H2S > HCN (iii) For a salt of strong acid and weak base, Example : NH4Cl, we
have
Relative Strength of Some Acids
(i) Oxy acid (Non-metal + O–H) NH 4 Cl ( s ) + H 2 O ( l) ® NH +4 ( aq ) + Cl - ( aq )
Case I : When non metals are different
Acidic character µ electronegativity of non-metal NH 4+ (aq) + H 2 O (l) ƒ NH 4OH (aq) + H + (aq)
Case–II : When non metals are similar Thus, the solution is acidic with pH < 7
Acidic character µ Oxidation Number
Kw
(ii) In hydride : EN increase, Acidic character increase Kh =
Kb
O
||
(iii) In carbonic acid : R – C – O – H Kh
Kh = ch2 Þ h =
1 c
Acidic character µ – I effect µ
+ I effect Kw Kw ´ c
(iv) Mathematical explanation of acids : [H+] = ch = c K ´ c = Kb
b
Acidic character µ [H+] µ Ka
Relative Strength of Some Bases é 1 1 ù
pH = ê7 - pKb - log c ú < 7
(i) Hydroxide (metal + O–H) ë 2 2 û
(iv) For a salt of weak acid and weak base, example : CH3COONH4
1 we have
Basic character µ Electronegativity of metal
CH3 COO - + NH +4 + H 2O ƒ CH 3COOH + NH 4OH
(ii) Basic strength of amine :
1 CH3 COOH ƒ CH 3COO - + H +
Basic strength µ +I effect µ
–I effect
NH 4 OH ƒ NH +4 + OH -
e.g. : (CH3)2NH > CH3NH2 > (CH3)3N > NH3
(C2H5)2NH > C2H5NH2 > NH3 > (C2H5)3N H 2 O ƒ H + + OH-
(iii) Explanation of mathematical bases : Kw
Kh =
Basic character µ [OH– ] µ Kb K a .Kb

Hydrolysis of Salts and pH of Their Solutions h= Kh


Hydrolysis may be defined as a process involving the action of
water on a salt to form a mixture of acid and alkali. Thus hydrolysis KW
is the reverse of neutralization. h=
K a .Kb
ˆˆ† Acid + Base
Salt + water ‡ˆˆ
The pH of the solution changes as a result of hydrolysis. 1 1
(i) The cations of strong bases and anions of strong acids simply pH = 7 + pKa – pKb
2 2
get hydrated and are not hydrolysed. As a result, pH of the
solution of the salt of strong acid and strong base is 7 i.e. pH > 7 if difference is positive and pH < 7 if difference is
neutral solution. negative
Equilibrium 155

Common - Ion Effect (b) Basic buffer mixture : A weak base with it's salt with strong
It states that if to the solution of a weak electrolyte, a solution of acid.
strong electrolyte is added which furnishes an ion common to e.g. : NH4OH + NH4Cl, glycine + Glycine Hydrochloride
that furnished by the weak electrolyte, the ionization of the weak pOH and pH values of basic buffers can be determined from
electrolyte is suppressed. the following Henderson's equation.
For example : Dissociation of acetic acid
pOH = pKb + log
[ Salt ]
CH 3 COOH ( aq ) ƒ H +
( aq ) + CH 3COO ( aq )
- [ Base]
é H + ù éCH3COO - ù Change in concentration or number of moles
Ka = ë û ë û
Buffer Capacity =
of acid or base added to litre of Buffer
[CH 3COOH ] Change in pH
Addition of acetate ions results in decrease in the concentration
Solubility Equilibria of Sparingly Soluble Salts
of H+ ions. Also, if H+ ions are added, equilibrium moves in the
For a salt to be soluble in a solvent, the solvation energy
direction of undissociated acetic acid, i.e., in a direction of reducing
(or hydration energy) should be greater than lattice energy.
the conc. of H+ ions.
Each salt has characteristic solubility which depends on
Buffer Solutions temperature.
The solutions which resist change in pH on dilution or with the On the basis of solubility, salts are classified as:
addition of small amounts of acid or alkali are called buffer 1. Soluble : Solubility > 0.1 M
solutions. 2. Slightly soluble : 0.01 M < Solubility < 0.1 M
Types of Buffer Solution 3. Sparingly soluble : Solubility < 0.01 M
(i) Simple buffers : Solubility Product (Ksp) : Solubility product of a sparingly soluble
(a) A salt of weak acid and weak base in water. electrolyte is equal to the product of ionic concentrations in a
Example CH3COONH4, NH4CN saturated solution at a given temperature.
(b) A mixture of an acid salt & normal salt of a poly basic acid. Solubility product constant
Example Na2HPO4 + Na3PO4 A solid salt of general formula AxBy with molecular solubility S in
(ii) Mixed buffers : These are of two types equilibrium with its saturated solution may be represented as:
(a) Acidic buffer mixtures : Solution of a weak acid and its salt AxBy ‡ˆˆˆˆ† xAy+ + yBx–
with strong base is acid buffer.
Ksp = [Ay+]x[Bx–]y
e.g. : NaHCO3 + H2CO3, CH3COOH + CH3COONa,
Let solubility of compound AxBy is s mol lit–1, then
pH of an acidic buffer : ˆˆ† xAy+ + yBx–
AxBy ‡ˆˆ
[Salt ] s xs ys
pH = pKa + log
[ Acid ] Ksp = (xs)x (ys)y
This equation is called Henderson equation Ksp = X xYy Sx+y
S(x + y) = Ksp / xx. yy.
S = (Ksp / xx . yy) 1/x+y.
EBD_7327
156 CHEMISTRY

1. The solubility of PbCl2 is given by 9. The KP/KC ratio will be highest in case of
1/ 3
é K sp ù 1
(a) ê ú (b) [8 Ksp]1/2 (a) CO (g) + O (g) ‡ˆˆ
ˆˆ† CO2 (g)
ë 4 û 2 2
-
(c) [Ksp]1/2 (d) K sp (b) H2 (g) + I2 (g) ‡ˆˆ
ˆˆ† 2HI (g)
2. For the following reaction in gaseous phase
(c) PCl5 (g) ‡ˆˆ
ˆˆ† PCl3(g) + Cl2 (g)
1
CO( g ) + O2 ( g ) ® CO2 ( g ), K P / K c is
2 (d) 7H2 (g) + 2NO2 (g) ‡ˆˆˆˆ† 2NH3 (g) + 4H2O(g)
(a) (RT)1/2 (b) (RT)–1/2 (c) (RT) (d) (RT)–1
10. A solution of FeCl3 in water acts as acidic solution due to :
3. The solubility of Ca3 (PO4)2 is 's' moles per litre. Its solubility
product is (a) hydrolysis of Fe3+ (b) acidic impurities
(a) 72 s5 (b) 108 s5 (c) dissociation (d) ionisation
(c) 9 s 2 (d) 8 s3 11. Pure ammonia is placed in a vessel at a temperature where its
4. Which of the following is correct for the reaction? dissociation constant (a) is appreciable. At equilibrium :
(a) Kp does not change significantly with pressure.
N 2 ( g ) + 3H 2 ( g ) 2NH3 ( g )
(b) does not change with pressure.
(a) Kp = Kc
(c) concentration of NH3 does not change with pressure.
(b) Kp < Kc
(d) concentration of hydrogen is less than that of nitrogen.
(c) Kp > Kc
(d) Pressure is required to predict the correlation 12. The value of K p for the reaction:
5. The equilibrium constant for a reaction A + 2B ‡ˆˆˆˆ† 2C
is 40. The equilibrium constant for reaction 2H 2S(g) = 2H 2(g) + S 2(g) is 1.2 ´ 10 -2 at

ˆˆ† B + 1 A is
C ‡ˆˆ 1065°C. The value for KC is:
2
2 (a) <1.2 ´ 10 -2 (b) >1.2 ´ 10 -2
æ 1 ö 1 1
(a) 40 (b) ç ÷ (c) (d)
è 40 ø 40 ( 40 )1/ 2 (c) 1.2 ´ 10 -2 (d) 0.12 ´ 10 -2
6. For which one of the following reactions Kp = Kc? 13. For a chemical reaction;
ˆˆ† PCl3 + Cl 2 (b) N 2 + O 2 ‡ˆˆ
(a) PCl5 ‡ˆˆ ˆˆ† 2NO A (g) + B (l) ƒ D (g) + E (g)
Hypothetically at what temperature, Kp = Kc
(c) N 2 + 3H 2 ‡ˆˆ
ˆˆ† 2NH 3 (d) 2SO3 ‡ˆˆ ˆˆ† 2SO 2 + O 2
(when, R = 0.08 l-atm/mole-K)
7. K1 and K2 are equilibrium constant for reactions (i) and (ii) (a) T = 0 K (b) T = 1K
ˆˆ† 2 NO (g)
N2(g) + O2 (g) ‡ˆˆ ...(i) (c) T = 12.5 K (d) T = 273 K
1 1 14. PCl5 is dissociating 50% at 250°C at a total pressure of P
ˆˆ†
NO(g) ‡ˆˆ N2 ( g ) + O2 ( g ) ...(ii) atm. If equilibrium constant is Kp, then which of the following
2 2
relation is numerically correct –
Then,
(a) Kp = 3P (b) P = 3Kp
2
æ ö
(a) K1 = ç 1 ÷ (b) K1 = K22 2K P 2P
çK ÷ (c) P = (d) Kp =
è 2ø 3 3
1 15. One mole of CH 3COOH and one mole of C2H5OH reacts to
(c) K1 = (d) K1 = (K2)0
K2
8. Some chemists at ISRO wished to prepare a saturated 2
produce mol of CH 3COOC 2 H 5 . The equilibrium
solution of a silver compound and they wanted it to have 3
the highest concentration of silver ion possible. Which of constant is :
the following compounds, would they use ? (a) 2 (b) + 2 (c) – 4 (d) + 4
Ksp(AgCl) = 1.8 × 10–10 16. 5 mole of NH4HS (s) start to decompose at a particular
Ksp(AgBr) = 5.0 × 10–13 temperature in a closed vessel. If pressure of NH3(g) in the
Ksp(Ag2CrO4) = 2.4 × 10–12 vessel is 2 atm, then Kp for the reaction,
(a) AgCl (b) AgBr NH4HS (s) ƒ NH3(g) + H2S (g), will be –
(c) Ag2CrO4 (d) Any of these (a) 2 (b) 4 (c) 0.4 (d) 0.8
Equilibrium 157

17. For homogeneous gaseous reaction, 26. According to Le-chatelier’s principle, adding heat to a
4NH3 + 5O2 ––––® 4NO + 6H2O ˆˆ† liquid equilibrium will cause the
solid ‡ˆˆ
The units of equilibrium constant Kc will be – (a) temperature to increase
(a) mol lit–1 (b) mol–1 lit–1 (b) temperature to decrease
2
(c) mol lit –2 (d) Kc is unitless (c) amount of liquid to decrease
18. If a is the fraction of HI dissociated at equilibrium in the (d) amount of solid to decrease.
reaction, 2 HI (g) H2 (g) + I2 (g), starting with 2 moles 27. The reaction quotient (Q) for the reaction
of HI, the total number of moles of reactants and products at ˆˆ† 2NH 3 ( g )
N 2 ( g ) + 3H 2 ( g ) ‡ˆˆ
equilibrium are
(a) 2 + 2a (b) 2 (c) 1 + a (d) 2 – a [ NH 3 ]2
is given by Q = . The reaction will proceed from
19. A solution having hydrogen ion concentration is 0.0005 g [ N 2 ][ H 2 ] 3
eqvt./litre, its pOH is :
(a) 8.2798 (b) 10.6990 right to left if
(c) 12.7854 (d) 13.3344 (a) Q = 0 (b) Q = Kc (c) Q < Kc (d) Q > Kc
20. Vapour density of the equilibrium mixture of the reaction where Kc is the equilibrium constant
SO2Cl2(g) SO2(g) + Cl2(g) is 50.0. Percent dissociation 28. A weak acid, HA is found to be 10% ionized in 0.01 M aqueous
of SO2Cl2 is : solution. Calculate the pH of a solution which is 0.1 M in HA
(a) 33.33 (b) 35.0 (c) 30.0 (d) 66.67 and 0.05 M in NaA.
(a) 5.365 (b) 6.355
21. In a gaseous reversible reaction
(c) 3.653 (d) 6.593
ˆˆ† 2NO + heat
N 2 + O 2 ‡ˆˆ
If pressure is increased then the equilibrium constant would 29. The equilibrium constant for a reaction, N2(g) + O2(g)
be : 2NO(g) is 4 × 10–4 at 2000 K. In the presence of catalyst, the
(a) unchanged equilibrium is attained 10 times faster. The equilibrium
(b) increased constant in presence of catalyst at 2000 K is :
(c) decreased (a) 10 × 10–4 (b) 4 × 10–2
(c) 4 × 10 –4 (d) 40 × 10–4
(d) sometimes increased, sometimes decreased
22. In which of the following reactions, the concentration of the 30. The solubility of PbF2 in water at 25ºC is ~ 10–3 M. What is
product is higher than the concentration of reactant at its solubility in 0.05 M NaF solution? Assume the latter to be
equilibrium? fully ionised.
(K = equilibrium constant) (a) 1.6 × 10–6 M (b) 1.2 × 10–6 M
(c) 1.2 × 10–5 M (d) 1.6 × 10–4 M
(a) A ƒ B; K = 0.001
31. The reaction A + B ‡ˆˆ ˆˆ† C + D + heat, has reached
(b) M ƒ N; K = 10
equilibrium. The reaction may be made to proceed forward
(c) X ƒ Y; K = 0.005 by
(d) R ƒ P; K = 0.01 (a) adding more C
23. For the gaseous reaction (b) adding more D
(c) decreasing the temperature
C2 H 4 + H 2 C2H6, DH = –130 kJ mol–1 carried in a
(d) increasing the temperature
closed vessel, the equilibrium concentration of C 2 H 6 can 32. Which of the following equilibria will shift to right side on
definitely be increased by : increasing the temperature
(a) increasing temperature and decreasing pressure ˆˆ† CO2 ( g ) + H 2 ( g )
(a) CO( g ) + H 2O( g ) ‡ˆˆ
(b) decreasing temperature and increasing pressure
(b) ˆˆ† 2SO3 ( g )
2SO 2 ( g ) + O 2 ( g ) ‡ˆˆ
(c) increasing temperature and pressure both
(d) decreasing temperature and pressure both
(c) ˆˆ† H 2 ( g ) + 1 (O 2 )( g )
H 2O( g ) ‡ˆˆ
24. On adding 0.1 M solution each of [Ag+], [Ba2+], [Ca2+] in a 2
Na2SO4 solution, species first precipitated is
[Ksp BaSO4 = 10–11, Ksp CaSO4 = 10–6, KspAg2SO4 = 10–5] (d) ˆˆ† 2H 2 O( g ) + 2Cl 2 ( g )
4HCl( g ) + O 2 ( g ) ‡ˆˆ
(a) Ag2SO4 (b) BaSO4 33. Solubility product of a salt AB is 1 × 10–8 in a solution in
(c) CaSO4 (d) All of these which the concentration of A+ ions is 10–3 M. The salt will
25. Which reaction is not affected by change in pressure? precipitate when the concentration of B– ions is kept
(a) between 10–8 M to 10–7 M
ˆˆ† 2HI
(a) H2 + I2 ‡ˆˆ ˆˆ† 2CO
(b) 2C + O2 ‡ˆˆ (b) between 10–7 M to 10–8 M
(c) > 10–5 M
ˆˆ† 2NH (d) PCl ‡ˆˆ
(c) N2 + 3H2 ‡ˆˆ ˆˆ† PCl + Cl
3 5 3 2 (d) < 10–8 M
EBD_7327
158 CHEMISTRY

34. Le-Chatelier principle is not applicable to 48. 100 mL of 0.04 N HCl aqueous solution is mixed with 100 mL
(a) H 2 ( g ) + I 2 ( g ) ‡ˆˆ
ˆˆ† 2HI( g ) of 0.02 N NaOH solution. The pH of the resulting solution is:
ˆˆ† FeS(s) (a) 1.0 (b) 1.7 (c) 2.0 (d) 2.3
(b) Fe(s) + S(s) ‡ˆˆ
49. Among BMe3, BCl3, BF3 and B2H6, which one will be the
(c) ˆˆ† 2NH 3 ( g )
N 2 ( g ) + 3H 2 ( g ) ‡ˆˆ best Lewis acid ?
(d) ˆˆ† 2NO( g )
N 2 ( g ) + O2 ( g ) ‡ˆˆ (a) BCl3 (b) BMe3 (c) B2H6 (d) BF3
35. A base when dissolved in water yields a solution with a 50. The value of the ionic product of water
hydroxyl ion concentration of 0.05 mol litre–1. The solution is (a) depends on volume of water
(a) basic (b) acidic (b) depends on temperature
(c) neutral (d) either (b) or (c) (c) changes by adding acid or alkali
36. Solution of 0.1 N NH4OH and 0.1 N NH4Cl has pH 9.25. (d) always remains constant
Then find out pKb of NH4OH 51. The following equilibrium is established when hydrogen
(a) 9.25 (b) 4.75 (c) 3.75 (d) 8.25 chloride is dissolved in acetic acid.
37. Which one of the following compounds is not a protonic
+
acid? HCl + CH3COOH Cl– + CH 3COOH 2
(a) SO2 (OH)2 (b) B (OH)3
The set that characterises the conjugate acid-base pairs is
(c) PO (OH)3 (d) SO (OH)2
+
38. In which of the following cases, pH is greater than 7? (a) (HCl, CH3COOH) and ( CH 2 COOH 2 , Cl–)
(a) 50 ml of 0.1 M HCl + 50 ml of 0.1 M NaCl
+
(b) 50 ml of 0.1 M H2SO4 + 50 ml of 0.2 M NaOH (b) (HCl, CH 3COOH 2 ) and (CH3COOH, Cl–)
(c) 50 ml of 0.1 M CH3COOH + 50 ml of 0.1 M KOH
+
(d) 50 ml of 0.1 M HNO3 + 50 ml of 0.1 M NH3 (c) ( CH 3COOH 2 , HCl) and (Cl–, CH3COOH)
39. Species acting as both Bronsted acid and base, is
+
(a) (HSO4)– (b) Na2CO3 (c) NH3 (d) OH– (d) (HCl, Cl–) and ( CH 3COOH 2 , CH3COOH)
40. In the equation I 2 + I - ¾
¾® I 3- , Lewis base is 52. Diacidic base is
(a) I2 (b) I– (c) I3 – (d) None (a) CH2(OH)2 (b) Ca(OH)2
41. –10
The pH of 10 M NaOH solution is nearest to: (c) CH3CH(OH)2 (d) All of these
(a) 6 (b) – 10 (c) 4 (d) 7
53. Which of the following behaves as Lewis acid and not as
42. The strongest Bronsted base is :
Bronsted acid
(a) ClO3- (b) ClO-2 (c) ClO-4 (d) ClO - (a) HCl (b) H2SO4 (c) HSO3– (d) SO3
43. The pH value for 1/1000 N-KOH solution is : 54. Water is well known amphoprotic solvent. In which chemical
(a) 3 (b) 10–11 (c) 1 (d) 11 reaction water is behaving as a base?
44. The pH of 0.1 M solution of the following salts increases in (a) ® H 3 O + + HSO 4 –
H 2SO 4 + H 2 O ¾¾
the order :
(b) ® H 3 O + + OH –
H 2 O + H 2 O ¾¾
(a) NaCl < NH 4 Cl < NaCN < HCl
(c) H2O + NH2 – ¾¾
® NH3 + OH –
(b) HCl < NH 4 Cl < NaCl < NaCN
(d) ® NH 4+ + OH –
H 2 O + NH 3 ¾¾
(c) NaCN < NH 4 Cl < NaCl < HCl
55. The pH of a 10–3 M HCl solution at 25°C if it is diluted 1000
(d) HCl < NaCl < NaCN < NH 4 Cl times, will be –
(a) 3 (b) zero (c) 5.98 (d) 6.02
45. The pH of 0.005 molar solution of H 2 SO 4 is approximately:
56. What will be the pH of a solution formed by mixing 50ml of
(a) 0.010 (b) 1 (c) 2 (d) 0.005
0.5 M HCl solution and 150ml of 0.5 M NaOH solution and
46. Which of the following has highest pH ?
300 ml H2O ?
M M (a) 13 (b) 12.7 (c) 7 (d) 11
(a) KOH (b) NaOH
4 4 57. The pH of a weak mono acidic base, neutralized upto 80%
M M with a strong acid in a dilute solution, is 7.40. The ionization
(c) NH 4 OH (d) Ca (OH ) 2
4 4 constant of the base is –
(a) 1.0 × 10–5 (b) 1.6 × 10–7
47. pH of 0.1 M NH3 aqueous solution is : ( K b = 1.8 ´10 -5 )
(c) 1.0 × 10–6 (d) None of these
(a) 11.13 (b) 12.5 (c) 13.42 (d) 11.55
Equilibrium 159

58. For the system : A(g) + 2B(g) ® C(g) 66. Given


K
a ® H O+ + F-
the equilibrium concentration is HF + H 2 O ¾¾¾ 3
A = 0.06 moles/l, B = 0.12 moles/l K
F- + H 2 O ¾¾¾
b ® HF + OH -
C = 0.216 moles/l
The Keq for the reaction is Which of the following reaction is correct
(a) 250 (b) 416 1
(a) Kb = Kw (b) Kb =
(c) 4 × 10–3 (d) 125 Kw
59. At 25°C, the dissociation constant of a base, BOH, is
Ka
1.0 ´ 10 -12 . The concentration of hydroxyl ions in (c) Ka × Kb = Kw (d) = Kw
Kb
0.01 M aqueous solution of the base would be
(a) 1.0 ´ 10- 5 mol L-1 (b) 1.0 ´ 10-6 mol L-1 67. At 298K a 0.1 M CH3COOH solution is 1.34% ionized. The
(c) 2.0 ´ 10-6 mol L-1 (d) 1.0 ´ 10-7 mol L-1 ionization constant Ka for acetic acid will be
60. Which of the following pKa value represents the strongest (a) 1.82 × 10– 5 (b) 18.2 × 10 –5
acid ? (c) 0.182 × 10 –5 (d) None of these
(a) 10–4 (b) 10–8 (c) 10–5 (d) 10–2 68. If the solubility of PbCl2 at 25°C is 6.3 × 10–3 mole/litre, its
61. The product of ionic concentration in a saturated solution solubility product at that temperature is
of an electrolyte at a given temperature is constant and is (a) (6.3 × 10–3) × (12.6 × 10–3)2
known as (b) (6.3 × 10–3)2 × (12.6 × 10–3)
(a) Ionic product of the electrolyte (c) (6.3 × 10–3) × (12.6 × 10–3)
(b) Solubility product (d) (12.6 × 10–3) × (12.6 × 10–3)
(c) Ionization constant 69. Which of the following is most soluble ?
(d) Dissociation constant (a) Bi2S3 (Ksp= 1 × 10–17) (b) MnS (Ksp= 7 × 10–16)
62. If degree of dissociation of pure water at 100°C is (c) CuS(Ksp= 8 × 10–37) (d) Ag2S (Ksp= 6 × 10–51).
1.8 × 10–8, then the dissociation constant of water will be 70. The pKa of acetylsalicylic acid (aspirin) is 3.5. The pH of
(density of H2O = 1 gm/cc) gastric juice in human stomach is about 2 – 3 and the pH in
the small intestine is about 8. Aspirin will be
(a) 1 × 10–12 (b) 1 × 10–14
(a) unionised in the small intestine and in the stomach
(c) 1.8 × 10–12 (d) 1.8 × 10–14
(b) completely ionised in the small intestine and in the
63. The dissociation constant of two acids HA1 and HA2 are
stomach
3.14 × 10– 4 and 1.96 × 10– 5 respectively. The relative strength
(c) ionised in the stomach and almost unionised in the
of the acids will be approximately
small intestine
(a) 1 : 4 (b) 4 : 1
(d) ionised in the small intestine and almost unionised in
(c) 1 : 16 (d) 16 : 1
the stomach
64. Ionisation of weak acid can be calculated by the formula
71. Which of the following solution cannot act as a buffer?
Ka 100 (a) NaH2PO4 + H3PO4
(a) 100 (b) ( pK a - pH ) (b) CH3COOH+CH3COONa
c 1 + 10
(c) HCl + NH4Cl
(c) Both (a) and (b) (d) None of these
(d) H3PO4 + NaH2PO4
65. ˆˆ† CH3COOC2 H5 + H 2 O
CH 3 COOH + C 2 H 5 OH ‡ˆˆ 72. Which one has maximum solubility in liquid CCl4?
In the above reaction one mole each of acetic acid and alcohol (a) NaCl (b) Cl2 (c) I2 (d) Br2
are heated in the presence of little conc. H2SO4. On 73. The solubility of AgCl in 0.2 M NaCl solution is : (Ksp of
equilibrium being attained AgCl = 1.20 × 10–10)
(a) 1 mole of ethyl acetate is formed (a) 6.0 × 10–10 M (b) 0.2 M
(b) 2 moles of ethyl acetate are formed (c) 1.2 × 10–10 M (d) 0.2 × 10–10 M
74. If s and S are respectively solubility and solubility product
1 of a sparingly soluble binary electrolyte then :
(c) mole of ethyl acetate is formed
3 (a) s = S (b) s = S2

2 1
(d) mole of ethyl acetate is formed (c) s = S1/ 2 (d) s= S
3 2
EBD_7327
160 CHEMISTRY

75. A buffer solution is prepared by mixing 10 ml of 1.0 M 85. If the equilibrium constant of the reaction of weak acid HA
CH3COOH and 20 ml of 0.5 M CH3COONa and then diluted with strong base is 107, then pOH of the aqueous solution
to 100 ml with distilled water. If pKa of CH3COOH is 4.76, of 0.1M NaA is
what is the pH of the buffer solution ? (a) 8 (b) 10 (c) 4 (d) 5
(a) 5.8 (b) 4.34 (c) 5.21 (d) 4.76 86. When equal volumes of the following solutions are mixed,
76. Solubility product of BaCl2 is 4 × 10–9 moles/litre. Its precipitation of AgCl (Ksp = 3 × 10–10) will occur with –
solubility would be : (a) 10–4 M AgNO3 and 10–5 M KCl
(a) 1 × 10–27 (b) 1 × 10–3 (b) 10–5 M AgNO3 and 10–5 M KCl
(c) 1 × 10–7 (d) 1 × 10–2 (c) 10–4 M AgNO3 and 10–4 M KCl
77. 100 ml solution of an acid of pH = 1, is mixed with 100 ml (d) Both (a) and (c)
solution of the same acid of pH = 2. The hydrogen ion 87. Solubility of BaF2 in a solution of Ba ( NO 3 ) 2 will be
concentration of the mixture will be :
represented by
(a) 5.5 × 10–2 (b) 10–2
(c) 10 –3 (d) 10–4 (a) [ Ba 2+ ] (b) [ NO 3 ]-2
78. The degree of hydrolysis in hydrolytic equilibrium
1 -
A - + H 2O HA + OH - (c) [F ] (d) [ Ba ]2
2
at salt concentration of 0.001 M is : ( K a = 1´10-5 ) 88. A buffer solution is prepared by mixing 0.1 M ammonia and
(a) 1 ´ 10 -3 (b) 1 ´ 10 -4 1.0 M ammonium chloride. At 298 K, the pKb of NH4OH is
5.0. The pH of the buffer is
(c) 5 ´ 10 - 4 (d) 1 ´ 10 -6 (a) 10.0 (b) 9.0 (c) 6.0 (d) 8.0
79. A litre of solution is saturated with AgCl. To this solution if 89. Which of the following is conjugate base of [C2H5NH3]+
(a) C2H5NH2 (b) C2N+H4(OH)–
1.0 ´ 10 -4 mole of solid NaCl is added, what will be the –
(c) [C2H2NH] (d) None of these
[Ag + ] , assuming no volume change? 90. A weak acid, HA, has a Ka of 1.0 × 10–5. If 0.1 mole of this
(a) More (b) Less (c) Equal (d) Zero acid dissolved in one litre of water, the percentage of acid
dissociated at equilbrium is closest to
80. The value of Kc for the reaction
(a) 1.0% (b) 99.9% (c) 0.1% (d) 99.0%
2NO(g) ® N2 (g) + O2 (g) is 1×8 × 10– 2. Calculate the value of
91. Calculate the pOH of a solution at 25°C that contains
K C for the reaction 1× 10– 10 M of hydronium ions, i.e. H3O+.
1 1 (a) 4.0 (b) 9.0 (c) 1.0 (d) 7.0
N 2(g) + O 2(g) ¾
¾® NO( g ) 92. If the concentration of OH– ions in the reaction
2 2
(a) 7×6 (b) 8×5 (c) 6×5 (d) 2 ˆˆ† Fe3+ (aq) + 3OH– (aq) is
Fe(OH) (s) ‡ˆˆ
3
81. The solubility of CaF2 in pure water is 2.3×10–4 mol dm–3. 1
decreased by times, then equilibrium concentration of
Its solubility product will be : 4
Fe3+ will increase by :
(a) 4.6 × 10–4 (b) 4.6 × 10–8
–12 (a) 8 times (b) 16 times (c) 64 times (d) 4 times
(c) 6.9 × 10 (d) 4.9 × 10–11
93. Equimolar solutions of the following were prepared in water
82. Amongst the following hydroxides, the one which has the
separately. Which one of the solutions will record the highest
lowest value of Ksp at ordinary temperature (about 25ºC) is :
pH ?
(a) Mg(OH)2 (b) Ca(OH)2
(a) SrCl2 (b) BaCl2
(c) Ba(OH)2 (d) Be(OH)2
(c) MgCl2 (d) CaCl2
83. In qualitative analysis, in III group NH4Cl is added before
94. The value of equilibrium constant of the reaction
NH4OH because
(a) to increase the concentration of NH4+ions 1 1
HI ( g ) ƒ H ( g ) + I 2 is 8.0
(b) to increase concentration of Cl– ions 2 2 2
(c) to reduce the concentration of OH– ions The equilibrium constant of the reaction
(d) to increase concentration of OH– ions H 2 ( g ) + I2 ( g ) ƒ 2HI( g ) will be:
84. The buffering action of an acidic buffer is maximum when its
pH is equal 1 1 1
(a) (b) (c) 16 (d)
(a) 5 (b) 7 (c) 1 (d) pKa 16 64 8
Equilibrium 161

95. Which of the following molecules acts as a Lewis acid ? 106. The pH of a 0.1 molar solution of the acid HQ is 3. The
(a) (CH3)2 O (b) (CH3)3 P value of the ionization constant, Ka of the acid is :
(c) (CH3)3 N (d) (CH3)3 B (a) 3 × 10–1 (b) 1 × 10–3 (c) 1 × 10–5 (d) 1 × 10–7
96. The ionization constant of ammonium hydroxide is 1.77 × 107. The pKa of a weak acid (HA) is 4.5. The pOH of an aqueous
10–5 at 298 K. Hydrolysis constant of ammonium chloride is: buffer solution of HA in which 50% of the acid is ionized is
(a) 6.50 × 10– 12 (b) 5.65 × 10–13
(c) 5.65 × 10–12 (d) 5.65 × 10–10 (a) 7.0 (b) 4.5 (c) 2.5 (d) 9.5
97. In which of the following equilibrium Kc and Kp are not 108. In a saturated solution of the sparingly soluble strong
equal? electrolyte AgIO3 (molecular mass = 283) the equilibrium
(a) 2 NO(g ) ƒ N 2 (g ) + O 2 (g )
ˆˆ† Ag + (aq) + IO3- (aq) . If the
which sets is AgIO3(s) ‡ˆˆ
(b) SO 2 (g )+NO 2 (g ) ƒ SO3 (g ) + NO(g )
solubility product constant Ksp of AgIO 3 at a given
(c) H 2 (g )+I2 (g ) ƒ 2 HI(g ) temperature is 1.0 × 10–8 , what is the mass of AgIO3
(d) 2C(s )+O 2 (g ) ƒ 2 CO 2 (g ) contained in 100 ml of its saturated solution?
98. In a buffer solution containing equal concentration of B– (a) 1.0 × 10– 4 g (b) 28.3 × 10–2 g
–3
(c) 2.83 × 10 g (d) 1.0 × 10–7 g.
and HB, the Kb for B– is 10–10. The pH of buffer solution is :
(a) 10 (b) 7 (c) 6 (d) 4 109. Four species are listed below :
99. Which one of the following molecular hydrides acts as a i. HCO3– ii. H3 O+ iii. HSO4– iv. HSO3F
Lewis acid?
Which one of the following is the correct sequence of their
(a) NH3 (b) H2O (c) B2H6 (d) CH4
100. Which of the following is least likely to behave as Lewis acid strength?
base ? (a) iv < ii < iii < i (b) ii < iii < i < iv
(a) H2O (b) NH3 (c) BF3 (d) OH– (c) i < iii < ii < iv (d) iii < i < iv < ii
101. pH of a saturated solution of Ba(OH)2 is 12. The value of 110. The pKa of a weak acid, HA, is 4.80. The pKb of a weak base,
solubility product (Ksp)of Ba(OH)2 is : BOH, is 4.78. The pH of an aqueous solution of the
(a) 3.3 × 10– 7 (b) 5.0 × 10–7 correspondng salt, BA, will be
(c) 4.0 × 10 –6 (d) 5.0 × 10–6 (a) 9.58 (b) 4.79 (c) 7.01 (d) 9.22
102. For the following three reactions a, b and c, equilibrium 111. Solid Ba(NO3)2 is gradually dissolved in a 1.0 × 10– 4 M
constants are given: Na2CO3 solution. At what concentration of Ba2+ will a
(i) CO( g ) + H 2O( g ) ƒ CO 2 ( g ) + H 2 ( g ); K1 precipitate begin to form? (Ksp for BaCO3 = 5.1 × 10–9)
(a) 5.1 × 10 –5 M (b) 8.1 × 10 –8 M
(ii) CH 4 ( g ) + H 2O( g ) ƒ CO( g ) + 3H 2 ( g );K 2 (c) 8.1 × 10 M –7 (d) 4.1 × 10 –5 M
(iii) CH 4 ( g ) + 2H 2O( g ) ƒ CO2 ( g ) + 4H 2 ( g );K 3 112. In aqueous solution the ionization constants for carbonic
acid are
(a) K1 K 2 = K3 (b) K 2 K3 = K1 K1 = 4.2 × 10–7 and K2 = 4.8 × 10–11.
Select the correct statement for a saturated 0.034 M solution
(c) K3 = K1 K2 (d) K3 .K 23 = K12 of the carbonic acid.
103. Three reactions involving H2PO4 – are given below :
(a) The concentration of CO32 - is 0.034 M.
(i) H3PO4 + H2O® H3O+ + H2PO4–
(b) The concentration of CO32- is greater than that of
(ii) H2PO4– + H2O ® HPO42– + H3O+
(iii) H2PO4– + OH– ® H3PO4 + O2– HCO3- .
In which of the above does H 2 PO4- act as an acid ? (c) The concentrations of H+ and HCO3- are approximately
(a) (ii) only (b) (i) and (ii) (c) (iii) only(d) (i) only equal.
104. The Ksp for Cr(OH)3 is 1.6 × 10–30. The solubility of this (d) The concentration of H+ is double that of CO32 - .
compound in water is : 113. Solubility product of silver bromide is 5.0 × 10–13. The
(a) 4
1.6 ´ 10 -30 (b) 4
1.6 ´ 10 -30 / 27 quantity of potassium bromide (molar mass taken as 120
g mol–1) to be added to 1 litre of 0.05 M solution of silver
(c) 1.6 ´ 10-30/ 27 (d) 1.6 ´ 10 -30 nitrate to start the precipitation of AgBr is
105. The equilibrium constant (K c ) for the reaction (a) 1.2 × 10–10 g (b) 1.2 × 10–9 g
N2(g) + O2(g) ® 2NO(g) at temperature T is 4 × 10–4. The (c) 6.2 × 10–5 g (d) 5.0 × 10–8 g
value of Kc for the reaction 114. At 25°C, the solubility product of Mg(OH)2 is 1.0 × 10–11.
1 1 At which pH, will Mg2+ ions start precipitating in the form
NO( g ) ® N 2 ( g ) + O 2 ( g ) at the same temperature is:
2 2 of Mg(OH)2 from a solution of 0.001 M Mg2+ ions?
(a) 0.02 (b) 2.5 × 102 (c) 4 × 10–4 (d) 50.0 (a) 9 (b) 10 (c) 11 (d) 8
EBD_7327
162 CHEMISTRY

115. An acid HA ionises as (c) W + X ƒ 2Y + 3Z; DH = + 92 kJ mol–1


ˆˆ† H + A + - (d) S + T ƒ E + F; DH = – 80 kJ mol–1
HA ‡ˆˆ
121. In the decomposition reaction
The pH of 1.0 M solution is 5. Its dissociation constant
would be : AB5 ( g ) ƒ AB3 ( g ) + B2 ( g ) , at equilibrium in a 10 litre
closed vessel at 227°C, 2 moles of AB3, 5 moles of B2 and 4
(a) 5 (b) 5 ´ 10 -8 (c) 1 ´ 10-5 (d) 1 ´ 10-10
moles of AB5, are present. The equilibrium constant Kc for
116. A vessel at 1000 K contains CO2 with a pressure of 0.5 atm. the formation of AB5(g) is
Some of the CO2 is converted into CO on the addition of (a) 0.25 (b) 4.0
graphite. If the total pressure at equilibrium is 0.8 atm, the (c) 0.04 (d) 2.5
value of K is : 122. Which one of the following will decrease the pH of
(a) 1.8 atm (b) 3 atm (c) 0.3 atm (d) 0.18 atm 50 mL of 0.01 M hydrochloric acid?
117. 2.5 ml of (2/5) M weak monoacidic base (Kb = 1 × 10–12 at (a) Addition of 50 mL of 0.01 M HCl
25°) is titrated with (2/15) M HCl in water at 25°C. The (b) Addition of 50 mL of 0.002 M HCl
concentration of H+ at equivalence point is (Kw = 1 × 10–14 (c) Addition of 150 mL of 0.002 M HCl
at 25°C) (d) Addition of 5 mL of 1 M HCl
(a) 3.7 × 10–14 M (b) 3.2 × 10–7 M 123. The equilibrium constant for the reaction
(c) 3.2 × 10 M–2 (d) 2.7 × 10–2 M A2 2A at 500 K and 700 K are 1 × 10–10 and 1 × 10–5
118. Solubility product constant (Ksp) of salts of types MX, MX2 respectively. The given reaction is [1996]
and M3X at temperature T are 4.0 × 10–8, 3.2 × 10–14 and (a) exothermic (b) slow
2.7 × 10–15, respectively. Solubilities (mol dm–3) of the salts (c) endothermic (d) fast
at temperature 'T' are in the order – 124. What is the H+ ion concentration of a solution prepared by
(a) MX > MX2 > M3X (b) M3X > MX2 > MX dissolving 4 g of NaOH (Atomic weight of Na = 23 amu) in
(c) MX2 > M3X > MX (d) MX > M3X > MX2 1000 ml? [1999]
119. How many litres of water must be added to 1 litre an aque- (a) 10–10 M (b) 10–4 M
ous solution of HCl with a pH of 1 to create an aqueous
(c) 10–1 M (d) 10–13 M
solution with pH of 2 ?
125. A physician wishes to prepare a buffer solution of pH =
(a) 0.1 L (b) 0.9 L (c) 2.0 L (d) 9.0 L 3.58 that efficiently resists a change in pH yet contains
120. Five gaseous homogenous equilibrium reactions are given only small concentrations of the buffering agents. Which
below. Choose the reaction in which both increase in one of the following weak acids together with its sodium
pressure and increase in temperature favour the formation salt would be the best to use ? [1997]
of products. (a) m-chlorobenzoic acid (pKa = 3.98)
(a) 2A + B ƒ C + D; DH = – 78 kJ mol–1 (b) p-chlorocinnamic acid (pKa = 4.41)
(b) 2M + 3N ƒ P + 2Q; DH = + 105 kJ mol–1 (c) 2, 5-dihydroxy benzoic acid (pKa = 2.97)
(d) Acetoacetic acid (pKa = 3.58)

1. In a closed system : A(s) 2B(g) + 3 C(g) , if the partial 3. Solubilities of three sparingly soluble salts XY (Ksp), XY2
pressure of C is doubled, then partial pressure of B will be: (K'sp) and X2Y3 (K''sp) are equal in water. What will be the
(a) two times the original value correct order of their solubility products –
(b) one-half of its original value (a) Ksp < K'sp < K''sp (b) Ksp < K''sp < K'sp
(c) K''sp < K'sp < Ksp (d) K''sp < Ksp < K'sp
1 4. Three sparingly soluble salts A2B, AB and AB3 have the
(c) times the original value
2 2 same values of solubility products (Ksp ). In a saturated
solution the correct order of their solubilities is
(d) 2 2 times its original value (a) AB2 > AB > A2B (b) AB3 > A2B > AB
2. The dissociation constants of two weak acids are K1 and (c) AB > AB3 > A2B (d) AB > A2B > AB3
K2. The relative strength of two acids is given by : 5. Solid AgNO3 is slowly added to a solution containing each
–3/ 2 +1/ 2 of 0.01 M NaCl and 0.001 M NaBr. What will be the
é K1 ù é K1 ù
(a) (b) ê ú concentration of Cl– ions in solution when AgBr will just
ê ú
ë K2 û ë K2 û start to precipitate ?
Ksp (AgBr) = 3.6 × 10–13, Ksp (AgCl) = 1.8 × 10–10.
K1 + K 2 K1 (a) 1.8 × 10–7 (b) 3.6 × 10–10
(c) (d)
K 2 + K1 K2 (c) 0.01 (d) 2 × 10–4
Equilibrium 163

6. If the Kp for the equilibrium, to yield NH 3 and H 2S gases in the flask. When the
M.5H2O(s) ƒ M.3H2O(s) + 2H2O(g) decomposition reaction reaches equilibrium, the total
is 1 × 10–4. Then M.5H2O(s) will show efflorescence when it pressure in the flask rises to 0.84 atm? The equilibrium
is exposed to an atmosphere where vapour pressure of water
is – constant for NH 4 HS decomposition at this temperature is
(a) more than 10–2 atm (b) below 10–2 atm (a) 0.11 (b) 0.17 (c) 0.18 (d) 0.30
(c) more than 10 atm–4 (d) below 10–4 atm 14. For the reversible reaction, N2(g) + 3H2(g) 2NH3(g) at
7. pH of a solution containing 0.3 M HX and 0.1M X–
500°C, the value of Kp is 1.44 ´10 -5 when partial pressure
(Kb for X– = 1.0 × 10–5) is –
(a) 5 + log 3(b) 5 – log 3 (c) 9 – log 3(d) 9 + log 3 is measured in atmospheres. The corresponding value of
8. For which one of the following systems at equilibrium, at Kc, with concentration in mole litre–1, is
constant temperature will the doubling of the volume cause 1.44 ´10 -5 1.44 ´ 10 -5
a shift to the right? (a) (b)
(0.082 ´ 500)- 2 (8.314 ´ 773)- 2
(a) H2(g)+Cl2(g) 2HCl(g)
(b) 2CO(g)+O2 (g) 2CO2(g) 1.44 ´10 -5 1.44 ´10-5
(c) (d)
(c) N2(g)+3H2(g) 2NH3(g) (0.082 ´ 773)2 (0.082´ 773)-2
15. Highest pH (14) is given by
(d) PCl5 ( g ) ƒ PCl3 ( g ) + Cl 2 ( g )
(a) 0.1 M H2SO4 (b) 0.1 M NaOH
9. If K1 and K2 are the respective equilibrium constants for the
(c) 1 N NaOH (d) 1 N HCl
two reactions
16. A physician wishes to prepare a buffer solution of pH = 3.58
XeF6 (g) + H2O (g) XeOF4 (g) + 2HF (g)
that efficiently resists a change in pH yet contains only
XeO4 (g) + XeF6 (g) XeOF4 (g) + XeO3F 2 (g)
small concentrations of the buffering agents. Which one of
the equilibrium constant of the reaction
the following weak acids together with its sodium salt would
XeO4 ( g ) + 2HF( g ) ƒ XeO3F2 ( g ) + H 2O( g ) will be be the best to use ?
(a) K1/(K2)2 (b) K1 . K2 (a) m-chlorobenzoic acid (pKa = 3.98)
(c) K1/K2 (d) K2/K1 (b) p-chlorocinnamic acid (pKa = 4.41)
10. For the reaction : 2 BaO 2( s) 2BaO( s) + O 2( g) ;
(c) 2, 5-dihydroxybenzoic acid (pKa = 2.97)
DH = +ve. In equilibrium condition, pressure of O2 is
(d) Acetoacetic acid (pKa = 3.58)
dependent on
(a) mass of BaO2 17. The dissociation constant of monobasic acids A, B and C
(b) mass of BaO are 10–4, 10–6 and 10–10 respectively. The concentration of
(c) temperature of equilibrium each is 0.1 M. Which of the following has been arranged in
(d) mass of BaO2 and BaO both order of increasing pH?
11. For the reaction : 2NO2(g) 2NO( g ) + O 2 ( g ), (a) A < B < C (b) C < A < B
(Kc = 1.8 × 10–6 at 184ºC and R = 0.0831 kJ/ mol. K) (c) B < C < A (d) B < A » C
18. In HS–, I–, RNH2 and NH3, order of proton accepting
When K p and K c are compared at 184°C, it is found that
tendency will be
(a) Whether Kp is greater than, less than or equal to Kc (a) I– > NH3 > RNH2 > HS– (b) HS– > RNH2 > NH3 > I–
depends upon the total gas pressure
(c) RNH2 > NH3 > HS– > I– (d) NH3 > RNH2 > HS– > I–
(b) Kp = Kc
(c) Kp is less than Kc 19. Which one of the following orders of acid strength is correct?
(d) Kp is greater than Kc (a) RCOOH > HC º CH > HOH > ROH
12. For the reactions (b) RCOOH > ROH > HOH > HC º CH
A B Kc =2 (c) RCOOH > HOH > ROH > HC º CH
B C Kc =4 (d) RCOOH > HOH > HC º CH > ROH
20. The rapid change of pH near the stoichiometric point of an
C D Kc =6 acid-base titration is the basis of indicator detection. pH of
K c for the reaction A D is the solution is related to ratio of the concentrations of the
conjugate acid (HIn) and base (In–) forms of the indicator
2´ 4
(a) 2 × 4 × 6 (b) by the expression
6
(a) [ In - ]
4´ 6 log = pK In - pH
(c) 2 + 4 + 6 (d) [ HIn ]
2
13. An amount of solid NH4HS is placed in a flask already [ HIn ]
containing ammonia gas at a certain temperature and 0.50 (b) log = pK In - pH
[ In - ]
atm pressure. Ammonium hydrogen sulphide decomposes
EBD_7327
164 CHEMISTRY

27. At 25ºC the pH of 0.01 M KOH is 12. If the temperature of


[ HIn ]
(c) log = pH - pK In this solution is raised to 50ºC without changing the volume,
[ In - ] which of the following is correct ?
(a) Both pH and pOH will remain constant
[ In - ] (b) pH will decrease while pOH willl remain constant
(d) log = pH - pK In
[ HIn ] (c) pH will increase while pOH will remain constant
21. What is the correct relationship between the pHs of (d) pH will increase while pOH will decrease.
isomolar solutions of sodium oxide (pH1 ), sodium 28. Equilibrium constant for the reaction,
sulphide (pH2 ), sodium selenide (pH3 ) and sodium CaCO3(s) ƒ CaO(s) + CO2(g), follows the equation
telluride (pH4)? 8400
(a) pH1 > pH2 > pH3 > pH4 ln Kp = 7 – , where T = absolute temperature. Find the
T
(b) pH1 > pH2 » pH3 > pH4 equilibrium temperature if decomposition of CaCO 3
(c) pH1 < pH2 < pH3 < pH4 produces CO 2 gas having partial pressure equal to
(d) pH1 < pH2 < pH3 » pH4 atmospheric pressure.
22. A solution of NH 4Cl and NH 3 has pH = 8.0. Which of the (a) 1200°C (b) 927 K (c) 927°C (d) None
following hydroxides may be precipitated when this solution 29. For a certain reaction, rate = k × [H+]n. If pH of reaction
is mixed with equal volume of 0.2 M of metal ion. changes from two to one, the rate becomes 100 times of its
value at pH = 2, the order of reaction is –
(a) Ba(OH)2 (K sp = 1.1 ´ 10-4 ) (a) 1 (b) 2 (c) 0 (d) 3
30. Which of the following solutions will have pH close
(b) Mg(OH)2 (K sp = 3.5 ´ 10-4 ) to 1.0 ?
(a) 100 ml of 0.1 M HCl + 100 ml of 0.1 M NaOH
(c) Fe(OH) 2 (K sp = 8.1´ 10-16 ) (b) 55 ml of 0.1 M HCl + 45 ml of 0.1 M NaOH
(c) 75 ml of 0.2 M HCl + 25 ml of 0.2 M NaOH
(d) Ca (OH) 2 (K sp = 2.1 ´ 10 -5 ). (d) 10 ml of 0.1 M HCl + 90 ml of 0.1 M NaOH
23. For a concentrated solution of a weak electrolyte AxBy of 31. At certain temperature Kw for water 4.0 × 10–14. Which of
concentration ‘c’, the degree of dissociation ‘a’ is given as the following is wrong for pure water at this temperature?
(a) pH = 6.699 ; water is acidic
(a) a = K eq / c( x + y) (b) pH = 6.699 ; water is neutral
(c) pOH = 6.699 ; water is neutral
(b) a = K eq c /( xy) (d) pH + pOH = 13.398 ; water is neutral
32. Ksp of Ca(OH)2 is 4.0 × 10–6. At what minimum pH, Ca2+
(c) a = (Keq / c x+ y -1x x y y )1/( x+ y) ions start precipitating in 0.01 M CaCl2?
(a) 12 – log 2 (b) 12 + log 2
(d) a = ( Keq / cxy) (c) 2 – log 2 (d) 2 + log 2
24. The pH of a solution containing equimolar amounts of 33. The following equilibrium constants are given:
CH3COOH (pKa = 4.74 at 25ºC) and sodium acetate at
50ºC is : ˆˆ† 2NH3 ; K1
N 2 + 3H 2 ‡ˆˆ
(a) 4.74 (b) <4.74
(c) >4.74 (d) none of these ˆˆ† 2NO; K 2
N 2 + O2 ‡ˆˆ
25. Two samples of CH3 COOH each of 10gm were taken
1
separately in two vessels containing water of 6 litre and 12 ˆˆ† H 2O; K3
H 2 + O2 ‡ˆˆ
litre respectively at 27°C. The degree of dissociation of 2
CH3COOH will be – The equilibrium constant for the oxidation of NH3 by oxygen
(a) More in 12 litre vessel to give NO is
(b) More in 6 litre vessel K 2 K32 K 22 K3
(c) Equal in both vessels (a) (b)
K1 K1
(d) Half in 6 litre vessel than in 12 litre vessel
26. Consider the following equilibrium at 25°C, K1 K 2 K 2 K33
2NO(g) ƒ N2(g) + O2(g) ; K1 = 4 × 1030 mol litre–1 and (b) (d)
K3 K1
1 34. Equal volumes of three acid solutions of pH 3, 4 and 5 are
NO(g) + Br (g) ƒ NOBr(g) ; K2 = 1.4 mol–1/2 litre1/2 mixed in a vessel. What will be the H+ ion concentration in
2 2
The value of Kc for the reaction (at the same temperature) the mixture ?
(a) 1.11 × 10–4 M (b) 3.7 × 10–4 M
1 1 1 (c) 3.7 × 10 M– 3 (d) 1.11× 10–3 M
N2(g) + O2(g) + Br2(g) ƒ NOBr(g) is
2 2 2 35. The dissociation equilibrium of a gas AB2 can be represented
(a) 3.5 × 10–31 (b) 2.8 × 1015 as :
(c) 7.0 × 10–16 (d) 5.6 × 1030
ˆˆ† 2AB( g ) + B2 ( g )
2AB2 ( g ) ‡ˆˆ
Equilibrium 165

The degree of dissociation is ‘x’ and is small compared to 1. 43. Buffer solutions have constant acidity and alkalinity because
The expression relating the degree of dissociation (x) with (a) these give unionised acid or base on reaction with
equilibrium constant Kp and total pressure P is : added acid or alkali.
(a) (2Kp/P) (b) (2Kp/P) 1/3 (b) acids and alkalies in these solutions are shielded from
(c) (2Kp/P) 1/2 (d) (Kp/P) attack by other ions.
36. The values of Kp1 and Kp2 for the reactions (c) they have large excess of H+ or OH– ions
(d) they have fixed value of pH
ˆˆ† Y + Z
X ‡ˆˆ ...(1) 44. Given that the equilibrium constant for the reaction
ˆˆ† 2B
and A ‡ˆˆ ...(2) 2SO2(g) + O2(g) ‡ˆˆ ˆˆ† 2SO (g) has a value of 278 at a
3
are in the ratio of 9 : 1. If degree of dissociation of X and A be particular temperature. What is the value of the equilibrium
equal, then total pressure at equilibrium (1) and (2) are in the constant for the following reaction at the same temperature ?
ratio :
(a) 3 : 1 (b) 1 : 9 (c) 36 : 1 (d) 1 : 1 ˆˆ† SO2 ( g ) + 1 O2 ( g )
SO3 ( g ) ‡ˆˆ
37. The dissociation constants for acetic acid and HCN at 25°C 2
(a) 1.8 × 10–3 (b) 3.6 × 10–3
are 1.5 × 10–5 and 4.5 × 10–10 respectively. The equilibrium
(c) 6.0 × 10–2 (d) 1.3 × 10–5
constant for the equilibrium
45. Given the reaction between 2 gases represented by A2 and
CN– + CH3COOH ƒ HCN + CH3COO– would be: B2 to give the compound AB(g).
(a) 3.0 × 10–5 (b) 3.0 × 10–4 A2(g) + B2(g) ‡ˆˆˆˆ† 2 AB(g).
(c) 3.0 × 104 (d) 3.0 × 105 At equilibrium, the concentration
38. If pH of a saturated solution of Ba (OH)2 is 12, the value of of A2 = 3.0 × 10–3 M
its K(sp) is : of B2= 4.2 × 10–3 M
(a) 4.00 × 10–6M3 (b) 4.00 × 10–7 M3 of AB = 2.8 × 10–3 M
(c) 5.00 × 10 M –6 3 (d) 5.00 × 10–7 M3 lf the reaction takes place in a sealed vessel at 527°C, then
+ the value of Kc will be :
39. What is [H ] in mol/L of a solution that is 0.20 M in
(a) 2.0 (b) 1.9 (c) 0.62 (d) 4.5
CH3COONa and 0.10 M in CH3COOH? Ka for CH3COOH =
46. The first and second dissociation constants of an acid H2 A
1.8 × 10–5.
are 1.0 × 10–5 and 5.0 × 10–10 respectively. The overall
(a) 3.5 × 10–4 (b) 1.1 × 10–5
dissociation constant of the acid will be
(c) 1.8 × 10–5 (d) 9.0 × 10–6
(a) 0.2 × 105 (b) 5.0 × 10–5
40. The reaction 2A( g ) + B( g ) ƒ 3C( g ) + D( g ) is began with (c) 5.0 × 10 15 (d) 5.0 × 10–15.
the concentrations of A and B both at an initial value of 1.00
M. When equilibrium is reached, the concentration of D is DIRECTIONS for Qs. 47 to 50 : These are Assertion-Reason
measured and found to be 0.25 M. The value for the type questions. Each of these question contains two statements:
equilibrium constant for this reaction is given by the Statement-1 (Assertion) and Statement-2 (Reason). Answer these
expression questions from the following four options.
(a) [(0.75)3 (0.25)] ¸ [(0.75)2 (0.25)] (a) Statement- 1 is True, Statement-2 is True, Statement-2 is a
(b) [(0.75)3 (0.25)] ¸ [(1.00)2 (1.00)] correct explanation for Statement -1
(c) [(0.75)3 (0.25)] ¸ [(0.50)2 (0.75)] (b) Statement -1 is True, Statement -2 is True, Statement-2 is
NOT a correct explanation for Statement - 1
(d) [(0.75)3 (0.25)] ¸ [(0.50)2 (0.25)]
(c) Statement - 1 is True, Statement- 2 is False
41. A buffer solution is prepared in which the concentration of (d) Statement -1 is False, Statement -2 is True
NH3 is 0.30M and the concentration of NH4+ is 0.20 M. If 47. Statement-1 : In a titration of weak acid and NaOH, the pH
the equilibrium constant, Kb for NH3 equals 1.8 × 10–5, what at half equivalence point is pKa.
is the pH of this solution ? (log 2.7 = 0.433). Statement-2 : At half equivalence point, it forms an acidic
(a) 9.08 (b) 9.43 (c) 11.72 (d) 8.73 buffer and the buffer capacity is maximum where [acid] = [salt]
42. In qualitative analysis, the metals of Group I can be separated [2009]
from other ions by precipitating them as chloride salts. A 48. Statement-1 : [Al(H2 O) 6 ]3+ is a stronger acid than
solution initially contains Ag+ and Pb2+ at a concentration [Mg(H2O)6]2+.
of 0.10 M. Aqueous HCl is added to this solution until the Statement-2 : Size of [Al(H2 O) 6 ]3+ is smaller than
Cl– concentration is 0.10 M. What will the concentrations of [Mg(H2O)6]2+ and possesses more effective nuclear charge.
Ag+ and Pb2+ be at equilibrium? [2010]
(Ksp for AgCl = 1.8 × 10–10, Ksp for PbCl2 = 1.7 × 10–5) 49. Statement-1 : Reaction quotient is defined in the same way
(a) [Ag+] = 1.8 × 10–7 M ; [Pb2+] = 1.7 × 10–6 M as equilbrium constant at any stage of the reaction.
(b) [Ag+] = 1.8 × 10–11 M ; [Pb2+] = 8.5 × 10–5 M Statement-2 : If Qc (reaction quotient) < KC (equilibrium
(c) [Ag+] = 1.8 × 10–9 M ; [Pb2+] = 1.7 × 10–3 M constant) reaction moves in direction of reactants. [2011]
50. Statement-1 : KOH is more soluble in water than NaOH.
(d) [Ag+] = 1.8 × 10–11 M ; [Pb2+] = 8.5 × 10–4 M
Statement-2 : NaOH is a stronger base than KOH. [2011]
EBD_7327
166 CHEMISTRY

Exemplar Questions (a) DH > 0 for the reaction


(b ) DH < 0 for the reaction
1. We know that the relationship between Kc and Kp is
Kp = Kc (RT)Dn (c ) DH = 0 for the reaction
What would be the value of Dn for the reaction? (d) The sign of DH cannot be predicted on the basis of
this information
ˆˆ† NH 3 ( g ) + HI ( g )
NH 4 Cl ( s ) ‡ˆˆ
7. The pH of neutral water at 25°C is 7.0. As the temperature
(a) 1 (b) 0.5 increases, ionisation of water increases, however, the
(c) 1.5 (d) 2 concentration of H+ ions and OH– ions are equal. What will
2. For the reaction, H 2 ( g ) + I2 ( g ) ‡ˆˆ
ˆˆ† 2HI ( g ) , the standard be the pH of pure water at 60°C?
(a) Equal to 7.0 (b) Greater than 7.0
free energy is DG s > 0. The equilibrium constant (K) would (c) Less than 7.0 (d) Equal to zero
be
(a) K = 0 (b) K > 1 8. The ionisation constant of an acid, Ka is the measure of
(c) K = 1 (d) K < 1 strength of an acid. The Ka values of acetic acid, hypochlorous
3. Which of the following is not a general characteristic of acid and formic acid are 1.74 × 10–5, 3.0 × 10–8 and 1.8 × 10–4
equilibria involving physical processes? respectively. Which of the following orders of pH of 0.1 mol
(a) Equilibrium is possible only in a closed system at a dm–3 solutions of these acids is correct?
given temperature. (a) Acetic acid > hypochlorous acid > formic acid
(b) All measurable properties of the system remain (b) Hypochlorous acid > acetic acid > formic acid
constant. (c) Formic acid > hypochlorous acid > acetic acid
(c) All the physical processes stop at equilibrium. (d) Formic acid > acetic acid > hypochlorous acid
(d) The opposing processes occur at the same rate and
there is dynamic but stable condition. 9. K a1 , Ka 2 and K a 3 are the respective ionisation constants
4. PCl5, PCl3, and Cl2 are at equilibrium at 500 K in a closed for the following reactions.
container and their concentrations are 0.8 × 10–3 mol L–1, ˆˆ† H + + HS-
H 2S ‡ˆˆ
1.2 × 10–3 mol L–1 and 1.2 × 10–3 mol L–1 respectively. The
value of Kc for the reaction H S - ‡ˆˆ
ˆˆ† H + + S 2 -
PCl5 ( g ) ‡ˆˆ
ˆˆ† PCl3 ( g ) + Cl2 ( g ) will be ˆˆ† 2H + + S 2 -
H 2 S ‡ˆˆ
(a) 1.8 × 103 mol L–1 (b) 1.8 × 10–3 The correct relationship between K a1 , Ka 2 and K a 3 is
–3
(c) 1.8 × 10 mol L –1 (d) 0.55 × 104
5. Which of the following statements is incorrect? (a) K a3 = K a1 ´ K a 2 (b) K a3 = K a1 + K a 2
(a) In equilibrium mixture of ice and water kept in perfectly
(c) K a3 = K a1 - K a 2 (d) K a3 = K a1 / K a 2
insulated flask, mass of ice and water does not change
with time 10. Acidity of BF3 can be explained on the basis of which of the
(b) The intensity of red colour increases when oxalic acid following concepts?
is added to a solution containing iron (III) nitrate and (a) Arrhenius concept
potassium thiocyanate (b) Bronsted Lowry concept
(c) On addition of catalyst the equilibrium constant value (c) Lewis concept
is not affected (d) Bronsted Lowry as well as Lewis concept
(d) Equilibrium constant for a reaction with negative DH 11. Which of the following will produce a buffer solution when
value decreases as the temperature increases. mixed in equal volumes?
6. When hydrochloric acid is added to cobalt nitrate solution (a) 0.1 mol dm–3 NH4OH and 0.1 mol dm–3 HCl
at room temperature, the following reaction takes place and (b) 0.05 mol dm–3 NH4OH and 0.1 mol dm–3 HCl
thus the reaction mixture becomes blue. On cooling the (c) 0.1 mol dm–3 NH4OH and 0.05 mol dm–3 HCl
mixture it becomes pink. On the basis of this information (d) 0.1 mol dm–3 CH3COONa and 0.1 mol dm–3 NaOH
mark the correct answer. 12. In which of the following solvent silver chloride is most
3+
é Co ( H 2O )6 ù ( aq ) + 4Cl- ( aq ) ‡ˆˆ
ˆˆ† soluble?
êë ( Pink) úû (a) 0.1 mol dm–3 AgNO3 solution
2- (b) 0.1 mol dm–3 HCl solution
é CoCl 4 ù ( aq ) + 6H 2O ( l) (c) H2O
ëê ( Blue) ûú
(d) Aqueous ammonia
Equilibrium 167

13. What will be the value of pH of 0.01 mol dm–3 CH3COOH 21. Which of these is least likely to act as Lewis base? [2013]
(Ka = 1.74 × 410–5)? (a) F– (b) BF3
(a) 3.4 (b) 3.6 (c) PF3 (d) CO
(c) 3.9 (d) 3.0 22. The values of K sp of CaCO 3 and CaC 2 O 4 are
14. Ka for CH3COOH is 1.8 × 10–5 and Kb for NH4OH is 4.7 × 10–9 and 1.3 × 10–9 respectively at 25°C. If the mixture of
1.8 × 10–5.The pH of ammonium acetate will be
these two is washed with water, what is the concentration of
(a) 7.005 (b) 4.75
Ca2+ ions in water? [NEET Kar. 2013]
(c) 7.0 (d) Between 6 and 7 –5
15. Which of the following options will be correct for the stage (a) 7.746 × 10 M (b) 5.831 × 10–5 M
ˆˆ† B? (c) 6.856 × 10 M –5 (d) 3.606 × 10–5 M
of half completion of the reaction A ‡ˆˆ
– – 23. At 100°C the Kw of water is 55 times its value at 25°C. What
(a) DG = 0 (b) DG > 0
– will be the pH of neutral solution? (log 55 = 1.74)
(c) DG < 0 (d) DGs = - RT ln K
16. On increasing the pressure, in which direction will the gas [NEET Kar. 2013]
phase reaction proceed to re-establish equilibrium, is (a) 6.13 (b) 7.00
predicted by applying the Le-Chatelier's principle. Consider (c) 7.87 (d) 5.13
the reaction, 24. The dissociation constant of a weak acid is
ˆˆ† 2NH3 ( g )
N 2 (g) + 3H 2 ‡ˆˆ 1 × 10– 4. In order to prepare a buffer solution with a pH = 5
the [Salt]/[Acid] ratio should be [NEET Kar. 2013]
Which of the following is correct, if the total pressure at
which the equilibrium is established, is increased without (a) 1 : 10 (b) 4 : 5
changing the temperature? (c) 10 : 1 (d) 5 : 4
(a) K will remain same 25. Accumulation of lactic acid (HC3H5O3), a monobasic acid in
(b) K will decrease tissues leads to pain and a feeling of fatigue. In a 0.10 M
(c) K will increase aqueous solution, lactic acid is 3.7% dissociated. The value
(d) K will increase initially and decrease when pressure is of dissociation constant, Ka, for this acid will be:
very high [NEET Kar. 2013]
17. What will be the correct order of vapour pressure of water, (a) 2.8 × 10–4 (b) 1.4 × 10–5
acetone and ether at 30°C? Given that among these (c) 1.4 × 10–4 (d) 3.7 × 10–4
compounds, water has maximum boiling point and ether has 26. Which of the following salts will give highest pH in water ?
minimum boiling point?
(a) KCl (b) NaCl [2014]
(a) Water < ether < acetone (b) Water < acetone < ether
(c) Ether < acetone <water (d) Acetone < ether < water (c) Na2CO3 (d) CuSO4
18. At 500 K, equilibrium constant, Kc, for the following reaction 27. Using the Gibbs energy change, DG° = + 63.3kJ, for the
is 5. following raction, [2014]
Ag2CO3 ‡ˆˆˆˆ† 2Ag+ (aq) + CO 2–(aq)
1 1
H 2 ( g ) + I 2 ( g ) ‡ˆˆ
ˆˆ† HI ( g ) 3
2 2 the Ksp of Ag2CO3(s) in water at 25°C is:-
What would be the equilibrium constant KC for the reaction ? (R = 8.314 J K–1 mol–1)
2HI(g) ƒ H2(g) + I2(g) (a) 3.2 × 10–26 (b) 8.0 × 10–12
(a) 0.04 (b) 0.4 (c) 2.9 × 10–3 (d) 7.9 × 10–2
(c) 25 (d) 2.5 28. For the reversible reaction, [2014]
19. In which of the following reactions, the equilibrium remains
N2(g) + 3H2(g) 2NH3(g) + heat
unaffected on addition of small amount of argon at constant
volume? The equilibrium shifts in forward direction
(a) H 2 ( g ) + I2 ‡ˆˆ
ˆˆ† 2HI ( g ) (a) By increasing the concentration of NH3(g)
(b) By decreasing the pressure
(b) PCl5 ( g ) ‡ˆˆ
ˆˆ† PCl3 ( g ) + Cl2 ( g )
(c) By decreasing concentration of N2(g) and H2(g)
(c) N 2 ( g ) + 3H 2 ( g ) ‡ˆˆ
ˆˆ† 2NH 3 ( g ) (d) By increasing pressure and decreasing temperature.
(d) The equilibrium will remain unaffected in all the three 29. For a given exothermic reaction, Kp and KP¢ are the equilibrium
cases constants at temperatures T1 and T2, respectively. Assuming
NEET/AIPMT (2013-2017) Questions that heat of reaction is constant in temperature range between
T1 and T2, it is readily observed that: [2014]
20. Identify the correct order of solubility in aqueous medium:
[2013] (a) Kp > K P¢ (b) Kp < K P¢
(a) ZnS > Na2S > CuS (b) Na2S > CuS > ZnS 1
(c) Na2S > ZnS > CuS (d) CuS > ZnS > Na2S (c) Kp = K P¢ (d) Kp =
K¢p
EBD_7327
168 CHEMISTRY

30. The K sp of Ag 2 CrO 4 , AgCl, AgBr an d AgI (b) The molar solubility of MY in water is less than that of
are respectively, 1.1 × 10–12, 1.8 × 10–10, 5.0 × 10–13, 8.3 × 10– NY3
17. Which one of the following salts will precipitate last if (c) The salts MY and NY3 are more soluble in 0.5 M KY
AgNO3 solution is added to the solution containing equal than in pure water.
moles of NaCl, NaBr, NaI and Na2CrO4? [2015] (d) The addition of the salt of KY to solution of MY and
(a) AgCl (b) AgBr NY3 will have no effect on their solubilities.
(c) Ag2CrO4 (d) AgI 37. Consider the nitration of benzene using mixed conc of H2SO4
and HNO3. If a large amount of KHSO4 is added to the mixture,
31. If the value of an equilibrium constant for a particular reaction the rate of nitration will be [2016]
is 1.6 × 1012, then at equilibrium the system will contain :- (a) faster (b) slower
(a) mostly reactants [2015] (c) unchanged (d) doubled
(b) mostly products 38. Consider the following liquid - vapour equilibrium. [2016]
(c) similar amounts of reactants and products ˆˆ† Vapour
Liquid ‡ˆˆ
(d) all reactants
Which of the following relations is correct ?
32. Which of the following statements is correct for a reversible
process in a state of equilibrium ? dlnG DH v dlnP DH v
(a) 2
= 2 (b) =
(a) DG = 2.30 RT log K [2015] dT RT dT RT
(b) DGº = –2.30 RT log K dlnP -DH v dlnP -DH v
(c) DGº = 2.30 RT log K (c) = (d) =
dT 2 T2 dT RT 2
(d) DG = –2.30 RT log K 39. The equilibrium constant of the following are :
33. Which one of the following pairs of solution is not an acidic N2 + 3H2 ƒ 2NH3 K1 [2017]
buffer ? [2015 RS] N2 + O2 ƒ 2NO K2
(a) HClO4 and NaClO4
1
(b) CH3COOH and CH3 COONa H 2 + O 2 ® H 2O K3
2
(c) H2CO3 and Na2CO3 The equilibrium constant (K) of the reaction :
(d) H3PO4 and Na3PO4
5 K
ˆˆˆ†
34. What is the pH of the resulting solution when equal volumes 2NH3 + O 2 ‡ˆˆˆ 2NO + 3H2O, will be;
2
of 0.1 M NaOH and 0.01 M HCl are mixed ? [2015 RS]
(a) 12.65 (b) 2.0 (a) K 2 K33 / K1 (b) K2K3/K1
(c) 7.0 (d) 1.04
(c) K32 K3 / K1 (d) K1K 33 / K 2
35. If the equilibrium constant for
N2(g) + O2(g) ‡ˆˆˆˆ† 2NO(g) is K, 40. Concentration of the Ag+ ions in a saturated solution of
Ag2 C2 O4 is 2.2 × 10–4 mol L–1 . Solubility product of
the equilibrium constant for [2015 RS] Ag2C2O4 is :- [2017]
1 1 (a) 2.66 × 10–12 (b) 4.5 × 10–11
N (g) + O2(g) ‡ˆˆ ˆˆ† NO(g) will be:
2 2 2 (c) 5.3 × 10–12 (d) 2.42 × 10–8
1 41. A 20 litre container at 400 K contains CO2(g) at pressure 0.4
1
(a) K2 (b) K atm and an excess of SrO (neglect the volume of solid SrO).
2
The volume of the container is now decreased by moving the
(c) K (d) K2 movable piston fitted in the container. The maximum volume
36. MY and NY3, two nearly insoluble salts, have the same Ksp of the container, when pressure of CO2 attains its maximum
values of 6.2 × 10–13 at room temperature. Which statement value, will be :-
would be true in regard to MY and NY3 ? [2016] (Given that : SrCO3 (s) ƒ SrO (s) + CO2(g), Kp = 1.6 atm)
(a) The molar solubilities of MY and NY3 in water are (a) 10 litre (b) 4 litre [2017]
identical. (c) 2 litre (d) 5 litre
Equilibrium 169

Hints & Solutions


EXERCISE - 1 6. (b) Kp = Kc ( RT )
Dng

1. (a) Let s be the solubility of PbCl2 and Ksp be its solubility


\ Kp = Kc only when Dng = 0
product
The solubility equilibrium of PbCl2 is represented as ˆˆ† 2NO( g )
For reaction, N 2 ( g ) + O2 ( g ) ‡ˆˆ
ˆˆ† Pb 2+ -
PbCl2 ‡ˆˆ + 2Cl Dn(g) = 2 – 2 = 0
s 2s
\ Ksp= [Pb2+] [Cl–]2 = (s) (2s)2 = 4s3 \ Kp = Kc (RT)0 or Kp = Kc
1/ 3 7. (a) For reation (i)
æ K sp ö
Þ s= ç ÷ [NO]2
è 4 ø K1 <
2. (b) For a gaseous phase reaction Kp and Kc are related as [N 2 ][O2 ]
Dn g and for reaction (ii)
K p = K c ( RT )
For the given reaction, [N 2 ]½ [O 2 ]½ 1
K2 < ; \ K1 <
1 [NO] K 22
CO(g) + O2 ( g ) ® CO2(g)
2 8. (c) To find which of the given compound among AgCl,
1 AgBr and AgCrO4 would they use, first find which has
Dng = 1– (1 + 0.5) = – 0.5 or - highest conc. of Ag+ ions. For this, we have to calculate
2
1
the solubility of Ag+ ions (let s be the solubility of Ag+
- ions)
\ Kp = Kc ( RT ) 2
For AgCl,
1
Kp -
2
AgCl A g+ + C l-
or = ( RT ) s s
Kc
Þ Ksp = (s)(s) = s2
3. (b) The solubility equilibrium of Ca3(PO4)2is represented
as, Þ s= K sp = 6 (1.8 ´10 -10 )3
Ca3(PO4)2 ‡ˆˆˆˆ† 3Ca2+ + 2PO 3– For AgBr,
4
3s 2s
\ Ksp = [Ca2+]3 [PO43–]2 s= K sp = 6 (5.0 ´10 -13 ) 3
= [3s]3 [2s]2 = 108 s5 For Ag2CrO4,
4. (b) N 2 ( g ) + 3H 2 ( g ) 2NH3 ( g )
s = 3 K sp < 6 (2.4≥10,12 )2
Dn = nproducts - nreactants = 2 – 4 = – 2
So, solution of silver compound can be made from Ag2
Kc
\ K p = K c ( RT ) -2 or,, K p = , CrO4 which has highest solubility among the given
( RT ) 2 silver halides.
Thus Kp < Kc. 9. (c) Using the relation KP = KC. (RT)Dn, we get
5. (d) Given A + 2B 2C KP
= ( RT ) Dn
KC
[ C ]2
K = (i)
[ A ] [ B] 2 KP
Thus K will be highest for the reaction having
C
1
C ƒ B+ A highest value of Dn.
2 The Dn values for various reactions are
[ B][ A ]1/ 2 æ 1ö
(a) Dn = 1– ç1 + ÷ = –
1
K' = (ii)
[ C] è 2 ø 2
Dividing (ii) by (i) (b) Dn = 2 – (1 + 1) = 0
(c) Dn = (1 +1) – 1= 1
1/ 2 1/ 2 (d) Dn = (2 + 4) – (7 + 2) = – 3
é1ù é1ù
K' =ê ú =ê ú Thus maximum value of Dn = 1
ëKû ë 40 û
EBD_7327
170 CHEMISTRY

10. (a) FeCl3 which is a salt of strong acid and weak base,
2 2
undergoes cationic hydrolysis in water as follows : V´ V
\ K= 3 3 =4
FeCl3 + 3H2O Fe(OH)3 + 3HCl
1 1
or Fe3+ + 3Cl – + 3H2O Fe(OH)3 V´ V
3 3
+ 3H+ + 3Cl–
or Fe3+ + 3H2O Fe(OH)3 + 3H+ 16. (b) PNH3 = PH 2S = 2 atm
+
So, it produces H ions due to which solution of FeCl3
in water is acidic. \ K p = PNH3 ´ PH2S = 4
11. (a) Statement (a) is correct and the rest statements are 17. (a) Unit of Kc Þ (mol lit–1)Dn
wrong. Kp depends only on temperature hence at For given reaction,
constant temp. Kp will not change. Dn = (4 + 6) – (4 + 5) = 1,
12. (a) For the given reaction, Kc ® mol lit–1
ˆˆ† 2H 2 ( g ) + S2 ( g ) 18. (b) According to equation
2H 2S (g) ‡ˆˆ
2HI H2 + I2
Dn = 3 – 2 = 1
At t = 0 (2 moles) 0 0
Kp -2 At equilibrium (2 – 2a ) moles a mole a mole
1.2 ´ 10
Kc = = Total moles at equilibrium
n
( RT ) ( RT )1
= 2 – 2a + a + a = 2 mole
\ Kc < K p or Kc < 1.2 ´ 10 –2 19. (b) pH = – log [H+] = – log [5 × 10–4]
= 4 – log 5 = 4 – [log 10 – log 2]
Dn
13. (c) As Kp = Kc RT g = 3 + log 2 = 3.3010
Here Dng = 1 pOH = 14 – pH = 14 – 3.3 = 10.7.
So, Kp = Kc when RT = 1 20. (b) SO2Cl2 ( g ) SO 2 ( g ) + Cl2 ( g )
Thus T = 12.5 K D-d
14. (b) PCl5 ƒ PCl3 + Cl2 a% = ´ 100 (y = 2)
d ( y - 1)
Moles at equilibrium
molar mass of SO 2 Cl 2 135
1 1 1 D= = = 67.5; d = 50.0
2 2
2 2 2 (given)
Mole fraction at equilibrium
67.5 - 50.0
1 1 1 \ a= ´ 100 = 35%
50.0(2 - 1)
3 3 3
21. ˆˆ† 2NO( g ) + Heat
(a) N 2 ( g ) + O 2 ( g ) ‡ˆˆ
Partial pressure at equilibrium
No. of moles of reactants = No. of moles of products.
P P P
The reaction in which number of moles of reactants are
3 3 3 equal to number of moles of products are not affected
P P by change in pressure (Le-chatelier's principle).
´
P
Kp = 3 3 = [Product]
P/3 3 22. (b) K=
[Reactant]
15. (d) CH3COOH + C2H5OH ® CH3COOC2H5 + H2O
Hence, [Product] = K [Reactant]
At start 1 mole 1 mole 0 0
\ [Product] > Reactant, when K > 1
2 2 2 2
At equb. 1– 1–
3 3 3 3 23. (b) C2H4 + H 2 C 2 H 6 , D H = - 130 kJ mol - 1
Let the total volume = V L 2 moles 1 mole

1 The reaction is exothermic and involves decrease in


\ [CH3OOH] = V mol/L
3 volume, so forward reaction will be favoured by
1 decreasing temperature and increasing pressure
[C2H5OH ] = V mol/L (Le Chatelier principle).
3
24. (b) The species having minimum value of Ksp will get
2 precipitated first of all because ionic product will exceed
[CH3COOC2H5] = V mol/L
3 the solubility product of such a species.
2 The Ksp value is minimum for BaSO4(10–11),
[H2O] = V mol/L so, BaSO4 will get precipitated first of all.
3
Equilibrium 171

25. (a) Reactions in which number of moles of gaseous -14


reactants and gaseous products is equal (i.e. D n = 0) or [ H + ] = 1´ 10 = 2 ´ 10 -13 mol L–1
0.05
are not affected by change in pressure (Le Chatelier
principle) We also know that
H 2 ( g ) + I2 ( g ) ƒ 2HI( g ) pH = - log[H + ] = - log [2 ´10 -13 ]
26. (d) Solid Liquid = - log 2 - log 10 -13 = - log 2 - (-13) log 10
It is an endothermic process. So when temperature is
raised, more liquid is formed. Hence adding heat will = -0.3010 + 13.0000 = 12.6990.
shift the equilbrium in the forward direction. Since the value of pH > 7, therefore the solution is
27. (d) For reaction to proceed from right to left basic.
Q > K c i.e the reaction will be fast in backward direction [Salt ]
36. (b) pOH = pK b + log
i.e rb > rf. [ Base ]
28. (c) a = 0.1, [Salt]
(0.1) 2 ´ (0.01) or pKb = pOH–log
a2 C [Base]
Ka = = = 1.11 × 10–4
1- a (1 - 0.1) but pOH+ pH = 14 or pOH = 14 – pH
Now pKa = –log 1.11 × 10–4 =3.9542 [Salt]
\ 14– pH – log = pK b
[salt] [Base]
pH = pKa + log
[acid]
0.1
14 - 9.25 - log = pK b
é 0.05 ù 0.1
= 3.9542 + log ê ú = 3.653
ë 0.10 û 14 – 9.25 – 0 = pKb pKb = 4.75
29. (c) Catalyst only changes time to achieve equilibrium, it 37. (b) B(OH)3 does not provide H+ ions in water instead it
does not affect the value of equilibrium constant. accepts OH– ion and hence it is Lewis acid.
30. (a) Solubility of PbF2 » 10–3 M
\ Ksp = 4S3 = 4 × 10–9
B(OH )3 + H 2O [B(OH) 4 ]- + H +
In 0.05 M NaF we have 0.05 M of F– ion contributed by 0.1
NaF. If the solubility of PbF2 in this solution is S M, 38. (c) (a) [ H 3O + ] = = 0.05 ;
2
then
total [F–] = [2S + 0.05] M. pH= – log 0.05 = 1.301
\ S[2S + 0.05]2 = 4 × 10–9 (b) Complete neutralization of strong acid and strong
Assuming 2S << 0.05, base, pH = 7.
S × 25 × 10–4 = 4 × 10–9 (c) Hydrolysis of the salt CH3 COOK,
\ S = 0.16 × 10–5 M Þ 1.6 × 10–6 M pH > 7 (salt of weak acid with strong base)
We observe that our approximation that 2S << 0.05 is (d) Hydrolysis of the salt NH 4 NO 3 ,
justified. pH < 7 (salt of strong acid with weak base)
31. (c) Exothermic reaction is favoured by low temperature to 39. (a) (HSO4)– can accept and donate a proton
proceed in forward direction. (HSO4)– + H+ H2SO4 (acting as base)
(HSO4)– – H+ SO42–. (acting as acid)
32. (c) ˆˆ† H 2 ( g ) + 1 O2 ( g )
H 2O( g ) ‡ˆˆ 40. (b) I– is electron donor. Hence it is Lewis base.
2
41. (d) Given concentration of NaOH = 10-10 M
In this reaction volume is increasing in the forward
direction. So on increasing temperature reaction will NaOH ¾¾ ® Na + + OH -
proceed in forward direction. 10-10 M 10-10
33. (c) ˆˆ† A + + B -
AB ‡ˆˆ \ [OH- ] from NaOH = 10-10
We have to consider dissociation of H2O
K sp = [A + ][B- ]
[OH- ] from H 2 O = 10 -7
Salt will precipitate if ionic conc. > Ksp
[A+][B–] > 1 × 10–8 Total [OH - ] = 10 -7 + 10-10
(1 × 10–3)[B–] > 1 ×10–8
æ 1001 ö
= 10-7 (0.001 + 1) = 10-7 ç ÷ = 10
-10
´ 1001
- 1 ´ 10-8 è 1000 ø
[B ] > or 1 × 10–5
1 ´ 10 -3 -
\ pOH = – log [OH ]
34. (b) Le chatelier principle is not applicable to solid-solid
equilibrium. = -(log1001´10-10 ) = – 3.004 + 10 = 6.9996
35. (a) Given : Hydroxyl ion concentration pH = 14 – pOH = 14 – 6.996 = 7.004
[OH–] = 0.05 mol L–1. We know that \ pH of 10–10 M NaOH solution is nearest to 7.
[H + ][OH - ] = 1´10 -14
EBD_7327
172 CHEMISTRY

42. (d) Strongest Bronsted base has weakest conjugate acid. 49. (a) In BCl3 and BF3 due to – I effect of F and Cl these are
Q HOCl is weakest acid among HOCl, HClO2, HClO3 better Lewis acids than BMe3 and B2H6. Among BCl3
and HClO4. and BF3, the former is better Lewis acid as in it pp - pp
\ Conjugate base of HOCl (i.e. OCl–) is strongest back bonding is lesser than in BF3.
Bronsted base. 50. (b) The value of ionic product of water changes with the
43. (d) In N/1000 KOH, [OH–] = 10–3 M temperature.
pOH = – log [OH–] = – log 10–3 = 3 51. (d) HCl is stronger acid than CH3COOH and Cl– is a
pH = 14 – pOH = 14 – 3 = 11 +
stronger base than CH 3COOH 2 and is the conjugate
44. (b) (i) HCl ¾¾ ® H + + Cl -
base of HCl.
0.1 M 0.1 M +
\ [H+] = 0.1 M HCl + CH3COOH Cl– + CH 3COOH 2
acid1 base2 base1 acid2
pH = – log [H+] = – log 0.1 = 1
52. (b) Diacidic base has two replaceable hydroxyl groups.
(ii) NaCl is a salt of strong acid and strong base so it
–OH groups present in compounds (a) and (c) represent
is not hydrolysed and hence its pH is 7.
alcoholic group, not base.
ˆˆ† NH OH + HCl
(iii) NH4Cl + H2O ‡ˆˆ 4 53. (d) Lewis acids are electron pair acceptor whereas Bronsted
\ The solution is acidic and pH is less than that acids are proton donor.
of 0.1 M HCl. . . 2– 2-
(iv) NaCN + H2O ‡ˆˆ ˆˆ† NaOH + HCN SO3 + : O . . : ¾¾® SO4
\ The solution is basic and pH is more than that Lewis acid
of 0.1 M HCl. 54. (a) Bronsted base is a substance which accepts proton. In
\ Correct order for increase in pH is option (a), H2O is accepting proton, i.e., acting as a
HCl < NH4Cl < NaCl < NaCN. base.
55. (c) On dilution [H+] = 10–6 M = 10–6 mol
45. (c) H2SO4 2H+ + SO42–
Now dissociation of water cannot be neglected,
Given concentration of H2SO4 = 0.005 M
Total [H+] = 10–6 + 10–7 = 11 × 10–7
\ [H+] = 0.005 × 2 = 0.01 = 10–2
pH = –log [H+]
pH = – log [H+] = – log 10–2 = 2
= –log (11× 10–7) = 5.98
46. (d) Among M/4 KOH, M/4 NaOH, M/4 NH4OH
56. (a) Acid and base will neutralise each other. Since volume
and M/4 Ca(OH)2, Ca(OH)2 furnishes highest number
of NaOH is more (150 ml) in solution.
of OH– ions
(Q Ca(OH)2 ¾¾ ® Ca2+ + 2OH–). So pH of M/4 - M 2 V2 - M1V1 150 ´ 0.5 - 50 ´ 0.5 1
\ [OH ] = = =
Ca(OH)2 is highest. V1 + V2 + V3 50 + 150 + 300 10
47. (a) NH 3 + H 2 O NH 4+ + OH - Now [H + ] [OH - ] = 10 -14
[NH +4 ] [OH - ] x ´ x
Kb = = Þ [ H+ ] = 10 -13
[NH 3 ] 0.1 pH = 13.
x2
or 1.8 ´ 10-5 = [Salt] 8 4
= =
0.1 57. (c) At 80% neutralization ,
[Base] 2 1
x2 = 1.8 × 10–6
x = 1.35 × 10–3 [Salt]
Thus at equilibrium, [OH–] = 1.35 × 10–3 \ pOH = pK b + log
[Base]
pOH = –log (1.35 × 10–3) = 2.87 14 – 7.4 = pKb + log 4
\ pH = 14.00 – 2.87 = 11.13 pKb = 6.6 – 0.6 = 6
48. (c) Number of meq. of the acid = 0.04 × 100 = 4 Kb = 10–6
Number of meq. of the base = 0.02 × 100 = 2
\ Number of meq. of the acid left on mixing = 4 – 2 = 2 [C] 0.216
58. (a) K= = = 250
Total volume of the solution = 200 mL [A][B] 2 0.06 ´ 0.12 ´ 0.12
\ No. of meq of the acid present in 1000 mL of the 59. (d) Given Kb = 1.0 × 10–12
solution = 10 [BOH] = 0.01 M [OH] = ?
or No. of eq. of the acid in 1000 mL of the solution
10 ˆˆ† B + + OH -
BOH ‡ˆˆ
= = 0.01 t =o c 0 0
1000 teq c(1- x) cx cx
Since the acid is monobasic and completely ionises in
solution c 2 x2 cx 2 0.01x 2
0.01 N HCl = 0.01 M HCl Kb = = Þ 1.0 × 10–12 =
c (1 - x) (1 - x ) (1 - x )
Thus [H + ] = 0.01 On calculation, we get, x = 1.0 × 10–5
\ pH = – log (0.01) = – (– 2) = 2 Now, [OH–] = cx = 0.01 × 10–5 = 1 × 10–7mol L–1
Equilibrium 173

60. (b) pKa = –log Ka 2


æ 1.34 ö
Smaller the value of pKa, stronger will be acid 67. (a) K = ca 2 = 0.1 ´ ç = 1.8 ´ 10 -5
\ Acid having pKa value of 10–8 is strongest acid. è 100 ÷ø
61. (b) Solubility product is the product of ionic concentration 68. (a) ˆˆ† Pb2+ + 2Cl- ;
PbCl2 ‡ˆˆ
in a saturated solution of an electrolyte at a given
temperature. Ksp = [Pb 2+ ] [Cl - ]2
wt. of solute per litre of solution
62. (d) As, molarity, = [Pb2 + ] = 6.3 ´10 -3 ; [Cl- ] = (2 ´ 6.3 ´10-3 )
Mol. wt. of solute
= 12.6 × 10–3
1000
Molarity of H2O = mole/litre Ksp = (6.3 × 10–3) (12.6 × 10–3)2
18
69. (b) For Bi2S3,
H2O ƒ H+ + OH– Ksp = (2s)2.(3s)3 = 4s2. 27s3 = 108s5
c (1 – a) ca ca
where s = solubility
ca 2
Thus, Ka = = ca 2 = 1.8 × 10–14 K sp
1- a \ s=5
108
a1 Ka1
3.14 ´ 10 -4 For MnS, Ksp = s2
63. (b) = = = 4 :1
a2 K a2 1.96 ´ 10-5 \ s = Ksp < 7≥10,16
64. (c) For weak acid dissociation equilibria, degree of For CuS, Ksp = s2
dissociation a is given as :
\ s < K sp < 8≥10,
37
Ka Ka
a= \ %a = 100 For Ag2S, Ksp = 4s3
c c
Ksp
[H + ][A - ] [H + ]ca [H + ]a \ s=3
Also, K a = = = 4
[HA] c (1 - a ) (1 - a)
Thus MnS has maximum solubility.
a 70. (d) Aspirin is a weak acid. Due to common ion effect, it is
log K a = log H + + log unionised in acid medium but completely ionised in
1- a
alkaline medium.
1- a 71. (c) HCl + NH4Cl. HCl is strong acid hence not used in
or pK a = pH + log
a buffer.
72. (b) (i) NaCl is ionic compound and therefore it is least
1- a soluble in liquid CCl4.
pK a - pH = log
a (ii) Covalent compounds are soluble in liquid CCl4.
1- a Solubility of covalent compounds
= 10pK a -pH
a 1
µ
1 Molecular mass
or, = 10pK a - pH + 1 \ Cl2 has highest solubility in liquid CCl4.
a
73. (a) Let solubility of AgCl = x moles/L
1 ˆˆ† Ag + + Cl-
\ a= AgCl ‡ˆˆ
[1 + 10pKa - pH ] before dissociation x 0 0
65. (d) In presence of little H2SO4 (as catalyst) about 2/3 mole after dissociation x x
of each of CH3COOH and C2 H5OH react to form 2/ NaCl ¾¾ ® Na + + Cl-
3 mole of the product at equilibrium. Connection of Cl– = 0.2

[H3O+ ][F- ] Total [Cl - ] = x + 0.2


66. (c) Ka = ...(i)
[HF] Ksp (AgCl) = [Ag + ][Cl - ] = (x) (x + 0.2) = x 2 + 0.2 x
[HF][OH - ] \
2
Ksp ; 0.2 x (Q x < < 1)
Kb = ...(ii)
-
[F ] \ 1.2 ´ 10 -10 = 0.2 x
From (i) and (ii), KaKb = [H3O+][OH–]=Kw \ x = 6 ´ 10 -10 M
(ionic product of water)
EBD_7327
174 CHEMISTRY

74. (c) Let binary electrolyte be AB 79. (b) The dissociation of a weak electrolyte (AgCl) is
ˆˆ† A + + B-
AB ‡ˆˆ suppressed on adding the strong electrolyte having a
s s s common ion (Cl–), hence concentration of Ag+ ion will
Hence, solubility product of AB be less.
Ksp = [A+] [B–]
80. (a) 2NO ( g ) ¾¾
® N 2 (g) + O 2 (g) .......(i)
S = [s.] [s] Þ s = S½
75. (d) Total volume = 100 mL Where, Kc = 1.8 ´ 10 2
1.0
[acid] = 10 mL ´ = 0.1 1 1
100 N 2 (g) + O 2 (g) ¾¾
® NO(g) .......(ii)
2 2
0.5 For equation (i),
[salt] = 20 mL ´ = 0.1
100 By Equilibrium’s Law
[ salt ] [ N 2 ] [O 2 ]
= Kc ............(iii)
pH of acidic buffer = pK a + log acid
[ ] [ NO] 2
For equation (ii)
0.1 By Equilibrium’s law
= 4.76 + log
= 4.76
0.1 [ NO(g)]
76. (b) Let solubility of BaCl2 = x moles/L K'c =
[ N 2 ]1 / 2 [O 2 ]1 / 2
BaCl2 Ba2+ + 2Cl–
On squaring,
No. of moles x x 2x
Ksp = [Ba2+] [Cl–]2 [ NO(g)]2
= (x) × (2x)2 = 4x3 (K 'c ) [ N ] [O ] ..........(iv)
2 =
2 2
\ 4 × 10–9 = 4x3 From equation (iii) and (iv),
4 ´ 10 –9 K c ´ ( K 'c ) 2 = 1
or, x= 3 = 1 ´ 10 -3
4
1 1
K'c = = = 7.6
[H+] of 100 ml solution which has (pH = 1), -1
77. (a) N1 = 10 Kc 1.8 ´ 102
(From CH+ = 10–pH)
81. (d) CaF2 Ca 2+ + 2F -
[H + ] of 100 ml solution which has
s 2s
(pH = 2) N2 = 10–2
Ksp = s ´ (2 s)2 = 4s3
N1V1 + N 2 V2 = N s Vs

10 -1 ´ 100 + 10 - 2 ´ 100 Ksp = 4 ´ (2.3 ´ 10-4 )3


= N s ´ (100 + 100) = 4.9 ´10 -11 (mol dm -3 )3
82. (d) In case of alkaline earth hydroxides solubility increases
11
10 + 1 = 200 N s or = Ns on moving down the group.
200 Be(OH)2 has lowest solubility and hence lowest Ksp.
83. (c) Due to common ion effect addition of NH4Cl in group
or N s = 5.5 ´ 10 -2
(III) suppresses the ionisation of NH4OH with the result
Hence, the hydrogen ion concentration of the mixture
concentration of OH– decreases.
solution is 5.5 ´ 10 -2 . 84. (d) Buffering action is maximum when
78. (a) A– + H2O ƒ HA + OH– [Salt] = [Acid]
0.001 M 0 0 (At start) i.e., pH = pKa
(0.001 – h) (0.001 × h) (0.001× h) (At eq.) 85. (c) Hydrolysis of a salt is reverse reaction of acid base
(0.001 ´ h)(0.001 ´ h)
neutralization reaction.
Kh =
(0.001 - h) K w 10-14
\ Kh = = -7 = 10–7
or, K h = 0.001 ´ h2 (as, 0.001 – h » 0.001) Ka 10
Kh
æ K w 10 -14 ö [OH - ] = ch = c ´ = c ´ K h = 10 -8 = 10 -4
10 -9
= 0.001 ´ h 2
ç Kh = = = 10 -9 ÷ c
Ka - 5
è 10 ø Þ pOH– = –log [OH–]
h2 = 10–6 \ h = 10 -3 = – log [10–4] = 4
Equilibrium 175

86. (c) Precipitation will occur when, ionic product > Ksp. = [Fe3+] [OH–]3 [\ [solid]=1].
Among thegiven choices, theionic product of 10–4 M 1
AgNO3 and 10–4 M KCl is 25 × 10–10 which exceeds If [OH–] is decreased by times then for reaction
4
the Ksp of AgCl which is shown below : equilibrium constant to remain constant, we have to
AgNO3 iionizes completely in the solution as increase the concentration of [Fe3+] by a factor of 43
AgNO3 ¾¾ ® Ag + + NO 3- i.e 4× 4 × 4 = 64. Thus option (c) is correct answer.
[Ag + ] = [AgNO3 ] = 10-4 M 93. (b) The highest pH will be recorded by the most basic
KCl ionizes completely in the solution of solution. The basic nature of hydroxides of alkaline
earth metals increase as we move from Mg to Ba and
KCl ¾¾ ® K + + Cl - thus the solution of BaCl2 in water will be most basic
\ [Cl ] = [KCl] = 10–4M

and so it will have highest pH.
Since equal volumes of two solutions are mixed 94. (b) Given : Equilibrium constant (K1) for the reaction
together, therefore, the concentration of Ag+ ions and
Cl– ions after mixing will be K1 1 1
ˆˆˆ
HI( g ) ‡ˆˆ †
ˆ H 2 ( g ) + I2 ( g ); K1 = 8; .........(i)
10-4 2 2
[Ag + ] = M To find equilibrium constant for the following reaction
2
H 2 ( g ) + I2 ( g ) ƒ 2HI( g ); K2 = ? .....(ii)
10-4
and [Cl- ] = M For this multiply (i) by 2, we get
2
2HI( g ) ƒ H 2 ( g ) + I 2 ( g );
10-4 10-4
\ Ionic product of AgCl = ´ K1 = 82 = 64. .....(iii)
2 2 [Note: When the equation for an equilibrium is
= 25 ´ 10 - 10 multiplied by a factor, the equilibrium constant must be
87. (c) Ba++ is common ion in the both solution. The solubility raised to the power equal to the factor]
will depend on more reactive element. Now reverse equation (iii), we get
Option (c) is correct.
1
88. (a) The pOH of a buffer consisting of NH3 (i.e. NH4OH) H 2 ( g ) + I2 ( g ) ƒ 2HI( g ); K = .....(iv)
and salt NH4Cl (salt) is given by the equation 64
[Note: For a reversible reaction, the equilibrium constant
pOH = pKb + log
[ammonia]
= 5.0 + log
[ 0.1] of the backward reaction is inverse of the equilibrium
[salt] [1.0] constant for the forward reaction.]
= 5.0 – log 10 = 5 – 1 = 4.0 Equation (iv) is the same as the required equation (ii),
\ pH = 14 – pOH = 14 – 4.0 = 10 1
thus K2 for equation (ii) is i.e. option (b) is correct.
..
+ 64
89. (a) C 2 H 5 N H 2 is conjugate base of [C 2 H 5 NH 3 ] 95. (d) (CH3)3 B - is an electron deficient, thus behave as a
[Conjugate base -acid differ by one proton] lewis acid.
.. 96. (d) Ammonium chloride is a salt of weak base and strong
[C 2 H 5 NH 3 ]+ C2 H5 N H 2 + H + acid. In this case hydrolysis constant K h can be
conjugate base acid calculated as
90. (a) Given Ka = 1.00×10–5, c = 0.100 mol Kw 1 ´ 10-14
for a weak electrolyte, Kh = = = 5.65 ´ 1010
K b 1.77 ´ 10 -5
degree of dissociation
97. (d) 2C(s )+O2 (g ) ƒ 2 CO 2 (g )
Ka 1 ´ 10 –5
(a) = = = 10 -2 = 1% Dn = 2 – 1 = + 1
c 0.100
91. (a) Given [H3O+] = 1 × 10–10 M \ Kc and Kp are not equal.
at 25º [H3O+] [OH–] = 10–14 98. (d) Kb = 10-10 ; Ka = 10-4 or pKa = 4
10 -14 For the buffer solution containing equal concentration
\ [OH - ] = = 10-4
10 -10 of B– and HB
pOH = – log [OH–] pH = pKa + log 1
= – log [10 –4]
pH = pKa = 4
=4
\ pOH = 4 99. (c) Boron in B2H6 is electron deficient
92. (c) For this reaction Keq. is given by 100. (c) BF3 behaves as lewis acid.
3
101. (b) Given pH = 12
éFe3+ ù éOH - ù or [H+] = 10–12
ë ûë û
K= Since, [H+] [OH–] = 10–14
[ Fe(OH)3 ]
EBD_7327
176 CHEMISTRY

10-14 éA- ù
\ [OH–] = = 10–2 ë û
10-12 or pH = pK a + log
[ HA ]
Ba(OH)2 Ba 2+ + 2OH Given pKa = 4.5 and acid is 50% ionised.
s 2s
[OH–] = 10–2 [HA] = [A–] (when acid is 50% ionised)
2s = 10–2 \ pH = pKa + log 1
\ pH = pKa = 4.5
10-2 pOH = 14 – pH = 14 – 4.5 = 9.5
s=
2 108. (c) Let s = solubility
Ksp = 4s3 ˆˆ† Ag + + IO3-
AgIO3 ‡ˆˆ
s s
æ 10 -2 ö 3 Ksp = [Ag+] [IO3–] = s × s = s2
= 4´ç ÷ Given Ksp = 1 × 10–8
è 2 ø
\ s = K sp = 1´10-8
= 5 × 10–7
102. (c) Reaction (c) can be obtained by adding reactions (a) = 1.0 × 10–4 mol/lit = 1.0 × 10–4 × 283 g/lit
and (b) therefore K3 = K1. K2 (Q Molecular mass of Ag IO3 = 283)
Hence (c) is the correct answer.
® H 3O + + H 2 PO 4-
103. (a) (i) H3PO 4 + H 2 O ¾¾ 1.0 ´10-4 ´ 283 ´100
= gm /100ml
acid1 base2 acid 2 base1 1000
-
® HPO 4-- + H 3O + = 2.83 × 10–3 gm/ 100 ml
(ii) H 2 PO 4 + H 2 O ¾¾
acid1 base 2 base1 acid 2 109. (c) The correct order of acidic strength of the given species

H 2 PO 4- + OH - ¾¾
® H3 PO 4 + O -- HSO3 F > H 3 O + > HSO 4 - > HCO3-
(iii) acid 2 base2 (iv) (ii) (iii) (i)
base1 acid1

Hence only in (ii) reaction H2PO4– is acting as an acid. or (i) < (iii) < (ii) < (iv)
ˆˆ† Cr (aq.) + 3OH (aq.)
104. (b) Cr(OH)3 (s) ‡ˆˆ 3+ - It corresponds to choice (c) which is correct answer.
110. (c) In aqueous solution BA(salt) hydrolyses to give
s 3s
BA + H2O ‡ˆˆˆˆ† BOH + HA
(s) (3s)3 = Ksp
Base acid
27 S 4 = K sp Now pH is given by
1 1 1
1/ 4 1/ 4 pH = pK w + pKa - pK b
æ K sp ö æ 1.6 ´ 10-30 ö 2 2 2
s=ç =ç ÷
è 27 ÷ø è 27 ø substituting given values, we get
1
105. (d) For the reaction pH = (14 + 4.80 - 4.78) = 7.01
N2 + O2 ¾¾ ® 2NO K = 4 × 10–4 2
Hence for the reaction 111. (a) Na 2 CO3 ¾¾ ® 2Na + + CO32-
1 1 1 1 1 ´10-4 M 1´10-4 M 1´10-4 M
NO ¾¾
® N2 + O2 K'= = = 50
2 2 K 4×10 – 4 Ksp(BaCO3 ) = [Ba 2+ ][CO32– ]
106. (c) ˆˆ† H+ + Q-
HQ ‡ˆˆ 5.1 ´ 10 -9
[Ba 2+ ] = -4
= 5.1 ´ 10 -5 M
c(1–a) ca ca 1 ´ 10
[H + ]
[H + ] = c a ; a = 112. (c) H 2CO3 (aq)+ H 2O(l ) ƒ HCO3– (aq) + H3O + (aq)
c 0.034-x x x

10 –3 [HCO3- ][H 3O + ] x´ x
or a = = 10-2 K1 = =
0.1 [H 2 CO 3 ] 0.034 - x

K a = c a 2 = 0.1´10 -2 ´10 -2 = 10–5 x2


Þ 4.2´10-7 ; Þ x = 1.195´10-4
0.034
é salt ù As H2CO3 is a weak acid so the concentration of
107. (d) For acidic buffer pH = pKa + log ê ú
ë acid û H2CO3 will remain 0.034 as 0.034 >> x.
Equilibrium 177

(As [H+]) = [A–] = ca


x = [H+] = [ HCO3- ] = 1.195 × 10–4
But, [H+] << c
Now, HCO3– (aq) + H 2 O(l ) ƒ CO32– ( aq ) + H3O+ ( aq ) \ Ka = (10–5)2 = 10–10
x- y y y
As HCO3- is again a weak acid (weaker than H2CO3) 116. (a) ˆˆ† 2CO
CO2 + C(graphite) ‡ˆˆ
with x >> y. Pinitial 0.5atm 0
Pfinal (0.5 – x)atm 2 x atm
[CO32– ][H3O+ ] y ´ (x + y)
K2 = = Total P at equilibrium = 0.5 – x + 2x = 0.5 + x atm
[HCO3– ] ( x - y)
0.8 = 0.5 + x
Note : [H3O+] = H+ from first step (x) and from second \ x = 0.8 – 0.5 = 0.3 atm
step (y) = (x + y)
[As x > > y so x + y ; x and x – y ; x] Now Kp = (PCO )2 /PCO
2
y´x (2 ´ 0.3) 2 (0.6) 2
So, K 2 ; =y = = = 1.8 atm
x (0.5 – 0.3) (0.2)
Þ K 2 = 4.8 ´ 10-11 = y = [CO32 - ] 117. (d) Let the weak monoacidic base be BOH, then the
reaction that occurs during titration is
So the con centration of [H + ] ; [ HCO3– ] = BOH + HCl ® BCl + H2O
concentrations obtained from the first step. As the Equilibrium : B+ + H 2 O ‡ˆˆ
ˆˆ† BOH + H +
dissociation will be very low in second step so there c (1- h) c.h c.h
will be no change in these concentrations. Using the normality equation, N1V1 = N 2 V2
Thus the final concentrations are (acid) (base)
[H+] = [ HCO3- ] = 1.195 × 10–4 & [CO23 - ] = 4.8 ´ 10-11 Substituting various given values, we get
2 2
113. (b) ˆˆ† Ag+ + Br-
AgBr ‡ˆˆ ≥ V1 = 2.5≥
15 5
Ksp = [Ag+] [Br–]
2 15
For precipitation to occur or V1 = 2.5≥ ≥ = 2.5 × 3 = 7.5 ml
Ionic product > Solubility product 5 2
Then the concentration of BCl in resulting solution is
K sp 5≥10,13
[Br, ] < < < 10,11 given by
[Ag∗ ] 0.05
2
i.e., precipitation just starts when 10–11 moles of KBr ´ 2.5 1
[BCl] = 5 = or 0.1 M
is added to 1l AgNO3 solution 10
10
\ Number of moles of Br– needed from KBr = 10–11
[Total volume = 2.5 + 7.5 = 10 ml]
\ Mass of KBr = 10–11 × 120 = 1.2 × 10–9 g
Kw
114. (b) ˆˆ† Mg ++ + 2 OH -
Mg(OH) 2 ‡ˆˆ Since K h <
Kb
Ksp = [Mg++][OH–]2
1.0 × 10–11 = 10–3 × [OH–]2 1≥10,14
[ Kh < = 10–2
1≥10,12
10,11
[OH, ] < < 10,4
10,3 0.1h 2
or 10,2 <
0.1h 2
Thus K h <
\ pOH = 4 (1, h) (1, h)
\ pH + pOH = 14 or 10–2 – 10–2 h = 0.1 h2
\ pH = 10 or 0.1 h2 + 10–2 h – 10–2 = 0
115. (d) pH = 5 (Solving this quadratic equation for h, we get)
\ [H+] = 10–5
é -b ± b 2 - 4ac ù
ˆˆ† H + + A -
HA ‡ˆˆ êx = ú
Using ê 2a ú
t=0 c 0 0 ë û
teq c (1 – a) ca ca
-10-2 ± (10-2 )2 + 4 ´10-1 ´ 10 -2
[H + ][A - ] (ca ) 2 [H + ]2 h=
Ka = = = 2 ´ 0.1
[HA] c (1 – a ) c - [H + ]
EBD_7327
178 CHEMISTRY

119. (d) Q pH = 1 ; H+ = 10–1 = 0.1 M


,2 ,4 ,3
,10 ° 10 ∗ 4≥10 pH = 2 ; H+ = 10–2 = 0.01 M
=
2≥ 0.1 \ M1 = 0.1 V1 = 1
M2 = 0.01 V2 = ?
,0.01 ° .0001 ∗ 0.004 From
=
0.2 M1V1 = M2V2
0.1 × 1 = 0.01 × V2
-0.01 ± 0.0041 V2 = 10 litres
=
0.2 \ Volume of water added = 10 – 1 = 9 litres
120. (a) The reaction (b) is an endothermic reaction hence
-0.01 ± 0.064 increase in temperature will favour the forward reaction
=
0.2 i.e. formation of products. Further Reaction is occuring
by decrease in volume, hence increase in pressure will
0.54 favour the formation of product.
= [Neglecting the negative term]
0.2
121. (a) ˆˆˆ†
AB5 ‡ˆˆˆ AB3 + B2
= 0.27
2 5
\ [H+] = c.h = 0.1 × 0.27 = 2.7 × 10–2 M ´
[AB3 ][B2 ] 10 10 10 10
Thus the correct answer is [d]. Kc = = = ´ = 0.25
[AB5 ] 4 100 4
10
118. (d) ˆˆ† M + + X -
MX ‡ˆˆ (Where s is the solubility) 122. (d) For 0.01 M HCl, pH = –log[10–2] = 2
s s
Using formula, N1V1 + N2V2 = NV
Then Ksp = s 2 or s = K sp where V = (V1 + V2)
50 × 0.01 + 5 × 1 = N × 55
↑ M 2 ∗ ∗ 2X ,
Similarly for MX 2 ¾¾ 0.5 + 5
N= = 0.1
s 2s 55
pH = –log [10–1] = 1 (pH decreased)
1
é K sp ù 3 123. (b) A2 2A Equilibrium constant is given by
2 3
K sp = s ´ (2 s ) = 4 s or s=ê ú [A]2
ë 4 û Kc =
[A 2 ]
ˆˆ† 3M + + X -3
and for M 3 X ‡ˆˆ Since the value given is very small, hence conc. of products
3s s
is less. It means the reaction is slow.
1 4
124. (d) No. of moles of NaOH = = 0.1
éK ù4 40
Ksp = (3s)3 ´ s = 27 s 4 or s = ê sp ú
ë 27 û [Molecular weight of NaOH = 40]
No. of moles of OH– = 0.1
From the given values of Ksp for MX, MX 2 and
0.1
M3 X, we can find the solubilities of those salts at Concentration of OH– = 1 litre = 0.1Mole / L
temperature, T.
As we know that, [H + ] [OH - ] = 10 -14
Solubility of MX < 4≥10,8 < 2≥10,4
\ [H + ] = 10 -13 (Q OH – = 10 –1 )
1 1
é 3.2 ≥10,14 ù 3 é 32,15 ù 3
Solubility of MX 2 < êê ú or ê ≥10 ú
[Salt]
ú 125. (d) pH = pK a + log10
ëê 4 úû ëê 4 ûú [Acid]
1 For small concentration of buffering agent and for maximum
= éê8≥10,15 ùú 3 or 2´10-5 [salt]
ë û buffer capacity @1
1 [acid]
é 2.7 ≥10,15 ù 4 \ pH = pKa
Solubility of M3 X < êê ú
ú
ëê 27 ûú EXERCISE - 2
2 3 2
1 1. (c) K p = pB (1) pC (1) = pB (2) pC3 (2) = pB(2)
2
(2 pC (1) )3 ;
= é10,16 ù 4 or 10–4
êë úû
pB2 (2) 1 pB (2) 1
Thus the solubilities are in the order MX > M3 X> MX2 hence = , Br =
i.e the correct answer is (d). pB2 (1) 8 pB (1) 2 2
Equilibrium 179

2. (b) For weak acids, Ka = a 2 or a = [ K a ]1/ 2 K=


[ XeO3F2 ][ H 2O]

\ ratio of relative strengths of two acids is [XeO 4 ][ HF]2

a1 æ K1 ö
1/ 2 \From eq. no. (i) and (ii) K = K 2 / K1
=
a 2 çè K 2 ÷ø 10. (c) For the reaction
3. (c) Since for sparingly soluble salts, solubility (s) will be ˆˆ† 2BaO(s ) + O2 (g); DH = + ve.
2BaO2 ( s ) ‡ˆˆ
less than one. Therefore as the powers of s increases In equilibrium k p < PO 2
in Ksp expression, magnitude of Ksp will become smaller.
Hence, the value of equilibrium constant depends only
4. (b) If the solubilities are S1, S2, S3 respectivley, then the
upon partial pressure of O2. Further on increasing
KSP values for them are.
temperature formation of O2 increases as this is an
For A2B, Ksp = 4 S13 [A2B ‡ˆˆ
ˆˆ† 2A+ + B 2–] endothermic reaction. Hence, pressure of O 2 is
dependent on temperature.
For AB, Ksp = S 22 ˆˆ† A+ + B–]
[AB ‡ˆˆ ˆˆ†
(d) For the reaction:- 2NO 2 (g ) ‡ˆ
11. ˆ 2NO(g ) + O 2 (g )
For AB3, Ksp = 27S34 ˆˆ† A3+ + 3B–]
[AB3 ‡ˆˆ Given Kc = 1.8 × 10–6 at 184 ºC
R = 0.0831 kj/mol. k
Since the value of Ksp is same
Kp = Kc (RT)Dng
\ The correct order of solubilities
Kp= 1.8 × 10–6 × 0.0831 × 457 = 6.836 × 10–5
is AB > A2B > AB3.
[ Q 184°C = (273 + 184) = 457 k, Dn = (2 + 1, –1) = 1]
5. (c) Ksp (AgBr) < Ksp (AgCl)
Hence it is clear that Kp > Kc
Therefore, AgBr will precipitate first and at that time all
Cl– will be present. [B ] [C ] éDù
2
12. (a) = 2, = 4 and ê ú = 6
6. (b) K p = (PH2O ) [A] [B] ëCû
Multiply the three equations,
PH2O (at equilibrium) = K p = 10 -2
[D]
Forward reaction will occur if value of PH2O is less 2×4× 6= = Kc
[ A]
than 10–2.
7. (c) Ka × Kb = Kw 13. (a) ˆˆ† NH3 ( g ) + H 2S( g )
NH 4HS( s) ‡ˆˆ
-14 start 0.5 atm 0 atm
K w 10
\ Ka for HX = = = 10 -9 At equib. 0.5 + x atm x atm
Kb 10-5 Then 0.5 + x + x = 2x + 0.5 = 0.84 (given)
pKa = 9
Þ x = 0.17 atm.
[X - ]
pH = pKa + log p NH3 = 0.5 + 0.17 = 0.67 atm ; p H 2S = 0.17 atm
[HX]
0.1 K= p NH3 ´ p H 2S = 0.67 ´ 0.17 atm 2 = 0.1139 = 0.11
pH = 9 + log = 9 – log 3
0.3
14. (d) ˆˆ† 2NH3
N 2 + 3H 2 ‡ˆˆ
8. (d) When volume is increased the conc. decreases & the
equilibrium shifts in the direction where more moles are Kp 1.44 ´10 -5
Dn = 2 – 4 = –2 K c = = ,
formed.
( RT ) Dn (0.082 ´ 773) - 2
9. (d) For the reaction
(R in L.atm.K–1 mole–1).
XeF (g) + H O (g) ‡ˆˆ ˆˆ† XeOF (g) + 2HF (g)
6 2 4 15. (c) 1 N NaOH = 1 M NaOH.
[XeOF4 ][HF] 2
K1 = ....(i) \[OH - ] = 100 ; [H + ] = 10-14 ; pH = 14. 66. (c)
[ XeF6 ][ H 2 O]
1 N NaOH = 1 M NaOH.
and for the reaction
ˆˆ† XeOF (g) + XeF (g)
XeO4 (g) + XeF6 (g) ‡ˆˆ \[OH - ] = 100 ; [H + ] = 10-14 ; pH = 14.
4 2
[XeOF4 ][ XeO3F2 ]
K2 = ....(ii) [salt]
[ XeO 4 ][ XeF6 ] 16. (d) pH = pK a + log
[acid]
For reaction :
For small concentration of buffering agent and for
XeO4 ( g ) + 2HF( g ) ® XeO3F2 ( g ) + H 2O( g )
[salt]
maximum buffer capacity @1
[acid]
EBD_7327
180 CHEMISTRY

\ pH = pKa [ A y + ]x [ B x- ] y
K eq = [ xca]x [ yca] y
Q pKa = 3.58, thus at this state pH = 3.58 =
[ Ax B y ] c (1 - a )
17. (a) A B C
Ka 10 –4 10–6 10–10 xx × c x × a x × y y × c y × a y
= [Q1 - a » 1]
The higher the value of Ka, the stronger the acid and c
the lower is pH. Hence order of pH A < B < C. = x x × y y × a x + y × c x + y -1
18. (c) Strong base has higher tendency to accept the proton. K eq
Increasing order of base and hence the order of a x+ y =
x × y y × c x + y -1
x

accepting tendency is I- < HS- < NH3 < RNH 2 æ 1 ö


æ K eq öçè x + y ÷ø
19. (c) The higher is the tendency to donate proton, stronger a = ç x y x + y -1 ÷
ç x × y ×c ÷
is the acid. Thus the correct order is è ø
R – COOH > HOH > R – OH > CH º CH [CH 3COO - ]
24. (b) pH = pK a + log = pK a ;
depending upon the rate of donation of proton. [CH 3 COOH]
20. (d) For an acid-base indicator
Þ at 50ºC pKa < 4.74
HIn + -
H + In 25. (a) CH3 COOH ƒ CH 3COO - + H +
[H + ][In - ]
\ K In = or [H + ] = K In ´ [HIn ] ca 2
[HIn ] [In - ] Kc =
(1 - a)
[HIn ] a will increase with dilution
or log H + = log K In + log
-
[ In ] 26. (c) 2NO(g) ƒ N2 (g) + O2 (g) ; K1 = 4 × 1030 ...(Given)
1 1
Taking negative on both sides N2 (g) + O2(g) ƒ 2NO (g) ; K ¢ = = .....(i)
K1 4 ´ 1030
[HIn] Divide (i) by 2, we get
, log[H∗ ] < , log K In , log
[In, ] 1 1 1
(ii) N 2 ( g ) + O2 ( g ) ƒ NO( g ) ; K" = K ¢ =
2 2 2 ´ 1015
[ In - ] 1
or we can write pH = pK In + log (iii) NO( g ) + Br2 ( g ) ƒ NOBr( g ) ; K2 = 1.4 mol–1/2 L1/2
[HIn ] 2
...(Given)
[In - ] On adding (ii) and (iii)
or log = pH - pK In
[HIn] 1 1 1
N (g) + O2(g) + Br2 (g) ƒ NOBr (g)
21. (a) The solution formed from isomolar solutions of sodium 2 2 2 2
oxide, sodium sulphide, sodium selenide are H2O, H2S, 1.4
H2 Se & H2Te respectively. As the acidic strength K = K'' × K2 = = 7.0 × 10–16
2 ´ 1015
increases from H2O to H2Te thus pH decreases and
27. (b) KOH, being strong electrolyte, ionizes completely.
hence the correct order of pHs is
Hence [OH–] = 0.01 M
pH1 > pH2 > pH3 > pH4.
which remains constant, even at 50ºC (volume of
22. (c) pH = 8, pOH = 6; [OH - ] = 10 -6 M; solution is supposed to be constant). Hence,
pOH(= –log 0.01 = 2) remains constant. At 50ºC
-6 2 pKw < 14. Hence pH at 50ºC = (pKw– 2) < 12.
Ionic product of Fe(OH)2 = 0.2 × (1´ 10 )
28. (c) CaCO3(s) ƒ CaO (s) + CO2(g)
-13 -16
= 2 ´ 10 > K sp ( = 8.1 ´ 10 ) K = PCO2 = 1 atm (given)
23. (c) The weak electrolyte AxBy dissociates as follows 8400 8400
\ By ln Kp = 7 – Þ 0= 7- [ln 1 = 0]
ˆˆ† xA y + + yB x -
Ax B y ‡ˆˆ T T
c 0 0 initially 8400
c (1–a) xca yca at equilibrium ÞT= = 1200 K = (1200 – 273) = 927°C
7
where, a = degree of dissociation 29. (b) pH = 2 : r1 = k × (10–2)n { [H+] = 10–pH}
c = concentration pH = 1 ; r2 = k × (10–1)n
Equilibrium 181
n Total [H3 O] + ion present in mixture solution
æ 10-1 ö
Given : r2 = 100 r1 Þ ç ÷ = 100 ; 10n = 100 = (10–3 + 10–4 + 10–5) moles
ç 10-2 ÷ Then [H3O]+ ion concentration of mixture solution
è ø
\ n=2 10 -3 + 10 -4 + 10-5 0.00111
30. (c) Milli moles of HCl = 75 × 0.2 = 15; = M= M
Milli moles of NaOH = 25 × 0.2 = 5 3 3
Milli moles of HCl left un-neutralized = 15 – 5 = 10 ; = 0.00037 M = 3.7 ×10–4 M.
35. (b) For the reaction
+ 10
[H3 O ] = = 0.1M ; pH = 1 2AB2 (g) ƒ 2AB(g) + B2 (g)
75 + 25 at equi 2(1- x ) 2x x

31. (a) [H + ] = [OH - ] = 40 ´ 10 -14 = 2.0 ´ 10 -7


Kc =
[ AB]2 [ B2 ] or K = (2 x)2 ´ x
pH = pOH = - log 2 ´ 10 -7 = 6.699
c
[AB2 ]2 {2(1 - x )}2
32. (b) Ksp[Ca(OH)2] = 4.0 ×10–6 =[Ca2+] [OH–]2 = x3 [(1– x) can be neglected in denominator (1– x)
= 0.01 ×[OH–]2 Þ [OH–] = 2×10–2 ; ; 1]
The partial pressure at equilibrium are calculated on
pOH = – log 2×10–2 = 2–log 2; pH = 14–(2–log2) the basis of total number of moles at equilibrium.
= 12 + 2 log 2 Total number of moles = 2 (1–x) + 2x + x = (2 + x)
33. (d) Given, 2(1 - x)
ˆˆ† 2NH 3 ; K1
N 2 + 3H 2 ‡ˆˆ ....(i) \ PAB = ´ P , where P is the total pressure.
2 (2 + x)
ˆˆ† 2NO; K 2
N 2 + O2 ‡ˆˆ ....(ii)
1 2x x
ˆˆ† H 2 O; K 3
H 2 + O2 ‡ˆˆ ....(iii) PAB = ´ P , PB = ´P
2 (2 + x ) 2 (2 + x )
We have to calculate Since x is very small so can be neglected in denominator
4NH 3 + 5O2 ¾¾ ® 4NO + 6H 2O; K = ? Thus, we get
5 PAB = (1 – x) × P PAB = x × P
or 2NH 3 + O 2 ¾¾ ® 2NO + 3H 2 O 2
2 x
PB = ´P
[NO]2 [H 2O]3 2 2
For this equation, K =
[NH3 ]2 [O 2 ]5 / 2 x
[NH3 ] 2
[NO] 2 (PAB )2 ( PB2 ) ( x )2 ´ P2 .P ´
2
but K1 = , K2 = Now, K P = =
(P 2 ) (1 - x )2 ´ P 2
2
[N 2 ] [H2 ]3 [N 2 ] [O2 ]
AB
[H 2O] [H 2O]3
& K3 = ½ or K33 = x 3 .P3
[H 2 ] [O2 ] [H 2 ]3 [O2 ]3/ 2 = [\ 1 – x ; 1 ]
2 ´ 1 ´ P2
K 2 . K 33
Now operate, 1
K1 x3 .P 2.K p æ 2K p ö 3
= or x3 = or x = ç
[NO]2 [H 2 O]3 [N ] [H2 ]3 2 P è P ÷ø
= ´ . 2
[N 2 ] [O2 ] [H 2 ]3 [O2 ]3 / 2 [NH3 ]2 36. (c) Given reactions are
2
[NO] [H 2 O] 3 X ƒ Y+Z ..... (i)
= =K
[NH3 ]2 [O2 ]5 / 2 and A ƒ 2B ......(ii)
Let the total pressure for reaction (i) and (ii) be P1 and
K 2 . K 33 P2 respectively, then
\ K=
K1 KP 9
1
= (given)
34. (b) [H3 O]+ for
a solution having pH = 3 is given by KP2 1
[H3O]+ = 1×10–3 moles/litre After dissociation,
[\ [H3O]+ = 10–pH]
Similarly for solution having pH = 4, X ƒ Y+Z
[H3O]+ = 1 × 10–4 moles/ litre and for pH=5 At equilibrium (1– a) a a
[H3O+] = 1×10–5 moles/ litre [Let 1 mole of X dissociate with a as degree of
Let the volume of each solution in mixture be IL, then dissociation ]
total volume of mixture solution L = (1 + 1 + 1) L Total number of moles = 1– a + a + a
= (1+ a)
=3L
EBD_7327
182 CHEMISTRY

æ1- a ö æ a ö é Salt ù
Thus PX = ç ÷ . P1 ; PY = ç P; 39. (d) pH = p Ka+ log ê
è1+ a ø è 1 + a ÷ø 1 ë Acid úû
æ a ö
PZ = ç .P log é H + ù = log Ka – log é Salt ù
è 1 + a ÷ø 1 ë û êë Acid úû

æ a ö a + é Acid ù
\ K P1 = ç .P ´
è 1 + a ÷ø 1 (1 + a )
. log éë H ùû = log Ka + log ê
ë Salt úû
æ1 - a ö
P1 / ç .P ....... (i) é H + ù = K é Acid ù
è 1 + a ÷ø 1 ë û aê
ë Salt úû
Similarly for A ƒ 2B 0.1
At equilibrium (1– a) 2a = 1.8 × 10 -5 ´ = 9 × 10-6 M
0.2
We have,
2
40. (c) 2A(g) + B(g) ƒ 3C(g) + D(g)
æ 2aP2 ö æ 1 - a ö Mole ratio 2 1 3 1
K P2 = ç / P ........(ii)
è 1 + a ÷ø çè 1 + a ÷ø 2 Molar concentration 1 1 0 0
Dividing (i) by (ii), we get at t = 0
KP 2 KP Equilibrium molar 1– 0.5 1– 0.25 0.75 0.25
1 = a .P1 1 = 1 . P1 concentration = 0.50 = 0.75
2 or
KP 4a .P2 KP 4 P2
2 2 [C ]3[ D] (0.75)3 (0.25)
Kc = =
1 P é KP ù [ A]2 [ B] (0.50) 2 (0.75)
or 9 = . 1 ê\ 1 = 9 ú
4 P2 ê KP 1ú 41. (b) Given [NH3] = 0.3 M, [NH4+] = 0.2 M, Kb = 1.8 × 10–5 .
ë 2 û
P1 36 [salt]
or = or P1 : P2 = 36 : 1 pOH = pK b + log [pKb = –log Kb;
P2 1 [base]
i.e. option (c) is correct answer. pKb = –log 1.8 × 10–5]
37. (c) Given, CH3COOH ƒ CH3COO– + H+ ; \ pKb = 4.74
Ka1 , = 1.5 × 10– 5 ....(i) 0.2
= 4.74 + log = 4.74 + 0.3010 – 0.4771 = 4.56
HCN ƒ H++ CN–; K a 2 = 4.5 × 10–10 0.3
pH = 14 – 4.56 = 9.436
or H+ + CN– ƒ HCN;
42. (c) Ksp = [Ag+] [Cl–]
1 1
K'a 2 = = ...(ii) 1.8 × 10–10 = [Ag+] [0.1]
Ka 2 4.5 ´ 10–10 [Ag+] = 1.8 × 10–9 M
\ From (i) and (ii), we find that the equilibrium constant Ksp = [Pb+2] [Cl–]2
(Ka) for the reaction , 1.7 × 10–5 = [Pb+2] [0.1]2
CN– + CH3COOH ƒ CH3COO– + HCN, is [Pb+2] = 1.7 × 10–3 M
K a = K a1 ´ K a' 2 43. (a) Lets take an example of an acidic buffer CH3COOH and
CH3COONa.
1.5×10 –5 1
= = ´ 105 = 3.33 ´ 10 4 CH3COOH CH3COO – + H + ;
–10 3
4.5×10 CH3COONa CH3COO– + Na+
2+ when few drops of HCl are added to this buffer, the H+
38. ® Ba (aq) + 2 OH - (aq)
(d) Ba (OH) 2 (s) ¾ ¾
pH = 12 or pOH = 2 of HCl immediatly combine with CH3COO– ions to
form undissociated acetic acid molecules. Thus there
[OH - ] = 10 -2 M will be no appreciable change in its pH value. Like wise
Ba(OH) 2 ¾¾ ® Ba 2 + + 2 OH– if few drops of NaOH are added, the OH – ions will
combine with H+ ions to form unionised water molecule.
0.5× 10-2 10 -2
Thus pH of solution will remain constant.
[\ Concentration of Ba 2 + is half of OH - ]
44. (c) ˆˆ† 2SO3 K = 278 (given)
2SO2 + O2 ‡ˆˆ
K sp = [Ba 2+ ] [OH - ]2
æ 1ö
ˆˆ† SO2 + 1 O2
SO3 ‡ˆˆ K' = ç
= [0.5 × 10-2 ] [1 × 10-2 ]2 2 ÷
è Kø
= 0.5 × 10-6 = 5 × 10-7 M 3
Equilibrium 183

3. (c) At the stage of equilibria physical processes does not


1 –2 stop but forward and reverse process occur at the
=
278
= 35.97 ´ 10-4 = 6 × 10 same rate.
4. (b) For the given reaction,
[AB]2 ˆˆ† PCl3 + Cl2
PCl5 ‡ˆˆ
45. ˆˆ† 2AB
(c) A2 + B2 ‡ˆˆ Kc = At 500 K in a closed container,
[A 2 ][B2 ]
[PCl5] = 0.8 × 10–3 mol L–1
[PCl3] = 1.2 × 10–3 mol L–1
(2.8 ´ 10-3 )2 (2.8)2 [Cl2] = 1.2 × 10–3 mol L–1
Kc = = = 0.62
3 ´ 10-3 ´ 4.2 ´ 10-3 3 ´ 4.2
Kc =
[ PCl3 ] [ Cl 2 ] = (1.2 ´ 10-3 ) ´ (1.2 ´ 10-3 )
46. (d) ˆˆ† H + HA
H 2A ‡ˆˆ + -
[ PCl5 ] ( 0.8 ´ 10-3 )
+ - = 1.8 ´ 10-3
\ K = 1.0 × 10 –5 = [H ][HA ] (Given) 2+
Fe3+ + SCN - ‡ˆˆ
ˆˆ† éë Fe (SCN ) ùû
1
[H 2 A] 5. (b)
( Red )
HA- ¾¾
® H+ + A-- When oxalic acid is added to a solution containing
iron (III) nitrate and potassium thiocynate then,
[H + ][A -- ] (Given) equilibrium shifts towards backward direction and
\ K 2 = 5.0 ´ 10 -10 =
[HA - ] intensity of red colour decreases.
Oxalic acid reacts with Fe3+ ions to form a stable
complex ion [Fe (C2 O4)3]3– . Thus, decreasing the
[H + ]2 [A 2 - ]
K= = K1 ´ K2 concentration of Fe3+ ions.
[H 2 A] 6. (a) For the given reaction,
= (1.0 × 10–5) × (5 × 10–10) = 5 × 10–15 3+
é Co ( H 2O ) ù ( aq ) + 4Cl - ( aq ) ‡ˆˆ
ˆˆ†
47. (a) Both Assertion and Reason are correct and Reason is ë 6û
( Pink)
the correct explanation of Assertion.
48. (a) The size of [Al(H2O)6]3+ is smaller than [Mg(H2O)6]2+. [ CoCl4 ]2- ( aq) + 6H2 O (l)
Also, the former possesses more effective nuclear ( Blue)
charge and thus, attracts electron pair from donor more Equilibrium shifts to backward direction on cooling
effectively. This gives rise to relatively strong acidic where as on heating, it shifts to forward direction. Thus,
nature of [Al(H2O)6]3+. reaction is endothermic.i.e., DH > 0.
7. (c) pH of neutral water at 25°C = 7.0
[C ]c [ D ]d At 25°C, [H+] = [OH–] = 10–7 and
49. (c) aA + bB c C + dD, Qc =
[ A]a [ B ]b Kw = [H+] = [OH – ] = 10–14
On heating, Kw increases, i.e., [H+] [OH–] > 10–14
If Qc > Kc, reaction will proceed in the direction of
As [H+] = [OH–]
reactants
\ [H+]2 > 10–14.
If Qc < Kc, reaction will move in direction of products.
or, [H+] > 10–7 M
If Qc = Kc, the reaction mixture is already at equilibrium.
\ pH < 7
50. (c) NaOH is a weaker base than KOH.
As temperature increases, pH of pure water decreases
EXERCISE - 3 hence, it will become less than 7 at 60°C.
8. (d) With the increase in acidity or Ka value of the given
Exemplar Questions acids pH decreases, hence the order of pH value of the
1. (d) Relationship between Kc and Kp is given as : acids will be :
Kp = Kc (RT)Dn hypochlorous acid < acetic acid < formic acid
where, Dn = (number of moles of gaseous products) – (3.8 ´10-8 ) (1.74 ´10-5 ) (1.8 ´10-4 )
(number of moles of gaseous reactants) 9. (a) For the reaction,
For given reaction,
ˆˆ† H + + HS-
H 2S ‡ˆˆ
ˆˆ† NH 3 ( g ) + HCl ( g )
NH 4 Cl ( s ) ‡ˆˆ
Dn = 2 – 0 = 2
[ H + ][ HS- ]
K a1 = … (i)
[ H 2S ]
2. (d) The relationship between DGs and K is :
For the reaction,
DGs = –RT ln K HS- ‡ˆˆ
ˆˆ† H + + S2 -
When G s > 0 it implies DGs must be positive, which [ H + ] [S2- ]
Ka2 = … (ii)
will be possible when Kc is negative i.e., Kc < 1. [ HS- ]
EBD_7327
184 CHEMISTRY

[ H + ] 2 [ S2 - ] 18. For the reaction,


1 1
H 2 ( g ) + I 2 ( g ) ‡ˆˆ
ˆˆ† HI ( g )
K a3 = … (iii) 2 2
[ H 2 S] [ HI ]
Kc = =5
Hence, K a3 = K a1 ´ K a 2
[ H 2 ] [ I2 ]1/ 2
1/ 2

10. (c) According to GN Lewis an acid is a species which Thus, for the reaction,
accepts an electron pair and base which donates an 2HI(g) ƒ H2(g) + I2(g)
electron pair. Since BF3 is an electron deficient species,
hence, it is a Lewis acid.
[ H ][ I ]
K c¢ = 2 22
11. (c) When the concentration of weak base (NH4OH) is [ HI ]
higher than the strong acid (HCl) a mixture of weak æ 1 ö
2
æ 1ö 1
2
base and its conjugate acid is obtained, which acts as K¢c = ç ÷ =ç ÷ = = 0.04
è cø
K è 5 ø 25
basic buffer.
19. (d) Addition of inert gas at constant volume does not affect
NH4OH + HCl NH4Cl + H2O any equilibrium.
Initially 0.1 M 0.05 M 0
NEET/AIPMT (2013-2017) Questions
After reaction 0.05 M 0 0.05 M 20. (c) Solubility of alkali metal is maximum among the following.
12. (d) AgCl, react with aqueous ammonia to form a complex, Among ZnS (1.7 × 10–5) & CuS (8 × 10–37) ZnS has higher
[Ag(NH3)2]+ Cl–. value of Ksp.
13. (a) Given, Ka = 1.74 × 10–5
21. (b) BF3 acts as Lewis acid.
Concentration CH3COOH (C) = 0.01 mol dm–3
[ H+ ] = 22. (a) CaCO3 ¾® Ca 2+ + CO32-
K a .C x x
= 1.74 ´ 10-5 ´ 0.01 = 4.17 ´ 10 -4 CaC2O4 ¾® Ca 2+
+ C 2 O 24-
pH = – log [H+] y y
= – log (4.17 × 10–4) = 3.4 \ [Ca2+] = x + y
14. (c) Given Ka for CH3COOH = 1.8 × 10–5
Kb for NH4OH = 1.8 × 10–5. Now, Ksp (CaCO3) = [Ca2+] [CO32-]
Ammonium acetate is a salt of weak acid and weak base. For or 4.7 × 10 –9 = (x + y) x
such salts similarly, Ksp (CaC2O4) = [Ca2+] [C2O42–]
1 or 1.3 × 10 –9 = (x + y) y
pH = 7 + ( pK a - pK b )
2 On solving, we get
éë - log1.8 ´ 10 -5 ùû - éë - log1.8 ´ 10 -5 ùû [Ca2+] = 7.746 × 10–5 M
= 7+ 23. (a) Kw at 25°C = 1 × 10–14
2
4.74 - 4.74 At 25ºC
= 7+ = 7.00 Kw = [H+] [OH–] = 10–14
2
At 100°C (given)
15. (a) DGs = - RT ln K
Kw = [H+] [OH–] = 55 × 10–14
At the stage of half completion of the reaction,
[A] = [B] Q for a neutral solution
Therefore, K = 1 [H+] = [OH–]
As we know that \ [H+]2 = 55 × 10–14
or [H+] = (55 × 10–14)1/2
DGs = - RT ln K
Q pH = – log [H+]
\ DG – = 0 On taking log on both side
16. (a) In the reaction N 2 (g) + 3H 2 ( g ) ‡ˆˆ
ˆˆ† 2NH 3 ( g ) – log [H+] = –log (55 × 10–14)1/2
If the total pressure at which the equilibrium is 1
pH = - log 55 + 14 log10
established, is increased without changing the 2
temperature, K will remain same but the equilibrium pH = 6.13
concentration will change. [Salt]
17. (b) The order of boiling point of compounds given is 24. (c) pH = pKa + log
[Acid]
Water > acetone > ether
Lower the boiling point, higher is the vapour pressure [Salt]
of the solvent. Hence, the correct order of vapour 5 = 4 + log [Q pKa = – log Ka]
[Acid]
pressure will be Given, Ka = 1 × 10– 4
Water < acetone < ether \ pKa = – log (1× 10– 4) = 4
Equilibrium 185

Now from Handerson equation Ksp = [Ag+] [Br–] = 5.0 × 10–13


[Salt] 5.3 ´ 10 –13
pH = pKa + log [Ag + ] =
[Acid] [Br – ]
Putting the values AgI
[Salt] Ksp = [Ag+] [I–] = 8.3 × 10–17
5 = 4 + log
[Acid] 8.3 ´ 10–17
[Ag + ] =
log
[Salt] [I – ]
= 5–4=1
[Acid]
If we take éë Cr2 O 4 ùû = [Cl ] = [Br ] = [I ] = 1
–2 – – –
Taking antilog
[Salt]/[Acid] = 10 = 10 : 1 than maximum [Ag+] will be required in case of Ag2CrO4.
Ka Ka 31. (b) Equilibrium constant for reaction:
25. (c) a = = 0.037 = ; [Product]
c 0.10
K = 1.6 × 1012 = [Reactant]
K a = (0.037)2 ´ 0.10 = 1.37 ×10–4
The value of K is very high so the system will contain
; 1.4 × 10–4 mostly products at equilibrium.
26. (c) Na2CO3 is a salt of strong base (NaOH) and weak acid 32. (b) DG° = –2.30RTlog K
(H2CO3). On hydrolysis this salt will produce strongly
because at equilibrium DG = 0
basic solution. i.e. pH will be highest (pH > 7) for this
33. (a) Among the given acids, HClO4 is a very strong acid,
sotluion. Others are combination of
completely dissociates
KCl = Strong acid + Strong base
\ [HA] ® 0 in this case, hence cannot be used for
® neutral solution (pH » 7)
acidic buffer.
NaCl = Strong acid + Strong base 34. (a) gm eq of NaOH = 0.1 × V = 0.1V
® neutral solution (pH » 7) gm eq of HCl = 0.01 × V = 0.01V
CuSO4 = Strong acid + wake base gm eq of NaOH > gm eq. HCl
® Acidic solution (pH < 7)
hence resultant solution should be basic, hence from
27. (b) DG = – 2.303 log K the eqn
here K = [Ag+]2 [CO3– –] = Ksp
M1V1 – M2V2 = MV
\ 63.3 × 10– 3 = – 2.303 × 8.314
0.1V – 0.01V = MV
× 298 log Ksp
0.09
63.3 ´ 10- 3 M= = 0.045 = 4.5 × 10–2
\ log Ksp = - = - 11.09 2
5705.8 Now pOH = – log [OH–]
\ Ksp = Antilog (– 11.09) = 8 × 10– 12 = – log 4.5 × 10–2 = 1.34
28. (d) Given reaction is exothermic reaction. Hence according Q pH + pOH = 14
to Le-Chatelier's principle low temperature favours the
\ pH = 14 – 1.34 = 12.65
forward reaction and on increasing pressure equilibrium
will shift, towards lesser number of moles i.e. forward 35. ˆˆ† 2NO(g)
(a) N2(g) + O2(g) ‡ˆˆ
direction.
[NO]2
29. (a) In exothermic reactions on increasing temperature value K<
[N 2 ][O 2 ]
of Kp decreases
\ Kp > Kp¢ (Assuming T1 < T2) 1 1
N 2 ∗ O2 ‡ˆˆ ˆˆ† NO
30. (c) Ag2CrO4 2 2
[NO]
Ksp = [Ag+]2 é Cr2 O4–2 ù = 1.1 × 10–12
ë û Kϒ < 12 12
< K1 2
[N 2 ] [O 2 ]
1.1 ´ 10 –12 36. (b) MY M+ + Y–
[Ag + ] =
éCr2 O4–2 ù 2
KSP = s = 6.2 × 10–13
ë û
AgCl s = 6.2´10-13
Ksp = [Ag+] [Cl–] = 1.8 × 10–10 s = 7.87 × 10–7 mol L–1
1.8 ´ 10 –10 NY 3 N3+ + 3Y–
[Ag+] =
[Cl – ] KSP = s × (3s)3 = 27s4 = 6.2 × 10–13
AgBr
EBD_7327
186 CHEMISTRY

1/4
æ 6.2 ´ 10-13 ö \ K = K2 × K 33 / K1
s= ç ÷
è 27 ø 40. ˆˆ† 2Ag + ( aq ) + C 2O 42 - ( aq )
(c) Ag 2C 2 O 4 (s) ‡ˆˆ
s = 3.89 × 10–4 mol L–1 2s s
\ molar solubility of NY3 is more than MY in water. + 2
KSP = [Ag ] [C2O4 ]2–
37. (b) The presence of large amount of KHSO4 will decrease [Ag+] = 2.2 × 10–4 M
ionisation of H2SO4 that result in lesser ionisation of Given that:
nitric acid and lesser formation of nitronium ion [NO2+]. \ Concentration of C2O42– ions,
Hence the rate of nitration will be slower. -4
38. (d) Clausius – Clapeyron's equation éë C2 O24 - ùû = 2.2 ´ 10 M
2
d ln P DH v
= = 1.1 ´ 10-4 M
dT RT 2 \ KSP = (2.2 × 10–4)2 (1.1 × 10–4)
39. (a)
= 5.324 × 10–12
ˆˆ† 2NH3 ; K1 =
(i) N 2 + 3H 2 ‡ˆˆ
[ NH3 ]2 41. (d) Max. pressure of CO2 = Pressure of CO2 at equilibrium
For reaction,
[ N2 ][ H2 ]3 ˆˆ† SrO(s) + CO2
SrCO3(s) ‡ˆˆ
[ NO]2 Kp = PCO2 = 1.6 atm = maximum pressure of CO2
ˆˆ† 2NO; K 2 =
(ii) N 2 + O 2 ‡ˆˆ
[ N 2 ][ O 2 ] volume of container at this stage.

1
(iii) H 2 + O 2 ¾¾
® H 2 O; K 3 =
[ H 2 O] V=
nRT
…(i)
P
2 [ H 2 ][ O 2 ]1/2 Since container is sealed and reaction was not earlier at
Applying (II + 3 × III – I) we will get equilibrium.
\ n = constant.
5 K
ˆˆˆ
2NH3 + O2 ‡ˆˆ†
ˆ 2NO + 3H 2O; PV 0.4 ´ 20
2 n= = …(ii)
RT RT
[ NO]2
´
[ H 2O ]3 [ NH3 ]2 Put equation (ii) in equation (i)
K=
[ N2 ][ O 2 ] [ H 2 ]3 ´ [ O2 ]3 / 2 [ N 2 ][ H 2 ]3 é 0.4 ´ 20 ù RT
V= ê = 5L
ë RT úû 1.6
v

Redox Reactions
8
Many chemical reactions involve transfer of electrons from one OXIDATION NUMBER
chemical substance to another. These electron-transfer reactions It is defined as the charge which an atom of the element has in its
are termed as oxidation-reduction or Redox reactions. ion or appears to have when present in the combined state with
other atoms
REDUCTION AND OXIDATION
Oxidation number is also called oxidation state.
Reduction Rules for Assigning Oxidation Number
Reduction is a process which involves (i) The oxidation number of all the atoms of different elements in
(i) removal of O/electronegative element their respective elementary states and allotropic forms is taken
D to be zero. For ex: in N2, Cl2, H2, He, P4, S8, O2, O3, Br2, Na,
Ex. 2HgO(s) 2Hg(l) + O2 (g) Fe, Ag, etc. the oxidation number of each atom is zero.
2FeCl3(aq) + H2(g) 2FeCl2 (aq) + 2 HCl(aq) (ii) The O.N. of a monoatomic ion is the same as the charge on it.
(ii) addition of H/electropositive element For ex: ON’s of Na+, Mg2+ and Al3+ ions are +1, +2 and +3
while those of Cl–, S2– and N3– ions are –1, –2 and –3.
Ex. CH2 = CH2(g) + H2(g) H3C – CH3(g) (iii) The O.N. of hydrogen is +1 when combined with non-metals
S + H2 H2S and is –1 when combined with active metals called metal
hydrides such as LiH, KH, MgH2, CaH2, etc.
(iii) involves gain of electrons by a species
(iv) The O.N. of O is –2 in most of its compounds except in
4+ – 2+
Sn + 2e 2Sn peroxides like H2O2, BaO2 etc. where it is –1 and in OF2 and
Oxidation O2F2 where it is +2 and +1.
(v) The O.N. of alkali metals (Li, Na, K, etc.) is always + 1 and
Oxidation is a process which involve
that of alkaline earth metals (Be, Mg, Ca, etc.) is + 2.
(i) addition of O/electronegative element
(vi) In all its compounds, fluorine has an O.N. of –1. Other halogens
Ex. 2 H 2S (g) + O2(g) 2S(s) + 2H 2O (l) (Cl, Br, I) also show O.N. of –1 when they occur as halide ions
Mg(s) + F2(g) MgF2(s) in their compounds.
(vii) The algebraic sum of the O.N. of all the atoms in a compound
(ii) removal of H/electropositive element
must be zero. In polyatomic ion, the algebraic sum of all the
Ex. 2H 2S (g) + O 2(g) 2S(s) + 2H 2O (l) O.N.’s of atoms of the ion must equal the charge on the ion.
2NaI + H2O2 2NaOH + I2 (viii) O.N. of an atom may be fractional negative, zero as well as
positive.
(iii) involves loss of electrons by a species
Applications of Oxidation Number
Ex. Fe2+
(i) To compare the strength of acid and base :
OXIDANT AND REDUCTANT Strength of acid µ Oxidation Number
1
Oxidising Agent or Oxidant : Acceptor of Electron(s) Strength of base µ
Oxidation number
Fluorine is the strongest oxidising agent. (ii) To determine the oxidising and reducing nature :
Reducing agent or reductant: Donor of Electron (s) Oxidising strength µ Oxidation number
In aq. solution, Li is the strongest reducing agent while in non aq. 1
Reducing strength µ
solution, Cs is the strongest reducing agent. Oxidation number
(iii) To determine the molecular formula of compound
EBD_7327
188 CHEMISTRY

TYPES OF REDOX REACTIONS Reduction


G55555555555555 H
Combination Reactions +1 -1 +1 -2 0
It is denoted by
(i) 2 H 2 O 2 ( aq ) ¾¾® 2 H 2 O ( l ) + O2 ( g )
E555555555555555555555 F
A + B ¾¾
®C Oxidation
For such a reaction to be a redox reaction, either A or B or both A 0 -3 +1
(ii) P4 ( s ) + 3OH - ( aq ) + 3H 2 O ( l ) ¾¾
® P H 3 ( g ) + 3H 2 P O 2-
and B must be in the elemental form.
0 +4 -2 0 -3 -1
D
Examples : C ( s ) + O02 ( g ) ¾¾® CO2 ( g ) (iii) Cl 2 (g) + 2OH - (aq) ¾¾ ® Cl O - (aq) + Cl - (aq) + H 2 O (l )

D +2 -3 BALANCING OF REDOX REACTIONS


3Mg0 ( s ) + N02 ( g ) ¾¾® Mg3 N 2 ( s )
There are two methods of balancing redox equations :
Decomposition Reactions Oxidation Number Method
They are the opposite of combination reactions. It leads to the (i) Write the correct formula for each reactant and product.
breakdown of a compound into two or more components at least (ii) Identify atoms which undergo change in O.N. in the reaction
one of which must be in the elemental state. by assigning O.N. to all elements in the reaction.
+1 -2 D 0 0 (iii) Calculate the increase or decrease in O.N. per atom and for
Example : 2 H 2 O ( l ) ¾¾ ® 2 H 2 ( g ) + O2 ( g ) the entire molecule / ion in which it occurs. If these are not
+1 -1 0 0 equal, then multiply by suitable coefficients so that these
D
2 Na H ( s ) ¾¾® 2 Na ( s ) + H 2 ( g ) become equal.
(iv) Add H+ or OH– ions on appropriate side of the reaction so
Displacement Reactions
that the total ionic charges of reactants and products are
In these reactions an ion in a compound is replaced by an ion of equal. In acidic solution, use H+ ions and in basic solution,
another element, i.e. use OH– ions.
X + YZ ¾¾ ® XZ + Y (v) Make the number of H-atoms on the two sides equal by adding
Types of displacement reaction H2O molecules to reactants or products.
(i) Metal displacement Half Reaction Method
In these reactions, a metal in compound is displaced by some (i) Produce unbalanced equation for the reaction in ionic form.
other metal in the elemental state. Examples : (ii) Separate the equation into two half reactions: oxidation half
0 reactionc and reduction half reaction.
(a) +2+4– 2
CuSO4 (aq) + Zn(s) (iii) Balance the atoms other than O and H in each half reaction
individually.
(iv) For reactions occurring in acidic medium, add H2O to balance
0 +2 +4 – 2
O atoms and H+ to balance H atoms. For basic medium, OH–
Cu(s) + ZnSO4 (aq) ions equal in number of H+ are added to both sides of balanced
equation.
+4 – 1 0 (v) Add e–s to one side of half reaction to balance the charges.
(b) TiCl4 (l ) + 2Mg(s) (vi) Add the two half reactions to achieve the overall reaction.
(vii) Verify that the equation contains the same type and number
0 +2 – 1 of atoms and the same charges on both sides of the equation.
Ti(s) + 2MgCl2(s)
ELECTRODE POTENTIAL
(ii) Non-metal displacement The potential of any electrode is the potential difference between
In these reactions, a metal or a non-metal displaces another it and the electrolyte surrounding the electrode. When the ions
non-metal from its compound. are at unit activity and the temperature is 25°C (298K), the potential
0 +1–2 difference is called a standard electrode potential (E°)
(a) 2Na(s) + 2H2O(l)
Important Informations from Standard Reduction
Potentials
+ 1 – 2+ 1 0 (i) It predicts relative reducing strength of reducing agents.
2NaOH (aq) + H2(g) Higher the standard reduction potential of a species, lesser
+ 1 –1 will be its reducing strength.
(b) Zn(s) + 2HCl (aq) (ii) Lesser the standard reduction potential of an element (metal),
greater will be its activity.
+2 – 1 0 (iii) Metal having positive value of E° oxide or negative value of
ZnCl2 (aq) + H2(g) E° red. (above H2 in the activity series) will displace hydrogen
from hydra acids and they themselves will be oxidised.
Disproportion Reactions (iv) An oxidising agent with a higher reduction potential will
When reduction and oxidation takes place on same element of a oxidise any reducing agent with a lower reduction potential.
compound, it is called disproportionation reaction. (v) A reducing agent with a lower reduction potential will reduce
Example : any oxidising agent with a higher reduction potential
Redox Reactions 189

ELECTROCHEMICAL SERIES
The arrangement of various elements in order of increasing values of standard reduction potential is called electrochemical series.
Standard reduction potentials at 298 K (Electrochemical Series)
TABLE
Element Electrode Reduction Reaction Standard Reduction Potential E° (in Volt)
Li Li+ + e– ¾® Li – 3.05
K K+ + e– ¾® K – 2.93
Ba Ba2+ + 2e– ¾® Ba – 2.90
Ca Ca2+ + 2e– ¾® Ca – 2.87
Na Na+ + e– ¾® Na – 2.71
Mg Mg2+ + 2e– ¾® Mg – 2.37
Al Al3+ + 3e– ¾® Al – 1.66
Mn Mn2+ + 2e– ¾® Mn – 1.18
Zn Zn2+ + 2e– ¾® Zn – 0.76
Cr Cr3+ + 3e– ¾® Cr – 0.74
Fe Fe2+ + 2e– ¾® Fe – 0.44
Cd Cd2+ + 2e– ¾® Cd – 0.40
Ni Ni2+ + 2e– ¾® Ni – 0.25
Sn Sn2+ + 2e– ¾® Sn – 0.14
Pb Pb2+ + 2e– ¾® Pb – 0.13
H2 2H+ + 2e– ¾® H2 0.00
Cu Cu2+ + 2e– ¾® Cu 0.34
I2 I2 + 2e– ¾® 2I– 0.53
Hg Hg2+2 + 2e– ¾® 2Hg 0.79
Ag Ag+ + e– ¾® Ag 0.80
Hg Hg+2 + 2e– ¾® Hg 0.85
Br2 Br2 + 2e– ¾® 2 Br– 1.08
Cl2 Cl2 + 2e– ¾® 2 Cl– 1.36
Au Au3+ + 3e– ¾® Au 1.50
F2 F2 + 2e– ¾® 2F– 2.87
190

CONCEPT MAP
CHEMISTRY

EBD_7327
Redox Reactions 191

1. Which of th e following involves transfer of five 9. In the reaction H2S + NO2 ® H2O + NO + S. H2S is
electrons ? (a) oxidised (b) reduced
(a) MnO -4 ® Mn 2+ (b) CrO 24- ® Cr 3+ (c) precipitated (d) None of these
10. The oxidation number of C in CH4, CH3Cl, CH2Cl2, CHCl3
(c) MnO42 - ® MnO 2 (d) Cr2O27- ® 2Cr3+ and CCl4 is respectively –
2. The standard reduction potential of Li+/Li, Ba2+/Ba, Na+/Na (a) + 4, – 2, 0, + 2, + 4
and Mg2+/Mg are – 3.05, – 2.73, –2.71 and – 2.37 volts (b) + 2, 4, 0, – 2, – 4
respectively. Which one of the following is strongest (c) 4, 2, 0, – 2, – 4
oxidising agent? (d) 0, 2, – 2, 4, 4
(a) Li+ (b) Na+ 11. In the reaction 3Mg + N2 ® Mg3N2
(c) Mg 2+ (d) Ba2+ (a) magnesium is reduced (b) magnesium is oxidized
3. Which reaction involves neither oxidation nor reduction? (c) nitrogen is oxidized (d) None of these
(a) CrO24- ¾¾ ® Cr2 O 72- (b) Cr ¾¾ ® CrCl3 12. The element that is easiest to be reduced:
+
(c) Na ¾¾ ® Na (d) 2S2 O3- ¾¾
2
® S4 O62- (a) Ag (b) Fe
4. In the following reaction (c) Cu (d) Sn
13. The conversion of sugar C12H22O11 ® CO2 is
4P + 3KOH + 3H 2 O ¾ ¾® 3KH 2 PO 2 + PH 3 (a) oxidation
(a) phosphorus is both oxidised and reduced. (b) reduction
(b) only phosphorus is reduced. (c) Both oxidation and reduction
(c) phosphorus is not oxidised (d) Neither oxidation nor reduction
(d) None of these 14. When Sn2+ changes to Sn4+ in a reaction
5. H2S acts only as a reducing agent while SO2 can act both as
(a) It loses two electrons (b) It gains two electrons
a reducing and oxidizing agent because
(c) It loses two protons (d) It gains two protons
(a) oxygen is more – ve in SO2
15. Oxidation number of N in HNO3 is :
(b) hydrogen in H2S is more + ve than oxygen
(c) S in SO2 has one oxidation state (a) – 3.5 (b) + 3.5
(d) S in H2S has – 2 oxidation state (c) –5 (d) + 5
6. In the reaction 16. Which of the following cannot act as reducing agent?
(a) NO2 (b) SO2
Cr2 O 72- + 14 H + + 6I - ¾
¾® 2Cr 3+ + 7H 2 O + 3I 2 (c) CO2 (d) ClO2
Which element is reduced 17. Phosphorus has the oxidation state of + 3 in
(a) I (b) O (a) phosphorous acid (b) orthophosphoric acid
(c) H (d) Cr (c) hypophosphorous acid (d) metaphosphoric acid.
7. Which one of the following reaction involves oxidation-
18. In which of the following reactions, there is no change in
reduction ?
valency ?
(a) H 2 + Br2 ® 2HBr
(b) NaBr + HCl ® NaCl + HBr (a) 4 KClO3 ¾
¾® 3KClO4 + KCl

(c) HBr + AgNO3 ® AgBr + HNO 3 (b) SO2 + 2H2S ¾


¾® 2H2O + 3S
(d) 2 NaOH + H 2 SO 4 ® Na 2SO 4 + 2H 2 O
(c) BaO2 + H2SO4 ¾
¾® BaSO4 + H2O2
8. The standard reduction potentials of some electrodes are
E°(K+, K) = – 2.9 V; E°(Zn2+, Zn) = – 0.76 V (d) 3 BaO + O2 ¾ ¾® 2 BaO2.
æ 1 ö 19. Which will reduce zinc oxide to zinc ?
E° ç H + , H 2 ÷ = – 0.0 ; (a) Mg (b) Pb
è 2 ø
(c) Cu (d) Fe
E°(Cu2+, Cu) = + 0.34 V 20. The oxidation number of chromium in potassium dichromate is
The electrode acting as strong reductant is (a) + 6 (b) – 5
(a) K (b) Cu (c) – 2 (d) + 2
(c) Zn (d) H2
EBD_7327
192 CHEMISTRY

21. The metal that does not displace hydrogen from an acid is : 35. The values of standard oxidation potentials of following
(a) Ca (b) Al reactions are given below :
(c) Zn (d) Hg (i) ¾® Zn 2 + + 2e - ; E º = 0.762 V
Zn ¾
22. A compound contains atoms of three elements A, B and C.
If the oxidation number of A is +2, B is +5, and that of C is (ii) ¾® Fe 2 + + 2e - ; E º = 0.440 V
Fe ¾
–2, the possible formula of the compound is :
¾® Cu 2 + + 2e - ; E º = -0.345 V
(iii) Cu ¾
(a) A2(BC3)2 (b) A3(BC4)2
+ -
(c) A3(B4C)2 (d) ABC2 (iv) Ag ¾ ¾® Ag + e ; E º = - 0 .800 V
23. The brown ring complex compound is formulated as Which of the following is most easily reduced
[Fe(H2O)5(NO)]SO4. The oxidation state of iron is : (a) Fe (b) Ag
(a) 1 (b) 2 (c) Zn (d) Cu
(c) 3 (d) 0 36. In which of the following compounds the oxidation number
24. The reaction of carbon is not zero?
2+ + (a) HCHO (b) CH3COOH
Ag (aq) ˆˆ† 2Ag (aq)
+ Ag (s) ‡ˆˆ
(c) C12H22O11 (d) CH3CHO
is an example of : 37. When SO2 is passed through acidified solution of potassium
(a) reduction (b) oxidation
dichromate, then chromium sulphate is formed. The change
(c) disproportionation (d) none of these
in valency of chromium is
25. The oxidation number of sulphur in S 8 , S 2 F2 , H2 S
respectively, are (a) +4 to +2 (b) +5 to +3
(a) 0, +1 and –2 (b) +2, +1 and –2 (c) +6 to +3 (d) +7 to +2
(c) 0, +1 and +2 (d) –2, +1 and –2 38. In which of the following transition metal complexes does
26. What is the oxidation state of P in Ba(H2PO2)2? the metal exhibit zero oxidation state ?
(a) + 1 (b) + 2 (a) [Co(NH3)6]Cl3 (b) [Fe(H 2 O) 6 ]SO 4
(c) + 3 (d) – 1
(c) Ni(CO)4 (d) [Fe(H2O)6]X3
27. Oxidation number of nitrogen in (NH4)2SO4 is
39. In the reaction
(a) –1/3 (b) –1
(c) +1 (d) –3 Ag 2 O + H 2 O 2 ¾¾® 2 Ag + H 2 O + O 2, the H2O2 acts as :
28. Oxidation numbers of chlorine atoms in CaOCl2 are (a) reducing agent (b) oxidising agent
(a) 0, 0 (b) –1, –1 (c) bleaching agent (d) none of the above
(c) –1, +1 (d) None of these 40. In which of the following compounds, the oxidation number
29. The oxidation number of nitrogen in (N2H5)+ is : of iodine is fractional ?
(a) + 2 (b) – 2 (a) IF7 (b) I -3
(c) + 3 (d) – 3
30. The oxidation number of sulphur in Na2S4O6 is (c) IF5 (d) IF3
(a) 1.5 (b) 2.5 41. A metal ion M3+ loses 3 electrons, its oxidation number will
(c) 3 (d) 2 be
31. H2C2O4 + KMnO4 in acidic medium reacts to have change in (a) +3 (b) +6
oxidation number of Mn from : (c) 0 (d) –3
(a) 7 to 5 (b) 2 to 7 42. For the reaction : NH 3 + OCl - ¾¾® N 2 H 4 + Cl - in basic
(c) 5 to 7 (d) 7 to 2
medium, the coefficients of NH3, OCl– and N2H4 for the
32. In which of the compounds does 'manganese' exhibit highest
oxidation number ? balanced equation are respectively
(a) MnO2 (b) Mn 3O4 (a) 2, 2, 2 (b) 2, 2, 1
(c) K2MnO4 (d) MnSO4 (c) 2, 1, 1 (d) 4, 4, 2
33. On reduction of KMnO4 by oxalic acid in acidic medium, the 43. C2H6 (g) + nO2(g) ® CO2(g) + H2O(l)
oxidation number of Mn changes. What is the magnitude of In this equation, the ratio of the coefficients of CO2 and
this change? H2O is
(a) From 7 to 2 (b) From 6 to 2 (a) 1 : 1 (b) 2 : 3
(c) From 5 to 2 (d) From 7 to 4 (c) 3 : 2 (d) 1 : 3
34. Among the following, identify the species with an atom in 44. Oxygen has an oxidation state of +2 in the compound :
+6 oxidation state
(a) H 2 O2 (b) CO2
(a) MnO 4– (b) Cr ( CN ) 36 –
(c) H2 O (d) F2 O
(c) NiF62 – (d) CrO 2 Cl 2
Redox Reactions 193

45. 2MnO -4 + 5H 2 O 2 + 6H + ® 2 Z + 5O 2 + 8H 2 O. In this (a) 2KI + H 2 O 2 ¾


¾® 2 KOH + I 2
reaction Z is (b) 2 FeSO 4 + H 2 SO 4 + H 2 O 2 ¾
¾® Fe 2 (SO 4 ) 3 + 2 H 2 O
(a) Mn +2 (b) Mn +4
(c) MnO2 (d) Mn (c) PbS + 4H 2 O 2 ¾
¾® PbSO 4 + 4H 2 O
46. Which one of th e following reactions in volves
(d) Ag 2 O + H 2 O 2 ¾
¾® 2 Ag + H 2 O + O 2
disproportionation?
(a) 2H2SO4 + Cu ® CuSO4 + 2H2O + SO2 57. Oxidation state of phosphorus in PO34- , P4 O10 and P2 O74 -
(b) As2O3 + 3H2S ® As2S3 + 3H2O
(c) 2KOH + Cl2 ® KCl + KOCl + H2O is :
(d) Ca3P2 + 6H2O ® 3Ca(OH)2 + 2PH3 (a) + 3 (b) + 2
47. The following species will not exhibit disproportionation (c) – 3 (d) + 5
reaction 58. When KMnO4 acts as an oxidising agent and ultimately
(a) ClO– (b) ClO-2 forms MnO 4–2 , MnO2, Mn2O3 and Mn +2, then the number
of electrons transferred in each case respectively is
(c) ClO3- (d) ClO-4
(a) 4, 3, 1, 5 (b) 1, 5, 3, 7
48. The standard electrode potentials of four elements A, B, C (c) 1, 3, 4, 5 (d) 3, 5, 7, 1
and D are –3.05, –1.66, –0.40 and +0.80. The highest chemical 59. The compound that can work both as an oxidising as well as
reactivity will be exhibited by : a reducing agent is
(a) A (b) B (a) KMnO4 (b) H2SO4
(c) C (d) D (c) BaO2 (d) H2O2
49. What is ‘A’ in the following reaction 60. Which of the following is not oxidizing agent?
2Fe3+(aq) +Sn2+(aq) ® 2Fe2+(aq) + A
(a) Sn3+ (aq) (b) Sn4+ (aq) (a) Na+ (b) [CrO4]2–
2+
(c) Sn (aq) (d) Sn –
(c) [MnO4] (d) Ozone
50. In a balance equation H2SO4 + xHI ® H2S + yI2 + zH2O, the 61. Which of the following behaves as both oxidising and
values of x, y, z are reducing agents ?
(a) x = 3, y = 5, z = 2 (b) x = 4, y = 8, z = 5 (a) H2SO4 (b) SO2
(c) x = 8, y = 4, z = 4 (d) x = 5, y = 3, z = 4 (c) H2O (d) HNO3
51. CrO4–2 aq. + 2H+ ® Cr2 O 72- (aq.) + H2O Element oxidised is 62. Which of the following substances acts as an oxidising as
well as a reducing agent?
(a) Oxygen (b) H
(c) Chromium (d) None (a) Na 2O (b) SnCl2
52. Which substance serves as reducing agent in the following (c) Na 2 O 2 (d) NaNO 2
reaction ?
63. Which of the following reactions depict the oxidising
14H+ + Cr2 O 27 - + 3Ni ¾
¾® 2Cr3+ + 7H2O + 3Ni2+ behaviour of H2SO4 :
(a) H2O (b) Ni (a) 2PCl5 + H 2SO4 ¾¾
® 2POCl3 + 2HCl + SO2Cl 2
(c) H+ (d) Cr2 O72,
(b) 2NaOH + H 2SO 4 ¾¾
® Na 2SO 4 + H 2 O
53. Which of these substances is a good reducing agent?
(a) HI (b) KBr (c) NaCl + H 2 SO4 ¾¾
® NaHSO 4 + HCl
(c) FeCl3 (d) KClO
54. Which reaction is not feasible? (d) 2HI + H 2 SO 4 ¾¾® I 2 + SO 2 + 2H 2 O
(a) 2KI + Br2 ® 2KBr + I 2 64. When NaCl is dissolved in water the sodium ion becomes
(b) 2KBr + I 2 ® 2KI + Br2 (a) oxidised (b) reduced
(c) 2KBr + Cl 2 ® 2KCl + Br2
(c) hydrolysed (d) hydrated
65. In the reaction
(d) 2H 2 O + 2F2 ® 4 HF + O 2
HAsO 2 + Sn 2+ ¾¾ ® As + Sn 4 + + H 2O oxidising agent is
55. Co(s) + Cu 2 + (aq) ¾¾® Co 2 + (aq) + Cu(s)
(a) Sn2+ (b) Sn4+
The above reaction is :
(c) As (d) HAsO2
(a) Oxidation reaction
66. A compound contains atoms X,Y,Z; the oxidation number of
(b) Reduction reaction
X is +2, Y is +5 and Z is –2, therefore a possible formula of
(c) Redox reaction
the compound is :
(d) None of these
56. In which of the following reactions, H2O2 acts as a reducing (a) XY1Z2 (b) X 2 (YZ3 )2
agent? (c) X3 (YZ4 )2 (d) X3 (Y4 Z)2
EBD_7327
194 CHEMISTRY

67. Stronger is oxidising agent, more is : (a) KMnO4 (b) H 2 O2


(a) standard reduction potential of that species
(b) the tendency to get it self oxidised (c) BaO2 (d) K 2 Cr2 O7
(c) the tendency to lose electrons by that species 72. The oxidation number of phosphorus in pyrophosphoric
(d) standard oxidation potential of that species acid is
68. Which of the following statements are correct concerning (a) +3 (b) +1
redox properties? (c) +4 (d) +5
(i) A metal M for which E° for the half life reaction 73. Zn gives H2 gas with H2SO4 and HCl but not with HNO3
Mn+ + ne– ‡ˆˆ ˆˆ† M is very negative will be a good because
reducing agent. (a) Zn acts as an oxidising agent when it reacts with HNO3
(ii) The oxidizing power of the halogens decreases from (b) HNO3 is weaker acid than H2SO4 and HCl
chlorine to iodine. (c) in electrochemical series, Zn is above hydrogen
(iii) The reducing power of hydrogen halides increases from (d) NO3– is reduced in preference to hydronium ion
hydrogen chloride to hydrogen iodide 74. The oxidation states of sulphur in the anions SO32–, S2O42–
(a) (i), (ii) and (iii) (b) (i) and (ii) and S2O62– follow the order
(c) (i) only (d) (ii) and (iii) (a) S2 O 6 2 - < S2 O 4 2- < SO 3 2-
69. In reaction of KMnO4 and Mohr’s salt, FeSO4 is oxidised to
(b) S2 O 4 2 - < SO 3 2- < S2 O 6 2-
(a) Fe2+ (b) Fe3+
(c) Fe (d) All of these (c) SO32- < S2 O4 2- < S2O 6 2-
70. The violent reaction between sodium and water is an example
of :
(d) S2 O 4 2 - < S 2 O 6 2- < SO 3 2-
(a) Reduction (b) Oxidation 75. Oxidation numbers of P in PO3– 2– and that
4 , of S in SO 4
(c) Redox reaction (d) Neutralization reaction
of Cr in Cr2 O 72– are respectively
71. The compound that can work both as an oxidizing and
reducing agent is : (a) + 3, + 6 and + 5 (b) + 5, + 3 and + 6
(c) – 3, + 6 and + 6 (d) + 5, + 6 and + 6

1. One mole of N2H4 loses 10 moles of electrons to form a new 0.9V respectively. Which of these is best oxidising agent and
compound, y. Assuming that all nitrogen appear in the new reducing agent respectively –
compound, what is the oxidation state of nitrogen in y (There (a) D2+/D and B2+/B (b) B2+/B and D2+/D
is no change in the oxidation state of hydrogen ) 2+
(c) D /D and C /C 2+ (d) C2+/C and D2+/D
(a) –1 (b) –3 6. Oxidation state for nitrogen is incorrectly given for compound
(c) +3 (d) +5 oxidation state
2. Which of the following reaction involves neither oxidation (a) [Co( NH 3 )5 Cl]Cl 2 0
nor reduction
(b) NH2OH –1
(a) CrO24- ® Cr2O72- (b) Cr ® CrCl3 (c) (N2H5)2SO4 –2
(c) Na ® Na + (d) 2S 2 O 32 - ® S4 O 26 - (d) Mg3N2 –3
7. FeS2 + O2 ® Fe2O3 + SO2
3. Among NH3, HNO3, NaN3 and Mg3N2 the number of
In the above equation the number of electrons lost by one
molecules having nitrogen in negative oxidation state is
molecule of FeS2 are –
(a) 1 (b) 2
(a) 6 (b) 2
(c) 3 (d) 4
(c) 1 (d) 11
4. The correct decreasing order of oxidation number of oxygen
8. The number of moles of KMnO4 that will be completely
in compounds BaF2, O3, KO2 and OF2 is :
reduced by one mole of ferrous oxalate in acidic medium –
(a) BaO2 > KO2 > O3 > OF2
(a) 2/5 (b) 3/5
(b) OF2 > O3 > KO2 > BaO2
(c) 4/5 (d) 1
(c) KO2 > OF2 > O3 > BaO2
9. The oxide, which cannot act as a reducing agent, is
(d) BaO2 > O3 > OF2 > KO2
5. Standard electrode potentials of redox couples (a) NO2 (b) SO2
A2+/A, B2+/B, C/C2+ and D2+/D are 0.3V, – 0.5V, – 0.75V and (c) CO2 (d) ClO2
Redox Reactions 195

10. In which of the following reaction, oxidation number of N 18. When Cl2 gas reacts with hot and concentrated sodium
remains unchanged ? hydroxide solution, the oxidation number of chlorine changes
(a) 2NO2 + H2O ¾¾ ® HNO3 + HNO2 from :
(b) N2 + O2 ¾¾ ® 2NO (a) zero to +1 and zero to –5
(c) 2NO2 ¾¾ (b) zero to –1 and zero to +5
® N2O4
(c) zero to –1 and zero to +3
(d) NH4NO2 ¾¾ ® N2 + 2H2O
(d) zero to +1 and zero to –3
11. Which of the following statements is not correct?
19. Standard reduction potentials of the half reactions are
(a) CaOCl2 is a mixed salt of two acids HCl and HOCl. given below :
(b) Oxidation number of chlorine in CaOCl2 is zero since it F2(g) + 2e– ® 2F– (aq); E° = + 2.85 V
exists as Cl2
Cl2(g) + 2e– ® 2Cl–(aq); E° = + 1.36 V Br 2(l) + 2e– ®
(c) Oxidation number of chlorine is – 1 since it exists as
2Br–(aq); E° = + 1.06 V I2(s) + 2e– ® 2I–(aq); E° = + 0.53 V
Cl–
The strongest oxidising and reducing agents respectively
(d) Oxidation number of chlorine is + 1 since it exists as are :
ClO– (a) F2 and I– (b) Br2 and Cl–
12. The following redox reaction is balanced by which set of (c) Cl2 and Br – (d) Cl2 and I2
coefficients ? 20. The compound YBa2Cu3O7 which shows superconductivity
aZn + bNO3- + cH + ® dNH 4+ + eH 2 O + fZn 2+ has copper in oxidation state .......... Assume that the rare
earth element Yttrium is in its usual +3 oxidation state
a b c d e f
(a) 3/7 (b) 7/3
(a) 1 1 10 1 3 1 (c) 3 (d) 7
(b) 2 2 10 2 3 2 21. The standard reduction potential values of three metallic
(c) 4 2 10 1 3 4 cations, X, Y and Z are 0.52,– 3.03 and – 1.18 V respectively.
(d) 4 1 10 1 3 4 The order of reducing power of the corresponding metals is
13. The oxidation states of the most electronegative element in (a) Y > Z > X (b) X > Y > Z
the products of the reaction, BaO2 with dil. H2SO4 is (c) Z > Y > X (d) Z > X > Y
(a) 0 and –1 (b) –1 and –2 22. Consider the following reaction :
(c) –2 and 0 (d) –2 and –1 z
14. In haemoglobin the iron is in the following oxidation state : xMNO4 - + yC2 O42 - + zH + ® xMn 2 + + 2yCO 2 + H 2O
2
(a) + 4 (b) + 2 The value’s of x, y and z in the reaction are, respectively :
(c) + 3 (d) + 1 (a) 5, 2 and 16 (b) 2, 5 and 8
15. In which of the following reactions, H2SO4 acts as an acid as (c) 2, 5 and 16 (d) 5, 2 and 8
well as an oxidant? DIRECTIONS for Qs. 23 to 25 : These are Assertion-Reason
(a) C12 H 22 O11 ¾H SO 4
¾2 ¾¾ ® 12C + 11H 2 O type questions. Each of these question contains two statements:
Statement-1 (Assertion) and Statement-2 (Reason). Answer these
(b) S + 2 H 2SO 4 ¾
¾® 3SO 2 + 2 H 2 O questions from the following four options.
(c) Cu + 2H 2SO 4 ¾ ¾® CuSO 4 + SO 2 + 2 H 2 O (a) Statement- 1 is True, Statement-2 is True, Statement-2 is a
(d) NaHCO 3 + H 2SO 4 ¾ ¾® NaHSO 4 + H 2 O + CO 2 correct explanation for Statement -1
16. Point out the correct statement of the following about (b) Statement -1 is True, Statement -2 is True ; Statement-2 is
Na2S4O6. NOT a correct explanation for Statement - 1
(a) Average oxidation number of S atom is +2 (c) Statement - 1 is True, Statement- 2 is False
(b) Oxidation number of two S atoms is zero each and that of (d) Statement -1 is False, Statement -2 is True
other two is +5 each 23. Statement-1 : Stannous chloride is a powerful oxidising agent
(c) Oxidation number of two S atoms is +1 each and that of which oxidises mercuric chloride to mercury.
other two is +4 each Statement-2 : Stannous chloride gives grey precipitate with
(d) None of these mercuric chloride, but stannic chloride does not do so.
17. The pair of compounds in which both the metals are in the 24. Statement-1 : HClO4 is a stronger acid than HClO3.
highest possible oxidation state is Statement-2 : Oxidation state of Cl in HClO4 is +VII and in
HClO3 +V.
(a) [Fe(CN ) 6 ]4 - , [Co(CN ) 6 ]3-
25. Statement-1 : In a reaction
(b) CrO2 Cl 2 , MnO-4 Zn(s) + CuSO4 (aq) ® ZnSO4(aq) + Cu(s)
Zn is a reductant but itself get oxidized.
(c) TiO 3 , MnO 2 Statement-2 : In a redox reaction, oxidant is reduced by
(d) [Co(CN) 6 ]3- , MnO3 accepting electrons and reductant is oxidized by losing
electrons.
EBD_7327
196 CHEMISTRY

Exemplar Questions (a) The oxidation number of hydrogen is always + 1


1. Which of the following is not an example of redox reaction? (b) The algebraic sum of all the oxidation numbers in a
(a) CuO + H 2 ¾¾ ® Cu + H 2 O
compound is zero
(c) An element in the free or the uncombined state bears
(b) Fe2 O3 + 3CO ¾¾ ® 2Fe + 3CO 2 oxidation number zero
(c) 2K + F2 ¾¾ ® 2KF (d) In all its compounds, the oxidation number of fluorine
(d) BaCl 2 + H 2SO 4 ¾¾ ® BaSO 4 + 2HCl is –1
Redox reactions are those reactions which involve change 7. In which of the following compounds, an element exhibits
in oxidation number of the reactants. two different oxidation states?
– (a) NH2OH (b) NH4NO3
2. The more positive the value of E , the greater is the tendency (c) N2H4 (d) N3H
of the species to get reduced. Using the standard electrode 8. Which of the following arrangements represent increasing
potential of redox couples given below find out which of the oxidation number of the central atom?
following is the strongest oxidising agent.
– – (a) CrO2- , ClO3- , CrO24 - , MnO4-
E values : Fe3+/Fe2+ = + 0.77V ; I2(s)/I = + 0.54;
Cu2+/Cu = + 0.34; Ag+/Ag = 0.80 V (b) ClO3- , CrO 42 - , MnO -4 , CrO -2
(a) Fe3+ (b) I2(s)
(c) CrO2- , ClO3- , MnO4- , CrO42 -
(c) Cu2+ (d) Ag+
– (d) CrO 24 - , MnO -4 , CrO -2 , ClO 3-
3. E values of some redox couples are given below. On the
basis of these values choose the correct option. 9. The largest oxidation number exhibited by an element
– depends on its outer electronic configuration. With which
E values: Br2/Br – = + 1.09 of the following outer electronic configurations the element
Ag+ / Ag(s) = + 0.80
will exhibit largest oxidation number ?
Cu2+/Cu(s) = + 0.34; I2 (s) / I – = + 0.54
(a) 3d1 4s2 (b) 3d2 4s2
(a) Cu will reduce Br – (b) Cu will reduce Ag
(c) 3d5 4s1 (d) 3d5 4s2
(c) Cu will reduce I – (d) Cu will reduce Br 2
10. Identify disproportionation reaction
4. Using the standard electrode potential, find out the pair
(a) CH 4 + 2O 2 ¾¾ ® CO 2 + 2H 2 O
between which redox reaction is not feasible.
– (b) CH 4 + 4Cl 2 ¾¾ ® CCl4 + 4HCl
E values: Fe3+/ Fe2+ = + 0.77; I2/I– = + 0.54; (c) 2F2 + 2OH - ¾¾
Cu2+ / Cu = +0.34; Ag+ / Ag = + 0.80 V ® 2F - + OF2 + H 2 O
-
(a) Fe3+ and I– (b) Ag+ and Cu (d) 2NO 2 + 2OH ¾¾ ® NO 2- + NO 3- + H 2 O
3+
(c) Fe and Cu (d) Ag and Fe3+ 11. Wh ich of the following elemen ts does n ot show
Determine the E°cell of the four redox reactions. If the value disproportionation tendency?
of E°cell of a reaction is negative, that reaction will not take (a) Cl (b) Br
place. (c) F (d) I
5. Thiosulphate reacts differently with iodine and bromine in
the reactions given below. NEET/AIPMT (2013-2017) Questions
12. Standard reduction potentials of the half reactions are given
2S2 O32 - + I 2 ® S4 O62 - + 2I - below :
S2 O32 - + 2Br 2 + 5H 2 O ® 2SO24 - + 2Br - + 10H + F2(g) + 2e– ® 2F– (aq); E° = + 2.85 V
Cl2(g) + 2e– ® 2Cl–(aq); E° = + 1.36 V
Which of the following statements justifies the above dual Br2(l) + 2e– ® 2Br–(aq); E° = + 1.06 V
behaviour of thiosulphate? I2(s) + 2e– ® 2I–(aq); E° = + 0.53 V
(a) Bromine is a stronger oxidant than iodine The strongest oxidising and reducing agents respectively
(b) Bromine is a weaker oxidant than iodine are : [2012 M]
(c) Thiosulphate undergoes oxidation by bromine and (a) F2 and I– (b) Br2 and Cl–
reduction by iodine in these reactions (c) Cl2 and Br– (d) Cl2 and I2
(d) Bromine undergoes oxidation and iodine undergoes 13. A mixture of potassium chlorate, oxalic acid and sulphuric
reduction in these reactions acid is heated. During the reaction which element undergoes
6. The oxidation number of an element in a compound is maximum change in the oxidation number ? [2012]
evaluated on the basis of certain rules. Which of the (a) S (b) H
following is incorrect in this respect? (c) Cl (d) C
Redox Reactions 197

14. When Cl2 gas reacts with hot and concentrated sodium 21. Which of the following involves a redox reaction?
hydroxide solution, the oxidation number of chlorine changes (a) Reaction of H2SO4 with NaOH [1997]
from : [2012] (b) Production of ozone from oxygen in the atmosphere by
(a) zero to +1 and zero to –5 lightning
(b) zero to –1 and zero to +5 (c) Production of nitrogen oxides from nitrogen and oxygen
(c) zero to –1 and zero to +3 in the atmosphere by lightning
(d) zero to +1 and zero to –3 (d) Evaporation of water
15. Oxidation numbers of P in PO3– 2– 22. The oxide, which cannot act as a reducing agent, is
4 , of S in SO 4 and that of Cr
[1995]
in Cr2 O 72– are respectively [2009] (a) NO2 (b) SO2
(a) + 3, + 6 and + 5 (b) + 5, + 3 and + 6 (c) CO2 (d) ClO2
(c) – 3, + 6 and + 6 (d) + 5, + 6 and + 6 23. The loss of electron is termed as [1995]
16. The oxidation states of sulphur in the anions SO32–, S2O42– (a) oxidation (b) reduction
and S2O62– follow the order [2003] (c) combustion (d) neutralization
(a) S 2 O 6 2 - < S 2 O 4 2- < SO 3 2- 24. The oxidation number of chromium in potassium dichromate
is [1995]
(b) S2 O 4 2 - < SO 3 2- < S2 O 6 2- (a) + 6 (b) – 5
(c) SO32- < S2 O4 2- < S2O 6 2- (c) – 2 (d) + 2
25. In which of the following reactions, there is no change in
(d) S2 O 4 2 - < S 2 O 6 2- < SO 3 2- valency ? [1994]
17. Zn gives H2 gas with H2SO4 and HCl but not with HNO3 (a) 4 KClO3 ¾ ¾® 3KClO4 + KCl
because [2002] (b) SO2 + 2H2S ¾ ¾® 2H2O + 3S
(a) Zn acts as an oxidising agent when it reacts with HNO 3 (c) BaO2 + H2SO4 ¾ ¾® BaSO4 + H2O2
(b) HNO3 is weaker acid than H2SO4 and HCl
(d) 3 BaO + O2 ¾ ¾® 2 BaO2.
(c) In electrochemical series, Zn is above hydrogen
26. Phosphorus has the oxidation state of + 3 in
(d) NO3- is reduced in preference to hydronium ion (a) Phosphorus acid [1994]
18. A compound contains atoms of three elements A, B and C. If (b) Orthophosphoric acid
the oxidation number of A is +2, B is +5, and that of C is –2, (c) Hypophosphorus acid
the possible formula of the compound is : [2000] (d) Metaphosphoric acid.
(a) A2(BC3)2 (b) A3(BC4)2 27. Which substance serves as a reducing agent in the following
(c) A3(B4C)2 (d) ABC2 reaction ? [1994]
19. The following redox reaction is balanced by which set of
coefficients ? [1999] 14H+ + Cr2 O 27 - + 3Ni ¾
¾®
- + +
aZn + bNO3 + cH ® dNH 4 + eH 2O + fZn 2+ 2Cr3+ + 7H2O + 3Ni2+
a b c d e f (a) H2O (b) Ni
(a) 1 1 10 1 3 1 (c) H+ (d) Cr2 O 72,
(b) 2 2 10 2 3 2 28. The oxidation state of Cr in is [1988]
(c) 4 2 10 1 3 4 (a) + 5 (b) + 3
(d) 4 1 10 1 3 4 (c) + 6 (d) + 7
20. The oxidation number of phosphorus in pyrophosphoric acid
is [1999]
(a) +3 (b) +1
(c) +4 (d) +5
EBD_7327
198 CHEMISTRY

Hints & Solutions


EXERCISE - 1 11. (b) In the given reaction oxidation state of Mg is changing
from 0 to +2 while in nitrogen it is changing from 0 to
1. (a) O.N. of Mn in MnO –4 is +7 and in Mn 2 + it is +2. The
– 3. So oxidation of Mg and reduction of nitrogen takes
difference is of 5 electrons. place.
2. (c) Higher values of reduction potential indicates the 12. (a) Ag is easiest to be reduced among the given choices
stronger oxidising agent. So, Mg2+ having reduction because its reduction potential is highest among the
potential –2.37 V, is the strongest oxidising agent among given choices.
the given choices. 13. (a) In this reaction oxidation occur.
3. (a) Ox. no. of Cr on both side is + 6. 14. (a) Sn2+ ® Sn4+ + 2e–. In this reaction Sn2+ change in
4. (a) 4P + 3KOH + 3H 2 O ® KH 2 PO 2 + PH 3 Sn4+ it is called an oxidation reaction.
O.N of P = 0, In KH2PO2 it is + 1, In PH3 it is –3. 15. (d) Let the oxidation no. of N in HNO3 = x
Hence P is oxidised and reduced. \ 1 + x + (3 × – 2) = 0
5. (d) H2S, the oxidation state of S is – 2. So it cannot accept \ x= +5
more electrons because on accepting 2 electrons S 16. (c) In all the given compounds oxidation number of non
accquires a noble gas configuration. So, it can acts metal is + 4. As C belongs to group IV and it is in its
only as a reducing agent by loosing electron. On the maximum oxidation state. So, reduction in oxidation
other hand, the oxidation state of S in SO2 is + 4 which number of nonmetal is not possible only in CO2. As we
is an intermediate oxidation state of sulphur so it can know that reduction is always accompained by an
reduce as well oxidise. increase in oxidation number of reducing agent. So,
6. (d) 2I- ® I2 is oxidation (loss of electrons) ; Cr (+6) CO2 cannot acts as reducing agent among the given
changes to Cr (+3) by gain of electrons. Hence Cr is choices.
reduced. 17. (a) O.N. of P in H3PO3 (phosphorous acid)
7. (a) In a redox reaction, one molecule is oxidised and other 3 × 1 + x + 3 × (– 2) = 0 or x = + 3
molecule is reduced i.e. oxidation number of reactants In orthophosphoric acid (H3PO4) O.N. of P is + 5, in
are changed. hypophosphorous acid (H3 PO2 ) it is + 1 while in
0 0 +1 -1 metaphosphoric acid (HPO3), it is + 5,
H 2 + Br 2 ¾¾ ® 2 HBr
∗2 ,1 ∗1 ∗6 ,2 ∗2 ∗6 ,2 ∗1 ,1
Here H2 is oxidised and Br 2 is reduced, thus it is 18. (c) Ba O2 ∗ H 2 S O4 ¾¾
↑ Ba S O4 ∗ H 2 O2
oxidation-reduction reaction. In this reaction, none of the elements undergoes a change
8. (a) The electrode for which the standard reduction in oxidation number or valency.
potential values are more negative are strong 19. (a) Mg which is placed at higher position in electrochemical
reductans. So, K electrode (E°red = – 2.9V) acts as strong series will reduce zinc oxide to zinc.
reductant among the given choices. 20. (a) Let x = oxidation no. of Cr in K2Cr2O7.
9. (a) In this reaction H2S is oxidised because the oxidation \ (2 × 1) + (2 × x) + 7 (– 2) = 0
state of ‘S’ change from – 2 to 0. or 2 + 2x – 14 = 0 or x = + 6.
10. (a) Let x1 be the oxidation state of C in CH4 21. (d) Hg which lies below the hydrogen in electrochemical
\ x1 + 4 (–1) = 0 Þ x1 – 4 = 0 series cannot displace hydrogen from acid.
Þ x1 = 4 22. (b) Oxidation number of a compound must be 0. Using the
Similarly, let x2 be oxidation state of C in CH3Cl. values for A, B and C in the four options, we find that
\ x2 + 3 (+ 1) + 1 (– 1) = 0 A3(BC4)2 is the answer.
Þ x2 + 3 – 1 = 0 Þ x2 + 2 = 0 Check : (+2)3 + [(+5)+4(–2)]2 = 6 + (5–8)2 = 0
Þ x2 = – 2 23. (b) Sum of oxidation state of all atoms in neutral compound
Let x3 be oxidation state of C in CH2Cl2 is zero.
\ x3 + 2(+ 1) + 2(– 1) = 0 Þ x3 = 0 Let the oxidation state of iron in the complex ion
Simlary, for CHCl3, 2-
[Fe(H2O)5(NO)]2+2SO4 be x; then x + 5 × 0 + 0= + 2.
\ x4 + 1(+ 1) + 3(– 1) = 0 \ x=+2
Þ x4 + 1 – 3 = 0 24. (d) The given reaction can be written as
x4 = 2
and for CCl4 Oxidation occurs
\ x5 + 4 (– 1) = 0 Þ x5 = 4 2+
Ag (aq) + Ag (s)
+ +
Ag (aq) + Ag (aq)
So, option (a) is correct choice.
reduction occurs
Redox Reactions 199

As shown above, one of the reactin g species 44. (d) In H2O2 : Þ 2 × (+1) + 2 × x = 0
undergoes reduction and other undergoes oxidation Þ x = –1
to give a same speices. These type of reactions are In CO2 : Þ 4 + 2x = 0 Þ x = –2
called comproportion reactions. In H2O : Þ 2 × (+1) + x = 0 Þ x = –2
25. (a) (i) Oxidation state of element in its free state is zero. In F2O : Þ 2 × (–1) + x = 0 Þ x = +2
(ii) Sum of oxidation states of all atoms in compound is
zero. 45. (a) 2MnO4– + 5H 2 O 2 + 6H + ® 2Mn 2 + + 5O2 + 8H 2 O.
O.N. of S in S8 = 0; O.N. of S in S2F2 = + 1; 46. (c) A reaction, in which a substance undergoes
O.N. of S in H2S = –2; simultaneous oxidation and reduction, is called
26. (a) Let oxidation state of P in Ba(H2PO2)2 be x disproportionation reaction. In these reactions, the same
\ (+ 2) + 2 [2 (+ 1) + x + 2 (– 2)] = 0 substance simultaneously acts as an oxidising agent
Þ 2 + 2 [2 + x – 4] = 0 an d as a reducing agent. Here Cl undergoes
Þ 2 + 2 [x – 2] = 0 simultaneous oxidation and reduction.
Þ x–2=–1 Þ x=+1 2KOH + Cl 2 ® KCl + KOCl + H 2O.
0 -1 +1
27. (d) (NH4)2 SO4 is split into ions. NH +4 . Let O.N. of N be x
47. (d) In disproportionation reaction, one element of a
then, 1 × (x) + 4 × (+1) = 1 \ x = –3 compound will simultaneously get reduced and
28. (c) CaOCl2 or Ca(OCl)Cl is the mixed salt of Ca(OH)2 with oxidised. In ClO-4 , oxidation number of Cl is + 7 and it
HCl and HOCl.
29. (c) Let O.N. of Nitrogen be x. can not increase it further. So, ClO-4 will not get
oxidised and so will not undergo disporportionation
6
2x – 5 = + 1 Þ x = =3 reaction.
2 48. (a) Standard electrode potential i.e. reduction potential of
30. (b) Na2S4O6 A is minimum (–3.05V) i.e. its oxidation potential is
Let O.N. of S be x then 2 × (+1) + 4 × (x) + 6 × (–2) = 0 maximum which implies ‘A’ is maximum reactive
\ x = 2.5. chemically.
31. (d) MnO-4 changes t1o Mn 2 + Reduction
49. (b)
MnO-4 + 8H + + 5e = Mn 2 + + 4H 2 O
2Fe3+ + Sn 2 + ® 2Fe2 + + Sn 4 +
Oxidation No of Mn charges from + 7 to + 2
32. (c) O.N. of Mn in K2MnO4 is +6 Oxidation
33. (a) In acidic medium MnO –4 changes to Mn2+, hence O.N. 50. (c) The value of x, y, z are 8, 4, 4 respectively hence the
reaction is
changes from +7 to +2.
H 2SO4 + 8HI ® H 2S + 4I2 + 4H 2O
34. (d) MnO –4 (O.S. of Mn +7); Cr (CN)36– (O.S. of Cr +3),
+6 -2 +6
51. (d) Cr O4 ( aqu.) + 2H +¾¾® Cr2 O -7 2 ( aqu.) + H 2 O None of
NiF62 – (O.S. of Ni +4) and CrO2Cl2 (O.S. of Cr +6)
the species is lossing e–
35. (b) As the oxidation potential of Ag is the lowest among the
\ none is showing oxidation.
given elements, hence it would be reduced most readily.
52. (b) The compound undergo oxidation itself and reduces
36. (d) O.N. of carbon in CH3CHO is –1; in other cases it is
others is known as reducing agent. In this reaction O.
zero.
N. of Ni changes from 0 to + 2 and hence Ni acts as a
37. (c) K2Cr2O7 + 3SO2 + 4H2SO4 ® reducing agent.
K 2 SO 4 + Cr2 (SO 4 )3 + 3SO 3 + 4H 2 O 53. (a) All the given compounds ionise in solution as
O.N.of chrominum changes from +6 to +3 HI ¾¾ ® H + + I-
38. (c) In metal carbonyls metal always has O.N. zero. ® K + + Br -
KBr ¾¾
39. (a) H2O2 is converting Ag2O into Ag,so it is reducing ® Fe3+ + 3Cl -
FeCl3 ¾¾
Ag2O. Hence it is a reducing agent here. KClO ¾¾ ® K + + ClO -
40. (b) O.N. of iodine in I3- is –1/3 A reducing agent is one which get oxidised easily and
41. (b) M3+ on losing 3 elections will become M+6 and O.N. = + 6. in this process, reducing agent provides the electrons
42. (c) The balanced equation : for reduction of the other substanc. Out of these given
ions I–, Br– and Cl– are in –1 oxidation state whereas in
2 NH 3 + OCl - ¾
¾® N 2 H 4 + Cl - + H 2 O
ClO–, Cl is in +1 oxidation state. ClO– ion can get,
43. (b) The balanced equation is oxidised in solution to ClO2, ClO3–ClO4– ions increasing
2C2 H6 + 7O2 ® 4CO 2 + 6H 2O. the oxidation state to +4, +5 and +7 state which are
Ratio of the coefficients of CO2 and H2O is 4 : 6 or 2 : 3. more stable. Hence, it acts as a good reducing agent.
EBD_7327
200 CHEMISTRY

54. (b) 2KBr + I 2 ¾


¾® 2KI + Br2 Stronger reducing agent Þ Easy to oxidise

reaction is not possible because Br– ion is not oxidised ß


Lower reduction potential Ü higher oxidation potential
in Br2 with I2 due to higher electrode (oxidation)
potential of I2 than bromine. (ii) Element F Cl Br I
55. (c) Co(s) + Cu 2+ (aq) ¾¾
® Co 2 + (aq) + Cu(s) Reduction potential +2.87 +1.36 +1.06
This reaction is a redox reaction as Co undergoes +0.54
oxidation whereas Cu+2 undergoes reduction. (E° volt)
56. (d) In (a), (b) and (c) H2O2 acts as oxidising agent. As reduction potential decreases from fluorine to iodine,
57. (d) In PO34- : x + 4(–2) = –3 Þ x = +5 oxidising nature also decreases from fluorine to iodine.
(iii) The size of halide ions increases from F – to I –. The
In P4O10 : 4x + 10(–2) = 0 Þ x = +5 bigger ion can loose electron easily. Hence the reducing
4x – 20 = 0 4x = 20 nature increases from HF to HI.
In P2 O74 - : 2x + 7(–2) = 4 Þ x = +5 69. (b) 2MnO 4 - + 10Fe 2 + + 16H + ¾¾
®
58. (c) In KMnO4 the O.N. of Mn is +7, in MnO4–2 +6, in MnO2 10Fe3+ + 2Mn 2+ + 8H 2O
70. (c) The violent reaction between sodium and water is an
+4 in Mn2O3 +3 and in Mn2+ is +2. The difference being
example of redox reaction :
1, 3, 4 and 5 respectively.
2Na + 2H 2 O ¾¾
® 2NaOH + H 2
59. (d) In H2O2 the O.N. of O is –1 which can be increased to 0 (Oxd. state) 0 +1 +1 0
or decresed to –2 hence H2O2 can work as reducing In this reaction, sodium (Na) is oxidised to NaOH while
and oxidising agent. H2O is reduced to H2.
60. (a) Na+ has a stable configuration of inert gas Ne, thus it 71. (b) H2O2 can act both as an oxidizing and reducing agent,
can neither lose nor gain e. hence does not undergo as oxygen has intermediate oxd. state in it.
oxidation or reduction. 72. (d) Pyrophosphoric acid H4P2O7
61. (b) In SO2 the O.N. of S can increase and decrease. Hence Let oxidation state of phosphorus is x
can behave as reducing and oxidising agent. Oxidation (4 × 1 + (– 2) × 7 + 2 x) = 0
state of S varies from –2 to 6. \ 2x = 10 or x = +5
62. (d) In Na2O, SnCl2 and Na2O2 central atom is either in 73. (d) Zinc gives H2 gas with dil H2SO4/HCl but not with
lowest or highest oxidation state, so it can function HNO3 because in HNO3, NO3– ion is reduced and give
either as an oxidising or a reducing agent but not both. NH4 NO 3 , N 2 O, NO and NO 2 (based upon the
However, the oxidation state of N in NaNO2 is +3 which concentration of HNO3)
lies between its highest (+5) and lowest (–3) values.
4Zn+10HNO3 ¾¾ ® 4Zn(NO3 ) 2 +NH 4 NO3 +3H 2 O
-1 +6 0 +4
63. (d) 2HI + H 2SO 4 ® I 2 + SO 2 + 2H 2 O Zn is on the top position of hydrogen in electrochemical
Oxidised Re duced series. So Zn displaces H2 from dilute H2SO4 and HCl
RA OA
with liberation of H2.
64. (d) NaCl + H 2O ® NaOH + HCl Zn + H2SO4 ® ZnSO4 + H2
Sodium ion hydrated in water. 74. (c) SO 32 - ® S is in + 4 oxidation state
65. (d) Oxidizing agent itself, undergoes reduction during a
redox reaction S2 O 24- ® S is in + 3 oxidation state
+3 +2 0 +4 S 2 O 62- ® S is in + 5 oxidation state
HAsO 2 + Sn ¾¾
® As+ Sn + H 2O
Hence, here HAsO2 is acting as oxidizing agent.
75. (d) PO3–
4 = x + 4 (– 2) = – 3; x – 8 = – 3; x = + 5
66. (c) On substituting the oxidation numbers of X, Y and Z in SO 2–
4 = x + 4 (– 2) = – 2; x – 8 = – 2; x = + 6
the formula, the sum is zero.
(a) (+ 2 × 3) + (+5 × 2) + (– 2 × 8) Cr2 O 72– = 2x + 7 (– 2) = – 2; 2x – 14 = – 2;
= 6 + 10 – 16 = 0, Hence, X3 (YZ4 )2 is the correct 2x =12; x = + 6
formula of the compound.
EXERCISE - 2
67. (a) More is E°RP , more is the tendency to get itself reduced
loss of 10 e
or more is oxidising power. 1. (c) N -2 4 H 4+4 ¾¾¾¾¾
® N 2+6 (y);
N
68. (a) (i) Mn n+ + ne– ‡ˆˆˆˆ† M, for this reaction, high O.N.of N changes from -2 to +3
negative value of E° indicates lower reduction potential, 2. (a) CrO24 - ® Cr2 O72– O.N of Cr does not change . It
that means M will be a good reducing agent. remains +6 on both sides . Hence there is no oxidation
or reduction.
Redox Reactions 201

3. (c) Calculating the oxidation state of nitrogen in given 13. (b) BaO2 + H2SO4 ® BaSO4 + H2O2
molecules; Oxygen is the most electronegative element in the
Oxidation state of N in NH3 is reaction and has the oxidation states of –1 (in H2O2)
x + 3 × (+ 1) = 0 or x = – 3 and –2 (in BaSO4). In H2O2, peroxo ion is present.
Oxidation state on N in NaNO3 is 14. (b) In haemoglobin, Fe is in +2 oxidation state.
1 + x + 3 × (– 2) = 0 or x = + 5
Oxidation state of N in NaN3 is 15. (c) ¾® CuSO 4 + H 2 (as acid)
Cu + H 2SO 4 ¾
1
+ 1 + 3x = 0 or x = – H 2 + H 2SO 4 ¾ ¾® 2H 2 O + SO 2 (as oxidant)
3
Oxidation state of N in Mg3N2 is In (a), H2SO4 acts just as dehydrating agent.
3 × 2 + 2x = 0 or x = –3 In (b), it acts just as an oxidant.
Thus 3 molecules (i.e. NH3, NaN3 and Mg3N2 have In (d), it neither acts as oxidant nor as acid.
nitrogen in negative oxidation state.
4. (b) Oxidation no. of O are + 2, 0, – 1/2 and – 1 respectively O O
5. (c) The redox couple with maximum reduction potential 16. (b) Na2S4O6 has the structure : Na – S*– S – S – S*– Na
will be best oxidising agent and with minimum reduction
potential will be best reducing agent. O O
+1
6. (a) [Co( NH 3 ) 5 Cl.]Cl 2 , H –1 –2 +1 O.N. of two S* atoms are +5 each and that of other two
N — O — H, S atoms is zero each.
–3 +1H
17. (b) CrO2Cl2, MnO4–; O.N. of Cr and Mn are +6 and +7
H .. H respectively.
H N-N , Mg3N2 18. (b) On reaction with hot and concentrated alkali a mixture
H H of chloride and chlorate is formed
– 1, – 2, –3
Hot
3Cl2 + 3 NaOH(excess) ¾¾¾ ®
7. (d) In FeS2, Fe2+ is converting into Fe3+ and sulphur is
changing from –1 oxidation state to +4 oxidation state. -1 +5
There are two S– and one Fe2+ in FeS2. Thus total no. 5NaCl + NaClO3 + 3H 2O
of electrons lost in the given reaction are 11. 19. (a) Higher the value of reduction potential higher will be
the oxidising power whereas the lower the value of
8. (b) KMnO 4 + FeC 2 O 4 ¾¾ ® Fe3+ + 2CO 2 + Mn 2 +
reduction potential higher will be the reducing power.
So half reaction,
20. (b) YBa2Cu3O7
MnO -4 ¾¾ ® Mn 2 + (decrease in O.N. = 5) 3 + 2 × 2 + 3x – (2 × 7) = 0
FeC2 O 4 ¾¾ ® Fe3+ + 2CO 2 (increase in O.N. = 1) 3 + 4 + 3x – 14 = 0
Now equating the change in O.N. and then by adding 3x = 7
both half reactions we get 7
x= .
5FeC 2 O 4 + MnO -4 ¾¾
® 5Fe3+ + 10CO 2 + Mn 2 + 3
On balancing equation, 21. (a) More negative or lower is the reduction potential , more
is the reducing property. Thus the reducing power of
3MnO -4 + 5FeC 2 O 4 + 24H + ¾¾
® 5Fe3+ + 3Mn 2+ the corresponding metal will follow the reverse order,
+10CO2 + 12H 2O i.e. Y > Z > X.
22. (c) On balancing the given equations, we get
\ 3 moles of KMnO4 = 5 moles of FeC2O4
3 2MnO 4- + 5C 2 O 4 2- + 16H + ¾¾
® 2Mn ++
1 mole FeC2O4 = moles of KMnO 4
5 +10CO 2 + 8H 2O
9. (c) Carbon has the maximum oxidation state of + 4,
So, x = 2, y = 5 & z = 16
therefore carbon dioxide (CO2) cannot act as a reducing
agent. 23. (d) Here, statement-1 is false, because stannous chloride
10. (c) The oxidation number of N in NO2 is + 4 and in N2O4 is is a strong reducing agent not strong oxidising agent.
also + 4 (i.e. no change) Stannous chlorides gives Grey precipitate with mercuric
11. (b) chloride. Hence statement-2 is true.
12. (d) Zn ® Zn +2 + 2e - ....(1) 24. (b) Both statement-1 and statement-2 are true but reason
- +
8e + 10H + NO 3- ® NH +4 + 3H 2 O ....(2) is not the correct explanation of assertion. Greater the
operate eq. (1) × 4 + eq. (2) × 1 number of negative atoms present in the oxy-acid make
4 Zn + 10H + + NO 3- ® 4 Zn +2 + NH +4 + 3H 2 O the acid stronger. In general, the strengths of acids
that have general formula (HO)m ZOn can be related to
EBD_7327
202 CHEMISTRY

the value of n. As the value of n increases, acidic –


Since, E of the overall reaction is positive, therefore,
character also increases. The negative atoms draw Cu can reduce Br2.
electrons away from the Z-atom and make it more 4. (d)
(a) 2Fe3+ + 2e– ® 2Fe2+; E° = + 0.77 V
positive. The Z-atom, therefore, becomes more effective
2I– ® I2 + 2e–; E° = – 0.54 V
in withdrawing electron density away from the oxygen 2Fe3+ + 2I– ® 2Fe2+ + I2; E°cell = + 0.23 V
atom that bonded to hydrogen. In turn, the electrons (b) Cu ®Cu2+ + 2e–; E° = – 0.34 V
of H – O bond are drawn more strongly away from the 2Ag+ + 2e– ® 2Ag; E° = + 0.80 V
Cu + 2Ag+ ® Cu2+ +2Ag; E° = + 0.46 V
H-atom. The net effect makes it easier from the proton
release and increases the acid a strength. (c) 2Fe3+ + 2e - ® 2Fe2 + ; E° = + 0.77V
25. (a) Both statement-1 and statement-2 are true and Cu ® Cu 2 + + 2e - ; E° = - 0.34 V
statement-2 is the correct explanation of statement-1.
2Fe3+ + Cu ® 2Fe 2 + + Cu 2 + ; E° = + 0.43V
Oxidation loss of 2e– (d) Ag ® Ag+ + e–; E° = – 0.80 V
Fe3+ + e– ® Fe2+; E° = + 0.77 V
Ag + Fe3+ ® Ag+ + Fe2+; E° = – 0.03 V
Zn(s) + Cu2+(aq) Zn2+ (aq) + Cu(s)
This reaction is not feasible since E°cell is negative.
+2 0 2.5
Reduction gain of 2e– 5. (a) 2 S 2 O32- ( aq ) + I 2 ( s ) ® S4 O62- ( aq ) + 2I - ( aq )
+2 0
EXERCISE - 3 S2 O32- ( aq ) + 2 Br2 ( l ) + 5H 2 O ( l ) ®
Exemplar Questions +6
2 S O 24- ( aq ) + 4Br - ( aq ) + 10H + ( aq )
1. (d) Following are the examples of redox reaction
Hence, bromine is a stronger oxidising agent than I2,
Red
as it oxidises S of S2 O32 - to SO 24 - whereas I2 oxidises
(a) CuO + H 2 ¾ ¾
® Cu + H 2 O
it only into S4 O62 - ion.
Oxi
Oxi 6. (a) Oxidation number of hydrogen is not always +1. It is
+1 in hydrogen halides, –1 in hydrides and zero in H2
(b) Fe2 O3 + 3CO ¾ ¾
® 2Fe + 3CO2
molecule.
Red 7. (b) Split NH4 NO3 into two ions NH +4 and NO3- . Now
Red
calculate the oxidation number of nitrogen in the two
(c) 2K + F2 ¾ ¾
® 2KF species.
Oxi +
Option (d) is not an example of redox reaction as there Let, oxidation number of N in N H 4 is x.
is no change in oxidation number of any of the Þ x + (4 × 1) = +1
reactants. or x + 4 = + 1 or x = – 3
2. (d) Since the redox couple Ag + /Ag has the highest
Let, oxidation number of N in NO3- is y
positive value of E o 0.80 V, therefore, Ag+ is the
strongest oxidising agent. Þ y + (3 × – 2) = – 1 or y – 6 = – 1 or y = + 5
– 8. (a) Calculating the oxidation number of central atom in the
3. (d) Given that E values of given species.
Br2/Br– = + 1.09 V
+3 +5 +6 +7
Ag+ / Ag(s) = 0.80 V
Cr O -2 , Cl O 3- , Cr O 42- , MnO 4-
Cu2+/Cu(s) = + 0.34V
I2 (s) / I– = + 0.54 V 9. (d) Highest oxidation number of any transition element is
Br2 / Br – = 1.09 V the sum of (n – 1) d-electrons and ns electrons. Hence,
– large the number of electrons in the 3d-orbitals, higher
The E values show that copper will reduce Br 2, if the is the maximum oxidation number.
E° of the following redox reaction is positive. 10. (d) Reactions in which the same substance undergo
The electrode reactions will be oxidation as well as reduction are called
Cu ® Cu 2 + + 2e - ; E° = -0.34V disproportionation reactions. Writing the O.N. of each
element above its symbol in the given reactions.
Br2 + 2e - ® 2Br - ; E° = +1.09 V
In reaction (d), N is both oxidised as well as reduced
Cu + Br2 ® CuBr2 ; E° = + 0.75 V
Redox Reactions 203

since the O. N. of N increases from +4 in NO2 to +5 in Zn is on the top position of hydrogen in electrochemical
series. So Zn displaces H2 from dilute H2SO4 and HCl
NO3- and decreases from +4 in NO2 to +3 in NO2- .
with liberation of H2.
oxidation Zn + H2SO4 ® ZnSO4 + H2
+4 +3 +5 18. (b) Oxidation number of a compound must be 0. Using the
NO 2 + 2OH - ¾ ¾
® NO2- + NO3- + H 2 O values for A, B and C in the four options we find that
A3(BC4)2 is the answer.
reduction Check : (+2)3 + [(+5)+4(–2)]2 = 6 + (5–8)2 = 0
11. (c) Fluorine is the strongest oxidising agent. It does not
show positive oxidation state hence it does not 19. (d) Zn ® Zn +2 + 2e - ....(1)
disproportionate. -
8e + 10H +
+ NO 3-
® NH +4
+ 3H 2 O .....(2)
NEET/AIPMT (2013-2017) Questions operate eq. (1) × 4 + eq. (2) × 1
12. (a) Higher the value of reduction potential higher will be 4Zn + 10H + + NO3- ® 4Zn +2 + NH 4+ + 3H 2 O
the oxidising power whereas the lower the value of
20. (d) Pyrophosphoric acid H4P2O7
reduction potential higher will be the reducing power. Let oxidation state of phosphorus is x
+5 +6 +6 (4 × 1 + (– 2) × 7 + 2 x) = 0
13. (c) KClO 3 + H 2C 2 O 4 + H 2SO 4 ® K 2SO 4
\ 2x = 10 or x = +5
–1
+ KCl + CO 2 + H 2 O 21. (c) 2NaOH+H 2SO 4 ¾¾
® Na 2SO 4 +2H 2 O
i.e. maximum change in oxidation number is observed (neutralization)
0 0
in Cl (+5 to –1). Light
3 O 2 ¾¾¾® 2 O 3 (not redox reaction)
14. (b) On reaction with hot and concentrated alkali a mixture
of chloride and chlorate is formed Light
N 2 + O2 ¾¾¾® 2 N O (redox reaction)
Hot
3Cl2 + 3 NaOH(excess) ¾¾¾ +2 -2
® 0 0
-1 +5 here oxidation of N2 & reduction of O2 is taking place
5NaCl + NaClO3 + 3H 2O D
H 2 O (l) ¾¾® H 2 O (g ) (not redox reaction)
15. (d) PO3–
4 = x + 4 (– 2) = – 3; x – 8 = – 3; x = + 5
22. (c) Carbon has the maximum oxidation state of + 4,
2– = x + 4 (– 2) = – 2; x – 8 = – 2; x = + 6 therefore carbon dioxide (CO2) cannot act as a reducing
SO 4
agent.
Cr2 O72– = 2x + 7 (– 2) = – 2; 2x – 14 = – 2; 23. (a) Losing of electron is called oxidation.
2x =12; x = + 6 24. (a) Let x = oxidation no. of Cr in K2Cr2O7.
16. (c) SO 32 - ® S is in + 4 oxidation state \ (2 × 1) + (2 × x) + 7 (– 2) = 0
or 2 + 2x – 14 = 0 or x = + 6.
S2 O 24- ® S is in + 3 oxidation state
+2 -1 + 1 +6 -2 +2 +6 -2 + 1 -1
S2 O 62- ® S is in + 5 oxidation state 25. (c) Ba O 2 + H 2 S O 4 ¾¾
® Ba S O 4 + H 2 O 2
17. (d) Zinc gives H2 gas with dil H2SO4/HCl but not with In this reaction, none of the elements undergoes a change
HNO3 because in HNO3, NO3– ion is reduced and give in oxidation number or valency.
NH4 NO 3 , N 2 O, NO and NO 2 (based upon the 26. (a) O.N. of P in H3PO3 (phosphorous acid)
concentration of HNO3) 3 × 1 + x + 3 × (– 2) = 0 or x = + 3
In orthophosphoric acid (H3PO4) O.N. of P is + 5, in
[ Zn + 2HNO 3 ¾
¾® Zn ( NO 3 ) 2 + 2H] ´ 4 hypophosphorous acid (H3PO2) it is + 1 while in
(nearly 6%) metaphosphoric acid (HPO3), it is + 5.
27. (b) The element undergo oxidation itself and reduces
HNO 3 + 8H ¾
¾® NH 3 + 3H 2 O others is known as reducing agent. In this reaction O.
N. of Ni Changes from 0 to + 2 and hence Ni acts as a
NH 3 + HNO 3 ¾
¾® NH 4 NO 3
reducing agent.
4Zn+10HNO3 ¾¾
® 4Zn(NO3 ) 2 +NH 4 NO 3 +3H 2 O 28. (c) = + 2 + 2x – 14 = 0 or x = + 6
EBD_7327
v
204 CHEMISTRY

9 Hydrogen

DIHYDROGEN, H2
H2 is the most abundant element in the universe and is the 2Na + ( aq ) + 2Cl- ( aq ) + 2H 2O ( l )
principal element in the solar atmosphere. In combined form it ¯
constitutes 15.4% of earth’s crust and the oceans.
Cl 2 ( g ) + H 2 ( g ) + 2Na + ( aq ) + 2OH - ( aq )
Isotopes of Hydrogen

Hydrogen has three isotopes: protium ( 11H ) , deuterium (iv) By reaction of steam on hydrocarbons or coke :
1270K
® nCO + ( 3n + 1) H 2
C n H 2n + 2 + nH 2O ¾¾¾¾
( 12 H or D) and tritium ( 13 H or T ) . Protium has no neutrons, Ni

1270K
deuterium has one and tritium has 2 neutrons in the nucleus. Only ex: CH 4 ( g ) + H 2O ( g ) ¾¾¾¾ ® CO ( g ) + 3H 2 ( g )
tritium is radioactive and emits low energy b– particles. Ni
Since the isotopes have the same electronic configuration, they The mixture of CO and H2 is called water gas and is also
have almost the same chemical properties. The only difference is called syn gas or synthetic gas.
in their rates of reactions, mainly due to their different enthalpy Properties of Dihydrogen
of bond dissociation. However, in physical properties these
(i) Physical properties: It is a colourless, odourless, tasteless,
isotopes differ considerably due to their large mass differences.
combustible gas. It is lighter than air and insoluble in water.
(ii) Chemical properties: (1) H2 is inert at room temperature due
Isotopes of hydrogen :
to high H–H bond enthalpy.
Protium Deuterium Tritium (2) Due to incomplete orbital with 1s1 configuration, H
(a) 11 H 2 H or D
1
3
1H or T combines with almost all elements either by (a) loss of one e–
(b) p = 1, e = 1, p = 1, e = 1, p = 1, e = 1, to give H+ (b) gain of an e– to form H– (c) sharing e–s to form
n=0 n=1 n=2 single covalent bond.
(c) Abundance Reactions of dihydrogen :
99% 0.0156% 10–15%
(d) Ordinary Heavy Radioactive used as tracer (a) H 2 ( g ) + X 2 ( g ) ¾¾
® 2HX ( g ) (X = F, Cl, Br, I)
hydrogen hydrogen to study reaction
mechanism. Reaction with iodine requires a catalyst
D
(b) 2H 2 (g ) + O2 (g ) ¾¾ ® 2H 2O (l )
Preparation of Dihydrogen or catalyst
DH = – 285.9 kJ mol–1
(i) Lab preparation:
673K
(a) Zn(s) + 2H + (aq) ¾¾
® Zn 2+ (aq) + H 2 (g) (c) 3H 2 ( g ) + N 2 ( g ) ¾¾¾® 2NH3 ( g )
200atm
(b) Zn(s) + 2NaOH(aq) ¾¾ ® Na 2 ZnO 2 (aq) + H 2 (g) Fe
(ii) Commercial preparation: DH = – 92.6 kJ mol–1
Electrolysis
2H2 O ( l ) ¾¾¾¾¾¾
® 2H 2 ( g ) + O2 ( g )
Traces of acid/base This is the Haber’s process for the manufacture of
(iii) By electrolysis of brine solution: ammonia.
At anode: 2Cl - ( aq ) ¾¾
® Cl 2 ( g ) + 2e -
(d) H 2 ( g ) + 2M ( g ) ¾¾
® 2MH ( s ) (M : alkali metal)
At cathode: 2H 2 O ( l ) + 2e - ¾¾
® H 2 ( g ) + 2OH - ( aq ) With many metals dihydrogen forms hydrides.
Overall reaction:
Hydrogen 205

(e) Reduction of metal ions (in aqueous solution) and oxides 3. Electron rich: They have excess e–1s which are present
of metals into corresponding metals. as lone pairs. Elements of group 15-17 form such
compounds.
H 2 (g) + Pd 2 + (aq) ¾¾
® Pd(s) + 2H + (aq)
They behave as Lewis bases, i.e., e– donors.
yH 2 (g) + M x O y (s) ¾¾
® xM (s) + yH 2 O(l ) Ex : NH3, H2O, etc.
(f) Hydroformylation of olefins. (iii) Metallic or interstitial hydrides
H 2 + CO + RCH = CH 2 ¾¾
® RCH 2 CH 2 CHO
(a) They are formed by d- and f-block elements.
(b) They are non-stoichiometric, being deficient in hydrogen
H 2 + RCH 2 CH 2 CHO ¾¾ ® RCH 2 CH 2 CH 2 OH ex: LaH2.87, NiH0.6-0.7
(g) Hydrogenation of unsaturated hydrocarbons In such hydrides the law of constant composition does
Ni or Pt not hold good.
H 2 + CH 2 = CH 2 ¾¾¾¾ ® CH 3 - CH 3 (c) Here, hydrogen occupies interstices in metal lattice
473
producing distortion. Hence, the name interstitial
Uses
hydrides.
It is used, (d) The property of absorption of hydrogen on transition
(i) in the synthesis of ammonia. metals is widely used in catalytic reduction or
(ii) in the manufacture of vanaspati fat by hydrogenation of hydrogenation reactions for preparing large number of
polyunsaturated vegetable oils. compounds.
(iii) in the manufacture of bulk organic chemicals, for preparing
metal hydrides, HCl and to reduce heavy metal oxides. WATER
(iv) as a rocket fuel in space research. Physical Properties
(v) in fuel cells for generating electrical energy. (i) It is colourless and tasteless liquid.
HYDRIDES (ii) Due to extensive H-bonding between water molecules, it has
Dihydrogen combines with almost all elements to form binary high freezing point, high boiling point, high heat of
compounds called hydrides, which can be represented as EHx vaporisation and high heat of fusion in comparison to H2S,
(ex. MgH2) or EmHn (ex: B2H6) (E is the symbol of the element). H2Se, H2Te etc.
(iii) In comparison to other liquids, water has a higher specific
Classification of Hydrides
heat, thermal conductivity, surface tension and dipole
The hydrides are classified into three categories moment.
(i) Ionic or saline hydrides Structure of water : In H2O each oxygen forms four sp3-hybridized
(a) They are formed with s-block elements (as they are orbital. Two of these form sp3-s, O–H–bonds while other two
electropositive in nature) contain a lone pair of electrons each. In gas phase, H2O is a bent
(b) They are crystalline, non-volatile and non-conducting molecule with a bond angle of 104.5° and O–H bond length of 95.7
in solid state pm.
(c) Their melts conduct electricity and on electrolysis
2d –
liberate dihydrogen gas at anode. O O
95.7 pm
2H (melt) electrolysis H2(g) + 2e–

104.5°
anode d+ d+
(d) Saline hydrides with water gives dihydrogen gas H H H H
Bent structure Dipolar structure
NaH(s) + H 2O(aq) NaOH(aq) + H 2(g)
Structure of Ice : Ice has a highly ordered 3D H-bonded structure
(e) Being unreactive at moderate temperature with O2 or
in which each O-atom is tetrahedrally surrounded by four other
Cl2, LiH is used in synthesis of other hydrides.
O-atoms at a distance of 276 pm. This gives ice an open cage like
ex: 8LiH + Al2Cl6 structure. Density of ice is less than that of water. Thus, ice floats
2LiH + B2H6 2LiBH4 on water.
(ii) Covalent or molecular hydrides Chemical Properties
(a) They are formed with p-block elements and some s-block (i) Amphoteric nature: H2O has the ability to act as an acid as
elements (Be and Mg) well as a base.
ex : CH4, NH3, H2O, HF
H 2 O ( l ) + NH 3 ( aq ) OH - ( aq ) + NH +4 ( aq )
(b) They are further classified as:
1. Eectron deficient: which have few e–s for writing Lewis H 2O ( l ) + H 2S ( aq ) H 3O + ( aq ) + HS- ( aq )
structure
The auto-protolysis ie. self ionization of water takes place as
Ex : B2H6. All elements of group 13 form e–, deficient
follows:
compounds. They act as Lewis acid.
2. Electron precise : They have the required no. of e– to H 2 O ( l )+ H 2 O ( l ) H3 O + ( aq ) + OH - ( aq )
acid -1 base - 2 acid - 2 base -1
write Lewis structure. All elements of group 14 (ex: CH4) (Conjugate
form such compounds.
( acid ) ( base ) (Conjugate
base)
acid)
EBD_7327
206 CHEMISTRY

(ii) Redox reactions involving water : (c) Ion-exchange method


(a) 2H 2 O ( l ) + 2Na ( s ) ¾¾
® 2NaOH ( aq ) + H 2 ( g ) 2NaZ(s) + M2+ (aq) ––®
i.e., highly electropositive metals reduce H2O to H2. MZ2(s) + 2Na+(aq) (M = Mg, Ca)
(b) It is oxidised to O2 during photosynthesis: where, NaZ ® NaAlSiO4 (zeolite)
6CO 2 (g) + 12H 2 O(l ) (d) Synthetic resins method.Cation exchange resins
exchange H+ ions with cations such as Ca 2+ and Mg2+
¾¾
® C6 H12 O 6 (aq) + 6H 2 O(l ) + 6O 2 (g) ions present in hard water.
(c) 2F2 (g) + 2H 2O(l ) ¾¾ ® 4H + (aq) + 4F- (aq) + O 2 (g) Anion exchange resins exchange OH– ion with anions
(iii) Hydrolysis reaction: Due to high dielectric constant, water such as Cl– and SO42–.
shows strong hydrating tendency. HEAVY WATER (D2O)
P4 O10 ( s ) + 6H 2 O ( l ) ¾¾
® 4H 3PO 4 ( aq ) It is extensively used as a moderator in nuclear reactors and in
Na 2CO3 (s) + 2H 2O (l ) ¾¾
® 2NaOH (aq ) + H 2CO3 (aq ) exchange reactions for the study of reaction mechanisms. It can
(iv) Hydrate formation : Many ionic compounds crystallise from be prepared by exhaustive electrolysis of water or as a by-product
water with one or more molecules of water associated with in some fertilizer industries. It is used for the preparation of other
them. These hydrates are classified as : deuterium compounds, for example:
(a) Coordinated water : ex., [Cr(H2O)6]3+ 3Cl– CaC2 + 2D2O ® C2D2 + Ca(OD)2
(b) Interstitial water : BaCl2.2H2O SO3 + D2O ® D2SO4
(c) H-bonded water : [Cu(H2O)4]2+SO42–.H2O Al4C3 + 12D2O ® 3CD4 + 4Al(OD)3
in CuSO4.5H2O
HYDROGEN PEROXIDE (H2O2)
HARD AND SOFT WATER
Preparation
Hard Water
(i) By acidifying barium peroxide
It does not give lather with soap. It contains salts of calcium and
magnesium in the form of chloride, sulphate and hydrogen BaO 2 .8H 2O ( s ) + H 2SO 4 ( aq )
carbonate. Hard water forms scum with soap which is unsuitable
® BaSO4 ( s ) + H 2O 2 ( aq ) + 8H 2O ( l )
¾¾
for laundry and boilers.
Soft Water (ii) By hydrolysis of peroxodisulphate
It gives lather with soap. It is free from soluble salts of calcium and
2HSO 4- ( aq ) ¾¾¾¾¾® HO3SOOSO3 H ( aq )
electrolysis
magnesium.
hydro -
Types of Hardness of Water ¾¾¾¾® HSO 4- ( aq ) + 2H + ( aq ) + H 2 O 2 ( aq )
Hardness of water is of two types lysis
(i) Temporary : It is due to presence of Mg and Ca hydrogen (iii) Industrial preparation
carbonates. It can be removed by O (air)
2
ˆˆˆˆˆˆ
†
2-Ethylanthraquinol ‡ˆˆˆˆˆ
(a) boiling ˆ H2O2 + oxidised product
H 2 Pd
Heating
Mg(HCO3 ) 2 ¾¾¾¾
® Mg(OH) 2 ¯ +2CO 2 ­ Properties
Heating
Ca(HCO3 ) 2 ¾¾ ¾¾ ® CaCO3 ¯ + H 2 O + CO 2 ­ Physical properties : In the pure state, H2O2 is an almost
(b) Clark’s method (lime is added to hard water which colourless (very pale blue) liquid. H2O2 is miscible with water in
precipitates out CaCO3 and Mg(OH)2. all proportions and forms a hydrate H2O2.H2O (mp 221K).
Ca(HCO 3 ) 2 + Ca(OH) 2 ¾¾
® 2CaCO 3 ¯ +2H 2 O A 30% solution of H2O2 is marketed as ‘100 volume’ hydrogen
Mg(HCO 3 ) 2 + 2Ca(OH) 2 ¾¾® peroxide. It means that one millilitre of 30% H2O2 solution will
2CaCO3 ¯ + Mg(OH) 2 ¯ +2H 2 O give 100 V of oxygen at STP.
(ii) Permanent : It is due to presence of soluble salts of Mg and Chemical properties
Ca as sulphates and chlorides. (i) Oxidising action in acidic medium
It can be removed by
(a) Treating with washing soda 2Fe 2 + ( aq ) + 2H + ( aq ) + H 2 O 2 ( aq )
MCl2 + Na 2 CO 3 ¾¾
® MCO3 ¯ +2NaCl (M = Mg,Ca) ® 2Fe3+ ( aq ) + 2H 2O ( l )
¾¾
MSO 4 + Na 2 CO3 ¾¾
® MCO 3 ¯ + Na 2SO 4
PbS(s) + 4H2O2 (aq) ¾¾ ® PbSO4 (s) + 4H 2O(l )
(b) Calgon’s method
(ii) Oxidising action in basic medium
® 2Na + + Na 4 P6 O18
Na 6 P6 O18 ¾¾ 2-
2Fe2+ + H 2O2 ¾¾
® 2Fe3+ + 2OH-
2+ 2- 2- +
M + Na 4 P6O18 ¾¾
® [Na 2 MP6O18 ] + 2Na
Mn 2+ + H 2O2 ¾¾
® Mn 4+ + 2OH-
(M = Mg, Ca)
Hydrogen 207

(iii) Reducing action in basic medium Uses of H2O2


I2 + H 2O 2 + 2OH - ¾¾ ® 2I- + 2H 2O + O2 (i) In daily life it is used as a hair bleach and as a mild
-
2MnO4 + 3H2O2 ¾¾ ® 2MnO2 + 3O2 + 2H2O + 2OH- disinfectant.
(iv) Reducing action in acidic medium (ii) It is used to manufacture chemicals like sodium perborate
2MnO-4 + 6H + + 5H 2O 2 ¾¾
® 2Mn 2+ + 8H 2O + 5O 2 and percarbonate, which are used in high quality
® H 3O + + Cl - + O 2
HOCl + H 2O 2 ¾¾ detergents.
(iv) H2O2 acts as a bleaching agent due to the release of nascent (iii) It is used in the synthesis of hydroquinone, tartaric acid
oxygen and certain food products and phar maceuticals
H 2O 2 ¾¾ ® H 2O + O (cephalosporin) etc.
Structure (iv) It is employed in the industries as a bleaching agent for
textiles, paper pulp, leather, oils, fats, etc.
Hydrogen peroxide has a non-planar structure.
(v) Nowadays it is also used in Environmental (Green)
Chemistry. For example, in pollution control treatment
H H of domestic and industrial effluents, oxidation of
95.0 pm 98.8 pm cyanides, restoration of aerobic conditions to sewage
147.5 pm 111.5° 145.8 pm 90.2°
O O O O wastes, etc.
94.8° 101.9°
H H Dihydrogen as a Fuel
Gas Phase Solid phase H2 releases large amount of heat on combustion. On a mass for
(i) H2O2 structure in gas phase, dihedral angle is 111.5°. mass basis, H2 releases more energy than petrol. Moreover, it
produces less pollution than petrol.
(ii) H2O2 structure in solid phase at 110 K, dihedral angle is
90.2°. It has promising potential for use as a non-polluting fuel of near
future. This is studied under ‘Hydrogen Economy’. The basic
Storage
principle of Hydrogen economy is transportation and storage of
H2O2 decomposes on exposure to light
energy in form of liquid or gaseous dihydrogen.
2H 2 O 2 ( l ) ¾¾
® 2H 2 O ( l ) + O 2 ( g ) Advantage of Hydrogen economy is that energy is transmitted in
The reaction is catalysed in presence of metal surfaces or traces form of H2 and not as electric power.
of alkali. Hence, it is stored in wax-lined glass or plastic vessels in Now a days H2 is also used in fuel cells for generation of electric
dark. Urea can be added as a stabiliser. It is kept away from dust power.
because dust can induce explosive decomposition of the
compound.
208

CONCEPT MAP
CHEMISTRY

EBD_7327
Hydrogen 209

1. Hydrogen is not more reactive at ordinary temperature, (c) Hydrogen molecule can form intermolecular hydrogen
because at ordinary temperature it is in its: bonds but chlorine molecule does not
(a) gaseous state (b) liquid state (d) Hydrogen molecule cannot participate in coordination
(c) molecular state (d) atomic state bond formation but chlorine molecule can
2. Hydrogen has high ionization energy than alkali metals 11. Hydrogen will not reduce
because it has : (a) heated cupric oxide (d) heated ferric oxide
(a) Ionic bond (b) Covalent bond (c) heated stannic oxide (d) heated aluminium oxide
12. Hydrogen is evolved by the action of cold dil. HNO3 on
(c) Large size (d) Small size
(a) Fe (b) Mn
3. The number of moles of H2 in 0.224 litre of hydrogen gas
(c) Cu (d) Al
at STP (273 K, 1 atm) is :
13. On industrial scale, H2O2 is now generally prepared by
(a) 0.1 (b) 0.01
(a) The action of H2SO4 on barium
(c) 0.001 (d) 1
(b) The action of H2SO4 on sodium peroxide
4. The ionization of hydrogen atom would give rise to (c) By the electrolysis of H2SO4
(a) hydride ion (b) hydronium ion (d) By burning hydrogen in an excess of O2.
(c) proton (d) hydroxyl ion. 14. Hydrogen can behave as a metal
5. Which of the following statement is not correct regarding (a) at very high temperature
hydrogen atom? (b) at very low temperature
(a) It resembles with halogens in some properties (c) at very high pressure
(b) It resembles with alkali metals in some properties. (d) at very low pressure
(c) It cannot be placed in first group of periodic table. 15. Which cannot be oxidised by H2O2 :
(d) It is the lightest element
(a) Na2SO3 (b) PbS
6. Which one of the following pairs of substances on reaction
(c) KI (d) O3
will not evolve H2 gas? 16. The property of hydrogen which distinguishes it from alkali
(a) Iron and H2SO4 (aqueous) metals is
(b) Iron and steam (a) its electropositive character
(c) Copper and HCl (aqueous) (b) its affinity for non metal
(d) Sodium and ethyl alcohol (c) its reducing character
7. When two ice cubes are pressed over each other, they unite to (d) its non-metallic character
form one cube. Which of the following forces is responsible to 17. Hydrogen accepts an electron to form inert gas
hold them together ? configuration. In this it resembles
(a) Hydrogen bond formation (a) halogen (b) alkali metals
(b) van der Waals forces (c) chalcogens (d) alkaline earth metals
(c) Covalent attraction 18. In Bosch’s process which gas is utilised for the production
(d) Ionic interaction of hydrogen gas ?
8. Which statement is correct for hydrogen ? (a) Producer gas (b) Water gas
(a) It has a very high ionisation potential (c) Coal gas (d) None of these
(b) It is always collected at cathode 19. The adsorption of hydrogen by metals is called
(c) It can form bonds in +1 as well as –1 oxidation state (a) Dehydrogenation (b) Hydrogenation
(d) It has same electronegativity as halogens (c) Occlusion (d) Adsorption
9. Hydrogen can be differentiated by other alkali metals due to 20. Which of the following ions can be replaced by H+ ions
its : when H2 gas is bubbled through the solutions containing
(a) nonmetallic character these ions ?
(a) Li+ (b) Ba2+
(b) affinity for non-metals 2+
(c) Cu (d) Be2+
(c) electropositive character 21. Which of the following terms is not correct for hydrogen ?
(d) reducing character (a) Its molecule is diatomic
10. Hydrogen molecules differs from chlorine molecule in the (b) It exists both as H+ and H– in different chemical
following respect compounds
(a) Hydrogen molecule is non-polar but chlorine molecule (c) It is the only species which has no neutrons in the
is polar nucleus
(b) Hydrogen molecule is polar while chlorine molecule is (d) Heavy water is unstable because hydrogen is
non-polar substituted by its isotope deuterium
EBD_7327
210 CHEMISTRY

22. Water is : 34. The boiling point of water is exceptionally high because
(a) more polar than H 2S (a) there is covalent bond between H and O
(b) water molecule is linear
(b) more or less identical in polarity with H 2S
(c) water molecules associate due to hydrogen bonding
(c) less polar than H 2S (d) water molecule is not linear
(d) None of these 35. What is formed when calcium carbide reacts with heavy
23. The reagent commonly used to determine hardness of water?
water titrimetrically is (a) C2D2 (b) CaD2
(a) oxalic acid (b) sodium thiosulphate (c) Ca2D2O (d) CD2
(c) sodium citrate (d) disodium salt of EDTA 36. Water possesses a high dielectric constant, therefore :
24. Calgon used as a water softener is (a) it always contains ions
(a) Na 2 [Na 4 (PO3 )6 ] (b) Na 4 [ Na 2 (PO 3 ) 6 ] (b) it is a universal solvent
(c) Na 4 [Na 4 (PO 4 )5 ] (d) Na 4 [ Na 2 (PO 4 ) 6 ] (c) can dissolve covalent compounds
(d) can conduct electricity
25. Which of the following groups of ions makes the water hard?
(a) Sodium and bicarbonate 37. Chemical A is used for water softening to remove temporary
hardness. A reacts with sodium carbonate to generate caustic
(b) Magnesium and chloride
soda. When CO2 is bubbled through a solution of A, it turns
(c) Potassium and sulphate
cloudy. What is the chemical formula of A.
(d) Ammonium and chloride.
(a) CaCO3 (b) CaO
26. Consider the following statements :
1. Atomic hydrogen is obtained by passing hydrogen (c) Ca(OH)2 (d) Ca(HCO3)2
through an electric arc. 38. Which of the following is not true?
2. Hydrogen gas will not reduce heated aluminium oxide. (a) D2O freezes at lower temperature than H2O
3. Finely divided palladium adsorbs large volume of (b) Reaction between H2 and Cl2 is much faster than D2
hydrogen gas and Cl2
4. Pure nascent hydrogen is best obtained by reacting Na
(c) Ordinary water gets electrolysed more rapidly than D2O
with C2H5OH
Which of the above statements is/are correct ? (d) Bond dissociation energy of D2 is greater than H2
(a) 1 alone (b) 2 alone 39. Which of the following statements do not define the
(c) 1, 2 and 3 (d) 2, 3 and 4 characteristic property of water “Water is a universal
27. Pure water can be obtained from sea water by solvent”
(a) centrifugation (b) plasmolysis (a) It can dissolve maximum number of compounds
(c) reverse osmosis (d) sedimentation (b) It has very low dielectric constant
28. The H–O–H angle in water molecule is about (c) It has high liquid range
(a) 90º (b) 180º (d) None of these
(c) 102.5° (d) 104.5º
40. The reaction
29. D2O is used in
(a) motor vehicles (b) nuclear reactor H 2S + H 2 O 2 ¾
¾® S + 2H 2 O manifests
(c) medicine (d) insecticide
(a) Acidic nature of H2O2
30. The process used for the removal of hardness of water is
(b) Alkaline nature of H2O2
(a) Calgon (b) Baeyer
(c) Oxidising action of H2O2
(c) Serpeck (d) Hoope
(d) Reducing action of H2O2.
31. When zeolite (hydrated sodium aluminium silicate) is treated
with hard water the sodium ions are exchanged with 41. Triple point of water is
(a) H+ ions (b) Ca2+ ions (a) 273 K (b) 373 K
(c) 203 K (d) 193 K
(c) SO 2- ions
4 (d) OH– ions
42. Which of the following is correct about heavy water ?
32. The percentage by weight of hydrogen in H2O2 is (a) Water at 4°C having maximum density is known as
(a) 5.88 (b) 6.25 heavy water
(c) 25 (d) 50
(b) It is heavier than water (H2O)
33. The alum used for purifying water is
(c) It is formed by the combination of heavier isotope of
(a) ferric alum (b) chrome alum
hydrogen with oxygen
(c) potash alum (d) ammonium alum
(d) None of these
Hydrogen 211

43. When sulphur trioxide is heated with heavy water the 58. Which of the following is wrong about H2O2? It is used
products are (a) As aerating agent in production of spong rubber
(a) Deutero-sulphuric acid (b) As an antichlor
(b) Deuterium sulphuric acid (c) For restoring white colour of blackened lead painting
(c) Deuterium and sulphuric acid
(d) None of these
(d) None of the above. 59. H2O2 ® 2H+ + O2 + 2e– ; E° = –0.68 V. This equation
44. The volume strength of 1.5 N H 2 O 2 solution is : represents which of the following behaviour of H2O2.
(a) 8.4 (b) 8.0 (a) Reducing (b) Oxidising
(c) 4.8 (d) 3.0 (c) Acidic (d) Catalytic
45. The decomposition of H2O2 is accelerated by – 60. Hard water when passed through ion exchange resin
(a) glycerine (b) alcohol containing R’COOH groups, becomes free from :
(c) phosphoric acid (d) Pt powder
46. Commercial 10 volume H2O2 is a solution with a strength of (a) Cl - (b) SO24-
approximately
(a) 15% (b) 3% (c) H3 O+ (d) Ca 2 +
(c) 1% (d) 10% 61. H2O2 when added to a solution containing KMnO4 and
47. Which of the following statements is not true for hydrogen H2SO4 :
peroxide? (a) Acts as an oxidizing agent
(a) Pure H2O2 is fairly stable
(b) Acts as a reducing agent
(b) It sometimes acts as a reducing agent
(c) It acts as an oxidising agent (c) Acts both as an oxidizing as well as a reducing agent
(d) Aqueous solution of H2O2 is weakly basic (d) Producing no reaction
48. The structure of H2O2 is 62. The O – O – H bond angle in H2O2 is
(a) planar (b) non planar (a) 106° (b) 109°28'
(c) spherical (d) linear
(c) 120° (d) 94.8°
49. In lab H2O2 is prepared by
(a) Cold H2SO4 + BaO2 (b) HCl + BaO2 63. When H2O2 is oxidised the product is
(c) Conc. H2SO4 + Na2O2 (d) H2 + O2 (a) OH– (b) O2
50. The reaction of H2S + H2O2 ® S + 2H2O manifests (c) O2– (d) HO2–
(a) Acidic nature of H2O2 (b) Alkaline nature of H2O2 64. Some statements about heavy water are given below:
(c) Oxidising nature of H2O2 (d) Reducing action of H2O2 (A) Heavy water is used as a moderator in nuclear reactors.
51. The strength in volumes of a solution containing 30.36 g/L (B) Heavy water is more associated than ordinary water.
of H2O2 is (C) Heavy water is more effective solvent than ordinary
(a) 10 V (b) 5 V water.
(c) 20 V (d) None of these Which of the above statements are correct?
52. Which one the following removes temporary hardness of
(a) (A) and (C) (b) (A) and (B)
water ?
(c) (A), (B) and (C) (d) (B) and (C)
(a) Slaked lime (b) Plaster of Paris
(c) Epsom (d) Hydrolith 65. Which of the following reaction produces hydrogen
53. 20 volume H2O2 solution has a strength of about (a) Mg + H2O (b) BaO2 + HCl
(a) 30% (b) 6% (c) H2S4O8 + H2O (d) Na2O2 + 2HCl
(c) 3% (d) 10% 66. Hydrogen can be fused to form helium at
54. Which substance does not speed up decomposition of H2O2 (a) High temperature and high pressure
(a) Glycerol (b) Pt (b) High temperature and low pressure
(c) Gold (d) MnO2 (c) Low temperature and high pressure
55. H2O2 is a
(d) Low temperature and low pressure
(a) Weak acid (b) Weak base
67. Which of the following gas is insoluble in water ?
(c) Neutral (d) None of these
56. Hydrogen is not obtained when zinc reacts with (a) SO2 (b) NH3
(a) Cold water (b) dil. HCl (c) H2 (d) CO2
(c) dil. H2SO4 (d) Hot NaOH (20%) 68. In which of the following reactions, H2O2 acts as a reducing
57. H2O2 is always stored in black bottles because agent
(a) It is highly unstable (a) PbO2(s) + H2O2 (aq) ® PbO(s) + H2O(l) + O2 (g)
(b) Its enthalpy of decomposition is high (b) Na2SO3 (aq) + H2O2(aq) ® Na2SO4(aq) + H2O(l)
(c) It undergo auto-oxidation on prolonged standing
(c) 2KI(aq) + H2O2 (aq) ® 2KOH (aq) + I2(s)
(d) None of these
(d) KNO2(aq) + H2O2 (aq) ® KNO3 (aq) + H2O(l)
EBD_7327
212 CHEMISTRY

69. Which one of the following statements is incorrect with (a) 17 (b) 51
regard to ortho- and para-dihydrogen? (c) 34 (d) 85
(a) They are nuclear spin isomers 72. The polymeric hydride is
(b) The ortho isomer has zero nuclear spin whereas the (a) CaH2 (b) NaH
para isomer has one nuclear spin (c) BaH2 (d) MgH2
(c) The para isomer is flavoured at low temperatures 73. The volume strength of a 3% w/v H2O2 sample is
(d) The thermal conductivity of the para isomer is 50% (a) 20 (b) 15
greater than that of the ortho isomer (c) 25 (d) 10
70. Permanent hardness of water is due to the presence of 74. The sum of the number of neutrons and protons in all the
(a) bicarbonates of sodium and potassium three isotopes of hydrogen is
(a) 6 (b) 5
(b) chlorides and sulphates of sodium and potassium (c) 4 (d) 3
(c) chlorides and sulphates of calcium and magnesium 75. Which of the following undergoes reduction with hydrogen
(d) bicarbonates of calcium and magnesium peroxide in alkaline medium?
71. The strength of H2O2 (in g/litre) in 11.2 volume solution of (a) Cl2 (b) I2
H2O2 is (c) PbS (d) Mn 2+

1. At its melting point ice is lighter than water because (a) the radius of the H nucleus
(a) H2O molecules are more closely packed in solid state (b) the electronegativity of H atom.
(b) ice crystals have hollow hexagonal arrangement of H2O
(c) the ionisation energy of the H-atom
molecules.
(c) on melting of ice the H2O molecule shrinks in size (d) the bond dissociation energyof H–H bond
(d) ice froms mostly heavy water on first melting. 7. D2O is preferred to H2O, as a moderator, in nuclear reactors
2. Which pair does not show hydrogen isotopes ? because
(a) D2O slows down fast neutrons better
(a) Ortho and para hydrogen
(b) D2O has high specific heat
(b) Protium & deuterium (c) D2O is cheaper
(c) Deuterium & tritium (d) None of these
(d) Tritium and protium 8. Saline hydrides react explosively with water, such fires can
3. The hydride ion, H–, is a stronger base than the hydroxide be extinguished by
ion, OH–. Which one of the following reactions will occur if (a) water (b) carbon dioxide
sodium hydride (NaH) is dissolved in water? (c) sand (d) None of these
(a) H - (aq ) + H 2 O(l) ® H3O - (aq ) 9. Which of the following chemicals is not present in clear
hard water?
(b) H - ( aq ) + H 2 O( l ) ® OH - ( aq ) + H 2 ( g )
(a) MgCO3 (b) MgSO4
(c) H - ( aq ) + H 2O(l ) ® OH - ( aq ) + 2 H + ( aq ) + 2e (c) Mg(HCO3)2 (d) CaCl2
(d) H - (aq ) + H 2O(l) ® No reaction 10. When same amount of zinc is treated separately with excess
4. HCl is added to following oxides. Which one would give of sulphuric acid and excess of sodium hydroxide solution
H2O2 the ratio of volumes of hydrogen evolved is
(a) MnO2 (b) PbO2 (a) 1 : 1 (b) 1 : 2
(c) BaO (d) None of these (c) 2 : 1 (d) 9 : 4
5. Heavy water reacts respectively with CO2, SO3, P2O5 and 11. Choose the correct statement :
N2O5 to give the compounds : The reason for use of polyphosphates as water softening
(a) D2CO3, D2SO4, D3PO2, DNO2
agents is, that
(b) D2CO3, D2SO4, D3PO4, DNO2
(c) D2CO3, D2SO3, D3PO4, DNO2 (a) they form soluble complexes with anionic species
(d) D2CO3, D2SO4, D3PO4, DNO3 (b) they precipitate out cationic species
6. The H– ion can be formed in ordinary chemical reaction (c) they precipitate out anionic species
under proper conditions, but the H+ ion cannot. The best (d) they form soluble complexes with cationic species
explanation for this difference is due to
Hydrogen 213

12. Which one of the following processes will produce hard (b) Two OH bonds lies in the same plane
water ? (c) Pale blue liquid
(a) Saturation of water with MgCO3 (d) Can be oxidised by ozone
(b) Saturation of water with CaSO4
DIRECTIONS for Qs. 19 to 25: These are Assertion-Reason
(c) Addition of Na2SO4 to water
type questions. Each of these question contains two statements:
(d) Saturation of water with CaCO3 Statement-1 (Assertion) and Statement-2 (Reason). Answer these
13. Which of the following species is diamagnetic in nature? questions from the following four options.
(a) H -2 (b) H +2 (a) Statement-1 is True, Statement-2 is True, Statement-2 is a
correct explanation for Statement -1
(c) H2 (d) He +2
(b) Statement-1 is True, Statement -2 is True ; Statement-2 is
14. Which of the following statements in relation to the NOT a correct explanation for Statement - 1
hydrogen atom is correct ? (c) Statement-1 is True, Statement- 2 is False
(a) 3s, 3p and 3d orbitals all have the same energy (d) Statement-1 is False, Statement -2 is True
(b) 3s and 3p orbitals are of lower energy than 3d orbital 19. Statement-1 : Decomposition of H2O2 is a disproportionation
(c) 3p orbital is lower in energy than 3d orbital reaction.
(d) 3s orbital is lower in energy than 3p orbital Statement-2 : H2O2 molecule simultaneously undergoes
15. In context with the industrial preparation of hydrogen from oxidation and reduction.
water gas (CO + H2), which of the following is the correct 20. Statement-1 : Temporary hardness can be removed by
statement? boiling.
(a) CO and H2, are fractionally separated using differences Statement-2 : On boiling the soluble bicarbonates change
in their densities to carbonates which being insoluble, get precipitated.
(b) CO is removed by absorption in aqueous Cu 2Cl 2 21. Statement-1 : H2O2 is not stored in glass bottles.
solution Statement-2 : Alkali oxides present in glass catalyse the
(c) H2 is removed through occlusion with pd decomposition of H2O2
(d) CO is oxidised to CO2 with steam in the presence of a 22. Statement-1 : Hydrogen combines with other elements by
catalyst followed by absorption of of CO2 in alkali losing, gaining or sharing of electrons.
16. The critical temperature of water is higher than that of O2 Statement-2 : Hydrogen forms electrovalent and covalent
because H2O molecule has bonds with other elements.
(a) Fewer electrons than oxygen 23. Statement-1 : Calgon is used for removing permanent
(b) Two covalent bonds hardness of water.
(c) V-shape Statement-2 : Calgon forms precipitates with Ca2+ and Mg2+.
(d) Dipole moment 24. Statement-1 : In alkaline solution, H2O2 reacts with
17. Blackened oil painting can be restored into original form by potassium ferricyanide.
the action of Statement-2 : H2O2 is a strong reducing agent.
(a) Chlorine (b) BaO2 25. Statement-1 : Hydrogen peroxide forms two series of salts
(c) H2O2 (d) MnO2 called peroxides.
18. Which of the following is false about H2O2 Statement-2 : Hydrogen peroxide molecule has two
(a) Act as both oxidising and reducing agent replaceable hydrogen atom.

Exemplar Questions 2. Why does H+ ion always get associated with other atoms or
molecules?
1. Hydrogen resembles halogens in many respects for which
(a) Ionisation enthalpy of hydrogen resembles that of alkali
several factors are responsible. Of the following factors
metals
which one is most important in this respect?
(b) Its reactivity is similar to halogens
(a) Its tendency to lose an electron to form a cation
(c) It resembles both alkali metals and halogens
(b) Its tendency to gain a single electron in its valence
(d) Loss of an electron from hydrogen atom results in a
shell to attain stable electronic configuration
nucleus of very small size as compared to other atoms
(c) Its low negative electron gain enthalpy value
or ions. Due to small size it cannot exist free.
(d) Its small size
EBD_7327
214 CHEMISTRY

3. Metal hydrides are ionic, covalent or molecular in nature.


C ( s ) + H 2 O ( g ) ¾¾¾¾
® CO ( g ) + H 2 ( g )
1270 K
(b)
Among LiH, NaH, RbH, CsH, the correct order of increasing
ionic character is CO ( g ) + H 2O ( g ) ¾¾¾¾
® CO2 ( g ) + H 2 ( g )
673K
(c) Catalyst
(a) LiH > NaH > CsH > KH >RbH
(b) LiH < NaH < KH < RbH < CsH (d) C 2 H 6 + 2H 2 O ¾¾¾¾
1270 K
® 2CO + 5H 2
(c) RbH > CsH > NaH > KH > LiH Ni
(d) NaH > CsH > RbH > LiH > KH 12. When sodium peroxide is treated with dilute sulphuric acid,
4. Which of the following hydrides is electron-precise hydride? we get ........ .
(a) B2H6 (b) NH3 (a) sodium sulphate and water
(c) H2O (d) CH4 (b) sodium sulphate and oxygen
5. Radioactive elements emit a, b and g rays and are (c) sodium sulphate, hydrogen and oxygen
characterised by their half-lives. The radioactive isotope of (d) sodium sulphate and hydrogen peroxide
hydrogen is 13. Hydrogen peroxide is obtained by the electrolysis of .... .
(a) protium (b) deuterium (a) water (b) sulphuric acid
(c) tritium (d) hydronium (c) hydrochloric acid (d) fused sodium peroxide
6. Consider the reactions 14. Which of the following reactions is an example of use of
(i) H2O2 + 2HI ¾¾ ® I2 + 2H2O water gas in the synthesis of other compounds?
CH 4 ( g ) + H 2 O ( g ) ¾¾¾¾
® CO ( g ) + H 2 ( g )
1270 K
(a)
(ii) HOCl + H2O2 ¾¾
® H3O + + Cl– + O2 Ni

CO ( g ) + H 2O ( g ) ( g ) + H 2 (g )
673 K
Which of the following statements is correct about H2O2 with (b) ¾¾¾¾ ® CO2
Catalyst
reference to these reactions ? Hydrogen peroxide is .....
C n H 2n + 2 + nH 2 O ( g ) ¾¾¾¾
1270 K
(a) an oxidising agent in both (i) and (ii) (c) ® nCO + ( 2n + 1) H 2
Ni
(b) an oxidising agent in (i) and reducing agent in (ii)
CO ( g ) + 2H 2 ( g )
Cobalt
(d) ¾¾¾¾ ® CH3OH ( l)
(c) a reducing agent in (i) and oxidising agent in (ii) Catalyst
(d) a reducing agent in both (i) and (ii) 15. Which of the following ions will cause hardness in water
7. The oxide that gives H2O2 on treatment with dilute H2SO4 is sample?
(a) PbO2 (b) BaO2,8H2O (a) Ca2+ (b) Na+
(c) MnO2 (d) TiO2 (c) Cl– (d) K+
8. Which of the following equations depict the oxidising nature 16. Which of the following compounds is used for water
of H2O2? softening?
(a) Ca3(PO4)2 (b) Na3PO4
(a) 2MnO -2 + 6H + + 5H 2 O 2 ¾¾
® 2Mn 2+ + 8H 2 O + 5O 2
(c) Na6P6O18 (d) Na2HPO4
(b) 2Fe3+ + 2H + + H 2 O 2 ¾¾
® 2Fe 2+ + 2H 2 O + O 2 17. Elements of which of the following group(s) of periodic table
do not form hydrides?
(c) 2I - + 2H + + H 2 O 2 ¾¾
® I 2 + 2H 2 O (a) Groups 7, 8, 9 (b) Group 13
(c) Groups 15, 16, 17 (d) Group 14
(d) KIO 4 + H 2 O 2 ¾¾ ® KIO3 + H 2 O + O 2 18. Only one element of ........ forms hydride.
9. Which of the following equation depict reducing nature of (a) group 6 (b) group 7
H2 O2 ? (c) group 8 (d) group 9
(a) 2[Fe(CN) 6 ]4 - + 2H + + H 2 O 2 ¾¾
® NEET/AIPMT (2013-2017) Questions
19. (i) H2O2 + O3 ® H2O + 2O2
2 [ Fe ( CN )6 ]
3-
+ 2H 2 O (ii) H2O2 + Ag2O ® 2Ag + H2O + O2
(b) I 2 + H 2 O 2 + 2OH - ¾¾
® 2I - + 2H 2 O + O 2 Role of hydrogen peroxide in the above reactions is
respectively - [2014]
(c) Mn 2 + + H 2 O 2 ¾¾
® Mn 4 + + 2OH -
(a) Oxidizing in (i) and reducing in (ii)
(d) PbS + 4H 2 O 2 ¾¾
® PbSO 4 + 4H 2 O (b) Reducing in (i) and oxidizing in (ii)
10. Hydrogen peroxide is .......... . (c) Reducing in (i) and (ii)
(a) an oxidising agent (d) Oxidizing in (i) and (ii)
(b) a reducing agent 20. Which of the following statements about hydrogen is
(c) both an oxidising and a reducing agent incorrect ? [2016]
(d) neither oxidising nor reducing agent (a) hydrogen has three isotopes of which tritium is the
11. Which of the following reactions increases production of most common.
dihydrogen from synthesis gas? (b) Hydrogen never acts as cation in ionic salts
CH 4 ( g ) + H 2 O ( g ) ¾¾ ¾¾
® CO ( g ) + 3H 2 ( g )
1270 K
(a) (c) Hydronium ion, H3O+ exists freely in solution
Ni
(d) Dihydrogen does not act as a reducing agent
Hydrogen 215

Hints & Solutions


EXERCISE - 1 17. (a) H + e– (1s1) ® H– (1s2 or [He])
F + e–([He]2s22p5) ® F– ( [He] 2s2 2p6 or [Ne])
1. (c) Hydrogen is not reactive at ordinary temperature
catalyst
because it is in molecular state i.e., as H2 with its 18. (b) CO + H 2+ H2O CO 2+ 2H2
complete duplet which makes it stable and lesser water gas
reactive. At very high temperature, hydrogen exists in
19. (c) Occlusion is the phenomenon of adsorption of
atomic form having one electron i.e., incomplete duplet
which makes it less stable and highly reactive. hydrogen by metal.
2. (d) Hydrogen has high ionisation energy in comparision 20. (c) Among the given ions, only Cu 2 + lies below H + in
with alkali metals because it has only one orbital i.e. electrochemical series.
smaller size. Due to smaller size, hold of nucleus on
outermost electron is greater in case of hydrogen. 21. (d) Heavy water is stable.
22. (a) Polarity of bond depends on difference in
3. (b) Q 22.4 L of H at STP = 1 mole of H2
2 electronegativity of the two concerned atoms. H2O is
1 more polar than H2S because oxygen (in O–H) is more
\ 0.224 L of H2 at STP = ´ 0.224 = 0.01 moles.
electronegative than sulphur (in S–H).
22.4
23. (d) It forms calcium and magnesium complex with Ca2+ and
4. (c) H( g ) ¾¾ ® H + ( g ) + e- . Mg2+ ions present in hard water.
5. (c) Actually hydrogen is in the first group of periodic table
24. (a) The complex salt of metaphosphoric acid sodium
due to its much resemblance with alkali metals
hexametaphosphate (NaPO3)6, is known as calgon. It
6. (c) Fe + dil. H 2SO 4 ® FeSO 4 + H 2 ­ is represented as Na2[Na4(PO3)6]
3Fe + 4H 2O ® Fe 3O 4 + 4H 2 ­ 25. (b) Temporary hardness is due to presence of bicarbonates
Steam of calcium and magnesium and permanent hardness is
Cu + dil. HCl ® No reaction due to the sulphates and chlorides of both of calcium
Copper does not evolve H2 from acid as it is below and magnesium.
hydrogen in electrochemical series. 26. (c) Pure hydrogen is evolved by reacting absolute alcohol
2Na + 2C 2 H 5OH ® 2C 2H 5ONa + H 2 ­ and Na
7. (a) Two ice cubes stick to each other due to H-bonding C 2 H 5 OH + Na ® C 2 H 5 ONa + ½ H 2
8. (c) The correct option is (c). In H2O, hydrogen has + 1 other statements are correct. See text.
oxidation no. In CaH2, hydrogen has – 1 oxidation no. 27. (c) Sea water is purified by reverse osmosis.
9. (a) Hydrogen can be differentiated from after group I 28. (d) The hybridisation in water is sp3 and bond angle 104.5º
elements due to its non-metallic character. 29. (b) D2O is used in nuclear reactors as moderator.
10. (d) Chlorine has lone pair which it can donate to form 30. (a) Calgon process is used to remove permanent hardness
coordinate bond while hydrogen cannot. of water
11. (d) H2 will not reduce heated Al2O3. As Al is more electro-
31. (b) Na zeolite + CaCl 2 ® Ca zeolite + 2NaCl
positive than hydrogen. therefore, its oxide will not be
reduced by hydrogen. 2
32. (a) % hydrogen in H2O2 = ´ 100 = 5.88%
12. (b) Mn + 2HNO3 (dil.) ® Mn(NO3)2 + H2 34
13. (c) By electrolysis of 50% ice cold H2SO4. 33. (c) Potash alum is used for purifying water.
+ 34. (c) The high boiling point of water is due to H-bonding.
2H 2SO 4 ¾
¾® 2H + 2HSO -4
35. (a) CaC 2 + 2D 2 O ® C 2 D 2 + Ca (OD) 2
At anode: 2HSO -4 ¾
¾® H 2S 2 O 8 + 2e - 36. (b) Due to high dielectric constant, water acts as a good
solvent therefore it is also called a universal solvent.
H 2S 2 O 8 + H 2 O ¾
¾® H 2SO 5 + H 2SO 4 37. (c) Ca(OH)2 is used for the softening of temporary hard
water.
H 2SO 5 + H 2 O ¾
¾® H 2SO 4 + H 2 O 2 Ca(OH) 2 (aq) ¾¾ ® CaCO3 (s)+ H 2O(l )
14. (c) Hydrogen behaves as a metal at very high pressure. cloudiness
15. (d) H2O2 can not oxidise O3. 38. (a) D2O actually has higher freezing point (3.8°C) than water
O3 oxidises H2O2 H2O (0°C)
H2O2 + O3 ® H2O + 2O2 39. (b) Water has high dielectric constant i.e., 78.39 C2/Nm2,
Option (d) is correct. high liquid range and can dissolve maximum number of
16. (d) Hydrogen is a non-metal while all other members of compounds. That is why it is used as universal solvent.
group 1 (alkali metals) are metals. 40. (c) H2S is oxidised to S by H2O2
EBD_7327
216 CHEMISTRY

41. (a) The triple point of any substance is that temperature 54. (a) Glycerol, phosphoric acid or acetanilide is added to
and pressure at which the material can exist in all three H2O2 to check its decomposition.
phases (Solid, liquid and gas) in equilibrium specifically 55. (a) H 2O 2 ® H 2O + [O]
the triple point of water is 273. 16K at 611.2 Pa. weak acid
42. (c) Heavy water is formed by the combination of heavier 56. (a) Zinc has no action on cold water.
isotope (1H2 or D) with oxygen. 57. (c) H2O2 is unstable liquid and decomposes into water
2D 2 + O 2 ® 2D 2 O and oxygen either on standing or on heating.
Heavy water 58. (d) H2O2 show all these properties.
43. (a) 59. (a) As H2O2 is loosing electrons so it is acting as reducing
SO 3 + D 2 O ¾
¾® D 2SO 4
agent.
(Deutero sulphuric acid) 60. (d) An ion exchange resin containing
vol. strength R–COOH group exchange cations like Ca2+, Mg2+, Na+,
44. (a) Normality of H2O2 =
5.6 Fe2+ with H+ when hard water is passed through it.
Volume of normal (1N) H2O2 solution = 5.6 volumes. This resin is called cation exchange resin.
\ Volume of strength of 1.5 N H2O2 61. (b) Acidified KMnO4 (pink or purple) is reduced to
= 1.5 × 5.6 = 8.4 volumes. colourless Mn++ ion :
45. (d) Decomposition of H2O2 can be accelerated by finely 2KMnO4 + 3H 2SO 4 ¾¾ ® K 2SO4
divided metals such as Ag, Au, Pt, Co, Fe etc. + 2MnSO4 + 3H2O + 5[O]
68 ´ 10 H2O2 + [O] H2O + O2] × 5
46. (b) Strength of 10V H2O2 = ´ 100 = 3.035%
22400 +7 –1
47. (d) It is weakly acidic in nature and pure hydrogen peroxide 2KMnO 4 + 3H 2SO 4 + 5H 2 O 2 ¾¾
®
turns blue litmus red. (Ka = 1.57 × 10–12 at 293 K). It Pink or purple

ionises in two steps. 0


H2O2 H+ + O2 K 2SO 4 + 2MnSO4 + 8H 2 O + 5O2
Colourless
HO -2 H + + O 22 - 62. (d) O – O – H bond angle in H2O2 is 94.8°.
So, option (d) is not true. 63. (b) Oxidation
H 2 O 2 + [O ] ¾¾ ¾ ¾ ¾® H 2 O + O 2 ­
48. (b) Structure of H2O2 is nonplanar
49. (a) H2SO4 + BaO2 ® BaSO4 + H2O2 64. (b) \ Correct choice : (b)
65. (a) Mg + 2H2O ® Mg(OH)2 + H2 ­
-2 0
50. (c) H 2 S + H 2O2 ® S+ 2H 2O 66. (a) A fusion reaction is difficult to occur because positively
charged nuclei repel each other. At very high
In this reaction H2O2 shows oxidising nature.
temperature of the order of 106 to 107 K, the nuclei may
51. (a) Eq. wt. of H2O2 = 17
have sufficient energy to overcome the repulsive forces
30.36 and fuse. It is for this reason, fusion reactions are also
N= = 1.78 N
17 called thermonuclear reactions. Hence, hydrogen can
Volume strength = 5.6 × normality be fused to form helium at high temperature and high
= 5.6 × 1.78 = 10 V pressure.
52. (a) This method is known as Clark's process. In this method 67. (c) Hydrogen is the lightest gas. It is insoluble in water.
temporary hardness is removed by adding lime water 68. (a) In the following reaction H2O2 acts as a reducing agent.
or milk of lime. PbO2(s) + H2O2 (aq) ® PbO(s) + H2O(l) + O2 (g)
Ca(OH)2 + Ca(HCO3 )2 ¾¾
® 2CaCO3 ¯ +2H 2O 69. (b) Ortho isomer has one nuclear spin whereas that para
ppt. isomer has zero nuclear spin.
53. (b) Q 22.4 litre O2 at N.T.P. obtained by 68 gm of H2O2 70. (c) Permanent hardness of water is due to chlorides and
sulphates of calcium and magnesium.
68
\ 1 litre O2 at N.T.P. obtained by gm of H2O2 71. (c) 1 mL of solution contains 0.03035 g H2O2
22.4 Strength of 11.2 volume of solutions.
\ 20 litre O2 at N.T.P. obtained by = 3.035 × 11.2 g/L = 34 g/L
68 72. (d) Due to its covalent nature MgH2 is Polymeric in nature.
´ 20 gm of H2O2 = 60.71 gm of H2O2
22.4 73. (d) The volume strength of 1% H2O2 = 3.3V
\ 1000 ml O2 at N.T.P. obtained by = 60.71 gm of H2O2 Then the volume strength of 3% H2O2 = 10V
\ 100 ml O2 at N.T.P. obtained by 74. (a) 1H1 1D2 1T3
60.71 no. of neutrons respectively are 0 , 1 , 2
= ´ 100 = 6.071% no. of protons respectively are 1 , 1 , 1
1000
Hence the sum of protons + neutrons = 1 + 2 + 3 = 6
Hydrogen 217

75. (b) It reduces iodine to iodine ion in alkaline medium. 14. (a) In one electron species, such as H-atom, the energy
I2 (s) + H2O2(aq) + 2OH– ¾® 2I– (aq) + 2H2O(l) + O2 (g) of orbital depends only on the principal quantum
number, n.
EXERCISE - 2 Hence answer (d)
i.e., 1s < 2s = 2p < 3s = 3p = 3d <
1. (b) In the structure of ice each molecule of H2 O is 4s = 4p Þ 4d = 4g
surrounded by three H2O molecules in hexagonal honey 15. (d) On the industrial scale hydrogen is prepared from water
comb manner which results an open cage like structure. gas according to following reaction sequence
As a result there are a number of 'hole' or open spaces. catalyst
In such a structure lesser number of molecules are CO + H 2 + H 2 O ¾¾¾¾
® CO2 + 2H 2
1424 3 {
packed per ml. When ice melts a large no. of hydrogen water gas (steam)
bonds are broken. The molecules therefore move into 2NaOH
¾¾¾¾
® Na 2 CO3 + H2O
(alkali)
the holes or open spaces and come closer to each other
than they were in solid state. This result sharp increase From the above it is clear that CO is first oxidised to
in the density. Therefore ice has lower density than CO2 which is then absorbed in NaOH.
water. 16. (d) Critical temperature of water is more than O2 due to its
2. (a) Ortho and para hydrogens are two forms of hydrogen dipole moment (Dipole moment of water = 1.84 D; Dipole
(1H1 ) which differ only in the direction of spin of moment of O2 = zero).
proton. 17. (c) H2O2. The key reactions are
3. (b) H - (aq)+ H 2O(l) ¾¾ ® OH - (aq)+ H 2 ( g ) PbO + H 2S ® PbS + H 2O
base 1 acid 1 base 2 acid2 (black)
In this reaction H– acts as bronsted base as it accepts PbS + 4H 2 O 2 ® PbSO 4 + 4H 2O
one proton (H+) from H2O and form H2. (white)
4. (d) MnO2 , PbO2 and BaO will not give H2O2 with HCl. When blackened statues are treated with H2O2, the
MnO2 and PbO2 will give Cl2 and BaO will react with PbS is oxidised to PbSO4, which is colourless (White).
HCl to give BaCl2 and water. 18. (b) The value of dipole moment of H2O2 is 2.1 D, which
5. (d) suggest the structure of H2O2 cannot be planar.
6. (c) Single e– of the H atom cannot be readily removed to An open-book structure is suggested for H2 O2 in
form H–
which O – H bonds lie in different plane.
7. (d) H2 O absorbs neutrons more than D2 O and this
decreases the number of neutrons for the fission 19. (a) Both statement-1 and statement-2 are true and statement-2
process. is the correct explanation of statement-1.
8. (c) Fire due to action of water on saline hydrides cannot 20. (a) 21. (a) 22. (a)
be extinguished with water or CO2. These hydrides can 23. (c) Both assertion (A) is correct reason (R) is not true.
reduce CO2 at high temperature to produce O2. Correct Reason : Calgon forms soluble complexes with
9. (a) MgCO3 is insoluble in water. Ca2+ and Mg2+ in which properties of these ions are
10. (a) Zn + H2SO4 ® ZnSO4 + H2
masked.
Zn + 2NaOH ® Na2ZnO2 + H2
24. (a)
\ Ratio of volumes of H2 evolved is 1 : 1
11. (d) Polyphosphates (e.g., sodium hexa metaphosphate) 25. (a) Both Assertion (A) and reason (R) are true. Hydrogen
form soluble complexes with cations such as Ca2+, peroxide forms two series of salts called hydroperoxides
Mg2+ and so they are used as water softening agents. and peroxides.

Na 2 [Na 4 (PO3 )6 ] + 2Ca 2+ ¾¾


® Na 2 [Ca 2 (PO3 ) 6 ]+ 4Na + EXERCISE - 3
"Calgon " Hardness "Soluble"
Sod. hexa metaphosphate Exemplar Questions
12. (b) Permanent hardness of water is due to chlorides and 1. (b) Hydrogen like halogens accept an electron readily to
sulphates of calcium and magnesium i.e CaCl2, CaSO4,
achieve nearest inert gas configuration.
MgCl2 and MgSO4.
2. (d) H+ ion always get associated with other atoms or
13. (c) A diamagnetic substance contains no unpaired
molecules because loss of an electron from hydrogen
electron.
atom results in a nucleus of very small size as compared
H2 is diamagnetic as it contains all paired electrons
to other atoms or ion. Due to small size it cannot exist
H 2 = s b2 , H 2+ = s1b , H 2- = s b2 , s *1
a ; free.
(diamagnetic) (paramagnetic) (paramagnetic) 3. (b) The stability of hydrides decreases from Li to Cs. It is
because of the fact that M-H bond becomes weaker
H +2 = s1b , H 2- = s b2 , He2+ = s b2 , s*1
a due to increase in size of alkali metals down the group.
aramagnetic) (paramagnetic) (paramagnetic) (paramagnetic)
Hence, ionic character increases as the size of the atom
EBD_7327
218 CHEMISTRY

increases i.e., the correct order of increasing ionic


CO ( g ) + H 2 O ( g ) ¾¾¾¾¾¾® CO 2 ( g ) + H 2 ( g )
4 FaCrO . 673 K
character is
LiH < NaH < KH < RbH < CsH CO2 is removed by scrubbing with a solution of sodium
4. (d) Electron-precise hydrides are the type of hydrides in arsenite.
which the number of electrons present is equal to the
12. (d) Na 2 O 2 + dil. H 2SO 4 ¾¾
® Na 2SO 4 + H 2 O 2
number of electrons required.
5. (c) The radioactive isotope of hydrogen is tritium. Its half 13. (b) Hydrogen peroxide is manufactured by electrolysis of
life is about 12.32 years. 50% sulphuric acid followed by distillation. The
6. (b) (i) distillate is 30% hydrogen peroxide. The first product
of electrolysis is peroxy disulphuric acid.
Reduction 2H2SO4 ----® 2H+ + HSO4–
-1 -1 0 -2
+ 2HI ¾ ¾ ® I2 + 2H 2 O 2HSO 4- ( aq ) ¾¾¾¾¾® H 2 S2 O8 + 2e -
H 2 O2 Electrolysis
Oxidation
® 2H 2 SO 4 + H 2 O 2 ( aq )
H2S2O8 + 2H2O ¾¾
Here H2O2 oxidises HI into I2 hence, behaves as an
oxidising agent. 14. (d) The water gas is a combination of carbon and hydrogen.
It is used in manufacturing of methanol.
Reduction CO ( g ) + 2H 2 ( g ) ¾¾¾¾
Cobalt
® CH3OH ( l)
+1 -1 0 Catalyst
+ -
(ii) HOCl + H 2O 2 ¾¾
® H 3O + Cl + O 2 15. (a) Bicarbonates, chlorides and sulphates of Ca and Mg
Oxidation are responsible for the hardness of water.
Here, H2O2 reduces HOCl to Cl, thus, behaves as 16. (c) Sodium hexametaphosphate also known as Calgon is
reducing agent. used.
7. (b) Oxides such as BaO2 , Na2O2 etc; which contain 2CaCl 2 + Na 6 P6O18 or Na 2 éë Na 4 ( PO3 ) 6 ùû ®
peroxide linkage on treatment with dilute H2SO4 give
H2O2 but dioxides such as PbO2, MnO2, TiO2 do not ( From hard
water ) Sodium
hexametaphosphate
give H2O2 on treatment with dilute H2SO4.
Na 2 éë Ca 2 ( PO3 )6 ùû + 4NaCl
BaO2 .8H 2 O ( s ) + H 2SO 4 ( aq ) ¾¾
® Complex Salt
Hydrated barium
peroxide 17. (a) Elements of group 7, 8 and 9 of d-block do not form
hydrides at all. The inability of metals of group 7, 8 and
BaSO 4 (s) + H 2 O 2 ( aq ) + 8H 2O ( l )
Hydrogen
9 of periodic table to form hydrides is referred to as
peroxide hydrides gap of d-block. In these compounds H atoms
are supposed to occupy interstitial position in the metal
Reduction
lattices. They are also called non-stoichiometric
- + -1 0 -2
8. (c) 2I + 2H + ¾¾® I2 + 2H 2 O
H2 O2 hydrides.
Oxidation 18. (a) Only one element of group 6, i.e., Cr forms hydride.
Here H2O2 oxidises I– to I2, hence behaves as oxidising NEET/AIPMT (2013-2017) Questions
agent.
9. (b) The given below reaction show the reducing action in Reduction
basic medium. –1 –2
Reduction 19. (c) (i) H2O2 + O3 H2O2 + 2O2

–1
I02 + H 2 O2 + 2OH - ¾ ¾
® 2I- + 2H 2 O + O02 Oxidation
Oxidation
Oxidation –1
10. (c) Hydrogen peroxide acts as an oxidising as well as (ii) H2O2 + Ag2O –2
2Ag + H2O +2O2
reducing agent in both acidic and alkaline media.
11. (c) The production of syn gas or synthesis gas from coal Reduction
is called coal gasification. Hence in both the reactions H2 O2 is acting as an
oxidising agent.
C ( s ) + H 2 O ( g ) ¾¾¾¾ ® CO ( g ) + H 2 ( g )
1270 K
Coal Steam Ni 1442443 20. (a & d) Among the three isotopes of hydrogen, Protium (1H1)
Syn gas is most common. It is an energetic reducing agent. It
In order to increase the production of hydrogen steam reduces oxides, chlorides and sulphides of certain metals
is added to carbon monoxide of the syn gas in the and produce free metals at ordinary temperature.
presence of iron chromate as a catalyst at 673 K. CuO + 2H ® Cu + H2O
10
The s-Block
Elements
GROUP 1 ELEMENTS : ALKALI METALS Chemical Properties:
(Li, Na, K, Rb, Cs, Fr) (i) Reactivity towards air:
Li forms mono oxide (M2O) and Na forms peroxide (M2O2)
Atomic and Physical Properties and K, Rb and Cs form superoxide (MO2).
(i) General electronic configuration: General configuration of
® 2Li 2 O ( oxide )
4Li + O 2 ¾¾
alkali metals is ns1. They are highly electropositive metals,
thus not found in free state in nature. ® Na 2 O 2 ( peroxide )
2Na + O ¾¾
(ii) Atomic and ionic radii : These elements have the largest size
in a particular period. Down the group their atomic size ® MO 2 ( superoxide ) (M = K, Rb, Cs)
M + O 2 ¾¾
increases. Basic nature, ionic character and reactivity of these oxides
(iii) Ionization enthalpy: Alkali metals have low value of IE which increases from Li to Cs. Due to high reactivity towards air
decreases on moving down the group. As a result, Cs is the and water, they are normally kept in kerosene oil.
most electropositive element in the whole of periodic table. (ii) Reactivity towards water:
(iv) Hydration enthalpy: Hydration energy decreases on going
® 2M + + 2OH - + H 2 (M = alkali metal)
2M + 2H 2 O ¾¾
down in the group, due to increase in the size of metal ion
Li+ > Na+ > K+ > Rb+ > Cs+ Due to its small size and high HE, Li reacts less
\ Li+ has maximum degree of hydration. Due to high degree vigorously with water as compared to other alkali metals
of hydration, lithium salts are mostly hydrated, example : LiCl. which react explosively with water. Basic nature of these
2H2O hydroxides increases from Li to Cs.
(v) Density : The density of alkali metals are quite low as LiOH < NaOH < KOH < RbOH < CsOH
compared to other metals. Order of densities of alkali metals Ionic character, melting point, boiling point, reactivity , thermal
Li < K < Na < Rb < Cs stability and solubility in water increases from Li to Cs.
(vi) Flame test : Elements and their respective colours imparted (iii) Reactivity towards dihydrogen:
to the flame are given below.
® 2M+ + 2H-
2M + H2 ¾¾
Element Li Na K Rb Ca
These hydrides are ionic solids with high m.pts.
Colour Crimson Golden Violet Red Blue
Thermal stability of LiH is highest. They are ionic hydrides
red yellow violet
and their stability depends on lattice energy.
(vii) Lustrous surface : Lustre is due to mobile electrons in the
LiH > NaH > KH > RbH > CsH
metallic lattice.
(iv) Reactivity towards halogens:
(viii) Tendency of forming complex compounds : These metals have
weak tendency of forming complex compounds due to large Alkali metals directly combine with halogen to form
size and low charge density. halides (MX)
(ix) Melting point and boiling point : Their melting and boiling ® 2M + + 2X -
2M + X 2 ¾¾
points are low due to weak metallic bonds. Strength of metallic Except lithium halides which are covalent, other alkali metal
bond decreases in the group from Li to Cs, due to which halides are ionic in nature.
hardness decreases from Li to Cs. The ionic compounds get dissolved in water, while the
Li > Na > K > Rb > Cs covalent compounds get hydrolysed.
(x) Photoelectric effect : Size of Cs and K is large and they LiCl gets hydrolysed due to its covalent nature. Decreasing
possess low I.E. Both contain one electron in outermost shell order of these halides in undergoing hydrolysis is as follows
which got emitted by absorption of visible light LiCl > NaCl > KCl > RbCl > CsCl
EBD_7327
220 CHEMISTRY

(v) Reducing nature: They are strong reducing agents, Li being (ii) Halides: They are high melting, colourless crystalline solids.
the strongest and Na the weakest. Electrode potential They have high –ve enthalpies of formation. M.pts and B.pts
depends upon sublimation energy, ionization energy and follow the order: F– > Cl– > Br– > I–
hydration energy. Li due to its small size has the highest Note : The low solubility of LiF in water is due to its high
hydration energy which accounts for its high negative E° lattice enthalpy while the low solubility of CsI is due to small
value. HE of the two ions.
(vi) Solution in liquid ammonia: Alkali metals have low I.E. and (iii) Salts of Oxo-acids : Oxo acids are those in which the acidic
large size thus they dissolve in liquid ammonia to give deep proton is on a hydroxyl group with an oxo group attached to
blue solution which are conducting in nature. the same atom. Ex: H2CO3, H2SO4.
- Alkali metals form salts with all the oxo-acids. As
+
M + ( x + y ) NH3 ¾¾ ® éë M ( NH3 ) x ùû + éêe ( NH3 ) y ùú electropositive character increases down the group, stability
ë û of carbonates and hydrogencarbonates increases.
The ammoniated electrons is responsible for the blue colour Li2CO3 being unstable to heat, decomposes to form Li2O
of the solution. These solutions are paramagnetic. and CO2.
On standing or in presence of impurities or catalyst the blue
Anomalous Properties of Lithium
colour solution forms amide.
The anomalous behaviour of Li is due to:
2M + ( NH3 ) x + 2e - (NH3 ) y ¾¾
® (i) its exceptionally small size
2M + NH -2 + H 2 + (x + y - 2)NH3 (ii) high polarising power
In concentrated solution, blue colour changes to bronze This give rise to covalent character in Li compounds.
colour and solution becomes becomes diamagnetic. Points of difference between Li and other alkali metals
(vii) Salts of oxoacids : Metal carbonates : (i) Li is much harder. Its m.pt and b.pt. are higher than other
(a) All these metals form M2CO3 type carbonates. alkali metals.
(Li2CO3, Na2CO3, K2CO3, Rb2CO3, Cs2CO3) (ii) It is the least reactive and is the strongest reducing agent
(b) Li 2 CO3 is least stable out of all these carbonates, among all the alkali metals.
because it is covalent and decomposes to Li2O and CO2 (iii) LiCl is deliquescent and crystallises as a hydrate.
at low temperature. Order of their stability is as follows : (iv) LiHCO3 is not obtained in solid state while other elements
Li2CO3 < Na2CO3 < K2CO3 < Rb2CO3 < Cs2CO3 form solid hydrogencarbonates.
Sulphates : Li2SO4 < Na2SO4 < K2SO4 < RbSO3 < Cs2SO4 (v) Unlike other alkali metals, Li does not form ethynide with
Nitrates : LiNO3 decomposes to Li2O at low temperature, ethyne
whereas NaNO3 gets decomposed to NaNO2 (vi) LiNO3 on heating gives Li2O while others decompose to
Bicarbonates : These metals form MHCO3 type bicarbonates nitrite.
and thermal stability of these bicarbonates increases from Li 4LiNO3 ¾¾
® 2Li 2 O + 4NO 2 + O 2
to Cs.
(viii) Nitrides : Among all alkali metals, only lithium directly 2NaNO3 ¾¾ ® 2NaNO 2 + O 2
combines with nitrogen to form nitride. (vii) LiOH, Li2CO3, LiF and Li2O are less soluble in water compared
to corresponding compounds of other alkali metals.
General Characteristics of Compounds of Alkali
Metals Diagonal Relationship Between Lithium and
(i) Oxides and hydroxides: On combustion in excess of air, Li Magnesium
forms Li2O, Na forms Na2O2 and K, Rb, Cs form superoxides (i) Li and Mg both are hard metals due to the presence of strong
of the formula, MO2. metallic bonds in them.
As the size of the metal ion increases, stability of peroxide or (ii) Li and Mg both are hard, therefore, their melting and boiling
superoxide increases due to stabilisation of large anions by points are high.
larger cations through lattice energy effects. (iii) LiOH and Mg(OH)2 both are weak bases.
These oxides are easily hydrolysed by water to form (iv) LiCl and MgCl2 are insoluble in water due to their covalent
hydroxides: nature, but soluble in organic solvents.
® 2M+ + 2OH-
M2O + H2 O ¾¾ (v) LiCl and MgCl2 get hydrolysed due to their covalent nature.
(vi) Li and Mg directly combine with O2 to form normal oxides
M 2 O 2 + 2H 2O ¾¾ ® 2M + + 2OH - + H 2O 2 (Li2O and MgO) while other members of their groups form
2MO 2 + 2H 2 O ¾¾ ® 2M + + 2OH - + H 2 O2 + O2 peroxide and super oxide.
The pure, oxides and peroxides are colourless while (vii) Li and Mg directly combine with N2 to form Li3N and Mg3N2
superoxides are yellow or orange in colour. Superoxides are (viii) Carbonates and nitrates of Li and Mg are unstable and readily
paramagnetic in nature. The alkali metal hydroxides are the decompose to form oxides.
strongest of all bases. They dissolve in water to produce (ix) Hydration energies of Li+1 and Mg+2 ions are higher due to
large amount of heat due to intense hydration. small size.
The-s-Block Elements 221

Some Important Compounds of Sodium (iii) Sodium Hydroxide, NaOH


(i) Sodium Carbonate (Washing soda); Na2CO3.10H2O Preparation:
Preparation: It is prepared by solvay process as follows: It is prepared by electrolysis of NaCl in castner-kellner cell.
® ( NH 4 ) 2CO3
2NH3 + H 2 O + CO 2 ¾¾ Brine solution is electrolysed using Hg cathode and C anode.

( NH 4 ) 2CO3 + H 2O + CO 2 ¾¾® 2NH 4 HCO 3 Hg


At cathode: Na + + e - ¾¾¾ ® Na - amalgam
NH 4 HCO3 + NaCl ¾¾
® NH 4 Cl + NaHCO3 1
-
D ® Cl2 + e-
At anode: Cl ¾¾
2NaHCO3 ¾¾® Na 2CO3 + CO 2 + H 2 O 2
Note : K2CO3 cannot be prepared by this method because
KHCO3 is too soluble to be precipitated by addition of 2Na - amalgam + 2H 2 O ¾¾
® 2NaOH + 2Hg + H 2
NH4HCO3 to saturated solution of KCl Properties:
Properties: (a) It is a white, translucent solid which melts at 591 K.
(a) It is white crystalline solid, existing as a decahydrate. (b) It dissolves in water to give an alkaline solution.
(b) It is readily soluble in water.
(c) Crystals of NaOH are deliquescent.
(c) On heating, it loses water of crystallisation to form
monohydrate. (d) It reacts with CO2 in the atmosphere to form Na2CO3.
Uses: It is used
D
Na 2CO3 .10H 2O ¾¾® Na 2CO3 .H 2O + 9H 2 O (a) in manufacture of soap, paper, artificial silk and chemicals.
Above 373 K, monohydrate becomes completely (b) in petroleum refining
anhydrous. (c) in purification of bauxite
>373K (d) for preparation of fats and oils
Na 2 CO3 .H 2 O ¾¾¾¾
® Na 2 CO 3 + H 2 O (e) as a laboratory reagent.
soda ash
(iv) Sodium Hydrogen Carbonate (Baking Soda), NaHCO3
(d) The carbonate part gets hydrolysed to form alkaline Preparation:
solution It is made by saturating a solution of Na2CO3 with CO2.
CO32 - + H 2 O ¾¾
® HCO 3- + OH -
Na 2CO3 + H 2O + CO2 ¾¾
® 2NaHCO3
Uses:
(a) It is used in water softening, laundering and cleaning. Properties:
(b) It is used in manufacture of glass, soap, borax and caustic It is known as baking soda because it decomposes on heating
soda. to generate bubbles of CO2.
(c) It is used in paper, paints and textile industries. Uses :
(d) It is an important laboratory reagent. (a) It is mild antiseptic for skin infections.
(ii) Sodium Chloride; NaCl (b) It is used in fire extinguishers.
Preparation : It is generally obtained by evaporation of sea
water. Crude NaCl, obtained by crystallisation of brine Biological Importance of Na and K
solution, contains Na2SO4, CaSO4, CaCl2 and MgCl2 as Importance of Na+ : Na+ ions are found on the outside of cells and
impurities. To obtain pure NaCl, crude salt is dissolved in participate in transmission of nerve signals, in regulating flow of
minimum amount of water and filtered to remove insoluble water across cell membranes and in transport of sugars and amino
impurities. The solution is then saturated with HCl gas, acids into cell.
crystals of pure NaCl separate out due to common ion effect, Importance of K+ : K+ ions are the most abundant cations within
leaving behind calcium chloride and magnesium chloride in cell fluids, where they activate many enzymes, participate in
solution.
oxidation of glucose to produce ATP and with sodium, are
ˆˆ† Na + + Cl –
NaCl ‡ˆˆ responsible for the transmission of nerve signals.
ˆˆ† H + + Cl –
HCl ‡ˆˆ GROUP 2 ELEMENTS: ALKALINE EARTH METALS
Properties: (Be, Mg, Ca, Sr, Ba, Ra ® radioactive)
(a) It melts at 1081 K. It is white crystalline solid.
(b) Its solubility is 36 g in 100 g of water at 273 K and the Atomic and Physical Properties
solubility does not increase appreciably with increase in (i) Electronic Configuration :General electronic configuration
temperature. of alkaline earth metals is [noble gas] ns2. Their compounds
Uses are ionic in nature.
(a) It is used as common salt or table salt for domestic (ii) Atomic and Ionic radii: Their radii are smaller than alkali
purposes. metals due to increased nuclear charge. The radii increases
(b) It is used for preparation of Na2O2, NaOH and Na2CO3.
down the group with increase in atomic number.
EBD_7327
222 CHEMISTRY

(iii) Ionization enthalpies: Basic nature (BeO < MgO < CaO < SrO < BaO), ionic character,
They have low IE due to large size and their IE decreases melting point, boiling point, thermal stability reactivity and
down the group. The first IE of group 2 elements are higher solubility in water of these oxides increases from Be to Ba,
than those of group 1 elements due to their smaller size than Ionic character, melting point, boiling point, reactivity, thermal
group 1 elements. Second IE’s of group 2 elements are smaller stability and solubility in water of hydroxides increases from
than those of group 1 elements. This is because removal of Be to Ba. Increasing order of basic character of these
second e– from group 2 elements results in stable noble gas hydroxides is as follows :
configuration. On other hand, second e– in group 1 has to be Be(OH)2 < Mg(OH)2 < Ca(OH)2 < Sr(OH)2 < Ba(OH)2
removed from noble gas core which needs large amount of (ii) Reactivity towards halogens:
energy. M + X 2 ¾¾ ® MX 2 (X = F, Cl, Br, I)
(iv) Hydration enthalpies BeF 2 is best prepared by thermal decomposition of
Their HE’s decreases with increase in ionic size, i.e., HE of (NH4)2BeF4 . BeCl2 is prepared by :
group 2 cations will be in following order : BeO + C + Cl 600-800K BeCl + CO
2 2
Be2+ > Mg2+ > Ca2+ > Sr2+ > Ba2+
Covalent character of these halides decreases from BeCl 2 to
Due to their smaller sizes than group 1 elements, HE’s of
BaCl2. Therefore, the amount of hydrolysis also decreases
group 2 elements are larger than those of group 1 elements.
from BeCl2 to BaCl2.
As a result, their compounds are more highly hydrated than
(iii) Reactivity towards hydrogen:
those of alkali metals.
Except Be, all elements combine with hydrogen to form
(v) Density : Atomic weight increases from Be to Ba in a group
hydride, MH2.
and volume also increases, but increase in atomic weight is BeH2 is prepared by:
more as compared to volume. Therefore, density increases
2BeCl2 + LiAlH 4 ¾¾ ® 2BeH 2 + LiCl + AlCl3
from Be to Ba.
Exception : These metals (except Be) combine with hydrogen to form
MH2 type hydrides. Thermal stability of these hydrides is as
Density of Mg is more as compared to Ca (Ca < Mg).
follows.
Density : Ca < Mg < Be < Sr < Ba
BeH2 < MgH2 > CaH2 > SrH2 > BaH2
(vi) Flame Test : Elements and their respective colour imparted
(iv) Reactivity towards acids:
to the flame are given below :
M + 2HCl ¾¾ ® MCl 2 + H 2
Element Be Mg Ca Sr Ba
(v) Reducing nature: They are strong reducing agents.
Flame – – Brick Blood Apple
(vi) Solutions in liquid ammonia:
green red red
2+ -
(vii) Photoelectric effect : Due to small size of these metals as ® éë M ( NH 3 ) x ùû
M + ( x + y ) NH 3 ¾¾ + 2 éêe ( NH 3 ) y ùú
ë û
compared to alkali metals, their ionisation potential is high.
Thus, electrons can be released only by high energy The solution is deep blue black in colour.
radiations. General Characteristics of Group 2 Compounds
(viii) Lustrous Surface : Lustre is due to mobile electron in the (i) Oxides and Hydroxides:
metallic lattice. They have high DH°f values and are thus very stable to heat.
BeO is amphoteric while other oxides are basic in nature.
(ix) Tendency of forming Complex Compounds : These metals
have higher tendency of forming complex compounds as ® M ( OH ) 2
MO + H 2 O ¾¾
compared to alkali metals, due to their relatively smaller size. BeO is covalent while other oxides are ionic in nature. The
This tendency decreases from Be to Ba. solubility, thermal stability and basic character of these
(x) Melting Point and Boiling Point : Melting and boiling points hydroxides increases down the group from Mg(OH)2 to
of these metals are low, but these metals are harder as Ba(OH)2
compared to metals of IA group. Be(OH)2 is amphoteric as shown below:
Hardness decreases from Be to Ba, due to which melting 2-
Be ( OH )2 + 2OH - ¾¾
® éë Be ( OH )4 ùû Berylate ion
and boiling points decrease.
Be > Ca > Sr > Ba > Mg Be ( OH )2 + 2HCl + 2H 2O ¾¾
® éë Be ( OH ) 4 ùû Cl 2
Chemical Properties: (ii) Halides
(i) Reactivity towards air and water : Be and Mg form normal Be-halides are covalent while other group 2 halides are ionic
MO type oxides, whereas, Ca, Sr and Ba form normal oxides in nature. In solid state, BeCl 2 has chain structure. In vapour
(MO) as well as peroxides MO2 . On burning in air they give phase, BeCl2 exists as a chloro-bridged dimer. Down the
corresponding oxides and nitrides. group, tendency to form halide hydrates decreases. Due to
They react with H2O (even in cold) to form hydroxides. high IE, fluorides are less soluble than chlorides.
The-s-Block Elements 223

(iii) Salts of Oxo-acids : Some Important Compounds of Calcium


Metal carbonates : (i) Calcium Oxide or Quick lime, CaO
(1) All these metals form MCO3 type carbonates Preparation
By heating limestone at 1070-1270 K
(BeCO3, MgCO3, CaCO3, SrCO3, BaCO3)
CaCO3 heat CaO + CO
(2) BeCO3 is least stable out of all these carbonates because 2
it is covalent and decomposes to BeO and CO2 at low For the reaction to proceed, CO2 is removed as soon as it is
temperature. Order of their stability is as follows : formed.
BeCO3 < MgCO3 < CaCO3 < SrCO3 < BaCO3 Properties:
(a) It is a white amorphous solid with a melting point of
(3) Stability of carbonates of IA group metals > stability of 2870 K.
carbonates of IIA group metals. (b) On exposure to air, it absorbs moisture and CO2
Sulphates : CaO + H 2 O ¾¾ ® Ca ( OH )2
Solubility of Sulphates : CaO + CO 2 ¾¾ ® CaCO3
BeSO4 > MgSO4 > CaSO4 > SrSO4 > BaSO4 (c) Being basic, it combines with acidic oxides at high
Increasing order of thermal stability temperature.
BeSO4 < MgSO4 < CaSO4 < SrSO4 < BaSO4 CaO + SiO2 ¾¾ ® CaSiO3
Nitrates : These metals also form M(NO3)2 and all nitrates 6CaO + P4O10 ¾¾ ® 2Ca 3 (PO4 )2
give oxides on decomposition. (d) The process of breaking the lump of lime by addition of
Bicarbonates : These metals form M(HCO 3 ) 2 type limited amount of water is called slaking of lime. Quick
bicarbonates. The thermal stability of bicarbonates increases lime slaked with soda gives solid sodalime.
from Be to Ba. Uses: It is used
(a) for manufacturing cement.
Different Behaviour of Beryllium as Compared to (b) in the manufacture of Na2CO3 from caustic soda.
Other Elements of its Group (c) in the purification of sugar and manufacture of dye stuffs.
(i) Ionisation potential and electronegativity of Be are higher (d) in the preparation of mortar, a building material.
than those of other metals. (ii) Calcium Hydroxide (Slaked lime), Ca(OH)2
(ii) BeCl2 is insoluble in water, due to its covalent nature, but It is prepared by adding water to quick lime.
soluble in organic solvents. Other chlorides (CaCl2, SrCl2 CaO + H2O ¾¾ ® Ca(OH)2
and BaCl2) get dissolved in water. Properties:
(iii) BeO and Be(OH)2 are amphoteric in nature. Therefore they (a) It is a white amorphous powder.
react with acids as well as bases. Other oxides react only with (b) It is sparingly soluble in water.
acids due to their alkaline nature. (c) Its aqueous solution is known as lime water and a
(iv) Beryllium forms single type of oxide (MO), Ca Sr and Ba form suspension of slaked lime in water is called milk of lime.
peroxides also. (d) Ca ( OH )2 + CO 2 ¾¾
® CaCO3 ¯ + H 2 O
(v) Beryllium does not give flame test, Ca, Sr and Ba impart milky
characteristic colours to the flame. ® Ca ( HCO3 ) 2
CaCO 3 + CO 2 + H 2 O ¾¾
(vi) Due to small size, Be forms complex compounds.
(vii) Hydrides and halides of Be get polymerized. (e) 2Ca ( OH ) 2 + 2Cl2 ¾¾
® CaCl 2 + Ca ( OCl) 2 + 2H 2O
Diagonal Relationship between Be and Al Bleaching
powder
(i) Be and Al both are hard due to strong metallic bonds.
Therefore, their melting and boiling points are high. Uses. Calcium hydroxide is used: (i) in the manufacture of
(ii) BeCl2 and AlCl3 both are covalent compounds. Therefore, bleaching powder and caustic soda, (ii) for white washing
they are insoluble in water and soluble in organic solvents. buildings and for softening of water, and (iii) in the preparation
of soda-lime (mixture of cal. hydroxide and caustic soda)
(iii) Melting points of BeCl2 and AlCl3 are low due to their covalent
(iii) Calcium Carbonate, CaCO3
tendency.
Preparation:
(iv) Be and Al both have tendency of forming complex compounds
due to small size. (a) Ca(OH) 2 + CO 2 ¾¾ ® CaCO3 + H 2O
(v) Both BeCl2 and AlCl3 are strong Lewis acids and are used as (b) CaCl 2 + Na 2 CO3 ¾¾® CaCO3 + 2NaCl
Friedel Crafts catalysts.
Properties
(vi) Be(OH)2.Al(OH)3, BeO and Al2O3 are amphoteric in nature.
(a) It is a white fluffy powder and is insoluble in water.
(vii) BeCl2 and AlCl3 from dimers, because both are electron
deficient compounds. 1200K
(b) Action of heat : CaCO3 ¾¾¾¾ ® CaO + CO2
EBD_7327
224 CHEMISTRY

(c) Action of acids : Properties:


CaCO3 + 2HCl ¾¾
®CaCl2 + H2O + CO2 With adequate quantity of water, it forms a plastic mass that
CaCO3 + H2SO4 ¾¾
® CaSO4 + H2 O + CO2 sets into a hard solid.
Uses: It is used Uses: It is used
(a) as a building material in form of marble (a) in building industry and in plasters.
(b) in manufacture of quick lime. (b) in dentistry, ornamental work and for making casts of
(c) as an antacid, mild abrasive in toothpaste, in chewing statues and busts.
gum and a filler in cosmetics. Biological Importance of Ca and Mg
(iv) Calcium Sulphate (Plaster of Paris), CaSO4.1/2H2O Magnesium is used as a cofactor by enzymes that utilise ATP in
Preparation: phosphate transfer. Chlorophyll, the main pigment for the
393K absorption of light in plants, contains Mg. 99% of body Ca is
2 ( CaSO4 .2H 2 O ) ¾¾¾® 2 ( CaSO 4 ) .H 2O + 3H 2O
present in bones and teeth. Ca plays role in neuromuscular
Gypsum
function, interneuronal transmission, cell membrane integrity and
Above 393 K, anhydrous CaSO4 is formed which is called blood coagulation.
‘dead burnt plaster’.
The-s-Block Elements 225

CONCEPT MAP
EBD_7327
226 CHEMISTRY

1. Which of the following represents a correct sequence of 13. Which of the following does not illustrate the anomalous
reducing power of the following elements? properties of lithium?
(a) Li > Cs > Rb (b) Rb > Cs > Li (a) The melting point and boiling point of Li are
(c) Cs > Li > Rb (d) Li > Rb > Cs comparatively high
2. Nitrolium is: (b) Li is much softer than the other group I metals
(a) CaC2 and graphite (b) CaCN2 and graphite (c) Li forms a nitride Li3N unlike group I metals
(c) Ca(CN)2 and graphite (d) CaCN2 + N2 (d) The ion of Li and its compounds are more heavily
3. The pair whose both species are used in ant- acid medicinal hydrated than those of the rest of the group
preparations is : 14. Which of the following statements is incorrect?
(a) NaHCO3 and Mg(OH)2 (a) Alkali metal hydroxide are hygroscopic
(b) Na2CO3 and Ca(HCO3)2 (b) Dissolution of Alkali metal hydroxide is endothermic
(c) Ca(HCO3)2 and Mg(OH)2 (c) Aqueous solution of alkali metal hydroxides are
(d) Ca(OH)2 and NaHCO3 strongly basic
4. Which one of the following properties of alkali metals (d) Alkali metal hydroxides form ionic crystals
increases in magnitude as the atomic number rises ? 15. Which property of sodium is being used in street lights ?
(a) Ionic radius (b) Melting point (a) It shows photoelectric effect
(c) Electronegativity (d) First ionization energy. (b) It has low melting point
5. Largest difference in radii is found in case of the pair (c) Sodium atom emits photons in the yellow region of
(a) Li, Na (b) Na, K visible spectrum, due to electrically stimulated transitions
(c) K, Rb (d) Rb, Cs. (d) Sodium vapours show golden colour
6. Which compound will show the highest lattice energy ? 16. Which is most basic in character ?
(a) RbF (b) CsF (a) CsOH (b) KOH
(c) NaF (d) KF (c) NaOH (d) LiOH
7. Strongest bond is in between 17. An inorganic compound which on heating first melts, then
(a) CsF (b) NaCl solidifies and liberates O2 gas, the inorganic compound is
(c) Both (a) and (b) (d) None of above
(a) Al2O3 (b) KMnO4
8. In crystals which one of the following ionic compounds
(c) MnO2 (d) KClO3
would you expect maximum distance between centres of
18. Which of the following has density greater than water?
cations and anions?
(a) Li (b) Na
(a) LiF (b) CsF
(c) K (d) Rb
(c) CsI (d) LiI
19. A metal salt solution forms a yellow precipitate with
9. Among the following components of cement which is present
potassium chromate in acetic acid, a white precipitate with
in highest amount?
(a) Ca2SiO4 (b) Ca3SiO5
dil H2SO4, but gives no precipitate with NaCl. The metal salt
(c) Al2O3 (d) Ca3Al2O6. solution will consist of
10. Which of the following is not correct ? (a) PbCO3 (b) BaCO3
heat (c) MgCO3 (d) CaCO3
(a) 2Li 2 O ¾¾ ¾® Li 2 O 2 + 2Li 20. When sulphur is heated with NaOH (aq)? The compounds
673k
heat formed are
(b) 2K 2O ¾¾¾® K 2 O2 + 2K
673k
heat (a) Na 2S + H 2O
(c) 2Na 2 O ¾¾ ¾® Na 2 O 2 + 2Na
673k
heat (b) Na 2SO 3 + H 2O
(d) 2Rb2 O ¾¾¾ 673k
® Rb2 O2 + 2Rb
11. When potassium dichromate crystals are heated with conc. (c) Na 2S + Na 2S2 O3 + H 2 O
HCl (d) Na 2S2 O3 + H 2O
(a) O2 is evolved
21. The first (IE1) and second (IE2) ionisation energies (kJ/mol)
(d) chromyl chloride vapours are evolved
of a few elements designated by Roman numerals are given
(c) Cl2 is evolved
below. Which of these would be an alkali metal?
(d) no reaction takes place
IE1 IE2
12. Which one of the alkaline earth metals shows some
anomalous behaviour and has same electronegativity as (a) I 2372 5251
aluminium? (b) II 520 7300
(a) Sr (b) Ca (c) III 900 1760
(c) Ba (d) Be (d) IV 16803 380
The-s-Block Elements 227

22. Washing soda has formula 36. Baking powder contains :


(a) Na2CO3.7H2O (b) Na2CO3.10H2O (a) NaHCO3, Ca (H2PO2)2 and starch
(c) Na2CO3.3H2O (d) Na2CO3 (b) NaHCO3, Ca(H2PO2)2
23. Which of the following is known as fusion mixture ? (c) NaHCO3, starch
(a) Mixture of Na2CO3 + NaHCO3 (d) NaHCO3
(b) Na2CO3.10H2O 37. Which of the following is used widely in the manufacture of
(c) Mixture of K2CO3 + Na2CO3 storage battery?
(d) NaHCO3 (a) Arsenic (b) Lithium
24. Aqueous solution of sodium carbonate absorbs NO and (c) Bismuth (d) Antimony
NO2 to give 38. Alkaline earth's metals are denser than alkali metals, because
(a) CO2 + NaNO3 (b) CO2 + NaNO2 metallic bonding in alkaline earth's metal, is :
(c) NaNO2 + CO (d) NaNO3 + CO (a) stronger (b) weaker
(c) volatile (d) not present
25. Mg on heating to redness in an atmosphere of N2 and then
39. The order of solubility of sulphates of alkaline earth metals
on treating with H2O gives:
in water is
(a) NH3 (b) N2
(a) Be > Mg > Ca > Sr > Ba
(c) PH3 (d) MgO
(b) Mg > Be >> Ba > Ca > Sr
26. Molecular formula of Glauber’s salt is : (c) Be > Ca > Mg > Ba >> Sr
(a) MgSO4.7H2O (b) CuSO4.5H2O (d) Mg > Ca > Ba >> Be > Sr
(c) FeSO4.7H2O (d) Na2SO4.10H2O 40. Arrange the following compounds in order of increasing
27. A mixture of KCl and KF is added to sodium chloride solubility
(a) to increase the conductivity of NaCl (i) MgF2 (ii) CaF2, (iii) BaF2
(b) to decrease the melting point of NaCl (a) (i) < (ii) < (iii) (b) (ii) < (i) < (iii)
(c) to supress the melting of dissociation of NaCl (c) (ii) < (iii) < (ii) (d) (iii) < (ii) < (i)
(d) to decrease the volatility of NaCl 41. Potassium is kept in:
28. What happens when carbonates of group IA elements are (a) alcohol (b) water
heated? (c) kerosene (d) liquid ammonia
(a) CO2 is given out 42. Which of the following atoms will have the smallest size ?
(b) Water vapours are given out (a) Mg (b) Na
(c) Carbon dioxide and water vapours are evolved (c) Be (d) Li
(d) None of these 43. The solubilities of carbonates decrease down the magnesium
29. A well known reagent which contains copper sulphate, group due to a decrease in
sodium potassium tartarate and sodium hydroxide is (a) hydration energies of cations
(a) Fenton’s reagent (b) Schiff’s reagent (b) inter-ionic attraction
(c) Fehling’s solution (d) Nessler’s reagent (c) entropy of solution formation
30. Baking soda is : (d) lattice energies of solids
(a) NaHCO3 (b) K2CO3 44. Which of the following relations is correct with respect to
(c) Na2CO3 (d) NaOH first (I) and second (II) ionization potentials of sodium and
31. On strong heating sodium bicarbonate changes into: magnesium?
(a) sodium monoxide (b) sodium hydroxide (a) I Mg = II Na (b) I Mg < II Na
(c) sodium carbonate (d) sodium peroxide
32. Which of the following compounds transforms baking soda (c) I Na > I Mg (d) II Na > IIMg
into baking powder? 45. Epsom salt is
(a) KCl (b) KHCO3 (a) CaSO4. 2H2O (b) BaSO4. 2H2O
(c) NaHCO3 (d) KHC4H4O6 (c) MgSO4. 2H2O (d) MgSO4. 7H2O
33. A certain metal M is used to prepare an antacid, which is 46. Which of the following has maximum ionization energy
used as a medicine in acidity. This metal accidently catches (a) Ba ¾ ¾® Ba + + e - (b) Be ¾ ¾® Be + + e -
fire which cannot be put out by using CO 2 based (c) Ca ¾¾ ® Ca 2+ + 2e- (d) Mg ¾ ¾® Mg 2 + + 2e -
extinguishers. The metal M is
(a) Ca (b) Mg 47. The value of x is maximum for
(c) C (d) All of these (a) MgSO4.x H2O
(b) CaSO4.x H2O
34. Which of the following is commercially known as oxone ?
(c) BaSO4.x H2O
(a) Na2O2 + HCl (b) Na2O + HCl
(d) All have the same value of x.
(c) Na2O2 + Na2 (d) none of these
48. Of the metals Be, Mg, Ca and Sr of group II A. In the periodic
35. Which pair cannot exist together in solution ?
table the least ionic chloride would be formed by
(a) NaHCO3 and NaOH (b) NaHCO3 and NaCl (a) Be (b) Mg
(c) NaHCO3 and Na2CO3 (d) NaCl and Na2CO3 (c) Ca (d) Sr
EBD_7327
228 CHEMISTRY

49. Which of the following is the component of most of the (a) Na+ (b) K+
kidney stones? (c) Ba 2+ (d) Ca2+
(a) (COO)2Ca (b) (COO)2Ba 63. Which one of the following is flourspar?
(c) (COONa)2 (d) (COO)2Mg (a) CaF2 (b) CaO
50. Which one is known as barytes? (c) H2F2 (d) CaCO3
(a) BaSO4 (b) BaCl2. 2H2O 64. Plaster of Paris is :
(c) BaO (d) BaCO3
51. Which one of the following salts does not impart colour to (a) CaSO 4 .2H 2 O (b) CaSO 4 .H 2 O
the flame ? 1
(a) Kl (b) LiCl (c) CaSO 4 . H 2 O (d) CaSO 4 .4H 2 O
2
(c) CaCl2 (d) MgCl2
65. Gypsum on heating at 120 – 130°C gives :
52. The Formula of Mohar’s Salt; FeSO4 (NH4)2 SO4×XH2O the
(a) anhydrous salt (b) hemihydrate
value of X :
(c) monohydrate (d) dihydrate
(a) 5 (b) 10
66. Plaster of Paris on making paste with little water sets to hard
(c) 6 (d) 8
mass due to formation of
53. Which of the following compounds is used in preparation
(a) CaSO4 (b) CaSO4.1/2H2O
of green fire ?
(a) K2SO4 (b) NaNO3 (c) CaSO4.H2O (d) CaSO4.2H2O
(c) Ba(NO3)2 (d) None of these 67. Which of the following compounds of cement sets at the
slowest rate ?
54. Amongst LiCl, RbCl, BeCl 2 and MgCl2 the compounds
(a) Dicalcium silicate
with the greatest and least ionic character respectively are : (b) Tricalcium silicate
(a) LiCl and RbCl (b) MgCl2 and BeCl 2 (c) Tricalcium aluminate
(d) Tetracalcium aluminoferrite.
(c) RbCl and BeCl 2 (d) RbCl and MgCl2
68. The chemical which is used for plastering the broken bones
55. Lithopone is is
(a) BaO + ZnSO4 (b) BaS + ZnSO4 (a) (CaSO4)2H2O (b) MgSO4.7H2O
(c) ZnS + BaSO4 (d) ZnO + BaSO4
(c) FeSO4. 7H2O (d) CuSO4. 5H2O
56. Mixture of MgCl2 and MgO is called :
(a) Portland cement (b) Sorel's cement 69. Colemanite is
(c) double salt (d) none of these (a) Ca[B3O4 (OH)2].2H2O (b) Ca2B6O11.5H2O
57. K2CS3 can be called potassium (c) Ca(OH)2 (d) Na2B4O7.2H2O
(a) Sulphocyanide (b) Thiocarbide 70. Dead burn plaster is
(c) Thiocarbonate (d) Thiocyanate (a) CaSO4.2H2O (b) MgSO4. 7H2O
58. Estimation of calcium and magnesium is done by (c) CaSO4.½ H2O (d) CaSO4
(a) EDTA (b) oxalate
71. Bone ash contains
(c) Phosphate (d) none of these
(a) CaO (b) CaSO4
59. Philosopher's wool on heating with BaO at 1100°C produces:
(a) Ba + ZnCl2 (b) BaCdO2 (c) Ca3 (PO4)2 (d) Ca(H2PO4)2
(c) BaZnO2 (d) BaO2+ Zn 72. The wire of flash bulb is made of
60. Which one of the following processes will produce hard (a) Mg (b) Cu
water? (c) Ba (d) Ag
(a) Saturation of water with MgCO3 73. Calcium cynamide is
(b) Saturation of water with CaSO4 (a) CaCHNH2 (b) CaCN2
(c) Addition of Na2SO4 to water (c) CaC2N2 (d) Ca(CN)2
(d) Saturation of water with CaCO3 74. Calcium is obtained by
61. In Solvay ammonia process, sodium bicarbonate is (a) roasting of lime stone
precipitate due to (b) reduction of CaCl2 with carbon
(a) presence of NH3 (c) electrolysis of a solution of CaCl2 in water
(b) reaction with CO2 (d) electrolysis of molten CaCl2
(c) reaction with brine solution 75. Mortar is a mixture of
(d) reaction with NaOH (a) CaCO3, sand and water
62. In India at the occasion of marriages, the fire works used (b) slaked lime and water
give green flame. Which one of the following radicals may (c) slaked lime, sand and water
be present? (d) CaCO3 and CaO
The-s-Block Elements 229

1. Lithium can not be stored in kerosene oil because 10. If NaOH is added to an aqueous solution of Zn2+ ions, a
(a) it is an alkali metal white precipitate appears and on adding excess NaOH, the
(b) it reacts with kerosene oil precipitate dissolves. In this solution zinc exists in the :
(c) it floats to the surface of kerosene oil (a) cationic part
(d) none of the above is correct (b) anionic part
2. Which one out of the NaOH and KOH, is a better absorber (c) both in cationic and anionic parts
of CO2 ? (d) there is no zinc left in the solution
(a) NaOH 11. Among LiI, NaI, KI, the one which is more ionic and more
(b) KOH soluble in water is :
(c) both absorb CO2 equally (a) KI (b) NaI
(d) can not be predicted (c) LiI (d) None of these
3. Which one of the following has minimum value of cation/ 12. A metal M readily forms its sulphate MSO4 which is water-
anion ratio ? soluble. It forms its oxide MO which becomes inert on heating.
(a) NaCl (b) KCl It forms an insoluble hyroxide M(OH)2 which is soluble in
(c) MgCl2 (d) CaF2 NaOH solution. Then M is
4. All of the following substances react with water. The pair (a) Mg (b) Ba
that gives the same gaseous product is (c) Ca (d) Be
(a) K and KO2 (b) Na and Na2O2 13. In curing cement plasters water is sprinkled from time to
(c) Ca and CaH2 (d) Ba and BaO2 time. This helps in
5. Which of the following is neither deliquescent nor (a) developing interlocking needle-like crystals of hydrated
efflorescent and is used for wool washing ? silicates
(a) NaOH (b) KOH (b) hydrating sand and gravel mixed with cement
(c) NaHCO3 (d) Na2CO3.NaCO3.2H2O (c) converting sand into silicic acid
6. Among the following oxides, which one is most basic (d) keeping it cool
(a) ZnO (b) MgO 14. On heating anhydrous Na2CO3,.......is evolved
(c) Al2O3 (d) N2O5 (a) CO2 (b) Water vapour
7. Which of the following statement is false ? (c) CO (d) No gas
(a) Strontium decomposes water readily than beryllium
15. The metals A and B form oxide but B also forms nitride when
(b) Barium carbonate melts at a higher temperature than both burn in air. The A and B are
calcium carbonate
(a) Cs, K (b) Mg, Ca
(c) Barium hydroxide is more soluble in water than
(c) Li, Na (d) K, Mg
magnesium hydroxide
(d) Beryllium hydroxide is more basic than barium 16. Based on lattice energy and other considerations which one
hydroxide. of the following alkali metal chlorides is expected to have
8. The general formula of an alum is the highest melting point
(a) LiCl (b) NaCl
M 2SO 4 .M 2¢ (SO 4 ) 3 .24H 2 O
(c) KCl (d) RbCl
where M is univalent and M¢ is a trivalent metal which of 17. An aqueous solution of salt ‘R’ when treated with dil HCl, a
the following does not form alum ? colourless gas is given out. The gas so evolved when passed
(a) Li (b) Na through acidified KMnO4 decolourises KMnO4 solution.
(c) K (d) all of these form alum The salt ‘R’ is
9. What is the function of potassium nitrate in gun powder ? (a) Na2CO3 (b) NaClO3
(a) It is a sublime substance (c) NaNO2 (d) Na2SO3
(b) It is added to act as instant explosive 18. Which of the following sulphates have the highest solubility
(c) It is added to provide oxygen in water?
(d) It reacts with sulphur to form another compound that (a) MgSO4 (b) BaSO4
is highly explosive in nature
(c) CaSO4 (d) BeSO4
EBD_7327
230 CHEMISTRY

19. On being placed in water, sodium peroxide not only produces 27. On being placed in water, sodium peroxide not only produces
an alkaline solution but also some bubbles. If we assume an alkaline solution but also some bubbles. If we assume
that the peroxide ion picks up two protons from water to that the peroxide ion picks up two protons from water to
produce a compound that can be seen as the dibasic produce a compound that can be seen as the dibasic
conjugate acid of peroxide ion and then this compound conjugate acid of peroxide ion and then this compound
undergoes a redox disproportion. undergoes a redox disproportion.
Using the above information complete the following Using the above information complete the following
equation. equation.
Na 2 O 2 (s) + H 2 O(l) ¾¾®( A) + ( B)
Na 2 O 2 (s) + H 2 O(l) ¾¾
®( A) + ( B)
(A) and (B) are
(A) and (B) are
(a) H2O2 and NaOH (b) H2O and O2
(c) NaOH and O2 (d) Na2O and NaOH (a) H2O2 and NaOH (b) H2O and O2
(c) NaOH and O2 (d) Na2O and NaOH
20. The metal X is prepared by the electrolysis of fused chloride.
28. BaSO4 is used in the X-ray investigation of intestinal track
It reacts with hydrogen to form a colourless solid from which
because it is opaque to X-rays. For this, one adds solid
hydrogen is released on treatment with water. The metal is :
BaSO4 in water to obtain a saturated solution with BaSO4 (s)
(a) Ca (b) Al (c) Zn (d) Cu
suspended in saturated solution.
21. The correct order of radii is –
BaSO4(solid) ¾¾ ® BaSO4(Soln.)
(a) Li < Be < Mg (b) H+ < Li+ < H–
3+
(c) Mn < Mn < Mn 2+ +7 (d) K+ > Cl– > S2– ¾¾ ® Ba+ ++ SO42–
However one patient is allergic to Ba2+ ions and one has to
22. Select the correct statements :
reduce the concentration of Ba2+ ions from the saturated
I. Cs+ is more highly hydrated that the other alkali metal
solution. The method adopted for this is :
ions
(a) Heat the system because ionization reaction
II. Among the alkali metals Li, Na, K and Rb, lithium has
BaSO4 = Ba+ + + SO42– is endothermic
the highest melting point
(b) Add more BaSO4 solid to the solution
III. Among the alkali metals only lithium forms a stable
(c) reduce the volume of saturated solution over
nitride by direct combination with nitrogen
the solid BaSO4
(a) I, II and III (b) I and II
(d) Add enough Na2SO4 to solution so that Ba2+ ion can
(c) I and III (d) II and III
be precipitate due to excess SO42–ions.
23. The melting point of lithium (181°C) is just double the melting 29. In which of the following the hydration energy is higher
point of sodium (98°C) because – than the lattice energy?
(a) down the group, the hydration energy decreases (a) MgSO4 (b) RaSO4
(b) down the group, the ionization energy decreases (c) SrSO4 (d) BaSO4
(c) down the group the cohesive energy decreases 30. The alkali metals form salt-like hydrides by the direct
(d) None of these synthesis at elevated temperature. The thermal stability of
+ these hydrides decreases in which of the following orders ?
24. Magnesium form Mg 2+ and Mg because :
(a) CsH > RbH > KH > NaH > LiH
(a) ionic radius of Mg(II) is smaller than of Mg(I). (b) KH > NaH > LiH > CsH > RbH
(b) hydration energy of divalent magnesium ion is higher. (c) NaH > LiH > KH > RbH > CsH
(c) magnesium (II) carbonate is insoluble in water. (d) LiH > NaH > KH > RbH > CsH
(d) generally higher oxidation states are preferred by metals. 31. Which of the following oxides is not expected to react with
25. A metal ‘M’ reacts with N2 to give a compound ‘A’ (M3N). sodium hydroxide?
‘A’ on heating at high temperature gives back ‘M’ and ‘A’; (a) CaO (b) SiO2
on reacting with H2O gives a gas B. ‘B’ turns CuSO4 solution (c) BeO (d) B2O3
blue on passing through it. M and B can be 32. Which of the following alkaline earth metal sulphates has
(a) Al and NH3 (b) Li and NH3 hydration enthalpy higher than the lattice enthalpy?
(c) Na and NH3 (d) Mg and NH3
(a) CaSO 4 (b) BeSO 4
26. Which is not correctly matched
(1) Basic strength Cs2O < Rb2O < K2O < Na2O < Li2O of (c) BaSO 4 (d) SrSO 4
oxides 33. The compound A on heating gives a colourless gas and a
(2) Stability of Na2O2 < K2O2 < Rb2O2 < Cs2O2 peroxides residue that is dissolved in water to obtain B. Excess of CO2
(3) Stability of LiHCO3 < NaHCO3 < KHCO3 bicarbonates is bubbled through aqueous solution of B, C is formed which
< RbHCO3 < CsHCO3 is recovered in the solid form. Solid C on gentle heating
(4) Melting point NaF < NaCl < NaBr < NaI gives back A. The compound is
(a) 1 and 4 (b) 1 and 3 (a) CaSO4.2H2O (b) CaCO3
(c) 1 and 2 (d) 2 and 3 (c) Na2CO3 (d) K2CO3
The-s-Block Elements 231

34. Which of the following compounds has the lowest melting 41. The products obtained on heating LiNO2 will be :
point ?
(a) Li 2 O + NO2 + O2 (b) Li 3 N + O 2
(a) CaCl2 (b) CaBr2
(c) CaI2 (d) CaF2 (c) Li 2 O + NO + O 2 (d) LiNO3 + O2
35. Which one of the following is present as an active ingredient in 42. What is the best description of the change that occurs when
bleaching powder for bleaching action ? Na2O(s) is dissolved in water ?
(a) CaOCl2 (b) Ca(OCl)2 (a) Oxide ion accepts sharing in a pair of electrons
(c) CaO2Cl (d) CaCl2 (b) Oxide ion donates a pair of electrons
36. Which of the following statements is incorrect? (c) Oxidation number of oxygen increases
(d) Oxidation number of sodium decreases
(a) Pure sodium metal dissolves in liquid ammonia to give
blue solution. 43. Which of the following on thermal decomposition yields a
basic as well as acidic oxide ?
(b) NaOH reacts with glass to give sodium silicate
(a) NaNO3 (b) KClO3
(c) Aluminium reacts with excess NaOH to give Al(OH)3
(c) CaCO3 (d) NH4NO3
(d) NaHCO3 on heating gives Na2CO3
44. The sequence of ionic mobility in aqueous solution is :
37. Match List – I with List –II for the compositions of
(a) K+ > Na+ > Rb+ > Cs+ (b) Cs+ > Rb+ > K+ > Na+
substances and select the correct answer using the code
given below the lists : (c) Rb+ > K+ > Cs+ > Na+ (d) Na+ > K+ > Rb+ > Cs+
List - I List - II 45. Property of the alkaline earth metals that increases with their
atomic number is
Substances Composition
(a) solubility of their hydroxides in water
(A) Plaster of paris (i) CaSO4.2H2O
(b) solubility of their sulphates in water
(B) Epsomite (ii) CaSO4.½ H2O
(c) ionization energy
(C) Kieserite (iii) MaSO4.7 H2O
(d) electronegativity
(D) Gypsum (iv) MgSO4. H2O
DIRECTIONS for Qs. 46 to 50 : These are Assertion-Reason
(v) CaSO4 type questions. Each of these question contains two statements:
Code : Statement-1 (Assertion) and Statement-2 (Reason). Answer these
(A) (B) (C) (D) questions from the following four options.
(a) (iii) (iv) (i) (ii) (a) Statement-1 is True, Statement-2 is True, Statement-2 is a
correct explanation for Statement -1
(b) (ii) (iii) (iv) (i)
(b) Statement-1 is True, Statement-2 is True ; Statement-2 is
(c) (i) (ii) (iii) (v) NOT a correct explanation for Statement-1
(d) (iv) (iii) (ii) (i) (c) Statement-1 is True, Statement-2 is False
38. Equimolar solutions of the following substances were (d) Statement-1 is False, Statement-2 is True
prepared separately. Which one of these will record the
46. Statement-1 : Potassium and caesium are used in
highest pH value ?
photoelectric cells.
(a) BaCl2 (b) AlCl3 Statement-2 : Potassium and caesium emit electrons on
(c) LiCl (d) BeCl2 exposure to light.
39. The ease of adsorption of the hydrated alkali metal ions on 47. Statement-1 : LiCl is predominantly a covalent compound.
an ion-exchange resins follows the order : Statement-2 : Electronegativity difference between Li and
(a) Li+ < K+ < Na+ < Rb+ Cl is too small.
(b) Rb+ < K+ <Na+ < Li+ 48. Statement-1 : Sulphate is estimated as BaSO4 and not as
(c) K+ < Na+ < Rb+ < Li+ MgSO4.
(d) Na+ < Li+ < K+ < Rb+ Statement-2 : Ionic radius of Mg2+ is smaller than that of
Ba2+.
40. KO2 (potassium super oxide) is used in oxygen cylinders in
49. Statement-1 : Mg continue to burn in nitric oxide
space and submarines because it
Statement-2 : During burning, heat evolved do not
(a) absorbs CO2 and increases O2 content
decompose NO.
(b) eliminates moisture 50. Statement-1 : Superoxides of alkali metals are paramagnetic.
(c) absorbs CO2 Statement-2 : Superoxides contain the ion O2– which has
(d) produces ozone. one unpaired electron.
EBD_7327
232 CHEMISTRY

Exemplar Questions (a) Be(OH)2 (b) Mg(OH)2


1. The alkali metals are low melting. Which of the following (c) Ca(OH)2 (d) Ba(OH)2
alkali metal is expected to melt if the room temperature rises 11. In the synthesis of sodium carbonate, the recovery of
to 30°C? ammonia is done by treating NH4Cl with Ca(OH)2. The
(a) Na (b) K by – product obtained in this process is
(c) Rb (d) Cs (a) CaCl2 (b) NaCl
2. Alkali metals react with water vigorously to form hydroxides (c) NaOH (d) NaHCO3
and dihydrogen. Which of the following alkali metals reacts 12. When sodium is dissolved in liquid ammonia, a solution of
with water least vigorously? deep blue colour is obtained. The colour of the solution is
(a) Li (b) Na due to
(c) K (d) Cs (a) ammoniated electron (b) sodium ion
3. The reducing power of a metal depends on various factors. (c) sodium amide (d) ammoniated sodium ion
Suggest the factor which makes Li, the strongest reducing 13. By adding gypsum to cement
agent in aqueous solution. (a) setting time of cement becomes less
(a) Sublimation enthalpy
(b) setting time of cement increases
(b) lonisation enthalpy
(c) colour of cement becomes light
(c) Hydration enthalpy
(d) shining surface is obtained
(d) Electron–gain enthalpy
4. Metal carbonates decompose on heating to give metal oxide 14. Dead burnt plaster is
1
and carbon dioxide. Which of the metal carbonates is most (a) CaSO4 (b) CaSO4 . H 2 O
2
stable thermally? (c) CaSO4.H2O (d) CaSO4.2H2O
(a) MgCO3 (b) CaCO3
15. Suspension of slaked lime in water is known as
(c) SrCO3 (d) BaCO3
(a) lime water
5. Which of the following carbonates given below is unstable
in air and is kept in CO2 atmosphere to avoid decomposition? (b) quick lime
(a) BeCO3 (b) MgCO3 (c) milk of lime
(c) CaCO3 (d) BaCO3 (d) aqueous solution of slaked lime
6. Metals form basic hydroxides. Which of the following metal 16. Which of the following elements does not form hydride
hydroxide is the least basic? by direct heating with dihydrogen?
(a) Mg(OH)2 (b) Ca(OH)2 (a) Be (b) Mg
(c) Sr(OH)2 (d) Ba(OH)2 (c) Sr (d) Ba
7. Some of the Group 2 metal halides are covalent and soluble 17. The formula of soda ash is
in organic solvents. Among the following metal halides, the (a) Na2CO3.10H2O (b) Na2CO3.2H2O
one which is soluble in ethanol is (c) Na2CO3.H2O (d) Na2CO3
(a) BeCl2 (b) MgCl2 18. A substance which gives brick red flame and breaks down
(c) CaCl2 (d) SrCl2 on heating to give oxygen and a brown gas is
8. The order of decreasing ionisation enthalpy in alkali metals (a) magnesium nitrate (b) calcium nitrate
is (c) barium nitrate (d) strontium nitrate
(a) Na > Li > K > Rb (b) Rb < Na < K < Li
19. Which of the following statements is true about Ca(OH)2?
(c) Li > Na > K > Rb (d) K < Li < Na < Rb
(a) It is used in the preparation of bleaching powder.
9. The solubility of metal halides depends on their nature, lattice
enthalpy and hydration enthalpy of the individual ions. (b) It is a light blue solid.
Amongst fluorides of alkali metals, the lowest solubility of (c) It does not possess disinfectant property.
Li in water is due to (d) It is used in the manufacture of cement.
(a) ionic nature of lithium fluoride 20. A chemical ‘A’ is used for the preparation of washing soda
(b) high lattice enthalpy to recover ammonia.When CO2 is bubbled through an
(c) high hydration enthalpy for lithium ion aqueous solution of ‘A’, the solution turns milky. It is used
(d) low ionisation enthapy of lithium atom in white washing due to disinfectant nature. What is the
10. Amphoteric hydroxides react with both alkalies and acids. chemical formula of ‘A’?
Which of the following Group 2 metal hydroxides is soluble (a) Ca(HCO3)2 (b) CaO
in sodium hydroxide? (c) Ca(OH)2 (d) CaCO3
The-s-Block Elements 233

21. Dehydration of hydrates of halides of calcium, barium and 23. Solubility of the alkaline earth's metal sulphates in water
strontium i.e., CaCl2. 6H2O, BaCl2.2H2O, SrCl2.2H2O can be decreases in the sequence :- [2015]
achieved by heating. These become wet on keeping in air. (a) Ca > Sr > Ba > Mg (b) Sr > Ca > Mg > Ba
Which of the following statements is correct about these (c) Ba > Mg > Sr > Ca (d) Mg > Ca > Sr > Ba
halides? 24. The function of "Sodium pump" is a biological process
(a) Act as dehydrating agent operating in each and every cell of all animals. Which of the
(b) Can absorb moisture from air following biologically important ions is also a consituent of
(c) Tendency to form hydrate decreases form calcium to this pump : [2015]
barium (a) Mg2+ (b) K+
(d) All of the above (c) Fe2+ (d) Ca2+
25. On heating which of the following releases CO2 most
NEET/AIPMT (2013-2017) Questions easily ? [2015 RS]
22. In Castner-Kellner cell for production of sodium hydroxide: (a) K2CO3 (b) Na2CO3
[NEET Kar. 2013] (c) MgCO3 (d) CaCO3
(a) Brine is electrolyzed with Pt electrodes 26. Which of the following statements is false ? [2016]
(b) Brine is electrolyzed using graphite electrodes (a) Mg2+ ions form a complex with ATP
(c) Molten sodium chloride is electrolysed (b) Ca2+ ions are important in blood clotting
(d) Sodium amalgam is formed at mercury cathode (c) Ca2+ ions are not important in maintaining the regular
beating of the heart.
(d) Mg2+ ions are important in the green parts of plants.
EBD_7327
234 CHEMISTRY

Hints & Solutions


EXERCISE - 1 14. (b) During the dissolution of alkali metal hydrides energy
is released in large amount, i.e., it is exothermic in nature.
1. (a) A reducing agent is a substance which can loose 15. (c) The property of sodium atom to emit photons in yellow
electron and hence a reducing agent should have low region of visible spectrum, due to electrically stimulated
ionisation energy. Now since ionisation energy electron transitions is used in street lights.
decreases from Li to Cs, the reducing property should 16. (a) Since the ionization energies of alkali metals decrease
increase from Li to Cs. The only exception to this is down the group, the ionic character and consequently
lithium. This is because the net process of converting basic property of their hydroxides increases in the same
an atom to an ion takes place in 3 steps. order, i.e. from LiOH to CsOH.
(i) M(s) ® M(g) DH = Sublimation energy
heat
(ii) M(g) ® M+(g) + e– DH = Ionisation energy 17. (d) 2KClO 3 ¾¾¾® 2 KCl + 3O 2
melts
(iii) M+(g)+H2O ® M+ (aq) DH = Hydration energy solid solid
The large amount of energy liberated in hydration of Li 18. (d) Li, Na, K are lighter than water but Rb is heavier than
(because of its small size) makes the overall DH negative. water.
This accounts for the higher oxidation potential of 19. (b) BaCO3 forms a yellow ppt of barium chromate. BaCO3
lithium i.e., its high reducing power. forms a white precipitate of BaSO4. BaCl2 is soluble in
2. (b) Nitrolim is CaCN2 + C. It is used as fertilizer since it water.
reacts with H2O to form NH3. 20. (c) 4S + 6 NaOH ® Na 2S 2O 3 + 2 Na 2S + 3H 2 O
CaCN 2 + 3H 2 O ¾¾ ® CaCO 3 + 2NH 3 . 21. (b) As outermost electronic configuration of alkali metals is
3. (a) NaHCO3 is used in medicine to neutralise the acidicty ns1 and also their size are largest in their respective
in the stomach. Mg(OH)2 is basic in nature and periods so their 1st I.E will be lowest among the given
dissolves in acids forming corresponding salts. So both options. As second electron is to be removed from
are used in antacid medicinal preparations. complete shell or noble gas core, so the IInd I.E. must be
4. (a) Within a group, ionic radius increases with increase in highest among the given options. So, option (b) is correct
atomic number. The melting points decrease down the choice.
group due to weakening of metallic bond. The 22. (b) Washing soda is Na2 CO3. 10 H2O.
electronegativity and the 1st ionization energy also 23. (c) Mixture of K2CO3 and Na2 CO3 is called fusion mixture
decreases down the group. 24. (b) Na 2CO3 ∗ NO ∗ NO 2 ↑ 2 NaNO 2 ∗ CO 2
5. (b) Radii of atoms increase as we go down the group due 25. (a) Magnesium on heating to redness in an atmosphere of
to the addition of extra energy shell. Nuclear charge N2 gives magnesium nitride.
also increases on moving down the group but the 3Mg + N2 ¾¾® D
Mg3N2
influence of addition of energy shell predominates. This
predomination is larger in case when we move from Na Then magnesium nitride is rapidly hydrolysed by water
to K among all the options given. to give magnesium hydroxide and ammonia.
6. (c) With the same anion, smaller the size of the cation, Mg3N2 + 6H2O ¾ ¾® 3Mg(OH)2 + 2NH3
higher is the lattice energy. The correct order of size of 26. (d) Glauber’s salt is Na2SO4.10 H2O.
cations is – 27. (b)
Na+ < K+ < Rb+ < Cs+ 28. (d) Carbonates of group IA are fairly stable and will melt
Hence, the lattice energy of NaF will be maximum. before they eventually decompose into oxides at
i.e., NaF. temperature above 1000°C. Li2CO3 is considerably less
7. (a) According to Fajan rules, ionic character increases with stable than other carbonates of group 1A and on
increase in size of the cation and decrease in size of the decomposition by heating, it gives CO2.
anion. Thus, CsF has higher ionic character than NaCl Li2CO3 ¾ ¾® Li2O + CO2
and hence bond in CsF is stronger than in NaCl. So, option (d) is more probable.
8. (c) As Cs+ ion has larger size than Li+ and I– has larger size 29. (c) Fehling’s solution is a mixture of Alk. CuSO4 + Na – K
than F–, therefore maximum distance between centres tartarate (Rochelle salt)
of cations and anions is in CsI. 30. (a) NaHCO3 (baking soda) is one of the major constituents
9. (b) Composition of cement depends upon its type. Portland of baking powder.
cement is most common kind and contains 51% 31. (c) Sodium bicarbonate on heating changes into sodium
tricalcium silicate, 24% of dicalcium silicate and 6% of carbonate by the loss of CO2 and H2O.
tricalcium aluminate.
10. (a) Lithium does not form peroxide. 2NaHCO3 D Na 2CO3 + H 2O + CO 2
Sodium ¾¾® Sodium
11. (c) bicarbonate carbonate
12. (d) Be shows diagonal relationship with Al 32. (d)
13. (b) Li is much softer than the other group I metals. Actually 33. (b) Magnesium hydroxide is used to prepare an antacid
Li is harder then other alkali metals which is used as medicine. Hence, the metal M is Mg.
The-s-Block Elements 235

34. (a) (Na2O2 + HCl) is commercially known as oxone and is 55. (c) ZnS + BaSO4 is lithopone. It is used as white pigment.
used for bleaching of delicate fibres. 56. (b) Mixture of MgO and MgCl2 is called Sorel’s cement. It
35. (a) NaHCO3 and NaOH cannot exist together. NaHCO3 is used for making magnesia cement.
being acid salt will react with alkali to from normal salt. 57. (c) In K2CO3 replacement of O by S gives K2CS3 which is
36. (a) Baking powder has starch, NaHCO3 and Ca(H2PO2)2. called potassium thiocarbonate.
37. (d) Lithium is widely used in the manufacture of storage 58. (a) Calcium and magnesium are estimated by EDTA.
battery. 59. (c) Philosopher’s wool is ZnO
38. (a) Metallic bonding in alkaline earth’s metals is stronger 1100°C
because of larger number of electrons in the outer shell ZnO + BaO ¾¾¾¾ ® BaZnO2
in alkaline earth’s metal atoms. 60. (b) Permanent hardness of water is due to chlorides and
39. (a) sulphates of calcium and magnesium i.e CaCl2, CaSO4,
40. (b) BaF2 > MgF2 > CaF2 MgCl2 and MgSO4.
41. (c) Potassium is kept in kerosene as it is highly reactive 61. (c) NH 4 HCO3 + NaCl ¾
¾® NaHCO3 ¯ + NH 4 Cl
and reacts with all other options given above namely Brine Sod. bicarbonat e ppt .
alcohol forming alkoxide, water forming hydroxide and 62. (c) 63. (a)
ammonia forming a complex but does not react with 64. (c) Chemically plaster of Paris is CaSO4.1/2H2O.
kerosene.
42. (c) Within a period, the size decreases from left to right, i.e., 120°C 1
65. (b) CaSO 4 × 2H 2 O ¾¾¾¾ ® CaSO4 × H 2O
Na > Mg > Li > Be. Atomic size increases down the 2
group. Plaster of Paris is hemihydrate.
43. (a) As we move down the group, the lattice energies of 1
carbonates remain approximately the same. However the 66. (d) Plaster of Paris (CaSO4 . H O) on making paste with
hydration energies of the metal cation decreases from 2 2
Be++ to Ba++, hence the solubilities of carbonates of little water sets to a hard mass due to formation of
the alkaline earth metal decrease down the group mainly gypsum (CaSO4.2H2O).
due to decreasing hydration energies of the cations from 1 3
Be++ to Ba++. CaSO4 . H2O + H2O ® CaSO4.2H2O + Heat
2 2
44. (d) The IInd ionisation potential of Na is higher than Mg Plaster of Paris Gypsum
because it requires more energy to remove an electron 67. (a) Dicalcium silicate 2CaO.SiO2 contains more SiO2 as
from a saturated shell or stable (fully filled) orbital. compared to tricalcium silicate 3CaO.SiO2. Therefore, it
I
11 Na ¾® 1s 2 , 2s 2 2p 6 , 3s1 ¾
¾ ¾® has slow setting rate.
68. (a) (CaSO4)2.H2O – Plaster of paris is used for plastering
II
1s 2 , 2s 2 2p 6 , 3s 0 ¾¾® 1s 2 , 2s 2 2p5 , 3s 0 the broken bones.
I 69. (b) Colemanite is a mineral of boron having composition
12 Mg ¾® 1s 2 , 2s 2 2p 6 , 3s 2 ¾
¾ ¾® as Ca2B6O11.5H2O
II 70. (d) 71. (c) 72. (a) Mg. 73. (b)
1s 2 , 2s 2 2p 6 , 3s1 ¾¾® 1s 2 , 2s 2 2p 6 , 3s 0
Here Na-I < Mg-I and Na-II > Mg-II. 74. (d) CaCl 2 ® Ca +2 + 2Cl -
Cathode Anode
Option (d) is correct.
45. (d) 46. (d) Cathode : Ca+2 + 2e– ® Ca
47. (a) Because of smaller size, Mg2+ ions are extensively Anode : 2Cl– ® 2e– + Cl2
hydrated. 75. (c)
48. (a) Because of small atomic size and high I.E. Be forms EXERCISE - 2
covalent chloride.
49. (a) 50. (a) 1. (c) Because of its very low density lithium floats to the
51. (d) Halides of group I and II impart characteristic colour to surface of kerosene oil.
the flame due to low IE of the central atom. However, 2. (b) KOH is better absorber of CO2 because K2CO3 formed
ionization energy of Be and Mg atoms is high due to is more soluble and does not separate out.
their small size, hence they can't be excited to higher 3. (c) Atomic size of K+ > Ca2+ > Mg2+ and that of Cl– > F–.
levels by Bunsen burner flame. Thus, their halides don't Therefore, Mg2+/Cl– ratio has the minimum value.
impart colour to flame. 4. (c) Ca and CaH2 both react with H2O to form H2 gas,
52. (c) The Mohr’s salt is represented by the molecular formula
Ca + 2H 2 O ¾
¾® Ca (OH ) 2 + H 2
FeSO 4 ( NH 4 ) 2 SO 4 .6H 2O .
Hence, it has six molecules of water. CaH 2 + 2H 2 O ¾
¾® Ca ( OH ) 2 + 2 H 2
53. (c) Ba(NO3)2 is used in preparation of green fire. whereas K gives H2 while KO2 gives O2 and H2O2
54. (c) According to Fajan's rule, 2K + 2 H2O ¾ ¾® 2KOH + H2
Size of cation µ Ionic character.. 2KO2 + 2H 2O ¾¾
® 2KOH + O2 + H 2O2
Recall that size of metal (ion) increases while going
Similarly, Na gives H2 while Na2 O2 gives H2O2
down the group, and decreases on crossing the periods
from left to right. Thus Rb+ (group I) is largest and 2 Na + 2 H 2 O ¾
¾® 2 NaOH + H 2
Be2+ (group II) is smallest in size. Hence RbCl has Na 2 O 2 + 2H 2 O ¾
¾® 2 NaOH + H 2 O 2
greatest, and BeCl2 has lowest ionic character.
EBD_7327
236 CHEMISTRY

Likewise Ba gives H2 while BaO2 gives H2O2 1


Na 2 O2 ( s) + H2 O(l) ¾¾
® 2NaOH(aq) + O2 ( g )
Ba + 2 H2O ¾ ¾® Ba (OH)2 + H2 2
BaO2 + 2H2O ¾ ¾® Ba (OH)2 + H2 O2
5. (d) Sodium sesquicarbonate (Na2CO3.NaCO3.2H2O) is 20. (a) Ca + H 2 ¾ ¾® CaH 2 + 2H 2 O ¾ ¾® Ca (OH) 2 + H 2
neither deliquescent nor efflorescent and is used for 21. (b) H cation is smaller than parent H-atom. H– and Li+ are
+

wool washing. isoelectronic species so Li+ with higher nuclear charge


6. (b) MgO. N2O5 is strongly acidic, ZnO and Al2O3 are (i.e. 3) will be smaller than H– having smaller nuclear
amphoteric, therefore, MgO is most basic. charge (i.e. 1).
7. (d) Be(OH)2 is amphoteric, but the hydroxides of other 22. (d) Amongst alkali metal Li ions are highly hydrated.
alkaline earth metals are basic. The basic strength 23. (c) The atom becomes larger on descending the group, so
increases gradually. the bonds becomes weaker (metallic bond), the cohe-
8. (a) Li does not form alum because of its small size. sive force/energy decreases and accordingly melting
9. (c) KNO3 is included in gun powder to provide a supply point also decreases.
of oxygen. 24. (b) Statement (b) is correct.
25. (b) From the given data ‘M’ is a metal from group 1 as it
10. (b) Zn 2∗ ∗ 2NaOH ¾¾ ↑ 2Na ∗ ∗ Zn(OH) 2 forms M3N. Since on heating to high temperature, M3N
Zn(OH) 2 ∗ 2NaOH ¾¾ ↑ Na 2 ZnO 2 ∗ 2H 2 O gives back the metal ‘M’ the metal is most likely to be
Thus Na2ZnO2 forms 2Na+ and [ZnO2]2– ions. Li.
11. (a) Larger cation (K+) develops less polarisation in anion Writing various reactions
and thus KI has more ionic nature and more soluble in 6Li + N2 ® 2Li3N ¾Heat ¾¾® 6 Li
water. ‘M’ ‘A’
12. (d) Sulphate of alkaline earth metal are sparingly soluble Li3N + 3H2O ® 3LiOH + NH3(g)
or almost not soluble in water whereas BeSO4 is ‘B’
soluble in water due to high degree of solvation. CuSO4 + 4NH3(g) ® [Cu(NH3)4]SO4
Be(OH)2 is insoluble in water but soluble in NaOH. Blue solution
13. (a) Setting of cement is exothermic process which develops Thus M is Li and B is NH3(g).
interlocking crystals of hydrated silicates 26. (a) Basic strength of the oxides increase in the order
14. (d) Anhydrous form of Na2CO3 does not decompose on Li2O < Na2O < K2O < Rb2O < Cs2O. The increase in
heating even to redness. It is a amorphous powder basic strength is due to the decrease in I.E. and increase
called soda ash. in electropositive character.
15. (d) K and Mg, both form oxides The melting points of the halides decrease in the order
NaF > NaCl > NaBr > NaI, as the size of the halide ion
K + O 2 ® KO 2 ;2Mg + O 2 ® 2MgO increases. The decrease in melting point is due to
Mg form nitride also 3Mg + N 2 ® Mg 3 N 2 increase in the covalent character with increase in the
K does not form nitride. size of anion according to Fajan's rule.
27. (c) From the given information, we can see that the reaction
16. (b) Although lattice energy of LiCl is higher than NaCl but
proceeds via formation of H2O2 (which is diabasic
LiCl is covalent in nature and NaCl ionic there after, the conjugate acid of peroxide ion), H 2 O 2 then
melting point decreases as we move towards NaCl disproportionates into water and oxygen.
because the lattice energy decreases as a size of alkali
metal atom increases (lattice energy a melting point of Na 2 O 2 (s) + H 2 O(l) ¾¾
® 2NaOH(aq) + H 2O 2 (aq)
alkali metal halide) 1
17. (d) Since ‘R’ gives a colourless gas on reaction with dil H 2O2 (aq) ¾¾ ® H 2O ( l) + O 2 (g)
HCl, so it contains CO32– or SO32– as anion (i.e., CO2 2
or SO2 is evolved) Thus over all reaction is
Since the gas decolourises acidified KMnO4 solution 1
so it is SO2 and thus the anion present is SO32– i.e., the Na 2O2 (s) + H 2O(l) ¾¾ ® 2NaOH(aq) + O 2 (g)
2
salt ‘R’ is Na2SO3. 28. (d) To reduce the concentration of Ba +2 ions, add enough
18. (d) BeSO4 is most soluble because hydration energy is
more than lattice energy. Na2SO4 to solution. As the concentration of SO -4 -
BeSO > MgSO > CaSO > SrSO > BaSO
¾¾ ¾ ¾4¾ ¾ ¾4¾ ¾ ¾
4 ¾ ¾4¾ ¾ ¾4 ¾¾
® increases, Ba +2 combine with SO -4 - and are
Hydration energy decreases hence,solubility decreases.
19. (c) From the given information, we can see that the reaction precipitated.
proceeds via formation of H2O2 (which is diabasic 29. (a) In alkaline earth metals ionic size increases down the
conjugate acid of peroxide ion), H 2 O 2 then group. The lattice energy remains constant because
disproportionates into water and oxygen. sulphate ion is so large, so that small change in cationic
Na 2 O 2 ( s ) + H 2 O( l ) ¾¾
® 2NaOH( aq ) + H 2 O 2 ( aq ) sizes do not make any difference. On moving down the
1 group the degree of hydration of metal ions decreases
H 2 O 2 (aq ) ¾¾ ® H 2 O ( l) + O 2 ( g ) very much leading to decrease in solubility
2
Thus over all reaction is \ BeSO4 > MgSO 4 > CaSO 4 > SrSO 4 > BaSO4
The-s-Block Elements 237

30. (d) The stability of alkali metal hydrides decreases from Li 44. (b) Smaller the ion more is its ionic mobility in aqueous
to Cs. It is due to the fact that M–H bonds become solution. Ionic radii of the given alkali metals is in the
weaker with increase in size of alkali metals as we move order Na+ < K+ < Rb+ < Cs+ and thus expected ionic
down the group from Li to Cs. Thus the order of stability mobility will be in the order Cs+ < Rb+ < K+ < Na+.
of hydrides is However due to high degree of solvation (or hydration)
LiH > NaH > KH > RbH > CsH because of lower size or high charge density, the
i.e. option (d) is correct answer. hydrated ion size follows the order Cs+ < Rb+ < K+ <
31. (a) NaOH is a strong alkali. It combines with acidic and Na+ and thus conductivity order is Cs+ > Rb+ > K+ >
amphoteric oxides to form salts. Since CaO is a basic Na+ i.e. option (b) is correct answer.
oxide hence does not reacts with NaOH. 45. (a) Lattice energy decreases more rapidly than hydration
energy for alkaline earth metal hydroxides. On moving
32. (b) Be 2+ is very small, hence its hydration enthalpy is
down a group\ solubility of their hydroxides increases.
greater than its lattice enthalpy
Sol. For Qs. 46-50
D
33. (b) CaCO3( s ) ¾¾® CO 2( g ) ­ + CaO ( s ) The thermal stabilities of carbonates increase down the group
A colourless B due to increase in metallic character i.e. electropositive character.
Further bigger cation stabilises bigger anion through crystal lattice
CaO(s) + H 2 O ¾¾ ® Ca (OH )2(aq) energy effect.
B Solution of alkali in liquid ammonia has high electrical conductivity
Ca (OH )2 + 2CO2 ¾¾ ® Ca (HCO3 )2(aq) due to the presence of ammoniated electrons.
C M + (x + y) NH3 ––––® [M (NH3)x]+ + [e (NH3)y]–
Ca (HCO3 )2(s) ¾¾ ® CaCO3(s) + CO2(g) + H 2 O(g) Ammoniated electrons
A Sodium carbonate is prepared by solvay process not potassium
\ Correct choice : (b) carbonate, as KHCO3 is quite soluble in water.
34. (c) Melting points of halides decreases as the size of the 46. (a) K and Cs emit electrons on exposure of light due to low
halogen increases. The correct order is ionisation potential.
CaF2 > CaCl2 > CaBr2 > CaI2 47. (c) Statement-1 is true but statement-2 is false.
35. (b) Active ingredient in bleaching powder for bleaching Due to high polarizing power of Li +, LiCl is a covalent
action is Ca (OCl)2 compound.
36. (c) 2Al(s) + 2NaOH (aq) + 2H2O (l) ¾¾ ® 2NaAlO2 + 3H2 48. (b) Sulphate is estimated as BaSO4 because Ba2+ being
sod. meta aluminate larger ion is hydrated to a lesser extent hence it has
1 high lattice energy group.
37. (b) (A) Plaster of paris = CaSO4. H 2 O
2 49. (c) When Mg is burnt in nitric oxide, it continue to burn
(B) Epsomite = MgSO4.7H2O because during burning the heat evolved decompose
(C) Kieserite = MgSO4.H2O NO to N2 and O2. Oxygen thus, produced helps Mg to
(D) Gypsum = CaSO4.2H2O burn.
38. (a) (AlCl 3, LiCl & BeCl2) ) all these solutions are acidic due 50. (a) Presence of unpaired electrons in superoxides of alkali
to cationic hydrolysis, whereas BaCl2, is salt of strong metals make them paramagnetic.
base and strong acid, hence its solution will almost neutral
i.e., pH » 7. EXERCISE - 3
39. (b) All alkali metal salts are ionic (except Lithium) and soluble Exemplar Questions
in water due to the fact that cations get hydrated by
water molecules. The degree of hydration depends upon 1. (d) Alkali metals have low melting and boiling points. The
the size of the cation. Smaller the size of a cation, greater melting point of alkali metals decreases from Li to Cs
is its hydration energy. as cohesive force decreases with increase in atomic
Relative ionic radii : Cs+ > Rb+ > K + > Na + > Li + size.
Relative degree of hydration: 2. (a) The reactivity of alkali metals with water increases on
moving down the group from Li to Cs due to increase
Li + > Na + > K + > Rb + > Cs +
in electropositive character.
40. (a) 4KO2 + 2 CO2 ® 2 K2CO3 + 3O2 .
3. (c) Reducing character is measured by tendency of an
KO2 is used as an oxidising agent. It is used as air purifier
element to lose electron in aqueous solution. Higher
in space capsules. Submarines and breathing masks as
it produces oxygen and remove carbon dioxide. the negative E°RP value, greater is the ability to lose
41. (a) 4LiNO3 ® 2Li 2 O + 4NO 2 + O 2 electrons.
42. (b) E°RP depends on:
43. (c) Calcium carbonate on thermal decomposition gives CaO (i) enthalpy of sublimation
(Basic oxide) and CO2 (Acidic oxide) (ii) ionisation of enthalpy
D (iii) enthalpy of hydration
CaCO 3 CaO + CO 2­ Thus, in aqueous medium, order of reactivity of alkali
Basic oxide Acidic oxide metals is Na < K < Rb < Cs < Li. E°RP value of Li is least
(–3.04 V) among all alkali metals.
EBD_7327
238 CHEMISTRY

The formation of Li+ (aq) from Li involves following


NaCl + NH 4 HCO3 ¾¾
® NaHCO3 ¯ + NH 4Cl
steps: Sodium bicarbonate
Sublimation D
(i) Li(s) ¾¾¾¾¾® Li(g); 2NaHCO3 ¾¾
® Na 2 CO3 + H 2O + CO 2
DHs = Enthalpy of sublimation Sodium carbonate
NH3 is recovered from NH4 HCO3 and NH4Cl
® Li + (s); IE1 = Ionisation enthalpy
(ii) Li(g) ¾¾
D
NH 4 HCO3 ¾¾® NH 3 + H 2 O + CO 2
+ +
(iii) Li (g) ¾¾ ® Li (aq); DHn = Enthalpy of hydration 2NH 4 Cl + Ca ( OH ) 2 ® 2NH3 + CaCl 2 + 2H 2O
For alkali metals, enthalpies of sublimation are almost Ammonium chloride Ammonia Calcium chloride
same. IE1 value of Li is positive & highest and 12. (a) Alkali metals dissolve in liquid NH3 giving deep blue
hydration enthalpy is negative and maximum for coloured solution.
smallest Li+ which makes it strongest reducing agent. +
4. (d) BaCO3 is thermally most stable due to the small size of ® éë Na ( NH3 ) x ùû +
Na + ( x + y) NH3 ¾¾ e- ( NH3 ) y
resulting oxide ion. As the atomic number increases Ammoniated cation Ammoniated electron
the stability of the metal ion decreases hence, stability When light fall on the solution, the ammoniated
of carbonates increases. electrons get excite to higher energy level by absorbing
5. (a) BeCO3 is unstable due to high polarising power of certain wavelength and transmit blue colour.
smallest Be2+ ion. Also, it shows reversible reaction, 13. (b) Gypsum (CaSO4.2H2O) is added to cement to increase
because of more stability of oxide formed. Hence, its setting time so that it gets sufficiently hardened.
BeCO3 has least stability and it is stable only in CO2 Setting of cement is an exothermic process and involves
atmosphere. hydration of calcium aluminates and silicates.
6. (a) The basic character of the given hydroxides increases 14. (a) On heating plaster of Paris at certain temperature, it
down the group due to decrease in ionization enthalpies. forms anhydrous calcium sulphate which is known to
On moving down the group, the ionic size increases, be dead burnt plaster and has no setting property as it
so M–O bond in metal hydroxides gets weaker and absorbs water very slowly.
easily gives OH– in the solution and I.E. further 1 200°C 1100° C
CaSO 4 . H 2O ¾¾¾¾ ® CaSO4 ¾¾¾¾ ® CaO + SO3
decreases. Hence, Mg(OH)2 will be least basic. The 2 ( Anhydrous)
order of basicity will be:
15. (c) Slaked lime [Ca(OH)2] is sparingly soluble in water and
Mg(OH)2 < Ca(OH)2 < Sr(OH)2 < Ba(OH)2
it forms a suspension of slaked lime in water which is
7. (a) Ethanol is a covalent compound and only a compound
called milk of lime and a clear solution obtained after
which has more covalent character can be dissolved in
the suspension settles is known as lime water.
it. Also the group ionic character increases and
16. (a) Except Be, all alkaline earth metals form hydrides (MH2)
beryllium halide have covalent character due to its small
on directly heating with H2. BeH2 can't be prepared by
size and high effective nuclear charge. Hence, among
direct heating but it can be prepared by the action of
the given halides, BeCl2 can be soluble in ethanol.
LiAlH4 on BeCl2.
8. (c) On moving down the group from Li to Rb increased
screening effect makes the removal of electron easier. 2BeCl2 + LiAlH 4 ® 2BeH 2 + LiCl + AlCl3
Thus, the order of decreasing ionization enthalpy will 17. (d) On heating washing soda, it loses its water of
be: Li > Na > K > Rb crystallisation. Above 373 K, it becomes completely
9. (b) Among fluorides, the order of solubility is LiF < NaF anhydrous white powder called soda ash.
< KF < RbF < CsF. On moving down the group solubility Above 373K
Na 2CO3.10H 2O ¾¾¾¾¾¾
® Na 2CO3
increases because lattice energy decreases. Washing soda Soda ash
Also, LiF exhibit very high lattice energy. ( Anhydrous form)
10. (a) Be(OH)2 is an amphoteric hydroxide thus gets dissolve 18. (b) Calcium gives brick red coloured flame. Also, calcium
both in acids and alkalies. nitrate on heating decomposes into calcium oxide and
Basic nature : evolution of a mixture of NO2 and O2 takes place.
Be ( OH ) 2 + 2HCl ¾¾ ® BeCl 2 + 2H 2 O 2Ca(NO3)2 ® 2CaO + 4NO2 + O2
brown coloured gas
Acidic nature: 19. (a) Ca(OH)2 is used in the manufacture of bleaching
Be ( OH ) 2 + 2NaOH ¾¾
® Na 2 BeO 2 + 2H 2 O powder.
Sodium beryllate
Cold
11. (a) Sodium carbonate can be synthesised by Solvay 2Ca ( OH ) 2 + 2Cl2 ¾¾¾
® CaCl2 + Ca(OCl) 2 + 2H 2 O
Slaked Bleaching
ammonia soda process. lime powder
The reactions involved in the process are : 20. (c) To recover NH3 in Solvay process Ca(OH)2 is used.
NH3 + H 2O + CO2 ¾¾
® NH 4 HCO3 On passing CO2 through Ca(OH)2, it turns milky due
Ammonium bicarbonate to the formation of CaCO3. Also, Ca(OH)2 is used for
white washing due to its disinfectant nature.
Ca(OH)2 + CO2 ® CaCO3¯+ H2O
The-s-Block Elements 239

21. (d) Chlorides of alkaline earth metals are hydrated salts. MgSO 4 > CaSO4 > SrSO 4 > BaSO 4
Due to their hygroscopic nature, they can be used as a Hydration Solubility
dehydrating agent and can absorb moisture from air. energy
Extent of hydration decreases from Ca to Ba. 24. (b) K+ ion is a constituent of sodium pump.
NEET/AIPMT (2013-2017) Questions 25. (c) Carbonates becomes more thermally stable down the
22. (d) In Castner Kellner cell, sodium amalgam is formed at group, therefore MgCO3 will leave CO2 easily.
mercury cathode. 26. (c)
23. (d) Solubility of alkaline earth metal sulphates decreases
down the group due decrease in hydration energy.
EBD_7327
240 CHEMISTRY

The p-block elements


11 (Group 13 and 14)
GROUP 13 ELEMENTS : THE BORON FAMILY Chemical Properties
It includes Boron (B) non-metal (i) Oxidation states
Aluminium (Al) metal Al forms Al3+ ions. Down the group, stability of +3 O.S.
Gallium (Ga) decreases while that of + 1 O.S. increases. This is because of
Indium (In) metals inert pair effect (i.e. due to poor shielding effect of intervening
Thalium (Tl) d- and f-orbitals, the ns e–1 s becomes inert towards bonding)
Atomic and Physical Properties (ii) Tendency of forming ionic compounds
(i) Electronic Configuration The tendency of formation of ionic compounds increases
All these elements have three electrons in the outermost from B to Tl. Boron forms only covalent compounds. This is
energy levels and their outer electronic configuration is due to its extremely high ionisation energy which is not
ns2 np1. compensated either by the lattice energies of its ionic
(ii) Atomic Radii compounds or by the hydration energies of ions in aqueous
The atomic radii are smaller than group 1 and 2 elements due solution.
to increased nuclear charge. (iii) Trends in chemical reactivity
Atomic radius increases B to Tl. The abrupt increase in the In trivalent state, they form e– deficient compounds. Thus,
atomic radius of Al is due to greater screening effect in Al
they show tendency to accept e– pair and hence act as Lewis
than in B. Atomic radius of Ga is less than that of Al. This is
acids. This tendency decreases down the group.
due to presence of d–e–1 s in Ga which have a poor shielding
effect thus resulting in poor screening of outer e–1 s from Ex:
nuclear charge. As a result, size of Ga < Al. (a) BCl 3 accepts lone pair of e – s from NH3 to form
(iii) Ionization enthalpy BCl3 .NH3.
Inspite of the large nuclear charge and small size, the first (b) AlCl3 becomes stable by forming a dimer Al2Cl6. In
ionisation energies of these elements are less than the trivalent state, most of their compounds are hydrolysed
corresponding value of s-block elements. This is because p- in water. For ex: the trichlorides on hydrolysis form
electrons are less penetrating and more shielded than s- -
electrons, i.e., they are farther from the nucleus and hence é M ( OH ) ù species.
ë 4û
held less tightly than free s-electrons and thus can be easily
(iv) Complex formation : On account of their smaller size and
removed.
(iv) Electronegativity greater charge, these elements have a much greater tendency
Down the group, electronegativity first decreases from B to to form complexes than the s-block elements.
Al and then increases marginally. This is due to difference in (v) Reactivity towards air:
atomic size of the elements. D
4M ( s ) + 3O2 ( g ) ¾¾
® 2M 2O3 ( s )
(v) Density : Density increases from B to Tl due to increase in
size of the atom. Aluminium has an extremely low density D
and hence used as an important structural material. 2M ( s ) + N 2 ( g ) ¾¾
® 2MN ( s ) (M = element)
(vi) Melting and boiling points : Melting points do not vary Aluminium forms a very thin oxide layer on surface which
regularly due to structural changes in the elements. Boron protect the metal from further attack.
has a very high melting point because it exists as a giant Boron trioxide is acidic. Al and Ga oxides are amphoteric and
covalent polymer in both solid and liquid states. The melting those of In and Tl are basic in their properties.
point decreases from B to Ga and then increases. Crystalline boron is unreactive. It is the amorphous boron
Due to its unusually low m. pt. (303 K), Ga can exist as liquid
which reacts with air.
during summers.
The p-block elements (Group 13 and 14) 241

(vi) Reactivity towards acids and alkalies: Some Important Compounds of Boron
B does not react with acids and alkalies. Al shows amphoteric Borax :
character and reacts with both acids and alkalies.
It is a white crystalline solid with formula Na2B4O7.10H2O.
2 Al(s) + 6HCl (aq) ¾¾ ® 2 Al3+ (aq) + 6Cl– (aq) + 3H2(g)
It contains tetranuclear units and the correct formula is therefore
2 Al(s) + 2NaOH (aq) + 6H2O (l) ¾¾ ®
Na 2 [B4O5 (OH) 4 ].8 H 2O
2 Na [Al(OH)4]– (aq) + 3H2 (g)
+

Conc HNO3 renders Al and Ga passive by forming a protective It dissolves in water to give an alkaline solution.
oxide layer on the surface. Preparation :
(vii) Reactivity towards halogens : (i) From colemanite :
With halogens they form trihalides of formula MX3
Ca 2B6O11 + 2Na 2CO3 ¾¾
®
2M (s) + 3X2 (g) ¾¾® 2 MX3 (s) Colemanite

(X = F, Cl, Br, I) 2CaCO3 ¯ + Na 2B4O7 +2NaBO2


Borax sod. metaborate
Anomalous Behaviour of Boron
Owing to its extremely small size, high electron density, high (ii) From boric acid :
electronegativity and non-avilability of d-electrons, boron exhibits 4H3 BO3 + Na 2 CO3 ¾¾
® Na 2 B4O7 + 6H 2O + CO 2
following anomalous properties :
(i) Boron is a typical non-metal, while other members of the Properties :
group are metals. (i) Aqueous solution of borax is alkaline due to hydrolytic
(ii) It forms only covalent compounds while other members can decomposition which produces NaOH and H3BO3.
form covalent as well as ionic compounds. The maximum
covalency of B is four while that of Al is six. Na 2 B4 O7 + 7H 2O ¾¾
® 2NaOH + 4H3BO 3
(iii) Its oxides and hydroxides have acidic nature while that of (ii) On heating, borax loses water and swells into a white mass
other members have amphoteric and basic nature. which on further heating melts to forms a transparent glassy
(iv) Its hydrides are quite stable while that of other members are solid called borax glass and borax bead.
not so stable.
(v) Like non-metals, it combines with metals to form metal borides D D
Na 2 B4 O7 .10H 2O ¾¾® Na 2B 4O 7 ¾¾
® 2NaBO 2 + B2O 3
(eg. Mg3B2) while other members form alloy with metals. Sodium Borax Boric
metaborate bead anhydride
(vi) It does not decompose steam while other members do so.
Trihalides of Boron : The metaborates of many transition metals have characteristic
Due to small size and high electronegativity of boron, all boron colours therefore borax bead test is used for qualitative
halides are covalent. analysis of various cation.
Boron atom, in BX3, has six electrons in the outermost orbit and Uses
thus it can accept a pair of electrons from a donor atom like N, O, (i) Borax is used in the manufacture of enamels, glazes, pyrex
P or S to complete its octet. Hence boron halides function as very glass.
efficient Lewis acids. (ii) It is also used as a cleansing and stiffening agent in laundry
The relative lewis acid character of boron trihalides is found to work and in candle industry.
follow the following order.
(iii) It is used for glazing paper, playing cards and also as an
BI3 > BBr3 > BCl3 > BF3
antiseptic.
This anomalous behaviour can be explained on the basis of the
(iv) It is also used in water softening.
relative tendency of the halogen atom to back donate its
unutilised electrons to the vacant p-orbitals of boron atom. Orthoboric Acid
It is white crystalline solid with a soapy touch.
Completely filled 2p Vacant 2p orbital Preparation
orbital of fluorine of boron (i) From colemanite,
F F
Ca 2 B6 O 11 + 2SO2 + 9H 2O ¾¾
® 2CaSO3 + 6H 3BO3

F B F+

B (ii) It is prepared by acidifying an aqueous solution of borax:
® 2NaCl + 4B ( OH )3
Na 2 B4O7 + 2HCl + 5H 2O ¾¾
F F Properties
Formation of back bonding between boron and (i) It is a weak monobasic acid and acts as a Lewis acid.
fluorine in BF 3 molecule. -
B ( OH )3 + 2HOH ¾¾
® éë B ( OH )4 ùû + H 3O +
EBD_7327
242 CHEMISTRY

(ii) Action of heat :


100° C
H 3BO 3 ¾¾¾® HBO 2 + H 2O
Orthoboric acid Metaboric acid H H
160°C C B
4HBO 2 ¾¾¾® H 2 B4 O 7 + H 2 O
Metaboric acid Tetraboric acid HC CH HN NH
red heat
H 2 B4O7 ¾¾¾¾ ® 2B2O3 + H 2O HC CH HB BH
Tetraboric acid Boron trioxide
C N
Uses
(i) It is used as an antiseptic, as eye-wash (eye-lotion) and as a H H
food preservative. Benzene Borazole
(ii) It is used in the preparation of glass, glazes and enamels. Empty 2p orbital
(iii) It is also used in leather industry.
Diborane
Its formula is B2H6. N B
Preparation :

4BF3 + 3LiAlH 4 ¾¾
® 2B2 H 6 + 3LiF + 3AlF3 B N

2NaBH 4 + I 2 ¾¾
® B2 H 6 + 2NaI + H 2
N B
(Lab method) Filled 2p
450K orbital
2BF3 + 6NaH ¾¾¾® B2 H 6 + 6NaF
(Industrial method)
Properties : (vii) Formation of complex borohydride :
(i) It is a colourless, highly toxic gas.
-
(ii) It catches fire spontaneously on exposure to air. ® 2M + [BH 4 ]
2MH + B2 H 6 ¾¾ ( M = Li or Na )
borohydride
(iii) Action of oxygen : B2 H 6 + 3O 2 ¾¾
® B2 O3 + 3H 2 O
Uses of diborane
(iv) Hydrolysis : (i) It is used as a catalyst in polymerisation reactions.
B2 H 6 ( g ) + 6H 2O ( l ) ¾¾
® 2B ( OH )3 ( aq ) + 6H 2 ( g ) (ii) It is used as a reducing agent in organic reactions.
(iii) It is used for welding torches.
(v) Reaction with Lewis base : (iv) Diborane is also used for preparing substances such as high
B2 H 6 + 2NMe3 ¾¾
® 2BH 3 . NMe3 energy fuels and propellents.
(v) Hydroboration is a useful method for preparing hydrocarbons,
B2 H 6 + 2CO ¾¾
® 2BH 3 .CO alcohols, ketones and acids.
Structure and bonding in diborane :
(vi) Action of ammonia :
Each B atom uses sp3 hybrid orbitals for bonding. Two of the four
+ sp3 orbitals of each of the boron atom overlap with 1 s orbitals of
® 3 éë BH 2 ( NH 3 ) 2 ùû
3B2 H 6 + 6NH3 ¾¾ [ BH 4 ]-
two terminal hydrogen atoms forming two normal B – H s bonds.
D
¾¾
® 2B3 N3 H6 + 12H 2 One of the remaining sp3 hybrid orbital ((either filled or empty) of
borazine or borazole one of the boron atoms, the 1s orbital of the bridge hydrogen atom
(Inorganic benzene)
and one of the sp3 hybrid orbitals (empty or filled respectively) of
Borazole has been called as inorganic benzene because it is the second boron atom overlap to form a delocalised orbital
very close to benzene in structure and properties. covering the three nuclei. Thus a pair of electrons (one electron
Borazole is isoelectronic with benzene. It is a planar molecule from B and other from H) acts as a bond between three atoms
in which B and H atoms are sp2 hybridized. The difference is (centres) ; such bond is known as three-centre two electron bonds
that in borazole the p-bonding is dative and it arises from (3 c – 2 e bonds).
overlap of empty p-orbitals of boron with filled p-orbitals of The 3-centre – 2 e– bridge bonds are also referred to as banana
nitrogen. bonds.
The p-block elements (Group 13 and 14) 243

(v) Physical properties


All the members of this group are solids. Their melting point
Empty sp3 and boiling points decreases form carbon to lead. C and Si
orbital of B are non-metals, Ge is a metalloid while Sn and Pb are soft
H H metals with low m. pts.
Chemical Properties
(i) Oxidation states and trends in chemical reactivity :
B B Their common O. S. are +4 and +2. C exhibits negative O. S.
also. Due to inert pair effect, the stability of +4 O.S. decreases
down the group while that of +2 O.S. increases. Thus, Pb Cl2
H is more stable than PbCl4.
Except C, other elements can expand their covalency beyond
4 due to presence of d-orbitals in them. Thus, their halides
Hb undergo hydrolysis and have tendency to form complexes
by accepting e– pairs from donor species.
Ht Ht 2-
For example : SiF62 - , [ GeCl6 ]2 -, éSn ( OH )6 ù
B B ë û
(ii) Reactivity towards oxygen :
Ht Ht They form two types of oxides : monoxide (MO) and dioxide
(MO2)
Hb Oxides in higher oxidation state are more acidic than those in
Structure of diborane showing two three centre - two electron bonds. lower oxidation state. Down the group the acidic character
Ht represents terminal H atom
Hb represents bridged H atom decreases.
CO2, SiO2, GeO2 are acidic while SnO 2 and PbO2 are
amphoteric. CO is neutral, GeO is acidic, SnO and PbO are
GROUP 14 ELEMENTS : THE CARBON FAMILY amphoteric.
It includes (iii) Reactivity towards water:
Non-metal : [Carbon (C) (a) C, Si and Ge are not affected by water.
Silicon (Si)] D
(b) Sn + 2H 2O ¾¾
® SnO 2 + 2H 2
Semi-metal : [Germanium (Ge)] (steam)
Metal : [Tin (Sn
(c) Pb is unaffected by water due to oxide film formation on
Lead (Pb)]
its surface.
Carbon is the important constituent of all organic matter while
(iv) Reactivity towards halogens
silicon is the main constituent of inorganic matter.
They form two types of halides : MX2 and MX4.
Atomic and Physical Properties Most of the MX4 are covalent in nature. However, SnF4 and
(i) Electronic Configuration : PbF4 are ionic in nature. PbI4 does not exist. Stability of
The valence shell electronic configuration of these elements dihalides increases down the group. Thus, GeX4 is more
is ns2 np2. stable than GeX2 but PbX2 is more stable than PbX4. Except
(ii) Covalent Radius : CCl4, other tetrachlorides are easily hydrolysed by water due
The radius increases from C to Si due to addition of a new to presence of d-orbitals in them. Ex: hydrolysis of SiCl4.
shell.
From Si to Pb, increase in size is small. This is due to presence
Cl Cl
of d-and f-e–1s which have a poor screening effect.
(iii) Ionization enthalpy :
The 1st ionisation energy of group 14 members is higher Si + O Si
–HCl
than group 13 members.
Ionisation energy decreases down the group. Small decreases Cl Cl Cl H H Cl OH
Cl
in ionisation energy from Si to Ge to Sn and slight increase in
OH
ionisation energy from Sn to Pb is due to poor shielding
effect of intervening d and f orbitals and increase in size of
atom. +3H2O Si
(iv) Electronegativity
–3HCl
Due to small size, they are more electronegative than HO OH
OH
group 13 elements. The electronegativity decreases from C
to Pb but not in regular manner due to filling of d- and Silicic acid
f-orbitals.
EBD_7327
244 CHEMISTRY

Important Trends and Anomalous Behaviour of Some Important Compounds of C and Si


Carbon Oxides of Carbon
(i) Due to its smaller size, higher electronegativity, higher (i) Carbon monoxide (CO)
ionisation energy and unavailability of d-orbitals, carbon (a) Preparation :
differs from rest of the members of its group.
D
(ii) Due to absence of d-orbitals, the maximum covalency of C is 1. 2C(s) + O 2 (g) ¾¾® 2CO(g)
4 while for other members, it can be more than 4 due to
373 K
presence of d-orbitals. 2. HCOOH ¾¾¾¾¾® H 2 O + CO
conc.H 2SO4
(iii) Carbon forms pp - pp multiple bonds with itself and with
other atoms of small size and high electronegativity. Examples 473 K
3. C(s) + H 2O(g) ¾¾¾¾
® CO(g) + H 2 (g)
are C = C, C º C, C = O, C = S and C º N. Heavier elements do 1273 K
Water or synthesis gas
not form pp-pp bonds because their atomic orbitals are too
large and diffuse to have effective overlapping. 4. 1273K
2C(s) + O 2 (g) + 4 N 2 (g) ¾¾¾¾
® 2CO(g) + 3 N 2 (g)
(iv) Carbon shows the property of catenation i.e. self-linking of E55555555555F
the atoms through covalent bonds. This is because C–C Producer gas
bonds are very strong. Down the group, catenation tendency 5. CO 2 + C ¾¾
® 2CO; CO 2 + Zn ¾¾
® ZnO + CO
decreases.
(v) Carbon remains unaffected by alkalies while other members 6. K 4 Fe(CN)6 + 6H 2SO 4 + 6H 2O ¾¾
® 2K 2SO 4 + FeSO 4
reacts with alkali as
+3(NH 4 )2 SO 4 + 6CO
1
Si + 2NaOH + O 2 ¾¾
® Na 2SiO 2 + H 2 (b)Properties :
2 Sodium silicate
1. It is colourless, odourless, water insoluble gas.
Allotropes of Carbon 2. It is a powerful reducing agent.
Carbon exhibits many allotropic forms: both crystalline and 3. It combines with haemoglobin (Hb) in the blood thus
amorphous. Diamond, graphite and fullerenes are the allotropes forming carboxyhaemoglobin which decreases oxygen
of carbon. carrying capacity of Hb.
Diamond 4. As a reducing agent
It has a rigid 3D network of C–atoms. 600- 900° C
Fe 2 O3 + 3CO ¾¾¾¾¾
® 2Fe + 3CO2
It has directional covalent bonds throughout the lattice.
It is very difficult to break extended covalent bonding and hence H 2 O (steam) + CO ¾¾¾¾¾¾
high temp.
® H 2 + CO2
pressure, catalyst
diamond is the hardest substance known. It is used as an abrasive
for sharpening hard tools, in making dies and in the manufacture 5. It behaves as an unsaturated compound and gives
of tungsten filaments for electric light bulbs. addition product with a number of substances.
In diamond all the electrons are used in bond formation, there is 150- 400° C
CO + 2H 2 ¾¾¾¾¾ ® CH3OH
no mobile electron in the system and hence the diamond crystals ZnO + Cu
(methanol)
are non-conductor of electricity.
sunlight
Graphite CO + Cl 2 ¾¾¾¾¾
or camphor
® COCl 2
Carbonyl chloride (phosgene)
It has a layered structure and the layers are held by
van der Waals forces. The C atoms undergo sp2 hybridisation, 50°C
ˆˆˆˆ†
Ni + 4CO ‡ˆˆˆˆ Ni(CO) 4
forming 3 s–bonds. The fourth e– forms a p- bond and is delocalised 180° C
Nickel tetracarbonyl
over the whole sheet. Electrons are mobile and this makes graphite
a good conductor of electricity. It is soft and slippery and is thus Uses :
used as a dry lubricant in machines running at high temperature. (i) Carbon monoxide is used as a fuel in the form of water
Graphite is thermodynamically the most stable allotrope of carbon gas (CO + H2) and producer gas (CO + N2).
and thus, DfH° for graphite is taken to be zero. (ii) It is used in making carbonyls which in turn are used in
Graphite is used (i) in making electrodes and carbon arcs, the extraction of iron and nickel.
(ii) as a lubricant, (iii) in crucible, (iv) in lead pencils, and
Ni + 4CO ¾¾
® Ni(CO) 4 ;Fe + 5CO ¾¾
® Fe(CO) 5
(v) as a moderator in nuclear reactors.
(iii) It is used in the manufacture of methanol, synthetic
Fullerenes
petrol and phosgene.
They are made by heating graphite in an electric arc in presence of
(iv) It is used as a reducing agent in metallurgy of iron and
inert gases such as He or Ar. They are pure form of carbon.
in the manufacture of hydrogen.
Fullerenes are large cage like spheroidal molecules with general
formula C2n (where n ³ 30). Two important member are C60 and Fe 2 O3 + 3CO ¾¾
® 2Fe + 3CO2
C70. C60 fullerene looks like a soccer ball (so called bucky ball). C60
catalyst
is called Buckminsterfullerene. H 2 O (steam) + CO ¾¾¾¾
® H 2 + CO 2
The p-block elements (Group 13 and 14) 245

(ii) Carbon dioxide (CO2) (vi) Silica gel is used as a drying agent and as a support for
(a) Preparation : chromatographic materials and catalysts.
D
(vii) Kieselghur, an amorphous form of silica, is used in filteration
1. C(s) + O 2 (g) ¾¾
® CO 2 (g) plants.
2. Commercial method : Silicones
heat They are a group of organosilicon polymers having –( R2 SiO )– as
CaCO3 ¾¾¾
® CaO + CO 2 ­
a repeating unit. The starting materials for their preparation are
heat
2NaHCO 3 ¾¾¾
® Na 2CO 3 + CO 2 + H 2 O alkyl or aryl substituted silicon chlorides RnSiCl(4 – n).
D
(i) Preparation of linear silicones :
3. CH 4 (s) + 2 O 2 (g) ¾¾® CO 2 (g) + 2 H 2O(g) Cu powder
2CH3Cl + Si ¾¾¾¾¾
® (CH3 ) 2SiCl 2
4. Laboratory method : 570K

CaCO 3 (s) + 2 HCl(aq) ¾¾


® +2H O
¾¾¾¾
2 ® (CH ) Si(OH)
3 2 2
-2HCl
CaCl 2 (aq) + CO2 (g) + 2H 2O(l)
(b) Properties : CH3 CH3 CH3
1. It is one of the gases responsible for greenhouse effect. | | |
It is not poisonous as CO. HO - Si - OH + HO - Si - OH + HO - Si - OH
| | |
2. It is obtained as a solid in form of dry ice by allowing CH3 CH3 CH3
liquefied CO2 to expand rapidly.
3. It exists as a resonance hybrid of two structures : –H2O Polymerisation
+ + –
O – C º O « O = C = O « Oº C – O
CH3 CH3 CH3 CH3
4. It turns lime water milky | 00000000000000
| | |
- O - Si - O - Si - O - Si - O - Si - O -
Ca(OH)2 + CO 2 ¾¾
® CaCO3 + H 2O | | | |
CH3 CH3 CH3 CH3
5. Action of NH3 : Linear silicon
453 - 473K (ii) Preparation of cyclic silicones: Cyclic (ring) silicones are
2NH3 + CO2 ¾¾¾¾¾
®[NH 2COONH 4 ]
200atm formed when water is eliminated from the terminal – OH
¯
NH2 CONH 2 + H2O groups of linear silicones.
urea
(iii) Preparation of cross linked silicones : Hydrolysis of alkyl
6. Photosynthesis : trichlorosilane, CH3SiCl3 gives the monomethylsillantriol
chlorophyll which then undergoes polymerisation to a very complex cross
6CO2 + 6H 2 O ¾¾¾¾¾
® C 6H12O6 + 6O2
light linked polymer.
Uses : CH3 CH 3
(i) Dry ice is used as a refrigerant. | |
(ii) Gaseous CO2 is used in cold drinks, in fire extinguisher Cl - Si - Cl + H 2O ¾¾
® HO - Si - OH
and to manufacture urea. | |
(iii) It is used in manufacture of white lead and Na2CO3 Cl OH
Methyl trichlorosilone Methyl silantriol
Silicon Dioxide, SiO2 |
(i) It is commonly known as silica. O CH3
(ii) It is a covalent, 3D network solid in which each Si atom is | |
covalently bonded in a tetrahedral manner to four O-atoms. OH CH3 – Si – O – S – O –
| | |
(iii) In normal form, silica is non-reactive due to high Si–O bond n CH3 - Si - OH ¾ ¾
® O O
enthalpy. | | |
(iv) It is attacked by F2 , HF and NaOH. OH CH3 – Si – O – Si – CH3
| |
SiO 2 + 2NaOH ¾¾
® Na 2SiO 3 + H 2 O O O
| |
SiO2 + 2F2 ¾¾
® SiF4 + O 2 Cross-linked silicone
Properties :
SiO2 + 4HF ¾¾ ® SiF4 + 2H 2 O (i) Being surrounded by non-polar alkyl groups, silicones are
(v) Quartz (crystalline form of silica) is used as a piezoelectric water repelling in nature.
material and has helped to develop extremely accurate clocks, (ii) They have high thermal stability, high dielectric strength and
modern radio and television broadcasting and mobile radio resistance to oxidation.
communications. (iii) Silicone is non-toxic.
EBD_7327
246 CHEMISTRY

(iv) Silicone products may be fluids, greases, resins or rubbers (iv) The various types of silicates are
depending upon the size and type of polymer. (a) Orthosilicates (b) Pyrosilicates
Uses (c) Cyclic or ring silicates (d) Chain silicates
They are used as sealant, greases, electrical insulators and for (e) Sheet silicates
water proofing of fabrics.
They are also used as non-stick coating for pans, and are used in (f) Three dimensional silicates
paints and varnish. (v) Two man-made silicates are glass and cement.
Silicates Zeolites
(i) Its examples are feldspar, zeolites, mica and asbestos. (i) They are formed when Al - atoms replace few Si-atoms in 3D
network of SiO2 such that the overall structure acquires a
(ii) Their basic structural unit is SiO44- . Each Si atom is bonded negative charge which is balanced by cations such as
to four O-atoms in tetrahedral fashion. Na+, K+ or Ca2+.
(iii) Silicates can be prepared by fusing an alkali metal carbonate (ii) They act as molecular sieves by trapping small molecules in
with silica. their interstices reversibly.
1500° C (iii) They are used as a catalyst in petrochemical industries for
Na 2 CO3 ¾¾¾¾
® CO 2 + Na 2 O cracking of hydrocarbons and isomerisation.
SiO 2
¾¾¾ ® Na 2SiO 4 ,(Na 2SiO3 ) n and others. (iv) Hydrated zeolites are used as ion exchangers in softening of
hard water.
The p-block elements (Group 13 and 14) 247
CONCEPT MAP
EBD_7327
248 CHEMISTRY

1. Alum is : 13. Thermite is a mixture of :


(a) FeSO4.(NH4)2SO4.24H2O
(a) Cr2 O3 + Al 2O3 (b) Fe 2 O 3 + Al
(b) Na2SO4. Al2(SO4)3.24H2O
(c) KCl.MgCl2.6H2O (c) Fe2 O3 + Al 2 O3 (d) Al2 O3 + 2Cr
(d) NaAlO2
2. In borax bead test which compound is formed? 14. Thermite is a mixture of iron oxide and
(a) Ortho-borate (b) Meta-borate (a) zinc powder (b) potassium metal
(c) Double oxide (d) Tetra-borate (c) sodium shavings (d) aluminium powder
3. Alum helps in purifying water by 15. Inorganic benzene is :
(a) forming Si complex with clay particles
(b) sulphate part which combines with the dirt and removes it (a) B3H 3 N 3 (b) BH 3 NH 3
(c) coagulaing the mud particles
(d) making mud water soluble. (c) B3 H 6 N 3 (d) H 3 B3 N 6
4. Which of the following halides is least stable and has 16. An aqueous solution of potash alum gives :
doubtful existence?
(a) CI4 (b) GeI4 (a) two types of ions (b) only one type of ion
(c) SnI4 (d) PbI4 (c) four types of ions (d) three types of ions
5. Unlike the other elements of its group carbon does not form
17. Ionisation enthalpy (D iH1kJ mol –1) for the elements of
CX2 type molecules because :
(a) energetically this is not possible Group 13 follows the order.
(b) carbon undergoes catenation (a) B > Al > Ga > In > Tl (b) B < Al < Ga < In < Tl
(c) it is non-metallic (c) B < Al > Ga < In > Tl (d) B > Al < Ga > In < Tl
(d) carbon does not contain d-orbital
6. AlI3, when react with CCl4, gives 18. The liquefied metal which expands on solidification is :
(a) AlCl3 (b) CI4 (a) Ga (b) Al
(c) Al4C3 (d) Al2O3 (c ) Zn (d) In
7. Which of the following statements about anhydrous
aluminium chloride is correct? 19. Amphoteric oxide among the following is –
(a) It exists as AlCl3 molecules (a) B2O3 (b) Ga2O3
(b) It is not easily hydrolysed (c) In2O3 (d) Tl2O3
(c) It sublimes at 180ºC 20. Which of the following has the least melting point ?
(d) It is a strong Lewis base
8. Orthoboric acid – (a) B (b) Al
(a) donate proton to form H2BO3– (c) Ga (d) In
(b) accept proton of form H4BO3+ 21. Borax is used as cleansing agent because on dissolving in
(c) donate OH– to form H2BO2+ water it gives
(d) accept OH– to form [B(OH)4]–
(a) Alkaline solution (b) Acidic solution
9. The approximate percentage of silica in cement is:
(a) 5 – 10% (b) 15 – 20% (c) Bleaching solution (d) Colloidal solution
(c) 20 – 25% (d) 25 – 30% 22. Which of the following properties of aluminium makes it
10. The strongest Lewis acid is – useful for food packaging ?
(a) BF3 (b) BCl3 (a) Good electrical conductivity
(c) BBr3 (d) BI3
11. Bell metal is an alloy of: (b) Good thermal conductivity
(a) Sn and Pb (b) Cu and Pb (c) Low density
(c) Sn and Cu (d) In, Sn and Cu (d) Non toxicity
12. In the Hoope’s process of aluminium extraction, the fused 23. Aluminium reacts with caustic soda to form:
materials remain in three different layers. These layers remain
separated even in electrolytic reduction, because : (a) aluminium hydroxide (b) aluminium oxide
(a) cell has the arrangement for separation (c) sodium meta-aluminate(d) sodium tetra-aluminate
(b) all the layers have different densities 24. In the reaction B2 O3 + C + Cl 2 ® A + CO. The A is
(c) all the layers are at different temperatures
(d) upper layer is attracted by cathode and lower layer is (a) BCl3 (b) BCl2
attracted by anode. (c) B2Cl2 (d) CCl2
The p-block elements (Group 13 and 14) 249

25. Non-oxide ceramics can be (c) It is prepared by heating Al2O3 in a stream of sulphur
(a) B4C (b) SiC chloride (S2Cl2) vapours and chlorine.
(c) Si3N4 (d) All of these (d) All of these
26. Which of the following metals burn in air at high 38. Aqueous solution of orthoboric acid can be titrated against
temperature with the evolution of heat ? sodium hydroxide using phenolphthalein indicator only in
(a) Cu (b) Hg presence of –
(c) Pb (d) Al (a) trans-glycerol (b) catechol
27. Which of the following product is formed when SiF4 reacts (c) cis-glycerol (d) Both (b) and (c)
with water? 39. Which of the following statement is correct with respect to
(a) SiF3 (b) H4SiO4 the property of elements with an increase in atomic number
(c) H2SO4 (d) H2SiF4 in the carbon family (group 14)?
28. Boron forms covalent compound due to (a) atomic size decrease
(a) higher ionization energy (b) ionization energy increase
(b) lower ionization energy (c) metallic character decrease
(c) small size (d) stability of +2 oxidation state increase
(d) Both (a) and (c) 40. H3BO3 on heating up to 373 K yields:
29. Which metal is protected by a layer of its own oxide (a) boric anhydride (b) orthoboric acid
(a) Al (b) Ag (c) metaboric acid (d) tetraboric acid
(c) In (d) Fe 41. Which of the following statements about H3BO3 is not
30. When Al is added to KOH solution
correct?
(a) no action takes place (b) oxygen is evolved
(a) It is a strong tribasic acid
(c) water is produced (d) hydrogen is evolved
(b) It is prepared by acidifying an aqueous solution of
31. Which of the following is not true about potash alum?
(a) Its empirical formula is KAl(SO4)2. 12H2O. borax
(b) Its aqueous solution is basic (c) It has a layer structure in which planar BO3 units are
(c) It is used in dyeing industries joined by hydrogen bonds
(d) All are correct. (d) It does not act as proton donor but acts as a Lewis acid
32. AlCl3 acts as a strong Lewis acid because AlCl3 is : by accepting a lone pair of electrons
(a) easily hydrolysed 42. Boron halides behave as Lewis base, because of their :
(b) a covalent compound (a) ionic nature (b) acidic nature
(c) on ionic compound (c) covalent nature (d) electron deficient nature
(d) an electron deficient compound 43. Aluminium chloride exists as dimer, (Al2Cl6) in solid state as
33. Which one of the following is a metalloid? well as in solution of non-polar solvents such as benzene.
(a) phosphorous (b) antimony When dissolved in water, it gives
(c) nitrogen (d) bismuth
34. In the electrolytic extraction of aluminium, cryolite is used
(a) [Al(OH) 6 ]3- + 3HCl (b) [ Al( H 2 O ) 6 ]3+ + 3Cl -
(a) to obtain more aluminium (c) Al 3+ + 3Cl - (d) Al2 O3 + 6HCl
(b) to decrease temperature to dissolve bauxite 44. Among the following the hardest is :
(c) to protect the anode (a) Peat (b) Lignite
(d) as reducing agent (c) Graphite (d) Anthractie
35. Select the correct statements for diborane : 45. Which one of the following exist in the oxidation state other
1. Boron is approximately sp3 hybridized than +3?
2. B – H – B angle is 180° (a) B (b) Al
3. There are two terminal B – H bonds for each boron (c) Ce (d) Ga
atom 46. In aqueous solution GaCl disproportionates to
4. There are only 12 bonding electrons (a) GaCl2 and GaCl3 (b) Ga and GaCl3
(a) 1, 2 and 4 (b) 1, 2 and 3
(c) GaCl2 and Ga (d) GaCl3 and GaCl5
(c) 2, 3 and 4 (d) 1, 3 and 4
47. Which of the following is a semiconductor ?
36. Action of concentrated nitric acid (HNO3) on metallic tin
produces (a) C (b) Pb
(a) stannic nitrate (b) stannous nitrite (c) Ge (d) Sn
(c) stannous nitrate (d) hydrated stannic oxide 48. Ordinary glass is :
37. Which of the following statements about anhydrous (a) Sodium silicate
aluminium chloride is correct? (b) Calcium silicate
(a) It fumes in moist air (c) Sodium and calcium silicate
(b) It exists as dimer both in the vapour state below 350°C (d) Mixed salt of Na and Ca
and in non-polar solvents 49. Which of the following types of forces bind together the
carbon atoms in diamond ?
EBD_7327
250 CHEMISTRY

(a) Ionic (b) Covalent (c) silica with magnesium


(c) Dipolar (d) Van der Waals. (d) None of these
50. Water gas is produced by 64. In the manufacture of glass addition of MnO2 gives :
(a) passing steam through a red hot coke bed (a) yellow colour (b) red colour
(b) saturating hydrogen with moisture (c) violet colour (d) pink colour
(c) mixing oxygen and hydrogen in the ratio of 1 : 2 65. Carborundum is
(d) heating a mixture of CO2 and CH4 in petroleum refineries. (a) SiC (b) CaC2
51. Activated charcoal is employed to remove colouring matter (c) Mg2C3 (d) None of these
from pure substances. It works by : 66. White lead is
(a) Bleaching (b) Oxidation (a) PbCO3 (b) PbCO3 .PbO
(c) Reduction (d) Adsorption (c) 2PbCO3 . Pb(OH)2 (d) 2PbSO4 .PbO
52. In graphite, electrons are 67. Pb reacts with dilute HNO3 and produces
(a) localised on every third C-atom (a) NO (b) NH4NO3
(b) present in anti-bonding orbital (c) N2O5 (d) NO2
(c) localised on each C-atom 68. In silica (SiO2), each silicon atom is bonded to
(d) spread out between the structure
(a) two oxygen atoms
53. Glass reacts with HF to produce
(b) four oxygen atoms
(a) SiF4 (b) H2SiF6
(c) one silicon and two oxygen atoms
(c) H2SiO3 (d) Na3AlF6
54. The oxide of lead used in lead accumulators is : (d) one silicon and three oxygen atoms
69. The inert pair effect is most prominent in –
(a) PbO (b) Pb 2 O3
(a) C (b) Pb
(c) Pb3O 4 (d) PbO 2 (c) Ge (d) Si
55. Producer gas is the mixture of : 70. Consider the following chemical reaction –
(a) CO + N2 (b) CO + H2 Fe2O3(s) + 3CO (g) ––® 2Fe (s) + 3CO2(g)
(c) CO + water vapours (d) N2 + CH4 (a) CO is an oxidising agent
56. Coal gas is a mixture of : (b) CO is reducing agent
(a) H 2 O and CO (b) H 2 ,CO, N 2 and CH 4 (c) CO is neutral towards litmus
(c) H 2 and CO (d) CH 4 and CO (d) CO acts as catalyst
57. The chemical name of phosgene is 71. Tetra ethyl lead is used as :
(a) Phosphorus (a) fire extinguisher (b) antiknock compound
(b) Carbonyl chloride (c) pain killer (d) mosquito killer
(c) Phosphorus oxychloride 72. To a piece of charcoal, sulphuric acid is added. Then:
(d) Phosphorus trichloride (a) there is no reaction
58. Least thermally stable is – (b) water gas is formed
(a) CCl4 (b) SiCl4 (c) SO2 and CO2 are evolved
(c) GeCl4 (d) GeBr4 (d) CO and SO2 are evolved
59. Be2C and Al4C3 are called – 73. Which of the following has the highest calorific value ?
(a) ethanides (b) methanides
(a) Coal gas (b) Water gas
(c) carbonides (d) acetylides
(c) Producer (d) Carbon dioxide gas
60. Which one of the following has the lowest m.p.?
(a) B (b) Al 74. Which one of the following statements about the zeolites is
(c) Ga (d) Tl. false ?
61. Red lead is: (a) They are used as cation exchangers
(a) Pb3O4 (b) Pb2O3 (b) They have open structure which enables them to take
(c) Pb2O (d) PbO up small molecules
62. Silicon dioxide is formed by the reaction of (c) Zeolites are aluminosilicates having three dimensional
(a) SiCl4 + H2O (b) SiO2 + HF network
(c) SiO2 + NaOH (d) SiCl4 + NaOH (d) None of the above
63. In laboratory silicon can be prepared by heating 75. When vapours of SiCl 4 are passed over hot Mg, the product
(a) carbon in electric furnance
(b) silica with sodium carbonate formed is :
(a) MgSiCl6 (b) SiCl2 + MgCl 2
(c) Si + MgCl 2 (d) MgSi + Cl 2
The p-block elements (Group 13 and 14) 251

Heat High temp. (d) washing soda reacts with aluminium to form insoluble
1. PbS ¾¾¾® A + PbS ¾¾¾ ¾¾® Pb + SO 2
in air No air aluminium oxide
Identify A 12. Which one of the following is correct statement ?
(a) PbO (b) PbSO4 (a) The hydroxide of aluminium is more acidic than that of
(c) None (d) (a) or (b) boron
2. Aluminium hydroxide is soluble in excess of sodium (b) The hydroxide of boron is basic, while that of aluminium
hydroxide forming the ion is amphoteric
(a) AlO +2 3 (b) AlO -2 3 (c) The hydroxide of boron is acidic, while that of aluminium
is amphoteric
(c) AlO-2 (d) AlO 3- (d) The hydroxide of boron and aluminium are amphoteric
3. The hardest substance amongst the following is 13. Newly shaped glass articles when cooled suddenly become
(a) Be2C (b) titanium brittle, therefore these are cooled slowly, this process in
(c) SiC (d) B4C known as
4. Moissan boron is (a) Tempering (b) Annealing
(a) amorphous boron of ultra purity (c) Quenching (d) Galvanising
(b) crystalline boron of ultra purity 14. CO2 and N2 are non-supporters of combustion. However
(c) amorphous boron of low purity for putting out fires CO2 is preferred over N2 because CO2
(d) crystalline boron of low purity
(a) Does not burn
5. Silicon carbide is used as a/ an
(b) Forms noncombustible products with burning
(a) abrasive (b) dehydrating agent
substances
(c) solvent (d) catalyst
(c) Is denser than nitrogen
6. Which of the following does not exist in free form
(d) Is a more reactive gas
(a) BF3 (b) BCl3
15. Aluminium has a great affinity for oxygen and its oxidation
(c) BBr3 (d) BH3
is an exothermic process. This fact is used in
7. The role of fluorspar ( CaF2) which is added in small
(a) preparing thin foils of aluminium
quantities in the electrolytic reducation of alumina dissolved
(b) making utensils
in fused cryolite (Na3AlF6) is
(c) preparing duralumin alloy
(a) As a catalyst
(d) thermite welding
(b) To make the fused mixture very conducting
16. Tin plague is the
(c) To increase the temperature of the melt.
(a) Conversion of stannous to stannic
(d) To decrease the rate of oxidation of carbon at the anode.
(b) Conversion of white tin to grey tin
8. The precious Ruby stone is
(c) Emission of sound while bending a tin rod
(a) Alumina
(d) Atmospheric oxidation of tin
(b) Aluminium silicate
17. Tin cry refers to
(c) Sodium aluminium silicate
(a) Conversion of white to grey tin
(d) Sodium silicate
(b) Tin plating
9. Anhydrous AlCl3 cannot be obtained from which of the
(c) Conversion of white tetrahedral tin to white
following reactions ?
rhombohedral tin
(a) Heating AlCl3.6H2O
(d) Emission of sound while bending a tin rod.
(b) By passing dry HCl over hot aluminium powder
18. Lead is not affected by dil. HCl in cold because
(c) By passing dry Cl2 over hot aluminium powder
(a) Pb is less electronegative than H
(d) By passing dry Cl2 over a hot mixture of alumina and coke
(b) PbO film is formed which resists chemical attack by
10. The factor responsible for weak acidic nature of B–F bonds
acid
in BF3 is
(a) Large electronegativity of fluorine (c) A protective coating of PbCl2 is formed on Pb surface
(b) Three centred two electron bonds in BF3 (d) PbO2 film is always present on Pb surface, which resist
(c) p p - d pp back bonding chemical attack
(d) p p - p p back bonding 19. Carbon and silicon belong to group 14. The maximum
11. Aluminium vessels should not be washed with materials coordination number of carbon in commonly occurring
containing washing soda since compounds is 4, whereas that of silicon is 6. This is due to
(a) washing soda is expensive (a) large size of silicon
(b) washing soda is easily decomposed (b) more electropositive nature of silicon
(c) washing soda reacts with aluminium to form soluble (c) availability of d-orbitals in silicon
aluminate. (d) Both (a) and (b)
EBD_7327
252 CHEMISTRY

20. A kettle which becomes furred-up in use has inside it a 32. In view of the signs of D r G ° for the following reactions :
deposit composed mainly of :
(a) Magnesium bicarbonate (b) Magnesium sulphate PbO 2 + Pb ® 2PbO, DrG° < 0
(c) Sodium sulphate (d) Calcium carbonate SnO 2 + Sn ® 2SnO , D r G° > 0
21. Colour is imparted to glass by mixing Which oxidation states are more characteristics for lead and
(a) synthetic dyes (b) metal oxides tin ?
(c) oxides of non-metals (d) coloured salts (a) For lead + 2, for tin + 2 (b) For lead + 4, for tin + 4
22. The form of BN which is as hard as diamond is (c) For lead + 2, for tin + 4 (d) For lead + 4, for tin + 2
(a) hexagonal form 33. When PbO2 reacts with conc. HNO3 the gas evolved is
(b) Cubic form with ZnS structure (a) NO2 (b) O2
(c) both of these (c) N2 (d) N2O
(d) none of these 34. How can the following reaction be made to proceed in
23. The type of glass used in making lenses and prisms is forward direction?
(a) Flint glass (b) Jena glass B(OH)3 + NaOH NaBO2 + Na[B(OH)4] + H2O
(c) Pyrex glass (d) Quartz glass (a) addition of borax (b) addition of cis -1,2-diol
24. SiF4 gets hydrolysed giving........ (c) addition of Na2HPO4 (d) addition of trans -1,2-diol
(a) SiO2 (b) Si(OH)2F2 35. The species present in solution when CO2 is dissolved in
(c) H2SiF6 (d) Si(OH)4 water are
25. When alumina is heated with carbon in the atmosphere of
nitrogen then products formed are (a) CO2 ,H 2CO3 ,HCO3- ,CO32-
(a) Al + CO (b) Al + CO + N2
(b) H 2CO3 , CO32-
(c) Al + CO2 (d) AlN + CO
26. Soldiers of Napolean army while at alps during freezing winter (c) CO32- , HCO3
suffered a serious problem as regards to the tin buttons of
their uniforms. White metallic tin buttons got converted to (d) CO2, H2CO3
grey powder. This transformation is related to 36. Unlike the other elements of its group carbon and silicon
(a) a change in the partial pressure of oxygen in the air does not form MX2 type molecules because :
(a) energetically this is not possible
(b) a change in the crystalline structure of tin
(b) carbon undergoes catenation
(c) an interaction with nitrogen of the air at very low
(c) it is non-metallic
temperatures
(d) carbon does not contain d-orbital
(d) an interaction with water vapours contained in the
37. Gas A is bubbled through slaked lime when a white
humid air
precipitate is formed. On prolonged bubbling the precipitate
27. On controlled hydrolysis and condensation R3SiCl yields
is dissolved. On heating the resultant solution, the white
(a) R3Si – O – SiR3 (b) -(R3Si – O – SiR3)- n
precipitate appears with evolution of gas B. The gases A
R R and B respectively are
| |
(a) CO and CO (b) CO 2 and CO
– Si – O – Si –
| | (c) CO and CO 2 (d) CO 2 and CO 2
(c) R3SiOH (d) O O
| | 38. Nitrolim is a mixture of
– Si – O – Si – (a) CaC 2 & Ga (b) CaCN 2 & C
| |
R R (c) Ca (CN) 2 & Ga (d) CaCN 2 & N 2
28. Carborundum is obtained when silica is heated at high 39. The straight chain polymer is formed by
temperature with (a) hydrolysis of CH3SiCl 3 followed by condensation
(a) carbon (b) carbon monoxide polymerisation
(c) carbon dioxide (d) calcium carbonate (b) hydrolysis of (CH3)4Si by addition polymerisation
29. Maximum pp-pp back bonding exists in (c) hydrolysis of (CH3)2SiCl2 followed by condensation
(a) BCl3 (b) BF3 polymerisation
(c) BBr3 (d) BI3 (d) hydrolysis of (CH3)3SiCl followed by condensation
30. Aluminium is extracted from alumina (Al2O3 ) by electrolysis polymerisation
of a molten mixture of : 40. The tendency of BF3, BCl3 and BBr 3 to behave as Lewis acid
(a) Al2O3 + HF + NaAlF4 decreases in the sequence:
(b) Al2O3 + CaF2 + NaAlF4 (a) BCl3 > BF3 > BBr3 (b) BBr3 > BCl3 > BF3
(c) Al2O3 + Na3AlF6 + CaF2 (c) BBr3 > BF3 > BCl3 (d) BF3 > BCl3 > BBr3
(d) Al2O3 + KF + Na3AlF6 41. Name the type of the structure of silicate in which one oxygen
31. Heating an aqueous solution of aluminium chloride to atom of [SiO4]4– is shared ?
dryness will give (a) Linear chain silicate (b) Sheet silicate
(a) Al(OH)Cl 2 (b) Al 2O 3 (c) Pyrosilicate (d) Three dimensional
(c) Al 2Cl 6 (d) AlCl3
The p-block elements (Group 13 and 14) 253

42. In silicon dioxide 46. Lead pipes are not suitable for drinking water because
(a) there are double bonds between silicon and oxygen (a) A layer of lead dioxide is deposited over pipes
atoms (b) Lead reacts with air to form litharge
(b) silicon atom is bonded to two oxygen atoms (c) Lead reacts with water containing air to form Pb(OH)2
(c) each silicon atom is surrounded by two oxygen atoms (d) Lead forms basic lead carbonate
and each oxygen atom is bonded to two silicon atoms DIRECTIONS for Qs. 47 to 50 : These are Assertion-Reason
type questions. Each of these question contains two statements:
(d) each silicon atom is surrounded by four oxygen atoms
Statement-1 (Assertion) and Statement-2 (Reason). Answer these
and each oxygen atom is bonded to two silicon atoms.
questions from the following four options.
43. The structure of diborane ( B 2 H 6 ) contains
(a) Statement-1 is true, Statement-2 is true, Statement-2 is a
(a) four 2c-2e bonds and four 3c-2e bonds correct explanation for Statement-1
(b) two 2c-2e bonds and two 3c-3e bonds (b) Statement-1 is True, Statement-2 is True ; Statement-2 is
(c) two 2c-2e bonds and four 3c-2e bonds NOT a correct explanation for Statement - 1
(d) four 2c-2e bonds and two 3c-2e bonds (c) Statement-1 is True, Statement-2 is False
44. Which one of the following is the correct statement? (d) Statement-1 is False, Statement-2 is True
(a) Boric acid is a protonic acid 47. Statement-1 : Boron is metalloid.
Statement-2 : Boron shows metallic nature.
(b) Beryllium exhibits coordination number of six
48. Statement-1 : Benzene is reactive while inorganic benzene
(c) Chlorides of both beryllium and aluminium have bridged
is unreactive compound.
structures in solid phase
Statement-2 : Inorganic benzene is, borazine, B3N3H6.
(d) B2H6.2NH3 is known as ‘inorganic benzene’ 49. Statement-1 : Silicones are hydrophobic in nature.
45. Name of the structure of silicates in which three oxygen Statement-2 : Si – O – Si linkages are moisture sensitive.
atoms of [SiO4]4– are shared. 50. Statement-1 : Pb4+ compounds are stronger oxidizing agents
(a) Pyrosilicate (b) Sheet silicate than Sn4+ compounds.
(c) Linear chain silicate (d) Three dimensional silicate Statement-2 : The higher oxidation states for the group 14
elements are more stable for the heavier members of the
group due to ‘inert pair effect’.

Exemplar Questions of the following elements is not likely to act as central atom
1. The elements which exists in liquid state for a wide range of in MF63- ?
temperature and can be used for measuring high temperature (a) B (b) Al
is (c) Ga (d) In
(a) B (b) Al 6. Boric acid is an acid because its molecule
(c) Ga (d) In (a) contains replaceable H+ ion
2. Which of the following is a Lewis acid? (b) gives up a proton
(a) AlCl3 (b) MgCl2 (c) accepts OH– from water releasing proton
(c) CaCl2 (d) BaCl2 (d) combines with proton from water molecule
3. The geometry of a complex species can be understood from 7. Catenation i.e., linking of similar atoms depends on size and
the knowledge of type of hybridisation of orbitals of central electronic configuration of atoms. The tendency of catenation
atom. The hybridisation of orbitals of central atom in in group 14 elements follows the order
[B(OH4)]– and the geometry of the complex are respectively. (a) C > Si > Ge > Sn (b) C > > Si > Ge » Sn
(a) sp3, tetrahedral (b) sp3, square planar (c) Si > C > Sn > Ge (d) Ge > Sn > Si > C
(c) sp3d2, octahedral (d) dsp2, square planar 8. Silicon has a strong tendency to form polymers like silicones.
4. Which of the following oxides is acidic in nature? The chain length of silicone polymer can be controlled by
(a) B2O3 (b) Al2O3 adding.
(c) Ga2O3 (d) In2O3 (a) MeSiCl3 (b) Me2SiCl2
5. The exhibition of highest coordination number depends on (c) Me3SiCl (d) Me4Si
the availability of vacant orbitals in the central atom. Which
EBD_7327
254 CHEMISTRY

9. Ionisation enthalpy (DiH kJ mol–1) for the elements of group 15. Cement, the important building material is a mixture of oxides
13 follows the order of several elements. Besides calcium, iron and sulphur, oxides
(a) B > Al > Ga > In > Tl (b) B < Al < Ga < In < Tl of elements of which of the group (s) are present in the
(c) B < Al > Ga < In > Tl (d) B > Al < Ga > In < Tl mixture?
10. In the structure of diborane, (a) Group 2 (b) Groups 2, 13 and 14
(c) Group 2 and 13 (d) Groups 2 and 14
(a) All hydrogen atoms lie in one plane and boron atoms
lie in a plane perpendicular to this plane NEET/AIPMT (2013-2017) Questions
(b) 2 boron atoms and 4 terminal hydrogen atoms lie in the 16. Which of the following structure is similar to graphite?
same plane and 2 bridging hydrogen atoms lie in the [2013]
perpendicular plane (a) B (b) B4C
(c) 4 bridging hydrogen atoms and boron atoms lie in one (c) B2H6 (d) BN
plane and two terminal hydrogen atoms lie in a plane 17. Which of these is not a monomer for a high molecular mass
silicone polymer? [2013]
perpendicular to this plane
(a) Me2SiCl2 (b) Me3SiCl
(d) All the atoms are in the same plane (c) PhSiCl3 (d) MeSiCl3
11. A compound X, of boron reacts with NH3 on heating to give 18. The basic structural unit of silicates is : [2013]
another compound Y which is called inorganic benzene. The
(a) SiO44- (b) SiO32-
compound X can be prepared by treating BF3 with lithium
aluminium hydride. The compounds X and Y are represented (c) SiO24- (d) SiO
by the formulas. 19. Which statement is wrong? [NEET Kar. 2013]
(a) B2H6, B3N3H6 (b) B2O3, B3N3H6 (a) Feldspars are not aluminosilicates
(c) BF3, B3N3H6 (d) B3N3H6, B2H6 (b) Beryl is an example of cyclic silicate
12. Quartz is extensively used as a piezoelectric material, it (c) Mg2SiO4 is orthosilicate
contains .............. . (d) Basic structural unit in silicates is the SiO4 tetrahedron
20. The stability of +1 oxidation state among Al, Ga, In and Tl
(a) Pb (b) Si
increases in the sequence : [2015 RS]
(c) Ti (d) Sn (a) Ga < In < Al < Tl (b) Al < Ga < In < Tl
13. The most commonly used reducing agent is (c) Tl < In < Ga < Al (d) In < Tl < Ga < Al
(a) AlCl3 (b) PbCl2 21. It is because of inability of ns2 electrons of the valence shell
(c) SnCl4 (d) SnCl2 to participate in bonding that:- [2017]
14. Dry ice is (a) Sn2+ is oxidising while Pb4+ is reducing
(b) Sn2+ and Pb2+ are both oxidising and reducing
(a) solid NH3 (b) solid SO2 (c) Sn4+ is reducing while Pb4+ is oxidising
(c) solid CO2 (d) solid N2 (d) Sn2+ is reducing while Pb4+ is oxidising
The p-block elements (Group 13 and 14) 255

Hints & Solutions


EXERCISE - 1
15. (c) H
1. (b) Alums are double salts. They have general formula |
B
M 2SO 4 .M '2 (SO 4 )3 .24H 2 O where M is monovalent

|
||
H N N H

|
|
cation and M ' is trivalent cation. | ||
\ Na 2SO4 .Al2 (SO4 )3 .24H 2O is alum. H B B H

|
N

||
|
2. D
(b) Na2B4O7. 10 H2O ¾¾¾¾® Na2B4O7 |
- 10H 2O H
Inorganic benzene, B 3N 3H6
D
Na 2 B4O7 ¾¾
® 2 NaBO2 + B2 O3
anhydrous sod.metaborate Boric anhydride It is isoelectronic with benzene.
water
CuO + B2 O3 ¾¾
® Cu(BO 2 )2 16. (d) K 2SO 4 .Al 2 (SO4 )3 .24H 2 O ¾¾¾®
cupric meta borate(Blue beed)
3. (c) Alum furnishes Al3+ ions which bring about coagulation 2K + + 4SO24 - + 2Al3+ + 24H 2O
of negatively charged clay particles, bacteria etc. 17. (d)
4. (d) The stability of +2 oxidation state increases and 18. (a) Gallium (Ga) is soft , silvery metal. Its melting point is
+4 oxidation state decreases as we move down in 30ºC. This metal expands by 3.1% when it solidifies
Group-14. So Pb(IV) is strongly oxidising and also I– is and hence, it should not be stored in glass or metal
strong reducing agent. Hence PbI4 is non existing. containers.
19. (b) Down the group basic character of oxides increases.
5. (a) The stability of dihalides (MX2) increases down the
B2O3 – Acidic
group. Except C and Si, the other members form
Al2O3 – Amphoteric
dihalides.
Ga2O3 – Amphoteric
6. (a) AlI3, on reaction with CCl4, gives the AlCl3
In2O3 – Basic
4AlI3 + 3CCl4 ¾¾ ® 4AlCl3 + 3CI 4 Tl2O3 – Basic
7. (c) AlCl3 exists as a dimer (Al2Cl6). It is a strong Lewis acid 20. (c) Element B Al Ga In
as it has an incomplete octet and has a tendency to m.p.(°C) 2076 660 30 157
gain electrons. AlCl3 undergoes hydrolysis easily and B have high m.p. due to giant polymeric structure. Ga
forms an acidic solution. has exceptionally low melting point due to unusual
AlCl3 + 3H2O ® Al(OH)3 + 3HCl structure consisting of only discrete Ga2 molecules.
Option (c) is true that AlCl3 sublimes at 180ºC. 21. (a) Borax is Na2B4O7.10H2O. It gives alkaline solution on
8. (d) H3BO3 acts as a Lewis acid and accepts OH– ions to dissolution in water as it is a salt of strong base and
form [B(OH)4]– weak acid.
9. (c) The approximate composition of cement is : Na2B4O7 + 7H2O ® 4H3BO3 + 2NaOH
22. (c) Due to the low density of aluminium it is useful for
Lime (CaO) = 50 – 60%
food packaging.
Silica (SiO2) = 20 – 25 % 23. (c) Aluminium reacts with caustic soda to form sodium
Alumina (Al2O3) = 5 – 10 % meta aluminate
Magnesia (MgO) = 2 – 3% 2Al + 2NaOH + 6H2O ¾¾ ® 2NaAlO2.2H2O + 3H2
Ferric oxide (Fe2O3) = 2 – 3% 24. (a) B2 O3 + 3C + 3Cl2 ® 2BCl3 + 3CO
10. (d) The order of strength of Lewis acid character for boron 25. (d) Ceramics are inorganic , non-metallic, solid minerals.
halides is, BF3 < BCl3 < BBr3 < BI3 (due to back bonding) They come in a variety of forms, including silicates
11. (d) Bell metal is an alloy of Cu and Sn. (silica, SiO2 with metal oxides), oxides (oxygen and
12. (b) All the layers have different densities and therefore metals), carbides (carbon and metals), aluminates
they remain separated from each other. (alumina, Al2O3 with metal oxides) and nitrides.
13. (b) Thermite is mixture of Fe2O3 + Al. The given ceramics are B4C (carbides), SiC (carbides),
14. (d) Thermite is a mixture of iron oxide and Al powder. Al Si3N4 (nitrides) and thus, none of these is an oxide . All
reduces iron oxide to iron giving out enormouns heat. of these are non-oxide ceramics.
Fe 2 O 3 + 2Al ¾¾ ® 2Fe + Al 2 O 3 + Heat 26. (d) 4Al + 3O2 ¾¾ ® 2Al2O3 DH = -ve
EBD_7327
256 CHEMISTRY

27. (b) In reaction with water, SiF4 (like SiCl4) gets hydrolysed 43. (b) Al2Cl6 + 12H2O 2[Al(H2O)6]3+ + 6 C1-
to form H4SiO4 (silicic acid).
44. (d) Anthracite is a coal of hard variety burning with little
SiF4 + 4H 2 O ¾¾ ® Si(OH) 4 or H 4SiO 4 + 4HF flame & smoke. Graphite layers are held together by
(Silicic acid)
weak van der waal forces so it is very soft. Lignite is
28. (d) 29. (a)
soft brown coal showing traces of plant structure,
30. (d) 2KOH + 2Al + 2H 2 O ® 2KAlO 2 + 3H 2
intermediate between coal & Peat. peat is vegetable
31. (a) KAl(SO4 )2 .12H 2O is a molecular formula of potash matter decomposed in water and partly carbonized.
alum. 45. (c) Boron (B), aluminium (Al) and gallium (Ga) are present
in group 13. They show + 3 oxidation state. While
32. (d) AlCl3 acts as a strong Lewis acid because AlCl3 is
cerium (Ce) is a lanthanoid. It presents in lanthanide
an electron deficient compound. Al has three electrons series. It shows + 3 and + 4 oxidation states.
in outer most electron. Through sharing it gets 3 46. (b)
electrons. Still it requires 2 electrons to complete its 47. (c) Ge is used as a semiconductor in transistors.
octet. 48. (c) Normal glass is calcium alkali silicate glass made by
33. (b) Antimony is a metalloid because it has some characters fusing the alkali metal carbonate, CaCO3 and SiO2.
of metals and some of non-metals. 49. (b) In diamond each carbon atom is sp3 hybridized and
34. (b) Cryolite Na3AlF6 is added thus forms covalent bonds with four other carbon atoms
(1) to decrease the melting temperature from 2323 K to lying at the corners of a regular tetrahedron.
1140 K
50. (a) H 2 O + C ¾¾ ® H 2 + CO
(2) to increase the electrical conductivity of solution
Steam Red hot coke water gas
H H H 51. (d) Activated charcoal is an allotropic form of carbon made
35. (d) B 197° B by heating or chemically oxidizing sawdust or peat. It
H H H has large surface area and acts as adsorbant.
B is sp3 hybridized 52. (d) In graphite, each carbon is sp2-hybridized and one
Only 12 bonding electrons available singly occupied unhybridized p-orbitals of C-atoms
BHB angle is 97° not 180°. overlap sidewise to form a p -electron cloud which is
36. (d) Action of concentrated nitric acid (HNO3) on metallic delocalized and thus the electrons are spread out
tin produces hydrated tin oxide (SnO2. 2H2O) which is between the structure.
also called meta stannic acid 53. (b) 6 HF + SiO 2 ® H 2SiF6 + 2 H 2 O
Sn + 4 HNO 3 ¾
¾® SnO 2 .H 2 O + 4 NO 2 + H 2 O 54. (d) PbO2 is a strong oxidising agent and is produced in
37. (d) (A) AlCl3 + 3H2O ––––® Al(OH)3 + 3HCl ­ (fumes) situ in lead storage batteries. The anode is oxidized to
(B) It exists as dimer in non-polar solvents like benzene PbO2 and cathode is reduced to spongy Pb.
and in vapour state below 400°C. 55. (a) Producer gas is a fuel gas and is mixture of CO and N2.
(C) 4Al2O3 + 3S2Cl2 + 9Cl2 ––––® 8AlCl3 + 6SO2 ­ 56. (b) Coal gas is a mixture of H2 + CO + N2 + CH4
So all options are correct. 57. (b) Carbonyl chloride (COCl2) is known as phosgene.
38. (d) It forms a stable cyclic complex with polyhydroxy 58. (d) The thermal stability of tetrahalides decreases in order
compounds like cis-glycerol, cathechol, etc. This helps CX4 > SiX4 > GeX4 > SnX4 and in terms of same metal
in the release of H+ as H3O+ and, therefore, boric acid with different halides is in order of
acts as a strong acid and hence can be titrated with MF4 > MCl4 > MBr4 > MI4.
sodium hydroxide using phenolphthalein. 59. (b) Because they react with water to form methane gas
39. (d) In the carbon family the stability of +2 oxidation state Al4C3 + 12H2O –––® 4 Al(OH)3 + 3CH4
increases with increase in atomic number due to inert Aluminium carbide Methane
pair effect.
Be 2C + 4H 2 O ¾¾
® 2Be(OH) 2 + CH 4
Therefore SnCl4 is more stable than SnCl2 & PbCl2 is Beryllium carbide Methane
more stable than PbCl4.
40. (c) H3BO3 on heating at 373K yields metaboric acid (HBO2) 60. (c) The m.p dcreases form B to Ga , hence gallium (Ga) has
least m.p. (303 K) among group of 13 element.
373K
H3BO3 ¾¾¾® HBO2 + H 2O 61. (a) Red lead is Pb3O4. It is a mixed oxide of Pb (II) and
D
metaboricacid Pb (IV). It acts as a powerful oxidising agent.
(orthorombic form)
41. (a) H3BO3 is a weak monobasic acid. D
62. (a ) SiCl4 ( s ) + 2H 2O( l) ¾¾¾¾
® Si(OH) 4 ( aq)
-4HCl
42. (d) Boron halides behave as Lewis acid because of their Silicic acid
electron deficient nature. Boron halide like BCl3 has
D
only six electrons around Boron, which is less than ¾¾
® SiO 2 .xH2O( s)
required for octat rule. So it has a tendency to attract Silica gel
electron from others. Thus it behaves as an acid. 63. (c) SiO2 + 2Mg ® Si + 2MgO.
The p-block elements (Group 13 and 14) 257

64. (c) Manganese salts are violet or purple coloured. So EXERCISE - 2


MnO2 gives violet colour to the glass. Fe3+ gives yellow
1. (d)
colour mixture of Fe3+ & Fe2+ gives green colour. CO2+
gives deep blue and colloidal particle of Cu gives ruby 2. (c) Al(OH)3 + NaOH ¾¾
® NaAlO2 + 2H 2O
Sodium meta-aluminate
-red colour.
3. (d) B4C is the hardest substance after diamond and BN.
65. (a)
4. (c) Moissan boron is amorphous boron, obtained by
66. (c) White lead is 2PbCO 3 .Pb ( OH ) 2 also known as basic reduction of B2O3 with Na or Mg. It has 95-98% boron
lead carbonate. and is black in colour.
67. (a) 3Pb + 8 HNO3 (dil) ¾¾
® 2NO + 3Pb(NO3)2 + 4H2O 5. (a) Abrasive is tough material used for sharpening and
nitric oxide grinding the uneven surfaces and silicon carbide is
Therefore, Pb reacts with dil HNO3 to produce NO. used for grinding glass
68. (b) In silica (SiO2); each Si atom is surrounded by four 6. (d) Boron form different hydride of general formula BnHn+4
and BnHn+6 but BH3 is unknown.
oxygen atom.
7. (b) CaF2 when added to fused Cryolite, lowers the m.p.
and increases the conductivity.
8. (a) Alumina Al2O3 is known as Ruby stone
— —

— —

— —

— —

— Si — O —Si — O — Si — O —Si —
9. (a) D , air
2Al + 6HCl ¾¾¾® 2AlCl3 + 3H 2
O O O O

2Al + 3Cl 2 ¾¾
® 2AlCl3
— Si — O —Si — O — Si — O —Si —

1000°C
O O O O Al 2O3 + 3C + 3Cl 2 ¾¾¾¾
® 2AlCl3 + 3CO
vapours
— —

— —

— —

— —

— Si — O —Si — O — Si — O —Si —
Cooling

Structure of SiO2 Anhydrous AlCl3


Only Si – O bonds exist and no Si = O. D
AlCl3 .6H 2O ¾¾
® Al(OH)3 + 3HCl + 3H 2O
69. (b) The inert pair effect is most prominent in the heavier
members of the group. Inert pair effect increases as we Thus AlCl3 cannot be obtained by this method
move the group down the group. 10. (d) It is pp - Pp back bonding involving B and F. The
70. (b) CO is reducing agent because it readily accepts oxygen smaller atoms show more back bonding.
to form CO2. CO is used as reducing agent to reduce 11. (c) Na 2CO3 + H 2O ® 2NaOH + CO 2
many metallic oxides into metals in metallurgy.
71. (b) Knocking (metallic sound or rattle) decreases the power 2Al + 2NaOH + 2H 2O ® 2NaAlO 2 + 3H 2 ­
and increases the wear of the engine. Knocking is more Sodium meta-aluminate
(soluble)
if petrol contains greater percentage of straight chain
hydrocarbon. It is least in case of alkene / branched 12. (c) B(OH)3 Þ H3BO3 Boric acid
Al(OH)3 Þ Amphoteric
chain HC/ aromatic HC. Tetra ethyl lead dissociates
13. (b) Annealing makes the glass soft.
into ethyl radicals which combine with some straight
14. (c) CO2 being more dense covers the igniting material more
chain HC and thus lowering the knocking property of
effectively than N2.
fuel by forming branched chain HC.
15. (d)
72. (c) Charcoal is a pure form of carbon, its reaction with hot
16. (b) It is the conversion of white tin to grey tin at low
conc. H2SO4 is as follows :
temperature which crumbles into powder.
C + 2H2SO4 ¾¾ ↑ 2H2O + 2SO2 + CO2 17. (d) Tin cry is the emission of sound on bending the tin.
73. (a) The larger the percentage of hydrogen present in a 18. (c) Pb with dil HCl forms protective coating of PbCl2
fuel, the higher will be its calorific value. Thus the 19. (c) Due to non-availability of vacant d-orbitals, it cannot
calorific values of coal gas, water gas and producer exceed its coordination number more than four. Thus
gas are of the order of 500, 350 and 103 BTH (British carbon never forms complexes e.g., [CCl6]2– deos not
Thermal Unit) per cubic foot respectively. exist but [SiCl6]2– exists.
1BTH = 252 calories. 20. (d) The bicarbonates of Ca and Mg are decomposed to Ca
74. (d) and Mg carbonates by boiling.
75. (c) SiCl4 + 2Mg ¾¾ ® Si + 2MgCl2 21. (b) Metal oxides or some salts are fused with glass to impart
colour.
(vapours) hot.
EBD_7327
258 CHEMISTRY

22. (b) The cubic form of BN with ZnS structure is as hard as 35. (a)
diamond. + - ˆˆˆ + 2-
CO 2 + H 2 O ‡ˆˆ
ˆˆˆ
† ˆ H + HCO3 ‡ˆˆ
ˆ H 2 CO 3 ‡ˆˆ
ˆˆˆ
† ˆ 2H + CO3
†
23. (a)
24. (d) It is hydrolysed with water to form a Si(OH)4. 36. (a) The stability of dihalides (MX2) increases down the
1800ºC
group. Except C and Si, the other members form
25. (d) Al2 O3 .2H 2 O + 3C + N 2 ¾¾¾¾
® dihalides.
2AlN + 3CO + 2H 2O 37. (d) CO 2 + Ca(OH) 2 ¾¾
® CaCO3 ¯+ H 2O
(A) white ppt
15.2°C 164°C 232°C
26. ˆˆˆˆˆ†
(b) a-tin ‡ˆˆˆˆ ˆˆˆˆˆ
†
ˆ b-Sn ‡ˆˆˆˆ ˆˆˆˆˆ
†
ˆ g -Sn ‡ˆˆˆˆˆ Liquid tin
(Grey) (White) (Britle) CaCO 3 + CO 2 + H 2 O ¾
¾® Ca ( HCO 3 ) 2
Calcium bicarbonat e
(most common and stable) (Rhombic) (Souble in water )
Although transitions of white to grey tin occurs at any Heat
temperature below 15.2°C, it becomes rapid only at Ca (HCO 3 ) 2 ¾¾¾® CaCO3 + CO 2 + H 2 O
( B)
–50°C, unless a catalyst is present. During the
conversion of white tin to grey tin (in cold countries) A and B are CO 2 and CO 2 .
volume increases. Grey Sn is very brittle easily crumbles 38. (b) Nitrolim is a mixture of CaCN 2 and graphite (C) which
down to powder. This phenomenon is called tin
diseases, tin past or tin plague. is used as fertilizer.
27. (a) R3SiCl on hydrolysis forms only a dimer. 39. (c) Hydrolysis of substituted chlorosilanes yield
R3SiOH + HOSiR3 ® R3Si – O – SiR3. corresponding silanols which undergo polymerisation.
28. (a) Silica on heating with carbon at elevated temperature, Cl H OH – 2HCl
CH3
gives carborundum (silicon carbide) Si +
D CH3 Cl H OH
SiO 2 + 3C ¾¾
® SiC + 2CO
carborundum
Carborundum is a very hard substance. CH3 OH
29. (b) In BF3, boron atom has vacant 2pz A.O. and fluorine Si
CH3 OH
atom has electron pair in 2p A.O. so, maximum
overlapping is possible between vacant 2pz A.O. of Dialkyl silandiol
boron and fully filled 2p A.O. of fluorine due to similar Polymerisation of dialkyl silandiol yields linear
size of A.O’s. thermoplastic polymer.
30. (c) Fused alumina (Al2O3) is a bad conductor of electricity.
CH3 CH3
Therefore, cryolite (Na 3AlF6) and fluorspar (CaF2) are
added to purified alumina which not only make alumina
HO — Si — OH + H O — Si — OH
a good conductor of electricity but also reduce the
melting point of the mixture to around 1140 K.
CH3 CH3
31. (b) The solution of aluminium chloride in water is acidic
due to hydrolysis. CH3 CH3
AlCl3 + 3H2O –––––® Al(OH)3 + 3HCl.
On heating it till dryness Al(OH)3 is converted into HO — Si — O — Si — OH
Al2O3
D Al O + 3H O CH3 CH3
Al(OH)3 ¾¾® 2 3 2
°
40. (b) p-p overlap between B and F is maximum due to identical
32. (c) Negative D r G value indicates that + 2 oxidation state size and energy of p-orbitals, so electron deficiency in
is more stable for Pb. Also it is supported by inert pair boron of BF3 is neutralized partially to the maximum
effect that + 2 oxidation state is more stable for Pb and extent by back donation. Hence BF3 is least acidic.
as DrG° value is positive in second reaction it indicates As the size of halogen atom increases from F to I, the
that + 4 oxidation state is more stable for Sn. extent of overlap between 2p-orbital of B and a bigger
i.e., Sn++ < Pb++, Sn4+ > Pb4+ p-orbital of halogen decreases. Therefore the electron
33. (b) PbO2 is a powerful oxidizing agent and liberate O2 when deficiency of B increases.
treated with acids.
2PbO 2 + 4HNO3 ¾¾ ® 2Pb(NO 3 ) 2 + 2H 2 O + O 2 ­ O– O–
34. (b) cis-1,2-diol forms chelated complex ion with the
41. (c)
product, [B(OH)4]– causing the reaction to proceed in Si Si

forward direction. O O O–
– O–
– –
CH2–OH HO OH HO–CH2 CH2–O O–CH2 O
+ B + B
CH2–OH HO OH HO–CH2 CH2–O O–CH2
Pyrosilicate [Si2O7]6–
Stable chelated complex ion
The p-block elements (Group 13 and 14) 259

42. (d) In SiO2 (quartz), each of O-atom is shared between two 49. (b)
SiO44– tetrahedra. 50. (c) Assertion is true because lower oxidation state becomes
more & more stable for heavier elements in p-block due
— —

— —

— —

— —
to inert pair effect. Hence Reason is false.
— Si — O —Si — O — Si — O —Si —
EXERCISE - 3
O O O O
Exemplar Questions


— Si — O —Si — O — Si — O —Si —
1. (c) Among these elements, in gallium, the crystal structure


O O O O is different which suggests that Ga consists of almost
discrete Ga2 molecule, so its melting point is lowest.
— —

— —

— —

— —
— Si — O —Si — O — Si — O —Si — Ga exists as liquid from 30°C upto 2000°C and hence, it
is used in high temperature measurement.
2. (a) Alkaline earth metals (Mg, Ca, Ba) form ionic chloride
Structure of SiO2 whereas aluminium form covalent chloride. Despite of
43. (d) In diborane (B2H6) structure there are four 2c-2e bonds sharing electrons with chlorine, the octet of aluminium
and two 3c–2e bonds (see structure of diborane). is incomplete. To complete the octet, it needs electrons
Structure of B2H6 : and thus, acts as a Lewis acid.
Hb 3. (a) Structure of B ( OH ) -4 is
Ht •• Ht OH
| s
B B HO B OH

|
|
Ht •• Ht OH
Hb 4 bond pair + 0 lone pair
44. (c) The correct formula of inorganic benzene is B3N3H6 so Hybridisation — sp3
(d) is incorrect statement Geometry —Tetrahedral
OH 4. (a) On moving down the group the acidic nature of oxides
| changes from acidic to basic through amphoteric
Boric acid (H3BO3 or B - OH ) is a lewis acid so (a) is
| B2 O 3 , AI2 O3 and Ga 2 O3 , In 2 O3 and TI2 O3
{ 14442444 3 144 42444 3
OH More acidic Amphoteric Basic
incorrect statement. less basic
The coordination number exhibited by beryllium is 4 5. (a) Among these elements boron has the lowest atomic
and not 6 so statement (b) is incorrect. number. It has atomic number 5 and do not have vacant
Both BeCl2 and AlCl3 exhibit bridged structures in solid d-orbital. Boron can show maximum coordination
state so (c) is correct statement. number of 4. The element M in the complex ion
Cl pm Cl Cl Cl Cl MF63- has a coordination number of six. Hence, B can
20
2 not form complex of the type MF63- .
Be 98° Be 263 pm Be Be 6. (c) Boric acid is a monobasic weak acid. It does not liberate
82° H+ ion but accepts electrons from OH– ion.
Cl Cl Cl Cl Cl
H3BO3+ H2O ® B ( OH ) -4 + H+
Cl Cl Cl
7. (b) The tendency of forming long open or closed chains
Al Al by combination of some atoms in themselves is known
as catenation. The tendency of catenation is maximum
Cl Cl Cl in carbon and decreases on moving down the group.
45. (b) Two dimensional sheet structures of the formula C > > Si > Ge » Sn > Pb
-
(SiO5 ) 2n
n are formed when tetrahedral are shared. This is due to high bond energy of C — C bonds. On
46. (c) moving down the group, atomic size increases and
47. (c) Boron is metalloid. Thus assertion is correct. Metalloids electronegativity decreases, hence, tendency to show
possess, metallic as well as non-metallic nature. Hence, catenation decreases.
reason is false. 8. (c) Silicon has a strong tendency to form polymers like
48. (d) Borazine has more polarity as compared to benzene silicones. The chain length of silicon polymer can be
because nitrogen is more and boron is less
controlled by adding (CH3)3SiCl which block the ends
electronegative than carbon. Thus due to polarity
as shown below
inorganic benzene, borazine is highly reactive while
benzene is much less reactive.
EBD_7327
260 CHEMISTRY

CH3 CH3 (ii) Diborane can be prepared by the reaction of BF3


| | with lithium aluminium hydride in diethyl ether.
n HO — Si — OH + Cl — Si — CH 3
| | 4BF3 + 3LiAIH4 ® 2B2H6 + 3AIF3 + 3LiF
CH3 CH3 12. (b) Quartz, is a crystalline forms of silica. Quartz is
–HCl Polymerisation extensively used as a piezoelectric material.
13. (d) Sn and Pb can show two oxidation states i.e. +2 and
CH3 CH3 +4. These elements in +4 oxidation state behave as
| | oxidising agent, where as in +2 oxidation state behave
— O — Si — O — Si — CH3 as reducing agent. The stability of +2 oxidation state
| | increases on moving down in a group i.e., reducing
CH3 CH3
character decreases from Ge (I) to Pb (II). i.e.,
Silicone
Germanium salts are strongest reducing agent and
9. (d) On moving down a group the ionization enthalpies
Pb(II) salt the weakest. Ge is much less abundant than
generally decreases due to increase in atomic size and
tin in nature. hence Sn (II) salts are widely used as
screening effect which is more than to compensate the
effect of increase in nuclear charge. Consequently the reducing agent.
electron becomes less and less tightly held by the 14. (c) Carbon dioxide can be obtained as a solid in the form
nucleus as we move down the group. The sharp of dry ice allowing the liquified CO2 to expand rapidly.
decrease in ionization enthalpy from B to Al is due to It is used to maintain the low temperature in laboratory.
increase in size. In case of Ga, there are 10 d-electrons Dry ice is solid CO2
in its inner electronic configuration. Since the 15. (b) Cement is obtained by combining a material rich in lime
d-electrons shield the nuclear charge less effectively (CaO) with other material like silica SiO2 alongwith the
than the s & p electrons, the outer electron are held oxides of aluminium, iron and magnesium. The average
strongly by the nucleus. As a result the ionization composition of cement is
enthalpy increases slightly inspite of the increase in CaO (50 – 60%)
atomic size as we move from Al to Ga. The similar
SiO2 (20 – 25%)
increase is also observed from In to Tl, which is due to
Al2O3 (5 – 10%)
presence of 14f electrons in inner electronic
configuration of Tl which has poor shielding effect. Fe2O3 (1 – 2%)
10. (b) In diborane, there are 12 valence electrons, three from SO2 (1 – 2%)
each boron atoms and six from the six hydrogen atoms. MgO (2 – 3)%
The structure of B2H6 can be represented as Thus, it contains elements of group 2(Ca), group
H H H 13 (Al) and group 14 (Si).

NEET/AIPMT (2013-2017) Questions


B B
16. (d) Boron nitride (BN) is known as inorganic graphite. The
most stable form is hexagonal one. It has layered
H H H structure similar to graphite.
Bonding in diborane 17. (b) Since Me3SiCl contains only one Cl, therefore it can’t
The four terminal hydrogen atoms and two boron atoms
form high molecular mass silicon polymer. It can form
lie in the same plane. These four terminal B-H bonds
only dimer.
are regular 3C-2e bonds. Above and below the plane,
there are two bridging hydrogen atoms. Each boron 18. (a) SiO44– is basic structural unit of silicates.
atom forms four bonds even though it has only three 19. (a) Feldspars are 3-dimensional alumino-silicates.
electrons. The terminal B — H bonds are regular bonds 20. (b) Lower oxidation state become more stable on moving
but the bridge B — H bonds are different. down the group
Each bridge hydrogen is bonded to the two boron atoms
Al < Ga < ln < Tl
only by sharing of two electrons. Such covalent bond
is called banana bond. 21. (d) Inertness of ns2 electrons of the valence shell to
11. (a) (i) Reaction of ammonia with diborane gives participate in bonding on moving down the group in
initially B2 H6 .2NH3 which is represented as heavier p-block elements is called inert pair effect.
[BH2 (NH3 )2]+[BH4]– which on further heating As a result, Pb(II) is more stable than Pb(IV)
gives borazine, B3N3H6 also called borazole. Sn(IV) is more stable than Sn(II)
473 K \ Pb(IV) is easily reduced to Pb(II) and can acts as an
3B2 H 6 + 6NH3 ¾¾¾® 2B3 N 3 H 6 + 12H 2
Diborane Barozole oxidising agent whereas Sn(II) is easily oxidised to
(X) (Y)
Sn(IV) and can acts as a reducing agent.
inorganic benzene
Organic Chemistry :

12 Some Basic Principles and


Techniques
TETRAVALENCY AND SHAPES OF ORGANIC sp, sp2 or sp3 hybridised.
MOLECULES Hybridisation influences the bond length and bond strength in
The tetravalence, formation of covalent bond and catenation organic compounds. The sp hybrid orbital contains more
property of carbon results in the formation of a large number of s-character and hence, it is closer to its nucleus and forms shorter
compounds of carbon. These compounds are covalent in nature and stronger bonds than sp3 hybrid orbital.
and are studied under a branch of chemistry called organic The bond angles and geometry associated with the three types of
chemistry. Carbon atoms present in organic compounds are either hybridization are summarized below :

3
Hybridisation sp sp2 sp

Angle 109º28' 120º 180º


Geometry Tetrahedral Trigonal Linear
Example Alkane, Cycloalkane Alkenes and other compounds Alkynes and all other
and in saturated part containing C=C, C=O, compounds containing
of all organic molecules C=N and C=S double bonds C C and C N triple bonds,

Bond Four- Three-s Two-s


One- Two-

s% 25 33.3 50

p% 75 66.7 50
Electronegativity 2.48 2.75 3.25

C(sp) – H > C(sp2)–H > C(sp3)–H For example few compounds are represented in all three
121 kcal mol –1 106 kcal mol –1 98.6 kcal mol–1 formulas below :
Ex:
C(sp) – C(sp) > C(sp ) – C(sp ) > C(sp3) – C(sp3)
2 2
(a) Butane (C4H10)
200 kcal mol–1 142 kcal mol–1 80-85 kcal mol–1 H H H H CH3CH2CH2CH3
Note : (i) Greater is the s - character of the hybrid orbitals, greater
Condensed
is the electronegativity. Thus, a C - atom having an sp hybrid H C C C C H
orbital with 50% s - character is more electronegative than sp2
(33% s-character) or sp3 (25% s-character) hybrid orbitals. H H H H Bond line
(ii) Since a bigger orbital forms a longer bond, therefore, C–C
single bond length decreases in the order: Complete
(b) 2 - Bromobutane
C(sp3) – C(sp3) > C (sp2) – C(sp2) > C(sp) – C(sp)
H H H H CH3CHBrCH2CH3
1.54 Å 1.34Å 1.20 Å
Condensed
STRUCTURAL REPRESENTATIONS OF ORGANIC H C C C C H
COMPOUNDS
H Br H H
(i) Complete, Condensed and Bond - line
Structural Formulas : Complete Br
Bond - line
EBD_7327
262 CHEMISTRY

(c) 1- Bromocyclopentane (ii) 3D Representation


H
| By using solid ( ) and dashed ( ) wedge formula, 3-D
C CH2 image of a molecule can be shown from 2D picture. The dashed
H – C – H | H – C – Br line depicts the bond projecting out of the plane of paper and
H CH2 CHBr away from the observer while solid-wedge depicts the bond
projecting out of the plane of paper and towards the observer.
H–C C–H The bonds lying in plane of paper are depicted by using a
| | CH2 CH2
normal line (–).
H H Condensed
Ex: Ethane
Complete
Bonds away Bonds in the
Br from observer C C plane of paper
Bonds toward
observer
Bond line
CLASSIFICATION OF ORGANIC COMPOUNDS
Organic Compounds

Open chain Closed chain


or or
Acyclic Cyclic
or or
Aliphatic compounds Ring compounds

Unsaturated Homocyclic Heterocyclic


Saturated e.g. Benzene e.g. pyrrole
e.g. Alkanes e.g. Alkenes & Alkynes

Alicyclic Aromatic

Saturated Unsaturated Benzenoids Nonbenzenoids


e.g. cyclopropane e.g. cyclopropene

NOMENCLATURE OF ORGANIC COMPOUNDS


CH3 CH2 CH CH2 CH CH3
Nomenclature of Alkanes
CH3 CH CH3
(i) Straight chain hydrocarbons:
CH3
The names of such compounds are based on their chain
structure, and end with suffix ‘-ane’ and carry a prefix 2 side chains (wrong)
indicating the number of C - atoms present in the chain. The CH3 CH2 - CH - CH2 - CH - CH3
alkanes differ from each other by no. of – CH2 groups in the l l
CH3 - CH CH3
chain. l
Some names are: CH3
Methane CH4 Butane C4H10 3-side chains (correct)
Ethane C2H6 Pentane C5H12 1 2 3 4 5 6 7 8 9
Propane C3H8 Hexane C6H14 CH3 — CH — CH2 — CH2CH2CH — CH2CH2 CH3 P Correct
(ii) Branched chain hydrocarbons: The rules for naming are: CH3 CH2CH3
(a) The longest C - chain in the molecule is identified. Out
1 2 3 4 5 6
of all the possible chains. If there are the possibilities of CH3 — CH CH2CH2CH2CH CH2CH2CH3 Incorrect
more than one longest chain the chain which possesses
more than one side chain is selected. CH3 CH2CH3
7 8
Organic Chemistry : Some Basic Principles and Techniques 263

(b) The numbering is done in such a way that branched carbon (2) If side chains are different numbering is given in
atoms get the lowest possible numbers. alphabetical preference order.
1 2 3 4 5 6 7 8 9 Ex.
C—C—C—C—C—C—C—C— C Correct P 7 6 5 4
H3C - CH2 - CH - CH2 - CH - CH2 - CH3 (Wrong)
3 2 1

C C—C l l
E H2C CH3 M
9 8 7 6 5 4 3 2 1 l
C—C—C—C—C—C— C—C— C Incorrect H3C
1 2 3 4 5 6 7
C C—C CH3 CH2 CH CH2 CH CH2 CH3 (Right)
In case, the parent chain has two or more substituents, numbering CH2 CH3 M
must be done in such a way that the sum of the locants on the E
parent chain is the lowest possible. Thus in following structures CH3
A and B numbering may be done in two ways : in one (A), the sum (f) Naming the complex substituent : In case the substituent
of locants is 9 while in other (B), it is 12, hence the former is correct
on the parent chain is complex (i.e., it has itself branched chain)
while the latter is wrong.
then it is named as a substituted alkyl group and its carbon chain
C C C C C C is numbered from the carbon atom attached to the main chain. The
| | | | | |
C- C- C - C- C - C C- C - C- C - C- C name of this complex substituent is written in bracket to avoid
1 2 3 4 5 6 6 5 4 3 2 1 confusion with the numbers of the main chain. e.g.,
Sum of locants : 2 + 3 + 4 = 9 (A) Sum of locants : 3 + 4 + 5 = 12 (B)
(correct) (incorrect) 3¢ CH3
(c) The names of alkyl groups attached as a branch are prefixed |
to the name of parent alkane and position of substituents is 2¢ CH - CH3
|
indicated by appropriate numbers. If different alkyl groups 1¢ CH - CH3
are present they are listed in alphabetical order. For example 1 2 3 4 5| 6 7 8 9
CH3 - CH - CH - CH 2 - CH3 CH3 - CH 2 - CH 2 - CH 2 - CH - CH 2 - CH 2 - CH 2 - CH3
| | 5- (1¢, 2¢ - dimethylpropyl) - nonane
CH3 C2H5 Nomenclature of Cyclic Compounds
3–Ethyl–2 methylpentane and not as
2 - Methyl -3- ethylpentane A saturated monocyclic compound is named by prefixing ‘cyclo’
(d) If two or more identical substituent groups are present, then to the corresponding straight chain alkane. If side chains are given,
numbers are separated by commas. The names of identical then rules given above are applied.
substituents are indicated with prefixes such as di, tri, tetra, For example : Cyclobutane
penta, hexa, etc. While writing names of substituents spelling
of these prefixes are not considered.
For example : 12
3 1 - methyl - 3 - propylcyclo - hexane
CH3
6 5 4| 3 2 1
CH3 - CH 2 - CH - CH 2 - CH - CH3 Nomenclature of Organic Compounds Containing
| | Functional Groups
C2 H 5 CH3
4 - Ethyl - 2, 4 - Dimethylhexane
(i) Numbering is given from the end from which functional group
is closest.
CH3 C2H 5 (ii) If the functional group is situated at same carbon from both
1 2 3| 4| 5 6 7 sides, follow the rules of saturated hydrocarbons.
CH 3 - CH 2 - C - CH - CH 2 - CH 2 - CH3 1 2 3 4 5
| Ex. C C C C C ( Wrong )
CH3
4 - Ethyl -3, 3- dimethylheptane C OH
5 4 3 2 1
(e) If side chains are at same carbon from both sides, C C C C C (Right)
following rules are adopted :
C OH
(1) For same side chains numbering may be given from any 7 6 5 4 3 2 1
end. C C C C C C C (Wrong)
1 2 3 4 5 C OH C
C- C- C- C- C
| | ( Right ) C
C C 1 2 3 4 5 6 7

5 4 3 2 1
C C C C C C C (Right)
C - C - C - C - C (Right) C OHC
| |
C C C
EBD_7327
264 CHEMISTRY

(iii) If the organic compound contains two or more functional (iv) — R, — C6H5, halogens (F, Cl, Br, I), –NO2, – OR etc. are
groups, one of the group is selected as principal functional always prefix substituents.
group containing longest possible chain of carbon atom (v) If more than one functional group of same type is present,
and all the remaining functional groups are treated as their number is indicated by adding di -, tri, etc. before the
substituents. class suffix.
Order of preferance used while selecting the principal Examples :
functional group (a) CH3CH2COCH2COCH3
Carboxylic acids > Sulphonic acids > anhydrides > esters Hexane - 2, 4 - dione
> acid chlorides > acid amides > nitriles > aldehydes > (b) CH º C - CH = CH - CH = CH 2
cyanides > isocyanides > ketones > alcohols > phenols > Hexa - 1, 3 - dien - 5 - yne
thiols > amines > ethers > alkenes > alkynes. See the following table for IUPAC names of certain organic
For example : compounds
CH3 – CH – CH2 – CH – CH3 CH3
| | |
NH2 OH CH3 – C – COCH3
4-Aminopentan–2–ol |
OH
–hydroxy –3–methyl
butan –2–one

Class of Functional group Functional Functional


compounds structure group prefix group suffix
Alkanes – – -ane
Alkenes >C=C< – -ene
Alkynes – C º C– – -yne
Arenes – – -
Halides –X halo -
(X = F, Cl, Br, I)
Alcohols –OH hydroxy -ol
Aldehydes –CHO formyl, -al
or oxo
Ketones >C=O oxo -one
Nitriles - CN º cyano nitrile
Ethers R-O-R alkoxy -
Carboxylic –COOH carboxy -oic acid
acids
Carboxylate – -oate
- COO-
ions
Esters –COOR alkoxycarbonyl -oate
Acyl halides –COX halocarbonyl -oyl halide
(X = F, Cl, Br, I)
Amines - NH2 , > NH, N - amino -amine
Amides - CONH,2 carbamoyl -amide
- CONHR,
- CONR 2
Nitro - NO2 nitro -
compounds
Sulphonic - SO3 H sulpho sulphonic acid
acids
Thio alcohol — SH Mercapto – – thiol

Anhydride — CO — – oic anhydride


O
— CO
Isocyanide —N C Carbylamine – iso nitrile
Organic Chemistry : Some Basic Principles and Techniques 265

O O O O (iii) For tri - or higher substituted benzene derivatives, the


compounds are named by identifying substituent positions
Note : C H, C OH, C Cl, C NH2 & – C º N on the ring by following lowest locant rule. In some cases,
are the groups of which carbons are also included in the common name of benzene derivatives is taken as base
longest chain. If these are alone in a compounds they comes compound. Substituent of base compound is assigned no. 1
on first carbon, if double, comes on first and last positions. and then the direction of numbering is chosen such that the
Nomenclature of Aromatic Compounds next substituent gets the lowest number. The substituents
(i) The substituent is placed as prefix to the word benzene. should be numbered according to alphabetical order.
Examples : ex:
Cl
CH3 OMe NH2 1 - Chloro - 2, 4 - dinitrobenzene

ON2 NO2

Methyl benzene Methoxy benzene Amino benzene NO2


(Toluene) (Anisole) (Aniline)
2 - Chloro - 1 - methyl - 4 - nitrobenzene
(ii) If benzene ring is disubstituted, position of substituents is
defined by numbering the C-atoms of ring such that
substituents are located at lowest numbers possible. Cl
Example : CH3

OMe
CH3 CH3 1
CH3 1 1 Cl
1 CH3 2 2 - Chloro - 4 - methyl anisole
2 3
3 CH3 4 4
1,2-Dimethylbenzene 1,3-Dimethylbenzene CH3 CH3
(o-Xylene) (m-xylene) 1,4-Dimethylbenzene
(p-xylene) (iv) When benzene ring is attached to an alkane with a functional
group, it is considered as substituent instead of parent and
its name as a substituent is phenyl

ISOMERISM
The phenomenon of existence of two or more compounds possessing same molecular formula but different properties is called
isomerism and the compounds are called isomers. The following chart shows different types of isomerism.
Types of Isomerism

Structural isomerism Stereoisomerism

Conformational Configurational
Chain Position Functional Metamerism
Tautomerism isomerism Isomerism
isomerism isomerism isomerism

Geometrical Optical
Structural Isomerism: It arises when compounds have same molecular formula but different structures. It arises because of the
difference in the sequence of covalently bonded atoms in the molecule without reference to space.
EBD_7327
266 CHEMISTRY

Types of Structural Isomerism (a) Molecular formula : C2H6O


(i) Chain isomerism : Two or more compounds with same CH3 – CH2 – OH and CH3 – O – CH3 : Functional isomers.
molecular formula but different carbon skeletons are called Ethyl alcohol Dimethyl ether
chain isomers and the phenomena is known as chain (Alcohol) (Ether)
isomerism.
(b) Molecular formula : C3H6O
Examples :
Pentane : C5H12 O O
|| ||
CH3CH2CH2CH2CH3 CH3CHCH 2CH3 CH3 – CH2 – C – H and CH3 – C – CH3 Functional isomers
|
CH3 Propanal Propanone
(Aldehyde) (Ketone)
n-Pentane Isopentane (c) Molecular formula : C3H6O2
CH3
| O
CH3 - C - CH3 ||
| CH3 – CH2 – COOH and CH3 – C – O – CH3
CH3 :Functional isomers
Neopentane Propanoic acid Methyl acetate
n-Pentane, isopentane and neopentane possess the chain of (Acid) (Ester)
five, four and three carbons, respectively. Hence they are (iv) Metamerism :This type of isomerism is due to unequal
chain isomers. distribution of substituents on either side of the polyvalent
Butyl alcohol : C4H9OH functional group.
CH3 – CH2 – CH2 – CH2OH CH3 - CH - CH 2 OH Members belong to the same homologous series.
| Ex.
CH3 (a) Diethyl ether and methyl propyl ether
n-Butyl alcohol Isobutyl alcohol CH3CH2OCH2CH3 CH3OCH2CH2CH3
These two butyl alcohols are chain isomers. Diethyl ether Methyl propyl ether
(ii) Position isomerism : It is shown by the compounds in which (b) Diethyl amine and methyl propyl amine
there is difference in the position of attachment of functional CH3CH2 – NH – CH2CH3 CH3CH2CH2 – NH – CH3
group, multiple bond or substituent along the same chain Diethyl amine Methyl propyl amine
length of carbon atoms. (v) Tautomerism :Tautomerism is a special type of functional
group isomerism which arises due to the migration of H-atom
Examples : CH3 – CH2 – CH2Cl & CH3 – CH – CH3
as proton from a polyvalent atom to other polyvalent atom
|
Cl with reshuffling of p bond.
Keto-Enol Tautomerism :
1-Chloropropane 2-Chloropropane
When the tautomers exist in the two forms keto & enol then, such
Molecular formula : C4H8
CH3 – CH2 – CH = CH2 and CH3 – CH = CH – CH3 : type of tautomerism is called keto-enol tautomerism.
1-Butene 2-Butene Keto means the compound has a keto group > C = O, and the enol
Note : In the disubstituted benzene derivatives also position form has both double bond and OH (hydroxy) group joined to the
isomerism exists because of the relative position occupied same carbon.
by the substituents on the benzene ring. Thus, H O OH
Chlorotoulene, C6H4(CH3)Cl exists in three isomeric forms - | || |
ortho, meta and para. –C–C– Û –C=C–
| |
CH3 CH3 CH3
Keto form Enol form
Cl
O H
P |
(a) CH3 - C - H Û CH 2 = C – OH
Cl
Cl Acetaldehyde Vinyl alcohol
o-Chlorotoluene m-Chlorotoluene p-Chlorotoluene O H OH
(iii) Functional group isomerism : When two or more compounds || | |
have same molecular formula but different functional groups. (b) CH3 – C – CH – COCH3 Û CH3 – C = CH – COCH3
Such compounds are called functional group isomers
(Keto) (Enol)
Organic Chemistry : Some Basic Principles and Techniques 267

(i) Geometrical isomerism : Arises due to different geometries


of compounds, i.e., different arrangement of atoms or groups
around doubly bonded C - atoms.
Two types:

Stereoisomerism
Two or more compounds which
have the same molecular formula Cis-isomer Trans-isomer
and the same connectivity of atoms
but different three dimensional Two identical atoms or Two identical atoms or
arrangement of their constituent groups lie on same side groups lie on opposite
atoms or groups are said to be of double bond i.e. side of double bond, i,e.
stereoisomers and the phenomenon
is termed as stereoisomerism. X X Y
X
C=C C=C
Y Y X X
If the two atoms or groups linked to same doubly linked carbon are similar
as in the molecule aaC = Cab, the compound does not show geometrical
isomerism.
(ii) Optical isomerism : It is shown by compounds which rotates the
plane of polarised light either towards right i.e clockwise or towards left,
i.e., anti - clockwise.
Optical activity : It is the ability of a substance to rotate the plane
of polarisation of plane polarised light. C2H5 C2H5 C2H5
(a) One rotating the plane of polarised light to the left. This form
Y H Y H
is named as laevorotatory. (Latin, laevous = left) or H
direction (–) form X
Y Y
(b) One rotating the plane of polarized light exactly to the same
extent but to the right is named dextrorotatory (Latin Dexter – CH3 CH3 CH3
right) or direction (+) – form. A
B Y
(c) An inactive form which does not rotate the plane of polarized
light at all. This is a mixture of equal amounts of (+) and (–) – A+ B
forms and hence it is optically inactive. It is named (±) - mixture if only (A) is obtained, process is called retention of configuration.
If only (B) is obtained, process is called inversion of configuration.
or Racemic mixture. (Latin, Racemic - mixture of equal If 50:50 mixture of above two is obtained then the process is called
compounds) Asymmetric carbon atom : A carbon atom racemisation and the product is optically inactive.
attached to four different atoms and groups is called
asymmetric carbon atom. e.g. CH3*CHOHCOOH. REACTION MECHANISM
Chirality : If the mirror image of the molecule is different from the A general reaction is depicted as:
molecule it is said to be a chiral molecule. In such case if one Attacking [Intermediate] Product (s)
configuration of the molecule is placed above its mirror image reagent
configuration, the similar atoms and groups do not fall over each By product(s)
other and the configurations are said to be nonsuperimposable. A sequential account of each step, describing details of electron
Chirality is the necessary condition for a compound to be optically movement, energetics during bond cleavage and bond formation,
active. and the rates of transformation of reactants into products is
Enantiomers : Pairs of nonsuperimposable mirror images are called referred to as reaction mechanism.
enantiomers. Enantiomers are non superimposable mirror Fission of a Covalent Bond
image isomers. They have identical physical properties (bpt, mpt, A covalent bond can be cleaved in two ways:
density etc.) except for their rotation of plane polarised light. They (i) Homolytic cleavage : On bond breaking, one of the electrons
are much more difficult to separate. In nature very often only one of shared pair in covalent bond goes with each of the bonded
enantiomer is produced. Living organisms are one of the best atoms.
sources of optically active compounds (plants, enzymes, animals, Homolytic fission
A+ B
A B ¾¾¾¾¾¾¾ ®
cells etc.). Free radicals
Diastereo Isomers : Stereo isomers which are not mirror images of It results in the formation of free radicals which contain an
each other are called diastereo isomers. They have different odd or unpaired electron. Such reactions are called free radical
physical properties (mpt, bpt, solubility) and are often easy to or homopolar or non-polar reactions.
separate by distillation, recrystallisation, chromatography etc.). Favourable conditions :
Racemiemixture : A mixture containing equal amounts of two (a) High temperature (b) Light of suitable wavelength
enantiomers will have zero optical rotation and is called racemic (c) Non polar solvent (d) Presence of peroxide or oxygen
mixture (e) Ability of substrate and attacking reagent to produce free-
radicals.
EBD_7327
268 CHEMISTRY

(ii) Heterolytic cleavage : When a covalent bond joining two +


atoms A and B breaks in such a way that both the electrons e.g. - N R 3 > – NO2 > – CN > – COOH > –F
of the covalent bond (i.e., shared pair) are taken away by one
of the bonded atoms, the mode of bond cleavage is called > – Cl > – Br > – I > – O && H > – O
&& R > –C H > – H
6 5
heterolytic fission or heterolysis. (b) +I Effect : Those atoms or group of atoms which attract the
shared electron pair (bond pair) less strongly than hydrogen
Heterolytic fission + –
A : B ¾¾¾¾¾¾¾ ® A +:B atom are said to have +I (electron repelling) effect. Such
(when B is more electronegative than A) groups when attached with a carbon chain displace the shared
It results in formation of cations and anions s electrons towards the chain and make it electron rich.
Favourable conditions : (a) Low temperature (b) Polar solvent (c) (CH3)3C – > (CH3)2CH – > CH3 – CH2 – > CH3 –
Presence of acid or base catalyst (d) Polar nature of the substrate (ii) Resonance effect : It is defined as the polarity produced in a
and attacking reagent. molecule by the interaction of two p - bonds or between a p -
Nucleophiles and Electrophiles bond and lone pair of e–1 s present on an adjacent atom. It is
A reagent that brings an electron pair is called a nucleophile i.e. also called mesomeric effect. It is a permanent effect.
nucleus seeking and the reaction is called nucleophilic. Two types of resonance effects:
They are of two types : (a) Positive resonance effect (+ R effect)
(a) Charged Nucleophiles : In this effect, transfer of electrons is away from an atom group
All the negative ions qualify as nucleophiles. attached to the conjugated system. This effect results into
F–, Cl–, Br–, I–, OH–, CN–, RCOO– increase in electron density at certain positions in molecule.
– – Example : Phenol
RO–, R–, R—C º C–, NH 2 , SH , etc.
H H
(b) Neutral Nucleophiles : Central atom has electron pair. O O
+

&& , R — NH &&
&& , R – NHR && , H O
, R 3N && &&
NH 3 2 2 :,R– O . . – H,
A reagent that takes away an electron pair is called an electrophile
(E+) i.e., electron seeking and the reaction is called electrophilic
Phenol
Electrophiles may be neutral or positively charged.
(a) Neutral electrophiles : Central atom has deficiency of
H
electrons. ex. BF3, BCl3, AlCl3, BeCl2, FeCl3, SO3, etc.. +
O
+
O
H H
O
(b) Charged electrophiles :
All the positive ions behave like electrophiles.
+
Cl+, Br+, I+, NO2+, NO+, H+, H3O+, N H 4 , R+,
+
R — C = O , etc.
Examples : halogen, – OH, – OR, – OCOR, – NH2, – NHR,
Electrophiles are generated by heterolysis of a covalent bond. – NR2, – NHR, – NR2, – NHCOR
Transition metal cations are electrophiles Decreasing order of activating (+R) groups
For example : Fe3+, Fe2+, Ag+, Hg2+, Cd2+, etc. –O– , –NH2 , –NHR , –NR2 , –OH , –OR , –NHCOR , –R ,
Electron Displacement –Ar , –X (Till –OH they are very strong activating groups)
The electron displacement in an organic molecule may take place (b) Negative resonance effect (– R effect)
either in ground state under the influence of an atom or a group or In this effect, transfer of electrons occurs towards atom or
in presence of an appropriate attacking reagent.
group attached to the conjugated system.
Electron displacement due to influence of an atom or group present
in molecule cause permanent polarisation of the bond. Example: Example : Nitrobenzene
– –
Inductive effect and resonance effect. O O O
O
Temporary electron displacement effects are seen in a molecule N N
when a reagent approaches attack it.
Example: electromeric or polarisability effect.
(i) Inductive Effect : The polarisation of s - bond caused by
polarisation of adjacent s - bond is called inductive effect. It – –
O O O O
is a permanent effect. The effect decreases as the number of
N N
intervening bonds increases and becomes very small after
three C - atoms. Inductive effect (I–effect) is of two types.
(a) –I effect : Atoms or group of atoms which attract the bonded O
electrons more strongly than hydrogen atom, are said to have N O
–I effect and are termed as electron attracting (–I groups).
Such groups when linked with a carbon chain make it electron
poor.
Organic Chemistry : Some Basic Principles and Techniques 269

Effects of hyperconjugation :
– (1) Stability of alkenes :
Examples : –COOH, – CHO, C = O, – CN, NO2
(a) Alkylated alkenes are more stable than others.

R - C = C - R > R 2C = CHR.
The very strong deactivating (–R) groups are | |
–NO2 , –CF3 , –NR3+ R R
Conditions for Resonance : [Tetraalkyl ethylene]
(1) The atomic arrangement is the same in all the canonical forms.
(2) Same number of paired or unpaired electrons must be present RCH = CHR > R2C = CH2 > R – CH = CH2 > CH2 = CH2
in each canonical form. Least stable
(3) The canonical forms must possess same or nearly same (b) Greater the number of a C – H bonds in an alkene, greater
energy. will be the number of hyperconjugated structures, hence
(4) The molecule must have a planar structure. greater will be the stability.
(iii) Electromeric effect (E - Effect) : It is defined as the complete (2) Bond lengths : Hyperconjugation in a molecule results in a
transfer of a shared pair of p - electrons to one of the atoms change in bond lengths.
(more electronegative) joined by a multiple bond on demand (3) Stability of alkyl free radicals :
of an attacking reagent. It is a temporary effect. t–alkyl > sec. alkyl > primary alkyl > CH3 (methyl radical)
Two types of electromeric effect: Greater the number of hyperconjugative structures of an alkyl
(a) + E effect radical, higher is the stability.
In this, p – electrons of multiple bond are transferred to that (4) Stability of alkyl carbonium ions : Greater the number of
atom to which reagent gets attached. alkyl groups attached to positively charged carbon atom,
For example : greater is the hyperconjugation interaction and stabilisation
of the cation.
CH3 CH CH2 H CH3 CH CH3 The relative stability of carbocations follows the order:
Propene Isopropyl carbocation CH3
(b) – E effect | + + +
CH3 — C + > ( CH3 ) 2C H > CH3C H 2 > C H3
In this, p – electrons of multiple bond are transferred to that |
atom to which attacking reagent does not get attached CH3
For example :
INTERMEDIATES
H O–
– H
O CN C Free Radicals
H H CN
| |
Homolytic fission
Formaldehyde Anion
Anion -C - X ¾¾¾¾¾¾¾¾ ® – C• + X•
Oxygen being more electronegative pulls p electrons of | |
multiple bond toward itself. Carbonyl C aquires positive If EN of C – ~ EN of X
charge thus got attacked by nucleophile. It is an atom or group of atoms possessing an odd or unpaired
(iv) Hyperconjugation : It involves delocalisation of s – electrons electron. It is electrically neutral and shows paramagnetism.
of C – H bond of an alkyl group directly attached to an atom Stability of free radicals is due to following reasons :
of unsaturated system or to an atom with an unshared p - (i) Resonance (ii) Hyperconjugation
orbital. It is a permanent effect. Hyperconjugation is also
Order of stability of free radicals :
called no -bond resonance.
t-butyl > isopropyl > ethyl > methyl
In propylene, there are three H—C sigma bonds in Stability on the basis of resonance :
conjugation with the p bond. So, only three resonating forms
Stability µ Number of resonating structures
can be written.
The order is :
H (f)3C· > (f)2CH· > f CH2· > CH2 = CH – CH2·
H+ [where f = C6H5]
H РC РCH = CH2 ¨ H РC = CH РCH2
|
Carbonium ion or Carbocation – C
H H
Propylene contains |
3H – C hyperconjugated bond
| |
Heterolytic fission
H H - C - X ¾¾¾¾¾¾¾¾ ® - CÅ + : X Q
| |
¨ H+ C = CH РCH2 ¨ H РC = CH РCH2

H
H+ If EN of X > EN of C
EBD_7327
270 CHEMISTRY

Organic species containing positively charged carbon atom is PURIFICATION OF ORGANIC COMPOUNDS
known as carbonium ion. The positively charged carbon atom A large number of methods are available for the purification of
contains six electrons in its valence shell. substances. The choice of method, however depends upon the
Stability of carbonium ions : nature of substance (whether solid or liquid) and the types of
1 impurities present in it. Following are some of the important
Stability of carbocation µ +I µ
–I methods which are commonly employed for the purification of
Å organic compounds:
> CH > NO – CH
Å Å
e.g. (i) CH3– CH 2 3 2 2 (1) Filtration, (2) Crystallization, (3) Fractional crystallization, (4)
Å Å Å Sublimation, (5) Distillation, (6) Fractional Distillation, (7)
(ii) (C6H5)3 C > (C6H5)2 CH > C6H5 CH 2 Distillation under reduced pressure, (8) Steam Distillation, (9)
Å Å Differential extraction, (10) Chromatography
> CH2 = CH – CH 2 > 3º > 2º > 1º > CH 3 QUALITATIVE AND QUANTITATIVE ANALYSIS
Stability µ No. of resonating structures
Qualitative Analysis
é | ù The elements present in organic compounds are C and H. In
Carbanion ê -C : - ú :
Q
ê | ú addition to these, they may also contain O, N, S, halogens and P.
ë û Detection of C and H: They are detected by heating the compound
| with copper (II) oxide. C is oxidised to CO2 and H to H2O. CO2
| Heterolytic fission
-C - X ¾¾¾¾¾¾¾¾ ® - C :Q + XÅ turns white lime water milky, while water formed turn white
| | anhydrous CuSO4 to blue hydrated CuSO4.
If EN of C > EN of X D
An organic species containing negatively charged carbon atom is C + 2CuO ¾¾® 2 Cu + CO 2
called carbanion. D
2H + CuO ¾¾® Cu + H 2O
Stability of carbanion is due to following reasons
(i) Inductive effect CO 2 + Ca( OH ) 2 ¾¾
® CaCO 3 ¯ + H 2 O
(ii) Resonance
(iii) s-character in Hybridisation
5H 2 O + CuSO 4 ¾¾
® CuSO4 .5H2 O
1
Stability of Carbanion µ –I , µ white blue
+I
.–. Detection of other elements
.–. .–.
Ex. (i) CH3– CH2 < CH3 < NO2 – CH2 Preparation of Lassaigne’s extract : N, S halogens and P are
detected by Lassaigne’s test. The elements present in organic
– – – –
(ii) CH 3 > CH3 CH2 > (CH3)2 CH > (CH3)3 C compounds are converted to ionic form by fusing with Na-metal.
The reactions involved are:
Note : Carbanions are nucleophiles.
D
TYPES OF ORGANIC REACTIONS Na + C + N ¾¾® NaCN

(i) Substitution or Displacement Reactions D


2Na + S ¾¾® Na 2S
Substitution or displacement reactions are those reactions in which
D
an atom or group of atoms attached to a carbon atom in a substrate Na + X ¾¾® NaX ( X = Cl, Br, I)
molecule is replaced by another atom or group of atoms. They are extracted from fused mass by boiling with distilled water.
(ii) Addition Reactions The extract is known as sodium fusion extract (Lassaigne’s extract).
Addition reactions are those in which the attacking reagent adds (a) Test for N : The extract is boiled with FeSO4 and then acidified
with conc. H2SO4. Prussian blue colour confirms presence
up to the substrate molecule. Such reactions are given by those
of nitrogen.
compounds which possess double or triple bonds.
4-
(iii) Elimination Reactions 6 CN - + Fe2 + ¾¾
® éë Fe ( CN )6 ùû
The reverse of addition reactions are termed as elimination
4- xH 2O
reactions. In these reactions generally atoms or groups from two 3 éë Fe ( CN )6 ùû + 4Fe3+ ¾¾¾¾ ® Fe 4 éë Fe ( CN )6 ùû . xH 2O
adjacent carbon atoms in the substrate molecule are removed and 3
multiple bond is formed. But if sulphur is also present along with nitrogen one gets
blood red colouration due to formation of ferric thiocyanate.
(iv) Rearrangement Reactions
Na + C + S + N ¾¾ ® NaSCN
The reactions which involve the migration of an atom or group
from one site to another within the molecule (nothing is added Fe3+ + 2NaSCN ¾¾ ® Fe(SCN)3 + 3Na+
from outside and nothing is eliminated) resulting in a new molecular Blood red
structure are known as rearrangement reactions.
Organic Chemistry : Some Basic Principles and Techniques 271

(b) Test for S: (ii) Nitrogen


(i) To extract, add CH3COOH and lead acetate. A black ppt (a) Duma,s method: The N-containing organic compound, when
of PbS confirms S. heated with CuO in an atmosphere of CO2, yields free N2 in
addition to CO2 and H2O.
S2- + Pb2+ ¾¾ ® PbS ¯
Let mass of organic compound = Wg
black Volume of nitrogen collected = V1 mL
(ii) Take small portion of Lassaigne's extract and add few Room temp = T1 K
drops of sodium nitroprusside solution, a violet
colouration is obtained which fades away on standing. P1V1 ´ 273
Volume of N2 at STP = (Let it be V mL)
2Na + S ¾¾ ® Na2S ; 760 ´ T1
Na2S + Na2[Fe(CN)5NO] ¾¾ ® Na4[Fe(CN)5NOS] P1 = Atm. pressure - Aqueous tension
violet color 22400 mL N2 at STP weighs 28 g.
(c) Test for halogens : The extract is acidified with HNO3 and 28 ´ V
then treated with AgNO3. A white ppt, soluble in NH4OH VmL N2 at STP weighs = g
22400
indicates the presence of Cl. A yellowish ppt, sparingly
soluble in NH4OH indicates the presence of Br. A yellow ppt, 28 ´ V ´ 100
insoluble in NH4OH indicates the presence of I % of N2 = 22400 ´ W

(b) Kjeldhal’s method: The compound is heated with conc


X - + Ag + ¾¾
® AgX ( X = Cl, Br, I )
H2SO4 when nitrogen gets converted to (NH4)2SO4. The
If N or S is also present, then extract is first boiled with conc. mixture is then heated with excess of NaOH. The liberated
HNO3 to decompose CN– and S2 – (they would otherwise NH3 gas is absorbed in an excess of standard solution of
interfere with AgNO3 test for halogens). H2SO4.
(d) Test for P : Phosphorus is detected by fusing the organic
D
compound with an oxidising agent like Na2O2 .P in the N (from compound) + conc. H2SO4 ¾¾ ® (NH4)2SO4
D
compound is oxidised to PO34- which is then extracted with (NH4)2SO4 + 2 NaOH ¾¾ ® Na2SO4 + 2H2O + 2NH3
water. The solution is boiled with HNO3 and then treated 2 NH3 + H2SO4 ¾¾ ® (NH 4)2SO4
with ammonium molybdate. A yellow coloured ppt indicates The volume of the acid left unused is found by titration against
presence of phosphorus. a standard alkali solution.
2 NaOH + H2SO4 ¾¾ ® Na2SO4 + 2H2O.
Na 3PO4 + 3HNO3 ¾¾
® H3PO 4 + 3 NaNO3 The difference between initial amount of acid taken and that
left after reaction gives the amount of acid reacted with NH3.
5Na2O2 +2 P ¾¾
® 2 Na3PO4 + 2 Na2O
æ V ö
14 ´ M ´ 2 ç V - 1 ÷
H3PO 4 +12 ( NH 4 ) 2 MoO4 + 21HNO3 ¾¾
® è 2 ø 100
% of N = ´
1000 W
( NH 4 )3PO 4 .12 MoO3 + 21 NH 4 NO3 + 12H 2 O Where M ® molarity
Ammonium V ® volume of H2SO4 taken
phosphomolybdate V1 ® volume of NaOH used for titration of excess of H2SO4
Quantitative Analysis W ® mass of organic compound
The percentage composition of elements present in an organic æ v ö
compound is determined in following ways: 2 ç v - 1 ÷ ® volume of NH solution
è 2 ø 3
(i) Carbon and Hydrogen
A known mass of an organic compound is burnt in presence (iii) Halogens
of excess of O2 and CuO, when C and H are oxidised to CO2 Carius method: A known mass of organic compound is heated
and H2O. Let mass of organic compound be ‘W’ g, mass of with fuming HNO3 in presence of AgNO3. C and H in the
H2O and CO2 produced be W1 and W2 g. Then, compound are oxidised to CO2 and H2O. The halogen present
12 ´ W2 ´ 100 forms Ag X which is filtered, washed, dried and weighed.
Percentage of C = Let mass of organic compound = Wg
44 ´ W
mass of AgX formed = W1 g
y D y z 1 mol of AgX contains 1 mol of X
Cx HyNz + (2x + 2 ) CuO ¾¾ ® x CO2 + H2O + N2
2 2
atomic mass of X ´ W1g
æ yö mass of X in W1 g of Ag = molecular mass of AgX
+ çè 2x + ÷ø Cu
2
2 ´ W1 ´ 100 at. mass of X ´ W1 ´ 100
Percentage of H= % of X = molecular mass if AgX ´ W
18 ´ W
EBD_7327
272 CHEMISTRY

(iv) Sulphur D
A known mass of organic compound is heated with sodium 2P + 5O (from HNO3) ¾¾ ® P2O5
peroxide or fuming HNO3. S in the compound is oxidised to D
P2O5 + 3H2O ¾¾ ® 2H3PO4
H2SO4. It is precipitated as BaSO4 by adding excess BaCl2
solution in water. Phosphoric acid
D H3PO4 + 12 (NH4)2MoO4 + 21HNO3 ¾¾ D
C + 2O (from HNO3) ¾¾ ® CO2 ®
(NH4)3PO4.12MoO3 + 21 NH4NO3 + 12H2O
D
2H + O (from HNO3) ¾¾ ® H2O Let mass of organic compound = Wg
D mass of ammonium phosphomolybdate = W1 g
S + H 2 O +3O (from HNO3 ) ¾¾
® H 2SO 4 Molar mass of (NH4)3PO4. 12 MoO3 = 1877 g
H2SO4 + BaCl2 ¾¾ ® BaSO4 + 2HCl 31 ´ W1 ´ 100
% of P = %
(white ppt) 1877 ´ W
The ppt is filtered, washed, dried and weighed. If P is estimated as Mg2P2O7.
The % of S is calculated from mass of BaSO4.
Let mass of organic compound taken = Wg 62 ´ W1 ´ 100
% of P = %
mass of BaSO4 formed = W1 g 222 ´ W
1 mol of BaSO4 = 233 g BaSO4 = 32 g S (vi) Oxygen
32 ´ W1 % of O2 = 100 – (sum of % of all other elements)
W1 g BaSO4 contains gS
233 D
Compound ¾¾¾¾ ® O2 + other gaseous products.
N gas 2
32 ´ W1 ´ 100
% of S = 233 ´ W 1373 K
2C + O2 ¾¾¾¾ ® 2 CO ] ´ 5 ... (A)
(v). Phosphorus
A known mass of an organic compound is heated with fuming I2O5 + 5CO ¾¾ ® I2 + 5 CO2 ] ´ 2 ... (B)
HNO3 Carbon and hydrogen are oxidised to CO2 and H2O On solving (A) and (B), we find that each mole of O2 liberated
respectively while P in compound is oxidised to H3PO4. It is produces 2 moles of CO2.
precipitated as ammonium phosphomolybdate, (NH4)3PO4. Let mass of organic compound = Wg
12 MoO3 by adding ammonium molybdate. The precipitate mass of CO2 produced = W1 g
of (NH4)3PO4. 12MoO3 is then filtered, washed, dried and
32 ´ W1
weighed. \ m1 g of CO2 is obtained from g O2
D
88
C + 2 O (from HNO3) ¾¾ ® CO2
32 ´ W1 ´ 100
D
2H + O (from HNO3) ¾¾ ® H2O \ % of O2 = 88 ´ W
%
Organic Chemistry : Some Basic Principles and Techniques 273

CONCEPT MAP
EBD_7327
274 CHEMISTRY

1. The number of en antiomers of th e compound 8. The IUPAC name of the compound shown below is :
(a) 3-bromo-1-chlorocyclohexene Cl
CH 3 CHBr CHBr COOH is :
(b) 1-bromo-3-chlorocyclohexene
(a) 2 (b) 3
(c) 2-bromo-6-chlorocyclohex-1-ene
(c) 4 (d) 6
(d) 6-bromo-2-chlorocyclohexene Br
2. The IUPAC name of CH 3 - C º C - CH(CH 3 ) 2 is: 9. In Lassaigne test a violet colour obtained indicates the
(a) 4-methyl-2-pentyne presence of
(b) 4, 4-dimethyl-2-butyne (a) sulphur (b) halogen
(c) methyl isopropyl acetylene (c) nitrogen (d) None of these
(d) 2-methyl-4-pentyne 10. IUPAC name of the compound is :
3. Which pairs have same percentage of carbon? O
(a) CH 3COOH and C 6 H12O 6 ||
H - C - CH 2 - CH 2 - OCH 3
(b) CH 3COOH and C12 H 22O11 (a) 2 – methoxypropanal (b) methoxypropanal
(c) CH 3COOH and C 2 H 5OH (c) 3 – methoxypropanal (d) 2 – formyl methoxyethane
11. The number of secondary hydrogens in 2, 2-dimethylbutane
(d) C 6 H12O 6 and C12 H 22O11 is :
4. The IUPAC name of the compound : (a) 8 (b) 6 (c) 4 (d) 2
CH 2 - CH - CH 2 is 12. The geometrical isomerism is shown by
| | | CH2
CN CN CN CH2
(a) 1, 2, 3-trinitropropane (a) (b)
(b) 1, 2, 3-tricyanopropane
(c) 3-cyanopentane-1, 5-dinitrile CHCl
(d) 1, 2, 3-pentanetrinitrile CCl2
5. The IUPAC name of the compound (c) (d)
CH 3OCH 2 CH 2 CH 2 OCH 2 CH 3 is :
13. The IUPAC name for
(a) 3-ethoxy-1-methoxypropane
(b) 1-ethoxy-3-methoxypropane C1
(c) 2, 5-dioxyhexane |
(d) ethoxypropane oxymethane CH3 - C - CH 2 - CH = CH - CH3 is
6. The absolute configuration of the following : |
H
CH3 (a) 5–chlorohex–2–ene
H Cl (b) 2–chlorohex–5–ene
is (c) 1–chloro–1–methylpent–3–ene
Cl H
(d) 5–chloro–5–methylpent–2–ene
C2H5 14. Among the following the most stable compound is :
(a) 2S, 3R (b) 2S, 3S (a) cis-1, 2-cyclohexanediol
(c) 2R, 3S (d) 2R, 3R (b) trans-1, 2-cyclohexanediol
7. Which of the following compounds has wrong IUPAC name? (c) cis-1,3-cyclohexanediol
(a) CH3–CH2–CH2 –COO–CH2CH3 ® ethyl butanoate (d) trans-1, 3-cyclohexanediol
CH3 - CH - CH 2 - CHO ® 3-methyl-butanal 15. Which of the following compounds contains 1°, 2°, 3° as
(b)
| well as 4° carbon atoms ?
CH3 (a) Neopentane (b) 2-methyl pentane
(c) 2,3-dimethyl butane (d) 2,2,3-trimethyl pentane
(c) CH3 - CH - CH - CH3 ® 2-methyl-3-butanol 16. Correct configuration of the following is
| |
OH CH3 CH3
O H OH
||
(d) CH3 - CH - C - CH 2 - CH3 CH3 OH
|
CH3
H
® 2-methyl-3- pentanone (a) 1S, 2S (b) 1S, 2R (c) 1R, 2R (d) 1R, 2S
Organic Chemistry : Some Basic Principles and Techniques 275

17. The correct name for the following hydrocarbon is 29. The chirality of the compound
(a) Tricyclo [4.1.0] heptane Br
(b) Bicyclo [5.2.1] heptane
(c) Bicyclo [4.1.0] heptane C
(d) Bicyclo [4.1.0] hextane H is
18. The Beilstein test for organic compounds is used to detect H3C Cl
(a) nitrogen(b) sulphur (c) carbon (d) halogens (a) R (b) S (c) E (d) Z
19. Which of the following is an optically active compound ? 30. Among the following four structures I to IV,
(a) 1-Butanol (b) 1-Propanol
(c) 2-Chlorobutane (d) 4-Hydroxyheptane CH3 O CH3
| || |
20. An important chemical method to resolve a racemic mixture C 2 H 5 - CH - C3 H 7 , CH 3 - C - CH - C 2 H 5,
makes use of the formation of ( II )
(a) a meso compound (b) enantiomers ( I)
(c) diasteromers (d) racemates
H CH3
21. Lassaigne’s test is used in qualitative analysis to detect | Å |
(a) nitrogen (b) sulphur H- C , C2H 5 - CH- C 2H5
(c) chlorine (d) All of these |
22. The process of separation of a racemic modification into d H ( IV)
( III)
and l -enantiomers is called
(a) Resolution (b) Dehydration it is true that
(c) Revolution (d) Dehydrohalogenation (a) only I and II are chiral compounds
23. Tautomerism will be exhibited by (b) only III is a chiral compound
(a) (CH3)2NH (b) (CH3)3CNO (c) only II and IV are chiral compounds
(c) R3CNO2 (d) RCH2NO2 (d) all four are chiral compounds
24. Lassaigne’s test for the detection of nitrogen fails in 31. Which of the following is optically inactive?
H H
(a) NH 2 CONHNH 2 .HCl (b) NH 2 NH 2 .HCl
H 3C Cl Cl CH3
(c) NH 2CONH 2 (d) C 6 H 5 NHNH 2 .HCl (a) Cl CH3 (b) H C Cl
3
25. A compound of molecular formula of C7H16 shows optical H H
isomerism, compound will be H
(a) 2, 3-Dimethylpentane (b) 2,2-Dimethylpentane H 3C Cl
(c) 2, 4 - Dimethylpentane (d) None of these (c) H C Cl (d) none of these.
3
.. .. H
26. CH2 – C – CH3 and CH2 = C – CH3 are 32. C8H16 that can form cis-trans geometrical isomers and also
|| |
O :O
. .: has a chiral centre, is
(a) resonating structures (b) tautomers
(c) geometrical isomers (d) optical isomers (a) H (b) H
27. Which of the following pairs of compounds are enantiomers?
CH3 CH3 (c) Both of these (d) None of these
(a)
HO H HO H 33. Which of the following will have a mesoisomer also?
and
H OH HO H (a) 2, 3- Dichloropentane (b) 2, 3-Dichlorobutane
CH3 CH3 (c) 2-Chlorobutane (d) 2-Hydroxypropanoic acid
CH3 CH3 34. Amongst the following compounds, the optically active
H OH HO H alkane having lowest molecular mass is
(b) and
HO H H HO H
CH3 CH3 |
(a) CH 3 - C -
CH3 CH3 |
H OH HO H C2H 5
(c) and
HO H HO H
CH3 CH3 CH3
|
CH3 CH3 (b) CH3 - CH 2 - CH - CH3
H OH H OH (c) CH 3 - CH 2 - CH 2 - CH 3
(d) and
HO H H OH
CH3 CH3 (d) CH 3 - CH 2 - C º CH
28. Kjeldahl’s method is used in the estimation of 35. The most suitable method for separation of a 1 : 1 mixture of ortho
(a) nitrogen (b) halogens and para nitrophenols is
(c) sulphur (d) oxygen (a) sublimation (b) chromatography
(c) crystallization (d) steam distillation
EBD_7327
276 CHEMISTRY

36. Which one of the following is a technique most suitable for 46. Geometrical isomerism is possible in case of :
purification of cyclohexanone from a mixture containing (a) pentene-2 (b) propane
benzoic acid, isoamyl alcohol, cyclohexane an d (c) pentane (d) ethene
cyclohexanone? 47. The number of isomers of the compound with molecular
(a) Crystallization (b) Sublimation formula C2H2Br2 is
(c) IR spectroscopy (d) Gas chromatography (a) 4 (b) 3 (c) 5 (d) 2
37. Which of the following is optically active ? 48. One among the following will show optical activity?
H H
H H
(a) C C CH3
(a) CH 3 C C C2 H5 (b) CH3 — CHOH — CH3 H C C
CH(Cl)CH3
H
OH OH
Br CH3 H
(b) C C CH(CH3)2
H C C
(c) CH3 C C COOH (d) CH3 CH2 C= O CH3 H

O Br OH H3C CHCl2
38. The correct stability order of following species is – (c) C C CH3
H C C
H3C CH3
(x) (y) H3C CH2Cl CH CH
C (d) C C 2 3
H C C
(z) (w) H CH2Cl2
(a) x > y > w > z (b) y > x > w > z 49. The number of ether isomers possible for C4H10O are :
(c) x > w > z > y (d) z > x > y > w (a) 2 (b) 5 (c) 4 (d) 3
39. An aromatic compound of formula C7H7Cl has in all ..... 50. Keto-enol tautomerism is observed in :
isomers :
(a) 5 (d) 2 (c) 4 (d) 3 O O
40. How many optically active stereoisomers are possible for || ||
(a) H 5 C 6 - C - CH 2 - C - C 6 H 5
lactic acid ?
(a) 1 (b) 2 (c) 4 (d) 3
O
41. The correct IUPAC name for ||
(b) H 5 C 6 - C - CH 3

O
||
(c) H 5C 6 - C - H
(a) 5-methyl -4-(1'-2'-demethylpropyl) heptane (d) Both (a) and (b)
(b) 3-methyl-4-(1',2'-dimethylpropyl) heptane 51. The best method for the separation of naphthalene and
(c) 2,3,5-trimethyl-4-propylheptane benzoic acid from their mixture is:
(d) 4-propyl-2,3,5-trimethylpeptane (a) distillation (b) sublimation
42. Geometrical isomerism is possible in : (c) chromatography (d) crystallisation
(a) CH 3CH(CH 3 )CH 2 CH 2 CH 3 52. Isomerism due to rotation around C–C single bond is :
(a) Enantiomerism (b) Position isomerism
(b) CH 3CH = CHCH 3
(c) Conformation (d) Diastereoisomerism
(c) CH 3 CH = CH 2 53. If a compound has n asymmetric carbon atoms, then maximum
(d) ClH 2 C - CH 2 Cl number of optical isomers are given by the formula
n
43. How many acyclic structural isomers are possible for the æ 1ö
(a) 2n (b) ç ÷ (c) 2n (d) 2 n
compound with molecular formula C3H3Cl3 ? è 2ø
(a) 3 (b) 2 (c) 5 (d) 6 54. Which is a chiral molecule?
44. The number of optical enantiomorphs of tartaric acid is/are: (a) 2, 2-dimethylbutanoic acid
(a) 1 (b) 4 (c) 3 (d) 2 (b) 4-methylpentanoic acid
45. Which of the cations in more stable ? (c) 3-methylpentanoic acid
H (d) 3,3-dimethylbutanoic acid
H 55. In Kjeldahl’s method, CuSO4 acts as
| | +
(a) R '- C + - OR (b) R '- C = OR (a) oxidising agent (b) reducing agent
(c) hydrolysing agent (d) catalytic agent
(c) both equal (d) both are unstable
Organic Chemistry : Some Basic Principles and Techniques 277

56. The following compounds differ in 63. Which of the substance is purified by sublimation?
H Cl H Cl (a) Benzoic acid (b) Camphor
C=C C=C (c) Naphthalene (d) All of these
H Cl Cl H 64. The most reactive compound for electrophilic nitration is
(a) configuration (b) conformation (a) Benzene (b) Nitrobenzene
(c) structure (d) chirality (c) Benzoic acid (d) Toluene.
57. A mixture of methyl alcohol and acetone can be separated 65. Which of the following molecules is achiral?
by NH2
(a) distillation H O
(b) fractional distillation
(a) CH2CH3 (b) H
(c) steam distillation
(d) distillation under reduced pressure H OH
58. The correct statement about the compounds (A), (B) and CH3
(C) is : OH
Cl SH
Cl SH
COOCH 3 COOH COOH (c) (d)
H OH H OH H OH
66. Which one of the following is a free-radical substitution
H OH H OH HO H
reaction?
COOH COOCH 3 COOCH 3
(C) (a) CH 3CHO + HCN ¾
¾® CH 3CH ( OH ) CN
(A) (B)
(a) (A) and (B) are identical CH3 CH2Cl
(b) Boiling
(b) (A) and (B) are diastereomers +Cl2
(c) (A) and (C) are enantiomers
(d) (A) and (B) are enantiomers Anh. AlCl3 CH3
59. Which compound on nitration will give highest amount of (c) +CH3Cl
m-substituted product ?
CH3 CH2–Cl CH2Cl CH2NO2
(d) + AgNO2
(a) (b) 67. Which of the following undergoes nucleophilic substitution
exclusively by SN1 mechanism?
Cl (a) Ethyl chloride (b) Isopropyl chloride
Cl
CH C Cl (c) Chlorobenzene (d) Benzyl chloride
Cl Cl 68. Styrene can be purified by
(a) Simple distillation (b) Fractional distilation
(c) (d)
(c) Steam distillation (d) Vacuum distillation
69. The correct order regarding the electronegativity of hybrid
60. The molecular formula of diphenyl methane, orbitals of carbon is
CH2 , is C13H12. (a) sp > sp2 > sp3 (b) sp < sp2 > sp3
2
(c) sp < sp < sp 3 (d) sp > sp2 < sp3
How many structural isomers are possible when one of the 70. The example of an electrophilic substitution reaction is :
hydrogens is replaced by a chlorine atom? NH2 N(C2H 5)2
(a) 6 (b) 4 (c) 8 (d) 7 (a) +C2H5B r®
61. Nitroethane can exhibit one of the following kind of
isomerism:
(a) metamerism (b) optical activity (b) OH + X– N2+ SO3– Na+
(c) tautomerism (d) position isomerism
+ –
62. Tautomerism is exhibited by – ® Na O N=N SO3- Na +

O HBr
(1) CH = CH – OH (2) O (c) H 2 C = CH – CH 3 ¾¾¾¾®
Peroxide
Br – CH 2 – CH 2 – CH3
(3) O (4) O O
||
NuH
(d) H 2C = CH - C - CH3 ¾¾¾ ®
O O
O
(a) (1), (3) and (4) (b) (2), (3), and (4) ||
(c) All of these (d) None of these NuCH2 CH 2 - C - CH3
EBD_7327
278 CHEMISTRY

71. Which one of the following does not have sp2 hybridized 84. Which of the following pairs represent electrophiles?
carbon ? (a) AlCl3, H2O (b) SO3, NO2+
(a) Acetonitrile (b) Acetic acid (c) BF3, H2O (d) NH3, SO3
(c) Acetone (d) Acetamide 85. Chromatography is a valuable method for the separation,
72. Rate of the reaction isolation, purification and identification of the constituents
O O – of a mixture and it is based on general principle of
R – C + Nu R – C + Z is fastest when Z is (a) phase rule (b) phase distribution
Z Nu (c) interphase separation (d) phase operation
(a) OC2H5 (b) NH2 (c) C l (d) OCOCH3 86. The shape of methyl carbanion is similar to that of –
73. The increasing order of stability of the following free radicals (a) BF3 (b) NH3
is (c) methyl free radical (d) methyl carbocation
• • •
(a) (C6H5)2 C H < (C6H5)3 C < (CH3)3 C < (CH3)2 C H
• 87. The replacement of chlorine of chlorobenzene to give phenol
• • • • requires drastic conditions, but the chlorine of
(b) (CH3)2 C H < (CH3)3 C < (C6H5)2 C H < (C6H5)3C 2,4-dinitrochlorobenzene is readily replaced since,
• • • •
(c) (CH3)2 C H < (CH3)3 C < (C6H5)2 C H < (C6H5)3 C (a) nitro groups make the aromatic ring electron rich at
• • • • ortho/para positions
(d) (C6H5)3C < (C6H5)2 C H < (CH3)3 C < (CH3)2 C H
(b) nitro groups withdraw electrons from the meta position
74. Steam distillation is used for the purification of :
of the aromatic ring
(a) covalent compounds
(c) nitro groups donate electrons at meta position
(b) electrovalent compounds
(d) nitro groups withdraw electrons from ortho/para
(c) co-ordinate compounds
positions of the aromatic ring
(d) volatile compounds
88. The optical inactivity due to internal compensation can be
75. Which of the following is strongest nucleophile –
exhibited by :
(a) Br– (b) : OH–
(a) dextro-tartaric acid (b) laevo-tartaric acid
(c) : CN - (d) C2 H 5O : (c) racemic-tartaric acid (d) meso-tartaric acid
76. Strongest acid among the following is : 89. Consider the following phenols :
(a) CCl 3COOH (b) CH 3COOH OH OH OH OH

(c) CF3 COOH (d) CBr3 COOH


77. The prussian blue colour obtained in the Lassaigne’s test
for nitrogen is due to the formation of NO2
(a) Iron (II) hexacyanoferrate (III) CH3 NO2
(b) Iron (III) hexacyanoferrate (II) I II III IV
(c) Iron (III) hexacyanoferrate (III) The decreasing order of acidity of the above phenols is
(d) Iron (II) hexacyanoferrate (II) (a) III > IV > II > I (b) II > I > IV > III
78. Which one of the following species is isoelectronic with (c) I > IV > II > III (d) III > IV > I > II
ammonia ? 90. Absolute alcohol cannot be obtained by simple fractional
distillation because
(a) CH -2 (b) +CH2 (c) –CH3 (d) +CH3 (a) Pure C2H5OH is unstable
79. A meta-directing functional group is : (b) C2H5OH form hydrogen bonds with water
(a) — COOH (b) OH (c) Boiling point of C2H5OH is very close to that of water
(c) — CH3 (d) — Br (d) Constant boiling azeotropic mixture is formed with water
80. The following compound will undergo electrophilic 91. Impure glycerine can be purified by
substitution more readily than benzene : (a) Steam distillation (b) Simple distillation
(a) Nitrobenzene (b) Benzoic acid (c) Vacuum distillation (d) Extraction with a solvent
(c) Benzaldehyde (d) Phenol 92. Consider the acidity of the carboxylic acids :
81. In Kjeldahl’s method for the estimation of nitrogen, the (A) PhCOOH (B) o-NO2C6H4COOH
foumula used is (C) p-NO2C6H4COOH (D) m-NO2C6H4COOH
Which of the following order is correct ?
1.4VW 1.4VW
(a) %N = (b) %N = (a) B > D > A > C (b) B > D > C > A
N V (c) A > B > C > D (d) B > C > D > A
1.4 VN
(c) %N = VNW (d) %N = 93. The decreasing order of nucleophilicity among the
1.8 W nucleophiles
82. The number of s and p bonds in a molecule of acetonitrile
are respectively (A) CH 3C - O - (B) CH 3 O -
(a) 2, 5 (b) 3, 4 (c) 4, 3 (d) 5, 2 ||
O
83. An asymmetric carbon atom is that in which : (C) CN -
(a) three similar and one dissimilar groups O
(b) two similar and two dissimilar groups ||
(c) all the different groups (D) H C
3 S – O – is
(d) none of the above ||
O
Organic Chemistry : Some Basic Principles and Techniques 279

(a) (C), (B), (A), (D) (b) (B), (C), (A), (D) 103. How many stereoisomers does this molecule have ?
(c) (D), (C), (B), (A) (d) (A), (B), (C), (D) CH3CH = CHCH 2CHBrCH 3
94. 1,2-Dihydroxybutane has the formula
(a) 4 (b) 6 (c) 8 (d) 2
(a) C(OH) – C(OH) – C – C (b) C – C(OH)2 – C – C
104. The stability of carbanions in the following :
(c) (OH)C – C – C – C(OH) (d) C – C(OH) – C(OH) – C
95. Which of the following represents the correct order of
stability of the given carbocations ? (I) RC = C (II)
+ |+ + + |+ +
(a) F3C > F3C - C > CH3 (b) H3C > F3C - C > F3C (III) R 2C = CH (IV) R 3C - CH 2
| | is in the order of :
(a) (I) > (II) > (III) > (IV) (b) (II) > (III) > (IV) > (I)
|+ + + |+ + + (c) (IV) > (II) > (III) > (I) (d) (I) > (III) > (II) > (IV)
(c) F3C - C > F3C > H3C (d) F3C - C > H3C > F3C 105. In the hydrocarbon
| |
CH3 – CH = CH – CH2 – C º CH
96. A bottle containing two immiscible liquids is given to you.
6 5 4 3 2 1
They may be separated by using a
The state of hybrization of carbons 1, 3 and 5 are in the
(a) Fractionating column (b) Separating funnel
following sequence :
(c) Vacuum distillation (d) Steam distillation
(a) sp2, sp, sp3 (b) sp, sp3, sp2
97. Distillation is used to separate liquids which differ in the b.p 2 3
(c) sp, sp , sp (d) sp3, sp2, sp
by
106. The state of hybridization of C2, C3, C5 and C6 of the
(a) 5°C (b) 10°C
hydrocarbon,
(c) 30 – 80°C (d) 100°C
CH3 CH3
98. Example of chain isomerism are : | |
(a) Ortho and meta toluic acids CH3 C CH = CH CH C º CH
(b) Methyl acetate and ethyl formate 7 6| 5 4 3 2 1
(c) Pentanoic acid and 2-methyl butanoic acid CH3
(d) 2-pentanone and 3-pentanone is in the following sequence:
99. CH3 – CHCl – CH2 – CH3 has a chiral centre. which one of (a) sp3, sp2, sp2 and sp (b) sp, sp2, sp2 and sp3
the following represents its R-configuration? 2 3
(c) sp, sp , sp and sp 2 (d) sp, sp3, sp2 and sp3
C2 H 5 C2 H5 107. Which of the following compounds will exhibit cis-trans
| | (geometrical) isomerism?
(a) H - C - CH3 (b) Cl - C - CH3 (a) Butanol (b) 2-Butyne
| | (c) 2-Butenol (d) 2-Butene
Cl H 108. Cyclohexanol (I), acetic acid (II), 2, 4, 6-trinitrophenol (III)
CH3 C2 H 5 and phenol (IV) are given. In these the order of decreasing
| | acidic character will be :
(c) H - C - Cl (d) H3C - C - Cl (a) III > II > IV > I (b) II > III > I > IV
| | (c) II > III > IV > I (d) III > IV > II > I
C2 H5 H 109. The correct order of increasing reactivity of C – X bond
100. For (i) I– , (ii) Cl– , (iii) Br – , the increasing order of towards nucleophile in the following compounds is:
nucleophilicity would be X X
(a) Cl– < Br– < I– (b) I– < Cl– < Br– NO2
(c) Br– < Cl– < I– (d) I– < Br– < Cl– (CH3)3 C – X, (CH3)2CH – X
101. The order of decreasing reactivity towards an electrophilic
reagent, for the following would be
(i) benzene (ii) toluene NO2
(iii) chlorobenzene (iv) phenol (I) (II) (III) (IV)
(a) (ii) > (iv) > (i) > (iii) (b) (iv) > (iii) > (ii) > (i) (a) I < II < IV < III (b) II < III < I < IV
(c) (iv) > (ii) > (i) > (iii) (d) (i) > (ii) > (iii) > (iv) (c) IV < III < I < II (d) III < II < I < IV
102. Which one of the following is most reactive towards 110. Among the given compounds, the most susceptible to
electrophilic attack ? nucleophilic attack at the carbonyl group is:
CH2OH (a) CH 3COOCH3 (b) CH 3CONH 2
(b) CH 3 COOCOCH 3 (d) CH3COCl
(a) (b)
111. The correct IUPAC name of the compound

Cl
OH is
(c) (d)
EBD_7327
280 CHEMISTRY

(a) 4-Ethyl-3-propyl hex-1-ene 117. The alkene that exhibits geometrical isomerism is :
(b) 3-Ethyl-4-ethenyl heptane (a) 2- methyl propene (b) 2-butene
(c) 3-Ethyl-4-propyl hex-1-ene (c) 2- methyl -2- butene (d) propene
(d) 3-(1-ethylpropyl) hex-1-ene 118. The number of stereoisomers possible for a compound of
112. The IUPAC name of the following compound is the molecular formula
Cl CH2CH3 CH3 – CH = CH – CH(OH) – Me is:
(b) 2 (c) 4 (d) 6 (d) 3
C C
119. Out of the following, the alkene that exhibits optical isomerism is
CH3 I (a) 3-methyl-2-pentene (b) 4-methyl-1-pentene
(a) trans-2-chloro-3-iodo-2-pentene (c) 3-methyl-1-pentene (d) 2-methyl-2-pentene
(b) cis-3-iodo-4-chloro-3-pentene 120. The IUPAC name of the following compound is OH
(c) trans-3-iodo-4-chloro-3-pentene (a) 4-Bromo-3-cyanophenol
(d) cis-2-chloro-3-iodo-2-pentene
(b) 2-Bromo-5-hydroxybenzonitrile
113. The IUPAC name of is (c) 2- Cyano-4-hydroxybromobenzene CN
(d) 6-Bromo-3-hydroxybenzonitrile Br
(a) 3-ethyl-4, 4-dimethylheptane 121. Hyperconjugation involves overlap of the following orbitals
(b) 1, 1-diethyl-2,2-dimethylpentane
(c) 4, 4-dimethyl-5,5-diethylpentane (a) s-s (b) s - p (c) p- p (d) p-p
(d) 5, 5-diethyl-4,4-dimethylpentane. 122. In allene (C3H4), the type(s) of hybridisation of the carbon
114. Presence of a nitro group in a benzene ring atoms is (are) :
(a) deactivates the ring towards electrophilic substitution (a) sp and sp3 (b) sp and sp2
(b) activates the ring towards electrophilic substitution 3
(c) renders the ring basic (c) only sp (d) sp2 and sp3
(d) deactivates the ring towards nucleophilic substitution. 123. Which one of the following conformations of cyclohexane
115. The correct decreasing order of priority for the functional is chiral?
groups of organic compounds in the IUPAC system of
(a) Boat (b) Twist boat (c) Rigid (d) Chair.
nomenclature is
(a) – COOH, – SO3H, – CONH2, – CHO 124. Directing influence of the methyl group can be explained on
(b) – SO3H, – COOH,– CONH2, – CHO the basis of
(c) – CHO, – COOH, – SO3H, – CONH2 (a) inductive effect (b) electromeric effect
(d) – CONH2, – CHO, – SO3H, – COOH (c) resonance effect (d) no-bond resonance
116. The IUPAC name of neopentane is 125. The number of isomers of the compound C 2 BrFCl is
(a) 2, 2 dimethylpropane (b) 2 methylpropane
(a) 3 (b) 4 (c) 5 (d) 6
(c) 2, 2 dimethylbutane (d) 2- methylbutane

1. Which of the following is true? (a) 3-pentanone, cyclopentanone, n-pentanal


(a) A cis isomer has lower b.p. than the trans (b) n-pentanal, 3-pentanone, cyclopentanone
(b) A cis isomer has lower m.p. than the tans (c) n-pentanal, cyclopentanone, 3-pentanone
(c) All isomers of nitrophenols are equal soluble in water (d) cyclopentanone, 3-pentanone, n-pentanal
(d) All the three are correct 6. The reaction :
2. Which is weakest base : CH3 (CH2 )5 (CH2)5CH3
H H
(a) C6H5 – CH2 – NH2 (b) C6H5 – CH2 – NH – CH3 OH
(c) O2N – CH2 – NH2 (d) CH3 – NH – CHO C – Br HO – C
3. Which of the following is maximum stable H3C CH3
(a) Conjugated alkadiene (CH2 = CH – CH = CH2) is described as
(b) Isolated alkadiene (CH2 = CH – CH2 – CH = CH2)
(c) Cumulated alkadiene (CH2 = C = CH2) (a) S (b) S (c) S (d) S
E2 N1 N0 N2
(d) All are equal 7. Which of the following is not correct?
4. Which is true regarding the relative basic character of the Order of basicity Order of nucleophilicity
following two pairs?
(a) NH3 > NH2OH (b) –CCl3 > –CF3
(a) F - > Cl - > Br - Br - > Cl - > F -
(c) Both are true (d) Nither is true (b) H - O - O - > OH - H - O - O - > OH -
5. Decreasing order of reactivity towards nucleophilic addition
to carbonyl group among cyclopentanone, 3-pentanone and (c) - CH 3 > NH -2 > OH - - CH 3 > NH 2- > OH -
n-pentanal is (d) H2O > H2S H2O < H2S
Organic Chemistry : Some Basic Principles and Techniques 281

8. Among the following compounds (I - III), the ease of their 17. Which of the following is the strongest base ?
reaction with electrophiles is,
OCH3 NO2 (a) (b) NHCH3

(c) NH2 (d) CH2NH2


I II III
18. Which type of isomerism is possible in the compound
(a) II > III > I (b) III > II > I
(c) II > I > III (d) I > II > III
COOH
9. Which one of the following compounds is resistant to ?
nucleophilic attack by hydroxyl ions ? (a) Geometrical (b) Optical
(a) Methyl acetate (b) Acetonitrile (c) Both (d) None
(c) Diethyl ether (d) Acetamide 19. Which pair of isomerism is not possible together?
10. Which one of the following orders is correct regarding the (a) Ring-chain and functional
–I effect of the substituents ? (b) Geometrical and optical
(a) —NR2 <—OR >—F (b) —NR2 > —OR >—F (c) Metamerism and functional
(c) —NR2 <—OR <—F (d) —NR2 >—OR <—F (d) Metamerism and chain
11. Which of the following is false regarding a good contributor 20. CH 3 Br + Nu - ¾¾® CH 3 - Nu + Br -
of a resonance hybrid?
(a) It must have as many bonds as possible The decreasing order of the rate of the above reaction with
(b) It must have as many octets as possible nucleophiles (Nu–) A to D is
(c) Negative charge, if any, should be on electronegative [Nu– = (A) PhO–, (B) AcO–, (C) HO–, (D) CH3O–]
atom (a) A > B > C > D (b) B > D > C > A
(d) None of the three (c) D > C > A > B (d) D > C > B > A
12. Correct order of stability is : 21. Lassaigne’s test is used to detect
(a) cis -2- butene > 1-butene > trans -2-butene (a) nitrogen and halogens (b) sodium and halogens
(b) trans-2-butene > cis-2-butene > 1-butene (c) halogens and sulphur (d) All of these
(c) 1-butene > cis-2-butene > trans-2- butene 22. The reason for the loss of optical activity of lactic acid when
(d) cis-2-butene > trans-2-butene > 1-butene OH group is changed by H is that :
13. The correct order of reactivity towards the electrophilic (a) Chiral centre of the molecule is destroyed
substitution of the compounds aniline (I), benzene (II) and (b) Molecule acquires asymmetry
nitrobenzene (III) is (c) Due to change in configuration
(a) I > II > III (b) III > II > I (d) Structural change occurs
(c) II > III > I (d) I < II > III 23. Which of the following does not exhibit cis-trans isomerism?
14. Which of the following is least reactive in a nucleophilic (a) C 6 H 5CH = NOH (b) C 6 H 5 N = NC 6 H 5
substitution reaction. (c) (d) None of the three
(a) (CH 3 )3 C - Cl (b) CH 2 = CHCl 24. C – H bond energy is about 101 kcal/mol for methane, ethane
(c) CH 3CH 2Cl (d) CH 2 = CHCH 2 Cl and other alkanes but is only 77 kcal/mol for C – H bond of
CH3 in toluene. This is because :
15. Which of the following is true regarding hyperconjugation,
(a) of inductive effect due to – CH3 in toluene
also known as no-bond resonance?
(b) of the presence of benzene ring in toluene.
(i) Like inductive effect it involves donation of electrons
(c) of resonance among the structures of benzyl radical in
through s bonds
toluene
(ii) Hyperconjugation involves overlapping of filled
(d) of aromaticity of toluene
orbitals with the empty p orbital of the carbocation
(iii) Like resonance, it involves displacement of p or lone 25. In Carius method 0.099 g organic compound gave 0.287 g
pair of electrons to the carbon bearing positive charge AgCl. The percentage of chlorine in the compound will be
(iv) It involves delocalisation of s and p- electrons (a) 28.6 (b) 71.7 (c) 35.4 (d) 64.2
(a) (ii) and (iv) (b) (ii) 26. Select the appropriate relation with respect to acidity of X, Y,
(c) (iii) (d) none Z for the given compound, with increasing order.
16. In the anion HCOO– the two carbon-oxygen bonds are found + +
to be of equal length. what is the reason for it ? H3N NH3
Z X
(a) The C = O bond is weaker than the C — O bond
(b) The anion HCOO– has two resonating structures COOH
(c) The anion is obtained by removal of a proton from the Y
acid molecule (a) Z > X > Y (b) Y > Z > X
(d) Electronic orbitals of carbon atom are hybridised (c) Z < X > Y (d) X > Y > Z
EBD_7327
282 CHEMISTRY

27. The shape of transition state is – COOH (ii)


R

Nu + H C X [Transition state]
R
R
NO2
–––––® Nu C H + X–
OH
R (iii)
(a) Triangular planar (b) Square pyramidal HO
(c) Trigonal bipyramidal (d) Tetrahedral (i)
28. Arrange the following alcohols in order of their decreasing
(iv)
tendency to form carbocation.
(a) (ii) > (i) > (iii) > (iv) (b) (ii) > (iii) > (i) > (iv)
(c) (i) > (ii) > (iii) > (iv) (d) (ii) > (iv) > (i) > (iii)
OH 33. Which of the following is most stable?
+
F OH F +
CH 3 (a) (b)
O O
I II
OH H3C CH 3 +
H2C +
C 6 H 5 CH 2 OH
(c) (d)
CH 3 OH O O
II III IV
34. Sodium nitroprusside, when added to an alkaline solution of
(a) I > II > III > IV (b) IV > III > I > II (c) sulphide ions, produces purple colour ion due to the
IV > III > II > I (d) II > I > IV > III formation of
29. Which molecule will be most reactive for SN1 reaction – (a) Na[Fe(H2O)5 NOS] (b) Na2[Fe(H2O)5 NOS]
Cl (c) Na3[Fe(CN)5NOS] (d) Na4[Fe(CN)5NOS]
CH3 CH3 35. Base strength of :
(a) (b) Br – , (B)
(A) H 3CCH 2
H 2C = CH and (C) H - C º C
CH3 is in the order of :
CH3 (a) (B) > (A) > (C) (b) (C) > (B) > (A)
(c) (A) > (C) > (B) (d) (A) > (B) > (C)
Cl CH3 36. Which one of the following is most reactive towards
(c)
CH3 (d) I electrophilic reagent ?
CH3 CH3
CH3
(a) (b)
30. Diazo compounds, sometimes do not respond Lassaigne’s
test for nitrogen because OCH3 OH
(a) these are quite stable compounds and do not CH3 CH3
decompose to elemental nitrogen
(b) these contain very little carbon (c) (d)
(c) these form organometallic compounds with sodium NHCOCH3 CH2OH
(d) their nitrogen contents is removed, during heating, in 37. Which one is a nucleophilic substitution reaction among
the form of nitrogen gas the following ?
(a) CH3 – CH = CH2 + H2O CH3 – CH – CH3
31. — X + Nu ¾¾
® — Nu + X , OH
(b) RCHO + R¢ MgX R – CH – R¢
the reaction is not possible because –
(a) Nu faces steric hinderance on attacking the leaving OH
CH3
group
(b) electrons are delocalised in the benzene ring (c) CH3 – CH2–CH–CH2Br + NH3
(c) reaction is thermodynamically controlled CH3
(d) C – X bond possesses a double bond character
CH 3–CH2–CH–CH2NH2
32. Which of the following represent the decreasing order of Ka
(d) CH3CHO + HCN ¾¾
® CH3CH (OH) CN
values?
Organic Chemistry : Some Basic Principles and Techniques 283

38. Which of the following compounds undergoes nucleophilic (c) (CH3 )3 C > (CH3 )2 CH > C6 H5 CH 2 > C Cl3
substitution reaction most easily ? (d) C6 H5 CH 2 > C Cl3 > (CH3 )3 C > (CH3 ) 2 CH
Cl 43. The correct order of increasing basicity of the given
Cl conjugate bases (R = CH3) is
(a) (b) (a) RCOO < HC º C < R < NH 2
NO2 (b) R < HC º C < RCOO < NH 2
CH3 (c) RCOO < NH 2 < HC º C < R
Cl (d) RCOO < HC º C < NH 2 < R
Cl 44. Among the following, the least stable resonance structure
(c) (d) is
Å O
(a) Å
OCH3 N
39. Which of the following molecules is expected to rotate the
plane of plane-polarised light? O
COOH
O
Å
(a) H2N (b)
H Å N
H O
CHO
O
(b) HO H (c) Å
Å N
CH2OH O
(c)
O
Å
SH
H2N NH2 (d)
Å N

(d) H H O
Ph Ph 45. The number of stereoisomers obtained by bromination of
40. The absolute configuration of
trans-2-butene is
HO2C CO2H (a) 1 (b) 2 (c) 3 (d) 4
46. The correct stability order for the following species is
OH
HO H H
+ + +
(a) S, S (b) R, R (b) R, S (c) S, R O O +
41. The electrophile, E Å attacks the benzene ring to generate (I) (II) (III) (IV)

the intermediate s-complex. Of the following, which s- (a) (II) > (IV) > (I) > (III) (b) (I) > (II) > (III) > (IV)
complex is lowest energy? (c) (II) > (I) > (IV) > (III) (d) (I) > (III) > (II) > (IV)
NO2 47. In the following carbocation, H/CH3 that is most likely to
H migrate to the positively charged carbon is
+ + E
(a) (b)
H H
H E 1 2 + 4 5
NO2 NO2 H3C — C —3C — C — CH3
H
(c) + E (d) + H HO H CH3
E
42. Arrange the carbanions, (a) CH3 at C-4 (b) H at C-4
(CH3 )3 C , C Cl3 , (CH3 ) 2 CH , C6 H5 CH 2 (c) CH3 at C-2 (d) H at C-2
in order of their decreasing stability :
(a) (CH3 ) 2 C H > C Cl3 > C6 H5 C H2 > (CH3 )3 C
(b) C Cl3 > C6 H5 CH 2 > (CH 3 )2 CH > (CH3 )3 C
EBD_7327
284 CHEMISTRY

DIRECTIONS for Qs. 48 to 50 : These are Assertion-Reason 48. Statement-1 : Aniline is better nucleophile than anilium ion.
type questions. Each of these question contains two statements: Statement-2 : Anilium ion have +ve charge.
Statement-1 (Assertion) and Statement-2 (Reason). Answer 49. Statement-1 : Trans-2-butene on reaction with Br2 gives
these questions from the following four options. meso-2, 3-dibromobutane.
(a) Statement-1 is true, Statement-2 is true, Statement-2 is a Statement-2 : The reaction involves syn-addition of bromine.
correct explanation for Statement-1 50. Statement-1 : Different number of electron pairs are present
(b) Statement-1 is True, Statement-2 is True ; Statement-2 is NOT in resonance structures.
a correct explanation for Statement-1 Statement-2 : Resonance structures differ in the location of
(c) Statement-1 is True, Statement-2 is False electrons around the constituent atoms.
(d) Statement-1 is False, Statement-2 is True

Exemplar Questions These are generally insoluble in water at room temperature


1. Which of the following is the correct IUPAC name? but are miscible with water vapour in vapour phase. A suitable
(a) 3-ethyl-4, 4-dimethylheptane method for the extraction of these oils from the flowers is
(b) 4, 4-dimethyl-3-ethylheptane (a) distillation
(c) 5-ethyl-4, 4-dimethylheptane (b) crystallisation
(d) 4, 4-bis(methyl)-3-ethylheptane (c) distillation under reduced pressure
O O (d) steam distillation
|| || 7. During hearing of a court case, the judge suspected that
2. The IUPAC name for CH3 - C - CH 2 - CH 2 - C - OH is some changes in the documents had been carried out. He
(a) 1-hydroxypentane-1, 4-dione asked the forensic department to check the ink used at two
(b) 1, 4-dioxopentanol different places. According to you which technique can give
(c) 1-carboxybutan-3-one the best results?
(d) 4-oxopentanoic acid (a) Column chromatography
3. The IUPAC name for (b) Solvent extraction
Cl (c) Distillation
(d) Thin layer chromatography
NO2 8. The principle involved in paper chromatography is
(a) adsorption (b) partition
(c) solubility (d) volatility
9. What is the correct order of decreasing stability of the
CH3 following cations?
(a) 1-chloro-2-nitro-4-methylbenzene Å Å
(b) 1-chloro-4-methyl-2-nitrobenzene I. CH3 - C H - CH3 II. CH3 - C H - OCH3
(c) 2-chloro-1-nitro-5-methylbenzene Å
(d) m-nitro-p-chlorotoluene III. CH3 - C H - CH 2 - OCH3
4. Electronegativity of carbon atoms depends upon their state (a) II > I > III (b) II > III > I
of hybridisation. In which of the following compounds, the (c) III > I > II (d) I > II > III
carbon marked with asterisk is most electronegative?
(a) CH3 – CH2 – *CH2 – CH3 10. Correct IUPAC name for H3C – CH – CH – CH3 is ......... .
| |
(b) CH3 – *CH = CH – CH3
C2H5 C2H5
(c) CH3 – CH2 – C º *CH
(d) CH3 – CH2 – CH = *CH2 (a) 2-ethyl-3-methylpentane
5. In which of the following functional groups, isomerism is (b) 3, 4-dimethylhexane
not possible? (c) 2-sec-butylbutane
(a) Alcohols (b) Aldehydes (d) 2, 3-dimethylbutane
(c) Alkyl halides (d) Cyanides 11. In which of the following compounds the carbon marked
6. The fragrance of flowers is due to the presence of some with asterisk is expected to have greatest positive charge?
steam volatile organic compounds called essential oils. (a) *CH3 — CH2 — Cl (b) *CH3 — CH2 — Mg+Cl–
(c) *CH3 — CH2 — Br (d) *CH3 — CH2 — CH3
Organic Chemistry : Some Basic Principles and Techniques 285

12. Ionic species are stabilised by the dispersal of charge. Which 17. Some meta-directing substituents in aromatic substitution
of the following carboxylate ion is the most stable? are given. Which one is most deactivating? [2013]
O O (a) –SO3H (b) –COOH
|| || (c) –NO2 (d) –C º N
(a) CH3 - C - O - (b) Cl - CH 2 - C - O -
18. Which of the following compounds will not undergo Friedal-
O O Craft’s reaction easily : [2013]
||
-
F ||
(a) Xylene (b) Nitrobenzene
(c) F - CH 2 - C - O (d) CH - C - O-
F (c) Toluene (d) Cumene
13. Electrophilic addition reactions proceed in two steps. The 19. The structure of isobutyl group in an organic compound is :
first step involves the addition of an electrophile. Name the
(a) CH3 - CH - CH 2 - CH3 [2013]
type of intermediate formed in the first step of the following ½
addition reaction. H3C – HC = CH2 + H+ ----® ? (b) CH3 - CH 2 - CH2 - CH 2 -
(a) 2° carbanion (b) 1° carbocation CH3
(c) 2° carbocation (d) 1° carbanion |
14. Covalent bond can undergo fission in two different ways. (c) CH3 - C-
|
The correct representation involving a heterolytic fission of CH3
CH3 – Br is CH 3
(a) Å (d) CH - CH 2 -
CH 3 - Br ¾ ¾
® CH 3 + Br
CH 3
r
(b) ® CH 3 + Br s
CH 3 - Br ¾ ¾ 20. Arrange the following in increasing order of stability
[NEET Kar. 2013]
r
(c) ® CH 3 + Br s
CH 3 - Br ¾ ¾ Å Å
(A) (CH3 )2 C - CH2CH3 (B) (CH3 )3 - C
(d) r s
CH3 - Br ¾ ¾ ® CH 3 + Br Å Å
15. The addition of HCl to an alkene proceeds in two steps. The (C) (CH3 )2 - CH (D) CH3 - CH 2
Å
first step is the attack of H+ ion to C—
—C portion (E) CH 3
which can be shown as (a) E < D < C < B < A (b) E < D < C < A < B
(c) D < E < C < A < B (d) A < E < D < C < B
(a) + —C 21. Given
H C—
Br Br H
H Br
(b) H
+
C—
—C CH3 H

(c) + —C H CH3 and CH3


H C— CH3
Br
(d) All of these are possible
NEET/AIPMT (2013-2017) Questions I II
16. Structure of the compound whose IUPAC name is 3-ethyl-2- I and II are [NEET Kar. 2013]
hydroxy-4-methylhex-3-en-5-ynoic acid is : [2013] (a) A pair of optical isomers
OH (b) Identical
(c) A pair of conformers
COOH (d) A pair of geometrical isomers
(a)
22. Homolytic fission of the following alkanes forms free radicals
OH
CH3 – CH3, CH3 – CH2 – CH3, (CH3)2 CH – CH3, CH3 – CH2
– CH (CH3)2. Increasing order of stability of the radicals is
COOH [NEET Kar. 2013]
(b) g g
(a) (CH3)3 C < (CH3)2 C – CH2CH3 <
g g
CH3 – C H – CH3 < CH3 – C H2
g g g
COOH (b) (CH3)2 C – CH2CH3 < CH3 – C H – CH3 < CH3 – C H2
(c) g
< (CH3)3 C
OH g g
OH (c) CH3 – C H2 < CH3 – C H – CH3 <
g g
(CH3)2 C – CH2 – CH3 < (CH3)3 C
g g g
(d) COOH (d) CH3 – C H2 < CH3 – C H – CH3 < (CH3)3 C < (CH3)2
g
C – CH2CH3
EBD_7327
286 CHEMISTRY

23. What is the hybridisation state of benzyl carbonium 30. Consider the following compounds [2015]
+ CH3 Ph
ion —CH2? [NEET Kar. 2013]
CH3—C—CH— Ph—C—PH
(a) sp3 (b) sp2 CH3
(c) spd 2 (d) sp2 d CH3
24. Nitrogen detection in an organic compound is carried out (I) (II) (III)
by Lassaigne’s test. The blue colour formed corresponds to Hyperconjugation occurs in :
which of the following formulae? [NEET Kar. 2013] (a) II only (b) III only
(a) Fe3[Fe(CN)6]3 (b) Fe3[Fe(CN)6]2
(c) I and III (d) I only
(c) Fe4[Fe(CN)6]3 (d) Fe4[Fe(CN)6]2
31. Which of the following is the most correct electron
25. Which of the following compounds will undergo racemisation
displacement for a nucleophilic reaction to take place?
when solution of KOH hydrolyses? [2014]
[2015]
CH2Cl
H H2
(i) (a) H3C—C = C – C – Cl
H
(ii) CH3CH2CH2Cl H H2
(b) H 3C—C = C – C – Cl
CH3 H
| H H2
(iii) H3C - C H - CH2Cl (c) H 3C—C = C – C – Cl
H
CH3 H H2
(d) H 3C—C = C – C – Cl
C H
(iv) H Cl 32. In Duma's method for estimation of nitrogen, 0.25 g of an
C2H3 organic compound gave 40 mL of nitrogen collected at 300
(a) (i) and (ii) (b) (ii) and (iv) K temperature and 725 mm pressure. If the aqueous tension
(c) (iii) and (iv) (d) (i) and (iv) at 300 K is 25 mm, the percentage of nitrogen in the compound
26. In the Kjeldahl’s method for estimation of nitrogen present is : [2015]
in a soil sample, ammonia evolved from 0.75 g of sample (a) 18.20 (b) 16.76
neutralized 10 mL of 1 M H2SO4. The percentage of nitrogen
(c) 15.76 (d) 17.36
in the soil is : [2014]
(a) 37.33 (b) 45.33 33. The pair of electron in the given carbanion, CH3C º C- , is
(c) 35.33 (d) 43.33
present in which of the following orbitals ? [2016]
27. Two possible stereo-structures of CH3CHOH.COOH, which
(a) 2p (b) sp3
are optically active, are called. [2015 RS]
(c) sp2 (d) sp
(a) Diastereomers (b) Atropisomers
34. Which of the following biphenyls is optically active ?
(c) Enantiomers (d) Mesomers
[2016]
28. Which of the following statements is not correct for a
(a) O2N
nucleophile? [2015 RS]
(a) Nucleophile is a Lewis acid
(b) Ammonia is a nucleophile
I
(c) Nucleophiles attack low e– density sites Br Br
(d) Nucleophiles are not electron seeking.
29. Given (b)
CH3 CH3 CH I I
3
I
CH3 CH3
(c)
CH3
O O O I
(I) (II) (III) CH3
Which of the given compounds can exhibit tautomerism?
(a) I and III (b) II and III [2015] (d)
(c) I, II and III (d) I and II CH3
Organic Chemistry : Some Basic Principles and Techniques 287

35. The correct statement regarding the comparison of 37. The IUPAC name of the compound [2017]
staggered and eclipsed conformation of ethane, is [2016] O O
|| ||
(a) The staggered conformation of ethane is less stable H–C
than eclipsed conformation, because staggered
conformation has torsional strain is :-

(b) The eclipsed conformation of ethane is more stable


than staggered conformation, because eclipsed (a) 5-formylhex-2-en-3-one
conformation has no torsional strain (b) 5-methyl-4-oxohex-2-en-5-al
(c) The eclipsed conformation of ethane is more stable (c) 3-keto-2-methylhex-5-enal
than staggered conformation even though the eclipsed (d) 3-keto-2-methylhex-4-enal
conformation has torsional strain 38. The correct statement regarding electrophile is:- [2017]
(a) Electrophile is a negatively charged species and can
(d) The staggered conformation of ethane is more stable form a bond by accepting a pair of electrons from another
than eclipsed conformation, because staggered electrophile
conformation has no torsional strain. (b) Electrophiles are generally neutral species and can form
36. The most suitable method of separation of 1 : 1 mixture of a bond by accepting a pair of electrons from a
ortho and para-nitrophenols is : [2017] nucleophile
(a) Chromatography (c) Electrophile can be either neutral or positively charged
species and can form a bond by accepting a pair of
(b) Crystallisation electrons from a nucleophile
(c) Steam distillation (d) Electrophile is a negatively charged species and can
(d) Sublimation form a bond by accepting a pair of electrons from a
nucleophile
EBD_7327
288 CHEMISTRY

Hints & Solutions


EXERCISE - 1 1°
CH 3
1. (c) No. of asymmetric carbon = 2 |
1° 4° 2° 1°
No. of enantiomers = 22 = 4. 11. (d) CH 3 - C - CH 2 - CH 3
|
CH 3
CH3 1°
1 2 3 4| 5
2. (a) Thus number of secondary hydrogens is two.
CH3 - C º C - C - CH3
| 12. (d) Geometrical isomerism is obseved when different groups
H are attached to each of the doubly bonded carbon atom.
13. (a) The IUPAC name of th e given compound is
4 -methyl-2-pentyne 5-chlorohex-2-ene.
3. (a) is the answer. 14. (d) Among 1, 2- and 1, 3- configurations, 1, 3- is more stable
Molecular formula of CH3COOH = C2H4O2 due to less repulsion.
Both the compounds given have same empirical formula Further among cis and trans isomers, trans is more
that is CH2O. So, percentage of carbon in both of them stable due to less crowding.
will be same. 15. (d) 2, 2,3-trimethyl pentane
4. (c) 2 3 4 1° 1°
CH 2 - CH - CH 2 CH3 CH3
| | | 1° 2° | | 1°
1 H3C - H 2 C - CH - C - CH 3
CN CN CN5 3° | 4°
IUPAC name is 3-cyanopentane-1,5-dinitrile CH3

1 2 3 16. (a)
5. (a) CH 3 - O - CH 2 - CH 2 - CH 2 - O - CH 2 CH 3
17. (c) 2
3-ethoxy-1-methoxypropane 3 1
7
S 4 6
1 5
CH3
Bicyclo [4, 1, 0] heptane
6. (b) H 2
Cl 18. (d)
Cl 3
H H
|
C2H5 19. (c) H3C - CH 2 - C*- CH 3
|
S Cl
The compound containing a chiral carbon atom i.e., (a
So compound is (2S, 3S) carbon atom which is attached to four different atoms
7. (c) The correct name is 3 - methylbutan - 2 - ol is known as a chiral carbon atom) is optically active. As
2-chlorobutane contains a chiral C* atom hence it is
Cl optically active.
1
6 2 20. (c) Diastereomers since they have different melting points,
8. (a) boiling points, solubilities etc.
5 3 Br 21. (d) Nitrogen, sulphur and halogens are tested in an organic
4
compound by Lassaigne's test. The organic compound
3-bromo-1chlorocyclohexene is fused with sodium metal as to convert these elements
9. (a) Violet colour in Lassaigne's test indicates the presence into ionisable inorganic substances,
of sulpur. Na + C + N ¾¾ ® NaCN
Na 2S + Na 2 [Fe(NO)(CN)5 ] ¾¾ ® 2Na + S ¾¾ ® Na 2S
sodium nitroprusside
Na4[Fe(CN)5NOS] 2Na + X 2 ¾¾
® 2NaX
violet coloured complex The cyanide, sulphide or halide ions can be confirmed
in aqueous solution by usual test.
O
||
22. (a) Resolution.
10. (c) H - C - CH 2 - CH2 - OCH3 23. (d) Tautomerism is exhibited by the oscillation of hydrogen
1 2 3 atom between two polyvalent atoms present in the
3-methoxypropanal
molecule. As (d) has a-hydrogen atom therefore it
Organic Chemistry : Some Basic Principles and Techniques 289

shows tautomerism whereas other structures do not. From the above structure, it is evident than structure
O OH (a) will show cis-trans isomerism and it also has a chiral
R CH2 N R CH N carbon. On the other hand, structure (b) although
Nitro form O Acinitro form O shows cis-trans isomerism, it has no chiral carbon.
24. (b) Hydrazine (NH2NH2) does not contain carbon and 33. (b) The compound has two similar assymmetric
hence on fusion with Na metal, it cannot form NaCN; C-atoms. It has plane of symmetry and exist in Meso
consequently hydrazine does not show Lassaigne’s form.
test for nitrogen.
25. (a) A compound is said to exhibit optical isomerism if it
atleast contains one chiral carbon atom, which is an
atom bonded to 4 different atoms or groups. plane of symmetry
CH3
|
(CH 3 ) 2 CH - C* - CH 2 CH 3
| Meso - 2, 3 dichlorobutane
H
– 34. (a) Only 2- cylcopropyl butane has a chiral centre.
26. (a) CH2 — C — CH3 CH2 = C — CH3 H
O O *
CH3 – C – chiral centre

I II C2H5
both are resonating structures.
27. (b) Compound which are mirror image of each other and 35. (d) The boiling point of o-nitrophenol is less than para-
are non superimposable are termed as enantiomers. nitrophenol due to presence of intramolecular hydrogen
CH3 CH3 bonding. Since p-nitrophenol is less volatile in steam
H OH HO
than o-nitrophenol hence they can be separated by
H
and steam distillation.
HO H H OH
36. (c) IR spectroscopy is used for the purification of
CH3 CH3 cyclohexanone from a mixture of benzoic acid, isoamyl
These are enantiomers alcohol, cyclohexane and cyclohexanone.because in this
28. (a) Kjeldahl's method is suitable for estimating nitrogen in method, each functional group appear at a certain peak.
those compounds in which nitrogen is linked to carbon So, cyclohexane can be identified by carbonyl peak.
and hydrogen. The method is not used in case of nitro, 37. (a) Optically active compound has asymmetric carbon
azo and azoxy compound. This method is basically used atom
for estimating nitrogen in food fertilizers and
H H
agricultural products. | |
29. (a) Clockwise rotation.
CH3 - C - C* - C 2 H5
*
1 Br | |
H 4 OH OH
C It has two asymmetric carbon (marked as *) atoms and
H3C Cl 2
3 is optically active.
Hence configuration is R. 38. (c) x is a conjugated diene system, w is an isolated diene
If the eye travel in a clockwise direction, the system, z is a cumulated diene system, y is antiaromatic
configuration is R as the order of priority is system.
Br > Cl > CH3 > H 39. (c) C7 H7 Cl has 4 isomers
30. (a) Carbon atom is connected with four different groups in
chiral structure. CH3 CH3
31. (c) It has centre of symmetry, so optically inactive. | |
Cl
32. (a) Redrawing the given structures we get,
|

H CH3 Cl
|

H H
| | | | o-Chlorotoluene m-Chlorotoluene
CH3 – C = C - CH 3 CH3 - C - C = C
| | | CH3 CH 2 Cl
*
H3C - C - H H H C-C-H | |
| 3
|
H - C-H CH3
|
CH3
|
benzyl chloride
Cl
(a) (b) p-Chlorotoluene
EBD_7327
290 CHEMISTRY

40. (b) No. of optical isomers = 2x (where x = No. of asymmetric 46. (a) Geometrical isomerism is shown by compounds having
atoms) C = C and the two groups attached to each of carbon
CH3 atom are different
HC3 2 5
H – *C – OH C=C
COOH H H
2-pentene
Lactic acid 47. (b) The parent hydrocarbon is C2H4 or CH2=CH2, so,
\ No. of optical isomers of lactic acid = (2)1 = 2 C2H2Br2 can be obtained by replacing two H by two Br
41. (c) In case two or more chains are of equal length, then the for which there are three possibilities.
chain with greater number of side chains is selected as Br H
the principal chain. C=C or
Br H
1, 1-dibromoethene
6 2
7 5 4 3 Br Br Br H
1
C=C or C=C
H H H Br
2, 3, 5-Trimethyl-4-propylheptane trans
cis
42. (b) Geometrical isomerism is shown by compounds which 1, 2-dibromoethene
have C = C and two groups attached to same C atoms 48. (a) Optical activity is the result of chirality (which comes
are different. Choice (b) fulfills both conditions. from the presence of chiral carbon in molecule) or the
H H H CH3 non–superimposibility of certain molecules on their
and mirror images. So, the compound given in option (a)
C=C C=C
which contains a chiral carbon atom(*) is the only chiral
H3C CH3 H3C H molecule and hence is optically active among the given
cis trans
molecules.
43. (d) The parent hydrocarbon of C3H3 Cl 3 is C3 H6 , i.e.
H H
3
CH3 2 CH = 1CH 2 . Three chlorine atoms can be C=C CH3
H C=C *
present in following ways.
H CH(Cl)CH3
(i) Cl3CCH = CH 2 * marked carbon atom is chiral.
Cl 49. (d) (i) C2H5OC2H5
| diethyl ether
(ii) Cl 2 CHC = CH 2 (ii) CH3OCH2 CH2 CH3
methyl propyl ether
(iii) Cl2 CHCH = CH 2 Cl
(iii) CH3 - O - CH - CH3
Cl |
| CH3
(iv) ClCH 2C = CHCl
methyl isopropyl ether
(v) ClCH 2CH = CCl2 50. (d) Keto-enol tautomerism is shown by carbonyl
compounds having a-hydrogen atom.
Cl
| O O
(vi) CH3 C = CCl 2 || a ||
(a) ˆˆ†
H5 C6 - C - C H 2 - C - C6 H5 ‡ˆˆ
44. (d) COOH keto form
|*
HO - C - H O OH
| || |
H - C* - OH H 5 C6 - C - C = C - C 6 H 5
| |
COOH H
Tartaric acid enol form
Tartaric acid is an example of a symmetrical molecule, O
|| a OH
having two asymmetric carbon atoms. In such cases |
(b) H 5C 6 - C - C H 3 ˆˆ† C H - C = CH
Number of optical enantiomorphs (d and l isomers) ‡ˆˆ 6 5 2
Keto form
= 2(n – 1) = 21 = 2 enol form
45. (b) Although (b) has positive ch arge on O (an O
electronegative element), it is more stable because here ||
(c) C6 H5 - C - H
every atom has octet of electrons.
no a- H atom
Organic Chemistry : Some Basic Principles and Techniques 291

51. (b) Among the given compounds naphthelene is volatile 61. (c) Nitroethane(C2H5NO2) can exhibit tautomerism among
but benzoic acid is non-volatile (it forms a dimer). So, the given kinds of isomerism due to the presence of a-
the best method for their separation is sublimation, hydrogen which is labile and form nitrolic acid.
which is applicable to compounds which can be O OH
converted directly into the vapour phase from its solid H3 C – CH2 – N H3 C – CH = N
O O
state on heating and back to the solid state on cooling.
62. (a).
Hence it is the most appropriate method.
52. (c) The free rotation across C – C gives rise to many spatial O
arrangements. These ar rangements are called (a) CH = CH – OH CH2 – CH
conformers and property is called conformation.
enol form keto form
53. (a) Maximum number optical isomers of a compound having
n assymetric carbon atoms is 2n.
54. (c) Chiral molecules are those molecules which have atleast (b) O O cannot tautomerise
one asymmetric carbon atom (a carbon atom attached
to 4 different groups). This is true in case of
3-methylpentanoic acid. (c) O OH O, OH

O keto form O OH OH
H enol form
|
C2 H5 – C - CH2 COOH
| (d) O OH , OH
CH3

55. (d) Kjeldahl’s method depends upon the fact that most of O O enol form OH
keto form
the organic compounds containing nitrogen are
63. (d) Benzoic acid, camphor and naphthalene sublime on
quantitatively decomposed to give (NH4)2SO4 when
heating hence, they are purified by sublimation method.
heated strongly with conc. H2SO4. In this method
64. (d) Due to + I-effect of the CH3 group, toluene has much
CuSO4 acts as catalytic agent.
higher electron density in the ring than benzene,
56. (a) Given structures are representing cis-trans isomerism
nitrobenzene and benzoic acid as they show – I-effect
(geometrical) hence differ only in configuration.
and hence is most reactive towards nitration.
57. (a) 65. (c) Compound given in option (c) is the only compound
58. (d) A and B are enantiomers. which do not have any chiral carbon atom. Thus, it is
59. (d) –CH3 group has electron donating inductive effect achiral i.e. do not show optical activity.
which decreases with replacement of each hydrogen 66. (b) In the presence of UV rays or energy, by boiling chlorine,
atom by chlorine atom. Hence highest amount of m– free radical is generated which attack the methyl carbon
substituted product will be obtained on nitration of atom of the toluene.
Cl CH3 CH2
C Cl D
+H
Cl
benzyl free
radical

Cl 2 ¾¾® 2Cl•
hn
60. (b) In diphenylmethane monochlorination at following •CH CH2Cl
positions will produce structured isomers 2 •
+ Cl

67. (d) SN1 reactions involve the formation of carbocation,


CH2 more is stability of carbocation, higher will be the
p roba bi l i ty of a l kyl ha l i de t o u n der g o S N 1
1 m ech a n ism . Th us decr ea si n g or der of S N 1
4 mechanisms follows order Benzyl > allyl > tertiary
2 3 alkyl halide> secondary alkyl halide > primary alkyl
halides
Cl CH2 CH2
Å
(1) Cl (2) CH2Cl CH2
– Cl
Cl
CH2 CH
(Benzyl cation)
Cl (3) (4)
Obtained from SN1 path. This molecule is resonance
stabilised.
EBD_7327
292 CHEMISTRY

68. (d) Styrene can be purified by vacuum distillation. O


69. (a) Among the three given hybrid orbitals, sp hybrid orbital ||
is most electronegative. Contribution of s in sp hybrid 79. (a) In present case, -C - OH is the only group whose
orbital is maximum so this orbital is closer to nucleus. key atom (atom attached to benzene carbon) is having
Naturally it will have greater tendency to pull electron a multiple bond (one of the characteristics of m-directing
towards it. Hence it becomes more electronegative and groups).
sp3 becomes least electronegative. 80. (d) Due to + M effect of the – OH group in phenol, electron
70. (b) It is a coupling reaction of diazonium salt. density in the ortho and para positions of the phenol
In it the aromatic ring undergoing attack by diazonium nucleus increases hence phenol will undergo
ion must contain a powerful electron releasing group, electrophilic substitution easily than benzene. The other
generally –OH, –NR2, –NHR or –NH2. Substitution three compounds have electron withdrawing groups,
usually secures para- to the activating group. hence they will undergo electrophilic substitution with
It has been established experimentally that coupling is a difficulty than benzene.
electrophilic aromatic substitutions. 81. (d)
H
O O s
sp 3 sp 3 sp3 ||
71.
||
(a) H3 C - C - CH 3 ; CH3 - C - OH ; 82. (d) H s C s s
CºN
s 2p
sp 2 sp 2
Acetone Acetic acid H
Therefore, the number of s and p bonds in acetonitrile
O are 5 and 2 respectively.
sp3 sp sp3 || 83. (c) An asymmetric carbon atom is that in which all different
C H3 - C º N ; CH3 - C - NH2 groups are attached with carbon. Option (c) is correct.
sp 2
Acetonitrile Acetamide
84. (b) Electrophiles are electron deficient or positively charged
species.
72. (c) Cl– is the best leaving group among the given option. 85. (b)
73. (b) The order of stability of free radicals 86. (b) Methyl carbanion is sp3 hybridised, with three bond
• • • •
(C6 H 5 ) 3 C > (C 6 H 5 ) 2 CH > (CH 3 ) 3 C > (CH 3 ) 2 C H pairs and one lone pair same is the case with NH3.
87. (d) —NO2 group is electron attractive group, so it is able
The stabilisation of first two is due to resonance and
to deactivate the benzene ring.
last two is due to inductive effect.
Cl
74. (d) Steam distillation is used for the purification of volatile
compounds. NO2
75. (c) The strength of nucleophile depends upon the nature
of alkyl group R on which nucleophile has to attack
and also on the nature of solvent. The order of strength NO2
of nucleophiles follows the order : hence withdrawl of electrons from ortho and para
CN– > I– > C6H5O– > OH– > Br– > Cl– position cause easy removal of –Cl atom due to
76. (c) CF3COOH has highest acidity due to –I effect of F. development of +ve charge on o- and p positions.
77. (b) The chemistry of the Lassaigne’s test of nitrogen is 88. (d) Optical inactivity due to internal compensation is
exhibited by meso-tartaric acid. Option (d) is correct.
D
Na + C + N ¾¾® NaCN 89. (a) Electron withdrawing group (–NO2) increases acidity
From organic
compound while electron releasing group (–CH3, –H) decreases
acidity. Also effect will be more if functional group
2NaCN + FeSO 4 ¾¾
® Fe(CN) 2 + Na 2SO 4 present at para position then ortho and then meta.
Fe(CN) 2 + 4NaCN ¾¾
® Na 4[Fe(CN) 6 ] 90. (d) Absolute alcohol cannot be obtained by simple
Sod. ferrocyanide fractional distillation because a Constant boiling
azeotropic mixture is formed. It is impossible to get pure
3Na 4 [Fe(CN)6 ] + 4Fe3+ ¾¾
® Fe 4[Fe(CN) 6 ]3 + 12Na +
(Ferric ferrocyanide)
ethanol by distiling any mixture of ethanol and water
(Prussian blue) containing less than 95.6% of ethanol. This particular
78. (c) Isoelectronic species has same number of electrons. mixture of ethanol and water boils as if it were a pure
No. of electrons in different species liquid. It has a constant boiling point, and the vapour
+ composition is exactly the same as the liquid.
NH3 = 7 + 3 = 10; CH 2 = 6 + 2 - 1 = 7 91. (c) Simple distillation is suitable for liquids which boil
- - without decomposition at atmospheric pressure. In case
CH 2 = 6 + 2 + 1 = 9; CH 3 = 6 + 3 + 1 = 10 of liquids which decompose before their boiling point
+ is reached, vaccum distillation is used. Glycerine
CH3 = 6 + 3 - 1 = 8 decomposes at its boiling point (298°C) but can be
distilled unchanged at 12 mm pressure at 180°C.
Organic Chemistry : Some Basic Principles and Techniques 293

92. (d) In aromatic acids presence of electron withdrawing 100. (a) Nucleophilicity increases down the periodic table.
substituent e.g. –NO2 disperses the negative charge
of the anion and stablises it and hence increases the I - > Br - > Cl - > F-
acidity of the parent benzoic acid. 101. (c) Electrophiles have high affinity for electrons. They
Further o-isomer will have higher acidity than attack at the site where electron-density is highest.
corresponding m and p isomers. Since nitro group at Electron donating groups increases the electron
p-position have more pronounced electron density. The electron donating tendency decreases in
withdrawing than –NO2 group at m-position hence the the order :
correct order is the one given above. –OH > –CH3 > –H > –Cl
COOH COOH Therefore, the correct order of reactivity towards
electrophile is
< C6H5OH > C6H5CH3 > C6H6 > C6H5Cl
NO2 102. (c) Out of the given compounds the most reactive towards

COOH COOH OH

NO2
< < electrophilic attack is . Phenol forms phenoxide

NO2 ion which is stable due to resonance i.e. the correct


93. (b) The stronger the base the more is the nucleophilic answer is option (c).
character and vice versa. 103. (a) In the molecule
Basic character
CH3 CH = CH CH 2 CH- CH 3,
- -
CH 3O > CN > CH3 COO > H 3C - - –
SO3 |
Br
94. (a) 1,2-dihydroxybutane has longest carbon atom chain of the number of stereoisomers is given by sum of
4 atoms and OH groups attached on it at first and
geometrical isomers (because of presence of C = C)
second carbon atoms. So the choice (a) is correct.
and optical isomers (because of presence of chiral
95. (b) – I group destablises carbocation and since inductive
effect decreases with increasing length of carbon chain. carbon atom).
Therefore (b) is the correct option. Number of geometrical isomers = 2 (one C = C is
96. (b) Being immiscible liquids, they form different layers present).
hence can be separated with the help of separating Number of optical isomers = 2 (one chiral carbon atom).
funnel. Total number of stereoisomers = 2 + 2 = 4
97. (c) Simple distillation can be applied for the separation 104. (d) The stability of carbanions is affected due to resonance,
and purification of a mixture of two or more liquids inductive effect and s - character of orbitals. Greater
provided their boiling points differ at least by 30 K. In the number of groups having + I group (alkyl group)
such cases, the low boiling (more volatile) liquid of the lesser stable would be the carbanion.
mixture will distil over first. The temperature of the Further stability of carbanion decreases with decrease
distillation flask is then raised when high boiling (less in s-character. Benzene carbanions are stablized due to
volatile) liquid of the mixture will distil, it is collected in resonance, hence the correct order is
a separate flask.
98. (c) Pentanoic acid and 2-methyl butanoic acid are chain
isomers. R - C = C > R 2C = CH> > R C – CH
3 2
CH3CH2CH2CH2COOH ¾¾ ®
The correct order of stability of given carbanion is in
CH3 the order I > III > II > IV.
|
H3C - CH 2 - HC - COOH 105. (b) C – 1 is sp hybridized (C º C)
C – 3 is sp3 hybridized (C– C)
2 C – 5 is sp2 hybridized (C = C)
C2H5 Thus the correct sequence is sp, sp3, sp2.
3
99. (b) Cl — C — CH3 CH3 CH3
1 sp 3 | sp 2 sp 3 | sp
106. (d) CH3 C CH = CH CH — C º C H
7 6| 5 4 3 2 1
H 4
CH3
R-configuration
EBD_7327
294 CHEMISTRY

107. (d) Alkenes with double bonds cannot undergo free 4 5


rotation and can have different geometrical shapes with 112. (a) Cl CH2CH3
two different groups on each end of the double bond. C=C
2 3
CH3 CH3 CH3 H CH3 I
1
C=C C=C Correct IUPAC name of above compound is
H H H CH3
trans-2-chloro-3-iodo-2-pentene
cis-But-2-ene trans-But-2-ene
CH3
OH 7 6 5 4| 3 2 1
ON2 113. (a) CH3 - CH 2 - CH 2 - C - CH - CH 2 - CH3
NO2 | |
108. (a) > CH3COOH >
CH3 CH 2
|
NO2 CH3
(III) (II) 3-ethyl -4,4- dimethyl heptane
OH OH 114. (a) Nitro group is electron withdrawing group, so it
deactivates the ring towards electrophilic substitution.
> 115. (a) The correct order of priority for the given functional
group is
(IV) (I) O O
|| ||
Explanation : Presence of three — NO 2 groups in –COOH > –SO3H > – C - NH 2 > - C - H
o–, p– positions to phenolic groups (in III) makes 116. (a) CH3
phenol strongly acidic because its corresponding 1 2
|
3
conjugate base is highly stabilised due to resonance. H3C- C - CH3
|
Conjugate base of CH3COOH, II (i.e. CH3 COO - ) is CH3
resonance hybrid of two equivalent structures. The Neopentane
or 2, 2- Dimethylpropane
conjugate base of phenol, IV is stabilized due to
resonance (note that here all resonating structures are 117. (b) H3C CH3 H3C H
not equivalent). The conjugate base of cyclohexanol, I C=C C=C
does not exhibit resonance, hence not formed. H H H CH3
109. (a) Tertiary alkyl halide is most reactive towards *
nucleophilic substitution because the corresponding 118. (b) CH 3 – CH = CH – CHCH 3
|
carbocation (3°) is most stable. Aryl halide is least OH
reactive due to partial double bond character of the C – exhibits both geometrical as well as optical isomerism.
Cl bond. cis - R cis - S
Presence of — NO 2 groups in ortho and para positions trans - R trans - S
increases the reactivity of the – Cl towards 119. (c) For a compound to show optical isomerism, presence
nucleophiles. of chiral carbon atom is a necessary condition.
H
(CH3)3 – C – X > (CH3)2 – CH2 – X > |
III IV
X H 2 C < CH — C* — CH 2 , CH3
|
X CH3
NO2
3- methyl-1-pentene
> 120. (b) – CN has highest priority. Further the sum of locants is
7 in (b) and 9 in (d).
NO2
II I 121. (b) Alkyl groups with at least one hydrogen atom on the
or I < II < IV < III a-carbon atom, attached to an unsaturated carbon
110. (d) Cl– is the weakest base and hence better leaving group atom, are able to release electrons in the following way.
111. (a) The given compound is
2 1
CH = CH2
CH3 — CH2 — CH2 — CH — CH — CH2 — CH3 Note that the delocalisation involves s and p bond
3 4 orbitals (or p orbitals in case of free radicals) ; thus
CH2 – CH3 it is also known as s – p conjugation. This type of
5 6 electron release due to the presence of the system
4 ethyl- 3-propyl hex – 1 – ene H—C—C = C is known as hyperconjugation
Organic Chemistry : Some Basic Principles and Techniques 295

In this reaction Walden Inversion takes place so it is an


sp 2 sp sp 2
122. (b) Allene (C3H4) is H 2 C = C = CH 2 exampleof SN 2-reaction.
7. (b) Peroxide ion is more nucleophilic but less basic than
123. (b) Chiral conformation will not have plane of symmetry.
OH–.
Since twist boat does not have plane of symmetry it is
8. (d) –OCH3 activates the benzene ring. –NO2 deactivates
chiral.
the ring. Hence the reaction of the given compounds
124. (d) Methyl group can’t produce electromeric and
with electrophiles is in the order, I > II > III.
resonance effects. Inductive effect affects all the three
9. (c) The compound is diethyl ether (CH3CH2)2O which is
positions of the ring in decreasing order of ortho >
resistant to nucleophilic attack by hydroxyl ion due to
meta > para. It shows hyperconjugation (no-bond
absence of double or triple bond, whereas all other
resonance) which explains beautifully the o, p-directing
compounds given are unsaturated
influence of the -CH 3 group.
O
125. (d) Taking any two halogens, the possible structural ||
isomers for the alkene can be three. C 2 H 5 OC2 H 5 CH3 - C - OCH3
Diethyl ether Methylacetate
F Br F Cl F Cl
C=C I C=C C=C O
Cl Br I I Br ||
Each of the three alkene can exist as E and Z isomer, CH3 - C º N CH3 - C - NH 2
making total number of isomers as six. Acetonitrile acetamide
10. (c) The atom or group which has more power to attract
EXERCISE - 2 electrons in comparision to hydrogen is said to have
1. (b) A cis-isomer has higher b.p. due to polar (or more polar) -I effect. Thus higher the electronegativity of atom
nature but lower m.p. due to asymmetry than the trans- stronger will be the -I effect. As electronegativity of N,
isomer. Among three isomeric nitrophenols, o-isomer O and F follow the order N < O < F hence based upon
forms intramolecular bonding so H-bonding with water electronegative character order of-I effect is
will be restricted. – NR2 < – OR < – F.
2. (d) Due to resonance of lone pair. 11. (d) All statements are ture.
3. (a) Due to resonance conjugated alkadiene is maximum 12. (b) Stability of an alkene depends upon the heat of
stable. Isolated is more stable than cumulated alkadiene. hydrogenation of an alkene. The heat of hydration is
4. (a) NH2OH is less basic than NH3 because of –I effect of the standard enthalpy change in hydrogenation of on
OH. However –CF3 is more basic than –CCl3 because in alkene. The lower the heat of hydrogenation of an
:–CCl3 the unshared electron pair of C in the p orbital alkene higher will be stability.
undergoes extented p–dp bonding into an empty d AO Order of stabilityHeat of hydrogenation
of the three chlorines, hence it is less available for (kJ/mol)
protonation. trans-2-butene – 115.5
cis-2-butene – 119.6 and
.. . .- . . -. . .. 1-butene – 126.8 respectively.
: .C.l - C - C
..l:¬¾® : C
.. l = C -C l : ¬¾® etc.
|
..
| 13. (a) Amino group is activating while nitro group is
:Cl
. .: :Cl
. .: deactivating. Hence, correct order is
aniline > benzene > nitrobenzene.
Since F lies in the 2nd period, it has no d orbital, hence
I > II > III
such dispersal of electrons of C is not possible.
Remember resonance stabilization is generally more NH2 NO2
important than the inductive effect.
5. (c) More is the electron - deficiency on the carbonyl
carbon, greater will be the reactivity of the carbonyl I II III
compounds towards nucleophilic addition. –NO2 is an electron attracting group hence decrease
O the electron density on ring, whereas –NH2 group is
electron releasing group hence increases electron
CH3CH 2CH 2CH 2CHO > > CH3CH2COCH2CH3 density on ring. Benzene is also electron rich due to
delocalization of electrons.
6. (d) When (–) 2-bromooctane is allowed to react with
sodium hydroxide under conditions, where second 14. (b) H 2 C = CHCl is capable of showing resonance which
order kinetics are followed, (+) 2 octanol is obtained. develops a partial double bond character on C–Cl bond,
thereby making it less reactive toward nucleophilic
C6H13 H C6H13 C6H13 substitution.

OH – .. _
H C Br HO -----C -----Br HO C H + Br
H 2 C = CH - Cl
.. : H 2 C - CH = Cl+
CH3 CH3 CH3
(–) 2-Bromooctane Transition state (+2) 2–octanol
EBD_7327
296 CHEMISTRY

.. H
H + 25. (b) % of chlorine = 35.5 ´ Mass of AgCl ´ 100
15. (a) C C 143.5 Mass of substance

..
H H

..
35.5 0.287
H = ´ ´100 = 71.71%
Carbocation Alkyl group 143.5 0.099
+
Hyperconjugation in CH2CH3 Å
H 26. (b) Carboxylic acids are stronger acids than - NH 3
H
because the corresponding conjugate base (–COO–) is
H2C – CH = CH 2 H2C – CH2 more stable than –NH2. Hence Y is the strongest acid.
Since – COOH has – I effect which decreases with
Hyperconjugation in propene Hyperconjugation
in free radical distance, therefore, effect is more pronounced in Z than
16. (b) HCOO – exists in following resonating structures in X. As a result, Z is more acidic than X. Hence the true
O O option is Y > Z > X.
|| | 27. (c) Shape of transition state is as follows:
,
H , C, O « H , C < O
Hence in it both the carbon oxygen bonds are found R
equal. Nu C X
17. (d) Lone pair of electrons present on the nitrogen of benzyl
amine is not involved in resonance. H R
COOH
4 2 Trigonal bipyramidal
18. (b) 5 . Here C 2 is chiral, hence optical CH 3
3 1 + +
isomerism is possible. However, the doubly bonded C4 28. (c) C 6 H5 C H 2 > CH >
has two similar (–CH3) groups, hence geometrical CH 3
isomerism is not possible.
19. (c) Metamerism shown among compounds of the same 23°
functional group. Benzyl
+ +
20. (c) The stronger the acid, the weaker the conjugate base > F - CH 2 -- C H
F - CH 2 - CH 2 - C H
formed. | |
The acid character follows the order : CH 3 CH 3
CH3COOH > C6H5OH > H2O > CH3OH 2° (more stable)
The basic character will follow the order F is more away from +ve charge
CH3COO– < C6H5O– < –OH < CH3O– 29. (d) Order of stability of carbocation 3° > 2° > 1° and I is the
Hence, decreasing order of rate of given reactions with best leaving group among halogens.
Nu– is 30. (d) Since diazo compounds may lose nitrogen in the form
D > C >A> B of nitrogen gas, they sometimes do not respond
21. (d) Lassaigne’s test for nitrogen.
22. (a) CH3* CH (OH)COOH
-OH
¾¾¾® CH 3CH(H )COOH 31. (d) Lone pair present at X can enter in the ring. This gives
Lactic acid +H
No chiral carbon rise to double bond character in C – X bond attached
Chiral carbon is present
(optically active)
(optically inactive ) to the ring .
32. (a) In given structures –
After releasing H+ from (ii) group it forms a more stable
23. (d) resonating structure of carboxylate ion. In case of (i)
the anion formed after release of H+ is more stable due
to present of e– withdrawing NO2 in benzene ring. In
(iii) phenoxide ion formed after release of H+ is more
stable in comparison to Acetylide ion of group (iv)
which is formed after releasing of (iv) H+ ion so acidic
structure of Ka is
(ii) > (i) > (iii) > (iv)
33. (b)
34. (d) Na 2 [Fe(CN)5 NO] + S2 - ® Na 4 [Fe(CN)5 NOS]
Sodium thio-nitroprusside
(violet)

24. (c) Because of partial double bond character (due to 35. (d) The amount of s-character in various hybrid orbitals is
resonance), C – H bond energy is less for toluene. as follows.
sp = 50%, sp2 = 33% and sp3 = 25%
Organic Chemistry : Some Basic Principles and Techniques 297

Therefore s character of the C – H bond in acetylene G releases electrons, intensifies –ve charge,
(sp) is greater than that of the C – H bond in alkene (sp2 destabi l izes car bani on, retards SN reaction
hybridized) which in turn has greater s character of the (deactivation)
C – H bond than in alkanes. Thus owing to a high s NO2 is activating group and CH3 and OCH3 are
character of the C – H bond in alkynes, the electrons deactiving group.
constituting this bond are more strongly held by the Hence, the correct order of nucleophilic substitution
carbon nucleus with the result the hydrogen present reactions
on such a carbon atom can be easily removed as proton.
Cl Cl Cl
The acidic nature of three types of C – H bonds follows
the following order Cl
-C º C - H > - C = C - H > - C - C - H
> > >
Further, as we know that conjugate base of a strong
acid is a weak base, hence the correct order of basicity
is NO2 CH3 OCH3
- - -
H - C º C < CH2 = CH < CH 2 - CH3 39. (b) The organic compounds which have chiral carbon atom
36. (b) Due to + M effect of – OH group and hyperconjugation (a carbon atom attached to four different group or atoms
of – CH3 group. and do not have plane of symmetry rotate plane
37. (c) Because of high electronegativities of the halogen atom, polarised light.
the carbon halogen (C – X) is highly polarised covalent CHO
bond. Thus, the carbon atom of the C – X bond becomes |
HO - C*- H (C* is asymmetric carbon)
a good site for attack by nucleophiles (electron rich |
species). Nucleophilic substitution reactions are the CH 2 OH
most common reactions of alkyl halides. 40. (b) The absolute configuration is (R, R)
38. (a) In SN Ar reactions, a carbanion is formed as an intermediate, (using priority rules to get the absolute configuration)
so any substituent that increases the stability of carbanion So the correct answer is (b)
and hence the transition state leading to its formation will 41. (b) In option (b) the complex formed is with benzene
enhance the SNAr reactions. To compare the rates of whereas in other cases it is formed with nitrobenzene
substitution in chlorobenzene, chlorobenzene having with –NO2 group in different position (o-, m-, p-). The
electron-withdrawing group, and chlorobenzene having complex formed with nitrobenzene in any position of
electron-releasing group, we compare the structures –NO2 group is less stable than the complex formed
carbanion I (from chlorobenzene), II (from chlorobenzene with benzene so the correct answer is (b). The most
containing electron-withdrawing group) and III (from stable complex has lowest energy.
chlorobenzene containing electron-releasing group). Cl
– – – –
Z Cl Z Cl 42. (b) C Cl > C6H5CH2 > (CH3)2 CH > (CH3)3C
Cl
–ve charge –M effect +I effect of CH 3 group
– – highly dispersed delocalises intensifies the –ve charge
due to – I effect –ve charge

43. (d) The correct order of basicity is


G
RCOO - < CH º C - < NH 2 - < R -
I II
44. (a) Due to similar charges on adjacent atoms, the structure
G withdraws electrons, neutralises (disperses) –ve (a) is least stable.
charge of the ring, stabilises carbanion, facilitates 45. (a) Anti addition of Br2 on trans alkene provides meso
SN reaction (activation effect) compound.
CH3
Z Cl H 3C H
CCl4 H Br
C= C + Br2 ¾¾ ¾®
– H CH 3 H Br
CH3
Therefore, no. of stereoisomers = 1
G
46. (d) The correct stability order is
III I > III > II > IV
EBD_7327
298 CHEMISTRY

Note : Carboxylic acid (—COOH) has more priority


+ + than ketone ( C= O).
O O 3. (b) Cl
I III 1 NO2
+ M effect, 6 hyper-
conjugative H’s, + M, 3 hyperconjugative 2
2 CH 3 groups . H’s, one CH 3 group
3
(+ I effect) 4

CH3
+
+ 1-chloro-4-methyl-2-nitrobenzene
II IV For tri or higher substituted benzene derivatives, the
5 hyperconjugative 5 hyperconjugative H’s compounds are named by identifying substituent
H’s 2 R groups one R group positions on the ring by following the lowest locant
rule.
47. (d) Migrating tendency of hydride is greater than that of 4. (c) Electronegativity of carbon atom depends on its state
alkyl group. Further migration of hydride from C–2 gives of hybridisation. More the s-character more will be the
more stable carbocation (stabilized by +R effect of OH electronegativity.
group and +I and hyperconjugative effects of methyl sp3 < sp2 < sp
group). s-character : 25% 33% 50%
Thus, sp-carbon (CH3 – CH2 – C º *CH) has the highest
H H
electronegativity.
1 2 3+ 4 5
1, 2 hydride
H3C – C — C– C – CH3 ¾¾¾¾¾ ® 5. (c) Two or more compounds having the same molecular
shift
formula but different functional groups are called
OH H CH3 functional isomers. Functional isomer of alcohol is ether,
aldehyde is ketone and cyanide is isocyanide. But alkyl
H H halides do not show functional isomerism.
+
6. (d) Essential oils are insoluble in water and have high
H3C – C — C– C – CH3
vapour pressure at 373K but are miscible with water
..OH H CH3 vapour in vapour phase, it means these are steam
..

volatile. Thus, steam distillation technique is a suitable


More stable carbocation method for the extraction of essential oils.
48. (a) It is fact that aniline is better nucleophile than anilium 7. (d) Thin layer chromatography (TLC) involves separation
ion. Anilium ion contain +ve charge, which reduces the of substances of a mixture over a thin layer of an
adsorbent coated on a glass plate.
tendency to donate lone pair of electron C6 H5 NH3+ .
A thin layer of an adsorbent is spread over a glass
Anilium ion
plate and glass plate is placed in an eluant. As eluant
Therefore , both assertion and reason are true. rises, components of the mixture move up along with
49. (c) The assertion that trans-2 butene reacts with Br 2 to the eluant to different distances depending on their
product meso-2, 3-dibromobutane is correct but it does degree of adsorption and separation takes place.
not involve syn-addition of Br 2. Therefore, TLC technique will give the best results in
50. (d) Resonance structures contain the same number of identifying the different types of ink used at different
unpaired electrons. However, they differ in the way of places in the documents.
distribution of electrons. 8. (b) Partition chromatography is based on continuous
EXERCISE - 3 differential partitioning of components of a mixture
between stationary and mobile phases. Paper
Exemplar Questions chromatography is a type of partition chromatography.
CH3 9. (a) Stability of the given cations can be understood by
7 6 5 4| 3 2 1
the following structures:
1. (a) CH3 – CH2 – CH2 – C – CH – CH2 – CH3 + + ..
| | CH3 CH CH3 ; CH3 - C H - O. . - CH3 ;
(I) (II)
CH3C2H5 + I – effect of the two Strong + R – effect of – OCH3
3-ethyl-4, 4-dimethyl heptane methyl groups stabilises group stabilises the carbocation
the carbocation
O O +
2. (d) 5 || 3 2 1 || CH 3 - C H - CH 2 ®- OCH 3
CH3 -4C - CH 2 - CH 2 - C - OH (III)
4-oxopentanoic acid - I effect of -OCH 3 group
destabilises the carbocation
Organic Chemistry : Some Basic Principles and Techniques 299

Hence, the correct order of decreasing stability of 15. (b) Addition of HCl to an alkene takes place in two steps as
carbocation will be : II > I > III follows:
10. (b) 4 3
Step I : p-bonds creates an electron cloud, electrophile
H3C — CH — CH — CH3 (H+) from H — Cl attacks the electron cloud and a
| | carbocation is formed.
5 CH 2 CH2
2
| | +
C—
+
C—C
H —C
6 CH3 1 CH3


3, 4-dimethylhexane H
(carbocation)
11. (a) Electronegativity of Cl, Br, C and Mg follows the order
Step II : The chloride ion attacks the carbocation
Cl > Br > C > Mg, thus chlorine has the greatest –I-
formed.
effect and disperse the positive charge on ‘C’ atom
most effectively.
Å –
Hence, * CH3 — CH2 — Cl has the greatest positive C—C + Cl C—C
charge. | | |
H Cl H
12. (d) In all the given ionic species, the negative charge is
dispersed which stabilises them. Here, the negative NEET/AIPMT (2013-2017) Questions
charge is dispersed by two factors, i.e., +R-effect of
OH
the carboxylate ion (conjugation) and – I-effect of the
halogens. 3 2
1
H3C COOH
As it can be clearly seen in the given structures, that 16. (a) 4
6
+R-effect is common in all the four species, therefore, 5 CH 3
overall dispersal of negative charge depends upon the
IUPAC name of th e structure is 3-eth yl-2-
number of halogen atoms and their electronegativity. hydroxy-4-methylhex-3-en-5-ynoic acid
Since, Fluorine has the highest electronegativity and 17. (c) Decreasing order of deactivating effect of the given
two F-atoms are present in option (d), thus, dispersal m-directing group is
of negative charge will be maximum in it, thus it is most NO2 > – CN > – SO3H > – COOH
stable. –NO2 group is most deactivating group due to
O O

strong – E, – I and – M effects.
F || F 18. (b) –NO2 is a powerful electron withdrawing group. Its


CH C—O CH C—
—O º presence on ring makes the ring less active.
F F CH 3
F O 19. (d) CH - CH 2 - (isobutyl group)
CH C –
CH 3
F O
20. (b) Greater the number of e– donating alkyl groups
Note : In structure (a), methyl group increases the (+I effect), greater will be the stability of carbocations.
density on C-atom.
21. (c) Conformers are form of stereoisomers in which isomers
13. (c) When an electrophile attacks CH3 – CH = CH2, there can be interconverted by rotation about single bonds.
are two possibilities of an intermediate formed: I and II are staggered and eclipsed conformers
+ Å respectively.
CH3 – CH = CH2 + H ® CH3 – C H – CH3 22. (c) Stability depends on number of hyperconjugative
(2°carbocation) structure.
+ Å
CH3 – CH = CH2 + H CH 3 - CH 2 - CH 2 23. (b) In the carbonium ion the carbon atom carrying the
(1° carbocation) positive charge is sp2 hybridized.
As 2° carbocation is more stable than 1° carbocation 24. (c) The prussian ferric ferrocyanide blue colour is of Fe4
thus first addition is more feasible. [Fe(CN)6]3
Note : Stability of carbocations is the basis of 26. (N) Out of the given four compounds only (iv) compound
Markownikoff's rule. is chiral and hence only this compound will undergo
racemisation. Therefore none of the given option is
14. (b) Arrow denotes the direction of movement of electrons.
correct.
Å
CH 3 - Br ¾ ¾
® CH 3 + Br 26. (a) 10 ml, 1 M H2SO4 = 20 ml, 1 M NH3
Q wt of N in one mole NH3 = 14
Since, Br is more electronegative than carbon, hence
heterolytic fission occurs in such a way that CH3 gets \ 20 × 10– 3 mol NH3 ¾®
a positive charge and Br gets a negative charge. 20 × 10– 3 × 14 nitrogen
EBD_7327
300 CHEMISTRY

\ 0.75 g of sample contains 273 ´ 700 ´ 40


= = 33.52 mL
14 ´ 20 ´ 10 -3 300 ´ 760
= ´ 100 = 37.33% Percentage of nitrogen
0.75
27. (c) H 28× volume of N 2 at STP ×100
= 22400 × wt. of organic substance
C COOH 28 ´ 33.52 ´ 100
CH3 = = 16.76%
OH 22400 ´ 0.25
Four different Substituent, only one chiral centre. 33. (d) CH3—C º C
Hence only enantiomers are possible. No.of s bp - 1 ù
28. (a) Nucleophile is a species that provide electron while lp -1 û
2 & hybridisation is sp
species which are deficient of electrons termed as lewis
acid, hence nucleophiles are usually lewis bases. Br Br
29. (c) All of these compounds show tautomerism
H3C CH3 H3 C CH3
34. (b) is optically active

H I I
O OH due to absence of plane of symmetry and center of
CH3 CH3 symmetry.
CH3 CH3 35. (d) In staggered conformation any two hydrogen atoms
on adjacent carbon atoms are as far apart as possible
H
there by minimising repulsion between the electron
O
O OH
OH
H clouds of s-bonds of two non-bonded H-atomic
H
(torsional strain)
H HH
CH3 CH3
H H
CH3 CH3
O OH
30. (b) Only structure (III) has hyperconjugation with free H
radical. H H H H
H
So, hyperconjugation is possible in III only. H
– Staggered form Eclipsed form
31. (b) p bond is transferred after leaving Cl
Å No torsional strain
CH3 – CH=CH.CH2 —Cl CH3 – CH=CH—CH2 36. (c) Steam distillation is the most suitable method of
Å separation of 1 : 1 mixture of ortho and para nitrophenols
CH 3—CH—CH=CH 2 as there is intramolecular hydrogen bonding in o-
32. (b) Wt. of organic substance = 0.25 g nitrophenol.
V1 = 40 mL, T1 = 300 K O
O
P1 = 725 – 25 = 700 mm of Hg C 2
P2 = 760 mm of Hg (at STP) 37. (d) H 3 4
1
T2 = 273 K 5
6
3-keto-2-methylhex-4-enal
P1V1 P2 V2 Aldehydes get higher priority over ketone and alkene
=
T1 T2 in numbering of principal carbon chain.
V2 (Volume of nitrogen at STP) 38. (c)
13 Hydrocarbons

CLASSIFICATION OF HYDROCARBONS
Depending upon the types of carbon-carbon bonds present, hydrocarbons are classified as

Hydrocarbons

Acyclic or open chain hydrocarbons Cyclic hydrocarbons


These compounds contain These compounds contain
open chains of carbon atoms closed chains or rings of
in their molecules carbon atoms in their molecules

Saturated hydrocarbons Unsaturated hydrocarbons Aromatic hydrocarbons Alicyclic hydrocarbons


Those compounds Those compounds Hydrocarbons which Hydrocarbons having
which contain C–C which contain C–C contain either fused or ring of three or more
and C–H single multiple bonds isolated benzene rings carbon atoms and
bonds. i.e., double bond, in their molecules properties similar to
For example : triple bond or both aliphatic hydrocarbons.
C2H6, CH3CH2CH3, etc. For example :
Ethane Propane CH3 - C º CH,
Benzene Naphthalene
Propyne
CH2 = CH2 , etc.
Ethylene

ALKANES
They have the general formula CnH2n+2. They exhibit structural and chain isomerism. All carbon atoms of alkanes are sp3 hybridized.
Methods of Preparation
(i) From unsaturated hydrocarbons:
Pt/Pd/ Ni
R - CH = CH - R '/ R - C º C - R '+ H 2 ¾¾¾¾® R - CH 2 - CH 2 - R '

Pt/Pd/Ni
CH 2 = CH 2 + H 2 ¾¾¾¾¾
® CH3 – CH3
Ethene Ethane
Pt/Pd/ Ni
CH3 – C º C – H + 2H 2 ¾¾¾¾¾
® CH 3CH 2CH 3
Propyne Propane
When the catalyst is Pt or Pd, the hydrogenation proceeds smoothly at ordinary temperature and pressure. With nickel as
catalyst, higher temperature (250º – 300ºC) and pressure are needed.
EBD_7327
302 CHEMISTRY

(ii) From alkyl halides: Note : If heavy water, D2O, is used, then the product is
(a) Reduction of Alkyl Halides : deuteroalkane (R – D).
R – MgX + DOD ¾® R – D + Mg(OD)X
Reductant
R - X ¾¾¾¾¾
® R - H+ HX (v) By the reduction of alcohols :
Alkyl halide Alkane
Re d P
Reductants : Zn–Cu couple/EtOH, Na–EtOH, Zn–HCl, R – OH + HI ¾¾¾® RI + H2O
150ºC
Pt or Pd or Ni/H2, Al–Hg/EtOH, LiAlH4 etc.
Zn /H + D
Example : CH3 – Cl + H2 ¾¾¾¾
® CH4 + HCl R – I + HI ¾¾® R – H + I2
(b) Wurtz Reaction : (vi) By the reduction of aldehydes and ketones :

D R Re d P
R
R – Br + 2Na + Br – R ¾¾¾¾¾
Dry Ether
® R - R + 2NaBr (a) C O + 4HI ¾¾¾ ® CH2 + H2O + 2I2
Alkane 150ºC
R' R'
Methane cannot be prepared by this method. The alkane (b) Clemmensen's reduction :
produced is higher and symmetrical. When the two
reacting alkyl halides are different, a mixture of three Zn - Hg
C O + 4HI ¾¾¾¾
® CH2 + H2O + 2I2
different alkanes is obtained. The Wurtz reaction is not conc. HCl
useful for preparing unsymmetrical alkanes.
(c) Wolff-Kishner reduction :
C2H5I + CH3I ¾¾ ® C2H6 + C3H8 + C4H10
R1 R1
Example : 2 2® H N.NH
C O ¾¾¾¾¾ C = N.NH2
(Hydrazine) R2
D , dry R2
CH3Br + 2Na + BrCH3 ¾¾¾® CH3 – CH3 + 2NaBr
ether
(c) Frankland's reaction : C H ONa
R1
¾¾¾¾¾
2 5 ® CH2 + N2
D 160ºC
RI + Zn + RI ¾¾
® R - R + ZnI R2
Alkane
(vii) By reduction of acids :
(iii) From carboxylic acids:
(a) Decarboxylation Re d P
RCOOH + 6HI ¾¾¾¾
® RCH3 + 3I2 + 2H2O
150ºC
D
RCOONa + NaOH (CaO) ¾¾
® R - H + Na 2CO3 (viii) From dialkyl zinc :
Sodium salt of Alkane
carboxylic acid R '- X
R2Zn ¾¾¾ ® R–R' + RZnX
Th e reaction is employed for steppin g down a
(ix) Corey-House Synthesis :
homologous series.
Ether
R – Br + 2Li ¾¾¾ ® R – Li + LiBr
Ex : CH3COONa + NaOH
Alkyl bromide Alkyl lithium
CaO, D 2R – Li + CuI ¾® R2LiCu + LiI
CH4 + Na2CO3 Lithium dialkyl
cuprate
(b) Kolbe’s electrolytic method R2LiCu + R' – X ¾® R – R' + RCu + LiX
Alkane
Electrolysis
2RCOONa + 2H2O ¾¾¾¾¾ ® Properties of Alkanes :
R – R + 2CO2 + H2 + 2NaOH Physical properties :
Methane cannot be prepared by this method. The side (i) They are non-polar molecules.
products are olefins, alcohols (particularly in alkaline (ii) C1 to C4 - gases; C5 to C17 - liquids, C18 and above solids.
solution) and esters. Presence of alkyl groups in (iii) Being non-polar, they are soluble in non-polar solvents.
a-position decreases the yield of alkane. (iv) Boiling point : The forces of attraction among the alkane
(iv) From Grignard reagent : molecules are the weakest van der Waal’s forces of attraction
Z- H as molecular mass increases, their van der Waal’s forces of
RMgX ¾¾¾ ® RH + Mg(Z)X attraction also increases resulting in increases of B.Pt.
ZH [compound containing active hydrogen]. Ex : Among isomeric alkanes, the boiling point decreases with
HOH, NH3, RC º CH, C6H5OH, CH3COOH, RNH2, R2NH, increasing branching.
Pyrrole, C2H5OH etc. Ex. n-Pentane > iso-Pentane > neo-Pentane
Hydrocarbons 303

(v) Melting Point : Note : Iodination of alkanes is reversible.


Unbranched alkanes containing 2, 4, 6, 8 etc. carbon atoms
have higher melting points than the unbranched alkanes CH4 + I2 CH3 – I + HI
containing 3, 5, 7, 9 etc. carbon atoms. Iodination may be carried out in the presence of an oxidising
M.P. with even no. of carbon atoms > agent such as HIO3, HNO3, HgO, etc. which destroys the HI
odd no. of carbon atoms as it is formed and so drives the reaction to the right.
(vi) Fuel Capacity : (b) Nitration :
Decreasing order of fuel capacity is
high temp.
more branched > less branched R – H + HO – NO2 ¾¾¾¾¾
® R – NO2 + H2O
> unbranched and lower mol.wt. > higher mol. wt. vapours form
Chemical properties : (c) Sulphonation :
(i) Substitution reactions R – H + HO – SO3H ¾¾® R – SO3H + H2O
(a) Halogenation :
Fuming H2SO4 Alkane sulphonic acid
hn (ii) Combustion and oxidation
R – H + X2 ¾¾® R – X + HX
Reactivity order of hydrogen atoms in alkanes is (a) Complete combustion :
3º > 2º > 1º æ 3n + 1 ö
Cn H2n +2 + ç ® nCO2 + ( n + 1) H2 O+ heat
÷ O2 ¾¾
Reactivity order of halogens is è 2 ø
F2 > Cl2 > Br2 > I2 (b) Incomplete combustion :
hn 2CH 4 + 3O 2 ¾¾ ® 2CO + 4H 2O + Heat
CH4 + Cl2 CH3Cl + HCl (c) Controlled Oxidation
Cl2 hu 1 Cu/523K
RCH3 + O 2 ¾¾¾¾¾ ® RCH 2 OH
2 100 atm
CH2Cl2 + HCl
Mo O
2 3 ® RCHO
RCH3 + O 2 ¾¾¾¾
Cl2 hu 54K,100 atm

CHCl3 + HCl 3 ( CH3COO )2 Mn


RCH 3 + O2 ¾¾¾¾¾¾¾ ® RCOOH
2 D
Cl2 hu
Note : Strong oxidising agents such as KMnO4 readily oxidise
a 3ºH atom to a hydroxy group.
CCl4 + HCl
Mechanism CH3 CH3
Step 1: Initiation : | Alk. KMnO 4 |
CH3 - C - H + [O] ¾¾¾¾¾® CH 3- C - OH
hu | |
Cl – Cl Cl + Cl
homolysis CH3 CH3
Step 2: Propagation :
Isobutane t-Butanol
hu
CH4 + Cl CH3 + H – Cl
(iii) Isomerisation
hu
CH 3 + Cl – Cl CH 3 – Cl + Cl anhy AlCl
CH3 ( CH 2 ) 4 CH3 ¾¾¾¾¾

®
CH3Cl + Cl HCl
CH2Cl + HCl
CH3 – CH – (CH2)2 – CH3 + CH3CH2 – CH – CH2 – CH3
CH2Cl + Cl – Cl CH2Cl2 + Cl
CH3 CH3
CH2Cl2 + Cl CHCl2 + HCl
For example : If we take n-heptane then it converts into more
CHCl2 + Cl – Cl CHCl3 + Cl stable form triptane.
Step 3: Termination: CH3
|
Cl + Cl Cl – Cl CH3 – (CH2)5 – CH3 ¾® CH3 – C –– CH – CH3
C H3 + CH 3 C H 3 – CH 3 | |
CH3 CH3
C H3 + Cl C H3 – Cl
n-heptane Triptane
EBD_7327
304 CHEMISTRY

(iv) Aromatization The infinite number of possible intermediate conformation


If an alkane having six or more carbon atoms is heated at between the two extreme conformation are referred to as skew
600ºC in the presence of a catalyst such as Cr 2O3 supported conformations.
over alumina or Pt, an aromatic hydrocarbon results. (ii) Newman Projections :
HH H HH
CH3
Cr2O3 or V2O5 H H
H2C CH3 or Mo2O3
773 K H
H2C CH2 10-20 atm H H
H H H H H
H H
CH2 Eclipsed Skew
H
(v) Reaction with steam Staggered
Ni Relative stabilities of conformations: staggered form of
CH 4 + H 2O ¾¾® CO + 3H 2 ethane is more stable because the e– clouds of C–H bonds
D
are as far apart as possible and thus, the repulsions are
(vi) Pyrolysis : minimum. In eclipsed form, e– clouds come closer thus causing
Decomposition of higher alkanes to lower alkanes is called increase in repulsions. Thus, staggered form is more stable
pyrolysis or cracking. than eclipsed form.
C6H12 + H2
ALKENES
773 K
C6H14 C4H8 + C2H6 They are unsaturated hydrocarbons containing atleast one double
bond. They have the general formula CnH2n. They exhibit chain
C3H6 + C2H4 + CH4 and position isomerism (structural isomerism). Besides these, they
CH3 CH CH2 H2 also exhibit geometrical isomerism (stereoisomerism).
C=C
400° – 600°C
CH3 CH2 CH3 Hybridisation : sp2
Bond angle = 120°
CH2 CH2 CH4 Methods of Preparation
Cracking is used for manufacture of petrol, petrol gas, oil (i) From alkynes :
gas, etc. R R¢
Pd/C
Conformations RC º CR ¢ + H 2 ¾¾¾¾¾¾¾¾® C=C
Sulphur compounds H H
The spatial arrangement of atoms which can be converted into or quinoline
cis-alkene
one another by rotation around a C–C single bond are called con-
formations or conformers or rotamers. If one CH3 group of ethane Na
R H
is kept constant and other methyl group is rotated through C – C RC º CR ¢ + H 2 ¾¾¾
® C=C
liquid NH3 H R¢
bond axis, an infinite no. of atomic arrangements are possible,
trans-alkene
which are called conformations.
The form in which H-atoms on the two carbons are as close as (ii) From alkyl halides: by b-elimination reaction
possible is called eclipsed. The form in which H-atoms on the two Alkyl halides on reaction with alcoholic KOH or NaOH forms
carbons are as far apart as possible is called staggered. corresponding alkenes.
Example : Conformations of ethane (CH3 – CH3): H H
(i) Sawhorse projections: R H
alc. KOH C=C
R–C–C–H
b a D H H
H H H X
H Reactivity of alkyl halides : 3º > 2º > 1º halide
C R – I > R – Br > R – Cl > R – F
Saytzeff's rule : According to this rule, “the dehydro-
H H halogenation of an alkyl halide results preferentially in the
H H
H 180º
¾¾¾
® production of more alkylated alkene, i.e., more stable alkene.”
H H
C (iii) from dihalides: by dehalogenation
(a) From vicinal dihalides :
H H CH 2 Br – CH 2 Br + Zn ¾¾ ® CH 2 = CH 2 + ZnBr2
Staggered conformation Eclipsed conformation CH 3CHBr – CH 2 Br + Zn ¾¾
® CH 3CH = CH 2 + ZnBr2
Hydrocarbons 305

(b) From gem dihalides : (ii) Addition of halogens:

X Zn X CCl4
C=C + X2 – C – C – (X = Cl, Br)
R-CH ++ + CH-R
-2 ZnX2 X X
X Zn X Reactivity order of halogens is Þ Cl2 > Br2 > I2
Note : A 5% solution of Br2 in CCl4 solvent is used for the
R-CH=CH-R detection of > C = C < bond since red brown colour of Br 2 is
discharged due to the formation of a colourless compound.
(iv) From alcohols by acidic dehydration: (iii) Addition of HX : (X = Cl, Br, I)
H H CH 2 = CH 2 + H – Br ¾¾
® CH3CH 2 Br
Conc. H2SO4 CH3 – CH – CH3 (major)
H–C–C–H CH2 = CH2 + H2O
D
Br (I)
H OH CH3CH = CH2 + HBr
Dehydrating agent :
conc. H2SO4 Þ 170ºC temp. CH3CH2CH2Br (minor)
P2O5 or H3PO4 Þ 200ºC (II)
Al2O3 Þ 350ºC
The addition of HX to an unsymmetrical alkene follows the
(vapour state)
Markovnikov’s rule according to which: ‘the negative part of
Order of reactivity of alcohols : = 3º > 2º > 1º alcohol
the addendum gets attached to that C-atom which has lesser
Dehydration occurs according to Saytzeff's rule
number of H-atoms’. Thus, (I) is the major product.
(b-elimination)
Mechanism:
D
CH3CH2CH2CH2OH + H2SO4 ¾¾
® Step 1: H3C – CH = CH 2 + HBr
160o C
+
CH3 – CH2 – CH = CH2 + CH3 – CH = CH – CH3 H
1–butene 2–butene
(minor product) (major/chief product) + – + –
H3C – CH2 – CH2 + Br H3C – CH – CH3 + Br
Properties of Alkenes 1° Carbocation 2° Carbocation
Physical properties : (less stable) (more stable)
(i) The first three members are gases, next fourteen liquids and Step 2:
higher ones solids.
+ –
(ii) They are insoluble in H2O and soluble in organic solvents. H3C – CH – CH3 + Br H3C – CH – CH3
(iii) Their boiling points, melting points and specific gravities
rise with the increase of molecular weight. The increase in Br
branching in carbon chain decreases the boiling point among 2-Bromopropane
isomeric alkenes. (major)
MP. and BP. µ mol. wt. However, in the presence of sunlight, air or an organic peroxide,
the addition of HBr to unsymmetrical alkene takes place
1
µ contrary to the Markovnikov rule. Ex:
branching in alkenes
( C6H5CO)
2 ® CH CH CH Br
O2
Chemical properties : CH3CH = CH 2 + HBr ¾¾¾¾¾¾¾ 3 2 2
(i) Addition of H2 : 1- Bromopropane
( major )
H2 Mechanism:
C=C –C–C– Step 1 : C6H5 – C – O – O – C – C6H5
Ni/Pd/Pt
H H O O
Heat of hydrogenation : The amount of heat liberated in the Benzoylperoxide
hydrogenation of one mole of alkene is known as heat of O
hydrogenation.
2C6H5 – C – O 2C6H5 + CO2
1 Benzoate free radical Phenyl
Stability of alkene µ free radical
Heat of hydrogenation
Step 2 : C6H5 + H – Br C6H6 + Br
EBD_7327
306 CHEMISTRY

Step 3 : Attack of Br free radical on propylene gives two free With acidic or hot KMnO4/K2Cr2O7
radicals. H3C H acidic K Cr O
2 2 7
C=C (CH3)2C = O + CO2 + H2O
CH3CH = CH2 + Br H or
H3C acidic KMnO4 Propan-2-one

4 KMnO /H+
CH3CH = CHCH3 ¾¾¾¾¾¾
® 2CH3COOH
CH3 – CH – CH2 CH3 – CH – CH2Br KMnO /H +
(more stable CH 2 = CH 2 ¾¾¾¾¾®4 2CO2 + 2H 2 O
Br
(less stable 2° free radical) With Osomium tetra oxide (OsO4) :–
1° free radical) H OH
· R – CH = CH2 + OsO4 ¾¾® R CH O O
Step 4 : CH3 – CH– CH 2Br + H – Br ¾¾
® Os
· CH2 O O
CH3CH 2 CH 2 Br + Br H OH
(Major product Osmic ester
formed)
® R – CH – OH + H2OsO4
2H2 O
¾¾¾
CH3CH – CH2 + H – Br CH3 – CH – CH3 +Br |
CH 2 – OH
Br Br cis-glycol
(Minor product formed)
Epoxidation :
Note : Out of HF, HBr, HCl and HI, only HBr shows peroxide
effect. C C - C-OH
C6 H5 COOOH H2O/OH
Symmetrical alkenes such as 2-butene. (CH3 – CH = CH – CH3), O
3-Hexene, etc., do not show peroxide effect. C Perbenzoic acid C HO-C
(iv) Addition of H2SO4 alkene oxirane
Addition of H2SO4 to unsymmetrical alkenes occur according (vii) Ozonolysis:
to Markovnikov’s rule. The addition of ozone on the double bonds and subsequent
hydrolysis of the ozonide formed is termed ozonolysis.
OSO3 H
| O
CH3 – CH = CH2 + H+ HSO4– ¾® CH3 – CH – CH3 C C
Zn
Propylene Isopropyl hydrogen C = C + O3
O O H 2O
sulphate
ozonide
OH
|
H 2O / D 2 C = O + H2O
¾¾¾¾
® CH3 – CH – CH3 + H2SO4
Isopropyl alcohol The hydrogen peroxide formed in the reaction is removed by
(v) Addition of water: adding zinc dust because it being an oxidising agent oxidises
Addition of H2O to unsymmetrical alkenes occurs according aldehydes into carboxylic acids. Ozonolysis of alkene offers
to Markonikov’s rule a valuable method of detection and location of double bonds
OH in complex organic molecules.
| (viii) Substitution reaction :
H3O+
R - CH = CH 2 + H - OH ¾¾¾® R - CH = CH3 a 500º–600ºC
(vi) Combustion and oxidation: CH2 = CH – CH 3 + Cl2 ¾¾¾¾¾ ®
(a) Combustion : Propylene
CH2 = CH – CH2Cl + HCl
3n Allyl chloride
Cn H 2n + O2 ¾¾ ® nCO2 + nH 2O + Heat
2 (ix) Isomerisation : Alkenes isomerise when heated at high
(b) Oxidation : temperature (500 – 700ºC) or at lower temperatures (200–300ºC)
With 1% alkaline KMnO4 (Baeyer’s reagent) in the presence of anhyd. AlCl3. or Al2(SO4)3/H2SO4
CH3 – CH2 – CH = CH2 ¾¾® CH3 – CH = CH – CH3
dil KMnO4 1-butene 2-butene
– C = C – + H2O + O –C–C–
273 K CH3
OH OH |
(cis-glycol) + CH3 – C = CH 2
vicinal glycol 2-methyl – 1-propene
Hydrocarbons 307

(x) Polymerisation (b) Gem dihalides :


High temperature and pressure
n(CH 2 = CH 2) –( CH 2 – C H2 –)n H X
Catalyst X
| | |
High temperature and pressure
¾¾¾ ®
R - C- C- H + alc. KOH - HX R – CH = C - H
n(CH3 – CH = CH2) – CH – CH2 – | |
Catalyst
CH3 n H X
NaNH

ALKYNES ¾¾¾¾ R–CºC–H
- HX
They are unsaturated HC’s with atleast one triple bond between (iii) From tetrahaloalkanes (Dehalogenation) :–
two C-atoms. Their general formula isCnH2n–2.
Dienes or Alkadienes X X
| |
They contain two >C = C< bonds. General formula CnH2n–2 R - C- C- H + Zn(dust) ¾¾
®
D

Types of alkadienes : | | 300°C

(i) Cumulative diene : In these dienes, the double bonds are X X


present on adjacent carbon atoms.
X X
Ex. CH2 = C = CH2 CH2 = C = CH – CH3 | |
D
Allene or 1, 2-Propadiene 1, 2-Butadiene R - C = C - H + Zn dust ¾¾
® R–CºC–H
300°C
(ii) Isolated dienes : In these dienes, the double bonds are Note : In the above reaction it is necessary that the four
separated by two or more C – C bonds. halogen atoms must be attached at vicinal carbons. If they
Ex. CH2 = CH – CH2 – CH = CH2 , are attached at the two ends, then the product cyclo alkene is
1, 4-Pentadiene obtained.
(iii) Conjugated dienes : The dienes in which double bonds are (iv) Synthesis of Higher Alkynes :
present in alternate manner are called conjugated dienes. With Na :
Ex. CH2 = CH – CH = CH2
1, 3-Butadiene liq.NH
3 ® 2 H - C º C - Na
2H - C º C - H + 2 Na ¾¾¾¾
Stability of alkadienes : - H2

Conjugated dienes are more stable than non-conjugated dienes. X-R


¾¾¾® H - C º C - R
This can be explained on the basis of hybridization and resonance.
All the carbon atoms in a conjugated diene (1, 3-Butadiene) are With Grignard Reagent :
sp2 hybridized. The delocalization of p electrons gives rise to H - C º CH + R - Mg - X ¾¾¾
- RH
® H - C º C - MgX
resonance in the molecule which as a result shows more stability
than the corresponding nonconjugated dienes. XR
¾¾¾
®H-CºC-R
- MgX 2
p p CH Å Q
CH2 CH CH 2 ¬¾® CH 2 – CH = CH – CH 2
Properties of Alkynes
1,3-Butadiene Physical properties
Q Å (i) The first three members are gases, C4 – C8 liquids and higher
¬¾® CH 2 – CH = CH – CH 2 ones solids.
They exhibit position isomerism. Hybridization of both carbons (ii) They are insoluble in H2O except few lower alkynes which
is sp. are partially soluble in H2O due to their polarizibility.
Methods of Preparation (iii) Their m.pt, b.pt and density increases with increase in molar
mass. M. pt. and b. pt decreases with branching.
(i) From calcium carbide:
Chemical properties
D
CaCO3 ¾¾® CaO + CO2 (i) Acidic nature: Alkynes are acidic as compared to alkenes
CaO + 3C ¾¾
® CaC2 + CO and alkanes. This is because the H-atoms in alkynes (ex:
ethyne) are attached to sp-hybridised C-atoms whereas they
® Ca ( OH ) 2 + C2 H 2
CaC2 + 2H 2O ¾¾ are attached to sp2 hybridised carbons in ethene and sp3
(ii) From dihalide (Dehydrohalogenation) : hybridised carbons in ethane. Due to maximum % of
(a) Vicinal dihalides : s-character in ethyne, the sp-hybridised orbitals of C-atoms
in ethyne have highest electronegativity. Hence they attract
H H X
| | | the shared e-pair of C–H bond of ethyne to a greater extent
R - C - C - H + alc. KOH ¾¾ ¾® R – CH = C - H than that of sp2 hybridised orbitals of C in ethene and sp3
| | - HX
X X hybridised orbitals of C in ethane. Thus, in ethyne, H-atoms
NaNH
2® R – C º C – H can be liberated as protons easily and hence ethyne (in
¾¾¾¾
- HX general alkynes) are acidic in nature.
EBD_7327
308 CHEMISTRY

1 (e) Addition of HCN :


® H – C º C - Na + +
HC º CH + Na ¾¾ H
2 2 Ba(CN)
CH 3 – C º C – H + NaNH 2 ¾¾
® R - C º CH + HCN ¾¾¾¾¾ 2 ® R - C = CH
2
or Cu 2 Cl2 |
CH 3 – C º C - Na + + NH 3 CN
Thus, we have the acidic strength of:
(f) Addition of water:
HC º CH > H2C = CH2 > CH3 – CH3
HC º CH > CH3 – C º CH > CH3 – C º C – CH3 Hg /H
2+ +

HC º CH + H2O Isomeri-
(ii) Addition reactions: CH2 = C – H
333K sation
(a) Addition of dihydrogen:
OH
Pt/Pd/Ni
® [ H 2C = CH 2 ]
HC º CH + H 2 ¾¾¾¾¾ CH 3 – C – H
H
2 ® CH - CH
¾¾¾ 3 3 O
(b) Addition of halogens: (iii) Polymerisation:
FeX 3 (a) Linear polymerisation : Under suitable conditions, po-
R – C º CH + X2 CH3CX = CHX lymerization of acetylene give polyacetylene or
X = F, Cl, Br
X2 polyethyne which is a high molecular weight polyene
with repeating units of (CH = CH – CH = CH) and is
X X
represented as –( CH = CH – CH = CH –)n . Thin films of
R–C–C–H polyacetylene can be used as electrodes in batteries.
(b) Cyclic polymerisation :
X X
(c) Addition of HX (X = Cl, Br, I) : HC
Alkynes form gem dihalide with HX because reaction CH
follows Markownikoff's Ist and IInd rule both. HC Red hot
or
Reactivity order : HI > HBr > HCl > HF HC CH Fe tube
873 K
R – C º C – H + H – X ¾¾
® R - C = CH 2
HC
Ethyne
(3 molecules)
X
HX
|
¾¾¾ ® R – C - CH3 AROMATIC HYDROCARBONS
| These hydrocarbons are also called ‘arenes’. They possess pleas-
X
gemdihalide ant odour and are thus called ‘aromatic compounds’. The aro-
matic compounds undergo substitution reactions more readily
Peroxide effect (Addition of HBr) :
than the addition reactions.
peroxide
CH3 – C º CH + HBr ¾¾ ¾¾
® CH3 – CH = CHBr Aromatic compounds have the cyclic system and contain (4n + 2)
1-bromopropene p electrons [Eric Huckel’s rule]
Peroxide / HBr
when n = 0, 1, 2, 3, 4 ................
¾¾¾¾¾¾ ® CH3 – CH(Br) – CH2Br Hence delocalisation alone is not sufficient criterion for aromaticity.
1, 2-dibromopropane Ex. (i) When n = 0, p e– = 2
(d) Reaction with Hypochlorous acid or Chlorine water : CH CH
OH Cyclopropenyl cation (Planar)
| CH
R–C Å
R– C || d– d+
d– d+ HO Cl Ex. (ii) When n = 1 ; p e– = 4 (1) + 2 = 6
||| + HO – Cl H– C
H– C |
Cl (Benzene)

OH
| H H
R – C – OH R – C =O H ¾¾¾ ® Å
Q –H
|
Cl
H – C – Cl –H2O H – C – Cl
| | Cycloheptatriene Cycloheptatrienyl cation
Cl Cl (Tropylium cation)
unstable
Hydrocarbons 309

Classification of Aromatic Compounds: (iv) Hydrolysis of benzene sulphonic acid :


(i) Aromatic homocyclic : SO3H
(a) Aromatic hydrocarbones [Arenes] :
(1) Monocyclic arenes : The Compounds having only one 150º –200ºC
+ HOH ¾¾¾¾¾
® + H2SO4
ring in the structural formula.
CH3 (v) From Grignard reagent :
MgBr
Benzene Toluene
Br
+ HO – H ¾® + Mg
(2) Polycyclic arenes : Aromatic hydrocarbons having more OH
than one ring in their structural formula are called
polycyclic arens. (vi) Reduction of benzenediazonium chloride :
N2Cl
Biphenyl (or Diphenyl)
C H OH
(a) ¾¾¾¾
2 5 ®
+ CH3CHO + N2 + HCl
(b) Aromatic hydrocarbon derivatives / Functional aromatic
compounds :
N2Cl
An aromatic compound with at least one functional group
in the structural formula is known as F.A.C. Cu +
(b) + H3PO2 + H2O ¾¾¾
®
NO2 NH2 OH

+ H3PO3 + N2 + HCl
Nitrobenzene Aniline Phenol Properties of Benzene
(Oil of mirbane) (Carbolic amine) Physical properties :
(ii) Heterocyclic aromatic compounds : (i) It is a colourless liquid with specific smell.
(ii) It is highly inflammable and burn with a sooty flame.
(iii) It is non-polar, immiscible with water and miscible with organic
solvents.
N S Chemical properties :
Pyridine
Pyridine(Azine)
(Azine) Thiophene (Thiole) (i) Electrophilic substitution reactions: The reaction can be
represented by :
Preparation of Benzene
H E
(i) Cyclic polymerisation of ethyne: Catalyst
+ E – Nu ¾¾¾¾
® + HNu
ºCH Red hot
CH Mechanism :
CH Fe Tube
or Å
º

CH CH 873 K (i) In first step, all electrophiles form E by substitution reaction


º

CH Å Q
E – Nu ¾® E + Nu &&
(ii) Decarboxylation of sodium benzoate :
(ii) In second step, electrophile attacks aromatic ring forming
COONa carbocation which is stablilised due to resonance.
H
CaO
+ NaOH D + Na2CO3 ¾¾¾¾¾¾
slow step
®
+ Å
E Rate determining step
(iii) Reduction of phenol:
OH E H E H E H H E

Å
D Å
¬® ¬® º
+ Zn + ZnO +
Å
Carbonium ion/(s-Complex)/
(Arenium ion)
EBD_7327
310 CHEMISTRY

(iii) In third step, elimination of proton from carbonium ion takes Benzoylation :
place to form substitution product.
Anhyd.
H E + ClCOC6H5 ¾¾¾¾
®
E AlCl3

NuQ O
¾¾¾® + H – Nu
+ Fast step
C
+ HCl
Product
Benzene gives electrophilic substitution while alkenes Benzophenone
undergo electrophilic addition. Note : The catalyst used may be AlCl3, FeCl3, SnCl4, BF3 or
(a) Nitration: ZnCl2 but anhydrous AlCl3 is commonly used. When excess
of electrophillic reagent is used for example treatment of
323-333K benzene with excess of Cl2 in presence of anhydrous AlCl3 in
+ Conc. HNO3 + Conc. H2SO4 dark.
NO2 Cl
Cl Cl
anhy. AlCl3
+ H2O + 6Cl2 + 6 HCl
dark, cold
Cl Cl
(b) Halogenations: Cl
Hexachlorobenzene (C6Cl6)
Cl
(ii) Addition reactions:
anhy. (a) Hydrogenation :
+ Cl2 + HCl
AlCl3
Ni/Pt/Pd
+ 3H2
Similarly benzene undergoes bromination when it is D
treated with Br2 in presence of FeBr3. Cyclohexane
(c) Sulphonation: Partial (Birch reaction) :– Reduction by Na/liquid NH3
SO3H
Na + liq.NH

+ H2 ¾¾¾¾¾
in C2 H5OH
+ H2SO4 (SO3) D
+ H2O
oleum 1, 4-Cyclohexadiene
(b) Halogenation :
(d) Friedel-crafts alkylation reactions: Cl
CH3 Cl Cl
UV
+ 3Cl2
anhy. 500 K
+ CH3Cl + HCl Cl
AlCl3 Cl
Cl
C2H5 Benzene hexachloride (BHC)
Similarly Br2 is used to prepare hexabromides. I2 cannot
anhy. give this reaction.
+ C2H5Cl + HCl (iii) Ozonolysis / Addition of ozone : When treated with ozone,
AlCl3
benzene triozonide is produced which on hydrolysis yields
(e) Friedel-Crafts acylation reaction: glyoxal.
COCH3
anhy. AlCl3
+ CH3COCl + HCl + 3O3 ¾¾®
D

COCH3

anhy. AlCl3 CH = O
+ (CH3CO)2O + CH3COOH 3Zn
D ¾¾¾ ® 3 | + 3H2O + 3ZnO
CH = O
Glyoxal
Hydrocarbons 311

(iv) Combustion: As a result, the ortho and para positions in the ring become the
points of high electron density or negatively charged. The new
15
C6 H 6 + O 2 ¾¾
® 6CO 2 + 3H 2O entering group which is an electrophile will prefer to come to ortho
2 and para positions rather than the meta position in the ring. These
Directive Influence of a Functional Group in groups are also called activating groups.
Monosubstituted Benzene Meta directing groups
Ortho and para directing groups: These groups direct the incoming group to meta position.
These groups direct the incoming group to o– and p– positions. Examples : – NO2, –CN, –CHO, –COR, –COOH, –COOR, –SO3H,
Examples : etc. In all meta directing groups the atom of the group which is
– NH2, – NHR, – NHCOCH3,– OCH3, – CH3, – C2H5, etc. directly attached to the ring has more electronegative atom linked
All o, p-directing groups except alkyl groups contain one or more to it by multiple bonds. The resonating structures for benzaldehyde
lone pair of electrons which are involved in resonance or configu- molecule are following.
ration with the p-electrons present in the ring. For example con- – – –
H O H O H O H O H O
sider resonance in aniline. C C C C C
..
NH2 NH2 + +

+
e–
Thus, overall density on benzene ring decreases making
further substitution difficult. The electron density at o– and
Å Å Å
NH2 NH2 NH2 p– positions is less than at m-position. Thus, E + attacks
the comparatively e– rich m-position resulting in meta substi-
Q Q
tution. These groups are also called ‘deactivating groups’.
Q
312

CONCEPT MAP
CHEMISTRY

EBD_7327
Hydrocarbons 313

1. Which of the following react with KMnO4 to produce pyruvic 9. The reaction of ethyl magnesium bromide with water would
acid? give
(a) butyne (b) propyne (a) ethane (b) ethyl alcohol
(c) 2-Pentyne (d) ethyne (c) ethyl bromide (d) ethyl ether
2. In Wurtz reaction if we take CH3Cl and C2H5Cl then product, 10. On mixing a certain alkane with chlorine and irradiating it
will be- with ultraviolet light, it forms only one monochloroalkane.
(a) propane + ethane This alkane could be
(b) propane (a) pentane (b) isopentane
(c) propane + ethane + butane (c) neopentane (d) propane
(d) propane + butane 11. 2-Methylbutane on reacting with bromine in the presence of
3. Reactivity of hydrogen atoms attached to different carbon sunlight gives mainly
atoms in alkanes has the order (a) 1-bromo-3-methylbutane
(b) 2-bromo-3-methylbutane
(a) Tertiary > Primary > Secondary
(c) 2-bromo-2-methylbutane
(b) Primary > Secondary > Tertiary
(d) 1-bromo-2-methylbutane
(c) Tertiary > Secondary > Primary.
(d) Both (a) and (b) CH
4. When CH3Cl and AlCl3 are used in Friedel-Crafts reaction,
12. ||| reacts with acetic acid in presence of Hg2+ to give :
CH
the electrophile is
(a) Cl+ (b) AlCl4– CH3
+ | CH ( CH 3COO ) 2
(c) CH3 (d) AlCl2+ (a) (b) |
C H(CH 3COO) 2
5. By which one of the following compounds both CH4 and CH (CH 3 COO ) 2
CH3 – CH3 can be prepared in one step? CH3
(a) CH3I (b) CH3OH |
(c) C H (CH COO) (d) None of these
(c) CH3CH2I (d) C2H5OH 2 3
6. In the free radical chlorination of methane, the chain initiating 13. Of the five isomeric hexanes, the isomer which can give two
step involves the formation of monochlorinated compounds is
(a) chlorine free radical (b) hydrogen chloride (a) 2-methylpentane (b) 2, 2-dimethylbutane
(c) methyl radical (d) chloromethyl radical. (c) 2, 3-dimethylbutane (d) n-hexane
7. Which one of the following reactions is expected to readily 14. Which one of the following contain isopropyl group?
give a hydrocarbon product in good yields ? (a) 2, 2, 3, 3-tetramethylpentane
Electrolytic
(b) 2-methylpentane
(a) RCOOK ¾¾ ¾¾¾
® (c) 2, 2, 3-trimethylpentane
oxidation
(d) 3, 3-dimethylpentane
(b) - + Br
RCOO Ag ¾ ¾
¾2® 15. Which types of isomerism is shown by 2, 3-dichlorobutane?
(a) Structural (b) Geometric
Cl
(c) CH 3 CH 3 ¾¾
¾2®
(c) Optical (d) Diastereo
hu
16. In mixture of iso-octane and n-heptane the percentage of
(d) (CH 3 ) 3 CCl ¾C¾2 H¾5OH
¾® n-heptane is 11, the octane number of this fuel is :
8. What will be the product in the following reaction? (a) 89 (b) 10
CH2 (c) 100 (d) 80
NBS 17. Which of the following isomers will have the highest octane
number?
CH 3
Br |
CH3 CH3 (a) CH3 - CH - CH 2 - C - CH 3
| |
(a) (b) CH 3 CH 3
Br (b) CH 3 - CH - CH - CH 2 - CH 2 - CH 3
| |
CH2Br CH3 CH 3 CH 3
(c) (d) (c) CH3 - CH - (CH 2 )4 - CH3
|
CH3
Br (d) CH3 – (CH2)6 – CH3
EBD_7327
314 CHEMISTRY

18. Aromatisation of n-hexane gives : 24. The highest boiling point is expected for :
(a) cyclohexane (b) benzene (a) iso-octane
(c) cycloheptane (d) toluene (b) n-octane
19. For the preparation of t-butyl methyl ether which one of the (c) 2, 2, 3, 3-tetramethylbutane
following methods should be recommended? (d) n-butane
25. The number of s and p bonds in
conc.H SO CH2 = CH – CH = CH2
(a) CH3OH+HO –C –(CH3)3 ¾¾¾¾¾®
2 4
(a) 8 s and 2 p bonds (b) 9 s and 1 p bonds
CH3 s
(c) 9 and 3 p bonds (d) 9 s and 2 p bonds
| +
HONa 26. 1, 2-Dibromocyclohexane on dehydrobromination gives :
(b) CH3 - C - Br + CH3OH ¾¾¾¾ ®
|
CH3 (a) (b)

CH3
| (c) (d) None of these
(c) CH3 Br + Na + O- - C - CH3
| HBr (i) Mg / ether
CH3 27. Propene ¾¾¾
® A ¾¾¾¾¾¾
®B
(ii) H 2 O / Boil
In the above sequence of reactions, B is
CH3 (a) propane (b) butane
| (c) propene (d) ethane
(d) CH3 - O- Na + + CH3 - C - Br 28. In reaction sequence
|
CH3 CH2OH
Hypochlorous R
CH 2 = CH 2 ¾¾ ¾¾¾ ¾® M ¾¾® |
20. At 130°C, normal butane reacts with bromine, the product
acid CH2OH
will be : molecule 'M' and reagent 'R' respectively are
(a) CH3CH2Cl and NaOH
Br (b) CH3CH2OH and H2SO4
|
(a) CH 3 - C - CH 3 (c) CH2Cl . CH2OH and aqueous NaHCO3
| (d) CH2 — CH2 and heat
CH 3
O
(b) CH 3 CH 2 CH 2 CH 2 - Br 29. Which of the following reagents convert propene to
Br 1-propanol?
| (a) H2O, H2SO4 (b) Aqueous KOH
(c) CH 3 - CH 2 - CH - CH 3
(c) MgSO4, NaBH4/H2O (d) B2H6, H2O2, OH–
(d) All of the above 30. The alkene that will yield 2, 3-dimethyl-2 butanol on hydration
21. When C 2 H 2 , CH 4 and C 2 H 4 pass through a test tube is :
which has ammonical Cu2Cl2, find out which gas comes out (a) (b)
unaffected from test tube?
(a) C2H2 and CH4 (b) C2H2 and C2H4
(c) C2H4 and CH4 (d) C2H2 (c) (d)
22. A hydrocarbon contains 10.5 g carbon and 1 g hydrogen. Its
31. Which alkene on ozonolysis gives CH3 CH2 CHO and
0.36 g has 1 L volume at 1 atm and 127°C, hydrocarbon is :
(a) C6H7 (b) C7H8 CH3 CCH3 ?
(c) C5H6 (d) None of these P
O
23. In cyclopropane, cyclobutane and cyclohexane, the common
group is CH3
(a) CH3CH2CH = C
| | CH3
(a) -C - (b) - CH
| | (b) CH3CH2CH= CHCH2CH3
(c) CH3 CH2 CH = CH CH3
| (d) CH 3 - C = CHCH 3
(c) -CH3 (d) CH 2 |
| CH3
Hydrocarbons 315

32. Aromatisation of n-heptane by passing over (Al2O3 + Cr2O3) 41. Which compound is aromatic ?
catalyst at 773 K gives:
(a) benzene (b) toluene OH
(c) mixture of both (d) heptylene
33. The compound (a) (b)
CH 3
|
CH 3— C = CH — CH 3 O
C6 H 5
on reaction with NaIO4 in the presence of KMnO4 gives |
(c) (d) Å
(a) CH3CHO + CO2 C
(b) CH3COCH3
(c) CH3COCH3 + CH3COOH HC = CC 6 H 5
(d) CH3COCH3 + CH3CHO
42. Cyclohexene on reaction with cold alkaline KMnO4 forms
34. Butene-1 may be converted to butane by reaction with
(a) Sn – HCl (b) Zn – Hg (a) trans-hexanediol (b) hexadiketone
(c) Pd/H2 (d) Zn – HCl (c) cis-hexanediol (d) None of these
43. Correct statement about 1, 3 -dibutene :
CrO Cl
35. C6H5CH3 ¾¾¾¾ 2 2
®Z (a) conjugated double bonds are present
In the given sequence Z is: (b) reacts with HBr
(a) benzaldehyde (b) toluic acid (c) forms polymer
(c) phenyl acetic acid (d) benzoic acid (d) All of these
36. Acid catalyzed hydration of alkenes except ethene leads to 44. The main product of the reaction is :
the formation of
(a) mixture of secondary and tertiary alcohols C
Na, NH 3 ( l )
C–R ¾¾¾¾¾ ® Product
(b) mixture of primary and secondary alcohols R 'OH
(c) secondary or tertiary alcohol
(d) primary alcohol C C–R CH2CH2R
(a) (b)
37. The compound most likely to decolourise a solution of
alkaline KMnO4 is :
(a) CH 3CH = CHCH 2 CH 2 CH 3 H H H R
(b) C10 H22 C C C C
(c) CH3 – CH3 (c) R (d) H
(d) (CH3)4C
38. The addition of HBr is easiest with :
45. Which of the following has highest knocking ?
(a) (CH3)2C = CH2 (b) CH3 - CH = CH 2
(a) Olefins (b) Branched chain olefins
(c) ClCH = CHCl (d) CH 2 = CH - Cl (c) Straight chain olefins (d) Aromatic hydrocarbons
39. Which of the production will be formed by the reaction? 46. Acetylene reacts with HCN in the presence of Ba(CN)2 to
Alkaline KMnO 4 yield :
H 2C = CH 2 ¾¾¾¾¾¾¾ ®X
Cold (a) 1, 1 -dicyanoethane (b) 1, 2-dicyanoethane
(a) Ethanol (b) Ethanoic acid (c) vinyl cyanide (d) None of these
(c) Ethylene glycol (d) Oxalic acid 47. Allene (C3H4) contains :
40. HOCl reacts on 3-methyl-2-pentene, the main product will (a) one double bond, one triple bond and one single bond
be : (b) one triple and two double bonds
(a) CH 3 - CH = C - CHOH - CH 3 (c) two triple and one double bonds
| (d) two double and four single bonds
Cl
48. Among the following, the aromatic compund is :
OH CH 3
| | (a) (b)
(b) CH 3 - CH - C - CH 2 - CH 3 + +
|
Cl
(c) (d)

Cl CH 3 49. A compound (X) on ozonolysis followed by reduction gives
| |
(c) CH 3 - CH - C - CH 2 - CH 3 an aldehyde, C2H4O and 2-butanone, compound (X) is
| (a) 3-methylpentene-2 (b) 3-methylpentene-3
OH (c) 3-methylhexene-3 (d) 3-ethylpentene-3
(d) None of these
EBD_7327
316 CHEMISTRY

50. Identify the product, P in the following reaction : 61. In the reaction
CH 3 - CH = CH 2 + NOCl ¾
¾® P CH 3 º C - - Na + + (CH3 ) 2 CH - Cl ¾¾®?
(a) CH 3 - C H - CH 2 (b) CH 3 - CH - C H 2 the product formed is :
| | | |
NO Cl Cl NO (a) propene (b) propyne
(c) propyne and propene (d) 4-methylpentyne-2
NO
| 62. Ammonical silver nitrate forms a white precipitate easily with
(c) C H 2 - CH 2 - CH 2 (d) CH 3 - CH 2 - C H (a) CH3C º CH (b) CH 3C º CCH 3
| | |
NO Cl Cl (c) CH3 CH = CH 2 (d) CH2 = CH2
51. Which of the following gives propyne on hydrolysis? 63. In the following sequence of the reations, what is D?
(a) Al4C3 (b) Mg2C3
CH3
(c) B4C (d) La4C3
[O] SOCl
52. The major product obtained on treatment of ¾¾¾
® A ¾¾¾¾
2® B

CH3CH2CH(F)CH3 with CH3O–/CH3OH is :


(a) CH3CH2CH(OCH3)CH3 NaN Heat
¾¾¾® 3 C ¾¾¾ ®D
(b) CH3CH = CHCH3
(c) CH3CH2CH = CH2 (a) Primary amine
(d) CH3CH2CH2CH2OCH3 (b) An amide
53. Which one of the following is the strongest bond? (c) Phenyl isocyanate
(a) > C = C < (b) — C º C — (d) A chain lengthened hydrocarbon
| | | 64. When acetylene passed through dil. H2SO4 in presence of
(c) — C — C = (d) — C — C — HgSO4, the compound formed is
| | | | (a) ether (b) acetaldehyde
Reagent (c) acetic acid (d) ketone
54. R - CH 2 - CCl 2 - R ¾¾¾¾® R - C º C - R
65. Identify the reagent from the following list which can easily
The reagent is distinguish between 1-butyne and 2-butyne
(a) Na (b) HCl in H2O (a) bromine, CCl4
(c) KOH in C2H5OH (d) Zn in alcohol.
(b) H2, Lindlar catalyst
55. Below, some catalysts and corresponding proceses/reactions
(c) dilute H2SO4, HgSO4
are matched. The mismatch is :
(d) ammonical Cu2Cl2 solution
(a) [RhCl(PPh3 )2 ] : Hydrogenation
66. (CH3)2C = CHCH3 + NOBr ¾¾¾ ® product. The structure
(b) TiCl 4 + Al(C 2 H 5 ) 3 : Polymerization of the product is
(c) V2O 5 : Haber-Bosch process (a) (CH3)2 C(NO) – CH(Br)CH3
(d) Nickel-Hydrogenation (b) (CH3)2 C(Br) – CH(NO)CH3
56. A compound is treated with NaNH2 to give sodium salt. (c) (CH3)2 CH – C(NO)(Br)CH3
Identify the compound
H
(a) C2H2 (b) C6H6 |
(c) C2H6 (d) C2H4. (d) H3C - C – CH - CH 3
57. Which of these will not react with acetylene? | |
(a) NaOH (b) Ammonical AgNO3 NO CH3
(c) Na (d) HCl.
67. Which C-atom is the most electronegative in this structure?
58. What is the product when acetylene reacts with
hypochlorous acid? III II I
(a) CH3COCl (b) ClCH2CHO CH 3 - CH 2 - C º CH
(c) Cl2CHCHO (d) ClCHCOOH. (a) I
59. Which of the following species participate in sulphonation (b) II
of benzene ring ? (c) III
(a) H2SO4 (b) SO3 (d) all are equal electronegative
(c) HSO3– (d) SO2–
68. What is the end product of the following sequences of
60. Which of the following is used for the conversion of 2-
hexyne into trans-2-hexene ? H 2O dil.H 2SO 4 Ni
operations CaC 2 ¾¾¾ ® A ¾¾¾¾¾ ++ ® B ¾¾¾ ®C
Hg H 2
(a) H2/Pd/BaSO4 (b) H2, PtO2
(a) Methyl alcohol (b) Acetaldehyde
(c) NaBH4 (d) Liq-NH3/C2H5OH
(c) C2H5OH (d) C2H4
Hydrocarbons 317

H
|
Br2 / hn Alcoholic (b) HC º C - C = CH 2
69. ¾¾¾¾ ® Major (X) ¾¾¾¾®
KOH / D
(c) CH 2 = CH – CH = CH 2

H - Br
(d) CH 2 = CH – CH 2 – CH 3
Major (Y) ¾¾¾® Major (Z) 77. Using anhydrous AlCl 3 as catalyst, which one of the
Peroxide
Major final product (Z) is following reactions produces ethylbenzene (PhEt)?
(a) H 3 C - CH 2 OH + C 6 H 6
Br (b) CH 3 - CH = CH 2 + C 6 H 6
(c) H 2 C = CH 2 + C 6 H 6
(a) (b)
Br (d) H 3 C - CH 3 + C 6 H 6
78. Nitration of toluene using fuming sulphuric and nitric acid
Br gives :
(a) p-nitrotoluene (b) o-nitrotoluene
Br (c) m-nitrotoluene (d) tri-nitrotoluene
(c) (d) 79. Which of the following compounds does not follow
Markownikoff’s law ?
(a) CH3CH = CH2 (b) CH2CHCl
70. The C - H bond length is minimum in the bond formed by (c) CH3CH = CHCH3 (d) None
(a) sp - s overlapping (as in alkynes) 80. Resonance energy of benzene represents :
(b) sp2 – s overlapping (as in alkenes) (a) stability of benzene
(c) sp3 – s overlapping (as in alkanes) (b) energy required to break C – C bond
(d) None of these (c) energy required to break C – H bond
71. The product(s) obtained via oxymercuration (HgSO4 +
H2SO4) of 1-butyne would be (d) energy for
O
P
(a) CH 3 - CH 2 - C - CH 3 light
81. C3H 8 + Cl 2 ¾¾¾® C3H 7 Cl + HCl is an example of
(b) CH3 – CH2 – CH2 – CHO which of the following type of reactions?
(c) CH3 – CH2 – CHO + HCHO (a) Substitution (b) Addition
(d) CH3CH2COOH + HCOOH (c) Elimination (d) Rearrangement
72. The ring structure of benzene was proposed by 82. Benzene can react with :
(a) Faraday (b) Davy (a) Bromine water (b) HNO3 + H2SO4
(c) Kekule (d) Wohler (c) H2O (d) CH3OH
73. Propyne on polymerisation gives 83. Toluene on treatment with CrO3 and (CH3CO)2O followed
(a) Mesitylene (b) Benzene by hydrolysis with dil HCl gives
(c) Ethyl benzene (d) Propyl benzene (a) benzaldehyde (b) benzoic acid
74. Ethyl alcohol is heated with conc. H 2SO4 . The product (c) phenol (d) phenylacetaldehyde
formed is : 84. Benzene can be obtained in the reaction
(a) Ethene + 1, 3-butadiene
(a) H3C - C - OC2 H5 (b) C2 H 6 (b) Trimerisation of ethyne
||
(c) Reduction of PhCHO
O
(d) All of these
(c) C 2 H 4 (d) C 2 H 2 85. Aromatic compounds burn with sooty flame because
75. Triple bond of ethyne is made of or Cylindrical shape of an (a) they have a ring structure of carbon atoms
alkyne is due to (b) they have a relatively high percentage of hydrogen
(a) Three s – bonds (c) they have a relatively high percentage of carbon
(b) Three p – bonds (d) they resist reaction with oxygen of air
(c) Two s and one p – bond 86. After ozonolysis of benzene (not hydrolysis), the product is
(d) Two p and one s – bond (a) Benzene triozonide (b) Glyoxal
76. Which of the following organic compound will give a mixture (c) Ethanediol (d) All of them
of 1-chlorobutane and 2-chlorobutane on chlorination? 87. The molecular formula of a compound in which double bond
is present between C & C :
(a) CH 3 - C H - CH = CH 2 (a) CnH2n + 2 (b) CnHn
|
CH3 (c) CnH2n (d) CnH2n–2
EBD_7327
318 CHEMISTRY

88. An organic compound ‘X’ - (molecular formula C 6 H 7 O 2 N ) 93. The IUPAC name of the compound having the formula
has six atoms in a ring system, two double bonds and a nitro CH º C – CH = CH2 is :
group as substituent; ‘X’ is: (a) 1-butyn-3-ene (b) but-1-yne-3-ene
(a) heterocyclic (c) 1-butene-3-yne (d) 3-butene-1-yne
(b) homocyclic and aromatic 94. Liquid hydrocarbons can be converted to a mixture of
(c) aromatic but not homocyclic gaseous hydrocarbons by :
(d) homocyclic but not aromatic (a) oxidation
89. The alkene R – CH = CH2 reacts readily with B2H6 and (b) cracking
formed the product B which on oxidation with alkaline (c) distillation under reduced pressure
hydrogen peroxide produces (d) hydrolysis
(a) R – CH2 – CHO (b) R – CH2 – CH2 – OH 1.Mg, Ether
(c) R - C = O (d) R - CH - CH2 95 In the following reaction, C6H5CH2Br ¾¾¾¾¾
+
® X, the
2.H 3O
| | |
CH3 product ‘X’ is
OH OH
(a) C6H5CH2CH2C6H5 (b) C6H5CH2OCH2C6H5
90. Compound X of molecular formula C4H6 takes up one
(c) C6H5CH2OH (d) C6H5CH3
equivalent of hydrogen in presence of Pt to form another
compound Y which on ozonolysis gives only ethanoic acid. 96. The IUPAC name of the compound
The compound X can be CH3CH = CHC º CH is
(a) Pent-l-yn-3-ene (b) Pent-4-yn-2-ene
(a) CH 2 = CH - CH = CH 2
(c) Pent-3-en-1-yne (d) Pent-2-en-4-yne
(b) CH2 = C = CHCH3 97. The compound formed as a result of oxidation of ethyl
(c) CH 3C º CCH 3 benzene by KMnO4 is
(d) All the three (a) benzyl alcohol (b) benzophenone
91. Predict the product C obtained in the following reaction of (c) acetophenone (d) benzoic acid.
butyne-1. 98. Which of the following reactions will yield
HI 2, 2-dibromopropane?
CH 3CH 2 - C º CH + HCl ¾¾ ® B ¾¾® C
I (a) CH3 – CH = CH2 + HBr ®
| (b) CH3 – C º CH + 2HBr ®
(a) CH3 - CH 2 - CH 2 - C - H (c) CH3CH = CHBr + HBr ®
| (d) CH º CH + 2HBr ®
Cl 99. The reaction of toluene with Cl2 in presence of FeCl3 gives
I
| predominantly
(b) CH 3 - CH 2 - CH - CH 2 Cl (a) m-chlorobenzene (b) benzoyl chloride
I (c) benzyl chloride (d) o- and p-chlorotoluene.
| 100. The hydrocarbon which can react with sodium in liquid
(c) CH3CH 2 - C - CH3 ammonia is
|
Cl (a) CH 3CH 2 CH 2 C º CCH 2 CH 2 CH 3
(d) CH3 - CH - CH 2CH 2 I (b) CH 3CH2C º CH
| (c) CH 3CH = CHCH3
Cl
92. Which of the compounds with molecular formula C5H10 (d) CH 3CH 2 C º CCH 2 CH 3
yields acetone on ozonolysis?
(a) 3-methyl-1-butene (b) cyclopentane
(c) 2-methyl-1-butene (d) 2-methyl-2-butene.

1. The alkene C6H10 producing (b) H+


C 6 H 6 + HOCl ¾¾
¾®
OHC(CH2)4CHO on ozonolysis is ZnCl
(a) Hexene-1 (b) Hexene-3
(c) C 6 H 6 + I - Cl ¾¾ ¾
¾2®

AlCl 3
(d) C 6 H 6 + Cl 2 ¾¾
(c) Cyclohexene (d) 1-Methylcyclohexene ¾®
2. Chlorination of benzene is not possible in the following 3. The conversion of ClCH=CHCl to Cl2CH–CHCl2 can be
reaction : carried out with
FeCl
(a) Cl2 (b) Cl 2/hn
(a) C 6 H 6 + Cl 2 ¾¾ ¾3 ® (c) Cl2/AlCl3 (d) Cl2/aq. NaOH
Hydrocarbons 319

4. A compound (X) (C5H8) reacts with ammonical AgNO3 to CHCH 3


give a white precipitate, and on oxidation with hot alkaline HO CH 2 CH 3 ||
KMnO4 gives the acid, (CH3)2CHCOOH, therefore X is –
(a) CH2 = CH – CH = CH – CH3 (a) (b)
(b) CH3 – CH = CH – CH2 – CH3
(c) (CH3)2CH – C º CH C º CH C CH 3
(d) (CH3)2C = C = CH2 CH 2 HO C
HO
5. Combustion of which of the following compound (in the
presence of excess of oxygen) does not result in the change (c) (d)
in hybrid state of C atom –
(a) CH4 (b) CH2 = CH2 10. In the following change,
(c) CH3 – CH3 (d) HC º CH
Pd
6. The reaction of propene with HOCl proceeds via the addition X+2 C=C ¾¾® +2 CH – CH
100°C
of
(a) H+ in the first step The X may be
(b) Cl+ in the first step (a) Cyclohexene (b) Cyclohexadiene
(c) OH– in the first step (c) Cyclogexatriene (d) None of these
(d) Cl+ and OH– in a single step 11. Me H Me
7. Consider the reaction
Light, 400K H
CH3 - CH - CH3 + Br2 ¾¾¾¾¾®
| Me H
CH3
Hydrogenation of the above compound in the presence of
CH3 poisoned palladium catalyst gives
| (a) an optically active compound
CH3 - CH - CH 2 Br + CH 3 - C - CH 3 (b) an optically inactive compound
| |
CH3 Br (c) a racemic mixture
(I) (II) (d) a diastereomeric mixture
(a) Compounds I and II are formed in nearly equal amounts 12. The nodal plane in the p-bond of ethene is located in
(b) Compounds I is formed in larger proportions (a) the molecular plane
(c) Compounds II is formed in larger proportions (b) a plane parallel to the molecular plane
(d) Relative amount of II cannot be predicted (c) a plane perpendicular to the molecular plane which bisects
8. Identify the end product Y – the carbon - carbon s -bond at right angle
(d) a plane perpendicular to the molecular plane which
AlCl
contains the carbon - carbon s -bond.
3
+ (CH3)2CHCH2Cl ¾¾¾® X
13. R - CH = CH 2 ¾Na liquid NH3
¾ ¾ ¾¾¾ ® R. CH 2 . CH 3
Et . OH
HNO /H SO
3 2 4® Y
¾¾¾¾¾¾ The reaction is called
330K (a) Birch's reduction (b) Clemensen's reduction
CH2–CH(CH3)2 CH2–CH(CH3)2 (c) Catalytic reduction (d) All of these
14. Consider the following reaction
·
(a) NO2 (b) H3C - CH - CH - CH3 + Br ¾¾ ® 'X '+ HBr
| |
D CH3
NO2 Identify the structure of the major product 'X'
·
C(CH3)3 C(CH3)3 (a) H3 C - CH - CH - CH 2
| |
D CH3
NO2 ·
(c) (d) (b) H 3C - CH- C - CH 3
| |
D CH3
NO2 ·
(c) H 3 C - C -CH - CH 3
9. The major product of the following reaction | |
D CH 3
O ·
H 3C - C º CH ¾CH
¾¾ 3 CH MgCl
¾2¾ ¾
¾® (d) H 3 C - CH - CH - CH 3
ether H 3O + |
CH3
EBD_7327
320 CHEMISTRY

15. Acetylenic hydrogens are acidic because 23. Reaction of one molecule of HBr with one molecule of
(a) Sigma electron density of C – H bond in acetylene is 1, 3-butadiene at 40°C gives predominantly
nearer to carbon, which has 50% s-character (a) 1-bromo-2-butene under kinetically controlled
(b) Acetylene has only open hydrogen in each carbon conditions
(c) Acetylene contains least number of hydrogens among (b) 3-bromobutene under thermodynamically controlled
the possible hydrocarbons having two carbons conditions
(d) Acetylene belongs to the class of alkynes with (c) 1-bromo-2-butene under thermodynamically controlled
molecular formula, CnH2n – 2. conditions
16. The probability factor for the replacement of hydrogen atom
(d) 3-bromobutene under kinetically controlled conditions
during chlorination/bromination is
24. The ease of nitration of the following three hydrocarbons
(a) more important in chlorination
(b) more important in bromination follows the order
(c) equally important in chlorination/bromination
CH3 CH3 CH3
(d) not definite
17. Which is the most suitable reagent among the following to
distinguish compound (3) from rest of the compounds ?
1. CH 3 - C º C - CH 3
CH3
2. CH3 - CH2 - CH2 - CH3
3. CH 3 - CH 2 C º CH CH3
4. CH3 - CH = CH2.
I II III
(a) Bromine in carbon tetrachloride
(b) Bromine in acetic acid (a) II = III » I (b) II > III > I
(c) Alk KMnO4 (c) III > II > I (d) I = III > I
(d) Ammonical silver nitrate. 25. Which of the reaction is not possible ?
18. Which is least reactive towards addition of HCl ?
(a) CH3CH2CH=CH2 (b) CH2=CH–CH=CH2 CH = CH2
(c) CH3CH2CºCCH3 (d) CH3CH=CHCHO AlCl3
19. In the presence of platinum catalyst, hydrocarbon A adds (a) + CH2 = CHCl
hydrogen to form n-hexane. When hydrogen bromide is
added to A instead of hydrogen, only a single bromo
C6H5
compound is formed. Which of the following is A? AlCl3
(b) + C6H5Cl
(a) CH 3 — CH 2 — CH = CH — CH 2 — CH 3
(b) CH 3 — CH 2 — CH 2 — CH = CH — CH 3
(c) CH 3 — CH = CH — CH 2 — CH 2 — CH 3 NH2 NH2
(d) CH 2 = CH — CH 2 — CH 2 — CH 2 — CH 3
AlCl3
20. A hydrocarbon A on chlorination gives B which on heating (c) + CH3Cl
with alcoholic potassium hydroxide changes into another
hydrocarbon C. The latter decolourises Baeyer's reagent and
on ozonolysis forms formaldehyde only. A is CH3
(a) Ethane (b) Butane
(c) Methane (d) Ethene (d) All the three
21. Which of the following is an aromatic species ? 26. In the following the most stable conformation of n-butane
NO2 CH3 is:

(a) H (b) H CH3 CH3


+ + H CH3 H H

(a) (b)
(c) (d) All of the three
N H H H H
H H CH3
22. Which one of the following alkenes will react faster with
H2 under catalytic hydrogenation conditions?
CH3 CH3
(R = Alkyl Substituent)
CH3 H
R R R H
(a) (b) (c) (d)
H H R H
H H
R R R R H H H
HH H3 C
(c) (d)
R H R R
Hydrocarbons 321

27. Which of the following conformers for ethylene glycol is CH3


most stable?
OH (b) C CH2 CH3
OH
H OH H H
OH CH3
CH3
(a) (b)
(c) C CH CH3
H H H H
H OH CH3 OH
OH OH CH3
OH H
(c) (d)
(d) C CH2 CH2

CH3 OH
H HO 30. Toluene is nitrated and the resulting product is reduced with
H H H H H H tin and hydrochloric acid. The product so obtained is
28. In the following reactions, diazotised and then heated wth cuprous bromide. The
CH3
+ /Heat
reaction mixture so formed contains
H
(a) CH
CH3–CH–CH–CH
–CH–CH–CH33 A + B (a) mixture of o- and p-bromotoluenes
Major Minor
OH products products (b) mixture of o- and p-dibromobenzenes
(c) mixture of o- and p-bromoanilines
HBr,dark
(b) A ¾¾¾¾¾¾¾¾
A¾¾¾¾¾¾¾¾ ® ® CC ++ D
D (d) mixture of o- and m-bromotoluenes
in absenceof peroxide
in absenceof peroxide æMajor ö æMinor ö
èproduct ø èproduct ø 31. In the following sequence of reactions, the alkene affords
the major products (A) and (C) are respectively : the compound ‘B’
CH3 CH3 O H O
CH 3 - CH = CH - CH 3 ¾¾3¾
® A ¾¾¾
2 ® B.
Zn
(a) CH2 2= C – CH2–2 CH33 and CH22– CH– CH22– CH33
The compound B is
Br
(a) CH3CH2CHO (b) CH3COCH3
CH3 CH3 (c) CH3CH2COCH3 (d) CH3CHO
(b) CH3–3 C = CH–CH33 and CH3– C – CH2– CH3 32. The treatment of CH3MgX with CH 3C º C - H produces
Br (a) CH 3 - CH = CH 2 (b) CH 3C º C - CH3
CH3 CH 3 H H
(c) | |
CH33– C = CH – CH33 and CH3 – CH – CH – CH 3 (c) CH 3 - C = C - CH3 (d) CH4
Br 33. One mole of a symmetrical alkene on ozonolysis gives two
CH3 CH3 moles of an aldehyde having a molecular mass of 44 u. The
(d) CH22= C – CH22– CH33 and CH33– C – CH22 – CH33 alkene is
Br
(a) propene (b) 1-butene
(c) 2-butene (d) ethene
29. In the following reaction :
34. Ozonolysis of an organic compound 'A' produces acetone
CH3 and propionaldhyde in equimolar mixture. Identify 'A' from
H 2O/H
+ the following compounds:
C CH —
— CH2 (a) 1 – Pentene
(b) 2 – Pentene
CH3 (c) 2 – Methyl – 2 – pentene
(d) 2 – Methyl – 1 – pentene
A B
Minor Product + Major Product 35. The non aromatic compound among the following is :
The major product is :
(a) (b)
CH3
S
(a) C CH CH3
(c) (d)
OH CH3

EBD_7327
322 CHEMISTRY

36. Cyclohexene on ozonolysis followed by reaction with zinc 43. Hyperconjugation is more pronounced in
dust and water gives compound E. Compound E on further (a) 2-methylpropene
treatment with aqueous KOH yields compound F. Compound (b) but-2-ene
F is
(c) 2, 3-dimethylbut-2-ene
(d) 2-methylbut-2-ene
(a) CHO (b) CHO
44. Reaction of hydrogen bromide with propene in the absence
of peroxide is a/an
CO 2 H (a) free radical addition
(c) COOH (d)
CO 2 H (b) nucleophilic addition
(c) electrophilic substitution
37. The synthesis of 3-octyne is achieved by adding a
(d) electrophilic addition
bromoalkane into a mixture of sodium amide and an alkyne.
The bromoalkane and alkyne respectively are 45. Among the three conformations of ethane, the order of
stability follows the sequence
(a) BrCH2CH2CH2CH2CH3 and CH3CH2C º CH
(a) eclipsed > gauche > staggered
(b) BrCH2CH2CH3 and CH3CH2CH2C º CH (b) eclipsed > staggered > gauche
(c) BrCH2CH2CH2CH2CH3 and CH3C º CH (c) staggered > gauche > eclipsed
(d) BrCH2CH2CH2CH3 and CH3CH2C º CH (d) gauche > staggered > eclipsed
38. The bond energy (in kcal mol–1) of a C–C single bond is DIRECTIONS for Qs. 46 to 50 : These are Assertion-Reason
approximately type questions. Each of these question contains two statements:
(a) 1 (b) 10 Statement-1 (Assertion) and Statement-2 (Reason). Answer these
(c) 100 (d) 1000 questions from the following four options.
39. The number of optically active products obtained from the (a) Statement-1 is True, Statement-2 is True, Statement-2 is a
complete ozonolysis of the given compound is : correct explanation for Statement-1
(b) Statement-1 is True, Statement-2 is True ; Statement-2 is
CH3 H NOT a correct explanation for Statement-1
(c) Statement-1 is True, Statement-2 is False
CH3 CH CH C CH CH C CH CH CH3
(d) Statement-1 is False, Statement-2 is True
46. Statement-1 : 1-Butene on reaction with HBr in the presence
H CH3
of a peroxide produces 1-bromobutane.
(a) 0 (b) 1 Statement-2 : It involves the free radical mechanism.
(c) 2 (d) 4 47. Statement-1 : CH4 does not react with Cl2 in dark.
40. 2-Hexyne gives trans-2-Hexene on treatment with : Statement-2 : Chlorination of CH4 takes place in sunlight.
(a) Pt/H2 (b) Li / NH3 48. Statement-1 : Iodination of alkanes is reversible.
(c) Pd/BaSO4 (d) Li AlH4 Statement-2 : Iodination is carried out in presence of iodic
41. The correct increasing order of acidity of the following acid.
alkynes
49. Statement-1 : All the hydrogen atoms in CH2 = C = CH2 lie in
(1) CH3 - C º C - CH3 one plane.
(2) CH3 - C º CH Statement-2 : Carbon atoms are sp2 and sp hybridized.
(3) CH º CH 50. Statement-1 : Tropylium cation is aromatic in nature
(a) 1 < 2 < 3 (b) 2 < 3 < 1
(c) 3 < 2 < 1 (d) 1 < 3 < 2 +
42. Which one of the following has the least boiling point?
Statement-2 : The only property that determines its aromatic
(a) 2, 2– dimethylpropane (b) n-butane
behaviour is its planar structure.
(c) 2-methylpropane (d) n-pentane
Hydrocarbons 323

Exemplar Questions A. H3C — C º C – B. H — C º C –


1. Arrange the following in decreasing order of their boiling C. H3C — CH 2
points. (a) A > B > C (b) B > A > C
1. n–butane 2. 2-methylbutane (c) C > B > A (d) C > A > B
3. n–pentane 4. 2, 2-dimethylpropane 9. Arrange the following alkyl halides in decreasing order of
(a) 1 > 2 > 3 > 4 (b) 2 > 3 > 4 > 1 the rate of b – elimination reaction with alcoholic KOH.
(c) 4 > 3 > 2 > 1 (d) 3 > 2 > 4 > 1
2. Arrange the halogens F2, Cl2, Br 2, I2, in order of their H
|
increasing reactivity with alkanes. A. CH3 - C - CH 2 Br
(a) I2 < Br2 < Cl2 < F2 (b) Br2 < Cl2 < F2 < I2 |
(c) F2 < Cl2 < Br2 < I2 (d) Br2 < I2 < Cl2 < F2 CH3
3. The increasing order of reduction of alkyl halides with zinc
B. CH3 — CH2 — Br
and dilute HCl is
C. CH3 — CH2 — CH2 — Br
(a) R – Cl < R – I < R – Br (b) R — Cl < R – Br < R – I
(a) A > B > C (b) C > B > A
(c) R – I < R – Br < R – Cl (d) R – Br < R – I < R – Cl
(c) B > C > A (d) A > C > B
4. The correct IUPAC name of the following alkane is
10. Which of the following reactions of methane is incomplete
H3 C - CH2 - CH - CH2 - CH2 - CH - CH2 - CH3
| | combustion?
CH CH 2 (a) Cu / 523 K /100 atm
2CH 4 + O 2 ¾¾¾¾¾¾¾¾
® 2CH 3OH
|
CH3 CH3 CH3 Mo O
(b) CH 4 + O 2 ¾¾¾¾
2 3 ® HCHO + H O
2
(a) 3, 6-diethyl-2-methyloctane
(b) 5-isopropyl-3-ethyloctane (c) ® C ( s ) + 2H 2 O ( l )
CH 4 + O 2 ¾¾
(c) 3-ethyl-5-isopropyloctane
(d) 3-isopropyl-6-ethyloctane (d) ® CO 2 ( g ) + 2H 2 O ( l )
CH 4 + 2O 2 ¾¾
5. The addition of HBr to 1-butene gives a mixture of products
A, B and C. NEET/AIPMT (2013-2017) Questions
Br C2H5
11. Which of the following chemical system is non aromatic?
C C [NEET Kar. 2013]
(A) H5C2 CH3 (B) H CH3
H Br (a) (b)
(C) CH3 - CH 2 - CH 2 - CH 2 - Br
The mixture consists of (c) (d)
(a) A and B as major and C as minor products S
(b) B as major, A and C as minor products 12. In the following reaction :
(c) B as minor, A and C as major products H SO
(d) A and B as minor and C as major products HC º CH ¾¾¾¾
2
2+
4 ® 'P'
Hg
6. Which of the following will not show geometrical isomerism?
Product ‘P’ will not give [NEET Kar. 2013]
F H F F (a) Iodoform test
(a) C=C (b) C=C
Cl D Cl Cl (b) Tollen’s reagent test
(c) Brady’s reagent test
H3C C2H5 CH3 CH3 (d) Victor Meyer test
(c) C=C (d) C=C
H5C2 CH3 C2H5 .
CH3 13. The radical, CH2 is aromatic because it has : [2013]
7. Arrange the following hydrogen halides in order of their
decreasing reactivity with propene. (a) 7 p-orbitals and 6 unpaired electrons
(a) HCl > HBr > HI (b) HBr > HI > HCl (b) 7 p-orbitals and 7 unpaired electrons
(c) HI > HBr > HCl (d) HCl > HI > HBr (c) 6 p-orbitals and 7 unpaired electrons
8. Arrange the following carbanions in order of their decreasing (d) 6 p-orbitals and 6 unpaired electrons
stability.
EBD_7327
324 CHEMISTRY

14. Which of the following organic compounds has same 20. The total number of p-bond electrons in the following
hybridization as its combustion product (CO2)? [2014] structure is : [2015]
(a) Ethane (b) Ethyne H H H
H3C
(c) Ethene (d) Ethanol CH3
H3C
15. The oxidation of benzene by V2 O5 in presence of air
produces : [2015 RS] H2C H CH3
(a) benzoic anhydride (b) maleic anhydride (a) 8 (b) 12
(c) benzoic acid (d) benzaldehyde (c) 16 (d) 4
16. 2,3-Dimethyl-2-butene can be prepared by heating which of 21. The reaction of C6H5CH = CHCH3 with HBr produces: [2015]
the following compounds with a strong acid ? [2015 RS] (a) C6 H5CH 2CHCH3
|
(a) (CH3)2 CH – CH – CH = CH2 Br
|
CH3 (b) C6 H5CH 2CH 2CH 2Br
(b) (CH3)3 C – CH = CH2 CH=CHCH3
(c) (CH3)2C = CH – CH2 – CH3
(d) (CH3)2CH – CH2 – CH = CH2 (c)
17. In the reaction with HCl, an alkene reacts in accordance with
the Markovnikov's rule, to give a product 1-chloro-1-
methylcyclohexane. The possible alkene is : [2015 RS] Br
CH2 CH3 (d) C 6 H 5 C HCH 2 CH 3
|
Br
(a) (b) 22. In the reaction
(1)NaNH 2 / liq.NH3 (1)NaNH 2 / liq.NH 3
H - C º CH X (2)CH CH Br Y
CH3 (2)CH3CH 2 Br 3 2
X and Y are [2016]
(a) X = 1-Butyne ; Y = 3-Hexyne
(c) (a) and (b) (d)
(b) X = 2-Butyne ; Y = 3-Hexyne
(c) X = 2-Butyne ; Y = 2-Hexyne
18. A single compound of the structure : [2015] (d) X = 1-Butyne ; Y = 2-Hexyne
CH3 CH3 23. Which one is the correct order of acidity ? [2017]
(a) CH º CH > CH3 – C º CH > CH2 = CH2 > CH3 – CH3
OHC C C
C H C (b) CH º CH > CH2 = CH2 > CH3 – C º CH > CH3 – CH3
O
H2 H2 (c) CH3 – CH3 > CH2 = CH2 > CH3 – C º CH> CH º CH
(d) CH2 = CH2 > CH3 – CH = CH2 > CH3 – C º CH > CH º CH
is obtainable from ozonolysis of which of the following cyclic
compounds ? 24. Predict the correct intermediate and product in the following
reaction : [2017]
H3C H3C CH3 H O, H SO
H3C - C º CH ¾¾¾¾¾¾
2 2 2 ® Intermediate ® product
HgSO4 (A)
(a) (b) (B)
H3C (a) A : H3C – C = CH B : H3C – C = CH2
| |
CH3 H3C OH SO4
(b) A : H3C – C = CH3 B : H3C – C º CH
(c) (d) CH3 ||
CH3 O
19. Given: (c) A : H3C – C = CH2 B : H3C – C = CH3
| ||
H3C CH3 H3C CH2 H2C CH2 OH O
(d) A : H3C – C = CH2 B : H3C - C - CH3
| ||
SO4 O
CH3 CH3 CH2
25. With respect to the conformers of ethane, which of the
(I) (II) (III) following statements is true ? [2017]
The enthalpy of the hydrogenation of these compounds will (a) Bond angle changes but bond length remains same
be in the order as : [2015] (b) Both bond angle and bond length change
(a) III > II > I (b) II > III > I (c) Both bond angles and bond length remains same
(c) II > I > III (d) I > II > III (d) Bond angle remains same but bond length changes
Hydrocarbons 325

Hints & Solutions


EXERCISE - 1 Br
9. (a) C 2 H 5 MgBr + H 2 O ¾
¾® C 2 H 6 + Mg
1. (b) Since Pyruvic acid has three carbons
OH
O
|| 10. (c) In neopentane all the H atoms are same (1º).
CH 3 - C - COOH , it can be prepared by propyne. CH3
|
2. (c) (1) CH3 Cl + 2Na + Cl - CH3 ® CH3 - CH3 CH 3 - C - CH 3
Ethane |
CH3
(2) CH3 Cl + 2Na + Cl - CH 2 - CH3
CH3 CH3
® CH3 - CH 2 - CH3 | |
Br2
Propane 11. (c) CH3 - CH - CH2 - CH3 ¾¾¾
® CH3 - C - CH2CH3
sunlight |
(3) CH3 - CH 2 Cl + 2Na + Cl - CH 2 - CH3 Br
2-bromo-2-methyl butane
® CH3 - CH 2 - CH 2 - CH3 Ease of replacement of H-atom 3° > 2° > 1°.
Butane
3. (c) The reactivity of H-atoms depends upon the stability CH Hg 2 +
12. (a) ||| + CH 3COOH ¾¾¾®
of free radicals follows the order :
CH
Tertiary > secondary > primary, therefore, reactivity of
H-atoms follows the same order, i.e., tertiary > secondary CH 2 CH 3
> primary. CH3COOH
|| ¾¾¾¾¾ ® |
Hg 2 + CH(COOCH3 ) 2
R R H H CH (CH3COO)
| | | |
R–C •
> R-C •
> R-C •
> H - C• CH3 CH3
| | | | | |
R H H H 13. (c) CH 3 - C H - C H - CH 3 . Since it contains only two
Tertiary Secondary primary free methyl
free radical free radical radical free radical types of H-atoms hence it will give only two mono
chlorinated compounds viz.
4. (c) CH3Cl + AlCl3 ¾
¾® CH3+ + AlCl4–
CH3 CH3
Electrophile | |
Here the electrophile is alkyl carbonium ion. Cl.CH 2 - C H - C H - CH 3
1- chloro - 2,3 - dimethyl butane
5. (a) CH3I is the compound.
CH 3 I + HI ¾
¾® CH 4 + I 2 CH3 CH3
| |
CH 3 I + 2 Na + ICH 3 ¾
¾® CH 3 .CH 3 + 2 NaI and CH 3 - C - C H - CH 3
|
hv Cl
6. (a) Cl2 ¾¾¾¾¾¾® 2 Cl• 2 -chloro - 2,3-dimethyl butane
Chain initiation
7. (a) Electrolysis of a concentrated aqueous solution of either 14. (b) CH 3 CH3
sodium or potassium salts of saturated mono-carboxylic |
acids yields higher alkane at anode. CH -, CH3 - CH - CH 2 - CH3
(2-methyl pentane)
CH 3
2RCOOK ¾¾ ¾ ¾¾® 2RCOO - + 2K +
Electrolyt ic
Oxidation Anode Cathode Isopropyl group
- –
At anode 2RCOO ® 2RCOO + 2e ¾
¾® It contains isopropyl group.
R — R + 2CO 2 Cl Cl
At Cathode +
2K + 2e ® 2 K – 15. (c) CH3 CH3. 2, 3-dichloro butane will exhibit optical
2K + H2O®2KOH + H2 ­ H H
isomerism due to the presence of two asymmetric
CH 2 Br
CH 2 NBS carbon atom.
8. (c) ¾¾¾ ® 16. (a) Octane number is the % of iso-octane in the mixture of
iso-octane and n-heptane which produces same
(Kolbe's Method) knocking effect as the fuel under observation.
EBD_7327
326 CHEMISTRY

Given % of n-heptane = 11% 21. (c) C2H4 and CH4 will come out unaffected. C2H2 forms
\ % of iso-octane = 100 – 11 = 89% copper acetylide with ammonical Cu2Cl2
\ Octane number is 89. Cu 2 Cl 2 + 2NH 4 OH ¾¾
® Cu 2 O + H 2 O + 2 NH 4 Cl
17. (a) More branched is the alkane, higher will be its octane
Cu 2O + C2 H 2 ¾¾
® Cu 2C 2 + H 2O
number because more branched chains have higher (copper acetylide)
antiknocting tendency than the corresponding straight
22. (b) Sum of masses of C and H = 10.5 + 1.0 = 11.5
chain hydrocarbons. Among the given options 2,2,4-
trimethyl pentane, will have the highest octane number, 10.5
% of C = ´ 100 = 91.3%
100. 11.5
18. (b) CH 3 - CH 2 - CH 2 - CH 2 - CH 2 - CH 3 1.0
% of H = ´ 100 = 8.7%
11.5
Aromatisation
¾¾¾¾¾¾
® Element % Ratio of atoms Simplest ratio
C 91.3 91.3/12 = 7.61 7.61/7.61 = 1 × 7 = 7
Benzene
H 8.7 8.7/1= 8.7 8.7/7.61 = 1.14 × 7= 8
Aromatisation is a process in which aromatic \ Empirical formula = C7H8
compounds are formed from open chain compounds.
19. (c) For the preparation of t-butylmethyl ether, the method PV
We know that n =
described in option (c) should be recommended. It is RT
due to the reason that alkoxides are not only
mass PV
nucleophiles but also strong bases. So, they tend to =
react with alkyl halides by elimination to yield alkenes. molar mass RT
Here the danger of competition between nucleophilic
2.81 1´1
substitution and elimination reaction increases with the = ; M = 92.168 » 92
M 0.082 ´ 400
tendency of alkyl halides to undergo elimination which
follows the order 3° > 2° > 1°. So, first of all we have to 84 + 8
select 1° halide and obviously the second choice will \ n= =1
92
be sodium t-butoxide.
\ Molecular formula = ( C7 H8 ) ´ 1 = C7 H8
CH3

CH3 C O– + CH3 – Br ¾¾
® 23. (d) ; ;

CH3
|
In all these we find CH2 group i.e. correct answer
CH3 |
is (d).
CH3 C O CH3+Br– 24. (b) For isomeric alkanes, the one having longest straight
CH3 chain has highest b.p. because of larger surface area.
p s p
25. (d) H s C s
C C s C H
s
Also the method described in option (a) is not suitable s s s s
as the ease of dehydration of alcohol follows the order H H H H
3° > 2° > 1°. So t-butanol will quickly give an alkene. As shown above, the given compound (1, 3-butadiene)
20. (c) The reaction of Br 2 on alkane proceeds via free radical contains 9s and 2p bonds.
mechanism; 2° free radical is more stable than 1° free Br
radical. - HBr
26. (b) ¾¾¾¾¾
®
Hence 2-bromobutane is major product. (alc)KOH
Br
Br2 1, 2-dibromo-
CH3CH 2CH 2 CH3 ¾¾¾® cyclohexane
130°C
n-Butane

g g - HBr
CH3CH 2 CHCH3 or CH 3CH 2CH 2 CH 2 ¾¾¾¾¾
®
(alc) KOH
2° free radical 1° free radical Br
(more stable) 1, 3-cyclo-
¯ hexadiene
CH3CH 2 CHBrCH 3
2-Bromobutane
Hydrocarbons 327

HBr
32. (b) n-heptane first undergo cyclisation by passing over
27. (a) CH 3 - CH = CH 2 ¾¾¾¾¾¾¾® Cr2O3 – Al2O3 at 770 K and 10 – 20 atm and gives
Markownikoff 's
propene addition take place methyl cyclohexane which on aromatization gives
toluene.
(i)Mg / ether
CH3 - CH - CH3 ¾¾¾¾¾¾¾ ® CH3
formation of
| Grignard reagent
Br CH2
iso-propylbromide
(A) CH2 CH3
Cr O - Al O
(ii)H2O / Boil ¾¾¾¾¾¾¾
2 3 2 3 ®
770 K,10 - 20 atm
CH3 CH - CH3 ¾¾¾¾¾¾®
hydrolysis of CH2 CH2 Cyclisation
| Grgnard reagent
MgBr CH2
n–heptane
Mg(OH)Br + CH3 - CH - CH3
| CH3 CH3
H
Propane Cr O - Al O
¾¾¾¾¾¾¾
2 3 2 3
®
(B) 770K,10 - 20atm
aromatisation
28. (c) We know that Cyclo toluene
NaHCO 3 hexane
CH2 CH2 + HOCl CH2OH CH2OH
CH3
CH2Cl CH2OH |
4 NaIO
33. (c) CH3 - C = CH - CH 3 ¾¾¾®
\ M = CH 2 OH - CH 2 Cl and R = NaHCO 3 KMnO 4

B H O
29. (d) 6 CH3 - CH = CH 2 ¾¾¾®
2 6
P
1, Pr opene ether, 0°C CH 3 - C - CH 3 + CH 3COOH
H O
34. (c) Alkenes combine with hydrogen under pressure and
2(CH3CH 2 CH 2 )3 B ¾¾¾®
2 2
in presence of a catalyst (Ni, Pt or Pd) and form alkanes.
OH -
H / Pd
Butene - 1 ¾¾2 ¾¾® Butane
6CH3CH 2 CH 2 OH + 2H 3 BO 3
35. (a) Toluene on oxidation with mild oxidising agent like
Pr opanol
chromyl chloride (CrO2Cl2) gives benzaldehyde.
30. (c) is the only compound among the given options CH3 CHO
which gives 2, 3-dimethyl-2-butanol as shown in the
table : CrO Cl
¾¾¾¾®
2 2
Compound Product obtained on hydration
toluene benzaldehyde
OH
2-methyl-2-propanol 36. H2SO4
(c) CH3 - CH = CH 2 + H 2O ¾¾¾¾ ® CH3 - CH - CH3
|
OH
2° alcohol
OH
2-methyl-2-butanol CH 3 CH3
| |
H 2SO 4
CH 3 - C = CH 2 + H 2 O ¾¾¾¾
® CH 3 - C - CH 3
|
OH OH
3° alcohol
2,3-dimethyl-2-butanol
Addition follows Markownikoff’s rule.
37. (a) Only alkenes and alkynes (or compounds having
OH C = C or C º C) decolourise alkaline KMnO4 solution.
3-methyl-2-butanol Only (a) is alkene, other three are alkanes.
31. (a) 38. (a) Addition of HBr to alkene is an electrophilic addition
reaction. The reaction proceeds via formation of
H CH3 H O CH3
O3
carbonium ion. Since 3° carbonium ion is most stable ,
CH3– CH 2– C C CH 3– CH 2– C– C hence the reactant which involves formation of 3°
CH3 carbonium ion will react most easily with HBr
CH3 O O
O +
H+
( CH3 ) 2 C = CH 2 ® ( CH3 ) 2 C - CH3
¾¾¾
CH 3– C – CH 3+ CH3 – CH 2 – CHO (–H2O) (a) 3° carbonium ion
EBD_7327
328 CHEMISTRY

39. (c) Completing the reaction The formula of an aldehyde with molecular formula
C2H4O is CH3CHO.
Alk.KMnO
H2C = CH2 ¾¾¾¾¾ 4 ® vicinal diol.
The compound X is
cold
[Alkenes are oxidised by cold neutral or alkaline KMnO4 CH3CH2 H O3
to give vicinal diols]. C=C
3H2C = CH2 + 2KMnO4 + 4H2O Zn/AcOH
H3C CH3
‘X’
® 3CH2 - OH
¾¾ + 2MnO2+ 2KOH
| CH3CH2
CH2 - OH C = O + CH3CHO
(Ethylene glycol) CH3
40. (c) The reaction follows Markownikoff rule i.e. negative 50. (b) Nitrosyl chloride adds on olefins according to
part adds to carbon atom having lesser number of Markownikov’s rule, where NO+ constitutes the
hydrogen atoms. positive part of the addendum.
CH3 H CH3 CH = CH 2 + NOCl ¾¾ ® CH3 CH CH 2
| | d- d+ | |
CH3 - CH 2 - C = C - CH3 + H OCl Cl NO
3 -methyl-2 -pentene 51. (b) Mg2C3 gives propyne.
Mg 2 C3 + 4H 2O ® 2Mg(OH) 2 + CH3 - C º CH
CH 3 H
| | 52. (b) CH3CH 2 - CH(F) - CH3 is a secondary halide. So, it
¾
¾® CH 3 - CH 2 - C - C - CH 3 will undergo dehydro- halogenation to form alkene.
| |
OH Cl CH O –
CH3CH 2 - CH(F) - CH3 ¾¾¾¾
3
®
41. (d) is the answer because it contains two benzene rings. CH3 - CH = CH - CH3
42. (c) Cyclohexene + KMnO4 (cold & alkaline) (Saytzeff product)
¾¾® cis-Hexanediol 53. (b) Greater the s character of C, higher is its
43. (d) CH 2 = CH - CH = CH 2 electronegativity, shorter and stronger will be the bond
1, 3-butadiene formed by it. Thus -C º C - is the strongest bond.
It is a conjugated diene (having alternate
54. (c) On heating ethylene chloride (1, 1 dichloro ethane) with
C = C and C – C). It reacts with HBr. It polymerises to
alcoholic potash followed by sodamide alkyne is
form synthetic rubber.
obtained
44. (d) It is a streoselective reaction and in it a trans product is
alc.KOH
formed. R - CH2 - CCl2 - R ¾¾¾¾® R - CH = CCl - R
Na, NH ( l) NaNH
C C–R ¾¾¾¾¾
3 ®
R 'OH
¾¾¾¾ 2®R - C º C - R
55. (c) V2O5 is used as a catalyst in contact process for the
H R manufacture of SO3 and hence H2SO4. In Haber-Bosch
C C process for the manufacture of NH3, finely divided Fe
H + molybdenum are used.
[It is a reduction. Metal dissolved r eduction 56. (a) Only C2H2 (acetylene) has acidic H-atoms and hence
Na/C2H5OH reaction is stereo selective]. reacts with NaNH 2 to form sodium salt, i.e.,
45. (c) Knocking depends upon the str uctur e of th e
HC º CH + NaNH2 ¾ ¾® HC º CNa + NH3.
hydrocarbon and follows the order
57. (a) Acetylene reacts with the other three as:
n-Alkanes > branched alkanes > straight chain alkenes
> branched alkenes > arenes CH2
Na
CH º CNa CH º CH +HCl
Ba ( CN ) liq. NH3
46. (c) CH º CH + H - CN ¾¾ ¾ ¾
¾2 ® CH = CHCN
2 CHCl
Vinyl cyanide
CH3
47. (d) Allene is H2C = C = CH2 +HCl
48. (a) According to Huckel's rule, the cyclic planar conjugated CHCl CHCl2
system having (4n +2) p electrons show aromaticity.
[AgNO3+NH4OH]
CH º CH AgCºCAg + NH4NO3
white ppt.

+ + – CHOH
2p electrons 4p electrons 4p electrons 4p electrons 58. (c) CH º CH + HOCl ¾¾
® ||
49. (a) Putting the given data in form of equation, we get CHCl
O
HOCl é
(i) O3 || CH ( OH ) 2 ù CHO
- H 2O
X ¾¾¾¾¾¾ ® C2 H 4 O + CH 3CH 2 C CH 3 ¾¾¾® ê | ú ¾¾ ¾ ¾® |
(ii)Zn / AcOH
Aldehyde 2 - Butanone ëê CHCl 2 ûú CHCl 2
dichloroacetaldehyde
Hydrocarbons 329

59. (b) SO3 participates in sulphonation of benzene. Br


60. (d) H2/Pd/BaSO4reduces an alkyne to cis-alkene, H2/Pt Br2 Alcoholic
reduces it to alkane, NaBH4 does not reduce an alkyne. 69. (c)
KOH/
Reduction of an alkyne by active metal in liq. NH3 gives (X) (Y)
trans-alkene.
H
61. (c) CH3 - CH - CH3 + CH3C º C - Na + ¾¾
® H – Br Br Br
| H
Peroxide
Cl
(Z)
CH 3 - CH = CH 2 + CH 3C º CH + NaCl 70. (a)
Propene Propyne
HgSO ,60°C
62. (a) Terminal alkyenes give a white precipitate easily on 71. (a) CH3 - CH 2 - C º CH ¾¾¾¾¾¾
4 ®
H 2SO4
reaction with ammonical silver nitrate solution.
O
CH3 COOH P
[O]
63. (c) CH3 - CH 2 - C - CH3
72. (c) Kekule in 1865 suggested a ring structure of benzene
Toluene Benzoic acid
(A)
in which the ring was composed of six carbon atoms,
each of which carries one atom of hydrogen. To satisfy
COCl the fourth valency of the carbon atom, he suggested
SOCl2
NaN
three alternate double bonds.
¾¾¾®
3
CH
Benzoyl chloride HC CH
(B) HC CH
CH
NCO
CON3
Heat
CH3 CH CH3
– N2 C C
(Curtius rearrangement) Phenyl isocyanate 73. (a) 3CH3 - C º CH ¾ ¾
®
(C) (D) CH CH
C
dil H SO
64. (b) CH º CH + H 2 O ¾¾¾¾¾¾
2 4 ®
HgSO4 .60°C
CH3
[CH 2 = CHOH] ¾¾
® CH3CHO Mesitylene or 1, 3, 5-trimethyl benzene
unstable
conc. H SO
65. (d) In 1-butyne terminal hydrogen is acidic where as in 74. (c) C2 H5 OH ¾¾¾¾¾¾
2 4® C H + H O
2 4 2
KMnO4
2-butyne there is no terminal hydrogen. Thus 2-butyne
Note : If ethyl alcohol is taken in excess and the reaction
will not react with ammonical Cu2Cl2. While 1-butyne,
is carried out at a temperature of 433-443 K diethyl ether
being terminal alkyne, will give red ppt. with ammonical
is formed.
cuprous chloride
66. (b) The reaction follows Markownikoff rule, namely the conc. H SO
2C2 H5OH ¾¾¾¾¾¾
2 4®
bromide ion adds on to the carbon having the least 433-443 K
(excess)
number of hydrogen and the more positive part namely
the – NO group adds to the other carbon of the double C2H5OC2H5 + H2O
bond. 75. (d) In CH º CH triple bond consists of one s and two p
67. (a) As the number of bonds between carbon atoms bonds.
increases, electronegativity of that carbon also 76. (c) When 1, 3-butadiene is reacted with Cl 2 , then a mixture
increases due to increasing active power of electrons. of 1-chlorobutane and 2-chlorobutane is formed.
Also sp hybrid is more e– negetive than sp2 which is Cl
more electronegative than sp3(Q s character decreases) CH 2 = CH - CH = CH 2 ¾¾2¾¾®
Hence, option (a) is correct. (1, 3-Butadiene)

H O Dil.H SO CH 3CH 2 CH 2 Cl + CH 3 - CH 2 - CH - CH 3
68. (c) CaC2 ¾¾¾
2 ® HC º CH ¾¾¾¾¾
2 4 ® CH CHO
+2 3
Acetylene Hg |
Acetaldehyde
Cl
Ni
¾¾¾
® C2 H5OH (1-chlorobutane) (2-chlorobutane)
H2
Hence, option (b) is correct.
EBD_7327
330 CHEMISTRY

AlCl
77. (c) C 6 H 6 + CH 2 = CH 2 ¾¾ ¾
3
® C 6 H 5 CH 2 CH 3 OCOCH3
CH3 CH
CH3 OCOCH3
| CrO3
83. (a)
78. (d) H2SO4
+ HNO3 ¾¾¾¾ (CH 3CO) 2O
®

CHO
CH3 CH3
| | H+
NO2 ¾¾¾¾¾¾®
| - (CH 3CO) 2 O
+
Benzaldehyde
| 84. (b) Benzene can be obtained by polymerisation of
NO2
acetylene.
o-nitrotoluene p-nitrotoluene
Under vigorous conditions, toluene produces TNT or Red hot tube
3HC º CH ¾¾¾¾¾®
500°C
trinitrotoluene.
85. (c) They have a relatively high percentage of carbon.
CH3 CH3
| NO2 |
| O O
HNO
+ ¾¾¾¾

O
H2SO4 O
| 86. (a) + O3 ¾¾
® O
NO2 O
O
O O
CH3 CH3
| O2N | 87. (c) Double bond in between carbon-carbon is present in
NO2 NO2
HNO3
|

|
|

¾¾¾¾
® alkenes whose general formula is CnH2n.
H2SO4 88. (d) The compound has –NO2 group, so N is not present in
| | ring and the six membered ring has C atoms only, so it
NO2 NO2 should be homocyclic. Further the six-membered cyclic
(TNT or Trinitrotoluene) compound has only two double bonds, it is not
79. (c) As per Markovnikoff’s law, the positive part (e.g. H of aromatic (for aromaticity six membered cyclic compound
HX) or the less negative part of the reagent adds to should have three alternate double bonds)
that carbon atom of alkene which has more number of B H
89. (b) 6R - CH = CH 2 ¾¾¾
2 6
® 2(RCH 2CH 2 )3 B
hydrogen atoms (the rich gets richer). So (c) is the Ether,0°C
correct option as the two carbons containing the double
H O
bond have one H atom each i.e. symmetric. ¾¾2 ¾
¾2 ® 6RCH CH OH + 2H BO .
2 2 3 3
NaOH
80. (a) Resonance energy is the difference in observed energy
and calculated energy of resonance hybrid. Greater its 90. (d) Formation of only CH3COOH by ozonolysis indicates
value, more is its stability. So, resonance energy that the compound Y should be CH3CH = CHCH3 which
represents stability of benzene. can be formed by all of the three given compounds
81. (a) That reaction in which an atom or a group of atoms 1H / Pt
CH 2 = CH - CH = CH 2 ¾¾¾¾
2 ®
attached to C-atom of the substrate molecule is replaced X
by another atom or group of atoms is called as CH 3 - CH = CH - CH 3
substitution reaction. Y
1H 2 / Pt
C3 H 8 + Cl 2 ¾ ¾® C3 H 7 Cl + HCl CH3C º CCH3 ¾¾¾¾
® CH3CH = CHCH3
Propane Chlorine Propyl chloride X Y
In the above reaction, ‘H’ atom of propane is replaced 1H 2 / Pt
CH 2 = C = CHCH3 ¾¾¾¾
®
by Cl-atom. X
Option (a) is correct. O
CH 3 CH = CHCH 3 ¾¾®
3 2CH 3 COOH
82. (b) Benzene undergoes electrophilic substitution with
Y
HNO3/H2SO4 (Nitiration).
91. (c) This reaction occurs according to Markownikoff’s rule
HNO NO 2 which states that when an unsymmetrical alkene
¾¾¾¾

H2SO4 undergo hydrohalogenation, the negative part goes to
Benzene Nitrobenzene
that C-atom which contain lesser no. of H-atom.
Hydrocarbons 331

CH3 - CH 2 - C º CH + HCl 99. (d) FeCl 3 is Lewis acid. In presence of FeCl 3 o- and
p-chlorotoluene is formed.
¾¾
® CH3 - CH 2 - C = CH 2 CH3 CH3 CH3
| Cl
Cl FeCl3
+ Cl2 ¾¾¾® +
I
| Toluene o-chlorotoluene
HI Cl
¾¾® CH3 - CH 2 - C - CH3 p-chloro toluene
|
Cl 100. (b) Alkynes having terminal –C º H react with Na in liquid
ammonia to yield H2 gas. CH3CH2C º CH can react
92. (d) H3C — C = CH — CH3
with Na in liquid NH3 so the correct answer is (b).
CH3 Na in
CH3CH 2 C º CH ¾¾¾¾¾
®
liquid NH 3
H3C O H
O3
C C 1
CH3 CH3CH 2 C º C – Na + + H 2 ( g )
H3C 2
O O
O O EXERCISE - 2
H2O
CH3 – C – CH3 + CH3 – C – H 1. (c) Since the alkene on ozonolysis gives a single
–H2O2 acetone acetaldehyde compound, having two aldehydic groups, it must be
93. (c) If both the double and triple bonds are present, the cyclic
compound is regarded as derivative of alkyne. Further
if double and triple bonds are at equidistance from either CH = O CH
(CH2)4 (CH2)4 or
side, the preference is given to double bond. CH = O CH
94. (b) During cracking higher hydrocarbons (liquid) are
Cyclohexene
converted to lower gaseous hydrocarbons.
2. (b) Reaction (b) does not involve the formation of Cl +, the
ether electrophile.
95. (d) C6H5CH2Br + Mg ¾¾¾® C6H5CH2MgBr
3. (a)
(in ether as solution) 4. (c) Compound X reacts with ammonical AgNO3 solution,
+ so it must be a terminal alkyne.
H
C 6 H 5 CH 2 Mg Br ¾¾¾® C H CH + MgBr(OH) Formation of (CH3)2CH COOH on oxidation of X with
H OH 6 5 3
hot alkaline KMnO4 further confirms that X is
96. (c) When both double and triple bonds are present, then (CH3)2 CHC º CH.
triple bond is considered as the principal group. 5. (d) Combustion of hydrocarbons change them to CO2 gas
in which C is sp-hybridised. Therefore, HC º CH, in
CH3 - CH = CH - C º CH
5 4 3 2 1 which carbon is in sp-hybrid state will not result in the
change in hybrid state of C-atom
97. (d) When alkyl benzene are oxidised with alkaline KMnO4,
6. (b)
(strong oxidising agent) the entire alkyl group is oxidised 7. (c) Bromination of alkanes in presence of UV light takes
to –COOH group regardless of length of side chain. place through free radical mechanism. Bromine
CH2CH3 COOH substitutes that hydrogen, abstraction of which leads
to formation of stablest free radical. (Reactivity
( O ), KMnO4 / OH- Selectivity Principle). So in isobutane, bromine will
¾¾¾¾¾¾¾¾ ®
come at second carbon, since after abstraction of
Ethyl benzene Benzoic acid
hydrogen it will form 3° free radical giving II as the
98. (b) The reaction follows Markownikov’s rule which states major product.
that when unsymmetrical reagent adds across
unsymmetrical double or triple bond the negative part
adds to carbon atom having lesser number of hydrogen 8. (c) + (CH3)2CHCH2Cl
atoms.
HBr
CH3 - C º CH + HBr ® CH3 - C = CH2 ¾¾¾ ® C(CH3)3 C(CH3)3
|
Br 3 AlCl
Br ¾¾¾® (HNO H SO )
3 2 4 ®
¾¾¾¾¾¾¾
|
CH3 - C - CH3
|
Br NO2
2, 2-dibromopropane
EBD_7327
332 CHEMISTRY

and hence it will be more influenced by the probability


9. (d) H 3C - C º C - H + CH 3CH 2 MgCl ¾
¾® factor.
17. (d) Br2 in CCl4 (a), Br2 in CH3 COOH (b) and alk. KMnO4
H 3C - C º C -MgCl + CH 3CH 3 (c) will react with all unsaturated compounds, i.e., 1, 3
and 4 while ammonical AgNO3 (d) reacts only with
terminal alkynes, i.e., 3 and hence 3 can be distinguished
from 1, 2 and 4 by. ammonical AgNO3 (d).
H 18. (d) Presence of electron-withdrawing substituent (e.g. C =
O, –COOH, –COOR, –CN, etc.) deactivates the carbon-
C º C - CH 3 carbon double bond towards electrophilic addition.
H2
10. (a) Two moles of alkene is reduced X must be 19. (a) CH3 CH2 CH CH CH2 CH3 Platinum

+ 4H CH3 CH2 CH2 CH2 CH2

HBr
CH3
which give 4H atoms needed for alkene.
CH3 CH2 CH CH2 CH2 CH3
11. (b) Addition on triplebond takesplaceby thesyn-addition
of hydrogen in presence of poisoned palladium Br
Cl 2 alc./ KOH O3 / H 2 O
catalyst. 20. (a) Given A ¾¾¾ ® B ¾¾¾¾¾ ® C ¾¾¾¾® CH 2O
Since the configuration of the double bond already Hydrocarbon

present is cis, the compound formed will have a plane Since hydrocarbon C give only CH2O, on ozonolysis,
C should be CH2 = CH2 hence on going backward, A
of symmetry and hence optically inactive. should be ethane. Thus the reactions are
Me Cl / hn alc.
H CH3CH3 ¾¾¾¾
2 ® CH3CH 2 Cl
Me KOH ...
Me H Me (A) (B)
Me
H CH 2= CH 2 ¾O / H2O
¾3 ¾ ¾® HCHO
(C ) D ( D)
H
Me H 21. (c) An aromatic compound have cyclic clouds of
H H delocalised (4n + 2) p electrons above and below the
plane of the molecule. Among the given three
12. (a) The p bond is formed by the sideways overlapping of
compounds, only compound (c) satisfies these
two p-orbitals of the two carbon atoms. conditions.
The molecular plane does not have any p electron
sp3
density as the p-orbitals are perpendicular to the plane
containing the ethene molecule. The nodal plane in the NO2 CH3
p-bond of ethene is located in the molecular plane.
H H
13. (a) The given reaction is Birch reduction.
+ +
14. (b) Br· is less reactive and more selective and so the most 3
Carbon bearing –NO2 (or –CH3) group is sp
stable free radical (3°) will be the major product. hybridised and does not have a p orbital with the
15. (a) The acidity of acetylene or 1--alkyne can be explained result delocalisation of p electrons over the complete
on the basis of molecular orbital concept according to ring is interrupted.
which formation of C—H bond in acetylene involves
sp-hybridised carbon atom. Now since s electrons are
closer to the nucleus than p electrons, the electrons N
present in a bond having more s character will be H
correspondingly more closer to the nucleus. 4p electrons and 2 non-bonding
Thus owing to high s character of the C—H bond in electrons (present in p orbital of N)
form a cyclic cloud of 6p electrons
alkynes (s = 50%), the electrons constituting this bond
are more strongly held by the carbon nucleus i.e., the 1
acetylenic carbon atom or the sp orbital acts as more 22. (a) Heat of hydrogenation of alkene µ
Stability of alkene
electronegative species than the sp2 and sp3 with the
Hence the alkene which will react fastest with H2 will be
result the hydrogen present on such a carbon atom the least stable. Order of stability will be
(ºC—H) can be easily removed as a proton.
R R R R R H R R
16. (a) More is the reactivity of a free radical, lesser is its
C=C > C=C > C=C > C=C
selectivity for different type of H atoms (1º, 2º, or 3º) R R R H R H H H
and hence the probability factor will be more in such Further the relative rates of hydrogenation decrease
reagent. Since Cl• is more reactive, it is less selective with increase of steric hindrance
Hydrocarbons 333

23. (c) In this case dehydration is governed by Saytezeff’s


Br rule according to which hydrogen is preferentially
eliminated from the carbon atom with fewer number of
CH2 = CH – CH – CH3 hydrogen atoms i.e., poor becomes poorer. Thus, 2
At –80°C the product is methyl butene-2 is the major product.
1, 2-addition
CH2 = CH – CH = CH2 + HBr CH3
HBr, dark
CH2 – CH = CH – CH3 CH3 — C = CH — CH3 (CH — CH — CH — C
in absence
(A) of peroxide
Br
At 40°C the product is CH3
1, 4-addition
HBr, dark
24. (b) All the three compounds have same electron-releasing (CH3)2 — CH — CH — CH3 + CH3 — C — CH2 – CH3
groups, but the presence of two such groups in II and
III make them more reactive than I. Further note that in Br Br
m-xylene (II), the two methyl groups cooperate each (Minor) (Major)
other at two positions, while in III no position is This reaction is governed by Markownikoff’s rule
activated by both the groups. according to which when an unsymmetrical reagent
25. (d) Vinyl and aryl halides can’t be used as the halide e.g. HBr adds to an unsymmetrical alkene, then the
negative part of the reagent is added to that carbon
component because they do not form carbocations
atom of the double bond which bears the least number
readily ; –NH2, –NHR and –NR2 groups react with of hydrogen atom. Thus, in above case. 2-methyl 2-
Lewis acids used in Friedal-Craft reaction to form bromo butane will be the major product.
electron-withdrawing groups, which deactivate the
CH3
benzene nucleus for electrophilic substitution.
.. + – CH CH2
NH2 NH2 AlCl3 29. (a) C

CH3
+ AlCl3 +
H
+
–NH2AlCl3– CH3
is electron-withdrawing
H2O
On the same reason, presence of electron-withdrawing C CH CH3 +
+ Å –H
group like –NO 2 , - N H3 , –CF 3 , – COOH, CH3
–COR, –SO3H etc. make the benzene ring less prone to 1,2-methyl shiftting
Friedel-Craft reaction.
26. (b) The bulky methyl groups are at maximum distance from
each other. CH3 CH3
27. (a) Due to hydrogen bonding between the two OH groups, Å
gauche conformation of ethylene glycol (a) is the most CH3 C CH CH3 C CH CH3
stable conformation.
H CH3 H3C OH
O 3° carbocaction A (Minor)
O (more stable)
H
H H2O

OH CH3
H H C CH CH3
H
\ Correct choice : (a) CH3
CH3 B (Major)
H+/ Heat CH3
28. (b) CH3 — CH — CH — CH3
OH 30. (a) Toluene ( ) contains –CH3 group which is
CH3 CH3
H / Heat
— CH CH3 — C = CH — CH3 + CH3 — CH — CH = CH2 o-, p- directing group so on nitration of toluene the
2 methyl 3 methyl
–NO2 group will occupy o-, p- positions.
butene-2 butene-1 (20%)
(80%) (B)
(A)
EBD_7327
334 CHEMISTRY

CH3 CH3 CH3 CH3 O


NO2 |
(HNO 3 + H2SO4) CH3 - C = O + CH3 - CH 2 - C
+
H
Acetone Propionaldehyde
o-
NO2
p- H H
35. (d) sp3 Carbon
on reduction with Sn/HCl they will form corresponding
anilines in which –NO2 group changes to –NH2. The

CH3 CH3 Cyclopentadiene is non aromatic, as it has sp3 carbon


NH2 in the ring.
mixture now contains and . These
1.O3 CHO
36. (a)
NH2 2.H2O/Zn CHO
anilines when diazotized and then treated with CuBr [E]
forms o-, p- bromotoluenes. KOH(aq), D
31. (d) Completing the sequence of given reactions, CHO
– H 2O
CH 3 – CH = CH - CH 3 ¾¾¾
®
O3 aldol condensation
[F]
O 37. (d) Only (d) can form 3-Octyne
Zn / H 2O
NaNH 2
CH 3CH 2 C º CH ¾¾¾¾
- NH
® CH 3CH 2 C º C - Na +
CH3– CH CH – CH3 ¾¾¾¾® 3
CH 3CH 2 CH 2CH 2 Br
¾¾¾¾¾¾¾(S N 2)
® CH3CH 2 C º CCH 2 CH 2CH 2CH3 + NaBr
O O
‘A’ 348
38. (c) C – C bond energy = 348 kJ/mol = kcal/mol
(ozonide) 4.2
= 82.85 kcal/mol » 100 kcal/mol.
2CH3CHO+ H 2 O + ZnO 39. (a)
'B '
CH3 H
Thus ‘B’ is CH3CHO
Hence (d) is correct answer.
32. (d) Writing the reaction we get H3C CH CH C CH CH C CH CH CH3

CH 3 MgX + CH 3 – C º C – H ¾¾
® H CH3
CH3 – C º CMgX + CH4 ( g ) O3/Zn/H2O
So we find that CH4 is produced in this reaction.
33. (c) The given molecular formula suggests that the aldehyde CH3
formed will be acetaldehyde hence the alkene will be
CH3CH = CHCH3
2 H3C CH O + 2O CH C CH O
2- butene
(achiral)
H O H
O
¾¾®
3
C C H
(achiral)
O O 40. (b) Anti addition of hydrogen atoms to the triple bond
occurs when alkynes are reduced with sodium (or
lithium) metal in ammonia, ethylamine, or alcohol at low
temperature. This reaction called, a dissolving metal
Zn / H 2O 2CH3CHO
¾¾¾¾® reduction, produces an (E)- or trans-alkene.
34. (c) From the products formed it is clear that the compound Sodium in liq. NH3 is used as a source of electrons in
has 5 carbon atoms with a double bond and methyl the reduction of an alkyne to a trans alkene.
group on 2nd carbon atom. CH3 CH2 CH2 C C CH3
CH3 2-Hexyne
| Li/NH3
O3 / Zn, H2O CH3 CH2 CH2 H
CH3 - C = CH - CH 2 - CH 3 ¾¾¾¾¾ ¾® Birch reduction
C C
(2– Methyl–2– pentene) H CH3
Trans-2-Hexene
(A)
Hydrocarbons 335

41. (a) Only terminal alkynes show acidic nature. Ethyne is peroxides, addition takes place via free radical
more acidic than propyne. But-2-yne is not acidic as it mechanism rather than the ionic mechanism.
does not contain any hydrogen attached to sp 47. (b) Chlorination of CH4 does not take place in dark is
hybridised carbon. correct because it is a free radical reaction and free
42. (a) Higher is the branching lesser will be the boiling point radicals are obtained in presence of sun light.
further increase in molecular weight increases boiling 48. (b) Iodination is reversible since formed HI is a strong
point in alkane. Hence 2, 2– dimethyl propane will have reducing agent and reduces the alkyl iodide back to
least boiling point. alkane. CH4 + I2 ƒ CH3I + HI
CH3 Iodination can be done only in presence of strong
| oxidising agents like iodic acid which destroys the
CH3 - C - CH 3
| hydriodic acid.
CH3 49. (d) The two hydrogen atoms on first carbon and the two
43. (c) The order of hyperconjugation is H-atoms on the third carbon atom lie in perpendicular
-CH3 > CH3CH2 - > (CH3 ) 2 CH - > (CH 3 )3 C - planes. The central carbon atom is sp-hybridized while
CH3 - C = CH 2 terminal carbon atoms are sp2-hybridized
| 50. (c) (4n + 2)p electrons and planar structure are the essential
(2 – methylpropene)
CH3 conditions for aromaticity.
CH3 - CH = CH - CH3 (but-2-ene) EXERCISE - 3
CH3 - C = C - CH 3
| | Exemplar Questions
(2, 3-dimethylbut-2-ene)
CH3 CH3
1. (d) We know that, as the number of carbon atom increases,
H3C - HC = C - CH3 boiling point increases while boiling point decreases
|
( 2-methylbut-2-ene) with branching.
CH3
2, 2-dimethyl propane, CH3
CH3 - CH 2 - C = CH 2 |
|
(2-methylbut-1-ene) H3C – C – CH3
CH3 |
Larger the number of methyl groups linked to carbon - CH3
carbon double bond, more is the hyperconjugative C – b.pt = 282.5 K
H bonds and greater is the stability of alkenes hence n – pentane, b.pt = 309.1 K
(c) will be more pronounced for hyperconjugation.
44. (d) Addition of hydrogen halide to alkene is an example of 2-methyl butane, H3 C — H 2 C — HC — CH3
|
electrophilic addition involving carbocations as CH3
intermediates. b.pt = 301 K
HBr
CH3 CH CH 2 ¾ ¾ ¾® CH3 —CH—CH3 n-butane, b.pt = 273 K
in absence of
peroxide
½ 2. (a) Reactivity of halogens decreases with decrease in
Br electronegativity which decreases down the group.
(Markownikoff addition)
45. (c) Among the infinite number of conformations the Hence the order of reactivity of alkanes with halogens is
staggered conformation in which hydrogen atoms are F2 > Cl2 > Br2 > I2.
as far apart as possible is the most stable while the Also with I2, the reaction is too slow that it requires a
eclipsed conformation in which hydrogen atoms are catalyst.
perfectly eclipsed is the least stable. 3. (b) The reactivity of halogens with alkanes follows the
Gauche conformation lie between these two in stability. order : F2 > Cl2 > Br2 > I2. Further, the reactivity of
Thus, order of stability is reduction of alkyl halides with Zn & dilute HCl
staggered > gauche > eclipsed increases as the strength of C – X (X = F, Cl , Br, I) bond
46. (a) This reaction is followed by anti Markownikoff rule decreases. Hence, reduction of alkyl halide with Zn
H H H H and dilute HCl follows the order :
H – C – C – C = C + HBr
Peroxide R – I > R – Br > R – Cl.
4. (a) The correct IUPAC name of the alkane will be :
H H H
1 Butene H3C - CH 2 - C H - CH 2 - CH2 - C H - CH2 - CH3
H H H H |3 4 5 6| 7 8
2
H–C–C–C–C–H CH CH2
1 |
H H H Br CH3 CH3 CH3
1-Bromobutane
In this reaction anti Markownikoff’s addition is 3, 6-diethyl -2-methyloctane.
explained on the basis of the fact that in the presence
EBD_7327
336 CHEMISTRY

5. (a) 1-butene is an unsymmetrical alkene and will follow NEET/AIPMT (2013-2017) Questions
Markownikoff's rule to give major product. 11. (a) Huckel rule is not obeyed. It has only four electrons.
CH3 - CH 2 - CH = CH 2 + H - Br ¾¾
® Further it does not have continuous conjugation.
I-Butene
O
P
+ H – Br + Hg +2 / H2SO 4
12. (d) CH º CH ¾¾¾¾¾¾
® CH3 - C - H
H CH3CHO does not give Victor Meyer test.
minor more stable (2°)
product(s) trigonal planar 13. (d) Presence of 6p orbitals, each containing one unpaired
carbocation electron, in a six membered cyclic structure is in accor-
H H Br s dance with Huckel rule of aromaticity.
+ 14. (b) The combustion reaction of ethylene is

Br Br 5
C2 H 2 + O2 ® 2CO2 + H 2 O
2
Racemic mixtures
(major products) Both HC CH and CO2 have sp hybridization.
VO
6. (d) In option (d), a carbon with double bond has two same 15. (b) 2C6 H6(g) + 9O2(g) ¾¾¾¾
2 5®
410°C
functional groups (CH3) attached to it. The rotation
O
around carbon will not produce a new compound.
Hence, geometrical isomerism is not possible. CH—C
7. (c) Lesser the bond energy of hydrogen halide more will O + 4CO2(g) + 4H2O(g)
be its reactivity. Hence, the order of reactivity of given CH—C
compounds with propene will be HI > HBr > HCl.
8. (b) + I-effect decreases the stability of carbanion. Since, – O
Maleic anhydride
CH3 group shows + I-effect, therefore, it intensifies
the negative charge and hence destabilises the 16. (b) CH 3 C = C - CH 3
| |
carbanion (A) relative to (B). CH 3 CH3
Also, sp hybridised carbanion is more stabilised than 2,3-Dimethyl-2-butene
sp3 .
(CH3)2CH – CH – CH = CH2 contain 7 carbon atoms
- - |
CH º C > CH 3 — C º C > CH 3 — CH 2 CH3
( B)sp ( A )sp ( C )sp 3
CH3 CH3
Hence, the order of decreasing stability will be : B > A > C
CH3 – C – CH = CH2+ H CH3 – C – CH – CH3
9. (d) Alkyl halides on heating with alcoholic KOH eliminates
halogen from a-carbon atom and hydrogen is CH3 CH3
eliminated from b-carbon atom to form an alkene. Rearrangement

3°b- carbon CH3


(A) CH3 — CH — CH 2 Br CH3 CH3 CH3
| C=C CH3 – C – C
CH3 CH3 CH3
CH3 H
(A) 2, 3-dimethyl-2-butene (Stable 3° carbocation)
1°b- carbon 17. (c) 1-chloro-1-methylcyclohexane.
(B) CH 3 — CH 2 — Br CH3
2°b - carbon Cl
(C) CH3 — CH2 — CH2 — Br
More the number of B-substituents (alkyl groups) more
stable alkene will be formed on b-elimination, more will CH2 CH3 CH3
be the reactivity. Thus the correct order is A > C > B Cl
10. (c) During incomplete combustion of alkanes with –
+ H – Cl + Cl
insufficient amount of air carbon black is formed which
is used in the manufacture of ink, printer ink, black
CH3 CH3 CH3
pigments and as filters.
Cl
H H
CH 4 ( g ) + O2 ( g ) ¾¾¾¾¾
Incomplete
® C ( s ) + 2H 2O ( l ) + H – Cl + Cl
combustion
Carbon black
Hydrocarbons 337

18. (d) = 2 × no. of double bond


H3C
=2× 4=8
H3C
O3 O=C 21. (d) C6H5—CH=CH—CH3 + HBr
CH3 Zn + H2O
CH3
OHC
Å
C6H5—CH—CH2—CH3
1 (Benzyl carbocation)
19. (a) Enthalpy of hydrogenation µ
stability of alkene Br

\ III > II > I C6H5—CH—CH2—CH3


20. (a) No. of double bonds = 4 Br
No. of p bond electrons

Å
NaNH H C–CH · Br
22. (a) 2 ® HCº CNa ¾¾¾¾¾¾¾¾¾¾¾¾¾¾¾¾
HCº CH ¾¾¾¾¾¾¾¾¾¾¾ 3 2 ® HCºC – CH 2 – CH3
liq. NH 3
(X)
1-Butyne NaNH2 liq. NH3
Å
3 2 H C - CH – Br
H3 C – CH 2 – Cº C – CH 2 – CH3 H3 C – CH 2 – CºC N a
3-Hexyne (Y)
23. (a) Correct order is H - C º C - H > H3C - C º C - H > H 2C = CH 2 > CH3 - CH3

( Two acidic
hydrogens ) ( One acidic
hydrogen )
24. (c) Hydration of alkynes give ketones.
OH
H2O, H 2SO2
H3 C - C º CH H3 C - C = CH 2
HgSO 4 (A)
O
Tautomerism
H3 C - C - CH3
(B)
25. (c) There is no change in bond angles and bond lengths in the conformations of ethane.
EBD_7327
338 CHEMISTRY

Environmental
14 Chemistry
ENVIRONMENTAL POLLUTION (I) Gaseous air pollutants
The addition of any undesirable material to air, water and soil by a (i) Oxides of sulphur:
natural source (or due to human activity) which affects the quality Produced when S containing fossil fuel (coal, petroleum etc.)
of environment is called an environmental pollution. is burnt in automobiles, industries, refineries, metallurgical
A substance which causes pollution is called pollutant. They can operations etc. SO2 is the more common. It causes respiratory
be solid, liquid or gaseous substances present in greater
diseases, eg. asthma, bronchitis. It causes irritation to eyes.
concentration than in natural abundance and are produced due to
human activities or due to natural happenings. SO2 get oxidised to SO3 by following reactions:
Types of Pollutants : particulate
2SO2(g) + O2(g) 2SO3(g)
(i) Primary and Secondary Pollutants: matter
(a) Primary pollutants: Those which enter the environment SO2(g) + O3(g) SO3(g) + O 2(g)
after their formation and remain as such. In presence of moisture SO3 is converted into highly
Ex. : NO, NO2, SO2 corrosive sulphuric acid.
(b) Secondary pollutants : The harmful materials which are SO 3 + (moisture ) H2SO4
formed by chemical reaction between primary pollutants
(ii) Oxides of nitrogen:
in the atmosphere.
Source - combustion of coal, gasoline, natural gas, petroleum
hn
Ex. Hydrocarbon + Oxide of nitrogen ¾¾ ® compound refining, chemical plants, manufacturing explosives and
(ii) Bio-degradable and Non Bio-degradable Pollutants : fertilizers, tobacco smoke.
(a) Bio degradable pollutants : They are thus not harmful,
but if present in excess in environment, they do not 1483 K
N2(g) + O2(g) 2NO(g)
undergo degradation completely and thus become
pollutant. 2NO(g) + O2(g) 2NO2(g)
(b) Non bio degradable pollutants : The materials (such as NO(g) + O3(g) NO2(g) + O2(g)
Hg, Al, DDT) which do not undergo degradation (or
degrade very slowly) but their presence even in very High conc. of NO2 damage the leaves of plants and retard
small amount in the environment is very harmful. the rate of photosynthesis. It can lead to acute respiratory
ATMOSPHERIC POLLUTION disease in children and is toxic to living tissues.
NO2 reacts with moisture to form acids.
Atmosphere consists of concentric layers of air and each layer
has a different density. The lowest region in which the human 2 NO 2 + H 2 O ¾
¾® HNO 2 + HNO 3 ,
beings along with other organisms live is called troposphere. It
extends upto a height of ~ 10 km from sea level. Above the 3 HNO 2 ¾
¾® 2 NO + HNO 3 + H 2 O
troposphere, between 10-50 km above sea level, lies stratosphere. (iii) Hydrocarbons
Atmospheric pollution is studied as tropospheric and stratospheric They are formed by incomplete combustion of fuel used in
pollution. automobiles. They are carcinogenic
Tropospheric Pollution. They harm plants by causing ageing, breakdown of tissues,
It occurs due to presence of undesirable solid or gaseous particles shedding of leaves, flowers and twigs.
in air. The major pollutants are:
Environmental Chemistry 339

(iv) (ii) It damage irons and steel structure.


Oxides of carbon (iii) It corrodes water pipes. As a result, heavy metals (like Fe,Pb,
Cu) are mixed with water which have toxic effect.
(iv) Acid rain increases the acidity of the lake, which is harmful
to fishes.
CO CO2 (v) It damages trees, plants and retards the growth of the plants.
(a) It is formed as a result of (a) CO2 is discharged (II) Particulate pollutants
incomplete combustion in to atmosphere by Particulates are the minute solid particles or liquid droplets
of hydrocarbons. respiration, burning in air. These particles are usually individually invisible to
(b) Mainly released from of fossil fuels, decomposition the naked eye. Collectively, however, small particles often
automobile exhaust. of limestone, during volcanic form a haze that restricts visibility. The viable particulates
(c) It is poisonous because it (b) eruptions. are the minute living organisms that are dispersed in
The increased amount of CO2 atmosphere. These include bacteria, fungi, moulds, algae
binds with Hb in blood
forming carboxyhaemoglobin, in air is responsible for etc. Non-viable particulates are formed either by the
Hb + CO HbCO global warming. breakdown of larger materials or by the condensation of
(Carboxy minute particles and droplets. There are four types of
haemoglobin) non-viable particulates in the atmosphere : mists, smoke,
Hb + O2 HbO2 fumes and dust.
(i) Mist is produced by particles of spray liquids and the
(oxyhaemoglobin)
Thus as a result O2 carrying condensation of vapours in air.
(ii) Smoke denotes very small soot particles produced by
capacity of blood is reduced. burning and combustion of organic matter.
This oxygen deficiency (iii) Fumes are condensed vapours Fumes of metals are
results in headache, weak well-known particulates of this type.
eyesight, nervousness and
(iv) Dust consists of the particles produced during crushing,
cardiovascular disorder.
Global Warming and Green House Effect grinding and attribution of solid materials.
The atmosphere traps the sun’s heat near the earth’s surface and Particulates are present in vehicle emissions, smoke particles,
dust and ash.
keeps it warm. This is called natural greenhouse effect as it
SOME EFFECTS OF AIR POLLUTION
maintains the temperature and makes the earth perfect for life.
Some of the gases present in earth’s atmosphere like CO2, CH4, Smog
O3, CFC’s, water vapours etc. which are capable of trapping solar It is a mixture of smoke and fog in suspended droplet form. It is of
energy are called green house gases. However, if the conc. of two types:
these gases increase beyond sustainable limit, the average global (i) Classical smog: It occurs in cool, humid climate. It consists
temperature will increase thereby causing global warming of a mixture of smoke fog and SO2 . Classical smog is of
Consequences of global warming : reducing nature and have high concentrations of SO2
(i) Global warming would result in rise in sea level due to (ii) Photochemical Smog : It occurs in warm, dry, sunny climate.
increased rate of melting of glaciers and floods. It is formed as a result of action of sunlight on unsaturated
(ii) Increase in infectious diseases like malaria, dengue, etc. hydrocarbons and nitrogen-oxide
Acid Rain Photochemical smog is of oxidising nature and have high
Normal pH of rain water is 5.6 if pH of rain water drops below 5.6, concentration of oxidising agents.
it is called acid rain. H2SO4, HNO3 (and small amount of HCl) Formation of photochemical smog
which are formed from the oxide of S and N2 present in the air is The chemistry of formation of photochemical smog centres
causes acid rain. These oxides are discharged into atmosphere as around NO, which is emitted into air with the exhaust gases.
a by products of various human activities This NO oxidises in air to NO2 which in turn absorbs energy
from sunlight and breaks up into nitric oxide and free oxygen
NO(g) + O3 (g) NO2(g) + O2(g)
atom. (Photochemical decomposition)
NO2(g) + O(g) NO(g) + O2(g)
SO2 and NO2 then undergo oxidation and react with water to form NO2(g) hv NO(g) + O2(g)
H2SO4 and HNO3 which are major contributors of acid rain. O(g) + O2(g) O3 (g)
NO(g) + O3(g) NO2(g) + O2(g)
2SO2 ( g ) + O2 ( g ) + 2H 2 O ( l) ¾¾
® 2 H 2SO4 ( aq ) Both NO and O3 are strong oxidising agents and can react
with the unburnt hydrocarbons in the polluted air to produce
4NO 2 ( g ) + O 2 ( g ) + 2H 2O ( l ) ¾¾
® 4 HNO3 ( aq ) chemicals such as formaldehyde, acrolein and peroxyacetyl
nitrate (PAN) that cause the eyes to water and burn and are
Ammonium salts are also formed and can be seen as an harmful to the respiratory system.
atmospheric haze. Aerosol particles of oxides or ammonium salts 3CH 4 + 2O 3 3HCHO + 3H2O
in rain drops result in wet deposition. Acrolein and peroxyacetyl nitrate (PAN) are particularly
Harmful effects of acid-rain: noxious.
(i) It damages buildings and statues which contain marble, lime CH 2 = CHCH = O CH 3COONO 2
stone, mortar etc. Acrolein ||
CaCO 3 + H 2SO 4 ¾¾
® CaSO 4 + H 2 O + CO 2 O
Peroxyacetyl nitrate (PAN)
EBD_7327
340 CHEMISTRY

Control of photochemical smog :


hv · ·
Catalytic converters are used in automobiles which prevent HOCl ( g ) ¾¾® OH ( g ) + Cl ( g )
the release of NO and hydrocarbons to atmosphere.
Stratospheric Pollution hv ·
Cl 2 ( g ) ¾¾® 2 Cl ( g )
Formation of ozone layer
O3 in stratosphere (11 -50 km) is a product of UV radiations acting ·
These Cl radicals thus formed initiate the chain reaction for O3
on O2 molecules.
depletion as discussed above.
UV
O 2 ( g ) ¾¾¾
® O (g ) + O ( g ) Effects of O3 layer depletion
Due to depletion of O3 layer, U.V. radiations fall on the earth.
O(g) + O2(g) UV O3(g) (i) The U.V. radiations, damage the cornea and lens of the eyes.
(ii) The U.V. radiations affect the plant proteins and thus reduce
Uv
O3 ( g ) ¾¾¾
® O 2 ( g ) + O(g)+x kcal the chlorophyll.
Depletion of ozone layer (iii) The U.V. radiations, disturb the heat balance of the earth.
The presence of chemicals like CFC’s and NO in stratosphere WATER POLLUTION
result in depletion of ozone layer. This is shown below: The contamination of water by foreign substances which would
UV · · constitute a health hazard and make it harmful for all purposes
CF2 Cl2 ( g ) ¾¾¾
® Cl ( g ) + CF2 Cl ( g ) (domestic, industrial or agriculture etc.) is known as water pollution.
The polluted water may have offensive odour, bad taste, unpleasant
Uv · ·
CFCl3 ( g ) ¾¾¾
® Cl(g)+ CFCl 2 ( g ) colour, murky oily, etc.
Sources of Water Pollution
· ·
Cl ( g ) + O3 ( g ) ¾¾
® ClO ( g ) + O 2 ( g ) Easily identified source or place of pollution is called point source,
ex: municipal and industrial discharge pipes. Non point sources of
· · pollution are those where a source of pollution cannot be easily
Cl O ( g ) + O ( g ) ¾¾
® Cl ( g ) + O 2 ( g )
identified, ex: agricultural run-off, acid rain, storm water drainage
etc. The major water pollution and their sources :
·
The Cl radicals are continuously regenerated and cause the Pollutant Source
breakdown of ozone. Thus, CFC’s are transporting agents for Microorganisms Domestic sewage
Organic wastes Domestic sewage, animal waste,
·
continuously generating Cl radicals into stratosphere and decaying animals and plants and
discharge from food processing
damaging the ozone layer.
factories
Ozone hole
Plant nutrients Chemical fertilizers
· · Toxic heavy metals Industries and chemical factories
In summers, NO2 and CH4 react with Cl O and Cl forming chlorine
Sediments Erosion of soil by agriculture and strip
sinks preventing ozone depletion. mining.
· Pesticides Chemicals used for killing insects,
Cl O ( g ) + NO 2 ( g ) ¾¾
® ClONO 2 ( g )
fungi and weeds
· · Radioactive Mining of uranium containing
Cl ( g ) + CH 4 ( g ) ¾¾
® CH3 ( g ) + HCl ( g ) substances minerals
In winters, polar stratospheric clouds are formed which provide Heat Cooling water used by industrial
surface on which chlorine nitrate formed gets hydrolysed to form plants (which is discharged as hot
hypochlorous acid. It also reacts with the HCl produced to water)
give Cl2. Important Causes of Water Pollution
ClONO 2 ( g ) + H 2 O ( g ) ¾¾
® HOCl ( g ) + HNO 3 ( g ) (i) Pathogens: They are disease causing agents and include
bacteria and other organisms that enter water from domestic
ClONO2 ( g ) + HCl ( g ) ¾¾
® Cl2 ( g ) + HNO3 ( g ) sewage and animal excreta.
(ii) Organic wastes: It includes organic matter such as leaves,
When sunlight returns in spring, sun’s warmth breaks up the clouds
grass, trash, etc. Excessive phytoplankton growth is also a
and HOCl and Cl2 are photolysed by sunlight as:
cause of water pollution. These wastes are biodegradable.
Environmental Chemistry 341

They consume oxygen dissolved in water. If excess of organic transportation. The accepted limit is 50 ppb. It can damage
matter isadded to water, all the availableO2 is used up. This kidney, liver, reproductive system, etc.
causes the death of aquatic life. (iii) Sulphate: Sulphate is harmless at moderate levels, but excess
It is defined as the amount of free oxygen required for
SO 24 - (> 500 ppm) in drinking water causes laxative effect.
biological oxidation of the organic matter by aerobic
conditions at 20°C for a period of five days. Its unit is mg/l or (iv) Nitrate: Maximum limit is 50 ppm. Excess level can cause
ppm. disease called methemoglobinemia (blue baby syndrome)
The amount of BOD in water is a measure of the amount of (v) Other Metals : The maximum recommended levels of
organic material in water, in terms of how much O2 will be common metals in drinking water are :
required to break it down biologically. Clean water would Metal Max. concentration (ppm or mg dm–3)
have a BOD value of less than 5 ppm whereas highly polluted Zn 5
river water could have a BOD value of 17 ppm or more. Fe 0.2
Mn 0.05
A large number of organic and inorganic compounds, Cu 3
however, are resistant to microbial oxidation. They, therefore, Cd 0.005
don’t contribute to the BOD, though their presence makes Al 0.2
water unfit for consumption. SOIL POLLUTION
Note : Estimation of BOD requires 5 days so another parameter Causes of Soil Pollution
called chemical oxygen demand (COD) can also be used. It is Pesticides: Pesticides are substances that are used to kill or block
a measure of all types of oxidisable impurities present in the the reproductive processes of unwanted organisms. The repeated
sewage. COD values are higher than BOD values.Oxidising use of same or similar pesticides make the pests resistant to them.
agent used in COD determination is acidified K2Cr2O7. Being water insoluble, non-biodegradable and high persistent
(iii) Chemical pollutants: toxins are transferred from lower trophic level to higher trophic
(a) Heavy metals water soluble inorganic chemicals that level through food chain, resulting in serious metabolic and
physiological disorders in higher animals.
include heavy metals like Cd, Hg, Ni, etc. constitute an
Insecticides: Control of insects by insecticides helps to cure
important class of pollutants. These metals are not
diseases (for example malaria and yellow fever) and protect crops.
excreted by the body and can damage kidneys, central
They are organophosphates and carbamates which are less
nervous system, liver, etc.
persistent and more biodegradable than pesticides. However, they
(b) Acid-polluted water (pH < 3) : This is deadly to most
are severe nerve toxins and are harmful to humans. Insects have
forms of aquatic life.Acid mine water principally contain
become resistant to them also.
sulphuric acid produced by the oxidation of iron pyrites
Herbicides: Herbicides are used to kill weeds. Ex: Sodium Chlorate
(FeS2 ). Industrial wastes and acid rain may also (NaClO3), sodium arsenite (Na3AsO3) etc. They are not persistent
contribute to the acidity of natural waters. and decompose in a few months. They are toxic to mammals, cause
(c) Organic chemicals like petroleum products, pesticides, birth defects.
industrial chemicals like polychlorinated biphenyl (PCB’s) Organic herbicides are, therefore, now used. They are much more
are also a source of water pollution. PCB’s are toxic to certain types of plants than to others.
carcinogenic. Fungicides: Fungicides are used to check the growth of fungi.
(d) Detergents and Fertilizers: These may contain (Fungi are the plants without chlorophyll). Organic compounds
phosphates as additives. The addition of phosphours of mercury have been used as fungicides.
to water, in the form of the phosphate anion PO43–, Industrial Waste
encourages the formation of algae, which reduces the (i) They are biodegradable and non-biodegradable.
dissolved oxygen concentration of water. The process, (ii) Biodegradable wastes are generated by cotton mills, food
known as eutrophication, impedes the development of processing units, paper mills and textile factories.
higher life forms, such as fish. (iii) Non-biodegradable wastes are generated by thermal power
International Standards for Drinking Water plants which produce fly ash, integrated iron and steel plants,
(i) Fluoride: Soluble F– is added to drinking water to bring its fertilizer industries, industries manufacturing Al, Zn, Cu,
conc upto 1 ppm or 1 mg dm–3. F– ions makes the teeth chemical, drugs, pharmaceuticals, dyes, pesticides, rubber
enamel much harder. However, F– ion conc above 2 ppm goods, etc.
causes brown motting of teeth. Above 10 ppm, it causes STRATEGIES TO CONTROL ENVIRONMENTAL
harmful effect to bones and teeth. POLLUTION
(ii) Lead: Drinking water gets contaminated with lead if water is (i) Waste management i.e., reduction of waste and proper
relatively acidic and lead pipes are used for water disposal, also recycling of materials and energy.
EBD_7327
342 CHEMISTRY

(ii) Adopting methods in day-to day life which result in reduction (ii) Bleaching of paper:
of the pollution, i.e., proper collection and disposal. Earlier, Cl2 gas was used but now H2O2 with a suitable catalyst
GREEN CHEMISTRY is being used for bleaching paper.
Green Chemistry focuses on processes and products that reduce (iii) Synthesis of Chemicals:
or eliminate the use and generation of hazardous substances.The
Catalyst
use of starting materials-reagents and solvents that pose less CH 2 = CH 2 + O2 ¾¾¾¾® CH 3CHO ( 90% )
hazard to man and his environment, is one aspect. Using raw Pd( II )
Cu ( II )
materials more efficiently and generating less waste is another. in water
Use of Green Chemistry in Day-to-Day Life: (iv) Development of a method for catalytic dehydrogenation of
(i) Dry cleaning of clothes: diethanolamine in which a new technique allows the
Earlier, tetrachloroethene was used as a solvent for dry production of an environmentally friendly herbicide in a less
cleaning but it contaminates ground water and is a carcinogen. dangerous way.
It is being replaced by liquified CO2 with a suitable detergent (v) Development of processes using carbon dioxide as the
which causes less harm to ground water. blowing agent, for manufacture of polystyrene foam sheet
Now, H2O2 is used for bleaching clothes which makes less packaging material.
use of water. Note : Thus, green chemistry is a cost effective approach which
involves reduction in material, energy consumption and waste
generation.
Environmental Chemistry 343

CONCEPT MAP
EBD_7327
344 CHEMISTRY

1. The pollutants which came directly in the air from sources 14. The region which is greatly affected by air pollution is
are called primary pollutants. Primary pollutants are (a) Thermosphere (b) Stratosphere
sometimes converted into secondary pollutants. Which of (c) Troposphere (d) Mesosphere
the following belongs to secondary air pollutants? 15. The substance which is a primary pollutant?
(a) C O (b) Hydrocarbon (a) H2SO4 (b) CO
(c) Peroxyacetyl nitrate (d) NO (c) PAN (d) Aldehydes
2. The green house effect is caused by 16. Formation of London smog takes place in
(a) CO2 (b) NO2 (a) winter during day time
(c) NO (d) CO (b) summer during day time
3. The gas responsible for ozone depletion : (c) summer during morning time
(a) NO and freons (b) SO2 (d) winter during morning time
(c) CO2 (d) CO
17. Green house gases –
4. Sewage mostly constitutes
(a) allow shorter wavelength to enter earth's atmosphere
(a) Non-biodegradable pollutants
while doesn't allow longer wavelength to leave the
(b) Biodegradable pollutants
earth's atmosphere.
(c) Effluents
(b) allow longer wavelength to enter earth atmosphere
(d) Air pollutants
while doesn't allow shorter wavelength to leave the
5. In Antarctica ozone depletion is due to the formation of
surface
following compound
(c) don't have wavelength specific character.
(a) acrolein (b) peroxyacetyl nitrate
(d) show wavelength specific behaviour near the earth
(c) SO2 and SO3 (d) chlorine nitrate
while far from earth these have wavelength indepen-
6. The main element of smog is dent behaviour.
(a) O3 and PAN (b) O3 18. Which of the following is/are the hazardous pollutant(s)
(c) PAN (d) PPN and PBN present in automobile exhaust gases?
7. Classical smog occurs in places of (i) N2 (ii) CO
(a) excess SO2 (b) low temperature (iii) CH4 (iv) Oxides of nitrogen
(c) high temperature (d) excess NH3 (a) (ii) and (iii) (b) (i) and (ii)
8. CFC which is a main reason behind air pollution, is produced (c) (ii) and (iv) (d) (i) and (iii)
by 19. Carbon monoxide (CO) is harmful to man because
(a) sewage pollutant (b) aerosols (a) it forms carbolic acid
(c) industrial remains (d) Above all (b) it generates excess CO2
9. Which is related to ‘Green House Effect’? (c) it is carcinogenic
(a) Farming of Green plants (d) it competes with O2 for haemoglobin
(b) Farming of Vegetables in Houses 20. Today the concentration of green house gases is very high
(c) Global Warming because of
(d) Biodegradable pollutant (a) use of refrigerator
10. Phosphate pollution is caused by (b) increased combustion of oils and coal
(a) sewage and agricultural fertilizers (c) deforestation
(b) weathering of phosphate rocks only (d) All of the above
(c) agriculutral fertilizers only 21. The non-viable particulate among the following is
(d) phosphate rocks and sewage (a) Dust (b) Bacteria
11. The uppermost region of the atmosphere is called (c) Moulds (d) Fungi
(a) Ionosphere (b) Stratosphere 22. BOD of pond is connected with
(c) Troposphere (d) Exosphere (a) microbes & organic matter
12. Which of the following is the coldest region of atmosphere (b) organic matter
(a) Thermosphere (b) Mesosphere (c) microbes
(c) Troposphere (d) Stratosphere (d) None of these
13. Acid rain is due to 23. Lead is
(a) CH3 (b) N2O5 (a) Radiological pollutant (b) Sound pollutant
(c) SO2 and NO2 (d) C2H5OH (c) Soil pollutant (d) Air pollutant
Environmental Chemistry 345

24. When rain is accompanied by a thunderstorm, the collected 36. Thermal pollution affects mainly –
rain water will have a pH value (a) vegetation (b) aquatic creature
(a) slightly lower than that of rain water without (c) rocks (d) air
thunderstorm 37. B.O.D. test or biochemical oxygen demand test is made for
(b) slightly higher than that when the thunderstorm is not measuring
there (a) air pollution (b) water pollution
(c) uninfluenced by occurrence of thunderstorm (c) noise pollution (d) soil pollution
(d) which depends upon the amount of dust in air 38. A dental disease characterised by mottling of teeth is due to
25. Minamata disease of Japan is due to pollution of presence of a certain chemical element in drinking water.
(a) Aresenic (b) Lead Which is that element?
(c) Cynide (d) Mercury (a) Boron (b) Chlorine
26. Select the process that does not add particulate materials to (c) Fluorine (d) Mercury
air. 39. The viable particulate among the following is
(a) Use of air conditioner (a) Fumes (b) Algae
(b) Burning of fosssil fuels (c) Smoke (d) Mist
(c) Paper industry
40. The high amount of E. coli in water is the indicator of
(d) Incomplete combustion of coal
(a) hardness of water
27. Which causes death of fish in water bodies polluted by
(b) industrial pollution
sewage?
(c) sewage pollution
(a) Foul smell (b) Pathogens
(c) Herbicides (d) Decrease in D.O. (d) presence of chlorine in water
28. Sewage water is purified by 41. A lake with an inflow of domestic sewage rich in organic
(a) aquatic plants (b) microoganisms waste may result in
(c) light (d) fishes (a) drying of the lake very soon due to algal bloom
29. Which pollutant is harmful for ‘Tajmahal’? (b) an increase production of fish due to lot of nutrients
(a) Hydrogen (b) O2 (c) death of fish due to lack of oxygen
(c) SO2 (d) Chlorine (d) increased population of aquatic food web organisms
30. The biggest particulate matter is 42. Which of the following is the major cause of global warming?
(a) HNO3 droplets (b) Soot (a) re-radiation of U.V. rays by CO2 and H2O
(c) H2SO4 droplets (d) Fly ash (b) re-radiation of I.R. rays by CO2 and H2O
31. Negative soil pollution is (c) re-radiation of I.R. rays by O2 and N2
(a) reduction in soil productivity due to erosion and over (d) re-radiation of U.V. rays by O2 and N2
use 43. Select the one that is an oxygen demanding waste.
(b) reduction in soil productivity due to addition of (a) Grease (b) Oil
(c) Pesticides (d) Domestic sevage
pesticides and industrial wastes
44. The greenhouse effect is because of the
(c) converting fertile land into barren land by dumping
(a) presence of gases, which in general are strong infrared
ash, sludge and garbage
absorbers, in the atmosphere
(d) None of the above
(b) presence of CO2 only in the atmosphere
32. The quantity of DDT in food chain
(c) pressure of O3 and CH4 in the atmosphere
(a) decreases (b) remains same
(d) N2O and chlorofluorohydrocarbons in the atmosphere
(c) increases (d) changes
45. Which one of the following statement is not true ?
33. Which is known as ‘Third poison of environment’ and also (a) pH of drinking water should be between 5.5 – 9.5.
creates ‘Blue baby syndrome’ (b) Concentration of DO below 6 ppm is good for the
(a) Nitrate present in water growth of fish.
(b) Phosphate and detergents found in water (c) Clean water would have a BOD value of less than
(c) Cynide 5 ppm.
(d) Pesticides (d) Oxides of sulphur, nitrogen and carbon are the most
34. The aromatic compounds present as particulates are widespread air pollutant.
(a) Polycyclic aromatic hydrocarbons 46. Identify the wrong statement in the following:
(b) Benzene (a) Chlorofluorocarbons are responsible for ozone layer
(c) Toluene depletion
(d) Nitrobenzene (b) Greenhouse effect is responsible for global warming
35. Water is often treated with chlorine to (c) Ozone layer does not permit infrared radiation from the
(a) remove hardness
sun to reach the earth
(b) increase oxygen content
(d) Acid rain is mostly because of oxides of nitrogen and
(c) kill germs
sulphur
(d) remove suspended particles
EBD_7327
346 CHEMISTRY

47. Identify the incorrect statement from the following : (c) O2 and O3
(a) Ozone absorbs the intense ultraviolet radiation of the (d) O3 and N2
sun. 49. What is DDT among the following ?
(b) Depletion of ozone layer is because of its chemical (a) Greenhouse gas
reactions with chlorofluoro alkanes. (b) A fertilizer
(c) Ozone absorbs infrared radiation.
(c) Biodegradable pollutant
(d) Oxides of nitrogen in the atmosphere can cause the
depletion of ozone layer. (d) Non-biodegradable pollutant
48. The smog is essentially caused by the presence of 50. The gas leaked from a storage tank of the Union Carbide
plant in Bhopal gas tragedy was :
(a) Oxides of sulphur and nitrogen
(a) Methyl isocyanate (b) Methylamine
(b) O2 and N2
(c) Ammonia (d) Phosgene

1. The statement which is not true 6. Air pollution causing photochemical oxidants production
(a) NO2 does not play any role in photochemical smog include
(b) SO3 is more harmful air polluntant than SO2 (a) Carbon monoxide, sulphur dioxide
(c) SO2 dos not affect larynx (voice box) (b) Nitrous oxide, nitric acid fumes, nitric oxide
(d) NO is more toxic to living tissues than NO2 (c) Ozone, peroxyacetyl nitrate, aldehydes
2. Black-foot disease is caused due to groundwater (d) Oxygen, chlorine, fuming nitric acid
contaminated with excess of 7. Photochemical smog formed in congested metropolitan cities
(a) Nitrate (b) Fluoride mainly consists of
(a) ozone, peroxyacetyl nitrate and NOx
(c) Arsenic (d) Sulphur
3. The false statement among the followings : (b) smoke, peroxyacetyl nitrate and SO2
(a) The average residence time of NO is one month (c) hydrocarbons, SO2 and CO2
(b) Limestone acts as a sink for SOx (d) hydrocarbons, ozone and SOx
(c) SOx can be removed from flue gases by passing through 8. Which of the following statements about polar stratosphere
a solution of citrate ions clouds (PSCs) is not correct?
(d) Ammonia acts as a sink for NOx (a) PSCs do not react with chlorine nitrate and HCl
4. Exposure of an organism to UV system causes (b) Type I clouds are formed at about –77ºC and contain
(a) photodynamic action solid HNO3 . 3H2O
(b) formation of thymidine (c) Type II clouds are formed at about –85ºC and contain
some ice
(c) splitting of H-bonds of DNA
(d) A tight whirlpool of wind called Polar Vortex is formed
(d) splitting of phosphodiester bonds which surrounds Antarctica
5. Under column-I, a list of gases that are known to have a 9. Which one of the following statements is correct ?
greenhouse effect’ is given. Relate them to their main source (a) Extensive use of chemical fertilizers may lead to
selecting from the list given under Column - II eutrophication of nearby water bodies
(b) Both Azotobacter and Rhizobium fix atmospheric
Column-I Column-II
A. Nitrous oxide 1. Secondary pollutant from nitrogen in root nodules of plants
(c) Cyanobacteria such as Anabaena and Nostoc are
car exhausts
important mobilizers of phosphates and potassium for
B . Chlorofluorocarbon 2. Combustion of fossil fuels,
plant nutrition in soil
(CFCs) wood, etc (d) At present it is not possible to grow maize without
C. Methane 3. Denitrification chemical fertilizers
D. Ozone (O3) 4. Refrigerators, aerosol, 10. Which of the following metal is a water pollutant and causes
sprays sterility in human being
E. Carbon dioxide 5. Cattle, rice fields, toilets. (a) As (b) Mn
(a) A – 3, B – 4, C – 5, D – 1, E – 2 (c) Mg (d) Hg
(b) A – 5, B – 1, C – 3, D – 4, E – 2 11. Eutrophication causes reduction in
(c) A – 4, B – 5, C – 1, D – 2, E – 3 (a) Dissolved oxygen (b) Nutrients
(d) A – 1, B – 3, C – 4, D – 5, E – 2 (c) Dissolved salts (d) All the above
Environmental Chemistry 347

12. Lichens do not like to grow in cities 19. Green chemistry means such reactions which :
(a) because of absence of the right type of algae and fungi (a) produce colour during reactions
(b) because of lack of moisture (b) reduce the use and production of hazardous chemicals
(c) because of SO2 pollution (c) are related to the depletion of ozone layer
(d) because natural habitat is missing
(d) study the reactions in plants
13. Which one of the following pairs is mismatched
(a) Fossil fuel burning – release of CO2 20. Which one of the following statements regarding
(b) Nuclear power – radioactive wastes photochemical smog is not correct?
(c) Solar energy – Greenhouse effect (a) Carbon monoxide does not play any role in
(d) Biomass burning – release of CO2 photochemical smog formation.
14. In a coal fired power plant electrostatic precipitators are (b) Photochemical smog is an oxidising agent in character.
installed to control emission of (c) Photochemical smog is formed through photochemical
(a) SO2 (b) NOx reaction involving solar energy.
(c) SPM (d) CO (d) Photochemical smog does not cause irritation in eyes
15. Presence of which fuel gas in the exhaust fumes shows and throat.
incomplete combustion of fuel.
(a) Sulphur dioxide DIRECTIONS for Qs. 21 to 25 : These are Assertion-Reason
(b) Carbon monoxide and water vapour type questions. Each of these question contains two statements:
(c) Carbon monoxide Statement-1 (Assertion) and Statement-2 (Reason). Answer these
(d) Nitrogen dioxide questions from the following four options.
16. The term “Bio-magnification” refers to the (a) Statement- 1 is True, Statement-2 is True, Statement-2 is a
(a) growth of organism due to food consumption correct explanation for Statement -1
(b) increase in population size (b) Statement -1 is True, Statement -2 is True ; Statement-2 is
NOT a correct explanation for Statement - 1
(c) blowing up of environmental issues by man
(d) increase in the concentration of non-degradable (c) Statement - 1 is True, Statement- 2 is False
pollutants as they pass through food chain (d) Statement -1 is False, Statement -2 is True
17. The statement which is not correct about control of 21. Statement-1 : Inhabitants close to very busy airports are likely
particulate pollution to experience health hazards.
(a) In electrostatic precipitator, the particulates are made Statement-2 : Sound level of jet aeroplanes usually exceeds
to acquire positive charge which are then attracted by 160 dB.
the negative electrode and removed 22. Statement-1 : Suspended particulate matter (SPM) is an
(b) Gravity settling chamber removes larger particles from important pollutant released by diesel vehicles.
the air Statement-2 : Catalytic converters greatly reduce pollution
(c) Cyclone collector removes fine particls in the diameter caused by automobiles.
range 5-20 microns 23. Statement-1 : Eutrophication shows increase in productivity
(d) Wet scrubbers are used to wash away all types of in water.
particulates Statement-2 : With increasing eutrophication, the diversity
18. Which of the following is/are the hazardous pollutant(s) of the phytoplankton increases.
present in automobile exhaust gases? 24. Statement-1 : The main cause of Bhopal gas tragedy was
(i) N2 (ii) CO phosgene.
(iii) CH4 (iv) Oxides of nitrogen Statement-2 : Phosgene is a volatile liquid.
(a) (ii) and (iii) (b) (i) and (ii) 25. Statement-1 : CO2 causes green house effect.
(c) (ii) and (iv) (d) (i) and (iii) Statement-2 : Other gases do not show such effect.

Exemplar Questions (a) NO2


1. Which of the following gases in not a green house gas? (b) O3
(a) CO (b) O3 (c) SO2
(c) CH4 (d) H2O vapour (d) Unsaturated hydrocarbon
3. Which of the following statements is not true about classical
2. Photochemical smog occurs in warm, dry and sunny climate.
smog?
One of the following is not amongst the components of
(a) Its main components are produced by the action of
photochemical smog, identify it.
sunlight on emissions of automobiles and factories
EBD_7327
348 CHEMISTRY

(b) Produced in cold and humid climate (a) CO (b) Hydrocarbon


(c) It contains compounds of reducing nature (c) Peroxyacetyl nitrate (d) NO
(d) It contains smoke, fog and sulphur dioxide 11. Which of the following statements is correct?
4. Biochemical Oxygen Demand, (BOD) is a measure of organic (a) Ozone hole is a hole formed in stratosphere from which
material present in water. BOD value less than 5 ppm indicates ozone oozes out
a water sample to be (b) Ozone hole is a hole formed in troposphere from which
(a) rich in dissolved oxygen ozone oozes out
(b) poor in dissolved oxygen (c) Ozone hole is thinning of ozone layer of stratosphere
(c) highly polluted at some places
(d) not suitable for aquatic life (d) Ozone hole means vanishing of ozone layer around
5. Which of the following statement(s) is/are wrong? the earth completely
(a) Ozone is not responsible for green house effect 12. Which of the following practices will not come under green
(b) Ozone can oxidise sulphur dioxide present in the chemistry?
atmosphere to sulphur trioxide (a) If possible, making use of soap made of vegetable oils
(c) Ozone hole is thinning of ozone layer present in instead of using synthetic detergents.
stratosphere (b) Using H2O2 for bleaching purpose instead of using
(d) Ozone is produced in upper stratosphere by the action chlorine based bleaching agents
of UV rays on oxygen
(c) Using bicycles for travelling small distances instead
6. Sewage containing organic waste should not be disposed
of using petrol/ diesel based vehicles
in water bodies because it causes major water pollution.
(d) Using plastic cans for neatly storing substances
Fishes in such a polluted water die because of
(a) large number of mosquitoes NEET/AIPMT (2013-2017) Questions
(b) increase in the amount of dissolved oxygen 13. Roasting of sulphides give the gas X as a by product. This is
(c) decrease in the amount of dissolved oxygen in water colorless gas with choking smell of burnt sulphur and caused
(d) clogging of gills by mud great damage to respiratory organs as a result of acid rain. Its
7. Which of the following statements about photochemical aqueous solution is acidic, acts as a reducing agent and its
smog is wrong? acid has never been isolated. The gas X is : [2013]
(a) It has high concentration of oxidising agents (a) SO2 (b) CO2
(b) It has low concentration of oxidising agent
(c) It can be controlled by controlling the release of NO2, (c) SO3 (d) H2S
hydrocarbons, ozone etc 14. Which one of the following statements is not true?
(d) Plantation of some plants like pinus helps in controlling [NEET Kar. 2013]
photochemical smog (a) Dissolved oxygen (DO) in cold water can reach a
8. The gaseous envelope around the earth is known as concentration upto 10 ppm.
atmosphere. The lowest layer of this is extended upto 10 km (b) Clean water would have a BOD value of 5 ppm.
from sea level, this layer is (c) Fluoride deficiency in drinking water is harmful. Soluble
(a) stratosphere (b) troposphere fluoride is often used to bring its concentration upto 1
(c) mesosphere (d) hydrosphere ppm.
9. Dinitrogen and dioxygen are main constituents of air but (d) When the pH of rain water is higher than 6.5, it is called
these do not react with each other to form oxides of nitrogen acid rain.
because ............... . 15. Which of the following is not a common component of
(a) the reaction is endothermic and requires very high Photochemical Smog? [2014]
temperature (a) Ozone
(b) the reaction can be initiated only in presence of a (b) Acrolein
catalyst (c) Peroxyacetyl nitrate
(c) oxides of nitrogen are unstable (d) Chlorofluorocarbons
(d) N2 and O2 are unreactive 16. Which of the following is a sink for CO ? [2017]
10. The pollutants which come directly in the air from sources (a) Microorganism present in the soil
are called primary pollutants. Primary pollutants are (b) Oceans
sometimes converted into secondary pollutants. Which of (c) Plants
the following belongs to secondary air pollutants? (d) Haemoglobin
Environmental Chemistry 349

Hints & Solutions


EXERCISE - 1 23. (c)
24. (a) Normal rain water has pH 5.6. Thunderstorm results
1. (c)
in the formation of NO and HNO 3 which lowers
2. (a) CO2 causes Green House Effect.
the pH.
3. (a) NO and freons are responsible for ozone depletion.
25. (d) Minamata is caused by Hg poisoning.
4. (b) Domestic sewage constitute biodegradable pollutants.
56. (a)
5. (a) In antarzctica ozone depletion is due to formation of
27. (d) Decrease in D.O causes death of fish
acrolein.
28. (b) Sewage water is purified by micro-organisms.
6. (a) 7. (b) 8. (b) 9. (c)
29. (c)
10. (a) Phosphate pollution is caused by sewage and
30. (d) Fly ash.
agricultural fertilizers.
31. (a) 32. (c) 33. (b)
11. (d) The uppermost region of atmosphere is exosphere.
34. (a) PAH (Poly Aromatic Hydrocarbon)
12. (b) The coldest region is mesosphere (temp. –27ºC to
– 92ºC) 35. (c) Water is often treated with Cl2 to kill germs.

13. (c) Acid rain is rain or any other form of precipitation that 36. (b) Thermal pollution is caused by power plants. Power
is unusually acidic. It has harmful effects on plants, plant requires a larger quantity of water for cooling.
aquatic animals, and infastructure. Acid rain is mostly The water after cooling is left in the water body. The
caused by human emissions of sulfur and nitrogen temperature of left water is generally very high and
compounds which react in the atmosphere to produce affects aquatic life.
acids. In recent years, many governments have 37. (b) Strength of sewage or degree of water pollution is
introduced laws to reduce these emissions. measured in terms of BOD (Biochemical oxygen
14. (c) Air pollution greatly affect the troposphere. demand) value.

15. (b) CO is primary pollutant. 38. (c) The excess of fluorine in water causes fluorosis. The
symptoms of fluorosis are mottling of teeth (yellowish
16. (d) London smog is formed in morning during winter.
streaks) and abnormal bones liable to fracture etc. It is
17. (a) Radiation coming from sun or outerspace have high
an example of endemic disease.
energy or short wavelength, which are allowed to enter
39. (b) Algae
by green house gases. However, radiation emitted by
earth is in infrared region, having long wavelength, are 40. (c) 41. (c) 42. (b)
reflected back by the envelope of green house gases. 43. (d) [Many organic substances break up into simpler
18. (c) CO and oxides of Nitrogen are poisnous gases present substan ces by taking up dissolved oxygen in
in automobile exhaust gases. presence of some bacteria. As they continue, dissolved
oxygen, runs short for aquatic life, which is then
19. (d) CO is highly toxic and impairs respiration. CO combine
badly affected. Moreover many harmful products are
with haemoglobin of blood and reduces its O2 carry
formed]
capacity.
44. (a) Green house gases such as CO2, ozone, methane, the
20. (d)
chlorofluorocarbon compounds and water vapour form
21. (a) Dust
a thick cover around the earth which prevents the IR
22. (a) BOD of pond is connected with microbes and organic rays emitted by the earth to escape. It gradually leads
matter. to increase in temperature of atmosphere.
EBD_7327
350 CHEMISTRY

45. (b) The ideal value of D.O for growth of fishes is 8 mg/ l . 13. (c) Solar energy is not responsible for green house effect
7mg /l is desirable range, below this value fishes get instead it is a source of energy for the plants and
susceptible to disease. A value of 2 mg/ l or below is animals.
lethal for fishes. 14. (c) SPM ® Suspended Particulates matter.
46. (c) Ozone layer acts as a shield and does not allow 15. (c) Prsence of CO in the exhaust fumes shows incomplete
ultraviolet radiation from sun to reach earth. It does combustion.
not prevent infra-red radiation from sun to reach earth. 16. (d)
Thus option (c) is wrong statement and so it is the 17. (a) Particulates acquire negative charge and are attracted
correct answer. by the positive electrode.
47. (c) The ozone layer, existing between 20 to 35 km above 18. (c) CO and oxides of Nitrogen are poisnous gases present
the earth’s surface, shield the earth from the harmful in automobile exhaust gases.
U. V. radiations from the sun.
19. (b) Green chemistry may be defined as the programme of
Depletion of ozone is caused by oxides of nitrogen developing new chemical products and chemical
N 2O + h u ¾
¾® NO + N processes or making improvements in the already
existing compounds and processes so as to make less
reactive nitric oxide
harmful to human health and environment. This means
NO + O 3 ¾
¾® NO 2 + O2 the same as to reduce the use and production of
hazardous chemicals.
O3 + h u ¾
¾® O 2 + O
i.e. correct answer is option (b).
NO 2 + O ¾
¾® NO + O 2
20. (d) The oxidised hydrocarbons and ozone in presence of
humidity cause photochemical smog.
2 O3 + h u ¾
¾® 3 O 2 (Net reaction)
Hydrocarbons + O2, NO2, NO, O, O3 ® Peroxides,
The presence of oxides of nitrogen increase the formaldehyde, peroxyacetylnitrate (PAN), acrolein etc.
decomposition of O3.
It is oxidising in nature and causes irritation to eyes,
48. (a) Smog is caused by oxides of sulphur and nitrogen. lungs, nose, asthamatic attack and damage plants.
49. (d) DDT is a non-biodegradable pollutant. 21. (a) Noise level upto 64 dB (decibel) is well tolerated.
50. (a) Methyl isocyanate, CH3 – N = C = O Prolonged exposure to noise level to 80 dB or more
leads to loss of hearing ability, fatigue, nervousness,
EXERCISE - 2
fever, hypertension, gastric disorder, increase in
1. (b) SO3 is more harmful pollutant than SO2. cholesterol level and dilation of pupil of the eye. As the
2. (c) jet aeroplanes have the noise upto 150-160 dB, the
inhabitants in the vicinity of busy airports are likely to
3. (a) The average residence time of NO is 4 days.
experience above health hazards. Maximum noise level
4. (c) 5. (a) 6. (c) 7. (a)
is recorded in rockets, i.e., 180 dB.
8. (a) PSCs react with chlorine nitrate and HCl to give HOCl
22. (b) SPM (Suspended Particulate Matter) is defined as
and Cl2.
particles floating in the air with a diameter below 10 mm.
9. (a) 10. (b) Studies have shown that high SPM concentrations in
11. (a) Eutrophication causes reduction in D.O. the air can have a detrimental impact on respiratory
organs. SPM is generated from natural sources (e.g.,
12. (c) Because they are very sensitive to sulphur dioxide and
volcanoes or dust storms) and human activities
in cities the amount of SO2 is high so lichen do not
(vehicles, incinerators and industrial plants).
grow in cities.
Environmental Chemistry 351

SPM Other aerosols sunlight on unsaturated hydrocarbons and nitrogen


oxides produced by automobiles and factories.
Less than 10 mm Less than 100 mm
Tend to float longer in Tend to settle fairly 3. (a) Classical smog occurs in cold humid climate. It is a
air due to small size quickly due to comparative mixture of smoke, fog and sulphur dioxide.

heaviness 4. (a) Water considered to be clean if it has BOD less than

Catalytic converters is a device designed to reduce the 5 ppm whereas highly polluted water has BOD more
amount of emissions from automobiles. The current than 17 ppm.
(so-called three-way) systems use a heated metal Therefore, water having BOD less than 5 ppm is rich in
catalyst to reduce the emissions of carbon monoxide dissolved oxygen.
(CO), hydrocarbons, and nitric oxide (NO), all of which
5. (a) O3 is responsible for greenhouse effect. Its contribution
contribute to the formation of photochemical smog. In
is about 8%.
an automobile’s exhaust system, a catalytic converter
provides an environment for a chemical reaction where 6. (c) The large population of bacteria decomposes organic
unburned hydrocarbons completely combust. matter present in water. They consume oxygen dissolved
in water. Hence, oxygen from water decreases. It is
23. (b) Eutrophication is a natural process which literally means
harmful for aquatic life.
well nourished or enriched. It is a natural state in many
lakes and ponds which have a rich supply of nutrients. 7. (b) Photochemical smog has high concentration of oxidants
Eutrophication become excessive, however when such as O3, organic oxidant and is therefore called as
abnormally high amount of nutrient from sewage, oxidising agent.
fertilizers, animal wastage and detergent, enter streams 8. (b) The lowest region of the atmosphere in which human
and lakes causes excessive growth or blooms of beings along with other organisms live is called
microorganisms. With increasing eutrophication, the troposhere. It extends upto the height of ~ 10 km from
diversity of the phytoplankton community of a lake sea level. Troposphere is a turbulent, dusty zone
increases and the lake finally becomes dominated by
containing air, much water vapour and clouds.
blue - green algae.
9. (a) Nitrogen and oxygen do not react with each other at
24. (d) Phosgene (COCl 2) is a poisonous and suffocating
normal temperature. At high altitude when lightning
volatile liquid. Release of methyl isocyanate in industrial
strikes, they combine to form oxides of nitrogen.
accident of Bhopal was the main reason of Bhopal gas
tragedy. 3000° C
N 2 ( g ) + O 2 ( g ) ¾¾¾¾® 2NO ( g )
25. (c) Other gases like CFCs, Ozone, water vapour and nitrous
oxide also show green house effect. 10. (c) Hydrocarbons present in atmosphere combine with
oxygen atom produced by the photolysis of NO2 to
EXERCISE - 3 form highly reactive intermediate called free radical. Free
Exemplar Questions radical initiates a series of reaction.
1. (a) Those gases which absorb sunlight near the earth's Peroxyacetyl nitrates are formed, which can be said as
surface and then radiates back to the earth are called secondary pollutants.
green house gases.
Carbon dioxide, water vapour, methane, ozone, oxides Hydrocarbon + O ® RCO. (free radicals)
of nitrogen, chlorofluoro carbons (CFCs) etc; are green
house gases. CO is not a green house gas. RCO. + O2 ® RCO3

2. (c) The smog which is formed in presence of sunlight is


called photochemical smog. The main components of RCO3g + NO2 ® RCO3 NO 2
Peroxyacetyl nitrate
the photochemical smog results from the action of
EBD_7327
352 CHEMISTRY

11. (c) Ozone hole is thinning of ozone layer of stratosphere 14. (d) Acid rain is the rain water containing sulphuric acid
at some place. NO and chlorofluorocarbon have been and nitric acid which are formed from the oxides of
found to be the most responsible for depleting the sulphur and nitrogen present in the air as pollutants
ozone layer. and rain water has a pH range of 4-5.
12. (d) Using plastic cans for neatly storing substances will 15. (d) The oxidised hydrocarbons and ozone in presence of
not come under green chemistry. The plastic materials humidity cause photochemical smong.
are non-biodegradable. Hydrocarbons + O2, NO2, NO, O, O3 ® Peroxides,
formaldehyde, peroxyacetyl-nitrate (PAN), acrodein etc.
NEET/AIPMT (2013-2017) Questions
Hence chlorofluoro carbons are not common component
13. (a) Based on the features given gas must be SO2 . of photochemical smog.
16. (a) Microorganisms present in the soil is a sink for CO.
15 The Solid State

SOLIDS Crystalline solids :


Solids are characterised by the state of matter in which particles A crystalline solid usually consists of a large number of small
are closely packed and held together by strong intermolecular crystals, each of them having a definite characteristic geometrical
attractive forces. shape.
Types of Solids : Amorphous solids :
A solid can be classified as
Solids Amorphous solids have no precise melting point but when heated,
become increasingly pliable until they assume the properties usu-
ally associated with liquids.
Crystalline Solids Amorphous Solids

Difference Between Crystalline and Amorphous Solids :

Property Crystalline Solids Amorphous Solids


Shape Definite, Characteristic geometrical shape Irregular shape
Melting Point Melt at a sharp and characteristic temperature Gradually soften over a range of
temperature
Cleavage property Give plain and smooth surfaces on cutting with Give irregular surfaces when cut with a
a sharp edged tool sharp edged tool
Heat of fusion Definite and characteristic Not definite
Nature True solids Pseudo solids or supercooled liquids

Order in arrangement of Long range order Only short range order


particles
Anisotropy and Isotropy Anisotropic in nature, crystalline solids show Isotropic in nature, Amorphous solids are
different values of physical properties along isotropic in nature because some of their
different directions and hence they are properties like electrical resistance,
anisotropic in nature. refractive index, thermal exapansion, etc.,
are same in all the directions.
Examples Crystalline solids:- All solids elements (metals Amorphous solids:- rubber, glass, fused
and non-metals) and compounds exist in this silica, plastics, etc.
form.
EBD_7327
354 CHEMISTRY

cooling.
Note : (c) Amorphous silicon is one of the best photovoltaic material
(a) Glass is considered a super cooled liquid because glass is a available for conversion of sunlight into electricity.
amorphous solid and hence, it shows the property to flow, CLASSIFICATION OF CRYSTALLINE SOLIDS
though slowly. This is supported by the fact that glass panes
fixed to windows or doors of old buildings are invariably Crystalline solids are classified on the basis of nature of
found to be slightly thicker at the bottom than at the top. intermolecular forces operating in them into four categories, viz.
(b) Any material can be made amorphous or glassy either by molecular, ionic, metallic and covalent solids. The different
rapidly cooling its melt or freezing its vapours. For example : properties of the four types of solids are given in the table below:
Silica (SiO2) which crystallises as quartz can be converted
into amorphous silica (i.e. silica glass) by melting and rapid

Type of Nature of Physical Electrical Melting


Constituent Examples
Solid Binding Nature Conductivity Point
Particles Forces
(1) Molecular Solids Atoms of noble Dispersion or Ar, H2, I2, Soft Insulator Very low
gas or non-polar London forces CH4, Solid CO2
(i) Non-Polar
molecules
(ii) Polar Polar molecules Dipole-dipole HCl, SO2 Soft Insulator Low
interactions

(iii) Hydrogen Molecules Hydrogen H2O (ice), NH3 Hard Insulator Low
bonded containing H bonding
linked to F,
O or N
(2) Ionic Solids Ions Coulombic or NaCl, LiF, Hard but Insulators in solid High
electrostatic Zns, CaF2, brittle state but conductors
forces MgBr2 in molten state and
in aqueous solution
state
(3) Metallic Solids Positive ions Metallic All metals and Hard but Conductors in Fairly High
in a sea of bonding alloys malleable solid as well
delocalised and as in molten state
electrons ductile

(4) Covalent or Atoms Covalent SiO2 (quartz), Hard Insulators


Network Solids bonding SiC, C(diamond), Very High
Soft Conductor
AlN, C(graphite) (exception)

Note : CRYSTAL LATTICES AND UNIT CELLS


(a) Ionic solids are insulators in the solid state because ions are
Crystal :
not free to move. However, in the molten state, when
dissolved in water, the ions become free to move about and A crystal is a homogeneous portion of a solid substance made by
they conduct electricity. regular pattern of structural units bonded by plane surface making
(b) Exceptional behaviour of graphite : definite angles with each other.
Graphite is soft, conductor of
Space Lattice :
electricity and a good solid
lubricant because of its typical The arrangement of constituents particles like atoms, ions or
structure. Carbon atoms are molecules in different sites in three dimensional space is called
arranged in different layers and space lattice.
each atom is covalently bonded to There are only 14 possible three dimensional lattices and these are
three of its neighbouring atoms in called Bravais Lattices. The characteristics of a crystal lattice are:
the same layer. The fourth valence (i) Each point in a lattice is called lattice point or lattice site.
electron of each atom is present (ii) Each point in a crystal lattice represents one constituent
between the layers and is free to
particle which may be an atom, a molecule or an ion.
move. This imparts graphite the
(iii) Lattice points are joined by straight lines to bring out the
above mentioned properties.
Structure of graphite geometry of the lattice.
The Solid State 355

Unit Cell : bcc unit cell :


It is the smallest portion of the crystal lattice which when repeated Total number of atoms per unit cell
in different directions, generates the entire lattice. A unit cell is (i) Eight Corner atoms contribute one atom per unit cell.
characterised by six parameters, i.e., a, b, c, a, b and g; where a, b (ii) Centre atom contribute one atom per unit cell.
and c are its dimensions along the edges while a, b and g are the (iii) So, total 1 + 1 = 2 atoms per unit cell.
angles between the edges. These are depicted in the figure given
below:
1
Z=8 × + 1=2
8

fcc unit cell :


c Total number of atoms per unit cells
b ab 1
(i) The eight corners atoms contribute for of an atom and
a g 8
thus one atom per unit cell.
(ii) Each of six face centered atoms is shared by two adjacent
Unit cells can be broadly divided into two categories: unit cells and therefore one face centred atom contribute half
Primitive unit cells : of its share. Means
Unit cell in which constituent particles are present only on the
corner positions of a unit cell.
Centred or non-primitive unit cells : 1
6× = 3 atom per unit cell.
Unit cell in which constituent particles are present at positions 2
other than corners in addition to those at corners
(iii) So, total Z = 3 + 1 = 4 atoms per unit cell.
Types of Non-Primitive Unit Cells :
(i) Body-centred unit cells(bcc) : CLOSE PACKING IN CRYSTALS :
Unit cell in which besides corners, one constituent particle is Close-packing of particles results in two highly efficient lattices,
present at the body-centre. hexagonal close packed (hcp) and cubic close packed (ccp). The
(ii) Face-centred unit cells (fcc) : latter is also called face centered cubic (fcc) lattice.
Unit cell in which besides corners, one constituent particle is Hexagonal Close Packing :
present at the centre of each face. In hcp, the pattern of spheres is repeated in alternate layers and is
(iii) End-centred unit cells : often written as ABAB _ _ _ _ pattern.
Unit cell in which besides corners, one constituent particle is
present at the centre of any two opposite faces.
Contribution of Atoms Present at Different Lattice
Sites :
In a crystal, atoms located at the corners and face-centre of a unit
cell are shared by other cells and only a portion of such an atom
actually lies within a given unit cell
(i) A point that lies at the corner of unit cell is shared among
eight unit cells and thus, it contributes for 1/8 of each such
point to unit cell.
(ii) A point along an edge is shared by four unit cells and thus
contribute for 1/4 of each such point to unit cell.
(iii) A face-centred point is shared by two unit cells and thus
contributes for 1/2 of each such point to unit.
(iv) A body-centred point lies entirely within the unit cell and
thus contributes for one each such point to unit cell.
Calculation Number of Atoms in a Unit Cell
Primitive cubic/simple cubic unit cell :
1
Total number of atoms in one unit cell is 8 ´ = (1)
8 Here each atom is surrounded by 12 others and is said to have
(8 corners × each corner is shared by 8 particles) co-ordination number of 12.
EBD_7327
356 CHEMISTRY

The hcp arrangement of atoms occupies 74% of the available space If the number of close packed spheres is N, then the number
and thus has 26% vacant space. of octahedral voids generated = N and, the number of
It has only one set of parallel close packed layers. tetrahedral voids generated = 2N
Hexagonal close packed structure is found in Be, Cd, Li, Ca, Cr, Note : In ionic compounds anions are present in the packing
Mo, V, Mg, Zn, Ce, Zr, Os, Ru, He, etc. whereas cations occupy the voids if the cations are small,
Cubic Close Packing : they may occupy tetrahedral voids while if cations are large
they occupy octahedral voids. Further, it is not necessary
In ccp, the pattern of layers is often written as ABCABC _ _
that all voids must be occupied. The formula of the compound
A can be calculated by knowing the fraction of voids occupied.
COORDINATION NUMBER :
In every ionic compound, positive ions are surrounded by negative
ions and vice versa. Normally each ion is surrounded by the largest
C possible number of oppositely charged ions. This number of
oppositely charged ions surrounding each ion is termed its
coordination number. The coordination number of positive and
negative ions of a compound are same when the two types of ions
are equal in number (e.g., NaCl, ZnS, etc.). On the other hand,
B when an ionic compound contains different number of positive
and negative ions (as in CaCl2 , Na2S etc.), the coordination
numbers of positive and negative ions are different. For example,
in CaCl2 since Cl– ions are twice the number of Ca2+ ions, the
coordination number of calcium ion is twice the coordination
A number of chloride ion.
As in hcp, each atom in ccp arrangement has 12 nearest RADIUS RATIO RULES :
neighbours. In other words, the co-ordination number of the atom
The ratio of the radii of the cation to the anion in crystal lattice
is 12.
is called radius ratio.
The ccp arrangement of atoms occupy 74% of the available space
and thus has 26% vacant space. r Radius of the cation
Radius ratio = =
It has four sets of parallel close packed layers. Hence the chances R Radius of the anion
for slipping of one layer over the other are more in the ccp Table shows the relationship of radius ratio, co-ordination num-
arrangement than in the hcp arrangement. Hence metals having ber and the geometrical shapes of some ionic crystals.
ccp structure (e.g. Cu, Ag, Au and Pt) are highly malleable and
ductile.
Cubic close-packed structure is found in Al, Cr, Co, Cu, Au, Ag, Limiting radius ratio C.N. Shape
Fe, Pb, Mn, Ni, Ca, Sr, Pt, all noble gases except He. (r+) / (r–)
Type and Number of Voids < 0.155 2 Linear
Close-packing of particles in hcp or ccp results in two types of 0.155 – 0.225 3 Planar triangle
voids : octahedral and tetrahedral. 0.225 – 0.414 4 Tetrahedral
(i) Tetrahedral void : Tetrahedral voids are formed when centres 0.414 – 0.732 4 Square planar
of four spheres are joined, i.e, 0.414 – 0.732 6 Octahedral
0.732 – 0.999 8 Body-centred cubic
Tetrahedral EFFECT OF PRESSURE AND TEMPERATURE ON
void CRYSTAL STRUCTURE
Effect of Pressure
Increase of pressure increases the co-ordination number during
If the number of close packed spheres is N, then the number crystallization e.g., by applying high pressure, the NaCl crystal
of tetrahedral voids generated = 2 N structure having 6 : 6 co-ordination number changes to CsCl
(ii) Octahedral void : A void surrounded by six spheres (i.e. crystal structure having co-ordination number 8 : 8.
when triangular voids in the second layer are above the
high pressure
triangular voids in the first layer) is called octahedral void, ¾¾¾¾¾® CsCl type crystal
NaCl type crystal ¬¾¾¾¾¾
i.e., heat, 760 K
Octahedral (6 : 6 co-ordination) (8 : 8 co-ordination)
void Effect of Temperature
Increase of temperature, however, decreases the co-ordination
number e.g., upon heating to 760 K, the CsCl crystal structure
having co-ordination of 8 : 8 changes to NaCl crystal structures
having co-ordination 6 : 6.
The Solid State 357

PACKING EFFICIENCY Points defect is of 3 types :


It is the percentage of total space filled by the particles. (i) Stoichiometric Defects :
Those point defects that do not disturb the stochiometry of
Relation between 'a 'and ' r ' the solid.
Type of Packing
a : edge length (a) Vacancy defect : This defect arises when some lattice
structure Efficiency sites are vacant. Thus generally arises due to absorption
r : radius of sphere
of heat from the surroundings. It decreases the density
hcp and a of a substance.
r= 74%
ccp ( fcc ) 2 2 (b) Interstitial defect : This defect arises when some
constituent particles occupy an interstitial site. It
3 increases the density of the substance.
bcc r= a 68% The above two types of defects are generally shown by
4
Simple
non-ionic solids. Ionic solids do not show simple vacancy
a and interstitial defects. Instead, they show these defects
cubic r= 52.4% as Schottky and Frenkel defects, as explained below:
2 (c) Frenkel defect : This defect arises when an ion is missing
lattice
from its lattice site and it occupies the interstitial site. It
Relationships between the nearest neighbour distance (d) and does not change the density of the crystal.
the edge (a) of unit cell of a cubic crystal. Frenkel defects are common in ionic compounds which
Simple Face-centred Body-centred have low co-ordination number and in which there is
large difference in size between positive and negative
a 3 ions.
d=a d= d= a
2 2 + – + – +
A B A B A
= 0.707 a = 0.866 a
+
A
DENSITY OF A UNIT CELL – – + –
For cubic crystals of elements B B A B
Z´M
r= g/cm3
3 -30 + – + – +
a ´ N 0 ´ 10 A B A B A
For cubic crystals of ionic compounds
ZM g/cm3 or kg m–3 B

A
+
B

A
+ –
B
r=
a3 NA
Fig. Frenkel defect
where r ® density of the unit cell Eg. ZnS, AgCl, AgBr, AgI etc.
Z ® no. of atoms present in one unit cell (d) Schottky defect : This defect arises when equal number
M ® molar mass of cations and anions are missing from their lattice sites.
a ® edge length of the unit cell It decreases the density of substance.
NA ® Avogadro’s number Schottky defect is more common in ionic compounds
BRAGG EQUATION : with high co-ordination number and where the size of
According to Bragg, a crystal (composed of series of equally positive and negative ions are almost equal.
spaced atomic planes) could be employed not only as a – –
+ + +
transmission grating (as suggested by Laue) but also as a A B A B A
reflection grating. When X-rays are incident on a crystal face,
these penetrate into the crystal and strike the atoms in successive – – + –
planes. From each of these planes the X-rays are reflected like the B B A B
reflection of a beam of light from a bundle of glass plates of equal
thickness. Based on this model, Bragg derived a simple relation + – + +
between the wavelength (l) of the X-rays used, the distance (d) A B A A
between the successive atomic planes and the angle of incident
X-rays or the angle of reflection (q). – + – + –
B A B A B
n l = 2d sin q (where n = 1, 2, 3, ......)
The equation is known as Bragg’s equation or Bragg’s law. The Fig. Schottky defect
reflection corresponding to n = 1 (for a given family of planes) is Eg. NaCl, KCl, CsCl, KBr etc.
called first order reflection; the reflection corresponding to n = 2 Note : AgBr shows both i.e., Frenkel and Schottky defect.
is the second order reflection and so on. (ii) Impurity defects :
IMPERFECTIONS IN SOLIDS These defects arise when foreign atoms are present at lattice
Point Defects and Line Defects : site (in place of host atoms) or at vacant interstitial sites.
Broadly the defects in crystals are of two types point and line (iii) Non-stoichiometric defects :
defects. Point defects are irregularities from ideal arrangement As a result of these defects, the ratio of cations to anions
around a point or an atom in lattice. However, when the deviation becomes different from that indicated by ideal chemical
from the ideal arrangement exist in the entire row of lattice points, formula.
the defect is called line defect. Example : VOx (where x can vary between 0.6 to 1.3)
EBD_7327
358 CHEMISTRY

Two types of non-stoichiometric defects are : Thus, the impurities added may be electron rich or electron deficit.
(a) Metal excess defect : Each of these is briefly described below :
Metal excess defects due to anion vacancies (i) Doping with electron rich impurities : Group 14 element like
It occurs due to anionic vacancies i.e. anions diffuse to the silicon or germanium has 4 electrons in the valence shell.
surface of the crystal resulting in anionic sites which are When it is doped with Group 15 element like P or As, the
occupied by unpaired electrons. These sites are called silicon or germanium atoms at some lattice sites are
F-centres. The e–1s absorb energy from the visible light and substituted by atoms of P or As. Now, as these atoms have
impart colour to crystal 5 electrons in the valence shell, after forming normal four
Examples : covalent bonds with the neighbouring silicon atoms, the fifth
(1) The excess sodium in NaCl makes the crystal appears extra electron is free and gets delocalized. These delocalized
yellow. electrons increase the conductivity of silicon or germanium,
(2) Excess potassium in KCl makes it violet. the silicon or germanium crystals doped with electron rich
(3) Excess lithium in LiCl makes it pink. impurities are called n-type semiconductors.
Note : Greater the number of F-centres, greater is the intensity (ii) Doping with electron deficit impurities : When Group 14
of colour. element like Si or Ge is doped with Group 13 element like B, Al
Metal excess defects due to interstitial cations : or Ga, the Si or Ge atom at some lattice sites are substituted
It may occur if an extra positive ion is present in an interstitial by those of B, Al or Ga. Now, as Group 13 elements have only
site. Electrical neutrality is maintained by the presence of an three valence electrons, they can form three covalent bonds
electron in the interstitial site. with the neighbouring silicon atoms. Thus, a hole is created
Example : When ZnO is heated, it loses oxygen and turns at the site where fourth electron is missing. This is called
yellow due to: electron hole or electron vacancy. Electron deficit doped
2+ 1 – silicon or germanium are called p-type semiconductors.
ZnO Zn + O2 + 2e
2 MAGNETIC PROPERTIES OF SOLIDS
The excess of Zn2+ ions get trapped into vacant interstitial
sites. Magnetic properties of materials are studied in terms of magnetic
(b) Metal Deficiency defect : moments which arise due to orbital motion and spinning motion
It occurs due to missing of cation from its lattice site and of electrons.
presence of higher charge cation in adjacent site. It occurs Based on their behaviour in the external magnetic field, substances
when metals show variable valency, Example : FeO, FeS and are classified as below :
NiO. Diamagnetic Substances :
Those substances which are weakly repelled by external magnetic
ELECTRICAL PROPERTIES OF SOLIDS field. It is shown by substances which contain fully-filled orbitals.
On the basis of conductivity, solids are classified as: For ex: TiO2, H2O, NaCl, benzene, etc.
(i) Conductors : The solids having conductivities ranging Paramagnetic Substances :
between 104 to 107 W–1 m–1. Metals are good conductors. Which are weakly attracted by external magnetic field. It is shown
(ii) Insulators : The solids having very low conductivities ranging by substances which contain unpaired e–1s. Example : O2, Cu2+,
between 10–20 to 10–10 W–1 m–1. Example : plastic, rubber, Fe3+, Cr3+. However, they loose their magnetism in the absence
non-metals etc. of magnetic field.
(iii) Semiconductors: The solids having conductivities in the Ferromagnetic Substances:
intermediate range from 10–6 to 104 W–1 m–1. The substances which are strongly attracted by magnetic field are
Band Theory : termed as ferromagnetic substances. This type of substance has
The behaviour of conductors, insulators and semiconductors are alignment of all the unpaired electrons in the same direction
explained on the basis of band theory. The atomic orbitals of (orientation) These substances are permanently magnetised i.e.
metals overlap to form a large number of molecular orbitals and these substances show magnetism even in the absence of
this set is called a band. Two bands are formed: magnetic field. Examples are Ni, Fe, Co and CrO2.
Valence band (lower energy) and conduction band (higher energy) Anti-Ferromagnetic Substances :
If valence band is partially filled or overlaps with higher energy Substances which possess zero net magnetic moment. It is due to
conduction band, e–1s can easily flow under the influence of presence of equal number of electrons in opposite directions. For
electric field. Thus, metals conduct electricity. ex: MnO, Mn2O3. Their magnetic moment will compensate each
If gap between filled valence band and unoccupied conduction others magnetic moment.
band is large, e–1s cannot jump from valence band to conduction Ferrimagnetic Substances :
band. Hence, the substance has extremely low conductivity and it When unequal number of unpaired electrons are aligned in
behaves as an insulator. opposite directions, the net magnetic moment is not zero. Such
If the gap between valence band and conduction band is small, substances are termed as ferrimagnetic substances e.g., ferrite
some e–1s may jump from valence band to conduction band. Hence, Fe2O3.
the substance shows some conductivity and it acts as a semi Note :
conductor. Electrical conductivity of semiconductor increases with (i) All magnetically ordered substances, i.e., ferromagnetic and
temperature as more electrons can jump to conduction band. Pure anti-ferromagnetic solids change into paramagnetic at high
substances like Si and Ge show this type of behaviour and are temperature. This is due to randomisation of domains (spins)
called intrinsic semiconductor. on heating. For ex: ferrimagnetic substance, Fe3O4, becomes
Extrinsic semiconductors are obtained by adding impurity to a paramagnetic at 850 K.
semiconductor so as to increase its conductivity. (ii) Each ferromagnetic substance has a characteristic
The process of adding impurities to a crystalline substance so as temperature above which no ferromagnetism is observed.
to change its properties is called doping. This is known as Curie temperature.
The Solid State 359

CONCEPT MAP
EBD_7327
360 CHEMISTRY

1. Frenkel and Schottky defects are : 12. The element which is used as semiconductor is
(a) nucleus defects (b) non-crystal defects (a) Al (b) Bi
(c) crystal defects (d) nuclear defects (c) Cr (d) Si
2. Which of the following compounds is a good conductor of 13. A solid has a structure in which ‘W’ atoms are located at the
electricity in solution state ? corners of a cubic lattice ‘O’ atoms at the centre of edges
(a) Covalent (b) Molecular and Na atoms at the centre of the cube. The formula for the
(c) Metallic (d) Ionic compound is
3. Most crystals show good cleavage because their atoms, ions (a) Na2WO3 (b) Na2WO2
or molecules are (c) NaWO2 (d) NaWO3
(a) weakly bonded together 14. A solid is made of two elements X and Z. The atoms Z are in
(b) strongly bonded together ccp arrangement while the atoms X occupy all the tetrahedral
(c) spherically symmetrical sites. What is the formula of the compound?
(d) arranged in planes (a) XZ (b) XZ 2
4. Certain crystals produce electric signals on application of
(c) X2Z (d) X 2 Z3
pressure. This phenomenon is called :
(a) pyroelectricity (b) ferroelectricity 15. The radius ratio in CsCl is 0.93. The expected lattice structure
(c) piezoelectricity (d) ferrielectricity is
5. A solid with high electrical and thermal conductivity is (a) tetrahedral (b) square planar
(a) Si (b) Li (c) octahedral (d) body-centred cubic
(c) NaCl (d) Ice 16. In NaCl crystal each Cl– ion is surrounded by
6. Solid CH4 is (a) 4 Na+ ions (b) 6 Na+ ions
(a) ionic solid (b) covalent solid (c) 1 Na+ ions (d) 2 Na+ ions
(c) molecular solid (d) does not exist 17. Edge length of a cube is 400 pm, its body diagonal would be
7. An example of a covalent crystalline solid is: (a) 566 pm (b) 600 pm
(a) Si (b) Al (c) 500 pm (d) 693 pm
(c) NaF (d) Ar 18. In stoichiometric defects, the types of compound exhibit
8. The pure crystalline substance on being heated gradually Frenkel defects have/has
first forms a turbid liquid at constant temperature and still at (a) Low co-ordination nos.
higher temperature turbidity completely disappears. The (b) High co-ordination.
behaviour is a characteristic of substance forming (c) Small difference in the size of cations and anions
(a) Allotropic (b) Liquid (d) None of these.
(c) Isomeric (d) Isomorphous 19. For orthorhombic system axial ratios are a ¹ b ¹ c and the
9. Na and Mg crystallize in the bcc and fcc type crystals axial angles are
respectively, then the number of atoms of Na and Mg present
in the unit cell of their respective crystal is : (a) a = b = g ¹ 90° (b) a = b = g = 90°
(a) 4 and 2 (b) 9 and 14 (c) a = b = g =90°, b =90° (d) a ¹ b ¹ g = 90°
(c) 14 and 9 (d) 2 and 4 20. In the fluorite structure, the coordination number of Ca2+
10. For fcc if AB is just like the rock salt like structure then, A+ ion is :
and B– are located at – (a) 4 (b) 6
(a) A+ - Tetrahedral voids ; B– - Corner (c) 8 (d) 3
(b) A+ - Corner and faces ; B– - Octahedral voids 21. Na and Mg crystallize in bcc and fcc type crystals
(c) A+ - Octahedral voids ; B– - Corner and faces respectively, then the number of atoms of Na and Mg present
(d) A+ - Corner and faces ; B– - Octahedral and tetrahedral in the unit cell of their respective crystal is
voids (a) 4 and 2 (b) 9 and 14
11. The ionic radii of X+ and Y– ions are 146 and 216 pm (c) 14 and 9 (d) 2 and 4
respectively. The probable type of structure shown by it is 22. In the solid state, MgO has the same structure as that of
(a) CsCl type (b) Rock salt type sodium chloride. The number of oxygens surrounding each
(c) Zinc blend type (d) CaF2 type magnesium in MgO is
(a) 6 (b) 1
(c) 2 (d) 4
The Solid State 361

23. Equal number of atoms or ion missing from normal lattice


r+
point creating a vacancy due to 35. For a cubic geometry the limiting is :
(a) Frenkel defect (b) Mass defect r–
(c) Schottky defect (d) Interstitial defect (a) 0.414 (b) 0.155
24. When molten zinc is converted into solid state, it acquires (c) > 0.731 (d) 0.731
hcp structure. The number of nearest neighbours of Zn will 36. CsBr crystallises in a body centered cubic lattice. The unit
be cell length is 436.6 pm. Given that the atomic mass of Cs =
(a) 6 (b) 12 133 and that of Br = 80 amu and Avogadro number being
(c) 8 (d) 4 6.02 × 1023 mol–1, the density of CsBr is
25. Hexagonal close packed arrangement of ions is described as (a) 0.425 g/cm3 (b) 8.5 g/cm3
(a) ABC ABA (b) ABC ABC (c) 4.25 g/cm 3 (d) 82.5 g/cm3
(c) ABABA (d) ABBAB 37. The pyknometric density of sodium chloride crystal is
2.165 × 103 kg m–3 while its X-ray density is 2.178 × 103 kg m–3.
26. CsBr has bcc structure with edge length 4.3. The shortest
The fraction of unoccupied sites in sodium chloride crystal
interionic distance in between Cs+ and Br– is
is
(a) 3.72 (b) 1.86
(a) 5.96 × 10–3 (b) 5.96 × 104
(c) 7.44 (d) 4.3 –2
(c) 5.96 × 10 (d) 5.96 × 10–1
27. Schottky defect generally appears in : 38. Potassium fluoride has NaCl type structure. What is the
(a) NaCl (b) KCl distance between K+ and F– ions if cell edge is ‘a’ cm.
(c) CsCl (d) all of these (a) 2a cm (b) a/2 cm
28. How many unit cells are present in a cube-shaped ideal (c) 4a cm (d) a/4 cm
crystal of NaCl of mass 1.00 g ? 39. In the Bragg’s equation for diffraction of X-rays, n represents
[Atomic masses : Na = 23, Cl = 35.5] for
(a) 5.14 × 1021 unit cells (b) 1.28 × 1021 unit cells (a) quantum number (b) an integer
(c) 1.71 × 1021 unit cells (d) 2.57 × 1021 unit cells (c) Avogadro’s numbers (d) moles
29. In a solid ‘AB’ having the NaCl structure, ‘A’ atoms occupy 40. The edge length of unit cell of a metal having molecular
the corners of the cubic unit cell. If all the face-centered weight 75 g/mol is 5Å which crystallizes in cubic lattice. If
atoms along one of the axes are removed, then the resultant the density is 2g/cc then find the radius of metal atom.
stoichiometry of the solid is (NA = 6 × 1023). Give the answer in pm.
(a) AB2 (b) A2B (a) 217 pm (b) 210 pm
(c) A4B3 (d) A3B4 (c) 220 pm (d) 205 pm
30. A solid AB crystallises as NaCl structure and the radius of 41. An element occuring in the bcc structure has 12.08 × 1023 unit
the cation is 0.100nm. The maximum radius of the anion can cells. The total number of atoms of the element in these cells
be: will be
(a) 0.137 nm (b) 0.241 nm (a) 24.16 × 1023 (b) 36.18 × 1023
(c) 6.04 × 10 23 (d) 12.08 × 1023
(c) 0.274 nm (d) 0.482 nm
42. The number of atoms in 100 g of an fcc crystal with density,
31. Edge length of a cube is 400 pm. Its body diagonal would be
d = 10 g/cm3 and cell edge equal to 100 pm, is equal to
(a) 500 pm (b) 566 pm
(a) 1 × 1025 (b) 2 × 1025
(c) 600 pm (d) 693 pm 25
(c) 3 × 10 (d) 4 × 1025
32. In the calcium fluoride structure, the coordination number 43. Pottasium has a bcc structure with nearest neighbour
of the cation and anion are respectively distance 4.52 Å. Its atomic weight is 39. Its density (in kg m–3)
(a) 6, 6 (b) 8, 4 will be
(c) 4, 4 (d) 4, 8 (a) 454 (b) 804
33. The limiting radius ratio for tetrahedral shape is: (c) 852 (d) 910
(a) 0 to 0.155 (b) 0.225 to 0.414 44. The radii of Na+ and Cl– ions are 95 pm and 181 pm
(c) 0.155 to 0.225 (d) 0.414 to 0.732 respectively. The edge length of NaCl unit cell is
34. The second order Bragg diffraction of X-rays with = 1.00 Å (a) 276 pm (b) 138 pm
from a set of parallel planes in a metal occurs at an angle 60º. (c) 552 pm (d) 415 pm
The distance between the scattering planes in the crystal is 45. The cubic unit cell of a metal (molar mass = 63.55g mol–1) has
(a) 0.575 Å (b) 1.00 Å an edge length of 362 pm. Its density is 8.92g cm–3.
(c) 2.00 Å (d) 1.15 Å The type of unit cell is
(a) primitive (b) face centered
(c) body centered (d) end centered
EBD_7327
362 CHEMISTRY

46. Coordination numbers of Zn2+ and S2– in the crystal structure 59. If NaCl is doped with 10– 4 mol % of SrCl2, the concentration
of wurtzite are of cation vacancies will be (NA = 6.02 × 1023 mol–1)
(a) 4, 4 (b) 6, 6 (a) 6.02 × 1016 mol–1 (b) 6.02 × 1017 mol–1
14
(c) 6.02 × 10 mol –1 (d) 6.02 × 1015 mol–1
(c) 8, 4 (d) 8, 8
47. Each of the following solids show, the Frenkel defect except 60. The fraction of total volume occupied by the atoms present
(a) ZnS (b) AgBr in a simple cube is
(c) AgI (d) KCl p p
48. Doping of silicon (Si) with boron (B) leads to : (a) (b)
3 2 4 2
(a) n-type semiconductor (b) p-type semiconductor
(c) metal (d) insulator p p
49. Schottky defect defines imperfection in the lattice structure (c) (d)
4 6
of
61. If ‘a’ stands for the edge length of the cubic systems : simple
(a) solid (b) gas
cubic, body centred cubic and face centred cubic, then the
(c) liquid (d) plasma
ratio of radii of the spheres in these systems will be
50. On doping Ge metal with a little of In or Ga, one gets
respectively,
(a) p-type semi conductor (b) n-type semi conductor
(c) insulator (d) rectifier 1 3 1 1 1
(a) a: a: a (b) a : 3a : a
51. If Z is the number of atoms in the unit cell that represents the 2 4 2 2 2 2
closest packing sequence ..... ABC ABC ......, the number of
tetrahedral voids in the unit cell is equal to : 1 3 3
(a) Z (b) 2Z (c) a: a: a (d) 1a : 3a : 2a
2 2 2
(c) Z/2 (d) Z/4
62. With which one of the following elements silicon should be
52. Schottky defect in crystals is observed when doped so as to give p-type of semiconductor ?
(a) an ion leaves its normal site and occupies an interstitial
(a) Germanium (b) Arsenic
site
(c) Selenium (d) Boron
(b) unequal number of cations and anions are missing from
the lattice 63. AB; crystallizes in a body centred cubic lattice with edge
(c) density of the crystal increases length ‘a’ equal to 387 pm. The distance between two
oppositely charged ions in the lattice is :
(d) equal number of cations and anions are missing from
the lattice (a) 335 pm (b) 250 pm
53. The appearance of colour in solid alkali metal halides is (c) 200 pm (d) 300 pm
generally due to 64. Among the following which one has the highest cation to
(a) Schottky defect (b) Frenkel defect anion size ratio?
(c) Interstitial positions (d) F-centre (a) NaF (b) CsI
54. The crystal system of a compound with unit cell dimensions (c) CsF (d) LiF
"a = 0.387, b = 0.387 and c = 0.504 nm and a = b = 90° and 65. A solid compound XY has NaCl structure. If the radius of
g = 120°" is the cation is 100 pm, the radius of the anion (Y–) will be :
(a) cubic (b) hexagonal (a) 275.1 pm (b) 322.5 pm
(c) orthorhombic (d) rhombohedral (c) 241.5 pm (d) 165.7 pm
55. Crystal defect indicated in the diagram below is 66. A metal crystallizes with a face-centered cubic lattice. The
Na + Cl - Na + Cl - Na + Cl - Cl– c Cl– Na+ c Na+ edge length of the unit cell is 408 pm. The diameter of the
Na+Cl– c Cl– Na+ Cl– Cl– Na+ Cl–Na+ c Na+ metal atom is :
(a) Interstitial defect (a) 288 pm (b) 408 pm
(b) Schottky defect (c) 144 pm (d) 204 pm
(c) Frenkel defect 67. In a compound, atoms of element Y form ccp lattice and
(d) Frenkel and Schottky defects those of element X occupy 2/3rd of tetrahedral voids. The
56. Schottky defect generally appears in : formula of the compound will be
(a) NaCl (b) KCl (a) X4Y3 (b) X2Y3
(c) CsCl (d) all of these (c) X2Y (d) X3Y4
57. Which defect causes decrease in the density of crystal 68. Copper crystallises in fcc with a unit cell length of 361 pm.
(a) Frenkel (b) Schottky What is the radius of copper atom?
(c) Interstitial (d) F – centre (a) 127 pm (b) 157 pm
58. What is the coordination number of sodium in Na2O? (c) 181 pm (d) 108 pm
(a) 6 (b) 4
(c) 8 (d) 2
The Solid State 363

69. The edge length of a face centered cubic cell of an ionic


substance is 508 pm. If the radius of the cation is 110 pm, the
radius of the anion is
(a) 288 pm (b) 398 pm
(c) 618 pm (d) 144 pm
2+ –
70. The Ca and F are located in CaF2 crystal, respectively at
face centred cubic lattice points and in L
(a) tetrahedral voids (b) half of tetrahedral voids (a) 39.27% (b) 68.02%
(c) octahedral voids (d) half of octahedral voids (c) 74.05% (d) 78.54%
71. KCl crystallises in the same type of lattice as does NaCl. 75. A compound M p X q has cubic close packing (ccp)
Given that r + / r - = 0.55 and r + / r - = 0.74. arrangement of X. Its unit cell structure is shown below. The
Na Cl K Cl
Calculate the ratio of the edge length of the unit cell for KCl empirical formula of the compound is
to that of NaCl.
(a) 1.123 (b) 0.891
(c) 1.414 (d) 0.414
72. Percentages of free space in cubic close packed structure
and in body centered packed structure are respectively M
(a) 30% and 26% (b) 26% and 32%
(c) 32% and 48% (d) 48% and 26% X
73. Lithium forms body centred cubic structure. The length of
the side of its unit cell is 351 pm. Atomic radius of the lithium
will be :
(a) 75 pm (b) 300 pm
(c) 240 pm (d) 152 pm
74. The packing efficiency of the two-dimensional square unit
cell shown below is : (a) MX (b) MX2
(c) M2X (d) M5X14

1. The correct statement regarding F– centre is 6. What is the number of tetrahedral voids per atom in a crystal?
(a) electron are held in the voids of crystals (a) 1 (b) 2
(b) F – centre produces colour to the crystals (c) 6 (d) 8
(c) conductivity of the crystal increases due to F – centre 7. Fe 3O 4 contains the magnetic dipoles of cations oriented as
(d) All of these
(a) ­¯­¯ (b) ­­­¯¯
2. The anions (A) form hexagonal closest packing and the
cations (C) occupy only 2/3 of octahedral holes. The simplest (c) ­­­­­ (d) ­­­¯¯¯
formula of the ionic compound is : 8. The intermetallic compound LiAg crystallizes in a cubic
(a) CA (b) C3A2 lattice in which both lithium and silver atoms have
(c) C4A3 (d) C2A3 coordination number of eight. To what crystal class does
3. Certain crystals produce electric signals on application of the unit cell belong
pressure. This phenomenon is called (a) Simple cubic (b) Face-centred cubic
(a) Pyroelectricity (b) Ferroelectricity (c) Body-centred cubic (d) None of these
(c) Piezoelectricity (d) Ferrielectricity 9. A compound is formed by elements A and B. The crystalline
r+ cubic structure has the A atoms at the corners of the cube
4. For a cubic geometry the limiting is : and B atoms at the body centre. The simplest formula of the
r–
compound is
(a) 0.414 (b) 0.155
(a) AB (b) A6B
(c) > 0.731 (d) 0.731
(c) AB6 (d) A8B4
5. Which of the following has maximum value of cation/anion
10. A compound formed by elements X and Y crystallizes in a
ratio?
cubic structure in which the X atoms are at the corners of a
(a) KCl (b) NaCl
cube and the Y atoms are at the face centres. The formula of
(c) CaF2 (d) MgCl2
the compound is
EBD_7327
364 CHEMISTRY

(a) XY3 (b) X3Y 18. In p-type semiconductor, the added impurity to silicon is ---
(c) XY (d) XY2 -and conduction of electric current is due to the movement
11. Na and Mg crystallize in bcc and fcc type crystals of -----
respectively, then the number of atoms of Na and Mg present (a) As, electrons (b) P, holes
in the unit cell of their respective crystal is (c) Ga, holes (d) Ga, electrons & holes
(a) 4 and 2 (b) 9 and 14 DIRECTIONS for Qs. 19 to 25 : These are Assertion-Reason
(c) 14 and 9 (d) 2 and 4 type questions. Each of these question contains two statements:
12. A match box exhibits Statement-1 (Assertion) and Statement-2 (Reason). Answer these
(a) Cubic geometry questions from the following four options.
(b) Monoclinic geometry (a) Statement-1 is True, Statement-2 is True; Statement-2 is a
(c) Orthorhombic geometry correct explanation of Statement-1
(d) Tetragonal geometry (b) Statement-1 is True, Statement-2 is True; Statement-2 is
13. Olive oil when allowed to cool slowly gets solidified and NOT a correct explanation of Statement-1
forms a solid over a wide range of temperature. Best (c) Statement-1 is True, Statement-2 is False
description of the solid based on above observation is (d) Statement-1 is False, Statement-2 is True
(a) Ionic solid (b) covalent network solid 19. Statement-1 : In crystal lattice, the size of the tetrahedral
(c) metallic solid (d) molecular solid hole is larger than an octahedral hole.
14. A crystalline solid Statement-2 : The cations occupy less space than anions in
(a) changes abruptly from solid to liquid when heated crystal packing.
(b) has no definite melting point 20. Statement-1 : In any ionic solid (MX) with Schottky defects,
(c) undergoes deformation of its geometry easily the number of positive and negative ions are same.
(d) has an irregular 3-dimensional arrangements Statement-2 : Equal number of cation and anion vacancies
15. Which of the following statements about amorphous solids are present.
is incorrect ? 21. Statement-1 : Space or crystal lattice differ in symmetry of
(a) They melt over a range of temperature the arrangement of points.
(b) They are anisotropic Statement-2 : nl = 2d sin q , is known as Bragg’s equation.
(c) There is no orderly arrangement of particles 22. Statement-1 : Crystalline solids have long range order.
(d) They are rigid and incompressible Statement-2 : Amorphous solids have short range order.
16. To get a n- type semiconductor, the impurity to be added to 23. Statement-1 : In close packing of spheres, a tetrahedral void
silicon should have which of the following number of valence is surrounded by four spheres whereas an octahedral void
electrons is surrounded by six spheres.
(a) 1 (b) 2 Statement-2 : A tetrahedral void has a tetrahedral shape
(c) 3 (d) 5 whereas an octahedral void has an octahedral shape.
17. Which set of following characteristics for ZnS crystal is 24. Statement-1 : Electrical conductivity of semiconductors
correct? increases with increasing temperature.
(a) Coordination number (4 : 4); ccp; Zn++ ion in the Statement-2 : With increase in temperature, large number of
alternate tetrahedral voids electrons from the valence band can jump to the conduction
(b) Coordination number (6 : 6); hcp; Zn++ ion in all band.
tetrahedral voids. 25. Statement-1 : On heating ferromagnetic or ferrimagnetic
(c) Coordination number (6 : 4); hcp; Zn++ ion in all substances, they become paramagnetic.
octahedral voids Statement-2 : The electrons change their spin on heating.
(d) Coordination number (4 : 4); ccp; Zn++ ion in all
tetrahedral voids.

Exemplar Questions 3. Which of the following is an amorphous solid?


1. Which of the following conditions favours the existence of (a) Graphite (C) (b) Quartz glass (SiO2)
(c) Chrome alum (d) Silicon carbide (SiC)
a substance in the solid state?
4. Which of the following arrangements shows schematic
(a) High temperature (b) Low temperature alignment of magnetic moments of antiferromagnetic
(c) High thermal energy (d) Weak cohesive forces substances?
2. Which of the following is not a characteristic of a crystalline
solid? (a)
(a) Definite and characteristic heat of fusion
(b)
(b) Isotropic nature
(c) A regular periodically repeated pattern of arrangement (c)
of constituent particles in the entire crystal
(d) A true solid (d)
The Solid State 365

5. Which of the following is true about the value of refractive 16. Graphite cannot be classified as ....... .
index of quartz glass? (a) conducting solid (b) network solid
(a) Same in all directions (c) covalent solid (d) ionic solid
(b) Different in different directions 17. Cations are present in the interstitial sites in ...... .
(c) Cannot be measured (a) Frenkel defect (b) Schottky defect
(d) Always zero (c) vacancy defect (d) metal deficiency defect
6. Which of the following statement is not true about 18. Schottky defect is observed in crystals when ....... .
amorphous solids? (a) some cations move from their lattice site to interstitial
(a) On heating they may become crystalline at certain sites
temperature (b) equal number of cations and anions are missing from
(b) They may become crystalline on keeping for long time the lattice
(c) Amorphous solids can be moulded by heating (c) some lattice sites are occupied by electrons
(d) They are anisotropic in nature (d) some impurity is present in the lattice
7. The sharp melting point of crystalline solids is due to 19. Which of the following is true about the charge acquired by
(a) a regular arrangement of constituent particles observed p – type semiconductors?
over a short distance in the crystal lattice (a) Positive
(b) a regular arrangement of constituent particles observed (b) Neutral
over a long distance in the crystal lattice (c) Negative
(c) same arrangement of constituent particles in different (d) Depends on concentrations of p impurity
directions 20. To get a n-type semiconductor from silicon, it should be
(d) different arrangement of constituent particles in doped with a substance with valency ............ .
different directions. (a) 2 (b) 1
8. Iodine molecules are held in the crystals lattice by......... . (c) 3 (d) 5
(a) London forces 21. The total number of tetrahedral voids in the face-centred
(b) dipole – dipole interactions unit cell is ......... .
(c) covalent bonds (a) 6 (b) 8
(d) coulombic forces (c) 10 (d) 12
9. Which of the following is a network solid? 22. Which of the following points defects are shown by AgBr(s)
(a) SO2 (solid) (b) I2 crystals?
(c) Diamond (d) H2O (ice) 1. Schottky defect
10. Which of the following solids is not an electrical conductor? 2. Frenkel defect
1. Mg(s) 2. TiO (s) 3. Metal excess defect
3. I2(s) 4. H2O(s) 4. Metal deficiency defect
(a) Only 1 (b) Only 2 (a) 1 and 2 (b) 3 and 4
(c) 3 and 4 (d) 2, 3 and 4 (c) 1 and 3 (d) 2 and 4
11. Which of the following is not the characteristic of ionic 23. In which of the following pair, most efficient packing is
solids? present?
(a) Very low value of electrical conductivity in the molten (a) hcp and bcc
state (b) hcp and ccp
(b) Brittle nature (c) bcc and ccp
(c) Very strong forces of interactions (d) bcc and simple cubic cell
(d) Anisotropic nature 24. The percentage of empty space in a body centred cubic
12. Graphite is a good conductor of electricity due to the arrangement is ............ .
presence of ........ . (a) 74 (b) 68
(a) lone pair of electrons (b) free valence electrons (c) 32 (d) 26
(c) cations (d) anions 25. Which of the following statement is not true about the
13. Which of the following oxides behaves as conductor or hexagonal close packing?
insulator depending upon temperature? (a) The coordination number is 12
(a) TiO (b) SiO2 (b) It has 74% packing efficiency
(c) TiO3 (d) MgO (c) Tetrahedral voids of the second layer are covered by
14. Which of the following oxides shows electrical properties the spheres of the third layer
like metals? (d) In this is arrangement spheres of the fourth layer are
(a) SiO2 (b) MgO exactly aligned with those of the first layer
(c) SO2(s) (d) CrO2 26. In which of the following structures coordination number
15. The lattice site in a pure crystal cannot be occupied by ..... for cations and anions in the packed structure will be same?
(a) molecule (b) ion (a) Cl– ions form fcc lattice and Na + ions occupy all
(c) electron (d) atom octahedral voids of the unit cell
EBD_7327
366 CHEMISTRY

(b) Ca2+ ions form fcc lattice and F– ions occupy all the 34. Which of the following defects is also known as dislocation
eight tetrahedral voids of the unit cell defect?
(c) O2– ions form fcc lattice and Na+ ions occupy all the (a) Frenkel defect
eight tetrahedral voids of the unit cell (b) Schottky defect
(d) S2– ions form fcc lattice and Zn 2+ ions go into alternate (c) Non – stoichiometric defect
tetrahedral voids of the unit cell (d) Simple interstitial defect
27. What is the coordination number in a square close packed 35. In the cubic close packing, the unit cell has ....... .
structure in two dimensions? (a) 4 tetrahedral voids each of which is shared by four
(a) 2 (b ) 3 adjacent unit cells
(c) 4 (d) 6 (b) 4 tetrahedral voids within the unit cell
28. Which kind of defects are introduced by doping? (c) 8 tetrahedral voids each of which is shared by four
(a) Dislocation defect (b) Schottky defect adjacent unit cells
(c) Frenkel defect (d) Electronic defect (d) 8 tetrahedral voids within the unit cells
29. Silicon doped with electron rich impurity forms..... . 36. The edge lengths of the unit cells in terms of the radius of
(a) p – type semiconductor spheres constituting fcc, bcc and simple cubic unit cells are
(b) n – type semiconductor respectively ...... .
(c) intrinsic semiconductor
(d) insulator 4r 4r
(a) 2 2r, , 2r (b) , 2 2r, 2r
30. Which of the following statements is not true? 3 3
(a) Paramagnetic substances are weakly attracted by
magnetic field. 4r 4r
(c) 2r, 2 2r, (d) 2r, , 2 2r
(b) Ferromagnetic substances cannot be magnetised 3 3
permanently. 37. Which of the following represents correct order of
(c) The domains in antiferromagnetic substances are conductivity in solids?
oppositely oriented with respect to each other. (a) kmetals >> kinsulators < ksemiconductors
(d) Pairing of electrons cancels their magnetic moment in (b) kmetals << kinsulators < ksemiconductors
the diamagnetic substances. (c) kmetals. > ksemmiconductors > kinsulators = zero
31. Which of the following is not true about the ionic solids? (d) kmetals < ksemiconductors > kinsulators ¹ zero
(a) Bigger ions form the close packed structure
(b) Smaller ions occupy either the tetrahedral or the NEET/AIPMT (2013-2017) Questions
octahedral voids depending upon their size
(c) Occupation of all the voids is not necessary 38. Which of the following statements about the interstitial
(d) The fraction of octahedral or tetrahedral voids compounds is incorrect ? [NEET 2013]
occupied depends upon the radii of the ions occupying (a) They are chemically reactive.
the voids (b) They are much harder then the pure metal.
32. A ferromagnetic substance becomes a permanent magnet (c) They have higher melting points than the pure metal.
when it is placed in a magnetic field because........ . (d) They retain metallic conductivity.
(a) all the domains get oriented in the direction of magnetic 39. The number of carbon atoms per unit cell of diamond unit cell
field is : [NEET 2013]
(b) all the domains get oriented in the direction opposite
(a) 8 (b) 6
to the direction of magnetic field
(c) domains get oriented randomly (c) 1 (d) 4
(d) domains are not affected by magnetic field 40. A metal has a fcc lattice. The edge length of the unit cell is
33. The correct order of the packing efficiency in different types 404 pm. The density of the metal is 2.72 g cm –3 . The
of unit cells is ...... . molar mass of the metal is : (NA, Avogadro’s constant
(a) fcc < bcc < simple cubic = 6.02 × 1023 mol-1) [NEET 2013]
(b) fcc > bcc > simple cubic (a) 30 g mol–1 (b) 27 g mol–1
(c) fcc < bcc > simple cubic (c) 20 g mol–1 (d) 40 g mol–1
(d) bcc < fcc > simple cubic
The Solid State 367

Hints & Solutions


EXERCISE - 1 15. (d) Since the radius ratio of CsCl is 0.93, it is expected to
have a body centred cubic structure.
1. (c) Frenkel and Schottky defects are crystal defects. It 16. (b) In NaCl crystal, each Cl– ion is surrounded by 6 Na+
arises due to dislodgement of cation or anion from their ions. Similarly, each Na+ is surrounded by 6 Cl– ions.
places in the crystal lattice.
2. (d) Ionic compounds are dissociated in solution state and
form ions. Ions are good carrier of charge which make –
solution conducting. CI
3. (d) Crystals show good cleavage because their constituent
particles are arranged in planes. Na+
4. (c) The phenomenon is called piezoelectricity.
5. (b) Out of the given substances, only Li has high electrical
and thermal conductivity as Li is a metallic solid.
6. (c) Solid CH4 is a molecular solid. In this, the constituent
molecules are held together by van der Waal’s forces.
7. (a) Si is an example of covalent crystalline solid among the 17. (d) For a simple cube, the relation can be obtained as under.
given choices. Si atoms are covalently linked in For a cube, body diagonal = 4 r
tetrahedral manner. (r = radius of sphere)
8. (b) Liquid crystals on heating first become turbid and then
clear. 4
For such a cube, we have a (edge length) = r or
9. (d) In a bcc cell eight atoms are located at the corners 3
whereas one lies at the centre.
3
æ 1ö r= a
\ n = ç8 ´ ÷ +1 = 2 4
è 8ø
Thus, body diagonal
whereas in an fcc cell eight atoms are at the corners and
one atom is located at each of the six faces which is shared 3
by two unit of cell. = 4r = 4 ´ ´a
4
1 æ 1ö
\ n = 8´ + ç 6 ´ ÷ = 4 = 3 a = 3 ´ 400 pm
8 è 2ø
10. (c) AB is just like NaCl. Thus twelve A+ are at edges and 1 = 1.73 ×400 pm ; 693 pm.
within body of fcc i.e. in octahedral voids and six B– at 18. (a) In stoichiometric Frenkel defects occurs in those
faces and 8 at corner. compound which have
r 146 (i) Low C.N.
11. (b) The radius ratio + = = 0.675 (ii) Large difference in size of cations and anions
r- 216
19. (b) For orthorhombic system a = b = g = 90 °
It lies between 0.414 – 0.732. Hence, it exhibits rock salt 20. (c) In fluorite structure each F– ion is surrounded by four
type structure. Ca++ ions whereas each Ca++ is surrounded by eight
12. (d) Semiconductors are those solids which are good F– ions, giving a body centred cubic arrangement. Thus
conductors of electricity at room temperature but the co-ordination number of Ca++ and F– are 8 and 4
behave as insulators at room temperature, e.g., silicon respectively.
and germanium. 21. (d) The bcc cell consists of 8 atoms at the corners and one
1 atom at centre. Contribution of each atom at each corner
13. (d) In a unit cell, W atoms at the corner = ´ 8 = 1
8 1
is equal to .
1 8
O-atoms at the centre of edges = ´ 12 = 3
4 æ 1 ö
Na-atoms at the centre of the cube = 1 \ n = ç8 ´ ÷ + 1 = 2
è 8ø
W : O : Na = 1 : 3 : 1
Hence, formula = NaWO3 The fcc cell consists of 8 atoms at the eight corners and
14. (c) Let the number of Z atoms in the ccp arrangement one atom at each of the six faces. This atom at the face
= 100 is shared by two unit cells.
Thus the number of tetrahedral sites = 200 1 æ 1ö
Since all the tetrahedral sites are occupied by X atoms, \n = 8´ + ç6´ ÷ = 4
8 è 2ø
the number of X atoms = 200
22. (a) Since MgO has a rock salt structure. In this structure
Hence ratio of X : Z = 2 : 1
each cation is surrounded by six anions and vice versa.
Thus the formula is X2Z
EBD_7327
368 CHEMISTRY

23. (c) The vacancy created due to missing of equal no. of 32. (b) In CaF2, Ca2+ ions have ccp arrangement and F– ions
atoms or ions form normal lattice point is called Schottky occupy the tetrahedral voids.
defect. In this type of defect electrical neutrality of ionic In ccp arrangement total no. of Ca2+ ions is 4 and
crystal is maintained. no. of tetrahedral voids is always 2n i.e. 8. Hence for
24. (b) hcp is a closed packed arrangement in which the unit every Ca2+ there are 8F– ions as C.N, for F– there are 4
cell is hexagonal and coordination number is 12. Ca2+ions.
25. (c) ABAB...... is hexagonal close packing. 33. (b) For tetrahedral shape radius ratio is 0.225 – 0.414.
34. (d) Order of Bragg diffraction (n) = 2; Wavelength (l) = 1Å
3
26. (a) For bcc structure, atomic radius, r = a and angle (q) = 60º.
4 We know from the Bragg’s equation
3 nl = 2d sin q
= × 4.3 = 1.86 or 2 × 1 = 2d sin 60º
4
Since, r = half the distance between two nearest 3 2
neighbouring atoms. Þ 2 ´1 = 2.d . Þd= = 1.15Å
2 3
Shortest interionic distance = 2 × 1.86 = 3.72 (where d = Distance between the scattering planes)
27. (d) Schottky defect occurs in ionic crystals of type A+ B– 35. (c) For cubic geometry the limiting ratio is
, when equal number of cations and anions are missing
from their lattice sites so that the electrical neutrality is r+
maintained. This defect generally appears in highly > 0.731 i.e. (c)
ionic compounds which have high coordination number. r-
NaCl, KCl and CsCl all have high coordination numbers 36. (b) For body centred cubic lattice Z = 2
i.e., 6, 6 and 8 respectively. So, Schottky defect appear Atomic mass of unit cell = 133 + 80 = 213 a.m.u
in all of the given compounds. Volume of cell = (436.6 × 10–10)3 cm3
28. (d) Since in NaCl type of structure 4 formula units form a ZM 2 ´ 213
cell. Density, r = 3 = -
a NA (436.6 ´ 10 10 )3 ´ 6.02 ´ 1023
58.5 gm. of NaCl = 6.023 × 1023 atoms = 8.50 g/cm3

6.023 ´ 1023 37. (a) Fraction of unoccupied sites in NaCl crystal


1 gm of NaCl = atoms
58.5 2.165 ´ 103
=1-
4 atoms constitute 1 unit cell 2.178 ´ 103
6.023 ´ 1023 2.178´103 , 2.165´103 0.13´103
\ atoms constitute < <
58.5 2.178´103 2.178

6.023 ´ 1023 130


= < = 5.96 × 10–3
58.5 ´ 4 2178
= 2.57 × 1021 unit cells. 1
38. (b) Distance between K+ and F– = ´ length of the edge
2
æ
1ö æ 1ö (b) Bragg’s equation is nl = 2d sin q
29. (d) Effective number of ‘A’ atoms = ç 8 ´ ÷ + ç 4 ´ ÷=3 39.
8ø è
è 2ø where n is a positive integer i.e., 1, 2, 3, 4 etc. which
æ 1ö stands for serial order of diffracted beams
Effective number of ‘B’ atoms = ç12 ´ ÷ + 1 = 4 ZM
è 4ø 40. (a) r =
\ Formula of the solid = A3B4. NA V
30. (b) Solid AB crystallizes as NaCl structure, so it has rNA V 2 ´ 6 ´ 1023 ´ (5 ´10-8 )3
coordination number 6 and its r +/r – ranges from Z= =
0.414 – 0.732. M 75
Z = 2, which represents bcc structure
For maximum radius of anion, we have to take the lower
3 3
r+ \ r= a= ´ 5 = 2.165Å = 216.5 pm
limit of the range 0.414– 0.732. So, = 0.414 4 4
r- » 217 pm
0.100 41. (a) There are two atoms in a bcc unit cell.
Þ r–= nm = 0.241 nm So, number of atoms in 12.08 × 1023 unit cells
0.414 = 2 × 12.08 × 1023 = 24.16 × 1023 atoms.
31. (d) Body diagonal (d) of a cubic crystal of edge length (a)
is given by, r ´ a3 ´ N A ´ 10-30
42. (d) M=
d= a 3 Z
putting a = 400 pm, we get
10 ´ (100)3 ´ 6.02 ´ 1023 ´ 10 -30
d= 3 ´ 400 pm = 692.8 pm » 693 pm. = = 15.05
4
The Solid State 369

53. (d) The appearance of colour in solid alkali metal halide is


6.02 ´1023 due to presence of F-centre found as defect in the
\ Number of atoms in 100 g = ´100
15.05 crystal structure.
= 4 × 1025 54. (b) a = b ¹ c , a = b = 90°, g = 120°
43. (d) For bcc, d = 3 a or a = 2d = 2 ´ 4.52 = 5.219Å These are the characteristics of a hexagonal system.
2 3 1.732 55. (b) When equal number of cations (Na+) and anions (Cl–) are
= 522 pm missing from their regular lattice positions, we have
schottky defect.
z´ M
r= 56. (d) Schottky defect occurs in ionic crystals of type A+ B,
a 3 ´ N A ´10 -30 when equal number of cations and anions are missing
from their lattice sites so that the electrical neutrality is
2 ´ 39
= maintained. This defect generally appears in highly
(522) ´ (6.023 ´1023 ) ´ 10 -30
3
ionic compounds which have high coordination number.
NaCl, KCl and CsCl all have high coordination numbers
= 0.91g / cm3 = 910 kg m -3 i.e., 6, 6 and 8 respectively. So, Schottky defect appear
44. (c) In a fcc lattice, the distance between the cation and in all of the given compounds.
anion is equal to the sum of their radii, which is equal to 57. (b) More is the Schottky defect in crystal more is the
half of the edge length of unit cell, decrease in density.
a 58. (b) In Na2O there is antifluorite structure. Here negative
i.e. r + + r - = (where a = edge length)
2 ions form the ccp arrangement so that each positive
r+ = 95 pm, r– = 181 pm ion is surrounded by 4 negative ions and each negative
Edge length = 2r+ + 2r– = (2 × 95 + 2 × 181) pm ion is surrounded by 8 positive ions. So coordination
= (190 + 362) pm = 552 pm. no. of Na+ is 4.
59. (b) Since each Sr++ ion provides one cation vacancy, hence
ZM
45. (b) r = Concentration of cation vacancies = mole % of SrCl 2
NA V added
= 10–4 mole %
rN A V 8.92 ´ 6.02 ´ 1023 ´ (362)3 ´10-30
Z= = 10-4
M 63.55 = ´ 6.023 ´ 1023 = 6.023 ´ 1017 mol–1
=4 100
\ It has fcc unit cell 60. (d) Number of atoms per unit cell = 1
46. (a) Wurtzite has face centred cubic structure in which each a
Atoms touch each other along edges. Hence r =
Zn2+ ion is attached to four S2– ions and each S2– ion 2
remains in contact with four Zn 2+ ions. Hence ( r = radius of atom and a = edge length)
coordination number of each ion is 4. 4 3
pr
47. (d) In KCl, co-ordination number of cation and anion is 6 3 p
Therefore % fraction = 3
=
and 6 respectively. KCl is highly ionic so Schottky (2 r ) 6
defect is common. 61. (a) Following generalization can be easily derived for
Note : Schottky defect is common in compounds having various types of lattice arrangements in cubic cells
high coordination nummber while Frenkel defect is between the edge length (a) of the cell and r the radius
common in compounds with low coordination number. of the sphere.
48. (b) a
For simple cubic : a = 2r or r =
49. (a) Schottky defects are found in solid. 2
50. (a) p-type of semiconductors are produced For body centred cubic :
(i) due to metal deficiency defects 4 3
(ii) by adding impurity containing less electrons (i.e., a= r or r = a
3 4
atoms of group 13) For face centred cubic :
Ge belongs to Group 14 and In to Group 13. Hence on 1
doping, p-type semicondutor is obtained. a = 2 2r or r = a
51. (b) In cubic closed pack system (CCP system) .......ABC 2 2
Thus the ratio of radii of spheres for these will be
ABC....... type of arrangement of layers is found. In this simple : bcc : fcc
system, there are atoms at the corners as well as centre 1
of the unit cell. a 3
= : a: a
1 2 4 2 2
\ No. of atoms per unit cell = 8 ´ + 1 = 2 i.e. option (a) is correct answer.
8 62. (d) The semiconductors formed by the introduction of
Hence, the no. of tetrahedral voids in a unit cell = 2Z impurity atoms containing one elecron less than the
52. (d) If in an ionic crystal of the type A+, B–, equal number of parent atoms of insulators are termed as p-type
cations and anions are missing from their lattice sites semiconductors. Therefore silicon containing 14
so that the electrical neutrality is maintained. The defect electrons is to be doped with boron containing 13
is called Schottky defect. electrons to give a p-type semi-conductor.
EBD_7327
370 CHEMISTRY

63. (a) For bcc lattice body diagonal = a 3 . 73. (d) For bcc structure 3 a = 4r
The distance between the two oppositely charged ions 3 3
a r= a= ´ 351 = 152 pm.
= 3 4 4
2
74. (d) Packing efficiency
387 ´ 1.732
= = 335pm Area occupied by circles within the square
2 =
M+ Area of square
64. (c) is highest in CsF 2pr 2 2pr 2 p
X- = ´ 100 = ´ 100 = ´100 = 78.54%
\ correct choice : (c) L2 2( 2 r ) 2 4
65. (c) Radius ratio of NaCl like crystal 1
75. (b) No. of M atoms = ´ 4 + 1 = 1 + 1 = 2
r+ 4
= - = 0.414
r 1 1
No. of X atoms = × 6 + × 8 = 3 + 1 = 4
- 100 2 8
r = = 241.5 pm
0.414 So, formula = M2X4 = MX2
66. (a) For ccp 2 a = 4r EXERCISE - 2
2 ´ 408 1. (d) All the given statements are correct about F-centres.
= 2r (2r = Diameter)
2 1
2. (d) The number of A ions per unit cell = ´ 12 (corners)
Diameter = 288.5 6
67. (a) From the given data, we have 1
Number of Y atoms in a unit cell = 4 + ´ 2 (end centres) + 3 (in the body)
2
2 16 Number of octahedral voids per unit cell = 1× 6 = 6.
Number of X atoms in a unit cell = 8 ´ =
3 3 2
From the above we get the formula of the compound as Number of cations (C) per unit cell = ´6 = 4
3
X16 / 3Y4 or X 4 Y3
Formula : C4 A6 or C2 A3
68. (a) For fcc unit cell, 4r = 2a 3. (c) When polar crystal is subjected to a mechanical stress,
2 ´ 361 electricity is produced – a case of piezoelectricity.
r= = 127 pm Reversely, if electric field is applied, mechanical stress
4
is developed. Piezoelectric crystal acts as a mechanical
69. (d) For an fcc crystal
electrical transductor.
edge length 4. (c) For cubic geometry the limiting ratio is
rcation + ranion =
2
r+
508 > 0.731 i.e., (c)
110 + ranion = r-
2
ranion = 254 – 110 = 144 pm 5. (c) Among the three options KCl, NaCl and MgCl2 , the
70. (a) size of anion is same. So larger the cation, larger will be
r + r + the cation/anion ratio i.e., KCl will have larger cation/
71. (a) Na
= 0.55 and K = 0.74
r - r - anion ratio among the three. So, we left with two options
Cl Cl
r + r + KCl and CaF2 . Among these two CaF2 will have
Na
+ 1 = 0.55 + 1 and K + 1 = 0.74 + 1 maximum value of cation/anion ratio because decrease
r - r -
Cl Cl in ionic radii of anion from Cl– to F– does not overcome
r +r r +r the effect of decrease in ionic radii of cation from K + to
Na + Cl- = 1.55 and K+ Cl - = 1.74
r - r -
Ca2+.
Cl Cl 6. (b) Number of tetrahedral voids (VT) in a crystal is twice
Now edge length ratio of KCl and NaCl is the number of atoms (n) in a crystal i.e.,
1.74 rK + + rCl- r -
Cl VT = 2n
= ´ = 1.123.
1.55 r - r + +r - So, number of tetrahedral voids per atom is given by
Cl Na Cl
72. (b) Packing fraction is defined as the ratio of the volume VT / n = 2
of the unit cell that is occupied by the spheres to the 7. (b)
total volume of the unit cell. 8. (c) A body-centred cubic system consists of all eight
P.F. for cpp and bcc are 0.74 and 0.68 respectively. corners plus one atom at the centre of cube.
So, the free space in ccp and bcc are 26% & 32% 9. (a) Atoms are present in the corners of cube = A and atom
respectively. present at body centre = B.
We know that a cubic unit cell has 8 corners. Therefore
The Solid State 371

1 by four spheres tetrahedrally while octahedral void is


contribution of each atom A at the corner = . Since so called because it is surrounded by six spheres
8
octahedrally.
number of atoms per unit cell is 8, therefore total 24. (a) In case of semiconductors, the gap between valence
1 band and the conduction band is small and therefore
contribution = 8 ´ = 1 . We also know that atoms in
8 some of the electrons may jump from valence band to
the body centre, therefore number of atoms of B per conduction band and thus on increasing temperature
unit cell = 1. Thus formula of the compound is AB. conductivity is also increased.
10. (a) For a face centred cubic structure, 25. (a) All magnetically ordered solids (ferromagnetic,
ferrimagnetic and antiferromagnetic solids) transform
No. of X atoms = 8 ´ 1 = 1 to the paramagnetic state at high temperature due to
8 the randomisation of spins.
1
No. of Y atoms = 6 ´ = 3
2 EXERCISE - 3
\ Formula of the compound = XY3
11. (d) In bcc - 8 atoms are at corners and one in the centre of Exemplar Questions
the unit cell. 1. (b) At low temperature existence of a substance in the
1 solid state is due to slow molecular motion and strong
Number of atoms per unit cell, n = 8 ´ + 1 = 2 . cohesive forces.
8
2. (b) Crystalline solid is anisotropic in nature as it shows
In fcc - 8 atoms are at the corners and also centre of the
different physical properties such as electrical
six faces of each cell.
resistance, refractive index in different directions.
1 1 3. (b) Quartz glass (SiO2) is an amorphous solid due to its
Number of atoms per unit cell , n = 8 ´ + 6 ´ = 4 .
8 2 short range order of constituent particles.
12. (c) Match box exhibits orthorhombic geometry i.e., 4. (d) Substances which have domain structure are
aa ¹¹ bb ¹¹ cc;; aa == bb == gg == 90
90°° oppositely oriented and cancel each other's magnetic
field are known as antiferromagnetic substances.
13. (d) It can best be described as molecular solid.
14. (a) In crystalline solid there is perfect arrangement of the
constituent particles only at 0 K. As the temperature
increases the chance that a lattice site may be Schematic arrangement of
unoccupied by an ion increases. As the number of antiferromagnetic substance
defects increases with temperature solid changes into 5. (a) Since, quartz glass is an amorphous solid having short
liquid. range order of constituents. Hence, value of refractive
15. (b) Amorphous solids are isotropic, because these index is same in all directions, measurable and not to
substances show same properties in all directions. be equal to zero always.
16. (d) For n-type, impurity added to silicon should have more 6. (d) Amorphous solids are isotropic in nature and any of
than 4 valence electrons. its physical property will be same in all direction.
17. (a) ZnS has cubic close packed (ccp) structure. The S2– 7. (b) Crystalline solids has a regular arrangement of
ions are present at the corners of the cube and at the constituent particles observed over a long distance in
centre of each face. Zinc ions occupy half of the the crystal lattice due to which they exhibit sharp
tetrahedral sites. Each zinc ion is surrounded by four melting point.
sulphide ions which are disposed towards the corners 8. (a) Iodine molecules belongs to a class of non – polar
of a regular tetrahedron. Similarly, S2– ion is surrounded molecular solids in which constituents molecule are
by four Zn 2+ ions. held together by London or dispersion forces.
18. (c) 9. (c) Diamond is a giant molecule in which constituent atoms
19. (d) Tetrahedral holes are smaller in size than octahedral are held together by covalent bonds.
holes. Cations usually occupy less space than anions. 10. (c) Iodine is a non–polar molecular solid and a non–
20. (a) Schottky defect is due to missing of equal number of
conductor of electricity. Water is a hydrogen bonded
cations and anions.
21. (b) Space or crystal lattice is a regular repeating molecular solid in which H and O are held together by
arrangement of points in space and forms the basis of polar covalent bond and each water molecule is held
classification of all structures. together by hydrogen bonding. Also, due to its non-
22. (b) In crystalline solids constituents are arranged in definite ionic nature, it also behaves as a non-conductor of
orderly arrangement. This regular arrangement of electricity.
constituents extends throughout the three dimensional 11. (a) Ionic solids get easily dissociated into its ions in molten
network of crystal. Thus crystalline substances said to state and show high electrical conductivity.
have long range order. Whereas amorphous solids have 12. (b) In graphite, each carbon is sp2 hybridised having one
no regular arrangement. free electron and due to the presence of free electrons
23. (c) Tetrahedral void is so called because it is surrounded graphite behaves as a good conductor of electricity.
EBD_7327
372 CHEMISTRY

13. (c) Certain metal oxides like VO2, VO3 and TiO3 etc. show 24. (c) Packing efficiency for bcc arrangement is 68% which
metallic or insulating property depending upon represents total filled space in the unit cell. Hence
temperature which is due to variation in energy gap empty space in a bcc arrangement will be 100 – 68
between conduction band and valence band. = 32%.
14. (d) Metal oxides like CrO2, TiO and ReO3 show electrical 25. (d) Hexagonal close packing can be diagramatically
conductivity similar to metals while SiO2, MgO and represented as :
SO2 do not show electrical properties.
15. (c) Each lattice point in a pure crystal is either an atom or B
molecule or ion which are joined together by a straight A
line to bring out geometry of lattice in pure crystal B
constituents which are arranged in fixed stoichiometric A
ratio.
We can see that lst layer (A) and 4th layer (B) are not
16. (d) Graphite is a covalent solid which is made up of carbon
exactly aligned.
atoms covalently bonded to three carbon atoms. As,
the formation of covalent bond occurs throughout the 26. (a) In NaCl crystal Cl– ions are present at fcc lattice points
crystal hence, it is a type of network solid. Also due to and face centre and Na+ occupies all the octahedral
the presence of free electrons graphite is a conducting voids of the unit cell.
solid. Thus, coordination number of Na+ and Cl– is 6.
17. (a) When smaller ion (usually a cation) is dislocated from 27. (c) Coordination number in a square closed packed
its normal site in crystal and move to interstitial site, it structure in 2-D must be equal to 4 as shown :
is known as Frenkel defect.
18. (b) Schottky defect is observed in crystal when equal A
A
number of cations and anions are missing from the A
lattice. A
When lattice site is occupied by electron, this type of
defect is known as metal excess defect. SquareCubic
closedclosed
packedstructure
structure
19. (b) For the formation of p-type semiconductors, group 13 28. (d) When electron rich or electron deficient impurity is
elements are doped with group 14 elements, which added to a perfect crystal it introduces electronic defect
creates a hole in a molecule but the molecule as a whole in it.
remain neutral. 29. (b) Sillicon doped with electron rich impurity to form a
20. (d) To get a n – type semiconductor from silicon, it should n – type semiconductor.
be doped with a substance with valency 5. 30. (b) Ferromagnetic substances can be permanently
e.g., silicon is doped with phosphorus to form a magnetised by applying magnetic field and they will
n – type semiconductor. exhibit the magnetic properties even after the removal
21. (b) In a fcc unit cell 8 tetrahedral voids are present at the of magnetic field.
centre of each 8 smaller cube of the unit cell. 31. (d) The fraction of octahedral or tetrahedral voids

.. .. .. . ..
Eight tetrahedral voids occupied depends upon the radii of the ions present at
per fcc unit cell the lattice points. As we know the radii of octahedral
1
or tetrahedral void is related to radii of atoms (r) as
2
3 Radius of octahedral void (Ro) = 0.414 r

. .. . .
4
Radius of tetrahedral void (Rt) = 0.225 r

.
Where, r = radius of bigger atom involved
Tetrahedral 32. (a) When a ferromagnetic substance is placed in a magnetic

.. .
void field it becomes a permanent magnet because all the
domains get oriented in the direction of magnetic field
Each cube contain one tetrahedral even after the removal of applied magnetic field.
5 6
7 void at its body centre 33. (b) Packing efficiency in different types of unit cells is as
8

Number of octahedral voids present = 4 follows :


Number of tetrahedral voids present = 8
22. (a) In AgBr, both Ag+ and Br– ions are absent from the Unit cell Packing efficiency
lattice causing Schottky defect. Also, Ag+ ions are fcc 74%
mobile so they have a tendency to move from their bcc 68%
lattice site to interstitial site which causes Frenkel Simple cubic 52%
defect.
23. (b) As, packing efficiency for hcp or ccp is calculated to Thus, the correct order of packing efficiency is: fcc
be 74%, it is maximum among all type of crystals. (74%) > bcc (68%) > simple cubic (52%).
The Solid State 373

34. (a) Frenkel defect is also known as dislocation defect NEET/AIPMT (2013-2017) Questions
because in this defect atoms present in crystal lattice
are dislocated to interstitial site. 38. (a) In interstitial compounds small atoms like H, B & C
35. (d) In the cubic close packing the unit cell has 8 tetrahedral enter into the void sites between the packed atoms of
voids are located at each eight smaller cube of an unit crystalline metal. They retain metallic conductivity and
cell. are chemically inert.
39. (a) Diamond is like ZnS. In diamond cubic unit cell, there
36. (a) Edge lengths for different types of unit cells are given
are eight corner atoms, six face centered atoms and four
as :
more atoms inside the structure (tetrahedral voids).
Total no. of atoms present per unit cell
Types of unit cell Edge length
1 1
fcc 2 2r = 8´ ∗ 6 ´ ∗ 4 < 8
8 2
4
bcc r (corners) (face (inside
3 centered) body)
Simple cubic 2r 40. (b) Density is given by
Z´M
37. (a) Conductivity of metal, insulator and semiconductors d= ; where Z = number of formula units present
can be represented in the terms of k (Kappa) which NA a 3
depends upon energy gap between valence band and in unit cell, which is 4 for fcc
conduction band. a = edge length of unit cell. M = Molecular mass
Conduction band 4´M
2.72 =
( )
Empty 3
band
Empty 6.02 ´1023 ´ 404 ´ 10-10
band
Forbidden zone
(Large energy gap) Small energy gap
(Q 1pm = 10 -10 cm)
Energy

Filled
Overlapping band 2.72 ´ 6.02 ´ (404) 3
M= = 26.99
band
Metal
bands
4 ´107
Insulator Semiconductor
Value of k 10 4 – 107 10 – 20 – 10
–10 10– 6 to 10 4
= 27 g mol–1
(in W – 1 m– 1)
Hence, correct order of conductivity in solids is :
kmetals >> kinsulators < ksemiconductors
EBD_7327
374 CHEMISTRY

16 Solutions

(iii) Weight – Volume Percentage (w/V)


COMPONENTS OF SOLUTION
A solution is a homogenous mixture of two or more components Weight of solute (gm)
called solute and solvent. Weight – volume percentage= × 100
Volumeof solution (ml)
(i) Solute : The component present in lesser amount than other (iv) Parts per Million
component in solution is called solute.
It is defined as:
(ii) Solvent : The component present in greater amount than any
or all other components is called the solvent. Number of parts of
A solution consisting of two components, i.e., one solute component
and one solvent is called a binary solution. Parts per = ´ 106
million Total number of parts of
TYPES OF SOLUTIONS all components of the
On the basis of physical state of solute and solvent : There may be solution
the following nine types of binary solutions :
(v) Normality
S.No. Solute Solvent Example
1. Gas Gas Air Number of gram equivalents of solute
Normality =
2. Gas Liquid CO2 in soda water Volumeof solution (lit)
3. Gas Solid Hydrogen in Pd/Pt.
Weight of solute (gm)
4. Liquid Gas Fog As Gram equivalents of solute =
5. Liquid Liquid Alcohol in water, Equivalent weight of solute
benzene in toluene (vi) Mole fraction
6. Liquid Solid Mercury in zinc It is defined as:
amalgam
7. Solid Gas Smog Number of moles of
8. Solid Liquid Sugar in water the component
Mole fraction of a =
9. Solid Solid Alloys component Total number of moles of
CONCENTRATION OF SOLUTIONS all the components
Composition of a solution can be described by expressing its (vii) Molarity (M)
concentration. There are several ways of expressing the It is defined as:
concentration of the solution quantitatively. Number of moles of solute
(i) Mass Percentage (w/W) Molarity =
Volume of solution ( in L )
It is given by
(viii) Molality (m)
Mass of the component It is defined as:
in the solution Moles of solute
mass% of a = ´ 100 Molality (m) =
component Total mass of the solution mass of solvent in kg
(ii) Volume Percentage (v/V) (ix) Formality
It is given by weight of solute (gm)
Formality =
Formula weight of solute
Volume of the component 1
volume% of a = ´ 100
Total volume of solution ×
component Volume of solution (lit.)
Solutions 375

Note: Effect of pressure: The solubility of a gas increases with increase


(i) Mass %, ppm, mole fraction and molality are independent of in pressure. Henry’s law states that at a constant temperature,
temperature, whereas molarity is a function of temperature. It solubility of a gas in a liquid is directly proportional to the pressure
is because volume depends on temperature and mass does of the gas, i.e. Expressing solubility in terms of mole fraction of gas
not. in the solution Henry’s law can be written as,
(ii) The molarity equation is M1V1 = M2V2 p = KHx
(for dilution from volume V1 to V2) p ® partial pressure of the gas in vapour phase
For a reaction between two reactants, x ® mole fraction of the gas in solution
KH ® Henry’s law constant
M1V1 M 2V2 • Henry’s constant, KH is a junction of the nature of the gas.
=
n1 n2 • Greater the value of KH, lower is the solubility of the gas at
where n1 and n 2 are their stoichiometric coefficients in the the same partial pressure.
balanced equation. • The value of KH increases with increase in temperature.
SOLUBILITY 1 1
KH µ and K H µ T \ x µ
Solubility of a substance, is its maximum amount that can be x T
dissolved in a specified amount of solvent. It depends on nature It is due to this reason that aquatic species are more
of solute and solvent, temperature and pressure. comfortable in cold water than in hot water.
Solubility of a Solid in a Liquid Applications of Henry’s Law
(i) To increase the solubility of CO2 in soft drinks and soda
When a solid solute is added to the solvent, some solute dissolves
water, the bottle is sealed under high pressure.
(dissolution) while some solute particles get separated
(ii) To avoid bends as well as toxic effects of high concentrations
(crystallization). A stage is reached when the two processes occur
of nitrogen in the blood, the tanks used by scuba divers are
at the same rate and a state of dynamic equilibrium is reached. The filled with air diluted with helium, nitrogen and oxygen.
solution which is in dynamic equilibrium with undissolved solute (iii) At high altitudes, the partial pressure of oxygen is less which
is the saturated solution and contains the maximum amount of leads to low concentration of oxygen in blood, causing
solute dissolved in a given amount of solvent. The concentration climbers to become weak and unable to think clearly,
of solute in such a solution is its solubility. symptoms of a condition known as anoxia.
Thus, the solubility of a solid in a liquid at any temperature is
defined as the maximum amount of the solid in grams which can be VAPOUR PRESSURE OF LIQUID SOLUTIONS AND
dissolved in 100g of the liquid to form the saturated solution. RAOULT’S LAW
Effect of temperature: If dissolution process is endothermic (DH Vapour Pressure of Liquid-liquid Solutions
> 0), solubility increases with increase in temperature and if it is When two volatile liquids are taken in a closed vessel, a stage is
exothermic (DH < 0), solubility decreases. reached when an equilibrium is established between vapour phase
Nature of the solute and solvent: Ionic compound dissolves in and liquid phase. The relationship between them is given by
polar solvents due to strong electrostatic forces of attraction Raoult’s law i.e.,
between ions and polar solvent molecules. Non-polar compound It states that at a given temperature, the partial vapour pressure of
dissolve in non polar solvents due to similar solute-solute, solute- any component of the solution is equal to the product of the
solvent and solvent-solvent interactions. vapour pressure of the pure component and its mole fraction in
the solution.
Effect of pressure: Solids and liquids are incompressible and
practically remain unaffected by changes in pressure. p1 µ x1 and p1 = p1°x1
Solubility of a Gas in a Liquid p2 µ x2 and p2 = p2° x2
It is generally expressed in terms of absorption coefficient which where p1 and p2 are partial pressures of components 1 and 2
is defined as the volume of gas (reduced to NTP) dissolved by p1° and p2° are their partial pressures in pure state
unit volume of solvent at particular temperature and 1 atm. pressure x1 and x2 are mole fractions of component 1 and 2.
of gas. According to Dalton’s law of partial pressure,
Nature of the gas: Easily liquifiable gases are more soluble. Example: pTotal = p1 + p2 = x1p1° + x2p2°
CO2 is more soluble than O2 and H2. = (1 – x2)p1° + x2p2°
= p1° + (p2° – p1°) x2
Nature of the solvent: Gases capable of forming ions in aqueous
Diagrammatically, it is represented as:
solution are more soluble in water than in other solvent.
Example : HCl, NH3 etc. p2 p2°
= p1+
Vapour pressure

Effect of temperature: Solubility of a gas in a liquid decreases p T


2
p

with increase in temperature. It is an exothermic process p1°


i.e., Gas + Solvent Solution + Heat
p1
Applying Le Chatelier, principle, increase in temperature would
shift the equilibrium in backward direction, i.e, solubility will x1 = 1 Mole x1 = 0
decrease. x2 = 0 fraction x2 = 1
EBD_7327
376 CHEMISTRY

The composition of the vapour phase is given by: Examples:


p1 = y1pTotal (i) Benzene and toluene
p2 = y2pTotal (ii) n-Hexane and n-Heptane
where y1 and y2 are the mole fractions of components 1 and (iii) Ethyl iodide and ethyl bromide
2 in the vapour phase. (iv) Chlorobenzene and bromobenzene
Vapour Pressure of Solutions of Solids in Liquids Non Ideal: Solution possesses the following characteristics:
If a non-volatile solute is added to a solvent to give a solution, the (i) The interactions between components are different from
vapour pressure of the solution will be from solvent alone and it is those in pure components.
lower than the vapour pressure of the pure solvent at the same ;
(ii) DV ¹ 0, DH ¹ 0 on mixing the components.
temperature. (iii) They do not obey Raoult’s law. They show positive or
Then, the total vapour pressure will be given by negative deviations from Raoult’s law, i.e., PA ¹ xAPA° and
pT = p1 + p2
PB ¹ xBPB°
as p2 = 0 (non-volatile solute)
(iv) They form azeotropes
\ pT = p1 = p1°x1
i.e., vapour pressure of solution µ mole fraction of the solvent in Azeotropic Mixtures
solution. They are binary mixtures of two liquids having same composition
Mole fraction of the solvent in solution, in liquid and vapour phase and boil at a constant temperature and
n1 can be distilled without any change in their composition. They are
x1 = n + n
1 2 formed by non–ideal solutions. They are of two type.
where n1 = moles of solvent and n 2 = moles of solute Minimum boiling azeotropes : it is formed by liquids showing
pT = p1°x1 positive deviation. Such azeotropes have boilig point lower than
pºT n1 either any of the pure component.
i.e., pº = n + n Maximum boiling azeotropes : It is formed by liquids showing
1 1 2
negative deviation such azeotropes have boiling point higher than
pº1 –pT n2
= either of the pure component.
pº1 n1 + n2 Non-Ideal Solutions are divided into two types:
pº1 –pT Non-ideal solutions showing positive deviations
where pº1 is called relative lowering of vapour pressure and 1. The interactions between components are less than in pure
components, i.e., A – B < A – A/B – B
n2 2. DVmixing = +ve °x
n1 + n 2 = mole fraction of the solute in the solution. ° x + pB B PB°
Vapour Pressure
3. DHmixing = – ve P = p A A lution
l S o
Limitations of Raoult’s Law 4. PA > xAPA°; PB > xBPB° I dea

(i) Raoult’s law is applicable only to very dilute solutions. Examples: PA°
(i) Acetone + ethyl alcohol
(ii) Raoult’s law is applicable to solutions containing non-volatile
solute only. (ii) Water + Ethylalcohol
(iii) Acetone + Benzene xA = 1 Mole Fraction xB = 1
(iii) Raoult’s law is not applicable to solutes which dissociates or xA = 0
xB = 0
associate in solution.
Non-ideal solutions showing negative deviations
IDEAL AND NON-IDEAL SOLUTIONS
1. The interactions between components are more than in pure
An Ideal solution possesses the following characteristics.
components, i.e., A – B > A – A/B – B
(i) The interactions between components are similar to those in
pure components. 2. DVmixing = – ve
(ii) There is no volume change and enthalpy change on mixing 3. DHmixing = – ve + p B°
xB PB°
Vapour Pressure

° xA
the components. (DV = 0, DH = 0) 4. PA < xAPA°, PB < xBPB° P = pA n
utio
(iii) Each component obeys Raoult’s law at all temperatures and Examples: dea l sol
PA° I
concentrations i.e, PA = xAPA° and PB = xBPB° (i) H2O + HCl
(iv) They do not form azeotropes (ii) CH3OH + CH3COOH
Their graphical behaviour is shown as:
(iii) H2O + HNO3
pB° (iv) Chloroform + Benzene
+ pB xA = 1 x =1
Vapour pressure

p Mole Fraction xB = 0
pT= p
A
xB = 0 A
B
pA° COLLIGATIVE PROPERTIES
pA Those properties of ideal solutions which depend only on the
number of particles of solute dissolved in a definite amount of the
xA = 1 Mole xA = 0 solvent and do not depend on the nature of the solute are called
xB = 0 fraction xB = 1
colligative properties.
Solutions 377

The important colligative properties are : (iii) Depression in Freezing Point


(i) Relative Lowering of Vapour Pressure Vapour pressure of the solution is less than that of pure
It is given by: solvent. As freezing point is the temperature at which the
p° - ps n2 w 2 / M2 vapour pressure of the liquid and solid phase are equal,
= = therefore, for the solution, this will occur at a lower temperature
p° n1 + n 2 w1 / M1 + w 2 / M 2 (Q lower the temperature lower is the vapour pressure)
p° - ps n 2 w 2 / M 2 Graphically, it is represented as:
for very dilute solution, ; =
p° n1 w1 / M1
p° - ps n 2 w 2 / M2 t
for all solutions, = = v en
ps n1 w1 / M1 id sol
u
(dilute or concentrated) Liq n
t utio
Where n1, w1, M1 ® number of moles, mass in g, mol. mass en Sol
solv
of solvent.

Vapour pressure
lid
n2, w2, M2 ® number of moles, mass in g, mol. mass of So
solute.
(ii) Elevation of Boiling Point
The boiling point of the solution is always higher than that DTf
of the pure solvent. This is because vapour pressure of
solution is lower than that of pure solvent and hence, the T f°
Tf
solution has to be heated more to make the vapour pressure
equal to the atmospheric pressure. Temperature/K
Graphically, it is represented as If Tf° is freezing point of pure solvent and T f freezing point
of the solution, then Tf < Tf°. The depression in freezing
Atmospheric point DTf is given by
pressure nt DTf = Tf° – Tf
. of solve Quantitatively, it is given by
B.P
tion DTf = Kf × m
f solu
Vapour pressure

B.P.
o Tf° – Tf = Kf × m (m ® molality)
w 2 1000
= Kf × ´
M 2 w1
DTb
1000 ´ K f ´ w 2
M2 = (Kf = molal depression constant
DTf ´ w1
or cryoscopic constant)

Relation for molal depression constant :

Tb° Tb M1RTf2
Temperature Kf =
If Tb° is vapour pressure of solvent and Tb is vapour pressure 1000DHf
of the solution than where, Tf = Freezing point of pure solvent
Tb > Tb° DHf = Enthalpy of fusion of solvent
The difference DTb is called elevation in boiling point Note:
DTb = Tb – Tb° Depression in freezing point is used in:
Quantitatively, it is given by (a) making antifreeze solutions
(b) melting of ice on the roads
DTb = K b ´ m , where m = molality
w 1000 (iv) Osmotic Pressure
Tb - Tb ° = K b ´ m = K b ´ 2 ´ The net spontaneous flow of the solvent molecules from the
M 2 w1
solvent to the solution or from a less concentrated solution
1000 ´ K b ´ w 2 to a more concentrated solution through a semipermeable
M2 = (Kb = molal elevation constant or
DTb ´ w1 membrane is called osmosis.
ebullioscopic constant) The minimum excess pressure that has to be applied on the
Molal elevation constant is also given as solution to prevent the entry of the solvent into the solution
through the semi-permeable membrane is called osmotic
RM1Tb2 pressure.
Kb =
1000 DH v Quantitatively, it is given by
where, Tb = b. pt. of pure solvent pµC where p ® osmotic pressure
DHv = Enthalpy of fusion of solvent pµT C ® conc. of solution
M1 = Molar mass of solvent \ p µ CT T ® temperature
EBD_7327
378 CHEMISTRY

(a) Relative lowering of vapour pressure


p = CRT R ® gas constant
p° - ps
n = ix 2
C= p°
V
where, n = number of moles of the solute (b) Elevation in boiling point
V = volume of solution DTb = Tb - Tb ° = ik b m
n (c) Depression in freezing point
p = RT
V DTf = Tf ° - Tf = ik f m
pV = nRT (d) Osmotic pressure
Note: n
p = iCRT = i RT
(i) When the semi-permeable membrane permits solvent v
molecules to come out through the membrane, it is called Note:
exo-osmosis and when the membrane permits solvent (i) i > 1 if there is dissociation of the solute in solution and i < 1
molecules to enter inside, it is called endo-osmosis. if there is association of the solute in solution.
(ii) If pressure higher than osmotic pressure is applied on the (ii) For 100% dissociation of the solute, i = number of ions
solution, the solvent will flow from the solution into the pure produced from one molecule of the solute.
solvent through the semi-permeable membrane. This process (iii) Degree of dissociation is defined as the fraction of the total
is called reverse osmosis and it is used in the desalination of substance that undergoes dissociation i.e.,
water i.e., removal of salts from sea water.
(iii) Such solutions which have the same osmotic pressure at the Number of moles dissociated
Degree of dissociation (a) =
same temperature are called isotonic solutions. If one solution Total number of moles taken
is of lower osmotic pressure, it is called hypotonic with respect (A)n nA
to more concentrated solution. The more concentrated Initial mole 1 0
solution is said to be hypertonic with respect to dilute Dissociation mole a na
solution. Equilibrium mole (1 – a) na
(iv) When an animal cell is placed in a hypotonic solution, the \ Total number of moles = [(1 – a) + na] mole
cell swells and bursts (hemolysis).When a cell is placed in = 1 + a(n – 1) mole
hypertonic solution, the cell contracts in size (plasmolysis).
1 + a (n –1)
When excess of fertilizers (like urea) are applied, plasmolysis \i= = 1 + na – a
takes place and plants dry up (wilt). 1

ABNORMAL MOLECULAR MASSES i -1


a= (n ® number of ions produced on dissociation)
n -1
In case of aqueous solutions of electrolytes, inorganic bases and
salts, which dissociate completely or partially in the solution, or if M c - M o é M c ® normal molecular mass ù
association takes place in the solution as in the case of acetic acid or, a = M ( n - 1) ê M ® observed molecular massú
and benzoic acid, the number of particles in the solution changes o ë o û
(increases in case of dissociation and decreases for association). (iv) Degree of association is defined as the fraction of the total
As a result, value of colligative property also changes and as substance which exists in the form of associated molecules
i.e.,
1 nA (A)n
molecular mass µ colligative properties
a
\ observed molecular mass comes out to be different from Equilibrium (1 – a) mole
n
theoretical value. This observed molecular mass is called abnormal moles mole
molecular mass. a
Van’t Hoff Factor \ Total no. of moles = 1 – a +
n
The extent of association or dissociation can be determined with a
the help of van’t Hoff factor (i) It is defined as the ratio of normal 1– a +
mass to the observed molecular mass of solute i.e., i= n
1
Normal molar mass Number of moles associated
i= Degree of association (a) =
Abnormal molar mass Total number of moles taken
Observed colligative property n
or i = \ a = (1 – i)
Calculated colligative property n -1
n = number of molecules of the solute associated
Total number of moles of particles after association / dissociation
or i = M o - Mc n
Number of moles of particles before association / dissociation
or, a = Mo ( n - 1)
Thus, the modified colligative properties are:
Solutions 379

CONCEPT MAP
EBD_7327
380 CHEMISTRY

1. Lowering in vapur pressrue is the highest for : 13. The osmotic pressure at 17ºC of an aq. solution containing
(a) 0.1 M glucose (b) 0.1 M BaCl2 1.75 gm of sucrose per 150 ml solution is :
(c) 0.2 M urea (d) 0.1 M MgSO4 (a) 0.08 atm (b) 8.1 atm (c) 0.8 atm (d) 9.1 atm
2. Formation of a solution from two components can be 14. An aqueous solution of glucose is 10% in strength. The
considered as volume in which 1 g mole of it is dissolved, will be :
(i) Pure solvent ® separated solvent molecules, DH1 (a) 9 litre (b) 1.8 litre (c) 8 litre (d) 0.9 litre
(ii) Pure solute ® separated solute molecules, DH2
15. 10 g of NaCl is dissolved in 10 6 g of the solution. Its
(iii) Separated solvent & solute molecules ® Solution,
DH3 concentration is :
Solution so formed will be ideal if (a) 100 ppm (b) 0.1 ppm(c) 1 ppm (d) 10 ppm
16. On adding a solute to a solvent having vapour pressure 0.80
(a) ΧHsoln < ΧH3 ,ΧH1 ,ΧH 2 atm, vapour pressure reduces to 0.60 atm. Mole fraction of
(b) ΧHsoln < ΧH1 ∗ ΧH 2 ∗ ΧH3 solute is :
(a) 0.25 (b) 0.75 (c) 0.50 (d) 0.33
(c) ΧHsoln < ΧH1 ∗ ΧH 2 ,ΧH3 17. If 0.44 gm of substance dissolved in 22.2 gm of benzene
(d) ΧHsoln < ΧH1 ,ΧH 2 ,ΧH3 lowers the freezing point of benzene by 0.567ºC, then the
3. An aqueous solution containing 1 gm of urea boils at molecular mass of substance is, the molal depression
100.25ºC. The aqueous solution containing 3 g of glucose in constant= 5.12ºC mol–1.
the same volume will boil at (mol. wt. of urea = 60 and of (a) 128.4 (b) 156.6 (c) 178.9 (d) 232.4
glucose = 180) : 18. 2.5 litres of NaCl solution contain 5 moles of the solute.
(a) 100.25ºC (b) 100ºC What is the molarity?
(c) 100.15ºC (d) 100.50ºC (a) 5 molar (b) 2 molar (c) 2.5 molar (d) 12.5 molar
4. The statement “If 0.003 moles of a gas are dissolved in 900 g 19. Which of the following statements, regarding the mole
of water under a pressure of 1 atmosphere, 0.006 moles will fraction (x) of a component in solution, is incorrect?
be dissolved under a pressure of 2 atmospheres”, illustrates (a) 0 £ x £ 1 (b) x £ 1
(a) Dalton’s law of partial pressure (c) x is always non-negative (d) None of these
(b) Graham’s law 20. When 10 g of a non-volatile solute is dissolved in 100 g of
(c) Raoult’s law benzene, it raises boiling point by 1ºC then molecular mass
(d) Henry’s law of the solute is (Kb for benzene = 2.53 km–1)
5. Which of the following modes of expressing concentration (a) 223 g (b) 243 g (c) 233 g (d) 253 g
is independent of temperature ? 21. The mole fraction of the solute in one molal aqueous
(a) Molarity (b) Molality solution is:
(c) Formality (d) Normality (a) 0.009 (b) 0.018 (c) 0.027 (d) 0.036
6. Molarity of liquid HCl will be, if density of solution is 22. When a solute is present in trace quantities the following
1.17 gm/cc expression is used
(a) 36.5 (b) 32.05 (c) 18.25 (d) 42.10 (a) Gram per million (b) Milligram percent
7. The molarity of a solution containing 5 gm of NaOH in 250 (c) Microgram percent (d) Parts per million
ml solution : 23. 5 ml of N HCl, 20 ml of N/2 H2SO4 and 30 ml of N/3 HNO3
(a) 0.5 (b) 1.0 (c) 1.5 (d) 2.0 are mixed together and volume made to one litre. The
8. Concentrated hydrochloric acid contains 37% HCl (by mass). normality of the resulting solution is
The density of this solution is 1.18 gm/ml. The molarity of N N N N
the solution is : (a) (b) (c) (d)
5 10 20 40
(a) 10 M (b) 12 M (c) 13 M (d) 14 M 24. The molal freezing point constant for water is 1.86ºC.
9. An X molal solution of a compound in benzene has mole Therefore, the change in freezing point of 0.1 molal of NaCl
fraction of solute equal to 0.2. The value of X is solution in water is expected to be
(a) 14 (b) 3.2 (c) 1.4 (d) 2 (a) 0.372ºC (b) –1.86ºC (c) –0.186ºC (d) –0.372ºC
10. The molarity of the solution containing 7.1 g of Na2SO4 in 25. Vapour pressure of benzene at 30°C is 121.8 mm Hg. When
100 ml of aqueous solution is 15 g of a non volatile solute is dissolved in 250 g of benzene
(a) 2 M (b) 0.5 M (c) 1 M (d) 0.05 M its vapour pressure decreased to 120.2 mm Hg. The molecular
11. The vapour pressure of pure benzene at 25°C is 640 mm Hg weight of the solute (Mo. wt. of solvent = 78)
and that of solution of solute A is 630 mm Hg. The molality (a) 356.2 (b) 456.8 (c) 530.1 (d) 656.7
of solution is – 26. If 20 g of a solute was dissolved in 500 ml. of water and
(a) 0.2 m (b) 0.4 m (c) 0.5 m (d) 0.1 m osmotic pressure of the solution was found to be 600 mm of
12. The molarity of pure water is : Hg at 15ºC, then molecular weight of the solute is :
(a) 50 M (b) 18 M (c) 55.6 M (d) 100 M (a) 1000 (b) 1200 (c) 1400 (d) 1800
Solutions 381

27. A solution containing components A and B follows Raoult's 39. In the accompanied diagram, the ideal behaviour of a solution
law when is shown by the line/s
(a) A – B attraction force is greater than A – A and B – B pA+pB
C D
(b) A – B attraction force is less than A – A and B – B p0
(c) A – B attraction force remains same as A–A and B –B A =x
A p
(d) volume of solution is different from sum of volume of A
solute and solvent p0 p p0
A B B
28. The vapour pressure of two liquids ‘P’ and ‘Q’ are 80 and 0 =x B
60 torr, respectively. The total vapour pressure of solution p B
obtained by mixing 3 mole of P and 2 mole of Q would be A Mole fractions B
(a) 72 torr (b) 140 torr xA = 1 xA = 0
(c) 68 torr (d) 20 torr xB = 0 xB = 1
29. A mixture of components A and B will show –ve deviation (a) AD (b) CB
when (c) CD (d) AD, CB and CD
(a) D Vmix > 0 (b) D Hmix < 0 40. Which one of the following is non-ideal solution
(c) A – B interaction is weaker than A – A and B – B (a) Benzene + toluene
interactions (b) n-hexane + n-heptane
(d) A–B interaction is stronger than A–A and B–B
(c) Ethyl bromide + ethyl iodide
interactions.
30. A non-ideal solution was prepared by mixing 30 mL (d) CCl4 + CHCl3
chloroform and 50 mL acetone. The volume of mixture will be 41. The molal elevation constant is the ratio of elevation in
(a) > 80 mL (b) < 80 mL(c) = 80 mL (d) ³ 80 mL boiling point to:
(a) molality (b) mole fraction
31. The volume of gas decreases from 500 cc to 300 cc when a
sample of gas is compressed by an average pressure of 0.6 (c) molarity (d) none
atm. During this process 10 J of heat is liberated . The change 42. The azeotropic mixture of water (b.p.100°C) and HCl
in internal energy is. (b.p.85°C) boils at 108.5°C. When this mixture is distilled it is
(a) –2.16 J (b) 12.15 J (c) 2.16 J (d) 101.3 J possible to obtain
32. Which of the following liquid pairs shows a positive (a) pure HCl
deviation from Raoult’s law ? (b) pure water
(a) Water - Nitric acid (c) pure water as well as pure HCl
(b) Benzene - Methanol (d) neither HCl nor H2O in their pure states
(c) Water - Hydrochloric acid 43. The molar freezing point constant for water is 1.86 °C/mol. If
(d) Acetone - Chloroform 342 g of cane sugar (C12H22O11) is dissolved in 1000 g of
33. 18 g of glucose (C6H12O6) is added to 178.2 g of water. The water, the solution will freeze at :
vapour pressure of water for this aqueous solution is (a) – 1.86°C (b) 1.86°C
(a) 76.00 torr (b) 752.40 torr (c) – 3.92°C (d) 2.42°C
(c) 759.00 torr (d) 7.60 torr 44. Which will form maximum boiling point azeotrope
34. If all the four following compounds were sold at the same (a) HNO3 + H2O solution
price, which would be cheapest for preparing an antifreeze (b) C2H5OH + H2O solution
solution for a car radiator?
(c) C6H6 + C6H5CH3 solution
(a) C3H5(OH)3 (b) C2H4(OH)2
(c) C2H5OH (d) CH3OH (d) None of these
35. The value of P° for benzene is 640 mm of Hg. The vapour 45. The vapour pressure at a given temperature of an ideal
pressure of solution containing 2.5gm substance in 39gm. solution containing 0.2 mol of a non-volatile solute and 0.8
benzene is 600mm of Hg the molecular mass of X is – mol of solvent is 60 mm of Hg. The vapour pressure of the
(a) 65.25 (b) 130 (c) 40 (d) 75 pure solvent at the same temperature is
36. An ideal solution is formed when its components (a) 150 mm of Hg (b) 60 mm of Hg
(a) have no volume change on mixing (c) 75 mm of Hg (d) 120 mm of Hg
(b) have no enthalpy change on mixing 46. When a hard boiled egg after shelling is immersed in
(c) Both (a) and (b) are correct saturated brine:
(d) Neither (a) nor (b) is correct (a) it shrinks
37. All form ideal solution except (b) it grows is size
(a) C6H6 and C6H5CH3 (b) C2H6 and C2H5I (c) its size remains unchanged
(c) C6H5Cl and C6H5Br (d) C2H5 I and C2H5OH. (d) it initially grows in size and then shrinks
38. For which of the following parameters the structural isomers 47. The vapour pressure of a solvent decreases by 10 mm of Hg
C2H5OH and CH3OCH3 would be expected to have the same when a non-volatile solute was added to the solvent. The
values?(Assume ideal behaviour) mole fraction of the solute in the solution is 0.2. What should
(a) Boiling points be the mole fraction of the solvent if the decrease in the
(b) Vapour pressure at the same temperature vapour pressure is to be 20 mm of Hg ?
(c) Heat of vaporization (a) 0.8 (b) 0.6 (c) 0.4 (d) 0.2
(d) Gaseous densities at the same temperature and pressure
EBD_7327
382 CHEMISTRY

48. A solution of acetone in ethanol 61. The freezing point of a 3% aq. solution of A is equal to the
(a) shows a positive deviation from Raoult’s law freezing point of 9% aq. solution of B. If the molecular weight
(b) behaves like a non ideal solution of ‘A’ is 60, then the molecular wt. of B is
(c) obeys Raoult’s law (a) 45 (b) 90 (c) 180 (d) 360
(d) shows a negative deviation from Raoult’s law 62. 1.00 g of a non-electrolyte solute (molar mass 250 g mol–1)
49. Azeotropic mixture of water and HCl has the percentage of was dissolved in 51.2 g of benzene. If the freezing point
HCl. depression constant, Kf of benzene is 5.12 K kg mol–1, the
(a) 20.2% (b) 26.4% (c) 48% (d) 68% freezing point of benzene will be lowered by
50. Which of the following aqueous solution has minimum (a) 0.3 K (b) 0.5 K (c) 0.4 K (d) 0.2
freezing point ? 63. The normality of mixture obtained by mixing 100 ml of 0.4 M
(a) 0.01 m NaCl (b) 0.005 m C2H5OH H 2SO4 and 200 ml of 0.2 M HCl is
(c) 0.005 m MgI2 (d) 0.005 m MgSO4.
(a) 0.0267 (b) 0.2670 (c) 1.0267 (d) 1.1670
51. On adding 0.750 gm of compound in 25 gm of solvent, the
64. Brownian movement is found in
freezing point is depressed by 0.502°C. The molecular wt. of
(a) unsaturated solution (b) saturated solution
the substance is [The molecular depression constant =
(c) colloidal solution (d) suspension solution
50.2°C per 100 gm of solvent]
65. A solution is formed by diluting 250 ml. of 0.400 N H2SO4 with
(a) 100 (b) 200 (c) 300 (d) 400
one litre of water. The normality of above formed solution is :
52. 5 gm of CH 3COOH is dissolved in one litre of ethanol. (a) 0.400 N (b) 0.899 N (c) 0.040 N (d) 0.080 N
Suppose there is no reaction between them. If the density of 66. In a 0.2 molal aqueous solution of a weak acid HX the degree
ethanol is 0.789 gm/ml then the molality of resulting solution of ionization is 0.3. Taking kf for water as 1.85, the freezing
is point of the solution will be nearest to
(a) 0.0256 (b) 0.1056 (c) 0.1288 (d) 0.1476 (a) – 0.360ºC (b) – 0.260ºC
53. Which one of the following is a colligative property ? (c) + 0.481ºC (d) – 0.481ºC
(a) Boiling point (b) Vapour pressure 67. Which one of the following aqueous solutions will exihibit
(c) Osmotic pressure (d) Freezing point highest boiling point ?
54. If 0.1 M solution of glucose and 0.1 M solution of urea are (a) 0.015 M urea (b) 0.01 M KNO3
placed on two sides of the semipermeable membrane to equal (c) 0.01 M Na2SO4 (d) 0.015 M glucose
heights, then it will be correct to say that 68. The amount of dibasic acid present in 100 ml of the aq.
(a) there will be no net movement across the membrane solution to give solution of [mol. wt. = 200] normality of 0.1
(b) glucose will flow towards urea solution N is
(c) urea will flow towards glucose solution (a) 0.5 gm (b) 1 gm (c) 1.5 gm (d) 2 gm
(d) water will flow from urea solution to glucose 69. The elevation in boiling point of a solution of 13.44 g of
55. When 800 gm of a 40% solution by weight was cooled, 100 CuCl2 in 1 kg of water using the following information will
gm of solute precipitated. The percentage composition of be (Molecular weight of CuCl2 = 134.4 g and Kb= 0.52 K
remaining solution is kg mol-1)
(a) 31.43% (b) 5.56% (c) 6.78% (d) 6.96% (a) 0.16 (b) 0.05 (c) 0.1 (d) 0.2
56. At 25°C, the highest osmotic pressure is exhibited by 0.1 M 70. The molecular weight of benzoic acid in benzene as
solution of determined by depression in freezing point method
(a) CaCl2 (b) KCl (c) glucose (d) urea corresponds to :
57. Which one of the following salts will have the same value of (a) ionization of benzoic acid.
van’t Hoff factor (i) as that of K4[Fe(CN)6]. (b) dimerization of benzoic acid.
(a) Al2(SO4)3 (b) NaCl (c) Al(NO3)3 (d) Na2SO4. (c) trimerization of benzoic acid.
58. Which of the following 0.10 m aqueous solutions will have (d) solvation of benzoic acid.
the lowest freezing point ? 71. A 0.5 molal solution of ethylene glycol in water is used as
(a) Al2(SO4)3 (b) C6H12O6 coolant in a car. If the freezing point constant of water be
(c) KCl (d) C12 H22 O11 1.86°C per mole, the mixture shall freeze at
59. Which of the following colligative property can provide (a) 0.93°C (b) –0.93°C(c) 1.86°C (d) –1.86°C
molar mass of proteins (or polymers or colloids) with greatest 72. The rise in the boiling point of a solution containing 1.8 g of
precision ? glucose in 100 g of solvent is 0.1°C. The molal elevation
(a) Osmotic pressure constant of the liquid is
(b) Elevation of boiling point (a) 0.01 K/m (b) 0.1 K/m (c) 1 K/m (d) 10 K/m
(c) Depression of freezing point 73. The solutions which have same osmotic concentrations are
(d) Relative lowering of vapour pressure known as
60. Camphor is often used in molecular mass determination (a) Normal (b) Isotonic
because (c) Hypotonic (d) Hypertonic
(a) it is readily available 74. The normal boiling point of the solution is the temperature
(b) it has a very high cryoscopic constant at which the vapour pressure of the solution is –
(c) it is volatile (a) equal to 1 torr (b) equal to 76 mm Hg
(d) it is solvent for organic substances (c) equal to 2.0 atm (d) equal to 1 atm
Solutions 383

75. The temperature at which 10% aqueous solution (w/v) of 87. The molal elevation constant for water is 0.52. What will be
glucose will exhibit the osmotic pressure of 14 bar is – the boiling point of 2 molar sucrose solution at 1 atm.
(a) 307.3°C (b) 307.3 K (c) 34 K (d) – 34°C pressure? (Assume B.P. of pure water as 100°C)
76. 4 gm castic soda is dissolved in 100 cc of solution. The (a) 101.04°C (b) 100.26°C
normality of solution is (c) 100.52°C (d) 99.74°C
(a) 0 (b) 0.5 (c) 1 (d) 1.5 88. A 0.0020 m aqueous solution of an ionic compound
77. If A contains 2% NaCl and is separated by a semipermeable Co(NH3)5(NO2)Cl freezes at – 0.00732 °C. Number of moles
membrane from B which contains 10% NaCl, then which of ions which 1 mol of ionic compound produces on being
event will occur ? dissolved in water will be (Kf = – 1.86°C/m)
(a) NaCl will flow from A to B (a) 3 (b) 4 (c) 1 (d) 2
(b) NaCl will flow from B to A 89. An aqueous solution is 1.00 molal in KI. Which change will
(c) Water will flow from A to B cause the vapour pressure of the solution to increase?
(d) Water will flow from B to A (a) Addition of NaCl
78. KBr is 80% ionized in solution. The freezing point of 0.4
molal solution of KBr is : (b) Addition of Na 2SO4
(c) Addition of 1.00 molal KI
K kg
K f (H 2 O) = 1.86 (d) Addition of water
mole 90. A solution of sucrose (molar mass = 342 g mol –1) has been
(a) 274.339 K (b) – 1.339 K prepared by dissolving 68.5 g of sucrose in 1000 g of water.
(c) 257.3 K (d) – 1.339°C The freezing point of the solution obtained will be
79. Which of the following pairs of solution are isotonic at the (Kf for water = 1.86 K kg mol–1).
same temperature ? (a) – 0.372°C (b) – 0.520°C
(a) 0.1 M Ca(NO3)2 and 0.1 M Na2SO4 (d) + 0.372°C (d) – 0.570°C
(b) 0.1 M NaCl and 0.1 M Na2SO4
(c) 0.1 M urea and 0.1 M MgCl2 91. 25.3 g of sodium carbonate, Na 2CO3 is dissolved in enough
(d) 0.2 M urea and 0.1 M NaCl water to make 250 mL of solution. If sodium carbonate
80. The relationship between the values of osmotic pressure of dissociates completely, molar concentration of sodium ions,
0.1 M solutions of KNO3 ( P1 ) and CH3COOH ( P2 ) is : Na + and carbonate ions, CO 32– are respectively (Molar mass
P1 P2 of Na2CO3 = 106 g mol–1)
(a) = (b) P1 > P2 (a) 0.955 M and 1.910 M (b) 1.910 M and 0.955 M
P1 + P2 P1 + P2 (c) 1.90 M and 1.910 M (d) 0.477 M and 0.477 M
(d) P2 > P1 (d) P1 = P2 92. The freezing point depression constant for water is
81. Which of the following solutions will exhibit highest boiling – 1.86ºC m–1. If 5.00 g Na2SO4 is dissolved in 45.0 g H2O, the
point? freezing point is changed by – 3.82ºC. Calculate the van’t
(a) 0.01 M Na2SO4 (aq) (b) 0.01 M KNO3 (aq) Hoff factor for Na2SO4
(c) 0.015 M urea (aq) (d) 0.015 M glucose (aq) (a) 2.05 (b) 2.62 (c) 3.11 (d) 0.381
82. What is the freezing point of a solution containing 8.1 g HBr 93. The van’t Hoff factor i for a compound which undergoes
in 100 g water assuming the acid to be 90% ionised ? dissociation in one solvent and association in other solvent
(Kf for water = 1.86 K kg mol–1) : is respectively :
(a) 0.85K (b) – 3.53K(c) 0K (d) – 0.35K (a) less than one and greater than one.
83. Osmotic pressure of 0.4% urea solution is 1.64 atm and that (b) less than one and less than one.
of 3.42% cane sugar is 2.46 atm. When the above two (c) greater than one and less than one.
solutions are mixed, the osmotic pressure of the resulting (d) greater than one and greater than one.
solution is : 94. Mole fraction of the solute in a 1.00 molal aqueous solution is
(a) 0.82 atm (b) 2.46 atm (a) 0.1770 (b) 0.0177 (c) 0.0344 (d) 1.7700
(c) 1.64 atm (d) 4.10 atm 95. A 0.1 molal aqueous solution of a weak acid is 30% ionized.
84. The freezing point of 1% solution of lead nitrate in water will If Kf for water is 1.86°C/m, the freezing point of the solution
be : will be :
(a) 2°C (b) 1°C (c) 0°C (d) below 0°C (a) – 0.18°C (b) – 0.54°C (c) – 0.36°C (d) – 0.24°C
85. A solution of urea (mol. mass 56 g mol-1) boils at 100.18°C 96. 200 mL of an aqueous solution of a protein contains its 1.26 g.
at the atmospheric pressure. If Kf and Kb for water are 1.86 The osmotic pressure of this solution at 300 K is found to be
and 0.512 K kg mol-1 respectively, the above solution will 2.57 × 10–3 bar. The molar mass of protein will be (R = 0.083 L
freeze at bar mol–1 K–1)
(a) 0.654°C (b) - 0.654°C (a) 51022 g mol–1 (b) 122044 g mol–1
(c) 6.54°C (d) - 6.54°C (c) 31011 g mol–1 (d) 61038 g mol–1
N N N 97. PA and PB are the vapour pressure of pure liquid components,
86. If 50 ml H2SO4, 30 ml HNO3, 10 ml HCl is mixed A and B, respectively of an ideal binary solution. If XA
10 3 2
and solution is made to 1L. Then normality of resultant represents the mole fraction of component A, the total
solution is pressure of the solution will be.
N N N (a) PA + XA (PB – PA) (b) PA + XA (PA – PB)
(a) (b) (c) (d) N (c) PB + XA (PB – PA) (d) PB + XA (PA – PB)
20 40 50
EBD_7327
384 CHEMISTRY

98. A mixture of ethyl alcohol and propyl alcohol has a vapour (a) The solution is non-ideal, showing – ve deviation from
pressure of 290 mm Hg at 300 K. The vapour pressure of Raoult’s Law.
propyl alcohol is 200 mm Hg. If the mole fraction of ethyl (b) The solution is non-ideal, showing + ve deviation from
alcohol is 0.6, its vapour pressure (in mm Hg) at the same Raoult’s Law.
temperature will be (c) n-heptane shows + ve deviation while ethanol shows
(a) 360 (b) 350 (c) 300 (d) 700 – ve deviation from Raoult’s Law.
99. A binary liquid solution is prepared by mixing n-heptane (d) The solution formed is an ideal solution.
and ethanol. Which one of the following statements is correct 100. The molarity of a solution obtained by mixing 750 mL of
regarding the behaviour of the solution? 0.5(M) HCl with 250 mL of 2(M) HCl will be :
(a) 0.875 M (b) 1.00 M (c) 1.75 M (d) 0.975 M

1. p1, p2, p3 and p4 atm are the osmotic pressures of 5% (mass/ 7. The mole fraction of water in 20% aqueous solution (By
volume) solutions of urea, fructose, sucrose and KCl weight) of H2O2 is
respectively at certain temperature. The correct order of their 77 68 20 80
magnitudes is : (a) (b) (c) (d)
68 77 80 20
(a) p1 > p 4 > p 2 > p3 (b) p1 < p 4 < p 2 < p3 8. Molarity of H2SO4 is 18 M. Its density is 1.8 g/ml. Hence
(c) p 4 > p1 > p 2 > p3 (d) p 4 > p1 > p3 > p 2 molality is
(a) 36 (b) 200 (c) 500 (d) 18
2. The osmotic pressure (at 27° C) of an aqueous solution 9. The solution containing 4.0 gm of a polyvinyl chloride
(200mL) containing 6 g of a protein is 2 × 10–3 atm. If polymer in 1 litre of dioxane was found to have an osmotic
R = 0.080 L atm mol–1K–1, the molecular weight of protein is pressure 6.0 × 10–4 atmosphere at 300K, the value of R used
(a) 7.2 × 105 (b) 3.6 × 105 is 0.082 litre atmosphere mole–1K–1. The molecular mass of
(c) 1.8 ×10 5 (d) 1.0 × 105 the polymer was found to be
3. The freezing point of equimolal aqueous solutions will be (a) 3.0 × 102 (b) 1.6 × 105 (c) 5.6 × 104 (d) 6.4 × 102
highest for : 10. At temperature 327°C and concentration C osmotic pressure
(a) C6H5NH3Cl (aniline hydrochloride) of a solution is P, the same solution at concentration C/2 and
(b) Ca(NO3)2 at temperature 427°C shows osmotic pressure of 2 atm, value
(c) La(NO3)3 of P will be
(d) C6H12O6 (glucose)
12 24 6 5
4. Which of the following has been arranged in the increasing (a) (b) (c) (d)
order of freezing point? 7 7 5 6
(a) 0.025M KNO 3 < 0.1M NH 2 CSNH 2 11. In countries nearer to polar region, the roads are sprinkled
with CaCl2. This is
< 0.05 M BaCl 2 < 0.1M NaCl (a) to minimise the snow fall
(b) 0.1M NaCl < 0.05M BaCl 2 (b) to minimise pollution
< 0.1M NH 2 CSNH 2 < 0.025 M KNO 3 (c) to minimise the accumulation of dust on the road
(c) 0.1M NH 2 CSNH 2 < 0.1M NaCl (d) to minimise the wear and tear of the roads
12. If a is the degree of dissociation of Na2SO4, the Vant Hoff’s
< 0.05M BaCl 2 < 0.025M KNO 3 factor (i ) used for calculating the molecular mass is
(d) 0.025M KNO 3 < 0.05M BaCl 2 (a) 1 + a (b) 1 – a (c) 1 + 2a (d) 1 – 2a
< 0.1M NaCl < 0.1M NH 2 CSNH 2 13. The correct relationship between the boiling points of very
5. The volume of 4 N HCl and 10 N HCl required to make 1 litre dilute solutions of AlCl3 (t1) and CaCl2 (t2), having the same
of 6 N HCl are molar concentration is
(a) 0.75 litre of 10 N HCl and 0.25 litre of 4 N HCl (a) t1 = t2 (b) t1 > t2 (c) t2 > t1 (d) t2 ³ t1
(b) 0.50 litre of 4 N HCl and 0.50 litre of 10 N HCl 14. If the various terms in the given below expressions have
(c) 0.67 litre of 4 N HCl and 0.33 litre of 10 N HCl usual meanings, the van’t Hoff factor (i) cannot be calculated
(d) 0.80 litre of 4 N HCl and 0.20 litre of 10 N HCl by which one of the following expressions
6. A solution of Al2(SO4)3 {d=1.253 gm / ml} contain 22% salt by (a) pV = inRT
weight. The molarity, normality and molality of the solution is (b) DT f = iK f .m
(a) 0.805 M, 4.83 N, 0.825 m (c) DTb = iK b .m
(b) 0.825 M, 48.3 N, 0.805 m P° -P æ n ö
(c) 4.83 M, 4.83 N, 4.83 m (d) solvent solution = i ç ÷
(d) None of these
°
Psolvent è N+nø
Solutions 385

15. A 0.001 molal solution of [Pt(NH3)4Cl4] in water had a freezing 26. A 5% solution of cane sugar (molar mass 342) is isotonic
point depression of 0.0054°C. If K f for water is 1.80, the with 1% of a solution of an unknown solute. The molar mass
correct formulation for the above molecule is of unknown solute in g/mol is :
(a) [Pt(NH3)4 Cl3]Cl (b) [Pt(NH3)4 Cl2]Cl2
(a) 171.2 (b) 68.4 (c) 34.2 (d) 136.2
(c) [Pt(NH3)4 Cl]Cl3 (d) [Pt(NH3)4 Cl4]
27. –1
Kf for water is 1.86 K kg mol . If your automobile radiator
16. The amount of urea to be dissolved in 500 c.c. of water
(K = 1.86°C mol–1) to produce a depression of 0.186°C in the holds 1.0 kg of water, how many grams of ethylene glycol
freezing point is : (C2H6O2) must you add to get the freezing point of the
(a) 9 g (b) 6 g (c) 3 g (d) 0.3 g solution lowered to –2.8ºC ?
17. Which has the minimum freezing point ? (a) 72 g (b) 93 g (c) 39 g (d) 27 g
(a) One molal NaCl aq. solution
28. When 20 g of naphthoic acid (C11H8O2) is dissolved in 50 g
(b) One molal CaCl2 aq. solution
(c) One molal KCl aq. solution of benzene (Kf = 1.72 K kg mol–1), a freezing point depression
(d) One molal urea aq. solution of 2 K is observed. The Van't Hoff factor (i) is
18. On mixing 3 g of non - volatile solute in 200 mL of water, its (a) 0.5 (b) 1 (c) 2 (d) 3
boiling point (100°) becomes 100.52°C. If Kb for water is 0.6 29. The Henry’s law constant for the solubility of N2 gas in
K/m then molecular wt. of solute is : water at 298 K is 1.0 × 105 atm. The mole fraction of N2 in air
(a) 10.5 g mol -1 (b) 12.6 g mol -1 is 0.8. The number of moles of N2 from air dissolved in
(c) 15.7 g mol -1 (d) 17.3 g mol -1 10 moles of water at 298 K and 5 atm pressure is
19. 1 mol each of the following compounds is dissolved in 1L of (a) 4.0× 10– 4 (b) 4.0 × 10–5
solution. Which will have the largest DTb value? (c) 5.0 × 10– 4 (d) 4.0 × 10–6
(a) HF (b) HCl (c) HBr (d) HI.
30. Dissolving 120 g of urea (mol. wt. 60) in 1000 g of water gave
20. Equal masses of methane and oxygen are mixed in an empty
container at 25°C. The fraction of the total pressure exerted a solution of density 1.15 g/mL. The molarity of the solution is
by oxygen is (a) 1.78 M (b) 2.00 M (c) 2.05 M (d) 2.22 M
1 273 31. The freezing point (in °C) of a solution containing 0.1 g
(a) 1/2 (b) 2/3 (c) ´ (d) 1/3.
3 298 of K3[Fe(CN)6] (Mol. wt. 329) in 100 g of water (Kf = 1.86 K
21. A 5.25% solution of a substance is isotonic with a 1.5% solu- kg mol–1) is
tion of urea (molar mass = 60 g mol–1) in the same solvent. If (a) –2.3 × 10–2 (b) –5.7 × 10–2
the densities of both the solutions are assumed to be equal to –3
(c) –5.7 × 10 (d) –1.2 × 10–2
1.0 g cm–3, molar mass of the substance will be
(a) 210.0 g mol–1 (b) 90.0 g mol–1 32. For a dilute solution containing 2.5 g of a non-volatile non-
(c) 115.0 g mol –1 (d) 105.0 g mol–1. electrolyte solute in 100 g of water, the elevation
22. At 80° C, the vapour pressure of pure liquid ‘A’ is 520 mm Hg in boiling point at 1 atm pressure is 2°C. Assuming
and that of pure liquid ‘B’ is 1000 mm Hg. If a mixture solution concentration of solute is much lower than the concentration
of ‘A’ and ‘B’ boils at 80° C and 1 atm pressure, the amount of solvent, the vapour pressure (mm of Hg) of the solution is
of ‘A’ in the mixture is (1 atm = 760 mm Hg) (take Kb = 0.76 K kg mol–1)
(a) 52 mol percent (b) 34 mol percent
(a) 724 (b) 740 (c) 736 (d) 718
(c) 48 mol percent (d) 50 mol percent
23. The vapour pressure of water at 20°C is 17.5 mm Hg. If 18 g 33. If sodium sulphate is considered to be completely
of glucose (C6H12O6) is added to 178.2 g of water at 20°C, dissociated into cations and anions in aqueous solution,
the vapour pressure of the resulting solution will be the change in freezing point of water (DTf ), when 0.01 mol
(a) 17.325 mm Hg (b) 15.750 mm Hg of sodium sulphate is dissolved in 1 kg of water, is (Kf = 1.86
(c) 16.500 mm Hg (d) 17.500 mm Hg K kg mol–1)
24. Two liquids X and Y form an ideal solution. At 300 K, vapour (a) 0.372 K (b) 0.0558 K
pressure of the solution containing 1 mol of X and 3 mol of Y
is 550 mm Hg. At the same temperature, if 1 mol of Y is further (c) 0.0744 K (d) 0.0186 K
added to this solution, vapour pressure of the solution 34. Which observation(s) reflect(s) colligative properties?
increases by 10 mm Hg. Vapour pressure ( in mm Hg) of X (i) a 0.5 m NaBr solution has a higher vapour pressure
and Y in their pure states will be, respectively: than a 0.5 m BaCl2 solution at the same temperature
(a) 300 and 400 (b) 400 and 600
(ii) pure water freezes at the higher temperature than pure
(c) 500 and 600 (d) 200 and 300
methanol
25. On mixing, heptane and octane form an ideal solution. At
373 K, the vapour pressures of the two liquid components (iii) a 0.1 m NaOH solution freezes at a lower temperature
(heptane and octane) are 105 kPa and 45 kPa respectively. than pure water
Vapour pressure of the solution obtained by mixing 25.0 g of Choose the correct answer from the codes given below
heptane and 35 g of octane will be (molar mass of heptane
(a) (i), (ii) and (iii) (b) (i) and (ii)
= 100 g mol–1 and of octane = 114 g mol–1)
(a) 72.0 kPa (b) 36.1 kPa (c) 96.2 kPa (d) 144.5 kPa (c) (ii) and (iii) (d) (i) and (iii)
EBD_7327
386 CHEMISTRY

35. When a gas is bubbled through water at 298 K, a very dilute 42. The system that forms maximum boiling azeotrope is
solution of the gas is obtained. Henry’s law constant for the (a) carbondisulphide – acetone
gas at 298 K is 100 kbar. If the gas exerts a partial pressure of (b) benzene – toluene
1 bar, the number of millimoles of the gas dissolved in one (c) acetone – chloroform
litre of water is (d) n-hexane – n-heptane
(a) 0.555 (b) 5.55 43. Which one of the following gases has the lowest value of
(c) 0.0555 (d) 55.5 Henry’s law constant?
36. 1.5 g of a non-volatile, non-electrolyte is dissolved in 50 g (a) N2 (b) He
benzene (Kb = 2.5 K kg mol–1). The elevation of the boiling (c) H2 (d) CO2
point of the solution is 0.75 K. The molecular weight of the 44. A solution is prepared by dissolving 10 g NaOH in 1250 mL
of a solvent of density 0.8 mL/g. The molality of the solution
solute in g mol–1 is
in mol kg–1 is
(a) 200 (b) 50
(a) 0.25 (b) 0.2
(c) 75 (d) 100
(c) 0.008 (d) 0.0064
37. Which one of the following statements is false? 45. The freezing point of equimolal aqueous solution will be
(a) Raoult’s law states that the vapour pressure of a highest for
component over a binary solution of volatile liquids is (a) C6H5NH3+Cl– (b) Ca(NO3)2
directly proportional to its mole fraction (c) La(NO3)2 (d) C6H12O6
(b) Two sucrose solutions of the same molality prepared
in different solvents will have the same depression of DIRECTIONS for Qs. 46 to 50 : These are Assertion-Reason
freezing point type questions. Each of these question contains two statements:
(c) The correct order of osmotic pressures of 0.01 M Statement-1 (Assertion) and Statement-2 (Reason). Answer these
questions from the following four options.
solution of each compound is BaCl2 > KCl > CH3COOH
> glucose (a) Statement-1 is true, Statement-2 is true, Statement-2 is a
(d) In the equation osmotic pressure p = MRT, M is the correct explanation for Statement-1
molarity of the solution (b) Statement-1 is True, Statement-2 is True ; Statement-2 is
38. At 300 K the vapour pressure of an ideal solution containing NOT a correct explanation for Statement-1
1 mole of liquid A and 2 moles of liquid B is 500 mm of Hg. (c) Statement-1 is True, Statement-2 is False
(d) Statement-1 is False, Statement-2 is True
The vapour pressure of the solution increases by 25 mm of
46. Statement-1 : If one component of a solution obeys Raoult’s
Hg, if one more mole of B is added to the above ideal solution
law over a certain range of composition, the other component
at 300 K. Then the vapour pressure of A in its pure state is
will not obey Henry’s law in that range.
(a) 300 mm of Hg (b) 400 mm of Hg Statement-2 : Raoult’s law is a special case of Henry’s law.
(c) 500 mm of Hg (d) 600 mm of Hg 47. Statement-1 : Lowering of vapour pressure is directly
39. Henry’s law constant of oxygen is 1.4 × 10–3 mol. lit–1. atm–1 at proportional to osmotic pressure of the solution.
298 K. How much of oxygen is dissolved in 100 ml at 298 K Statement-2 : Osmotic pressure is a colligative property.
when the partial pressure of oxygen is 0.5 atm? 48. Statement-1 : If a liquid solute more volatile than the solvent
(a) 1.4 g (b) 3.2 g is added to the solvent, the vapour pressure of the solution
(c) 22.4 mg (d) 2.24 mg may increase i.e., ps > po.
40. 1 g of a non-volatile, non-electrolyte solute of molar mass Statement-2 : In the presence of a more volatile liquid solute,
250 g/mol was dissolved in 51.2 g of benzene. If the only the solute will form the vapours and solvent will not.
freezing point depression constant Kf of benzene is 5.12 49. Statement-1 : Azeotropic mixtures are formed only by
kg K mol–1. The freezing point of benzene is lowered by non-ideal solutions and they may have boiling points either
(a) 0.3 K (b) 0.5 K greater than both the components or less than both the
(c) 0.2 K (d) 0.4 K components.
41. Negative deviation from Raoult’s law is observed in which Statement-2 : The composition of the vapour phase is same
one of the following binary liquid mixtures? as that of the liquid phase of an azeotropic mixture.
(a) Ethanol and acetone 50. Statement-1 : Camphor is used as solvent in the
(b) Benzene and toluene determination of molecular masses of naphthalene,
(c) Acetone and chloroform anthracene etc.
(d) Chloroethane and bromoethane Statement-2 : Camphor has high molal elevation constant.
Solutions 387

Exemplar Questions 10. The unit of ebullioscopic constant is


1. Which of the following units is useful in relating (a) K kg mol–1 or K (molality)–1
concentration of solution with its vapour pressure? (b) mol kg K–1 or K–1 (molality)
(a) Mole fraction (b) Parts per milion (c) kg mol–1 K–1or K–1 (molality)–1
(c) Mass percentage (d) Molality (d) K mol kg–1or K (molality)
2. On dissolving sugar in water at room temperature solution 11. In comparison to a 0.01 M solution of glucose, the depression
feels cool to touch. Under which of the following cases in freezing point of a 0.01 M MgCl2 solution is ....... .
dissolution of sugar will be most rapid? (a) the same (b) about twice
(a) Sugar crystals in cold water (c) about three time (d) about six times
(b) Sugar crystals in hot water 12. An unripe mango placed in a concentrated salt solution to
(c) Powdered sugar in cold water prepare pickle shrivels because .......... .
(d) Powdered sugar in hot water (a) it gains water due to osmosis
3. At equilibrium the rate of dissolution of a solid solute in a (b) it loses water due to reverse osmosis
volatile liquid solvent is ....... (c) it gains water due to reverse osmosis
(a) less than the rate of crystallisation (d) it loses water due to osmosis
(b) greater than the rate of crystallisation 13. At a given temperature, osmotic presure of a concentrated
(c) equal to the rate of crystallisation solution of a substance ............ .
(d) zero (a) is higher than that of a dilute solution
4. A beaker contains a solution of substance 'A'. Precipitation (b) is lower than that of a dilute solution
of substance 'A' takes place when small amount of 'A' is (c) it gains water due to reverse osmosis
added to the solution. The solution is ........ . (d) it loses water due to osmosis
(a) saturated (b) supersaturated
14. Which of the following statements is false?
(c) unsaturated (d) concentrated
(a) Two different solutions of sucrose of same molality
5. Maximum amount of a solid solute that can be dissolved in
prepared in diffferent solvents will have the same
a specified amount of a given liquid solvent does not
depression in freezing point.
depends upon ......... .
(b) The osmotic pressure of a solution is given by the
(a) temperature (b) nature of solute
(c) pressure (d) nature of solvent equation p = CRT (where, C is the molarity of the
6. Low concentration of oxygen in the blood and tissues of solution)
people living at high altitude is due to ............ . (c) Decreasing order of osmotic pressure for 0.01 M
(a) low temperature aqueous solutions of barium chloride, potassium
(b) low atmospheric pressure chloride, acetic acid and sucrose is
(c) high atmospheric pressure BaCl2 > KCl > CH3COOH > sucrose
(d) both low temperature and high atmospheric pressure (d) According to Raoult's law, the vapour pressure exerted
7. Considering the formation, breaking and strength of by a volatile component of a solution is directly
hydrogen bond, predict which of the following mixtures will proportional to its mole fraction in the solution
show a positive deviation from Raoult's law? 15. The value of van't Hoff factors for KCl, NaCl and K2SO4
(a) Methanol and acetone respectively are ............ .
(b) Chloroform and acetone (a) 2, 2 and 2 (b) 2, 2 and 3
(c) Nitric acid and water (c) 1, 1 and 2 (d) 1, 1 and 1
(d) Phenol and aniline 16. Which of the following statement is false?
8. Colligative properties depend on ............ . (a) Units of atmospheric pressure and osmotic pressure
(a) the nature of the solute particles dissolved in solution are same
(b) the number of solute particles in solution (b) In reverse osmosis, solvent molecules move through a
(c) the physical properties of the solution particles semipermeable membrane from a region of lower
dissolved in solution concentration of solute to a region of higher
(d) the nature of solvent particles concentration
9. Which of the following aqueous solutions should have the (c) The value of molal depression constant depends on
highest boiling point? nature of solvent
(a) 1.0 M NaOH (b) 1.0 M Na2SO4 (d) Relative lowering of vapour pressure, is a dimensionless
(c) 1.0 M NH4NO3 (d) 1.0 M KNO3 quantity
EBD_7327
388 CHEMISTRY

17. Value of Henry's constant KH ............. . 22. Two beakers of capacity 500 mL were taken. One of these
(a) increases with increase in temperature beakers labelled as "A" was filled with 400 mL water whereas
(b) decreases with increase in temperature the beaker labelled "B" was filled with 400 mL of 2M solution
of NaCl. At the same temperature both the beakers were
(c) remains constant
placed in closed containers of same material and same
(d) first increases then decreases capacity as shown in figure.
18. The value of Henry's constant, KH is ............. .
(a) greater for gases with higher solubility
(b) greater for gases with lower solubility A B
(c) constant for all gases
(d) not related to the solubility of gases Water NaCl solution
19. Consider the figure and mark the correct option.
At a given temperature, which of the following statement is
correct about the vapour pressure of pure water and that of
Piston (A) Piston (B)
NaCl solution?
SPM (a) Vapour pressure in container (A) is more than that in
container (B)
(b) Vapour pressure in container (A) is less than that in
Fresh water Concentrated container (B)
(A) sodium chloride (c) Vapour pressure is equal in both the containers
solution in
water (B) (d) Vapour pressure in container (B) is twice the vapour
pressure in container (A)
(a) Water will move form side (A) to side (B) if a pressure 23. If two liquids A and B form minimum boiling azeotrope at
lower than osmotic pressure is applied on piston (B) some specific composition then
(b) Water will move from side (B) to side (A) if a pressure (a) A — B interactions are stronger than those between
greater than osmotic pressure is applied on piston (B) A — A or B — B
(c) Water will move from side (B) to side (A) if a pressure (b) vapour pressure of solution increases because more
equal to osmotic pressure is applied on piston (B) number of molecules of liquids A and B can escape
from the solution
(d) Water will move from side (A) to side (B) if pressure
(c) vapour pressure of solution decreases because less
equal to osmotic pressure is applied on piston (A)
number of molecules of only one of the liquids escape
20. We have three, aqueous solutions of NaCl labelled as 'A', 'B' from the solution
and 'C' with concentrations 0.1 M,0.01M and 0.001 M,
(d) A — B interactions are weaker than those between
respectively. The value of van't Hoff factor for these solutions
A — A or B — B
will be in the order ........... .
24. 4 L of 0.02 M aqueous solution of NaCl was diluted by adding
(a) iA < iB < iC (b) iA > iB > iC
1 L of water. The molality of the resultant solution
(c) iA = iB = iC (d) iA < iB > iC is .................
21. On the basis of information given below mark the correct (a) 0.004 (b) 0.008
option. (c) 0.012 (d) 0.016
Information 25. On the basis of information given below mark the correct
(i) In bromoethane and chloroethane mixture option.
intermolecular interactions of A — A and B — B type Information On adding acetone to methanol some of the
are nearly same as A — B type interactions. hydrogen bonds between methanol molecules break.
(ii) In ethanol and acetone mixture A — A or B — B type (a) At specific composition methanol-acetone mixture will
intermolecular interactions are stronger than A — B form minimum boiling azeotrope and will show positive
type interactions. deviation from Raoult's law
(iii) In chloroform and acetone mixture A — A or B — B (b) At specific composition methanol-acetone mixture will
type intermolecular interactions are weaker than form maximum boiling azeotrope and will show positive
A — B type interactions. deviation from Raoult's law
(a) Solution (ii) and (iii) will follow Raoult's law (c) At specific composition methanol-acetone mixture will
(b) Solution (i) will follow Raoult's law form maximum boiling azeotrope and will show negative
(c) Solution (ii) will show negative deviation from Raoult's deviation from Raoult's law
law (d) At specific composition methanol-acetone mixture will
(d) Solution (iii) will show positive deviation from Raoult's form minimum boiling azeotrope and will show negative
law deviation from Raoult's law
Solutions 389

26. KH value for Ar (g), CO2 (g), HCHO (g) and CH4 (g) are (b) Molecular mass of X is less than the molecular mass of
40.39, 1.67, 1.83 × 10–5 and 0.413 respectively. Y.
Arrange these gases in the order of their increasing solubility. (c) Y is undergoing dissociation in water while X undergoes
(a) HCHO < CH4 < CO2 < Ar no change.
(b) HCHO < CO2 < CH4 < Ar (d) X is undergoing dissociation in water.
(c) Ar < CO2 < CH4 < HCHO 32. Which one is not equal to zero for an ideal solution: [2015]
(d) Ar < CH4 < CO2 < HCHO (a) DSmix (b) DVmix
NEET/AIPMT (2013-2017) Questions (c) DP = Pobserved - PRaoult (d) DHmix
27. How many grams of concentrated nitric acid solution should 33. What is the mole fraction of the solute in a 1.00 m aqueous
be used to prepare 250 mL of 2.0M HNO3 ? The concentrated solution ? [2015 RS]
acid is 70% HNO3 [2013] (a) 0.177 (b) 1.770
(a) 90.0 g conc. HNO3 (c) 0.0354 (d) 0.0177
(b) 70.0 g conc. HNO3 34. At 100°C the vapour pressure of a solution of 6.5g of a solute
(c) 54.0 g conc. HNO3 in 100 g water is 732 mm. If Kb = 0.52, the boiling point of this
(d) 45.0 g conc. HNO3 solution will be [2016]
28. Which condition is not satisfied by an ideal solution? (a) 101°C (b) 100°C
(a) Dmix H = 0 [NEET Kar. 2013] (c) 102°C (d) 103°C
(b) Dmix V = 0 35. If molality of the dilute solutions is doubled, the value of
(c) Dmix S = 0 molal depression constant (Kf ) will be:- [2017]
(d) Obeyance to Raoult’s Law (a) halved (b) tripled
29. Of the following 0.10 m aqueous solutions, which one will (c) unchanged (d) doubled
exhibit the largest freezing point depression? [2014] 36. Which of the following is dependent on temperature? [2017]
(a) KCl (b) C6H12O6 (a) Molarity (b) Mole fraction
(c) Weight percentage (d) Molality
(c) Al2(SO4)3 (d) K2SO4
37. Which of the following statement about the composition of
30. Which one of the following electrolytes has the same value the vapour over an ideal 1 : 1 molar mixture of benzene and
of van't Hoff's factor (i) as that of the Al2(SO4)3 (if all are toluene is correct? Assume that the temperature is constant
100% ionised) ? [2015] at 25°C. (Given : Vapour Pressure Data at 25°C, benzene
(a) K3[Fe(CN)6] (b) Al(NO3)3 = 12.8 kPa, toluene = 3.85 kPa) [2016]
(a) The vapour will contain a higher percentage of benzene
(c) K4[Fe(CN)6] (d) K2SO4 (b) The vapour will contain a higher percentage of toluene
31. The boiling point of 0.2 mol kg–1 solution of X in water is (c) The vapour will contain equal amounts of benezene
greater than equimolal solution of Y in water. Which one of and toluene
the following statements is true in this case ? [2015] (d) Not enough information is given to make a predication
(a) Molecular mass of X is greater than the molecular mass
of Y.
EBD_7327
390 CHEMISTRY

Hints & Solutions


EXERCISE - 1 9. (b) Relation between molality and mole fraction is
1000 ´ x2 1000 ´ .2
1. (b) Lowering in vapour pressure is the highest for that m= = = 3.2
x1M1 .8 ´ 78
molecules for which Vont-Hoff factor is maximum be-
cause. Now, Von’t Hoff factor is maximum for BaCl2 [As Thus, X (m) = 3.2
number of ions formed here after dissociation in water Wt ´ 1000 7.1 ´ 1000
10. (b) M = ; M= = 0. 5M
is zero], this will have the highest lowering of vapour M.Wt . ´ V 142 ´ 100
pressure. Dp n2 n2 640 - 630 0.0156
2. (b) For an ideal solution, DHmixing = 0 11. (a) = or = =
p ° n1 + n2 n1 + n2 640 1
DH = DH1 + DH2 + DH3 (According to Hess's law)
i.e., for ideal solutions there is no change in magnitude or 1 mole i.e., 78g benzene contains solute = 0.0156 mol
of the attractive forces in the two components present. 0.0156 ´ 103
\ Molality of solution = = 0.2 m
m 1000 78
3. (a) DT = K b . ´
M W Number of moles
12. (c) Molarity =
Volume of solution (L)
1 1000 1 1000
0.25 = K b . ´ = Kb ´ ´
M W 60 W Mass 1000
Moles of water = = = 55.6
Molar mass 18
0.3 1000
DT = K b ´ ´ 55.6
180 W Molarity = = 55
1
DT 3 13. (c)
= ´ 60 = 1 Þ DT = 0.25 °C pV = nRT
0.25 180
nRT 1.75 ´ 0.0821 ´ (273 + 17)
New boiling point of glucose solution = 100.25 °C. p= =
4. (d) V 342 ´150 /1000
5. (b) The molality involves weights of the solute and the [M.W. of sucrose = 342]
solvent. Since weight does not change with the 1.75 1000
temperature, therefore molality does not depend upon =p= ´ ´ 0.0821 ´ 290 = 0.8 atm.
342 150
the temperature.
14. (b) Q 10 g glucose is dissolved in = 100 ml solution.
6. (b) Density = 1.17 gm/cc (Given) \ 180 g (g mole) is dissolved in
Mass
As d < 100
Volume = ´ 180 = 1800 ml = 1.8 L
10
volume = 1cc \ mass = d = 1.17g
No. of moles 1.17 ≥1000 Mass of solute
Molarity < < 15. (d) ppm = ´ 106
Mass of solution
Volume in litre 36.5 ≥1
10
<
1170
=32.05M
\ ppm = ´ 106 = 10 ppm
36.5 106
mole 5 / 40 p° - p
7. (a) Molarity = = 16. (a) = xsolute
litre 250 /1000 p°
Given, p° = 0.8 atm, p = 0.6 atm, xsolute = ?
5 1000
= ´ = 0.5 M 0.8 - 0.6 0.2
40 250 = xsolute or = xsolute ,
0.8 0.8
mole 37 / 36.5 or xsolute = 0.25
8. (b) Molarity = =
litre 100 / 1.18 ´1000
m ´ 1000
[37 gm HCl in 100 gm of solution 17. (c) DT = K b
MW
100 100 ù
= C.C = lú 0.44 ´ 1000
1.18 1.18 ´ 1000 û Þ 0.567 = 5.12 ´
M ´ 22.2
37 1.18 ´ 1000 512 ´ 0.44 ´ 1000
= ´ = 11.96 » 12. ÞM = = 178.9
36.5 100 0.567 ´ 22.2
Solutions 391

5 é ù
18. (b) Molarity = No. of moles of solute = =2M
600
Volume in litres 2.5 ê 600mm = 760 atmosphere ú
ë û
19. (a) Mole fraction of any component A in solution
38 ´ 20 ´ 1000 ´ 0.083 ´ 288
x=
No. of moles of A ÞM =
Total No. of moles of solution
30 ´ 500
As total no. of moles of solution > No. of moles of A = 1211 » 1200
Thus x can never be equal to one or zero. 27. (c) These two components A and B follows the condition of
Raoult’s law if the force of attraction between A and B is
m 1000
20. (d) DT = k b ´ equal to the force of attraction between A and A or B and
M W B.
10 1000 28. (a) Given V.PP = 80 torr
Þ 1 = 2.53 ´ ´ V.PQ = 60 torr
M 100
Ptotal = V·PP × xp + V·Pq × xq
Þ M = 2.53 ´ 100 = 253 g é 3 2ù
1 = ê80 ´ + 60 ´ ú = 16 × 3 + 12 × 2
ë 5 5û
21. (b) One molal solution means one mole of solute is present
Ptotal = 48 + 24 = 72 torr
in 1 kg (1000 g) solvent
29. (d) A solution containing A and B components shows
i.e., mole of solute = 1
negative deviation when A–A and B–B interactions
1000g 1000 are weaker than that of A–B interactions. For such
Mole of solvent (H2O) = =
18g 18 solutions.
DH = –ve and DV = –ve
1 18 30. (b) Chloroform and acetone form a non-ideal solution in
Mole fraction of solute = = = 0.018. which A......B type of interactions are readily seen due
æ 1000 ö 1008
ç1 + ÷ to intensive H-bonding instead of A......A and B......B
è 18 ø
type. Therefore, the solution shows negative deviation
22. (d) from Raoult's law i.e.
23. (d) NV = N1V1 + N2V2 + N3V3
DVmix = - ve ; DH mix = - ve
1 1 1 \ Total volume of solution is less than 80 ml.
or, 1000N = 1 ´ 5 + ´ 20 + ´ 30 or N = .
2 3 40 31. (c) Here, DV = 500 - 300 = 200 cc = 0.2 L
m 1000 p = 0.6 atm
24. (d) DT = k f ´ ´
M W q = 10 J
W = p D v = (0.2 ´ 0.6) = 0.12 Latm
0.1 ´ 1000
= 1.86 ´ = 0.186 Also, 1 L atm = 101.3 J
1000
\ W = 0.12 × 101.3 = 12.156 J
Since NaCl is 100% ionised so Vant Hoff factor = 2 But q = –ve, when heat is liberated
So, DT ' = i ´ DT = 2 ´ 0.186 = – 0.372°C \ DE = q + W = -10 + 12.16 = 2.16 J
25. (a) Given vapour pressure of pure solvent
32. (b) Positive deviations are shown by such solutions in
(P°) = 121.8 mm Hg; Weight of solute (w) = 15 g
which solvent-solvent and solute-solute interactions
Weight of solvent (W) = 250 g; Vapour pressure of
are stronger than the solute-solvent interactions. In
solution (P) = 120.2 mm Hg and Molecular weight of
such solution, the interactions among molecules
solvent (M) = 78
becomes weaker. Therefore their escaping tendency
From Raoult’s law
increases which results in the increase in their partial
Po - P
w M vapour pressures.
= = ´
Po m W In pure methanol th ere exists intermolecular
121.8 - 120.2 15 78 H–bonding.
= ´
121.8 m 250 ---O – H--- O—H --- O—H---
15 ´ 78 121.8 | | |
or m= ´ = 356.2 CH3 CH3 CH3
250 1.6
26. (b) p = CRT On adding benzene, its molecules come between
ethanol molecules there by breaking H-bonds which
600 20 ´ 1000 ´ 0.083 ´ (273 + 15) weaken intermolecular forces. This results in increase
=
760 M ´ 500 in vapour pressure.
EBD_7327
392 CHEMISTRY

43. (a) Freezing point of water is, T = 0°C


33. (b) Moles of glucose = 18 = 0.1 When 342 g of cane sugar (C12H22O11) is dissolved in
180 1000 g of water, the freezing point of water is depressed
by DTf which is given by,
Moles of water = 178.2 = 9.9
18 w sol 1000
DTf = Kfm = K f ´
Total moles = 0.1 + 9.9 = 10 M.W w solvent

pH2O = Mole fraction × Total pressure = 9.9 ´ 760 342 g 1000


= (1.86 °C /mol)× ´
10 (342g / mol) 1000
= 752.4 Torr = 1.86°C.
34. (d) Antifreeze is a water based liquid coolant used in So, freezing point of solution
gasoline and diesel engines. Compounds like methanol, = T – DTf
ethanol, glycol, glycerol are added to the water to = (0 – 1.86) °C
reduce the freezing point of the mixture below the lowest = – 1.86 °C
temperature that the engine is likely to be exposed and 44. (a) The solutions (liquid mixture) which boils at constant
to inhibit corrosion in cooling systems which often temperature and can distil as such without any change
contain a range of electrochemically incompatible in composition are called azeotropes.
metals. If all the mentioned compounds are available at Solution of HNO3 and H2O will form maximum boiling
same price then the compound having lowest molecular point azeotrope. Maximum boiling azeotropes show
weight will be the choice because negative deviation from Raoult’s law.
w 1000 Composition (%) Boiling Point
DTf = iK f ´ HNO3 68.0 359 K
M.W. w solvent H2O 32.0 373 K
So, less quantity of the compound will result in large Boiling point of the azeotrope of these two solutions is
depression in freezing point of antifreeze solution. 393.5 K.
P° - P n2 45. (c) According to Raoult's law
35. (d) =
P° n1 + n2 p° - p
= xB
640 - 600 2.5 x p°
=
640 39 78 é .2 1ù
ê xB = Mole fraction of solute = .2 + .8 = 5 ú
640 ´ 78 ´ 2.5 ê ú
x= = 80 ë p = 60 mm of Hg û
39 ´ 40
36. (c) For ideal solution,
p° - p 1
DVmixing = 0 and DH mixing = 0 . = or 4 p° = ( p) ´ 5
p° 5
37. (d) C2H5I and C2H5OH form non-ideal solution.
38. (d) Gaseous densities of ethanol and dimethyl ether would 60≥5
Þ p° < < 75mm of Hg
be same at same temperature and pressure. The heat 4
of vaporisation, V.P. and b.pts will differ due to 46. (a) When hard boiled egg after shelling is immersed in
H-bonding in ethanol. saturated brine, its size remains same because of the
39. (c) The given vapour pressure graph shows that on mixing cagulation of inner liquid there is no flow of solvent
two components having vapour pressures pA and pB molecules across the membrane.
in any ratio, the total vapour pressure of the solution, 47. (b) According to Raoult's law
shown by the line CD, does not show any deviation
from ideal behaviour. Lines AD and CB are also do not Dp n
= (mole fraction of solute)
show any deviation from their expected behaviour but p° n + N
these lines show the individual beheviour of 10
components A and B. = 0.2 \ p° = 50 mm of Hg

40. (d)
For other solution of same solvent
41. (a) Molal elevation constant is given by the formula,
20 n
DTb = (Mole fraction of solute)
Kb = , where m is molality and DTb is elevation in p° n + N
m
boiling point. 20
Þ = Mole fraction of solute
42. (d) Azeotropic mixture is constant boiling mixture, it is not 50
possible to separate the components of azeotropic Þ Mole fraction of solute = 0.4
mixture by boiling. As mole fraction of solute + mole fraction of solvent = 1
Hence, mole fraction of solvent = 1 – 0.4 = 0.6
Solutions 393

48. (a) A solution of acetone in ethanol shows positive 58. (a) Depression in freezing point µ No. of particles.
deviation from Raoult's law. It is because ethanol (when concentration of different solutions is equal)
molecules are strongly hydrogen bonded. When Al 2 (SO4 )3 provides five ions on ionisation
acetone is added, these molecules break the hydrogen
bonds and ethanol becomes more volatile. Therefore ¾® 2 Al 3+ + 3SO 42 –
Al 2 (SO 4 ) 3 ¾
its vapour pressure is increased. while KCl provides two ions
49. (a) Liquid mixture having a definite composition and boil KCl ¾¾ ↑ K ∗ ∗ Cl,
like a pure liquid is called an azeotrope. C6H12O6 and C12H22O11 are not ionised so they have
Þ Mixture showing +ive deviation from Raoult's Law single particle in solution.
forms minimum boiling azeotrope while mixture showing Hence, Al2(SO4)3 have maximum value of depression
–ive deviation from Raoult's law forms maximum boiling in freezing point or lowest freezing point.
azeotrope. 59. (a) Molecular masses of polymers are best determined by
50. (a) DTf = i × Kf × m osmotic pressure method. Firstly because other
Van't Hoff factor, i = 2 for NaCl, m = 0.01 colligative properties give so low values that they
hence DTf = 0.02 Kf which is maximum in the present cannot be measured accurately and secondly, osmotic
case. pressure measurements can be made at room
Hence DTf is maximum or freezing point is minimum.
temperature and do not require heating which may
51. (c) We know that
change the nature of the polymer.
100 ´ Kf ´ w \ m = 100 ´ Kf ´ w 60. (b) Solvent having high cryoscopic constant (camphor)
DTf =
m´ W DTf ´ W can be used in determination of molecular masses of
Now, Kf = molecular depression constant organic compounds like naphthalene, anthracene etc.,
= 50.2°C per 100 gm by cryoscopic method.
DTf = 0.502°C
61. (c) DTA = Kf mA , DTB = Kf MB
W = 25 gm
100 ´ 50.2 ´ 0.750 DTA = DTB , mA = m B
m= = 300
0.502 ´ 25 Mass of solute A Mass of B
52. (b) density of ethanol = 0.789 gm/ml. =
Molecular mass of A Molecular mass of B
wt. of 1000 ml = volume × density = 789g
Mass of CH3COOH 1000 3 9
\ Molality = Mol. mass of CH COOH ´ wt. of ethanol =
3 60 Molecular mass of B
5 1000 Molecular mass of B = 180.
= ´ = 0.1056 mol/g
60 789
K f ´ W2 ´1000 1 1000
53. (c) Osmotic pressure is a colligative property. 62. (c) DT = K f m = = 5.12 ´ ´ = 0.4K
54. (a) As both the solutions are isotonic hence there is no M 2W1 250 51.2
net movement of the solvent occurs through the 63. (b) Total no. of milli equivalents of acids
semipermeable membrane between two solutions. = (100 × 0.4 + 200 × 0.2) meq.
55. (a) Total wt. of solution = 800 gm. = 40 + 40 = 80 meq.
40 Total volume of mixture = 300
Wt. of solute = ´ 800 = 320 g.
100 \ N × 300 = 80
So, 100 gm solute precipitated 80
Remining total wt. of solute = 320 – 100 g N= = 0.2670 N.
300
= 220 g
Remining total wt. of solution = 800 – 100 64. (c) Colloidal particles exhibit ceaseless chaotic and random
= 700 g motions, such motions of the particles are called
Brownian motion.
220
\ % composition = ´ 100 = 31.43% 65. (d) V1 = 250ml; N1 = 0.4N
700
56. (a) Concentration of particles in CaCl2 solution will be V2 = 1250 ml; N 2 = ?
maximum as i = 3 for CaCl2 and i = 2 for KCl.
Applying the law of equivalence :–
Glucose and Urea do not dissociate into ions, as they
No. of equivalents before dilution = No. of equivalents
are nonelectrolytes.
after dilution
57. (a) K 4 [Fe(CN) 6 ] and Al 2(SO 4 )3 both dissociates to
\ N1V1 = N 2 V2
give 5 ions or i = 5
(0.25) ´ (0.4) = (1.25) ´ N 2
ˆˆ† 4K+ + [Fe(CN)6]4–
K4 [Fe(CN)6] ‡ˆˆ
N 2 = 0.080 N .
ˆˆ† 2Al3∗ ∗ 3SO 4,,
and Al2 (SO4 )3 ‡ˆˆ
EBD_7327
394 CHEMISTRY

+ nRT
66. (d) ( HX H + X - , i = 1.3); DTf = Kf × m × i 75. (b) p= or T = pV
1- 0.3 0. 3 0.3 V nR
DTf = 1.85 × 0.2 × 1.3 = 0.481º C 10% (w/v) aqueous solution of glucose means
\ Tf = T f° - DT f = 0 – 0.481ºC = – 0.481ºC 10 g glucose is present in 100 mL of H2O

67. (c) DTb = Kb ´ m 14 ´ 100 ´ 10 -3 ´ 180


= = 307.3 K
Elevation in boiling point is a colligative property, 0.082 ´ 10
which depends upon the no. of particles (concentration 76. (c) Normality
of solution). Thus greater the number of particles,
Mass 1000
greater is the elevation in boiling point and hence = ´
greater will be its boiling point. Equivalant mass Volume of solution
Na2SO4 ƒ 2Na+ + SO42– 4 1000
Since Na2SO4 has maximum number of particles (c) = ´ =1
40 100
hence has maximum boiling point.
77. (c) Solvent molecules flow from low concentration solution
No. of equivalents to high concentration solution.
68. (b) Normality =
Volume of solution in litre 78. (d) DT f = iK f m
Þ No. of equivalents
® K + + Br -
KBr ¾¾
= Normality × Volume of solution in litre 1 0 0
= 0.1 × 0.1 = 0.01 (1-a) a a
i=1+a
Wt Here a = 0.8 (80% ionization)
Þ = 0.01
Equivalent wt. i = 1.8
DTf = (1.8) (1.86) (0.4) = 1.339
Molecular wt. Tf = – 1.339°C
Þ w t = 0.01´
basicity 79. (a) The solution which provide same number of ions are
isotonic.
200 Ca (NO3)2 ¾¾ ® Ca2+ + 2 NO3–
wt. = 0.01´ = 1gm
2 Total ions produced = 3
No. of particles after ionisation Na2SO4 ¾¾ ® 2 Na+ + SO42–
69. (a) (i) i = Total ions produced = 3
No. of particles before ionisation
\ 0.1 M Ca (NO3)2 and 0.1 M Na2SO4 are isotonic.
(ii) DTb = i × Kb × m 80. (b) KNO3 is a strong electrolyte which dissociates into
CuCl2 ¾
¾® Cu + 2Cl 2+ - two ions. Therefore, its van’t Hoff factor is 2. Acetic
1 0 0 acid (CH3COOH) is a weak electrolyte, it does not
(1-a) a 2a dissociate completely. So, its van’t Hoff factor is less
than that of KNO3
1 + 2a \ Osmotic pressure of 0.1 M KNO3 >
i= , i = 1 + 2a
1 Osmotic pressure of 0.1 M CH3COOH
Assuming 100% ionization or P1 > P2
So, i = 1 + 2 = 3 81. (a) For highest boiling point of a solution, the DTb should
be highest, Since, DTb is directly proportional to the
13.44 molality of solution.
DTb = 3 ´ 0.52 ´ 0.1 = 0.156 » 0.16 [m = = 0.1]
134.4 DTb µ m
70. (b) Benzoic acid exists as dimer in benzene. Of the given solutions urea and glucose remain
71. (b) DT f = K f ´ m = 1.86 ´ 0.5 = 0.93°C; T f = -0.93°C unionized and remain as one particle in solution. In
case of Na2SO4 (a strong electrolyte) there are three
0.1 ´180 ´100 particles for each molecule (2 Na + and one SO42–)
72. (c) Kb = =1 K / m
1.8 ´1000 whereas for KNO3 (strong electrolyte) there are 2
73. (b) Isotonic solutions are those solutions which have same particles (one K+ and one NO3–). Thus for equimolar
osmotic concentrations. solutions of these two, the number of particles is more
The solution with higher osmotic concentration is called in Na2SO4 (aq) and so is molality. Thus it will exhibit
hypertonic and lower is called hypotonic solution. highest boiling point.
74. (d) Boiling temperature is a temperature at which vapour 82. (b) Given mass of solute = 8.1 g
pressure is equal to the atmospheric pressure when Mass of solvent = 100 g
external pressure is equal to 1 atm. For HBr
Solutions 395

® H + + Br -
HBr ¾¾ Þ Boiling pt. of solution (2M sucrose solution)
1 0 0 = DTb + Boiling pt. of pure water
(1-a ) a a
a = 90% = 0.9 = 1.04°C + 100°C = 101.04°C
88. (d) DTf = 0 – (0.00732°C) = 0.00732
i = 1 + a = 1 + 0.9 = 1.9
DTf = i × Kf × m
DT f = Kf × m × i
DT f 0.00732
moles of solute i= = = 1.97 » 2
= 1.86 × × 1.9 K f ´m 1.86 ´ 0.002
mass of solvent in kg
89. (d) When the aqueous solution of one molal KI is diluted
8.1/ 81 with water, concentration decreases, therefore the
= 1.86 × ´ 1.9 vapour pressure of the resulting solution increases.
100 /1000
= 1.86 × 1 × 1.9 = 3.534K 1000W2 1.86×1000×68.5
90. (a) DT f = K f = = 0.372
T f = T f ° - DT f M 2W1 342×1000
or Tf = 0 – 3.534 K Tf = T °f –DTf
\ Tf = – 3.534K Tf = – 0.372°C
83. (d) Osmotic pressure is a colligative property. Hence 25.3 1000
resulting osmotic pressure of the solution is given by 91. (b) Concentration of Na 2CO3 = ´ = 0.955 M
106 250
pT = p1 + p2 + p3 ..........
pT = 1.64 + 2.46 = 4.10 atm. [Na + ] = 2 × 0.955 = 1.91 M
84. (d) Addition of solute to water decreases the freezing point
of water (pure solvent). é CO32– ù = 0.955 M
ë û
\ When 1% lead nitrate (solute) is added to water, the 92. (b) Given Kf = – 1.86ºC m–1, mass of solute = 5.00 g,
freezing point of water will be below 0°C. mass of solvent = 45.0 g and DTf = – 3.82°C
85. (b) As DTf = Kf. m DTf = i × Kf .m
DTb = Kb. m 5 ´ 1000
3.82 = i × 1.86 ×
DT f DTb 142 ´ 45
Hence, we have m = =
Kf Kb \ i = 2.62 (Molecular mass of Na2SO4 = 142 g)
93. (c) If compound dissociates in solvent i > 1 and on
Kf association i < 1.
or DT f = DTb
Kb 94. (b) 1 molal solution means 1 mole of solute dissolved in
1000 gm solvent.
Þ [DTb = 100.18 - 100 = 0.18°C] \ nsolute = 1 wsolvent = 1000 gm
1.86 1000
= 0.18 × = 0.654°C \ nsolvent = = 55.56
0.512 18
As the Freezing Point of pure water is 0°C,
1
DTf = 0 –Tf xsolute = = 0.0177
0.654 = 0 – Tf 1 + 55.56
\ Tf = – 0.654 95. (d) Given a = 30% i.e., 0.3
Thus the freezing point of solution will be – 0.654°C. HA ¾¾ ® H+ + A–
86. (c) Applying the law of equivalence, 1–a a a
1 – 0.3 0.3 0.3
N1V1 + N 2V2 + N3V3 = N RVR
i = 1 – 0.3 + 0.3 + 0.3
N N N i = 1.3
´ 50 + ´ 30 + ´ 10 = N R ´ 1000
10 3 2 DTf = 1.3 × 1.86 × 0.1 = 0.2418
5 N + 10 N + 5 N = 1000 ´ N R Tf = 0 – 0.2418 = – 0.2418 °C
N WRT 1.26 ´ 0.083 ´ 300
Þ NR = 96. (d) M = =
50 pV 2.57 ´10-3 ´ 200 ´ 10-3
87. (a) Given : Kb = 0.52, molarity = 2m
Here, molality = molarity = 2 molal 31.374 ´ 10 6
= = 61038 g
Now, using DTb = Kb × m, we get 514
DTb = 0.52 × 2 = 1.04 °C 97. (d) p = pAxA + pBxB
Now, DTb = pAxA + pB (1 – xA)
= Boiling pt. of solution – Boiling pt. of Þ pAxA + pB – pBxA
solvent (pure water) Þ pB+ xA (pA – pB)
EBD_7327
396 CHEMISTRY

98. (b) pA° = ? , Given p B° = 200 mm of Hg , xA = 0.6, 8. (c)


Molarity
Molality (m)
xB = 1 – 0.6 = 0.4, P = 290 of Hg Molarity ´ Molecular mass
Density -
1000
P = PA + PB = PA°x A + PB°xB
18
= = 5000
Þ 290 = PA° × 0.6 + 200 × 0.4 \ p°A = 350 mm of Hg. 1.8 -
18 ´ 98
99. (b) For this solution intermolecular interactions between 1000
n-heptane and ethanol are weaker than w
9. (b) pV = RT
n-heptane-n-heptane & ethanol-ethanol interactions m
hence the solution of n-heptane and ethanol is non- 4
ideal and shows positive deviation from Raoult’s law. \ 6 ´ 10-4 ´1 = ´ 0.0821´ 300 ;m = 1.64 ´105.
m
100. (a) From molarity equation : 10. (b) pV = CRT
M1V1 + M2V2 = M × V
p1 C1T1
M V + M 2V2 =
M= 1 1 where V = total volume p 2 C2T2
V
C
750 ´ 0.5 + 250 ´ 2 p1 = P, p 2 = 2 atm. C1 = C , C2 =
= 2
1000 T1 = 600 K, T2 = 700 K
= 0.875 M
P 2 ´ C ´ 600
=
EXERCISE - 2 2 C ´ 700
24
P= .
5 5 7
1. (c) Moles of urea = ; moles of fructose = ; 11. (a) CaCl 2 acts as a non-volatile solute and results in
60 180
depression in freezing point. Thus, snow fall is reduced
5 and prevents blocking of roads in the polar region.
moles of sucrose = ;
342 12. (c) Na2SO4 ¾¾ ® 2Na+ + SO 24-
Mol. before dissociation 1 0 0
5 5
moles of KCl(effective) = 2 ´ = Mol. after dissociation 1 – a 2a 1a
74.5 37.25
i = 1 - a + 2a + a = 1 + 2a
2. (b) Using the relation,
13. (b) AlCl3 furnishes more ions than CaCl 2 and thus possess
w.R.T 6 ´ 0.080 ´ 300 higher boiling point i.e., T1 > T2.
M = , we get M = = 3.6 × 105
pV -3 14. (a) Van’t Hoff equation is
2 ´ 10 ´ 0.2
3. (d) The salt that ionises to least extent will have highest pV = inRT
freezing point. [ i.e., minimum D Tf ] For depression in freezing point.
D Tf = i × Kf × m
4. (b) Greater is the effective molarity (i × C), higher the DTf
For elevation in boiling point.
value and lower the freezing point.
D Tb = i × Kb × m
5. (c) N1V1 + N2V2 = NV
For lowering of vapour pressure,
4x + 10 (1 – x) = 6 × 1; –6x = –4 ; x = 0.67
Thus 0.67 litre of 4N HCl °
Psolvent - Psolution æ n ö
= iç .
è N + n ÷ø
1 – x = 1 – 0.67 = 0.33 litre of 10 N HCl P°
solvent
% ´ 10 ´ d 22 ´ 10 ´ 1.253 15. (b) DTf = i.Kf .m ; 0.0054 = i × 1.8 × 0.001
6. (a) Molarity = = = 0.805M.
GMM 342 i = 3 so it is [Pt(NH3)4Cl2]Cl2.
% ´ 10 ´ d 22 ´ 10 ´ 1.253 16. (c) Given, W = 500 cc = 500 g
Normality = = = 4.83N
GEM 342 / 6 (Q density of water = 1g/cc)
22 ´ 1000 Weight
Molality = = 0.825m As, Density =
342(100 - 22) Volume
80 1g
Weight = Density × Volume = ´ 500cc = 500g
68 18 cc
7. (b) Mole fraction of H2O = = .
80 20 77 w = amount of urea = ?
+
18 34 Kf = 1.86°C; DTf = 0.186°C
M = Molecular mass of urea = 60
Solutions 397

1000 ´ K f ´ w 22. (d) At 1 atmospheric pressure the boiling point of mixture


M = is 80°C.
W ´ DT f At boiling point the vapour pressure of mixture, pT = 1
atmosphere = 760 mm Hg.
1000 ´ 1.86 ´ w
60 = Using the relation,
500 ´ 0.186
60 = 20 × w pT = p A° x A + p B° xB , we get
pT = 520 x A + 1000(1 - x A )
60
\ w = = 3g
{Q pAo = 520 mm Hg ,
20
17. (b) Among the given options, CaCl 2 solution will produce
pB° = 1000 mm Hg , x A + xB = 1 }
maximum (three) ions per molecule, so it will show
minimum freezing point. or 760 = 520 x A + 1000 - 1000 x A or 480 x A = 240
w 1000 240 1
18. (d) DTb = Kb ´ or x A = = or 50 mol. percent
M W 480 2
3 1000 i.e., the correct answer is (d)
0.52 = 0 .6 ´ ´ (W = 200 ´ 1)) 23. (a) The vapour pressure of a solution of glucose in water
M 200
can be calculated using the relation
1.8 ´ 5
M= = 17.3g mol -1 p° - ps Moles of glucose in solution
0.52 =
ps Moles of water in solution
19. (d) The value of ΧTb depends upon two factors 'i ' and
'm'. It is given that 1 mol of each compound is dissolved 17.5 - ps 18/180
in 1 L of solution. Hence molarity is same for all the or = [Q p° = 17.5 ]
ps 178.2/18
compounds. Now the van't Hoff factor depends on
number of particle i.e. on degree of ionisation which 0.1´ ps
further depends on the bond dissociation energy which or 17.5 – ps = or ps = 17.325 mm Hg.
is in the order 9.9
Hence (a) is correct answer.
HI < HBr < HCl < HF
i.e., bond dissociation energy of HI is least. Lower the 24. (b) p = pA° xA + pB° xB
total
bond dissociation energy, higher is the degree of 1 3
ionisation and hence higher the number of particles, 550 = pA° ´ + pB° ´
4 4
thus i will be maximum for HI and hence ΧTb value will pA° + 3 pB° = 550 ´ 4 ...(i)
be larger for HI. In second case
20. (d) Let the mass of methane and oxygen = m gm. 1 4
Mole fraction of O2 ptotal = pA° ´ + pB° ´
5 5
Moles of O2 pA° + 4 pB° = 560 ´ 5 ...(ii)
=
Moles of O 2 + Moles of CH 4 Subtract (i) from (ii)
m / 32 m / 32 1 \ pB° = 560 ´ 5 - 550 ´ 4 = 600
= = =
m / 32 + m /16 3m / 32 3 Q p° = 400
A
Partial pressure of O2 = Total pressure × mole fraction
25. (a) p Total = p °A x A + p °B x B
1 1
of O2 , PO2 = P × = P = p°Heptane xHeptane + p°Octane x Octane
3 3
21. (a) Osmotic pressure (p) of isotonic solutions are equal.
25 /100 35 /114
For solution of unknown substance C1(concentration). = 105 ´ 25 35 + 45 ´ 25 35
5.25 / M + +
C1 = 100 114 100 114
V
0.25 0.3
Where M represents molar mass. = 105 ´ + 45 ´
0.25 + 0.3 0.25 + 0.3
1.5 / 60
For solution of urea, C2 (concentration) = 105 ´ 0.25 45 ´ 0.3 26.25 + 13.5
V = + =
0.55 0.55 0.55
Given, p1 = p2 Q p = CRT = 72 kPa
5.25 / M 1.5 / 60 26. (b) For isotonic solutions
\ C1RT = C2RT or C1 = C2 or =
V V p1 = p 2
\ M = 210 g/mol
EBD_7327
398 CHEMISTRY

C1 = C2 31. (a) DTf = i × Kf × m


Where m = Molality of the solution
5 / 342 1/ M (i.e. number of moles of solute per 1000 g of the solvent)
=
0.1 0.1 0.1
Here m = ´10
5 1 329
=
342 M 0.1´10
Thus DTf = 4 × 1.86 × = 2.3 × 10–2
329
342
Þ M = = 68.4 gm/mol Thus Tf = 0 – 2.3 × 10–2 = – 2.3 × 10–2 ºC
5
32. (a) From Raoult law
27. (b) DTf = i × Kf × m
Given DTf = 2.8, Kf = 1.86 K kg mol–1 i = 1 p° – p No.of moles of solute
=
(ethylene glygol is a non- electrolyte) p° No. of moles of solvent+ No. of moles of solute
wt. of solvent = 1 kg When the concentration of solute is much lower than
Let of wt of solute = x the concentration of solvent,
Mol. wt of ethylene glycol = 62
p° - p No. of moles of solute
x =
2.8 = 1 × 1.86 × p° No. of moles of solvent
62 ´ 1
DTb = Kb × m
2.8 ´ 62 Number of moles of the solute
or x = = 93 gm m= ´ 1000
1.86 Mass of solvent in grams
28. (a) Molecular weight of naphthoic acid
Number of moles of the solute
C11H8O2 = 172 g mol–1. DTb = K b ´ × 1000
The theoretical value of depression in freezing point Mass of solvent in grams
Number of moles of solute
20×1000
= Kf × molality = 1.72× = 4K DT ´ Mass of solvent in grams
172×50 = b
K b ´ 1000
Van't Hoff factor, 2 ´100
= = 0.26,
Observed value of colligative property 0.76 ´1000
i= 100
Theoretical value of colligative property Number of moles of solvent = = 5.56
18
2 From equation (i) we get, 760 - p = 0.26
= = 0.5 . 760 5.56
4 On solving, p = 724.46 » 724
29. (a) PN 2 = k H x N 2 33. (b) Sodium sulphate dissociates as
0.8 × 5 = 1 × 105 × xN Na 2SO 4 ¾¾ ® 2Na + + SO -4 -
2
hence van’t hoff factor i = 3
\ x N = 4 × 10–5
2 Now DT f = i K f .m
Solubility in 10 moles = 4 × 10–4.
= 3 × 1.86 × 0.01 = 0.0558 K
120 34. (d) Colligative properties depends upon the no. of particles.
30. (c) Number of moles of urea = =2
60 Since methanol is non electrolyte hence cannot be
Total mass of solution = 1000 + 120 = 1120 g considered.
Mass 35. (a) kH = 100 kbar = 105 bar, p = 1 bar
Total volume of solution (in L) = p = k H × xA
Density
p 1
1120 112 xA = = = 10-5
k H 100 ´ 10 3
= = L
3 115
1.15 ´ 10 1000
Moles of water = = 55.5
Number of moles 18
Molarity of the solution = Weight of water = 1000 g (Q 1000 mL = 1000 g)
Volume of solution in litre
x
2×115 Mole fraction = 10–5 =
= = 2.05 mol L–1 55.5 + x
112 As 55.5 >>> x, thus neglecting x from denominator
Solutions 399

x M2 = molar mass of solute


10 -5 = Þ x = 55.5 ´ 10 -5 moles w1 = mass of the solvent
55.5
or 0.555 millimoles. 5.12 ´ 1000 ´ 1
On substituting given values, DT f =
36. (d) DTb = kb.m 51.2 ´ 250
weight of solute ´ 1000 \ DTf = 0.4K
DTb = k b ´ 41. (c) Acetone and chloroform shows negative deviation from
molecular weight of solute ´
weight of solvent Raoult's law when these are mixed, the hydrogen
bonding takes place between the two molecular species
1.5 1000
0.75 = 2.5 ´ ´ due to which escaping tendency of either liquid
M 50 molecules becomes less and boiling point of solution
2.5 ´ 1.5 ´ 1000 increases.
M= = 100
0.75 ´ 50 42. (c)
37. (b ) DTf = Kf × m 43. (d) According to Henry’s law the mass of a gas dissolved
Kf is a characteristic of a particular solvent i.e., it will be per unit volume of solvent is proportional to the
different for different solvents. pressure of the gas at constant temperature m = K p i.e.
38. (a) According to Raoult’s law, as the solubility increases, value of Henry’s law
PT = xAp°A + xBp°B constant decreases. Since CO2 is most soluble in water
Given, = PT1 500 mm Hg among the given set of gases. Therefore CO2 has the
nA = 1 and nB = 2 \ xA = 1/3 and xB = 2/3 lowest value of Henry’s law constant.
44. (a) Given w = 10 g Mol. mass = 40
1 2
Þ 500 = p° + p ° Weight of solvent = 1250 × 0.8 g = 10000 g = 1 kg
3 A 3 B
10
\ molality = = 0.25
Þ 1500 = p°A + 2p°B ... (i) 40 ´ 1
Also given Qn that one more mole of B is added to the 45. (d) Glucose is non electrolyte hence depression in freezing
solution, the pressure of the ideal solution increases point will be minimum, hence freezing point will be
by 25 mm Hg. highest.
46. (b)
\ PT2 = 500 + 25 = 525 mm Hg 47. (b) Both assertion and reason are correct but reason is not
Also, nB = 3 \ xA = 1/4 and xB = 3/4 the correct explanation of assertion.
1 3 The relationship between lowering of vapour pressure
525 = p°A = p°B ... (ii) and osmotic pressure can be derived as follows:
4 4
Van’t Hoff equation for dilute solutions is
2100 = PA° + 3PB° n
Subtract (i) and (ii), p = RT ....(i)
V
p° = 600 mm Hg In case of a dilute solution, the volume of solution can
B
p° + 2p° = 1500 Þ p° = 300 mm Hg. be taken as equal to that of solvent. If N is the number
A B A of moles of solvent of molecular weight M and density
39. (d) According to Henry’s law, r , the volume V is given by
m=k×p
given KH = 1.4 × 10–3 NM
V= ....(ii)
pO2 = 0.5 or r
n pM
p O2 = K H ´ x O2 or = ....(iii)
N r.R.T .
0.5 From Raoult’s law,
\ x O2 =
1.4 ´ 10- 3 P° – P n
= ....(iv)
m P° N
No. of moles; n = P° – P pM
M \ =
P° r RT [From (iii) and (iv)]
-4 m
0.7 ´ 10 =
32 pM
or ( P° – P ) = r RT ´ P°
m = 22.4 ´ 10-4 g = 2.24 mg
40. (d) Mass of non-volatile solute = 1g MP °
Molar mass of solute = 250g mol–1 The factor r RT is constant at constant temperature.
Mass of benzene = 51.2g, Kf = 5.12 K kg mol–1
\ ( P° – P ) µ p
K ´ 1000 ´ w 2
DTf = f or lowering of V.P. µ osmotic pressure
M 2 ´ w1 Thus assertion is correct.
where, w2= mass of the solute Osmotic pressure is a colligative property is correct.
EBD_7327
400 CHEMISTRY

48. (c) Both the solute and solvent will form the vapours but unit of DTb K
vapour phase will become richer in the more volatile Unit of Kb = unit of m = molality
component.
K
49. (b) = = K mol-1 kg
-1
mol kg
50. (c) Camphor has high molal depression constant.
11. (c) DTf = iKf m i.e., depression in freezing point is directly
EXERCISE - 3 related to van't Hoff factor (i). Glucose is a non-
electrolyte hence will remain undissociated. Hence for
Exemplar Questions glucose value of i is 1 whereas for MgCl2 value of i is 3.
1. (a) According to Henry's law partial pressure of a gas in Hence, depression in freezing point of MgCl2 is about
the solution is proportional to the mole fraction of gas 3 times of glucose.
in the solution. 12. (d) When an unripe mango is placed in a concentrated salt
p = KHx ; KH = (Henry's constant) solution to prepare pickle then mango loose water due
2. (d) Dissolution of sugar in water will be most rapid when to osmosis and get shrivel.
powdered sugar is dissolved in hot water because in 13. (a) Osmotic pressure (p) = CRT.
powdered form it can easily insert in the vacancies of For concentrated solution C has higher value than
water particle. dilute solution.
Also, dissolution of sugar in water is an endothermic Hence, as concentration of solution increases osmotic
process. So it is favourable at high temperature. pressure also increase.
3. (c) 14. (a) The value of molal depression constant, Kf depends
4. (b) When solute gets precipitated in the solution, then the upon nature of solvent. Therefore two different
solution is known as supersaturated solution. solutions of sucrose of same molality prepared in
5. (c) Maximum amount of solid that can be dissolved in a different solvents will have different depression in
specified amount of a given solvent does not depend freezing point.
upon pressure. The reason is solid and liquid are highly 15. (b) Number of total ions present in the solution is known
incompressible and practically remain unaffected by as van't Hoff factors (i).
change in pressure. for KCl, i= 2
6. (b) At high altitude, the partial pressure of oxygen is less for NaCl, i= 2
than at the ground level. This decreased atmospheric for K2SO4, i= 3
pressure causes release of oxygen from blood. Hence 16. (b) In reverse osmosis, solvent molecules move through
people living at high altitude have low concentration a semipermeable membrane from a region of higher
of oxygen in the blood and tissues. concentration of solute to lower concentration.
7. (a) Mixture of methanol and acetone show a positive 17. (a) The value of Henry's constant (KH) increases with
deviation from Raoult’s law. Molecules in pure increase in temperature.
methanol are hydrogen bonded. On adding acetone, 18. (b) According to Henry's law
its molecules enters in between the host molecules and pµx
break some of the hydrogen bonds between them. Þ p = KHx
Therefore, the intermolecular attractive forces between As value of KH rises solubility of gases decreases.
the solute-solvent molecules are weaker than those 19. (b) If a pressure higher than the osmotic pressure is applied
between the solute-solute and solvent-solvent on the solution, the solvent will flow from the solution
molecules. into the pure solvent through the semi-permeable
Other three remaining options will show negative membrane. This process is called reverse osmosis.
deviation. Thus, in this case, if a pressure greater than osmotic
8. (b) Colligative properties depend upon number of solute pressure is applied on piston (B). Water will move from
particles in solution irrespective of their nature. side (B) to side (A).
9. (b) Colligative properties depends upon the number of 20. (c) As van’t hoff factor depends only on no. of dissociated
particles. As we know greater the value of van't Hoff ions. Hence, i will be independent on concentration of
factor higher will be elevation in boiling point and hence solutions.
higher will be the boiling point of solution. Among the 21. (b) A mixture of bromoethane and chloroethane is an
given solution 1.0M Na 2 SO4 have highest no. of example of ideal solution. For an ideal solution, the
particles i.e highest value of ‘i’. A — A or B — B type intermolecular interaction is
Hence, 1.0 M Na2SO4 has highest value of boiling nearly equal to A — B type interaction.
point. Chloroform and acetone mixture is an example of
10. (a) DTb = Kbm elevation in boiling point non-ideal solution having negative deviation while
ethanol-acetone mixture shows positive deviation.
DTb
Kb = 22. (a) On adding salt to water to make the salt solution the
m vapour pressure of solution gets lowered. This is due
Solutions 401

to the decrease in surface covered by solvent molecule n


which leads to decrease in number of solvent molecule < (where n solute = W/M)
V(kg)
escaping from the surface corresponding to pure
solvent. n
1.00 m < i.e., 1 mole in 1 kg of water
Hence, vapour pressure also get reduces. V(kg)
23. (a) If two liquids A and B form minimum boiling azeotrope Moles of 1 kg H2O
at some specific composition then A — B interactions 1000g
are weaker than those of A — A and B — B. Because in < < 55.55 mole
18g / mol
case of positive deviation, we get minimum boiling
azeotropes whereas in case of negative deviation we Moles of solute = 1
get maximum boiling azeotropes. Mole fraction
24. (d) As we know, M1V1 = M2V2 nsolute 1
< <
On putting values, we get n solute ∗ n water (1 ∗ 55.55)
0.02 × 4 L = M2 × 5L
0.08 = 0.01768 = 0.0177
M2 = = 0.016 M
5 æ P°- Ps ö n W1 M 2
25. (a) At specific composition methanol-acetone mixture will 34. (a) çç ÷÷ = = ´
è P° ø N M1 W2
form minimum boiling azeotrope and will show positive
deviation. This is due to weaker A — B interaction Where, W1 = wt of solute
than A — A and B — B interaction. W2 = wt of solvent
26. (c) Value of KH depends upon nature of gases dissolved M1 = Mass of solute
in water. Higher the value of KH at a given temperature
M2 = Mass of solvent
the lower is the solubility of the gas in the liquid.
Hence, correct order is : at 100°C, P° = 760 mm
Ar < CO2 < CH4 < HCHO.
760 - 732 6.5 ´18
NEET/AIPMT (2013-2017) Questions =
760 M1 ´ 100
wt ´ 1000
27. (d) Molarity (M) = M1 = 31.75 g mol–1
mol. wt. ´ vol (ml)
wt. 1000 W1 ´1000
2= × DTb = m × Kb = M ´ W × Kb
63 250
1 2
63
wt. = gm 0.52 ´ 6.5 ´1000
2
100 DTb = = 1.06°C
wt. of 70% acid = ´ 31.5 = 45 gm 31.75 ´100
70 \ boiling point of solution
28. (c) An ideal solution is that solution in which each
= 100°C + 1.06°C = 101°C
component obeys Raoult’s law under all conditions of
temperatures and concentrations. For an ideal solution. 35. (c) Kf (molal depression constant) only depends on the
DHmix = 0 and DVmix = 0. nature of the solvent and is independent of the
concentration of the solution.
29. (c) Colligative properties µ no. of particles. Since
Al2(SO4)3 contains maximum number of particles, hence 36. (a) Molarity depends on the volume of a solution which
will have the largest value of freezing point depression. can be changed with change in temperature.
37. (a) Let us consider that A is benzene and B is toluene
30. ¾¾
(c) K4[Fe(CN)6] ¬¾
¾® 4K+ + [Fe(CN)6]– 1 : 1 molar mixture of A and B
and Al2(SO4)3 ® 2Al3+ + 3SO2– 1 1
4
\ xA = and xB =
\ van’t Hoff factor is 5 for both Al2 (SO4)3 and K4[Fe(CN)6] 2 2
31. (d) DTb = iKb m Total pressure of solution (P) = PA0 x A +PB0 x B
Given, (DTb)x > (DTb)y 1 1
\ ix Kb m > iyKb m P = 12.8 × + 3.85 × = 8.325 kPa
2 2
(Kb is same for same solvent) 1
ix > iy 0 12.8´
YA = PA x A = 2 = 0.768
So, x is undergoing dissociation in water. P 8.325
32. (a) For an ideal solution DSmix > 0 \ yB = 1 – yA = 1 – 0.768 = 0.232
W 1 so, the vapour will contain higher percentage of
33. (d) Molality < ≥
M V(kg) benzene.
EBD_7327
402 CHEMISTRY

17 Electrochemistry
Electrochemistry is that branch of chemistry which deals with the Note:
relationship between electrical energy and chemical changes taking (i) Electrode on which oxidation occurs is called anode (negative
place in redox reactions i.e., how chemical energy produced in a pole) while the electrode on which reduction occurs is called
redox reaction can be converted into electrical energy or how cathode (positive pole)
electrical energy can be used to bring about a redox reaction which (ii) Electrons flow from anode to cathode in the external circuit.
is otherwise non-spontaneous. (iii) Inner circuit is completed by flow of ions through the salt
Under electrochemistry, we study two types of cells: bridge.
electro chemical and electrolytic cell. (iv) Representation of electrochemical cell is like:
Difference Between Electrochemical and Electrolytic
Electrons flow
Cell: Left electrode Right electrode
Electrochemical cell Electrolytic cell
Metal Metal ion (conc) Metal ion (conc) Metal
1. It converts chemical 1. It converts electrical energy
energy into electrical into chemical energy. Oxidation occurs Salt Reduction occurs
energy 2. The redox reaction is non- Anode bridge cathode
2. It is based upon redox spontan eous and takes negative pole positive pole
reaction which is place only when electrical
spontaneous. energy is supplied. Characteristics of Salt Bridge
3. The electrode on which 3. The electrode which is (i) It allows the flow of current by completing the circuit.
oxidation takes place is connected to negative (ii) It maintains electrical neutrality of electrolytes in two half
called Anode (negative terminal of the battery is cells.
pole) and electrode on called cathode; the cations
which reduction takes migrate to it, gain electrons ELECTROMOTIVE FORCE (EMF)
place is called cathode and hence reduction takes The difference in the electrode potentials of two electrodes of the
(positive pole). place. The other electrode cell is termed as electromotive force (EMF).
is called anode. Cell potential or EMF of the cell is given by:
4. A salt bridge is used to set 4. No salt bridge is used in this Ecell = Eright – Eleft = ER – EL
up the cell. case. = Ecathode – Eanode = Ered – Eox
5. Two electrolytes are used. 5. Only one electrolyte is Conventionally, all electrode potentials are expressed at top as
taken. reduction potentials, and
GALVANIC CELL Reduction potential = – Oxidation potential
It is also called voltaic cell. It converts chemical energy liberated For a redox reaction to be spontaneous , EMF of the cell must be
during redox reaction to electrical energy. positive.
For example reaction between Zn and CuSO4. Reduction occurs at the electrode having higher reduction
potential and oxidation occurs at the electrode having lower
Zn ( s ) + Cu 2 + ( aq ) ¾¾
® Zn 2 + ( aq ) + Cu ( s ) reduction potential.
The chemical reaction responsible for production of electricity Electrode potential is a fixed quantity and is not multiplied by ‘n’
takes place in two separate compartments called half cells which i.e. number of e–1s involved in a reaction.
consists of a electrolyte solution and a electrode. Standard electrode potential for fluorine is the highest indicating
Anode: Zn ® Zn2+ + 2e– that fluorine gas (F2) has the maximum tendency to get reduced to
(Ist half cell reaction) F– ions and thus, F2 gas is the strongest oxidising agent. Lithium
Cathode: Cu2+ + 2e– ® Cu has lowest electrode potential indicating that Li+ ion is the weakest
(IInd half cell reaction) oxidising agent while lithium metal is the most powerful reducing
Two half cells are connected by salt bridge. agent in an aqueous solution.
Electrochemistry 403

Note : Galvanic cell has an external voltage < 1.1.V when Eext = On passing electric current, positively charged ions move towards
1.1V, the reaction stops and no current flows. And, if Eext > 1.1V, cathode and negatively charged ions move towards anode; where
the reaction occurs in opposite direction and the cell functions as they loose their charge and become neutral species. Oxidation
an electrolytic cell where the electrons flow from Cu to Zn and occurs at anode while reduction takes place at cathode.
current flows from Zn to Cu. Faraday’s Laws
NERNST EQUATION The quantitative aspects of electrolysis were studied by Faraday.
He put forward two laws as follows:
The relationship between the concentration of ions and electrode
(i) Faraday’s First law of Electrolysis:
potential is given by Nernst equation.
According to this law, the amount of chemical reaction and
(i) For electrode potential, the reaction is,
hence the mass of any substance deposited at any electrode
M n + + ne- ¾¾®M during electrolysis by a current is directly proportional to the
Then Nernst equation, is applied as quantity of electricity passed through the electrolyte i.e.,
mµQ
RT [M]
E = E° – ln m = ZQ
nF [M n + ] as, Q = It
for pure solids liquids or gases at 1 atm pressure [M] = 1 \ m = ZIt
where m ® amount of subs. deposited
RT [M] Q ® quantity of electricity in ‘C’
E=E°– ln
nF [M n + ] Z ® Constant called Electrochemical equivalent
I ® Current in ampere
putting R = 8.314 Jk–1 mol–1
t ® time in seconds
F = 96500C
Electrochemical equivalent can be defined as the mass of
T = 298 K
the substance deposited by one coulomb of charge or by
0.0591 1 one ampere of current passed for one second.
E Re d = E° Re d – log
n éMn+ ù equivalent wt.of element
ë û Electro-chemical equivalent (Z) =
96500
(ii) For EMF of the cell, the cell reaction is:
(ii) Faraday’s second law of electrolysis
aA + bB xX + yY When same amount of electricity is passed through solutions
Then by Nernst equation at 298 K, of different electrolytes connected in series, the weight of
x y the substances produced at the electrodes are directly
E cell = E°cell -
0.0591
log
[X ] [ Y ] proportional to their equivalent weights.
n [ A ]a [ B]b Ex: for AgNO3 and CuSO4 solutions connected in series, if
same quantity of electricity is passed, then
Where n is the number of electrons involved in the cell
reaction. wt. ofAg deposited Eq. wt. of Ag
=
(iii) For a cell reaction in equilibrium, Wt. of Cu deposited Eq. wt. of Cu
Ecell = 0 Note:
(i) If n electrons are involved in the electrode reaction, the
[X]x [Y]y
KC = passage of n faradays (i.e., nF = n × 96500 C) of electricity will
[A]a [B]b liberate one mole of the substance.
(ii) Equivalent weight of a substance can be calculated as:
0.0591
E°cell = log K c at 298 K m é m ® amount of substance deposited ù
n Eq. wt. = ´ 96500 ê ú
where n = no. of electrons involved in the cell reaction Q ëQ ® Quantity of electricity û
KC = equilibrium constant for the cell reaction. CONDUCTANCE OF ELECTROLYTIC SOLUTIONS
Gibb’s Free Energy and Cell Potential
Conductance (G)
DG° = – nFE°cell
The amount of electric current that can be passed through the
RT solution is called conductance. Conductance is inverse of
as E°cell = ln K C
nF resistance.
DG° = – 2.303 RT log KC 1 1
Conductance = or G =
where DG° = standard free energy change of the reaction Resistance R
F = Faraday’s constant = 96500 C Unit of conductance is inverse of ohm. It is represented as mho or
R = 8.314 JK–1mol–1 ohm–1 or siemens (s).
ELECTROLYSIS (i) Resistance of a conductor is given as follows:
It is a process of decomposition of an electrolyte by the passage l
R=r
of electricity through its aqueous solution of molten state. A
EBD_7327
404 CHEMISTRY

R = resistance; r = resistivity; l = length, Effect of Dilution on Conductance, Specific


A = area of cross-section.
Conductivity and Lm
(ii) The reciprocal of resistivity is called conductivity
(i) Conductance increases with increase in dilution due to
1 1 l increase in number of ions in solution.
k= = k ® specific conductivity or conductivity.
r RA (ii) Conductivity decreases with the decrease in concentration.
l
This is because conductivity is the conductance of one unit
or k = G ´ = observed conductance × cell constant volume of solution. As the number of ions per unit volume
A
decrease on dilution, hence conductivity decreases with
= G × G* (where cell constant = l/A)
decrease in concentration.
k Conductivity (iii) Molar conductivity and equivalent conductivity increases
\ Cell constant (G*) = =
G Conductance with decrease in concentration. This is because L m = k V
Equivalent Conductivity
and Leq = k V. Though ‘ k ’ decreases with decrease in
If one equivalent of an electrolyte is dissolved in a solution, then
the conductivity of two parallel electrodes situated 1 cm apart is concentration but it is compensated by increase in volume

called equivalent conductivity. It is denoted by Leq . and hence L m and Leq increase with dilution.

Leq = k × V (where V = volume) The variation of L m for strong and weak electrolytes with
In terms of concentration, concentration is as follows:

k ´ 1000 k ´ 1000
Leq = =
Ceq Normality
Where Ceq = Concentration
Unit of Leq = cm2 ohm–1 gm eq–1
Lm
Molar Conductivity
If one mole of an electrolyte is dissolved in a solution, then the Strong electolyte
conductivity of that solution between two parallel electrodes
situated 1 cm apart, is called molar conductivity. It is represented Weak electrolyte
by lm.
lm = k × Vm 1
where Vm = Volume of the solution in which 1 mole solute is C2
æ 1ö
dissolved. çè Vm = ÷ø The increase in L m for strong electrolyte with dilution is
C
k = specific conductivity not very sharp. This is because strong electrolyte is already
completely dissociated into ions. On dilution, ions get
1000 cm3
lm = ×k separated and their movement becomes easier and hence,
M
or Unit = cm mol ohm–1 = S cm2 mol–1
2 –1 L m increases but not sharply.
Unit of Different Quantities On the other hand, weak electrolyte has lower degree of
dissociation at higher concentrations. On dilution, the degree
Commonly used of dissociation increases and number of ions in the solution
Physical Quantity SI units
units increases. Thus, L m increases sharply.
1. Resistance(R) ohm ( W) ohm ( W)
KOHLRAUSCH’S LAW
2. Resistivity ( r) ohm cm ( W cm ) ohm m ( W m )
According to this law, limiting molar conductivity of an electrolyte
3. Conductance ( G ) W -1 siemen ( S) is the sum of the limiting ionic conductivities of the cation and the
anion each multiplied with the number of ions present in one formula
4. Conductivity ( k ) W-1cm -1 S m -1
unit of the electrolyte.
5. Equivalent
W -1cm 2 eq -1 S m2eq -1 Mathematically, Lom for AxBy = xl°y+(A) + yl°x–(B)
( )
Conductivity L eq
where,
6. Molar
W -1cm 2mol -1 S cm 2mol -1 Lom = Limiting molar conductivity of the electrolyte
Conductivity ( L m )
l°y+(A) and l°x–(B) = Limiting molar conductivities of cation and
( )
7. Cell constant G* cm -1 m -1 anion respectively.
Electrochemistry 405

Applications of Kohlrausch’s Law Dry cell or Leclanche cell : Particulars are:


Anode - Zinc Container
(i) Calculation of Lom for weak electrolytes.
Cathode - graphite rod surrounded by MnO2 powder
l° (CH3COOH) = l°CH3COO¯ + l ° + .......(1) Electrolyte - paste of NH4Cl + ZnCl2
H
for strong electrolytes : Cathode Reaction : MnO2 + NH +4 + e– ¾¾® MnO(OH) + NH3
lº (CH3COOK) = l º CH3COO¯ + l ° + .....(2)
K Anode Reaction : Zn – 2e - ¾
¾® Zn + +
lº (HCl) = l ° + + lºCl– .....(3)
H Zn 2 + + 2 NH 3 ¾
¾®[ Zn ( NH 3 ) 2 ] 2 +
lº (KCl) = l ° + + l ° .....(4) Cell potential 1.25V to 1.5V
K Cl -
\ eqn (2) + eqn (3) – eq (4) = eqn (1)
n Mercury Cell
¥
+ l¥ ¥ ¥ Anode - Zn-Hg amalgam
i.e., l (CH3COOK) (HCl) – l (KCl) = l (CH3COOH)
Cathode - paste of (HgO + C)
(ii) Calculation of degree of dissociation (a):
Electrolyte - moist paste of KOH-ZnO
It L cm is the molar conductivity of solution at any Cathode Reaction :
concentration C and Lom is the molar conductivity at infinite HgO(s) + H 2 O(l ) + 2e - ¾¾
® Hg(l) + 2OH -
dilution the degree of dissociation will be:
Anode Reaction : Zn + 2OH - ¾¾
® ZnO + H 2 O( l) + 2e -
Lc
a= m Net Reaction : Zn(Hg) + HgO(s) ¾¾
® ZnO(s) + Hg(l)
L °m
(iii) Calculation of dissociation constant of a weak electrolyte cell potential = 1.30 volt

Ca 2 Secondary Batteries or Cells


i.e., K c = These can be recharged by passing current in opposite direction
1- a
where C is concentration and a is degree of dissociation of so that they can be used again.
weak electrolyte. Examples: (i) Lead storage battery and Nickel cadmium storage
(iv) Calculation of solubility of a sparingly soluble salt: Salts cell.
which dissolve to a very small extent in water are called Lead storage battery :
sparingly soluble salts. Their solutions are considered as Anode - Spongy lead
infinite dilute solutions as they dissolve in very little amount. Cathode - grid of lead packed with PbO2
Their solubility is given as: Electrolyte - 38% H2SO4 by mass
k ´ 1000
Solubility = Anode Reaction : Pb + SO -4 - ¾
¾® PbSO 4 + 2e -
L om
Cathode Reaction :
PREDICTING THE PRODUCTS OF ELECTROLYSIS PbO 2 + SO 4- - + 4 H + + 2e - ¾
¾® PbSO 4 + 2H 2 O
In electrolysis, the conversion of ions into neutral species at their Net Reaction :
respective electrodes is called primary change. The product formed
as a result of primary change may be collected as such or it may Pb + PbO 2 + 4H + + 2SO 4- - ¾
¾® 2PbSO 4 + 2H 2 O
undergo secondary change to form final products.
When recharged the cell reactions are reversed.
When electrolysis is carried out in molten state, the products are
actually the substances obtained from the ions of the electrolyte. Nickel-cadmium storage cell
However, in aqueous solution, the situation is different. At each Anode - Cadmium
electrode, two different ions can be discharged which ion gets Cathode - metal grid containing NiO2
discharged depends: Electrolyte - KOH solution
(i) On their relative discharge potentials Anode Reaction Cd ( s ) + 2OH - ® Cd(OH) 2 ( s ) + 2e -
(ii) Sometimes on the material of the electrode used.
Usually, the ions with lower discharge potential are Cathode Reaction :
discharged in preference to those which have high discharge
NiO2 (s ) + 2H 2 O ( l ) + 2e- ® Ni ( OH )2 (s ) + 2OH - ( aq )
potentials.
SOME COMMERCIAL CELLS (Batteries) Net Reaction :
They are mainly of two types: Cd ( s ) + NiO2 ( s ) + 2H 2 O ( l )
Primary Batteries or Cells ® Cd(OH)2 ( s ) + Ni(OH) 2 ( s )
Primary cells are those in which redox reaction occurs only once
Cell potential = 1.4V.
and the cell becomes dead after some time and cannot be used
again. Examples:
EBD_7327
406 CHEMISTRY

Fuel Cells The reactions involved in the rusting of iron are:


Galvanic cells which are designed to convert energy of combustion Cathode: O2(g) + 4H+ (aq) + 4e– ® 2H2O (l)
of fuels like H2, CH4, CH3OH, etc. directly into electrical energy E° = 1.23V
are called fuel cells. Ex: H2 – O2 fuel cell. H + / O2 /H2O

H2O Anode: 2Fe(s) ® 2Fe2+ + 4e– E° = -0.44V


Cathode (+) Fe2+ /Fe
Anode (–) Overall reaction: 2Fe(s) + O2(g) + 4H+ (aq) ®
Porous carbon
Porous carbon cathode containing 2Fe2+ (aq) + 2H2O (l)
anode containing suitable catalyst The ferrous ions are further oxidised by atmospheric oxygen to
suitable catalyst ferric ions which come out as rust in form of hydrated ferric oxide
Conc. (Fe2O3.xH2O).
KOH/NaOH 2Fe2+ + ½O2 + H2O ® Fe2O3 + 4H+
Fe2O3 + xH2O ® Fe2O3 .xH2O
H2 O2 Rust
Factors Which Promote Corrosion
(i) More active metals are readily corroded.
Fig : H2 – O2 fuel cell (ii) Presence of impurities enhance corrosion. Pure metals do
not corrode.
The electrode reactions are: (iii) Air and moisture accelerate corrosion. In vacuum, there is no
cathode: O2 (g) + 2H2O (l) + 4e– ® 4OH– (aq) corrosion.
anode: 2H2(g) + 4OH– (aq) ® 4H2O(l) + 4e– (iv) Corrosion occurs rapidly at bands, scratches, nicks and cuts
Overall reaction: in the metal.
2H2(g) + O2(g) ® 2H2O (l) (v) Electrolytes also increase the rate of corrosion.
Advantages of fuel cells
Prevention of Corrosion
(i) The cell runs continuously as long as the reactants are
supplied. (i) Painting, oiling, greasing (i.e., barrier protection) prevent
(ii) Their efficiency is about 70% compared to thermal plants corrosion
whose efficiency is 40%. (ii) Covering with a layer of more reactive metal (called sacrificial
(iii) They are pollution free. protection). For ex: iron is coated with more active zinc. The
process is called Galvanisation.
CORROSION (iii) Coating of iron surface with phosphate or other chemicals
It is the slow eating away of the metal due to attack of the which forms a protective, tough, adherent in soluble film.
atmospheric gases on the surface of the metal resulting into the Note:
formation of compounds such as oxides, sulphides, carbonates, (i) Iron in contact with a solution whose pH is above 9-10 does
sulphates etc. not corrode. This is because H+ ions are involved in the
Examples: rusting of iron, tarnishing of silver, green coating on reduction of O2. As concentration of H+ decreases, i.e., pH is
silver and bronze, etc. increased, the reduction of O2 becomes less favourable.
Water (ii) If water is saline, it helps in the flow of current in the miniature
O2(g) cell and hence enhances the process of corrosion.
Droplet
Rust (iii) Rust is formed at a site other than the site where pitting has
Fe2O 3. xH2O occurred.
Anode (iv) Iron corrodes more rapidly then it does in the absence of
Cathode
copper and tin. This is because when iron is coated with
2+ –
e– Fe Fe +2e copper or tin; if the coating is broken, iron is exposed and it
+ –
being more reactive than both copper and tin, gets corroded.
O2+4H +4e 2H2O
+ –
O2+2H2O +4e 4OH¯
Electrochemistry 407

CONCEPT MAP
EBD_7327
408 CHEMISTRY

1. The amount of chlorine evolved by passing 2A of current in 11. A 0.5 M NaOH solution offers a resistance of 31.6 ohm in a
aq. solution of NaCl for 30 minutes is : conductivity cell at room temperature. What shall be the
(a) 0.84 gm (b) 1.32 gm approximate molar conductance of this NaOH solution if cell
(c) 3.34 gm (d) 5.47 gm constant of the cell is 0.367 cm–1 .
2. The conjugate acid of NH3 is : (a) 234 S cm2 mole–1 (b) 23.2 S cm2 mole–1
2
(c) 4645 S cm mole –1 (d) 5464 S cm2 mole–1
(a) NH +4 (b) NH2OH
12. For the electrochemical cell,
(c) NH3 (d) N2H4
3. If 0.01 M solution of an electrolyte has a resistance of 40 M | [M + || X – | X, E°(M + /M) = 0.44 V an d
ohms in a cell having a cell constant of 0.4 cm–1, then its E° (X/X–) =0.33 V. From this data one can deduce that:
molar conductance in ohm–1 cm2 mol–1 is (a) M + X ¾¾ ® M + + X– is spontaneous reaction
(a) 102 (b) 104 (b) M + X ¾¾
+ – ® M + X is spontaneous reaction
(c) 10 (d) 103 (c) Ecell = 0.77V
4. Specific conductance of a 0.1 N KCl solution at 23ºC is 0.012 (d) Ecell = – 0.77V
ohm–1 cm–1. Resistance of cell containing the solution at 13. If 0.5 amp current is passed through acidified silver nitrate
same temperature was found to be 55 ohm. The cell constant solution for 100 minutes. The mass of silver deposited on
is cathode, is (eq.wt.of silver nitrate = 108)
(a) 0.0616 cm–1 (b) 0.66 cm–1 (a) 2.3523 g (b) 3.3575 g
(c) 6.60 cm –1 (d) 660 cm–1 (c) 5.3578 g (d) 6.3575 g
5. The highest electrical conductivity of the following aqueous 14. Using same quantity of current, which among Na, Mg, and
solutions is of Al is deposited to the highest extent during electrolysis.
(a) 0.1 M difluoroacetic acid (b) 0.1 M fluoroacetic acid (a) Mg (b) Al
(c) 0.1 M chloroacetic acid (d) 0.1 M acetic acid (c) Na (d) All in same amount
6. When an electric current of 0.5 ampere is passed through 15. Aluminium oxide may be electrolysed at 1000°C to furnish
acidulated water for two hours, then volume of hydrogen aluminium metal (At. Mass = 27 amu; 1 Faraday = 96,500
(H2) at NTP produced will be (1 Coulomb of electricity
deposits 0.00001 gm of hydrogen) Coulombs). The cathode reaction is– Al
3+
+ 3e - ® Al
(a) 0.1 litre (b) 0.6 litre To prepare 5.12 kg of aluminium metal by this method we
(c) 0.4 litre (d) 0.8 litre require electricity of
Electrolyte: KCl KNO 3 HCl NaOAc NaCl (a) 5.49 × 101 C (b) 5.49 × 10 4 C
7.
L ¥ (S cm mol ) : 149.9
2 –1
145 426.2 91 126.5
(c) 1.83 × 10 7 C (d) 5.49 × 10 7 C
Calculate L¥
HOAc using appropriate molar conductances of 16. When electric current is passed through acidified water,
the electrolytes listed above at infinite dilution in H 2 O at 112 ml of hydrogen gas at STP collected at the cathode in
965 seconds. The current passed in amperes is
25°C
(a) 1.0 (b) 0.5
(a) 217.5 (b) 390.7
(c) 0.1 (d) 2.0
(c) 552.7 (d) 517.2
17. An electrolytic cell contains a solution of Ag2SO4 and has
8. The unit of equivalent conductivity is
platinum electrodes. A current is passed until 1.6 gm of O2
(a) ohm cm
has been liberated at anode. The amount of silver deposited
(b) ohm–1 cm2 (g equivalent)–1
at cathode would be
(c) ohm cm2 (g equivalent)
(a) 107.88 gm (b) 1.6 gm
(d) S cm–2
(c) 0.8 gm (d) 21.60 gm
9. The resistance of 0.01 N solution of an electrolyte was found
18. Which of the following will form a cell with the highest
to be 220 ohm at 298 K using a conductivity cell with a cell
voltage?
constant of 0.88cm–1. The value of equivalent conductance
(a) 0.1 M Ag+, 2 MCo2+ (b) 2 M Ag+, 0.1 MCo2+
of solution is – + 2+
(c) 1 M Ag , 1 M Co (d) 2 M Ag+, 2 M Co2+
(a) 400 mho cm2 g eq–1 (b) 295 mho cm2 g eq–1
(c) 419 mho cm g eq2 –1 (d) 425 mho cm2 g eq–1 19. When 9650 coulombs of electricity is passed through a
solution of copper sulphate, the amount of copper deposited
10. How much chlorine will be liberated on passing one ampere
is (given at. wt. of Cu = 63.6)
current for 30 min. through NaCl solution?
(a) 0.66 mole (b) 0.33 mole (a) 0318g (b) 3.18 g
(c) 0.66 g (d) 0.33 g (c) 31.8g (d) 63.6g
Electrochemistry 409

20. Find the charge in coulombs required to convert 0.2 mole 31. On passing C ampere of electricity through a electrolyte
VO3–2 into VO4–3 – solution for t second. m gram metal deposits on cathode.
(a) 1.93 × 104 (b) 9.65 × 104 The equivalent weight E of the metal is
(c) 1.93 × 105 (d) 9.65 × 105 C ´t C´m
(a) E= (b) E =
21. Iron pipes, lying in acidic soil, are often attached to the m ´ 96500 t ´ 96500
blocks of magnesium for their protection from rusting, 96500 ´ m C ´ t ´ 96500
because magnesium : (c) E= (d) E =
C ´t m
(a) is lighter than iron 32. What is the time (in sec) required for depositing all the silver
(b) is readily converted into positive ion present in 125 mL of 1 M AgNO3 solution by passing a
(c) forms a corrosion-resistant alloy with iron current of 241.25 A? (1 F = 96500 C)
(d) prevents air from reaching the surface of iron (a) 10 (b) 50
(c) 1000 (d) 100
22. A silver cup is plated with silver by passing 965 coulombs of
33. The number of electrons passing per second through a cross-
electricity. The amount of Ag deposited is : section of copper wire carrying 10–6 amperes of current per
(a) 107.89 g (b) 9.89 g second is found to be
(c) 1.0002 g (d) 1.08 g (a) 1.6 × 10–19 (b) 6 × 10–35
23. Total charge on 1 mole of a monovalent metal ion is equal to (c) 6 × 10–16 (d) 6 × 1012
(a) 9.65 × 104 coulombs (b) 1.6 × 10–19 coulombs 34. What is the Eºcell for the reaction
(c) 6.2 × 1018 coulombs (d) None of these ˆˆ† Cu(s ) + Sn 4+ (aq )
Cu 2+ ( aq ) + Sn 2+ (aq ) + ‡ˆˆ
24. On passing current through two cells, connected in series
at 25ºC if the equilibrium constant for the reaction is 1 × 106 ?
containing solution of AgNO3 and CuSO4, 0.18 g of Ag is (a) 0.5328 V (b) 0.3552 V
deposited. The amount of the Cu deposited is: (c) 0.1773 V (d) 0.7104 V
(a) 0.529 g (b) 10.623 g 35. Which of the following reaction is possible at anode?
(c) 0.0529 g (d) 1.2708 g (a) 2Cr3+ + 7H2O ® Cr2O72– + 14H+
25. How many atoms of calcium will be deposited from a solution (b) F2 ® 2F –
of CaCl2 by a current of 25 mA flowing for 60 seconds? (c) (1/2) O2 + 2H+ ® H2O
(a) 4.68 × 1018 (b) 4.68 × 1015 (d) None of these.
(c) 4.68 × 10 12 (d) 4.68 × 109 36. In electrolysis of dilute H2SO4, what is liberated at anode?
26. To deposit one equivalent weight of silver at cathode, the (a) H2 (b) SO 24 -
charge required will be (c) SO2 (d) O2
(a) 9.65 × 104 C (b) 9.65 × 103 C 37. The reference electrode is made by using
(c) 9.65 × 10 C5 (d) 9.65 × 107 C (a) ZnCl2 (b) CuSO4
27. In the electrolysis of water, one faraday of electrical energy (c) HgCl2 (d) Hg2Cl2
38. The standard hydrogen electrode potential is zero, because
would liberate
(a) hydrogen oxidized easily
(a) one mole of oxygen (b) electrode potential is considered as zero
(b) one gram atom of oxygen (c) hydrogen atom has only one electron
(c) 8 g oxygen (d) hydrogen is a very light element
(d) 22.4 lit. of oxygen 39. The element that is easiest to be reduced:
(a) Ag (b) Fe
28. When an acid cell is charged, then :
(c) Cu (d) Sn
(a) voltage of cell increases 40. Which one is not called a anode reaction from the following
(b) electrolyte of cell dilutes 1
- -
(c) resistance of cell increases (a) Cl ® Cl2 + e (b) Cu ® Cu ++ + 2e -
2
(d) none of the above (c) Hg+ ® Hg++ + e- (d) Zn 2+ + 2e -® Zn
29. The volume of oxygen gas liberated at NTP by passing a 41. The chemical reaction,
current of 9650 coulombs through acidified water is :
2AgCl(s) + H 2(g) ¾¾
® 2HCl(aq) + 2Ag(s)
(a) 1.12 litre (b) 2.24 litre
(c) 11.2 litre (d) 22.4 litre taking place in a galvanic cell is represented by the notation
30. Three faradays electricity was passed through an aqueous (a) Pt (s) | H 2(g) ,1 bar |1M KCl(aq) | AgCl(s) | Ag(s)
solution of iron (II) bromide. The weight of iron metal (b) Pt (s) | H2(g) ,1 bar | 1M HCl(aq) | 1MAg +(aq) | Ag (s)
(at. wt = 65) deposited at the cathode (in gm) is
(c) Pt (s) | H 2(g) ,1 bar | 1M HCl(aq) | AgCl (s) | Ag (s)
(a) 56 (b) 84
(c) 112 (d) 168 (d) Pt (s) | H 2(g) ,1 bar | 1M HCl(aq) | Ag (s) | AgCl(s)
EBD_7327
410 CHEMISTRY

42. The cell reaction Cu + 2Ag+ ® Cu +2 + Ag is best 51. Cu + (aq) is unstable in solution and undergoes
represented by simultaneous oxidation and reduction according to the
(a) Cu(s ) | Cu +2 (aq) | | Ag + (aq) | Ag( s ) reaction :

Pt | Cu +2 | | Ag + ( aq ) | Ag( s) 2Cu +(aq) Cu 2+ (aq) + Cu( s ) choose correct Eº for


(b)
given reaction if Eº Cu2+/Cu = 0.34 V and Eº Cu2+/Cu+ = 0.15
(c) Cu +2 | Cu | | Pt | Ag V
(d) None of the above representations (a) –0.38 V (b) +0.49 V
43. The resistance of 1 N solution of acetic acid is 250 ohm, (c) +0.38 V (d) –0.19 V
when measured in a cell of cell constant 1.15 cm–1. The 52. In the silver plating of copper, K[Ag(CN)2] is used instead
equivalent conductance (in ohm–1 cm2 equiv–1) of 1 N acetic of AgNO3. The reason is
acid will be (a) a thin layer of Ag is formed on Cu
(a) 4.6 (b) 9.2 (b) more voltage is required
(c) 18.4 (d) 0.023 (c) Ag+ ions are completely removed from solution
(d) less availability of Ag+ ions, as Cu cannot displace Ag
44. Specific conductance of 0.1 M HNO3 is 6.3×10–2. The molar
from [Ag(CN)2]– ion
conductance of the solution is
53. On passing electric current of one ampere for 16 min and 5
(a) 100 ohm–1 cm2 (b) 515 ohm–1 cm2
–1 2 sec through one litre solution of CuCl2, all copper of solution
(c) 630 ohm cm (d) 6300 ohm–1 cm2
was deposited at cathode. The strength of CuCl2 solution
45. Which of the following statements about galvanic cell is was (molar mass of Cu = 63.5, Faraday constant = 96500 C/
incorrect mol):
(a) anode is positive (a) 0.2 N (b) 0.01 N
(b) oxidation occurs at the electrode with lower reduction (c) 0.1 N (d) 0.02 N
potential 54. The standard e.m.f. of a galvanic cell involving cell reaction
(c) cathode is positive with n = 2 is found to be 0.295 V at 25°C. The equilibrium
(d) reduction occurs at cathode constant of the reaction would be (Given F = 96500 C mol –1;
46. Which of the following reactions occurs at the cathode? R = 8.314JK–1mol–1)
(a) Sn2+ ¾¾
® Sn4+ + 2 e– (a) 2.0 ´ 1011 (b) 4.0 ´ 1012
(b) Ag+ + e– ¾¾
® Ag (c) 1.0 ´ 10 2 (d) 1.0 ´ 1010
(c) Zn ¾¾
® Zn2+ + 2 e– 55. What will be the emf for the given cell
Pt | H2 (P1) | H+ (aq) | | H2 (P2) | Pt
1
(d) 2OH– ¾¾
® H2O + O + 2e– RT P RT P1
2 2 (a) log e 1 (b) 2 F log e P
47. At 298K the standard free energy of formation of H2O (l) is F P2 2
–237.20 kJ/mole while that of its ionisation into H+ ion RT P2
and hydroxyl ions is 80 kJ/mole, then the emf of the following (c) log e (d) None of these.
F P1
cell at 298 K will be [Take Faraday constant F = 96500C]
H2 (g , 1 bar) | H+ (1M) | | OH– (1M) | O2(g, 1 bar) 56. In a cell that utilises the reaction
(a) 0.40V (b) 0.81V Zn( s) + 2H + (aq) ® Zn 2+ (aq) + H 2 ( g ) addition of H2SO4
(c) 1.23 V (d) – 0.40 V to cathode compartment, will
48. If the following half cells have E° values as (a) increase the E and shift equilibrium to the right
A3+ + e– ––––® A2+, E° = y2V (b) lower the E and shift equilibrium to the right
A2+ + 2e– ––––® A, E° = –y1V (c) lower the E and shift equlibrium to the left
The E° of the half cell A3+ + 3e ––––® A will be (d) increase the E and shift equilibrium to the left
2 y1 - y2 y - 2 y1 57. The same amount of electricity was passed through two
(a) (b) 2 cells containing molten Al2O3 and molten NaCl. If 1.8 g of Al
3 3
were liberated in one cell, the amount of Na liberated in other
(c) 2y1 – 3y2 (d) y2 – 2y1 cell is:
49. To deposit 0.634 g of copper by electrolysis of aqueous (a) 2.8 g (b) 3.2 g
cupric sulphate solution, the amount of electricity required (c) 4.6 g (d) 6.8 g
(in coulombs) is 58. According to Nernst equation, which is not correct if
(a) 1930 (b) 3960 Q = Kc :
(c) 4825 (d) 9650 RT
50. Without losing its concentration ZnCl2 solution cannot be (a) Ecell = 0 (b) ln Q = E ocell
nF
kept in contact with
nFEocell
(a) Au (b) Al
(c) Pb (d) Ag (c) Kc = e RT (d) E cell = Eocell
Electrochemistry 411

59. The volume of oxygen gas liberated at NTP by passing a 69. Following cell has EMF 0.7995V.
current of 9650 coulombs through acidified water is : Pt | H2 (1 atm) | HNO3 (1M) || AgNO3 (1M) | Ag
(a) 1.12 litre (b) 2.24 litre If we add enough KCl to the Ag cell so that the final Cl– is
(c) 11.2 litre (d) 22.4 litre 1M. Now the measured emf of the cell is 0.222V. The K sp of
60. The emf of the cell AgCl would be –
Ni | Ni2+ (1.0 M)| | Au3+ (1.0M)| Au is (a) 1 × 10–9.8 (b) 1 × 10–19.6
[Given E°Ni2+ / Ni = – 0.25 V and E°Au3+/ Au = + 1.5 V] (c) 2 × 10 –10 (d) 2.64 × 10–14
(a) 2.00 V (b) 1.25 V
RT
(c) – 1.25 V (d) 1.75 V 70. E° = In K eq
61. What is the e.m.f for the given cell ? nF
The above equation is called :
Cr | Cr 3+ (1.0M ) || Co 2+ (1.0M) | Co (a) Gibb’s equation
(b) Gibb’s–Helmholtz equation
(E° for Cr 3+ / Cr = -0.74 volt and E° for
(c) Nernst equation
Co 2+ / Co= - 0.28 volt) (d) Van der Waal’s equation
(a) – 0.46 volt (b) – 1.02 volt 71. Standard cell voltage for the cell Pb | Pb2+ || Sn2+ | Sn is
(c) + 0.46 volt (d) 1.66 volt – 0.01 V. If the cell is to exhibit Ecell = 0, the value of
62. If a salt bridge is removed between the two half cells, the [Sn 2+] / [Pb2+] should be antilog of –
voltage (a) + 0.3 (b) 0.5
(a) Drops to zero (b) Does not change (c) 1.5 (d) – 0.5
(c) Increase gradually (d) Increases rapidly 72. An electrochemical cell is set up as: Pt; H2 (1atm)|HCl(0.1 M)
63. The standard emf of a cell, involving one electron change is || CH3COOH (0.1 M)| H2 (1atm); Pt. The e.m.f of this cell will
found to be 0.591 V at 25°C. The equilibrium constant of the not be zero, because
reaction is (F = 96500 C mol–1) (a) the temperature is constant
(a) 1.0 × 101 (b) 1.0 × 105 (b) e.m.f depends on molarities of acids used
(c) 1.0 × 10 10 (d) 1.0 ×1030 (c) acids used in two compartments are different
64. Given the electrode potentials (d) pH of 0.1 M HCl and 0.1 M CH 3COOH is not same
Fe3+ + e– ¾¾ ® Fe2+, E° = 0.771 volts 73. The quantity of electricity needed to liberate 0.5 gram
I2 + 2e ¾¾
– ® 2I–, E° = 0.536 volts equivalent of an element is
E°cell for the cell reaction (a) 48250 faraday (b) 48250 coulomb
2Fe3+ + 2I– ¾¾ ® 2Fe2+ + I2, is (c) 19300 faraday (d) 19300 coulomb
(a) 1.006 V (b) 0.503 V 74. For the cell reaction,
(c) 0.235 V (d) –0.235 V Cu2+ (C1, aq) + Zn(s) ® Zn2+ (C2, aq) + Cu(s) of an
65. For the galvanic cell electrochemical cell, the change in free energy, DG, at a given
temperature is a function of
Zn | Zn2+ (0.1M) || Cu2+ (1.0M)|Cu the cell potential increase
(a) ln (C1) (b) ln (C2/C1)
if:
(c) ln (C2) (d) ln (C1 + C2)
(a) [Zn2+] is increased
75. The most durable metal plating on iron to protect against
(b) [Cu2+] is increased
corrosion is
(c) [Cu2+] is decreased
(a) nickel plating (b) copper plating
(d) surface area of anode is increased
(c) tin plating (d) zinc plating
66. Adding powered lead and iron to a solution that is 1.0 M in
76. A certain current liberated 0.504 gm of hydrogen in 2 hrs.
both Pb2+ and Fe2+ ions, would result in a reaction, in which:
How many grams of copper can be liberated by the same
(a) More iron and Pb2+ ions are formed current flowing for the same time in a copper sulphate
(b) More lead and Fe2+ ions are formed solution ?
(c) Concentration of both Pb2+and Fe2+ ion increases (a) 12.7 gms (b) 15.9 gms
(d) There is no net change (c) 31.8 gms (d) 63.5 gms
67. The standard cell potential of Zn | Zn 2+ ( aq ) | | Cu 2+ ( aq ) | Cu 77. Several blocks of magnesium are fixed to the bottom of a ship to
cell is 1.10 V. The maximum work obtained by this cell will be (a) make the ship lighter
(a) 106.15 kJ (b) –212.30 kJ (b) prevent action of water and salt
(c) –318.45 kJ (d) – 424.60 kJ (c) prevent puncturing by under-sea rocks
68. A hydrogen electrode is immersed in a solution with (d) keep away the sharks
pH = 0 (HCl). By how much will the potential (reduction) 78. In a hydrogen-oxygen fuel cell, combustion of hydrogen
change if an equivalent amount of NaOH is added to the occurs to
(a) produce high purity water
solution. (Take pH2 = 1 atm), T = 298 K.
(b) create potential difference between two electrodes
(a) increase by 0.41 V (b) increase by 59 mV (c) generate heat
(c) decrease by 0.41 V (d) decrease by 59 mV (d) remove adsorbed oxygen from elctrode surfaces
EBD_7327
412 CHEMISTRY

79. During the charging of lead storage battery, the reaction at (c) infinite dilution, each ion makes definite contribution
anode is represented by : to conductance of an electrolyte whatever be the nature
(a) Pb2 + + SO24 - ¾¾
® PbSO 4 of the other ion of the electrolyte.
(d) infinite dilution, each ion makes definite contribution
(b) ® PbO 2 + SO 42 - + 4H + + 2e -
PbSO 4 + 2H 2 O ¾¾ to equivalent conductance of an electrolyte, whatever
(c) ® Pb 2 + + 2e -
Pb ¾¾ be the nature of the other ion of the electrolyte.
88. Standard free energies of formation (in kJ/mol) at 298 K are
(d) Pb2 + + 2e - ¾¾ ® Pb – 237.2, – 394.4 and – 8.2 for H2O(l), CO2(g) and pentane (g),
80. Two electrolytic cells, one containing acidified ferrous respectively. The value E°cell for the pentane-oxygen fuel
chloride and another acidified ferric chloride, are connected cell is :
in series. The ratio of iron deposited at cathodes in the two (a) 1.968 V (b) 2.0968 V
cells will be : (c) 1.0968 V (d) 0.0968 V
(a) 3 : 1 (b) 2 : 1
89. Given:
(c) 1 : 1 (d) 3 : 2
(i) Cu2+ + 2e– ® Cu, Eo = 0.337 V
81. Which colourless gas evolves, when NH4Cl reacts with zinc
(ii) Cu2+ + e– ® Cu+, Eo = 0.153 V
in a dry cell battery
(a) NH4 (b) N2 Electrode potential, Eo for the reaction,
(c) H2 (d) Cl2 Cu + + e– ® Cu, will be :
82. When a lead storage battery is discharged (a) 0.90 V (b) 0.30 V
(a) SO2 is evolved (c) 0.38 V (d) 0.52 V
(b) Lead sulphate is consumed 90. Al2O3 is reduced by electrolysis at low potentials and high
(c) Lead is formed currents. If 4.0 × 104 amperes of current is passed through
(d) Sulphuric acid is consumed molten Al2O3 for 6 hours, what mass of aluminium is
produced? (Assume 100% current efficiency. At. mass of
83. Zn 2+ ® Zn (s) ; E 0 =- 0.76 V Al = 27 g mol–1)
(a) 8.1 × 104 g (b) 2.4 × 105 g
Cu 2+ ® Cu (s) ; E 0 =- 0.34 V 4
(c) 1.3 × 10 g (d) 9.0 × 103 g
Which of the following is spontaneous? M
91. The equivalent conductance of solution of a weak
(a) Zn 2+ + Cu ® Zn + Cu 2 + 32
monobasic acid is 8.0 mho cm2 and at infinite dilution is 400
(b) Cu 2 + + Zn ® Cu + Zn 2 + mho cm2. The dissociation constant of this acid is:
(c) Zn 2+ + Cu 2+ ® Zn + Cu (a) 1.25 × 10–6 (b) 6.25 × 10–4
(c) 1.25 × 10 –4 (d) 1.25 × 10–5
(d) None of these
84. The efficiency of a fuel cell is given by 92. For the reduction of silver ions with copper metal, the
DG standard cell potential was found to be + 0.46 V at 25°C.
(a) DG (b) The value of standard Gibbs energy, DG° will be
DS DH
DS DH (F = 96500 C mol–1 )
(c) (d)
DG DG (a) – 89.0 kJ (b) – 89.0 J
85. The equilibrium constant of the reaction: (c) – 44.5 kJ (d) – 98.0 kJ
93. An increase in equivalent conductance of a strong electrolyte
ˆˆ† Cu 2+ (aq) + 2Ag(s) ;
Cu(s ) + 2Ag+ (aq) ‡ˆˆ
with dilution is mainly due to:
E° = 0.46 V at 298 K is
(a) increase in ionic mobility of ions
(a) 2.0 × 1010 (b) 4.0 × 1010
(b) 100% ionisation of electrolyte at normal dilution
(c) 4.0 × 1015 (d) 2.4 × 1010
86. On the basis of the following E° values, the strongest (c) increase in both i.e. number of ions and ionic mobility
oxidizing agent is : of ions
[Fe(CN)6]4– ®[Fe(CN)6]3– + e– ; E° = – 0.35 V (d) increase in number of ions
Fe2+ ® Fe3+ + e–; E° = – 0.77 V 94. Which of the following expressions correctly represents the
(a) [Fe(CN)6]4– (b) Fe2+ equivalent conductance at infinite dilution of Al2(SO4)3,
(c) Fe3+ (d) [Fe(CN)6]3–
87. Kohlrausch’s law states that at : Given that L °Al3+ and L°SO2- are the equivalent
4
(a) finite dilution, each ion makes definite contribution to conductances at infinite dilution of the respective ions?
equivalent conductance of an electrolyte, whatever be
1 ° 1
the nature of the other ion of the electrolyte. (a) L 3+ + L° 2- (b) 2L°Al3+ + 3LSO ° 2-
3 Al 2 SO4 4
(b) infinite dilution each ion makes definite contribution to
equivalent conductance of an electrolyte depending æ ° 2- ö ´ 6
(c) L°Al3+ + LSO
° 2-
(d) èL°Al3+ + LSO
on the nature of the other ion of the electrolyte. 4 4 ø
Electrochemistry 413

95. Consider the following relations for emf of a electrochemical (a) 0.500 V (b) 0.325 V
cell: (c) 0.650 V (d) 0.150 V
(i) emf of cell = (Oxidation potential of anode) – 98. Standard electrode potential for Sn4+ / Sn2+ couple is + 0.15 V
(Reduction potential of cathode) and that for the Cr3+ / Cr couple is – 0.74 V. These two couples
(ii) emf of cell = (Oxidation potential of anode) + (Reduction in their standard state are connected to make a cell. The cell
potential of cathode) potential will be :
(iii) emf of cell = (Reduction potential of anode) + (Reduction (a) + 1.19 V (b) + 0.89 V
potential of cathode) (c) + 0.18 V (d) + 1.83 V
(iv) emf of cell = (Oxidation potential of anode) – (Oxidation 99. If the E°cell for a given reaction has a negative value, then
potential of cathode) which of the following gives the correct relationships for
the values of DG° and Keq ?
Which of the above relations are correct?
(a) DG° > 0 ; Keq > 1 (b) DG° < 0 ; Keq > 1
(a) (ii) and (iv) (b) (iii) and (i)
(c) DG° < 0 ; Keq < 1 (d) DG° > 0 ; Keq < 1
(c) (i) and (ii) (d) (iii) and (iv)
100. Limiting molar conductivity of NH4OH
96. Standard electrode potential of three metals X, Y and Z are
– 1.2 V, + 0.5 V and – 3.0 V, respectively. The reducing power
of these metals will be :
(i.e., L°
m(NH 4OH ) is equal to :
(a) L°m( NH Cl) + L°m( NaCl) -L°m( NaOH)
(a) Y > Z > X (b) X > Y > Z 4

(c) Z > X > Y (d) X > Y > Z (b) L°m( NaOH) + L°m( NaCl) -L °m( NH Cl)
4
97. The electrode potentials for
(c) L °m( NH °
) + L m( NH 4Cl) -L°m( HCl)
Cu2+(aq) + e– ¾¾ ® Cu+(aq) 4OH

and Cu+(aq) + e– ¾¾ ® Cu(s) (d) L °m( NH °


) + L m( NaOH)-L°m( NaCl)
4Cl
are + 0.15 V and + 0.50, respectively. The value of
E°Cu2+ / Cu will be :

1. Molar conductances of BaCl2, H2SO4 and HCl at infinite (b) Current will flow from electrode A to B in the external
dilutions are x 1 , x 2 and x 3 respectively. Equivalent circuit
conductance of BaSO4 at infinite dilution will be : (c) A will act as cathode and have positive polarity
(a) (x1 + x2 – x3) /2 (b) x1 + x2 – 2x3 (d) None of these
(c) (x1 – x2 – x3) /2 (d) (x1 + x2 – 2x3) /2 5. The electric charge for electrode decomposition of one gram
2. Resistance of a conductivity cell filled with a solution of an equivalent of a substance is
electrolyte of concentration 0.1 M is 100 W. The conductivity (a) one ampere per second
of this solution is 1.29 S m–1. Resistance of the same cell (b) 96500 coulombs per second
when filled with 0.02 M of the same solution is 520 W. The (c) one ampere for one hour
molar conductivity of 0.02 M solution of electrolyte will be (d) charge on one mole of electrons
(a) 1.24 × 10–4 S m2 mol–1 (b) 12.4 × 10–4 S m2 mol–1 6. On passing a current of 1.0 ampere for 16 min and 5 sec through
(c) 124 × 10–4 S m2 mol–1 (d) 1240 × 10–4 S m2 mol–1 one litre solution of CuCl2, all copper of the solution was
3. What is the amount of chlorine evolved when 2 amperes of deposited at cathode. The strength of CuCl2 solution was
current is passed for 30 minutes in an aqueous solution of (Molar mass of Cu= 63.5; Faraday constant = 96,500 Cmol–1)
NaCl ? (a) 0.01 N (b) 0.01 M
(a) 66 g (b) 1.32 g (c) 0.02 M (d) 0.2 N
(c) 33 g (d) 99 g 7. In electrolysis of NaCl when Pt electrode is taken then H2 is
4. A galvanic cell is set up from electrodes A and B liberated at cathode while with Hg cathode it forms sodium
amalgam. This is because
Electrode A : Cr2O 27 - / Cr 3 + , Ered
o = + 1.33 V
(a) Hg is more inert than Pt
o (b) more voltage is required to reduce H+ at Hg than at Pt
Electrode B : Fe 3 + / Fe 2 + , Ered = 0.77 V (c) Na is dissolved in Hg while it does not dissolve in Pt
Which of the following statements is false ? (d) conc. of H+ ions is larger when Pt electrode is taken
(a) Standard e.m.f of the cell is 0.56 V
EBD_7327
414 CHEMISTRY

= –2.37 with increasing voltage, the sequence of deposition


8. Given : 2Br - ® Br2 + 2e - E o = - 1.09 V;
of metals on the cathode will be
I 2 + 2e - ® 2I - E o = 0.54 V (a) Ag, Hg, Cu, Mg (b) Mg, Cu, Hg, Ag
(c) Ag, Hg, Cu (d) Cu, Hg, Ag
Fe 2 + + 2e ® Fe E o = - 0.44 V 16. The e.m.f. of the cell Zn | Zn 2+ (0.01M) | | Fe2+ (0.001M) | Fe
at 298 K is 0.2905 then the value of equilibrium for the cell
Which of the following reactions will not be spontaneous?
reaction is
(a) Fe + Br2 ® FeBr2 (b) Fe + I 2 ® FeI 2 0.32 0.32
(a) (b)
- - -
(c) I 2 + 2Br ® 2I + Br2 (d) Br2 + 2I ® 2Br + I 2 - e 0.0295 10 0.0295
9. Sn4+ + 2e– ¾¾ ® Sn2+ E° = 0.13 V 0.26 0.32
(c) (d)

Br2 + 2e ¾¾ ® 2Br– E° = 1.08 V 10 0.0295 10 0.0591
17. Aluminium displaces hydrogen from acids but copper does
Calculate Keq for the cell formed by two electrodes
not. A galvanic cell prepared by combining Cu / Cu2+ and
(a) 1041 (b) 1032
Al / Al3+ has an e.m.f. of 2.0 V at 298 K. If the potential of
(c) 10–32 (d) 10–42 copper electrode is + 0.34 V, that of aluminium is
10. The e.m.f. of a Daniell cell at 298 K is E1. (a) + 1.66 V (b) – 1.66 V
ZnSO4 CuSO4 (c) + 2.34 V (d) – 2.3 V
Zn Cu
(0.01 M) (1.0 M) 18. 1. 0 L each of a buffer containing 1 mole NH 3 and 1 mol of
When the concentration of ZnSO4 is 1.0 M and that of CuSO4 NH +4 were placed in the cathodic and anodic half-cells and
is 0.01 M, the e.m.f. changed to E2. What is the relationship
965 C of electricity was passed. If anodic and cathodic half
between E1 and E2?
cells reactions involve oxidation and reduction of water only
(a) E 2 = 0 ¹ E1 (b) E1 > E 2 as
(c) E1 < E 2 (d) E1 = E 2
¾® 4H + + O 2 - 4e - ;
2H 2 O ¾
11. Sodium is made by the electrolysis of a molten mixture of
about 40% NaCl and 60% CaCl2 because ¾® H 2 + 2 OH -
2H 2 O + 2e ¾
(a) CaCl2 helps in conduction of electricity Then pH of
(b) this mixture has a lower melting point than NaCl (a) cathodic solution will increase
(c) Ca++ can displace Na from NaCl (b) anodic solution will decrease
(d) Ca++ can reduce NaCl to Na (c) both the solutions will remain practically constant
12. In anodising : (d) both the solutions will increase
(a) The metal ion present in electrodyte gets reduced and 19. A gas X at 1 atm is bubbled through a solution containing a
a film of metal gets coated on the surface of metal mixture of 1MY– and 1 MZ– at 25°C. If the reduction potential
of Z > Y > X, then
cathode
(a) Y will oxidize X and not Z
(b) The anode metal gets oxidised to form a metal oxide
(b) Y will oxidize Z and not X
coat (c) Y will oxidize both X and Z
(c) both the above are correct (d) Y will reduce both X and Z
(d) none of the above is correct 20. Aluminium oxide may be electrolysed at 1000°C to furnish
13. At 25°C, the molar conductance at infinite dilution for the aluminium metal (At. Mass = 27 amu; 1 Faraday = 96,500
strong electrolytes NaOH, NaCl and BaCl2 are 248 × 10–4, Coulombs). The cathode reaction is
126 × 10 –4 and 280 × 10 – 4 Sm 2 mol –1 respectively. Al3+ + 3e– ® Al
To prepare 5.12kg of aluminium metal by this method would
L 0m Ba(OH)2 in S m2 mol –1 is
require
(a) 52.4 × 10– 4 (b) 524 × 10– 4 (a) 5.49 × 107 C of electricity
(c) 402 × 10– 4 (d) 262 × 10– 4 (b) 1.83 × 107 C of electricity
14. The specific conductance of a 0.1 N KCl solution at 23°C is (c) 5.49 × 104 C of electricity
0.012 ohm–1cm–1. The resistance of cell containing the (d) 5.49 × 101 C of electricity
solution at the same temperature was found to be 55 ohm. 21. At pH = 2, E o (quinhydron) = 1.30 V, E(quinhydron) will be
The cell constant will be
OH O
(a) 0.142 cm–1 (b) 0.66 cm–1
(c) 0.918 cm–1 (d) 1.12 cm–1 + 2H + + 2e -
15. A solution containing one mole per litre of each Cu(NO3)2,
AgNO3, Hg2(NO3)2 and Mg(NO3)2, is being electrolysed OH O
by using inert electrodes. The values of standard electrode (a) 1.20 V (b) 1.42 V
potentials in volts (reduction potentials) are Ag+ / Ag (c) 1.36 V (d) 1.30 V
= +0.80, Hg22+ / 2Hg = +0.79, Cu2+ / Cu= +0.34, Mg2+ / Mg
Electrochemistry 415

22. The cell, 30. The standard reduction potentials for Zn 2+ /Zn,
Ni2+/Ni and Fe2+/Fe are –0.76,–0.23 and –0.44 V respectively.
Zn | Zn 2+ (1 M) || Cu 2+ (1 M) | Cu ( E °cell = 1.10 v)
was allowed to be completely discharged at 298 K. The The reaction X +Y 2 + ¾¾ ® X 2+ + Y will be spontaneous
when :
æ [Zn 2+ ] ö (a) X = Ni, Y = Fe (b) X = Ni, Y = Zn
relative concentration of Zn2+ to Cu2+ çç 2+ ÷
÷ is
è [Cu ] ø (c) X= Fe, Y = Zn (d) X= Zn, Y = Ni
(a) 9.65 × 104 (b) antilog (24.08) 31. Electrolysis of dilute aqueous NaCl solution was carried out
(c) 37.3 (d) 1037.3. by passing 10 milli ampere current. The time required to
liberate 0.01 mol of H2 gas at the cathode is (1 Faraday =
23. Given Eº = –0.72 V, Eº 2 + = – 0.42 V. The 96500 C mol–1)
Cr 3+ / Cr Fe / Fe
potential for the cell (a) 9.65 × 104 sec (b) 19.3 × 104 sec
4
(c) 28.95 × 10 sec (d) 38.6 × 104 sec
Cr|Cr3+ (0.1M)|| Fe2 + (0.01 M)| Fe is
32. AgNO 3 (aq.) was added to an aqueous KCl solution
(a) 0.26 V (b) 0.336 V gradually and the conductivity of the solution was measured.
(c) – 0.339 (d) 0.26 V
24. In a fuel cell methanol is used as fuel and oxygen gas is used The plot of conductance ( L ) versus the volume of AgNO3
as an oxidizer. The reaction is is

CH 3OH(l ) + 3/2O 2 (g) ¾¾® CO2 (g) + 2H 2O(l )


At 298 K standard Gibb’s energies of formation for CH3OH(l),
H2O(l) and and CO2 (g) are –166.2 –237.2 and –394.4 kJ mol–
1 respectively. If standard enthalpy of combustion of
methonal is – 726 kJ mol–1, efficiency of the fuel cell will be:
(a) 87% (b) 90%
(c) 97% (d) 80%
25. The Gibbs energy for the decomposition of Al 2O3 at 500°C
is as follows :
2 4
Al2 O3 ® Al + O2 , D r G = + 966 kJ mol -1
3 3
The potential difference needed for electrolytic reduction of
Al2O3 at 500°C is at least
(a) 4.5 V (b) 3.0 V
(c) 2.5 V (d) 5.0 V (a) (P) (b) (Q)
26. The correct order of E ° 2 + values with negative sign (c) (R) (d) (S)
M /M
for the four successive elements Cr, Mn, Fe and Co is 33. Given : E° = -0.74 V; E° = 1.51 V
Cr3+ / Cr MnO-
4 / Mn
2+
(a) Mn > Cr > Fe > Co (b) Cr < Fe > Mn > Co
(c) Fe > Mn > Cr > Co (d) Cr > Mn > Fe > Co E° = 1.33 V; E° = 1.36 V
Cr2O72 - / Cr 3+ Cl / Cl-
27. The reduction potential of hydrogen half-cell will be negative
Based on the data given above, strongest oxidising agent
if :
will be :
(a) p(H2) = 1 atm and [H+] = 2.0 M (a) Cl (b) Cr3+
(b) p(H2) = 1 atm and [H+] = 1.0 M (c) Mn 2+ (d) MnO4 –
(c) p(H2) = 2 atm and [H+] = 1.0 M 34. Four successive members of the first row transition elements
(d) p(H2) = 2 atm and [H+] = 2.0 M are listed below with atomic numbers. Which one of them is
28. Consider the following cell reaction: expected to have the highest E° 3+ 2+ value ?
2Fe( s) + O2 (g) + 4H+ (aq) ® 2Fe2+ (aq) + 2H2O(l);E° =1.67V M /M
(a) Cr(Z = 24) (b) Mn(Z = 25)
At [Fe2+] = 10–3 M, p(O2) = 0.1 atm and pH = 3, the cell (c) Fe(Z = 26) (d) Co(Z = 27)
potential at 25ºC is
35. A solution contains Fe2+, Fe3+ and I– ions. This solution
(a) 1.47 V (b) 1.77 V
(c) 1.87 V (d) 1.57 V was treated with iodine at 35°C. E° for Fe3+ / Fe2+ is + 0.77 V
29. Resistance of 0.2 M solution of an electrolyte is 50 W. The and E° for I2/2I– = 0.536 V. The favourable redox reaction is :
specific conductance of the solution is 1.3 S m –1 . If (a) I2 will be reduced to I–
resistance of the 0.4 M solution of the same electrolyte is (b) There will be no redox reaction
260 W, its molar conductivity is : (c) I– will be oxidised to I2
(a) 6.25 × 10–4 S m2 mol–1 (b) 625 × 10–4 S m2 mol–1 (d) Fe2+ will be oxidised to Fe3+
(c) 62.5 S m2 mol–1 (d) 6250 S m2 mol–1
EBD_7327
416 CHEMISTRY

36. Standard reduction potentials of the half reactions are given 43. The electrode potential E of a zinc electrode at
below : (Zn 2+ Zn)
F2(g) + 2e– ® 2F– (aq); E° = + 2.85 V 25°C with an aqueous solution of 0.1 M ZnSO 4 is
Cl2(g) + 2e– ® 2Cl–(aq); E° = + 1.36 V
2.303RT
Br2(l) + 2e– ® 2Br–(aq); E° = + 1.06 V [ E° = – 0.76 V. Assume = 0.06 at 298 K].
(Zn 2+ Zn) F
I2(s) + 2e– ® 2I–(aq); E° = + 0.53 V
The strongest oxidising and reducing agents respectively are : (a) + 0.73 (b) – 0.79
(a) F2 and I– (b) Br2 and Cl– (c) – 0.82 (d) – 0.70
(c) Cl2 and Br – (d) Cl2 and I2 44. The electrical properties and their r espective
37. Molar conductivities ( L° m ) at infinite dilution of NaCl, HCl SI units are given below. Identify the wrongly matched pair.
and CH3COONa are 126.4, 425.9 and 91.0 S cm2 mol–1 Electrical property SI unit
respectively. L°m for CH3COOH will be : (a) Specific conductance S m–1
(a) 425.5 S cm2 mol–1 (b) 180.5 S cm2 mol–1 (b) Conductance S
2
(c) 290.8 S cm mol –1 (d) 390.5 S cm2 mol–1
(c) Equivalent conductance S m2 gequiv–1
38. The equivalent conductances of two strong electrolytes at
infinite dilution in H2O (where ions move freely through a (d) Cell constant m
solution) at 25°C are given below : 45. The ion of least limiting molar conductivity among the
LoCH3COONa = 91.0 S cm2 / equiv. following is

L o HCl = 426.2 S cm 2 / equiv. (a) SO 24 - (b) H+


What additional information/ quantity one needs to calculate
Lo of an aqueous solution of acetic acid? (c) Ca 2+ (d) CH 3COO -
(a) L o of chloroacetic acid (ClCH2COOH) 46. The reduction potential (in volt) of a hydrogen electrode set
(b) L o of NaCl up with a 2 × 10–2 M aqueous solution of a weak mono basic
(c) L o of CH3COOK acid (Ka = 5 × 10–5) at one atmosphere and 25°C is
(d) the limiting equivalent coductance of H + (l° ). (a) + 0.09 (b) + 0.18
H+
39. Given: E ° = –0.036V, E ° = –0.439 V (c) – 0.09 (d) – 0.18
Fe 3+ / Fe Fe 2+ / Fe
47. A current strength of 3.86 amp was passed through molten
The value of standard electrode potential for the change, calcium oxide for 41 minutes and 40 seconds. The mass of
Fe 3+ ( aq ) + e – ¾¾
® Fe 2+ ( aq ) will be: calcium in grams deposited at the cathode is (Atomic mass
(a) 0.385 V (b) 0.770 V of Ca is 40 g/mol, 1 F = 96500 C)
(c) –0.270 V (d) –0.072 V (a) 4 (b) 2
40. Consider the following four electrodes: (c) 6 (d) 8
P = Cu2+ (0.0001 M)/Cu(s) DIRECTIONS for Qs. 48 to 50 : These are Assertion-Reason
Q = Cu2+ (0.1 M)/Cu(s) type questions. Each of these question contains two statements:
R = Cu2+ (0.01 M)/Cu(s) Statement-1 (Assertion) and Statement-2 (Reason). Answer these
S = Cu2+ (0.001 M)/Cu(s) questions from the following four options.
If the standard reduction potential of Cu2+/Cu is +0.34 V, (a) Statement-1 is true, Statement-2 is true, Statement-2 is a
the reduction potentials in volts of the above electrodes correct explanation for Statement-1
follow the order. (b) Statement-1 is True, Statement-2 is True, Statement-2 is NOT
(a) P > S > R > Q (b) S > R > Q > P a correct explanation for Statement-1
(c) R > S > Q > P (d) Q > R > S > P (c) Statement-1 is True, Statement-2 is False
41. Which pair of electrolytes could not be distinguished by (d) Statement-1 is False, Statement-2 is True
the products of electrolysis using inert electrodes?
48. Statement-1 : The resistivity for a substance is its resistance
(a) 1 M CuSO4 solution, 1 M CuCl2 solution
when it is one meter long and its area of cross section is one
(b) 1 M KCl solution, 1 M KI solution
square meter.
(c) 1 M AgNO3 solution, 1 M Cu(NO3)2 solution
Statement-2 : The SI units of resistivity is ohm metre (Wm).
(d) 1 M KCl solution, 1 M NaCI solution
42. A current is passed through two cells connected in series. 49. Statement-1 : On increasing dilution, the specific
The first cell contains X(NO3)3(aq) and the second cell conductance keep on increasing.
contains Y(NO3)2(aq). The relative atomic masses of X and Statement-2 : On increasing dilution, degree of ionisation of
Y are in the ratio 1 : 2. What is the ratio of liberated mass of weak electrolyte increases and molality of ions also increases.
X to that of Y? 50. Statement-1 : Galvanised iron does not rust.
(a) 3 : 2 (b) 1 : 2 Statement-2 : Zinc has a more negative electrode potential
(c) 1 : 3 (d) 3 : 1 than iron.
Electrochemistry 417

Exemplar Questions (d) Ecell is an extensive property while DrG of cell reaction
is an intensive property.
1. Which cell measure standard electrode potential of copper 4. The difference between the electrode potentials of two
electrode? electrodes when no current is drawn through the cell is
(a) Pt ( s ) |H 2 ( g, 0.1 bar ) | |H + ( aq, 1 M )| |Cu 2+ ( aq, 1 M )| Cu called......
(a) cell potential (b) cell emf
(b) Pt ( s ) |H 2 ( g,1 bar ) | |H + ( aq, 1 M )| |Cu 2+ ( aq, 2 M )| Cu (c) potential difference (d) cell voltage
5. Which of the following statement is not correct about an
(c) Pt ( s) |H2 ( g,1 bar ) | |H+ ( aq,1 M)| |Cu 2+ ( aq, 1 M )| Cu inert electrode in a cell?
(a) It does not participate in the cell reaction
(d) Pt ( s) |H2 ( g,0.1bar) | |H+ ( aq, 0.1M)| |Cu 2+ ( aq,1 M)| Cu (b) It provides surface either for oxidation or for reduction
reaction
2. Electrode potential for Mg electrode varies according to the
(c) It provides surface for conduction of electrons
equation
(d) It provides surface for redox reaction
E 2+ = Es 2+ 0.059 1 6. An electrochemical cell can behave like an electrolytic cell
Mg / Mg Mg / Mg - 2 log . The graph
éëMg 2 + ùû when ......
(a) Ecell = 0 (b) Ecell > Eext
of E vs log [Mg2+] is
Mg 2 + / Mg (c) Eext > Ecell (d) Ecell = Eext
7. Which of the statements about solutions of electrolytes is
not correct?
®

(a) E (a) Conductivity of solution depends upon size of ions


Mg 2 + / Mg (b) Conductivity depends upon viscosity of solution
(c) Conductivity does not depend upon solvation of ions
present in solution
log éë Mg2 + ùû ® (d) Conductivity of solution increases with temperature
8. Using the data given below find out the strongest reducing
agent.
– –
®

(b) E E = 1.33V; E = 1.36V


Mg 2 + / Mg Cr2O 27 - / Cr 3+ Cl2 / Cl -
– –
E = 1.51V; E = - 0.74V
MnO -4 / Mn 2 + Cr 3+ / Cr -

log éë Mg2 + ùû ® (a) Cl (b) Cr
(c) Cr3+ (d) Mn2+
9. Use the data given in Q. 8 and find out which of the following
is the strongest oxidising agent?
®

(c) E (a) Cl– (b) Mn2+


Mg 2 + / Mg
(c) MnO4- (d) Cr3+
10. Using the data given in Q. 8 find out in which option the
log éë Mg2 + ùû ® order of reducing power is correct.
(a) Cr3+ < Cl - > Mn 2 + < Cr
(b) Mn 2 + < Cl - > Cr 3+ < Cr
®

(d) E 2+
Mg / Mg
(c) Cr3+ < Cl - < Cr2O72 - < Mn 2 +
(d) Mn 2 + < Cr 3+ < Cl - < Cr
11. Use the data given in Q.8 and find out the most stable ion in
log éë Mg2 + ùû ®
its reduced form.
3. Which of the following statement is correct?
(a) Cl– (b) Cr3+
(a) Ecell and D rG of cell reaction both are extensive
(c) Cr (d) Mn2+
properties.
12. Use the data of Q. 8 and find out the most stable oxidised
(b) Ecell and D rG of cell reaction both are intensive
species.
properties.
(c) Ecell is an intensive property while DrG of cell reaction (a) Cr3+ (b) MnO4-
is an extensive property.
(c) Cr2 O72 - (d) Mn2+
EBD_7327
418 CHEMISTRY

13. The quantity of charge required to obtain one mole of The cell potential will be : [2013]
aluminium from Al2O3 is ....... . (a) 0.42 V (b) 0.84 V
(a) 1 F (b) 6 F (c) 1.34 V (d) 1.10 V
(c) 3 F (d) 2 F 21. How many grams of cobalt metal will be deposited when a
14. The cell constant of a conductivity cell ........ . solution of cobalt (II) chloride is electrolyzed with a current
(a) changes with change of electrolyte of 10 amperes for 109 minutes (1 Faraday = 96,500 C; Atomic
(b) changes with change of concentration of electrolyte mass of Co = 59 u) [NEET Kar. 2013]
(c) changes with temperature of electrolyte (a) 0.66 (b) 4.0
(d) remains constant for a cell (c) 20.0 (d) 40.0
15. While charging the lead storage battery ......... . 22. Consider the half-cell reduction reaction :
(a) PbSO4 anode is reduced to Pb Mn2+ + 2e– ® Mn, E° = –1.18 V
(b) PbSO4 cathode is reduced to Pb Mn2+ ® Mn3+ + e–, E° = –1.51 V
(c) PbSO4 cathode is oxidised to Pb The E° for the reaction 3Mn 2+ ® Mn 0 + 2Mn 3+, and
(d) PbSO4 anode is oxidised to PbO2 possibility of the forward reaction are, respectively
°
16. L m( NH4OH ) is equal to ....... . [NEET Kar. 2013]
(a) – 2.69 V and no (b) – 4.18 V and yes
(a) L °m( NH + L °( NH - L °m( HCI ) (c) + 0.33 V and yes (d) + 2.69 V and no
4OH ) 4Cl) 23. When 0.1 mol MnO42– is oxidised the quantity of electricity
L °m( NH Cl) + L °m( NaOH ) - L °m( NCI ) required to completely oxidise MnO42– to MnO4– is:
(b) 4
(a) 96500 C (b) 2 × 96500 C [2014]
(c) L °m( NH + L °m( NaCl) - L °m ( NaOH ) (c) 9650 C (d) 96.50 C
4Cl )
24. The weight of silver (at wt. = 108) displaced by a quantity of
(d) L °m( NaOH ) + L °m( NaCl) - L °m( NH electricity which displaces 5600 mL of O2 at STP will be :
4Cl )
(a) 5.4 g (b) 10.8 g [2014]
17. In the electrolysis of aqueous sodium chloride solution which
of the half cell reaction will occur at anode? (c) 54.9 g (d) 108.0 g
25. A device that converts energy of combustion of fuels like
Na + ( aq ) + e - ¾¾
°
(a) ® Na ( s ) ; E cell = - 2.71V hydrogen and methane, directly into electrical energy is
known as : [2015]
(b) 2H 2 O ( l ) ® O2 ( g ) + 4H + ( aq ) + 4e - ; E°cell = 1.23V (a) Electrolytic cell (b) Dynamo
1 (c) Ni-Cd cell (d) Fuel Cell
(c) H + ( aq ) + e - ¾¾
® H 2 ( g ) ; E°cell = 0.00 V 26. Aqueous solution of which of the following compounds is
2
the best conductor of electric current ? [2015 RS]
1 (a) Acetic acid, C2H4O2
(d) Cl – ( aq ) ¾¾
® Cl2 ( g ) + e - ; E°cell = 1.36 V
2 (b) Hydrochloric acid, HCl
(c) Ammonia, NH3
NEET/AIPMT (2013-2017) Questions (d) Fructose, C6H12O6
18. A hydrogen gas electrode is made by dipping platinum wire 27. The pressure of H2 required to make the potential of
in a solution of HCl of pH = 10 and by passing hydrogen gas H2-electrode zero in pure water at 298 K is : [2016]
around the platinum wire at one atm pressure. The oxidation (a) 10–14 atm (b) 10–12 atm
potential of electrode would be ? [2013] (c) 10–10 atm (d) 10–4 atm
(a) 0.59 V (b) 0.118 V 28. Ionic mobility of which of the following alkali metal ions is
(c) 1.18 V (d) 0.059 V lowest when aqueous solution of their salts are put under
19. At 25°C molar conductance of 0.1 molar aqueous solution an electric field ? [2017]
of ammonium hydroxide is 9.54 ohm-1 cm2mol-1 and at infinite (a) K (b) Rb
dilution its molar conductance is 238 ohm-1 cm2 mol-1. The (c) Li (d) Na
degree or ionisation of ammonium hydroxide at the same 29. In the electrochemical cell :- [2017]
concentration and temperature is : [2013] Zn | ZnSO4 (0.01M) | | CuSO4 (1.0 M) | Cu, the emf of this
(a) 20.800% (b) 4.008% Daniel cell is E1. When the concentration of ZnSO4 is
(c) 40.800% (d) 2.080% changed to 1.0M and that of CuSO4 changed to 0.01M, the
20. A button cell used in watches functions as following emf changes to E2. From the followings, which one is the
Zn(s) + Ag2O(s) + H2O(l) relationship between E1 and E4?
2Ag(s) + Zn2+(aq) + 2OH–(aq) RT
If half cell potentials are : (Given, = 0.059)
F
Zn2+(aq) + 2e– ® Zn(s); Eo = – 0.76 V
Ag2O(s) + H2O (l) + 2e– ® 2Ag(s) + (a) E1 < E2 (b) E1 > E2
2OH–(aq); Eo = 0.34 V (c) E2 = 0 ¹ E1 (d) E1 = E2
Electrochemistry 419

Hints & Solutions


EXERCISE - 1 8. (b) ohm–1 cm2 (geq)–1
1. (b) Charge passed 1000 1 l 1000
9. (a) L eq = k ´ = ´ ´
= 30 ´ 60 ´ 2 = 3600 coulombs N R a N
Chlorine liberated 1 1000
= ´ cell constant ´
35.5 ´ 3600 R N
= = 1.32gm
96500 1 1000
= ´ 0.88 ´ = 400 mho cm2 g eq–1
2. (a) NH 4+ is conjugate acid of NH3 as it is capable of giv- 220 0.01
ing H+ ion. 10. (c) Charge = 1 × 30 × 60 = 1800 coulomb
96500 liberates 35.5 gm of Cl
NH3 + H + ¾¾
® NH 4+ 1800 coulomb liberates
Base Conjugate acid
35.5 ´ 1800
3. (d) Molarity = 0.01 M ; Resistance = 40 ohm; = 0.66 gm of Cl
l 96500
-1
Cell constant = 0.4cm . 11. (b) Here, R = 31.6 ohm
A
Specific conductivity (k ) 1 1
\ Conductance = = ohm -1 = 0.0316 ohm–1
R 31.6
cell constant 0.4
= = = 0.01 ohm -1 cm -1 Specific conductance = conductance × cell constant.
resistance 40 = 0.0316 ohm–1 × 0.367 cm–1 = 0.0116 ohm–1 cm–1
1000k Now, molar concentration = 0.5M (given)
Molar conductance ( Ù m ) = = 0.5 × 10–3 mole cm–3
Molarity
k 0.0116
1000 ´ 0.01 \ Molar conductance = =
= = 103 ohm -1 cm 2 mol-1 C 0.5 ´ 10 -3
.01
4. (b) Specific conductance of the solution (k) = 0.012 ohm–1 cm– = 23.2 S cm2 mol–1
1 and resistance (R) = 55 ohm. 12. (b) For the given cell
Cell constant = Specific conductance × Observed
M M + X - X, the reaction can be given by
resistance
= 0.012 × 55= 0.66cm–1. R.H.S. : Reduction -
5. (a) Difluoroacetic acid being strongest acid will furnish
maximum number of ions showing highest electrical X + e - ® X - ; E 0 = 0.33 V
conductivity. L.H.S. : Oxidation
6. (c) Volume of 1 × 10 –5 gm of hydrogen
M ® M + + e - ; E0 = 0.49 V
-5
22400 ´10 Adding both half cell reactions, we get
= c.c.
2
When 0.5 amp flows for 2 hours, charge passed M + X ® M+ + X -
= 0.5 × 60 × 60 × 2 = 3600 coulomb E°cell = E°RHS - E°LHS = 0.33 - 0.44 = -0.11 V
Now 1 coulomb of electricity releases
Since ECell = –ve, the reaction can not be spontaneous
22400 ´10 -5 in forward direction but can proceed spontaneously if
= c.c. of gas the direction is reversed.
2
3600 coulomb of electricity releases, 13. (b) Given current (i) = 0.5 amp;
Time (t) = 100 minutes × 60 = 6000 sec
22400 ´ 10 -5 3600 Equivalent weight of silver nitrate (E) = 108.
= ´ litres = 0.4 litres
2 10 3 According to Faraday's first law of electrolysis
L¥ Eit 108 ´ 0.5 ´ 6000
7. (b) HCl = 426.2 (i) W= = = 3.3575 g.
96500 96500

AcONa = 91.0 (ii)
14. (c) According to Faraday's law of electrolysis, the amount
L¥ = 126.5 (iii) of metal deposited on an electrode is directly
NaCl
proportional to the amount of electricity passed and
¥
L AcOH = (i) + (ii) - (iii) = [426.2 + 91.0 - 126.5] = 390.7 equivalent weight of metal. Now equivalent weight of
EBD_7327
420 CHEMISTRY

24.3 23. (a) Total charge on one mole of monovalent metal ion
sodium is 23, magnesium is i.e. 12.15 and of = Charge on N0 electrons
2
27 1F = 96500 coulombs = 9.65 ´104 coulombs .
aluminium is i.e. 9. So, sodium which has highest
3 24. (c) Using Faraday’s second law of electrolysis,
equivalent weight among Na, Mg and Al, is deposited
to highest extent during electrolysis. Weight of Cu deposited Equ. wt. of Cu
=
15. (d) 1 mole of e– = 1F = 96500 C Weight of Ag deposited Equ. wt. of Ag
27g of Al is deposited by 3 × 96500 C
5120 g of Al will be deposited by w Cu 63.5 1
Þ = ´
0.18 2 108
3 ´ 96500 ´ 5120
= = 5.49 ´ 10 7 C
27 63.5 ´ 18
Þ wCu = = 0.0529 g.
16. (a) 2H+ + 2e– ¾ ¾ ® H2 2 ´108 ´ 100
2 25. (a) Charge passed = 25 × 10–3 × 60
EH (Eq. wt) = =1 g
2 = 15 ´ 10 -1 = 1.5 coulomb

=
22400
= 11200 ml (STP) Ca 2+ + 2e ¾¾ ® Ca
2 Now for calcium ion,
96500´112 2 × 96500 coulomb is required for
Total charge passed = = 965 6.02 × 1023 atoms
11200
Q = It = 965 6.02 ´10 23 ´1.5
0.3 coulomb will be required for
965 2 ´ 96500
I= = 1amp.
965 = 4.68 × 1018
26. (a) For deposition of silver, reaction is
17. (d) WA WB 1.6 Wt. of Ag
= ; =
EA EB 8 108 Ag+ + e– ¾¾ ® Ag
1 mol of Ag will be deposited by
\ Wt . of Ag = 21.6 g = 1 F = 96500 C = 9.65 × 104 C
18. (b) Here Ag+ acts as oxidising agent and Co2+ acts as Since 1 equivalent weight of Ag is also equal to the
reducing agent. So, cell can be represented as weight of its 1 mol, hence 1 equivalent weight of Ag
Co2+ | Co3+(aq.) || Ag+(aq.) | Ag will be deposited by = 9.65 × 104 C
Using Nernst equation, 27. (c) According to the definition 1 F or 96500 C is the charge
carried by 1 mol of electrons when water is electrolysed
0.0591 [Co3+ (aq)][Ag]
Ecell = Eocell - log 2H2O ¾¾ ® 4H+ + O2 + 4e–
n [Ag + (aq)][Co 2+ ] So, 4 Faraday of electricity liberate = 32 g of O2.
So, lesser the logarithmic term higher will be the value Thus 1 Faraday of electricity liberate
of Ecell. Among the given combinations logarithmic
term is lesser for 0.1 M Co3+ and 2M Ag+ and hence it 32
= g of O2 = 8 g of O2
will form a cell of highest voltage. 4
19. (b) Cu+2 + 2e– ® Cu (s) 28. (a) Charging increases the voltage. When the discharged
2 × 96500 C 63.6g cell is recharged, it acts as electrolytic cell.
29. (b) A current of 96500 coulombs liberate 1 mole of O2.
63.6
9650 C will deposit = ´ 9650 = 3.18 g Þ 96500 C liberates = 22.4 L of O2 at NTP
2 ´ 96500
22.4
20. (a) Charge = 0.2 × 1 Faraday Þ 9650 C liberates = ´ 9650
= 0.2 × 96500 coulombs 96500
= 19300 = 1.93 × 104 coulombs = 2.24 L of O2 at NTP
21. (b) Magnesium which is more electropositive than iron 56
prevents the rusting of iron by converting themselves 30. (b) Fe2+ + 2e - ® Fe; E Fe = = 28
2
into positive ion and thus reverses the process of
1 Faraday liberates = 28 g of Fe
conversion of iron to Fe 3+ ion.
3 Faraday liberates = 3 × 28 = 84 gm
22. (d) Ag + + e - ¾¾
® Ag 31. (c)
96500 coulombs deposit = 108 g of Ag 32. (b) Amount of silver present in 125 ml of 1 M. AgNO3
108 108
\ 965 coulombs deposit = ´ 965 = 1.08 g Ag = ´ 125 g
96500 1000
Electrochemistry 421

Q needed for deposition of 108 g (1 g equivalent of Ag) 42. (a)


= 1 F = 96500 C Cell constant 1.15
\ Q needed for deposition of 43. (a) k= =
R 250
108 1 108 1.15 1000
´ 125 g Ag = ´ ´ 125 F = 0.125 F L eq = ´ = 4.6
1000 108 1000 250 1
= 0.125 × 96500 C 44. (c) Molar conductance of solution is related to specific
conductance as follows :
Since, Q = i × t
0.125 ´ 96500
\ t=
Ùm = K ´ 1000 ....(1)
= 50 s C
241.25 where C is molar concn.
[Given: i = 241.25 amperes] Putting K = 6.3 × 10–2 ohm–1 cm–1 and
33. (d) Charge (Coulombs) pass per second = 10–6 C = 0.1M in equ (1), we get
number of electrons passed per second
1000
-6 Ùm = (6.3 × 10–2 ohm–1 cm–1) ×
=
10
= 6.24 ´ 1012 (0.1mol / cm3 )
1.602 ´ 10 -19 = 6.3 × 10–2 × 104 ohm–1cm2 mol–1
= 630 ohm–1 cm2 mol–1
° = 2.303RT .0591 45. (a) Anode has negative polarity.
34. (c) Ecell log K eq = log K eq
nF n 46. (b) Reduction occurs at cathode. Among the given choices,
the reaction shown in option (b) is the reduction of
.0591 Ag+ to Ag. So, it occurs at cathode.
= log 106 = .0591 × 3 = 0.1773 V
2 47. (a) Cell reaction
35. (a) 2Cr 3+
+ 7 H 2 O ® Cr2 O 72 -
+ 14H + cathode :
O.S. of Cr changes from +3 to +6 by loss of electrons. 1
H 2O(l) + O2 ( g ) + 2e - ¾¾
® 2OH - (aq)
At anode oxidation takes place. 2
36. (d) In electrolysis of dilute H2SO4, O2 is liberated at anode.
It can be summarised as follows :-
anode : ® 2H + ( aq ) + 2e-
H 2 ( g ) ¾¾
H2SO4 ‡ˆˆˆˆ† 2H+ + SO 2– ––––––––––––––––––––––––––––––––––––––––
4
ˆˆ†
H2O ‡ˆˆ H+ + OH– 1
During electrolysis, the hydrogen ions migrate towards ® 2H + (aq) + 2OH - (aq )
H 2 O(l) + O 2 ( g ) + H 2 ( g ) ¾¾
2
the cathode and are collected here in the form of Also we have
hydrogen gas.
Following reactions take place 1 o
® H 2O(l) ; DG f = -237.2 kJ / mole
H 2 ( g ) + O 2 ( g ) ¾¾
At cathode : 2H+ + 2e– ¾¾ ® H2 2
4H+ + 4e– ¾¾ ® 2H2
® H + ( aq ) + OH - ( aq );
H 2 O( l) ¾¾
At anode : 2H2O ¾¾ ® O2 + 4H++ 4 e–
Overall reaction is DG° = 80 kJ / mole
4H+ + 2H2O ‡ˆˆˆˆ† 2H2 + O2 + 4H+ Hence for cell reaction
Thus, we see that hydrogen is produced at cathode DG° = -237.2 + (2 ´ 80) = -77.20 kJ / mole
and oxygen at anode during electrolysis.
37. (d) Calomel electrode is used as reference electrode. DG° 77200
\ E° = - = = 0.40V
38. (b) Electrode potential is considered as zero nF 2 ´ 96500
39. (a) Ag is easiest to be reduced among the given choices 48. (b) (i) A3+ + e– ––––® A2+, DG1 = – 1 F y2
because its reduction potential is highest among the (ii) A2+ + 2e– ––––® A, DG2 = –2F(–y1) = 2Fy1
given choices. Add, (i) and (ii) we get
Reduction A3+ + 3e– ––––® A ;
DG3 = DG1 + DG2
40. (d) It shows reduction reaction. –3FE° = –Fy2 + 2Fy1
Zn 2∗ 2e, Zn
–3FE° = –F (y2 – 2y1)
41. (b) 2AgCl(s) + H 2(g) ® 2HCl(aq) + 2Ag(s) y2 - 2 y1
E° =
The activities of solids and liquids are taken as unity 3
and at low concentrations, the activity of a solute is 49. (a) In the electrolysis of cupric sulphate, the reaction that
approximated to its molarity. occurs at cathode is
The cell reaction will be
Cu 2+ + 2e- ¾¾® Cu
Pt (s) | H 2(g) ,1bar | H + (aq) 1M | AgCl(aq) 1M | Ag(s) Thus 2F or 2 × 96500 C of electricity is required to
deposit
EBD_7327
422 CHEMISTRY

= 1 mol of Cu = 63.5 g of Cu
2 ´ 0.295
It means that to deposit 63.5 g of Cu, the amount of \ log K = = 9.98 » 10 or K = 1010
electricity required = 2 × 96500 C 0.0591
So, to deposit 0.634 g of Cu, the amount of electricity 55. (b) RHS : 2H+ + 2e– ‡ˆˆˆˆ† H2 (P2)
ˆˆ† 2H+ + 2e–
LHS : H2(P1) ‡ˆˆ
2 ´ 96500
required = ´ 0.634 » 1930C ˆˆ† H2(P2)
overall reaction : H2 (P1) ‡ˆˆ
63.5
RT P2 RT P2 RT P1
50. (b) Without losing its concentration ZnCl2 solution cannot E = E°- l n = 0- ln = ln
kept in contact with Al because Al is more reactive nF P1 nF P1 nF P2
than Zn due to its highly negative electrode reduction
potential. 56. (a) Zn( s) + 2H+ (aq) Zn 2+ (aq) + H 2 ( g )

51. (c) 2 Cu + ¾¾
® Cu +2 + Cu 0.059 [Zn 2 + ][H 2 ]
Ecell = E °cell - log
º 2 [H + ]2
2 e - + Cu +2 ¾
¾® Cu ; E1 = 0.34V; ...(i)
Addition of H2SO4 will increase [H+]and Ecell will also
-
e + Cu +2
¾® Cu ;
¾ + E2º = 0.15V; ...(ii) increase and the equilibrium will
shift towards RHS
Cu + + e- ® Cu; E3º = ? ... (iii) 57. (c) According to Faraday’s law of electrolysis
W = ZQ
Now, DG1º = -nFE1º = -2 ´ 0.34F where W is weight of substance liberated at an electrode,
Z is electrochemical equivalent and Q is quantity of
DG2º = -1 ´ 0.15F , DG3º = -1 ´ E3º F
electricity passed.
Again, DG1º = DG2º + DG3º When Q coloumbs of electricity passed through molten
Al2O3, the amount of Al liberated is
Þ -0.68F = -0.15F - E3º F WAl = ZAl Q
equ.wt of Al
Þ E3º = 0.68 - 0.15 = 0.53V Þ WAl = Q
96500
º
Ecell º
= Ecathode (Cu + / Cu) - Eanode
º
(Cu +2 / Cu + )
27
= 0.53 – 0.15 = 0.38 V. Þ 1.8 = Q .........(i)
3 ´ 96500
52. (d) In the silver plating of copper, K[Ag(CN ) 2 ] is used Similarly when Q cloumbs of electricity is passed
instead of AgNO3. Copper being more electropositive through molten NaCl, the amount of Na liberated is
readily precipitate silver from their salt solution
23
WNa = Q ........(ii)
Cu ∗ 2AgNO3 ¾¾ ↑ Cu(NO 3 ) 2 ∗ Ag 96500
whereas in K[Ag(CN)2 ] solution a complex anion Divide (ii) by (i), we get
[Ag(CN)2]– is formed and hence Ag+ are less available WNa 23 96500 ´ 3
in the solution and therefore copper cannot displace = ´
Ag from its complex ion. 1.8 96500 27
53. (b) I = 1A, t = 16 min. 5 sec = 965 sec 23
Quantity of electricity pased, Þ WNa = 1.8 ´ 3 ´ = 4.6 g.
27
Q = It = 1 × 965 C = 965 C
The reaction involved at the cathode is o RT
58. (d) Ecell = Ecell - ln Q
Cu2+ + 2e– ¾¾ ® Cu nF
Thus, 2F (= 2 × 96500 C) deposits At equilibrium,
= 1 mol of Cu = 63.5g Cu Ecell = 0 and Q = Kc
63.5 ´ 965 \
o
Ecell ¹ Ecell
Þ 965 C deposits = g Cu = 0.3125 g Cu
2 ´ 96500 59. (b) A current of 96500 coulombs liberate 1 mole of O2.
Now, Normality of the solution Þ 96500 C liberates = 22.4 L of O2 at NTP
Þ 9650 C liberates = 2.24 L of O2 at NTP
0.3125 1000
= ´ = 0.01 N 60. (d) Here Ni is anode and Au is cathode
(63.5 / 2) 1000
Given E° ° 3+
= -0.25V and EAu = +1.5V
0.0591 Ni 2 + / Ni / Au
54. (d) E ° = log K
n
E°cell = E°C - E°A = 1.5 – (– 0.25) = 1.5 + 0.25 = 1.75 V
Here, n = 2, E ° = 0.295
Electrochemistry 423

61. (c) E° °
= – 0.74 V, E Co 2+ / Co = – 0.28 V 0.222 = 0.7995 -
0.0591 1
Cr 3+ / Cr log
The given cell reaction is 2 [Ag + ]2
[Ag+] = 10–9.8
Cr | Cr 3+ (1.0 M ) || Co 2 + (1.0 M) | Co Ksp = [Ag+] [Cl–] = (10–9.8) × (1) = 10–9.8
\ Cr is anode and Co is cathode 70. (b) Nernst equation is
° = EC° – E °A = – 0.28 – (– 0.74)
Ecell 2.303 RT [C]c [d]d
Ecell = E ocell - log
= – 0.28 + 0.74 = + 0.46 V nF [A]a [B]b
62. (a) On removing the salt bridge between the two half cells When Ecell drops to zero the concentration of ions will
the circuit is broken. Hence, emf becomes zero. be equilibrium concentrations. We have for general cell
63. (c) The E°cell is given by reaction
0.0591 aA + bB cC + dD
E°cell = log Keq
n 2.303RT [C]c [D]d
o
E cell = E cell - log
0.0591 nF [A]a [B]b
\ 0.591 = log K eq
1 2.303RT
E ocell = log K c
0.591 nF
or log Keq = = 10 [Since at equilibrium Ecell = 0]
0.0591
or Keq = 1 × 1010 0.0591
E ocell = log K c at 298K.
n
64. (a) Given : [Fe3 + e– ¾¾ ® Fe2+] × 2 71. (a) Apply Nernst equation to the reaction
E° = 0.771 × 2 V = 1.542 V Pb + Sn2+ ® Pb2+ + Sn
2I – ¾¾ ® I2 + 2e– E° = – 0.536 V 0.059 [Sn 2+ ]
or E° + log = Ecell
2Fe3+ + 2I– ¾¾
2 [Pb 2 + ]
® 2Fe2+ + I2
E°cell = 1.542 – 0.536 = 1.006 V [Sn 2 + ] 0.01 ´ 2
65. (b) For the given cell or log 2+
= = 0.3 (Q Ecell = 0)
[Pb ] 0.059
0.059V [Zn 2 + ( aq )]
Ecell = E°cell - log [Sn 2+ ]
2 [Cu 2 + ( aq )] or = antilog (0.3)
The cell potential will decreases with increase in [Pb 2 + ]
[Zn 2+ (aq)] and will increases with increase in 72. (d) For a concentration cell having different
[Cu2+(aq)]. concentrations of ions.
66. (b) Fe is above Pb in the electrochemical series i.e. Eooxid E=
0.0591 c
log 1
o
n c2
of Fe (+ 0.44 V) is higher than Eoxid of Pb (+ 0.129 V). If all the concentrations are identical then obviously
Hence, on addition of powdered Fe and Pb to a solution the cell voltage is zero. But as the pH of 0.1 M HCl
of Fe++ ion and Pb ++ ion, the following reaction will (strong acid) and pH of 0.1M CH3COOH is (weak acid)
take place : not same, therefore the cell voltage is not zero.
73. (b) Electricity required = No. of g equiv. × 96500 coulombs
Fe(s) + Pb + + (aq) ¾¾ ® Fe + + (aq) + Pb(s) = 0.5 × 96500 = 48250 C
++
So, more Pb and Fe ions will be formed.
67. (b) Maximum work done = Wmax 74. (b) DG = - nE°F
RT C2
° = -2´ 96500´1.10 = -212.30 KJ
= -nFEcell For concentration cell, E = ln
nF C1
68. (c) pH changes from 0 to 7.
\ [H+] changes from 1 to 10–7 M. In it R, T, n and F are constant

-0.059 1 C2
Accordingly Ered. = log + So E is based upon l n
n C1
[H ]
= 0.059 log 10–7 RT
Now DG = -nEF = - nF ´ ln C2 / C1
i.e., 0.059 × (–7) = – 0.41 volt. nF
69. (a) 2Ag+ + H2 ¾¾ ® 2H+ + 2Ag = –RTlnC2/C1
At constant temperature DG is a function of
0.0591 [H + ]2
E = E° - log ln C2/C1.
2 PH 2 ´ [Ag+ ]2
EBD_7327
424 CHEMISTRY

75. (d) This is because zinc has higher oxidation potential than ion migrates independent of its co-ion and contributes
Ni, Cu and Sn. The process of coating of iron surface to the total equivalent conductance of an electrolyte a
with zinc is known as galvanization. Galvanized iron definite share which depends only on its own nature.
sheets maintain their lustre due to the formation of From this definition we can see that option (d) is the
protective layer of basic zinc carbonate. correct answer.
1 1 88. (c) Writing the equation for pentane-oxygen fuel cell at
76. (b) H + + e - ¾¾ ® H 2 ; E H = ´ 2 = 1; respective electrodes and overall reaction, we get
2 2
At Anode:
63.5
Cu 2 + + 2e - ¾¾
® Cu; ECu = = 31.75
2 C5 H12 + 10H 2O ® 5CO 2 + 32H + + 32e -
(pentane)
WCu WH 0.504
= or WCu = ´ 31.75 = 16 g At Cathode:
E Cu EH 1
77. (b) Magnesium provides cathodic protection and prevent 8O 2 + 32H + + 32e - ® 16H 2O
rusting or corrosion. Overall :C5H12 + 8O2 ® 5CO2 + 6H 2O
78. (b) In H 2 - O 2 fuel cell, the combustion of H2 occurs to
Calculation of DG° for the above reaction
create potential difference between the two electrodes DG° = [5×(–394.4) + 6× (–237.2)]
79. (b) During charging, the lead storage battery behaves like – [–8.2]
an electrolytic cell. So, at anode the reaction is = – 1972.0 – 1423.2 + 8.2 = – 3387.0 kJ
PbSO 4 + 2H 2 O ¾¾ ® PbO 2 + 4H + + SO 42 - + 2e - = – 3387000 Joules.
80. (d) At cathodes : Fe+ + 2e - ¾¾ ® Fe; From the equation we find n = 32

Fe3+ + 3e - ¾¾
® Fe ° and substituting
Using the relation, DG° = – nFEcell
At. wt. At. wt. various values, we get
(EFe )1 = ; (E Fe ) 2 =
2 3 ° (F = 96500C)
– 3387000 = –32×96500× Ecell
(E Fe )1 3
Hence, = ° = 3387000
(E Fe ) 2 2 or Ecell
32 ´ 96500
81. (c) 2NH 4 Cl + Zn ® 2NH 3 + ZnCl 2 + H 2 ­ .
3387000 3387
Disch arg e = or V = 1.0968 V
82. (d) ˆˆˆˆˆˆ†
Pb + PbO 2 + 2H 2SO 4 ‡ˆˆˆˆˆˆ 2PbSO 4 + 2H 2 O. 3088000 3088
Rech arg e
Thus option (c) is correct answer.
Sulphuric acid is consumed on discharging. 89. (d) Cu2+ + 2e– ® Cu;
83. (b) For spontaneous reaction E0cell must be positive. DGo = – nEoF = – 2 × F × 0.337 = – 0.674 F ....(i)
Cu+ ® Cu2+ + e– ;
Zn 2+ ¾¾
® Zn; E 0 = -0.76V DGo = – nEoF = – 1 × F × – 0.153 = 0.153 F ....(ii)
On adding eqn (i) & (ii)
Cu 2 + ¾¾® Cu; E 0 = -0.34V Cu+ + e– ® Cu ;
Here Cu acts as cathode and zinc acts as anode. DGo = – 0.521 F = – nEoF;
Here n = 1 \ Eo = + 0.52 V
E°cell = E C
°
- E°A = (-0.34) - (-0.76) 90. (a) Q Q = i × t
= 0.42 V. \ Q = 4.0 × 104 × 6 × 60 × 60 C = 8.64 × 10 8 C
DG Al3+ + 3e- ¾¾
® Al
84. (b) Efficiency of a fuel cell (h) = 3´ 96500C 27g
DH
[3 × 96500 C liberates = 27 g of Al]
o 0.0591
85. (c) As Ecell = log K c \ 96500 C liberates 9 g of Al
n
9
8.64 × 108 C liberates = ´ 8.64 ´ 108 g Al
0.0591 96500
\ 0.46 = log K c
2 = 8.1 × 104 g of Al
91. (d) Degree of dissociation
2 ´ 0.46
\ log Kc = = 15.57 L 8.0
0.0591 a= = = 2 ´ 10 -2
or Kc = Antilog 15.57 = 3.7 × 1015 » 4 × 1015 L ¥ 400
86. (c) From the given data we find Fe3+ is strongest oxidising
ca 2
( )
1 2
agent. More the positive value of E°, more is the Ka = » ca 2 = ´ 2 ´ 10 -2 = 1.25 ´ 10 -5
tendency to get oxidized. Thus correct option is (c). (1 - a ) 32
87. (d) Kohlrausch’s Law states that at infinite dilution, each
92. (a) Cu + 2 Ag + (aq) ¾¾
® Cu 2+ (aq ) + 2 Ag (s)
Electrochemistry 425

Here, n = 2 , E°cell = + 0.46 V = L°m ° °


( NH4+ ) + L m(Cl- ) + L m( Na + )
DG° = – nE°F = – 2×0.46×96500 kJ ; – 89 kJ
1000 é ù
+L°m - ê L°m + L °m
93. (a) Dilution of strong electrolytes increases ionisation, ( OH )-
ëê ( Na )
+
(Cl )
- ú
ûú
hence ionic mobility of ions which in turn increases
equivalent conductance of the solution.
L °m( NH )L °
(
°
) + L m( NaOH) -L°m( NaCl)
94. (c) Conductivity of an electrolyte depends on the mobility 4OH = m NH 4Cl
of ions and concentration of ions. The motion of an
ionic species in an electric field is retarded by the EXERCISE - 2
oppositely charged ions due to their interionic
attraction. On dilution, concentration of electrolyte 1. (d) l¥
m ( BaSO4 ) = l
¥
+ l¥
Ba 2 + SO42 -
decreases and the retarding influence of oppositely
charged ions decreases. Therefore mobility of ions = l¥ ¥ - ¥
m ( BaCl2 ) + l m ( H 2 SO4 ) - 2l ( HCl )
increases.
95. (a) Option (ii) and (iv) are correct 1 º
= x1 + x2 - 2 x3 ; lºe = l m (BaSO 4 )
96. (c) As the value of standard reduction potential decreases 2
the reducing power increases i.e., l
Z > X > Y 2. (a) Conductivity (k) =
R.a.
( -3.0) ( -1.2) ( +0.5)

97. (b) Cu 2+ + 1e - ® Cu + E1o = 0.15V; DG1o = -n1E1o F l ælö


k= ; cell constant. ç ÷ = 1.29 ´ 100 = 129
R.a è aø
o o o
Cu + + 1e - ® Cu E2 = 0.50V; DG2 = - n2 E2 F 1
Again conductivity of 0.02M solution k = ´ 129
520
Cu 2+ + 2e - ® Cu DG° = DG°1 + DG °2 k ´ 1000 129 1000
Lm = = ´ = 1.24 ´ 10-4 S m2mol-1
– nE°F = –1 n1 E1 F + (–1) n2 E2oF
o
M 520 0.02
– nE°F = –1 (n1 E1o F + n2 E2oF) 1
3. (b) At Anode, Cl - ® Cl2 + e -
n E o + n2 E2o 0.15 ´ 1 + 0.50 ´ 1 2
E° = 1 1 =
n 2 35.5 ´ 2
Equivalent wt. of chlorine (ECl2 ) = = 35.5
E° = 0.325 V 2
98. (b) Given ESn +4 Sn +2 = + 0.15 V ECl2 ´ I ´ t 35.5 ´ 2 ´ 30 ´ 60
WCl2 = = = 1.32 gm.
96500 96500
E Cr+3 = – 0.74 V
Cr 4. (d)
5. (d) Charge on one mole of electrons = 96500 C
E °cell = E ox
° + E °red = 0.74 + 0.15 = 0.89 V W Q
6. (a) By Faraday's Ist Law of electrolysis, E = 96500
99. (d) Standard Gibbs free energy is given as DG° = – nE°F
If E°cell < 0 i.e., – ve (where Q = it = charge of ion )
DG° > 0 We know that no. of gram equivalent
Further DG° = – RT ln Keq W it 1 ´ 965 1
\ DG° > 0 and Keq < 0 = = = =
E 96500 96500 100
(where i= 1 A, t = 16×60+5 = 965 sec.)
100. (d) L °m( NH Cl) = L°m + + L°m -
4 NH 4 Cl Since, we know that
1
L°m( NaOH ) = L°m + + L°m - No. of gram equivalent 100
Na OH Normality = = = 0.01 N
Volume (in litre) 1
L°m( NaCl) = L°m + L°m 7. (b) In electrolysis of NaCl when Pt electrode is taken then
Na + Cl-
H2 liberated at cathode while with Hg cathode it forms
sodium amalgam because more voltage is required to
\ L°m °
( NH4+ ) + L m(OH- ) reduce H+ at Hg than Pt.
8. (c) As per reaction in (c), reduction occurs at iodine
electrode and oxidation at bromine electrode.
EBD_7327
426 CHEMISTRY

o
Hence, Ecell = Eo - Eo 0.0591 c 0.0591 c
I2 / I – Br2 / Br - E = E° - log 1 ; E ° = E + log 1
n c2 n c2
= 0.54 – 1.09 = – 0.55V
The negative cell potential suggests the non- 0.0591 10-2
= 0.2905 + log -3 = 0.32 V.
spontaneity of the reaction. In other cases Ecell will be 2 10
positive.
0.0591 0.32 ´ 2 0.32
9. (b) Sn 4+ 2e - ¾¾
® Sn 2+ E ° = 0.13V E° = log K c ; log K c = =
2 0.0591 0.0295
Br2 + 2e - ¾¾® 2Br - E ° = 1.08V 0.32
E° value shows Br 2 has higher reduction potential. \ Kc = 10 0.0295.
Hence 17. (b) 18. (c)
Ecell = E - - E +4 +2 19. (a) An oxidising agent with a higher reduction potential
Br2 / Br Sn / Sn
= 1.08 – 0.13 = 0.95 V will oxidise only reducing agent with a lower reduction
potential.
Now -DG = nF Ecell . 20. (a) 27 gm of Al is obtained by charge of 3 × 96500 C.
n = 2, F = 96500.
96500
-DG = 2 ´ 96500 ´ 0.95 kJ / mol. Q 1 gm of Al is obtained by charge of 3 ´ C.
27
Also, DG = -2.303RTlog K eq \ 5.12 × 103 gm of Al is obtained by charge of
96500
DG - ( -2 ´ 96500 ´ 0.95) 3´ ´ 5.12 ´ 1000 = 5.49 × 107C.
log K eq = - = 27
2.303 ´ R ´ T 2.303 ´ 8.314 ´ 298
0.059
= 32.13 21. (b) DE = E° - log [H + ]2
Keq = antilog 32.682 » 1032 2
(b) Cell reaction is, Zn + Cu 2 + ® Zn 2 + + Cu 0.059
10. = 1.30 - log (10 - 2 ) 2 = 1.418 V
2+
2
° - RT ln [Zn ]
Ecell = Ecell 22. (d) Ecell = 0; when cell is completely discharged.
nF [Cu 2+ ]
æ é Zn 2 + ù ö
0.059
é ( Zn 2 + ) ù Ecell = E°cell - log ç ë û÷
2 ç é 2+ ù ÷
Greater the factor ê 2+
ú , less is the EMF ç Cu
èë ûø
÷
ëê (Cu ) ûú
Hence E1 > E2 æ é Zn 2 + ù ö
0.059
11. (b)
or 0 = 1.1 - log ç ë û÷
12. (b) It is the correct statement. 2 ç é 2+ ù ÷
ç Cu ÷
èë ûø
13. (b) L° + L° = 248 ´ 10 -4 Sm 2 mol-1
Na + OH -
æ é Zn 2+ ù ö
log ç ë û ÷ = 2 ´1.1 = 37.3
L° + L° = 126 ´ 10 Sm mol -4 2 -1
ç é 2+ ù ÷ 0.059
Na + Cl - ç Cu ÷
èë ûø
L° + L° = 280 ´ 10 -4 S m 2 mol-1
Ba 2 + 2Cl - é Zn 2+ ù
ë û = 1037.3
\
Now, L °Ba(OH)2 = L °BaCl2 + 2L°NaOH - 2L NaCl
0
éCu 2+ ù
ë û
23. (d) From the given representation of the cell, Ecell can be
L°Ba(OH)2 = 280 ´ 10-4 + 2 ´ 248 ´ 10-4 - 2 ´ 126 ´ 10-4 found as follows.

[ Cr 3+ ]
2
L °Ba(OH) 2 = 524 ´ 10 -4 Sm 2 mol -1.
1
(
Ecell = E o
Fe2 + / Fe
- Eo
Cr 3+ / Cr ) -
0.059
6
log
[Fe 2 + ]3
14. (b) k = ´ Cell constant [Nernst -Equ.]
R
Cell constant = k × R; 0.012 × 55 = 0.66 cm–1. 0.059 (0.1) 2
15. (c) A cation having highest reduction potential = –0.42 – (–0.72) - log
will be reduced first and so on. However, Mg 2+
6 (0.01)3
in aqueous solution will not be reduced 0.059 0.1´ 0.1
= –0.42 + 0.72 - log
æ E° ö. 6 0.01´ 0.01´ 0.01
<E
çè Mg 2 + / Mg 1
H 2 O / H 2 + OH - ÷ø Instead water
2 0.059 10-2 0.059
would be reduced in preference. = 0.3 - log = 0.3 - ´4
6 - 6 6
16. (b) For this cell, reaction is; Zn + Fe2+ ® Zn2+ + Fe 10
= 0.30 – 0.0393 = 0.26 V
Electrochemistry 427

Hence option (d) is correct answer. 0.059


= 1.67 - log107 = 1.67 – 0.103 = 1.567
3 4
24. (c) CH3OH (l) + O (g) ® CO2 (g) + 2H2O (l)
2 2 1 l
DGr = [DGf (CO2 , g ) + 2DGf (H 2O, l)] - 29. (a) k = ´
R A
é 3 ù 1 l
êë DGf (CH3OH, l )+ 2 DGf (O 2 , g ) úû 1.3 = ´
50 A
= – 394.4 + 2 (–237.2) – (–166.2) – 0 l
= – 394.4 – 474.4 + 166.2 = – 702.6 k J = 65m-1
A
702.6
% efficiency = ´ 100 = 97% k ´ 1000
726 L=
25. (c) DG = – nFE molarity
For 1 mol of Al, n = 3 [molarity is in moles/litre but 1000 is used to convert
liter into cm3]
4 4
For mol of Al, n = ´ 3 = 4 æ 1 ö
3 3 ´ 65 m -1 ÷ ´ 1000 cm 3
çè ø
260
DG 966´103 =
or E = = = -2.5 V 0.4 moles
-nF -4´96500
\ The potential difference needed for the reduction 650 m-1 1
= 2.5 V. = ´ m3 = 6.25 × 10–4 S m2 mol–1
260 ´ 4 mol 1000
26. (a) The value of E ° for given metal ions are 30. (d) For a spontaneous reaction DG must be –ve
M 2+ M
Since DG = – nFE°
E° = -1.18 V, Hence for DG to be -ve DE° has to be positive. Which
Mn 2+ Mn is possible when X = Zn, Y = Ni

E° = -0.9 V, Zn + Ni++ ¾¾
® Zn++ + Ni
2+
Cr Cr
E° + E°
Zn / Zn +2 Ni 2+ / Ni= 0.76 + (–0.23)
E° = -0.44 V and = +0.53
Fe2+ Fe
(positive)
E° = -0.28 V. 31. (b) ˆˆ† H+ + OH-
H 2O ‡ˆˆ
Co 2+ Co
1
The correct order of E ° 2+ values without H+ + e- ¾¾ ® H2
M M 2
considering negative sign would be \ 0.5 mole of H2 is liberated by 1 F = 96500 C
Mn2+ > Cr2+ > Fe2+ > Co2+. 0.01 mole of H2 will be liberated by
1 96500
27. (c) H+ + e– ¾¾
® H2 = ´ 0.01 = 1930 C
2 0.5
Q=I ×t
0.059 [P(H2 )]1/ 2
E = Eº – log Q 1930 C
1 [H + ] t= = = 19.3´10 4 sec
I 10 ´10-3 A
Now if pH 2 = 2 atm and [H+] = 1M
32. (d) AgNO3 ( aq ) + KCl( aq ) ¾¾
® AgCl(s ) + KNO 3 (aq )
1/ 2 Conductivity of the solution is almost compensated
0.059 2 -0.059
then E = 0 – log = log 2 due to formation of KNO3(aq). However, after end
1 1 2
28. (d) Here n = 4, and [H+] = 10– 3 (as pH = 3) point, conductivity increases more rapidly due to
Applying Nernst equation addition of excess AgNO3 solution.
33. (d) Higher the value of standard reduction potential,
0.059 [Fe ]2+ 2 stronger is the oxidising agent, hence MnO4– is the
E = Eº – log + 4 strongest oxidising agent.
n [H ] ( pO2 )
E° = – 0.41 V E°Fe3+ / Fe 2 + = + 0.77 V
34. (d) Cr 3+ / Cr 2 +
0.059 (10-3 )2
= 1.67 - log E°
4 (10-3 ) 4 ´ 0.1 Mn3+ / Mn 2 + = + 1.57 V,, E°Co3+ / Co 2 + = + 1.97 V
EBD_7327
428 CHEMISTRY

35. (c) Given Fe+3/Fe2+ = + 0.77 V \ Equivalent masses are in the ratio
and I2/2I– = 0.536V 1 2 1
2 (e– + Fe+3 ¾¾ ® Fe+2) E° = 0.77 V : or :1 or 1 : 3
3 2 3
2I – ¾¾ ® I2 + 2e– E° = – 0.536 V 43. (b) For Zn 2+ ® Zn
2Fe+3 + 2I– ¾¾ ® 2Fe+2 + I
2
E = E° -
2.303RT
log
[ Zn ]
E° = E°ox + E°red Zn 2 + /Zn Zn 2+ /Zn nF é Zn 2+ ù
= 0.77 – 0.536 ë û
= 0.164 V 0.06 1
\ Since value of E° is + ve reaction will take place. = -0.76 - log = -0.76 - 0.03
2 [ 0.1]
36. (a) Higher the value of reduction potential higher will be
the oxidising power whereas the lower the value of E = -0.79V
Zn 2 + /Zn
reduction potential higher will be the reducing power. 44. (d) Cell constant = l/a
Unit = m/m2 = m–1.
37. (d) L °CH3COOH = L°CH3COONa + L°HCl - L °NaCl
45. (d) Larger the size, lower the speed.
= 91 + 425.9 – 126.4 = 390.5 S cm2mol–1 46. (d) 2H + + 2e - ¾¾® H 2
38. (b) According to Kohlrausch’s law, molar conductivity of
weak electrolyte acetic acid (CH3 COOH) can be Ecell = 0.06 log [H+] = 0.06 ´ log K a C
calculated as follows: = 0.06 × log 10–3 = –0.18 V
(
LoCH3COOH = LoCH3COONa + LoHCl - L oNaCl ) 47. (b) Given I = 3.86 amp
t = 41 min 40 seconds = 2500 seconds
o
\ Value of L NaCl should also be known for
Mass deposited = ZIt
o
calculating value of L CH 3COOH . Where
39. (b) Given Equivalent weight
Z=
Fe3+ + 3e - ® Fe , 96500
E° = –0.036 V … (i) 40
Fe3+ / Fe Equivalent weight of Ca = (Ca2+ ® Ca) = 20
2+ - 2
Fe + 2e ® Fe ,
20
E° 2+ = –0.439V … (ii) Z= ´ 3.86 ´ 2500 = 2
Fe / Fe 96500
we have to calculate
48. (b) We know, R µ l or R = r æç l ö÷ , where proportionality
Fe3+ + e - ® Fe 2 + , DG = ? A è Aø
To obtain this equation subtract equ (ii) from (i) we get constant r is called resistivity. If l = 1m and A = 1m2,
Fe3+ + e – ® Fe 2 + … (iii) then R = r i.e., Resistance = Resistivity..
As we know that DG = – nFE 49. (d) The specific conductivity decreases while equivalent
Thus for reaction (iii) and molar conductivities increase with dilution.
DG = DG1 - DG2 50. (a) Zinc metal which has a more negative electrode
potential than iron will provide electrons in preference
– nFE° = – nFE1 – (–nFE2) of the iron, and therefore corrodes first. Only when all
–nFE° = nFE2 – nFE1 the zinc has been oxidised, the iron start to rust.
–1FE° = 2× (– 0.439) – 3 × (– 0.036)
–1 FE° = – 0.770 F EXERCISE - 3
\ E° = + 0.770V Exemplar Questions
0.591
40.
o
(d) E red = E red + log[M n + ] 1. (c) Standard electrode potential of copper electrode can
n
Lower the concentration of Mn+, lower is the reduction be calculated by constructing a concentration cell
potential. composed of two half cell reactions in which
Hence order of reduction potential is : concentration of species on left hand and right hand
Q> R>S >P side are unity.
41. (d) Electrolysis of both KCl and NaCl liberates H2 at the Pt ( s) | H 2 ( g,1 bar ) | | H + ( aq., 1 M ) || Cu 2 + ( aq, 1 M ) | Cu
144444424444443 144424443
cathode and Cl 2 at the anode. But the resulting Oxidation half cell Reduction half cell
solutions will be different. 2. Electrode potential for Mg electrode varies according to
42. (c) The oxidation states of X and Y are X3+ and Y2+. Given the equation :
atomic masses are in the ratio of 1 : 2
0.059 1
Atomic mass E 2+ = E° 2 + - log
Q Eq. mass = Mg / Mg Mg / Mg 2 éëMg 2 + ùû
Oxidation state
Electrochemistry 429

0.059 +3e
E = E° + log éë Mg 2 + ùû 13. (c) Al3+ ( aq ) ¾¾¾
® Al ( s)
Mg 2 + / Mg Mg 2+ / Mg 2
Hence, total 3F is required.
0.059
E 2+ = log éë Mg 2 + ùû + E° 2 + l
Mg / Mg 2 Mg / Mg 14. (d) Cell constant (G) =
This equation represents equation of straight line. A
where l = length of object and A = area of cross section
0.059
Hence intercept (c) = E°Mg 2 + / Mg and slope = Since, l and A remain constant for any particular object
2 hence value of cell constant always remains constant.
Thus, equation can be diagrammatically represented as: 15. (a) When the lead storage battery is recharged the
reaction occurring on cell is reversed and PbSO4 (s)
on anode and cathode is converted into Pb and PbO2
respectively
®

E
Mg 2 + / Mg The electrode reactions are as follows
Anode reaction:
PbSO4 (s) +2e– ® Pb(s) + SO42– (aq) (Reduction)
log éë Mg2 + ùû ® Cathode reaction:
3. (c) Ecell is an intensive property as it is independent of PbSO4 (s) + 2H2O ® PbO2(s) + SO42– + 4H+ + 2e–
the mass of species but D rG of cell reaction is an (Oxidation)
extensive property because this depends upon mass Net reaction:
of species. 2PbSO4 (s) + 2H2O ® Pb(s) + PbO2(s) + 4H+ (aq.) +
2SO42– (aq.)
4. (b) Cell emf is the difference between the electrode
L ° m( NH = l° °
potential of two electrodes when no current is drawn
through the cell.
16. (b) 4Cl) ( NH 4+ ) + l ( Cl– ) ...(i)

L °m ( NaOH ) = l °( + l (°
Na + ) OH + )
5. (d) Inert electrode does not participate in redox reaction ...(ii)
and acts only as source or sink for electrons. It
L °m ( NaCl) = l °( + l °(
Na + ) Cl– )
provides surface either for oxidation or for reduction ...(iii)
reaction. On adding (i) & (ii) and subtract (iii), we get :
6. (c) If an external opposite potential is applied on the L °m( NH + L °m( NaOH ) - L °m ( NaCl) = L °m( NH OH )
galvanic cell and increased slowly. It is observed that 4Cl ) 4

the reaction continues to take place till the opposing 17. (d) In the electrolysis of aqueous NaCl, following reactions
voltage reaches the value 1.1V. At this stage no current are possible at anode.
flow through the cell. Any further increase in the 1
At anode; Cl - ( aq ) ¾¾ ® Cl2 ( g ) + e - ;
external potential restarts the reaction but in the 2
opposite direction. E°cell = 1.36V
Hence, now the cell will behave like an electrolytic cell.
7. (c) Greater the solvation of ions of an electrolyte lesser ® O 2 ( g ) + 4H + ( aq ) + 4e - ;
At cathode; 2H 2 O ( l ) ¾¾
will be the electrical conductivity of the solution. E°cell = -1.23 V
8. (b) Among the given options chromium has highest The reaction at anode with lower value of E° is preferred
negative value of E° hence, it is the most strongest and therefore water should get oxidised in preference
reducing agent. to Cl–(aq). However on account of overpotential of
9. (c) Higher the positive value of standard reduction oxygen reaction (d) is preferred.
potential of metal ion higher will be its oxidising
capacity. NEET/AIPMT (2013-2017) Questions
Since, E° has highest positive value 18. ® 2H+ + 2e–
(a) H2 ¾¾
MnO4- / Mn 2 +
hence it is the strongest oxidising agent. 1 atm 10-10
10. (b) On moving down in electrochemical series reducing

( )
power decreases as the value of electrode potential 2
E 0.059 10-10
decreases. H 2 /H + =0– log
2 1
11. (d) E° has highest positive value. So Mn2+
MnO4- / Mn 2 +
is most stable ion in its reduced form. E = +0.59 V
H 2 /H +
12. (d) E° has the lowest value of SRP. Hence, Cr3+
Cr3+ / Cr
Lm 9.54
is the most stable oxidised species. 19. (b) a = L ¥ = = 0.04008 = 4.008 %.
m 238
EBD_7327
430 CHEMISTRY

20. (d) E°Cell = E°OP + E°RP 26. (b) HCl completely dissociates to give Hr and Clss ions,
hence act as very good electrolyte. While others are
= 0.76 + 0.314 = 1.10 V non- electrolytes.
21. (c) Applying, 27. (a) 2H+(aq) + 2e– ® H2(g)
Eit 0.0591 PH2
w = Zit = \ E = E0 – log
96500
Equivalent weight of cobalt (II) = 59/2
2 [ H + ]2
I = 10 A PH 2
Time (t) = 109 min = 109 × 60 sec 0 = 0 – 0.0295 log
Substituting these values we get, (10-7 )2
59 ´ 10 ´ 109 ´ 60 PH 2
w= = 20.0
2 ´ 96500 (10-7 )2 = 1
22. (a) DE° = E°red + E°oxd PH2 = 10–14 atm
= -1.81 - 1.51 = -2.69 28. (c) Li+ being smallest, has maximum charge density.
Since DE° is negative \ Li+ is most heavily hydrated among all alkali metal
\ DG = –nFE°, DG will have positive value so, forward ions. Effective size of Li + in aqueous solution is
reaction is not possible. therefore, largest. So, moves slowest under electric field.
29. (b) For cell,
( +6) ( +7)
Zn|ZnSO4(0.01 M) || CuSO4(1.0 M)|Cu
23. (c) Mn O 24 - ¾® MnO4- + e -
0.1 mole 2.303RT log [ Zn 2+ ]
E cell = E°cell -
Quantity of electricity required = 0.1F nF [ Cu 2+ ]
= 0.1 × 96500 = 9650 C ( 0.01)
2.303RT
24. (d) wO2 = nO2 × 32 \ E1 = E°cell - ´ log
2´ F 1
5600 When concentrations are changed for ZnSO4 and
wO2 = ´ 32 = 8g = 1 equivalent of O2
22400 CuSO4, we can write
= 1 equivalent of Ag = 108 2.303RT 1
25. (d) A device that converts energy of combustion of fuels, E2 = E°cell - ´ log
2F 0.01
directly into electrical energy is known as fuel cell.
\ E1 > E2
Chemical Kinetics 431

18 Chemical Kinetics

RATE OF A CHEMICAL REACTION


It is defined as the change in the concentration of reactant or
product in unit time. It is always a positive quantity. It can be
expressed in terms of: Conc. of Conc. of
(i) rate of decrease in concentration of any one of the reactant Reactant Product
(ii) rate of increase in concentration of any one of the product.
For ex; For a reaction, A ® B
Time Time
Decrease in conc. of A
Rate of disappearance of A = FACTORS AFFECTING RATE OF A REACTION
Time taken
(i) Nature of reactants : Reactions involving lesser bond
-D [ B ] rearrangement proceed much faster than those which involve
= larger bond rearrangement.
Dt
(ii) Concentration of reactants : More will be the concentration
Increase in conc. of B +D [ B] of reactant greater will be the rate of reaction.
Rate of appearance of B = =
Time taken Dt (iii) Temperature : When temperature increases, the rate of
reaction increase because large number of molecules cross
where D[A] = A2 – A1 = Afinal – Ainitial
the energy barrier. For all reactions, rate of reaction increases
D[B] = B2 – B1 = Bfinal – Binitial
with increase in temperature whether reaction is exothermic
Dt = t2 – t1 = change in time or endothermic.
This is average rate of reaction, rav. which is rate of reaction (iv) Presence of catalyst: Catalyst increased rate without itself
over a certain measurable period of time during the course of getting consumed in reaction
reaction. (v) Surface area of reactants : The more the surface area, the
To express the rate at a particular moment of time, we determine more is the rate of reaction.
the instantaneous rate, rinst. It is given by: (vi) Presence of light : Some reactions do not occur in dark but
-D [ A ] D [ B] occur in presence of light. Such reactions are known as
rav = = (Q time interval is the smallest ) photochemical reactions.
Dt Dt
As t ® 0, RATE LAW, RATE CONSTANT, ORDER OF REACTION
- d [ A ] d [ B] Rate Law
rinst = =
dt dt For a general reaction,
aA + bB ® cC + dD
Rate of Reaction in the Form of Stoichiometry of a
the rate expression is given by:
Chemical Reaction Rate µ [A]a[B]b (Law of mass action)
Let us consider a reaction :
x
m1A + m2B + ------- ¾¾® n1P + n2Q + n3R + --------- Rate µ [ A ] [ B] y Rate Law or Rate expression

Rate of reaction where x and y may or may not be equal to a and b


Again, Rate = k [A]x [B]y
1 d [ A] 1 d [ B ] 1 d [ P ] 1 d [Q ] -d [ R ]
= k [ A ] [ B]
x y
=– =- = =
m1 dt m2 dt n1 dt n2 dt dt
This is called Differential rate equation. This equation which
Units of rate of reaction are conc/time i.e., mol L–1s–1. relates the rate of reaction to concentration of reactants is called
Graphically, it is shown as: rate law or rate expression. Rate law or for any reaction is determined
experimentally.
EBD_7327
432 CHEMISTRY

Rate Constant reaction are:


k is constant of proportionality called Rate constant/ Velocity Difference Between Order and Molecularity of
constant/ specific reaction rate. Reaction
[A] = [B] = 1
Rate = k Order of reaction Molecularity of reaction
In general rate constant may be defined as the rate of the reaction 1. It can be fractional as 1. It is always a whole
when the concentration of each of the reactants is unity. well as zero. number. It cannot be
Characteristics of rate constant : zero or fractional.
(i) The value of k is different for different reactions.
2. It can be determined 2. It can be calculated by
(ii) At fixed temperature the value of k is constant.
(iii) It is in dependent of concentration but depends on experimentally only. simply adding the
temperature. molecules of the slowest
(iv) The larger the value of k the faster is the reaction and vice step.
versa. 3. It is applicable to 3. It is applicable only to
‘k’ for different reaction order are given below: elementary as well as elementary reactions.
Order of complex reactions. For a complex reaction,
Units of rate constant molecularity of the
reaction
slowest step is same as
Zero
(mol L-1 )1-0 s-1 = mol L-1s -1 the overall order of
( n = 0) reaction.
First order
(mol L-1)1-1s -1 = s-1 4. It cannot be obtained 4 It can be obtained.
( n = 1) from balanced or
Second order stoichiometric
(mol L-1 )1- 2 s -1 = mol L-1s -1
( n = 2) equation.
5 It is changed when one 5 It undergoes no change.
Order of reaction reactant is taken in
It is defined as the sum of the exponents to which the concentration large excess
terms in the rate law expression are raised to express the observed
rate of reaction. INTEGRATED RATE EQUATIONS
For the reaction The integrated rate equations are different for reactions of different
aA + bB ¾¾® Products orders. Some of these are:
then, Rate = k [A]a [B]b, Zero Order Reaction
then the sum of exponents of concentration terms X®Y
= a + b = Order of reaction
-d [ X ] 0
where a = Order with respect to reactant A Rate = = k [ X]
b = Order with respect to reactant B dt
(i) The order of a reaction may be zero, (–)ve, (+)ve or fractional. -d [ X ]
= k Þ d [ X ] = - kdt
(ii) The order of a reaction is determined by the slowest step of dt
mechanism. On integrating both sides
(iii) Reactions of higher order (more than 3) are rare. [X] = – kt + C .......... (i)
Note : Order of a reaction is experimentally determined quantity. It At t = 0 [X] = [X]0
can be obtained from rate law equation not from balanced chemical [X]0 = C
equation. Substituting in equation (i) we get,
[X] = –kt + [X]0
Molecularity of a Reaction
[X]0 - [X]
The reactions taking place in one step are called elementary k= .......... (ii)
reactions. The reactions which do not occur in one step are called t
complex reactions. Half-life for zero order reaction :
The total number of moles of atoms or ions or molecules or reacting 1
At t = t1 2 [X] = [X]0
species in a balanced chemical equation is called molecularity of 2
the reaction. By putting these values in equation (ii)
For an elementary reaction, molecularity is the sum of the molecules
1
of different reactants as represented by balanced chemical equation [X]0 - [X]0
2 [X]0
whereas complex reaction, molecularity has no meaning. k= t1 2 =
It can only be defined for an elementary reaction. t1 2 2k
The main points of different between order and molecularity of a Graphical representation for zero order reaction
Chemical Kinetics 433

[R]0

Zero order Slope =


Conc 2.303
Rate Rate log [X]
[R]

Time, t Conc Conc t

1
t1 Slope = log t½
2R
2 [X] Slope =
2.303

[R]0
First Order Reaction t [X]0
X®Y Second Order Reaction
These are of two types :
-d [ X ] d[X] (i) 2A ¾¾ ® Products
Rate = = k [ X] , = –kdt Rate µ [A]2
dt [X]
(ii) A + B ¾¾ ® Product
On integrating both sides Rate µ [A] [B]
ln [X] = –kt + C .......... (i) For reaction, 2X ¾¾ ® Product
when t = 0, [X] = [X]0 1 X
ln [X]0 = C k= ...... (i)
t [X]0 ([X]0 - [X])
Substituting in equation (i) we get
ln [X] = –kt + ln [X]0 Half life period for second order rection :
ln [X] – ln [X]0 = – kt [ X ]0
kt = ln [X]0 – ln [X] When t = t1/2 , [X] =
2
By putting values in equation (i)
[ X]0 1 [ X ]0
1
kt = ln Þ k = ln t1 2 =
[X] t [X] k[X]0
2.303 [ X ]0 Graphical representation for second order reaction
k= log .......... (ii)
t [X]
Half-life for first order reaction :
Rate
[ X ]0 1
When t = t1/2 , [X] = [X]
2
By putting values in equation (ii) (conc)
2
t
2.303 [ X]0
k= log
t1 2 [X]0 2
t1/2
2.303 0.693
t1 2 = log 2 =
k k
Graphical representation for first order reaction 1/[X]0

On summarising we get,
Order Reaction Differential Rate Law Integrated Rate Law Half - life Units of R
d [X] Conc. time-1or
0 X®Y = -k kt = [ X ]0 - [ X ] [ X ]0 / 2k
dt mol L-1s -1
[ X ] = [ X ]0 e-kt
d [ X]
1 X®Y = -k [ X] [ X ]0 ln 2 / k time -1or s -1
dt or kt = ln
[X]
[X] 1
2 2X ® Product k= conc -1 time -1or mol s -1
t[X]0 ([X]0 - [X]) k[X]0
EBD_7327
434 CHEMISTRY

Half-life Period for the nth Order Reaction Temperature Dependence of Reaction
When the order of reaction is n, t1/2 is given by For a chemical reaction, with 10°C rise in temperature, the rate
1 é 2n -1 - 1 ù constant is nearly doubled. This is because on increasing the
t1/ 2 = ê ú temperature, the fraction of molecules colliding with energies
k(n - 1) êë a n -1 úû
greater than Ea increases. At (T + 10)°C, the fraction of molecules
1 having energy equal to or greater than E a gets doubled leading to
\ t1 / 2 µ n -1 doubling the rate of a reaction.
a
Unit of k = (conc)1–n. time–1 Diagrammatically, it is shown as:
For a first order gas phase reaction, A(g) ® B(g) + C(g)

2.303 p 2.303 pi T
k= log i = log
t pA t 2pi - pt T + 10°

Fraction of molecules
where pi is initial pressure of A, pt ® total pressure at time t
(pt = pA + pB + pC)
PSEUDO FIRST ORDER REACTIONS
Reactions are not truly of first order but under certain conditions
become reactions of first order are called pseudo-first order Most
reactions. probable
For ex: kinetic
(i) acid-catalysed hydrolysis of ethyl acetate energy

H+ Kinetic energy
CH 3COOC2 H 5 + H 2 O ¾¾¾
® CH 3COOH + C 2 H 5OH

Rate of reaction µ [ CH3COOC2 H5 ] Mathematically, temperature dependence of rate of reaction is


(ii) acid-catalysed inversion of cane sugar given by Arrhenius equation given as:

H+ k = Ae -Ea / RT ...(i)
C12 H 22 O11 + H 2O ¾¾¾
® C6 H12O6 + C6 H12O6
Where A ® Arrhenius factor or frequency factor or pre-exponential
Rate of reaction µ [C12 H 22O11 ] factor
This is because that water is present in such large excess R ® gas constant
that its concentration remains almost constant during the Ea ® activation energy
reaction. If rate constant of a reaction at a particular temperature is known,
ACTIVATION ENERGY its value at another temperature can be calculated, provided Ea is
The minimum energy which the colliding molecules must have in known. It is as follows:
order that the collision between them may be effective is called Taking logerithm on both sides of equation (i)
threshold energy. Ea
The minimum extra amount of energy absorbed by reactant lnk = ln A –
RT
molecules so that their energy becomes equal to threshold value
is called activation energy. Thus, If the value of rate constant at temperatures T1 and T2 are k1 and
Activation energy = Threshold energy – Avg KE of reactant k2
For fast reaction activation energy is low while for slow reaction lnk1 = lnA – Ea/RT1
activation energy is high. lnk2 = lnA – Ea/RT2
Graphically, it is shown as: On substracting the two equations we get
Energy barrier
Et k2 Ea æ 1 1 ö
(Activated complex)
ln = R çT –T ÷
k1 è 1 2 ø
Threshold Activation
Potential energy

energy Ea æ 1
Ea energy k 1ö
ER log 2 = -
k1 2.303R çè T1 T2 ÷ø
Reactants Energy EFFECT OF CATALYST ON RATE OF REACTION
evolved
DH A catalyst alters the rate of a reaction without itself undergoing
EP any permanent chemical change.
Products
The catalyst provides an alternate pathway or reaction mechanism
by reducing the activation energy between reactants and products
Reaction of coordinate and hence lowering the potential energy barrier.
Chemical Kinetics 435

The graphical representation is as follows: Collision Frequency


Total number of collisions which occur among the reacting
molecules per second per unit volume is called collision frequency.
Its value is given by
Activated Complex
z= 2 p n s2 n 2

Ea Ea n = average velocity, s = molecular diameter in cm.,


n = number of molecules per cc.
Ea(c) Rate of Reaction from Collision Theory
E
Ea (c) It is given by
Reactants Rate of reaction = f × z
DH
z = collision frequency,
Products f = fraction of effective collisions
Progress of Reaction
Dn
= = e - E a / RT
N
Where,
Ea = Activation energy without catalyst \ Rate (k) = Ze- E a /RT
Ea(c) = Activation energy with catalyst. For effective collision another factor P, called steric factor has to
COLLISION THEORY OF CHEMICAL REACTIONS be taken into consideration.
Then rate of reaction can be expressed as
Chemical reaction occurs as a result of effective collision between
reaction molecules. For this two things are important. Rate = PZAB e–Ea/RT
Where P ® Probability or steric factor
(i) proper orientation of reaction molecules
ZAB ® collision frequency of reactants A and B
(ii) possession of energy more than threshold energy.
e–Ea/RT ® fraction of molecules with energies equal to or
greater than Ea.
436

CONCEPT MAP
CHEMISTRY

EBD_7327
Chemical Kinetics 437

1. The rate constant of first order reaction is 3 × 10–6 per second. 7. For reaction aA ® xP , when [A] = 2.2 mM, the rate was
The initial concentration is 0.10 M. The initial rate is: found to be 2.4 mMs–. On reducing concentration of A to
(a) 3 × 10–7 moles/litre/sec half, the rate changes to 0.6 mMs–1. The order of reaction
(b) 3 × 10–8 moles/litre/sec with respect to A is :
(c) 3 × 10–5 moles/litre/sec (a) 1.5 (b) 2.0
(d) 3 × 10–8 moles/litre/sec (c) 2.5 (d) 3.0
2. Which of the following statement is true for the reaction,
8. The rate of reaction is doubled for every 10°C rise in
dx
H 2 + Br2 ® 2HBr . The rate law is = k [ H 2 ][ Br2 ]1/ 2 : temperature. The increase in reaction rate as a result of
dt temperature rise from 10°C to 100°C is :
(a) order of reaction is 1.5
(a) 614 (b) 400
(b) molecularity of the reaction is 2
(c) by increasing the concentration of Br 2 four times the (c) 512 (d) 112
rate of reaction is doubled 9. In the reaction 2A + B ® A 2 B, if the concentration of A is
(d) all the above are correct.
doubled and that of B is halved, then the rate of the reaction will:
3. The rate constant k, for the reaction
1 (a) increase 2 times (b) increase 4 times
N 2 O 5 (g ) ¾
¾® 2 NO 2 (g ) + O 2 ( g )
2 (c) decrease 2 times (d) remain the same
is 1.3 × 10–2s–1. Which equation given below describes the 10. Which of the following graphs represent relation between
change of [N2 O 5 ] with time ? [N2 O 5 ]0 and [N2 O5 ] t initial concentration of reactants and half-life for third order
corrospond to concentration of N2O5 initially and at time t. reaction?
(a) [N2O5]t = [N2O5]0 + kt
(b) [N2O5]0 = [N2O5]t ekt (a) (b)
(c) log [N2O5]t = log [N2O5]0 + kt
[N 2 P5 ]0 t1/2 t1/2
(d) In [N P ] = kt
2 5 t
4. For the reaction 2 A + B ® 3C + D
Which of the following does not express the reaction rate ? a
a
d[ B] d[ D]
(a) - (b)
dt dt (c) (d)
1 d[A] 1 d[C]
(c) – (d) , t1/2
2 dt 3 dt t1/2
5. Consider the reaction : N2 (g) + 3H2 (g) ® 2 NH3 (g)
d[ NH 3 ] d[H 2 ]
The equality relationship between and - is 1/a
2
dt dt 1/a
d[ NH 3 ] 2 d[ H 2 ] d[ NH 3 ] 3 d[ H 2 ] 11. The differential rate law for the reaction
(a) + =- (b) + =-
dt 3 dt dt 2 dt H2 (g) + I2 (g) ® 2HI (g) is
d[ NH 3 ] d[H 2 ] d[ NH 3 ] 1 d[H 2 ] d[ H 2 ] d[ I 2 ] d[ HI ]
(c) =- (d) =- (a) - =- =-
dt dt dt 3 dt dt dt dt
6. For the reaction system :
d[H 2 ] d[I 2 ] 1 d[Hl ]
2NO( g) + O 2 ( g ) ® 2 NO2 ( g ) volume is suddenly reduced (b) = =
dt dt 2 dt
to half its value by increasing the pressure on it. If the reaction
is of first order with respect to O2 and second order with 1 d[H 2 ] 1 d[I 2 ] d[Hl]
respect to NO, the rate of reaction will (c) = =-
2 dt 2 dt dt
(a) diminish to one-eighth of its initial value
(b) increase to eight times of its initial value d[ H 2 ] d[ I 2 ] d[ HI ]
(c) increase to four times of its initial value (d) -2 = -2 =
dt dt dt
(d) diminish to one-fourth of its initial value
12. Select the rate law that corresponds to data shown for the
EBD_7327
438 CHEMISTRY

following reaction initially with [A] = 2.00 M and after 200 min, [A] becomes
A+B ¾ ¾® products. 0.15 M. For this reaction t1/2 is
(a) 53.72 min (b) 50.49 min (c) 48.45 min (d) 46.45 min
Exp. [A] [B] Initial rate
22. After how many seconds will the concentration of the
1 0.012 0.035 0.1
reactants in a first order reaction be halved, if the decay
2 0.024 0.070 0.8 constant is 1.155 × 10–3 sec–1.
3 0.024 0.035 0.1 (a) 200 (b) 400 (c) 600 (d) 800
4 0.012 0.070 0.8 23. The plot of concentration of the reactant vs time for a reaction
(a) rate = k [B]3 (b) rate = k [B]4 is a straight line with a negative slope. The reaction follows
(c) rate = k [A] [B]3 (d) rate = k [A]2 [B]2 a rate equation
(a) zero order (b) first order
13. For the reaction 2N 2O5 ® 4NO2 + O 2 , rate and rate (c) second order (d) third order
constant are 1.02 × 10–4 mol lit–1 sec–1 and 3.4 × 10–5 sec–1 24. Half life of a first order reaction is 4s and the initial
respectively then concentration of N2O5 at that time will be concentration of the reactant is 0.12 M. The concentration
(a) 1.732M (b) 3M of the reactant left after 16 s is
(c) 3.4 × 105M (d) 1.02 × 10–4M (a) 0.0075 M (b) 0.06 M (c) 0.03 M (d) 0.015 M
14. For a first order reaction, to obtain a positive slope, we need 25. 3A ® B + C , it would be a zero order reaction when
to plot {where [A] is the concentration of reactant A} (a) the rate of reaction is proportional to square of
(a) – log10[A] vs t (b) – loge[A] vs t concentration of A
(c) log10[A] vs log t (d) [A] vs t (b) the rate of reaction remains same at any concentration
15. In a first order reaction the concentration of reactant of A
decreases from 800 mol/dm3 to 50 mol/dm3 in 2 × 104 sec. (c) the rate remains unchanged at any concentration of B
The rate constant of reaction in sec–1 is: and C
(a) 2 × 104 (b) 3.45 × 10–5 (d) the rate of reaction doubles if concentration of B is
(c) 1.386 × 10–4 (d) 2 × 10–4 increased to double
26. A first order reaction is 50% completed in 30 minutes at 27ºC.
16. The rate constant for the reaction 2 N 2 O 5 ¾¾® 4 NO 2 + O 2
Its rate constant is :
is 3.10 × 10–5 sec–1. If the rate is 2.4 × 10–5 mol litre–1 sec–1 (a) 2.31 × 10–2 minute–1 (b) 3.21 × 10–5 minute–1
then the concentration of N 2 O 5 (in mol litre–1) is : –2
(c) 4.75 × 10 minute –1 (d) 1.33 × 10–3 minute–1
(a) 0.04 (b) 0.8 (c) 0.07 (d) 1.4 27. The reaction A ® B follows first order kinetics. The time
17. T50 of first -order reaction is 10 min. Starting with 10 mol L–1, taken for 0.8 mole of A to produce 0.6 mole of B is 1 hour.
rate after 20 min is What is the time taken for conversion of 0.9 mole of A to
(a) 0.0693 mol L–1 min–1 produce 0.675 mole of B?
(b) 0.0693 × 2.5 mol L–1 min–1 (a) 2 hours (b) 1 hour (c) 0.5 hour (d) 0.25 hour
28. Out of the following reaction which will proceed in forward
(c) 0.0693 × 5 mol L–2 min–1
direction if volume of container is increased?
(d) 0.0693 × 10 mol L–1 min–1
18. Velocity constant K of a reaction is affected by (a) H 2 + Cl 2 ¾ ¾® 2HCl
(a) Change in the concentration of the reactant (b) CaCO 3 ¾
¾® CaO + CO 2
(b) Change of temperature (c) N 2 + 3H 2 ¾¾ ® 2NH 3
(c) Change in the concentration of the product (d) N 2 + 2O 2 ¾ ¾® N 2 O 4
(d) None of the above 29. The rate of a first order reaction is 1.5 × 10–2 mol L–1 min–1 at
19. The first order rate constant for a certain reaction increases 0.5 M concentration of the reactant. The half life of the
from 1.667 × 10–6 s–1 at 727ºC to 1.667 × 10–4 s–1 at 1571ºC. reaction is
The rate constant at 1150ºC, assuming constancy of (a) 0.383 min (b) 23.1 min (c) 8.73 min (d)7.53 min
activation energy over the given temperature range is 30. For a first order reaction A ¾® B the reaction rate at
[Given : log 19.9 = 1.299] reactant concentration of 0.01 M is found to be
(a) 3.911 × 10–5 s–1 (b) 1.139 × 10–5 s–1 2.0 ´10-5 mol L-1 s-1. The half life period of the reaction
(c) 3.318 × 10 s –5 –1 (d) 1.193 × 10–5 s–1 is
20. For the following reaction scheme (homogeneous), the rate (a) 30 s (b) 220 s (c) 300 s (d) 347 s
31. For a chemical reaction which can never be a fraction :
constant has units : A + B ¾¾
K
®C (a) rate constant (b) order
(a) sec–1 mole (b) sec –1 (c) molecularity (d) half life
(c) sec–1 litre mole–1 (d) sec 32. If half-life of a substance is 5 yrs, then the total amount of
21. A substance 'A' decomposes by a first order reaction starting substance left after 15 years, when initial amount is 64 grams
Chemical Kinetics 439

is [A] [B] [C] rate (M/sec.)


(a) 16 grams (b) 2 grams (c) 32 grams (d) 8 grams. 0.2 0.1 0.02 8.08 × 10–3
33. In a first order reaction, the concentration of the reactant, 0.1 0.2 0.02 2.01 × 10–3
decreases from 0.8 M to 0.4 M in 15 minutes. The time taken 0.1 1.8 0.18 6.03 × 10–3
for the concentration to change from 0.1 M to 0.025 M is 0.2 0.1 0.08 6.464 × 10–2
(a) 7.5 minutes (b) 15 minutes (a) –1, 1, 3/2 (b) –1, 1, 1/2
(c) 30 minutes (d) 60 minutes (c) 1, 3/2, –1 (d) 1, –1, 3/2
34. The concentration of a reactant X decreases from 0.1 M to 44. For a chemical reaction t1/2 is 2.5 hours at room temperature.
0.005 M in 40 min. If the reaction follows first order kinetics, How much of the reactant will be left after 7.5 hours if initial
the rate of the reaction when the concentration of X is 0.01 weight of reactant was 160 gm?
M will be (a) 10 gm (b) 40 gm (c) 80 gm (d) 20 gm
45. For the reaction,
(a) 1.73 × 10–4 M min–1 (b) 3.47 × 10–4 M min–1
(c) 3.47 × 10–5 M min–1 (d) 7.5 × 10–4 M min–1 H+
CH3COCH3 + I2 ¾¾¾ ® products
35. The order of a reaction with rate equal to k[A]3/2 [B]–1/2 is : The rate is governed by expression
1 3 dx
(a) 1 (b) - (c) - (d) 2 = k[acetone][H + ]
2 2 dt
36. The rate constant for a first order reaction whose half-life, is The order w.r.t. I2 is:
480 seconds is : (a) 1 (b) 0 (c) 3 (d) 2
(a) 2.88 × 10–3 sec–1 (b) 2.72 × 10–3 sec–1 46. In the following first order competing reactions
(c) 1.44 × 10 sec–3 –1 (d) 1.44 sec–1 X + reagent ® product
37. In a reaction, when the concentration of reactant is increased Y + reagent ® product
two times, the increase in rate of reaction was four times. k1
Order of reaction is : The ratio of if only 50% of Y will have been reacted
k2
(a) zero (b) 1 (c) 2 (d) 3
when 94% of X has been reacted is
38. The rate constant of a first order reaction is 6.9 ´10-3 s -1 . (a) 3.06 (b) 1.06
How much time will it take to reduce the initial concentration (c) 4.06 (d) 0.06
to its 1/8th value? 47. The rate constant of a reaction is 3.00 × 10 3 L mol–1 sec–1.
(a) 100 s (b) 200 s (c) 300 s (d) 400 s The order of this reaction will be:
39. In a 1st order reaction, reactant concentration C varies with (a) 0 (b) 1 (c) 2 (d) 3
time t as : 48. Point out the wrong statement:
1 For a first order reaction
(a) increases linearly with t (a) time for half-change (t1/2) is independent of initial
C
concentration
(b) log C decreases linearly with t
(b) change in the concentration unit does not change the
1 rate constant (k)
(c) C decreases with
t (c) time for half-change × rate constant = 0.693
1 (d) the unit of k is mole–1 min–1
(d) log C decreases with 49. Which one of the following plots is true for the first order
t
decomposition of N2O5 ?
40. A substance ‘A’ decomposes by a first order reaction starting
initially with [A] = 2.00 mol/litre and after 200 sec, [A] = 0.15
mol/litre. For this reaction, what is the value of k ?
(a) 1.29 × 10–2 sec–1 (b) 2.03 × 10–3 sec–1
rate
rate

(c) 1.80× 10 sec–3 –1 (d) 1.55 × 10–2 sec–1


41. A reaction proceeds by first order, 75% of this reaction (a) (b)
was completed in 32 min. The time required for 50%
completion is
(a) 8 min (b) 16 min (c) 20 min (d) 24 min CN C N2O5
2O5
42. For the third order reaction, 3 A ¾¾ ® products, with 0.1 M
as the initial concentration of A, t1/2 is 8 hr 20 minute. The
rate constant of the reaction is
rate

rate

(a) 5 × 10–3 lit2 mole –1 sec–1


(b) 50 lit2 mole–2 sec–1
(c) 5.0 × 10–3 lit2 mole–2 sec–1 (c) (d)
(d) 5 × 10–2 lit2 mole–2 sec–1
43. What is order with respect to A, B, C, respectively CN2O5 C N2O5
50. The first order rate constant for the decomposition of N2O5
EBD_7327
440 CHEMISTRY

is 6.2 × 10–4 sec–1. The half life period for this decomposition IV. In k vs T is a straight line.
in seconds is V. In k vs 1/T is a straight line.
(a) 1117.7 (b) 111.7 (c) 223.4 (d) 160.9 Correct statements are
51. For a certain reaction, rate = k × [H+]n. If pH of reaction (a) I and IV (b) II and V
changes from two to one, the rate becomes 100 times of its (c) III and IV (d) II and III
value at pH = 2, the order of reaction is – 60. Activation energy of a chemical reaction can be determined
(a) 1 (b) 2 (c) 0 (d) 3 by
52. The half-life of a reaction is inversely proportional to the (a) evaluating rate constant at standard temperature
square of the initial concentration of the reactant. Then the (b) evaluating velocities of reaction at two different
order of the reaction is temperatures
(a) 0 (b) 1 (c) 2 (d) 3 (c) evaluating rate constants at two different temperatures
53. Ethylene is produced by C 4 H8 ¾¾ D (d) changing concentration of reactants
® 2C 2 H 4
61. The rate law for the reaction
The rate constant is 2.3 × 10–4 sec–1. Approximately in what
time will the molar ratio of ethylene to cyclobutane in mixture xA + yB ¾¾
® mP + nQ is Rate = k [A]c [B]d. What is the
attain the value equal to one ?
(log 2 = 0.3, log 3 = 0.47) total order of the reaction?
(a) 1700 sec (b) 1600 sec (a) (x + y) (b) (m + n)
(c) 2000 sec (d) 1200 sec (c) (c + d) (d) x/y
62. The temperature dependence of rate constant (k) of a
54. t 1 can be taken as the time taken for the concentration of a chemical reaction is written in terms of Arrhenius equation,
4 *
3 k = Ae - Ea / RT. Activation energy ( Ea* ) of the reaction can
reactant to drop to of its initial value. If the rate constant
4 be calculated by plotting
t
for a first order reaction is k, the 1 can be written as
4 1
(a) log k vs (b) k vs T
(a) 0.75/k (b) 0.69/k (c) 0.29/k (d) 0.10/k log T
55. What is the half life of a radioactive substance if 87.5% of
any given amount of the substance disintegrates in 40 1 1
(c) k vs (d) log k vs
minutes ? log T T
(a) 20 minutes (b) 10 minutes 63. For a first order reaction, the time taken to reduce the initial
(c) 13 minutes 32 sec (d) 160 minutes 1
56. The rate law for the reaction 2X + Y ® Z is Rate = k[X][Y]. concentration by a factor of is 20 minutes. The time
4 1
The correct statement with regard to this relation is required to reduce initial concentration by a factor of is
(a) the unit of k is s–1 16
(a) 20 min (b) 10 min
(b) the rate of the reaction is independent of [X] and [Y]
(c) 80 min (d) 40 min
(c) for this reaction t1/2 is independent of initial
64. The activation energy for a simple chemical reaction
concentrations of reactant
(d) the rate of formation of Z is half of the rate of A ® B is Ea in forward direction. The activation energy for
disappearance of X reverse reaction
57. For an exothermic reaction, the energy of activation of the (a) is always double of Ea
reactants is (b) is negative of Ea
(a) equal to the energy of activation of products (c) is always less than Ea
(b) less than the energy of activation of products (d) can be less than or more than Ea
(c) greater than the energy of activation of products 65. Rate of a reaction can be expressed by Arrhenius equation as
(d) sometimes greater and sometimes less than that of the : k = Ae–Ea/RT
products In this equation, Ea represents
58. A chemical reaction is catalyzed by a catalyst X. Hence X (a) the total energy of the reacting molecules at a
(a) reduces enthalpy of the reaction temperature, T
(b) decreases rate constant of the reaction (b) the fraction of molecules with energy greater than the
(c) increases activation energy of the reaction activation energy of the reaction
(d) does not affect equilibrium constant of the reaction (c) the energy below which all the colliding molecules will
59. Consider the following statements: react
I. Increase in concentration of reactant increases the (d) the energy below which colliding molecules will not react
rate of a zero order reaction. 66. The minimum energy a molecule should possess in order to
II. Rate constant k is equal to collision frequency A if enter into a fruitful collision is known as
Ea = 0. (a) reaction energy (b) collision energy
III. Rate constant k is equal to collision frequency A if (c) activation energy (d) threshold energy
Ea = ¥. 67. Which one of the following reactions is a true first order
Chemical Kinetics 441

reaction? 298 K. What is the half-life period of the reaction in minutes?


(a) Alkaline hydrolysis of ethyl acetate (a) 50 (b) 15 (c) 45 (d) 25
(b) Acid catalyst hydrolysis of ethyl acetate 75. The reaction of hydrogen and iodine monochloride is given
(c) Decomposition of N2O as:
(d) Decomposition of gaseous ammonia on a hot platinum H 2 ( g ) + 2ICl( g ) ¾¾
® 2HCl( g ) + I 2 ( g )
surface
The reaction is of first order with respect to H2(g) and ICI(g),
68. The reason for almost doubling the rate of reaction on
following mechanisms were proposed.
increasing the temperature of the reaction system by 10°C is
Mechanism A:
(a) The value of threshold energy increases
(b) Collision frequency increases H 2 ( g ) + 2ICl( g ) ¾¾
® 2HCl( g ) + I 2 ( g )
(c) The fraction of the molecule having energy equal to Mechanism B:
threshold energy or more increases
(d) Activation energy decreases H 2 ( g ) + ICl( g ) ¾¾
® HI( g );slow
69. For a reaction, activation energy (Ea) = 0 and rate constant
HI( g ) + ICl( g ) ¾¾® HCl( g ) + I 2 ( g );fast
( k ) = 3.2 ´ 106 s -1 at 300 K. What is the value of the rate Which of the above mechanism(s) can be consistent with
constant at 310 K the given information about the reaction?
(a) A and B both (b) neither A nor B
(a) 3.2 ´ 10-12 s -1 (b) 3.2 ´ 106 s -1
(c) A only (d) B only
(c) 6.4 ´ 1012 s -1 (d) 6.4 ´ 106 s -1 76. In a first-order reaction A ® B, if k is rate constant and inital
concentration of the reactant A is 0.5 M, then the half-life is
1
70. In the Arrhenius plot of ln k vs , a linear plot is obtained
T
log 2 log 2 ln 2 0.693
with a slope of –2 × 104 K. The energy of activation of the (a) (b) (c) (d)
k k 0.5 k 0.5k
reaction (in kJ mole–1) is (R value is 8.3 J K–1 mol–1)
77. The bromination of acetone that occurs in acid solution is
(a) 83 (b) 166 (c) 249 (d) 332 represented by this equation.
71. A graph plotted between log k vs 1/T for calculating CH3COCH3 (aq) + Br 2 (aq) ® CH3COCH2Br (aq) + H+
activation energy is shown by (aq) + Br– (aq)
These kinetic data were obtained for given reaction
concentrations.
(a) log k (b) log k Initial Initial rate,
Concentrations, M disappearance of
1/T 1/T Br2, Ms–1
[CH3 COCH3] [Br2] [H+]
0.30 0.05 0.05 5.7×10–5
(c) log k (d) log k 0.30 0.10 0.05 5.7 × 10–5
0.30 0.10 0.10 1.2 × 10–4
1/T 1/T 0.40 0.05 0.20 3.1 × 10–4
72. Consider an endothermic reaction X ® Y with the activation Based on given data, the rate equations is:
energies, respectively, in general (a) Rate = k[CH3COCH3][H+]
(a) Eb < Ef (b) Rate = k [CH3COCH3][Br2]
(b) Eb > Ef
(c) Rate = k [CH3COCH3] [Br2] [H+]2
(c) Eb = Ef
(d) There is no definite relation between Eb and Ef (d) Rate = k [CH3COCH3][Br2] [H+]
73. When a biochemical reaction is carried out in laboratory in 78. For the reaction, N2 + 3H2 ® 2NH3,
the absence of enzyme then rate of reaction obtained is 10–
6 times, then activation energy of reaction in the presence of d [ NH3 ] – d [H2 ]
= 2 × 10–4 mol L–1 s–1 , the value of
enzyme is dt dt
6 would be :
(a) (a) 4 × 10–4 mol L–1 s–1 (b) 6 × 10–4 mol L–1 s–1
RT
(b) different from Ea obtained in laboratory –4
(c) 1 × 10 mol L s –1 –1 (d) 3 × 10–4 mol L–1 s–1
(c) P is required 79. In the reaction
(d) can't say anything
74. In a first order reaction, the concentration of the reactant is
1
reduced to of the initial concentration in 75 minutes at
8
EBD_7327
442 CHEMISTRY

by the rate equation rate = k [NO]2 [Cl2]


BrO3– (aq ) + 5Br - ( aq) + 6H + ( aq ) ® 3Br2 (l ) + 3H 2O(l ) The value of the rate constant can be increased by:
The rate of appearance of bromine (Br 2) is related to rate of (a) increasing the concentration of NO.
disappearance of bromide ions as following: (b) increasing the temperature.
(c) increasing the concentration of the Cl 2
d[Br2 ] 5 d[Br – ] d[Br2 ] 5 d[Br – ] (d) doing all of these
(a) =– (b) =
dt 3 dt dt 3 dt 86. Which one of the following statements for the order of a
reaction is incorrect ?
d[Br2 ] 3 d[Br – ] d [Br2 ] 3 d[Br – ] (a) Order can be determined only experimentally.
(c) = (d) =-
dt 5 dt dt 5 dt (b) Order is not influenced by stoichiometric coefficient of
80. Half life period of a first-order reaction is 1386 seconds. The the reactants.
specific rate constant of the reaction is : (c) Order of reaction is sum of power to the concentration
(a) 0.5 × 10–2 s–1 (b) 0.5 × 10–3 s–1 terms of reactants to express the rate of reaction.
(c) 5.0 × 10–2 s–1 (d) 5.0 × 10–3 s–1 (d) Order of reaction is always whole number.
87. The unit of rate constant for a zero order reaction is
81. For the reaction A + B ¾¾ ® products, it is observed that: (a) mol L–1 s–1 (b) L mol–1 s–1
2
(c) L mol s –2 –1 (d) s – 1
(1) On doubling the initial concentration of A only, the rate
of reaction is also doubled and 88. In a zero-order reaction for every 10° rise of temperature, the
(2) On doubling the initial concentrations of both A and B, rate is doubled. If the temperature is increased from 10°C to
100°C, the rate of the reaction will become :
there is a change by a factor of 8 in the rate of the reaction.
(a) 256 times (b) 512 times (c) 64 times (d) 128 times
The rate of this reaction is given by:
89. Activation energy (Ea) and rate constants (k1 and k2) of a
(a) rate = k [A] [B]2 (b) rate = k [A]2 [B]2 chemical reaction at two different temperatures (T1 and T2)
(c) rate = k [A] [B] (d) rate = k [A]2 [B] are related by :
82. For the reaction [ N 2O5 (g) ¾¾
® 2NO2 (g) + 1/2 O2 (g)]
k2 E æ1 1ö k2 E æ 1 1ö
(a) ln = - a ç - ÷ (b) ln =- a ç - ÷
the value of rate of disappearance of N 2O5 is given as k1 R è T1 T2 ø k1 R è T2 T1 ø
6.25 × 10–3 mol L–1s–1. The rate of formation of NO 2 and O 2 k2 E æ 1 1ö k E æ1 1ö
is given respectively as : (c) ln = - a ç + ÷ (d) ln 2 = a ç - ÷
k1 R è T2 T1 ø k1 R è T1 T2 ø
(a) 6.25 × 10–3 mol L–1s–1 and 6.25 × 10–3 mol L–1s–1
90. The energies of activation for forward and reverse reactions
(b) 1.25 × 10–2 mol L–1s–1 and 3.125 × 10–3 mol L–1s–1
for A2 + B2 ƒ 2AB are 180 kJ mol–1 and 200 kJ mol–1
(c) 6.25 × 10–3 mol L–1s–1 and 3.125 × 10–3 mol L–1s–1
respectively. The presence of a catalyst lowers the activation
(d) 1.25 × 10–2 mol L–1s–1 and 6.25 × 10–3 mol L–1s–1
energy of both (forward and reverse) reactions by 100 kJ mol–
2 ↑ 2AB) in
83. For an endothermic reaction, energy of activation is Ea and 1. The enthalpy change of the reaction (A + B
2
the presence of a catalyst will be (in kJ mol–1)
enthalpy of reaction of DH (both of these in kJ/mol). Minimum
(a) 20 (b) 300 (c) 120 (d) 280
value of Ea will be. 91. Consider the reaction, 2A + B ® products. When
concentration of B alone was doubled, the half-life did not
(a) less than DH (b) equal to DH
change. When the concentration of A alone was doubled,
(c) more than DH (d) equal to zero the rate increased by two times. The unit of rate constant for
84. During the kinetic study of the reaction, 2A + B ® C + D, this reaction is
following results were obtained: (a) s – 1 (b) L mol–1 s–1
(c) no unit (d) mol L–1 s–1.
Run [A ](mol L–1 ) [B](mol L– 1 ) Initial rate of
1
formation of 92. For a reaction A ® 2B, rate of disappearance of ‘A’ is
–1 –1 2
D (mol L min )
–3
related to the rate of appearance of ‘B’ by the expression
I 0.1 0.1 6.0 × 10
d[A] 1 d[B] d[A] 1 d[B]
II 0.3 0.2 7.2 × 10
–2 (a) – = (b) – =
–1
dt 2 dt dt 4 dt
III 0.3 0.4 2.88 × 10
–2 d[A ] d[B] d[A] d[B]
IV 0.4 0.1 2.40 × 10 (c) – = (d) – =4
dt dt dt dt
Based on the above data which one of the following is correct? 93. The half life period of a first order chemical reaction is 6.93
(a) rate = k [A]2 [B] (b) rate = k[A] [B] minutes. The time required for the completion of 99% of the
chemical reaction will be (log 2 = 0.301)
(c) rate = k [A]2 [B]2 (d) rate = k [A] [B]2 (a) 23.03 minutes (b) 46.06 minutes
85. The rate of the reaction 2NO + Cl2 ¾¾ ® 2NOCl is given (c) 460.6 minutes (d) 230.03 minutes
Chemical Kinetics 443

94. For a first order reaction (A) ® products the concentration (b) rate = k [B]4
of A changes from 0.1 M to 0.025 M in 40 minutes. (c) rate = k [A] [B]3
The rate of reaction when the concentration of A is 0.01 M (d) rate = k [A]2 [B]2
is : 100. Activation energy of a chemical reaction can be determined
by [1998]
(a) 1.73 × 10–5 M/min (b) 3.47 × 10–4 M/min
–5
(a) evaluating rate constant at standard temperature
(c) 3.47 × 10 M/min (d) 1.73 × 10–4 M/min (b) evaluating velocities of reaction at two different
95. The rate of a reaction doubles when its temperature changes temperatures
from 300 K to 310 K. Activation energy of such a reaction (c) evaluating rate constants at two different temperatures
will be : (R = 8.314 JK–1 mol–1 and log 2 = 0.301) (d) changing concentration of reactants
(a) 53.6 kJ mol –1 (b) 48.6 kJ mol–1
(c) 58.5 kJ mol–1 (d) 60.5 kJ mol–1
96. The rate of reaction depends upon the [1995]
(a) volume (b) force
(c) pressure (d) conc. of reactants
97. Half life of a first order reaction is 4 s and the initial
concentration of the reactants is 0.12 M. The concentration
of the reactant left after 16 s is [1999]
(a) 0.0075 M (b) 0.06 M
(c) 0.03 M (d) 0.015 M
98. The plot of concentration of the reactant vs. time for a
reaction is a straight line with a negative slope. The reaction
follows a [1996]
(a) zero order rate equation
(b) first order rate equation
(c) second order rate equation
(d) third order rate equation
99. Select the rate law that corresponds to data shown for the
following reaction [1994]
A+B ¾ ¾® products.
Exp. [A] [B] Initial rate
1 0.012 0.035 0.1
2 0.024 0.070 0.8
3 0.024 0.035 0.1
4 0.012 0.070 0.8
(a) rate = k [B]3

1. The rate equation for a reaction, concentration of B, the ratio of the new rate to the earlier rate
N2O ¾® N2 + 1/2O2 of the reaction will be as
is Rate = k[N2O]0 = k. If the initial concentration of the reactant 1
(a) (m + n) (b) (n – m) (c) 2(n – m) (d) (m + n )
is a mol Lit–1, the half-life period of the reaction is 2
a 3. The rate equation for the reaction 2A + B ® C is found to
t1 = - t 1 = ka be : rate = k[A][B]. The correct statement in relation to this
(a) 2k (b)
2 2 reaction is that the
a k (a) rate of formation of C is twice the rate of disappearance of
(c) t1 = (d) t 1 = a A
k
2 2 (b) t1/2 is a constant
2. The rate law for a reaction between the substances A and B (c) unit of k must be s–1
is given by Rate = k [A]n [B]m (d) value of k is independent of the initial concentrations
On doubling the concentration of A and halving the of A and B
4. For a first order reaction the rate constant is 6.909 min–1.
EBD_7327
444 CHEMISTRY

The time taken for 75% conversion in minutes is 13. Lead is the final product formed by a series of changes in
3 2 which the rate determining stage is the radioactive decay of
(a) log 2 (b) log 3 uranium-238 with a half-life of 4.5 × 109 years. What would
2 3
be the age of a rock sample originally lead free in which the
2 3 æ 3ö molar proportion of uranium to lead is now 1 : 3?
(c) log 2 (d) log ç ÷
3 2 è 4ø (a) 1.5 × 109 years (b) 2.25 × 109 years
5. Rate of a reaction can be expressed by following rate (c) 4.5 × 108 years (d) 9.0 × 109 years
expression 14. For the reaction H2(g) + Br2 (g) ® 2HBr (g), the experimental
Rate = k[A]2 [B], if concentration of A is increased by 3 data suggest, rate = k[H2][Br2]1/2. The molecularity and order
times, and concentration of B is increased by 2 times, how of the reaction are respectively
many times rate of reaction increases? 3 3 3 1
(a) 9 times (b) 27 times (a) 2, (b) , (c) 1, 1 (d) 1,
2 2 2 2
(c) 18 times (d) 8 times 15. A reaction was found to be second order with respect to the
6. The decomposition of N2O5 occurs as
concentration of carbon monoxide. If the concentration of
2N2O5 ¾¾ ® 4NO2 + O2 and follows Ist order kinetics, carbon monoxide is doubled, with everything else kept the
hence: same, the rate of reaction will
(a) the reaction is unimolecular (a) remain unchanged (b) triple
(b) the reaction is bimolecular (c) increase by a factor of 4 (d) double
(c) t1/2 µ a0 16. Which among the following plots are linear (a – x) is the
(d) None of these concentration of reactant remaining after time, t
7. The decomposition of ammonia on tungsten surface at 500 K
(1) (a – x) vs t, for a first order reaction
follows zero order kinetics. The half-life period of this reaction
(2) (a – x) vs t, for a zero order reaction
is 45 minutes when the initial pressure is 4 bar. The half-life
(3) (a – x) vs t, for a second order reaction
period (minutes) of the reaction when the initial pressure is
16 bar at the same temperature is (4) 1/(a – x) vs t, for a second order reaction
(a) 120 (b) 60 (c) 240 (d) 180 (a) 1 and 2 (b) 1 and 3
8. Diazonium salt decomposes as (c) 2 and 3 (d) 2 and 4
17. A reaction was observed for 15 days and the percentage of
C6 H 5 N +2 Cl - ® C6 H5Cl + N 2 the reactant remaining after the days indicated was recorded
At 0°C, the evolution of N2 becomes two times faster when in the following table :
the initial concentration of the salt is doubled. Therefore, it
Time(days 0 2 4 6 8 10 12 14 15
is
(a) a first order reaction % reactant
100 50 39 25 21 18 15 12.5 10
(b) a second order reaction remaining
(c) independent of the initial concentration of the salt
Which one of the following best describes the order and the
(d) a zero order reaction
9. If ‘I’ is the intensity of absorbed light and ‘C’ is the half-life of the reaction?
concentration of AB for the photochemical process, Reaction Order Half-life (days)
(a) First 2
AB + hv ¾ ¾® AB*, the rate of formation of AB* is directly (b) First 6
proportional to
(a) C (b) I (c) I2 (d) C.I (c) Second 2
10. The rate constant of a reaction becomes equal to the (d) Zero 6
pre-exponential factor when 18. A first order reaction is half completed in 45 minutes. How
(a) the absolute temperature is zero long does it need 99.9% of the reaction to be completed
(b) the activation energy is infinity (a) 5 hours (b) 7.5 hours
(c) the absolute temperature is infinity (c) 10 hours (d) 20 hours
(d) the temperature in Celsius is zero. 19. In the first order reaction, the concentration of the reactant
11. The rate of reaction is doubled for every 10°C rise in is reduced to 25% in one hour. The half life period of the
temperature. The increase in reaction rate as a result of reaction is
temperature rise from 10°C to 100°C is (a) 2 hr (b) 4 hr (c) 1/2 hr (d) 1/4 hr
(a) 112 (b) 512 (c) 400 (d) 614 20. The half-life of 2 samples are 0.1 and 0.4 seconds. Their
12. For the reaction N2 + 3H2 ® 2NH3 respective concentration are 200 and 50 respectively. What
D[NH3 ] -D[H 2 ] is the order of the reaction
if = 2 ´ 10-4 mol l–l s–1 , the value of (a) 0 (b) 2 (c) 1 (d) 4
Dt Dt
21. The rate constant of a first order reaction is doubled when
would be
the temperature is increased from 20°C to 25°C. How many
(a) 1 × 10– 4 mol L–1 s–1 (b) 3 × 10– 4 mol L–1 s–1 times the rate constant will increase if the temperature is
(c) 4 × 10– 4 mol L–1 s–1 (d) 6 × 10– 4 mol L–1 s–1 raised from 20°C to 40°C ?
(a) 4 (b) 8 (c) 16 (d) 32
Chemical Kinetics 445

22. Cyclopropane rearranges to form propene (a) 4 h (b) 0.5 h (c) 0.25 h (d) 1 h
32. Consider the reaction :
¾¾ ® CH3 - CH = CH 2
This follows first order kinetics. The rate constant is Cl2 ( aq ) + H 2S( aq ) ® S(s ) + 2H + (aq) + 2Cl- (aq)
2.714 × 10 –3 sec –1. The initial concentration of cyclopropane The rate equation for this reaction is
is 0.29 M. What will be the concentration of cyclopropane rate = k[Cl2 ][H 2S]
after 100 sec
Which of these mechanisms is/are consistent with this rate
(a) 0.035 M (b) 0.22 M
equation?
(c) 0.145 M (d) 0.0018 M
23. The rate constant is doubled when temperature increases A. Cl 2 + H 2S ® H + + Cl - + Cl+ + HS- (slow)
from 27°C to 37°C. Activation energy in kJ is Cl + + HS- ® H + + Cl- + S (fast)
(a) 34 (b) 54 (c) 100 (d) 50 + -
24. The rate constant, the activation energy and the arrhenius B. H 2S ƒ H + HS (fast equilibrium)
parameter of a chemical reaction at 25°C are 3.0 × 10–4s–1, Cl 2 + HS- ® 2Cl - + H + + S (Slow)
104.4 kJ mol–1 and 6.0 × 1014 s–1 respectively. The value of
(a) B only (b) Both A and B
the rate constant as T ® ¥ is
(c) Neither A nor B (d) A only
(a) 2.0 × 1018 s–1 (b) 6.0 × 1014 s–1
33. The rate of a chemical reaction doubles for every 10°C rise
(c) Infinity (d) 3.6 × 1030 s–1
of temperature. If the temperature is raised by 50°C, the rate
25. The activation energy for a reaction is 9.0 kcal/mol. The
of the reaction increases by about :
increase in the rate constant when its temperature is
(a) 10 times (b) 24 times (c) 32 times (d) 64 times
increased from 298K to 308K is
34. Consider a reaction aG + bH ® Products. When
(a) 63% (b) 50% (c) 100% (d) 10%
concentration of both the reactants G and H is doubled, the
26. For the following reaction: NO2(g) + CO(g) ® NO(g) +
rate increases by eight times. However, when concentration
CO2(g), the rate law is: Rate = k [NO2]2. If 0.1 mole of gaseous
of G is doubled keeping the concentration of H fixed, the
carbon monoxide is added at constant temperature to the
rate is doubled. The overall order of the reaction is
reaction mixture which of the following statements is true?
(a) 0 (b) 1 (c) 2 (d) 3
(a) Both k and the reaction rate remain the same
35. Under the same reaction conditions, initial concentration of
(b) Both k and the reaction rate increase
1.386 mol dm–3 of a substance becomes half in 40 seconds
(c) Both k and the reaction rate decrease
and 20 seconds through first order and zero order kinetics,
(d) Only k increases, the reaction rate remain the same
respectively. Ratio (k1/k0) of the rate constant for first order
27. If 60% of a first order reaction was completed in 60 minutes,
(k1) and zero order (k0) of the reaction is –
50% of the same reaction would be completed in aproximately
(a) 0.5 mol–1 dm3 (b) 1.0 mol dm–3
(a) 45 minutes (b) 60 minutes –3
(c) 1.5 mol dm (d) 2.0 mol–1 dm3
(c) 40 minutes (d) 50 minutes
36. For a first order reaction A®P, the temperature (T) dependent
28. The rate constants k1 and k2 for two different reactions are
rate constant (k) was found to follow the equation
1016 . e–2000/T and 1015 . e–1000/T, respectively. The temperature
at which k1 = k2 is : 1
log k = – (2000) + 6.0 . The pre-exponential factor A and
2000 1000 T
(a) 1000 K (b) K (c) 2000 K (d) K the activation energy Ea, respectively, are
2.303 2.303 (a) 1.0 × 106 s–1 and 9.2 kJ mol–1
29. The rate of the reaction 2N2O5 ® 4NO2 + 2O2 can be written (b) 6.0 s–1 and 16.6 kJ mol–1
in three ways : (c) 1.0 × 106 s–1 and 16.6 kJ mol–1
- d[N 2O5 ] d[NO 2 ] (d) 1.0 × 106 s–1 and 38.3 kJ mol–1
= k [N 2O5 ] , = k ¢ [N 2O 5 ]
dt dt 37. Plots showing the variation of the rate constant (k) with
d[O 2 ] temperature (T) are given below. The plot that follows
= k ¢¢ [N 2O 5 ] Arrhenius equation is
dt
The relationship between k and k' and between k and k¢¢ are:
(a) k¢ = 2k ; k¢ = k (b) k¢ = 2k ; k¢¢ = k / 2
(c) k¢ = 2k ; k¢¢ = 2k (d) k¢ = k ; k¢¢ = k
30. A radioactive element gets spilled over the floor of a room. (a) (b)
Its half-life period is 30 days. If the initial velocity is ten
times the permissible value, after how many days will it be
safe to enter the room?
(a) 100 days (b) 1000 days
(c) 300 days (d) 10 days.
31. The time for half life period of a certain reaction (c) (d)
A ¾¾ ® Products is 1 hour. When the initial concentration
of the reactant ‘A’, is 2.0 mol L–1, how much time does it take
for its concentration to come from 0.50 to 0.25 mol L–1 if it is 38. For a reaction A + B ¾® C + 2D, experimental results were
a zero order reaction ? collected for three trials and the data obtained are given
EBD_7327
446 CHEMISTRY

below: 1
concentration by a factor of is 20 minutes. The time
4
Trial [A], M [B], M Initial Rate, M s–1 1
1 0.40 0.20 5.5 × 10–4 required to reduce initial concentration by a factor of is
16
2 0.80 0.20 5.5 × 10–4 (a) 20 min (b) 10 min
3 0.40 0.40 2.2 × 10–3 (c) 80 min (d) 40 min
45. The rate law for the reaction
The correct rate law of the reaction is xA + yB ¾® mP + nQ is Rate = k [A]c [B]d. What is the total
(a) rate = k[A]0 [B]2 (b) rate = k[A] [B]2 order of the reaction?
(c) rate = k[A] [B] (d) rate = k[A] [B]0 (a) (x + y) (b) (m + n)
39. The decomposition of ammonia on tungsten surface at 500 K (c) (c + d) (d) x/y
follows zero order kinetics. The half-life period of this reaction 46. Consider the following statements:
is 45 minutes when the initial pressure is 4 bar. The half-life I. Increase in concentration of reactant increases the
period (minutes) of the reaction when the initial pressure is rate of a zero order reaction.
16 bar at the same temperature is II. Rate constant k is equal to collision frequency A if
(a) 120 (b) 60 Ea = 0.
(c) 240 (d) 180 III. Rate constant k is equal to collision frequency A if
Ea = ¥.
40. The initial rates of reaction
3A + 2B + C ¾® Products, at different initial concentrations IV. In k vs T is a straight line.
are given below: V. In k vs 1/T is a straight line.
Initial rate, [A]0, M [B]0, M [C]0, M Correct statements are
Ms–1 (a) I and IV (b) II and V
5.0 × 10–3 0.010 0.005 0.010 (c) III and IV (d) II and III
5.0 × 10–3 0.010 0.005 0.015 DIRECTIONS for Qs. 47 to 50 : These are Assertion-Reason
1.0 × 10–2 0.010 0.010 0.010 type questions. Each of these question contains two statements:
1.25 × 10–3 0.005 0.005 0.010 Statement-1 (Assertion) and Statement-2 (Reason). Answer these
The order with respect to the reactants, A, B and C are questions from the following four options.
respectively (a) Statement- 1 is True, Statement-2 is True, Statement-2 is a
(a) 3, 2, 0 (b) 3, 2, 1 correct explanation for Statement -1
(c) 2, 2, 0 (d) 2, 2, 1 (b) Statement -1 is True, Statement -2 is True ; Statement-2 is
NOT a correct explanation for Statement - 1
41. Which one of the following reactions is a true first order
reaction? (c) Statement - 1 is True, Statement- 2 is False
(d) Statement -1 is False, Statement -2 is True
(a) Alkaline hydrolysis of ethyl acetate
47. Statement-1 : If the activation energy of a reaction is zero,
(b) Acid catalyst hydrolysis of ethyl acetate
temperature will have no effect on the rate constant.
(c) Decomposition of N2O Statement-2 : Lower the activation energy, faster is the
(d) Decomposition of gaseous ammonia on a hot platinum reaction.
surface 48. Statement-1 : If in a zero order reaction, the concentration of
42. In a first order reaction, the concentration of the reactant is the reactant is doubled. the half-life period is also doubled.
1 Statement-2 : For a zero order reaction, the rate of reaction is
reduced to of the initial concentration in 75 minutes at
8 independent of initial concentration.
298 K. What is the half-life period of the reaction in minutes? 49. Statement-1 : If the activation energy of a reaction is zero,
temperature will have no effect on the rate constant.
(a) 50 (b) 15 Statement-2 : Lower the activation energy, faster is the
(c) 45 (d) 25 reaction.
1 50. Statement-1 : According to steady state hypothesis, in a
43. In the Arrhenius plot of ln k vs , a linear plot is obtained multistep reaction, the change in concentration with time for
4 T
with a slope of –2 × 10 K. The energy of activation of the reactive intermediates is zero.
reaction (in kJ mole–1) is (R value is 8.3 J K–1 mol–1) Statement-2 : The intermediates are so reactive that after a
(a) 83 (b) 166 brief initial period their concentrations rise from zero to a
(c) 249 (d) 332 small value and remains constant for most of the duration of
the reaction.
44. For a first order reaction, the time taken to reduce the initial
Chemical Kinetics 447

Exemplar Questions 6. According to Arrhenius equation rate constant k is equal to


1. The role of a catalyst is to change ........... . A e - E a/RT
/ RT
T. Which of the following options represents the

(a) Gibbs energy of reaction 1


graph of ln k vs ?
(b) enthalpy of reaction T
(c) activation energy of reaction
(d) equilibrium constant
2. In the presence of a catalyst, the heat evolved or absorbed
during the reaction ........ .
(a) increases (a) ln k
(b) decreases
(c) remains unchanged
(d) may increase or decrease
3. Activation energy of a chemical reaction can be determined 1/T
by ...... .
(a) determining the rate constant at standard temperature
(b) determining the rate constant at two temperatures
(c) determining probability of collision
(d) using catalyst (b) ln k
4. Consider figure and mark the correct option.
Activated complex
1/T
E1
Products
Energy

E2

Reactants (c) ln k
Reaction coordinate
(a) Activation energy of forward reaction is E 1 + E2 and
1/T
product is less stable than reactant
(b) Activation energy of forward reaction is E 1 + E2 and
product is more stable than reactant
(c) Activation energy of both forward and backward
reaction is E1 +E2 and reactant is more stable than (d) ln k
product
(d) Activation energy of backward reaction is E 1 and
product is more stable than reactant
5. Consider a first order gas phase decomposition reaction 1/T
7. Consider the Arrhenius equation given below and mark the
given below
correct option.
A(g) ® B(g) + C(g)
E
The initial pressure of the system before decomposition of - a
k = Ae RT
A was pi. After lapse of time 't' total pressure of the system
(a) Rate constant increases exponentially with increasing
increased by x units and became 'pt'. The rate constant k for
activation energy and decreasing temperature
the reaction is given as ........... .
(b) Rate constant decreases exponentially with increasing
2.303 p 2.303 pi activation energy and decreasing temperature
(a) k= log i (b) k = log
t pi - x t 2pi - p t (c) Rate constant increases exponentially with decreasing
activation energy and decreasing temperature
2.303 pi 2.303 p
(c) k= log (d) k = log i (d) Rate constant increases exponentially with decreasing
t 2pi + pt t pi + x activation energy and increasing temperature
EBD_7327
448 CHEMISTRY

8. A graph of volume of hydrogen released vs time for the V5 - V2 V4 - V2


reaction between zinc and dil. HCl is given in figure. On the (a) (b)
50 - 30 50 - 30
basis of this mark the correct option.
V3 - V2 V3 - V1
V5 (c) (d)
40 - 30 40 - 20
V4
11. Which of the following statements is correct?
(a) The rate of a reaction decreases with passage of time
V3 as the concentration of reactants decreases.
(b) The rate of a reaction is same at any time during the
reaction.
V2 (c) The rate of a reaction is independent of temperature
V1 change.
(d) The rate of a reaction decreases with increase in
concentration of reactant(s).
O 12. Which of the following expressions is correct for the rate of
20 30 40 50
reaction given below?
V3 - V2
(a) Average rate upto 40s is 5Br - ( aq ) + BrO 3- ( aq ) + 6H + ( aq ) ® 3Br2 ( aq ) + 3H 2 O ( l )
40

(b) Average rate upto 40s is 3


V - V2 D [ Br - ] D [ H+ ] D [ Br - ] 6 D [ H + ]
40 - 30 (a) =5 (b) =
Dt Dt Dt 5 Dt
V3
(c) Average rate upto 40s is D [ Br - ] 5 D [ H + ] D [ Br - ] D [ H+ ]
40 (c) = (d) =6
Dt 6 Dt Dt Dt
V - V1
(d) Average rate upto 40s is 3 13. Which of the following graphs represents exothermic
40 - 20
reaction?
9. Which of the following statements is not correct about order
of a reaction?
(a) The order of a reaction can be a fractional number Activated complex
(b) Order of a reaction is experimentally determined quantity
(c) The order of a reaction is always equal to the sum of
the stoichiometric coefficients of reactants in the
balanced chemical equation for a reaction
Energy

(d) The order of a reaction is the sum of the powers of


molar concentration of the reactants in the rate law
expression (I)
Reactants
10. Consider the graph given in figure. Which of the following
options does not show instantaneous rate of reaction at Products
40th second?
Reaction coordinate

V5
V4 Activated complex

V3
Energy

V2 (II)
V1 Products
Reactants

O Reaction coordinate
20 30 40 50
Chemical Kinetics 449

Activated complex (b) It alters DG of the reaction


(c) It is a substance that does not changes the equilibrium
constant of a reaction
(d) It provides an alternate mechanism by reducing
activation energy between reactants and products.
Energy

(III) 19. The value of rate constant of a pseudo first order


reaction.............
Products (a) depends on the concentration of reactants present in
Reactants
small amount
(b) depends on the concentration of reactants present in
excess
Reaction coordinate (c) is independent of the concentration of reactants
(a) Only (I) (b) Only (II) (d) depends only on temperature
(c) Only (III) (d) Both (I) and (II) 20. Consider the reaction A ® B. The concentration of both the
14. Rate law for the reaction A + 2B ¾® C is found to be reactants and the products varies exponentially with time.
Rate = k[A] [B] Which of the following figures correctly describes the change
Concentration of reactant 'B' is doubled, keeping the in concentration of reactants and products with time?
concentration of 'A' constant, the value of rate constant will [B]
be ...... .
(a) the same (b) doubled
(c) quadrupled (d) halved

Concentration
15. Which of the following statements is incorrect about the
collision theory of chemical reaction?
(a) It considers reacting molecules or atoms to be hard (a)
spheres and ignores their structural features [A]
(b) Number of effective collisions determines the rate of
reaction Time
(c) Collision of atoms or molecules possessing sufficient [B]
threshold energy results into the product formation
(d) Molecules should collide with sufficient threshold
energy and proper orientation for the collision to be
Concentration

effective
16. A first order reaction is 50% completed in 1.26 × 1014 s. How
much time would it take for 100% completion? (b)
(a) 1.26 × 1015 s (b) 2.52 × 1014 s [A]
(c) 2.52 × 10 s 28 (d) Infinite
17. Compounds 'A' and 'B' react according to the following Time
chemical equation.
[B]
A(g) + 2B(g) ¾® 2C(g)
Concentration of either 'A' or 'B' were changed keeping the
concentrations of one of the reactants constant and rates
Concentration

were measured as a function of initial concentration.


Following results were obtained. Choose the correct option
for the rate equations for this reaction. (c)
Experiment Initial Initial Initial
[A]
concentration concentration concentration of
–1 –1 –1 –1 Time
of [A]/mol L of [B]/mol L [C]/mol L s
1. 0.30 0.30 0.10
2. 0.30 0.60 0.40 [A]
3. 0.60 0.30 0.20
Concentration

(a) Rate = k [A]2[B] (b) Rate = k [A] [B]2


(c) Rate = k [A] [B] (d) Rate = k [A]2[B]0
(d)
18. Which of the following statement is not correct for the
[B]
catalyst?
(a) It catalyses the forward and backward reactions to the
Time
same extent
EBD_7327
450 CHEMISTRY

NEET/AIPMT (2013-2017) Questions (a) First


(b) Second
21. What is the activation energy for a reaction if its rate doubles
(c) More than zero but less than first
when the temperature is raised from 20°C to 35°C? (R = 8.314
J mol–1 K–1) [2013] (d) Zero
(a) 269 kJ mol–1 (b) 34.7 kJ mol–1 27. The rate constant of the reaction A ® B is 0.6 × 10–3 mole per
(c) 15.1 kJ mol–1 (d) 342 kJ mol–1 second. If the concentration of A is 5 M then concentration
22. A reaction having equal energies of activation for forward of B after 20 minutes is : [2015 RS]
and reverse reaction has : [2013] (a) 1.08 M (b) 3.60 M
(a) DG = 0 (b) DH = 0 (c) 0.36 M (d) 0.72 M
28. The rate of a first -order reaction is 0.04 mol l–1s–1 at 10
(c) DH = DG = DS = 0 (d) DS = 0 seconds and 0.03 mol l–1s–1 at 20 seconds after initiation of
23. A reaction is 50% completed in 2 hours and 75% completed the reaction. The half-life period of the reaction is [2016]
in 4 hours. The order of reaction is [NEET Kar. 2013] (a) 24.1 s (b) 34.1 s
(a) 0 (b) 1 (c) 44.1 s (d) 54.1 s
(c) 2 (d) 3 29. The addition of a catalyst during a chemical reaction alters
24. For a reaction between A and B the order with respect to A is which of the following quantities? [2016]
2 and the order with respect to B is 3. The concentrations of (a) Entropy (b) Internal energy
both A and B are doubled, the rate will increase by a factor (c) Enthalpy (d) Activation energy
of: [NEET Kar. 2013]
30. Mechanism of a hypothetical reaction [2017]
(a) 10 (b) 12 X2 + Y2 ® 2XY is given below :
(c) 16 (d) 32 (i) X2 ® X + X(fast)
25. The activation energy of a reaction can be determined from (ii) X + Y2 ƒ XY + Y (slow)
the slope of which of the following graphs ? [2015] (iii) X + Y ® XY (fast)
ln K l The overall order of the reaction will be :
(a) vs.T (b) ln K vs. (a) 2 (b) 0
T T
T l (c) 1.5 (d) 1
(c) ln K vs. T (d) ln K vs . T 31. A first order reaction has a specific reaction rate of 10 –2
sec–1. How much time will it take for 20g of the reactant to
26. When initial concentration of a reactant is doubled in a reduce to 5 g ? [2017]
reaction, its half-life period is not affected. The order of the (a) 138.6 sec (b) 346.5 sec
reaction is : [2015] (c) 693.0 sec (d) 238.6 sec
Chemical Kinetics 451

Hints & Solutions


EXERCISE - 1 2
(
= k 2[A]2[B] = 2r1 Q r1 = k [A] [B] )
dc
1. (a) = k [c ] = 3 ´ 10- 6 ´ 0.1 \ Rate of reaction is increased two times.
dt 10. (d) Graph (d) represents graph between t½ and initial
dc concentration for 3rd order reaction :
= 3 ´ 10 -7 mol litre -1 sec -1 (a) Zero order reaction
dt
(b) 1st order reaction
2. (d) All the statements are correct.
(c) 2nd order reaction.
Order of reaction = 1 +
1
= 1.5 1 d [ HI]
2 11. (d) rate of appearance of HI =
2 dt
Molecularity = 1 + 1 = 2
-d ëé H 2 ûù
dx rate of formation of H2 =
µ [Br2 ]1/2 dt
dt -d [ I 2 ]
So, reaction rate will be doubled if concentration of Br 2 rate of formation of I2 =
is increased by 4 times. dt
3. (d) As the unit of rate constant is sec–1, so the reaction is -d [ H 2 ] d [ I2 ] 1 d [ HI]
first order reaction. Hence hence =- =
dt dt 2 dt
1 [N O ]
k = = log a or kt = log 2 5 0 2d [ H 2 ] 2d [ I2 ] d [ HI]
t ( a - x) [N 2 O5 ]t or – =- =
dt dt dt
d[C] 12. (a) From data 1 and 3, it is clear that keeping (B) const, [A] is
4. (d) In the given options - will not represent the
3.dt doubled, rate remains unaffected. Hence rate is
reaction rate. It should not have –ve sign as it is a product. independent of [A]. From 1 and 4, keeping [A] constant,
[B] is doubled, rate become 8 times. Hence rate µ[B]3.
1 d[C]
shows the rate of formation of product C. 13. (b) 2 N 2O 5 ® 4 NO 2 + O 2
3 dt
5. (a) If we write rate of reaction in terms of concentration of from the unit of rate constant it is clear that the reaction
NH3 and H2,then follow first order kinetics. Hence
by rate law equation, r = k [N2O5]
1 d[ NH 3 ] 1 d[H 2 ] where r = 1.02 × 10–4, k = 3.4 × 10–5
Rate of reaction = =-
2 dt 3 dt 1.02 × 10–4 = 3.4 × 10–5 [N2O5]
[N2O5] = 3M
d[ NH 3 ] 2 d[H 2 ]
So, =- 14. (b) For a first order reaction the positive slope is obtained
dt 3 dt when we plot – loge [A] vs t.
6. (b) r = k [O2][NO]2. When the volume is reduced to 1/2, the 15. (c)
conc. will double 2.303 [ A]
k= log 0
\ New rate = k [2O2][2 NO]2 = 8 k [O2][NO]2 t [ A]t
The new rate increases to eight times of its initial.
7. (b) When the concentration of reactant is reduced to half 2.303 800
\ k= = 1.386 ´ 10-4 s -1
log
its initial value, the rate is reduced by 2.4 = 4 times 2 ´ 10 4 50
0.6 16. (b) Given dx/dt = 2.400 × 10–5 mol litre–1 sec–1
It means, rate µ [ reactant]2 k = 3.10 × 10–5 sec–1
So, order of reaction = 2 For first order reaction
8. (c) Increase in temperature = 100°C – 10°C = 90°C
\ n=9 2N 2 O5 ¾¾
® 2NO 2 + O 2
Increase in reaction rate = 2n = 29 = 512 times
dx
9. (a) 2A + B ¾ ¾® A2B = k [N 2 O 5 ]
r1 = k[A]2 [B] ...(i) dt
or 2.4 × 10–5 = 3.0 × 10–5 [N2O5]
é Bù
When, [A] = [2A], [B] = ê ú
ë2û 2.4 ´ 10 -5
or [N2O5] = = 0.8 mol. litre–1
3.0 ´ 10 -5
2 éBù 2 [B]
r2 = k [ 2A ] ê ú = k 4[A] 17. (b) Initial concentration = 10 mol L–1
ë2û 2 \ Conc. after 20 min (two half lives) = 2.5 mol L–1
EBD_7327
452 CHEMISTRY

0.693 0.693 23. (a) Plots of conc. [A] Vs time, t


Now, k = =
t1 / 2 10 min Zero First
order order
or 0.0693 min–1 1 1

log [A]
\ rate = k × [reactant] [A] Second Third
[ A] [ A]2
order order
= 0.0693 × 2.5 mol L–1 min–1 t t t
18. (b) Velocity constant ‘K’ is characteristic constant of a
24. (a) t1/2 = 4s T = 16s
reaction and depends only on temperature and catalyst.
k2 E a é T2 - T1 ù T 16
n= = =4 (\ T = n × t½)
19. (c) log =
k1 2.303R êë T1T2 úû
t1/ 2 4

æ 1 ön æ 1 ö4 0.12
or 2.303log k 2 = Ea é T2 - T1 ù [ A] < [ A]o çç ÷÷÷ < 0.12≥çç ÷÷÷ < < 0.0075 M
k1 R êë T1T2 úû çè 2 ø èç 2 ø 16
Where [A]o = initial concentration and [A] = concentration
é1.667 ´ 10 -4 ù E é 1 1 ù left after time t
2.303 log ê =– a ê -
-6 ú R ë1844 1000 úû
ëê1.667 ´ 10 ûú 25. (b) For reaction 3A ¾ ¾® B + C
Ea 844 If it is zero order reaction r = k= [A]0, i.e., the rate remains
2.303 × 2 = × 1844 ´ 1000 ........(1) same at any concentration of 'A'. i.e., independent of
R concentration of A.
Ea 4.606 ´ 1844 ´ 1000
\ = 26. (a) k = 0.693 = 0.693 = 2.31 ´ 10 -2 min -1
R 844 T1/2 30
é k3 ù E a 1423 - 1000 27. (b) A ® B For a first order reaction
2.303 log ê -6 ú
= ×
êë1.667 ´ 10 úû R 1423 ´ 1000 Given a = 0.8 mol, (a – x) = 0.8 – 0.6 = 0.2
2.303 0.8
Ea 423 k= log or k = 2.303 log 4
= × 1423 ´ 1000 ........(2) 1 0.2
R again a = 0.9, a – x = 0.9 – 0.675 = 0.225
Dividing equation (2) by equation (1) 2.303 0.9
é ù k= log
k3 t 0.225
log ê -6 ú
ë1.667 ´ 10 û 2.303
2.303log 4 = log 4
2 t
423 1844 ´ 1000 Hence t = 1 hour
= 1423 ´ 1000 ×
844 28. (b) CaCO 3 ¾¾ ® CaO + CO 2
423 ´ 1844 This reaction will proceed in forward direction if volume
é k3 ù
\ log ê ú =2× = 1.299 of container is increased.
-6
êë1.667 ´ 10 úû 1423 ´ 844 29. (b) For a first order reaction, A ® products
On taking Antilog, k3 = 19.9 r
r = k[A] or k =
\ k3 = 19.9 × 1.667 × 10–6 = 3.318 × 10–5 s–1 [A]
20. (c)
21. (a) Given initial concentration (a) = 2.00 M; Time taken 1.5 ´ 10-2
Þk = = 3 × 10–2
(t) = 200 min and final concentration (a – x) = 0.15 M. 0.5
For a first order reaction rate constant, 0.693 0.693
Further, t1/ 2 = = = 23.1
2.303 a 2.303 2.00 k 3 ´ 10-2
k= log = log
t a-x 200 0.15 30. (d) Given [A] = 0.01 M
Rate = 2.0 × 10–5 mol L–1 s–1
2.303 -2 -1
= ´ (0.301 + 0.824) = 1.29 ´ 10 min . For a first order reaction
200 Rate = k[A]
0.693 0.693 2.0 ´ 10 -5
Further (t1/ 2 ) = = = 53.72 min . k= = 2 × 10–3
k 1.29 ´ 10-2 [0.01]
0.693 0.693 693 0.693
22. (c) T1/ 2 = = = = 600 sec t1/2 = = 347 sec
k 1.155 ´ 10 -3 1.155 2 ´ 10-3
Chemical Kinetics 453

31. (c) Molecularity can never be fraction. It is the number of \ T = n ´ t1/ 2 = 3 × 100 = 300 sec.
reacting species undergoing simultaneous collision in
the elementary or simple reaction. It is a theoretical
concept. Slope = - k / 2.303
32. (d) t1/2 = 5 years, T = 15 years hence total number of half
39. (b)
log C
life periods = 15 = 3 .
5
64 t
\ Amount left = = 8g
3
( 2) 40. (a) For a first order reaction, we have
33. (c) As the concentration of reactant decreases from 0.8 to 2.303 a
k= log
0.4 in 15 minutes hence the t1 / 2 is 15 minutes. To fall the t (a - x)
concentration from 0.1 to 0.025 we need two half lives as Substituting the given values
2.303 2.0 2.303
æ 1ö
n k = log = (log 2.0 - log 0.15) sec -1
[A] = çè ÷ø [A]0 200 sec 0.15 200
2
2.303
n = ´ 1.1249 sec -1 = 1.29 × 10–2 sec–1
0.1 æ 1ö 200
=ç ÷
0.025 è 2 ø 41. (b) Given: 75% reaction gets completed in 32 min
n=2 2.303 a 2.303 100
i.e., 30 minutes Thus, k = log = log
t (a - x ) 32 (100 - 75)
34. (d) For a first order reaction, we have
2.303
2.303 N = log 4 = 0.0433 min–1
k= log 0 32
t N Now we can use this value of k to get the value of time
2.303 0.1 required for 50% completion of reaction
\ k= log
40 min 0.005 2.303 a 2.303 100
t= log = log
k (a - x ) 0.0433 50
2.303 2.303
= ´ log 20 = ´ 1.3010 2.303
40 min 40 = log 2 = 16 min
Now rate = k × [reactant] 0.0433
When [x] = 0.01 M 42. (a) The half life period of a 3rd order reaction,
2.303 3[A] ¾¾ ↑ product,
\ rate = × 1.3010 × 0.01 M min–1 is given by the equation
40
= 7.5 × 10–4 M min–1. 3
t1/2 =
35. (a) Given r = k [A]3/2 [B]–1/2 2k[A]0 2
3 -1 2 where [A]0 is initial concentration.
Order = 3/2 – 1/2 = = =1
2 2 Here t1/2 = 8 hr 20 min = 500 min. = 30000 sec and [A0]
0.693 = 0.1M
36. (c) For first order reaction, k = 3 3
t1/ 2 [ k= =
2t1/ 2 [A]0 2 2 ´ (30000 sec)(0.10 mol L-1 ) 2
where k = rate constant
t1/2 = half life period = 480 sec. 3
= = 5 × 10–3 L2 mol–2 sec–1
0.693 60000 ´ 0.01
\ k = = 1.44 × 10–3 sec–1 43. (d) If rate = k[A]x [B]y [C]z
480
37. (c) Since rate of reaction becomes four times on doubling From first two given data
concentration of reactant, it is second order reaction. 8.08 × 10–3 = k [0.2]x [0.1]y [0.02]z .... (1)
n
2.01 × 10–3 = k [0.1]x [0.2]y [0.02]z .... (2)
æ 1ö Divide (1) by (2) we get, 4 = 2x (1/2)y
38. (c) [A] = [A]0 ç ÷ where [A]0 = initial concentration
è 2ø Similarly, from second and third data
(9)y (9)z = 3
n
1 æ 1ö 2y + 2z = 1 .
= 1ç ÷ ; n = 3 From first and fourth data 4z = 8 = 23
8 è 2ø
2z = 3. So z = 3/2, y = – 1, x = 1
0.693 44. (d) Using the relation
t1/ 2 = = 100 sec
6.9 ´ 10-3 [A] = [A]0 (1/2)n [n = number of half-lives]
T = n × ty2
EBD_7327
454 CHEMISTRY

7.5 line is giong up. Hence graph shown in option (d) is the
Here, n = =3 correct choice.
2.5
3 0.693 0.693
æ1ö 1 50. (a) t1/ 2 = = = 1117.7 sec.
\ [A] = 160 ´ ç ÷ = 160 ´ k 6.2 ´ 10 -4
è2ø 8
51. (b) pH = 2 ; r1 = k × (10–2)n {Q [H+] = 10–pH}
or 20 gm i.e., (d) pH = 1 ; r2 = k × (10–1)n
45. (b) The order w.r.t. I 2 is zero because the rate is not
dependent on the concentration of I2. n
æ 10-1 ö
46. (c) For the reaction, Given r2 = 100 r1 Þ ç ÷ = 100
ç 10-2 ÷
k
è ø
X + reagent ¾¾ 1 ® product,
Þ 10n = 100
the value of k1 when 94% of X has been reacted is \ n=2
given by
1
52. (d) t1/2 µ 2
2.303 100 a
k1 = log
t 100 - 94 1
We know that t1/ 2 µ n -1
a
2.303 æ 100 ö ......(1) i.e. n = 3
Þ k1 = log ç ÷
t è 6 ø Thus reaction is of 3rd order.
Similarly, for the reaction 53. (a) C 4 H 8 ¾¾
® 2C 2 H 4
k2 Moles at t = 0 a 0
Y + reagent ¾¾® product,
the value of k2 when 50% of Y have been reacted is Moles at t = t (a – x) 2x
given by, 2x a
when = 1 then x =
2.303 100 a-x 3
k2 = log
t 100 - 50 2.303 a
\ t= log
k ( a - x)
2.303 2.303 a
Þ k2 = log(2) .........(2) = log = 1700 sec
t 2.3 ´ 10 -4 a
a-
Dividing equ. (1) by equ. (2), we get 3
2.303 1 2.303 4
æ 100 ö 54. (c) t1/ 4 = log = log
log ç ÷ k 3/ 4 k 3
k1 è 6 ø = 4.059
=
k2 log 2 2.303 2.303
= (log 4 - log 3) = (2 log 2 - log 3)
k k
47. (c) As we know that, units of rate constant.
= (unit of conc.)1–n (unit of time)–1 2.303 0.29
= (mol L–1)1–n (sec)–1 = (2 ´ 0.301 - 0.4771) =
k k
On comparing these units with the given units of rate
55. (c) Given initial mass = a
constant, we get
Mass left after time t = x = 0.875 a
(mol L–1)1–n (sec)–1 = L mol–1 sec–1
t = 40 minutes.
Þ Ln–1 mol1–n sec–1 = L mol–1 sec–1
On comparing the powers, we get 2.303 a = 2.303 log a
n–1=1 Þ n=2 k= log
t a-x 40 a - 0.875 a
So, reaction is of second order.
48. (d) Unit of k for Ist order reaction is (Time)–1 2.303
= log 8 = 0.052 min–1
49. (d) For first order decomposition of N2O5, rate law is given 40
by
0.693 0.693
rate, r = k[N2O5] t1/ 2 = = = 13 minutes 32 sec.
k 0.052
On plotting rate on y-axis and concn [N2O5] on x-axis and
56. (d) None of the given options is correct.
then comparing it with y = mx + c, we get a line starting The given reaction is : 2X + Y —® Z
from origin with slope 'k'. Its positive slope suggests that
Chemical Kinetics 455

d[X] d[Z] 20
– = t1/ 2 = = 10
2dt dt 2
\ Rate of formation of Z is half of the rate of disappearance For second condition
of X. n
æ 1 ö æ 1ö
-d[X] -d[Y] çè ÷ø = çè ÷ø
= 16 2
2dt dt
Rate of X is not equal to rate of disappearance of Y.
n = 4 \ T = 4 × 10 = 40 min
64. (d) The activation energy of reverse reaction will depend
57. (b) Activation energy of reactant is less than the energy of
upon whether the forward reaction is exothermic or
activation of products.
endothermic.
58. (d) A catalyst affects equally both forward and backward
As DH = Ea (forward reaction) – Ea(backward reaction)
reactions, therefore it does not affect equilibrium constant
For exothermic reaction
of reaction.
DH = –ve
59. (b) According to Arrhenius equation, k = Ae–Ea/RT
\ –DH = Ea(f) – Ea(b)
\ when Ea = 0, k = A
or Ea(f) = Ea(b) – DH
Also ln k us 1/T is a straight line with slope = –Ea/R. \ Ea(f) < Ea(b)
\ Statements (ii) and (v) are correct. for endothermic reaction
60. (c) We know that the activation energy of chemical reaction DH = + ve
k2 Ea é T2 - T1 ù \ DH = Ea(f) – Ea(b) or Ea(f) = DH + Ea(b)
is given by formula = = ê ú , where k1 is \ Ea(f) > Ea(b).
k1 2.303R ë T1T2 û
the rate constant at temperature T1 and k2 is the rate 65. (d) In Arrhenius equation k = Ae - Ea / RT , Ea is the energy of
constant at temperature T2 and Ea is the activation energy. activation, which is required by the colliding molecules
Therefore activation energy of chemical reaction is to react resulting in the formation of products.
determined by evaluating rate constant at two different 66. (d) The definition of threshold energy.
temperatures. 1
67. (c) N 2 O ¾¾® N 2 + O2
61. (c) Order is the sum of the powers to which the concentration 2
terms are raised in the rate equation. dx
µ [N 2 O]1
62. (d) k = Ae–Ea/RT dt
i.e. order of reaction = 1
Ea
or log k = log A - 68. (b) When the temperature is increased, energy in form of
2.303 RT heat is supplied which increases the kinetic energy of the
Comparing the above equation with reacting molecules. this will increase the number of
y = mx + c collisions and ultimately the rate of reaction will be
1 enhanced.
y = log k, x = 69. (b) When Ea = 0 rate constant is independent of temperature.
T
E /RT
Thus a plot of log k vs 1/T should be a straight line, with 70. (b) k = Ae– a
slope equal to – Ea/2.303 RT and intercept equal to log A lnk = ln A – Ea/RT
For ln k vs 1/T
ln A = intercept
– Ea – Ea/R = slope = –2 × 104 K
Slope =
2.303RT \ Ea = 8.3 × 2 × 104 J mol–1
log k = 16.6 × 104 J mol–1 or 166 kJ mol–1
1
71. (b) A graph plotted between log k vs for calculating
T
1/T activation energy is shown as

- Ea
\ Slope =
2.303R log k
or Ea = –2.303R ´ Slope
n
1/T
N æ1ö from Arrhenius equation
63. (d) Q =ç ÷
N0 è 2 ø Ea
log k = log A -
For first condition 2.303 RT
n
1 æ1ö 72. (a) For endothermic reaction DH = + ve
=ç ÷ \ n =2
4 è2ø Then from equation DH = EFR - EBR ; EBR < E FR
EBD_7327
456 CHEMISTRY

73. (b) The presence of enzyme (catalyst) increases the speed of Actually this reaction is autocatalyzed and involves
reaction by lowering the energy barrier, i.e., a new path is complex calculation for concentration terms.
followed with lower activation energy. We can look at the above results in a simple way to find
the dependence of reaction rate (i.e., rate of
ET disappearance of Br2).
E'T From data (1) and (2) in which concentration of
CH3COCH3 and H+ remain unchanged and only the
Ea concentration of Br 2 is doubled, there is no change in
Products
Energy

Ea rate of reaction. It means the rate of reaction is


1
independent of concentration of Br 2.
Again from (2) and (3) in which (CH3CO CH3) and (Br2)
Reactants + catalyst remain constant but H+ increases from 0.05 M to 0.10
Progress of reaction i.e. doubled, the rate of reaction changes from 5.7×10–5
to 1.2 × 10–4 (or 12 × 10–5), thus it also becomes almost
Here ET is the threshold energy.
doubled. It shows that rate of reaction is directly
Ea and Ea is energy of activation of reaction in absence
1 proportional to [H+]. From (3) and (4), the rate should
and presence of catalyst respectively.
have doubled due to increase in conc of [H+] from 0.10
74. (d) For a first order reaction, M to 0.20 M but the rate has changed from 1.2× 10–4 to
2.303 a 3.1×10–4. This is due to change in concentration of
k= log
t a-x CH3COCH3 from 0.30 M to 0.40 M. Thus the rate is
2.303 1 2.303 ´ 0.903 directly proportional to [CH3 COCH3]. We now get
k= log = min -1 rate = k [CH3COCH3]1[Br2]0[H+]1
75 1/ 8 75
First order reaction = k [CH3COCH3][H+].
78. (d) Rate of disappearance of H2 = rate of formation of NH3.
0.693 0.693 ´ 75
t1/2 = = = 25 min 1 d[H 2 ] 1 d[NH3 ]
k 2.303 ´ 0.903 – =
75. (d) As the slowest step is the rate determining step thus the 3 dt 2 dt
mechanism B will be more consistent with the given – d[H 2 ] 3 d[NH3 ] 3 –4
information also because it involve one molecule of H2 Þ = = × 2×10
dt 2 dt 2
and one molecule of ICl it can expressed as
r = k [H2][ICl] = 3×10 –4 mol L–1s –1
Which shows that the reaction is first order w.r.t. both H2 79. (d) Rate of disappearance of Br –
& ICl. = rate of appearance of Br 2
76. (c) For a first order reaction 1 d[Br – ] 1 d[Br2 ]
Þ– =
2.303 a 5 dt 3 dt
k= log
t a-x 1 d [Br2 ] 1 d [Br – ]
when t = t½ Þ =-
3 dt 5 dt
2.303 a
k= log d[Br2 ] 3 d[Br – ]
t½ -
a a/2 Þ =-
dt 5 dt
2.303 ln 2 80. (b) For a first order reaction
or t½ = log 2 =
k k
0.693 0.693
77. (a) Rewriting the given data for the reaction t1/2 = ; k= = 0.5 × 10–3s–1
k 1386
H+
CH 3COCH 3 ( aq ) + Br2 ( aq ) ¾¾® 81. (a) When concentration A is doubled, rate is doubled. Hence
CH3COCH 2 Br(aq) + H + ( aq) + Br - ( aq) order with respect to A is one.
When concentrations of both A and B are doubled, rate
S. Initial concent Initialconcentr Initialconcentr Rate of increases by 8 times hence order with respect to B is 2.
No. -ration of -ation of Br2 -ation of H + disappearance \ rate = k [A]1 [B]2
CH3COCH3 in M in M of Br2 in Ms -1 Total order = 1 + 2 = 3
in M d dx
i.e. - [Br2 ]or 82. (b) N 2O5 (g) ¾¾
® 2NO2 (g) + 1/2 O 2 (g)
dt dt
d 1d d
1
2
0.30
0.30
0.05
0.10
0.05
0.05
5.7 ´ 10 -5
-5

dt
[ N2 O5 ] = +
2 dt
[ NO2 ] = 2 [O2 ]
dt
5.7 ´ 10
3 0.30 0.10 0.10 1.2 ´ 10-4 d
4 0.40 0.05 0.20 3.1 ´ 10-4 dt
[ NO2 ] = 1.25 ´ 10-2 mol L–1s–1 and
Chemical Kinetics 457

From the above, we have


d
dt
[ O2 ] = 3.125 ´ 10-3 mol L–1s–1 d 1 d
–2 [A] = [B]
83. (b) dt 2 dt
84. (d) In case of (II) and (III), keeping concentration of [A] d 1 d
or – [A] = [B]
constant, when the concentration of [B] is doubled, the dt 4 dt
rate quadruples. Hence it is second order with respect to i.e., correct answer is (b)
B. In case of I & IV Keeping the concentration of [B] 93. (b) For first order reaction,
constant. when the concentration of [A] is increased four
times, rate also increases four times. Hence, the order 0.693 0.693
k= =
with respect to A is one. hence t1/ 2 6.93
Rate = k [A] [B]2 2.303 100
k= log
85. (b) 2 NO (g) + Cl2(g) ƒ 2 NOCl(g) t 100 - 99
Rate = k [NO]2 [Cl] 0.693 2.303 100
The value of rate constant can be increased by = log
6.93 t 1
increasing the temperature.
\ Correct choice : (b) 0.693 2.303 ´ 2
=
86. (d) order of reaction may be zero, whole number or fractional. 6.93 t
87. (a) Rate= k[A] ° t = 46.06 min
Unit of k = mol L–1 sec–1 94. (b) For a first order reaction
r °C æ T2 -T1 ö æ 100 -10 ö 2.303 a 2.303 0.1
çè ÷ø çè ÷ k= log = log
100 10 ø
88. (b) = 2 10 =2 = 29 = 512 (where 2 is t a-x 40 0.025
r
10°C
2.303 2.303 ´ 0.6020
temperature coefficient of reaction) = log 4 =
89. (b, d) 40 40
According to Arrhenius equation –1
= 3.47 ´ 10 –2 min
k E æ1 1ö Rate = k[A] = 3.47 × 10–2 × 0.01
ln 2 = a ç - ÷
k1 R è T1 T2 ø = 3.47 × 10–4 M/min
95. (a) Activation energy can be calculated from the equation
Ea æ 1 1 ö log k2 - Ea æ 1 1 ö
=– - = -
R çè T2 T1 ÷ø log k1 2.303 R çè T2 T1 ÷ø

k1 E æ1 1ö k2
ln - aç - ÷
= Given = 2 ; T2 = 310 K ; T1 = 300 K
K2 R è T1 T2 ø k1
90. (a) Presence of catalyst does not affect enthalpy change of - Ea æ 1 1 ö
reaction DH R = E f - E b = 180 – 200 = – 20 kJ/mol = log 2 = ç - ÷
2.303 ´ 8.314 è 310 300 ø
91. (b) Since doubling the concentration of B does not change Ea = 53598.6 J/mol = 53.6 kJ/mol.
half life, the reaction is of 1st order w.r.t. B. 96. (d) The rate of a reaction is the speed at which the reactants
Order of reaction with respect to A = 1 because rate of are converted into products. It depends upon the
reaction doubles when concentration of A is doubled concentration of reactants. e.g for the reaction
keeping concentration of B constant.
A ∗ B ¾¾
↑ Product ; r ×[A][B]
\ Order of reaction = 1 + 1 = 2 and units of second order
reaction are L mol–1 sec–1. 97. (a) t1/2 = 4 s T = 16
92. (b) The rates of reactions for the reaction T 16
n= = =4 (Q T = n × t½)
1 t1 / 2 4
A ¾¾ ® 2B
2 n 4
can be written either as æ1ö æ1ö 0.12
A = Ao ç ÷ = 0.12 ´ ç ÷ = = 0.0075 M
è2ø è2ø 16
d
-2 [A] with respect to ‘A’’ where Ao = initial concentration &
dt
A = concentration left after time t.
1 d
or [B] with respect to ‘B’ dx
2 dt 98. (b) For a first reaction < k(a , x) on intergration
dt
EBD_7327
458 CHEMISTRY

dx Hence rate increases by 18 times.


ò (a - x ) ò
= k dt 6. (c) Half life time (t1/2) for nth order reaction is given by,
t1/2 µ [a]1– n
i.e – ln (a –x) = kt + c or kt = ln a – ln (a–x) where n is the order of reaction and a is concentration
or kt = 2.303 [log a – log (a – x)] of reactant.
Thus if we plot a graph between log a & t we get As decomposition of N2O5 follows 1st order kinetic.
So, Þ t1/2 µ [a]1 – 1 Þ t1/2 µ a0
st 7. (d) For a zero order reaction,
1 order t1/2 µ a0 (initial concentration or initial pressure)
log[a]
(t1/2)1 µ P1
(t1/2)2 µ P2
time t ( t1/2 )2 P2 ( t1/2 )2 16
99. (a) From data 1 and 3, it is clear that keeping (B) const, = , =
When [A] is doubled, rate remains unaffected. Hence
( t1/2 )1 P1 45 4
rate is independent of [A]. from 1 and 4, keeping [A] 16
(t1/2)2 = ´ 45 = 180 min
constant, when [B] is doubled, rate become 8 times. 4
Hence 8. (a) As doubling the initial conc. doubles the rate of
rate µ [ B]3 . reaction, order =1
9. (b) The rate of photochemical process varies with the
100. (c) We know that the activation energy of chemical reaction
intensity of absorption.
k2 E a é T2 , T1 ù Since greater the intensity of absorbed light more
is given by formula = log k < 2.303R êê T T úú , where photons will fall at a point, and further each photon
1 ë 1 2 û causes one molecule to undergo reaction.
k1 is the rate constant at temperature T1 and k2 is the Photolysis
® OH - + H +
H 2 O ¾¾ ¾ ¾¾
rate constant at temperature T2 and Ea is the activation
energy. Therefore activation energy of chemical reaction 10. (c) According to Arrhenius equation, the rate constant of
reaction is given by,
is determined by evaluating rate constant at two different
k = Ae – Ea /RT
temperatures.
where A is pre-exponential factor, Ea is activation energy
EXERCISE - 2 and T is absolute temperature.
E
1. (a) For a zero order reaction when T ® ¥, a ® 0
RT
a
t1/ 2 = then k = Ae0
2k or k=A
2. (c) Rate1 = k [A]n [B]m; Rate2 = k [2A]n [½B]m 11. (b) As the rate of reaction get doubled for every 10°C rise
n m in temperature. Hence the increase in reaction rate as a
Rate2 k[2A] [½B] result of temperature rise from 10°C to 100°C is equal to
\ = = [2]n [½]m = 2n.2–m = 2n–m
Rate1 k[A]n [B]m = 29 = 512
12. (b) N2 + 3H2 ‡ˆˆˆˆ† 2NH
3. (d) The velocity constant depends on temperature only. It is 3
independent of concentration of reactants. -D[N 2 ] 1 D[H 2 ] 1 D[NH3 ]
4. (c) Given, k =6.909 min–1 =- =
Dt 3 Dt 2 Dt
For a first order reaction
2.303 æ a ö -D[H 2 ] 3 D[NH3 ] 3
k= log ç \ = ´ = ´ 2 ´ 10-4
t è a - x ÷ø Dt 2 Dt 2
2.303 æ 100 ö 2.303 æ 100 ö = 3 × 10–4 mol litre–1sec–1
log ç = log ç
6.909 = t è 100 - 75 ÷ø t è 25 ÷ø 13. (d) Here, t1/2 = 4.5 × 109 years
2.303 1 2 Molar ratio of U238 : Pb206 = 1 : 3
Þt= log 4 Þ t = log(2 2 ) = log 2.
6.909 3 3 Age of the rock can be determined according to the
5. (c) Reaction Rate R1 = k [A]2 [B] formula
Now increase conc. of A by three times and conc. of B 2.303 ´ t1/ 2 é Pb 206 ù 2.303 ´ 4.5 ´ 109
by two times. Then new rate t= log ê1 + ú = log [1 + 3]
0.693 ë U 238 û 0.693
R2 = k [3A]2 [2B]
2.303 ´ 4.5 ´ 109
R1 k[A]2 [B] 1 1 1 = ´ 0.6020
= 2
= 2´ = 0.693
R 2 k[3A] [2B] 3 2 18 \ t = 9 × 109 years.
R2 = 18 × R1
Chemical Kinetics 459

3 æ 300 ´ 310 ö
14. (a) The order of reaction is and molecularity is 2. Ea = 0.3010 ´ 2.303 ´ 8.314 ç ÷ø
2 è 10
dx = 53598.59 = 54 kJ.
15. (c) Since - µ [CO]2 so on doubling the concentration 24. (b) T2 = T (say), T1 = 25°C = 298K,
dt Ea = 104.4 kJ mol–1 = 104.4 × 103 J mol–1
of CO the rate of reaction will increase by 4 times. k1=3 × 10 – 4, k2 = ?,
16. (d)
k2 Ea é 1 1 ù
17. 2 days
(c) 100 ¾¾¾ 4 days log = -
®50 50 ¾¾¾
® 25 k1 2.303 R êë T1 T2 úû
8 days
25 ¾¾¾ ®12.5
1 k2 104.4 ´ 103 J mol-1
é 1 1ù
From the data given above we conclude t1/ 2µ . If log = - ú
[A]0 3 ´10 -4
2.303 ´ (8.314 J K mol ) ê
ë 298 -1
T û -1
we plot a graph between half life and 1/[A]0 it will look 1
like As T® ¥, ®0
T
k2 104.4 ´ 103 J mol -1
\ log =
t1/2 3 ´ 10-4 2.303 ´ 8.314 ´ 298
k2 k2
log -4
= 18.297, = 1.98 ´ 1018
3 ´ 10 3 ´ 10-4
1/[A]0 k2 = (1.98 ´ 1018 ) ´ (3 ´ 10-4 ) = 6 ´ 1014 s -1
Above graph is a characteristic of second order
reaction. t1/2 of reaction = 2 days. k 2 E a é T2 - T1 ù
25. (a) 2.303log =
k1 R êë T1T2 úû
0.693 2.303 a
18. (b) k= min -1 = log or
45 t99.9% a - 0.999a k 2 9.0 ´103 é 308 - 298 ù
log =
k1 2.303 ´ 2 êë 308 ´ 298 úû
2.303 ´ 45
t99.9% = log103 = 448 min » 7.5 hrs
0.693 k2
= 1.63; k 2 = 1.63k1;
k1
19. (c) 2.303 100 2.303 100
k= log = log k 2 - k1
1 hr 25 t 50 Increase in k1 = ´100
k1
1 1 1 1.63k1 - k1
\ log 4 = log 2 \ 2log 2 = log 2; t = hr. = ´ 100 = 63.0%
t t 2 k1
1-n n -1
0.1 (200)1-n 1 é4ù é1ù 26. (a) k remains constant at constant temperature and CO
20. (b) t1/ 2 µ a1- n Þ = 1- n
Þ =ê ú =ê ú does not effect the rate of reaction.
0.4 (50) 4 ë1 û ë4û 27. (a) For a first order reaction
1 1 2.303 a
Þ = n -1 \ n - 1 = 1; n = 2
4' 4 k= log
t a-x
21. (c) Increase in rate = 2(T2 - T1)/5 = 2(40 - 20) / 5 = 2 4 = 16 times when t = 60 and x = 60%
2.303 100 2.303 100
2.303 a k= log = log
22. (b) k= log 60 100 - 60 60 40
t (a - x ) Now,
(a – x) is the concentration left after 100 sec.
2.303 100 2.303
2.303 0.29 t1 = log = ´ log 2
2.7 ´ 10-3 = log 2 0.0153 100 - 50 0.0153
100 ( a - x)
0.27 0.29 0.29 2.303
Þ = log Þ 0.117 = log = ´ 0.3010 = 45.31 min .
2.303 ( a - x) (a - x ) 0.0153
2000
Þ (a – x) = 0.22 M. -
28. (d) Given, k1 = 1016.e T
k2 Ea é 1 1 ù
log = - 1000
23. (b) k1 2.303 R êë T1 T2 úû and k2 = 1015.e T
-

k2 when k1 and k2 are equal at any temperature T, we have


If =2
k1 2000 1000
- -
1016.e T 15
= 10 .e T
Ea é 1 1 ù
log 2 = - 2000 1000
2.303 ´ 8.314 êë 300 310 úû 15
-
15
-
or 10 ´ 10 .e T
= 10 .e T
EBD_7327
460 CHEMISTRY

2000 1000 For equation,


- -
or 10.e T
=e T ˆˆ† H+ + HS–
H2S ‡ˆˆ
2000 1000 é H + ù é HS- ù K [ H 2 S]
or ln 10 - =-
K = ë ûë û -
T T or éë HS ùû =
H 2S H+
2000 1000
or ln 10 = - Substituting this value in equation (i) we find
T T
1000 Rate = k [ Cl2 ] K
[ H 2S] = k ' [ Cl2 ][ H 2S]
or 2.303 log 10 = H+ éH+ ù
T ë û
or 2.303 ×1×T=1000 [\ log 10= 1]
Thus slow step should involve 1 molecule of Cl2 and
1000 1 molecule of H2S.
or T = K
2.303 hence only , mechanism (A) is consistent with the given
29. (b) Rate of disappearance of reactant = Rate of appearance rate equation.
of products 33. (c) Since for every 10ºC rise in temperature rate doubles
1 d[N 2 O5 ] 1 d[NO 2 ] d[O 2 ] for 50ºC rise in temperature increase in reaction rate
- = = = 25 = 32 times
2 dt 4 dt dt
34. (d) Overall order = sum of orders w.r.t each reactant.
1 1 Let the order be x and y for G and H respectively
k[N 2O5 ] = k ¢[N 2O5 ] = k ¢¢[N 2O5 ]
2 4
[G]mole [H]mole rate(mole
k k¢ k Exp.No.
= = k ¢¢ k¢ = 2k, k ¢¢ = litre-1 litre -1 litre- time -1 )
2 4 2
30. (a) Since initial velocity is ten times the permissible value 1 a b r
\ A0 = 10A 2 2a 2b 8r
0.693 0.693 3 2a b 2r
l= =
t1/ 2 30 Q For (1) and (3), the rate is doubled when conc. of
G is doubled keeping that of H constant i.e.,
2.303 A 2.303 10A
t1/2 = log 0 = log rate µ [G] \ x = 1
l A 0.693 / 30 A
From (2) and (3), y = 2
2.303 ´ 30 \ Overall order is 3.
= ´ log10 = 100 days.
0.693 35. (a) The values of rate constants k0, k1 for zero order and
31. (c) For the reaction first order reaction, respectively, are given by the
A ® Product following equation:
given t1/ 2 = 1 hour
A0
for a zero order reaction k0 = [where A0 = initial concentration,
2×t1/2
t1 / 2 =
[ A0 ]
and t1/2 = half-life period]
2k
0.693
or k =
[ A0 ] =
2
= 1 mol lit –1 hr–1
and k1 =
t1/ 2
2 t1/ 2 2 ´1 Substituting various given values, we get
Further for a zero order reaction
1.386 mol litre –1
dx change in concentration k0 = ... (i)
k= = 2 ´ 20 sec
dt time
0.693
0.50 - 0.25 and k1 = ... (ii)
1= 40sec
time
Dividing (ii) by (i), we get
\ time = 0.25 hr.
32. (d) Since the slow step is the rate determining step hence k1 0.693 2´ 20
= ´
if we consider option (A) we find k0 40 1.386 mol dm -3
Rate = k [ Cl2 ][ H 2S]
0.693
Now if we consider option (B) we find = = 0.5
1.386
Rate = k [ Cl2 ] éë HS ùû
- = 0.5 mol–1 dm3
...(i)
Thus the correct answer is (a).
Chemical Kinetics 461

Ea 2.303 a
36. (d) log k = log A - …(1) k= log
2.303RT t a-x
1 2.303 1 2.303 ´ 0.903
Also given log k = 6.0 - (2000) …(2) k= log = min -1
T 75 1/8 75
On comparing equations, (1) and (2) First order reaction
log A = 6.0 Þ A = 106 s–1 0.693 0.693 ´ 75
t1/2 = = = 25 min
Ea k 2.303 ´ 0.903
and 2.303 R = 2000 ; E /RT
43. (b) k = Ae– a
ÞEa = 2000 × 2.303 × 8.314 = 38.29 kJ mol–1 lnk = ln A – Ea/RT
37. (a) As per Arrhenius equation (k = Ae - Ea / RT ) , the rate For ln k vs 1/T
ln A = intercept
constant increases exponentially with temperature. – Ea/R = slope = –2 × 104 K
38. (a) From the first set of data (i) and (ii) it is observed that \ Ea = 8.3 × 2 × 104 J mol–1
on keeping concentration of [B] constant and on = 16.6 × 104 J mol–1 or 166 kJ mol–1
doubling the concentration of [A] rate does not n
changes hence order of reaction with respect to A is N æ1ö
44. (d) Q =ç ÷
zero. N0 è 2 ø
From the second set of data (i) and (iii) it is observed that
For first condition
rate becomes 4 times on doubling the concentration of
n
[B] keeping [A] constant hence order with respect to [B] 1 æ1ö
will be 2 =ç ÷ \ n =2
4 è2ø
rate = k[A]0 [B]2
39. (d) For a zero order reaction, 20
t1/ 2 = = 10
t1/2 µ a0 (initial concentration or initial pressure) 2
(t1/2)1 µ P1 For second condition
(t1/2)2 µ P2 n
æ 1 ö æ 1ö
( t1/2 )2 P2 ( t1/2 )2 16 çè ÷ø = çè ÷ø
= , = 16 2
( t1/2 )1 P1 45 4 n = 4 \ T = 4 × 10 = 40 min
45. (c) Order is the sum of the powers to which the
16 concentration terms are raised in the rate equation.
(t1/2)2 = ´ 45 = 180 min
4 46. (b) According to Arrhenius equation, k = Ae–Ea/RT
40. (d) From 1st and 2nd sets of data - no change in rate is \ when Ea = 0, k = A
observed with the change in concentration of ‘C’. So Also ln k us 1/T is a straight line with slope = –Ea/R.
the order with respect to ‘C’ is zero. \ Statements (ii) and (v) are correct.
From 1st and 4th sets of data
Dividing eq. (4) by eq. (1) 47. (b) According to Arrhenius equation, k = Ae - E / RT
x
When Ea = 0, k = A
1.25 ´ 10 -3 é 0.005 ù 48. (b) For a zero order reaction, t1/2[A0]/2k

5.0 ´ 10 -3
ë 0.010 úû 49. (b) According to Arrhenius equation, k = Ae–Ea/RT.
or 0.25 = (0.5) or (0.5)2 = (0.5)x
x
EXERCISE - 3
\ x=2
The order with respect to ‘A’ is 2 from the 1st and 3rd Exemplar Questions
sets of data Dividing eq. (1) by eq. (3) 1. (c) The role of a catalyst is to change the activation energy
5.0 ´ 10-3
y
é 0.005 ù of reaction.

1.0 ´ 10 -2
ë 0.010 úû 2. (c) The heat absorbed or evolved during the reaction
or (0.5) = (0.5)y Þ y = 1
1 remains unchanged in presence of catalyst as there is
The order with respect to ‘B’ is 1 no change in stability of reactant and product.
So the order with respective the reactants A, B and C is 3. (b) Arrhenius equation relates activation energy of a
2, 1 and 0. chemical reaction with rate constant of a reaction at
two different temperatures.
1
41. (c) N 2 O ¾¾® N 2 + O2
2
æk ö E é1 1 ù
dx ln ç 1 ÷ = a ê - ú
µ [N 2 O]1 è k2 ø R ë T1 T2 û
dt
i.e. order of reaction = 1 4. (a) The minimum energy required to convert reactant
42. (d) For a first order reaction, molecules into product molecules.
EBD_7327
462 CHEMISTRY

i.e. Activation energy = E1 + E2 12. (c)


Energy of product is greater than the reactants so less 5Br - ( aq ) + BrO3- ( aq ) + 6H + ( aq ) ® 3Br2 ( aq ) + 3H 2 O ( l )
stable.
Rate law expression as
5. (b) A(g) ¾® B(g) + C(g)
Initially pi 0 0 1 D [ Br - ] D é BrO3- ùû -1 D [ H + ] +1 D [ Br2 ]
At time t pi – x x x - =- ë = =
5 Dt Dt 6 Dt 3 Dt
Total pressue at equilibrium, pt = pi – x + x + x = pi + x
For first order reaction x = pt – pi D [ Br - ] D é BrO3- ùû -5 D [ H + ]
Þ– =- ë =
2.303 p Dt Dt 6 Dt
k= log i
t pi - x D [ Br - ] 5 D [ H + ]
Þ =
2.303 pi Dt 6 Dt
= log
t pi - ( p t - p i ) 13. (a) Ea(F.R.) + DH = Ea(B.R.)
DH = – ve
2.303 pi
= log Activated complex
t 2pi - p t
6. (a) According to Arrhenius equation, k = A e - E a / RT

Taking log on both side ln k = ln A e( -


Ea
RT ) Ea(B.R.)

Energy
Ea(F.R.)
Ea DH
ln k = ln A -
RT
Ea 1
ln k = - ´ + ln A Reaction coordinate
R T
14. (b) Rate of reaction w.r.t B is of first order
- Ea R1 = k [A] [B]
From the graph, it is clear that slope = and when concentration of reactant 'B' is doubled then rate
R
intercept = ln A. (R2) will be :
R2= k [A] [2B]
7. (d) As per Arrhenius equation k = A e - Ea / RT R2= 2k [A] [B]
So, k µ e - Ea R2 = 2 R1
15. (c) In collision theory of reaction rates the key factors
1
- which determine whether a particular collision will
kµe T
result in a reaction is the energy of collision and the
µ eT orientation of the collision.
These relations show that rate constant increases with 16. (d) The time in which the concentration of a reactant is
decreasing activation energy and increasing reduced to half of its original value is called half-life
temperature. period of the reaction.
But it is impossible to perform 100% of the reaction.
8. (c) Zn + dil. HCl ¾¾ ® ZnCl 2 + H 2 ­ Because substance never reacts completely as in every
Rate of reaction half-life, 50% of the substance reacts. So, time taken
Change in concentration of H 2 V3 - 0 V3 for 100% completion of a reaction is infinite.
= = = 17. (b) r = k [A]x[B]y
Change in time 40 - 0 40
x y
Rate of exp .1 [ 0.30 ] [ 0.30]
9. (c)
Order of reaction may or may not be equal to sum of =
stoichiometric coefficients of reactants in the balanced Rate of exp. 2 [ 0.30]x [ 0.60] y
chemical equation. 0.10 [ 0.30 ]
y
10. (b) Instantaneous rate of reaction =
0.40 [ 0.60 ]y
Change in volume
= Time interval close to 40s y
1 é1ù
=
V4 - V2 V - V2 4 êë 2 úû
rinst ( 20s ) = incorrect, correct is 5 2 y
50 - 30 50 - 30 é1ù é1ù
êë 2 úû = êë 2 úû
11. (a) Concentration of reactant decreases with passage of y= 2
time, so rate of reaction decreases because rate
x y
changes with change in concentration of reactant or Rate of exp.1 [ 0.30] [ 0.30 ]
=
product with time. Rate of exp. 3 [ 0.60]x [ 0.30 ]y
Chemical Kinetics 463

x y For first order t1/2 is independent of initial concentration


0.10 é 0.30 ù é 0.30 ù
= of reactant.
0.20 êë 0.60 úû ëê 0.30 ûú
27. (d) Rate constant k = 0.6 × 10–3 mole per second. (unit mole
x
1 é1ù per second shows zero order reaction)
= [1] y
2 êë 2 ûú For a zero order reaction
x
1 é1ù [A] = [A]0 – kt
=
2 êë 2 úû and [A0] – [A] = [B] = kt
i.e., x = 1 = 0.6 × 10–3 × 20 × 60 = 0.72 M
\ Rate = k [A]x[B]y 28. (a) For a first order reaction
Rate = k [A]1[B]2
2.303 ( a - x1 )
18. (b) Catalyst does not change Gibbs free energy because K = t - t log a - x
Gibbs free energy is related to concentration of reactant ( 2 1 ) ( 2 )
& product which is not changed by catalyst. 2.303 æ 0.04 ö
K= ( log ç ÷
19. (b) Pseudo first order reaction is a chemical reaction in 20 - 10 ) è 0.03 ø
which rate of reaction depends upon concentration of 2.303 ´ 0.1249
only one reactant while concentration of another K=
10
reactant has no effect on rate of reaction. 0.6932 2.303 ´ 0.1249
20. (b) A®B =
t1/2 10
As time increases concentration of reactant decreases
and concentration of product increases exponentially. 0.6932 ´ 10
t1/2 = = 24.1 sec
2.303 ´ 0.1249
NEET/AIPMT (2013-2017) Questions
29. (d) A catalyst provides an alternative route for the reaction
k Ea æ 1 1ö with a lower activation energy.
21. (b) log 2 = ç - ÷
k1 2.303R è T1 T2 ø 30. (c) The overall reaction rate depends on the rate of the
slowest step.
Ea é 1 1 ù
log 2 = - i.e., Overall rate = Rate of slowest step (ii)
2.303 ´ 8.314 ë 293 308 úû
ê
Ea 15 = k[X][Y2] … (1)
0.3 = × k = rate constant
2.303 ´ 8.314 293 ´ 308
0.3 ´ 2.303 ´ 8.314 ´ 293 ´ 308 Assuming step (i) to be reversible, its equilibrium
Ea = . constant,
15
= 34673 J mol–1 = 34.7 kJ mol–1 [ X ]2
k eq = Þ [ X ] = k eq [ X 2 ] ;
2
22. (b) DH = Ea f - Ea b = 0 [ X2 ]
23. (b) For a first order reaction, 1 1
t75% = 2 × t50% [ X] = k eq2 [ X2 ] 2 … (2)
24. (d) Rate1 = k[A]2[B]3 From eq (1) and (2)
when concentrations of both A and B are doubled then 1 1
Rate2 = k[2A]2[2B]3 = 32 k[A]2[B]3 Rate = kk eq 2 [ X 2 ] 2 [ Y2 ]
\ rate will increase by a factor of 32. 1 3
Overall order = + 1 = = 1.5
25. (b) Arrhenius equation 2 2
31. (a) Half life for a first order reaction,
- E a /RT Ea
K=A.e Þ ln K = ln A – 0.693
RT t1/2 =
K
–E a 0.693
slope = sec .
R So, t1/2 =
10-2
so, activation energy of reaction can be determined from
Also, for the reduction of 20 g or reactant to 5 g, two
1 half lives will be required.
the slope of ln K vs
T \ For 20 g of the reactant to reduce to 5g, time taken,
0.693 0.693
26. (a) t1/ 2 = t=2× sec = 138.6 sec.
k 10-2
EBD_7327
464 CHEMISTRY

19 Surface Chemistry

Surface chemistry deals with the phenomena that occur at the Mechanism of Adsorption
surfaces or interfaces. Particles at the surface of adsorbent are in different state than
ADSORPTION inside the bulk. On the surface particles are not surrounded by
atoms or molecules of their kind on all sides and hence they
The accumulation of molecular species at the surface rather than
possess unbalanced or residual attractive forces. Th ese
in the bulk of a solid or a liquid is termed Adsorption. The substance
unbalanced inward forces or free valencies at the surface of
which accumulates at the surface is called adsorbate and the material
adsorbent are responsible for attracting the adsorbate particles
on the surface of which adsorption takes place is called adsorbent.
on its surface.
Examples of adsorbents are activated charcoal, Pt, Ni, Pd. etc.
The process of removing an adsorbed substance from a surface Characteristics of Adsorption:
on which it is adsorbed is called Desorption. (i) Adsorption is a surface phenomenon
Absorption (ii) It is spontaneous process.
Adsorption is different from absorption. In adsorption, the (iii)· Adsorption is exothermic because during adsorption, residual
substance is concentrated only at the surface while in absorption, forces of the surface decrease i.e., surface energy decreases
the substance is uniformly distributed throughout the bulk of the which appears as heat i.e., DH is negative.
solid. For ex: water vapour. Water vapours are absorbed by (iv) For adsorption, DS is negative because the molecules of
anhydrous CaCl2 but adsorbed by silica gel. adsorbate are held on the surface of the adsorbent.
Both absorption and adsorption can take place simultaneously. (v) As DG = DH – TDS, \ for adsorption to be spontaneous, DG
This process in which absorption and adsorption take place must be negative. As DS is negative, DG can be negative
simultaneously is called sorption. only if DH = –ve and DH > TDS.
\ for adsorption, DH = –ve
DS = –ve
Types of Adsorption DG = –ve if DH > TDS
Depending upon the nature of forces between molecules of adsorbate and adsorbent, adsorption is of two types :
(i) Physical adsorption (ii) Chemical adsorption

Phys ical ads orption (Phys ios orption) Chemical A ds orption (Chemis orption)
1 The forces operating in these cases are weak van der 1 The forces operating are s imilar to thos e of a chemical
waals forces bond.
2 The heats of adsorptio n are low, viz about 20-40 kJ 2 The heats of ads orption are high, viz, about 40-400 kJ
–1 –1
mol mol
3 No co mpound formation takes place. 3 Surface compounds are formed.
4 The proces s is revers ible 4 It is an irrevers ible proces s .
5 It does not require any activation energy 5 It requires activation energy.
6 It us ually occurs at low temperature and decreas es with 6 It firs t increases with increas e of temperature.
increase of temperature.
7 It is not s pecific in nature i.e., all gas es are ads orbed on 7 It is s pecific in nature and occurs only when there is
all solids to some extent. some pos s ibility of compound formation between
ads orbate and ads orbent.
8 It forms multimolecular layer 8 It forms unimolecular layer.
Surface Chemistry 465

Factors Affecting Adsorption Langmuir Adsorption Isotherm


(i) Nature of adsorbate: Readily liquefiable gases such as HCl, Acc to his theory of adsorption,
SO2, CO2, CH4, NH3 etc. are more easily adsorbed by the (i) Adsorption could take place on the surface of a solid only till
adsorbent than the permanent gases H2, N2 etc. whole of the surface was completely covered with a
(ii) Nature of absorbent: Activated charcoal is a better adsorbent unimolecular layer of the gas molecules.
than transitional metals. (ii) Adsorption consists of two opposing process, namely
(iii) Surface area of adsorbent: The greater the surface area of condensation of gas molecules on solid surface and
adsorbent, the greater will be the extent of adsorption. evaporation of gas molecules from surface back into gaseous
(iv) Pressure: Extent of physical adsorption increases as the phase.
pressure of the gas increases, till a saturation point is reached. (iii) The rate of condensation depends upon the bare surface
(v) Temperature: Adsorption is accompanied by evolution of available for condensation whereas rate of evaporation
heat i.e., DH is negative, so the rate of adsorption should depends upon the covered surface.
decrease with rise in temperature. It is found to be so in case and, mathematically, the langmuir adsorption isotherm is given
of physical adsorption. The effect of temperature is as :
represented by an adsorption isobar. x ap
(vi) Activation of adsorbent: An adsorbent can be activated either = ...(1)
m 1 + bp
by heating or by bringing it in finely divided state, or by
where p ® pressure of the gas at any instant and
making its surface rough by rubbing. For example charcoal is
a and b are constants whose values depend upon the nature
activated by heating it in vacuum at 1000ºC.
of the gas adsorbed, the nature of adsorbent and temperature.
ADSORPTION ISOTHERM The above equation can be written as
The variation in the amount of gas adsorbed by adsorbent with x m a p 1 + bp 1 b
pressure at constant temperature is expressed with help of = or = = + p
p 1 + bp x m a a a
Adsorption Isotherm. Thus a plot of p/x/m vs p is a straight line with slope
Freundlich Adsorption Isotherm
b 1
A relationship was given by Freundlich which is as follows: = and intercept on y-axis =
x a a
= kp1/ n (n > 1)
m Application of Adsorption
Where x ® mass of gas adsorbed (i) Production of high vacuum (by using charcoal)
m ® mass of adsorbent (ii) Gas masks
p ® pressure (iii) Control of humidity (by using silica and aluminium gels)
k, n ® Constants which depend on nature of adsorbent (iv) Removal of colouring matter from solutions (using animal
and the gas at particular temperature. charcoal)
(v) Heterogeneous catalysis
(vi) Separation of inert gases (using coconut charcoal)
(vii) In curing diseases
(viii) In froth floatation process
(ix) Adsorption indicators (Example : Silver halides)
(x) Chromatographic analysis.
CATALYSIS
Catalyst is a substance which can change the speed of a chemical
reaction without itself undergoing any change in mass and
(i) At low pressure : x/m is directly proportion to pressure i.e. chemical composition at the end of the reaction and the
x/m µ p. phenomenon is known as catalysis.
(ii) At high pressure : The extent of adsorption x/m becomes Certain substances which enhance the activity of a catalyst are
independent of pressure i.e., x/m µ p°.
called promoters. Those which lower the activity of the catalyst
(iii) At intermediate pressure : x/m will depend upon the power
are called poisons and process is called catalytic poisoning.
of pressure which lies between 0 and 1.
The curves seem to approach saturation at high pressure. Types of Catalysis
x 1 Catalysis is of two types :
Taking log, we have log = log k + log p (i) Homogeneous catalysis: When catalyst, reactants and the
m n
products are in same phase the pr ocess is called
homogeneous catalysis.
NO(g)
Example : 2SO2 (g) + O2 (g) ¾¾¾® 2SO3 (g)
(ii) Heterogeneous catalysis : When the catalyst is in different
phase than that of reactants and products the process is
known as heterogeneous catalysis.
Fe(s)
Example: N 2 (g) + 3H 2 (g) ¾¾¾ ® 2NH3 (g)
EBD_7327
466 CHEMISTRY

Classification of Catalysts cracks have more active centres and more effective e.g. finely
Catalysts are divided into four types: divided nickel or platinum.
(i) Positive catalysts: The substance which increases the rate Important Features of Solid Catalysts
of a reaction is known as a positive catalyst. It decreases the (i) Activity: It is the capacity of a catalyst to increase the speed
energy of activation for the reaction. of a chemical reaction.
(ii) Negative catalysts: The substance which decreases the rate (ii) Selectivity: It is ability of a catalyst to direct a reaction to
of chemical reaction is called negative catalyst or inhibitor. It yield a particular product.
increases the activation energy for the reaction. Shape - Selective Catalysis
(iii) Autocatalyst: When one of the products of the reaction The catalytic reaction which depends upon the pore structure of
begins to act as a catalyst, it is called auto-catalyst. the catalyst and the size of reactant and product molecules is
(iv) Induced catalyst: When a chemical reaction enhances the called shape-selective catalysis. zeolites are good shape - selective
rate of another chemical reaction, it is called induced catalysis. catalyst because of their honeycomb - like structures.
General Characteristics of Catalysts: ENZYME CATALYSIS
(i) A catalyst remains unchanged in mass and chemical Many complex chemical reactions like oxidation, reduction or
composition. hydrolysis take place in presence of enzymes which are highly
(ii) Only a very small amount of catalyst is sufficient to catalyze complex, nitrogenous non living organic substances. They are
a reaction. highly specific in nature.
(iii) A catalyst does not initiate a reaction & does not control a
chemical reaction. Mechanism of Enzyme Catalysed Reactions:
(iv) When a catalyst is a solid, it is usually more efficient when The most accepted mechanism of enzyme catalysed reaction is
used in finely divided form. lock and key mechanism. Enzymes are highly specific in their action
(v) Generally catalyst does not change the nature of products. due to presence of active sites on their surface. The shape of the
active site in any enzyme is like a cavity such that a specific
(vi) A catalysts does not change the equilibrium state of a
substrate can fit into it in the same way as one key can fit into a
reversible reaction but helps to timely achieve equilibrium
particular lock. This binding gives enzyme-substrate complex.
state or position of equilibrium.
The reaction takes place as:
Theory of Catalysis: E + S ƒ ES ® P + E
(i) Intermediate theory
COLLOIDS
(ii) Adsorption theory
(iii) Modern theory A colloid is a heterogeneous system in which one substance is
(i) Intermediate theory - A catalyst combines with one of the dispersed (dispersed phase) as very fine particles in another
reactants and forms the unstable intermediate compound substance called dispersion medium. Colloidal particles are larger
than simple molecules but small enough to remain suspended.
which in turn combines with another reactant and generates
Their range of diameter in between and 1000 nm (10–9 to 10–6 m).
the catalyst.
Dispersed phase : Discontinuous phase of colloidal system is
(ii) Adsorption theory - By the adsorption of the reactants at the known as dispersed phase.
surface of a catalyst, their concentrations are increased. The Dispersed medium : The continuous phase of colloidal system
rate being directly proportional to concentration, is increased. known as dispersion medium.
(iii) Modern theory - It is the combination of intermediate theory
and adsorption theory. At the surface of a catalyst there are
Classification of Colloids
residual or free valencies and reactant molecules are retained Colloids are classified on the basis of following criteria:
at the surface in distorted or strained positions. These (i) Physical state of dispersed phase and dispersion medium
molecules then react vigrously to give products and make (ii) Nature of interaction between dispersed phase and
space for other reactant molecules. The greater the surface dispersion medium.
area of the catalyst, the more is its activity. Further catalysts (iii) Type of particles of dispersed phase.
with rough surface, having greater number of corners, peaks, These are discussed as follows:
(i) On the basis of physical state of dispersed phase and dispersion medium:
Dis persed phas e Dis persion M edium Type of colloid Examples
1 Solid Solid Solid s ol gem stones alloys
2 Solid Liquid Sol Paints , cell fluids, colloidal gold
3 Solid Gas Aeros ol Smoke, dus t, occluded gases
4 Liquid Solid Gel Chees e, butter, jellies
5 Liquid Liquid Emuls ion M ilk, hair cream
6 Liquid Gas Aeros ol Fog, mist, cloud, insecticide s prays
7 Gas Solid Solid s ol Pumice s tone, foam rubber
8 Gas Liquid Foam Froth, whipped cream, s oap lather
Surface Chemistry 467

Note: A colloidal dispersion of one gas in another is not possible (ii) A sol of sulphur is prepared by passing H2S into SO2 solution.
since the two gases would give a homogeneous molecular oxidation
structure. SO 2 + 2H 2S ¾¾¾¾ ® 3S(sol) + 2H 2O
(ii) Based on the nature of interaction between dispersed phase (iii) Ferric hydroxide sol is prepared by pouring dilute solution of
and dispersed medium ferric chloride into boiling water
hydrolysis
(a) Lyophilic colloids : When dispersed phase has certain FeCl3 + 3H 2 O ¾¾¾¾® Fe(OH)3 (sol) + 3HCl
affinity for the dispersion medium it is known as lyophilic (iv) Sols of gold, platinum and silver are prepared by reduction of
sol. their compounds in H2O using tannic acid or hydrozine.
Characteristics of lyophilic colloids: Bredig’s arc method :
1. They can be prepared easily by directly mixing with liquid
dispersion medium An electric arc is struck between electrodes of the metal immersed
2. They are stable and cannot be easily precipitated or in dispersion medium. The intense heat produced vapourises the
coagulated. metal which then condenses to form particles of colloidal size. It is
3. They are highly hydrated. used for metals like Au, Ag, Pt, etc.
4. Their particles carry a characteristics charge either Peptization :
positive or negative. It is defined as a process of converting a precipitate into colloidal
5. Reversible in nature, i.e., once precipitated can reform sol by shaking it with dispersion medium in the presence of a
the colloidal sol by simply remixing with the dispersion small amount of electrolyte. The electrolyte used for this purpose
is called peptizing agent.
medium. Usually formed by organic substances like
Note:
starch, gum, proteins, etc.
The cause of peptization is that during peptization, the ppt adsorbs
6. Surface tension is almost same as that of solvent.
(b) Lyophobic colloids: When dispersed phase has no one of the ions of the electrolyte on its surface. This causes the
affinity for dispersion medium it is known as lyophobic development of positive or negative on ppt, which ultimately break
sol. up into smaller particles of colloidal size.
Characteristics of lyophobic colloids. Purification of Colloidal Solutions
1. Prepared by special methods only and gets easily The process used for reducing the amount of impurities to a
precipitated by addition of a small amount of a suitable requisite minimum amount known as purification of colloidal sol.
electrolyte. Difference methods of purification of colloidal solutions are :
2. Not much hydrated. (i) Dialysis: It is a process of removing a dissolved substance
3. The particles have little charge or no charge at all. from a colloidal solution by means of diffusion through a
4. Irreversible in nature i.e., cannot be reformed by simple suitable membrane. The membrane used can be animal
addition of dispersion medium usually formed by membrane, or parchment paper or cellophane sheet as
inorganic substances. colloidal particles cannot pass through them.
5. Like metals, their sulphides, etc. A mixture containing colloidal particles and true solution is
6. The particles have little or no charge at all. placed in parchment bag. The bag is hanged in water vessel
Note: through which it is continously flowing. True solution comes
(i) In associated colloids, micelle formation takes place only out of membrane leaving behind colloidal solution.
above a particular temperature called Kraft temperature (Tk) (ii) Electro-dialysis: To increase the speed of dialysis process,
and above a particular concentration called critical micelle an electric field is applied (if impurity is an electrolyte) and
concentration (CMC). the process is called electro-dialysis.
(ii) Micelle formation: In this process vessel is fitted with electrodes which makes
Soap is sodium or potassium salt of a higher fatty acid RCOO– the removal of electrolytes fast.
Na+. The RCOO– ions consist of a long hydrocarbon chain R (iii) Ultrafiltration: It is a process of separating the colloidal
which is hydrophobic and a polar group COO– which is particles from the solvent and soluble solutes present in
hydrophilic. The cleansing action of soap is due to the fact colloidal solution by specially prepared filters which are
that soap molecules form micelle around the oil droplet in permeable to all substances except colloidal particles.
such a way that hydrophobic part is in the oil droplet and Properties of Colloidal Solutions
hydrophilic part projects out of the grease droplet. Since the
(i) Colligative properties: Values of colligative properties are
polar groups can interact with water, the oil droplet is pulled
smaller as compared to true solutions because colloidal
in water and removed from the dirty surface. Thus, soap helps
particles being bigger aggregates, the no. of particles in the
in emulsification and washing away of oils and fats.
solution are small as compared to true solution.
Preparation of Colloids (ii) Tyndall effect: It is defined as the scattering of light by
Some important methods for preparation of colloids are: colloidal particles present in colloidal sol. It is due to this
Chemical methods scattering of light that the path of light is illuminated with
(i) Double decomposition: An arsenic sol is prepared by passing bluish light when viewed at right angles to the direction of
H2S through cold solution As2O3 till yellow colour deepens light.
to it maximum. Ex: Tyndall effect is observed during projection of picture in
double
cinema hall due to scattering of light by dust and smoke
As 2 O3 + 3H 2S ¾¾¾®
decom.
As2S3 (sol) + 3H 2O particles.
EBD_7327
468 CHEMISTRY

(iii) Brownian movement: It is defined as continuous zig-zag (c) By boiling : Sols such as sulphur and silver halides
movement of colloidal particles in a colloidal sol. disperse in water, get coagulated when boiled due to
(a) It does not depend on nature of colloid. increased collisions between sol particles and water
(b) It depends on size of particles and viscosity of solution. molecules which removes the adsorbed charged layer
Smaller the size of particles and lesser the viscosity, faster from the sol and therefore the sol particles settle down.
the movement. (d) By addition of electrolytes: When excess of an electrolyte
(c) It arises due to unbalanced bombardment of particles by is added, colloidal particles get precipitated because
particles of dispersion medium. particles take up ions which are oppositely charged and
(d) It has a stirring effect which does not allow the particles thus get neutralised.
to settle and is thus responsible for stability of sols.
(iv) Colour: The colour of colloidal solution depends upon the Coagulation Ion and Coagulating Value
wavelength of light scattered by colloidal particles which in The ion responsible for neutralization of charge on the particles is
turn depends upon size and nature of particles. called coagulating ion.
(v) Charge on colloidal particles: The stability of colloidal sol The minimum amount of an electrolyte (in millimoles per litre)
is due to the fact that the particles are electrically charged, required to cause precipitation of a sol in two hours is called
they repel one another and do not coagulate. All particles of coagulating value or flocculation value. Smaller is the coagulation
dispersed phase carry same charge (which can be either +ve value of an electrolyte, greater is its coagulating power.
or –ve) while dispersed medium carries equal and opposite Hardy - Schulze Rule
charge.
Note : It states that the precipitating effect of an ion on dispersed phase
(a) The best explanation for the origin of electric charge is particles of opposite charge increases with the valence of the ion.
the preferential adsorption of ions from solution. The (i) For precipitation of negatively charged sol the flocculating
sol particles acquire positive or negative charge by power of Al3+, Ba2+ and Na+ is in the order: Al3+ > Ba2+ > Na+.
preferential adsorption of positive or negative ions. (ii) For precippitation of positive sol, flocculating power is in the
When two or more ions are present in dispersion medium, 4-
preferential adsorption of the ion common to colloidal order: éë Fe ( CN 6 ) ùû > PO34- > SO 42- > Cl-
particles usually takes place. Coagulation of Lyophobic and Lyophilic Colloids:
(b) Electrical double layer :
When one type of ions are adsorbed by colloidal particles, Lyophilic sols are more stable than lyophobic colloids. This is
a fixed layer is formed. It attracts counter ions from the because stability of lyophilic colloids is due to two factors:
medium forming a second layer which is mobile and is (i) Same charge on all colloidal particles
called diffused layer. The double layer of opposite (ii) Solvation of colloids
charges thus formed is called Helmholtz electrical However, stability of lyophobic sol is only due to charge
double layer. As a result, a potential exists between fixed which can be removed by adding electrolyte only. Hence,
and diffused layers and is called electrokinetic or zeta they can be easily coagulated.
potential. On the other hand, to coagulate lyophilic colloids, both
(vi) Electrophoresis: It is the movement of colloidal particles factors (i) and (ii) have to be removed. This is done by adding
under the influence of an electric field. Positively charged (a) electrolyte
particles move towards cathode (–ve) while negatively (b) suitable solvent (like alcohol or acetone which causes
charged particles move towards anode (+ve). On reaching dehydration of colloidal particles)
the oppositely charged electrode, they get neutralised and Protection of Colloids
coagulated.
Lyophobic sols are readily precipitated by small smounts of
It confirms the existence of charge on colloidal particles.
(vii) Electro-osmosis: It is the phenomenon in which the molecules electrolytes. However these sols are often stabilized by addition
of dispersion medium are allowed to move under the influence of lyophilic sols.
of an electric field whereas colloidal particles are not allowed Lyophilic colloids have a unique property of protecting lyophobic
to move. colloids. When a lyophilic sol is added to a lyophobic sol, lyophilic
(viii) Coagulation or precipitation: It is a process of aggregating / particles form a layer around lyophobic particles and thus protect
settling down of colloidal particles is called coagulation or the latter from electrolytes lyophilic colloids used for this purpose
precipitation. are called protective colloids.
It is done in the following ways: Gold Number
(a) By electrophoresis : During electrophoresis the charged The number of milligrams of a hydrophilic colloid that will just
sol particles migrate towards the electrode of opposite prevent the precipitation of 10 ml of a gold sol on the addition of
sign. There they deposit their charge and then get
1 ml of 10% sodium chloride solution is known as gold number.
coagulated
The smaller the value of gold number of lyophilic sol, the greater
(b) By mixing two oppositely charged sols : The neutral
is the protective action.
coagulation of two sols of opposite charge can be
effected by mixing them., For eg Fe(OH)3 (positive sol) 1
and Arsenious sulphide (negative sol) when mixed join Protection capacity µ
Gold number
and coagulate.
Surface Chemistry 469

Colloids Around us Examples are: proteins, gums, natural and synthetic soaps (for O/
Examples of colloids are: W emulsions) and heavy metal salts of fatty acid, long chain
(i) Blue colour of sky alcohols, lampblack, etc. (for W/O type)
(ii) Fog, mist and rain Properties of Emulsions :
(iii) Food articles (i) They show Tyndal effect and Brownian movement.
(iv) Blood
(ii) They can be demulsified (broken) by heating, adding
(v) Soils
(vi) Formation of delta electrolytes, freezing and centrifuging
(iii) They can be diluted with dispersion medium.
Application of Colloids
(i) Electrical precipitation of smoke GELS
(ii) Purification of drinking water A gel is a jelly like colloidal system in which a liquid is dispersed in
(iii) Medicines solid medium. Gels are of two types:
(iv) Tanning (i) Elastic gels - They can be temporarily deformed by applying
(v) Cleansing action of soaps and detergents
force eg Gelatin, starch and soaps.
(vi) Photographic plates and films
(vii) Rubber industry (ii) Non elastic gels - They are rigid eg silica gel.
(viii) Industrial products like paints, inks, rubber, cement etc. Properties of Gels
EMULSIONS (i) Syneresis - Shrinkage of gels on standing by exudation of
It is a colloidal dispersion in which both dispersed phase and solvent is known as syneresis
dispersion medium are liquids (two liquids involved are otherwise (ii) Thixotropy - Certain gels when shaken form a sol and on
immiscible). standing are converted into the form of gel They are known
They are of two types: as thixotropic gels and sol - gel transofrmation is known as
(i) Oil dispersed in water (O/W type) Ex: milk, vanishing cream thixotropy.
(ii) Water dispersed in oil (W/O type) Ex: butter, cream. (iii) Swelling or imbibition of gels - The property of adsorbing
Emulsions are usually not stable. To stabilise an emulsion, other definite amount of water and causing the volume of gel to
substances are added called emulsifying agents or emulsifiers.
increase is known as swelling or imbibition.
470

CONCEPT MAP
CHEMISTRY

EBD_7327
Surface Chemistry 471

1. What is the name given to a phenomenon in which both 9. Which of the following statements is not correct ?
adsorption and absorption take place? (a) Physical adsorption is due to van der Waal’s forces
(a) Chemisorption (b) Physiosorption (b) Chemical adsorption first decreases with increase in
(c) Desorption (d) Sorption temperature.
2. The dispersed phase, dispersion medium and nature of (c) Physical adsorption is reversible
colloidal solution (lyophilic or lyophobic) of 'gold sol' (d) Adsorption energy for a chemical adsorption is
respectively, are generally greater than that of physical adsorption
(a) solid, solid, lyophobic 10. The adsorption of a gas on a solid surface varies with
(b) liquid, liquid, lyophobic pressure of the gas in which of the following manner
(c) solid, liquid, lyophobic (a) Fast ® slow ® independent of the pressure
(d) soild, liquid, lyophilic (b) Slow ® fast ® independent of the pressure
3. In Langmuir's model of adsorption of a gas on a solid surface (c) Independent of the pressure ® fast ® slow
(a) the mass of gas striking a given area of surface is (d) Independent of the pressure ® slow ® fast
proportional to the pressure of the gas 11. Adsorption is always
(b) the mass of gas striking a given area of surface is (a) endothermic
independent of the pressure of the gas (b) exothermic
(c) the rate of dissociation of adsorbed molecules from the (c) exothermic in case of physical and endothermic in case
surface does not depend on the surface covered of chemical
(d) the adsorption at a single site on the surface may involve (d) Either (a) or (b)
multiple molecules at the same time 12. Adsorption of gases on solid surface is exothermic reaction
4. Pick out the statement which is not relevant in the discussion because
of colloids: (a) free energy increases
(a) Sodium aluminium silicate is used in the softening of (b) enthalpy is positive
hard water (c) entropy increases
(b) Potash alum is used in shaving rounds and as a styptic (d) enthalpy is negative
13. The role of a catalyst in a reversible reaction is to
in medicine
(a) increase the rate of forward reaction
(c) Artificial rain is caused by throwing electrified sand on
(b) decrease the rate of backward reaction
the clouds from an aeroplane
(c) alter the equilibrium constant of the reaction
(d) Deltas are formed at a place where the river pours its
(d) allow the equilibrium to be achieved quickly
water into the sea
14. Which of the following kind of catalysis can be explained by
5. How many layers are adsorbed in chemical adsorption ?
the adsorption theory ?
(a) One (b) Two
(a) Homogeneous catalysis
(c) Many (d) Zero
(b) Acid - base catalysis
6. Adsorption due to strong chemical forces is called (c) Heterogeneous catalysis
(a) Chemisorption (b) Physisorption (d) Enzyme catalysis
(c) Reversible adsorption (d) Both (b) and (c) 15. "The greater the charge on an ion, the greater its coagulating
7. In physical adsorption, gas molecules are bound on the solid power" is a statement of :
surface by (a) Tyndall's effect (b) Faraday's law
(a) chemical forces (b) electrostatic forces (c) Mosley's law (d) Hardy-Schulze law
(c) gravitational forces (d) van der Waal’s forces 16. Catalytic poisons act by :
8. Which among the following statements are correct with (a) making the products chemically inactive.
respect to adsorption of gases on a solid? (b) increasing the rate of the backward reaction.
(i) The extent of adsorption is equal to kpn according to (c) chemical combination with any one of the reactants.
Freundlich isotherm (d) preferential adsorption on the catalyst surface.
(ii) The extent of adsorption is equal to to kp1/n according 17. Which one is a colloid?
to Freundlich isotherm (a) Sodium chloride (b) Urea
(iii) The extent of adsorption is equal to (1+ bp)/ap (c) Cane sugar (d) Blood
according to Langmuir isotherm 18. The extra stability of lyophilic colloids is due to
(iv) At low pressure x /m is directly proportional to the (a) Charge on their particles
pressure (b) A layer of medium of dispersion on their particles
(a) (i) and (iii) (b) (i) and (iv) (c) The smaller size of their particles
(c) (ii) and (iii) (d) (ii) and (iv) (d) The large size of their particles
EBD_7327
472 CHEMISTRY

19. The action of enzymes in living system is to : 31. Which of the following forms cationic micelles above certain
(a) supply energy to tissues concentration?
(b) enhance immunity (a) Sodium dodecyl sulphate
(c) circulate oxygen (b) Sodium acetate
(d) enhance the rate of biochemical reactions. (c) Urea
20. Active charcoal is a good catalyst because it (d) Cetyl trimethyl ammonium bromide
(a) is made up of carbon atoms. 32. Which one of the following statements is incorrect in the
(b) is very reactive. case of heterogeneous catalysis ?
(c) has more adsorption power. (a) The catalyst lowers the energy of activation
(d) has inert nature toward reagents. (b) The catalyst actually forms a compound with the
21. Which one of the following substances gives a positive reactant
charged sol? (c) The surface of the catalyst plays a very important role
(a) Gold (b) A metal sulphide (d) There is no change in the energy of activation.
(c) Ferric hydroxide (d) An acidic dye 33. The cause of Brownian movement is
22. During dialysis (a) heat changes in liquid state
(a) only solvent molecules can diffuse (b) convectional currents
(b) solvent molecules, ions and colloidal particles can (c) the impact of molecules of the dispersion medium on
diffuse the colloidal particles.
(c) all kinds of particles can diffuse through the (d) attractive forces between the colloidal particles and
semi-permeable membrane molecules of dispersion medium.
(d) solvent molecules and ions can diffuse 34. Small liquid droplets dispersed in another liquid is called
23. At the critical micelle concentration (CMC) the surfactant (a) gel (b) suspension
molecules (c) emulsion (d) true solution
(a) decompose 35. Milk is a colloid in which a
(b) dissociate (a) liquid is dispersed in a liquid
(c) associate (b) solid is dispersed in a liquid
(d) become completely soluble (c) gas is dispersed in a liquid
24. The colloidal solution of gelatin is known as (d) sugar is dispersed in a liquid
(a) Solvent loving sol (b) Reversible sol 36. Butter is a colloid formed when
(c) Hydrophilic sol (d) All the above (a) Fat is dispersed in water
25. Hardy-Schulze rule explains the effect of electrolytes on the (b) Fat globules are dispersed in water
coagulation of colloidal solution. According to this rule, (c) Water is dispersed in fat
coagulation power of cations follow the order (d) None of the above
(a) Ba+2 > Na+ > Al+3 (b) Al+3 > Na+ > Ba+2 37. Commonly used catalyst in the preparation of acrylonitrile
+3
(c) Al > Ba > Na+2 + (d) Ba+2 > Al+3 > Na+ from propylene, ammonia and oxygen is
26. Which of the following is used for neutralising charge on (a) silver (b) bismuth molybdates
colloidal solution? (c) caesium chloride (d) nickel
(a) Electrons 38. Hair cream is an example of
(b) Electrolytes (a) gel (b) sol
(c) Positively charged ions (c) aerosol (d) foam
(d) Compounds 39. Surface tension of lyophilic sols is
27. Which of the following methods is used for the destruction (a) lower than that of H2O
of sol ? (b) more than that of H2O
(a) Condensation (c) equal to that of H2O
(b) Dialysis (d) either less or more than H2O depending upon the nature
(c) Diffusion through animal membrane of disperse phase
(d) Addition of an electrolyte. 40. Which of the following ions can cause coagulation of
28. Colloidal solutions are not purified by proteins ?
(a) Dialysis (b) Electrodialysis (a) Ag + (b) Na+
(c) Ultrafiltration (d) Electrophoresis (c) Mg2+ (d) Ca2+
29. Pure water can be obtained from sea water by 41. Which of the following is a lyophilic colloid ?
(a) Centrifugation (b) Plasmolysis (a) Milk (b) Gum
(c) Reverse osmosis (d) Sedimentation (c) Fog (d) Blood
30. How non-polar and polar part in micelle are arranged ?
42. When a strong beam of light is passed through a colloidal
(a) Polar at outer surface and non-polar at inner surface
solution, the light will
(b) Polar at inner surface and non-polar at outer surface
(a) be reflected (b) be scattered
(c) Both polar and non-polar at inner surface
(c) be refracted (d) give a rainbow
(d) Distributed all over the surface
Surface Chemistry 473

43. Adsorption is accompanied by 58. Which of the following will be most effective in the
(a) decrease in enthalpy and increase in entropy coagulation of Al(OH)3 sol ?
(b) increase in enthalpy and increase in entropy (a) KCN (b) BaCl2
(c) decrease in enthalpy and decrease in entropy (c) NaCl (d) Mg3(PO4)2
(d) increase in enthalpy and decrease in entropy 59. Tyndall effect shown by colloids is due to :
44. In coagulating the colloidal solution of As2S3 which has the (a) scattering of light by the particles
minimum coagulating value ? (b) movement of particles
(a) NaCl (b) KCl (c) reflection of light by the particles
(c) BaCl2 (d) AlCl3 (d) coagulation of particles
45. The simplest way to check whether a system is colloidal or 60. Alloy is an example of
not is by (a) gel (b) solidified emulsion
(a) Tyndall effect (b) Brownian movement (c) solid solution (d) sol
(c) Electrodialysis (d) Measuring particle size 61. The separation of colloidal particles from particles of
46. Which of the following is most effective in causing the molecular dimensions is known as
coagulation of ferric hydroxide sol? (a) sedimentation (b) dispersion
(a) KCl (b) KNO3 (c) pyrolysis (d) dialysis
(c) K2SO4 (d) K3[Fe(CN)6] 62. According to Freundlich adsorption isotherm, the amount
47. The ability of an ion to bring about coagulation of a given of gas adsorbed at very high pressure
colloid depends upon (a) reaches a constant limiting value
(a) its size (b) goes on increasing with pressure
(b) the magnitude of its charge (c) goes on decreasing with pressure
(c) the sign of its charge (d) increase first and decreases later with pressure
(d) both magnitude and sign of its charge 63. The Langmuir adsorption isotherm is deduced by using the
48. The formation of micelles takes place only above assumption
(a) inversion temperature (b) Boyle temperature (a) the adsorption sites are equivalent in their ability to
adsorb the particles
(c) critical temperature (d) Kraft temperature
(b) the heat of adsorption varies with coverage
49. Gold number is the index for :
(c) the adsorbed molecules interact with each other
(a) electroplated gold (b) metallic gold
(d) the adsorption takes place in multilayers.
(c) protective colloid (d) purity of gold
64. If x is amount of adsorbate and m is amount of adsorbent,
50. Which of the following electrolytes is least effective in which of the following relations is not related to adsorption
coagulating ferric hydroxide solution? process ?
(a) KBr (b) K2SO4 (a) x / m = f (p) at constant T.
(c) K2CrO4 (d) K4 [Fe(CN)6] (b) x / m = f (T) at constant p.
51. An aerosol is a : (c) p = f (T) at constant (x / m).
(a) dispersion of a solid or liquid in a gas x
(b) dispersion of a solid in a liquid (d) = p ´T
m
(c) dispersion of a liquid in a liquid 65. In Freundlich Adsorption isotherm, the value of 1/n is :
(d) solid solution (a) between 0 and 1 in all cases
52. Colloidion is a 4% solution of which one of the following (b) between 2 and 4 in all cases
in an alcohol-ether mixture? (c) 1 in case of physical adsorption
(a) nitroglycerine (b) celluloseacetate (d) 1 in case of chemisorption
(c) glycoldinitrate (d) nitrocellulose 66. Which one of the following, statements is incorrect about
53. A precipitate is changed to colloidal solution by the following enzyme catalysis?
process : (a) Enzymes are mostly proteinous in nature.
(a) dialysis (b) ultrafiltration (b) Enzyme action is specific.
(c) peptization (d) electrophoresis (c) Enzymes are denaturated by ultraviolet rays and at high
54. The movement of colloidal particles towards their respective temperature.
electrodes in the presence of an electric field is known as : (d) Enzymes are least reactive at optimum temperature.
(a) electrolysis (b) Brownian movement 67. The protecting power of lyophilic colloidal sol is expressed
(c) electrodialysis (d) electrophoresis in terms of :
55. Which of the following acts as protective colloid? (a) coagulation value
(a) Silica gel (b) Gelatin (b) gold number
(c) Sodium acetate (d) None of these (c) critical miscelle concentration
56. Which of the following is the best protective colloid? (d) oxidation number
(a) Gelatin (Gold No. = 0.005) 68. Which of the following statements is incorrect regarding
(b) Gum arabic (Gold No. = 0.15) physisorptions?
(c) Egg albumin (Gold No. = 0.08) (a) More easily liquefiable gases are adsorbed readily.
(d) All possess same protective power (b) Under high pressure it results into multimolecular layer
57. Which one of the following is correctly matched? on adsorbent surface.
(a) Emulsion-smoke (b) Gel-butter (c) Enthalpy of adsorption ( DH adsorption) is low and
positive.
(c) Aerosol-hair cream (d) Sol-whipped cream (d) It occurs because of van der Waal’s forces.
EBD_7327
474 CHEMISTRY

69. Bredig arc method cannot be used to prepare colloidal 73. Gold numbers of protective colloids A, B, C and D are 0.50,
solution of which of the following 0.01, 0.10 and 0.005, respectively. The correct order of their
(a) Pt (b) Fe protective powers is
(c) Ag (d) Au (a) D < A < C < B (b) C < B < D < A
70. Which one is an example of multimolecular colloid system (c) A < C < B < D (d) B < D < A < C
(a) Soap dispersed in water 74. Among the electrolytes Na2SO4, CaCl2, Al2(SO4)3 and
(b) Protein dispersed in water NH4Cl, the most effective coagulating agent for Sb2S3 sol is
(c) Gold dispersed in water (a) Na2SO4 (b) CaCl2
(d) Gum dispersed in water (c) Al2(SO4)3 (d) NH4Cl
71. Metals like Pt and Pd can adsorb large volume of hydrogen 75. Among the following, the surfactant that will form micelles
under specific conditions. Such adsorbed hydrogen by the in aqueous solution at the lowest molar concentration at
metal is known as ambient condition is : -
(a) Occluded hydrogen (b) Absorbed hydrogen (a) CH3(CH2)15N+(CH3)3Br–
(c) Reactive hydrogen (d) Atomic hydrogen (b) CH3(CH2)11OSO–3 Na+
72. The electrical charge on a colloidal particle is observed by: (c) CH3(CH2)6COO–Na+
(a) Ultramicroscope (b) Scattering (d) CH3(CH2)11N+(CH3)3Br–
(c) Brownian movement (d) Electrophoresis

1. Which one of the following impurities present in colloidal (c) aluminium which coagulates the mud particles
solution cannot be removed by electrodialysis? (d) making mud water soluble
(a) Sodium chloride (b) Potassium sulphate 6. In petrochemical industry alcohols are directly converted to
(c) Urea (d) Calcium chloride gasoline by passing over heated
2. The dispersed phase and dispersion medium in soap lather (a) Platinum (b) ZSM-5
are respectively
(c) Iron (d) Nickel
(a) gas and liquid (b) liquid and gas
7. The function of gum arabic in the preparation of an Indian ink is
(c) solid and gas (d) solid and liquid
(a) coagulation (b) peptization
3. Which of the following is most powerful to coagulate the
negative colloid? (c) absorption (d) protective action
(a) ZnSO4 (b) Na3PO4 8. Which is adsorbed in maximum amount by activated charcoal ?
(c) AlCl3 (d) K4[Fe(CN)6] (a) N2 (b) CO2
4. The disperse phase in colloidal iron (III) hydroxide and (c) Cl2 (d) O2
colloidal gold is positively and negatively charged, 9. A colloidal solution is subjected to an electric field. The
respectively. Which of the following statements is NOT particles move towards anode. The coagulation of same sol
correct? is studied using NaCl, BaCl2 and AlCl3 solutions. The order
(a) Coagulation in both sols can be brought about by of their coagulation power should be –
electrophoresis (a) NaCl > BaCl2 > AlCl3 (b) BaCl2 > AlCl3 > NaCl
(b) Mixing the sols has no effect (c) AlCl3 > BaCl2 > NaCl (d) BaCl2 > NaCl > AlCl3
(c) Sodium sulphate solution causes coagulation in both 10. Associated colloid among the following is
sols
(a) enzymes (b) proteins
(d) Magnesium chloride solution coagulates, the gold sol
(c) cellulose (d) sodium stearate
more readily than the iron (III) hydroxide sol
11. Coagulation value of the electrolytes AlCl3 and NaCl for
5. Alum helps in purifying water by
As2S3 sol are 0.093 and 52 respectively. How many times
(a) forming Si complex with clay particles AlCl3 has greater coagulating power than NaCl ?
(b) sulphate part which combines with the dirt and removes (a) 930 (b) 520
it
(c) 560 (d) None of these
Surface Chemistry 475

19. When KClO3 is heated, it decomposes into KCl + O2. If


12. The density of gold is 19 g/cm3. If 1.9 ´ 10 -4 g of gold is
some MnO2 is added, the reaction goes much faster because
dispersed in one litre of water to give a sol having spherical
gold particles of radius 10 nm, then the number of gold (a) MnO2 decomposes to give O2
particles per mm3 of the sol will be : (b) MnO2 provides heat by reacting
(c) better contact is provided by MnO2
(a) 1.9 ´ 1012 (b) 6.3 ´ 1014 (d) MnO2 acts as a catalyst
(c) 6.3 ´1010 2.4 ´ 10 6
(d) 20. According to Langmuir adsorption isotherm, the amount of
gas adsorbed at very high pressures
13. The disease kala azar is caused by
(a) reaches a constant limiting value
(a) colloidal antimony (b) milk of magnesia
(b) goes on increasing with pressure
(c) argyrols (d) colloidal gold
(c) goes on decreasing with pressure
14. Which is correct about physical adsorption?
(d) increases first and then decreases with pressure
(a) High temperature and high pressure favour adsorption
21. The charge on colloidal particles is due to
(b) High temperature and low pressure favour adsorption
(a) presence of electrolyte
(c) Low temperature and high pressure favour adsorption
(b) very small size of particles
(d) Low temperature and low pressure favour adsorption (c) adsorption of ions from the solution
15. The rate of oxidation of oxalic acid by acidified KMnO4 (d) None of these
increases as the reaction progresses. It is an example of
22. The efficiency of an enzyme in catalysing a reaction is due
(a) promoter (b) catalytic poison to its capacity
(c) autocatalysis (d) inhibitor (a) to form a strong enzyme-substrate complex
16. At the high pressure, Langmuir adsorption isotherm takes (b) to decrease the bond energies of substrate molecule
the form (c) to change the shape of the substrate molecule
x ap x a (d) to lower the activation energy of the reaction
(a) = (b) = 23. Purple of cassius is colloidal solution of :
m 1 + bp m b
(a) Silver (b) Lead
x m b 1
(c) = ap (d) = + (c) Gold (d) Mercury
m x a ap
DIRECTIONS for Qs. 24 and 25 : These are Assertion-Reason
17. Freundlich equation for adsorption of gases (in amount of x g) type questions. Each of these question contains two statements:
on a solid (in amount of m g) at constant temperature can be Statement-1 (Assertion) and Statement-2 (Reason). Answer these
expressed as
questions from the following four options.
x 1 (a) Statement-1 is true, Statement-2 is true, Statement-2 is a
(a) log = log p + log K
m n correct explanation for Statement-1
x 1 (b) Statement-1 is True, Statement-2 is True ; Statement-2 is
(b) log = log K + log p NOT a correct explanation of Statement-1
m n
(c) Statement-1 is True, Statement-2 is False
x
(c) µ pn (d) Statement-1 is False, Statement-2 is True
m
24. Statement-1 : The enthalpy of physisorption is greater than
x 1 chemisorption.
(d) = log p + log K
m n
Statement-2 : Molecules of adsorbate and adsorbent are
18. Which one of the following is an example for multimolecular held by van der Waal’s forces in physisorption and by
colloid? chemical bonds in chemisorption.
(a) Aqueous starch sol x 1/ n
(b) Aqueous enzyme sol 25. Statement-1 : According to Freundlich: = K . p .
m
(c) Alcoholic polystyrene sol Statement-2 : The isotherm shows variation of the amount
(d) Aqueous sol of sodium laurylsulphate of gas adsorbed by the adsorbent with temperature.
EBD_7327
476 CHEMISTRY

Exemplar Questions 11. Which of the following is an example of absorption?


1. Which of the following process does not occur at the (a) Water on silica gel
interface of phases? (b) Water on calcium chloride
(a) Crystallisation (c) Hydrogen on finely divided nickel
(b) Heterogeneous catalysis (d) Oxygen on metal surface
12. On the basis of data given below predict which of the
(c) Homogeneous catalysis
following gases shows least adsorption on a definite amount
(d) Corrosion
of charcoal?
2. At th e equilibrium position in the process of
adsorption ................. . Gas CO 2 SO 2 CH 4 H2
(a) DH > 0 (b) DH = TDS Critical temp./K 304 630 190 33
(c) DH > TDS (d) DH < TDS
(a) CO2 (b) SO2
3. Which of the following interface cannot be obtained?
(c) CH4 (d) H2
(a) Liquid-liquid (b) Solid-liquid
13. In which of the following reactions heterogeneous catalysis
(c) Liquid-gas (d) Gas-gas
is involved?
4. The term 'sorption' stands for ................ .
NO( g )
(a) absorption (i) 2SO2 ( g ) + O 2 ( g ) ¾¾¾¾ ® 2SO3 ( g )
(b) adsorption Pt ( s )
(c) Both absorption and adsorption (ii) 2SO2 ( g ) ¾¾¾® 2SO3 ( g )
(d) desorption Fe( s )
(iii) N 2 ( g ) + 3 H 2 ( g ) ¾¾¾® 2NH3 ( g )
5. Extent of physisorption of a gas increases with ............. .
HCI( l)
(a) increase in temperature (iv) CH3COOCH 3 ( l ) + H 2O ( l ) ¾¾¾¾ ® CH3COOH ( aq )
(b) decrease in temperature
+ CH3 OH ( aq )
(c) decrease in surface area of adsorbent
(d) decrease in strength of van der Waal's forces (a) (ii), (iii) (b) (ii), (iii) and (iv)
6. Extent of adsorption of adsorbate from solution phase (c) (i), (ii) and (iii) (d) (iv)
increases with ............... . 14. At high concentration of soap in water, soap behaves as ......
(a) increase in amount of adsorbate in solution (a) molecular colloid (b) associated colloid
(b) decrease in surface area of adsorbate (c) macromolecular colloid (d) lyophilic colloid
(c) increase in temperature of solution 15. Which of the following will show Tyndall effect?
(a) Aqueous solution of soap below critical micelle
(d) decrease in amount of adsorbate in solution
concentration
7. Which one of the following is not applicable to the
(b) Aqueous solution of soap above critical micelle
phenomenon of adsorption?
concentration
(a) DH > 0 (b) DG < 0
(c) Aqueous solution of sodium chloride
(c) DS < 0 (d) DH < 0 (d) Aqueos solution of sugar
8. Which of the following is not a favourable condition for 16. Method by which lyophobic sol can be protected
physical adsorption? (a) by addition of oppositely charged sol
(a) High pressure (b) by addition of an electrolyte
(b) Negative DH (c) by addition of lyophilic sol
(c) Higher critical temperature of adsorbate (d) by boiling
(d) High temperature 17. Freshly prepared precipitate sometimes gets converted to
9. Physical adsorption of a gaseous species may change to collodial solution by ................... .
chemical adsorption with ............. . (a) coagulation (b) electrolysis
(a) decrease in temperature (c) diffusion (d) peptisation
(b) increase in temperature 18. Which of the following electrolytes will have maximum
(c) increase in surface area of adsorbent coagulating value for Ag/Ag+ sol?
(d) decrease in surface area of adsorbent (a) Na2S (b) Na3PO4
10. In physisorption adsorbent does not show specificity for (c) Na2SO4 (d) NaCl
any particular gas because ............. . 19. A colloidal system having a solid substance as a dispersed
(a) involved van der Waal's forces are universal phase and a liquid as a dispersion medium is classified
(b) gases involved behave like ideal gases as .............. .
(c) enthalpy of adsorption is low (a) solid sol (b) gel
(d) it is a reversible process (c) emulsion (d) sol
Surface Chemistry 477

20. The values of colligative properties of colloidal solution are (c) Formation of Helmholtz electrical double layer
of small order in comparison to those shown by true solutions (d) Absorption of ionic species from solution
of same concentration because of colloidal particles ............ 25. Which of the following phenomenon is applicable to the
(a) exhibit enormous surface area process shown in the figure?
(b) remain suspended in the dispersion medium
(c) form lyophilic colloids
(d) are comparatively less in number
21. Arrange the following diagrams in correct sequence of steps
involved in the mechanism of catalysis, in accordance with Solution of raw sugar
modern adsorption theory. yellowish brown
A
| | |
— O —O —O —
| | |
(I) —O—O—O— Column of
| | | animal
B charcoal
| | |
A — O —O —O —
| | |
(II) B — O — O — O —
Colourless
| | | solution of
| | | sugar
A — O —O —O —
| | |
(III) B — O — O — O — (a) Absorption (b) Adsorption
| | | (c) Coagulation (d) Emulsification
A NEET/AIPMT (2013-2017) Questions
| | | |
B — O — O —O — 26. Which of the following statements is correct for the
(IV) | | | spontaneous adsorption of a gas ? [2014]
— O—O—O—
(a) DS is negative and, therefore, DH should be highly
| | |
| | | positive
A — O — O —O — (b) DS is negative and therefore, DH should be highly
| + | | | negative
(V) B — O — O — O —
| | | (c) DS is positive and, therefore, DH should be negative
(a) I ® II ® III ® IV ® V (b) I ® III ® II ® IV ® V (d) DS is positive and, therefore, DH should also be highly
(c) I ® III ® II ® V ® IV (d) I ® II ® III ® V ® IV positive
22. Which of the following process is responsible for the 27. Which property of colloidal solution is independent of charge
formation of delta at a place where rivers meet the sea? on the colloidal particles:- [2014, 2015]
(a) Emulsification (b) Colloid formation (a) Electrophoresis (b) Electro-osmosis
(c) Coagulation (d) Peptisation (c) Tyndall effect (d) Coagulation
23. Which of the following curves is in according with Freundlich
28. Which one of the following characteristics is associated with
adsorption isotherm?
adsorption ? [2016]
(a) DG is negative but DH and DS are positive
log x/m ®

log x/m ®

(b) DG, DH and DS all are negative


(a) (b) (c) DG and DH are negative but DS is positive
(d) DG and DS are negative but DH is positive
log p ® log p ® 29. Fog is colloidal solution of [2016]
(a) Liquid in gas (b) Gas in liquid
log x/m ®
log x/m ®

(c) Solid in gas (d) Gas in gas


(c) (d) 30. Which one of the following statements is not correct? [2017]
(a) The value of equilibrium constant is changed in the
log p ® log p ® presence of a catalyst in the reaction at equilibrium
24. Which of the following process is not responsible for the (b) Enzymes catalyse mainly bio-chemical reactions
presence of electric charge on the sol particles? (c) Coenzymes increase the catalytic activity of enzyme
(a) Electron capture by sol particles (d) Catalyst does not initiate any reaction
(b) Adsorption of ionic species from solution
EBD_7327
478 CHEMISTRY

Hints & Solutions


EXERCISE - 1 magnitude and allow the equilibrium to be achieved quickly.
14. (c) Adsorption theory is applied to heterogeneous catalysis
1. (d) Sorption : Absorption and adsorption occur together.
15. (d) The above statement is the law proposed by Hardy-
Absorption : Particles of gas or liquid get uniformly
distributed throughout the body of solid and the force Schulze. According to this law, the coagulating power
of attraction between them is weak vander waal’s force of an ion is directly proportional to the charge it
of attraction or by some bond formation. possesses.e.g. Na+ < Mg2+ < Al3+
Adsorption ® Physical adsorption (vander waal’ss 16. (d) The catalytic poisons decrease the activity of the
force of a attraction) catalyst because they are preferentially adsorbed on
Chemical adsorption (bond formation between adsobed the surface of catalyst.
and adsorbate molecules). 17. (d) Blood is a –vely charged colloidal system. Rest of the
2. (c) In case of gold sol, we have compounds, i.e., NaCl, urea & cane sugar form true
Dispersed phase – solid. solution in water.
Dispersion medium – liquid 18. (b) The stability of a lyophilic sol is due to both the charge
Gold sol is lyophobic colloid. i.e., (c) is correct. and solvation of particles, but largely to the latter factor.
19. (d) Enzymes are biological catalysts and enhance the rate
3. (a) According to Langmuir's model of adsorption of a gas
of biochemical reactions.
on a solid surface the mass of gas adsorbed( x)per gram
20. (c) Active charcoal has more adsorption power due to
of the adsorbent (m) is directly proportional to the
greater surface area.
pressure of the gas (p) at constant temperature.
4. (a) This statement is not relevant in discussion of colloids 21. (c) Fe(OH)3 particles absorb Fe3+ ions and get peptized to
as softening of hard water involves removal of salts give a positively charged sol.
that cause hardness of water i.e., salts of Ca2+ and 22. (d) The use of membrane for separating colloidal particles
Mg2+. from those of crystalloids is termed as dialysis. Hence
5. (a) Chemical adsorption involves formation of monolayer. it is clear that colloidal particles cannot pass through
6. (a) Chemisorption involves strong chemical forces. animal parchment membrane. Hence only solvent
7. (d) In physisorption, gas molecules are held on the solid molecules and ions (in case of electrodialysis) can
surface by weak van der Waal’s forces. diffuse through membrane.
8. (d) (i) This statement is incorrect. 23. (c) The critical micelle concentration is the lowest
x concentration at which micelle formation appears when
(ii) At intermediate pressure, = k.p1 n , so this surfactants are present above that CMC, they can act
m as emulsifiers that will solubilise a compound which is
statement is correct.
(iii) This statement is incorrect because the correct normally insoluble in the solvent being used.
expression for Langmuir adsorption isotherm is 24. (d) Gelatin is a lyophilic solution ( lyo-liquid philic -love).
x Lyophilic colloids are also known as reversible colloids
= k1 kp (1 + kp), which may also be expressed since on evaporating the dispersion medium (i.e. water),
m
x ap the residue can again be easily reconverted into
as = colloidal state simply by addition of the liquid i.e. water.
m (1 + bp)
x 25. (c) According to this law the coagulating effect of an ion
(iv) At low pressure = kp on dispersed phase of opposite charge increases with
m the increase in valency of the ion. The precipitating
These statements (ii) and (iv) are correct and so option
(d) is correct. power of Al3+, Ba++, Na+ ions is in order Al3+ > Ba2+ > Na+.
9. (b) 26. (b) Electrolytes are used for neutralising charge on colloidal
10. (a) Adsorption of a gas on solid is represented by following particles.
equilibria, 27. (d) Addition of electrolytes causes precipitation
Gas (Adsorbate) + solid (Adsorbent) ‡ˆˆ ˆˆ† (coagulation) of a sol.
Gas adsorbed on solid + Heat 28. (d) Colloidal solutions are not purified by electrophoresis.
Initially adsorption increases with increase in pressure Movement of colloidal particles under the influence of
at a particular temperature then got slow. After attaining electric field is called electrophoresis. So, it can make
equilibrium adsorption become independent of easier. Electrophoresis is the property of colloids not
pressure. the purification method.
11. (b) Adsorption is an exothermic process. 29. (c) The osmotic pressure of sea water is 25 atm at 15°C.
12. (d) Adsorption is accompanied by evolution of heat as When pressure greater than 26 atm is applied on sea
the residual forces acting along the surface of adsorbent water separated by a rigid emipermeable membrane,
decrease i.e., adsorption is accompanied by decrease pure water is obtained.
in enthalpy.
13. (d) For a reaction in equilibrium, the increase in rate of 30. (a) Polarhead
reaction in forward direction by catalyst increases the Non-polar tail
concentration of product(s) and thus the rate of (micelle)
backward reaction also increases with the same
Surface Chemistry 479

31. (d) Cetyl trimethyl ammonium bromide, 56. (a) Lower is the gold number, higher will be the protective
+ - power. Thus, gelatin has maximum protective power.
[C16 H33 (CH3 )3 N Br ] is a cationic micelle.
57. (b) Butter is an example of gel.
32. (c) The theory of heterogeneous catalysis is based upon
58. (d) Al(OH)3 is a positive sol so salt having anion with
the phenomenon of adsorption. The activity of catalyst
maximum negative charged (i.e. phosphate ion) will be
is due to the presence of free valencies on its sufrace
most effective in coagulation.
due to which surface of catalyst has force of attraction.
59. (a)
33. (c) It is due to impact of molecules of dispersion medium
60. (c) Alloy is an example of solid solution.
on the colloidal particles.
61. (d) The separation of colloidal particles of molecular
34. (c) When the dispersed phase and dispersion medium both
dimension is known as dialysis. It is a purification
are liquid, the colloidal system is called as an emulsion
method of colloid.
like milk, vasnishing cream etc.
62. (a) According to Freundlich adsorption isotherm
Option (c) is correct.
x
35. (a) Milk is a emulsion in which liquid is dispersed in liquid. = kp1/n (wheren n > 1)
36. (c) In butter (liquid - solid) water is dispersed in fat. m
37. (b) Bismuth cerium molybdate catalysts promoted with At very high pressure x/m = kp0
alkali metal and other optional ingredients provide high 63. (a) Langmuir adsorption isotherm is based on the
yields of acrylonitrile in the ammoxidation of propylene. assumption that every adsorption site is equivalent
38. (d) Emulsions are liquid-liquid colloidal systems, Generally and the ability of a particle to bind there is independent
one of the two liquids is water. of whether nearby sites are occupied or not.
39. (a) Surface tension of lyophilic sols is lower than water 64. (d)
(dispersion medium). 65. (a) According to Freundlich Adsorption isotherm
1
40. (a) Proteins are coagulated by some heavy metal ions like x
Ag+, Hg2+ and Pb2+ = KP n
m
41. (b) Gum is lyophilic colloid. 1
42. (b) It is due to Tyndall effect. at low pressure = 1
n
43. (c) Adsorption is an exothermic process i.e. DH of x
adsorption is always negative. When a gas is adsorbed, \ µ P1
m
the freedom of movement of its molecules becomes 1
restricted i.e. DS is negative. Hence adsorption at high pressure = 0
n
accompanied by decrease in enthalpy as well as x
decrease in entropy of the system. µ P°
m
44. (d) As2S3 is negative sol. For coagulating negative sol i.e., the value of n varies between 0 to 1
Al 3+ is most effective. It is because higher the 66. (d) Enzymes are most reactive at optimum temperature. The
magnitude of the charge, the lower is coagulating value. optimum temperature for enzyme activity lies between
45. (a) Tyndall effect is the simplest way to check colloidal 40°C to 60°C.
system since path of light beam becomes visible due to 67. (b) The lyophobic sols are less stable than lyophilic sols.
scattering of light. The lyophilic sols are thus used to protect the
46. (d) Fe(OH)3 is positive sol. K3 [Fe(CN)6] will provide lyophobic sols. This property of lyophilic sols is known
[Fe(CN)6]3– for coagulation having highest magnitude as protective action of lyophilic sols which is
of –ve charge among given options. represented by gold number.
47. (d) According to the Hardy schulze rule the coagulating 68. (c) Adsorption is an exothermic process, hence DH will
effect of an ion on dispersed phase of opposite charge always be negative.
increases with the valency of the ion. Therefore more 69. (b) Bredig’s arc method is suitable for the preparation of
the charge on oppositely charged ion higher is the colloidal solution of metals like gold, silver, platinum
coagulation value. etc. An arc is struck between the metal electrode under
48. (d) The formation of micelles takes place only above a the surface of water containing some stabilzing agent
particular temperature called kraft temperature (TK). such as a trace of KOH. However, Fe does not react
49. (c) Gold number is measure of protective power of lyophilic with alkalies that is why it is not obtained by Bredig’s-
colloid. arc method.
1 70. (c) Example of multimolecular colloid system is a gold
Protective power of colloid µ dispersed in water.
Gold number
50. (a) Smaller the charge on anion, lesser will be its 71. (a)
coagulating power. 72. (d) The electrical change on a colloidal particle is observed
by electrophoresis. Under electric field, charged
\ KBr have Br – with least charge of – 1 on Br thus
particles move in a particular direction.
KBr is least effective in coagulating Fe(OH)3.
73. (c) For a protective colloid lesser the value of gold number
51. (a) more will be the protective power. Thus the correct
52. (d) Colloidion is a flammable, syrupy solution of proxyline order of protective power of A, B, C and D is
(nitrocellulose) in ether and alcohol. Þ (A) < (C) < (B) < (D)
53. (c) 54. (d) 55. (b) Gold number 0.50 0.10 0.01 0.005
EBD_7327
480 CHEMISTRY

Hence (c) is the correct answer 1´ 10-5


74. (c) As Sb2S3 is a negative sol, so Al2(SO4)3 will be the = = 2.38 ´ 1012
most effective coagulant due to higher positive charge 4.19 ´ 10 -18
on Al (Al3+) – Hardy-Schulze rule. \ No. of gold sol particles in one mm3
75. (b) 2.38 ´ 1012
= = 2.38 ´ 106
6
EXERCISE - 2 10
13. (a) Colloidal antimony is used in curing kala-azar.
1. (c) Electrodialysis involves movement of ions towards 14. (c) Physical adsorption involves weak forces, physical in
oppositely charged electrodes. nature with small heat of adsorption. Thus low
Urea being a covalent compound does not dissociate temperature and high pressure favours physical
to give ions and hence it cannot be removed by adsorption.
electrodialysis.However all the other given compounds 15. (c) Since the rate of oxidation of oxalic acid by acidified
are ionic which can undergo dissociation to give KMnO4 increases with the progress of the reaction so the
oppositely charged ions and thus can be separated. reaction is catalysed by one of the products of reaction
2. (a) Soap lather is a colloid containing gas as a dispersed 2 MnO -4 + 5C 2 O 42 + 16H + ¾¾ ®
phase and liquid as a dispersion medium. 2Mn 2+ + 10 CO 2 + 8H 2 O
3. (c) According to Hardy-Schulze rule "The amount of The Mn2+ ion formed acts as catalyst.
electrolyte required to coagulate a fixed amount of a 16. (b) Langmuir adsorption isotherm, is
sol depends upon the sign of charge and valency of x ap
the flocculating ion." =
m 1 + bp
Thus, the coagulating power vary in the order.
Al3+ > Zn++ > Na+ At high pressure, 1 + bp = bp
x ap a
4. (b) When oppositely charged sols are mixed their charges or = =
are neutralised. Both sols may be partially or completely m bp b
precipitated. 17. (b) According to Freundlich equation.
5. (c) Alum coagulates mud particles and helps in purifying x x
µ p1/ n or = Kp1/ n
water. m m
6. (b) ZSM-5 is a shape selective catalyst. Zeolites are good x x 1
shape selective catalysts because of the honey comb or log = log Kp1/ n or log = log K + log p
m m n
like structure. 18. (a) Multimolecular colloids consist of aggregates of atoms
7. (d) Gum arabic has protective power. or small molecules. Sulphur sol is an example of
8. (b) The gases having higher values for critical temperature multimolecular colloids
are easily liquified and are adsorbed to the greater extent. MnO 2 ® 2KCl + 3O
CO2 has highest critical temperature of 304K. 19. (d) 2KClO 3 ¾¾ ¾¾ 2
9. (c) As colloidal particles move towards anode so these 20. (a) According to Langmuir Adsorption isotherm the
particles are negatively charged and coagulated by amount of gas adsorbed at very high pressure reaches
cations of electrolyte. a constant limiting volume.
According to Hardy Schulze rule, 21. (c)
Coagulation power µ charge of ion 22. (d) Efficiency of catalysing property of a catalyst is
\ Order of coagulation power is Al3+ > Ba2+ > Na+ inversely proportional to activation energy.
10. (d) Sodium stearate is a soap. Soaps and detergents are 23. (c) Purple of cassius is colloidal solution of gold.
surface agents which when dissolved in a medium, 24. (d) Statement - 1 is false but Statement - 2 is true. The
forms aggregated particles, called associated colloids. enthalpy of chemisorption is of the order of
Coagulation power of AlCl3 40 - 400 kJmol–1 while for physical adsorption it is of the
11. (c) order of 20 - 40 kJmol–1.
Coagulation power of NaCl 25. (c) Statement - 1 is true but Statement - 2 is false.
Coagulation value of NaCl Freundlich adsorption isotherm gives an empirical
=
Coagulation value of AlCl3 relationship between the quantity of gas adsorbed by
52 unit mass of solid adsorbent and pressure at a particular
= = 559.13 » 560 temperature.
0.093
12. (d) Volume of gold dispersed in 1 L water EXERCISE - 3
mass 1.9 ´ 10- 4 g
= = = 1 × 10–5 cm3 Exemplar Questions
density 19gm cm-3
Radius of gold sol particle 1. (c) In homogeneous catalysis reactant and catalyst have
= 10 nm = 10 × 10–7 cm = 10–6 cm same phase so homogeneous catalysis can not occur
4 at the inertface of phases
Volume of gol sol particle = p r3 2. (b) Equilibrium DG = 0
3
4 22 -6 3 DH – TDS = 0
= ´ ´ (10 ) = 4.19 × 10–18 cm3 DH = TDS
3 7
\ No. of gold sol particles in 1 × 10–5 cm3
Surface Chemistry 481

3. (d) Gas-gas interface can not be obtained as gases are 17. (d) Peptisation is a process in which by addition of a
completely miscible in nature. suitable peptising agent precipitate gets converted into
colloidal solution.
4. (c) When both absorption and adsorption occur it is called
sorption 18. (b) According to Hardy-Schulze law, greater the charge
on anion greater will be its coagulating power.
+ + + + + + + + + +

+ + + + + +
+ + + + Electrolytes Anionic part Charge on anion
+ + + +
+ + + + adsorption Na 2S S2– 2
+ + + +
+ + + + Na 3 PO4 PO3–
4 3
+ + + + 2–
absorption sorption Na 2 SO4 SO4 2

NaCl Cl 1
5. (b) In physisorption particles are attached to the surface
by weak van der Waal's force of attraction so on
increasing temperature they get desorbed. Here PO3– 3–
4 have highest charge. Hence, PO4 have
highest coagulating power.
6. (a) When amount of adsorbate increases the interaction
19. (d) Sol is a colloidal system in which solid substance is a
of adsorbate adsorbent increases which lead to
dispersed phase and a liquid is a dispersion medium
increase in adsorption.
e.g., aint cell fluids etc.
7. (a) Enthalpy change during adsorption is negative. As in
In paints solid colouring particles are dissolved in
adsorption, there is decrease in residual forces of the
liquid dispersion medium.
surface which result in decrease in surface energy in
form of heat. 20. (b) The value of colligative properties of colloidal solution
are of small order in comparison to those of true
8. (d) Adsorbate get adsorbed on the adsorbent surface by solutions of same concentration because colloidal
weak van der Waal's force of attraction in physisorption. particles are comparatively less in number.
Rise in temperature will break the interaction between
adsorbate and adsorbent so adsorbate particles get 21. (b) The Correct sequence of steps is I ® III ® II ® IV ® V
desorbed. Each step represents a meaningful process as follows
9. (b) With increase in temperature physisorption changes I ® adsorption of A and B on surface
to chemisorption, because energy of activation of III ® II interaction between A and B to form
adsorbate particles increases which lead to formation intermediate
of chemical bond.
III ® IV starting desorption of A–B
10. (a) The extent of van der Waal's interaction between
absorbate and adsorbent is constant for all gases. IV ® V complete desorption
A
11. (b) Absorption is a bulk phenomena in which particles
goes to the bulk instead of residing only to surface. A
® ®
12. (d) Critical temperature of gases is directly proportional to B
the extent of adsorption.
B
13. (a) Reaction in which catalyst and reactant/product is in (i) (ii)
different phase then it is known as heterogeneous A
catalysis. A |
® B ®
14. (b) At high concentration of soap in water, soap particles B
present in the solution get associated and it leads to
the formation of associated colloid. (iii) (iv)
15. (b) Tyndall effect is a characteristic of colloidal solution,
as aqueous solution of soap above cmc form a colloidal A
solution. |
B
16. (c) Lyophobic sol can be protected by addition of lyophilic
sol as lyophobic sols are readily precipitated on
(v)
addition of small amount of electrolytes or shaking, or
22. (c) River water is a colloidal solution of clay and sea. Water
heating hence they are made stable by adding lyophillic
contains various electrolytes. When river water comes
sol which stabilises the lyophobic sols.
in contact with sea water, then the electrolytes present
EBD_7327
482 CHEMISTRY

in sea water coagulate the suspended colloidal particles Here, aqueous solution of raw sugar is filtered by using
which ultimately settle down at the point of contact. animal charcoal. Yellowish brown colour of raw sugar
23. (c) According to the Freundlich adsorption iotherm is adsorbed and filterate is colourles which gives white
1 colour on cystallisation. Hence, this phenomenon is
x adsorption.
= kp n
m NEET/AIPMT (2013-2017) Questions
x æ 1ö 26. (b) For adsorption DS < 0 and for a spontaneous change
Taking log on both side log = ç ÷ log p + log k
m è nø DG = – ve
This equation is comparable with straight line equation, hence DH should be highly negative which is clear from
y = mx + c, where m represents slope of the line and c the equation
represents intercept on y axis. DG = DH – TDS
= – DH – T(– DS) = – DH + TDS
So if DH is highly negative DG will also be (– ve)
27. (c) Tyndall effect is an optical property, and it is independent
log x/m

Slope = 1/n of charge on colloidal particles.


28. (b) Adsorption is spontaneous process, therefore change
log k (intercept)
in the free energy (DG) for the process is negative.
log p According to Gibbs's Helmholtz eqn.
24. (d) Absorption of ionic species from solution is not DG = DH – TDS
responsible for the presence of electric charge on the DS is negative because adhering of gas molecules to
sol particles. Charge on the sol particles is due to the surface lowers the randomness.
(i) electrons capture by sol particles during electro \ DG can be –ve only when DH is –ve.
dispersion of metal. 29. (a) Fog is a colloidal system having dispersed phase as
(ii) preferential adsorption of ionic species from liquid and dispersion medium as gas.
solution.
30. (a) A catalyst speeds up both forward and backward
(iii) formation of Helmholtz electrical double layer.
reaction with the same rate.
25. (b) Above figure represent adsorption of yellowish brown
colour of raw sugar by animal charcoal. So, equilibrium constant is not affected by the presence
of a catalyst at any given temperature.
General Principles and Processes of Isolation of Elements 483

General Principles

20 and Processes of
Isolation of Elements
OCCURENCE OF METALS Concentration of ores
All naturally occuring chemical substances in the earth crust are Removal of unwanted materials from the ore is known as
called minerals. Minerals from which metal can be extracted concentration, dressing or beneficiation. It involves various steps
profitably are called ores. Thus, all ores are minerals but all minerals which depend upon the differences in the physical properties of
are not ores. the compound of the metal present and that of the gangue.
Elements vary in abundance. Among metals, Al is the most Some important procedures are following :
abundant. Fe is the second most abundant metal in the earth’s (i) Hydraulic Washing : It is a type of gravity separation.It is
crust. The principal ores of some metals are given in the table based on the differences in gravities of the ore and gangue
below : particles.
In this process, a stream of running water is used to wash the
powdered ore. The lighter gangue particles are washed away
and the heavier ore particles are left behind. Oxide ores of Fe,
Metals Ores Composition
Sn and native ores of Au, Ag etc. are concentrated by this
Aluminium (i) Bauxite Al2O3.2H2O method.
(ii) Kaolinite [Al2(OH)4 Si2O 5] (ii) Magnetic Separation : It is based on the differences in the
magnetic properties of the ore components. It is carried out if
Iron (i) Haemetite Fe2O3 either the ore or the gangue is magnetic in nature.The finely
(ii) Magnetite Fe3O4 powdered ore is dropped over electromagnetic rollers with
(iii) Siderite FeCO3 conveyor belt. As the belt moves the magnetic particles of
the ore are attracted by the magnetic roller and fall nearer to
(iv) Iron pyrites FeS the roller while non-magnetic impurities fall away from the
Copper (i) Copper pyrites CuFeS2 roller, giving rise to two heaps, one having magnetic particles
(ii) Malachite CuCO3.Cu(OH) 2 and other with non-magnetic particles. For example, chromite
[Fe(CrO2)2] being magnetic is separated from silicious
(iii) Cuprite Cu2O impurities by this method.
(iv) Copper glance Cu2S (iii) Froth floatation method : It is used for sulphide ores. In this
process, a suspension of powdered ore is made with water.
Zinc (i) Zinc blende ZnS
To it, collectors and froth stabilizers are added. The mineral
(ii) Calamine ZnCO 3
particles are wetted by oil while the gangue particles got wet
(iii) Zincite ZnO
with water. The mixture is then violently agitated by passing
a current of compressed air. As a result, froth is formed which
carries the mineral particles. The froth is light and skimmed
METALLURGICAL PROCESSES off. It is then dried for recovery of the ore particles.
The ore is contaminated with earthly or undesired materials known The role of collectors is to enhance the non-wettability of
as gangue. The extraction and isolation of metals from ores involve mineral particles. Examples : pine oils, fatty acids, xanthates,
the following steps: etc. The role of froth stabilisers is to stabilise the froth.
(a) Concentration of ore Examples : cresols, aniline
(b) Isolation of metal from concentrated ore It is possible to separate two sulphide ores by adjusting the
proportion of oil to water or by using ‘depressants’. For
(c) Purification of metal
example in an ore containing ZnS and PbS, NaCN is used as
The entire scientific and technological process used for isolation
a depressant. It selectively prevents ZnS from coming to the
of the metal from its ore is known as metallurgy. froth but allows PbS to come with the froth.
EBD_7327
484 CHEMISTRY

(iv) Leaching : It is used if the ore is soluble in a suitable solvent. (a) Carbon reduction method : Smelting
(a) Leaching of alumina from bauxite: SnO2 + 2C ® Sn + 2CO ­
Bauxite contains SiO2, iron oxide and TiO2 as impurities. Tin oxide Tin
The powdered ore is digested with conc. NaOH solution at
PbO + C ® Pb + CO ­
473-523 K and 35-36 bar pressure Al2O3 is leached out as
sodium aluminate (SiO2 as sodium silicate) leaving the (b) Alumina thermic process :
impurities behind. Cr2O3 + 2Al ® 2Cr + Al2O3
Al2O3(s) + 2NaOH (aq) + 3H2O(l) ® 2Na[Al(OH)4] (aq) 3Mn 3O4 + 8 Al ® 9Mn + 4Al2O3
The solution is then neutralised by passing CO2 gas and (c) Precipitation method :
hydrated Al2O3 is precipitated out. Ag2S + 4NaCN ® 2Na[Ag(CN)2] + Na2S
2Na[Al(OH)4] (aq) + CO2 (g) ® Al2O3. xH2O(s) + 2NaHCO3 (aq) Silver glance Sodium argentocyanide
Sodium silicate remains in the solution and hydrated Al2O3 2Na[Ag(CN)2] + Zn ® 2Ag + Na2 [Zn(CN)4]
is filtered, dried and heated to give back pure Al2O3. (d) Electrolytic method : If metal oxides, chlorides or hydroxides
Al2O 3 . xH 2O ( s ) ¾¾ ¾¾ ® Al2 O 3 ( s ) + xH 2 O ( g )
1470K in fused state are electrolysed, the metal deposits at cathode.
The reduction of active elements such as alkali metals, alkaline
(b) Leaching of Au and Ag: earth metals and the oxides of Al with carbon is difficult
In metallurgy of Ag and Au, the respective metal is leached because these form carbides with carbon at high temperature.
with a dilute solution of NaCN or KCN in the presence of air For these metals, electrolytic method is used.
(for O2) to form soluble cyanide complex while impurities
remains uneffected. The metal is later obtained by its THERMODYNAMIC PRINCIPLES OF METALLURGY
replacement with more electropositive Zn metal. Some concepts of thermodynamics, like Gibbs free energy (G) help
4M(s) + 8CN– (aq) + 2H2O (aq) + O2 (g) in understanding metallurgical transformations.
® 4 [M(CN)2] –(aq) + 4OH– (aq) (M = Ag or Au) The change in Gibbs energy is given by:
-
DG = DH – TDS
2-
2 éë M ( CN )2 ùû ( aq ) + Zn ( s ) ¾¾
® éë Zn ( CN ) 4 ùû (aq)+2M(s) Also, DG = – RT lnK
For a reaction to be spontaneous, DG should be negative.
Extraction of crude Metal from concentrated ore If on coupling the two reactions, sum of their DG comes out to be
It involves two major steps: negative, then the overall reaction will occur.
(i) Conversion to oxide H.J.T. Elligham diagram (plots of DG Vs T) provides a sound basis
(a) Calcination : It involves heating in absence or limited for considering the choice of reducing agent in the reduction of
supply of air. Thus the volatile matter (moisture, S, As, P oxides.
etc.) escapes leaving behind metal oxide: This method is The reducing agent forms its oxide when the metal oxide is reduced.
commonly used to convert metal carbonates and The role of reducing agent is to provide DG negative and large
hydroxides to their respective oxides. enough to make the sum of DG° of the two reactions negative.
The temperature is chosen such that the sum of DG° in the two
D
Fe 2 O3 . xH 2 O ( s ) ¾¾
® Fe 2 O3 ( s ) + xH 2 O ( g ) combined redox process is negative.
D
ZnCO3 ( s ) ¾¾
® ZnO (s ) + CO 2 ( g )
D
CaCO3 .MgCO3 ( s ) ¾¾® CaO ( s ) + MgO ( s ) + 2CO 2 ( g )
(b) Roasting: The ore is heated in a regular supply of air in
a furnance at a temperature below the melting point of
the metal. This process is commonly used for sulphide
ores. In this process moisture organic matter and non-
metallic impurities got removed.
2ZnS + 3O 2 ¾¾
® 2ZnO + 2SO 2

2PbS + 3O 2 ¾¾
® 2PbO + 2SO 2

2Cu 2S + 3O 2 ¾¾
® 2Cu 2 O + 2SO 2
(ii) Reduction of oxide to the metal : It involves heating the
metal oxide with some reducing agent (C or CO or other metal).
The reducing agent (ex: C) combines with oxygen of the metal
oxide.
Fig. Gibbs energy (DGº) vs T plots (schematic) for formation
M x O y + yC ¾ ¾
® xM + yCO of some oxides (Ellingham diagram)
In DG° vs T plots, this is indicated by the point of intersection of
Some metals are easily reduced while some are reduced with
difficulty. the two curves. After that point, DG value becomes more negative
for the combined process including the reduction of MxO.
General Principles and Processes of Isolation of Elements 485

The difference in the two DG° values after that point determines At 900 – 1500 K, the reactions are:
whether reductions of the oxide of the upper line is feasible by the
C + CO 2 ¾¾
® 2CO
element represented by the lower line. If the difference is large, the
reduction is easier. FeO + CO ¾¾ ® Fe + CO 2
The reduction of metal oxide can be accomplished with the help of
Limestone is decomposed to CaO which removes silicate
the following reaction impurity as slag. The slag is in the molten state and separates
out from iron. The iron obtained is called pig iron and contain
M x O ( s ) + C ( s ) ¾¾
® xM ( s or l ) + CO ( g )
about 4% carbon and many impurities in smaller amount.
(i) Cast iron has slightly lower C-content and is very hard
M x O ( s ) + CO ( g ) ¾¾
® xM ( s or l ) + CO 2 ( g ) and brittle.
(ii) Wrought iron or malleable iron is the purest form of
1 1
M x O ( s ) + C ( s ) ¾¾
® xM ( s or l ) + CO2 ( g ) commercial iron and is prepared from cast iron by
2 2 oxidising impurities in a reverberatory furnance.
ELECTROCHEMICAL PRINCIPLES OF METALLURGY Fe 2 O3 + 3C ¾¾
® 2Fe + 3CO
Reduction of metal in the molten state or in solution state is
(b) Extraction of Copper From Cuprous Oxide
carried out by using electrolysis or by adding suitable reducing
agent. The equation used is DG° = – nFE° The ore is concentrated by froth flotation method.
where n ® no. of electrons The oxide ores of Cu are easily reduced to the metal by heating
with coke. However, most of the ores are sulphide and may
E° ® electrode potential of the redox couple
also contain iron. The sulphide ore is first roasted / smelted
If the difference of two E° values is positive, DG becomes
to give oxide:
–ve and thus, a less reactive metal will be displaced by a more
reactive metal. The less reactive metal will come out of the solution 2Cu 2S + 3O 2 ¾¾ ® 2Cu 2O + 2SO 2
and the more reactive will go to the solution. The oxide can then be easily reduced to metallic copper using
coke:
Example : Cu 2+ ( aq ) + Fe ( s ) ¾¾
® Cu(s) + Fe 2+ ( aq )
Cu 2 O + C ¾¾ ® 2Cu + CO
In electrolysis, M n+ ions are discharged at the cathode
(–ve electrode). Sometimes a flux is added to make the molten The ore is heated in a reverberatory furnance after mixing
mass more conducting. with silica. Iron oxide ‘slags of’ as iron silicate and copper is
produced in the form of copper matte. This contains Cu2S
EXTRACTION OF SOME METALS FROM THEIR ORES and FeS.
(a) Extraction of Iron From its Oxides: FeO + SiO2 ¾¾ ® FeSiO3 ( slag )
Iron ores are concentrated by magnetic separation method. Copper matte is then charged into silica lined convertor. Some
Iron oxide is reduced by using coke in a blast furnance. One silica is also added and hot air blast is blowed to convert the
of the main reduction step is: remaining FeS, FeO to FeSiO3 and Cu2S, Cu2O to metallic
copper. Following reactions take place:
FeO ( s ) + C ( s ) ¾¾
® Fe ( s / l ) + CO ( g )
This reaction can be written as: 2FeS + 3O 2 ¾¾
® 2FeO + 2SO 2

1 FeO + SiO 2 ¾¾
® FeSiO 3
FeO ( s ) ¾¾
® Fe ( s ) + O2 ( g ) DG( FeO, Fe )
2 2Cu 2S + 3O 2 ¾¾
® 2Cu 2 O + 2SO 2
1
C ( s) + O 2 ( g ) ¾¾
® CO ( g ) DG C, CO 2Cu 2 O + Cu 2 S ¾¾® 6Cu + SO 2
2 The solidified copper obtained has blistered appearance due
The net Gibbs energy change becomes: to evolution of SO2 and is also called blister copper.
DG ( C, CO ) + D G ( FeO,Fe ) = D r G Copper from low grade ores and scraps : Cu is extracted by
hydrometallurgy from low grade ores. It is leached out using acid
From Ellingham diagram, it is found that at temperatures
or bacteria. The solution containing Cu2+ is treated with scrap
above 1073 K (approx), coke reduces FeO and itself gets
iron or H2.
oxidised to CO.
In Blast furnance, reduction of iron oxide takes place at Cu 2+ ( aq ) + H 2 ( g ) ¾¾
® Cu ( s ) + 2H + ( aq )
different temperature ranges.
(c) Extraction of Zinc from Zinc Oxide
At 500 – 800 K, the reactions taking place are:
The reduction is done by using coke. For the purpose of
3Fe 2O3 + CO ¾¾
® 2Fe3O 4 + CO 2 heating, the oxide is made into brickettes with coke and clay.
coke, 673K
Fe3O 4 + 4CO ¾¾
® 3Fe + 4CO 2 ZnO + C ¾¾¾¾¾® Zn + CO
The metal is distilled off and collected by rapid chilling
Fe2 O3 + CO ¾¾
® 2FeO + CO 2
EBD_7327
486 CHEMISTRY

(d) Extraction of Aluminium from Alumina in the same direction. At one end, impurities get concentrated.
Al is obtained from alumina by the process of electrolysis This end is cut off. This method is very useful for producing
which is known as Hall-Heroult process. Steel cathode and semi-conductor and other metals of very high purity.
graphite anode are used. Alumina is mixed with Na3AlF6 or Ex. Ge, Si, B, Ga and In.
CaF2 (they lower the melting point of the mix and brings (v) Vapour Phase Refining
conductivity). The fused matrix is electrolysed. In this method, the metal is converted into its volatile
The overall reaction can be written as: compound and collected elsewhere. It is then decomposed
2Al2 O 3 + 3C ¾¾ ® 4Al + 3CO 2 to give pure metal.
Two requirements are :
The oxygen liberated at anode reacts with C of anode to (i) Metal should form a volatile compound with an available
produce CO and CO2. For each kg of Al produced, about reagent.
0.5 kg of C anode is burnt away. The reactions are: (ii) The volatile compound should be easily decomposable,
Cathode: Al 3+ ( melt ) + 3e - ¾¾
® Al ( l ) so that the recovery is easy.
Mond process for refining nickel:
Anode: C ( s ) + O 2- ( melt ) ¾¾
® CO ( g ) + 2e - Ni is heated in a stream of CO to form volatile Ni(CO)4 :
330-350K
C ( s ) + 2O 2- ( melt ) ¾¾
® CO 2 ( g ) + 4e - ® Ni ( CO )4
Ni + 4CO ¾¾¾¾¾
The carbonyl compound is subjected to higher temperature
REFINING
so that it is decomposed giving pure metal:
Some methods of purifying the metals are:
450 - 470K
(i) Distillation: Ni (CO) 4 ¾¾¾¾¾ ® Ni + 4CO
It is useful for low boiling metals like Zn and Hg. The impure Van Arkel method for refining Zr or Ti:
metal is evaporated to obtain the pure metal as distillate. The crude metal in heated in an evacuated vessel with iodine.
(ii) Liquation: The metal iodide being more covalent, volatilises:
In this, low melting metal like Sn is made to flow on a sloping
surface. Being low melting, the metal melts and flows down Zr + 2I 2 ¾¾ ® ZrI4
the slope while the higher melting impurities stay behind. The metal iodide is decomposed on a tungsten filament, when
(iii) Electrolytic Refining electrically heated to about 1800 K. The pure metal is thus
Many metals are purified electrolytically. The metal salt is deposited on the filament:
taken as electrolyte, impure metal rod as anode and thin pure
ZrI 4 ¾¾
® Zr + 2I 2
metal rod or strip as cathode. On electrolysis pure metal is
deposited at cathode. Cu, Ag, Al, Zn, Cr etc. metals are (vi) Chromatographic Methods
purified by this method. It is based on the principle that different components of a
(iv) Zone Refining mixture are differently adsorbed on the adsorbent. The mixture
It is based on the principle that impurities are more soluble in is put in a liquid or gaseous medium which is moved through
the melt than in the solid state of the metal. A circular mobile the adsorbent.
heater is fixed at one end of a rod of impure metal. The molten Different components are adsorbed at different levels on the
zone moves along with the heater which is moved forward. column. Depending upon the physical state of the moving
As the heater moves forward, pure metal crystallises out of medium and the adsorbent material and also on the process
the melt and impurities pass on into the adjacent molten zone. of passage of the moving medium, the chromatographic
The process is repeated several times and the heater is moved method is given the name.
General Principles and Processes of Isolation of Elements 487

CONCEPT MAP
EBD_7327
488 CHEMISTRY

1. Which one of the following is an ore of silver ? 16. Calcination is used in metallurgy for removal of
(a) Argentite (b) Stibnite (a) moisture (b) water and CO2
(c) Haematite (d) Bauxite (c) CO 2 and H 2S (d) H 2 O and H 2S
2. Leaching is a process of 17. Which one of the following is not a sulphide ore?
(a) reduction (b) concentration (a) Magnetite (b) Iron pyrites
(c) refining (d) oxidation (c) Copper glance (d) Sphalerite
3. Copper can be extracted from (e) Galena
(a) Kupfernical (b) Dolomite 18. Roasting is generally done in case of the
(c) Malachite (d) Galena (a) oxide ores (b) silicate ores
4. Autoreduction is employed in the metallurgy is (c) sulphide ores (d) carbonate ores
(a) Hg (b) Al (c) Ti (d) Zn 19. Heating of pyrites in air for oxidation of sulphur is called
5. The most abundant metal on the surface of the earth is (a) roasting (b) calcination
(a) Fe (b) Al (c) Ca (d) Na (c) smelting (d) slagging
6. Which of the following is an ore of tin ? 20. Pyrolusite is an ore of
(a) Carborundum (b) Epsomite (a) polonium (b) manganese
(c) Cassiterite (d) Spodumene (c) tin (d) lead
7. Which of the following is chalcopyrite? 21. Which one of the furnaces among the following can produce
(a) CuFeS2 (b) FeS2 the highest temperature?
(c) KMgCl3.6H2O (d) Al2O3.2H2O (a) muffle furnace (b) blast furnace
8. Which one of the following does not occur as sulphide ore? (c) reverberatory furnace (d) electric furnace
(a) Zn (b) Cr (c) Ag (d) Fe 22. The role of calcination in metallurgical operations is
9. Which of the following metal is correctly matched with its (a) to remove moisture
ore? (b) to decompose carbonates
Metal Ore (c) to drive off organic matter
(a) Zinc Calamine (d) to decompose carbonates and drive off moisture and
(b) Silver Ilmenite organic matter
(c) Magnesium Cassiterite 23. Solder is an alloy of lead with :
(d) Tin Azurite (a) copper (b) zinc (c) nickel (d) tin
10. Composition of azurite mineral is 24. General method for the extraction of metal from oxide ore is
(a) CuCO3CuO (b) Cu(HCO3)2. Cu(OH)2 (a) carbon reduction (b) reduction by aluminium
(c) 2CuCO3.Cu(OH)2 (d) CuCO3. 2Cu(OH)2 (c) reduction by hydrogen (d) electrolytic reduction
11. Which one of the following ores is best concentrated by 25. Aluminothermic process is used for the extraction of metals,
froth-flotation method ? whose oxides are
(a) Galena (b) Cassiterite (a) fusible
(c) Magnetite (d) Malachite (b) not easily reduced by carbon
12. Germanium of very high purity is obtained by (c) not easily reduced by hydrogen
(a) liquation (b) vapour phase refining (d) strongly basic
(c) distillation (d) zone refining 26. In the blast furnace iron oxide is reduced by
13. Iron is obtained on large scale from Fe2O3 by (a) Silica (b) CO
(a) reduction with Na (b) reduction with Al (c) carbon (d) Limestone
(c) reduction with CO (d) passing H2 27 Which of the following is the chief ore of copper?
14. Cassiterite is concentrated by (a) Cu2S (b) Cu2S
(a) levigation (c) CuFeS2 (d) CuCO3.Cu(OH)2
(b) electromagnetic separation 28. Furnaces are lined with calcium oxide because
(c) floatation (a) it gives off oxygen on heating
(d) liquefaction (b) it gives strong light on heating
15. Among of the following statements the correct one is (c) it is refractory and basic
(a) zinc blende and iron pyrites are sulphides (d) it is not affected by acids
(b) calamine and sidherite are carbonates 29. The substance used in the thermite process of reducing
(c) malachite and azoxite are ores of copper metal ores is
(d) argentite and cuprite are oxides (a) aluminium (b) thorium
(c) heated Pt gauge (d) carbon
General Principles and Processes of Isolation of Elements 489

30. Which of the following is not an ore of magnesium? (a) volatile stable compound
(a) Carnallite (b) Magnesite (b) volatile unstable compound
(c) Dolomite (d) Gypsum (c) non volatile stable compound
31. Process followed before reduction of carbonate ore is – (d) None of the above
(a) calcination (b) roasting 42. A metal which is refined by poling is
(c) liquation (d) polling (a) sodium (b) blister copper
32. Which of the following set of equations represent correct (c) zinc (d) silver
mode of obtaining Pb from PbS by self-reduction process? 43. In the extraction of Cu, the metal is formed in the bessemer
converter due to the reaction :
Air
(A) PbS ¾¾¾
D
® PbO + SO 2 (a) Cu2S + 2Cu2O ¾¾ ® 6Cu + SO2
(b) Cu2S ¾¾ ® 2Cu + S
2PbS (c) Fe + Cu2O ¾¾ ® 2Cu + FeO
2Pb + SO2 (d) 2Cu2O ¾¾ ® 4Cu + O2
Air 44. If the impurities in a metal has a greater affinity for oxygen
(B) PbS ¾¾¾
D
® PbO + SO 2 and is more easily oxidised than the metal, then the
purification of metal may be carried out by
C
(a) distillation (b) zone refining
Pb + CO2 (c) electrolytic refining (d) cupellation
(a) (A) (b) (B) 45. The sulphide ore of copper is concentrated by
(c) Both (A) and (B) (d) Neither (A) nor (B) (a) chemical method (b) froth floatation method
33. Which of the following species has the highest electron (c) magnetic method (d) none of these
affinity? 46. In order to refine “blister copper” it is melted in a furnace
(a) F (b) O (c) O– (d) Na+ and is stirred with green logs of wood. The purpose is
(a) to expel the dissolved gases in blister copper.
34. Flux is used to :
(b) to bring the impurities to surface and oxidize them.
(a) remove silica
(c) to increase the carbon content of copper.
(b) remove silica and undesirable metal oxides
(d) to reduce the metallic oxide impurities with hydrocarbon
(c) remove all impurities from ores
gases liberated from the wood.
(d) reduce metal oxide 47. Hydro-metallurgical process of extraction of metals is based
35. Thermite process is used in reduction of on
(a) Cr2O3 (b) Al2O3 (c) PbO2 (d) CuO (a) complex formation (b) hydrolysis
36. Which of the following reactions represents calcination? (c) dehydration (d) dehydrogenation
(a) 2Ag + 2HCl + [O] ® 2AgCl + H2O 48. In Hall's process, the main reagent is mixed with :
(b) 2Zn + O2 ® 2ZnO (a) NaF (b) Na3AlF6
(c) 2ZnS + 3O2 ® 2ZnO + 2SO2 (c) AlF3 (d) none of these
(d) MgCO3 ® MgO + CO2 49. Match List I with List II and select the correct answer using
37. The electrolytic method of reduction is employed for the the codes given below the list
preparation of metals that List I List II
(a) are weakly electropositive 1. Ti A. Bauxite
(b) are moderately electropositive 2. Si B. Cerussite
(c) are strongly electropositive 3. Al C. Van-Arkel method
(d) form oxides 4. Pb D. Zone refining
38. Froth floatation process is used for the concentration of (a) 1–B, 2–A, 3–C, 4–D (b) 1–B, 2–C, 3–A, 4–B
(a) Chloride ore (b) Amalgams (c) 1–C, 2–A, 3–B, 4–D (d) 1–C, 2–D, 3–A, 4–B
(c) Oxide ore (d) Sulphide ore 50. Which of the following statements, about the advantage of
39. The cryolite is represented by roasting of sulphide ore before reduction is not true?
(a) BaSO 4 (b) Al 2O 3 (a) The DG of of the sulphide is greater than those for CS2
(c) CuFeS 2 (d) Na 3AlF6 and H2S.
40. During the process of electrolytic refining of copper, some (b) The DG of is negative for roasting of sulphide ore to
metals present as impurity settle as ‘anode mud’. These are oxide.
(a) Fe and Ni (b) Ag and Au (c) Roasting of the sulphide to the oxide is
(c) Pb and Zn (d) Sn and Ag thermodynamically feasible.
41. Van Arkel method of purification of metals involves (d) Carbon and hydrogen are suitable reducing agents for
converting the metal to a reduction of metal sulphides.
EBD_7327
490 CHEMISTRY

1. The elements present in the core of earth are collectively 9. Consider the following statements –
known as (A) In the Aluminothermite process, aluminium acts as
(a) lithophiles (b) nucleophiles reducing agent.
(c) chalcophiles (d) siderophiles
(B) The process of extraction of gold involves the formation
2. Method used for obtaining highly pure silicon which is used
as a semiconductor material, is of [Au(CN)2]– and [Zn(CN)4]2–.
(a) oxidation (b) electrochemical (C) In the extractive metallurgy of zinc, partial fusion of
(c) crystallization (d) zone refining ZnO with coke is called sintering and reduction of ore
3. Refractory metals are used in construction of furnaces to the molten metal is called smelting.
because (D) Extractive metallurgy of silver from its ore argentine
(a) they can withstand high temperature involves complex formation and displacement by more
(b) they are chemically inert electropositive metal.
(c) their melting point is high Choose the correct options –
(d) None of these (a) A and B (b) B and C
4. In electro-refining of metal the impure metal is used to make (c) A, B and C (d) A, B, C and D
the anode and a strip of pure metal as the cathode, during 10. In Goldschmidt alumino thermic process which of the
the electrolysis of an aqueous solution of a complex metal following reducing agents is used :
salt. This method cannot be used for refining of (a) calcium (b) coke
(a) Silver (b) Copper (c) Al-powder (d) sodium
(c) Aluminium (d) Sodium 11. Which statement is correct ?
5. According to Ellingham diagram, the oxidation reaction of (a) Gangues are carefully chosen to combine with the slag
carbon to carbon monoxide may be used to reduce which present in the ore to produce easily fusible flux to carry
one of the following oxides at the lowest temperature ? away the impurities
(b) Slags are carefully chosen to combine with the flux
(a) Al2O3 (b) Cu2O
present in the ore to produce easily fusible gangue to
(c) MgO (d) ZnO carry away the impurities
6. While extracting an element from its ore, the ore is grounded (c) Gangues are carefully chosen to combine with the flux
and leached with dil. potassium cyanide solution to form the present in the ore to produce easily fusible slag to carry
soluble product potassium argento cyanide. The element is away the impurities
(a) Lead (b) Chromium (d) Fluxes are carefully chosen to combine with the gangue
present in the ore to produce easily fusible slag to carry
(c) Manganese (d) Silver away the impurities
7. Calcination is the process in which : 12. Sulfide ores are common for the metals
(a) ore is heated above its melting point to expel H2O or (a) Ag, Cu and Pb (b) Ag, Mg and Pb
CO2 or SO2 (c) Ag, Cu and Sn (d) Al, Cu and Pb
(b) ore is heated below its melting point to expel volatile 13. Bauxite ore is made up of Al2O3 + SiO2 + TiO2 + Fe2O3. This
ore is treated with conc. NaOH solution at 500 K and 35 bar
impurities
pressure for few hours and filtered hot. In the filtrate the
(c) ore is heated above its melting point to remove S, As species present, is/are
and Sb as SO2 ,AS2O3 and Sb2O3 respectively (a) NaAl(OH)4 only
(d) ore is heated below its melting point to expel H2O or (b) Na2Ti(OH)6 only
CO2 (c) NaAl(OH)4 and Na2SiO3
(d) Na2SiO3 only
8. When CuSO 4 is electrolysed using platinum electrodes, 14. Which is not a basic flux ?
(a) CaCO3 (b) Lime
(a) copper is liberated at cathode, sulphur at anode
(c) SiO2 (d) CaO
(b) copper is liberated at cathode, oxygen at anode 15. The compound that can work both as an oxidising agent
(c) sulphur is liberated at cathode, oxygen at anode and a reducing agent is
(a) KMnO4 (b) H2S
(d) oxygen is liberated at cathode, copper at anode
(c) BaO2 (d) H2O2.
General Principles and Processes of Isolation of Elements 491

16. Aluminium is prepared in large quantities by DIRECTIONS for Qs. 21 to 25: These are Assertion-Reason
(a) heating cryolite in a limited quantity of air type questions. Each of these question contains two statements:
(b) reducing aluminium oxide with coke Statement-1 (Assertion) and Statement-2 (Reason). Answer these
(c) reducing aluminium oxide with sodium questions from the following four options.
(d) electrolysing aluminium oxide dissolved in fused (a) Statement-1 is True, Statement-2 is True, Statement-2 is a
electrolyte correct explanation for Statement-1
17. Duraluminium is an alloy of (b) Statement-1 is True, Statement-2 is True ; Statement-2 is
(a) Al and Mg (b) Al, Mg and Ni NOT a correct explanation for Statement-1
(c) Al, Mg, Mn and Cu (d) Al and Ni. (c) Statement-1 is True, Statement-2 is False
18. Which of the following factors is of no significance for (d) Statement-1 is False, Statement-2 is True
roasting sulphide ores to the oxides and not subjecting the 21. Statement-1 Zinc can be used while copper cannot be used
sulphide ores to carbon reduction directly? in the recovery of Ag from the complex [Ag(CN)2]–.
(a) Metal sulphides are thermodynamically more stable Statement-2 Zinc is a powerful reducing agent than copper.
than CS2 22. Statement-1 Coke and flux are used in smelting.
(b) CO2 is thermodynamically more stable than CS2 Statement-2 The phenomenon in which ore is mixed with
(c) Metal sulphides are less stable than the corresponding suitable flux and coke is heated to fusion is known as
oxides smelting.
(d) CO2 is more volatile than CS2 23. Statement-1 Leaching is a process of reduction.
19. Pb and Sn are extracted from their chief ore by Statement-2 Leaching involves treatment of the ore with a
(a) carbon reduction and self reduction. suitable reagent so as to make it soluble while impurities
(b) self reduction and carbon reduction. remains insoluble.
(c) electrolysis and self reduction. 24. Statement-1 Lead, tin and bismuth are purified by liquation
(d) self reduction and electrolysis. method.
20. Extraction of zinc from zinc blende is achieved by Statement-2 Lead, tin and bismuth have low m.p. as compared
(a) electrolytic reduction to impurities.
(b) roasting followed by reduction with carbon 25. Statement-1 Levigation is used for the separation of oxide
(c) roasting followed by reduction with another metal ores from impurities.
(d) roasting followed by self-reduction Statement-2 Ore particles are removed by washing in a
current of water.

Exemplar Questions (a) Al and Fe (b) Al and Cu


(c) Fe and Cu (d) Cu and Ag
1. In the extraction of chlorine by electrolysis of brine ........... 5. Zone refining is based on the principle that ..................
(a) oxidation of Cl– ion to chlorine gas occurs (a) impurities of low boiling metals can be separated by
(b) reduction of Cl– ion to chlorine gas occurs distillation.
(c) for overall reaction D G s has negative value (b) impurities are more soluble in molten metal than in solid
(d) a displacement reaction takes place metal.
2. When copper ore is mixed with silica in a reverberatory (c) different components of a mixture are differently
furnace copper matte is produced. The copper matte contains adsorbed on an adsorbent.
(a) sulphides of copper (II) and iron (II) (d) vapours of volatile compound can be decomposed in
(b) sulphides of copper (II) and iron (III) pure metal.
(c) sulphides of copper (I) and iron (II) 6. In the extraction of copper from its sulphide ore, the metal is
(d) sulphides of copper (I) and iron (III) formed by the reduction of Cu2O with
3. Which of the following reactions is an example of auto- (a) FeS (b) CO
reduction? (c) Cu2S (d) SO2
(a) Fe3 O 4 + 4CO ¾¾ 7. Brine is electrolysed by using inert electrodes. The reaction
® 3Fe + 4CO 2
at anode is .................
(b) Cu 2 O + C ¾¾ ® 2Cu + CO 1
(a) Cl - ( aq ) ¾¾ ® Cl2 ( g ) + e - ; Escell = 1.36 V
(c) Cu 2+ ( aq ) + Fe ( s ) ¾¾
® Cu ( s ) + Fe 2 + ( aq ) 2
(b) 2H 2 O ( l ) ¾¾ ® O 2 ( g ) + 4H + + 4e - ; Escell = 1.23 V
1 1
(d) Cu 2 O + Cu 2S ¾¾ ® 3Cu + SO 2
2 2 (c) Na + ( aq ) + e - ¾¾
® Na ( s ) ; Escell = 2.71 V
4. A number of elements are available in earth's crust but most 1
abundant elements are ................ (d) H + ( aq ) + e - ¾¾
® H 2 ( g ) ; Escell = 0.00 V
2
EBD_7327
492 CHEMISTRY

8. In the metallurgy of aluminium .............. (b) DG value for the overall reduction reaction with a
(a) Al3+ is oxidised to Al (s). mixture of 1 mol carbon and 1 mol oxygen is positive.
(b) graphite anode is oxidised to carbon monoxide and (c) DG value for the overall reduction reaction with a
carbon dioxide. mixture of 2 mol carbon and 1 mol oxygen will be
(c) oxidation state of oxygen changes in the reaction at positive.
anode. (d) DG value for the overall reduction reaction with carbon
(d) oxidation state of oxygen changes in the overall monoxide is negative.
reaction involved in the process.
9. Electrolytic refining is used to purify which of the following NEET/AIPMT (2013-2017) Questions
metals? 14. The metal oxide which cannot be reduced to metal by carbon
(a) Cu and Zn (b) Ge and Si is [NEET Kar. 2013]
(c) Zr and Ti (d) Zn and Hg (a) Fe2O3 (b) Al2O3
10. Extraction of gold and silver involves leaching the metal (c) PbO (d) ZnO
with CN – ion. The metal is recovered by .......... 15. "Metals are usually not found as nitrates in their ores".
(a) displacement of metal by some other metal from the Out of the following two ((i) and (ii)) reasons which is/are
complex ion. true for the above observation ? [2015]
(b) roasting of metal complex. (i) Metal nitrates are highly unstable.
(c) calcination followed by roasting.
(ii) Metal nitrates are highly soluble in water.
(d) thermal decomposition of metal complex.
(a) (i) and (ii) are false
Direction (Q. No. 11-13) Answer the questions on the basis of
figure. (b) (i) is false but (ii) is true
–200 eO . . (c) (i) is true but (ii) is false

. A D. 2F (d) (i) and (ii) are true


D G°/kJ mol of O2 ®

®
–300 O 2 16. In the extraction of copper from its sulphide ore, the metal
. ... E e+
2 F C + O2 ® CO2 finally obtained by the reduction of cuprous oxide with :
–400
. B
–1

[2015 RS]
–500 .
2C
+
O2 +
O
(a) iron (II) sulphide (b) carbon monoxide
O O2 2 ®
2C 2 C 2C (c) copper (I) sulphide (d) sulphur dioxide
–600 ® O
17. Match items of Column I with the items of Column II and
asign the correct code : [2016]
–700 Column-I Column-II
0 400 800 1200 1600 2000
Temperature (°C) ® (A) Cyanide (i) Ultrapure Ge
11. Choose the correct option of temperature at which carbon process
reduces FeO to ion and produces CO. (B) Froth flotation (ii) Dressing of ZnS
(a) Below temperature at point A process
(b) Approximately at the temperature corresponding to (C) Electrolytic (iii) Extraction of Al
point A reduction
(c) Above temperature at point A but below temperature (D) Zone refining (iv) Extraction of Au
at point D (v) Purification of Ni
(d) Above temperature at point A Code :
12. Below point 'A' FeO can .....................
(a) (b) (c) (d)
(a) be reduced by carbon monoxide only.
(a) (iv) (ii) (iii) (i)
(b) be reduced by both carbon monoxide and carbon.
(c) be reduced by carbon only. (b) (ii) (iii) (i) (v)
(d) not be reduced by both carbon and carbon monoxide. (c) (i) (ii) (iii) (iv)
13. For the reduction of FeO at the temperature corresponding (d) (iii) (iv) (v) (i)
to point D, which of the following statements is correct? 18. Extraction of gold and silver involves leaching with CN–ion.
(a) DG value for the overall reduction reaction with carbon Silver is later recovered by [2017]
monoxide is zero. (a) distillation (b) zone refining
(c) displacement with Zn (d) liquation
General Principles and Processes of Isolation of Elements 493

Hints & Solutions


EXERCISE - 1 18. (c) In this process sulphide ores are converted into oxide
ores.
1. (a) Argentite or silver glance (Ag2S) is an ore of Ag. 2ZnS + 3O2 ® 2ZnO + 2SO2­
2. (b) Leaching is a process used for concentration of ore. In 19. (a)
this process, a powdered ore is treated with a suitable 20. (b) Pyrolusite is MnO2. So, it is an ore of manganese. It is
reagent (such as acids, bases or other chemicals) which used in making dry batteries and also in brick industry.
can selectively dissolve the ore, but not the impurities. 21. (d) 22. (d)
3. (c) Malachite is CuCO3 . Cu(OH)2 it is ore of copper. 23. (d) Solder is an alloy of lead (Pb) with tin (Sn). It contains
4. (a) Autoreduction or self reduction is mainly used for about 30% Sn and 70% Pb.
sulphide ores of less electropositive metals like Hg, Pb, 24. (a) Carbon reduction, Fe2O3 + 3 C ® 2Fe + 3CO
Cu, etc. 25. (b) Aluminothermite process involves reduction of oxides
2PbS + 3O2 ¾¾® 2PbO + 2SO2 which are not satisfactorily reduced by carbon such as
2PbO + PbS ¾¾® 3Pb + 2SO2. Fe2O3, Mn3O4, Cr 2O3, etc. to metals with aluminium.
5. (b) Al is most abundant metal on the surface of the earth.
Cr2 O3 + 2Al ® Al 2O3 + 2Cr DH = - ve
6. (c) Carborundum - SiC
Epsomite or Epsom salt - MgSO4.7H2O 26. (b) Fe2 O3 + 3CO ® 2Fe + 3CO2
Cassiterite - SnO2 27. (c) The most common ore of copper is CuFeS2 which is
Spodumene - Ore of lithium commonly known as chalcopyrite.
7. (a) Chalcopyrite : CuFeS2 28. (c)
Fool's gold : FeS2 29. (a) A mixture of Al powder and metallic oxide (Cr2O3,
Carnalite : KMgCl3.6H2O Mn3O4 etc.) is called thermite.
Bauxite : Al2O3.2H2O 30. (d) Gypsum which is chemically represented as CaSO4.2H2O
8. (b) Except chromium all the given metals exists as their is not an ore of magnesium among the given choices.
sulphides. Note: Carnallite is KCl.MgCl2 .6H2O, magnesite is
Zn exists as zinc blende ZnS. MgCO3 and dolomite is MgCO3.CaCO3.
Silver exists as silver glance Ag2S. 31. (a) Calcination is heating ore in absence of air to remove
Iron exists as iron pyrites FeS2. moisture and volatile impurities. Carbonate ores
9. (a) (a) Zinc Calamine is ZnCO3 decomposed to corresponding oxides as a result of
(b) Silver Ilmenite is FeTiO3 calcination.
(c) Magnesium Cassiterite is SnO2 32. (a) PbS reduces PbO to Pb and SO2 is liberated. This is
(d) Tin Azurite is called as self-reduction.
[2CuCO3.Cu (OH)2] 33. (a) Fluorine is the most electronegative element. Hence its
10. (c) Azurite is a basic carbonate ore of copper. electron affinity is highest. By gaining one electron it
2CuCO3. Cu(OH)2 completes its octet. Whereas oxygen needs electrons,
11. (a) Galena is PbS and thus purified by froth floatation O– is already a negatively charged species hence have
method. some repulsion for the incoming electron. Na+, will again
Froth flotation method is used to concentrate sulphide loose its octet by gaining electron.
ores. This method is based on the fact that the surface 34. (b) Flux is used during metallurgy to remove silica and
of sulphide ores is preferentially wetted by oils while undesirable metal oxides.
that of gangue is preferentially wetted by water. 35. (a) Due to high affinity for oxygen aluminium reduces
12. (d) Metals of high purity are obtained by zone refining several metallic oxides like Fe2O3, Cr 2O3, Mn2O3 etc.
e.g., silicon, germanium, boron, gallium, indium.
Cr2 O3 + 2Al ¾¾ ® Al 2O 3 + 2Cr + Heat
13. (c) Fe2O3 + 3CO ® 2Fe + 3CO2 ( in blast furnace)
This process is known as thermite process.
14. (b) Cassiterite contains the magnetic impurities of FeSO 4 36. (c)
and thus concentrated by electromagnetic separation. 37. (d) Calcination is a process of heating the ore in a limited
15. (a) Zinc blende is ZnS and iron pyrite is FeS. supply of air below its melting point.
16. (b) Calcination is used for removal of volatile impurities D
and decompose carbonates into their respective oxides. MgCO3 ¾¾ ® MgO + CO 2
D 38. (d) Froth floatation process is used for sulphide ores in
Al2O3 · 2H2O ¾¾ ® Al2O3 + 2H2O which ore particles are accumulated in froth and the
Bauxite Alumina impurities remain in solution.
CaCO3 ¾¾ D 39. (d) Na 3AlF6 (cryolite) is used to decrease the melting
® CaO + CO2
Limestone Calcium oxide point of Al 2 O3 prior to electrolysis to get pure
17. (a) The formula of magnetite is Fe3O4. aluminium.
EBD_7327
494 CHEMISTRY

40. (b) During the process of electrolytic refining of copper


¾® Na 2 [ Zn (CN ) 4 ] + 2 Ag ¯
2 Na[ Ag (CN ) 2 ] + Zn ¾
Ag and Au are obtained as anode mud.
7. (d) Calcination is a process of heating a substance to a
523K 1700K
41. (a) Ti + 2I2 ¾¾¾® TiI4 ¾¾¾¾
® Ti + 2I 2 high temperature but below the melting or fusion point,
Volatile Pure metal causing loss of moisture, reduction or oxidation and
Stable compound
dissociation into simpler substances.
42. (b) Poling is used for purification of metal which contain
their own oxide as impurity e.g., Cu2O in Cu, SnO2 in 8. (b) CuSO 4 Cu 2+ + SO 24 - ;
Sn etc.
43. (d) Decomposition of carbonates and hydrated oxides. H2O H + + OH -
44. (d) At cathode : Cu 2 + + 2e - ® Cu
45. (b) Froth floatation process is used for the concentration
of lighter ores (sulphides) like CuFeS2 (copper pyrite). At anode : 4OH - ® 2H 2 O + O 2 + 4e -
46. (d) 2Cu2S + 3O2 ® 2Cu2O + 2SO2
+3
3Cu 2 O + CH 4 ® 6Cu + 2H 2 O + CO 9. (d) (A) Cr2 O3 + 2Al ¾¾
® Al2 O3 + 2Cr
(From green
logs of wood)
- 1
47. (a) For example, Ag2S is converted into Na[Ag(CN)2]. (B) Au + 2CN + H 2 O + O2
When Zn is added, Ag is displaced. 2
48. (b) Na 3 AlF6 ® Cryolite ® [Au(CN) 2 ]- + 2OH -
¾¾
Al 2 O3 is mixed with cryolite.
Cryolite improves the electrical conductivity of the cell 2[Au(CN) 2 ]- + Zn ® [Zn(CN) 4 ]2 - + 2Au
as Al 2 O3 is a poor conductor. In addition, the cryolite (C) Statement is true
serves as an added impurity and lowers the melting O2
(D) Ag 2S + 4NaCN ¾¾¾ ® 2Na[Ag(CN) 2 ] + Na 2S
point of the mixture to about 950°C.
49. (d) Ti – van-Arkel method 2Na[Ag(CN) 2 ] + Zn ¾¾ ® Na 2 [Zn(CN) 4 ] + 2Ag
Si – Zone refining method
10. (c) Reduction by powdered aluminium is known as Gold-
Al – Bauxite (Al2O3)
Schmidt aluminothermic process. This process is
Pb – Cerussite (PbCO3)
employed in cases where metals have very high m.p.
50. (d) The sulphide ore is roasted to oxide before reduction
and are to be extracted from their oxides.
because the DGof of most of the sulphides are greater 11. (d)
than those of CS2 and H2S, therefore neither C nor H 12. (a) Silver, copper and lead are commonly found in earth's
can reduce metal sulphide to metal. Further, the standard crust as Ag2S (silver glance), CuFeS2 (copper pyrites)
free energies of formation of oxide are much less than and PbS (galena)
those of SO2. Hence oxidation of metal sulphides to 13. (a) In bauxite ore, only Al2O3 reacts with conc. NaOH and
metal oxide is thermodynamically favourable. formed sodium meta aluminate. This, further dissolves
EXERCISE - 2 in water.
500K
1. (d) Al2O3 + 2H2O + 2NaOH ¾¾¾®
35 bar 2NaAlO2 + 3H2O
2. (d) Si obtained by reduction of SiCl4 with H2 is further
NaAlO2 + 2H2O ® NaAl(OH)4
purified by zone refining method to get Si of very high
14. (c) SiO2 is an acidic flux.
purity. Silicon is purified by zone-refining process
15. (b) In froth floatation process, ore particles float because
because the impurities present in it are more soluble in
their surface is hydrophobic i.e., not easily wetted by
the liquid phase than in the solid phase.
oil.
3. (a) Refractory metals are used in the construction of
16. (d)
furnaces because they can withstand high temperature
17. (c) Duraluminium alloy contains Al (95%), Cu (3%), Mn
e.g. silica, flint, lime, etc.
(1%), Mg (1%).
Option (a) is correct.
18. (c) The reduction of metal sulphides by carbon reduction
4. (d) Na reacts vigorously with water (exothermic process )
process is not spontaneous because DG for such a
5. (b) In the graph of Dr G° vs T for formation of oxides, the
process is positive. The reduction of metal oxide by
Cu2O line is almost at the top. So, it is quite easy to
carbon reduction process is spontaneous as DG for
reduce oxide ores of copper directly to the metal by
such a process is negative. From this we find that on
heating with coke both the lines of C, CO and C, CO 2
thermodynamic considerations CO2 is more stable than
are at much lower temperature (500 - 600 K).
CS2 and the metal sulphides are more stable than
Cu 2 O + C ¾¾
® 2Cu + CO corresponding oxides.
6. (d) Cyanide process is used in the metallurgy of Ag In view of above the factor listed in choice (c) is incorrect
and so is of no significance.
2Ag 2S + 8 NaCN + O 2 + 2 H 2 O ¾ ¾®
19. (b) PbO & PbSO4 get reduced by PbS itself which is already
4 Na[Ag(CN ) 2 ] + 4 NaOH + 2S present in mixture so because the reduction took place
General Principles and Processes of Isolation of Elements 495

by itself, hence is known as self reduction. to O2 than that needed for oxidation of Cl– ions to Cl2.
So reaction (i) is taking place at anode.
D
2PbO + PbS ¾¾
® 3Pb + SO 2 ­ 8. (b) In metallurgy of aluminium, graphite anode is oxidised
D to CO and CO2 and this process is known as Hall-
PbSO 4 + PbS ¾¾ ® 2Pb + 2SO 2 ­ Heroult process.
20. (b) Extraction of Zn from ZnS (Zinc blende) is achieved by The electrolytic reactions the process are :
roasting followed by reduction with carbon. At cathode : Al3+ (melt) + 3e– ¾¾ ® Al(l)
2ZnS + 3O2 ¾¾ ® 2ZnO + 2SO2 At anode : C(s) +O2– (melt) ¾¾ ® CO (g) + 2e–
ZnO + C ¾¾ ® Zn + CO C(s) + 2O2– (melt) ¾¾ ® CO2 (g) + 4e–
21. (a) 9. (a) Two metals which are generally purified by using
22. (b) Both statemen-1 and statement-2 are true but statement- electrolytic refining are copper and zinc.
2 is not the correct explanation of statement-1. Non 10. (a) In the process of extraction of Ag and Au involving
fusible mass present in ore in mixing with suitable flux leaching with cyanide ion, the metal can be recovered
are fused which are then reduced by coke to give free by displacement of Ag/Au by some other metal in the
metal. complex ion. Reactions involved in the process are as
23. (d) Statement-1 is false but statement-2 is true. Leaching is follows :
a process of concentration. 4 Au (s) + 8CN– (aq) + 2 H2O (aq) + O2 (g) ¾¾ ®
24. (a) 4 [Au (CN)2]– (aq) + 4 OH– (aq)
25. (c) Statement-1 is true but Statement-2 is false. -
2 éë Au ( CN ) 2 ùû ( aq ) + Zn (s ) ¾¾
®
Oxide ores being heavier than the earthy or rocky
gangue particles, settle down while lighter impurities 2-
are washed away. 2Au ( s ) + éë Zn ( CN ) 4 ùû ( aq )
11. (d) Above point A : Df G°(C,CO) < Df G°(Fe, FeO)
EXERCISE - 3 Therefore, reduction of FeO by carbon occurs above
Exemplar Questions point A.
12. (a) Gibbs free energy change for the formation of CO2
1. (a) Chlorine extraction by electrolysis can be represented from CO has lesser value than Gibbs free energy change
as :- for the formation of FeO (below point A). As a result ,
2H2O(l) + 2Cl–(aq) ® H2(g) + Cl2(g) + 2OH–(aq) FeO will be reduced by CO below point A.
So, the oxidation of Cl– to Cl2 occurs. 13. (a) DG curve for the formation of CO2 and FeO intersect
2. (c) Copper matte contains sulphide of copper (I) and each other at point D thus, net reduction of FeO with
iron (II). CO is zero.
1 1
3. (d) Cu 2 O + Cu 2S ¾¾ ® 3Cu + SO 2 NEET/AIPMT (2013-2017) Questions
2 2
In this given reaction, copper is reduced by itself hence 14. (b) Al2O3 cannot be reduced by carbon.
it is known as auto-reduction. 15. (b) Metal nitrates are highly soluble in water and are very
4. (a) The most abundant elements are aluminium (8.3% by stable for e.g. NaNO3 and KNO3.
weight) and iron (4.2% by weight). 16. (c) 2 Cu2S (s) + 3O2 (g) ® 2Cu2O(s) + 2SO2(g)
5. (b) The principle behind zone refining is that the impurities The unchanged Cu2S, mixed with Cu2O and heated
are more soluble in molten state than in solid state of strongly in absence of air
the metal. 2 Cu2S + 2Cu2O ® 6Cu + SO2
6. (c) In the extraction of copper from its sulphide ore, the 17. (a) Highly electropositive metals like Al, K, Na etc. are
metal is formed according to the reaction given : extracted by the electrolytic reduction.
• zone refining method is used for obtaining metals
1 1
Cu 2O + Cu 2S ¾¾ ® 3Cu + SO 2 of high purity e.g. Ge
2 2 • Froth flotation process is suitable for sulphide ores
7. (a) Possible reactions occuring at anode are : • Cyanide process is used for the extraction of gold.
1 18. (c) Zn being more reactive than Ag and Au, displaces them.
(i) Cl - ( aq ) ¾¾
® Cl2 ( g ) + e - ; Escell = 1.36 V Leaching
2 4Ag + 8NaCN + 2H2O + O2 ¾¾¾¾®
4Na[Ag(CN)2] + 4NaOH
® O2 (g) + 4H+ + 4e– ; Escell = 1.23 V
(ii) 2H2O(l) ¾¾ Soluble Sodium dicyanoargentate (I)
The reaction at anode with lower value of Es is Soluble cyanide compound can be treated with Zn to
preferred and therefore water should get oxidised in give metal by displacement.
preference to Cl– (aq). However, Cl2 is produced instead Displacement
2Na[Ag(CN)2] + Zn ¾¾¾¾¾¾ ®
of O2. This unexpected result is explained on the basis Na2[Zn(CN)4] + 2Ag¯
of the fact that water needs greater voltage for oxidation
EBD_7327
496 CHEMISTRY

The p-Block Elements


21 (Group 15, 16, 17 and 18)
GROUP 15 ELEMENTS N shows a maximum covalency of 4 since only four orbitals
It includes are available for bonding. The heavier elements show higher
covalency due to availability of vacant d-orbitals. ex: PF6 –.
Nitrogen (N) (ii) Reactivity towards hydrogen:
Phosphorus (P) Non-metals
They form hydrides of the type EH3 where E = N\ P\ As\ Sb\
Arsenic (As) Bi. The stability of hydrides decreases from NH3 to BiH3.
Antimony (Sb) Metalloids This is because with increase in bond length from N to Bi,
Bismuth (Bi) Metal bond strength decreases due to which reducing character of
Elements of this group are called pinacogen hydrides increases down the group.
Hydrides are formed by the action of water or dilute acids on
Atomic Properties compounds : Mg3N2, Ca3P2, Zn3As2, Mg3Sb2 and Mg3Bi2.
(i) Electronic Configuration: Their valence shell electronic All hydrides are coloured gases. The order of basicity of
configuration is ns2 np3. Due to half-filled p-orbitals, they are hydrides is NH3 > PH3 > AsH3 > SbH3 > BiH3.
extra stable. This is because EH3 has a lone pair of e–1 s on E which is
(ii) Atomic and Ionic radii: Covalent and ionic radius increases available for donation. As the e– density of NH3 is high due
down the group. Increase in covalent radii from As to Bi is to smaller size of N, thus availability of lone pair of electrons
very small. This is due to poor shielding of the valence e–1 s for donation is more and hence, basicity of NH3 is more
by d- and f- orbitals in heavier members. compared to other hydrides of the group.
(ii) Ionisation enthalpy: Down the group, IE decreases due to Trends from NH3 to BiH3
increase in size. NH3 PH3 AsH3 SbH3 BiH3
The IE of group 15 is much greater than group 14 in
corresponding periods. This is because of extra stable filled Basic character decreases
¾¾¾¾¾¾¾¾¾¾¾¾ ®
p-orbital configuration and smaller size.
Thermal stability decreases
(iv) Electronegativity : With increase in size down the group, ¾¾¾¾¾¾¾¾¾¾¾¾®
electronegativity decreases. Reducing nature increases
¾¾¾¾¾¾¾¾¾¾¾¾®
Physical Properties
(i) All the elements are polyatomic Dipole moment decreases
¾¾¾¾¾¾¾¾¾¾¾¾ ®
(ii) N2 is a gas while other elements are solids
Bond angle decreases
(iii) Metallic character increases down the group due to decrease ¾¾¾¾¾¾¾¾¾¾ ®
in IE and increase in atomic size. (iii) Reactivity towards oxygen:
(iv) Except nitrogen, all elements show allotropy. Nitrogen forms five oxides with oxidation states ranging from
Chemical Properties +1 to +5 while other elements form oxides in + 3 and + 5
(i) Oxidation states: Common oxidation states are –3, +3, and oxidation states.
+5. The stability of +5 oxidation state decreases down the The oxide in higher oxidation state of the element is more
group while that of + 3 oxidation state increases down the acidic than that of lower oxidation state.The oxides of the
group. This is due to inert pair effect. type E2O3 of N and P are purely acidic, that of As and Sb are
For N, +1 to + 4 oxidation states disproportionate in acid amphoteric and those of Bi are predominaently basic.
solution. (iv) Reactivity towards halogens:
Ex: 3 HNO2 ¾¾ ® HNO3 + H2O + 2NO They form two series of halides : EX3 and EX5. N does not
For P, all intermediate oxidation states disproportionate into form pentahalide due to non-availability of
+ 5 and – 3 both in alkali and acid. d-orbitals in its valence shell. All trihalides except those of N
+3
D
+5 –3 are stable. In case o f N, only NF3 is known to be stable. The
Example : 4H PO ¾¾® 3H 3 PO 4 + PH 3 reason assigned to unstable nature of NCl3, NBr3, NI3 is the
3 3
From As to Bi tendency to undergo disproportionation weakness of the N–X bond due to large difference in the size
decreases. of nitrogen and halogen atom.
The p-Block Elements (Group 15, 16, 17 and 18) 497

Pentahalides are more covalent than trihalides. This is Chemical properties :


because it is very difficult to form E5+ due to very high. I.E. (i) Li, Mg, Ca, Al form nitrides
Bi does not form pentahalides due to inert pair effect 3Ca + N2 ¾¾ D
Note : ® Ca3N2
(a) Pentahalides are thermally less stable than trihalides (ii) N2 + O2 ƒ 2NO (electric arc) – 43.2 kcal
down the group stability of + 5 oxidation state decreases This reaction forms the basis for the manufacture of HNO3
due to inert pair effect. by Birkland and Eyde process.
(b) As the size of the halogen increases from F to I strength 1273K
(iii) CaC2 + N2 ¾¾¾¾ ® Ca(CN)2 + C
E –X of bond decreases and steric hinderance increases. calcium carbide calcium cyanamide
(v) Reactivity towards metals: Ca(CN)2 gives ammonia on reaction with water, therefore
All these elements react with metals to form binary [Ca(CN)2 + C] is used as fertilizer under the name nitrolim.
compounds exhibiting –3 oxidation state such as Ca3N2, (iv) Combination with hydrogen to form NH3 (Haber's process)
under pressure 100 – 1000 atmospheres.
Ca3P2, Na3As2, Zn3Sb2 and Mg3Bi2.
Fe + Mo; 400-550°C
Anomalous properties of nitrogen ˆˆˆˆˆˆˆˆˆˆˆ†
N 2 + 3H 2 ‡ˆˆˆˆˆˆˆˆˆˆˆ 2NH3
Nitrogen shows different behaviour from rest of the members of Uses :
the group due to its: (i) Used for the manufacture of NH3, HNO3, Ca(CN)2 etc.
(i) smaller size (ii) To provide inert atmosphere N2 gas is used in metallurgical
(ii) high electronegativity processes.
(iii) high IE (iii) It is used in filling electric bulbs.
(iv) non-availability of d-orbitals. Ammonia, NH3
Thus, the properties of N which are different from other elements Preparation:
of the group are:
(i) NH2CONH2 + 2H2O (NH4)2 CO3 2 NH3
(i) N can form pp – pp multiple bonds with itself and other urea
elements of small size and high electronegativity. As a + H2O + CO2
result, N exists as a diatomic molecule with a triple bond
(ii) 2NH 4 Cl + Ca ( OH ) 2 ¾¾
® 2NH 3 + 2H 2 O + CaCl 2
between the two atoms. On the other hand, P, As and Sb
form single bonds as P–P, As–As, Sb–Sb. (iii) ( NH 4 )2SO 4 + 2NaOH ¾¾® 2NH 3 + 2H 2O + Na 2SO 4
(ii) Due to non-availability of d-orbitals, N cannot form (iv) Haber’s process:
dp-pp bond while other elements can. Thus, R3P = O exists
but R3N = O cannot. N 2 ( g ) + 3H 2 ( g ) ƒ 2NH3 ( g ) .
(iii) Due to small size and high electronegativity, nitrogen forms D fHº = – 46.1 kJ mol–1.
nitride ion (N3–). Phosphorus to some extent also gives Optimum conditions for this reaction are high pressure (200 ×
phosphide ion (P3–) in some compounds. This tendency is 105 Pa) and temperature (~ 700 K) and use of catalyst like iron
absent in rest of the elements. oxide with small amounts of K2O and Al2O3.
Physical Properties:
NITROGEN AND SOME OF ITS IMPORTANT (i) It is a colourless gas with pungent odour.
COMPOUNDS (ii) In solid and liquid states, it is associated through
Dinitrogen, N2 H-bonds which results in its high M.Pt and B.Pt.
(iii) It is trigonal pyramidal, i.e.,
Preparation:
. . Lone pair
(i) NH4Cl (aq) + NaNO2 (aq)¾¾
® N2 (g) + 2H2O (l) + NaCl (aq)
N
(ii) ( NH 4 )2Cr2 O7 ¾¾
D ® N + 4H O + Cr O
2 2 2 3
H 107.8°
H
H
(iii) Ba ( N3 ) 2 D®
¾¾ Ba + 3N 2 (give very pure N2)
Chemical properties :
(iv) 2NaN3 ¾¾ ® 2Na+3N2 (i) Its aqueous solution is weakly basic due to formation of OH–
Commercially, it is produced by liquefaction and fractional ions:
distillation of air. Liquid N2 distils out first leaving behind NH 3 ( g ) + H 2 O ( l ) ƒ NH +4 ( aq ) + OH - ( aq )
liquid oxygen. (ii) It forms ammonium salts with acids and being a weak base,
Physical Properties: precipitates the hydroxides of many metals from their salt
(i) It is colourless, odourless, tasteless and non-toxic gas. solutions. For ex:
(ii) It has two stable isotopes 14N and 15N. 2FeCl3 ( aq ) + 3NH 4OH ( aq ) ¾¾
®
(iii) It is inert at room temperature due to high bond energy of
N º N bond. Fe(OH)3 (s) ¯+ 3NH 4Cl ( aq )
(iv) At higher temp, it becomes reactive brown
EBD_7327
498 CHEMISTRY

ZnSO4(aq) + 2 NH4OH(aq) (v) With Nessler’s reagent :


¾¾® Zn(OH) 2 (s) ¯ + (NH 4 ) 2 SO 4 (aq) 2K2HgI4 + NH3 + 3KOH ® NH2 – HgO.HgI¯ + 7KI + 2H2O
white (red brown ppt.)
(iii) It acts as a Lewis base due to presence of lone pair of
Uses
e–1 s on N and combines with Lewis acids ex:
2+
(i) Due to lone pair of electrons it acts as a Lewis base.
Cu 2+ ( aq ) + 4NH 3 ( aq ) ƒ éëCu ( NH 3 )4 ùû ( aq ) (ii) Preparation of fertilizers.
( deep blue ) (iii) In making artificial silk.
Ag + ( aq ) + Cl - ( aq ) ¾¾
® AgCl ( s ) (iv) As a laboratory reagent.
white ppt (v) In refrigeration.
(vi) In making HNO3, (NH4)2CO3, Na2CO3 etc.
® éëAg ( NH 3 )2 ùû Cl ( aq )
AgCl ( s ) + 2NH 3 ( aq ) ¾¾
(vii) Liquid NH3 is a good solvent for the alkali and alkaline earth
(iv) Oxidation (reducing property) : metals. The solution is conducting and blue in color. The
(a) 3CuO + 2NH3 ¾¾ ® 3Cu + N2 + 3H2O electrical conductivity is due to ammoniated cations as well
as electrons.
Pt gauze
(b) 4NH3 + 5O2 ¾¾¾
800°C
¾® 4NO + 6H2O M + (x + y)NH3 ¾¾® [M (NH3)x]+ + [e (NH3)y]–
Oxides of nitrogen these are tabulated below :

Name of oxide Formula O.S. of N Common methods of preparation Physical appearance and chemical nature
D
(1) Dinitrogen oxide N2O +1 NH 4 NO3 ¾¾
® N 2 O + 2H 2 O colourless gas, neutral
(2) Nitrogen monoxide NO +2 2NaNO 2 + 2FeSO4 + 3H 2SO 4 ® colourless gas, neutral
Fe2 ( SO4 ) + 2NaHSO4 + 2H2O + 2NO
3
250K
(3) Dinitrogen trioxide N 2 O3 +3 2NO + N 2O 4 ¾¾¾® 2N 2 O3 blue solid, acidic

(4) Nitrogen dioxide NO 2 +4 2Pb ( NO 3 ) 2 ® 4NO 2 + 2PbO + O2 brown gas, acidic


cool
(5) Dinitrogen tetraoxide N2O4 +4 2NO 2 ƒ N 2 O 4 colourless solid, liquid, acidic
heat
(6) Dinitrogen pentaoxide N 2 O5 +5 4HNO3 + P4O10 ® 4HPO3 + 2N 2 O5 colourless solid, acidic

Nitric acid, HNO3. (ii) In aqueous solution, it behaves as a strong acid.


Preparation: HNO3 ( aq ) + H 2 O ( l ) ¾¾
® H3O+ ( aq ) + NO3- ( aq )
(i) Lab method:
(iii) It is a strong oxidising agent.
D
NaNO3 + H 2 SO 4 ¾¾
® NaHSO 4 + HNO3
® 3Cu ( NO3 ) 2 + 2NO + 4H 2 O
3Cu + 8HNO3 (dilute) ¾¾
(ii) Ostwald’s process:

4NH3 ( g ) + 5O 2 ( g ) ¾¾¾¾¾¾
® 4NO ( g ) + 6H 2 O ( g )
Pt /Rh catalyst ® Cu ( NO3 )2 + 2NO 2 + 2H 2O
Cu + 4HNO3 ( conc ) ¾¾
500 K, 9 bar
® 4Zn ( NO 3 ) 2 + 5H 2 O + N 2 O
4Zn + 10 HNO 3 ( dilute ) ¾¾
2NO ( g ) + O2 ( g ) ƒ 2NO2 ( g )
® Zn ( NO3 ) 2 + 2H 2 O + 2NO 2
Zn + 4 HNO3 ( conc) ¾¾
3NO2 ( g ) + H 2 O ( l ) ¾¾
® 2HNO3( aq ) + NO ( g )
(iv) It oxidises non-metals and their compounds. ex:
Properties:
(i) In gaseous state, it exists as a planar molecule as shown I 2 + 10HNO3 ¾¾
® 2HIO 3 + 10NO 2 + 4H 2 O
below:
C + 4HNO3 ¾¾
® H 2 CO3 + H 2 O + 4NO 2
O
H 102°
O N 130° S8 + 48HNO3 ( conc ) ¾¾
® 8H 2SO4 + 48NO 2 + 16H 2O
O P4 + 20HNO3 ¾¾
® 4H3PO4 + 20NO 2 + 4H 2 O
It is a resonance hybrid of the following structures : ( conc)
–. . (v) Semi metals or metalloids such as As and Sb are oxidised to
O O
..

..

.. ..
+ + corresponding acids with conc. HNO3.
HO N HO N As + 5 HNO3 ¾¾® H3AsO4 + 5NO2 + H2O
. .– ..
O O Sb + 5 HNO3 ¾¾® H3SbO4 + 5NO2 + H2O
..
..
..
The p-Block Elements (Group 15, 16, 17 and 18) 499

Brown ring test: It is for the detection of NO3- ions in a salt. (iv) Reaction with aqueous metallic salts:
In this test, dilute FeSO4 is added to an aqueous solution of
3CuSO 4 + 2PH3 ¾¾
® Cu 3 P2 + 3H 2SO 4
NO3- ions and then conc H2SO4 is added along the sides of Copper
phosphide
test-tube. A brown ring at the interface indicates NO3- ion.
3 HgCl 2 + 2PH 3 ¾¾ ® Hg 3 P2 + 6HCl
NO3- + 3Fe2 + + 4H + ¾¾
® NO + 3Fe3+ + 2H 2 O
(v) It is weakly basic and gives phosphonium compounds with
2+ 2+
é Fe ( H 2O ) ù ® éë Fe ( H 2O)5 ( NO) ùû
+ NO ¾¾ + H2O acids. ex:
ë 6û
brown PH 3 + HBr ¾¾
® PH 4 Br
ALLOTROPES OF PHOSPHORUS Uses :
Phosphorous exists in many allotropic forms, important ones being (i) It is used in smoke screens.
white, red and black. (ii) Containers containing calcium carbide and calcium phosphide
(i) White P: are pierced and thrown in the sea when the gases evolved
It is translucent waxy solid, poisonous, insoluble in H2O but burn and serve as a signal. These are called Holme's signals.
soluble in CS2 and shows chemiluminescence (glowing in Phosphorus Halides
dark). In white phosphorus P4 molecules are held by weak
P forms two types of halides, PX3 and PX5.
Van der Waal’s forces. Therefore it is low melting and low
boiling in nature. It readily catches fire in air give dense fumes (X = F, Cl, Br, I)
of P4O10. Phosphorus Trichloride (PCl3)
Reaction with aqueous alkalies: Preparation:
P4 + 3NaOH + 3H 2 O ¾¾
® PH 3 + 3NaH 2 PO 2
(white) (i) P4 + 6Cl 2 ¾¾
® 4PCl 3
White phosphorus acts as a strong reducing agent. It reduces (white)
HNO3 to NO2, H2SO4 to SO2 and CuSO4 to Cu and itself
(ii) P4 + 8 SOCl2 ¾¾
® 4PCl3 + 4SO 2 + 2S2 Cl2
oxidised to H3PO4.
(ii) Red Phosphorus Properties:
(i) Reaction with H2O:
It is obtained by heating white phosphorus in an inert
atmosphere for several days. PCl3 + 3H 2 O ¾¾ ® H 3 PO 3 + 3HCl
(a) It possesses iron grey lustre. (ii) Reaction with organic compounds:
(b) It is odourless, non-poisonous, insoluble in H2O as well
as CS2. PCl3 + 3CH 3 COOH ¾¾
® 3CH 3 COCl + H 3 PO3
(c) Chemically, it is less reactive than white phosphorus. PCl 3 + 3CH 3CH 2 OH ¾¾ ® 3C 2 H 5 Cl + H 3 PO3
(d) Red phosphorus does not react with caustic alkalis. (iii) It has a pyramidal shape in which P is sp3-hybridised
(e) Red phosphorus react at ordinary temp. and on heating
..
respectively with chlorine giving PCl3 and PCl5.
(iii) Black Phosphorus P 204 pm
It has two forms : a - and b - a-black phosphorus is formed
Cl
Cl
by heating red phosphorus. It has opaque monoclinic or Cl
100.4°
rhombohedral crystals. It does not oxidise in air.
b-black phosphorus is prepared by heating white phosphorus Phosphours Pentachloride (PCl5)
under high pressure. It does not burn in air upto 673 K. Preparation:

SOME IMPORTANT COMPOUND OF PHOSPHORUS (i) P4 + 10Cl 2 ¾¾


® 4PCl5
Phosphine [PH3] ( white ) ( excess)
Preparation (ii) P4 + 10SO 2 Cl 2 ¾¾ ® 4PCl5 + 10SO 2
(i) Ca 3P2 + 6H 2O ¾¾
® 3Ca(OH)2 + 2PH3 Properties:
Ca 3P2 + 6HCl ¾¾
® 3CaCl 2 + 2PH 3 (i) In moist air, it hydrolyses to POCl3 and finally gets converted
(ii) P4 + 3NaOH + 3H 2O ¾¾ ® PH3 + 3NaH 2 PO 2 to phosphoric acid.
Properties (Physical and Chemical) : PCl5 + H 2 O ¾¾
® POCl3 + 2HCl
(i) It is colourless gas and is highly poisonous.
(ii) It explodes in contact with oxidising agents like HNO3, Cl2 PCl5 + 3H 2 O ¾¾ ® H 3 PO 4 + 3HCl
and Br2 vapours. (ii) On heating, it sublimes and decomposes on stronger heating.
(iii) In water, it decomposes in presence of light to give red P and D
H2. PCl5 ¾¾® PCl3 + Cl 2
EBD_7327
500 CHEMISTRY

(iii) It reacts with organic compounds The P–H bonds are not ionisable to give H+ and do not play
C 2 H 5 OH + PCl 5 ¾ ¾
® C 2 H 5 Cl + POCl 3 + HCl any role in basicity. The H–atoms attached to O in P–OH are
ionisable and cause basicity. Thus, H3PO4 is tribasic (three-
CH 3 COOH + PCl5 ¾ ¾
® CH 3 COCl + POCl3 + HCl
OH bonds) H3PO3 is dibasic (2 – OH bonds) while H3PO2 is
(iv) Finely divided metals on heating with PCl5 give corresponding monobasic (one – OH bond)
chlorides.
· 2Ag + PCl5 ¾¾ ® 2AgCl + PCl3 Oxides of Phosphorus
(i) Phosphorus trioxide (ii) Phosphorus pentoxide
Sn + PCl5 ¾¾ ® SnCl4 + 2 PCl3
(P4O6) (P4O10)
(v) In gaseous and liquid phases, it has
trigonal bipyramidal Cl
Cl O
structure as shown :
1 43
The two axial bonds are longer than P P P pm
equitorial bonds. This is because axial bond Cl O O O 102° O
Cl 127°
pairs suffer more repulsion than equatorial O O 123°
Cl
bond pairs. 160 pm

(vi) In solid state, it exists as an ionic solid, [ PCl 4 ]+ [ PCl6 ]- in P P P P


- O O O O
which [ PCl4 ] is tetrahedral and [ PCl6 ] is octahedral.
+ 100°
O P O O P O
Oxoacids of Phosphorus 166 pm

Phosphorus forms two types of oxoacids namely phosphorus and O


phosphoric acids. In oxoacids, P is tetrahedrally surrounded by
other atoms. These acids contain one P = O and atleast one P–OH GROUP 16 ELEMENTS
bond. The acids in + 3 oxidation state of P, ex: H4P2O6, H3 PO2, It includes
H3PO3, tend to disproportionate. ex:
D Oxygen (O)
4 H 3 PO3 ¾¾® 3H3 PO 4 + PH 3 Sulphur (S) Non-metals
The hydrogen atoms in P–OH bonds are ionisable and are therefore
acidic. However these are not ionisable in P–H bonds and are Selenium (Se)
Metalloids
responsible for the reducing properties of acids.
Ex: H3PO2 reduces AgNO3 to Ag (it has two P–H bonds) Polonium (Po) Metal
They are also called chalcogens (ore forming elements) because
4 AgNO 3 + 2H 2 O + H 3 PO 2 ¾¾
® 4 Ag + 4HNO 3 + H 3 PO 4
these are involved in chemical formulas of the ores such as oxides,
Structures of some oxoacids are: sulphides, tellurides etc.
O O O Atomic Properties
(i) Electronic Configuration:
P P P Their general electronic configuration is ns2 np4.
OH OH OH (ii) Atomic and Ionic radii.
HO OH H OH H H Atomic and ionic radii increases down the group. The size of
H3PO4 H3PO3 H3PO2 O is exceptionally small.
(iii) Ionisation enthalpy :
O Down the group, IE decreases due to increase in size.
O O O O OH IE values of group 16 elements are much lower than those of
P
P P P HO P OH group 15 elements. This is because group 15 elements have
OH HO O OH HO O OH extra stable half-filled p-orbital electronic configuration
OH OH O (iv) Electron gain enthalpy:
Pyrophosphoric Metaphosphoric Hypophosphoric
acid acid They have high tendency to accept two e–1s to become stable
acid
and hence have large negative electron gain enthalpies.
Electron gain enthalpy of oxygen is least negative in this
O O O O O
O group due to its small size resulting in large e–– e– repulsions
P P P P P in 2p-subshell due to which the incoming e–1 s are not
HO OH O O O O accepted with ease.
O O HO HO OH (v) Electronegativity :
P
Polymetaphosphoric (i) Electronegativity value decreases down the group with
O OH acid (HPO3)n increase in size. Oxygen is the second most
(HPO3)3 electronegative element in the periodic table. This is due
Cyclo meta phosphoric to its very small size and urge to take up two electrons to
acid
acquire a ns2 configuration.
The p-Block Elements (Group 15, 16, 17 and 18) 501

Physical Properties : All elements except Se form dichlorides and dibromides which
(i) O exists as a diatomic molecule while other elements exist as have sp3 hybridisation and tetrahedral structure. Monohalides
octaatomic solids. are dimeric in nature and undergo disproportionation.
(ii) O forms pp–pp double bonds while other elements do not. ExamplesareS2F2, S2Cl2, S2Br2, Se2Cl2, Se2Br2.
(iii) S has stronger tendency for catenation than O (due to 2Se 2 Cl2 ¾¾
® SeCl 4 + 3Se
stronger S–S bonds compared to O–O bonds)
(iv) All elements of this group show allotropy Anomalous Behaviour of Oxygen
(v) M.pts. and B.pts. increases down the group with increase in The anomalous behaviour of oxygen is due to:
atomic number. (i) its small size
(ii) high electronegativity
Chemical Properties (iii) absence of d-orbitals (which limits its covalency to 4)
(i) Oxidation states:
OXYGEN AND SOME OF ITS IMPORTANT
They show both negative and positive oxidation states. COMPOUNDS
The stability of – 2 O.S. decreases down the group. Due to
high electronegativity of O it shows only negative O.S. of Dioxygen, O2
–2 except in OF2 and O2F2 where it shows + 2 and +1 oxidation Preparation :
states respectively. In peroxides like H2O2, O has – 1 oxidation (i) By heating oxygen containing salts like chlorates, nitrates
states and in O2, it has zero oxidation states. Other elements and permanganates.
show +2, +4, +6 oxidation states. S, Se, Te show +4 oxidation
D
2KMnO4 ¾¾® K2MnO4 + MnO2 + O2
states with O and +6 with F. Due to inert pair effect, stability
of + 6 oxidation state decreases down the group while that of D
+ 4 oxidation states increases. Bonding in higher oxidation 2KClO3 ¾¾® 2KCl + 3O2
states, i.e. +4 and +6, is primarily covalent. MnO2
(ii) Reactivity with hydrogen : (ii) By thermal decomposition of oxides of metals low in the
They form hydrides of the type H2E. Bond angle decreases electrochemical series and higher oxides of some metals.
from H2O to H2Te. This is because as electronegativity 2Ag 2 O ( s ) ¾¾
® 4Ag ( s ) + O 2 ( g )
decreases from O to Te, electron density also decreases i.e.
electron pair repulsion in electron pairs around the central 2PbO 2 ( s ) ¾¾
® 2PbO ( s ) + O 2 ( g )
metal atom decreases bond angle also decreases.
Their acidic character increases from H2O to H2Te. This is (iii) By catalytic decomposition of H2O2
due to decrease in bond dissociation energy down the group. MnO
2H 2 O2 ( aq ) ¾¾¾¾ 2 ( ) 2( )
2 ® 2H O l + O g
For the same reason, thermal stability of hydrides decreases
from H2O to H2Po. All hydrides except H2O possess reducing (iv) On large scale, electrolysis of water leads to release of
property and this character increases from H2S to H2Te. This hydrogen at cathode and oxygen at anode.
is due to decrease in bond dissociation energy due to increase (v) By liquefaction of air followed by its fractional distillation
in bond length value. where N2 distills off at 77K and O2 remains as liquid.
(iii) Reactivity with oxygen: Properties
Ozone is considered as oxide of oxygen O. Oxides of other (i) It has three stable isotopes, 16O, 17O and 18O
elements are as follows (ii) O2 is paramagnetic inspite of having even number of e–1s.
This is due to unpaired e–1s in anti-bonding orbitals.
Element Mono Oxide Dioxide Tri Oxide (iii) The reaction of O2 with other elements is highly exothermic
S SO SO2 SO3 which sustains the reaction. As the bond dissociation
Se - SeO2 SeO3 enthalpy of O = O is high, some external heating is required
Te TeO TeO2 TeO3 to initiate the reaction.
Po PoO PoO2 - (iv) Reaction with metals:
O3 and SO2 are gases while SeO3 is solid. Reducing property 2Ca + O 2 ¾¾
® 2CaO
of dioxides decreases from SO2 to TeO2, SO2 is reducing
while TeO2 is an oxidising agent. 4Al + 3O 2 ¾¾ ® 2Al 2O3
(iv) Reactivity towards halogens: (v) Reaction with non-metals:
They form a large number of halides of the type EX6, EX4,
EX 2 . The stability of halides decreases in the order P4 + 5O 2 ¾¾
® P4 O10

F- > Cl- > Br - > I- . C + O2 ¾¾ ® CO2


Among hexahalides, hexafluorides are the only stable halides (vi) Reaction with metal sulphides:
All hexaflourides are gaseous in nature and have octahedral
structure. SF6 is exceptionally stable due to steric reasons. 2ZnS + 3O2 ¾¾ ® 2ZnO + 2SO2
Among tetrafluorides, SF4 is a gas, SeF4 is a liquid and TeF4 (vii) Reaction with hydrocarbons:
a solid. They have sp3d hybridisation and see-saw geometry
(with a lone pair at one of equatorial positions). CH 4 + 2O 2 ¾¾
® CO 2 + 2H 2 O
EBD_7327
502 CHEMISTRY

Uses: Properties :
(i) In respiration and combustion processes. (i) It is a pale blue gas, dark blue liquid and violet-black solid.
(ii) In oxyacetylene welding and manufacture of steel. (ii) In small concentrations, it is harmless. If concentration rises
(iii) As oxygen cylinders in hospitals, high altitude flying above 100 ppm, breathing becomes uncomfortable resulting
and in mountaineering.
in headache and nausea.
Oxides (iii) It is thermodynamically unstable with w.r.t O 2 (as its
A binary compound of oxygen with other element is called oxide. decomposition into O2 results in liberation of heat (i.e.,
Simple oxides negative DH) and increase in entropy (i.e. +DS), resulting in
Simple oxides are classified on the basis of their acidic, basic or negative DG (i.e. spontaneous process)
amphoteric character. An acidic oxide combines with H2O to give (iv) O3 is diamagnetic while O2 is paramagnetic.
an acid.
(v) It is powerful oxidising agent as it readily releases atoms of
Example : SO2, Cl2O7, CO2, N2O5.
nascent oxygen i.e., O3 ¾¾ ® O2 + O
CO2 + H2O ¾¾® H2CO3
Ex : PbS ( s ) + 4O3 ( g ) ¾¾
® PbSO 4 ( s ) + 4O 2 ( g )
SO 2 + H 2 O ¾¾® H 2SO3
These acidic oxides react with bases to form salts and water. 2H g(l) + O 3 (g) ¾¾® H g 2 O(s) + O 2 (g)
Non-metal oxides and oxides of some metals in high oxidation
2I - ( aq ) + H 2 O ( l ) + O 3 ( g ) ¾¾
® 2OH - ( aq ) + I 2 ( s ) + O 2 ( g )
state (ex: Mn2O7, CrO3, V2O5) have acidic character. A basic oxide
combines with H2O to give base. For ex: Na2O, CaO, BaO (vi) Reducing action
Na2O + H2O ¾¾® 2NaOH
H 2 O 2 + O 3 ® H 2 O + 2O 2
® Ca ( OH )2
CaO + H 2 O ¾¾
Basic oxides form salts and water with acids Metallic oxides are BaO 2 + O 3 ® BaO + 2O 2
basic NaOCl + O 3 ® NaCl + 2O 2
Some oxides show characteristics of acidic as well as basic oxides.
They are called amphoteric oxides. For ex: Al2O3,ZnO, PbO and (vii) Addition reactions
SnO. It reacts with acids as well as alkalies.
O
Al 2O3 ( s ) + 6HCl ( aq ) + 9H 2 O ( l ) ¾¾
®
3+ CH 2 = CH 2 + O3 ® H 2C CH2
2 é Al ( H 2 O)6 ù
ë û ( aq ) + 6Cl- ( aq )
O O
Al 2O3 ( s ) + 6NaOH ( aq ) + 3H 2O ( l ) ¾¾
® Ozonide

2Na 3 ëé Al ( OH )6 ûù ( aq ) Note :
Some oxides are neither acidic nor basic. They are called neutral (i) Nitrogen oxides react with O3 rapidly and result in depletion
oxides. For ex: CO, NO and N2O. of O3 in upper atmosphere.
Polyoxides - These contain more oxygen than normal oxidation
number of M and M – O and O – O bonds. They are further NO ( g ) + O3 ( g ) ¾¾
® NO2 ( g ) + O2 ( g )
classified as
(ii) O3 can be quantitatively estimated using KI and Na2S2O3
(i) Peroxides - They contain O22- ion, produce hydrogen peroxide solutions as follows:
with dil. acids and O2 with concentrated acids eg BaO2, Na2O2 When O3 is treated with excess of KI solution buffered with
borate buffer (pH 9.2), I2 is liberated quantitatively.
(ii) Super oxides - They contain O 2– ion. With water they give
hydrogen peroxide and oxygen 2I - ( aq ) + H 2 O ( l ) + O3 ( g ) ¾¾
® 2OH - ( aq ) + I2 ( s ) + O 2 ( g )
(iii) Dioxides - They give chlorine with conc HCl and oxygen
with Conc H2SO4 eg MnO2, PbO2 etc. The I2 thus liberated in titrated against a standard solution of
(iv) Suboxides - They contain lower percentage of oxygen eg Na2S2O3 using starch as an indicator.
N2O, C3O2 2Na2S2O3 + I2 ¾¾® Na2S4O6+2NaI
They have M – M and M – O bonds.
For example : O = C = C = C = O (carbon suboxide) SULPHUR-ALLOTROPIC FORMS
(v) Mixed oxides - Formed by the combination of two simple
The yellow rhombic (a – sulphur) and monoclinic (b– sulphur)
oxides eg Red lead, Pb3O4 (PbO2.2PbO), Fe3O4 (FeO+Fe2O3)
allotropes of sulphur are the most important. At room temperature,
Ozone, O3
the stable form is rhombic sulphur which when heated above 369
Preparation: K, transforms to monoclinic sulphur. Conversely, a-sulphur is
When a slow dry stream of oxygen is passed through a silent stable below 369 K and transforms into b-sulphur above this. At
electrical discharge, conversion of O2 to O3 occurs. 369 K, both the forms are stable. This temperature is called
3O 2 ¾¾® 2O3 , D Hº = + 142kJmol –1 transition temperature.
The p-Block Elements (Group 15, 16, 17 and 18) 503

SULPHUR AND SOME OF ITS IMPORTANT


O O
COMPOUNDS O O
Sulphur S
S S S
Prepration : O O
O O O O O
(i) Main source of sulphur are : Sicily and Louisiana the OH
OH OH OH
processes employed are called :
Peroxodisulphuric acid Pyrosulphuric acid (Oleum)
Sicilian process and Louisiana (or Frasch) process
(H2S 2O8) (H2S2O7)
(ii) Sulphur is also obtained from alkali wastes, spent oxides of
coal gas and iron pyrites :
(iii) Flowers of sulphur : In the purification process, vapours of S O
boiling sulphur (444ºC) are condensed on the cold walls of P
S HO - S - S - OH
the vessel as a light yellow powder called flowers of sulphur.
P
Both the forms have S8 molecules. The S8 ring in both the HO O
forms is puckered and has a crown shape. In vapour state, S OH O
partly exist as S2 molecule which has two unpaired e–1s in the Thiosulphuric acid Peroxomonosulphuric acid
antibonding p* orbitals like O2 , and hence, exhibits (H2S2O3) or caro’s acid (H2S2O5)
paramagnetism. O O
Sulphur Dioxide, SO2
Preparation S S

(i) S ( s ) + O2 ( g ) ¾¾
® SO 2 ( g ) O O
OH OH
(ii) Lab method : By heating Cu with conc H2SO4 Dithionic acid
Cu + 2H 2SO 4 ® CuSO 4 + SO2 + 2H 2O (H2S2O6)
(iii) From iron pyrite : Sulphuric acid, H2SO4
4FeS2 ( s ) + 11O 2 ( g ) ¾¾
® 2Fe 2 O3 ( s ) + 8SO 2 ( g ) Manufacture:
(Industrial method) (i) By contact process
It involves the following steps:
Properties
(a) Production of SO2 by burning sulphur or roasting iron pyrites.
(i) As oxidising agent
2H 2S + SO 2 ® 2H 2 O + 3S ¯ S8 + 8O 2 ¾¾
® 8SO2

SO 2 + 2Mg ( burning ) ® 2MgO + S ¯ 4FeS2 + 11O2 ¾¾ ® 2Fe 2O3 + 8SO2


(ii) Burning of potassium - Potassium burns in SO2 giving (b) Catalytic oxidation of SO2 by air to give SO3.
sulphate and thiosulphate
2SO2 ( g ) + O2 ( g ) 2SO3 ( g ) DH = – 196.6 kJ mol–1
4K + 3SO 2 ® K 2SO3 + K 2S2O3
(iii) Bleaching action - Its bleaching action is due to reduction According to Le Chatelier’s principle, the yield of SO2 is
increased with high P, low T and use of catalyst like Pt or
SO 2 + 2H 2O ® H 2SO 4 + 2H V2O5.
(iv) When moist, it behaves as a reducing agent. For ex: (c) Absorption of SO3 in 98% H2SO4 to form oleum.
(a) 2Fe3+ + SO 2 + 2H 2 O ¾¾
® 2Fe2 + + SO 24- + 4H + ® H 2S2O7 ( oleum )
SO3 + H 2SO 4 ¾¾
Dilution of oleum give H2SO4 of desired concentration.
(b) 5SO 2 + 2MnO 4- + 2H 2 O ¾¾
® 5SO42 - + 4H + + 2Mn 2+
Oxoacids of Sulphur H 2S2O7 + H 2O ¾¾
® 2H 2SO 4
S forms a number of oxoacids such as H2SO3, H2S2O3, H2S2O4, Lead chamber process : Here oxidation of SO2 is affected
H2S2O5, H2SxO6 (x = 2 to 5), H2SO4, H2S2O7, H2SO5, H2S2O8. catalytically by means of oxides of nitrogen in the presence of
Structure of some important oxoacids are: water.
2SO2 + O2(air) + 2H2O + [NO] (catalyst) ® 2H2SO4 + [NO]
O
· · Mechanism may be described as :
S 2NO + O2 ® 2NO2
S O NO2 + SO2 + H2O ® H2SO4 + NO
HO O HO
OH The unreacted gases (NO, NO2, O2 and N2) are absorbed in H2SO4
OH
Sulphurous acid Sulphuric acid and nitrosyl sulphuric acid. The product on decomposition gives
(H2SO3) (H2SO4) H2SO4.
EBD_7327
504 CHEMISTRY

2H2SO4 + NO + NO2 ® 2NO+H SO4– + H2O (iii) Ionisation enthalpy


They have very high IE as they have little tendency to loose.
2NOHSO4– + H2O ® 2H2SO4 + NO­ + NO2­
electron(s) IE decreases down the group with increase in
Physical properties : atomic size.
(i) It is colourless, dense, oily liquid with a specific gravity of (iv) Electron gain enthalpy:
1.84. As they need only one e– to become stable, halogens show
(ii) It dissolves in H2O with the evolution of lot of heat. a high tendency to accept e–1s and hence they have large
Chemical properties : negative values of electron gain enthalpy.
The chemical reactions of H2SO4 are due to its (a) low volatility (b) Down the group, electron gain enthalpy becomes less
strong acidic character (c) strong affinity for water and (d) ability negative. However, electron gain enthalpy of F is less than
to act as an oxidising agent. that of Cl. This is due to small size of F atom resulting in
(i) In aqueous solution, H2SO4 ionises in two steps: strong interelectronic repulsion in the relatively small 2p-
orbitals of F.
® H 3O + ( aq ) + HSO 4- ( aq ) ;
H 2 SO 4 ( aq ) + H 2 O ( l ) ¾¾ (v) Electronegativity
They have high electronegativity. F is the most
K a = 1 ´ 103 electronegative element in the periodic table.
1
Physical properties
® H 3O + ( aq ) + SO 2–
HSO 4– ( aq ) + H 2 O ( l ) ¾¾ 4 ( aq ) ; (i) F2 and Cl2 are gases, Br2 is a liquid and I2 is a solid.
(ii) All the halogens are coloured. F2 is yellow, Cl2 greenish
K a = 1.2 ´ 10-2 yellow, Br2 is red and I2 is violet. The colour arises due to
2
Large value of Ka shows that H2SO4 is a strong acid. excitation of outer e–1s to higher energy levels by absorbing
(ii) 2MX + H 2SO 4 ¾¾ ® 2HX + M 2SO 4 (X = F, Cl, NO3) radiations in visible region.
(iii) In halogen molecules, the bond dissociation enthalpies are
(M = metal) expected to decrease with the increase in bond length values.
(iii) It is a strong dehydrating agent.Ex. (iv) F2 has smaller bond dissociation enthalpy as compared to
Conc.H SO
C12 H 22 O11 ¾¾¾¾¾¾
2 4 ® 12C + 11H O Cl2. This is due to large e– – e– repulsion among the lone
2
pairs in F2 molecule.
Conc.H SO
HCOOH ¾¾¾¾¾¾
2 4 ® CO + H O
2 Chemical Properties
Conc.H SO (i) Oxidation states:
C 2 H 5 OH ¾¾¾¾¾¾ 2 ® C2 H 4 + H 2 O
All halogens exhibit –1 oxidation state Cl, Br and I exhibit +1,
(iv) It is a strong oxidising agent and oxidises both metals and +3, +5 and +7 oxidation state also. F exhibits only – 1 oxidation
non-metals state being the most electronegative element in the table.
Exs: Halogens are highly reactive and react with metals and non-
metals to form halides.
Cu + 2H 2SO 4 ( conc ) ¾¾
® CuSO 4 + SO 2 + 2H 2O
(ii) Oxidising nature :
Halogens are strong oxidising agents due to their tendency
3S + 2H 2SO 4 ( conc.) ¾¾
® 3SO2 + 2H 2O to accept e–1s. F2 is the strongest oxidising agent. In general,
a halogen oxidises halide ions of higher atomic number.
C + 2H 2SO4 ( conc.) ¾¾
® CO 2 + 2SO 2 + 2H 2O
F2 + 2X - ¾¾ ® 2F- + X 2 (X = Cl, Br or I)
(v) Displaces more volatile acids :
Cl 2 + 2X - ¾¾
® 2Cl - + X 2 (X = Br or I)
NaCl + H2SO4 ® NaHSO4 +HCl
NaNO3 + H2SO4 ® NaHSO4 + 2HNO3 Br2 + 2X - ¾¾ ® 2Br - + X 2 (X = I)
(iii) Reaction with water.
GROUP 17 ELEMENTS
® 4H + ( aq ) + 4F- ( aq ) + O2 ( g )
2F2 ( g ) + 2H 2O ( l ) ¾¾
It includes
Fluorine (F) X 2 ( g ) + H 2 O ( l ) ¾¾
® HX ( aq ) + HOX ( aq ) (X = Cl or Br)
Chlorine (Cl)
Bromine (Br) I 2 ( g ) + H 2 O ( l ) ¾¾
® No reaction
Iodine (I) Instead, I– ions can be oxidised by oxygen in acidic medium.
Astatine (At) 4I - ( aq ) + 4H + ( aq ) + O 2 ( g ) ¾¾
® 2I 2 ( s ) + 2H 2 O ( l )
They are collectively known as halogens.
(iv) Reaction with alkalies :
Atomic Properties Cl2, Br2 and I2 behave similarly when treated with alkali (They
(i) Electronic configuration undergo disproportionation reaction)
Their general electronic configuration is ns2 np5. (a) Cold and dilute alkali :
(ii) Atomic and Ionic radii: X2 + 2NaOH ® NaX + NaOX + H2O
They have the smallest atomic radii in their respective periods (b) Hot and concentrated alkali :
due to maximum effective nuclear charge. Down the group,
3X2 + 6NaOH ® 5NaX + NaXO3 + 3H2O
atomic and ionic radii increases
The p-Block Elements (Group 15, 16, 17 and 18) 505

F2 behaves differently with alkalies : (iii) 2KMnO 4 + 16HCl ¾¾ ® 2KCl + 2MnCl 2 + 8H 2 O + 5Cl 2
F2 + 2NaOH (dil) ® 2NaF + OF2 + H2O (iv) Deacon’s process - In this process HCl is oxidised by O2 in
2F2 + 4NaOH (conc) ® 4NaF + O2 + 2H2O presence of CuCl2 as catalyst at 400° C
(v) Reaction with hydrogen :
4HCl + O 2 ® 2Cl 2 + 2H 2 O
All halogens directly combine with hydrogen to form HX
known as hydracid. (v) Electrolytic process - By the electrolysis of brine solution
dark in Nelson cell
H2 + F2 ¾¾¾ ® 2HF
NaCl Na+ + Cl–
diffused sunlight
H2 + Cl2 ¾¾¾¾¾¾¾
® 2HCl
D 2Na + + 2e ® 2Na + 2H2O ® 2NaOH + H2 (at cathode)
H2 + Br2 ¾¾® 2HBr
Pt
H2 + I2 ¾¾® 2HI 2Cl – - 2e – ® 2Cl ® Cl 2 (at anode)
Affinity for hydrogen decreases from F to I. They dissolve in Properties:
water to form hydrohalic acids. The acid strength follows the (i) It is about 2-5 times heavier than air.
order: HF < HCl < HBr < HI. (ii) It is soluble in water.
This is because down the group, size of the halogen atom (iii) It reacts with metals and non-metals to form chlorides.
increases thus H–X bond length increases and hence bond
strength decreases. 2Al + 3Cl2 ¾¾
® 2AlCl3
The order of stability of halides is HF > HCl > HBr > HI. This
is because down the group, bond dissociation enthalpy 2Na + Cl 2 ¾¾
® 2NaCl
decreases.
(vi) Reactivity towards oxygen: 2Fe + 3Cl 2 ¾¾
® 2FeCl3
Halogens do not combine directly with oxygen hence they
are prepared by indirect methods. Fluorine forms two oxides, P4 + 6Cl 2 ¾¾
® 4PCl3
OF2 and O2F2 of which only OF2 is thermally stable. The
higher oxides are more stable than the lower ones. The S8 + 4Cl 2 ¾¾ ® 4S2Cl 2
decreasing order of stability of oxides formed by halogens is (iv) It reacts with compounds containing hydrogen to form HCl.
I > Cl > Br. Chlorine oxides, Cl2O, ClO2, Cl2O6 and Cl2O7 are
highly reactive oxidising agents and tend to explode. The H 2 + Cl 2 ¾¾
® 2HCl
bromine oxides, Br2O, BrO2, BrO3 are the least stable and
exist only at low temperatures. Iodine oxides, I2O4, I2O5 and H 2S + Cl2 ¾¾
® 2HCl + S
I2O7 are insoluble solids and decompose on heating.
C10 H16 + 8Cl 2 ¾¾ ®16HCl + 10C
(vii) Reactivity towards metals:
They react with metals to form metal halides. (v) Reaction with NH3:
When NH3 is in excess
Ex: Mg ( s ) + Br2 ( l ) ¾¾
® MgBr2 ( s )
Their ionic character decreases in the order: 8NH3 + 3Cl 2 ¾¾
® 6NH 4Cl + N 2
MF > MCl > MBr > MI ( excess )
The halides in higher oxidation state are more covalent than When Cl2 is in excess :
in lower oxidation state. For ex: SnCl4, PbCl4 and SbCl5 are
more covalent than SnCl2, PbCl2 and SbCl3. This is according NH3 + 3Cl 2 ¾¾
® NCl3 + 3HCl
to Fajan’s rule i.e. compounds with metal ion in higher ( excess )
oxidation state are expected to be more covalent than the
(vi) Reaction with NaOH:
compounds in lower oxidation state.
Anomalous Behaviour of Fluorine 2NaOH + Cl2 ¾¾
® NaCl + NaOCl + H2 O
It is due to its (i) small size (ii) highest electronegativity (iii) low ( cold and dilute)
F–F bond dissociation enthalpy, and (iv) non-availability of d-
orbitals in valence shell. Due to these reasons, most of the reactions 6NaOH + 3Cl2 ¾¾
® 5NaCl + NaClO3 + 3H 2O
of F are exothermic, it forms only one oxoacid. HF is a liquid due to ( hot and conc)
strong H-bonding while other hydrogen halides are gases. (vii) Reaction with slaked lime:
CHLORINE AND SOME OF ITS IMPORTANT 2Ca ( OH ) 2 + 2Cl 2 ¾¾
® Ca ( OCl ) 2 + CaCl2 + 2H 2O
COMPOUNDS
slaked lime bleaching
Chlorine, Cl2 powder
Preparation: (viii) Reaction with hydrocarbons:
(i) MnO2 + 4HCl ¾¾ ® MnCl2 + Cl2 + 2H 2O hn
CH 4 + Cl 2 ¾¾® CH3Cl + HCl
(ii) 4NaCl + MnO 2 + 4H 2SO 4 ¾¾
® room temperature
C2 H 4 + Cl 2 ¾¾¾¾¾¾¾¾
® C2 H 4Cl 2
MnCl 2 + 4NaHSO 4 + 2H 2O + Cl2
EBD_7327
506 CHEMISTRY

(ix) Oxidising action: Examples. (c) Chloric acid (d) Perchloric acid
2FeSO 4 + H 2SO 4 + Cl 2 ¾¾ ® Fe 2( SO 4 )3 + 2HCl (HClO3) (HClO4)
Na 2SO3 + Cl2 + H 2 O ¾¾ ® Na 2SO 4 + 2HCl H H
SO 2 + 2H 2 O + Cl2 ¾¾ ® H 2SO 4 + 2HCl O O
I 2 + 6H 2 O + 5Cl 2 ¾¾
® 2HIO3 + 10HCl
Cl Cl
(x) Bleaching action: Bleaching action is due to oxidation.

..
Cl 2 + H 2 O ¾¾ ® 2HCl + O O O O
O O
Coloured substance + O ¾¾ ® Colourless substance Perchloric acid (no lp on Cl)
The bleaching by Cl2 is permanent. The colour of bleached (i) Acid strength and oxidising power
articles cannot be restored. (a) Acid strength of hypohalous acids:
Notes : On standing, Cl2 water loses its yellow colour due to +1 +1 +1
formation of HCl and HOCl. HOCl so formed, gives nascent oxygen HOCl > HOBr > HOI
which is responsible for oxidising and bleaching properties of As electronegativity of halogens decreases electron
chlorine. density on O increases and thus, O–H bond polarity
decreases there by decreasing the acid strength.
Uses: (b) Oxidising power of hypohalous acids:
(i) For bleaching pulp, cotton and textiles. HOCl > HOBr > HOI
(ii) Extraction of Au and Pt. As size of halogen increases thermal stability of
(iii) Manufacture of dyes, drugs and organic compounds like O–X bond also increases thereby decreasing the
CCl4, CHCl3, DDT, refrigerants, etc. oxidising power.
(iv) Sterilising drinking water. (ii) (a) Acid strength of perhalic acids
Hydrogen Chloride, HCl HClO4 > HBrO4 > HIO4
As electronegativity of halogen decreases, tendency of
Preparation: XO3 to withdraw e–1 s of O–H bond towards itself
(ii) 420 K decreases thereby decreasing the acid strength.
NaCl + H 2SO4 ¾¾¾¾
® NaHSO4 + HCl (b) Oxidising power of perhalates
(ii) 823K
NaHSO4 + NaCl ¾¾¾® Na 2SO4 + HCl BrO -4 > IO 4- > ClO 4-
Properties : It is on the basis of their electrode potential values.
(i) It is soluble in water and ionises as: (iii) Acid strength of oxoacids of same halogen in different
oxidation states. It increases with increase in oxidation number
® H 3O + ( aq ) + Cl - ( aq ) Ka = 107
HCl ( g ) + H2 O ( l ) ¾¾ of X -atom i.e,
(ii) The aqueous solution of HCl is called hydrochloric acid. It is +7 +5 +3 +1
a strong acid as indicated by high value of Ka. HClO 4 > HClO 3 > HClO 2 > HClO .
(iii) Action with NH3: It is on the basis of the stability of the anion formed after the
NH3 + HCl ¾¾ ® NH 4Cl removal of proton. Greater the no. of O-atoms in the anion,
(iv) Aqua regia (3 parts conc. HCl + 1 part conc HNO3) is used to greater is the dispersal of negative charge and hence, greater
dissolve noble metals, ex: gold, platinum is the stability of the anion.
Au + 4H + + NO3- + 4Cl - ¾¾ ® AuCl4- + NO + 2H 2 O Interhalogen Compounds
They are compounds of two different halogens. They have
3Pt + 16H + + 4NO 3- + 18Cl - ¾¾
® 3PtCl62 - + 4NO + 8H 2 O compositionslikeXX¢, XX¢3, XX¢5 and XX¢7 where X is the halogen
(v) It decomposes salts of weaker acids, examples of larger size and X¢ is halogen of smaller size and X is more
Na 2 CO3 + 2HCl ¾¾ ® 2NaCl + H 2 O + CO 2 electropositive than X¢.
Fluorine being smallest and most electronegative element forms
NaHCO 3 + HCl ¾¾ ® NaCl + H 2 O + CO 2 the maximum number of interhalogen compounds.
Na 2SO3 + 2HCl ¾¾ ® 2NaCl + H 2 O + SO 2 Preparation:
Oxoacids of Halogens 437K
Cl 2 + F2 ¾¾¾¾
® 2ClF
F forms only one oxoacids, HOF (due to high electronegativity
and small size of F). Cl, Br and I form four series of oxoacids with 573K
Cl 2 + 3F2 ¾¾¾® 2ClF3
formula HOX, HOXO, HOXO2 and HOXO3. Structures of oxoacids
of chlorine are given below: I2 + Cl 2 ¾¾® 2ICl
(a) Hypochlorous acid (b) Chlorous acid I 2 + 3Cl2 ¾¾
® 2ICl3
(HOCl) (HClO2) Br2 + 3F2 ¾¾ ® 2BrF3
H H Br2 + 5F2 ¾¾ ® 2BrF5
O O Note: The less electronegative halogen is always written first.
Properties:
Cl Cl
(i) They are covalent and diamagnetic in nature.
.. .. (ii) Except ClF which is a gas, rest all are volatile solids or liquids.
..

..

..

O
The p-Block Elements (Group 15, 16, 17 and 18) 507

(iii) They are more reactive than halogens (except F2) because 873 K
the X–X¢ bond is weaker than X––X bond. Xe ( g ) + 2F2 ( g ) ¾¾¾¾
® XeF4 ( s )
1 : 5 7 bar
(iv) XX ¢ + H 2O ¾¾ ® HX ¢ + HOX
573 K
(v) The XX¢3 compounds have bent T-shape, XX¢5 compounds Xe ( g ) + 3F2 ( g ) ¾¾¾¾
® XeF6 ( s )
square pyramidal and XX¢7 has pentagonal bipyramidal 1 : 20 60 - 70 bar
structure. 143K
XeF4 + O2F2 ¾¾¾®XeF6 + O2
GROUP 18 ELEMENTS
It includes : Properties
Helium (He) (i) XeF2, XeF4 and XeF6 are colourless crystalline solids and
Neon (Ne) powerful fluorinating agents.
Argon (Ar) (ii) They are readily hydrolysed even with traces of water.
Krypton (Kr) Ex: 2XeF2 ( s ) + 2H 2O ( l ) ¾¾
®
Xenon (Xe)
Radon (Rn) 2Xe ( g ) + 4HF ( aq.) + O2 ( g )
Because of their inert nature, they are termed noble gases. (iii) Reactions with fluoride ion acceptors
+ -
Some atomic, physical and chemical properties are discussed as XeF2 + PF5 ¾¾ ® [ XeF ] [ PF6 ]
+ -
follows: XeF4 + SbF5 ¾¾ ® [ XeF3 ] [SbF6 ]
Atomic Properties (iv) Reactions with fluoride ion donors
-
(i) Electronic configuration XeF6 + MF ¾¾ ® M + [ XeF7 ] (M = Na, K, Rb or Cs)
Their general electronic configuration is ns2np6. He has
configuration of 1s2 Structures of fluorides of Xe:
(ii) Ionisation enthalpy XeF2 is linear, XeF4 is square planar and XeF6 is distorted
Due to stable electronic configuration, they have very high octahedral as shown below:
ionisation enthalphy. Ionisation enthalpy decreases down (a) XeF2 (b) XeF4 (c) XeF6
the group with increase in atomic size.
(iii) Atomic radii F
.. F
The atomic radii are the largest in their respective periods. .. F F F .. F
This is because they have van der waal’s radii while others Xe
..

have covalent radii and van der waal’s radii are larger than Xe Xe
..

covalent radii. Down the group, atomic radii increases due to F .. F F F


increase in number of shells. F F
(iv) Electron gain enthalpy Linear Square planar Distorted octahedral
Due to their stable electronic configurations, they have no
Xenon-Oxygen Compounds
tendency to accept e– and thus, they have large positive
values of electron gain enthalpy. Preparation :
Physical properties: (i) 6XeF4 + 12H 2O ¾¾® 4Xe + 2XeO3 + 24HF + 3O2
(i) Due to their stable outer electronic configuration noble gases (ii) XeF6 + 3H 2O ¾¾
® XeO3 + 6HF
are monoatomic (iii) XeF + H O ¾¾ ® XeOF4 + 2HF
(ii) They are colourless, odourless and tasteless. 6 2
(iv) XeF6 + 2H 2O ¾¾
® XeO2 F2 + 4HF
(iii) Due to weak dispersion forces holding the atoms together
noble gases have low M. pts. and B.pts. (He has the lowest Structures :
b.pt of any known substance)
O ..
(iv) Due to weak van der Waal’s forces holding the atoms together F F F
noble gases cannot be liquefied easily. Down the group, ease
of liquifaction increases. Xe Xe Xe = O
(v) They are sparingly soluble in water. F .. F O O
Chemical properties : O F
(i) Noble gases are least reactive due to: XeOF4 XeO3 XeOF2
(a) stable outer shell electronic configuration. Square pyramidal trigonal pyramidal T – shaped
(b) high ionisation energy and more positive electron gain (sp3d)
enthalpy. (sp3d2) (sp3)
(ii) The first compound of noble gases was prepared by Neil
Bartlett by mixing PtF6 and Xe. The compound formed was F
Xe+[PtF6]– (red colour). Xe
Xe
XENON-FLUORINE COMPOUNDS:
F
Preparation:
XeO2F2 XeO4
673 K
Xe ( g ) + F2 ( g ) ¾¾¾¾
® XeF2 ( s) See-Saw Tetrahedral
(excess) 1 bar (sp3d) (sp3)
508

CONCEPT MAP
CHEMISTRY

EBD_7327
The p-Block Elements (Group 15, 16, 17 and 18) 509

1. The gas used in air ships is : O O


(a) He (b) Ne (c) Ar (d) Xe
P P
2. When orthophosphoric acid is heated to 600°C, the product (c) HO (d) H
formed is OH OOH
OH OH
(a) PH3 (b) P2O5 (c) H3PO3 (d) HPO3 17. With which of the given pairs CO2 resembles?
3. Which one has the lowest boiling point ? (a) HgCl2, C2H2 (b) HgCl2, SnCl4
(a) NH3 (b) PH3 (c) AsH3 (d) SbH3 (c) C2H2, NO2 (d) N2O and NO2
4. Least stable oxide of chlorine is : 18. The number of hydrogen atom(s) attached to phosphorus
(a) Cl2O (b) ClO2 (c) Cl2O7 (d) ClO3 atom in hypophosphorous acid is
5. Which of the following statements is not correct for (a) three (b) one (c) two (d) zero
nitrogen? 18. Nitrogen dioxide cannot be obtained by heating :
(a) Its electronegativity is very high (a) KNO3 (b) Pb(NO3)2 (c) Cu(NO3)2(d) AgNO3
(b) d-orbitals are available for bonding 20. Which of the following has the highest pp – pp bonding
(c) It is a typical non-metal tendency ?
(d) Its molecular size is small (a) N (b) P (c) As (d) Sb
6. P2O5 is heated with water to give 21. Concentrated sulphuric acid is always diluted by adding
(a) hypophosphorous acid(b) phosphorous acid (a) water to acid
(c) hypophosphoric acid (d) orthophosphoric acid (b) alcohol to acid
7. The gas not absorbed by coconut charcoal is : (c) acid to water
(a) He (b) Ne (c) Ar (d) Kr (d) distilled water to acid
8. PCl3 reacts with water to form 22. Most acidic oxide among the following is –
(a) PH3 (b) H3PO4 and HCl (a) N2O5 (b) P2O5 (c) N2O4 (d) As 2 O 3
(c) POCl3 (d) H3PO4 23. Nitrogen reacts with calcium and carbon to give –
9. PH4I + NaOH forms (a) calcium nitride (b) calcium cyanide
(a) PH3 (b) NH3 (c) P4O6 (d) P4 O10 (c) calcium cyanamide (d) calcium nitrate
10. Pure nitrogen is prepared in the laboratory by heating a mixture 24. The most efficient agent for the absorption of SO3 is :
of (a) 80% H2SO4 (b) 98% H2SO4
(a) NH4OH + NaCl (b) NH4NO3 + NaCl (c) 50% H2SO4 (d) 20% H2S2O7
(c) NH4Cl + NaOH (d) NH4Cl + NaNO2.
25. NH 4 ClO4 + HNO 3 (dil.) ¾¾ ® HClO 4 + [X]
11. Sugarcane on reaction with nitric acid gives
D
(a) CO2 and SO2 (b) (COOH)2 [X] ¾¾
® Y(g)
(c) HCOOH (d) No reaction occurs [X] and [Y] are respectively –
12. The acid which cannot be kept in glass bottles is : (a) NH4NO3 & N2O (b) NH4NO2 & N2
(a) HCl (b) HBr (c) HF (d) HI (c) HNO4 & O2 (d) None of these
13. Nitrogen is relatively inactive element because 26. NH3 gas can be dried by ?
(a) its atom has a stable electronic configuration (a) conc. H2SO4 (b) P2O5
(b) it has low atomic radius (c) CaO (d) CaCl2
(c) its electronegativity is fairly high
(d) dissociation energy of its molecule is fairly high 28. In Haber’s process for the manufacture of NH 3 :
14. H3PO2 is the molecular formula of an acid of phosphorus. (a) finely divided nickel is used as a catalyst
Its name and basicity respectively are (b) finely divided iron is used as a catalyst
(a) phosphorus acid and two (c) finely divided molybdenum is used as a catalyst
(b) hypophosphorous acid and two (d) no catalyst is necessary
(c) hypophosphorous acid and one 29. Chlorine upon reaction with NaOH in cold yields:
(d) hypophosphoric acid and two (a) NaCl, NaClO, H 2 O (b) NaCl, NaClO3 , H 2 O
15. As compared to nitrogen, oxygen is
(c) NaClO, NaClO3 , H 2 O (d) NaCl, H 2 O
(a) less electronegative and less reactive
(b) more electronegative and less reactive 30. Ammonia on reaction with hypochlorite anion can form :
(c) more electronegative and more reactive (a) NO (b) N2H4
(d) less electronegative and more reactive (c) NH4Cl (d) Both (b) and (c)
16. The structural formula of hypophosphorous acid is 30. How many bonding electron pairs are there in white
phosphorus ?
O O (a) 2 (b) 4 (c) 3 (d) 5
P P 31. Nitrogen is obtained when NaNO2 reacts with :
(a) H (b) H (a) NH4Cl (b) NH4NO3
OH OH
H OH (c) (NH4)2CO3 (d) NH4OH
EBD_7327
510 CHEMISTRY

32. The sides of safety matches contains : 47. In the manufacture of bromine from sea water, the mother
(a) red phosphorous + sand powder liquor containing bromides is treated with
(b) P4S3 (a) CO2 (b) I2 (c) Cl2 (d) H2O
(c) Ca3(PO4) + glass pieces 48. Oxidation of thiosulphate by iodine gives
(d) KClO3, KNO3, sulphur + antimony (a) tetrathionate ion (b) sulphide ion
33. One mole of magnesium nitride on reaction with an excess of (c) sulphate ion (d) sulphite ion
water gives 49. When PbO2 reacts with conc. HNO3 the gas evolved is
(a) one mole of NH3 (b) two moles of NH3 (a) NO2 (b) O2 (c) N2 (d) N2O
(c) one mole of HNO3 (d) two moles of HNO3 50. The compound which gives off oxygen on moderate heating
34. The bonds present in N2O5 are : is :
(a) only ionic (b) covalent and coordinate (a) cupric oxide (b) mercuric oxide
(c) only covalent (d) covalent and ionic (c) zinc oxide (d) aluminium oxide
35. Which of the following oxides of nitrogen is a coloured gas? 51. The oxide which form dimer is :
(a) N2O (b) NO (c) N2O5 (d) NO2 (a) N2O5 (b) N2O (c) NO2 (d) N2O3
36. One mole of calcium phosphide on reaction with excess water 52. On electrolysis of dilute sulphuric acid using platinum
gives electrodes, the product obtained at the anode will be
(a) one mole of phosphine (a) hydrogen (b) oxygen
(b) two moles of phosphoric acid (c) hydrogen sulphide (d) sulphur dioxide.
(c) two moles of phosphine 53. Which compound is used in photography?
(d) one mole of phosphorus pentoxide (a) Na2SO5 (b) Na2S2O8
37. Which of the following is not a chalcogen : (c) Na2S2O6 (d) Na2S2O3
(a) Se (b) O (c) S (d) Na 54. Which among the following is strongest acid?
38. The number of P – O – P bonds in cyclic metaphosphoric (a) H(ClO)O2 (b) H(ClO)O3
(c) H(ClO)O (d) H(ClO)
acid is
55. Sodium thiosulphate is a
(a) zero (b) two (c) three (d) four
(a) reducing agent (b) oxidising agent
39. Nitrogen is obtained by the thermal decomposition of :
(c) complexing agent (d) bleaching agent
(a) NH4NO2 (b) NH4NO3
56. A gas that cannot be collected over water is :
(c) AgNO3 (d) Pb(NO3)2
(a) N2 (b) O2 (c) SO2 (d) PH3
40. The gases respectively absorbed by alkaline pyrogallol and
57. Sodium thiosulphate is prepared by
oil of cinnamon are
(a) reducing Na2SO4 solution with H2S
(a) O3 and CH4 (b) O2 and O3
(b) boiling Na2SO3 solution with S in alkaline medium
(c) SO2 and CH4 (d) N2O and O3. (c) neutralising H2S2O3 solution with NaOH
41. It is possible to obtain oxygen from air by fractional (d) boiling Na2SO3 solution with S in acidic medium
distillation because 58. Which of the following is not oxidized by O3 ?
(a) oxygen is in a different group of the periodic table from (a) KI (b) FeSO4 (c) KMnO4 (d) K2MnO4
nitrogen 59. Laughing gas is prepared by heating :
(b) oxygen is more reactive than nitrogen (a) NH4Cl + NaNO3 (b) NH4Cl
(c) oxygen has higher b.p. than nitrogen (c) (NH4)2SO4 (d) NH4NO2
(d) oxygen has a lower density than nitrogen. 60. 4HNO3 + P4 O10 ® 4HPO3 + X In the above reaction, the
42. Which would quickly absorb oxygen ? product X is
(a) Alkaline solution of pyrogallol (a) NO 2 (b) N 2 O3 (c) N 2 O 4 (d) N 2 O5
(b) Conc. H 2SO 4 61. In the manufacture of bromine from sea water, the mother
(c) Lime water liquor containing bromides is treated with
(d) Alkaline solution of CuSO 4 (a) carbon dioxide (b) chlorine
(c) iodine (d) sulphur dioxide
43. Which one of the following is a correct statement : 62. Which of the following species has four lone pairs of
(a) all metal nitrates are insoluble in water electrons?
(b) solubility depends on temperature (a) I (b) O - (c) Cl - (d) He
(c) all metal nitrates are soluble in water 63. A solution of potassium bromide is treated with each of the
(d) all metal nitrates are soluble in alcohol following. Which one would liberate bromine ?
44. Polyanion formation is maximum in (a) Hydrogen iodide (b) Sulphur dioxide
(a) nitrogen (b) oxygen (c) Chlorine (d) Iodine
(c) sulphur (d) boron 64. Which one of the following oxides of chlorine is obtained
45. The acid which has a peroxy linkage is by passing dry chlorine over silver chlorate at 90°C ?
(a) sulphurous acid (b) pyrosulphuric acid
(a) Cl2 O (b) ClO 3 (c) ClO 2 (d) ClO 4
(c) dithionic acid (d) caro’s acid
65. Ammonia and sodium hypochlorite reacts to produce
46. By passing H 2S gas in acidified KMnO4 solution, we get
(a) NH 2OH (b) NH 2 NH 2
(a) S (b) K2S (c) MnO2 (d) K2SO3 (c) N 2 (d) NO
The p-Block Elements (Group 15, 16, 17 and 18) 511

66. Which one is the correct order of the size of iodine species? 84. Hydrolysis of NCl3 gives NH3 and X. Which of the following
(a) I > I+ > I– (b) I > I– > I+ is X ?
+ –
(c) I > I > I (d) I– > I > I+ (a) HClO4 (b) HClO3 (c) HOCl (d) HClO2
67. A one litre flask is full of brown bromine vapours. The 85. Which one of the following noble gases is not found in the
intensity of brown colour of vapours will not decrease atmosphere
appreciably on adding to the flask some (a) Rn (b) Kr (c) Ne (d) Ar
(a) pieces of marble (b) animal charcoal powder 86. Noble gases are group of elements which exhibit
(c) carbon tetrachloride (d) carbon disulphide (a) high chemical activity
68. Which of the following is the best description for the
(b) low chemical activity
behaviour of bromine in the reaction given below ?
(c) minimum electronegativity
H 2 O + Br2 ® HOBr + HBr (d) paramagnetic properties
(a) Proton acceptor only 87. PbO2 on reaction with conc. HNO3 gives
(b) Both oxidized and reduced
(a) PbNO2 + O2
(c) Oxidized only
(d) Reduced only 1
(b) Pb(NO3)2 + H2O + O 2
69. P2 O5 is an anhydride of 2
(c) Pb + N2 + H2O
(a) HPO 3 (b) H 3 PO 4 (d) Pb(OH)2 + N2O
(c) H 3PO 3 (d) H2 P2O7 88. In nitrogen family, the H-M-H bond angle in the hydrides
70. The correct order of the thermal stability of hydrogen halides gradually becomes closer to 90º on going from N to Sb. This
(H–X) is shows that gradually
(a) HI > HCl < HF > HBr (b) HCl< HF > HBr < HI (a) The basic strength of the hydrides increases
(c) HF > HCl > HBr > HI (d) HI < HBr > HCl < HF (b) Almost pure p-orbitals are used for M-H bonding
71. Cl2 reacts with hot and conc. NaOH to give – (c) The bond energies of M-H bonds increase
(a) NaClO (b) NaClO3 (c) NaClO2 (d) NaClO4 (d) The bond pairs of electrons become nearer to the central
72. Oleum is a mixture of
atom
(a) H 2SO 4 + SO 2 (b) H 2SO 4 + SO 3 89. Perdisulphuric acid has the following bond
(c) H 2 S 2 O 3 + SO 3 (d) H 2 S 2 O 3 + SO 2
73. Conc. HNO3 reacts with I2 to form : (a) O ¬¾ ¾O=O (b) ¬¾ ¾ O = O ¾¾ ®
(a) HI (b) HOI (c) HIO2 (d) HIO3 (c) > O ® O < (d) – O – O –
74. For a given alcohol the order of reactivity of halogen acids 90. Which one of the following statements regarding helium is
is: incorrect ?
(a) HI > HBr > HCl (b) HCl > HBr < HI (a) It is used to pr oduce an d sustain powerful
(c) HI < HBr < HCl (d) HCl < HI < HBr superconducting magnets.
75. Which of the following has maximum bond energy? (b) It is used as a cryogenic agent for carrying out
(a) Cl2 (b) F2 (c) Br2 (d) I2 experiments at low temperatures.
76. Which of the following product is formed by the reaction of (c) It is used to fill gas balloons instead of hydrogen
sulphur dioxide with chlorine in presence of sunlight :
because it is lighter and non-inflammable.
(a) SO2Cl (b) SO2Cl2 (c) SOCl2 (d) SO3Cl
77. Fluorine exhibits an oxidation state of only –1 because (d) It is used in gas-cooled nuclear reactors.
(a) it can readily accept an electron 91. The formation of O2+[PtF6]– is the basis for the formation of
(b) it is very strongly electronegative xenon fluorides. This is because
(c) it is a non-metal (a) O2 and Xe have comparable sizes
(d) it belongs to halogen family (b) both O2 and Xe are gases
78. In XeF2, XeF4, XeF6 the number of lone pairs on Xe are (c) O2 and Xe have comparable ionisation energies
respectively (d) Both (a) and (c)
(a) 2, 3, 1 (b) 1, 2, 3 (c) 4, 1, 2 (d) 3, 2, 1. 92. The species not containing peroxide ions is :
79. Formula for tear gas is : (a) H2O2 (b) PbO2 (c) SrO2 (d) BaO2
(a) COCl2 (b) CCl3NO2 93. Which one of the following orders correctly represents the
(c) N2O (d) none of these increasing acid strengths of the given acids?
80. Which of the following is least polarisable ? (a) HOClO < HOCl < HOClO3 < HOClO2
(a) Ne (b) He (c) Xe (d) Kr (b) HOClO2 < HOClO3 < HOClO < HOCl
81. End-product of the hydrolysis of XeF6 is (c) HOClO3 < HOClO2 < HOClO < HOCl
(a) XeF4O (b) XeF2O2 (c) XeO3 (d) XeO3 – (d) HOCl < HOClO < HOClO2 < HOClO3
82. For advertisement, the coloured discharge tubes contain : 94. Which one of the following arrangements does not give the
(a) He (b) Ne (c) Ar (d) Kr correct picture of the trends indicated against it ?
83. Noble gases do not react with other elements because
(a) F2 > Cl2 > Br2 > I2 : Oxidizing power
(a) they have completely filled valence shell (ns2np6)
(b) F2 > Cl2 > Br2 > I2 : Electron gain enthalpy
(b) the sizes of their atoms are very small
(c) they are not found in abundance (c) F2 > Cl2 > Br2 > I2 : Bond dissociation energy
(d) they are monoatomic (d) F2 > Cl2 > Br2 > I2 : Electronegativity.
EBD_7327
512 CHEMISTRY

95. In the case of alkali metals, the covalent character decreases (a) Cl 2 O < ClO 2 < ClO 2– (b) ClO 2 < Cl 2 O < ClO 2–
in the order: (c) Cl 2 O < ClO 2– < ClO 2 (d) ClO 2– < Cl 2 O < ClO 2
(a) MF > MCl > MBr > MI
(b) MF > MCl > MI > MBr 98. Oxidation states of P in H4 P2O5 , H4 P2O6 , and H4 P2O7 ,
(c) MI > MBr > MCl > MF are respectively:
(d) MCl > MI > MBr > MF (a) + 3, + 5, + 4 (b) + 5, + 3, + 4
96. Among the following which is the strongest oxidising agent? (c) + 5, + 4, + 3 (d) + 3, + 4, + 5
(a) Br2 (b) I2 (c) Cl2 (d) F2 99. Which one of the following compounds is a peroxide ?
97. The correct order of increasing bond angles in the following (a) KO 2 (b) BaO2 (c) MnO 2 (d) NO 2
species are : 100. How many bridging oxygen atoms are present in P4O10?
(a) 5 (b) 6 (c) 4 (d) 2

1. In which of the following, NH4OH is not used? 11. The ease of liquefaction of noble gases increases in the
(a) Tollen’s reagent order
(b) Nessler’s reagent (a) He < Ne < Ar < Kr < Xe (b) Xe < Kr < Ne < Ar < He
(c) Group reagent for the analysis of IV group basic radicals (c) Kr < Xe < He < Ne < Ar (d) Ar < Kr < Xe < Ne < He
(d) Group reagent for the analysis of III group basic radicals 12. Which of the following represents correct sequence of
2. Which of the following oxides will be the least acidic? decreasing acidic character of oxides?
(a) As 4 O 6 (b) As 4 O10 (a) N2O5 > NO > N2O (b) MnO > MnO2 > MnO4–
(c) P4 O10 (d) P4O6 (c) Cr2O3 > CrO4– (d) Fe2O3 > FeO > Fe3O4
3. Noble gases are used in discharge tubes to gives different 13. The solubility of silver bromide in hypo solution is due to
colours. Reddish orange glow is due to the formation of :
(a) Ar (b) Ne (c) Xe (d) Kr (a) Ag2SO3 (b) Ag2S2O3
4. Which of the following species has the highest dipole moment ? (c) Ag+ (d) NH +4
(a) NH3 (b) PH3
(c) AsH3 (d) SbH3 14. The correct decreasing order of basic strength is:
5. Which of the following bonds will be most polar? (a) AsH3 > SbH3 > PH3 > NH3
(a) N – Cl (b) O – F (b) SbH 3 > AsH 3 > PH 3 > NH 3
(c) N – F (d) N – N (c) NH 3 > PH 3 > AsH 3 > SbH 3
6. When Br2 is treated with aqueous solutions of NaF, NaCl (d) PH 3 > AsH 3 > SbH 3 > NH 3
and NaI separately 15. Oxygen and sulphur both are the members of the same group
(a) F2, Cl2 and I2 are liberated in periodic table but H2O is liquid while H2S is gas because
(b) only F2 and Cl2 are liberated (a) molecular weight of water is more
(c) only I2 is liberated (b) electronegativity of sulphur is more
(d) only Cl2 is liberated (c) H2S is weak acid
7. Regarding F– and Cl– which of the following statements is/ (d) water molecules are having weak hydrogen bonds
are correct? between them
(i) Cl– can give up an electron more easily than F– 16. Euchlorine is a mixture of
(ii) Cl– is a better reducing agent than F– (a) Cl2 and SO2 (b) Cl2 and ClO2
(iii) Cl– is smaller in size than F– (c) Cl2 and CO (d) None of these
(iv) F– can be oxidized more readily than Cl – 17. The correct order of heat of formation of halogen acids is
(a) (i) and (ii) (b) (i), (ii) and (iv) (a) HI > HBr > HCl > HF (b) HF > HCl > HBr > HI
(c) (iii) and (iv) (d) Only (i) (c) HCl > HF > HBr > HI (d) HCl > HBr > HF > HI
8. The p-block element that forms predominantly basic oxide is 18. Which of the following acts as pickling agent?
(a) N (b) P (c) As (d) Bi (a) HNO3 (b) HCl (c) H2SO4 (d) HNO2
9. In case of nitrogen, NCl3 is possible but not NCl5 while in 19. KF combines with HF to form KHF2. The compound contains
case of phosphorus, PCl3 as well as PCl5 are possible. It is the species.
due to (a) K+, F– and H+ (b) K+, F– and HF
+
(c) K and [HF2] – (d) [KHF]+ and F–
(a) availability of vacant d orbitals in P but not in N
(b) lower electronegativity of P than N 20. Which of the following iodide is least stable and has doubtful
(c) lower tendency of H-bond formation in P than N existence?
(d) occurrence of P in solid while N in gaseous state at (a) CI4 (b) GeI4 (c) SnI4 (d) PbI4
room temperature. 21. In the preparation of sulphuric acid, V2 O 5 is used in the
10. In which one of the following oxides of nitrogen, one reaction, which is
nitrogen atom is not directly linked to oxygen? (a) S + O 2 ¾¾® SO 2 (b) SO2 + H 2O ¾ ¾® H 2SO4
(a) NO (b) N2O4 (c) N2O (d) N2O3 (c) 2SO2 + O2 ¾ ¾® 2SO3 (d) N 2 + 3H 2 ¾¾® 2 NH 3
The p-Block Elements (Group 15, 16, 17 and 18) 513

22. In compounds of type ECl3, where E = B, P, As or Bi, the (b) Ozone reacts with SO2 to give SO3.
angles Cl - E- Cl for different E are in the order (c) Silicon reacts with NaOH(aq) in the presence of air to
(a) B > P = As = Bi (b) B > P > As > Bi give Na2SiO3 and H2O.
(c) B < P = As = Bi (d) B < P < As < Bi (d) Cl2 reacts with excess of NH3 to give N2 and HCl.
23. Match List - I (substances) with List - II (processes) employed 34. Regular use of which of the the following fertilizers increases
in the manufacture of the substances and select the correct the acidity of soil?
option. (a) Ammonium sulphate (b) Potassium nitrate
List - I List - II (c) Urea (d) Superphosphate of lime.
Substances Processes 35. Which of the following has maximum number of lone pairs
(A) Sulphuric acid (i) Haber’s process associated with Xe ?
(B) Steel (ii) Bessemer’s process (a) XeF4 (b) XeF6 (c) XeF2 (d) XeO3
(C) Sodium hydroxide (iii) Leblanc process 36. Which of the following statements regarding sulphur is
(D) Ammonia (iv) Contact process incorrect?
Options: (a) S2 molecule is paramagnetic.
(A) (B) (C) (D) (b) The vapours at 200°C consists mostly of S8 rings.
(a) (iv) (ii) (iii) (i) (c) At 600°C the gas mainly consists of S2 molecules.
(b) (i) (iv) (ii) (iii) (d) The oxidation state of sulphur is never less than +4 in
(c) (i) (ii) (iii) (iv) its compounds.
(d) (iv) (iii) (ii) (i) 37. In nitrogen family, the H-M-H bond angle in the hydrides
24. Which of the following statements is not valid for oxoacids gradually becomes closer to 90º on going from N to Sb. This
of phosphorus? shows that gradually
(a) Orthophosphoric acid is used in the manufacture of (a) The basic strength of the hydrides increases
triple superphosphate. (b) Almost pure p-orbitals are used for M-H bonding
(b) Hypophosphorous acid is a diprotic acid. (c) The bond energies of M-H bonds increase
(c) All oxoacids contain tetrahedral four coordinated (d) The bond pairs of electrons become nearer to the central
phosphorus. atom
(d) All oxoacids contain atleast one P = O and one P — OH 38. Which acid has P – P linkage ?
group. (a) Hypophosphoric acid
25. Sulphur trioxide can be obtained by which of the following (b) Pyrophosphoric acid
reaction : (c) Metaphosphoric acid
(d) Orthophosphoric acid
Fe 2 ( SO4 )3 ¾¾
Δ Δ
(a) CaSO 4 + C ¾¾
® (b) ® 39. The no. of S-O-S bonds in cyclic SO3 is
Δ Δ (a) 1 (b) 2
(c) S + H 2SO 4 ¾¾ ® (d) H 2SO 4 + PCl5 ¾¾ ® (c) 3 (d) None of these
26. Which one of the following reactions of xenon compounds 40. Which of the following fluorides does not exist?
is not feasible? (a) NF5 (b) PF5
(a) 3XeF4 + 6H 2 O ¾¾ ® 2Xe + XeO 3 +12HF +1.5O 2 (c) AsF 5 (d) SbF5
(b) 2XeF2 + 2H 2O ¾¾ ® 2Xe + 4HF + O 2 41. Which of the following shows nitrogen with its increasing
order of oxidation number?
(c) XeF6 + RbF ¾¾ ® Rb[XeF7 ] (a) NO < N2O < NO2 < NO3– < NH4+
(d) XeO3 + 6HF ¾¾ ® XeF6 + 3H 2 O (b) NH4+ < N2O < NO2 < NO3– < NO
27. Aqueous solution of Na2S2O3 on reaction with Cl2 gives – (c) NH4+ < N2O < NO < NO2 < NO3–
(a) Na2S4O6 (b) NaHSO4 (d) NH4+ < NO < N2O < NO2 < NO3–
(c) NaCl (d) NaOH 42. When Br2 is treated with aqueous solutions of NaF, NaCl
28. The reaction of P4 with X leads selectively to P4O6. The X is and NaI separately
(a) Dry O2 (a) F2, Cl2 and I2 are liberated
(b) A mixture of O2 and N2 (b) only F2 and Cl2 are liberated
(c) Moist O2 (c) only I2 is liberated
(d) O2 in the presence of aqueous NaOH (d) only Cl2 is liberated
29. Extra pure N2 can be obtained by heating 43. In which one of the following oxides of nitrogen, one
(a) NH3 with CuO (b) NH4NO3 nitrogen atom is not directly linked to oxygen?
(c) (NH4)2Cr2O7 (d) Ba(N3)2 (a) NO (b) N2O4
30. The shape of XeO2F2 molecule is (c) N2O (d) N2O3
(a) trigonal bipyramidal (b) square planar 44. In which of the following, NH4OH is not used?
(c) tetrahedral (d) see-saw (a) Tollen’s reagent
31. Which of the following oxide is amphoteric ? (b) Nessler’s reagent
(a) SnO2 (b) CaO (c) SiO2 (d) CO2 (c) Group reagent for the analysis of IV group basic radicals
32. In which of the following compounds, nitrogen exhibits (d) Group reagent for the analysis of III group basic radicals
highest oxidation state ? 45. The geometry of XeF6 is
(a) N2H4 (b) NH3 (c) N3H (d) NH2OH (a) planar hexagon
33. Identify the incorrect statement among the following. (b) regular octahedron
(a) Br2 reacts with hot and strong NaOH solution to give (c) distorted octahedron
NaBr and H2O. (d) square bipyramid
EBD_7327
514 CHEMISTRY

46. Noble gases are used in discharge tubes to gives different (b) Statement -1 is True, Statement -2 is True ; Statement-2 is
colours. Reddish orange glow is due lo NOT a correct explanation for Statement - 1
(a) Ar (b) Ne (c) Statement - 1 is True, Statement- 2 is False
(c) Xe (d) Kr (d) Statement -1 is False, Statement -2 is True
47. The oxidation state of phosphorus in 48. Statement-1 : White phosphorus is more reactive than red
cyclotrimetaphosphoric acid is phosphorus.
(a) +3 (b) +5 Statement-2 : Red phosphorus consists of P4 tetrahedral
(c) –3 (d) +2 units linked to one another to form linear chains.
DIRECTIONS for Qs. 48 to 50 : These are Assertion-Reason 49. Statement-1 : When a metal is treated with conc. HNO3 it
type questions. Each of these question contains two statements: generally yields a nitrate, NO2 and H2O.
Statement-1 (Assertion) and Statement-2 (Reason). Answer these Statement-2 : Conc. HNO3 reacts with metal and first
questions from the following four options. produces a metal nitrate and nascent hydrogen. The nascent
(a) Statement- 1 is True, Statement-2 is True, Statement-2 is a hydrogen then further reduces HNO3 to NO2.
correct explanation for Statement -1 50. Statement-1 : Bond angle of H2S is smaller than H2O.
Statement-2 : Electronegativity of the central atom increases,
bond angle decreases.

Exemplar Questions strongest reducing agent?


1. On addition of conc. H2SO4 to a chloride salt, colourless Compound NH3 PH3 AsH3 SbH3
fumes are evolved but in case of iodide salt, violet fumes
D diss ( E — H ) / kJ mol -1
389 322 297 255
come out. This is because
(a) H2SO4 reduces HI to I2 (b) HI is of violet colour (a) NH3 (b) PH3
(c) HI gets oxidised to I2 (d) HI changes to HIO3 (c) AsH3 (d) SbH3
2. In qualitative analysis when H2S is passed through an 8. On heating with concentrated NaOH solution in an inert
aqueous solution of salt acidified with dil. HCl, a black atmosphere of CO2, white phosphorus gives a gas. Which
precipitate is obtained. On boiling the precipitate with of the following statement is incorrect about the gas?
dil. HNO3, it forms a solution of blue colour. Addition of (a) It is highly poisonous and has smell like rotten fish
excess of aqueous solution of ammonia to this solution (b) It's solution in water decomposes in the presence of light
gives ............ . (c) It is more basic than NH3
(a) deep blue precipitate of Cu(OH)2 (d) It is less basic than NH3
(b) deep blue solution of [Cu(NH3)4]2+ 9. Which of the following acids forms three series of salts?
(c) deep blue solution of Cu(NO3)2 (a) H3PO2 (b) H3BO3
(d) deep blue solution of Cu(OH)2. Cu(NO3)2 (c) H3PO4 (d) H3PO3
3. In a cyclotrimetaphosphoric acid molecule, how many single 10. Strong reducing behaviour of H3PO2 is due to
and double bonds are present? (a) low oxidation state of phosphorus
(a) 3 double bonds; 9 single bonds (b) presence of two — OH groups and one P — H bond
(b) 6 double bonds; 6 single bonds (c) presence of one — OH group and two P — H bonds
(c) 3 double bonds; 12 single bonds (d) high electron gain enthalpy of phosphorus
(d) Zero double bond; 12 single bonds 11. On heating lead nitrate forms oxides of nitrogen and lead.
4. Which of the following elements can be involved in The oxides formed are ................. .
pp – dp bonding? (a) N2O, PbO (b) NO2, PbO
(a) Carbon (b) Nitrogen (c) NO, PbO (d) NO, PbO2
(c) Phosphorus (d) Boron 12. Which of the following elements does not show allotropy?
5. Which of the following pairs of ions are isoelectronic and (a) Nitrogen (b) Bismuth
isostructural? (c) Antimony (d) Arsenic
13. Maximum covalency of nitrogen is ................... .
(a) CO32 - , NO3- (b) ClO3- , CO32-
(a) 3 (b) 5
(c) SO32 - , NO3- (d) ClO3- , SO32 - (c) 4 (d) 6
6. Affinity for hydrogen decreases in the group from fluorine 14. Which of the following statements is wrong?
to iodine. Which of the halogen acids should have highest (a) Single N — N bond is stronger than the single P — P
bond dissociation enthalpy? bond.
(a) HF (b) HCl (b) PH3 can act as a ligand in the formation of coordination
(c) HBr (d) HI compounds with transition elements.
7. Bond dissociation enthalpy of E — H (E = element) bonds is (c) NO2 is paramagnetic in nature.
given below. Which of the following compounds will acts as (d) Covalency of nitrogen in N2O3 is four.
The p-Block Elements (Group 15, 16, 17 and 18) 515

15. A brown ring is formed in the ring test for NO3- ion. It is due (d) ionic solid with [PCl 4 ]+ tetrahedral and [PCl 6 ] –
to the formation of octahedral
(a) [Fe (H2O)5 (NO)]2+ (b) FeSO4 . NO2 26. Reduction potentials of some ions are given below. Arrange
(c) [Fe (H2O)4 (NO)2]2+ (d) FeSO4 . HNO3 them in decreasing order of oxidising power.
16. Elements of group- 15 form compounds in +5 oxidation state. - - -
However, bismuth forms only one well characterised Ion ClO4 IO4 BrO4
compound in +5 oxidation state. The compound is Reduction potential
E° = 1.19V E° = 1.65V E° = 1.74V
(a) Bi2O5 (b) BiF5 E° / V
(c) BiCl5 (d) Bi2S5
17. On heating ammonium dichromate and barium azide (a) ClO -4 > IO 4- < BrO 4- (b) IO 4- > BrO4- < ClO 4-
separately we get (c) BrO4- > IO-4 < ClO4- (d) BrO4- > ClO 4- < IO4-
(a) N2 in both the cases
(b) N2 with ammonium dichromate and NO with barium 27. Which of the following is isoelectronic pair?
azide (a) ICl2, ClO2 (b) BrO -2 , BrF2+
(c) N2O with ammonium dichromate and N2 with barium (c) ClO2, BrF (d) CN–, O3
azide
NEET/AIPMT (2013-2017) Questions
(d) N2O with ammonium dichromate and NO2 with barium
azide 28. Which is the strongest acid in the following : [2013]
18. In the preparation of HNO3 , we get NO gas by catalytic (a) HClO3 (b) HClO4
oxidation of ammonia. The moles of NO produced by the (c) H2SO3 (d) H2SO4
oxidation of two moles of NH3 will be ................. . 29. Which of the following does not give oxygen on heating?
(a) 2 (b) 3 (a) Zn(ClO3)2 (b) K2Cr2O7 [2013]
(c) 4 (d) 6 (c) (NH4)2Cr2O7 (d) KClO3
19. The oxidation state of central atom in the anion of compound 30. Identify the incorrect statement, regarding the molecule
NaH2PO2 will be ................. . XeO4: [NEET Kar. 2013]
(a) + 3 (b) + 5 (a) XeO4 molecule is tetrahedral
(c) + 1 (d) – 3 (b) XeO4 molecule is square planar
20. Which of the following is not tetrahedral in shape? (c) There are four pp – dp bonds
(a) NH+4 (b) SiCl4 (d) There are four sp3 – p, s bonds
(c) SF4 (d) SO42– 31. Acidity of diprotic acids in aqueous solutions increases in
21. Which of the following are peroxoacids of sulphur? the order : [2014]
(a) H2SO5 and H2S2O8 (b) H2SO5 and H2S2O7 (a) H2S < H2Se < H2Te (b) H2Se < H2S < H2Te
(c) H2S2O7 and H2S2O8 (d) H2S2O6 and H2S2O7 (c) H2Te < H2S < H2Se (d) H2Se < H2Te < H2S
22. Hot conc. H2SO4 acts as moderately strong oxidising agent. 32. Nitrogen dioxide and sulphur dioxide have some properties
It oxidises both metals and non-metals. Which of the in common. Which property is shown by one of these
following element is oxidised by conc. H2 SO4 into two compounds, but not by the other? [2015]
gaseous products? (a) is a reducing agent
(a) Cu (b) S (b) is soluble in water
(c) C (d) Zn (c) is used as a food-preservative
23. A black compound of manganese reacts with a halogen acid (d) forms 'acid-rain'
to give greenish yellow gas. When excess of this gas reacts 33. Which of the statements given below is incorrect?
with NH3 an unstable trihalide is formed. In this process the (a) Cl2O7 is an anhydride of perchloric acid [2015 RS]
oxidation state of nitrogen changes
(b) O3 molecule is bent
from ........... .
(c) ONF is isoelectronic with O2N–.
(a) – 3 to + 3 (b) – 3 to 0
(d) OF2 is an oxide of fluorine
(c) – 3 to + 5 (d) 0 to – 3
34. The variation of the boiling point of the hydrogen halides is
24. In the preparation of compounds of Xe, Bartlett had taken
- in the order HF > HI > HBr > HCl. [2015 RS]
O+2 Pt F6 as a base compound. This is because What explains the higher boiling point of hydrogen fluoride?
(a) both O2 and Xe have same size.
(a) The electronegativity of fluorine is much higher than
(b) both O2 and Xe have same electron gain enthalpy.
for other elements in the group.
(c) both O2 and Xe have almost same ionisation enthalpy.
(b) There is strong hydrogen bonding between HF
(d) both Xe and O2 are gases.
molecules
25. In solid state PCl5 is a ................. .
(c) The bond energy of HF molecules is greater than in
(a) covalent solid
other hydrogen halides.
(b) octahedral structure
(d) The effect of nuclear shielding is much reduced in
(c) ionic solid with [PCl 6 ]+ octahedral and [PCl 4 ] –
fluorine which polarises the HF molecule.
tetrahedral
EBD_7327
516 CHEMISTRY

35. Strong reducing behaviour of H3PO2 is due to [2015 RS] 39. Which is the correct statement for the given acids? [2016]
(a) presence of one –OH group and two P–H bonds (a) Phosphinic acid is a diprotic acid while phosphonic
(b) high electron gain enthalpy of phosphorus acid is a monoprotic acid
(c) high oxidation state of phosphorus (b) Phosphinic acid is a monoprotic acid while phosphonic
(d) presence of two –OH groups and one P–H bond. acid is a diprotic acid
36. Among the following, the correct order of acidity is [2016] (c) Both are triprotic acids
(d) Both are diprotic acids
(a) HClO3 < HClO4 < HClO2 < HClO
40. The product obtained as a result of a reaction of nitrogen
(b) HClO < HClO2 < HClO3 < HClO4
with CaC2 is [2016]
(c) HClO2 < HClO < HClO3 < HClO4 (a) Ca(CN)2 (b) CaCN
(d) HClO4 < HClO2 < HClO < HClO3 (c) CaCN3 (d) Ca2CN
37. Which one of the following orders is correct for the bond 41. Match the interhalogen compounds of column-I with the
dissociation enthalpy of halogen molecules? [2016] geometry in column II and assign the correct code. [2017]
(a) I2 > Br2 > Cl2 > F2 (b) Cl2 > Br2 > F2 > I2 Column-I Column-II
(c) Br2 > I2 > F2 > Cl2 (d) F2 > Cl2 > Br2 > I2 1. XX' (i) T-shape
38. Match the compounds given in column I with the 2. XX'3 (ii) Pentagonal bipyramidal
hybridisation and shape given in column II and mark the 3. XX'5 (iii) Linear
correct option. [2016] 4. XX'7 (iv) Square-pyramidal
Column-I Column-II (v) Tetrahedral
1. XeF6 (i) Distorted octahedral Code :
2. XeO3 (ii) Square planar 1 2 3 4
3. XeOF4 (iii) Pyramidal (a) (iii) (i) (iv) (ii)
4. XeF4 (iv) Square pyramidal (b) (v) (iv) (iii) (ii)
Code : (c) (iv) (iii) (ii) (i)
(d) (iii) (iv) (i) (ii)
1 2 3 4
42. In which pair of ions both the species contain S – S bond?
(a) (i) (iii) (iv) (ii)
(b) (i) (ii) (iv) (iii) (a) S4 O62- ,S2 O32 - (b) S2 O72- ,S2 O82 - [2017]
(c) (iv) (iii) (i) (ii)
(d) (iv) (i) (ii) (iii)
(c) S4 O62- ,S2 O72 - (d) S2 O72- ,S2 O32 -
The p-Block Elements (Group 15, 16, 17 and 18) 517

Hints & Solutions


EXERCISE - 1 16. (a) We know that empirical formula of hypophosphrus acid
is H3PO2. In this only one ionisable hydrogen atom is
1. (a) Hydrogen is lighter than helium but still helium is used present i.e. it is monobasic. Therefore option (a) is
in ballons because hydrogne is highly explosive and correct structural formula of it.
reactive gas. It may lead to accident.
17. (d) CO2 is oxide of non-metal. Both N2O and NO2 are also
600°C oxide of non-metal. They form acids on dissolution in
2. (d) 2H 3 PO 4 ¾¾ ¾
¾® 2 HPO 3
- 2H 2O water. In this way CO2 resembles N2O and NO2. Option
3. (b) The order of boiling points of the group 15 hydrides is : (d) is the answer.
BiH3 > SbH3 > NH3 > AsH3 > PH3 18. (c) Structure of hypophosphorous acid
4. (b) ClO2 is least stable. The order of stability is
H
Cl2O < ClO2 < ClO3 < Cl2O7 |
The higher the oxygen atom involved the greater the H-O- P ® O
|
stability. The oxygen atom being electro-negative does H
not allow electron of chlorine atom to be showed with
others in the formation of compound. So Cl2O7 is most Two H-atoms are attached to P atom.
inactive or highly stable. 19. (a) Only nitrates of heavy metals and lithium decompose
5. (b) In case of nitrogen, d-orbitals are not available. on heating to produce NO2.
D 20. (a) Nitrogen due to small size is able to show pp-pp lateral
6. (d) P2O5 + 3H 2O ¾¾® 2H3 PO4
7. (a) He being the smallest sized atom, can not be absorbed overlap forming N º N, rest elements due to bigger size
by coconut charcoal. are not able to show pp-pp lateral overlap.
8. (b) PCl3 + H2O ¾¾ ® POCl3 + 2HCl 21. (c) concentrated sulphuric acid is always diluted by adding
POCl3 + 3H2O ¾¾ ® H3PO4 + 3HCl acid to water because the process of mixing generates
9. (a) PH 4 I + NaOH ® NaI + PH 3 + H 2 O a lot of heat. If we add water to acid due to generation
Heat of large amount of heat acid may spill over to its
10. (d) NH 4 Cl + NaNO 2 ¾¾¾¾
® NH 4 NO 2 surroundings.
- NaCl
Heat 22. (a) Oxide in which central atom has higher charge and more
¾¾¾® N 2 + 2 H 2 O.
electronegativity is more acidic, i.e.
11. (b) C12 H 22O11 + ® 6 (COOH) 2 + 5H 2 O.
18[O] ¾¾ N2O5 > N2O4 > P2O5 > As2O3.
Canesugar From HNO3 Oxalic acid
23. (c) Ca + 2C ––® CaC2
12. (c) HF (Hydrofluoric acid or Hydrogen flouride) is the acid
CaC2 + N2 ¾¾¾¾1273K
which cannot be kept in glass because it reacts with ® CaCN2 + C
glass Calcium
6 HF + SiO 2 ¾
¾® H 2SiF6 + 2 H 2 O. cyanamide
13. (d) N2 molecule contains triple bond between N atoms 24. (b) Most efficient agent for the absorption of SO3 is 98%
having very high dissociation energy (946 kJ mol–1) H2SO4. They form oleum.
due to which it is relatively inactive.
¾® H 2S 2 O 7 (oleum)
SO 3 + H 2SO 4 ¾
14. (c) H 3PO 2 is named as hypophosphorous acid. It is Sulpher trioxide is not directly absorbed in water to
monobasic as it contains only one P – OH bond, its form sulphuric acid because it forms dense fog of
basicity is one. sulphuric acid.
O 25. (a) NH4ClO4 + HNO3 ® HClO4 + NH4NO3
||
P NH4NO3 ¾¾ D
H | OH ® N2O + 2H2O
|

H 26. (c) NH3 gas can be dried by CaO (calcium oxide or quick
15. (c) As compared to nitrogen , oxygen is mor e lime).
electronegative and more reactive because as we go
27. (b) In Haber’s process for manufacture of NH3, finely
from left to right in a group electronegativity increases
divided iron is used as catalyst and molybdenum is
and reactivity decreases. So, nitrogen should be more
used as catalytic promoter
reactive than oxygen but in nitrogen
Fe + Mo
p-orbitals are exactly half filled which make it stable N2 ( g ) + 3H2 ( g ) ¾¾¾¾¾¾ ® 2NH 3 ( g )
800K, High P
and less reactive as compared to oxygen.
EBD_7327
518 CHEMISTRY

cold major constituents of liquid air i.e., liquid oxygen and


28. (a) 2NaOH + Cl2 ¾¾¾
® NaCl + NaClO + H2O
Sod. hypo- liquid nitrogen are separated by means of fractional
chlorite distillation (b.p. of O2 = –183°C : b. P. of N2 = – 195.8°C)
29. (d) N2H4 and NH4Cl are obtained by reaction of ammonia 42. (a) Alkaline solution of pyrogallol absorbs oxygen quickly.
with hypochlorite anion. 43. (b) Solubility depends upon temperature. In most cases it
3NH 3 + NaOCl ¾¾
® N 2 H 4 + NH 4 Cl + NaOH increases with rise in temperature but in those cases
.. where the process of dissolution is exothermic,
30. (c) P solubility decreases with rise in temperature.
44. (c) Due to greater tendency for catenation, sulphur shows
P P property of polyanion formation to a greater extent.
..

..

..P For example, in polysulphides such as S32 - , S24 - , S52 -


P4 (White phosphorus)
45. (d) Caro’s acid is H 2SO 5 which contains one S – O – OH
Q Each P-atom forms 3 bonds. Thus it has 3 bond peroxy linkage. It is also known as permonosulphuric
pairs of electrons. acid.
D O
31. (a) NH 4 Cl + NaNO 2 ¾¾® NH 4 NO 2 + NaCl
||
D HO – S – O – OH
NH 4 NO 2 ¾¾® N 2 + 2H 2 O ||
32. (a) The sides of safety matches contain red phosphorus O
and sand powder. 46. (a) 2KMnO 4 + 5H 2 S + 3H 2 SO 4 ¾¾®
33. (b) Mg3N2 + 6H2O ® 3Mg(OH)2 + 2NH3 K2 SO 4 + 2MnSO 4 + 5S + 8H 2 O.
1 mol (excess) 2 mol
Thus in this reaction S2– is oxidised to S.
O O 47. (c) In the manufacture of bromine from sea water, the
mother liquor which contains bromides is treated with
34. (b) N–O–N chlorine which oxidies bromide ion to bromine
O O 2KBr + Cl 2 ¾
¾® 2 KCl + Br2
The structure clearly shows the presence of covalent
and co-ordinate bonds. 48. (a) 2S2 O3-2 + I2 ® S4 O6-2 + 2I -
Thiosulphate Tetrathionate
35. (d) NO2 is reddish brown coloured gas. Rest of the oxides
are colourless. 49. (b) PbO2 is a powerful oxidizing agent and liberate O2 when
36. (c) Ca3P2 + 6H2O ® 3Ca(OH)2 + 2PH3 ; i.e 2 moles of treated with acids.
phosphine are produced from one mole of calcium 2PbO 2 + 4HNO3 ¾¾
® 2Pb(NO 3 ) 2 + 2H 2 O + O 2 ­
phosphide.
50. (b) Oxygen can be prepared by heating oxides of Hg, Pb,
37. (d) Group 16, elements are known as Chalcogens Na is not
Ag, Mn and Ba.
group 16 element. So it is not Chalcogen.
D
38. (c) In cyclic metaphosporic acid number of P–O–P bonds 2HgO ¾¾® 2Hg + O2
is three. 51. (c) NO2 is the compound which forms dimer N2O4. NO2
has one unpaired electron. So, it dimerises to form
O OH
paired electrons.
P
O O 1
O O 52. (b) At anode : 2OH - H 2O + O 2
P P 2
O OH 53. (d) Hypo (Na2S2O3) is used in photography. It is used
HO
during fixing of image. It dissolves AgBr that has not
been affected by light during exposure leaving metallic
39. (a) Nitrogen is obtained by the thermal decomposition of
silver (Ag) as such
NH4NO2
AgBr + 2Na 2S2O3 ¾¾ ® Na 3 [Ag(S2O 3 ) 2 ]+ NaBr
D
NH 4 NO 2 ¾¾® N 2 + 2H 2 O Hypo Soluble complex

40. (b) Alkaline pyrogallol absorbs O2 and oil of cinnamon 54. (b) HClO4 is the strongest acid.
absorbs O3. 55. (a) Sodium thiosulphate is a reducing agent. It is used in
41. (c) Air is liquified by making use of the joule-Thompson volumetric titration (Iodimetry) to reduce I2 to I–.
effect (cooling by expansion of the gas) Water vapour 2Na2S2O3 + I2 ¾¾ ® Na2S4O6 + 2NaI
and CO2 are removed by solidification. The remaining Sod. thiosulphate Sod. tetrathionate
The p-Block Elements (Group 15, 16, 17 and 18) 519

56. (c) SO2 is highly soluble in water and therefore cannot be is least stable. The decreasing stability of the hydrogen
collected over water. halide is also reflected in the values of dissociation
In alkaline energy of the H–X bond
57. (b) Na2SO3 + S ¾¾ ¾ ¾
¾® Na S O
2 2 3 medium H-F H - Cl H - Br H-I
58. (c) In KMnO4 manganese is already present in its highest 135kcal mol-1 103kcal mol-1 87 kcal mol-1 71kcal mol-1
possible oxidation state i.e. +7.So no further oxidation 71. (b) Cl 2 + 2NaOH ® NaCl + NaClO + H2O
is possible. (cold & dil)
59. (a) Laughing gas (N2O) is prepared by heating mixture of 3Cl2 + 6NaOH ® 5NaCl + NaClO3 + 3H2O
NH4Cl and NaNO3 (hot & conc.)
D
NH 4 Cl + NaNO3 ¾¾® NH 4 NO3 + NaCl 72. (b) H 2SO4 + SO3 ¾¾
® H 2S2 O7
Oleum
D
NH 4 NO3 ¾¾® N 2 O + 2H 2 O 73. (d) I2 + 10HNO3 ¾¾
® 2HIO 3 + 10NO 2 + 4H2O
60. (d) N 2O5 is colourless deliquiscent solid. It is highly (Conc) Periodic acid
74. (a) Longer bond length means weaker bond energy i.e.
reactive, a strong oxidizing agent and is light sensitive.
less bond dissociation energy or more reactivity. Larger
It is anhydride of HNO3 . the size of anion more will be bond length thus the
61. (b) Bromide in the mother liquor is oxidised to Br2 by order of reactivity is HI > HBr > HCl.
75. (a) The order of bond energy is
Cl 2 which is a stronger oxidising agent. Cl – Cl > F – F > Br – Br > I – I
2Br - + Cl 2 ® Br2 + 2Cl - The small value of bond energy in F2 than Cl2 (anomaly)
62. (c) Outer electronic configuration of Cl atom is due to smaller size of F atom which leads to
inter-electronic repulsions.
= 3s 2 3 px2 p 2y p1z 76. (b)
Sun light
Outer electronic configuration of Cl– ion = 3s 2 3 px2 p2y pz2 77. (b) SO 2 + Cl2 ¾¾¾¾ ®
Hence Cl– has 4 unshared pair of electrons. SO 2Cl 2 (Sulphuryl chloride)
63. (a) A stronger oxidising agent (Cl 2 ) displaces a weaker 78. (d)
oxidising agent (Br2 ) from its salt solution. 79. (b) CCl3 NO 2
2KBr + Cl 2 ® 2KCl + Br2 80. (b) Polarizibility of noble gases increases down the group
because of increase of distance of electrons from
64. (c) 2AgClO 3 + Cl 2 (dry ) ¾90°C nucleus. Thus He (first member of group) is least
¾¾® 2AgCl + 2ClO2 + O 2
polarized.
65. (b) fast
NH 3 + NaOCl ¾¾¾
® NH 2 Cl + NaOH
81. (c) XeF6 + 3H2O ¾¾ ® XeO3 + 6HF
2NH 3 + NH 2 Cl ¾¾
® NH 2 NH 2 + NH 4 Cl \ Complete hydrolysis of XeF6 gives XeO3 (an
66. (d) We know that positive ion is always smaller and explosive).
negative ion is always larger than the corresponding 82. (b)
atom. Therefore the correct order of the size is 83. (a)
I- > I > I + 84. (c) Completing the reaction, we get
67. (a) Due to absence of reaction between marble and
bromine. NCl3 + 3H 2O ¾¾
® 3HClO+ NH 3
'X'
0 +1 -1 85. (a) Rn because it is radioactive element obtained by the
68. (b) H 2O + Br2 ¾¾
® HOBr + HBr
disintegration of radium
Thus here oxidation number of Br increases from 0 to 226
+1 and also decreases from 0 to –1. Thus it is oxidised 88 Ra ®86 Rn 222 + 2 He4
as well as reduced. 86. (b)
69. (b) 87. (b) PbO2 is a strong oxidising agent. It reacts with conc.
H3PO4
HNO3 as shown:
P2 O5 exists as dimer ( P4 O10 )
1
PbO2 + 2HNO3 ¾¾ ® Pb(NO3 ) 2 + H 2O + O 2
P4 O10 + 6H 2 O ¾¾
® 4H 3 PO 4 2
88. (b) With the decrease in the electronegativity of central atom
70. (c) The H–X bond strength decreases from HF to HI. i.e.
the bond angle decreases
HF > HCl > HBr > HI. Thus HF is most stable while HI
EBD_7327
520 CHEMISTRY

89. (d) Perdisulphuric acid is represented as


O O
Cl2O
2 ClO2 2
ClO2 2

|| || O .. . Cl . .

..
..
H -O -S- O - O -S-O - H Cl
|| ||
.. 110° ..
O O .. 118°
Cl Cl O O O O

..

..
..

..
..
.. ..

..
..

..
..

..
So, it contains a peroxo linkage – O – O –
..
90. (c) Helium is twice as heavy as hydrogen it is inflammable * In ClO2– there are 2 lone pairs of electrons present on
but not lighter than hydrogen. Helium has the lowest
the central chlorine atom. Therefore the bond angle in
melting and boiling point of any element which makes
liquid helium an ideal coolant for many extremely low ClO2– is less than 118° which is the bond angle in ClO2
temperature application such as super conducting which has less number of electrons on central chlorine
magnet and cryogenic research where temperature close atom.
to absolute zero are needed. He is used in gas cooled 98. (d)
atomic reactors as a heat transfer agent.
91. (d) (i) The first ionization energy of xenon (1, 170 kJ mol–1) O
is quite close to that of dioxygen (1,180 kJ mol–1).
99. (b) Ba
(ii) The molecular diameters of xenon and dioxygen are
almost identical.
O
Based on the above similarities Barlett (who prepared
O2+[PtF6]– compound) suggested that since oxygen
combines with PtF6, so xenon should also form similar 100. (b) O
compound with PtF6.
bridging P bridging
92. (b) PbO2 does not contain peroxide ion. It is lead dioxide.
O
93. (d) HO Cl < HO Cl O < HO Cl O 2 < HO Cl O3 O
+1 +3 +5 +7 O
In case of oxyacids of similar element as the oxidation
P O
number of the central atom increases, strength of acid
O P
also increases. O
94. (b, c) From the given options we find option (a) is correct. O P O
bridging
The oxidising power of halogens follow the order
O bridging
F2 > Cl2 > Br2 > I2. Option (b) is incorrect because it in oxygen
not the correct order of electron gain enthalpy of
halogens. \ Correct choice : (b)
The correct order is Cl2 > F2 > Br 2 > I2. The low value
EXERCISE - 2
of F2 than Cl2 is due to its small size.
Option (c) is incorrect. The correct order of bond 1. (b) Potassium tetraiodo mercurate (II) (K2HgI4) dissolves
in KOH solution to give Nessler’s reagent. Nessler’s
dissociation energies of halogens is
Cl2 > Br2 > F2 > I2. reagent is used to test NH +4 ions.
Option (d) is correct. It is the correct order of 2. (a) Acidic strength of oxyacids increases with increase in
oxidation number and on moving from top to bottom in
electronegativity values of halogens. Thus option (b)
a group acidic strength of oxides also decrease due to
and (c) are incorrect. decrease in electronegativity down the group.
95. (c) MI > MBr > MCl > MF. As the size of the anion decreases +5 +3 +5 +3
covalent character also decreases. P4 O10 > P4 O6 > As 4 O10 > As 4O6
96. (d) Since all the halogens have a strong tendency to accept 3. (b) Neon gives a distinct reddish glow when used in either
electrons. Therefore halogens act as strong oxidising low-voltage neon glow lamps or in high voltage
agents and their oxidising power decreases from fluorine discharge tube.
to iodine. 4. (a) Order of dipole moment
NH3 > PH3 > AsH3 > SbH3
97. (c) The correct order of increasing bond angle is
(Based upon electronegativity)
Cl2 O < ClO2- < ClO 2 5. (c) Polarity of the bond depends upon the electronegativity
difference of the two atoms forming the bond. Greater
The p-Block Elements (Group 15, 16, 17 and 18) 521

the electronegativity difference, more is the polarity of angle £ 109°28¢, since the central atoms belong to the
the bond. same group, the bond angle of the chlorides decreases
N – Cl O–F N–F N– N as we go down the group. Thus the order of bond
3.0–3.0 3.5–4.0 3.0–4.0 3.0–3.0 angle is, BCl3 > PCl3 > AsCl3 > BiCl3.
6. (c) Br2 reacts with NaI only to get I2. 23. (a)
2NaI + Br2 ® 2NaBr + I2
7. (d) (A) Sulphuric acid (iv) Contact process
8. (d) Bi forms basic oxides whereas N and P form acidic and (B) Steel (ii) Bessemer’s
As and Sb form amphoteric oxides. process
9. (a) 7N = 1s2 2s2 2p3; 15P = 1s2 2s2 2p6 3s2 3p3 (C) Sodium hydroxide (iii) Leblanc process
In phosphorous the 3d- orbitals are available. Hence
(D) Ammonia (i) Haber’s process
phosphorus can from pentahalides also but nitrogen
cannot form pentahalide due to absence of d-orbitals
10. (c) In N2O (nitrous oxide) two N atoms are covalently O
bonded through triple bond
[ N º N ¾¾® O ] 24. (b) H P H Hypophosphorus acid (H3PO2) is a
11. (a) As size increases, van der Waal's forces of attraction
between noble gas atoms also increases. Consequently, O
ease of their liquefaction increases. H
12. (a) For oxides of same element higher the oxidation state,
more will be acidic character. monobasic acid. i.e., it has only one ionisable hydrogen
+5 +2 +1 atom or one OH is present.
N 2O5 > NO > N 2O
25. (b) Fe 2 (SO4 )3 ¾¾
® Fe2O3 + 3SO3
13. (b) Hypo solution is Na2S2O3 solution which is used in Δ
photography for fixing films & prints. Photographic 26. (d) The products of the concerned reaction react each
emulsions are made of AgBr. After developing, the film other forming back the reactants.
is put into hypo solution. This forms soluble complex
XeF6 + 3H 2 O ¾¾ ® XeO3 + 6HF .
with Ag.
Na2S2O3+Ag Br ® Ag2S2O3 27. (b) The following reaction occurs
Na S O
Na2S2O3 + 4Cl2 + 5H2O ¾¾ ↑ 2 NaHSO4 + 8HCl.
¾¾¾¾¾2 2 3 ® Na [Ag(S O ) ]
5 2 3 3
28. (b) In presence of N
14. (c) As the size of central atom increases the lone pair of P4 + 3O 2 ¾¾¾¾¾¾¾
2® P O
4 6
electrons occupies a larger volume. In other words Here N2 acts as a diluent and thus retards further
electron density on the central atom decreases and oxidation. Reaction of P4 under other three conditions.
consequently its tendency to donate a pair of electrons (a) 2O
P4 + 3O 2 ¾¾
® P4 O 6 ¾¾¾
2® P O
4 10
decreases along with basic character from NH3 to BiH3.
15. (d) H2O is liquid but H2S is a gas. This can be attributed to (c) In moist air, P4O6 is hydrolysed to form H3PO3
the presence of intermolecular hydrogen bonding in P4 O 6 + 6H 2 O ¾¾
® 4H 3 PO 3
case of H2O. (d) In presence of NaOH,
16. (b) Mixture of Cl2 and ClO2 acts as a bleaching agent and P4 + 3OH - + 3H 2O ¾¾
® PH 3 + 3H 2 PO 2-
is called euchlorine.
29. (d) Very pure N2 can be obtainted by thermal decomposition
17. (b) On moving from top to bottom of halogen group the
bond dissociation energy of hydrogen halides of sodium or barium azide. Ba(N 3 ) 2 ¾ ¾ ® Ba + 3N 2
decreases and so the heat of formation of halogen acids 30. (d) XeO2F2 has trigonal bipyramidal geometry, but due to
also decreases. presence of lone pair of electrons on equitorial position,
18. (c) Pickling agent is H2SO4. Pickling is cleaning the surface its actual shape is see-saw.
of metals before enameling, electroplating and F
galvanising. O
19. (c) KF + HF ® KHF2 K+ + [HF2]– Xe
20. (d) Due to inert pair effect. O
V2O 5 F
21. (c) 2SO 2 + O 2 ¾¾ ¾® 2SO 3

1 31. (a) SnO2 is an amphoteric oxide because it reacts with acids


22. (b) In BCl3, H = (3 + 3 + 0 - 0) = 3 ; sp2 hybridization
2 as well as with bases to form corresponding salts.
(bond angle = 120°) similarly PCl3 AsCl3 and BiCl3 are SnO2 + 2H2SO4(conc) ¾¾ ® Sn(SO4)2 + 2H2O
found to have sp3 hybridized central atom with one SnO2 + 2NaOH ¾¾ ® Na2SnO3 + H2O
lone pair of electrons on the central atom. The bond
EBD_7327
522 CHEMISTRY

32. (c) Compound Oxidation number of nitrogen OH OH


N2H4 = –2
NH3 = –3 O P O P O
N3H = –1/3
NH2OH = –1 OH OH
33. (d) Chlorine reacts with excess of ammonia to produce Pyrophosphoric acid
ammonium chloride and nitrogen. HO O OH
3Cl2 + 8NH3 (excess)¾¾ ® 6NH4Cl + N2 P P
O O O
34. (a) (NH4)2SO4 + 2H2O¾¾ ® 2H2SO4 + NH4OH
Metaphosphoric acid
H2SO4 being strong acid increases the acidity of soil. O
In case of potassium nitrate and superphosphate of
lime on hydrolysis strong base also form along with HO P OH
strong acid on hydrolysis results into neutralisation.
F OH
Orthophosphoric acid
39. (c) The structure of solid sulphur trioxide is complex, it
possesses either cyclic trimer structure or an infinite

35. (c) XeF2 : Xe helical chain made up of linked SO4 tetrahedron.


O O
S

F O
O O
O
3lp S S

F F O O O
total no of S - O - S bonds = 3
40. (a) NF 5 does n ot exist because N does not form
XeF4 : Xe pentahalides due to the absence of d-orbital in its
valence shell. While P, As and Sb form pentahalides of
F F the general formula MX5 (where, M = P, As and Sb) due
to the presence of vacant d-orbitals in their respective
2lp valence shell.
41. (c) Compound O.S. of N
F N2O +1
F NO +2
NO2 +4
NO3– +5
F Xe F
XeF6 : NH4+ –3
Therefore increasing order of oxidation state of N is:
F
F NH +4 < N2O < NO < NO2 < NO3- .
42. (c) Br2 reacts with NaI only to get I2.
2NaI + Br2 ® 2NaBr + I2
Xe 43. (c) In N2O (nitrous oxide) two N atoms are covalently
bonded through triple bond
[ N º N ¾¾® O ]
XeO3 : O O 44. (b) Potassium tetraiodo mercurate (II) (K2HgI4) dissolves
O in KOH solution to give Nessler’s reagent. Nessler’s
1lp reagent is used to test NH +4 ions.
Hence XeF2 has maximum no. of lone pairs of electrons. 45. (c) The geometry of XeF6 is distorted octahedral in which
36. (d) Oxidation state of sulphur varies from – 2 to + 6 in its all the six positions are occupied by fluorine atoms and
various compounds. the lone pair of electrons of Xe atom is present at the
37. (b) With the decrease in the electronegativity of central corner of one of the triangular faces.
atom the bond angle decreases F
38. (a) O O F F

HO — P — P — OH Xe
HO OH
F F
Hypophosphoric acid F
The p-Block Elements (Group 15, 16, 17 and 18) 523

46. (b) Neon gives a distinct reddish glow when used in either
low-voltage neon glow lamps or in high voltage 5. (a) Both CO32 - and NO3- have same number of electrons.
discharge tube.
47. (b) Formula of cyclotrimetaphosphoric acid is (HPO3)3 CO32 - = 6 + 8 × 3 +2 = 32
Oxidation state of ‘P’ is 3(+ 1 + x + 3 (– 2)) = 0
x+ –6+1 =0 Þ x= +5 NO3- = 7 + 8 × 3 + 1 = 32
48. (b) White phosphorus exists as P4 tetrahedral molecule
Hence, CO32 - and NO3- are isoelectronic. Also, both
having P-P-P bond angle 60º. Hence the molecule is
under strain and more reactive. On the other hand red have same type of hybridization (sp2) and have trigonal
phosphorus exists as P4 tetrahedra which are joined planar structure. Hence, they are also isostructural.
together through covalent bonds giving polymeric 6. (a) On moving down the group size of halogen atom
structure. increases hence the H–X bond length increases. As a
49. (a) Both assertion and reason are true and reason is the result, bond dissociation enthalpy decreases.
correct explanation of assertion. Hence, the correct order of bond enthalpy is :
M + HNO3 ¾¾
® MNO3 + H
H – F > H – Cl > H – Br > H – I.
(metal) (conc.) (metal nitrate) (nascent hydrogen) 7. (d) On moving down the group size of central atom
2HNO3 + 2H ¾¾
® 2NO2 + 2H 2 O
increases thus bond length of E—H bond increases
(nascent hydrogen) and bond dissociation energy decreases. Hence,
50. (c) Bond angle of H2S (92°) < H2O (104°31). As the reducing nature increases in the order : NH3 < PH3 <
electronegativity of the central atom decreases, bond AsH3 < SbH3
angle decreases. In the present case, S is less 8. (c) White phosphorus on reaction with NaOH solution in
electronegative than oxygen. Thus bond pairs in H2S the presence of inert atmosphere of CO2 produces
are more away from the central atom than in H2O and phosphine gas which is less basic than NH3.
thus repulsive forces between bond pairs are smaller
producing smaller bond angle. P4 + 3NaOH + 3H 2 O –® PH3 + 3 NaH 2 PO 2
(Sodium hypophosphite)
EXERCISE - 3
9. (c) Structure of H3PO4 is
Exemplar Questions O
1. (c) Hydrogen iodide (HI) is stronger reducing agent than ||
H2SO4. Hence, it reduces H2SO4 to SO2 and itself P
oxidises to I2. HO OH
OH
H2SO4 + 2HI –® SO2 + I2 + 2H2O H3PO4 has 3 – OH groups i.e., three ionisable H-atoms
(Violet
colour) and hence it forms three series of salts: NaH2PO4,
Na2HPO4 and Na3PO4
2. (b) When H2S gas is passed through an aqueous solution
10. (c) The acids which contain P–H bond have strong
of copper sulphate acidified with dil. HCl a black ppt.
reducing properties. Thus, H3PO2 is a strong reducing
of CuS is obtained.
agent due to the presence of two P — H bonds and
dil. HCl one —OH group
CuSO 4 + H 2S ¾¾¾¾
® CuS + H 2SO 4
black ppt O
On boiling CuS with dil. HNO3 it forms a blue coloured ||
solution and the following reactions occur P H
3CuS + 8HNO3 –® 3Cu(NO3)2 + 2NO + 3S + 4H2O H OH
Hypophosphorus acid
2+
+
Cu 2aq + 4NH3 ( aq ) ® éë Cu ( NH3 ) 4 ùû 11. (b) On heating, lead nitrate produces brown coloured
( ) nitrogen dioxide (NO2) and lead (II) oxide.
(Deep blue solution)
D
3. (c) 2Pb ( NO3 )2 ¾¾® 4NO2 + 2PbO + O 2
O O H
P 12. (a) Nitrogen does not show allotropy due to its small size
O O O and high electronegativity. The N–N bond is weak due
O
to high inter–electronic repulsions among non-bonding
P P electrons due to the small bond distance. Hence it does
H O O O H not show allotropy.
i.e, 3 double and 12 single Bonds 13. (c) Maximum covalency of nitrogen is 4 in which one
4. (c) Among carbon, nitrogen, phosphorus and boron only electron is from s-orbital and 3 electrons are from
phosphorus has vacant d-orbital h ence only p-orbitals. Hence, total four electrons are available for
phosphorus has the ability to form pp – dp bonding. bonding.
EBD_7327
524 CHEMISTRY

14. (a) The single N — N bond is weaker than the single Cl2 on further treatment with NH3 produces NCl3.
P — P bond. This is why phosphorus show allotropy -3 +3
but nitrogen does not. N H3 + 3Cl2 –® NCl3 + 3HCl
15. (a) When freshly prepared solution of FeSO4 is added to Hence, NH3 (– 3) changes to NCl3 (+ 3).
an aqueous solution containing NO3- ion, it leads to
the formation of a brown coloured complex. This is 24. (c) Bertlett had taken O+2 Pt F6- as a base compound
known as brown ring test for nitrate ion. because O2 and Xe both have almost same ionisation
enthalpy.
NO3- + 3Fe2+ + 4H+ –® NO + 3Fe3+ + 2H2O
25. (d) In solid state PCl5 exists as an ionic solid with the
2+ 2+ cation [PCl 4]+ (tetrahedral) and the anion [PCl6]–
éFe ( H2 O) ù + NO –® éëFe ( H2 O )5 ( NO ) ùû + H2O
ë 6û (octahedral).
Brown ring +
Cl Cl
16. (b) The only well characterised compound having + 5 Cl Cl
oxidation state of Bi is BiF5. It is due to smaller size P P
and high electronegativity of fluorine. Cl Cl Cl
Cl
17. (a) On heating ammonium dichromate and barium azide Cl Cl
+
both produces N2 gas separately. [PCl6]

[PCl4]
( NH 4 )2Cr2 O 7 ¾¾
D
® N 2 + 4H 2 O + Cr2 O3
26. (c)
octahedral tetrahedral
Greater the SRP value, higher will be the oxidising
Ba ( N3 )2 –® Ba + 3N2 power. Hence the correct order of oxidising power is:

Pt / Rh gauge catalyst
BrO4- > IO-4 > ClO4-
18. (a) 4NH 3 + 5O 2 ¾¾¾¾¾¾¾¾® 4NO(g)+ 6H2O (l) 27. (b) Isoelectronic pair have same number of electrons
500K, 9 bar
\ Two moles of NH3 will produce 2 moles of NO on Total number of electrons for the given compounds :
catalytic oxidation. BrO2– BrF2+
19. (c) Let oxidation state of P in NaH2PO2 is x. = = 52 = = 52
1+2×1+x+2×–2=0 35 + 2 ´ 8 + 1 35 + 9 ´ 2 – 1
1+2+x–4=0 ICl 2 ClO2 BrF
+x–1=0
53 + 2 ´ 17 = 87 17 + 16 = 33 35 + 9 = 44
x=+1
20. (c) SF4 has sea-saw shape as shown below: CN – O3
6 + 7 + 1 = 14 8 ´ 3 = 24
.. NEET/AIPMT (2013-2017) Questions
28. (b) HClO4 is the strongest acid amongst all because the
F S F oxidation state or Cl is maximum (+7).
D
F F 29. (c) (NH4)2Cr2O7 ¾¾® N2 + Cr 2O3 + 4H2O
It has trigonal bipyramidal geometry having sp3 d D
Zn(ClO3)2 ¾¾® ZnCl2 + 3O2
hybridisation.
D
21. (a) Peroxoacids of sulphur must contain one — O — O — 2 KClO3 ¾¾® 2KCl + 3O2
bond as shown below: D
4K2Cr2O7 ¾¾® 4K2CrO4 + 2Cr2O3 + 3O2
O O O

S S S 30. (b) (Tetrahedral sp3)


OH O O
H O O O
O O OH OH 31. (a) The weaking of M—H bond with increase in size of M
H2SO5 H2S2O8 (where M = S, Se, Te) explains the acid character of
22. (c) Carbon on oxidation with H2SO4 produces two types hydrides. Since on moving down the group atomic size
of oxides CO2 and SO2. increases hence bond length increases and hence
C + 2H2SO4 (conc) –® CO2 + 2SO2 + 2H2O removal tendency of H also increases.
23. (a) MnO2 reacts with HCl to produce greenish yellow 32. (c) SO2 is widely used in food and drinks industries for its
coloured gas of Cl2. property as a preservative and antioxidant while NO2 is
MnO2 + 4HCl –® MnCl2 + 2H2O + Cl2 not used as food preservative.
( Black ) (greenish
yellow gas)
The p-Block Elements (Group 15, 16, 17 and 18) 525

33. (d) OF 2 ; among the following O and F, F is more O


electronegative than oxygen. F F F F

So OF2 cannot be called oxide because in that case Xe Xe


fluorine is in +1 oxidation state which is not possible, so F F
F F
OF2 is called oxygen difluoride.
2lp
34. (b) Th e H-bonding is present in HF due to high
electronegativity of fluorine atom. While H-bonding is Square pyramidal Square planar
not present in HI, HBr and HCl. 39. (b) Phosphinic acid as shown in structure below has one
35. (a) The acids which contain P-H bond have strong reducing P—OH bend thus it is monobasic or monoprotic
properties. Thus H3PO2 acid is good reducing agent as O
it contains two P–H bonds and reduces, for example,
AgNO3 to metallic silver. P
H OH (Monoprotic)
4 AgNO3 + 2H2O + H3PO2 —® 4Ag + 4HNO3 + H3PO4 H
36. (b) Acidic strength increases as the oxidation number of Phosphonic acid as shown in structure has two P–OH
central atom increases. bonds thus it is dibasic or diprotic
HClO < HClO2 < HClO3 < HClO4
O
+1 +3 +5 +7
37. (b) Bond dissociation enthalpy decreases as the bond P
H (Diprotic acid)
distance increases from F2 to I2. This is due to increase OH
OH
in the size of the atom, on moving from F to I.
F – F bond dissociation enthalpy is smaller then Cl – Cl 40. (a) (Bonus)
and even smaller than Br – Br. This is because F atom is CaC2 + N2 ® Ca(CN)2 + C
very small and hence the three lone pairs of electrons 41. (a) XX' ® Linear (e.g. ClF, BrF)
on each F atom repel the bond pair holding the XX3' ® T-Shape (e.g. ClF3, BrF3)
F-atoms in F2 molecules. XX5' ® Square pyramidal (e.g. BrF5 IF5)
The increasing order of bond dissociation enthalphy is XX7' ® Pentagonal bipyramidal (e.g. IF7)
I2 < F2 < Br2 < Cl2 O O O
38. (a) XeF6 XeO3 –
F 42. (a) O – S – S – S – S – O S
F O O S O
O
S4 O6 2 - S2 O32 -
Xe
F Xe F
O O
O
F F
1lp
distorted octahedral Pyramidal
XeOF4 XeF4
EBD_7327
526 CHEMISTRY

The d- and f-Block


22 Elements
THE TRANSITION ELEMENTS (d-BLOCK) (ii) Melting and boiling points :
Position in the Periodic Table Strong metallic bonds between the atoms of these elements
The elements which lie in between s and p - block elements are are responsible for the high melting and boiling points. This
called transition elements. They are called d - block elements since is clear from their high enthalpies of atomization. (i.e., heat
in them, 3d, 4d, 5d and 6d sub - shells are incomplete and the last required to break the metal lattice to get free atoms)
electrons enters the (n – 1) d subshell. A transition element is Metals of 4d and 5d series have greater enthalpies of
defined as an element whose atoms in ground state or ions in one atomisation than 3d series due to much more frequent
of the common oxidation states, has incomplete d - subshell. metal-metal bonding in their compounds. Greater is the number
Classification of d-Block Elements of unpaired d-electrons stronger will be metallic bonding.
Transition elements consist of the following four series Thus in a particular series metallic strength increased up to
middle and then decreases.
(i) 3dseries 21 Sc to 30 Zn 3d1-10 4s1- 2
Note : In Zn, Cd, and Hg there is no unpaired electron present
(ii) 4d series 39 Y to 48Cd 4d 1-105s0 - 2 in d-orbital, so the metallic bond is weak and their m.pt. and
(iii) 5d series 57 La 72 Hf to 80 Hg 5d1-10 6s1- 2 b.pt. are very low. (Volatile metals Zn, Cd, Hg)
(iv) 6d series 89 Ac 104 Rf to 111Rg (iii) Density
Electronic Configuration of d-Block Elements The atomic volume of the transition elements are low compared
with s-block, so their density is comparatively high.
General electronic configuration of these elements is
(n – 1) d1–10 ns1–2. There is a normal increase in density from 3d to 4d series.
The last shell electronic configuration of Cr and Cu are However from 4d to 5d, it becomes almost double due to
3d54s1 (instead of 3d4 4s2) and 3d10 4s1 (instead of 3d9 4s2). This lanthanoid contraction.
is because of extra stability associated with half and completely In 3d series
filled orbitals. Sc ® Cr density increases
The electronic configurations of Zn, Cd and Hg are represented Cu ® Zn decreases
by general formula (n – 1) d10 ns2. The orbitals in these elements, (iv) Atomic and ionic sizes
in ground as well as in common oxidation state, are completely They lie between those of s - and p - block elements. Atomic
filled and hence, they are not regarded as transition elements but
radii in a series decreases with increase in atomic number but
they show properties of transition elements to appreciable extent.
the decrease is small after mid-way. This is because in the
General Properties of Transition Elements begining, the nuclear charge increases but screening effect
Since only penultimate shell i.e., (n – 1) is expanding, they have of d-electron is less. After mid-way screening effect of
resemblance in their physical and chemical properties. d-electrons increases which counter balances increased
(i) All the d-block elements behave as metals. nuclear charge and hence increase in size is not much.
They are hard, malleable and ductile. i.e., Cu, Ag and Au are At the end of period, there is a slight increase in atomic radii.
most ductile. They are good conductors of heat and electricity This is because near the end of series, the increased e– – e–
(due to free electrons).
repulsions between added electrons in same orbital are greater
Explanation :
than attractive forces due to increased nuclear charge.
The electronic configuration of these elements reveals that
they have one or more unpaired electrons present in either Amongst the dipositive ions of 3d series, Cu2+ is the smallest
ns or (n – 1) d-orbitals which are available for bond formation. in size.
In general, greater the number of such electrons available, The elements of 4d and 5d series have almost similar atomic
more will be the chances of their mutual combination and radii. This is due to Lanthanoid contraction. E.g. atomic radii
more will be strength of the metallic bond. of Zr ; Hf; Tc ; Re; Nb ; Ta; Ru ; Os etc.
The d- and f-block Elements 527

(v) Ionisation enthalpies (vi) Oxidation states


The IE increases with increase in atomic number due to Transition metals exhibit a large no. of O.S. This is because
increased nuclear charge. However, some irregularities are there is little difference in energies of (n – 1) d and ns orbitals
observed. This is because removal of an electron alters the and hence, both the levels can be used for bond formation.
relative energies of 4s and 3d orbitals. Thus, there is a Most common O.S. of first row transition metals is +2 due to
reorganisation energy accompanying ionisation. This results loss of two ns2 electrons (exception: Sc, it has + 3). In +2 and
into the release of exchange energy which increases as the +3 O.S, ionic bonds are formed. In higher O.S., bonds formed
number of electrons increases in the d n configuration and are covalent as they are formed by sharing of d-electrons.
also from the transference of s-electrons into d-orbitals. The elements which show maximum no. of O.S. occur in or
Difference between IE of any two successive d-block near middle of series. For ex: Mn ® + 2 to + 7. Highest O.S. is
elements is very less as compared to s or p-block elements. It + 8 (shown by osmium). In a group, higher O.S. are more
is because addition of d electrons in last but one [(n – 1) or stable for heavier elements. For ex: in group 6, Mo (VI) and W
penultimate] shell with increase in atomic number provides a (VI) are found to be more stable than Cr (VI). This is why Cr
screening effect and thus shields the outer s electrons from (VI) in Cr2O72– is a stronger oxidizing agent in acidic medium
inward nuclear pull. whereas MoO3 and WO3 are not.
The 1st IE of Zn, Cd and Hg are very high due to fully filled Low O.S. are found when a complex compound has ligands
(n – 1) d10 ns2 configuration. capable of p - acceptor character in addition to s – bonding.
The order for IE2 for 1st series is: The compounds of metals with F and O exhibit highest O.S.
23V < 24Cr > 25Mn and 28Ni < 29Cu > 30Zn as F and O are small in size and highly electronegative. O
This is because after removal of one e–, Cr and Cu acquire stabilizes the highest O.S. even more than F. This is due to
stable configuration (d5 and d10), so removal of 2nd e– the ability of O to form multiple bonds with metal atoms.
becomes difficult. Usually transition metal ions in their lower oxidation state
Third IE of Mn (25, 3d5 4s2) is very high as the third e– has behave as reducing agents and in higher oxidation state, they
to be removed from stable half - filled 3d – orbital. behave as oxidising agents.
Third IE for change from Fe2+ to Fe3+ (26Fe, 3d6 4s2) is small E.g. : Ti+2, V+2, Fe+2, Co+2 etc are reducing agents
because loss of third e– gives stable configuration of 3d5. Cr+6, Mn+7, Mn+4 Mn+5, Mn+6 etc are oxidising agents.
The high values of third IE for Cu, Zn and Ni explain why (vii) Standard electrode potentials (E°) and chemical reactivity
they have a maximum O.S. of +2.
Thermodynamic stability of compound of transition metals
The first IE’s of 5d elements are higher as compared to those
can be evaluated in terms of its IE. Smaller the IE of metal,
of 3d and 4d elements. This is because of the weak shielding
stabler is its compound. In solution, the stability of the
effect of electrons present in 4f-orbitals which results in
compounds depends upon electrode potentials which
greater effective nuclear charge. Thus in 5d elements outer
depends on Dsub H, IE and Dhyd H.
valence electrons are more tightly held.
The lower the electrode potential, i.e., more negative the standard reduction potential of the electrode, more stable is the oxidation
state of the transition metal ion in aqueous medium.

Element ® Ti V Cr Mn Fe Co Ni Cu Zn
E°(M 2+
/ M) in volts (V) - 1.63 - 1.18 - 0.91 - 1.18 - 0.44 - 0.28 - 0.25 + 0.34 - 0.76

E°(M 3+
/ M 2+ ) in volts (V) - 0.37 - 0.26 - 0.41 + 1.57 + 0.77 + 1.97 - - -

The irregular trend is due to variation in ionization energies E° (Mn3+ / Mn2+) is due to stable d5 configuration of Mn 2+.
and sublimation energies. E° values along the series becomes Low value of E°(Fe3+ / Fe2+) is due to extra stability of
less negative generally it is due to increase in sum of IE1 and Fe3+ (d5). Low value for V is due to stability of V2+ due to its
IE2. 3
half filled t 2g configuration. E° values for the redox couple
Cu has a +ve value of E° (M2+ / M). As a result, it does not
liberate H2 gas from acids. M3+ / M2+ indicate that Mn 3+ and Co3+ ions are the
The E° (M2+ / M) values for Mn, Ni and Zn are more –ve strongest oxidising agents in aqueous solution whereas Ti2+,
than expected. This is due to greater stability of half-filled V2+ and Cr 2+ are the strongest reducing agents and can
3d-orbital in Mn2+ (d5) completely filled 3d-orbital in Zn2+ liberate hydrogen from a dilute acid, Ex :
(d10) and exceptional behaviour of Ni is due to high –ve 2Cr 2+ ( aq ) + 2H + ( aq ) ¾¾
® 2Cr 3+ ( aq ) + H 2( g )
enthalpy of hydration for Ni2+. (viii) Magnetic properties
A very low E° (Sc3+ / Sc2+) value reflects the stability of Transition metals contain unpaired electrons in (n – 1) d orbitals
Sc3+ which has noble gas configuration. The highest and hence most of transition metal ions and their compounds
E° (M3+ / M2+) for Zn is due to high stability of Zn2+ with are paramagnetic.
d 10 configuration. Comparitively high value of
EBD_7327
528 CHEMISTRY

Transition metals which have paired electrons are Some important alloy :
diamagnetic. Magnetic moment of transition metal ions is 1. Bronze – Cu (75 - 90 %) +Sn ( 10 - 25 %)
calculated by using ‘spin - only’ formula given as: 2. Brass – Cu ( 60 - 80 %) +Zn (20 - 40 %)
m = n ( n + 2 ) B. M. where ‘n’ is the no. of unpaired 3. Gun metal – (Cu + Zn + Sn) (87 : 3 : 10)
electrons. As the number of unpaired electrons increases the 4. German Silver – Cu + Zn + Ni ( 2 : 1 : 1)
magnetic moment also increases. Ions with d5 configuration 5. Bell metal – Cu (80%) + Sn (20%)
have maximum magnetic moment. 6. Nichrome – (Ni + Cr + Fe)
(ix) Coloured ions Some Important Compounds of Transition Metals
Most transition metal compounds are coloured both in solid Oxides and oxoanions
and in aqueous solution. Colour is due to presence of Oxides are formed by reaction of O 2 with metals at high
incomplete d - subshell. The d - orbitals split into two sets, temperatures. They are formed in O.S. from +1 to +7 As oxidation
one with lower energy and other with higher energy. The e– number of metal increases, ionic character decreases.
absorbs energy and gets excited to higher energy level. When e.g., MnO, Mn O , Mn O , MnO , Mn O
this excited e– falls back to the ground state, it emits radiation 3 4 2 3 2 2 7
which falls in the visible range of spectrum. +2 + 8 / 3 + 3 +4 +7
Thus, colour of transition metals is due to d-d transitions. Ionic character decreases
Sc3+ and Ti4+ have empty d - orbitals and are colourless. The oxides in lower OS of metals are basic and in higher OS they
Cu+, Ag+, Au+, Zn2+, Cd2+, Hg2+ have completely filled are acidic whereas in intermediate OS, they are amphoteric.
d-orbitals, there are no vacant d - orbitals for promotion of For ex:
electrons, hence they are also colourless. (i) MnO Mn 3O 4 Mn 2 O 3 MnO 2 Mn 2 O7
(x) Complex formation Basic Amphoteric Amphoteric Amphoteric Acidic
Transition metal ions form a large no. of complex compounds
because:
(\ Mn 2O7 + H 2O )
(a) of comparatively smaller size of their metal ions. HMnO4
(b) of their high positive charge. (ii) V2O3 V2O 4 V2 O5
(c) of availability of vacant d - orbitals so that they can basic less basic amphoteric (mainly acidic)
accept lone pair of e–1 s donated by ligands. (\ V 2 O 5 r eacts with alkalies and acids to give
(xi) Catalytic properties
VO34- and VO+4 )
Transition metals act as catalysts due to following reasons:
(a) presence of unpaired electrons in their incomplete (iii) CrO Cr2 O3 CrO3
d-orbitals and posses the capacity to absorb and re-emit basic amphoteric acidic
wide range of energies.
(CrO3 dissolves in water to give the acids H2 CrO4 and
(b) Transition metals exhibit variable oxidation states and
H2Cr2O7)
may form intermediate compounds with one of reactants
providing a new path with lower activation energy. Potassium dichromate (K2 Cr2 O7)
(c) Transition metals provides a suitable large surface are Preparation : It is prepared from ore called chromite or ferrochrome
with free valencies on which reactants are adsorbed. or chrome iron, FeO. Cr 2O3. The various steps involved are:
Note : When transition elements and their compounds are in (i) 4FeCr2 O 4 + 8 Na 2 CO3 + 7O 2 ¾¾ ®
powdered state, their catalytic behaviour increases. This is 8Na 2 CrO 4 + 2Fe2 O3 + 8CO2
due to greater surface area available in the powdered state.
(xii) Interstitial compounds (ii) 2Na 2 CrO4 + H 2SO4 ¾¾
®
Sodium chromate
Transition elements form interstitial compounds with smaller
sized non metal elements like hydrogen, carbon, boron, Na 2 Cr2 O 7 + Na 2SO 4 + H 2O
nitrogen etc. The smaller sized atoms get entrapped in between (iii) Na 2 Cr2O7 + 2KCl ¾¾
® K 2 Cr2 O7 + 2NaCl
the interstitial spaces of the metal lattices. These interstitial Sodium dichromate Potassium chromate
compounds are non stoichiometric in nature and hence cannot Properties:
be given any definite formula. (i) It forms orange crystals which melt at 669 K.
Some important characteristics of these compounds are: (ii) It is moderately soluble in cold water and freely soluble in
(a) they have high m. pts. higher than those of pure metals. hot water.
(b) they are very hard and rigid. (iii) When heated, it decomposes with the evolution of oxygen.
(c) they show conductivity like that of pure metal. D
4K 2Cr2 O7 ¾¾® 4K 2CrO4 + 2 Cr2O3 + 3O2
(d) they are chemically inert.
(iv) The chromates and dichromates are interconvertible in
(xiii)Alloy formation aqueous solution depending on pH of the solution.
Alloys are homogeneous solid solutions of two or more metals
obtained by melting the components and then cooling the 2CrO4 2 - + 2H + ¾¾
® Cr2 O 27 - + H 2O
melt. CrO 27 - + 2OH - ¾¾
® 2CrO 24 - + H 2 O
Transition metals have similar atomic radii and other
At pH = 4
characteristics, hence they form alloys readily. Alloys are ˆˆˆˆˆˆ† 2CrO24 - + 2H +
Cr2 O27 - + H 2O ‡ˆˆˆˆˆˆ
generally harder, have higher m. pts. and more resistant to orange yellow
corrosion than individual metals.
The d- and f-block Elements 529

(v) Na2Cr2O7 and K2Cr2O7 are strong oxidising agents. In acidic Potassium permanganate (KMnO4)
medium, it furnishes 3 atoms of available oxygen as shown Preparation
by the equation : (i) Conversion of MnO2 into potassium manganate :
K 2 Cr2 O 7 + 4H 2SO 4 ¾¾
® Pyrolusite is fused with KOH or K2CO3 in presence of air or
K 2SO 4 + Cr2 (SO 4 )3 + 4H 2 O + 3O
oxidising agent (i.e., KNO3 or KClO3)
2MnO2 + 4KOH + O 2 ¾¾ ® 2K 2MnO 4 + 2H 2O
or Cr2 O 27 - + 14H + + 6e - ¾¾
® 2Cr 3+ + 7H 2O MnO 2 + 2KOH + KNO3 ¾¾ ® K 2 MnO 4 + KNO 2 + H 2 O
It oxidises Oxidation of K2MnO4 to KMnO4 :
(a) Chemical oxidation :
2I - ¾¾
® I 2 + 2e -
3K 2 MnO 4 + 2CO 2 ® 2KMnO4 + MnO 2 ¯ +2K 2CO3
Fe 2 + ¾¾
® Fe3+ + e - 2K 2 MnO 4 + Cl 2 ® 2KMnO 4 + 2KCl
+ -
3H 2S ¾¾
® 6H + 3S + 6e 2K 2 MnO 4 + H 2 O + O 3 ® 2KMnO 4 + 2KOH + O 2
(b) Electrolytic oxidation : The alkaline manganate solution
SO 32 - + H 2 O ¾¾
® SO 42 - + 2H + + 2e -
is electrolysed between iron electrodes :
NO 2- + H 2 O ¾¾
® NO 3- + 2e - + 2H + ˆˆ† 2K + + MnO 24 -
K 2 MnO 4 ‡ˆˆ
® SO 24 - + 2H + + 2e -
SO 2 + 2H 2 O ¾¾ ˆˆ† H + + OH -
H 2O ‡ˆˆ
At anode : MnO 24 - ¾¾
® MnO -4 + e -
Sn 2 + ¾¾
® Sn 4 + + 2e -
At cathode : H + + e- ¾¾
®H
C 2 O 24 - ¾¾
® 2CO 2 + 2e -
2H ¾¾ ® H2
® O 2 + 2H + + 2e -
H 2 O 2 ¾¾ (ii) Laboratory method :
Uses - 2Mn 2+ + 5 S2O82- + 8H 2O ¾¾
®
(i) In chrome tanning,
(ii) In dyeing-calico printing, 2MnO -4 + 10 SO 42 - + 16H +
(iii) In photography Properties
(iv) Chromic acid ( mixture of K 2 Cr2 O 7 + H 2 SO 4 ) used as (i) It exists as deep purple black prisms with greenish lustre
which become dull in air due to superficial reduction.
cleaning agent,
(ii) It is moderately soluble in H2O at room temperature.
(v) In preparation of compounds such as
(iii) When heated, it decomposes at 513 K
K 2 SO 4 .Cr2 ( SO 4 )2 .24H 2 O, CrO 2 Cl 2 etc. 2KMnO 4 ¾¾ ® K 2 MnO 4 + MnO 2 + O 2
Structures of chromate and dichromate ions : At red heat.
2– red
O O 2K 2 MnO 4 ¾¾¾
® 2K 2 MnO3 + O3
heat
(iv) It is a powerful oxidising agent (in neutral, alkaline and acidic
Cr or Cr medium)
– – In neutral medium, the reaction is
O O O O
O O MnO 4- + 2H 2O + 3e - ¾¾
® MnO 2 + 4OH -
Chromate ion – Tetrahedral structure In alkaline medium:
2– MnO -4 + e - ¾¾
® MnO 24 -
O O O O
I - + 6OH - ® IO3- + 3H 2 O + 6e -
115º
NO 2
+ 7OH -
NO 2
Cr Cr or Cr Cr C6 H 4 ® C6 H 4 + 5H 2 O + 6e -
CH3 COO -
– –
O O O O O O In acidic medium :
O O O O
Dichromate ion – Two tetrahedra showing one oxygen atom MnO-4 + 8H + + 5e - ¾¾
® Mn 2+ 4H 2 O
at one corner S 2- ¾¾® S + 2e -
Note :
SO 2 + 2H 2 O ¾ ¾® SO 24 - + 4H + + 2e -
(i) OS of Cr in chromate and dichromate is same, i.e., +6.
(ii) It is used in the detection of Cl – in qualitative analysis in the SO 32 - + H 2 O ¾
¾® SO 24- + 2H + + 2e -
-
chromyl chloride test. NO 2 + H 2 O ¾ ¾® NO3- + 2H + + 2e -
2+
(iii) Na2Cr2O7 is more soluble in water than K2Cr2O7. Fe ¾ ¾® Fe 3+ + e -
(iv) Na 2 Cr 2 O7 is not used in volumetric analysis as it is C 2 O 4 2- ¾¾® 2CO 2 + 2e -
deliquescent H 2O2 ¾ ¾® O 2 + 2 H + + 2e -
EBD_7327
530 CHEMISTRY

Note : Volumetric titrations involving KMnO4 are carried out General characteristics
only in presence of H2SO4 not HCl or HNO3. This is because (i) They are silvery white soft metals and tarnish rapidly in air.
oxygen produced from KMnO4 + dil. H2SO4 is used only for (ii) They have high densities and melting points.
oxidising the reducing agent. Moreover, H2SO4 does not (iii) They have typical metallic structure and are good conductors
give any oxygen of its own to oxidize the reducing. In case of heat and electricity.
HCl is used, the oxygen produced from KMnO4 + HCl is (iv) The lanthanoid ions have unpaired electrons in their
partly used up to oxidise HCl to chlorine and in case HNO3 is incomplete 4f orbitals. Thus these electrons absorbs energy
used, it itself acts as oxidising agent and partly oxidizes the in visible region of light and undergo f-f transition and hence
reducing agent. exhibit colour. The colour exhibited depends on the number
of unpaired electrons in the 4f orbitals. Lanthanoid ions having
THE INNER TRANSITION ELEMENTS (f-BLOCK) 4f 0, 4f 7 and 4f 14 are colourless.
The elements in which the last e– enters the anti - penultimate (v) Their ions [except La 3+ (f0 ) and Lu 3+ (f14 )] show
energy level, i.e., (n – 2) f-orbitals are called f - block elements. paramagnetism due to unpaired electrons in 4f - subshell.
They are also called inner transition elements. (vi) They have low IE and are highly electropositive. They readily
Their general electronic configuration is : loose e–1 s and are thus good reducing agents.
(n – 2) f1–14 (n – 1) d0–1 ns2. (vii) Because of their large size and low charge density they do
The f - block consists of two series, lanthanoids and actinoids. not have much tendency to form complexes.
The Lanthanoids (Ln) (viii) Chemical behaviour
Halogen
Electronic configurations LnX3
They have electronic configuration with 6s2 common but with
variable occupancy of 4 f-level. The electronic configuration of C, 2773 K
Ln3C, Ln2C3 and LnC2
their tripositive ions are of the form 4f n.
Atomic and ionic sizes dilute acids
Ln liberate H2 gas
In lanthanoids with increasing atomic number, there is a
progressive decrease in the atomic as well as ionic radii. This burn in O2
regular decrease in size with increasing atomic number is called Ln2O3
lanthanoid contraction. It arises due to imperfect shielding of the
electrons by the f - subshell. D, N2 LnN
In the lanthanoid series with increasing atomic number, there is a
progressive decrease in the size from lanthanum to lutetium or D, S Ln2S3
from La+3 to Lu+3. This contraction in size is known as lanthanoid
contraction. H2O
Cause of lanthanoid contraction : Ln(OH)3+ H2
As we move along lanthanoid series, the nuclear charge increases Uses of lanthanoids
by one unit at each element and is added into same sub-shell (4 f ). (i) Lanthanoids are used in production of alloy steels for plates
Due to shape of f -orbitals, there is imperfect shielding of one and pipes. Ex: misch metal (Ln metal (95%), iron (5%) and
electron by another. This imperfect shielding is unable to traces of S, C, Ca and Al. Misch metal is used in making
counterbalance the effect of increased nuclear charge, thus magnesium based alloys, bullets, shells and lighter flints.
resulting in contraction in size. (ii) Their oxides are used in glass industry, for polishing glass
and for making coloured glasses for goggles as they give
Consequences of lanthanoid contraction
protection against UV light.
(i) Difficulty in the separation of lanthanoids due to very small (iii) Mixed oxides of lanthanoids are used as catalysts in
change in their atomic radii petroleum cracking.
(ii) Similarity in size of elements belonging to same group of (iv) Lanthanoid compounds like cerium molybdate, cerium
second and third transition series. tungstate are used as paints and dyes.
(iii) As the size decreases from La 3+ to Lu3+, covalent character The Actinoids (Ac)
of hydroxides increases and hence, the basic strength Electronic configurations
decreases. Thus La(OH)3 is most basic while Lu(OH)3 is least The general configuration of actinoides may be given as
basic.. [Rn] 5f 1–14 6d 0–1 7s 2. They have common 7s2 configuration
Oxidiation states and variable occupancy of 5f and 6d subshells. Though 4f and 5f
The typical OS of Ln is +3. Oxidation states + 2 and +4 occur orbitals have similar shapes but 5f is less deeply buried than 4f.
particularly when they lead to : Hence, 5f electrons can participate in bonding to a far greater extent
(i) A noble gas configuration e.g. Ce4+ (f 0) Ionic sizes
(ii) A half filled ‘f ’ orbital e.g. Eu2+, Tb4+, (f 7) They show a general trend of decrease in size like lanthanoids.
(iii) A completely filled ‘f ’ orbital e.g. Yb2+ (f 14) This is referred to as actinoid contraction. The contraction is greater
They show limited OS due to large energy gap between 4f and 5d from element to element resulting from poor shielding by
5f - electrons.
subshells.
The d- and f-block Elements 531

This is on account of the reason that the 5f orbitals are more (ii) Both are electropositive and highly reactive.
diffused than the orbitals present in 4f sub-shell. Consequently, (iii) Both exhibit magnetic and spectral properties.
the shielding by 5f electrons is much less than by 4f electrons in Differences:
lanthanoids. The magnitude of actinoid contraction is more than
that of lanthanoid contraction. Lanthanoids Actinoids
Oxidation states 1 Besides +3 OS, they 1 Besides +3 OS, they
They show larger no. of OS due to small energy gap between 5f, show +2 and +4 OS in show higher OS of +4,
6d and 7s subshells. General OS is +3. They also show OS of +4. few cases. +5, +6, +7 also.
Compounds with +3 and +4 oxidation state tend to hydrolyse. 2 Most of their ions are 2 Most of their ions are
General characteristics colourless. coloured.
(i) Actinoid metals have silvery appearance. 3 They have less 3 They have greater
(ii) Actinoid cations are coloured due to f - f transitions.Cations tendency to form tendency to form
with f 0, f 7 and f 14 configurations are colourless. complexes. complexes.
(iii) They have high m.pt and b.pt., high densities, low IE and are 4 Lanthanoid compounds 4 Actinoid compounds
highly electropositive. are less basic. are more basic
(iv) They are strongly paramagnetic, strong reducing agents and
5 Non-radioactive, except 5 Radioactive
are radioactive.
promethium
Note : The actinoids have lower ionization enthalpies than
lanthanoids because 5f is less penetrating than 4f. 6 Their magnetic 6 Their magnetic
Comparison of lanthanoids and actinoids properties can be properties cannot be
explained easily. explained easily.
Similarities:
(i) Both show an OS of +3
532

CONCEPT MAP
CHEMISTRY

EBD_7327
The d- and f-block Elements 533

1. The oxides, CrO3, MoO3, and WO3 are strongly 14. Which of the following configuration is correct for iron ?
(a) neutral (b) acidic
(c) basic (d) none of these (a) 1s2 ,2s2 2 p6 ,3s2 3 p6 3d 4

2. 4K 2 Cr2 O 7 ¾¾¾
heat
® 4K 2 CrO 4 + 3O 2 + X . In the above (b) 1s2 , 2s2 2 p6 ,3s2 3 p6 3d 6 4s2
reaction X is: (c) 1s2 ,2s2 2 p6 ,3s2 3 p6 3d 2
(a) CrO3 (b) Cr2O7
(c) Cr2O3 (d) CrO5 (d) 1s2 , 2s2 2 p6 ,3s2 3 p6 3d 2 4s 2
3. Which one of the following ionic species will impart colour
to an aqueous solution? 15. Percentage of silver in German silver is :
(a) Ti4+ (b) Cu+ (a) 0% (b) 1%
(c) Zn2+ (d) Cr3+ (c) 5% (d) none of these
4. The aqueous solution containing which one of the 16. In first transition series, the melting point of Mn is low
following ions will be colourless? (Atomic number: because
Sc = 21, Fe = 26, Ti = 22, Mn = 25) (a) due to d10 configuration, metallic bonds are strong
(a) Sc3+ (b) Fe2+ (b) due to d7 configuration, metallic bonds are weak
(c) Ti 3+ (d) Mn 2+ (c) due to d5 configuration, metallic bonds are weak
5. When tin is treated with concentrated nitric acid: (d) None of these
(a) it is converted into stannous nitrate 17. The number of unpaired electrons in gaseous species of
(b) it is converted into stannic nitrate Mn3+, Cr 3+ and V3+ respectively are.
(c) it is converted into metastannic acid (a) 4, 3 and 2 (b) 3, 3 and 2
(d) it becomes passive (c) 4, 3 and 2 (d) 3, 3 and 3
6. In which of the following pairs both the ions are coloured in 18. The liquified metal expanding on solidification is
aqueous solutions ? (a) Ga (b) Al
(a) Sc3+, Ti3+ (b) Sc3+, Co2+ (c) Zn (d) Cu
2+
(c) Ni , Cu + (d) Ni2+, Ti3+ 19. In which of the following metallic bond is strongest ?
(At. no. : Sc = 21, Ti = 22, Ni = 28, Cu = 29, Co = 27) (a) Fe (b) Sc
7. Which of the following ions has the maximum magnetic (c) V (d) Cr
moment? 20. Stainless steel contains iron and
(a) Mn +2 (b) Fe+2 (a) Cr + Ni (b) Cr + Zn
(c) Ti 3+ (d) Cr+2. (c) Zn + Pb (d) C +Cr + Ni
8. For the ions Zn 2+, Ni2+ and Cr3+ which among the following 21. Among th e following, the compound th at is both
statements is correct? paramagnetic and coloured, is
(atomic number of Zn = 30, Ni = 28 and Cr = 24) (a) KMnO4 (b) CuF2
(a) All these are colourless (c) K2Cr2O7 (d) All are coloured
(b) All these are coloured 22. Which of the following elements shows maximum number of
(c) Only Ni2+ is coloured and Zn 2+ and Cr3+ are colourless different oxidation states in its compounds?
(d) Only Zn2+ is colourless and Ni2+ and Cr3+ are coloured (a) Eu (b) La
9. Copper and con. H 2SO 4 react to produce (c) Gd (d) Am
(a) Cu (b) SO 2 23. K2Cr2O7 on heating with aqueous NaOH gives

(c) H 2 (d) O 2 (a) CrO 24- (b) Cr(OH)3


10. The volatile metal is : (c) Cr2 O 72- (d) Cr(OH)2
(a) Fe (b) Zn
24. CrO3 dissolves in aqueous NaOH to give
(c) Cu (d) Ag
(a) Cr2O72– (b) CrO42–
11. Which of the following metal ions is not coloured?
(c) Cr(OH)3 (d) Cr(OH)2
(a) Ti3+ (b) Fe3+
(c) V 2+ (d) Cu+ 25. To prevent corrosion, iron pipes carrying drinking water are
12. The transition metal with least atomic number covered with zinc. The process involved is
is : (a) alloy formation (b) electroplating
(a) Os (b) Zr (c) galvanising (d) soldering
(c) Pt (d) Ru 26. When KMnO4 acts as an oxidising agent and ultimately
13. Which of the following element is not a member of transition forms [MnO4]–2, MnO2, Mn2O3, Mn+2 then the number of
elements ? electrons transferred in each case respectively is
(a) Zn (b) Pt (a) 4, 3, 1, 5 (b) 1, 5, 3, 7
(c) Ce (d) Mo (c) 1, 3, 4, 5 (d) 3, 5, 7, 1.
EBD_7327
534 CHEMISTRY

27. The oxidation state of chromium in the final product formed 40. Addition of iron fillings to CuSO 4 solution caused
by the reaction between KI and acidified potassium precipitation of Cu owing to the
dichromate solution is: (a) Reduction of Cu2+ (b) Oxidation of Cu2+
(a) + 3 (b) + 2 (c) Reduction of Fe (d) Reduction of Fe3+
(c) + 6 (d) + 4 41. The final products formed on the addition of KI to copper
28. When the same amount of zinc is treated separately with sulphate solution are :
excess of H 2SO4 and excess of NaOH, the ratio of volumes (a) K2SO4, CuI2 and I2 (b) K2SO4, Cu2I2 and I2
(c) K2SO4 and Cu2O (d) K2SO4, CuO and I2
of H2 evolved is 42. KMnO4 (acidic/alkaline) is not decolourised by
(a) 1 : 1 (b) 1 : 2 (a) mohr salt (b) oxalic acid
(c) 2 : 1 (d) 9 : 4 (c) benzene (d) propene
29. Calomel (Hg 2Cl 2 ) on reaction with ammonium hydroxide 43. Bessemer converter is used in the manufacture of
gives (a) Pig iron (b) Steel
(c) Wrought (d) Cast iron.
(a) HgO (b) Hg 2O
44. Among the lanthanoides the one obtained by synthetic
(c) NH 2 – Hg – Hg – Cl (d) Hg2NH2Cl method is
30. Copper becomes green when exposed to moist air for longer (a) Lu (b) Pm
period – (c) Pr (d) Gd
(a) because of the formation of a layer of cupric oxide on 45. Both acid and base may react with which of the following
the surface of copper oxides ?
(b) because of the formation of a layer of basic carbonate (a) CaO (b) Na 2O 2
of copper on the surface of copper
(c) because of the formation of a layer of cupric hydroxide (c) ZnO (d) Mn 3O 4 .
on the surface of copper 46. The lanthanoide contraction is responsible for the fact that
(d) because of the formation of a layer of cupric acetate on (a) Zr and Y have about the same radius
the surface of copper (b) Zr and Nb have similar oxidation state
31. Copper displaces which of the metal from their salt solutions ? (c) Zr and Hf have about the same radius
(a) AgNO 3 (b) ZnSO 4 (d) Zr and Zn have the same oxidation state
(Atomic numbers : Zr = 40, Y = 39, Nb = 41, Hf = 72, Zn = 30)
(c) FeSO 4 (d) All of the above
47. Which one of the following elements shows maximum
32. Gun metal is an alloy of : number of different oxidation states in its compounds?
(a) Cu and Al (b) Cu and Sn (a) Eu (b) La
(c) Cu, Zn and Sn (d) Cu, Zn and Ni (c) Gd (d) Am
33. Which of the following has lowest percentage of carbon ? 48. Among the following which is used as an electrolyte in
(a) Cast iron electroplating a base metal with gold ?
(b) Wrought iron
(c) Steel iron (a) NH 4Cl (b) K[Au(CN) 2 ]
(d) All have same percentage. (c) HgCN (d) AgCN
34. Black jack is an ore of : 49. Cuprous chloride is used in gas analysis to absorb?
(a) Cr (b) Sn (a) CO2 (b) CO
(c) Zn (d) Ni (c) H2 (d) CH4
35. Which one of the following is coinage metal ? 50. Lanthanoids are
(a) Zn (b) Cu (a) 14 elements in the sixth period (atomic no. = 90 to 103)
(c) Sn (d) Pb. that are filling 4f sublevel
36. When excess of SnCl2 is added to a solution of HgCl2, a (b) 14 elements in the seventh period (atomic no. = 90 to
white precipitate turning to grey is obtained. This grey colour 103) that are filling 5f sublevel
is due to the formation of (c) 14 elements in the sixth period (atomic no. = 58 to 71)
(a) Hg 2Cl 2 (b) SnCl4 that are filling 4f sublevel
(c) Sn (d) Hg. (d) 14 elements in the seventh period (atomic no. = 58 to
37. Bronze is an alloy of : 71) that are filling 4f sublevel
(a) Pb + Sn + Zn (b) Cu + Sn 51. Aufbau principle does not give the correct arrangement of
(c) Pb + Zn (d) Cu + Zn filling up of atomic orbitals in :
38. The treatment of Cu with dilute HNO3 gives : (a) Cu and Zn (b) Co and Zn
(a) N2O (b) NO (c) Mn and Cr (d) Cu and Cr
52. The radius of La3+ (Atomic number of La = 57) is 1.06Å.
(c) NH +4 (d) NO2 Which one of the following given values will be closest to
39. White vitriol has the formula : the radius of Lu3+ (Atomic number of Lu = 71) ?
(a) CaSO4.2H2O (b) CuSO4.5H2O (a) 1.40 Å (b) 1.06 Å
(c) ZnSO4.7H2O (d) FeSO4.7H2O (c) 0.85 Å (d) 1.60 Å
The d- and f-block Elements 535

53. Which of the following compounds is formed when a mixture (c) Shielding power of 4f electrons is quite weak.
of K 2Cr2 O 7 and NaCl is heated with conc. H 2SO 4 ? (d) There is a decrease in the radii of the atoms or ions as
one proceeds from La to Lu.
(a) CrO2 Cl 2 (b) CrCl3 65. The correct order of decreasing second ionisation enthalpy
(c) Cr2 (SO 4 ) 3 (d) Na 2CrO4 of Ti (22), V(23), Cr(24) and Mn (25) is :
(a) Cr > Mn > V > Ti (b) V > Mn > Cr > Ti
54. Which of the following factors may be regarded as the main (c) Mn > Cr > Ti > V (d) Ti > V > Cr > Mn
cause of lanthanoide contraction? 66. Which one of the elements with the following outer orbital
(a) Greater shielding of 5d electrons by 4f electrons configurations may exhibit the largest number of oxidation
(b) Poorer shielding of 5d electrons by 4f electrons states?
(c) Effective shielding of one of 4f electrons by another in (a) 3d 54s1 (b) 3d 54s2
the subshell (c) 3d 24s2 (d) 3d 34s2
(d) Poor shielding of one of 4f electron by another in the 67. Which one of the following ions has electronic configuration
subshell [Ar] 3d 6 ?
55. For making Ag from AgNO3, which of the following is used
(a) PH3 (b) phosphonium iodide (a) Ni 3+ (b) Mn 3+
(c) Na2CO3 (d) NH3 (c) Fe3+ (d) Co3+
56. The titanium (atomic number 22) compound that does not (At. Nos. Mn = 25, Fe = 26, Co = 27, Ni = 28)
exist is 68. Which of the following pairs has the same size?
(a) TiO (b) TiO2
(c) K2TiF6 (d) K2TiO4 (a) Fe 2+ , Ni 2+ (b) Zr 4+ , Ti 4+
57. The Ce (Z = 58) belongs to IIIrd group of periodic table. If it
(c) Zr 4+ , Hf 4+ (d) Zn 2+ , Hf 4+
furnish one a particle to form an element 'X', then X belongs 69. Which of the following oxidation states is the most common
to among the lanthanoids?
(a) IIIrd group (b) IInd group (a) 3 (b) 4
st
(c) I group (d) zero group (c) 2 (d) 5
58. Lanthanum is grouped with f-block elements because 70. For the four successive transition elements (Cr, Mn, Fe and
(a) it has partially filled f-orbitals Co), the stability of +2 oxidation state will be there in which
(b) it is just before Ce in the periodic table of the following order?
(c) it has both partially filled f and d-orbitals (a) Mn > Fe > Cr > Co (b) Fe > Mn > Co > Cr
(d) properties of lanthanum are very similar to the elements (c) Co > Mn > Fe > Cr (d) Cr > Mn > Co > Fe
of f-block 71. Acidified K2Cr2O7 solution turns green when Na2SO3 is
59. The approximate percentage of iron in mischmetal is added to it. This is due to the formation of :
(a) 10 (b) 20 (a) Cr2(SO4)3 (b) CrO42–
(c) 50 (d) 5 (c) Cr2(SO3)3 (d) CrSO4
60. Which of the following is not an actinide ? 72. Which of the statements is not true?
(a) Curium (b) Californium (a) On passing H2S through acidified K2Cr2O7 solution, a
(c) Uranium (d) Terbium milky colour is observed.
61. A reduction in atomic size with increase in atomic number is (b) Na2Cr2O7 is preferred over K2Cr2O7 in volumetric
a characteristic of elements of analysis.
(a) high atomic masses (b) d-block (c) K2Cr2O7 solution in acidic medium is orange.
(c) f-block (d) Radioactive series (d) K2Cr2O7 solution becomes yellow on increasing the
62. Identify the product and its colour when MnO2 is fused pH beyond 7.
with solid KOH in the presence of O2. 73. Which one of the following does not correctly represent the
(a) KMnO4, purple (b) K2MnO4, dark green correct order of the property indicated against it?
(c) MnO, colourless (d) Mn2O3, brown (a) Ti < V < Cr < Mn : increasing number of oxidation states
63. Which one of the following ions is the most stable in (b) Ti3+ < V3+ < Cr3+ < Mn3+ : increasing magnetic moment
aqueous solution? (c) Ti < V < Cr < Mn : increasing melting points
(a) V3+ (b) Ti3+ (d) Ti < V < Mn < Cr : increasing 2nd ionization enthalpy
(c) Mn 3+ (d) Cr3+ 74. The catalytic activity of transition metals and their
(At.No. Ti = 22, V = 23, Cr = 24, Mn = 25) compounds is mainly due to :
64. Identify the incorrect statement among the following : (a) their magnetic behaviour
(a) Lanthanoid contraction is the accumulation of (b) their unfilled d-orbitals
successive shrinkages. (c) their ability to adopt variable oxidation state
(b) As a result of lanthanoid contraction, the properties of (d) their chemical reactivity
4d series of the transition elements have no similarities 75. Which of the following exhibit only + 3 oxidation state ?
with the 5d series of elements. (a) U (b) Th (c) Ac (d) Pa
EBD_7327
536 CHEMISTRY

1. To an aqueous solution containing anions a few drops of 10. When a small amount of KMnO4 is added to concentrated
acidified KMnO4 are added. Which one of the following H2SO4, a green oily compound is obtained which is highly
anions, if present will not decolourise the KMnO4 solution? explosive in nature. Compound may be :
(a) I– (b) CO32 - (a) MnSO4 (b) Mn2O7
(c) S 2– (d) NO-2 (c) MnO2 (d) Mn2O3
11. For d block elements the first ionization potential is of the
2. Consider the following statements
order :
(I) La(OH) 3 is the least basic among hydroxides of
lanthanides. (a) Zn > Fe > Cu > Cr (b) Sc = Ti < V = Cr
(II) Zr4+ and Hf4+ posses almost the same ionic radii. (c) Zn < Cu < Ni < Co (d) V > Cr > Mn > Fe
(III) Ce4+ can as an oxidizing agent. 12. A blue colouration is not obtained when
Which of the above is/are true ? (a) ammonium hydroxide dissolves in copper sulphate
(a) (I) and (III) (b) (II) and (III) (b) copper sulphate solution reacts with K 4 [Fe(CN) 6 ]
(c) (II) only (d) (I) and (II) (c) ferric chloride reacts with sod. ferrocyanide
3. The basic character of the transition metal monoxides follows (d) anhydrous CuSO4 is dissolved in water
the order
13. Which of the following combines with Fe (II) ions to form a
(Atomic Nos.,Ti = 22, V = 23, Cr = 24, Fe = 26)
brown complex?
(a) TiO > VO > CrO > FeO (b) VO > CrO > TiO > FeO
(a) NO (b) N2O
(c) CrO > VO > FeO > TiO (d) TiO > FeO > VO > CrO
(c) N2O3 (d) N2O5
4. The correct order of ionic radii of Y3+, La3+, Eu3+ and Lu3+ is
14. Oxidation states of the metal in the minerals haematite and
(a) La3+ < Eu3+ < Lu3+ < Y 3+
magnetite, respectively, are
(b) Y 3+ < La 3+ < Eu 3+ < Lu 3+ (a) II, III in haematite and III in magnetite
(c) Y 3+ < Lu 3+ < Eu 3+ < La 3+ (b) II, III in haematite and II in magnetite
(d) Lu 3+ < Eu 3+ < La 3+ < Y 3+ (c) II in haematite and II, III in magnetite
(Atomic nos. Y =39, La = 57, Eu = 63, Lu = 71) (d) III in haematite and II, III in magnetite
5. Which one of the following metals has a different lattice 15. A solution of sodium thiosulphate on addition of few drops
from those of the others? of ferric chloride gives violet colour due to the formation of
(a) Fe (b) Co (a) Na2S4O6 (b) Fe2(SO4)3
(c) Ni (d) Cu (c) Fe2(S2O3)3 (d) Fe2(S2O3)2
6. Among the following series of transition metal ions, the one 16. Precipitate of AgCl is soluble in liquid NH3, the compound
where all metal ions have 3d2 electronic configuration is forms
(At. nos. Ti = 22; V = 23; Cr = 24; Mn = 25) (a) Ag(NH4)2OH (b) Ag(NH4)2Cl
(a) Ti 3+ , V 2 + , Cr 3+ , Mn 4 + (c) Ag(NH3)2OH (d) Ag(NH3)2Cl
(b) Ti + , V 4 + , Cr 6 + , Mn 7 + 17. Philosopher's wool is :
(a) ZnCl2 (b) ZnS
(c) Ti 4 + , V 3+ , Cr 2 + , Mn 3+
(c) Zn(NO3)2 (d) ZnO
(d) Ti 2+ , V 3+ , Cr 4 + , Mn 5 + 18. Zinc and mercury do not show variable valency like d-block
7. A red solid is insoluble in water. However it becomes soluble elements because
if some KI is added to water. Heating the red solid in a test (a) they are soft
tube results in liberation of some violet coloured fumes and (b) their d-shells are complete
droplets of a metal appear on the cooler parts of the test (c) they have only two electrons in the outermost subshell
tube. The red solid is
(d) their d-shells are incomplete
(a) HgI2 (b) HgO
19. Silver nitrate produces a black stain on skin due to
(c) Pb3O4 (d) (NH4)2Cr2O7
(a) being a strong reducing agent
8. Potassium dichromate when heated with concentrated
sulphuric acid and a soluble chloride, gives brown-red (b) its corrosive action
vapours of : (c) formation of complex compound
(a) CrO3 (b) CrCl3 (d) its reduction to metallic silver
(c) CrO2Cl2 (d) Cr2O3 20. KI and CuSO4 solutions on mixing produce :
9. Of the following outer electronic configurations of atoms, (a) Cu2I2 + K2SO4 (b) Cu2I2 + I2 + K2SO4
the highest oxidation state is achieved by which one of (c) CuI2 + K2SO4 (d) CuI2 + I2 + K2SO4
them ? 21. Which one is most acidic?
(a) (n – 1)d 3 ns2 (b) (n – 1)d 5 ns1 (a) Cr2O3 (b) V2O5
(c) (n – 1)d 8 ns2 (d) (n – 1)d 5 ns2 (c) Mn2O7 (d) Fe2O3
The d- and f-block Elements 537

22. Which of the following statement is incorrect ? (d) more reactive nature of the actionids than the
(a) Across a period from Sc to Cu the densities increase lanthanoids
with increasing atomic number. 31. Amount of oxalic acid present in a solution can be
(b) The melting point of transition elements rise to a determined by its titration with KMnO4 solution in the
maximum from Sc to Cr and then decreases from Fe to presence of H2SO4.The titration gives unsatisfactory result
Zn. when carried out in the presence of HCl, because HCl
(c) Transition elements have high enthalpies of atomization (a) gets oxidised by oxalic acid to chlorine
and in 3d series increase regularly from Sc to Cu. (b) furnishes H+ ions in addition to those from oxalic acid
(d) On going down a group from 3d to 6d series the stability (c) reduces permanganate to Mn 2+
of higher oxidation state increases with increasing (d) oxidises oxalic acid to carbon doxide and water
atomic number. 32. Knowing that the chemistry of lanthanoids(Ln) is dominated
23. Lithopone is a mixture of : by its + 3 oxidation state, which of the following statements
(a) ZnCO3, BaCO3 (b) ZnS, Na2SO4 is incorrect?
(c) ZnSO4, BaSO4 (d) ZnS, BaSO4 (a) The ionic size of Ln (III) decrease in general with
24. Out of TiF62– , CoF63– , Cu2Cl2 and NiCl2– 4
(Z of Ti = 22, increasing atomic number
Co = 27, Cu = 29, Ni = 28), the colourless species are: (b) Ln (III) compounds are generally colourless.
(c) Ln (III) hydroxide are mainly basic in character.
(a) Cu2Cl2 and NiCl 2– 4 (b) TiF62– ,and Cu2Cl2
(d) Because of the large size of the Ln (III) ions the bonding
(c) CoF63– ,and NiCl 2–
4 (d) TiF62– ,and CoF63– , in its compounds is predominantly ionic in character.
25. Which of the following ions will exhibit colour in aqueous 33. In context of the lanthanoids, which of the following
solutions? statements is not correct?
(a) There is a gradual decrease in the radii of the members
(a) La 3+ (Z = 57) (b) Ti3+ (Z = 22)
with increasing atomic number in the series.
(c) Lu 3+ (Z = 71) (d) Sc3+ (Z = 21) (b) All the members exhibit +3 oxidation state.
26. Oxidation states of the metal in the minerals haematite and (c) Because of similar properties the separation of
magnetite, respectively, are lanthanoids is not easy.
(a) II, III in haematite and III in magnetite (d) Availability of 4f electrons results in the formation of
(b) II, III in haematite and II in magnetite compounds in +4 state for all the members of the series.
(c) II in haematite and II, III in magnetite 34. The outer electron configuration of Gd (Atomic No. : 64) is :
(d) III in haematite and II, III in magnetite (a) 4f 3 5d5 6s2 (b) 4f 8 5d 0 6s2
27. Which of the following arrangements does not represent 4
(c) 4f 5d 6s 4 2 (d) 4f 7 5d 1 6s2
the correct order of the property stated against it ? 35. Iron exhibits +2 and + 3 oxidation states. Which of the
(a) V2+ < Cr2+ < Mn2+ < Fe2+ : paramagnetic behaviour following statements about iron is incorrect ?
(b) Ni2+ < Co2+ < Fe2+ < Mn2+ : ionic size (a) Ferrous oxide is more basic in nature than the ferric
(c) Co3+ < Fe3+ < Cr3+ < Sc3+ : stability in aqueous solution oxide.
(d) Sc < Ti < Cr < Mn : number of oxidation states
(b) Ferrous compounds are relatively more ionic than the
28. Four successive members of the first row transition elements
corresponding ferric compounds.
are listed below with atomic numbers. Which one of them is
(c) Ferrous compounds are less volatile than the
expected to have the highest E° 3+ 2 + value ? corresponding ferric compounds.
M /M
(a) Cr(Z = 24) (b) Mn(Z = 25) (d) Ferrous compounds are more easily hydrolysed than
(c) Fe(Z = 26) (d) Co(Z = 27) the corresponding ferric compounds.
29. Identify the incorrect statement among the following: 36. Four successive members of the first series of the transition
(a) 4f and 5f orbitals are equally shielded. metals are listed below. For which one of them the standard
(b) d-Block elements show irregular and erratic chemical
properties among themselves. ( º
)
potential E M 2 + / M value has a positive sign?
(c) La and Lu have partially filled d-orbitals and no other
partially filled orbitals. (a) Co (Z = 27) (b) Ni (Z = 28)
(d) The chemistry of various lanthanoids is very similar. (c) Cu (Z = 29) (d) Fe (Z = 26)
30. Larger number of oxidation states are exhibited by the 37. Which of the following species is/are paramagnetic?
actinoids than those by the lanthanoids, the main reason Fe2+, Zn0, Hg2+, Ti4+
being (a) Fe2+ only (b) Zn 0 and Ti4+
(a) 4f orbitals more diffused than the 5f orbitals (c) Fe2+ and Hg2+ (d) Zn 0 and Hg2+
(b) lesser energy difference between 5f and 6d than 38. The titanium (atomic number 22) compound that does not
between 4f and 5d orbitals exist is
(c) more energy difference between 5f and 6d than between (a) TiO (b) TiO2
4f and 5d orbitals (c) K2TiF6 (d) K2TiO4
EBD_7327
538 CHEMISTRY

39. The colour of the following ions V2+, V3+, V4+, Fe2+, Fe3+ DIRECTIONS for Qs. 45 to 50 : These are Assertion-Reason
are respectively type questions. Each of these question contains two statements:
(a) green, violet, blue, green, yellow Statement-1 (Assertion) and Statement-2 (Reason). Answer
(b) yellow, green, violet, green, blue these questions from the following four options.
(c) violet, green, yellow, green, blue (a) Statement-1 is True, Statement-2 is True, Statement-2 is a
(d) yellow, green, blue, green, violet correct explanation for Statement-1
40. Which of the following is not a characteristic of interstitial (b) Statement-1 is True, Statement-2 is True ; Statement-2 is
compounds of transition elements? NOT a correct explanation for Statement-1
(a) The formulae of these compounds do not correspond (c) Statement-1 is True, Statement-2 is False
to any normal oxidation state
(d) Statement-1 is False, Statement-2 is True
(b) They have melting points higher than those of pure
elements 45. Statement-1 : Magnetic moment values of actinides are
(c) They are very hard and some compounds approach lesser than the theoretically predicted values.
diamond in hardness Statement-2 : Actinide elements are strongly paramagnetic.
(d) They are insulators in contrast to the transition metals 46. Statement-1 : Cu2+ and Cd2+ are separated from each other
41. In the following salts the lowest value of magnetic moment by first adding KCN solution and then passing H2S gas.
is observed in Statement-2 : KCN reduces Cu2+ to Cu+ and forms a complex
(a) MnSO4. 4H2O (b) CuSO4.5H2O with it.
(c) FeSO4.6H2O (d) ZnSO4.7H2O 47. Statement-1 : Cuprous ion (Cu+) has unpaired electrons while
42. Which one of the following transition elements does not cupric ion (Cu++) does not.
exhibit variable oxidation state? Statement-2 : Cuprous ion (Cu+) is colourless whereas cupric
(a) Ni (b) Cu ion (Cu++) is blue in the aqueous solution
(c) Fe (d) Sc 48. Statement-1 : Transition metals show variable valency.
43. Identify the product and its colour when MnO2 is fused
Statement-2 : Transition metals have a large energy
with solid KOH in the presence of O2.
difference between the ns2 and (n – 1)d electrons.
(a) KMnO4, purple (b) K2MnO4, dark green
49. Statement-1 : Transition metals are good catalysts.
(c) MnO, colourless (d) Mn 2O3, brown
Statement-2 : V2O5 or Pt is used in the preparation of H2SO4
44. Which one of the following ions has the maximum
magnetic moment? by contact process.
(a) Sc3+ (b) Ti3+ 50. Statement-1 : Magnetic moment values of actinides are
(c) Cr3+ (d) Fe3+ lesser than the theoretically predicted values.
Statement-2 : Actinide elements are strongly paramagnetic.

Exemplar Questions 4. Generally, transition elements form coloured salts due to the
1. Electronic configuration of a transition element X in +3 presence of unpaired electrons. Which of the following
oxidation state is [Ar]3d5. What is its atomic number? compounds will be coloured in solid state?
(a) 2 5 (b) 2 6 (a) Ag2SO4 (b) CuF2
(c) 2 7 (d) 2 4 (c) ZnF2 (d) Cu2Cl2
2. The electronic configuration of Cu(II) is 3d9 whereas that of 5. On addition of small amount of KMnO4 to concentrated
Cu(I) is 3d10. Which of the following is correct. H2SO4, a green oily compound is obtained which is highly
(a) Cu(II) is more stable explosive in nature. Identify the compound from the
(b) Cu(II) is less stable following.
(a) Mn 2O7 (b) MnO2
(c) Cu(I) and Cu(II) are equally stable
(c) MnSO4 (d) Mn 2O3
(d) Stability of Cu(I) and Cu(II) depends on nature of
6. The magnetic nature of elements depends on the presence
copper salts
of unpaired electrons. Identify the configuration of transition
3. Metallic radii of some transition elements are given below.
element, which shows highest magnetic moment.
Which of these elements will have highest density?
(a) 3d7 (b) 3d5
Element Fe Co Ni Cu (c) 3d8 (d) 3d2
Metallic radii/pm 126 125 125 128 7. Which of the following oxidation state is common for all
lanthanoids?
(a) Fe (b) Ni (a) + 2 (b) + 3
(c) Co (d) Cu (c) + 4 (d) + 5
The d- and f-block Elements 539

8. Which of the following reactions are disproportionation 17. Which of the following statements is not correct?
reactions? (a) Copper liberates hydrogen from acids
(i) Cu+ ® Cu2+ + Cu (b) In its higher oxidation states, manganese forms stable
(ii) 3MnO -4 + 4H + ® 2MnO 4- + MnO 2 + 2H 2 O compounds with oxygen and fluorine
(iii) 2KMnO4 ® K2MnO4 + MnO2 + O2 (c) Mn3+ and Co3+ are oxidising agents in aqueous solution
(iv) 2MnO 4- + 3Mn 2+ + 2H 2O ® 5MnO 2 + 4H + (d) Ti2+ and Cr 2+ are reducing agents in aqueous solution
18. When acidified K2Cr2O7 solution is added to Sn2+ salt then
(a) (i) (b) (i), (ii) and (iii)
Sn2+ changes to
(c) (ii), (iii) and (iv) (d) (i) and (iv)
9. When KMnO4 solution is added to oxalic acid solution, the (a) Sn (b) Sn3+
(c) Sn 4+ (d) Sn+
decolourisation is slow in the beginning but becomes
instantaneous after some time because 19. Highest oxidation state of manganese in fluoride is
(a) CO2 is formed as the product +4 (MnF4 ) but highest oxidation state in oxides is +7
(b) reaction is exothermic (Mn2O7) because
(c) MnO -4 catalyses the reaction (a) fluorine is more electronegative than oxygen
(b) fluorine does not possess d-orbitals
(d) Mn2+ acts as autocatalyst
10. There are 14 elements in actinoid series. Which of the (c) fluorine stabilises lower oxidation state
following elements does not belong to this series? (d) in covalent compounds, fluorine can form single bond
(a) U (b) Np only while oxygen forms double bond
(c) Tm (d) Fm 20. Although zirconium belongs to 4d transition series and
11. KMnO4 acts as an oxidising agent in acidic medium. The hafnium to 5d transition series even then they show similar
number of moles of KMnO4 that will be needed to react with physical and chemical properties because ................ .
one mole of sulphide ions in acidic solution is (a) both belong to d-block
2 3 (b) both have same number of electrons
(a) (b) (c) both have similar atomic radius
5 5
(d) both belong to the same group of the periodic table
4 1
(c) (d) 21. Why is HCl not used to make the medium acidic in oxidation
5 5
reactions of KMnO4 in acidic medium?
12. Which of the following oxides are amphoteric?
(a) Both HCl and KMnO4 act as oxidising agents
Mn2O7, CrO3, Cr 2O3, CrO, V2O5, V2O4
(a) V2O5, Cr 2O3 (b) Mn2O7, CrO3 (b) KMnO4 oxidises HCl into Cl2 which is also an oxidising
(c) CrO, V2O5 (d) V2O5, V2O4 agent
13. Gadolinium belongs to 4f series. Its atomic number is 64. (c) KMnO4 is a weaker oxidising agent than HCl
Which of the following is the correct electronic configuration (d) KMnO4 acts as a reducing agent in the presence of
of gadolinium? HCl
(a) [Xe]4f 7 5d16s2 (b) [Xe]4f 6 5d26s2
8 2 NEET/AIPMT (2013-2017) Questions
(c) [Xe]4f 6d (d) [Xe]4f 9 5s1
14. Interstitial compounds are formed when small atoms are 22. Which of the following lanthanoid ions is diamagnetic ?
trapped inside the crystal lattice of metals. Which of the (At nos. Ce = 58, Sm = 62, Eu = 63, Yb = 70) [2013]
following is not the characteristic property of interstitial (a) Sm2+ (b) Eu2+
compounds? (c) Yb2+ (d) Ce2+
(a) They have high melting points in comparison to pure 23. KMnO4 can be prepared from K2MnO4 as per the reaction:
metals
3MnO 24 - + 2H 2 O 2MnO24 - + MnO2 + 4OH -
(b) They are very hard
(c) They retain metallic conductivity The reaction can go to completion by removing OH– ions by
(d) They are chemically very reactive adding. [2013]
(a) KOH (b) CO2
15. The magnetic moment is associated with its spin angular
(c) SO2 (d) HCl
momentum and orbital angular momentum. Spin only
magnetic moment value of Cr3+ ion is 24. The outer electronic configuration of Gd (At. No. 64) is:
(a) 2.87 BM (b) 3.87 BM [1997, NEET Kar. 2013]
(c) 3.47 BM (d) 3.57 BM (a) 4 f 4 5d 5 6s1 (b) 4 f 5 5d 4 6s1
16. KMnO4 acts as an oxidising agent in alkaline medium. When (c) 4 f 7 5d1 6s2 (d) 4 f 3 5d5 6s2
alkaline KMnO4 is treated with KI, iodide ion is oxidised 25. Sc (Z = 21) is a transition element but Zn (Z = 30) is not
to ................. . because [NEET Kar. 2013]
(a) I2 (b) IO– (a) both Sc and Zn do not exhibit variable oxidation states
(b) both Sc3+ and Zn2+ ions are colourless and form white
(c) IO3- (d) IO-4 compounds
EBD_7327
540 CHEMISTRY

(c) in case of Sc, 3d orbitals are partially filled but in Zn 35. The angular momentum of electron in 'd' orbital is equal to :
these are completely filled [2015]
(d) last electron is assumed to be added to 4s level in case (a) 2h (b) 2 3 h
of Zn
26. The pair of compounds that can exist together is: [2014] (c) 0h (d) 6h
(a) FeCl3, SnCl2 (b) HgCl2, SnCl2 36. Assuming complete ionization, same moles of which of the
(c) FeCl2, SnCl2 (d) FeCl3, KI following compounds will require the least amount of acidified
27. In acidic medium, H2O2 changes Cr2O7–2 to CrO5 which has KMnO4 for complete oxidation [2015 RS]
two (–O–O) bonds. Oxidation state of Cr in CrO5 is:- [2014] (a) FeSO4 (b) FeSO3
(a) + 5 (b) + 3 (c) FeC2O4 (d) Fe(NO2)2
(c) + 6 (d) – 10 37. Gadolinium belongs to 4f series. It's atomic number is 64.
28. The reaction of aqueous KMnO 4 with H2O2 in acidic Which of the following is the correct electronic configuration
conditions gives: [2014] of gadolinium ? [2015 RS]
(a) Mn4+ and O2 (b) Mn2+ and O2 (a) [Xe]4f 86d2 (b) [Xe]4f 95s1
(c) Mn2+ and O3 (d) Mn4+ and MnO2 (c) [Xe] 4f 75d16s2 (d) [Xe] 4f 65d26s2
29. Magnetic moment 2.83 BM is given by which of the following 38. Which is the correct order of increasing energy of the listed
ions ? [2014] orbitals in the atom of titanium ? [2015 RS]
(At. nos. Ti = 22, Cr = 24, Mn = 25, Ni = 28):- (a) 3s 4s 3p 3d (b) 4s 3s 3p 3d
(a) Ti3+ (b) Ni2+ (c) 3s 3p 3d 4s (d) 3s 3p 4s 3d
(c) Cr3+ (d) Mn 2+ 39. Which one of the following statements is correct when SO2
30. Reason of lanthanoid contraction is:- [2014] is passed through acidified K2Cr2O7 solution ? [2016]
(a) Negligible screening effect of ‘f ’ orbitals (a) The solution turns blue
(b) Increasing nuclear charge (b) The solution is decolourized
(c) Decreasing nuclear charge
(c) SO2 is reduced
(d) Decreasing screening effect (d) Green Cr2(SO4)3 is formed
31. Magnetic moment 2.84 B.M. is given by :- [2015] 40. The electronic configurations of Eu(Atomic No. 63),
(At. nos, Ni = 28, Ti = 22, Cr = 24, Co = 27) Gd(Atomic No. 64) and Tb (Atomic No. 65) are [2016]
(a) Ti3+ (b) Cr2+ 7 2 8 2
(a) [Xe]4f 6s , [Xe]4f 6s and [Xe]4f 5d 6s 8 1 2
(c) Co 2+ (d) Ni2+ (b) [Xe]4f75d16s2, [Xe]4f7 5d1 6s2 and [Xe]4f96s2
32. The number of d-electrons in Fe2+ (Z = 26) is not equal to the (c) [Xe]4f65d16s2, [Xe]4f75d16s2 and [Xe]4f85d16s2
number of electrons in which one of the following? [2015]
(d) [Xe]4f76s2, [Xe]4f75d16s2 and [Xe]4f96s2
(a) p-electrons in Cl (Z = 17)
41. Which copper is heated with conc. HNO3 it produces [2016]
(b) d-electrons in Fe (Z = 26)
(a) Cu(NO3)2 and NO2
(c) p-electrons in Ne (Z = 10)
(b) Cu (NO3)2 and NO
(d) s-electrons in Mg (Z = 12)
33. Which of the following processes does not involve oxidation (c) Cu(NO3)2, NO and NO2
of iron ? [2015] (d) Cu(NO3)2 and N2O
(a) Decolourization of blue CuSO4 solutution by iron 42. Name the gas that can readily decolourise acidified KMnO4
(b) Formation of Fe(CO)5 from Fe solution : [2017]
(c) Liberation of H2 from steam by iron at high temperature (a) SO2 (b) NO2
(c) P2O5 (d) CO2
(d) Rusting of iron sheets
43. The reason for greater range of oxidation states in actinoids
34. Because of lanthanoid contraction, which of the following
is attributed to :- [2017]
pairs of elements have nearly same atomic radii ? (Numbers
(a) actinoid contraction
in the parenthesis are atomic numbers). [2015]
(b) 5f, 6d and 7s levels having comparable energies
(a) Zr (40) and Nb (41) (b) Zr (40) and Hf (72)
(c) 4f and 5d levels being close in energies
(c) Zr (40) and Ta (73) (d) Ti (22) and Zr (40) (d) the redioactive nature of actinoids
The d- and f-block Elements 541

Hints & Solutions


EXERCISE - 1 Ni 2 + = 1s2, 2s2p6, 3s2 p6 d8
(No. of e - s = 26)
1. (b) The transition metal oxides in which metal is in lower
oxidation states are basic and in which metal is in higher Thus Zn 2+ , Cr 3+ and Ni 2+ have zero, 3 and 2 unpaired
oxidation states are acidic while with intermediate electrons respectively.
oxidation states are amphoteric. 9. (b) Copper reacts with H2SO4 to produce SO2
+6 +6 +6 Cu + 2H 2SO 4 ¾¾ ® CuSO 4 + SO 2 + 2H 2 O
Cr O3 Mo O3 W O3
But Cu does not displace H2 from dilute H2SO4 as it is
So, the given transition metal oxides are acidic in nature. less reactive than H 2 (comes below H 2 in
2. (c) Potassium dichromate dissociates with evolution of O2 electrochemical series)
on heating. 10. (b) Zn is volatile metal
4 K2Cr2O7 ¾¾ ® 4 K CrO + 2Cr O + 3O
2 4 2 3 2
11. (d) The ion having all electrons paired is not coloured.
So, X is Cr2O3. (a) Ti3+ = 1s2, 2s2, 2p6, 3s2, 3p6, 3d1,4s0
3. (d) (b) Fe3+ = 1s2, 2s2, 2p6, 3s2, 3p6, 3d5, 4s0
3d 4s
(c) V3+ = 1s2, 2s2, 2p6, 3s2, 3p6, 3d2, 4s0
4. 3+
(a) Sc ® 3d 4s 0 0
(d) Cu+ = 1s2, 2s2, 2p6, 3s2, 3p6, 3d10,4s0
Q Only Cu+ does not have any unpaired electron, it is
Fe2+® 3d 6 4s 0 not coloured.
12. (b) Zirconium (Z = 40).
Ti3+® 3d 14s 0 13. (c) Cerium (Ce) belongs to lanthanide series and is member
of inner-transition metals.
Mn2+ ® 3d 5 4s 0
14. (b) Configuration of Fe ( Z = 26)
In Sc3+ there is/are no unpaired electrons. So the 1s2, 2s2, 2p6, 3s2, 3p6, 3d 6, 4s2
aqueous solution of Sc3+ will be colourless. 15. (a) German silver is
5. (c) Tin on reaction with conc. HNO3 to form meta stannic Cu = 25 - 50%
acid and nitrogen dioxide. Zn = 25 - 35%
Ni = 10 - 35%
Sn + 4HNO3 ¾¾
® H 2SnO3 + 4NO2 + H 2O Used as utensils, resistance wire.
tin nitric Meta nitrogen
acid stannic dioxide 16. (c) Due to d 5 configuration, Mn has exactly half filled
(conc.) acid d-orbitals. As a result the electronic configuration is
6. (d) Sc : 1s , 2s p , 3s p d0, 4s0; no unpaired electron.
3+ 2 2 6 2 6 stable means 3d electrons are more tightly held by the
Cu+ : 1s2, 2s2p6, 3s2p6d 10, 4s0; no unpaired electron. nucleus and this reduces the delocalization of electrons
Ni2+: 1s2, 2s2p6, 3s2p6d 8, 4s0; resulting in weaker metallic bonding.
17. (c) Mn3+ = [Ar]3d4
unpaired electrons are present.
Ti3+ : 1s2, 2s2p6, 3s2p6d 1, 4s 0; = [Ar]
unpaired electron is present Number of unpaired electrons = 4
Co2+ : 1s2, 2s2p6, 3s2p6d 7, 4s0; Cr3+ = [Ar]3d3
unpaired electrons are present
= [Ar]
So from the given options the only correct combination
is Ni2+ and Ti3+. No. of unpaired electrons = 3
7. (a) Mn++ –5 unpaired electrons V3+ = [Ar]3d2
Fe++ – 4 unpaired electrons = [Ar]
Ti++ – 2 unpaired electrons No. of unpaired electrons = 2
Cr++ – 4 unpaired electrons 18. (a) Among the given options Ga is the only liquid metal
Hence maximum no. of unpaired electron is present in which expands on solidification. It has an unusual
Mn++. structure. Each metal atom has one close neighbour at
Magnetic moment µ number of unpaired electrons a distance of 2.43Å, and six more neighbours at distances
8. (d) The ions with unpaired electrons are colourled and within range of 2.70Å to 2.79Å. Due to this structure it
those with paired electrons are colourless. exists as discrete diatomic molecules rather than a
metallic structure in crystal form which is less dense
Zn 2 + = 1s 2, 2s2p6, 3s2 p6 d10
(No. of e - s = 28) than its liquid form. As its solid form is heated to 30°C,
due to weak metallic bonds this unusual structure no
Cr 3+ = 1s 2, 2s2p6, 3s2 p6 d3 longer exists and melts to liquid. This property is unique
-
(No. of e s = 21) to Ga, Ge and Bi.
EBD_7327
542 CHEMISTRY

19. (d) Strength of metallic bond directly depends on the 40. (a) Addition of iron filling to CuSO4 solution caused ppt.
number of unpaired electrons. More is number of of Cu owing to the reduction of Cu2+ e.g.
unpaired electrons stronger is metallic bond. Hence, CuSO 4 + Fe ¾ ¾® FeSO 4 + Cu
metallic bond is strongest in Cr having maximum of 6 (reduction of copper).
unpaired electrons among the given options, i.e., Sc (1 41. (b) 2CuSO4 + 4KI ¾¾ ® 2K 2SO4 + Cu 2 I2 + I 2
unpaired electron), V (3 unpaired electrons) and Fe (4
42. (c) Benzene does not decolourise KMnO4 (acidic/alkaline).
unpaired electrons).
It is due to the delocalization of p-electron in benzene.
20. (d)
Mohr’s salt and oxalic acid gives redox reaction with
21. (b) CuF2 is both paramagnetic and coloured.
KMnO4. Propene decolorises KMnO4 due to presence
22. (d) Eu La Gd Am of C = C bond.
O.S = +2 ,+3 +3 +3 +3,+4,+5,+6 43. (b) Bessemer converter is used in the manufacture of steel.
44. (b) Pm is obtained by synthetic method.
23. (a) ® 2CrO 42 - + H 2 O
Cr2 O 72– + 2OH – ¾¾ 45. (c) ZnO is amphoteric and dissolves in both acids and
bases
Hence CrO 24- ion is obtained.
ZnO + 2 HCl ¾
¾® ZnCl 2 + H 2 O
24. (b) CrO3 + 2NaOH ® Na 2CrO 4 + H 2 O ZnO + 2 NaOH ¾
¾® Na 2 ZnO 2 + H 2 O
25. (c) Galvanisation is the process of deposition of zinc metal 46. (c) A regular decrease in the size of the atoms and ions in
on the surface of Fe to prevent corrosion. lanthanoid series from La3+ to Lu3+ is called lanthanide
+3 +7 +6 contraction. The similarity in size of the atoms of Zr
26. (c)
– –
– 4e
Mn2O3 ¬ ¾ –e¾® [MnO ] - 2
¾ ¾ [KMnO 4 ] ¾ ¾ 4 and Hf is due to the lanthanide contraction.
– 5e– – 47. (d) We know that lanthanides La, Gd shows +3, oxidation
– 3e
2+ +4 state, while Eu shows oxidation state of +2 and + 3. Am
Mn MnO2
shows +3, +4, +5 and +6 oxidation states. Therefore
2- - + Americium (Am) has maximum number of oxidation
27. (a) ® 3I 2 + 7H 2 O + 2 Cr 3+
Cr2 O 7 + 6I + 14H ¾¾ states.
oxidation state of Cr is +3. 48. (b) Complex soluble salts of the metal to be deposited are
28. (a) Zn + H 2SO 4 ® ZnSO 4 + H 2 usually employed as electrolytes in electroplating.
Hence K[Au(CN) 2 ] is used as an electrolyte in gold
Zn + 2NaOH ® Na 2 ZnO 2 + H 2
plating.
1 mole atom of zinc produces the same amount of H2 49. (b) A solution of CuCl in NH4OH is often used to measure
(2 g) in each case. the amount of CO in gas samples, by measuring the
29. (d) Hg 2 Cl 2 + 2NH 4 OH ¾¾
® Hg 2 NH 2Cl +NH 4 Cl + 2H 2O change in volume of the gas.
50. (c) Lanthanides are 4 f-series elements starting from cerium
30. (b) 2Cu + H2O + CO2 + O2 –® CuCO3.Cu(OH)2 (Z= 58) to lutetium (Z = 71). These are placed in the
(green coloured basic carbonate of copper is formed) sixth period and in third group.
31. (a) Cu is more electropositive than Ag therefore, it displaces 51. (d) Electronic configuration of Cr is [Ar] 3d54s1 because
Ag from their salt solution AgNO3. this gives it a stable half filled d5 configuration. Similarly,
Cu + 2 AgNO 3 ¾ ¾® Cu ( NO 3 ) 2 + 2Ag electronic configuration of Cu is [Ar]3d104s1. This filling
32. (c) Gun metal is an alloy of Cu, Zn and Sn. It contains 88% of electrons is not in accordance with Aufbau principle.
Cu, 10% Sn and 2% Zn. 1
33. (b) Wrought iron is the purest form of iron containing 52. (c) Ionic radii µ
z
lowest percentage of C (0.2-0.5%). z2 1.06 71
34. (c) Black jack or Zinc blende is ZnS. It is ore of Zn. Thus, Þ <
z1 3+
(Ionic radii of Lu ) 57
35. (b) Cu, Ag and Au are called coinage metals.
36. (d) 2HgCl 2 + SnCl 2 ¾ ¾® HgCl 2 + SnCl 4 Þ Ionic radii of Lu3+ = 0.85 Å
White ppt Heat
53. (a) K 2 Cr2 O 7 + 4 KCl + 6 H 2SO 4 ¾¾¾®
Hg 2 Cl 2 + SnCl 2 ¾
¾® 2Hg + SnCl 4
grey 2CrO 2Cl 2 + 6KHSO 4 + 3H 2 O
37. (b) Bronze is an alloy of Cu and Sn. 54. (b) In lanthanides, there is poorer shielding of 5d electrons
38. (b) 3Cu+ (dil) 8 HNO3 ® 3Cu(NO3)2 + 2NO + 4H2O by 4 f electrons resulting in greater attraction of the
39. (c) ZnSO4.7H2O is White vitriol nucleus over 5d electrons and contraction of the atomic
FeSO4.7H2O is Green vitriol radii.
CuSO4.5H2O is Blue vitriol
The d- and f-block Elements 543

55. (a) When AgNO 3 reacts with PH 3 , then Ag is obtained. 67. (d) Ni 3+ : [Ar] 3d 7
6AgNO3
+
2PH3
¾
¾® Mn 3+ : [Ar] 3d 4
silver nitrate phosphene
Fe3+ : [Ar] 3d 5
6Ag + 2H 3 PO 3 + 6NO2
phosphorous acid Co3+ : [Ar] 3d 6
56. (d) Oxidation state of Ti in the given compounds. 68. (c) Due to lanthanide contraction, the size of Zr and Hf
as follows: (atom and ions) become nearly similar
TiO – +2 69. (a) 70. (a)
TiO2 – +4 71. (a) The green colour appears due to the formation of
Cr+++ion
TiCl3 – +3
K2TiF6 – + 4 Cr2 O72– + 3SO32– + 8H + ¾¾
® 3SO 2–
4 + 2Cr
3+
+ 4H 2 O
K2TiO4 – + 6 72. (b) Na2Cr2O7 is hygroscopic.
The oxidation states exhibited by Ti is + 2, + 3, + 4. So 73. (c) The melting points of the transition element first rise to
K2TiO4 does not exist. a maximum and then fall as the atomic number increases
57. (b) 58Ce is lanthanide. Lanthanides are from 57 to 71 all manganese have abnormally low melting point.
present in IIIrd group. Hence 58Ce forms 56X element 74. (c) The transition metals and their compounds are used as
on emission of one a particle with belongs to IInd group. catalysts. Because of the variable oxidation states they
58. (d) may form intermediate compound with one of the
59. (d) Mischmetal is an alloy which contains rare earth readtants. These intermediate provides a new path with
elements (94-95%), iron (5%) and traces of sulphur, lowe activation energy. V2O5 + SO2 ® V2O4 + SO3
carbon, silicon, calcium and aluminium. It is used in 2V2O4+ O2 ® 2V2O5
gas lighters, tracer bullets and shells. 75. (c) Ac (89) = [Rn] [6d1] [7s2]
60. (d) Terbium is lanthanide as it belongs to 4f – series having
configuration [Xe]4s96s2. However the remaining EXERCISE - 2
members belong to 5f – series (actinides). 1. (b) Thus they decolourise the KMnO4 solution.
61. (c) Lanthanide contraction results into decrease in atomic Except CO32– all other ions I–, S2–, NO2–, Cl– are oxidised
and ionic radii. by acidified KMnO4 forming I2, S, NO3– and Cl2
62. (b) 2MnO2 + 4KOH + O2 ¾¾
® 2K 2 MnO4 + 2H 2O respectively.
dark green 2MnO4– + 10I– + 16H+ ® 2Mn2+ + 5I2 + 8H2O.
S2– is oxidised to S in presence of KMnO4
63. (d) For chromium ion + 3 oxidation state is most stable.
2KMnO4 + 3H2SO4 + 5H2S ®
64. (b) As a result of lanthanoid contraction change in ionic
K2SO4 + 2MnSO4 + 5S + 8H2O.
radii on going from elements of 4d to 5d transition series
Nitrites are oxidised to nitrates
is very small. Thus chemical properties of 4d and 5d
2KMnO4 + 5KNO2 + 3H2SO4 ®
series of transition elements are similar.
K2SO4 + 2MnSO4 + 5KNO3 + 3H2O
65. (a) Ti ; Z (22) is 1s22s22p63s23p63d 24s2 Chloride is oxidised to chlorine
V ; Z (23) is 1s22s22p63s23p63d 34s2 2KMnO4 + 3H2SO4 + 10HCl ®
Cr ; Z (24) is 1s22s22p63s23p63d 54s1 K2SO4 + 2MnSO4 + 5Cl2 + 8H2O
Mn ; Z (25) is 1s22s22p63s23d 54s2 CO32– is not oxidised by KMnO4 because in CO32–, C
The second electron in all the cases (except Cr) has to is already in its highest + 4 oxidation state. Thus CO2– 3
be removed from 4s-orbital and for Cr it has to be does not decolourise KMnO4 solution.
removed from completely half filled 3d-orbital which 2. (b) As a result of lanthanide contraction Zr 4+ and Hf4+
have extra stability thus I.E2 is highest for Cr. Now possess almost the same ionic radii. Ce4+ is an
among rest elements more will be the nuclear charge oxidising agent. Ce4+ gains electron to acquire more
more will be the value of I.E2. As nuclear charge stable Ce3+state. La(OH)3 is the most basic among
depends upon atomic number. Therefore correct order lanthanide hydroxides.
of I.E2 is Mn > V > Ti. 3. (a) The order of basic character of the transition metal
i.e. Cr > Mn > V > Ti. So correct answer is (a). monoxide is TiO > VO > CrO > FeO because basic
character of oxides decreases with increase in atomic
66. (b) Mn - 3d 5 4s2 number.
The no. of various oxidation states possible are + 2, + 4. (c) In lanthanide series there is a regular decrease in the
3, + 4, + 5, + 6 and + 7. atomic as well as ionic radii of trivalent ions (M3+) as
the atomic number increases. Although the atomic radii
EBD_7327
544 CHEMISTRY

do show some irregularities but ionic radii decreases 13. (a) We know that when nitrogen oxide (NO) combines with
from La(103 pm) to Lu (86pm). Y3+ belong to second Fe (II) ions, a brown complex is formed. This reaction is
transition series there fore have greater ionic radii then called brown ring test.
other ions of third transition series. 14. (d) (i) Haematite is Fe2O3 in which Fe is present in III
5. (a) Fe has hcp lattice while all other given metals have ccp oxidation state.
lattice. (ii) Magnetite (Fe3O4) is an equimolar mixture of FeO
6. (d) The electronic configuration of different species given and Fe2O3.
in the question are : Oxidation state of Fe in FeO is II.
3+ Oxidation state of Fe in Fe2O3 is III.
(a) 22 Ti :1s 2 2s 2 p6 3s 2 p 6 d1 15. (c) The reaction can be represented as
(b) + 2 2 6 2 6 2 1 3Na 2S2O3 + 2FeCl3 Fe2 (S2O3 )3 + 6NaCl
22 Ti :1s 2s p 3s . p d 4s
(Violet)
4+
(c) 22 Ti :1s 2 2s 2 p 6 3s 2 p6 16. (d) AgCl + 2NH3 ¾¾
® Ag(NH 3 ) 2 Cl
2+ 2 2 6 2 6 2 diammine silver (I) chloride
(d) 22 Ti :1s 2s p 3s p d
17. (d) Philosopher’s wool is ZnO
Thus options (a) and (c) are discarded; now let us
18. (b) 30Zn and 80Hg have their d orbitals completely filled
observe the second point of difference.
so they do not show any variable valency.
4+
23 V :1s 2 2s 2 p6 3s 2 p 6 d1 19. (d) Its reduction to metallic silver.
Thus option (b) is discarded 2AgNO3 ® 2Ag + N 2 + 3O 2
3+ 2 2 6 2 6 2 black stain
23 V :1s 2 s p 3s p d
20. (b) [2KI + CuSO4 ® CuI2 + K2SO4] × 2
4+
24 Cr :1s 2 2s 2 p6 3s 2 p 6 d 2 2CuI2 ® Cu2I2 + I2
5+
:1s 2 2 s 2 p 6 3s 2 p 6 d 2 4KI +2CuSO4 ® 2K2SO4 + Cu2I2 + I2
25 Mn
21. (c) As the oxidation state of metal associated with oxygen
7. (a) HgI 2 ∗ 2KI ® K 2 [HgI 4 ] increases, the acidic character of oxide increases. So
red, solid (soluble) Mn2O7 is most acidic as it has highest +7, oxidation
(insolub le)
state.
HgI 2 ƒ Hg + I 2 22. (c) (a), (b) and (d) are correct statements.
(violet vapours)
(c) There is drop in enthalpy of atomization at
8. (c) Solid potassium dichromate when heated with manganese because of half filled stable electronic
concentrated sulphuric acid and a soluble chloride gives configuration leading to less mobilisation of electrons.
orange red vapours of a volatile oily liquid CrO2Cl2 This results in weaker metallic bonds.
K2Cr2O7 + 4NaCl + 6H2SO4
¾® 2KHSO4 + 4NaHSO4 + 2CrO2Cl2 23. (d) Lithopone is a mixture of ZnS + BaSO 4 which is a
chromyl chloride white paint.
9. (d) (n – 1)d 5ns2 attains the maximum O.S. of + 7. 24. (b) The colour exhibited by transition metal ions is due to
10. (b) KMnO4 reacts with H2SO4 to form Mn2O7 which is the presence of unpaired electrons in d-orbitals which
highly explosive substance. permits the d - d excitation of electrons.
2KMnO4 + H2SO4 ¾¾® K2SO4 + Mn2O7 + H2O
In TiF62– ,– Ti is in + 4 O.S. ; 3d 0 = colourless
11. (a) The ionisation energies increase with increase in atomic
number. However, the trend is some irregular among d-
In CoF63– –, Co is in + 3 O.S ; 3d 5 = coloured
block elements. On the basis of electronic configuration,
the In Cu2Cl2– Cu is in +1 O.S. ; 3d 10 – colourless
Zn : 1s 2 2s 2 p 6 3s 2 p6 d 10 4s 2 In NiCl2– 8
4 – Ni is in + 2 O.S ; 3d – coloured

Fe : 1s 2 2s 2 p 6 3s 2 p 6 d 6 4s 2 25. (b) La 3+ : 54 e– = [Xe]


Cu : 1s 2 2s 2 p6 3s 2 p 6 d 10 4s1 Ti3+ : 19 e– = [Ar] 3 d 1 (Coloured)
Cr : 1s 2 2s 2 p6 3s 2 p 6 d 5 4s1 Lu 3+ : 68 e– = [Xe] 4 f 14
IE1 follows the order : Zn > Fe > Cu > Cr
12. (b) Sc3+ : 18 e– = [Ar]
2CuSO 4 + K 4 [ Fe (CN ) 6 ] ¾
¾®
Cu 2 [ Fe (CN ) 6 ] + 2 K 2SO 4
26. (d) (i) Haematite is Fe2O3 in which Fe is present in III
oxidation state.
Chocolate ppt.
The d- and f-block Elements 545

(ii) Magnetite (Fe3O4) is an equimolar mixture of FeO Eo


and Fe2O3. Ni + + / Ni = – 0.25V
o
Oxidation state of Fe in FeO is II. E ++
Fe / Fe = – 0.44V
Oxidation state of Fe in Fe2O3 is III. 37. (a) The outermost electronic configuration of Fe is
27. (a) Fe = [Ar] 3d6 4s2
(a) V = 3d 3 4s 2 ; V2+ = 3d 3 = 3 unpaired electrons Fe2+ = [Ar] 3d6 4s0
Cr = 3d 5 4s 1 ; Cr2+ = 3d 4 = 4 unpaired electrons
Mn = 3d 5 4s2 ; Mn2+ = 3d 5 = 5 unpaired electrons
Fe = 3d 6 4s 2 ; Fe2+ = 3d 6 = 4 unpaired electrons Since Fe2+ has 4 unpaired electrons it is paramagnetic
Hence the correct order of paramagnetic behaviour in nature.
V2+ < Cr 2+ = Fe2+ < Mn2+ Zn = [Ar] 3d10 4s2 —— no unpaired e–
(b) For the same oxidation state, the ionic radii generally Hg2+ = [Ar] 4f 14 5d10 —— no unpaired e–
decreases as the atomic number increases in a particular Ti4+ = [Ar] 3d0 4s0 —— no unpaired e–
transition series. hence the order is 38. (d) Oxidation state of Ti in the given compounds.
Mn++ > Fe++ > Co++ > Ni++ as follows:
(c) In solution, the stability of the compound depends TiO – +2
upon electrode potentials, SEP of the transitions metal TiO2 – +4
ions are given as TiCl3 – +3
Co3+ / Co = + 1.97, Fe3+ / Fe = + 0.77 ; K2TiF6 – +4
Cr3+ / Cr 2+ = – 0.41, Sc 3+ is highly stable as it does not K2TiO4 – +6
show + 2 O. S. The oxidation states exhibited by Ti is + 2, + 3, + 4. So
(d) Sc – (+ 2), (+ 3) K2TiO4 does not exist.
Ti – (+ 2), (+ 3), (+ 4) 39. (d) V2+ – violet, V3+ – green V4+ – blue
Cr – (+ 1), (+ 2), (+ 3), (+ 4), (+ 5), (+ 6) Fe2+ – green Fe3+ – yellow
Mn – (+ 2), (+ 3), (+ 4), (+ 5), (+ 6), (+ 7) 40. (d) Interstitial compounds of transition metal exhibit
i.e. Sc < Ti < Cr = Mn metallic conductivity.
E° = – 0.41 V E°Fe3+ / Fe 2 + = + 0.77 V 41. (a) Mn++ = 3d5 i.e. no. of unpaired e– = 5
28. (d) Cr 3+ / Cr 2 +
Cu++ = 3d9 i.e. no. of unpaired e– = 1
E° = + 1.57 V,, E°Co3+ / Co 2 + = + 1.97 V Fe++ = 3d6 i.e. no. of unpaired e– = 4
Mn3+ / Mn 2 +
Zn++ = 3d10 i.e. no. of unpaired e– = 0
29. (a) 4f orbital is nearer to nucleus as compared to 5 f orbital
Ni++ = 3d8 i.e. no. of unpaired e– = 3
therefore, shielding of 4 f is more than 5 f.
Higher the number of unpaired electrons higher will be
30. (b) The main reason for exhibiting larger number of oxidation the magnetic moment. Hence Mn ++ having maximum
states by actinoids as compared to lanthanoids is lesser unpaired electrons will have the maximum magnetic
energy difference between 5 f and 6d orbitals as moment.
compared to that between 4f and 5d orbitals. 42. (d) Sc does not show variable valency.
In case of actinoids we can remove electrons from 5f as 43. (b) 2MnO2 + 4KOH + O2 ¾¾ ® 2K 2 MnO4 + 2H 2O
well as from d and due to this actinoids exhibit larger dark green
number of oxidation state than lanthanoids. Thus the 44. (d) Fe3+(d5) has 5 unpaired electrons therefore magnetic
correct answer is option (b) moment = n(n + 2) = 5(5 + 2) = 5.91 which is
31. (c) The titration of oxalic acid with KMnO4 in presence of
maximum among given options. As Sc3+, Ti3+, Cr 3+,
HCl gives unsatisfactory result because of the fact that
V3+ contains 0, 1, 3, and 2 number of unpaired electrons
KMnO4 can also oxidise HCl along with oxalic acid.
respectively.
HCl on oxidation gives Cl2 and HCl reduces KMnO4 to 45. (b) The magnetic moments are lesser than the theoetically
Mn 2+ predicted values. This is due to the fact that 5f electrons
32. (b) Most of the Ln 3+ compounds except La3+ and Lu3+ of actinides are less effectively shielded which result
are coloured due to the presence of f-electrons. in quenching of orbital contribution.
33. (d) 46. (b) KCN forms complexes with Cu + and Cd 2+ as
K3 [Cu(CN)4 ] and K2 [Cd(CN)4 ] respectively. On
34. (d) The configuration of Gd is [xe] 4f 7 5d1 6s2.
passing H2S, only Cd2+ complex is decomposed to give
35. (d) Fe3+ is easily hydrolysed than Fe2+ due to more yellow CdS precipitate.
positive charge. 47. (d)
36. (c) E o +2 = 0.34 V 48. (c) The statement-1 is correct but the statement-2 is false.
Cu / Cu
Actually transition metal show variable valency due to
other has – ve E oR.P. very small difference between the ns2 and (n – 1)d
Eo electrons.
Co + + / Co = – 0.28 V
EBD_7327
546 CHEMISTRY

49. (b) Due to larger surface area and variable valencies to 5 moles of S2– ions react with 2 moles of KMnO4 thus,
form intermediate absorbed complex easily, transition
2
metals are used as catalysts. 1 mole of S2– ion will react with moles of KMnO4.
50. (b) The magnetic moments are lesser than the fact that 5f 5
electrons of actinides are less effectively shielded 12. (a) Amphoteric oxide react with alkalies as well as acids.
which results in quenching of orbital contribution. alkali 3-
V2O5 ¾¾¾ ® VO 4
EXERCISE - 3 acid +
V2O5 ¾¾¾ ® VO 4
Exemplar Questions
acid
Cr2O3 ¾¾¾ 3+
1. (b) Electronic configuration of X3+ is [Ar]3d5 ® [Cr(H2O)6]
Atomic number of X = 18 + 5 + 3 = 26 Cr2O3 ¾¾¾ alkali
® [CrO2]

2. (a) Anion carrying a higher charge and smaller size is
13. (a) The E.C. of gadolinium is :
associated with greater lattice and hydration energy
Gd = [Xe]4f 75d16s2
but it is expected on th e basis of electronic.
14. (d) Interstitial compounds are chemicaly inter.
configuration Cu+ should be more stable because it
has completely filled orbitals. But the lattice and 15. (b) Spin only magnetic moment ( m ) = n ( n + 2 ) BM
hydration energy factors dominate and as a result Cu2+
is more stable. = 3 ( 3 + 2 ) = 15
3. (d) Along the period, decrease in metallic radius is coupled = 3.87 BM
with increase in atomic mass therefore results in 16. (c) When alkaline KMnO4 oxidising agent is treated with
increase in density of metal. Hence, Cu will exhibit
highest density. KI, iodide ion is oxidised to IO3- .
4. (b) CuF2 is coloured in solid state because Cu(II) has d9 2KMnO4 + H2O + KI ® 2MnO2 + 2KOH + KIO3
configuration thus, due to the presence of unpaired 17. (a) Copper lies below hydrogen in the electrochemical
electron, it exists as a coloured salts. series and hence does not liberate H2 from acids.
5. (a) The compound formed can be explained by the 18. As mentioned in the reaction below Sn 2+ changes to
following reaction : Sn4+ :
2KMnO4 + 2H2SO4 (conc.) ® Mn2O7 + 2KHSO4 + H2O
Oxidation
(green oily)
6. (b) Greater the number of unpaired electrons, higher will (c) Cr2+ 6 O72 - +
+ 14H + 3Sn 2+
® 2Cr 3+
+ 3Sn 4+ + 7H 2 O
be the value of magnetic moment. Reduction
19. (d) In covalent compounds fluorine can form only single
Configuration No. of Unpaired es s bond while oxygen forms double bond.
3d 7 20. (c) Zr and Hf possess almost same atomic and ionic radii
3
5
because of lanthanoid contraction. Hence they exhibit
3d 5 similar properties.
3d 8
2 21. (b) HCl is not used in oxidation reactions of KMnO4 in
2
acidic medium because KMnO4 + HCl produce nascent
3d 2 oxygen which will partly oxidise HCl to Cl2. So the end
7. (b) Lanthanoids show common oxidation state of +3. point will not be correct in the titration.
8. (a) The reaction in which oxidation as well as reduction of NEET/AIPMT (2013-2017) Questions
same metal occur is known as disproportionation
22. (c) Sm2+(Z = 62) [Xe]4f6 6s2 – 6 unpaired e–
reaction.
Eu2+(Z = 63) [Xe]4f7 6s2 – 7 unpaired e–
Oxidation 2+
Yb (Z = 70) [Xe]4f14 6s2 – 0 unpaired e–
+ 2+
Cu Cu + Cu 2+
Ce (Z = 58) [Xe]4f1 5d1 6s2 – 2 unpaired e–
Reduction
Only Yb2+ is diamagnetic.
9. (d) Mn2+ acts as autocatalyst. 23. (b) HCl and SO2 are reducing agents and can reduce
2MnO-4+ 16H +
+ 5C2O42-
® 2Mn + 10CO2 + 8H2O 2+ MnO4–. CO2 which is neither oxidising and nor reduc-
ing will provide only acidic medium. It can shift reaction
10. (c) Tm ; Thulium Z = 69 do not belong to actinoid series.
in forward direction and reaction can go to completion.
11. (a) Reaction KMnO4 with sulphide ions in acidic medium
24. (c) Gd [64] = [Xe] 4f 75d16s2
is as follows :
2KMnO4 + 3H2SO4 ® K2SO4 + 2MnSO4 + 3H2O + 5[O] 25. (c) A transition element must have incomplete d-subshell.
Zinc have completely filled d subshell having 3d10
H 2 S+ [ O ] ® H 2 O + S] ´ 5 configuration. Hence do not show properties of
2KMnO 4 + 3H 2 SO 4 + 5H 2S® K 2SO 4 + 2MnSO 4 + 8H 2 O + 5S transition elements to any appreciable extent except for
their ability to form complexes.
The d- and f-block Elements 547

26. (c) Both are reducing agents. angular momentum = 2(2 + 1)h = 6h
27. (c) The structure of CrO5 is
O O 36. (a) Considering the same moles of compounds Fe2+ SO2–
4
Cr ( +6)
only Fe2+ is oxidised by KMnO4. SO2–
4 in which
O O
sulphur is in highest oxidation state cannot be oxidised.
O
Hence CrO5 has two proxy linkage. Fe 2+ SO32– ¾¾
® Fe 2Å
Now suppose the oxi. no. of Cr is x then (+4)
2–
(+6)
2– –
x + (– 1 × 4) + (– 2) = 0 SO3 —® SO4 + 2e
\ x= +6 Sulphur can be oxidised to (+6) i.e to SO 2–
4
28. (b) 2 KMnO4 + 3H2SO4 + 5H2O2 ¾®
® Fe 2Å
In FeC2 O4 ¾¾
K2SO4 + 2MnSO4 + 8H2O + 5O2
Carbon can be oxidised to (+IV) i.e to CO2
i.e., Mn++ ion and O2.
29. (b) Magnetic moment Fe(NO 2 )2 ¾¾® Fe 2Å

m= n(n + 2) (+3)
+ 2NO2– Nitrogen can be
2.83 = n(n + 2) oxidised to (+V)
¯ – – state
On solving n = 2 2NO3 + 2e
­
Ni2+ have two unpaired electron. (+5)
30. (a) The shape of f-orbitals is very much diffused and they 37. (c) Gd (64) = [Xe]4f7 5d1 6s2
have poor shielding effect. The effective nuclear charge 38. (d) (n + l) rule can be used. Titanium is a multi electron system
increases which causes the contractiion in the size of (n + l) 3s < 3p < 4s < 3d
electron charge cloud. This contraction in size is quite
¯ ¯ ¯ ¯
regular and known as lanthanoid contraction.
(3 + 0) (3 + 1) (4 + 0) (3 + 2)
31. (d) Magnetic moment = 2.84 B.M. This indicates that 2 || || || ||
unpaired electrons are present. 3 4 4 5
If (n + l) values are same, than value of "n" has to be
For Ni+2 = 4s0 3d8
considered.
Number of unpaired electrons (n) = 2 39. (d) K2Cr2O7 + SO2 + H2SO4
Hence Ni2+ gives magnetic moment 2.84 B.M. ® K2SO4 + Cr 2(SO4)3 + H2O
32. (a) Fe+2 = 3d6 (number of ‘d’ electrons = 6) green colour
In Cl = 1s2 2s2 2p6 3s2 3p5 40. (d) Eu (63) = [Xe] 4f7 6s2
total p electrons = 11, which are not equal to number of Gd (64) = [Xe] 4f7 5d1 6s2
‘d’ electrons in Fe+2 Tb (65) = [Xe] 4f9 6s2
p-electrons in Ne = 1s2 2s2 2p6 = 6 41. (a) Less active metals like Cu, Ag etc react with conc acid
s-electrons in Mg = 1s2 2s2 2p6 3s2 = 6 and form nitrate and nitrogen dioxide.
42. (a) Potassium permanganate has a purple colour. When
33. (b) Formation of Fe(CO)5 from Fe involves no change in
oxidation state of iron. sulphur dioxide reacts with potassium permanganate
the solution decolourizes.
34. (b) Due to lanthanoid contraction atomic radii of Zr and Hf
SSO2 + 2KMnO4 + 2H2O ® 2H2SO4 + 2MnSO4 + K2SO4
is almost similar.
43. (b) Minimum or comparable energy gap between 5f, 6d and
35. (d) Orbital angular momentum = l (l + 1).h
7s subshell makes electron excitation easier, hence there
For d-orbital l =2 is a greater range of oxidation states in actinoids.
EBD_7327
548 CHEMISTRY

23
Coordination
Compounds
Coordination compounds in which the central metal atom is linked Representation of Complexes
to a number of ions or neutral molecules by coordinate bonds, i.e. Werner’s representation for Fe(NH3)6Cl3
by donation of ions pairs of e–1 s by these ions or neutral molecules
to the central metal atom. Cl
NH3
DOUBLE SALT AND COMPLEX COMPOUND H 3N
NH3
The main difference between a double salt (like alums, Mohr’s Fe Cl
salt, etc.) and a complex (like ferrocyanide) ion is that former H3N NH3
dissociates completely into ions in aqueous solution and does NH3 Cl
not contain any coordinate bond while later does not dissociates Dotted lines indicate primary valency and solid lines indicate
completely into ions and always contains coordinate bonds. secondary valency of metal ion.
WERNER’S THEORY OF COMPLEXES SOME IMPORTANT TERMS USED IN
The main postulates are: COORDINATION COMPOUNDS
(i) Every metal forming a complex exhibits two types of valencies
Primary valency and Secondary valency. Coordination Entity/ Coordination Sphere
(ii) Primary valency is ionisable in nature whereas secondary The central metal atom/ ion and the ligands which are directly
valency is non-ionisable. attached to it are enclosed in square brackets and are collectively
(iii) Primary valency is also called oxidation state. Secondary called coordination entity. The ionizable groups are written outside
valency is also called Coordination number of metal the brackets and are called counter ions.
ex: éë Pt ( NH 3 ) 6 ùûCl 4 , primary valency is 4 (from 4Cl– ) Central Atom / ion:
In a Coordination entity, the atom / ion to which a fixed number of
whereas secondary valency is 6 (from 6NH3)
ions / groups are bound in a definite geometrical arrangement
(iv) Primary valencies are satisfied by negative ions whereas
secondary valencies are satisfied by negative ion or neutral around it, is called central atom / ion.
molecules. They are also referred to as Lewis acids
(v) Secondary valencies are directional and determine geometry Ligands
of the complex. On the other hand, primary valencies are non These are ions / molecules which bound to central atom / ion in
directional. Coordination entity. Ligands may be ions (ex: Cl–), small molecules
Experimental evidence to Werner’s theory of complexes can be like H2O or NH3, larger molecules like H2N CH2CH2NH2 or
provided based on : N(CH2CH2NH2)3 or even macromolecules.
(i) Precipitation of primary valencies on the addition of a suitable The number of co-ordinating or ligating groups present in a ligand
reagent. is called its denticity. Based on this, ligands are classified as:
(ii) Electrical conductance of complexes

Ligands

Unidentate /
Didentate/Bidentate Polydentate
monodentate
If two donor atoms are If several donor atoms are
If one donor atom is present in its molecule present in its molecule
present in its molecule which can coordinate which can coordinate
which can coordinate 2–
– Ex ; NH2CH2CH2NH2,C2O4 Ex: EDTA (hexadentate)
Ex: Cl , H2O, NH3
Coordination Compounds 549

Chelating Ligands NOMENCLATURE OF COORDINATION


If the ligands with two or more electron donor groups (or co- COMPOUNDS
ordinating groups) positioned in such a way that they form five or
Formulas of Mononuclear Coordination Compounds
six membered ring with central metal ion, then the ligands are
known as chelating ligands and the ring formed is called chelate Following rules are applied:
(i) The central atom is listed first.
ring. The complex which is thus formed is termed as metal chelate
(ii) The ligands are then listed in alphabetical order. The
or simply chelate while this property is called chelation.
placement of a ligand in the list does not depend on its charge.
Chelating ligands form more stable complexes. Complexes formed
(iii) Polydentate ligands are also listed alphabetically.
by ligands with smaller groups are more stable as compared to (iv) In case of abbreviated ligands (e.g., en for ethane-1, 2-
complexes formed by ligand with larger group. diamine),the first letter of the abbreviation is used to determine
Chelates are used in softening of hard water, in separation of the position of the ligand in the alphabetical order.
lanthanoids and actinoids, in qualitative analysis of metals etc. (v) The formula of the entire coordination entity, whether charged
Ambidentate Ligands or not, is enclosed in square brackets.
Ligands which can bind through two different sites are called (vi) There should be no space between the ligands and the metal
ambidentate ligands. For ex: NO 2- , SCN - , etc. within a co-ordinatiion sphere.
(vii) If the formula is written without the counter ion, the charge is
Coordination Number indicated outside the square bracket as a superscript.
The number of coordinate bonds formed with the central metal (viii) The charge of the cation (s) is balanced by the charge of the
atom/ion by the ligands. For determining Coordination number anion (s).
only sigma bonds are considered not pi-bonds even if formed. Naming of Mononuclear Coordination Compounds
Following rules are used:
For ex: [ Pt Cl6 ]2- , Coordination number of Pt = 6 (i) The cation is named first in both positively and negatively
charged Coordination entities.
3-
é Fe ( C 2 O 4 ) ù , Coordination number of Fe = 6 (ii) The ligands are named in alphabetical order before the name
ë 3û
of the central atom / ion.
Coordination Polyhedron (iii) The ligands can be neutral, anionic or cationic.
It is the spatial arrangement of ligand atoms which are directly (a) Neutral ligands are named as the molecule e.g. C5H5N
attached to the central atom / ion. is named as pyridine,
The polyhedron can be octahedral, square planar, square (C6H5)3P as Triphenyl phosphine,
pyramidal, trigonal bipyramidal or tetrahedral. H2N–CH2–CH2–NH2 as ethylene diamine.
Oxidation Number of Central Atom The neutral ligands which are not named as the molecule
are
It is the charge which the central atom would carry if all the ligands
CO carbonyl
are removed along with the e– pairs that are shared with the central
NO nitrosyl
3-
atom. For example : Oxidation number of Cu in éë Cu ( CN ) 4 ùû is H2O aqua
NH3 amine
+1 and is written as Cu (I). (b) Negative ligands end in ‘O’, i.e.,
Homoleptic and Heteroleptic Complexes Cl– chloro, SO42– sulphato NO2– Nitro
Complexes in which a metal is bound to only one kind of donor ONO– nitrito, OH– hydroxo, NH2– amido
2–
NH imido NO3– nitrato, CO32– carbonato
3+
groups are called homoleptic. For ex: é Cu ( NH 3 ) ù – –
ë 6û CH3COO acetato, CN cyano
Complexes in which a metal is bound to more than one kind of +
(c) Positive ligands end in ‘ium’ ex : NH2 - NH3
donor groups are heteroleptic : For ex: [Co(NH3)4Cl2] + +
Homonuclear and Polynuclear Complexes : Hydrazinium, NO2 (nitronium), NO (nitrosonium)
Complexes in which only one metal atom is present are (iv) Prefixes mono, di, tri, etc are used to indicate the number of
homonuclear. individual ligands in Coordination entity.
Example : K3[Fe (C2O4)3] (v) Oxidation state of the metal in cation, anion or neutral
Complexes in which more than one metal atoms are present are Coordination entity is indicated by Roman numeral in
known as polynuclear. parenthesis.
(vi) If the complex ion is a cation, the metal is named same as the
NH2 element.
For example : [Co (NH3)6] Cl3
(en)2 Co (Cationic complex is present so metal is without any suffix)
Example : Co(en)2 (SO4)2
Hexaaminecobalt (III) chloride.
If the complex ion is an anion, the name of the metal ends with
OH the suffix - ate.
EBD_7327
550 CHEMISTRY

For example : K4[Fe(CN)6] (ii) Mabcd Three isomers are possible


(anionic complex is present so suffix ate is added with metal’s a b a d a b
name) Potassium hexacyanoferrate (II)
(vii) The neutral complex molecule is named similar to that of the
M M M
complex cation.
(viii) When bidentate ligands are coordinated to the metal ion,
their repetition is indicated by prefixes like bis, tris, tetrakis d c c b c d
(i) (ii) (iii)
etc. For example :
(a) [Fe (en)3] Cl3 Example : [Pt(NH3)(NH2OH)(NO2)(py)NO2.
Trisethylenediamineiron (III) chloride Octahedral complexes showing this behaviour are of type:
(b) [Ni (gly)2] (i) Ma 4 b2
Bisglycinatonickel (II) b b
ISOMERISM IN COORDINATION COMPOUNDS a b a a
+
Isomers are two or more compounds that have the same chemical ex: éë Co ( NH3 ) 4 Cl2 ùû M M
formula but a different arrangement of atoms.Coordination
compounds show two main types of isomerism: a a a a
a b
Structural Isomerism
Cis Trans
(i) Linkage isomerism: It arises in complexes containing [ Ma4 b2 ]
ambidentate ligand. For ex: SCN. It can bind through N or S
to give M - NCS or M - SCN. For example [Cr(H2O)5(SCN)]2+ (ii) M a2 (b – b)2 i.e. with bidentate ligands b – b (cis and trans
and [Cr(H2O)5(NCS)]2+ are linkage isomers. forms)
(ii) Coordination isomers : This type of isomerism occurs when ex: CoCl2(en)2.
both cation and anion of a salt are complex. The isomerism (iii) Ma3b3 (facial and meridional isomers).
arises by the interchange of ligands between the two complex
b b
ions. a a
b b
For example : [Co(NH3)6] [Cr(CN)6] and [Cr(NH3)6][Co(CN)6]
(iii) Ionisation Isomerism: It arises when the counter ion in a M M
complex salt is itself a potential ligand and can displace ligand
which can then become the counter ion. This type of isomers a b a a
a b
yields different types of ions in solution.
Cis Trans
For ex: éë Co ( NH 3 )5 SO 4 ùû Br and éë Co ( NH 3 )5 Br ùû SO 4 [ Ma3b3 ]

(iv) Solvate/ Hydrate Isomerism: Octahedral complexes of general formula, Mabcdef, can have
The complexes which differ with respect to the number of fifteen geometrical isomers.
water molecules attached to the metal ion as ligands are called (II) Optical Isomerism
hydrate isomers. It is shown by compounds which cannot be superimposed.An
For e.g. - Cr(H2O)6Cl3 has three possible structures optically active complex is one which is asymmetric in nature
i.e., not divisible into two identical halves.
(a) [Cr(H2O)6]Cl3 violet It has two forms, dextro (d) and laevo (l), arise depending
(b) [Cr(H2O)5Cl] Cl2.H2O green upon the direction in which they rotate the plane of polarised
(c) [Cr(H2O)4Cl2]Cl . 2H2O dark green. light.
This isomerism is common in octahedral complex involving
Stereo Isomerism
bidentate ligands.
(I) Geometrical Isomerism
3+ 2+
Geometrical isomerism is most important in compounds with ex: éë Co ( en )3 ùû , cis is isomer of éë PtCl 2 ( en ) 2 ùû
coordination numbers 4 and 6. 4-coordinated complexes with
For complexes with coordination number 4, square planar
tetrahedral geometry do not exhibit cis-trans isomerism.
complexes do not show optical isomerism but tetrahedral
Square planar complexes showing this behaviour are of type: complexes having bidentate ligands, ex : [Ni(gly)2] shows
(i) Ma 2 b2 optical isomerism.
a b a a Other examples are [Co(en)3]3+, [CoCl2(en)2]+ etc.
BONDING IN COORDINATION COMPOUNDS
M M
Valence Bond Theory (VBT)
b a b b
Trans–isomer Cis–isomer
(i) According to this theory, metal - ligand bonds are formed by
the overlap of metal orbitals with those of ligands i.e., by
[Ma2b2 ] donation of e– -pairs by the ligands to empty hybridized
ex: Pt (NH3)2 Cl2 orbitals of metal.
Coordination Compounds 551

For e.g. -In the formation of [Fe(NH3)6]3+ , Fe+3 ion provides Inner and Outer Orbital Complexes
six vacant orbitals. The empty ‘d’ orbitals involved in hybridisation may be inner
In [Cu(NH3)4]2+, Cu2+ ion provides four vacant orbitals. (n – 1)d or outer nd orbitals. The complexes thus formed are
(ii) The metal orbitals undergo hybridisation, to give a set of called as Inner orbital complexes and Outer orbital complexes
hybrid orbitals of equal energy. respectively.
In case of inner orbital complex the electrons of the metal are made
(iii) The number of vacant orbitals provided is equal to the to pair up, so the complex will be either diamagnetic or less
coordination number of metal ion. paramagnetic. This type of complex is also known as low spin
(iv) Octahedral, square planar and tetrahedral complexes are formed complex. For outer orbital complex number of unpaired electrons
as a result of d2sp3 (or sp3d2), dsp2 and sp3 hydridisation will be large as the configuration of the metal remains unchanged.
respectively of the central metal atom or the ion. This type of complex is also known as high spin complex.
A few examples are:

Complex Hybridisation Magnetic behaviour Nature

(CN = 6)
[Cr(NH3)6]
3+ 2
d sp
3 paramagnetic inner orbital
3– 2 3
[Fe(CN)0] d sp octahedral slightly paramagnetic inner orbital
4– 2 3
[Fe(CN)6] d sp inner orbital
diamagnetic
3–
paramagnetic outer orbital
[CoF6] 3 2
sp d
(CN = 4)
0 3 diamagnetic —
[Ni (CO4)] sp tetrahedral
2+ 3
[Zn(NH3)4] sp diamagnetic —
2–
[Ni(CN)4] dsp
2
Square diamagnetic —
2+ 2 planar
[Cu(NH3)4] dsp paramagnetic —
(CN = 5)
3 trigonal diamagnetic
Fe (CO)5 dsp bipyramidal
The d-orbitals present in metal have the same energy in the free
Note : Number of unpaired electrons present (m = n(n + 2) BM)
state. This is called degenerate state of d-orbital. But, when a
can be calculated by determining magenatic moment value complex is formed the ligands destroy the degeneracy of these
experimentally. Thus type of hybridisation involved in complex orbitals. This effect is known as Crystal field splitting of d- orbitals.
can be predicted.
Crystal Field Theory for Octahedral Complexes :
Crystal Field Theory When a ligand approaches the metal ion, the ‘d’ orbitals of the
This theory considers M - L bond to be ionic arising from metal ion undergo splitting forming two sets i.e., higher energy
electrostatic interaction. set, eg of dx 2– y 2 and dz 2 and a lower energy set, t2g of dxy’ dyz,
and dxz
This is shown diagrammatically as:
Crystal field
splitting energy ( 0
)
dx2–y 2 dz 2

0.6
Energy

0 eg

0
Average energy of
orbitals in a spherical 0.4 0 dxy dyz dxz
crystal field

t2g
d-orbitals in free ion
Figure : Splitting of d-orbitals in octahedral field
EBD_7327
552 CHEMISTRY
The difference of energy between the two sets of d-orbitals is STABILITY OF COORDINATION COMPOUNDS
called crystal field splitting energy or crystal field stabilization
energy (CFSE). It is usually represented by the symbol Do. The stability of a complex in solution can be expressed in terms of
For any given metal cation, the magnitude of Do depends upon equilibrium constant of the dissociation equilibrium
the nature of the ligands For a general complex dissociating reaction
Spectro chemical series ˆˆ† M a + + nLx -
MLbn+ ‡ˆˆ
In general, ligands can be arranged in a series in order of increasing
field strength as given below: [M a + ][Lx - ]n
I– < Br– < SCN– < Cl– < S2– < F– < OH– Ki =
[MLbn+ ]
< C 2 O24 - < O2– < H2O < NCS– < EDTA4– < NH3 < en < NO2–
Ki = instability constant
< CN– < CO
This series is called spectrochemical series. ˆˆ† MLbn+
M a + + nLx- ‡ˆˆ
Pairing of the two sets of d-orbital depends on the magnitude of
Do and P (Do ® , CFSE and P ® pairing energy) [MLbn+ ]
bn =
[M a + ][Lx - ]n
If Do > P, EC is t 2g 4 e0g . Upto d6 pairing will occur in the t2g and
eg. will remain vacant. Ligands which produce this effect are called bn ® stability constant (\ b = 1/Ki)
strong field ligands and form low spin complex. Thus more will be stability constant more will be stability of
complex.
If Do < P, EC is t 2g3 e1g . For d5 configuration t 32 g eg2 ligands
For the reaction, M + 4L ƒ ML4, overall stability constant is
which produce this effect are called weak field ligands and form
high spin complexes. given by b4 = [ ML 4 ] / [ M ][ L ]4 whereas the stepwise stability
4 constants are:
For tetrahedral complex, D t = D0
9 (i) ˆˆ† ML
M + L ‡ˆˆ
Splitting pattern for tetrahedral complexes is just the reverse of
the splitting patterns of the octahedral complexes Dt is so small K1 = [ ML] / [ M ] [ L]
that it is unable to force the electrons to pair up.
Here, energy of eg < energy of t2g. (ii) ˆˆ† ML 2
ML + L ‡ˆˆ
Colour of the complexes
The coloured nature of solutions of coordination compounds can K 2 = [ ML2 ] / [ ML] [ L ]
also be explained on the basis of crystal field theory, because in
coordination compounds the energy difference between two sets ˆˆ† ML3
(iii) ML 2 + L ‡ˆˆ
of d-orbitals is usually small (as explained by crystal field splitting)
thus, excitation of an electron from lower energy to higher energy K3 = [ ML3 ] / [ ML 2 ][ L ]
is very easy and can be achieved even by the absorption of low
energy radiations of visible region. As the result of the absorption ˆˆ† ML 4
(iv) ML3 + L ‡ˆˆ
of such selected wavelengths of visible light, the complexes K 4 = [ ML4 ] / [ ML3 ] [ L ]
appeared coloured.
Different complexes exhibit different colours when either metal is and, b4 = K1 ´ K 2 ´ K3 ´ K 4
different or ligands are different.
In absence of ligand, crystal field splitting does not occur and or, bn = K1 ´ K 2 ´ K 3 - - - - - - K n
hence, the substance is colourless. For ex: CuSO4. 5H2 O is blue
but anhydrous CuSO4 is white. Note (i) Greater is the charge on central metal ion, greater will be
The size of D depends on - the stability of the complex
(i) Nature of the ligand : In Spectrochemical series D decreases (ii) Greater will be the basic strength of ligand stable will be
as shown below the complex
(iii) Formation of chelate rings increases the stability of the
I – < Br – < Cl – < OH – < F- < C2O 42- < H 2O < CNS– < complex.
weak ligands IMPORTANCE OF COORDINATION COMPOUNDS
< NH 3 < en < NO2 – < CN – < CO (i) In qualitative analysis : In salt analysis, the presence of a
strong ligands number of basic radicals is confirmed by converting them
(ii) Oxidation state of the metal : D is greater for M3+ than for into suitable complexes which have definite colours.
M2+ For ex : Fe3+ forms a prussian blue complex with potassium
(iii) Position of the metal in the periodic table: For a given ligand ferrocyanide solution :
and oxidation state of the metal, D increases going down in
a group eg. 4FeCl3 + 3K 4 [Fe(CN)6 ] ¾¾
® Fe4 [Fe(CN) 6 ]3 + 12KCl
D is greater in [Ru (NH3)6]3+ than in [Fe(NH3)6]3+ Ferri-ferrocyanide
(Prussian blue)
Coordination Compounds 553

(ii) In extraction of metals : The noble metals like silver and gold Bonding in Metal Carbonyls
are extracted from their ores through the formation of cyanide Bond is formed between transition metal and C of CO group.
complexes, [Ag(CN)2]– and [Au(CN)2]–. These carbonyls can have tetrahedral, trigonal bipyramidal or
(iii) In biological systems : A number of biologically important octahedral structure.
compounds are Coordination compounds, i.e., metal The M - C bond possesses both s - and p - character.
complexes. For ex : chlorophyll is a coordination compound The M - C s - bond is formed by donation of lone pair of electrons
of magnesium; haemoglobin is a complex compound of iron; from a filled bonding p2p orbital on the carbonyl carbon of CO into
vitamin B12 is a Coordination compound of cobalt, etc. a vacant d-orbital of metal.
The M - Cp - bond is formed by donation of a pair of electrons
ORGANOMETALLIC COMPOUNDS
These are compounds containing one or more metal-carbon bonds. from a filled d - orbital of metal into vacant antibonding p*2p orbital
The compounds of metalloids (Ge, Sb) and non metals (B, Si, P, As of CO. Here, donation of electrons from a filled metal d-orbital into
etc.) which are less electronegative than carbon are also a vacant antibonding p* orbital of CO occurs, thus this bonding is
categorised under this. called back bonding.
e.g., R - Zn - R Dialkyl zinc (Frankland reagent) The effect of s-bond formation strengthens the p bond and vice
R - Mg - X Alkyl Mg halide (Grignards reagent) - versa. This is called synergic effect which strengthens the bond
Compounds like between CO and metal.
O p*
P p
Sodium acetate CH3 - C- ONa
M s
Sodium ethoxide C2H5–O–Na C O
Sodium Mercaptide H3C–SNa
are not organometallic compounds as in these compounds p
metal is not directly attached with carbon atom. Synergic bonding
H3CO Synthesis of Organometallic Compounds
B(OCH3)3 is not OMC ® B-OCH3
(i) s – bonded organometallic compounds
H3CO
(CH3CH 2 )2 O
R — X + 2M ¾¾¾¾¾¾ ® R —M + MX
OCH3
(CH3CH 2 )2 O
R — X + Mg ¾¾¾¾¾¾
while CH3B(OCH3)2 is OMC ® H3C - B ® R — Mg — X
OCH3 SnCl4 + 4BuLi ¾¾
® Bu4Sn + 4 LiCl
Cyanides, carbides and carbonates are not considered OMC Tetrabutyltin
because their properties are very different from those belonging
to class of OMC’s. PbCl4 + 4C2 H5 Br ¾¾
® (C2 H5 ) 4 Pb + 4MgBrCl
Classification of Organometallic Compounds Tetraethyllead (TEL)

OMC can be classified into three main categories D


(ii) K 2 [PtCl4 ] + CH 2 = CH 2 ¾¾® K[PtCl3 (C 2 H 4 )]+ KCl
(i) s - bonded OMC : Zeise's salt
These compounds are formed mostly by non transition and
metalloid elements. 2C5 H5 MgBr + FeCl2 ¾¾
® [(C5 H 5 ) 2 Fe] + 2MgBrCl
Ferrocene
eg. R - Mg - X, (Grignard reagent)
(CH3)2Zn (Dimethyl zinc or Frankland’s reagent) Applications of Organometallic Compounds
(R)2Cd, (Dialkyl Cadmium) (i) Wilkinsons Catalyst
(ii) p - bonded OMC : [RhCl(Ph3P)] is used for selective hydrogenation of alkenes.
These are usually formed by transition metals. (ii) As heterogenous catalyst
eg. Ferrocene [Fe(h5 - C5H5)2] Zeigler – Natta Catalyst is used for low temperature
polymerisation of alkenes.
where h = No. of carbon atoms combined with metal atom.
(iii) Organic synthesis - OMC like R - Mg - X, R2 - Cd, CH3 - Li
Zeise’s salt K[PtCl3(h2 - C2H4)]
etc. are used for preparation of almost all type of organic
(iii) s and p - bonded OMC : compounds. .
Transition metals of gp. 6, 7,8,9 and 10 in their zero oxidation (iv) As petrol additive - TEL(Tetraethyl lead) is used as an
state form such type of OMC. antiknock agent in petrol.
The carbonyl compounds of these metals have both s, and p (v) In medicine - Organo arsenic compounds are used as medicine
bonds. e.g. for syphilis disease.
[Ni(CO)4], [Fe(CO)5], [Mn2(CO)10] (vi) In agriculture - Seeds are treated with ethyl mercury chloride
Metal carbonyls are used as industrial catalysts. They are to protect the plants against infection.
also used as precursors in synthesis of organic compounds.
554

CONCEPT MAP
CHEMISTRY

EBD_7327
Coordination Compounds 555

1. Which one of the following shows maximum value of 12. The number of ions formed when triamminetrichloro
paramagnetic behaviour : chromium (III) is dissolved in water :
(a) [Sc(CN)6]3– (b) [Co(CN)6]3– (a) 2 (b) 3
(c) [Fe(CN)6]3– (d) [Cr(CN)6]3– (c) 1 (d) None of these
13. The IUPAC name for the complex [Co(ONO)(NH3)5]Cl2 is
2. K 3[Al(C 2O 4 )3 ] is called
(a) pentaamminenitrito-N-cobalt(II) chloride
(a) Potassium aluminooxalate (b) pentaamminenitrito-N-cobalt(III) chloride
(b) Potassium trioxalatoaluminate (III) (c) nitrito-N-pentaamminecobalt(III) chloride
(c) Potassium aluminium (III) oxalate (d) nitrito-N-pentaamminecobalt(II) chloride
(d) Potassium trioxalatoaluminate (VI)
3. The formula for the complex, dichlorobis (urea) copper (II) is 14. K 4 [Fe(CN) 6 ] is a :
(a) [Cu{O = C (NH2)2}] Cl2 (a) double salt (b) complex compound
(b) [Cu{O = C (NH2)2}Cl]Cl (c) acid (d) base
(c) [CuCl2 {O = C(NH2)2}2]
15. [EDTA]4 - is a :
(d) [CuCl2] [{O = C (NH2)2}]2
4. Which of the following is not an organometallic compound? (a) monodentate ligand (b) bidentate ligand
(a) sodium ethoxide (c) quadridentate ligand (d) hexadentate ligand
(b) trimethyl aluminium 16. What is incorrect about homoleptic metal carbonyls ?
(c) tetraethyl lead (a) M – C s- bond is formed by donation of lone pair of
(d) ethyl magnesium bromide electrons from CO
5. The geometry of Ni(CO)4 and Ni(PPh3)2Cl2 are : (b) M – C p- bond is formed by back donation of electron
(a) both square planar from filled d-orbital of metal to vacant p- orbital of carbon
(b) tetrahedral and square planar respectively (c) M – CO bonding product synergic effect
(c) both tetrahedral (d) Metal carbonyl contain only s- bonds
(d) square planar and tetrahedral respectively 17. The compound having the lowest oxidation state of iron is:
6. The hypothetical complex chlorodiaquatriamminecobalt (III) (a) (b) K 2 FeO 4
K 4 Fe(CN) 6
chloride can be represented as
(a) [CoCl(NH3)3 (H2O)2]Cl2 (c) Fe 2O 3 (d) Fe(CO)5
(b) [Co(NH3)3 (H2O)Cl3] 18. The IUPAC name of K2[PtCl6] is
(c) [Co(NH2)3 (H2O)2 Cl] (a) hexachloroplatinate potassium
(d) [Co(NH3)3 (H2O)3]Cl3 (b) potassium hexachloroplatinate (IV)
7. According to IUPAC nomenclature sodium nitroprusside is (c) potassium hexachloroplatinate
named as (d) potassium hexachloroplatinum (IV)
(a) Sodium pentacyanonitrosylferrate (III) 19. In K4[Fe(CN)6], the E.A.N. of Fe is
(b) Sodium nitroferrocyanide (a) 33 (b) 35 (c) 36 (d) 26
(c) Sodium nitroferricyanide 20. The organometallic compound is :
(d) Sodium pentacyanonitrosylferrate (II)
8. In the coordination compound, K4[Ni(CN)4], the oxidation (a) Ti (OCOCH 3 ) 4 (b) Ti(C2 H5 )4
state of nickel is (c) Ti (OC 6 H 5 ) 4 (d) Ti (OC 2 H 5 ) 4
(a) 0 (b) +1 (c) +2 (d) –1
21. In [Ni(NH3)4]SO4, the E.A.N. of Ni is
9. Which does not give a precipitate with AgNO3 solution ?
(a) 34 (b) 35
(a) [Co(NH3)6] Cl3 (b) [Co(NH3)5Cl] Cl2
(c) 36 (d) 37
(c) [Co(NH3)4 Cl2] Cl (d) [Co(NH3)3Cl3]
22. The effective atomic number of cobalt in the complex
10. The oxidation state of Cr in [Cr ( NH 3 ) 4 Cl 2 ]+ is [Co(NH3)6]3+ is
(a) 0 (b) + 1 (c) + 2 (d) + 3 (a) 36 (b) 33 (c) 24 (d) 30
11. The IUPAC name of the coordination compound 23. The ligand called p acid is :
(a) C O (b) NH3
K 3[Fe(CN ) 6 ] is
(c) C2O42– (d) Ethylenediamine
(a) Tripotassium hexacyanoiron (II) 24. Pick out the complex compound in which the central metal
(b) Potassium hexacyanoiron (II) atom obeys EAN rule strictly
(c) Potassium hexacyanoferrate (III) (a) K4[Fe(CN)6] (b) K3[Fe(CN)6]
(d) Potassium hexacyanoferrate (II) (c) [Cr(H2O)6]Cl3 (d) [Cu(NH3)4]SO4
EBD_7327
556 CHEMISTRY

25. The number of geometrical isomers for [Pt (NH3)2 Cl2] is 36. Coordination isomerism is caused by the interchange of
(a) 2 (b) 1 (c) 3 (d) 4 ligands between the
26. Which one of the following forms, with an excess of CN– (a) cis and trans structure
(cyanide), a complex having co-ordination number two? (b) complex cation and complex anion
(a) Cu+ (b) Ag+ (c) Ni2+ (d) Al(OH)3 (c) inner sphere and outer sphere
(d) low oxidation and higher oxidation states
27. Which of the following will give maximum number of isomers? 37. The pair in which both species have same magnetic moment
(a) [Ni (C2O4) (en)2]2– (b) [Ni (en) (NH3)4]2+ (spin only value) is :
(c) [Cr (SCN)2 (NH3)4]+ (d) [Co (NH3)4 Cl2] (a) [Cr(H2O)6]2+ , [CoCI4]2–
28. Which one of the following octahedral complexes will not
show geometric isomerism? (A and B are monodentate (b) [Cr(H 2 O 6 ) 2 + , [Fe(H 2 O) 6 ]2 +
ligands) (c) [Mn(H 2 O)6 ]2+ ,[Cr(H 2O)6 ]2+
(a) [MA5B] (b) [MA2B4]
(c) [MA3B3] (d) [MA4B2] (d) [CoCl 4 ) 2 - , [Fe(H 2 O) 6 ]2+
29. Which of the following coordination compounds would 38. The ligands in anti cancer drug cis-platin are:
exhibit optical isomerism? (a) NH3, CI (b) NH3, H2O
(a) pentamminenitrocobalt(III) iodide (c) Cl, H2O (d) NO, Cl
(b) diamminedichloroplatinum(II) 39. The number of precipitable halide ions in the sample
(c) trans-dicyanobis (ethylenediamine) chromium (III) [Pt(NH3)Cl2Br]Cl will be
chloride (a) 2 (b) 3 (c) 4 (d) 1
40. Wavelength of red light is absorbed by the complex
(d) tris-(ethylendiamine) cobalt (III) bromide
(a) [Cu(CN)4]2– (b) [Cu(NH3)4]2+
30. The type of isomerism present in Pentaminenitrochromium (c) CuSO4 (d) Cu(CN)2
(III) chloride is 41. Which of the following is considered to be an anticancer
(a) optical (b) linkage species ?
(c) ionisation (d) polymerisation.
Cl CH Cl Cl
31. The correct order for the wavelength of absorption in the
visible region is : (a) Pt CH2 (b) Pt

[ Ni( NO 2 ) 6 ]4- < [ Ni( NH 3 ) 6 ]2 + Cl Cl Cl Cl


(a)
< [ Ni (H 2 O) 6 ]2 + H3N Cl H3N Cl
4- 2+ (c) Pt (d) Pt
(b) [ Ni( NO 2 ) 6 ] < [ Ni (H 2 O) 6 ]
H3N Cl Cl NH3
< [ Ni ( NH 3 ) 6 ]2 +
42. Which one of the following will show paramagnetism
2+ 2+ corresponding to 2 unpaired electrons?
(c) [ Ni( H 2 O ) 6 ] < [ Ni( NH 3 ) 6 ]
(Atomic numbers : Ni = 28, Fe = 26)
< [ Ni( NO 2 ) 6 ]4 - (a) [FeF6]3– (b) [NiCl4]2–
(c) [Fe (CN)6]3– (d) [Ni (CN)4]2–
2+ 2+
(d) [ Ni( NH 3 ) 6 ] < [ Ni ( H 2 O) 6 ] 43. [Fe(NO2)3Cl3] and [Fe(ONO)3Cl3] shows
(a) linkage isomerism (b) geometrical isomerism
< [ Ni( NO 2 ) 6 ]4 - (c) optical isomerism (d) none of the above
32. Which of the following compounds shows optical 44. The number of unpaired electrons in the complex
isomerism? [Cr(NH3)6]Br3 is (Atomic number Cr = 24)
(a) 4 (b) 1 (c) 2 (d) 3
(a) [Co(CN) 6 ]3 - (b) [Cr(C 2 O 4 )3 ]3 - 45. Atomic number of Cr and Fe are respectively 25 and 26,
which of the following is paramagnetic?
(c) [ ZnCl4 ]2 - (d) [Cu ( NH 3 ) 4 ]2 + (a) [Cr(CO)6] (b) [Fe(CO)5]
33 Which of the following ions can exhibit optical isomerism– (c) [Fe(CN)6]–4 (d) [Cr(NH3)6]+3
(a) [Co(NH3)4Cl2]+ (b) [Co(NH3)2Cl4]– 46. [Cr(H2O)6]Cl3 (at. no. of Cr = 24) has a magnetic moment of
+ 3.83 B. M. The correct distribution of 3d electrons in the
(c) Cis-[Co(en)2Cl2] (d) trans-[Co(en)2Cl2]+
Chromium of the complex is
34. Which would exhibit co-ordination isomerism
(a) [Cr(NH3)6][Co(CN)6] (b) [Co(en)2Cl2] (a) 3d 1 , (3d 2 2 )1,3d 1
xy x -y yz
(c) [Cr(NH3)6]Cl3 (d) [Cr(en)2Cl2]+ (b) 3d ,3d 1 ,3d 1
35. [Co(NH3)5NO2]Cl2 and [Co(NH3)5(ONO)]Cl2 are related to xy1 yz xz
each other as (c) 3d , 3d ,3d
xy1 yz1 z2
(a) geometrical isomers (b) optical isomers (d) (3d 1
) ,3d 2 , 3d
(c) linkage isomers (d) coordination isomers x2 - y2 z xz1
Coordination Compounds 557

47. Which one of the following cyano complexes would exhibit (c) Co must be in +3 oxidation state
the lowest value of paramagnetic behaviour ? (d) None of these
(a) [Co(CN) 6 ]3 - (b) [Fe(CN) 6 ]3 - 56. Which one of the following complexes is an outer orbital
complex ?
(c) [Mn (CN) 6 ]3 - (d) [Cr (CN) 6 ]3 - (a) [Co(NH3)6]3+ (b) [Mn(CN)6]4–
(c) [Fe(CN)6] 4– (d) [Ni(NH3)6]2+
(At. Nos : Cr = 24, Mn = 25, Fe = 26, Co = 27)
48. A chemist wants to determine the molecular geometry of the (Atomic nos. : Mn = 25; Fe = 26; Co = 27, Ni = 28)
[CoCl 4 ]2– ion. Which of the following gives the best 57. A square planar complex is formed by hybridisation of which
suggestion for a measurement and for the interpretation of atomic orbitals?
that measurement? (a) s, p , p , d (b) s, px , p y , d 2 2
(a) Using absorption spectroscopy, measure lmax then x y yz x -y
calculate Do for octahedral geometry (c) s, px , p y , d 2 (d) s, py , pz , dxy
z
(b) Measure the molecule’s magnetic moment and use the 58. Which of the following may be considered to be an
result to estimate the number of unpaired spins in the organometallic compound?
molecule. If this number is low, the geometry is likely to be (a) Nickel tetracarbonyl (b) Chlorophyll
square planar; otherwise, it is likely to be tetrahedral (c) K3 [Fe (C2O4)3] (d) [Co (en)3] Cl3
(c) Measure the molecule’s magnetic moment and use the 59. Which of the following organometallic compound is s and p
result to estimate the number of unpaired spins in the bonded?
molcule.If this number is low, the geometry is likely to be (a) [Fe (h5 – C5H5)2] (b) Fe (CH3)3
tetrahedral; otherwise, it is likely to be square planar
(c) K [PtCl3(h2 – C2H4)] (d) [Co(CO)5 NH3]2+
(d) Measure the molecule’s magnetic moment and use the
result to estimate the number of unpaired spins in the 60. In Fe2(CO)9, the two iron atoms are
molecule. If this number is low, the geometry is likely to (a) linked only directly
be tetrahedral; otherwise, it is likely to be octahedral (b) linked directly along with 3 CO molecules as bridging
49. The value of the ‘spin only’ magnetic moment for one of the ligands
following configurations is 2.84 BM. The correct one is (c) linked only through 3 CO molecules as bridging ligands
(d) joined through one CO group as bridging ligands.
(a) d 5 (in strong ligand field)
61. Among the following, which is not the p-bonded
(b) d 3 (in weak as well as in strong fields) organometallic compound?
(c) d 4 (in weak ligand fields) (a) ( CH 3 ) 4 Sn (b) K [ PtCl 3 ( h 2 - C 2 H 4 )]
(d) d 4 (in strong ligand fields) (c) Fe ( h5 - C 5 H 5 ) 2 (d) Cr ( h6 - C 6 H 6 ) 2
50. The "spin-only" magnetic moment [in units of Bohr 62. Which one of the following is the correct order of field
magneton, (µB)] of Ni2+ in aqueous solution would be strength of ligands in spectrochemical series?
(At. No. Ni = 28)
(a) I – < Cl– < F – < H2O < CN–
(a) 6 (b) 1.73 (c) 2.84 (d) 4.90
(b) F – < H2O < I – < CN – < Cl–
51. How many pairs of enantiomers are possible for following
complex compound, [M (AB) (CD) ef]n± (c) CN – < I – < F – < Cl– < H2O
(where AB, CD– Unsymmetrical bidentate ligand, e, f– (d) H2O < F – < CN – < Cl– < I–
Monodentate ligands) 63. Which of the following does not have a metal- carbon bond?
(a) 20 (b) 5 (c) 10 (d) 8 (a) Al(OC 2 H 5 ) 3 (b) C 2 H 5MgBr
52. Which has maximum paramagnetic nature ?
(c) K[Pt (C 2 H 4 )Cl 3 ] (d) Ni(CO) 4
(a) [Mn (H 2 O) 6 ]2 + (b) [Cu( NH 3 ) 4 ]2 +
64. Coordination compounds have great importance in biological
(c) [Fe(CN) 6 ]4- (d) [Cu(H 2 O) 4 ]2 + systems. In this context which of the following statements
53. The compound which is not coloured is is incorrect ?
(a) Cyanocobalamin is B12 and contains cobalt
(a) K 4 [Fe(CN)6 ] (b) K3[Fe(CN)6 ]
(b) Haemoglobin is the red pigment of blood and contains
(c) Na 2 [ CdCl 4 ] (d) Na 2 [ CuCl 4 ] iron
2+
54. Which complex of Co will have the weakest crystal field (c) Chlorophylls are green pigments in plants and contain
splitting – calcium
(a) [CoCl6]4– (b) [Co(CN)6]4– (d) Carboxypeptidase - A is an enzyme and contains zinc.
(c) [Co(NH3)6]2+ (d) [Co(en)3]2+ 65. Ammonia will not form complex with
55. A complex [CoL6]n+ where L is neutral ligand has a magnetic (a) Ag2+ (b) Pb2+ (c) Cu2+ (d) Cd2+
moment m = 4.5 B. M. Hence, 66. In Fe(CO)5, the Fe – C bond possesses
(a) Co must be in +2 oxidation state (a) ionic character (b) s-character only
(b) L must be a strong ligand
(c) p-character (d) both s and p characters
EBD_7327
558 CHEMISTRY

67. Which method can be used to distinguish 77. Which of the following complex ions is expected to absorb
visible light?
[Co(NH3 )6 ][Cr(NO 2 ) 6 ] and [Cr(NH3)6][Co(NO2)6]
(a) [Ti (en)2(NH3)2]4 + (b) [Cr (NH3)6]3 +
(a) by measurement of their conductivity
(b) by titration method (c) [Zn (NH3)6]2 + (d) [Sc (H2O)3 (NH3)3]3+
(c) by precipitation method with AgNO3 (At. no. Zn = 30, Sc = 21, Ti = 22, Cr = 24)
(d) by electrolysis of their aqueous solutions 78. Which of the following complex ion is not expected to absorb
68. The organometallic compound is : visible light ?
(a) Ti (OCOCH 3 ) 4 (b) Ti (C 2 H 4 ) 4
(a) [ Ni(CN)4 ]2- (b) [Cr(NH3 )6 ]3+
(c) Ti (OC 6 H 5 ) 4 (d) Ti (OC 2 H 5 ) 4
69. Which of the following is an organometallic compound? (c) [ Fe(H2 O)6 ]2+ (d) [ Ni(H 2O)6 ]2+
(a) Lithium methoxide
(b) Lithium acetate 79. Which one of the following complexes is not expected to
exhibit isomerism?
(c) Lithium dimethylamide
(d) Methyl lithium 2+
70. The formula of ferrocene is
(a) [ Ni(en)3 ]2+ (b) éë Ni ( NH3 )4 ( H 2O ) 2 ùû

(a) [Fe(CN)6]4– (b) [Fe(CN)6]3–


(c) é Pt ( NH3 ) Cl 2 ù (d) é Ni ( NH3 ) Cl 2 ù
(c) [Fe(CO)5] (d) [(C5H5)2Fe] ë 2 û ë 2 û
71. The p– bonded organometallic compound which has ethene 80. Of the following complex ions, which is diamagnetic in
as one of its component is nature?
(a) Zeise’s salt (b) Ferrocene
(a) [NiCl4]2– (b) [Ni(CN)4]2–
(c) Dibenzene chromium (d) Tetraethyl tin
(c) [CuCl4]2– (d) [CoF6]3–
72. Which of the following will give a pair of enantiomorphs?
(a) [Cr(NH3)6][Co(CN)6] 81. The complexes [Co(NH3)6] [Cr(CN)6 ] and [Cr(NH3 )6 ]
[Co(CN)6] are the examples of which type of isomerism?
(b) [Co(en)2Cl2]Cl
(c) [Pt(NH3)4] [PtCl6] (a) Linkage isomerism
(d) [Co(NH3)4Cl2]NO2. (en =NH2CH2CH2NH2) (b) Ionization isomerism
73. The d electron configurations of Cr 2+, Mn2+, Fe2+ and Ni2+ (c) Coordination isomerism
are 3d 4, 3d 5, 3d 6 and 3d 8 respectively. Which one of the (d) Geometrical isomerism
following aqua complexes will exhibit the minimum
paramagnetic behaviour? 82. The complex, [Pt(py)(NH3 )BrCl] will have how many
geometrical isomers ?
(a) [Fe(H2O)6]2+ (b) [Ni(H2O)6]2+
(c) [Cr(H2O)6]2+ (d) [Mn(H2O)6]2+ (a) 3 (b) 4
(At. No. Cr = 24, Mn = 25, Fe = 26, Ni = 28) (c) 0 (d) 2
74. Which of the following complexes exhibits the highest 83. Which of the following complex compounds will exhibit
paramagnetic behaviour ? highest paramagnetic behaviour?
(a) [V(gly)2(OH)2(NH3)2]+ (At. No. : Ti = 22, Cr = 24, Co = 27, Zn = 30)
(b) [Fe(en)(bpy)(NH3)2]2+ (a) [Ti (NH3)6]3+ (b) [Cr (NH3)6]3+
(c) [Co(ox)2(OH)2]2–
(c) [Co (NH3)6]3+ (d) [Zn (NH3)6]2+
(d) [Ti(NH3)6]3+
84. Red precipitate is obtained when ethanol solution of
where gly = glycine, en = ethylenediamine and bpy =
dimethylglyoxime is added to ammoniacal Ni(II). Which of
bipyridyl moities)
the following statements is not true ?
(At.nosTi = 22, V = 23, Fe = 26, Co = 27)
(a) Red complex has a square planar geometry.
75. In which of the following coordination entities the magnitude
D0 (CFSE in octahedral field) will be maximum? (b) Complex has symmetrical H-bonding
(a) [Co(H2O)6]3+ (b) [Co(NH3)6]3+ (c) Red complex has a tetrahedral geometry.
(c) [Co(CN)6] 3– (d) [Co (C2O4)3]3– (d) Dimethylglyoxime functions as bidentate ligand.
(At. No. Co = 27)
76. Which of the following does not show optical isomerism? OH
(a) [Co(NH3)3Cl3]0 (b) [Co (en) Cl2 (NH3)2]+ H3C C N
dimethylglyoxime =
(c) [Co (en)3]3+ (d) [Co (en)2Cl2]+ H3C C N
(en = ethylenediamine) OH
Coordination Compounds 559

85. Low spin complex of d 6 -cation in an octahedral field will (a) The complex involves d 2 sp3 hybridisation and is
have the following energy : octahedral in shape.
(b) The complex is paramagnetic.
-12 -12
(a) D0 + P (b) D 0 + 3P (c) The complex is an outer orbital complex
5 5
(d) The complex gives white precipitate with silver nitrate
-2 -2 solution.
(c) D 0 + 2P (d) D0 + P
5 5 93. Among the ligands NH3, en, CN– and CO the correct order
(D0= Crystal Field Splitting Energy in an octahedral field, of their increasing field strength, is :
P = Electron pairing energy)
(a) NH3 < en < CN - < CO
86. Which of the following has a square planar geometry?
(a) [PtCl4]2– (b) [CoCl4]2– (b) CN - < NH3 < CO < en
(c) [FeCl4]2– (d) [NiCl4]2–
(At. nos.: Fe = 26, Co = 27, Ni = 28, Pt = 78) (c) en < CN - < NH3 < CO

87. The coordination number and the oxidation state of the (d) CO < NH3 < en < CN -
element ‘E’ in the complex
94. Which among the following will be named as dibromidobis
[E (en)2 (C2O4)]NO2 (where (en) is ethylene diamine) are,
(ethylene diamine) chromium (III) bromide?
respectively,
(a) [Cr (en)3]Br3 (b) [Cr(en)2Br2]Br
(a) 6 and 2 (b) 4 and 2
(c) [Cr(en)Br4 ]– (d) [Cr(en)Br2]Br
(c) 4 and 3 (d) 6 and 3
95. As per IUPAC nomenclature, the name of the complex
88. Which of the following pairs represent linkage isomers? [Co(H2O)4(NH3)2]Cl3 is :
(a) [Pd(PPh3)2(NCS)2] and [Pd ( PPh3)2 (SCN)2] (a) Tetraaquadiaminecobalt (III) chloride
(b) [Co(NH3)5NO3] SO4 and [Co(NH3)5SO4]NO3 (b) Tetraaquadiamminecobalt (III) chloride
(c) [PtCl2(NH3)4] Br2 and [Pt Br 2 (NH3)4] Cl2 (c) Diaminetetraaquacoblat (II) chloride
(d) [Cu(NH3)4][Pt Cl4] and [Pt(NH3)4][CuCl4] (d) Diamminetetraaquacobalt (III) chloride
89. A solution containing 2.675 g of CoCl3. 6 NH3 (molar mass = 96. The charge on the central metal ion in the complex [Ni(CO)4]
267.5 g mol–1) is passed through a cation exchanger. The is
chloride ions obtained in solution were treated with excess
(a) + 2 (b) + 4
of AgNO3 to give 4.78 g of AgCl (molar mass = 143.5 g mol–
1). The formula of the complex is (c) 0 (d) + 3
(At. mass of Ag = 108 u) 97. The ligand N(CH2CH2NH2)3 is
(a) tridentate (b) pentadentate
(a) [Co(NH3 )6 ]Cl3 (b) [CoCl2 (NH3 )4 ]Cl
(c) tetradentate (d) bidentate
(c) [CoCl3 (NH3 )3 ] (d) [CoCl(NH3 )5 ]Cl2 98. The IUPAC name of the complex [Co(NH3)4(H2O)Cl]Cl2 is
90. Which one of the following has an optical isomer? (a) aquatetramminechloridocobalt (III) chloride
(a) [Zn(en) (NH3)2]2+ (b) [Co(en)3]3+ (b) chloridoaquatetramminechloridocobalt (III) chloride
(c) [Co(H2O)4(en)]3+ (d) [Zn(en)2 ]2+ (c) chloridoaquatetramminechloridocobalt (III) chloride
(en = ethylenediamine) (d) tetrammineaquachloridocobalt (III) chloride
91. Which one of the following complex ions has geometrical 99. The magnetic moment of [NiCl4]–2 is expected to be
isomers ? (a) 2.88 BM (b) 1.44 BM
(a) [Ni(NH3)5Br]+ (b) [Co(NH3)2(en)2]3+ (c) 5.25 BM (d) 4.91 BM
(c) [Cr(NH3)4(en)2]3+ (d) [Co(en)3]3+ 100. Ammonia will not form complex with
(en = ethylenediamine) (a) Ag2+ (b) Pb2+
92. Which of the following facts about the complex (c) Cu2+ (d) Cd2+
[Cr(NH3)6]Cl3 is wrong?
EBD_7327
560 CHEMISTRY

1. The crystal field splitting energy for octahedral (D0) and (a) Diamminedichloroplatinum (II)
tetrahedral (Dt) complexes is related as (b) Tetraamminedichlorocobalt (III) chloride
1 4 (c) Potassium hexacyanoferrate (II)
(a) D t = – D 0 (b) D t = – D 0 (d) Hexaaquochromium (III) chloride
2 9
3 2 9. One mole of the complex compound Co(NH3)5Cl3, gives 3
(c) D t = – D 0 (d) D t = – D 0 moles of ions on dissolution in water. One mole of the same
5 5
2. Among the following complexes, optical activity is possible complex reacts with two moles of AgNO3 solution to yield
in two moles of AgCl (s). The structure of the complex is
(a) [Co(NH3)3Cl3]. 2 NH3 (b) [Co(NH3)4Cl2] Cl . NH3
(a) [ Co ( NH 3 ) 6 ]3+
(c) [Co(NH3)4Cl] Cl2. NH3 (d) [Co(NH3)5Cl] Cl2
(b) [ Co ( H 2 O) 2 ( NH 3 ) 2 Cl 2 ]+ 10. The ligand N(CH2CH2NH2)3 is
(c) [ Cr ( H 2 O) 2 Cl 2 ]+ (a) tridentate (b) pentadentate
(c) tetradentate (d) bidentate
(d) [ Co ( CN ) 5 NC ] 11. Among the properties (a) reducing (b) oxidising (c)
3. Facial and meridional isomerism will be exhibited by complexing, the set of properties shown by CN– ion towards
(a) [Co(NH3)3Cl3] (b) [Co(NH3)4Cl2]Cl metal species is
(c) [Co(en)3]Cl3 (d) [Co(NH3)5Cl]Cl2 (a) c, a (b) b, c (c) a, b (d) a, b, c
4. A co-ordination complex compound of cobalt has the 12. Which one of the following has largest number of isomers ?
molecular formula containing five ammonia molecules, one
nitro group and two chlorine atoms for one cobalt atom. One (a) [ Ir ( PR 3 ) 2 H ( CO )]2 +
mole of this compound produces three mole ions in an (b) [Co( NH3 )5 Cl]2+
aqueous solution. On reacting this solution with excess of
AgNO3 solution, we get two moles of AgCl precipitate. The (c) [Ru( NH3 ) 4 Cl2 ]+
ionic formula for this complex would be (d) [Co(en ) 2 Cl 2 ]+ (R = alkyl group, en = ethylenediamine)
(a) [Co(NH3)4 (NO2) Cl] [(NH3) Cl] 13. Which among the following statements are true for the
(b) [Co (NH3)5 Cl] [Cl (NO2)]
complex [Co(NH3)6][Cr(CN)6] ?
(c) [Co (NH3)5 (NO2)] Cl2
1. It is a non-electrolyte
(d) [Co (NH3)5] [(NO2)2Cl2]
2. The magnitude of the charge on each complex ion is 3
5. Which of the following complexes exists as pair of
3. The complex will not conduct current
enantiomers?
4. The complex will exhibit coordination isomerism
(a) trans-[Co(en)2Cl2]+
5. The magnitude of the charge on each complex ion is 1
(b) [Co(NH3)4Cl2]+
(a) 1 and 4 (b) 1 and 2
(c) [Co{P(C2H5)3}2ClBr]
(c) 1 and 3 (d) 2 and 4
(d) [Cr(en)3]+3
6. Which one of the following is an inner orbital complex as 14. The correct order of magnetic moments (spin only values in
well as diamagnetic in behaviour? (Atomic number: B.M.) among is
Zn = 30, Cr = 24, Co = 27, Ni = 28) (a) [ Fe (CN ) 6 ]4 - > [ MnCl 4 ] 2 - > [CoCl 4 ] 2 -
(a) [Zn(NH3 )6]2+ (b) [Cr(NH3)6]3+ (b) [ MnCl 4 ] 2- > [ Fe (CN ) 6 ] 4 - > [CoCl 4 ] 2 -
(c) [Co(NH3)6] 3+ (d) [Ni(NH3)6]2+
7. [Co(NH3)4 (NO2)2] Cl exhibits (c) [ MnCl 4 ] 2- > [ CoCl 4 ] 2 - > [ Fe (CN ) 6 ] 4 -
(a) linkage isomerism, ionization isomerism and geometrical (d) [ Fe (CN ) 6 ]4 - > [CoCl 4 ]2 - > [ MnCl 4 ] 2 -
isomerism (Atomic nos. : Mn = 25, Fe = 26, Co = 27)
(b) ionization isomerism, geometrical isomerism and optical 15. Among the following coordination compounds/ions
isomerism 3-
(c) linkage isomerism, geometrical isomerism and optical (i) é Fe ( CN ) ù (ii) é Pt ( NH3 ) Cl 2 ù
ë 6û ë 2 û
isomerism
3+
(d) linkage isomerism, ionization isomerism and optical (iii) éë Co ( NH 3 )6 ùû (iv) éë Cr ( H 2 O )6 ùû Cl3
isomerism Which species exhibit geometrical isomerism?
8. Which one of the following has the highest molar (a) (ii) only (b) (i) and (ii)
conductivity? (c) (ii) and (iv) (d) (i) and (iii)
Coordination Compounds 561

16. Nickel (Z = 28) combines with a uninegative monodentate 24. Which of the following carbonyls will have the strongest
ligand X– to form a paramagnetic complex [NiX4]2–. The C – O bond ?
number of unpaired electron(s) in the nickel and geometry (a) [Mn (CO)6]+ (b) [Cr (CO)6]
of this complex ion are, respectively (c) [V (CO)6] – (d) [Fe (CO)5]
(a) one, square planar (b) two, square planar 25. Which one of the following is an outer orbital complex and
(c) one, tetrahedral (d) two, tetrahedral exhibits paramagnetic behaviour ?
17. A complex cation is formed by Pt (in some oxidation state) (a) [Ni(NH3)6]2+ (b) [Zn(NH3)6)]2+
with ligands (in proper number so that coordination number (c) [Cr(NH3)6]3+ (d) [Co(NH3)6]3+
of Pt becomes six). Which of the following can be its correct 26. In which of the following complexes of the Co (at. no. 27),
IUPAC name ? will the magnitude of Do be the hightest?
(a) Diammineethylenediaminedithiocyanato-S-platinum (II) (a) [Co(CN)6]3– (b) [Co(C2O4)3]3–
(b) Diammineethylenediaminedithiocyanato-S-platinate (c) [Co(H2O)6]3+ (d) [Co(NH3)6]3+
(IV) ion 27. Which of the following has an optical isomer
(c) Diammineethylenediaminedithiocyanato-S-platinum (a) [Co(en) (NH3)2]2+ (b) [Co(H2O)4(en)]3+
(c) [Co(en)2 (NH3)2] 3+ (d) [Co(NH3)3Cl] +
(IV) ion
(d) Diamminebis (ethylenediamine) dithiocyanato-S-plati- 28. Among the following metal carbonyls, the C–O bond order
num (IV) ion is lowest in
(a) [Mn(CO)6]+ (b) [Fe(CO)5]
18. [Fe(en)2 (H2 O)2 ]2+ + en ® complex (X). The correct
(c) [Cr(CO)6] (d) [V(CO)6]–
statement about the complex (X) is –
29. The IUPAC name of [Ni (NH3)4] [NiCl4] is
(a) it is a low spin complex
(a) Tetrachloronickel (II) - tetraamminenickel (II)
(b) it is diamagnetic
(b) Tetraamminenickel (II) - tetrachloronickel (II)
(c) it shows geometrical isomerism (c) Tetraamminenickel (II) - tetrachloronickelate (II)
(d) (a) and (b) both (d) Tetrachloronickel (II) - tetrachloronickelate (0)
19. Among the following, the species that is both paramagnetic 30. Both [Ni(CO)4] and [Ni(CN)4]2– are diamagnetic. The
and coloured is – hybridisations of nickel in these complexes, respectively,
(a) [MnO4]2– (b) [TiCl6]2– are
(c) [VO4] 3– (d) CrO2Cl2 (a) sp3, sp3 (b) sp3, dsp2(c) dsp2, sp3 (d) dsp2, sp2
20. 0.02 mole of [Co(NH 3 ) 5 Br]Cl 2 and 0.02 mole of 31. The spin only magnetic moment value (in Bohr magneton
units) of Cr(CO)6 is
[Co(NH3)5Cl]SO4 are present in 200 cc of a solution X. The
(a) 0 (b) 2.84 (c) 4.90 (d) 5.92
number of moles of the precipitates Y and Z that are formed
32. The ionisation isomer of [Cr(H2O)4Cl(NO2)]Cl is
when the solution X is treated with excess silver nitrate and (a) [Cr(H2O)4(O2N)]Cl2
excess barium chloride are respectively (b) [Cr(H2O)4Cl2](NO2)
(a) 0.02, 0.02 (b) 0.01, 0.02 (c) [Cr(H2O)4Cl(ONO)]Cl
(c) 0.02, 0.04 (d) 0.04, 0.02 (d) [Cr(H2O)4Cl2(NO2)].H2O
33. The correct structure of ethylenediaminetetraacetic acid
21. Crystal field stabilization energy for high spin d 4 octahedral
(EDTA) is
complex is:
HOOC – H2C CH2– COOH
(a) – 1.8 D 0 (b) – 1.6 D 0 + P (a) N – CH = CH – N
HOOC – H2C CH2– COOH
(c) – 1.2 D 0 (d) – 0.6 D 0
HOOC COOH
22. The existence of two different coloured complexes with the (b) N – CH2 – CH2 – N
HOOC COOH
composition of [Co(NH3 )4 Cl2 ]+ is due to : HOOC–H2C CH2–COOH
(c) N – CH2 – CH2 – N
(a) linkage isomerism (b) geometrical isomerism HOOC–H2C CH2–COOH
(c) coordination isomerism(d) ionization isomerism
COOH
23. The d-electron configurations of Cr2+, Mn2+, Fe2+ and Co2+ CH2
are d 4 , d 5, d 6 and d 7, respectively. Which one of the (d) HOOC–H2C H
following will exhibit minimum paramagnetic behaviour? N–CH–CH–N
H CH2–COOH
(a) [Mn(H2O)6]2+ (b) [Fe(H2O)6]2+ CH2
(c) [Co(H2O)6]2+ (d) [Cr(H2O)6]2+ HOOC
(At, nos. Cr = 24, Mn = 25, Fe = 26, Co = 27)
EBD_7327
562 CHEMISTRY

34. The complex showing a spin-only magnetic moment of (a) Diamminedichloroplatinum (II)
2.82 B.M. is : (b) Tetraamminedichlorocobalt (III) chloride
(a) Ni(CO)4 (b) [NiCl4]2– (c) Potassium hexacyanoferrate (II)
(c) Ni(PPh3)4 (d) [Ni(CN)4]2– (d) Hexaaquochromium (III) chloride
35. Among the following complexes (K-P) 44. Among the following coordination compounds/ions
K3[Fe(CN)6] (K), [Co(NH3)6]Cl3 (L), Na3[Co(oxalate)3] (M),
3-
[Ni(H2O)6]Cl2 (N), K2[Pt(CN)4] (O) and [Zn(H2O)6](NO3)2 (P) (i) é Fe ( CN ) ù (ii) é Pt ( NH3 ) Cl 2 ù
ë 6û ë 2 û
the diamagnetic complexes are
(a) K, L, M, N (b) K, M, O, P 3+
(iii) é Co ( NH 3 )6 ù (iv) éë Cr ( H 2 O )6 ùû Cl3
(c) L, M, O, P (d) L, M, N, O ë û
36. [NiCl2 {P(C2H5)2(C6H5)}2] exhibits temperature dependent Which species exhibit geometrical isomerism?
magnetic behaviour (paramagnetic/diamagnetic). The (a) (ii) only (b) (i) and (ii)
coordination geometries of Ni2+ in the paramagnetic and (c) (ii) and (iv) (d) (i) and (iii)
diamagnetic states are respectively 45. The crystal field splitting energy for octahedral (D0) and
(a) tetrahedral and tetrahedral tetrahedral (Dt) complexes is related as
(b) square planar and square planar
1 4
(c) tetrahedral and square planar (a) D t = – D0 (b) D t = – D0
2 9
(d) square planar and tetrahedral
37. Which of the following complex species is not expected to 3 2
(c) D t = – D0 (d) D t = – D0
exhibit optical isomerism ? 5 5
(a) [Co(en)3]3+ (b) [Co(en)2 Cl2]+ 46. Facial and meridional isomerism will be exhibited by
(c) [Co(NH3)3 Cl3] (d) [Co(en) (NH3)2 Cl2]+ (a) [Co(NH3)3Cl3] (b) [Co(NH3)4Cl2]Cl
38. Which one of the following is the correct order of field (c) [Co(en)3]Cl3 (d) [Co(NH3)5Cl]Cl2
strength of ligands in spectrochemical series? DIRECTIONS for Qs. 47 to 50 : These are Assertion-Reason
(a) I – < Cl– < F – < H2O < CN– type questions. Each of these question contains two statements:
(b) F – < H2O < I – < CN – < Cl– Statement-1 (Assertion) and Statement-2 (Reason). Answer these
(c) CN – < I – < F – < Cl– < H2O questions from the following four options.
(d) H2O < F – < CN – < Cl– < I– (a) Statement-1 is True, Statement-2 is True, Statement-2 is a
39. Which of the following organometallic compound is s and p correct explanation for Statement-1
bonded? (b) Statement-1 is True, Statement-2 is True ; Statement-2 is
(a) [Fe (h5 – C5H5)2] (b) Fe (CH3)3 NOT a correct explanation for Statement-1
(c) K [PtCl3(h – C2H4)] (d) [Co(CO)5 NH3]2+
2 (c) Statement-1 is True, Statement-2 is False
40. Which one of the following coordination compounds is used (d) Statement-1 is False, Statement-2 is True
to inhibit the growth of tumours? 47. Statement-1 : [FeF6]3– is a low spin complex.
(a) Trans-platin (b) EDTA complex of calcium Statement-2 : Low spin complexes have lesser number of
(c) [(Ph3P)3RhCl] (d) Cis-platin unpaired electrons.
41. Which of the following complexes exists as pair of 48. Statement-1 : NF3 is a weaker ligand than N(CH3)3.
enantiomers? Statement-2 : NF3 ionizes to give F– ions in aqueous
(a) trans-[Co(en)2Cl2]+ (b) [Co(NH3)4Cl2]+ solution.
(c) [Co{P(C2H5)3}2ClBr] (d) [Cr(en)3]+3 49. Statement-1 : [Fe(CN)6]3– is weakly paramagnetic while
42. Which of the following complex ions is diamagnetic? [Fe(CN)6]4– is diamagnetic.
(a) [FeF6]3– (b) [CoF6]3– Statement-2 : [Fe(CN)6]3– has +3 oxidation state while
(c) [Co(C2O4)3] 3– (d) [Fe(CN)6]3– [Fe(CN)6]4– has +2 oxidation state.
43. Which one of the following has the highest molar 50. Statement-1 : [Ti(H2O)6]3+ is coloured while [Sc(H2O)6]3+
conductivity? is colourless.
Statement-2 : d-d transition is not possible in [Sc(H2O)6]3+.
Coordination Compounds 563

Exemplar Questions 11. A chelating agent has two or more than two donor atoms to
1. Which of the following complexes formed by Cu2+ ions is bind to a single metal ion. Which of the following is not a
most stable? chelating agent?
(a) Cu2+ + 4NH3 ® [Cu(NH3)4]2+, log K = 11.6 (a) Thiosulphato (b) Oxalato
(b) Cu2+ + 4CN– ® [Cu(CN)4]2–, log K = 27.3 (c) Glycinato (d) Ethane-1, 2-diamine
(c) Cu2+ + 2en ® [Cu(en)2]2+, lok K = 15.4 12. Which of the following species is not expected to be a ligand?
(d) Cu2+ + 4H2O ® [Cu(H2O)4]2+, log K = 8.9 (a) NO (b) NH +4
2. The colour of the coordination compounds depends on the
crystal field splitting. What will be the correct order of (c) NH2CH2CH2NH2 (d) CO
absorption of wavelength of light in the visible region, for 13. What kind of isomerism exists between [Cr(H2O)6] Cl 3
the complexes, [Co(NH3)6]3+, [Co(CN)6]3–, [Co(H2O)6]3+. (violet) and [Cr (H2O)5Cl] Cl2 . H2O (greyish-green)?
(a) [Co(CN)6]3– > [Co(NH3)6]3+ > [Co(H2O)6]3+ (a) Linkage isomerism
(b) [Co(NH3)6]3+ > [Co(H2O)6]3+ > [Co(CN)6]3– (b) Solvate isomerism
(c) [Co(H2O)6]3+ > [Co(NH3)6]3+ > [Co(CN)6]3– (c) Ionisation isomerism
(d) [Co(CN)6]3– > [Co(NH3)6]3+ < [Co(H2O)6]3+ (d) Coordination isomerism
3. When 0.1 mol CoCl3 (NH3)5 is treated with excess of AgNO3, 14. IUPAC name of [Pt(NH3)2Cl(NO2)] is
0.2 mol of AgCl are obtained. The conductivity of solution (a) Platinum diamminechloronitrite
will correspond to (b) Chloronitrito-N-ammineplatinum (II)
(a) 1 : 3 electrolyte (b) 1 : 2 electrolyte (c) Diamminechloridonitrito-N-platinum (II)
(c) 1 : 1 electrolyte (d) 3 : 1 electrolyte (d) Diamminechloronitrito-N-plantinate (II)
4. When 1 mole of CrCl3 . 6H2O is treated with excess of AgNO3,
3 moles of AgCl are obtained. The formula of the complex is NEET/AIPMT (2013-2017) Questions
(a) [CrCl3(H2O)3] . 3H2O (b) [CrCl2(H2O)4]Cl . 2H2O
15. A magnetic moment of 1.73 BM will be shown by one among
(c) [CrCl(H2O)5]Cl2 . H2O (d) [Cr(H2O)6]Cl3
the following : [2013]
5. The correct IUPAC name of [Pt (NH3)2Cl2] is
(a) Diamminedichloridoplatinum (II) (a) [Ni(CN)4]2– (b) TiCl4
(b) Diamminedichloridoplatinum (IV) (c) [CoCl6]4– (d) [Cu(NH3)4]2+
(c) Diamminedichloridoplatinum (0) 16. An excess of AgNO3 is added to 100 mL of a 0.01 M solution
(d) Dichloridodiammineplatinum (IV) of dichlorotetraaquachromium (III) chloride. The number of
6. The stabilisation of coordination compounds due to moles of AgCl precipitated would be : [2013]
chelation is called the chelate effect. Which of the following (a) 0.002 (b) 0.003
is the most stable complex species? (c) 0.01 (d) 0.001
(a) [Fe(CO)5] (b) [Fe(CN)6]3– 17. The correct IUPAC name for [CrF2(en)2]Cl is:
(c) [Fe(C2O4)3]3– (d) [Fe(H2O)6]3+ [NEET Kar. 2013]
7. Indicate the complex ion which shows geometrical isomerism. (a) Chlorodifluoridobis (ethylene diamine) chromium (III)
(a) [Cr(H2O)4Cl2]+ (b) [Pt(NH3)3Cl]3– (b) Chlorodifluoridoethylenediaminech-romium (III) chlo-
(c) [Co(NH3)6]3– (d) [Co(CN)5(NC)]3– ride
8. The CFSE for octahedral [CoCl6]4– is 18,000 cm–1. The CFSE
(c) Difluoridobis (ethylene diamine) chromium (III) chloride
for tetrahedral [CoCl4]2– will be
(a) 18,000 cm–1 (b) 16,000 cm–1 (d) Difluorobis-(ethylene diamine) chromium (III) chloride
(c0 8,000 cm–1 (d) 20,000 cm–1 18. Crystal field splitting energy for high spin d4 octahedral
9. Due to the presence of ambidentate ligands coordination complex is: [NEET Kar. 2013]
compounds show isomerism. Palladium complexes of the (a) – 1.6 D0 (b) – 1.2 D0
type [Pd(C6H5)2 (SCN)2] and [Pd(C6H5)2 (NCS)2] are (c) – 0.6 D0 (d) – 0.8 D0
(a) linkage isomers (b) coordination isomers 19. Which is diamagnetic? [NEET Kar. 2013]
(c) ionisation isomers (d) geometrical isomers (a) [Fe(CN)6] 3– (b) [Co(F6)]3–
10. The compounds [Co(SO4) (NH3)5] Br and [Co(SO4) (NH3)5] (c) [Ni(CN)4] 2– (d) [NiCl4]2–
Cl represent 20. In a particular isomer of [Co(NH3)4Cl2]0, the Cl-Co-Cl angle
(a) linkage isomerism (b) ionisation isomerism is 90°, the isomer is known as [NEET Kar. 2013]
(c) coordination isomerism (d) no isomerism (a) Linkage isomer (b) Optical isomer
(c) cis-isomer (d) Position isomer
EBD_7327
564 CHEMISTRY

21. The anion of acetylacetone (acac) forms Co(acac)3 chelate (c) Tricyanoferrate (III) ion
with Co3+. The rings of the chelate are [NEET Kar. 2013] (d) Hexacyanidoferrate (III) ion
(a) three membered (b) five membered 29. Number of possible isomers for the complex [Co(en)2Cl2]Cl
(c) four membered (d) six membered will be (en = ethylenediamine) [2015 RS]
22. Which among the following is a paramagnetic complex? (a) 2 (b) 1
[NEET Kar. 2013] (c) 3 (d) 4
(a) Mo(CO)6 (b) [Co(NH3)6]3+ 30. Which of the following has longest C–O bond length? (Free
(c) [Pt(en)Cl2] (d) [CoBr4]2– C–O bond length in CO is 1.128Å) [2016]
(At. No. of Mo = 42, Pt = 78) (a) Ni(CO)4 (b) [Co(CO)4]–
23. Among the following complexes the one which shows zero (c) [Fe(CO)4]2– (d) [Mn(CO)6]+
crystal field stabilization energy (CFSE): [2014] 31. An example of a sigma bonded organometallic compound is :
(a) [Mn(H2O)6]3+ (b) [Fe(H2O)6 ]3+ [2017]
(c) [Co(H2O)6]2+ (d) [Co(H2O)6]3+ (a) Grignard's reagent (b) Ferrocene
24. Which of the following complexes is used as an anti-cancer (c) Cobaltocene (d) Ruthenocene
agent: [2014] 32. HgCl2 and I2 both when dissolved in water containing I ions
(a) mer-[Co(NH3)3Cl3] (b) cis-[PtCl2(NH3)2] the pair of species formed is : [2017]
(c) cis-K2[PtCl2Br2] (d) Na2CoCl4 (a) HgI2, I– (b) HgI24 - , I3-
25. Cobalt (III) chloride forms several octahedral complexes with
ammonia. Which of the following will not give test of chloride (c) Hg2I2, I– (d) HgI2 , I3-
ions with silver nitrate at 25ºC ? [2015]
33. The correct order of the stoichiometries of AgCl formed when
(a) CoCl3·4NH3 (b) CoCl3·5NH3
AgNO3 in excess is treated with the complexes : CoCl3.6NH3,
(c) CoCl3·6NH3 (d) CoCl3·3NH3 CoCl3.5NH3, CoCl3.4NH3 respectively is :- [2017]
26. Which of these statements about [Co(CN)6]3– is true ? [2015] (a) 3 AgCl, 1 AgCl, 2 AgCl
(a) [Co(CN)6 ]3– has four unpaired electrons and will be in (b) 3 AgCl, 2 AgCl, 1 AgCl
a low-spin configuration.
(c) 2 AgCl, 3 AgCl, 1 AgCl
(b) [Co(CN)6]3– has four unpaired electrons and will be in
(d) 1 AgCl, 3 AgCl, 2 AgCl
a high spin configuration.
34. Correct increasing order for the wavelengths of absorption
(c) [Co(CN)6]3– has no unpaired electrons and will be in a
in the visible region the complexes of Co3+ is :- [2017]
high-spin configurtion.
(a) [Co(H2O)6]3+, [Co(en)3]3+, [Co(NH3)6]3+
(d) [Co(CN)6]3– has no unpaired electrons and will be in a
low-spin configuration. (b) [Co(H2O)6]3+, [Co(NH3)6]3+, [Co(en)3]3+
27. The sum of coordination number and oxidation number of the (c) [Co(NH3)6]3+, [Co(en)3]3+, [Co(H2O)6]3+
metal M in the complex [M(en)2(C2O4)]Cl (where en is (d) [Co(en)3]3+, [Co(NH3)6]3+, [Co(H2O)6]3+
ethylenediamine) is: [2015 RS] 35. Pick out the correct statement with respect to [Mn(CN)6]3-
(a) 9 (b) 6 (a) It is sp3d2 hybridised and tetrahedral [2017]
(c) 7 (d) 8 (b) It is d2sp3 hybridised and octahedral
28. The name of complex ion, [Fe(CN)6 ]3– is : [2015 RS] (c) It is dsp2 hybridised and square planar
(a) Hexacyanoiron (III) ion (d) It is sp3d2 hybridised and octahedral
(b) Hexacyanitoferrate (III) ion
Coordination Compounds 565

Hints & Solutions


EXERCISE - 1 9. (d) In [Co(NH3)3Cl3], all chlorine atoms are inside the co-
ordination sphere so they will not ionise and this com-
1. (c) CN– is a strong ligand so it forces the electrons to pair pound will not give precipitate with AgNO3.
up.
10. (d) Oxidation state of Cr in [Cr ( NH 3 ) 4 Cl 2 ]+ .
Sc3+ Let it be x, 1 × x + 4 × 0 + 2 × (–1) = 1 Therefore x =3.
3d 4s 11. (c) K 3[Fe(CN ) 6 ] is Potassium hexacyanoferrate (III).
Cr3+
ƒ ƒ ƒ
ƒ ƒ †
12. (d) The given complex is [Cr(NH3)3Cl3]3+. When it is
dissolved in water it forms none other ion.
Fe3+
ƒ †

13. (b) [Co(ONO)(NH3)5]Cl2


pentaamminenitrito-N-cobalt (III) chloride
Co3+ 14. (b) Complex compounds do not dissociate into constituent
4s ions.
In Sc3+, no electron is there in d orbital, in Cr 3+ one
electron is unpaired. In Fe3+ also one electron is K4[Fe(CN)6] ¾¾ ® 4K+ + [Fe(CN6)]4–
unpaired, Co3+ has no unpaired electrons. As both Cr 3+ \ It is a complex because no CN– is formed on
dissociation.
and Fe3+ contain one electron in unpaired state so both
15. (d) [EDTA]4– is a hexadentate ligand, because it has six
may be paramagnetic. But as we know Fe has more
donor atoms and donate 6 pairs of electrons to central
magnetic properly. Fe3+ will be more paramagnetic. So metal atom in the complex.
option (c) is correct. 16. (c) It is incorrect statement.
2. (b) IUPAC name is Potassium trioxalatoaluminate (III). All other statements i.e. (a), (b) and (d) are correct.
3. (c) [CuCl2{(O = C(NH2)2}2] 17. (d) In Fe(CO)5, Fe is in minimum oxidation state (zero).
4. (a) Sodium ethoxide is not an organometallic compound 18. (b) K2[Pt Cl6]
as in this compound carbon is not directly attached Potassium hexachloroplatinate (IV)
with metal. Oxidation state of Pt is + 4 in the complex and anion is
5. (b) Both are tetrahedral. CO is a strong ligand and P(Ph 3) present in form of complex.
is a weak ligend. So, CO pair up electrons while P(PH3) 19. (c) EAN = Atomic number – Oxidation state + 2 × number
do not pair up. Oxidation state of Ni is zero in carboxyl of Ligands = 26 – 2 + 2 (6) = 36.
compound which in other Ni has + 2 oxidation state. 20. (b) Organo-metallic compound is (b) as it contains metal-
Ni(28) carbon bonds. In others, direct link of carbon with metal
is not present.
21. (a) EAN = (atomic no) – (oxidation state) + (2 × number of
Ligands) = 28 – 2 + 2 × 4 = 34
22. (a) EAN = at. no. of central atom – oxidation state + 2 ×
1442443
(no. of ligands) = 27 – 3 + 2 × 6 = 36.
sp 3 hybridised
23. (a) Due to back bonding present between metal and
Shape is tetrahedral. carbonyl ligand in metal carbonyl, CO is termed as p-
Ni2+ acid ligand.
24. (a) In complex K4[Fe(CN)6] the Fe obey EAN rule strictly.
1442443 As in this complex EAN of Fe is 36, which corresponds
sp 3 hybridised to the atomic number of krypton
Hence, according to sidwick the complex will be stable
Shape is tetrahedral. 25. (a) Pt (NH3)2 Cl2 is a disubstituted complex and shows
6. (a) Chlorodiaquatriamminecobalt (III) chloride is only cis-& trans-isomers
[CoCl( NH 3 )3 (H 2O) 2 ]Cl 2 . Cl NH3 NH3 Cl
7. (a) IUPAC name of sodium nitroprusside Na2[Fe(CN)5NO]
is sodium pentacyanonitrosylferrate (III) because in it Pt Pt
NO is neutral ligand. Hence Cl
Cl NH3 NH3
2×O.N. of Na + O.N. of Fe + 5×O.N. of CN cis trans
1×O.N. of NO = 0 26. (b) Among the given metal ions only Ag+ forms complex
2×(+1) + O.N. of Fe + 5 ×(–1) +1×0 = 0 with CN– having co-ordination number 2.
O.N. of Fe = 5 – 2 = +3, Hence ferrate (III) Ag + 2CN – ® [Ag(CN)2 ]–
8. (a) Let the O. No. of Ni in K4[Ni(CN)4] be = x then
Coordination number of metals is defined as the number
4 (+ 1) + x + (–1) × 4 = 0 Þ 4 + x – 4 = 0 of s bonds by which ligands are attached to the metal
x=0 atom.
EBD_7327
566 CHEMISTRY

in first one and nitrogen in second, so they show


27. (c) [Cr(SCN ) 2 ( NH 3 ) 4 ]+ shows linkage, geometrical and linkage isomerism.
optical isomerism. Hence produces maximum no. of 36. (b) Co-ordination isomerism is caused by the interchange
isomers. of ligands between cation and anion complexes.
28. (a) MA3B3 – 2 geometrical isomers 37. (b) [Cr(H2O)6]2+. Here Cr is in Cr2+ form
MA2B4 – 2 geometrical isomers
MA4B2 – 2 geometrical isomers
The complexes of general formula MA6 and MA5B Cr 2 +
having octahedral geometry do not show geometrical 3d 4s
isomerism.
29. (d) The optical isomers are pair of molecules which are Fe 2 +
non superimposable mirror images of each other. 3d 4s
In [Fe(H2O)]2+, Fe is in Fe2+ form. Both will have 4
en en unpaired electrons.
38. (a) The ligands in cis-platin [PtCl2(NH3)2] are Cl and NH3.
39. ˆˆ† [Pt(NH3 )Cl 2 Br]+ + Cl-
(d) [Pt(NH3 )Cl2 Br]Cl ‡ˆˆ
en Co Co en
Cl– ion is precipitable.
40. (b) The compound which appears blue green, absorb red
en en light as blue-green is complementary to red colour.
Here [Cu(NH3)4]2+ appears blue-green, so it absorbs
[Co(en)3]3+ [Co(en)3]3+ red colour.
Mirror
(dextro) (laevo) 41. (c) Diaminodichloroplatinum (II) commonly known as cis -
platin is found to have anticancer property.
The two optically active isomers are collectivity called 42. (b) As in [NiCl4]–2 Chloride ion being a weak ligand is not
enantiomers. able to pair the electrons in d orbital.
30. (b) The chemical formula of Pentaminenitrochromium (III) 43. (a) The given compound contains –NO2 group which can
chloride is donate electrons either from O– or from N– . Thus, it
é Cr ( NH3 ) NO 2 ù Cl2 shows linkage isomerism. Thus the two given
ë 5 û compounds are linkage isomers, one is nitrite –O–N =
It can exist in following two structures
O
éCr ( NH3 ) NO2 ù Cl2 and O form and the other is nitro, –N form.
ë 5 û
O
éCr ( NH3 ) ONOù Cl2 44. (d) In [Cr (NH3)6]Br3, Cr is in +3 oxidation state
ë 5 û
Therefore the type of isomerism found in this compound 3d 4s 4p
is linkage isomerism as nitro group is linked through N Cr
24
as –NO2 or through O as – ONO.
31. (a) For any metal cation, the magnitude of D0 depends d2sp3 hybridized
upon the nature of ligand. Higher the value of D0, lower 3d 4s 4p
will be the wave length absorbed. D0 is crystal field Cr
+3
stabilisation energy. 21
The value of D 0 for ligands varies as follows Its ion is octahedral in nature. Due to the presence of
H 2 O < NH3 < NO2 - three unpaired electrons it is paramagnetic.
45. (d) Cr 3+ has 4so 3d3 electronic configuration with 3
So, the wavelength absorbed will vary in reverse order unpaired electrons, hence paramagnetic. In other cases
or NO 2- < NH 3 < H 2 O pairing of d-electrons take place in presence of strong
32. (b) field ligands such as CO or CN–.
33. (c) cis-[Co(en)2Cl 2]+: cis-[M(aa)2b2] can show optical In Cr(CO)6 molecule 12 electrons are contributed by
isomerism. CO group and it contain no odd electron
34. (a) 46. (b) m = n(n + 2)
35. (c) NH3 NH3 3.83 = n(n + 2)
H3N ONO H3N NO2 on solving n = 3
Co Co
as per magnetic moment, it has three unpaired electron.
H3N NH3 H3N NH3
Cr3+ will have configuration as
NH3 NH3
Cr 1s2 2s2 2p6 3s2 3p6 3d 4 4s2
Here more than one atom function as donor, as oxygen Cr3+ 1s2 2s2 2p6 3s2 3p6 3d 3
Coordination Compounds 567

52. (a) Paramagnetic species has unpaired electron. More the


no. of unpaired electrons, more will be paramagnetic
character.
dxy dyz dzx
Complex O. S. of metal
So, 3dxy1, 3dyz1, 3dxz1 2+
é Mn ( H 2 O ) ù Mn 2+
: ë 6û
47. (a) Co3+
2+
[Co(CN)6]3– : éCu ( NH3 ) ù Cu2+
ë 4û
CN– is a strong field ligand and it causes pairing of
4-
electrons as a result number of unpaired electrons in é Fe ( CN ) ù Fe2+
ë 6û
Co3+ becomes zero and hence it has lowest value of
paramagnetic behaviour. 2+
éCu ( H 2 O ) ù Cu2+
48. (b) In the complexes of C.N. 4, square planar geometry gives ë 4û
rise to low spin (spin paired) configurations whereas Electronic configuration of the ion present in complex
tetrahedral complexes display high spin configurations. 3d 4s 4p
49. (d) d 5 –––– strong ligand field Mn2+

No. of unpaired electron = 5


t2g eg
Cu2+
m = n(n + 2) = 3 = 1.73BM
No. of unpaired electron = 1
d 3–– in weak as well as in strong field
Fe2+

t2g eg However, CN– is a strong ligand, so pairing of electrons


will occur in the complex having CN– ions.
m = 3(5) = 15 = 3.87 B.M. \ Fe2+ in presence of CN–
d 4– in weak ligand field

\ No. of unpaired electron = 0


t2g eg Thus [Mn (H2O)6]2+ having maximum no. of unpaired
electrons has maximum paramagnetic nature.
m = 4(4+2) = 24 = 4.89 53. (c) In Na2[CdCl4], Cd has oxidation state +2.
d 4– in strong ligand field So, its electronic configuration is 4d105s0
or all the 4d orbitals are fully filled.
Hence, there will be no d-d transition. So, it is colourless.
t2g eg 54. (a) Cl– is a weak field ligand.
55. (c) Co must be in +3 oxidation state and the ligand L should
m = 2(4) = 8 = 2.82.
be a weak ligand.
50. (c) The number of unpaired electrons in Ni2+(aq) = 2
Water is weak ligand hence no pairing will take place Co3+ = [Ar]3d6
spin magnetic moment = n(n + 2) = 2(2 + 2) n = 4, m = 4.5 BM.
= 8 = 2.83 56. (d) Hybridisation
51. (c) [Fe(CN)6 ]4 -,[Mn(CN)6 ]4 - ,
e e f f d 2 sp3 d 2 sp3
A f A f A e A e

B
M
D B
M
C B
M
D B
M
C
[Co(NH 3 ] 3+ ,[Ni(NH 3 ) 6 ] 2+
C D C D d 2 sp 3 sp 3d 2

Hence [ Ni( NH 3 ) 6 ]2 + is outer orbital complex.


e e f f
B f B f B e B e
M M M M 57. (b) A square planar complex is formed by hybridisation of
A D A C A D A C
s, px, py and d atomic orbitals.
C D C D x2 - y 2
e e
C A D
58. (b) Organometallic compounds are those compounds in
A
M M which a metal is bonded directly to a carbon atom of a
B D B C molecule. In chlorophyll there is bond involving
f f metal Mg
EBD_7327
568 CHEMISTRY

59. (d) [Co(CO) 5 NH 3 ]2+ . In this complex Co-atom attached


71. (a)
with NH3 through s bonding and with CO with dative H –
p-bond.
C H
Cl
O C H K+ Fe
O C O Pt
C C H
Fe Cl Cl
Ferrocene
60. (b) O=C O=C C=O Zeise’s salt
Fe
C
O C C O (C2H5)4Sn
O tetraethyl tin
Cr
61. (a) The number of carbon atom found in p bonded
organometallic compounds is indicated by the greek
letter 'h' with a number. The prefixes h2, h5 and h6
indicate that 2, 5 and 6 carbon atom are bound to the dibenzene chromium
metal in the compound.(CH3)4Sn does not involve any
pi (p) bond formation. This is a s bonded organometallic 72. (b) Non superimposable mirror images are called optical
compound. isomers and may be described as “chiral’. They are
62. (a) also called enantiomers and rotate plane polarised light
63. (a) Triethoxyaluminium has no Al – C linkage in opposite directions.
O - CH 2 CH 3 Cl Cl
Al O - CH 2 CH 3 Cl Cl
O - CH 2 CH 3
en Co Co en
64. (c) The chlorophyll molecule plays an important role in
photosynthesis, contain porphyrin ring and the metal
Mg not Ca.
65. (b) The complex formation is a characteristic of d-block elements. en en
Lead is a p-block element hence does not forms complex 73. (b) Lesser is the number of unpaired electrons smaller will
compounds.
be the paramagnetic behaviour. As Cr ++, Mn++, Fe++
66. (d) Due to some backbonding by sidewise overlapping and Ni++ contains.
between d-orbitals of metal and p-orbital of carbon, the
Fe–C bond in Fe(CO)5 has both s and p character. Cr++ (3d 4) =
67. (d) In one case, on electrolysis of aqueous solution, the
= 4 unpaired e–.
complex ion of cobalt [i.e., {Co(NH3)6}3+] of the complex
[Co(NH3)6][Cr(NO2)6] moves towards cathode (i.e., Mn++ (3d 5)=
negative electrode) and on this electrode finally cobalt
would be deposited. = 5 unpaired e–.
In another case, on electrolysis of aqueous solution,
Fe++ (3d 6) =
the complex ion of chromium
[i.e., {Cr(NH 3 ) 6 } 3+ ] of the complex = 4 unpaired e–.
[Cr(NH3)6][Co(NO2)6] moves towards cathode (i.e.,
negative electrode) and on this electrode chromium Ni++ (3d 8) =
would finally be deposited. = 2 unpaired e–.
68. (b) Organometallic compound is (b) as it contains metal- As Ni++ has minimum no. of unpaired e– thus this is
carbon bonds. In others, direct link of carbon with metal least paramagnetic.
is not present. 74. (c)
69. (d) Compounds that contain atleast one carbon metal bond 75. (c) In octahedral field the crystal field splitting of d- orbitals
are called organometallic compound. Hence, based on of a metal ion depends upon the field produced by the
above definition methyl lithium is an organometallic ligands. In general ligands can be arranged in a series
compound. In other chemical compounds carbon is not in the order of increasing fields and splittings which
linked with metal. they produce around a central metal ion. A portion of
70. (d) the series is given below.
Coordination Compounds 569

cyanide > ethylene - diamine > ammonia > pyridine > 81. (c) Coordination isomerism occurs when cationic and
thiocyanate > water > oxalate > hydroxide > fluoride > anionic complexes of different metal ions are present in
chloride > bromide > iodide. a salt. The two isomers differ in the distribution of
Out of the given ligands water, ammonia, cyanide and ligands in cation and anion e.g.,
oxalate, we can find from the above series of ligands that [Co (NH3)6] [Cr (CN)6] is an isomer of [Co (CN)6]
the maximum splitting will occur in case of cyanide (CN–) [Cr (NH3)6]
i.e. the magnitude of D0 will be maximum in case of 82. (a) Complexes of the type MABCD may exist in three
[Co(CN)6]3+. isomeric forms.
76. (a) The octahedral coordination compounds of the type A B A C
MA3B3 exhibit fac-mer isomerism.
77. (b) Since Cr3+ in the complex has unpaired electrons in the
d orbital, hence it will absorb visible light and will be
M M
coloured
Ti = [Ar]3d 2 4 s2 ; Ti4 + = 3d 0
5
Cr = [Ar] 3d 4s ; 1 Cr3+ = 3d 3
Zn= [Ar] 3d 10 4s2; Zn2+= 3d 10 D C D B
(I) (II)
Sc = [Ar] 3d 1 4s2; Sc3+ = 3d 0
2- A C A C
78. (a) [ Ni(CN)4 ] : Number of unpaired electrons = 0

[Cr(NH3 )6 ]3+ : Number of unpaired electrons = 3


M M
[ Fe(H 2 O)6 ]2+ : Number of unpaired electrons = 4
[ Ni(H 2O)6 ]2+ : Number of unpaired electrons = 2
D B B D
(III)
79. (d) In ë (
é Ni NH 3 )2 Cl ù
2 û , Ni
2 + 3
is in sp hybridisation,
thus tetrahedral in shape. Hence the four ligands are Similarly, [Pt (py) (NH3) BrCl] may exist in three isomeric
not different to exhibit optical isomerism. In tetrahedral form in which
geometry all the positions are adjacent to one another M = Pt, A = Py, B = NH3, C = Br, D = Cl.
\ geometrical isomerism is not possible.
80. (b) Ni++ = 3d 8 4s0 83. (b)
(a) [Ti(NH3)6]3+ : 3d1 configuration and thus has one
unpaired electron.
(b) [Cr(NH3)6]3+ : In this complex Cr is in +3 oxidation
Since, the coordination number of Ni in this complex is state.
4, the configuration of Ni++ at first sight shows that the Cr3+ : 3d 3 4s0
complex is paramagnetic with two unpaired electron.
However, experiments show that the complex is
diamagnetic. This is possible when the 3d electrons It has 3 unpaired electrons thus complex is paramagnetic
rearrange against the Hund’s rule as shown below. This NH3 NH3 NH3 NH 3 NH3 NH3
is in accordance with the fact that the ligand involved
here is strong i.e., CN– ion.
Ni++ (after rearrangement) (c) [Co(NH3)6]3+ : In this complex cobalt ion is in + 3
4s 4p oxidation state with 3d 6 configuration.
Co3+,[Ar]3d 6
4s 4p

Hence, now dsp2 hybridization involving one 3d, one [Co(NH3)6]3+


4s and two 4p orbitals, takes place leading to four dsp2
hybrid orbitals, each of which accepts four electron
pairs from CN– ion forming [Ni (CN)4]2– ion. NH3 NH3 NH3 NH3 NH3 NH3
[Ni (CN)4]2–
(inner orbital or d 2sp3 hybrid orbital, low spin complex)
\ diamagnetic
×× ×× ×× ×× ××
(d) In this complex Zn exists as
Zn++ ion
four dsp2 hybrid bonds Zn++ ion : 3d 10 4s0
Thus, the complex is diamagnetic as it has no unpaired
electron.
EBD_7327
570 CHEMISTRY

Zn++ ion in [Zn(NH3)4]2+ 90. (b) Option (b) shows optical isomerism [Co(en)3]3+
NH 3 NH 3 NH 3 NH 3
en 3+ en 3+

Due to presence of paired electrons complex is en Co Co en


diamagnetic in nature.
84. (c) Nickel ions are frequently detected by the formation of
en
red precipitate of the complex of nickel en
dimethylglyoxime, when heated with dimethylglyoxime. d–form Mirror –form
CH3 C NOH Complexes of Zn++ cannot show optical isomerism as
+ Ni++
CH3 C NOH they are tetrahedral complexes with plane of symmetry.

Dimethylglyoxime [Co(H2 O)4 (en)]3+ have two planes of symmetry


hence it is also optically inactive.
OH O [Zn(en)2]2+ cannot show optical isomerism

CH3 C N N C CH3 en 3+
91. (b)
Ni NH3
CH3 C N N C CH3

O OH Co

Nickel dimethylglyoxime NH3


6 2, 2, 2 0,0 en
85. (b) d – t2g eg (in low spin)
cis
12 3+
C.F.S.E = – 0.4 × 6D0 + 3P = - D + 3P NH 3
5 0
86. (a) Complexes with dsp2 hybridisation are square planar.
So, [PtCl4]2– is square planar in shape.
87. (d) In the given complex we have two bidentate ligands en Co en
(i.e en and C2O4), so coordination number of E is 6
(2 × 2 + 1 × 2 = 6)
Let the oxidation state of E in complex be x, then
[x + (–2) = 1] or x – 2 = 1
NH 3
or x = + 3, so its oxidation state is + 3
Thus option (d) is correct. trans-
88. (a) The SCN– ion can coordinate through S or N atom 92. (c) [Cr (NH3)6]Cl3 is an inner orbital complex, because in
giving rise to linkage isomerism this complex inner d-orbital is used for hybridisation
M ¬ SCN thiocyanato (d2 sp3 )
M ¬ NCS isothiocyanato. 93. (a) Ligands can be arranged in a series in the orders of
89. (a) CoCl3 .6NH3 ¾¾ ® x Cl- increasing field strength as given below :
2.675g Weak field ligands :

x Cl- + AgNO3 ¾¾
® x AgCl ¯ I- < Br - < S2 - < SCN < Cl - < N3- , F -
4.78g < Urea, OH– < oxalate
Number of moles of the complex Strong field ligands
2.675 O -- < H 2O < NCS- < EDTA < Py, NH3 <
= = 0.01 moles
267.5
Number of moles of AgCl obtained en = SO 3 – < bipy, Phen < NO2- < CH3-
4.78
= 0.03 moles < C6 H 5- < CN - < CO
=
143.5 Such a series is termed as spectrochemical series. It is
\ No. of moles of AgCl obtained an experimentally determined series based on the
= 3 × No. of moles of complex absorption of light by complexes with different ligands.
0.03 94. (b) [Cr(en)2Br2]Br
\n= =3 dibromidobis (ethylenediamine) chromium (III) bromide.
0.01
\ 3Cl– ions are precipitable 95. (d) [Co(H2O)4(NH3)2]Cl3
Hence the formula of the complex is [Co(NH3)6]Cl3 = Diamminetetraaquacobalt (III) chloride.
Coordination Compounds 571

96. (c) In case of [Ni(CO)]4, the ligand CO, is neutral thus the [Ni(NH3 )6]2+ , Ni2+ (28 – 2 = 26)
charge on Ni is zero.
97. (c) Number of donor atoms (N) in N(CH2CH2NH2)3 are
2
four. So, N(CH2CH2NH2)3 is a tetradentate ligand. sp3d ®(outer octahedral
98. (d) complex & paramagnetic)
99. (a) In [NiCl4]–2, Ni is in +2 oxidation state [Zn(NH3)6]2+ , Zn 2+ (30 – 2 = 28)
Ni2+ = 3d8 i.e. 2 unpaired e–
Magnetic moment (m) = n(n + 2) = 8 = 2.82 B.M. sp3d 2 ® (outer octahedral
100. (b) The complex formation is a characteristic of d-block complex &diamagnetic)
elements. 7. (a) The given compound may have linkage isomerism due
Lead is a p-block element hence does not forms complex to presence of NO2 group which may be in the form
compounds. –NO2 or –ONO.
EXERCISE - 2 It may have ionisation isomerism due to presence of
two ionisable group –NO 2 & –Cl. It may have
1. (b) The crystal field splitting in tetrahedral complexes is lower
geometrical isomerism in the form of cis-trans form as
4 follows :
than that in octahedral complexes, and D t = - D0 .
9 [Co(NH3)4Cl(NO2)]NO2 & [Co(NH3) (NO2)2]Cl
2. (b) It is optically active. ––– ionisation isomers.
3. (a) Octahedral complexes of the type [MA3 B 3 ] like [Co(NH3)5(NO2)2]Cl & [Co(NH3)5(ONO)2]Cl
[Co(NH3)3Cl3] exhibit fac-mer isomerism. ––– Linkage isomers
Cl Cl
NO2 NO2
H3N Cl H3N NH3
H3N NH3 H3N NO2
Co Co Co Co
H3N Cl Cl NH3 NH3 NH3 NH3 NH3

NH3 Cl NO2 NH3


fac-isomer mer-isomer Trans-form cis-form
4. (c) As it forms two moles of silver chloride thus it has two
Geometrical isomers
moles of ionisable Cl. 8. (c) K4[Fe(CN)6] produces maximum number of ions (5) in the
[Co ( NH 3 ) 5 NO 2 ]Cl 2 ® [Co ( NH 3 ) 5 NO 2 ]+ + + 2Cl - solution.
ˆˆ† 4K+ + [Fe(CN)6] –4
K4[Fe(CN)6] ‡ˆˆ
2Cl - + 2AgNO3 ® 2AgCl + 2NO 3-
9. (d) Co (NH3)5 Cl3 [Co(NH3)5Cl]+2 + 2Cl-
5. (d) Optical isomerism is generally shown by octahedral complexes
of the formula, [Ma 2 b2 c2] n±, [Mabcdef] th [M(AA) 3] n± , \ Structure is [Co (NH3)5 Cl] Cl2.
[M(AA) 2 a 2 ] n± , [M(AA) 2ab] n± and [M(AB) 3 ] th . Thus, Now [Co(NH3 )5 Cl]Cl2 + 2AgNO3
among the given compounds, only [Cr(en)3]3+ exhibits optical
isomerism and exists as a pair of enantiomers. ® [Co(NH3 )5 Cl](NO3 )2 + 2AgCl
10. (c) Number of donor atoms (N) in N(CH2CH2NH2)3 are four.
en 3+ en 3+
So, N(CH2CH2NH2)3 is a tetradentate ligand.
Cr en en Cr
11. (a) CN– ion acts good complexing as well as reducing
en en agent.
6. (c) [Co(NH3 )6 ]3+ , Co3+ (27 –3 = 24) 12. (d) Isomers
[Ru(NH3 )4 Cl2 ]+ ,[Co(NH3 )5 Cl]2+ ,
cis and trans none
2
d sp3 ® (inner octahedral
complex & diamagnetic) [ Ir ( PR 3 ) 2 H ( CO )] 2+ , [ Co ( en ) 2 Cl 2 ]+
cis and trans cis and trans and
[Cr(NH3 )6]3+ , Cr 3+ (24 –3 = 21) optical isomers
13. (d) [Co(NH 3)6][Cr(CN)6] is an ionic compound. Both cation
and anion carry 3 unit charges. It exhibits co-ordination
2
d sp3 ® (inner octahedral isomerism.
complex & paramagnetic) 14. (c) [Fe (CN)6]
4–
®
– no. of unpaired electrons = 0
EBD_7327
572 CHEMISTRY

2–
[MnCl4] ® [Co(NH 3 )5 Cl]SO 4 + BaCl 2 ®
1mole
– no. of unpaired electrons = 5 0.02 mole
2–
[CoCl4] ® [Co(NH3 )5 Cl]Cl2 + BaSO 4 (ppt.) (Z)
– no. of unpaired electrons = 3 1mole
0.02mole
The greater the number of unpaired electrons, greater
the magnitude of magnetic moment. Hence the correct 21. (d) d 4 in high spin octahedral complex
order will be eg —
[MnCl4]– – > [CoCl4]– – > [Fe(CN)6]4–
15. (a) Geometrical isomerism is possible only in square planar t 2g
complexes of the type MA2 B 2 and MA2 BC and for
octahedral complexes of the type MA4B 2 and MA4BC. CFSE = (–0.4x + 0.6y)D0
Hence only (ii) will show geometrical isomerism. Where, x ® electrons in t2g orbital
16. (d) [Ni X4]2–, the electronic configuration of Ni2+ is y ® electrons in eg orbital
CFSE = [0.6 × 1] + [–0.4 × 3] = – 0.6 D 0

22. (b)

3d 4s 4p
It contains two unpaired electrons and the hybridisation
is sp3 (tetrahedral).
17. (c) (1) [Pt(en) (SCN)2 (NH3)2]0 ; it is not ion
(2) [Pt(en) (SCN)2 (NH3 )2 ] 2+ ; it should not be
platinate trans (green)
(3) [Pt (en) (SCN)2 (NH3)2]2+ : correct name
(4) [Pt(en)2 (SCN) 2 (NH3 )2]2+ ; its coordination Cl +
HN
3 Cl
number is 8
18. (d) Complex X is [Fe(en)3]2+ ; as 'en' is a strong field ligand
pairing of electrons will take place. Co
[Fe(en)3]2+ :
3d HN
3 NH3
4s 4p NH3

cis (violet)

d 2sp3 hybridisation 23. (c) Cr2+ d4 4


Hence, hybridisation is d 2sp3 and complex is diamag-
Mn2+ d5 5
netic. As it has 3 bidentate symmetrical 'en' ligands so
it will not show geometrical isomerism.
Fe2+ d6 4
19. (a) [MnO4]2– ; Mn is in +6 oxidation state. Electronic
configuration is [Ar]18 3d 14s0 . As it contains one
unpaired electron it is paramagnetic and green coloured Co2+ d7 3
[TiCl6]2–, [VO4]3– and CrO2Cl2 are diamagnetic as all Minimum paramagnetic behaviour = [Co (H2O)6]2+
electrons are paired. 24. (a) As positive charge on the central metal atom increases,
20. (d) When excess of AgNO3 and BaCl2 are added to solution X. the less readily the metal can donate electron density
into the p* orbitals of CO ligand (donation of electron
[Co(NH3 )5 Br]Cl2 + 2AgNO3 ® density into p* orbitals of CO result in weakening of
1mole 2 moles
0.02 mole C – O bond). Hence, the C – O bond would be strongest
in [Mn(CO)6]+.
[Co(NH3 )5 Br](NO3 )2 + 2AgCl(ppt.)(Y) 25. (a) [Ni(NH3)6]2+
1mole 2 moles
0.02 ´ 2 = 0.04 mole Ni2+ = 3d 8, according to CFT = t 62g eg2 therefore,
hybridisation is sp3d 2 and complex is paramagnetic.
26. (a) In octahedral complex the magnitude of Do will be
highest in a complex having strongest ligand. Out of
Coordination Compounds 573

the given ligands CN– is strongest. So, Do will be 34. (b) [NiCl4]2–, O.S. of Ni = +2
highest for [Co(CN)6]3–. Thus option (a) is correct.
Ni(28) = 3d 8 4s2
NH3 NH3
27. (c) NH3 3d 4s 4p
NH3

en Co Co en

3d 4s 4p
en en

Enantiomers of cis- éë Co(en) 2 (NH 3 ) 2 ùû 3+


28. (d) An anionic carbonyl complex can delocalise more sp3

electron density to antibonding pi-orbital of CO and No. of unpaired electrons = 2


hence, lowers the bond order.
Magnetic moment, m = 2.82 BM.
29. (c) The correct IUPAC name of the given compound is
tetramminenickel (II) - tetrachloronickelate (II) thus (c) 35. (c) Complex No. of electrons No. of unpaired
is the correct answer. in outer d electron (s)
30. (b) In carbonyls O.S. of metal is zero orbital
In [Ni(CO)4], the oxidation state of nickel is zero. Its [Fe(CN)6]
3–
3d
5 –
1 (CN causes
configuration in Ni(CO)4 is pairing of
electrons)
3d 4s 4p
[Co(NH3 )6]
3+
3d
6 0
[Ni(CO)4] ;
[Co(oxal.)3]
3–
3d
6 0
3
sp hybridisation 2+ 8 2
[Ni(H2 O)6 ] 3d
In [Ni(CN)4]2– the oxidation state of Ni is 2+ and its [Pt(CN)4 ]
2–
5d
8 –
0 (CN causes
configuration is pairing of
3d 4s 4p electrons)
2
[Ni(CN)4] [Zn(H2 O)6]
2+
3d
10 0

2 Thus L, M, O and P are diamagnetic.


dsp hybridisation
36. (c) In both states (paramagnetic and diamagnetic) of the
Thus the hybridisations of nickel in these compounds given complex, Ni exists as Ni2+ whose electronic
are sp3 and dsp2 respectively. configuration is [Ar] 3d 84s0.
Hence (b) is the correct answer. 3d 4s 4p
31. (a) Chromium in Cr(CO)6 is in zero oxidation state and has Ni2+ :
[Ar]18 3d 54s 1 as the electronic configuration. However,
CO is a strong ligand, hence pairing up of electrons sp3
takes place leading to following configuration in In the above paramagnetic state the geometry of the
Cr(CO)6. complex is sp3 giving tetrahedral geometry.
3d 4s 4p The diamagnetic state is achieved by pairing of
­¯ ­¯ ­¯ ´ ´ ´ ´ ´´ ´´ ´´ ´´ electrons in 3d orbital.
1444444 424444444 3 3d 4s 4p
d 2 sp3 Hybridisation
Since the complex has no unpaired electron, its magnetic
moment is zero.
dsp2
32. (b) Ionisation isomer of [Cr(H2 O) 4 Cl(NO 2 )]Cl is
[Cr(H2O)4Cl2]NO2. Thus the geometry of the complex will be dsp2 giving
square planar geometry.
33. (c) The correct structure of EDTA is
37. (c) Octahedral coordination entities of the type Ma3b3
HOOC–H2C CH2–COOH exhibit geometrical isomerism. The compound exists
N – CH2 – CH2 – N
HOOC–H2C CH2–COOH both as facial and meridional isomers, both contain
plane of symmetry
EBD_7327
574 CHEMISTRY

NH3 NH3 48. (c) It is correct statement that NF3 is a weaker ligand than
NH3 N(CH3 ) 3 , the reason is that fluorine is highly
Cl NH3 Cl
electronegative therefore, it with draw electrons from
Co Co nitrogen atom. Hence, the lone pair of nitrogen atom
NH3 Cl Cl cannot be ligated. While N(CH3)3 is a strong ligand
Cl
Cl NH3 because CH3 is electron releasing group.
49. (b) Both statement-1 and statement-2 are true but
fac- mer
38. (a) statement-2 is not the correct explanation of statement-
1. [Fe(CN)6 ]3– is weakly paramagnetic as it has
39. (d) [Co(CO) 5 NH 3 ]2+ . In this complex. Co-atom attached unpaired electrons while [Fe(CN)6]2– has no unpaired
with NH3 through s bonding while with CO it is attached electron.
with dative p-bond. \ It is diamagnetic.
40. (d) 50. (a) Both statement-1 and statement-2 are true and
statement-2 is the correct explanation of statement-1.
41. (d) Optical isomerism is generally shown by octahedral
complexes of the formula, [Ma2b2c2]n±, [Mabcdef]th [Sc(H2O6]3+ has no unpaired electron in its d subshell
[M(AA) 3 ] n± , [M(AA) 2 a 2 ] n± , [M(AA) 2 ab] n± and and thus d–d transition is not possible whereas
[M(AB)3]th. Thus, among the given compounds, only [Ti(H2O)6]3+ has one unpaired electron in its d subshell
[Cr(en)3]3+ exhibits optical isomerism and exists as a which gives rise to d–d transition to impart colour.
pair of enantiomers. EXERCISE - 3
en 3+ en 3+ Exemplar Questions
Cr en en Cr
1. (b) Higher the value of log K, higher will be stability of
en en complex compound formed.
42. (c) [Co (C2O4)3]3– has Co3+ (d6 system) 2. (c) Strength of ligand increases as crystal field splitting
energy increases.
due to presence of stronger C2O42– chelating ligand
Ligand strength order : CN– > NH3 > H2O
pairing of electrons occurs in this case.
As energy separation increases, wavelength decreases.
Co3+ : Thus, the correct order is :
; Diamagnetic. [Co(H2O)6]3+ > [Co(NH3)6]3+ > [Co(CN)6]3–
3. (b) In above reaction, when 0.1 mol CoCl3 (NH3)5 is treated
43. (c) K4[Fe(CN)6] produces maximum number of ions (5) in with excess of AgNO3, 0.2 mol of AgCl are obtained
the solution. thus, there must be two free chloride ions in the solution
ˆˆ† 4K+ + [Fe(CN)6] –4
K4[Fe(CN)6] ‡ˆˆ of electrolyte as one mole of AgNO3 precipitates one
44. (a) Geometrical isomerism is possible only in square planar mole of chloride. So, molecular formula of complex will
complexes of the type MA2B2 and MA2BC and for be [Co(NH3)5 Cl] Cl2 and electrolytic solution must
octahedral complexes of the type MA4B2 and MA4BC. contain [Co(NH3)5Cl]2+ and two Cl– as constituent
Hence only (ii) will show geometrical isomerism. ions. Thus, it is 1 : 2 electrolyte.
45. (b) The crystal field splitting in tetrahedral complexes is 2Å
éëCo( NH3 )5 Clùû Cl2 ® éëCo( NH3 )5 Clùû (aq) + 2Cl - (aq)
lower than that in octahedral complexes, and
4 4. (d) 1 mole of AgNO3 precipitates one free chloride ion
Dt = - D0 . (Cl–). Here, 3 moles of AgCl are precipitated by excess
9
46. (a) Octahedral complexes of the type [MA 3 B3] like of AgNO3. Hence, there must be three free Cl– ions.
So, the formula of the complex can be [Cr(H2O)6]Cl3
[Co(NH3)3Cl3] exhibit fac-mer isomerism.
5. (a) Ligands present in the compound are
Cl Cl (i) NH3
(ii) Cl (chlorido; di prefixed to represent two ligands.)
H3N Cl H3N NH3 The oxidation number of platinum in the compound is
Co Co 2. Hence, correct IUPAC name is
Diamminedichloridoplantinum (II)
H3N Cl Cl NH3 6. (c) Ligand which chelates the metal ion are known as
chelating ligand. Here, only oxalate ion is a chelating
NH3 Cl ligand. Hence, it stabilises coordination compound by
fac-isomer mer-isomer chelating Fe3+ ion.
47. (d) [FeF6]3– is a high spin complex since F– is a weak 7. (a) [Cr(H2O)4Cl2]+ shows geometrical isomerism. The
ligand. possible geometrical isomers are
Coordination Compounds 575

Cl Cl Coordin ation compound [Cr(H 2 O) 6 ]Cl 3 and


H2O H2O H2O Cl [Cr(H2O)5Cl] H2O × Cl2 are solvate isomers, because
water is exchanged by chloride ion. This is why both
Cr Cr of them show different colour on exposure to sunlight.
14. (c) Ligands (based on priority) present are :
H2O H2O H2O H 2O (i) NH3
Cl H2O
(ii) Cl s
trans-isomer cis-isomer
8. (c) Relation between CFSE of octahedral and tetrahedral (iii) NO s 2
So, IUPAC name will be Diamminechloridonitrito-N-
4
complexes is D t = Do platinum (II).
9
NEET/AIPMT (2013-2017) Questions
According to question, D o = 18, 000cm –1 15. (d) [Cu(NH3)4]2+ hybridisation dsp2
Cu+2 – 3d9 has one unpaired e-
4 4
\ Dt = D o = ´ 18, 000cm –1
9 9
= 4 × 2,000 cm–1 = 8,000 cm–1 So magnetic moment
9. (a) NCS (thiocyanate) can bind to the metal ion in two
ways : m= n ( n + 2 ) = 1(1 + 2 )
M ¬ NCS or M ® SNC
= 3 = 1.73
Thus, coordination compounds [Pd(C6H5)2 (SCN)2]
16. (d) [Cr(H2O)4Cl2] Cl + AgNO3 ¾¾ ® AgCl +
and [Pd(C6H5)2 (NSC)2] are linkage isomers.
[Cr(H2O)4Cl2]NO3
10. (d) Isomers are the compounds having same molecular
wt 1000
formula but different structural formula. [Co(SO4)2 Molarity = ´
(NH3)5] Br and [Co(SO4)(NH3)5]Cl do not have same mol. mass vol.
molecular formula. Hence, they are not isomers. wt vol. 0.01 ´100
= molarity ´ =
11. (a) Chelating ligand has two or more donor atoms to a mol. mass 1000 1000
single metal ion e.g., = 0.001
17. (c) IUPAC name of [CrF2 (en)2]Cl is
NH2 Difluoridobis(ethylenediamine) chromium (III) chloride.
O NH2 CH2 18. (c) CFSE = (– 0.4x + 0.6y) Do
C O CH2 where
CH2 x ® e–s in t2g orbital
C O C O NH2 y ® e–s in eg orbital
O For high spin 4d complex, x = 3 & y = 1
O
\ – 0.4 × 3 + 0.6 × 1 = – 0.6 D0
Oxalato Glycinato Ethane-1, 2 diamine
Here (¬) denotes binding site. 19. (c) Ni+2 ® 3d 8 =
S
|| .. – CN– is a strong ligand and causes pairing of 3d electrons
|| |....–
S O of Ni2+.
.. ..|

O .. O \ It is diamagnetic.
Thiosulphato S2O32-( ) is not a chelating ligand
Cl
90°
Cl
NH3
because geometrically it is not favourable for S2 O32- 20. (c) Co
to chelate a metal ion. NH3
NH3
12. (b) Ligand should have a pair of electron which is loosely NH3
held and form a M – L bond. cis-isomer
e.g., :N O, NH2CH 2CH 2NH 2, :CO,
:

21. (d) Acetylacetone forms six membered stable ring


:
||

+ complexes.
Amongst the species given NH 4 does not have any
22. (d) Co2+ Þ [Ar]3d74s0, here, Br– is a weak field ligand so
pair of electron.
will not cause pairing of d-electrons in Co2+.
So, it is not expected to be a ligand.
13. (b) The compounds having same molecular formula but \ [CoBr4]2– will exhibit paramagnetic behaviour due
differ in ligands inside & outside the coordination to unpaired electrons.
sphere are solvate (hydration) isomers. 23. (b) Due to d5 configuration CFSE is zero.
EBD_7327
576 CHEMISTRY

24. (b) Since formation constant of [HgI4]2– is very large (1.9


× 10 30 ) as compared with I 3
25. (d) CoCl3 . 3NH3 will not give test for chloride ions with
(Kf = 700)
silver nitrate due to absence of ionisable chloride atoms. \ I– will preferentially combine with HgCl2.
CoCl3 . 3NH3 Þ [Co(NH3)3 Cl3] HgCl2 + 2I– ® HgI2 ¯ + 2Cl–
Red ppt
AgNO 3
[Co(NH3)3 Cl3] ¾¾¾¾ ® no ppt 2-
HgI2 + 2I ® [ HgI 4 ]
-

26. (d) In [Co(CN)6 ]–3, O.N. of Co is +3 soluble


\ Co+3 = 3d6 4s0
(b) éëCo ( NH3 ) 6 ùû Cl3 ¾¾¾¾
AgNO
3 ® 3mol of AgCl
33.
CN– is a strong field ligand
éë Co ( NH3 )5 Cl ùû Cl2 ¾¾¾¾
AgNO
\ Pairing of electrons occurs so in this complex no 3 ® 2mol of AgCl

unpaired electron is present and it is low spin complex.


éëCo ( NH3 )4 Cl2 ùû Cl ¾¾¾¾
AgNO
3 ® 1mol of AgCl
27. (a) [M (en)2 (C2O4)] Cl
Complexes are respectively [Co(NH 3 ) 6 Cl 3 ,
C2O4 = bidentate ligand, carry – 2 charge
[Co(NH3)5Cl]Cl2 and [Co(NH3)4Cl2]Cl
en = bidentate ligand, carry 0 charge
34. (d) The order of the ligand in the spectrochemical series is :
\ M carry+ 3 charge; coordination number = 6 H2O < NH3 < en
\ Sum = + 3 + 6 = 9 Hence, the wavelength of the light observed will be in
28. (d) Hexacyanidoterrate (III) ion. the order
29. (c) [Co (en)2 Cl2] Cl [Co(H2O)6]3+ < [Co(NH3)6]3+ < [Co(en)3]3+
Cl N Thus, wavelength absorbed will be in the opposite order
N N N N i.e., [Co(en)3]3+, [Co(NH3)6]3+, [Co(H2O)6]3+
Co and Co
N Cl 35. (b) In the complex [Mn(CN)6]3–, O.S. of Mn is + 3
N N
Cl Cl E.C. of Mn+3 ® 3d4
trans cis 3d 4s 4p
(I) (II)
Mirror image of (II)

N The presence of a strong field ligand CN– causes pairing


N of electrons.
N
Co
Cl N Þ
®
®

Cl 3d 4s 4p
2 3
d sp
30. (c) [Fe(CO)4]2– As, coordination number of Mn = 6, so it will form an
Since metal atom is carrying maximum –ve charge octahedral complex.
therefore it would show maximum synergic bonding as \ [Mn(CN)6]3– =
a resultant C—O bond length would be maximum.
31. (a) Grignard's reagent (RMgX) is a s-bonded organometallic [Ar] ´´ ´´ ´´ ´´ ´´ ´´
®
®

compound. 3d 4s 4p
32. (b) In a solution containing HgCl2, I2 and I–, both HgCl2
d 2 sp3
and I2 compete for I–.
Haloalkanes and Haloarenes 577

24
Haloalkanes and
Haloarenes
The replacement of H-atom(s) in a hydrocarbon, aliphatic or (b) Aryl halides : The halogen atom is attached directly to
aromatic, by halogen atom(s) results in the formation of alkyl the carbon atom of the benzene ring.
halide (haloalkane) and aryl halide (haloarene).
X X
CLASSIFICATION
On the Basis of No. of Halogen Atoms : H3C
Mono, di, tri, tetra, etc. depending on whether they contain one, Note :
two, three, four halogen atoms in structures. (a) Down the group, size of X increases, \ C– X bond length
increases down the group from F to I.
C2H5X CH2X CH2X
| | (b) Although F is more electronegative than Cl, yet dipole
CH2X CHX moment of CH3Cl is more than CH3F. This is because of
| small size of F due to which C-F bond distance (d) becomes
CH2X small in comparison to C – Cl bond distance dipole moment
mono di tri
is given as µ = q × d. Thus dipole moment of C – Cl bond is
Classification Based on Nature of C – X Bond greater in comparison to dipole moment of C – F bond.
(i) Compounds containing sp3 hybridised C–X bond METHODS OF PREPARATION OF ALKYL HALIDES
(a) Alkyl halides or halo alkanes : (i) From Alcohols
RCH2X R2CH X R3CX
1° 2° 3° R – OH + HX RX + H2O
(b) Allylic halides : In these compounds the halogen atom
is linked to an sp3 hybridised carbon atom which has a 3
C = C bond attached to it.
R – OH + PCl5 R – Cl + POCl3 + HCl
X

CH2X
Thionyl chloride method is preferred over hydrogen
(c) Benzylic halides : The tetrahedral carbon involved in chloride or phosphorus pentachloride method for the
C – X bond is linked to an aromatic ring. preparation of chloroalkanes since both the by-products
(SO2 and HCl) in this reaction being gases escape out
CH2X R
X leaving behind the chloroalkanes in almost pure state.
R' Note :
(a) Order of reactivity among HX : HI > HBr > HCl >> HF
(b) Order of reactivity among ROH : 3° > 2° > 1° > CH3OH
(ii) Compounds containing sp2 hybridised C–X bond
(c) Mixture of conc. HCl and anhydrous ZnCl2 is used for
(a) Vinylic halides : The halogen atom is attached to sp2
differentiating three types of alcohols (3° > 2° > 1°)
hybridised carbon atom of C = C bond.
under the name of Lucas reagent.
X (d) SOBr2 is less stable and SOI2 does not exist, PBr5 and
X
PI5 are highly unstable hence not used.
EBD_7327
578 CHEMISTRY

(ii) From Alkenes Note:

CCl4 (a) Chlorination and bromination can be achieved by above


C=C + X2 C C method while iodination is done in presence of
X X oxidising agent (i.e., HNO 3 or HIO 3). Direct
A vic-dihalide flourination is highly exothermic. Thus it is done by
(X = Cl, Br)
heating alkyl chlorides with inorganic fluorides (Hg2F2,
When above reaction is carried out by using Br 2/CCl4 the AgF, SbF3 etc.).
reddish brown colour of Br2 is discharged. Therefore this
(b) Benzylic hydrogens (hydrogen present on C attached
reaction is used as a test for detection of unsaturation in
directly to benzene) are more reactive, hence easily
organic molecules.
replaced than 1°, 2° or 3° hydrogens.
C=C + HX H C C X (iv) By Halogen Exchange
(with unsymmetrical alkenes, (a) Finkelstein reaction
Markovnikov’s rule followed)
R – X + NaI acetone R – I + NaX
Alkenes react with halogen acids to form haloalkanes. The (X = Cl, Br)
order of reactivity is
(b) Swarts reaction
HI > HBr > HCl > HF
Markownikoff's rule : Alkyl chlorides/ bromides is heated in presence of a
When an unsymmetrical alkene or alkyne reacts with metallic fluoride such as AgF, Hg2F2, CoF2 or SbF3 to
unsymmetrical reagent, then negative part of reagent attach form alkyl flourides.
with that carbon atom which contains lesser number of
D
hydrogen atom during the addition. R - X + AgF ¾¾
® R - F + AgX(X = Cl, Br)
For example:
PHYSICAL PROPERTIES OF ALKYL HALIDES
Br
| (i) Lower halides are gaseous in nature whereas higher halides
CH3 - CH = CH 2 + HBr ® CH3 - CH - CH3 are either liquid or solids (having 18 or more C–atoms)
Pr opene 2 - bromopropane
(ii) Alkyl halides are colourless when pure. The bromides and
Anti-Markownikoff's rule : iodides develop colour when exposed to light.
Addition of HBr (not HCl, HI and HF) on alkenes in presence
(iii) Melting and Boiling points
of peroxides takes place in anti-Markownikoff's way
(Peroxide effect). Here addition takes place via free- The m.pts and b.pts of chlorides, bromides and iodides are
radical mechanism. higher than those of analogous hydrocarbons due to
peroxide presence of dipole-dipole interactions in them besides van
CH3CH = CH 2 + HBr ¾¾¾¾® CH3CH 2CH 2Br
der waal’s forces.
(iii) From Alkanes The m.pts. and b.pt. follows the order: RI > RBr > RCl >
By free radial halogenation RF. This is because with increase in size and mass of
hv halogen, magnitude of van der Waal’s forces also increases.
CH3CH2CH3 + Cl2 CH3CHCH3 + CH3CH2CH2Cl The b.pts of isomeric haloalkanes decrease with increase
|
Cl in branching. For isomeric alkyl halides, order of boiling
(a mixture of products point is
is obtained)
Primary > Secondary > Tertiary
However, compounds containing only one type of hydrogen
(iv) Solubility
atom can be converted into monohalogenated products in
good yield by taking excess of the concerned hydrocarbon; The haloalkanes are only very slightly soluble in water. This
is because less energy is released when new interactions
examples of such compounds are CH4, CH3CH3, (CH3)4C,
are set up between haloalkane and water molecules and these
C6H5CH3 etc. The reactivity of the alkanes follows the
are not as strong as original H-bonds in water. Haloalkanes
following order: Tertiary alkane > Secondary alkane > are completely soluble in organic solvents.
Primary alkane.
Haloalkanes and Haloarenes 579

CHEMICAL PROPERTIES OF ALKYL HALIDES alc. KOH


R – CH 2CH(Br)CH3 ¾¾¾¾
D
®
1. Nucleophilic Substitution Reactions
R – CH 2 CH = CH 2 + R – CH = CH – CH 3
R – X + Nu ® R – Nu + X – (major) (minor)
This is also called b-elimination or Dehydrohalogenation
reaction.
The product formed is determined by Saytzeff’s rule i.e.
the preferred alkene is that which has greater no. of alkyl
groups attached to doubly bonded C-atoms. Ease of
dehydrohalogenation among halides is : 3° > 2° > 1°
(Willamson’s ether synthesis) Note :
Elimination reactions dominate over substitution when
strong base i.e., Bronsted base [e.g. NH 2- , Me 3CO - , OC 2 H 5
etc.) is used and alkyl halide is 3° or 2°.
(iii) Reaction with Metals
(a) Grignard reaction:
dry ether
R – X + Mg R – MgX
Note : (b) Wurtz reaction:
2R – X + 2Na R – R + NaX
dry ether
(c) 4CH 3 CH 2 Cl + 4 Pb / Na ¾¾¾¾ ®
(C 2 H5 )4 Pb+ 4NaCl + 3Pb
TEL
(d) With Zn dust : (Frankland reaction)
R X 2Zn X R ¾¾® R– R + ZnX2
alkane
(e) With Li :
dry ether
R – Cl + 2Li ¾¾¾¾® RLi + LiCl
Note : alkyl lithium
Alkyl lithiums react with copper halides to form higher
alkanes (Corey-House synthesis)
CuI R 'X
2RLi ¾¾¾
® R 2CuLi ¾¾¾
® R - R'
( - LiI) (1° halide)
(iv) Reduction
Haloalkanes on reduction produces alkanes frequently.
R – X + 2H ¾¾® R – H + HX
Alkane

MECHANISM OF NUCLEOPHILIC SUBSTITUTION


Nucleophilic substitution rxns can proceed via two mechanism
SN1 or SN2.
¢
¢ SN1 (Unimolecular Nucleophilic Substitution)
This reaction occurs in two steps. In first step, a carbocation is
O O formed from alkyl halide molecule. First step is slow step so it
||
Ag – O – C– R '
|| is also rate determining step. In second step, an attacking
R '– C– O – R + AgX nucleophile attacks on this carbocation and forms the final
+
K SH– product.
R – SH + KX
Ar – H + AlCl3 ® Ar – R (Friedel-craft reaction) CH 3 CH3
(ii) Elimination Reactions | | –
Slow step
(i) CH3 - C - X ¾¾¾¾® CH3 - CÅ + X
Alkyl halide loses a molecule of hydrogen halide when | |
heated with alc KOH and alkene is formed. CH3 CH3
Ter. butyl carbocation
EBD_7327
580 CHEMISTRY

(b) For a given alkyl group, the reactivity of halide, R – X follows


CH3 the same order in both the mechanisms:
| CH3
| R – I > R – Br > R – Cl > R – F
(ii) CH3 - CÅ + O H ¾¾¾¾
Fast step
® CH3 - C - OH (c) Polar solvents favour SN1 reactions while non-polar
| | solvents favour SN2 reactions.
CH3 CH3 Stereochemical aspects of nucleophilic substitution.
If the alkyl halide is optically active, the product formed in SN1
Rate of reaction µ [(CH3)3 C – X] It is a unimolecular
reaction is always a racemic mixture. This is due to the formation
substitution reaction. of carbocations as intermediates which, being planar (sp 2
In S N1 mechanism, carbocations are formed as hybridised) can be equally attacked by the nucleophile on either
intermediate, hence more the stability of the intermediate side of the face forming two enantiomers.
carbocation, greater are chances for their formation and R2

hence more reactive will be the parent alkyl halide for SN1 +X : |
R2 R1 - C - Z
reaction. Hence the order of reactivity of alkyl halides R1 R2 |
| +
toward SN1 reaction follows the order : 3° > 2° > 1° R3
R1 - C - X C
When the intermediate carbocation is capable of undergoing | R2
R3 |
rearrangement, lesser stable carbocation (1° < 2° < 3°) R3 Z - C - R1
Alkyl halide Carbocation |
rearranges to the more stable carbocation and hence under (sp 3 hybridised C) (sp2 hybridised) R3
such conditions unexpected product is formed. Enantiomers in
equal amounts
Note: Allylic and benzylic halides show high reactivity
Since the nucleophile attacks from the back side and the halide
towards the SN1 reaction because carbocation formed gets
ion leaves from the front side, the product obtained will have an
stabilised via resonance.
inverted configuration [Walden inversion]. This implies that if
SN2 (Bimolecular Nucleophilic Substitution) : the alkyl halide is optically active, the product will also be optically
The rate of SN2 reactions depends on the concentration of alkyl active, although the sign of rotation may be same or different.
halide as well as nucleophile, i.e. r = k [ RX ][ Nu ] . This implies H H
that both the reactants are involved in the rate-determining step, - HO C X
H C X + OH
i.e. the reaction occurs in one step only or it is a concerted
reaction. Concerted reactions occur through a transition state H H H
transition state
(an imaginary state in which both the reactant molecules are
H
partially linked to each other).
H HO C H + X¯
é H H ù
H ê d- d- ú
Nu : + C–X ê N u - - - - C| - - - - X ú H
H ê ú (Walden inversion)
ë H û
Transition state METHODS OF PREPARATION OF ARYL AND ARALKYL
H HALIDES
H H ù
d- ú -X :
H (i) By Electrophilic Substitution
---X ú ¾¾¾® Nu – C
ú H
û
Inverted product
The nucleophile attacks from the back side of the halide ion, bulkier
the alkyl group present on the carbon bearing halogen lesser will be
its tendency to undergo SN2 reaction. Thus the reactivity of alkyl
Chlorobenzene and bromobenzene can be prepared by above
halides towards SN2 mechanism is methods whereas iodobenzene cannot be done in the same
The reactivity of alkyl halides in SN2 reactions is: way because HI formed in the reaction is a powerful reducing
1° > 2° > 3° agent due to low bond dissociation enthalpy (299 kJ mol–1).
The order of reactivity among various 1° alkyl halides is It will therefore, make the reaction reversible in nature.
CH3X > C2H5X > n – C3H7X, etc.
Bulkier the alkyl group, more is the steric hindrance in the
formation of transition state and less is the reactivity of alkyl halide.
Note :
(a) 3° alkyl halides react by SN1, 1° by SN2 and 2° by either or On account of this, HI formed in the reaction is oxidised by
both of these mechanism depending upon the nature of the carrying the reaction in the presence of iodic acid (HIO3)
alkyl halide and the reagent. or conc. HNO3.
Haloalkanes and Haloarenes 581

heat CHEMICAL PROPERTIES OF ARYL AND ARALKYL


2HI + 2HNO3 ¾¾¾ ® 2NO2 + 2H 2O + I 2
HALIDES
heat
5HI + HIO3 ¾¾¾ ® 3H 2O + 3I 2 (i) Nucleophilic Substitution :
Fluorination being extremely violent is difficult to control. The halogen atom is firmly attached with the benzene
Substitution in alkyl group nucleus and acquires extra stability due to resonance (+M)
effect. Hence, the halogen atom cannot be easily replaced
CH3 CH2Cl by other atoms or group of atoms. So, aryl halides are less
reactive the than alkyl halides.
Sunlight, 383K + + +
+ Cl2
Cl Cl Cl Cl

(ii) By the Decomposition of Diazonium Salts.


(a) Sandmeyer’s reaction

+ – The halogen atom is replaced by other nucleophiles under


N2 Cl Cl drastic conditions.
CuCl Cl OH
HCl + N2
(i) NaOH, 623 K, 300 atm
+ – +
N2 Cl Br (ii) H

CuBr Cl NH2
HBr + N2
200º C
2 + 2NH3 + Cu2O 2 + 2CuCl +
High
pressure
(b) Gattermann reaction: Aniline

C6 H5 N 2+ Cl –
Cu/HCl
¾¾¾¾
® C 6 H5Cl H2O

Cl CN
C6 H5 N 2+ Cl – ¾¾¾¾
Cu /HBr
® C6 H5 Br
+ – Pyridine, 300º C
N 2 Cl I + CuCN + CuCl
(c) high pressure

KI Phenyl cyanide
+ N2
Note : Haloarenes does not undergo nucleophilic
substitution as clevage of C – X bond is difficult. When
+ –
NH2 N 2 BF4 F electron withdrawing groups like –NO2, – CN, – CHO, –
NaNO2 COOH etc. are present at ortho or para positions the bond
D
(d) + N2 + BF3 cleavage becomes easier.
HBF4
0 – 5°C (ii) Electrophilic Substitution Reactions
(a) Halogenation :
Thermal decomposition of benzenediazonium
tetrafluoroborate to give fluorobenzene is called Balz-
Schiemann reaction.
(iii) Hunsdiecker Method :

C 6 H 5 COOAg + Cl 2 (or Br2 ) ¾


¾® C 6 H 5 Cl + CO 2 + AgCl

PHYSICAL PROPERTIES OF ARYL AND ARALKYL HA- (b) Nitration :


LIDES
(i) Like alkyl halides, aryl halides are insoluble in water due to
their incapability of forming H-bonds.
(ii) Aryl halides are less polar than alkyl halides because in aryl
halides, halogen is present on sp2 hybridised carbon which
is more electronegative than the sp3 hybridised carbon of
alkyl halides.
EBD_7327
582 CHEMISTRY

(c) Sulfonation : (iii) From carbon tetrachloride (Pyrene) :


Fe / HCl reduction
CCl4 + 2[H] ¾¾ ¾ ¾ ¾ ¾ ¾® CHCl3 + HCl
Properties
(i) Chloroform is colourless with sweet smelling, liquid.
(ii) Inslouble in water and soluble in organic solvent.
(iii) Boiling point of CHCl3 is 61ºC.
(iv) It is best solvent for fats, oil and wax.
(d) Friedel-crafts reaction : (v) On exposure to air and sunlight, chlorofrom, a colourless
heavy liquid, oxidises to carbonyl chloride (phosgene), a
highly poisonous gas used in warfare.
1 light
CHCl3 + O 2 ¾¾¾® CCl3 (OH) ¾¾® COCl2 + HCl
2 unstable
To avoid this oxidation chlorofrom is always stored in dark
coloured bottles filled to the brim to exclude any air. Further
nearly 1% alcohol is also added to destroy traces of
phosgene, if formed, to harmless diethyl carbonate.
2C 2 H 5OH + Cl 2 C = O ¾
¾® (C 2 H 5O) 2 C = O+ 2HCl
Diethyl carbonate
Uses :
(iii) Reaction with Metals:
(a) As an anaesthetic agent. However, it has been replaced by
(a) Wurtz-Fiting reaction less toxic and safer anaesthetic agents.
X R (b) CHCl3 acts as a solvent for fat, waxes, rubber etc.
+ 2Na + RX + 2NaX Iodoform (Triiodomethane) CHI3 :
CHI3 was earlier used as an antiseptic for dressing wounds. Its
(b) Fittig reaction antiseptic properties are due to the liberation of iodine when
X iodoform comes in contact with skin.
2 ether
Any compound containing CH3CO– or CH3CH(OH)– group,
+ 2Na
when heated with iodine and aqueous NaOH or NaOI (sodium
(iv) Ullmann Reaction: hypoiodite) gives yellow precipitate of iodoform, this reaction
I + 2Cu + I is called iodoform reaction.
I / NaOH
CH 3.CH(OH ).R ¾¾
2 ¾¾
¾® CH 3 .CO.R
D , in a + 2CuI I / NaOH I / NaOH
¾¾ ¾ ¾¾
® ¾¾
2 ¾¾
¾® I 3C.CO.R ¾¾
2 ¾¾
¾® CHI 3 + RCOONa
Sealed tube
Iodoform
Diphenyl Carbon Tetrachloride (Tetrachloro Methane or
POLYHALOGEN COMPOUNDS Perchloromethane) :
Chloroform Preparation :
Preparation of CHCl3
hv
(i) Haloform reaction : Aldehydes and ketones with CH3CO CHCl 3 + Cl 2 ¾¾® CCl 4 + HCl
group, and alcohols with CH3CH(OH) group give this Properties :
reaction. (i) It is a colourless, non-inflammable, poisonous liquid,
CH3 - CH - R + NaOCl ¾¾ ® CH3 - C - R + H 2O + NaX soluble in alcohol and ether.
| P (ii) On heating with steam at about 773K, it undergoes oxidation
OH O
forming carbonyl chloride.
CH3 - C - R + 3NaOCl ¾¾ ® RCOO - + CHCl3 + 3NaOH
P 773 K
CCl 4 + H 2 O (steam ) ¾¾¾® COCl 2 + 2 HCl
O Uses :
(ii) Preparation of pure chloroform :
Carbon tetrachloride is used
Cl OH (i) as a solvent for oils, fats, resins
| | NaOH (ii) in dry cleaning
Cl - C - C - H ¾¾¾®
distillation CHCl3 + HCOONa + H2O (iii) as a laboratory reagent
| |
(iv) as anthelmentic (removal of worms) for hook worms and
Cl OH
Chloralhydrate (v) as a fire extinguisher under the name of pyrene.
Haloalkanes and Haloarenes 583

DDT
Preparation:

H Cl Cl
HH2 SO
SO4, ,
CCl3CH O + Cl3CHC
-H2O
H Cl
Cl Cl
DDT
(p, p'-Dichlorodiphenyl
trichloroethane)

Uses
It is priorly widely used as a insecticide but later researches have shown that DDT is very harmful for aquatic life as it is
non-biodegradeble.
584

CONCEPT MAP
CHEMISTRY

EBD_7327
Haloalkanes and Haloarenes 585

1. Under basic conditions which one suffers elimination the (a) vinyl chloride (b) benzyl chloride
most (c) chlorobenzene (d) ethyl bromide
CH3 11. When hydrochloric acid gas is treated with propene in
(a) (CH3)3 CCl (b) CH–CH2Cl presence of benzoyl peroxide, it gives
CH3 (a) 2-chloropropane (b) allyl chloride
(c) CH 3CH 2 - CH - CH 3 (d) CH3CH2CH2–CH2–Cl (c) n-propyl chloride (d) No reaction occurs
|
Cl 12. C 6 H 6Cl 6 on treatment with KOH produces

2. The total number of acyclic isomers including the (a) C 6 H 6 (b) C 6 H 6Cl 4
stereoisomers with the molecular formula C4H7Cl (c) C 6 H 3Cl 3 (d) C 6 H 6OH
(a) 11 (b) 12
13. 2-Bromopentane is heated with potassium ethoxide in
(c) 9 (d) 10
ethanol. The major product obtained is
3. Gem dihalides on treatment with alcoholic KOH give :
(a) alkyne (b) alkene (a) 2-ethoxypentane (b) pentene-1
(c) alkane (d) all of these (c) trans-2-pentene (d) cis-pentene-2
4. When two halogen atoms are attached to same carbon atom 14. When CH3CH2CHCl2 is treated with NaNH2, the product
then it is : formed is
(a) vic-dihalide (b) gem-dihalide (a) CH3 — CH = CH2 (b) CH 3 — C º CH
(c) a, w -halide (d) a , b -halide
NH2 Cl
5. Full name of DDT is (c) CH3CH2CH (d) CH3CH2CH
(a) 1, 1, 1-trichloro-2, 2-bis(p-chlorophenyl) ethane NH2 NH2
(b) 1, 1-dichloro-2, 2-diphenyl trimethylethane 15. When 2-bromobutane reacts with alcoholic KOH, the
(c) 1, 1-dichloro-2, 2-diphenyl trichloroethane reaction is called
(d) None of these (a) halogenation (b) chlorination
(c) hydrogenation (d) dehydro-halogenation
6. How many structural isomers are possible for a compound 16. Elimination of bromine from 2-bromobutane results in the
with molecular formula C3H7Cl formation of –
(a) 2 (b) 5 (a) predominantly 2-butyne
(c) 7 (d) 9 (b) predominantly 1-butene
7. Which one of the following is least reactive in a (c) predominantly 2-butene
nucleophilic substitution reaction? (d) equimolar mixture of 1 and 2-butene
(a) CH3CH2Cl (b) CH2 = CHCH2Cl 17. Light
C 3 H 8 + Cl 2 ¾¾¾® C 3 H 7 Cl + HCl is an example of
(c) (CH3)3 C – Cl (d) CH2 = CHCl
8. The compound which contains all the four 1°, 2°, 3° and 4° (a) substitution (b) elimination
carbon atoms is (c) addition (d) rearrangement reaction
(a) 2, 3-dimethyl pentane 18. The reaction of tert-butyl bromide with sodium methoxide
(b) 3-chloro-2, 3 dimethylpentane produces mainly –
(c) 2, 3, 4-trimethylpentane (a) iso-butane (b) iso-butylene
(d) 3,3-dimethylpentane (c) tert-butyl methyl ether (d) sodium tert butoxide
9. Benzene hexachloride is alc. HBr Na
19. CH3 – CH2 – CH2 – Cl ¾¾¾ ® B ¾¾¾ ® C ¾¾® D
(a) 1, 2, 3, 4, 5, 6-hexachlorocyclohexane KOH ether
(b) 1, 1, 1, 6, 6, 6-hexachlorocyclohexane In the above sequence of reactions, the product D is –
(c) 1, 6-phenyl-1, 6-chlorohexane (a) propane (b) 2, 3-dimethylbutane
(d) 1, 1-phenyl-6, 6-chlorohexane (c) hexane (d) allyl bromide
10. A compound on treatment with NaOH followed by addition 20. The compounds CHCl3 and HF lead to the formation of a
of Ag NO3 produces white precipitate at room compound of fluorine of molecular weight 70. The
compound is
temperature. The precipitate is soluble in NH 4OH . The
(a) fluoroform (b) fluoric monoxide
compound is identified as (c) fluoride dioxide (d) fluoro methanol
EBD_7327
586 CHEMISTRY

21. When two halogens are attached to same carbon atom, it is (a) X = dilute aqueous NaOH, 20°C; Y = HBr/acetic acid,
known as : 20°C
(a) vic-dihalide (b) gem-dihalide (b) X = concentrated alcoholic NaOH, 80°C; Y = HBr/
(c) a, w -dihalide (d) a, b-dihalide acetic acid, 20°C
22. Reaction of alkyl halides with aromatic compounds in (c) X = dilute aqueous NaOH, 20°C ; Y = Br2/CHCl3,
0°C
presence of anhydrous AlCl3 is known as : (d) X = concentrated alcoholic NaOH, 80°C; Y = Br2/
(a) Friedel Craft reaction (b) Corey house synthesis CHCl3, 0°C
(c) Kolbe’s synthesis (d) Beckmann rearrangement 30. n-Propyl bromide on treatment with ethanolic potassium
23. Chlorobenzene gives aniline with : hydroxide produces
(a) NH3 + Cu2O (b) NH3 + H2SO4 (a) propane (b) propene
(c) propyne (d) propanol
(c) NaNH2 (d) None of these
31. The compound which forms acetaldehyde when heated with
24. In the following sequence of reactions
dilute NaOH, is
AgCN Reduction (a) 1, 1-dichloroethane (b)1, 1, 1-trichloroethane
C 2 H 5 Br ¾¾¾¾ ® Y ; Y is
® X ¾¾¾¾¾
(c) 1-chloroethane (d)1, 2-dichloroethane
(a) n-propyl amine (b) isopropylamine
32. The number of structural and configurational isomers of a
(c) ethylamine (d) ethylmethyl amine
bromo compound, C5H9Br, formed by the addition of HBr
25. Ethanol can be prepared more easily by which reaction ? to 2-pentyne respectively are
(i) CH 3 CH 2 Br + H 2 O ¾¾
® CH 3 CH 2 OH (a) 1 and 2 (b) 2 and 4
(c) 4 and 2 (d) 2 and 1
(ii) CH3CH 2 Br + Ag 2O (in boiling water) 33. Chlorination of toluene in the presence of light and heat
followed by treatment with aqueous KOH gives
¾¾
® CH3CH 2OH (a) o-cresol (b) m-cresol
(a) by (i) reaction (c) p-cresol (d) benzyl alcohol
(b) by (ii) reaction 34. Isobutyl magnesium bromide with dry ether and ethyl
alcohol gives :
(c) Both reactions proceed at same rate
(d) by none (a) CH 3 CHCH 2 OH & CH 3CH 2 MgBr
|
26. The reaction conditions leading to the best yields of C2H5Cl CH3
are :
UV light
(b) CH3 CHCH3 & MgBr(OC2 H5 )
(a) C2H6 (excess) + Cl2 ¾¾¾¾® |
CH3
dark
(b) C2H6 + Cl2 ¾room
¾ ¾ ¾ ¾ ¾¾®
temperatur e (c) CH3CHCH = CH 2 & Mg(OH)Br
|
UV light CH3
(c) C2H6 + Cl2 (excess) ¾¾¾¾®
UV light (d) CH 3 CHCH 3 & CH 3CH 2 OMgBr
(d) C2H6 + Cl2 ¾¾¾¾® |
27. The best method for the conversion of an alcohol into an CH3
alkyl chloride is by treating the alcohol with 35. During debromination of meso-2,3-dibromobutane, the
(a) PCl5 major compound formed is
(b) dry HCl in the presence of anhydrous ZnCl2 (a) n-butane (b) 1-butene
(c) SOCl2 in presence of pyridine (c) cis-2-butene (d) trans-2-butene
(d) None of these 36. Which of the following isomeric heptanes can yield seven
28. (CH3)3CMgCl on reaction with D2O produces : different monochlorinated products upon free radical
(a) (CH3)3 CD (b) (CH3)3OD chlorination?
(c) (CD3)3CD (d) (CD3)3OD. (a) 3-methylhexane (b) 2,2-dimethylpentane
(c) 2-methylhexane (d) 2,3-dimethylpentane
29. Identify the set of reagents 'X' and 'Y' in the following set 37. Benzene reacts with n-propyl chloride in the presence of
of transformations anhydrous AlCl3 to give
X Y (a) 3 – Propyl – 1 – chlorobenzene
CH3 - CH 2 - CH 2 Br ¾¾® Product ¾¾® CH3 - CH - CH3 (b) n-Propylbenzene
|
Br (c) Isopropylbenzene.
(d) No reaction occurs
Haloalkanes and Haloarenes 587

38. Methyl bromide reacts with AgF to give methyl fluoride (a) N, N-dimethyl aniline
and AgBr. This reaction is called (b) phenyl lithium (C 6 H 5Li)
(a) Finkelstein reaction (b) Swarts reaction
(c) Fittig reaction (d) Wurtz reaction (c) para chloro-N, N-dimethyl aniline
39. Bromobenzene reacts with Mg in dry ether to give a (d) meta chloro-N, N-dimethyl aniline
compound (A) which further reacts with ethanol to yield 47. To prepare 3-ethylpentan-3-ol the reagents needed are –
(a) phenol (b) benzene (a) CH3CH2MgBr + CH3COCH2CH3
(c) ethylbenzene (d) phenyl ether. (b) CH3MgBr + CH3CH2CH2COCH2CH3
40. Phenyl magnesium bromide reacts with methanol to give (c) CH3CH2MgBr + CH3CH2COCH2CH3
(a) a mixture of toluene and Mg(OH)Br (d) CH3CH2CH2MgBr + CH3COCH2CH3
(b) a mixture of phenol and Mg(Me)Br 48. Which one of the following alkyl halides has the lowest
(c) a mixture of anisole and Mg(OH)Br boiling point?
(d) a mixture of benzene and Mg(OMe)Br (a) n-Butyl chloride (b) iso-Butyl chloride
41. In alkaline hydrolysis of a tertiary alkyl halide by aqueous (c) sec-Butyl chloride (d) tert-Butyl chloride
alkali, if concentration of alkali is doubled, then the 49. When phenyl magnesium bromide reacts with tert -butanol,
reaction rate of constant temperature the product would be
(a) will be doubled
(a) benzene (b) phenol
(b) will be halved
(c) ter-butylbenzene (d) ter-butyl phenyl ether
(c) will become four times greater
50. Which Chloride is least reactive with the hydrolysis point
(d) will remain constant
42. Bromination of toluene gives of view
(a) only m-substituted product (a) CH3Cl (b) CH3CH2Cl
(b) only p-substituted product (c) (CH3)3CCl (d) CH2 = CH – Cl
(c) mixture of o-and p-substituted products 51. 2-Bromopentane is treated with alcoholic KOH solution.
(d) mixture of o-and m-substituted products The major product formed in this reaction and the type of
43. On sulphonation of C6H5Cl reaction respectively are
(a) m-chlorobenzenesulphonic acid is formed (a) pent-2-ene, b-elimination
(b) benzenesulphonic acid is formed (b) pent-1-ene, b-elimination
(c) o-chlorobenzenesulphonic acid is formed (c) 2-pentanol, nucleophilic substitution
(d) mixture of o- and p-Chlorobenzenesulphonic acid is (d) pent-1-ene, nucleophilic substitution
formed 52. Chloropicrin is obtained by the reaction of
44. An alkyl halide with molecular formula C 6 H13 Br on (a) steam on carbon tetrachloride
dehydrohalogenation gave two isomeric alkenes X and Y (b) nitric acid on chlorobenzene
with molecular formula C6H12. On reductive ozonolysis, (c) chlorine on picric acid
X and Y gave four compounds CH3COCH3 , CH3CHO,
(d) nitric acid on chloroform
CH3CH2CHO and (CH3)2CHCHO. The alkyl halide is
(a) 2-bromohexane
(b) 2, 2-dimethyl-1-bromobutane 53. Cl
(c) 4-bromo-2-methylpentane C–CCl3
(d) 3-bromo-2-methylpentane
45. Silver benzoate reacts with bromine to form Cl H
COOBr
The above structural formula refers to
(a) BHC (b) DNA
(a) (b)
(c) DDT (d) RNA
54. The pesticide DDT slowly changes to
COOAg (a) CCl3-CHO and chlorobenzene
(b) p, p'-Dichlorodiphenylethene
(c) p, p'-Dichlorodiphenyldichloroethane
(c) (d) C 6 H 5Br
Br (d) p, p'-Dichlorodiphenyldichloroethene
55. Which one of the following on hydrolysis produces a
46. What is the product of the following reaction ? ketone?
Cl (a) Isobutylidene chloride
(b) Secondarybutylidene chloride
(CH ) NLi (c) Benzylidene chloride
¾¾ ¾
3 ¾
2 ¾®
?
(CH 3 ) 2 NH (d) Ethylidene chloride
EBD_7327
588 CHEMISTRY

56. Pure chloroform is prepared by (c) RX + Mg ® RMgX


(a) distilling chloral hydrate with aqueous sodium (d) RX + KOH ® ROH + KX
hydroxide 67. Benzene reacts with CH3Cl in the presence of anhydrous
(b) heating ethanol with bleaching powder AlCl3 to form:
(c) heating acetone with bleaching powder (a) chlorobenzene (b) benzylchloride
(d) reducing carbon tetrachloride (c) xylene (d) toluene
57. The organic compound used as feedstock in the synthesis 68. Trichloroacetaldehyde, CCl 3 CHO reacts with
of chlorofluorocarbons is chlorobenzene in presence of sulphuric acid and produces
(a) CH2Cl2 (b) CHCl3 :
Cl
(c) CH3Cl (d) CCl4
58. If chloroform is left open in air in the presence of sunlight,
it gives
(a) carbon tetrachloride (b) carbonyl chloride
(c) mustard gas (d) lewisite
59. On warming with silver powder, chloroform is converted (a) Cl C Cl
into
(a) acetylene H
(b) hexachloroethane
(c) 1,1,2,2-tetrachloroethane OH
(d) ethylene
60. Cl2 reacts with CS2 in presence of I2 to form (b) Cl C Cl
(a) CHCl3 (b) CCl4
(c) C2H5Cl (d) Cl3C – NO2 Cl
61. Reaction of chloroform with KOH in presence of a primary
aromatic amine is called : (c) Cl CH Cl
(a) carbylamine reaction (b) reduction
(c) hydrolysis (d) Wurtz reaction CCl3
62. The product formed by heating iodoform with KOH is :
(a) HCHO (d) HCOOK Cl
(c) CH3COOK (d) CH3CHO
(d) Cl C Cl
63. Ethyl alcohol is used as a preservative for chloroform
because it :
CH2Cl
(a) prevents aerial oxidation of chloroform
(b) prevents decomposition of chloroform 69. Which one is most reactive towards SN 1 reaction ?
(c) decomposes phosgene to CO and Cl2 (a) C 6 H 5 CH(C 6 H 5 )Br
(d) removes phosgene by converting it to ethyl carbonate
64. When chlorobenzene is reacted with acetyl chloride in the (b) C6 H5 CH(CH3 )Br
presence of anhydrous AlCl3, the major product formed is (c) C6 H5C(CH3 )(C6 H5 )Br
(a) 2-chloroacetophenone (d) C6 H5CH 2 Br
(b) 3-chloroacetophenone 70. In the following sequence of reactions
(c) 4-chloroacetophenone
(d) 1, 4-dichlorobenzene KCN 3 ® H O+
CH3 - Br ¾¾¾® A ¾¾¾¾
65. In a SN2 substitution reaction of the type
LiAlH
4 ® C,
R - Br + Cl - ¾¾¾ ® R - Cl + Br -
DMF
B ¾¾¾¾ the end product (C) is :
ether
which one of the following has the highest relative rate ? (a) acetone (b) methane
(a) CH3 – CH2 – CH2Br (b) CH3 - CH - CH 2 Br (c) acetaldehyde (d) ethyl alcohol
| 71. Which of the following is the correct order of decreasing
CH3 SN2 reactivity?
(a) R2CHX > R3CX > RCH2X
CH3 (b) RCHX > R3CX > R2CHX
| (c) RCH2X > R2CHX > R3CX
(c) CH3 - C - CH 2 Br (d) CH3CH2Br (d) R3CX > R2CHX > RCH2X.
| (X is a halogen)
CH 72. Iodoform can be prepared from all except :
3
(a) Ethyl methyl ketone
66. Which of the following reactions is an example of (b) Isopropyl alcohol
nucleophilic substitution reaction?
(c) 3-Methyl 2-butanone
(a) 2 RX + 2 Na ® R – R + 2 NaX
(d) Isobutyl alcohol
(b) RX + H2 ® RH + HX
Haloalkanes and Haloarenes 589

73. Which one of the following is not an allylic halide? 75. Consider the following bromides :
(a) 4-Bromopent-2-ene Me Me
(b) 3-Bromo-2-methylbut-1-ene Me Br Me
(c) 1-Bromobut-2-ene Br Br
(d) 4-Bromobut-1-ene (A) (B)
74. The organic chloro compound, which shows complete The correct order of SN1 reactivity is
stereochemical inversion during a SN2 reaction, is (a) B > C > A (b) B > A > C
(a) (C2H5)2CHCl (b) (CH3)3CCl (c) C > B > A (d) A > B > C
(c) (CH3)2 CHCl (d) CH3Cl

1. Which chloro derivative of benzene among the following NaCN Ni / H


would undergo hydrolysis most readily with aqueous 5. CH 3 CH 2 Cl ¾¾ ¾
¾® X ¾¾ ¾
¾2®Y

sodium hydroxide to furnish the corresponding hydroxy Acetic


Y ¾¾ ¾
¾® Z
derivative? anhydride

Z in the above reaction sequence is


NO2
(a) CH3CH2CH2NHCOCH3
(a) O N
2 Cl (b) ON
2 Cl (b) CH3CH2CH2NH2
(c) CH3CH2CH2CONHCH3
NO (d) CH3CH2CH2CONHCOCH3
2
6. Bottles containing C6H5I and C6H5CH2I lost their original
labels. They were labelled A and B for testing. A and B were
(c) MeN
2
Cl (d) C6H5Cl. separately taken in test tubes and boiled with NaOH solution.
The end solution in each tube was made acidic with dilute
2. The alkyl halide that undergoes SN1 reaction more readily HNO3 and then some AgNO3 solution was added. Substance
is B gave a yellow precipitate. Which one of the following
(a) ethyl bromide (b) isopropyl bromide statements is true for this experiment ?
(c) vinyl bromide (d) n=propyl bromide (a) A was C6H5CH2I
(b) B was C6H5I
3. CH 3 - CH 2 - C H - CH 3 obtained by chlorination of
| (c) Addition of HNO3 was unnecessary
Cl
(d) A was C6H5I
n-butane, will be
7. Which of the following is the correct method of
(a) l-form (b) d-form preparation of methyl fluoride?
(c) meso form (d) racemic mixture (a) CH4 + HF ® (b) CH3OH + HF ®
4. Aryl halides do not undergo nucleophilic substitution (c) CH4 + F2 ® (d) CH3Br + AgF ®
reactions under ordinary conditions because 8. The structure of the major product formed in the following
1. approach of nucleophile is retarded reaction
2. carbon carrying halogen atom is sp3 hybridised CH2Cl
NaCN
3. the substrate molecule is destabilised due to resonance is
DMF
4. partial double bond character between carbon and
I
halogen
CH2Cl CH2CN
(a) 2 and 4 only (b) 1 and 4 only
(a) (b)
(c) 2 and 3 only (d) 2, 3 and 4 only
CN I
EBD_7327
590 CHEMISTRY

CH2CN CH2Cl HBr


KCN
(c) (d) 15. Isobutene ¾peroxide
¾ ¾¾® A ¾¾¾®
CN
CN dil. H SO
I B ¾¾¾2¾¾

C + inorganic salt D
9. Identify Z in the following series
C and D are
Alc. KOH Br2 KCN
C 2 H 5 I ¾¾ ¾ ¾
¾® X ¾¾
¾® Y ¾¾¾® Z (a) Me 2 CH.CH 2 COOH, ( NH 4 ) 2 SO 4
(a) CH3CH2CN (b) NCCH2–CH2CN (b) Me 2 CH.COOH, ( NH 4 ) 2 SO 4
(c) BrCH2–CH2CN (d) BrCH=CHCN
10. Which of the following pairs is/are correctly matched? (c) Me 2CH.CH 2 COOK, NH 4OH
Reaction Product (d) Me 2CH.CH 2 COOK, K 2SO 4
I. RX + AgCN RNC
II. RX + KCN RCN 16. How many isomeric naphthylamines are expected in the
following reaction ?
O
III. RX + KNO2 R–N Br
O NaNH , NH
¾¾¾¾
2 ¾¾

IV. RX + AgNO2 R–O–N = O – 33°C
(a) Only I (b) I and II
(a) two (b) only single product
(c) III and IV (d) I, II, III and IV
(c) four (d) three
11. Identify Z in
17. 2-phenyl-2-hexanol can be prepared by Grignard synthesis.
aq. NaOH
CH 3CH 2 CH 2 Br ¾¾¾¾¾
®X The pair of compounds giving the desired product is
Al O Cl / H O
¾¾2¾ ¾3 ® Y ¾¾2¾¾ 2¾® Z

(a) Mixture of CH3CHClCH2Cl and CH3CHOHCH2Cl C


(b) CH3CHOHCH2Cl (a) Br and Ph CH 3
(c) CH3CHClCH2OH
(d) CH3CHClCH2Cl
12. Which compound in each of the following pairs is most
reactive to the conditions indicated ? C
(b) Br and Ph
CH 2 Br CH 3
Br
(A) and (B) (KOH in CH3OH)
(c) Ph and PhBr
(d) None of these
CH 3 CH 3 18. Which will undergo SN2 reaction fastest among the following
Cl Cl halogen compounds?
(C) and (D) (NaNH2 in NH3) (a) CH3CH2F (b) CH3CH2Cl
CH 3 (c) CH3CH2Br (d) CH3CH2I
(a) A and C (b) B and C 19. In the following sequence of reactions
(c) A and D (d) B and D
P +I Mg HCHO
13. The correct kinetic rate equation for the addition- CH 3CH 2 OH ¾¾¾
2 ® A ¾¾¾® B ¾¾¾¾
®
ether
elimination mechanism of nucleophilic aromatic
H O
substitution C ¾¾¾
2 ®D
(a) rate = K [aryl halide] [nucleophile] the compound D is
(b) rate = K [aryl halide] (a) propanal (b) butanal
(c) rate = K [aryl halide] [nucleophile]2 (c) n-butyl alcohol (d) n-propyl alcohol.
(d) rate = K [ nucleophile] 20. X in the following reaction is –
14. Which of the following reagents react readily with CH3 - C - H CCl4
bromobenzene ? Br2 + || ¾¾¾ ®X
(a) NaNH 2 / NH 3 at - 33°C H - C - CH3

(b) (CH 3 )2 NH at. 25°C (a) (+) 2, 3-Dibromobutane


(b) (–) 2, 3-Dibromobutane
(c) CH 3CH 2ONa at. 25°C (c) Rac. 2, 3-Dibromobutane
(d) NaCN / DMSO at. 25°C (d) Meso-2, 3-Dibromobutane
Haloalkanes and Haloarenes 591

21. Consider the reactions :


CH2Br
(i) C H OH
(CH 3 ) 2 CH - CH 2 Br ¾¾2 ¾¾¾
5 ® CH2Br
(CH 3 )2 CH - CH 2OC 2H 5 + HBr (c) (d)
CH3
C H O-
(ii) (CH 3 ) 2 CH - CH 2 Br ¾¾2 ¾5 ¾¾
® CH3

(CH 3 )2 CH - CH 2OC 2 H 5 + Br - DIRECTIONS for Qs. 23 to 25 : These are Assertion-Reason


type questions. Each of these question contains two statements:
The mechanisms of reactions (i) and (ii) are respectively : Statement-1 (Assertion) and Statement-2 (Reason). Answer these
(a) SN1 and SN2 (b) SN1 and SN1 questions from the following four options.
(a) Statement-1 is True, Statement-2 is True, Statement-2 is a
(c) SN2 and SN2 (d) SN2 and SN1 correct explanation for Statement-1
22. Compound (A), C8H9Br, gives a white precipitate when (b) Statement-1 is True, Statement-2 is True ; Statement-2 is
warmed with alcoholic AgNO3. Oxidation of (A) gives an NOT a correct explanation for Statement-1
acid (B), C8H6O4. (B) easily forms anhydride on heating. (c) Statement-1 is True, Statement-2 is False
(d) Statement-1 is False, Statement-2 is True
Identify the compound (A).
23. Statement-1 : CHCl3 is stored in dark bottles.
Statement-2 : CHCl3 is oxidised in dark.
CH2Br C2H5 24. Statement-1 : Addition of bromine to trans-2-butene yields
(a) (b) meso-2, 3-dibromobutane
Br Statement-2 : Bromine addition to an alkene is an
electrophilic addition.
CH3
25. Statement-1 : CCl4 is not a fire extinguisher.
Statement-2 : CCl4 is insoluble in water.

Exemplar Questions CH3


1. The order of reactivity of following alcohols with halogen |
acids is .............. . (d) CH3CH2 – C – OH
|
(A) CH3CH 2 — CH 2 — OH CH3

(B) CH3CH 2 - CH - OH 3. Identify the compound Y in the following reaction.


| +
NH3 N 2 Cl
CH3 Na NO 2 + HCl
¾¾ ¾ ¾ ¾ ¾® Cu Cl
¾ ¾ 2¾ 2¾® Y + N 2
273- 278 K
CH3
| Cl
(C) CH3CH2 – CH – OH
| (a) (b)
CH3
(a) (A) > (B) > (C) (b) (C) > (B) > (A) Cl
(c) (B) > (A) > (C) (d) (A) > (C) > (B) Cl
2. Which of the following alcohols will yield the
corresponding alkyl chloride on reaction with concentrated (c) (d)
HCl at room temperature?
Cl
Cl
(a) CH3CH 2 — CH 2 — OH
4. Toluene reacts with a halogen in the presence of iron (III)
(b) CH3CH 2 - CH - OH chloride giving ortho and para halo compounds. The
| reaction is
CH3 (a) electrophilic elimination reaction
(b) electrophilic substitution reaction
(c) CH3CH2 – CH—CH2OH (c) free radical addition reaction
|
CH3 (d) nucleophilic substitution reaction
EBD_7327
592 CHEMISTRY

5. Which of the following is halogen exchange reactions? (a) (i), (ii), (iii) and (iv) (b) (i), (ii) and (iii)
(a) RX + NaI ® RI + NaX (c) (ii), (iii) and (iv) (d) (i), (iii) and (iv)
10. Which of the following structures is enantiomeric with the
(b) C C + HX C C
|| molecule (A) given below?

|
| |
H X H
CH3
(c) ZnCl C
R — OH + HX ¾¾¾¾
2®R — X + H O
2 Br
H5C2(A)
CH3 CH3 CH3
Fe H
(d) + X 2 ¾¾¾ ® H
dark C2H5 C
X (a) C (b)
Br
CH3 H3C Br C2H5
H Br
+ Br H
X (c) C (d) C
6. Which reagent will you use for the following reaction? H3C C2H5 CH3
H5C2
CH3CH2CH2CH3 ® CH3CH2CH2CH2Cl + CH3CH2CHClCH3 11. Which of the following is an example of vic-dihalide?
(a) Cl2 /UV light (a) Dichloromethane (b) 1, 2-dichloroethane
(b) NaCl + H2SO4 (c) Ethylidene chloride (d) Allyl chloride
(c) Cl2 gas in dark 12. The position of Br in the compound CH3CH = CHC(Br)
(d) Cl2 gas in the presence of iron in dark (CH3)2 can be classified as .............. .
7. Arrange the following compounds in the increasing order (a) allyl (b) aryl
of their densities. (c) vinyl (d) secondary
Cl 13. Chlorobenzene is formed by reaction of chlorine with
benzene in the presence of AlCl3. Which of the following
(i) (ii) species attacks the benzene ring in this reaction?
(a) Cl – (b) Cl+
(c) AlCl3 (d) [AlCl4]–
Cl Br 14. Ethylidene chloride is a/an ................ .
(a) vic-dihalide (b) gem-dihalide
(iii) (iv) (c) allylic halide (d) vinylic halide
15. What is 'A' in the following reaction?
Cl Cl
CH2 — CH — CH2
(a) (i) < (ii) < (iii) < (iv) (b) (i) < (iii) < (iv) < (ii)
(c) (iv) < (iii) < (ii) < (i) (d) (ii) < (iv) < (iii) < (i)
8. Arrange the following compounds in increasing order of + HCl A
their boiling points. (a) CH2 — CH — CH2
CH3
(i) CH — CH2Br (ii) CH3CH2CH2CH2Br
Cl
CH3

CH3 (b) CH2 — CH2 — CH2 — Cl


|
(iii) H3C — C| — CH3
Br
(c) CH2 — CH — CH3
(a) (ii) < (i) < (iii) (b) (i) < (ii) < (iii)
(c) (iii) < (i) < (ii) (d) (iii) < (ii) < (i) Cl
9. In which of the following molecules carbon atom marked
with asterisk ( * ) is asymmetric?
H D (d) Cl
CH — CH2 — CH3
(i) C* (ii) C*
I Cl I Cl
Br Br
H H
16. A primary alkyl halide would prefer to undergo .................
(iii) C* (iv) H C* (a) SN1 reaction (b) SN2 reaction
OH CH3 CH3
C2H 5 C2H 5 (c) a-elimination (d) racemisation
Haloalkanes and Haloarenes 593

17. Which of the following alkyl halides will undergo SN1 25. Which of the following compounds will give racemic mixture
reaction most readily? on nucleophilic substitution by OH – ion?
(a) (CH3 )3 C — F (b) (CH3 )3 C — Cl
Br
(c) (CH3 )3 C — Br (d) (CH3 )3 C — I |
18. Which is the correct IUPAC name for (i) CH3 — CH — Br (ii) CH3 — CH — CH3
| |
CH3 — CH — CH 2 — Br ? C2H5 C2H5
|
C2H5
(iii) CH 3 — CH — CH3Br
(a) 1-bromo-2-ethylpropane |
(b) 1-bromo-2-ethyl-2-mehylethane C2H5
(c) 1-bromo-2-methylbutane (a) (i) (b) (i), (ii) and (iii)
(d) 2-methyl-1-bromobutane (c) (ii) and (iii) (d) (i) and (iii)
19. What sh ould be th e correct IUPAC name for Direction (Q. No. 26 to 29) : In the questions 26 to 29 arrange
diethylbromomethane? the compounds in increasing order of rate of reaction towards
(a) 1-bromo-1, 1-diethylmethane nucleophilic substitution.
(b) 3-bromopentane
(c) 1-bromo-1-ethylpropane Cl Cl Cl
(d) 1-bromopentane NO2
20. The reaction of toluene with chlorine in the presence of 26. (i) (iii)
(ii)
iron and in the absence of light yields ............ . NO2
CH2Cl CH3 (a) (i) < (ii) < (iii) (b) (iii) < (ii) < (i)
Cl (c) (i) < (iii) < (ii) (d) (iii) < (i) < (ii)
(a) (b)
Cl Cl Cl
CH3
(c) H 3C Cl (d) Mixture of (b) and (c) 27. (i) (ii) (iii)
CH3
21. Chloromethane on treatment with excess of ammonia yields
mainly (a) (i) < (ii) < (iii) (b) (i) < (iii) < (ii)
(c) (iii) < (ii) < (i) (d) (ii) < (iii) < (i)
æ CH3ö
(a) N, N-dimethylmethanamine çCH3 —N ÷ Cl
è CH3ø Cl Cl
O2N NO2
(b) N - methylmethanamine (CH3 — NH — CH3)
(c) methanamine (CH3NH2) 28. (i) (ii) (iii)
(d) mixture containing all these in equal proportion
22. Molecules whose mirror image is non – superimposable NO2 NO2
over them are known as chiral. Which of the following
molecules is chiral in nature? (a) (iii) < (ii) < (i) (b) (ii) < (iii) < (i)
(a) 2 - bromobutane (b) 1 - bromobutane (c) (i) < (iii) < (ii) (d) (i) < (ii) < (iii)
(c) 2 - bromopropane (d) 2 -bromopropan - 2 - ol
Cl Cl Cl
23. Reaction of C6H5CH2Br with aqueous sodium hydroxide
follows .......... . CH3
(a) SN1 mechanism 29. (i) (ii) (iii)
(b) SN2 mechanism
(c) Any of the above two depending upon the temperature
CH3 CH3
of reaction
(d) Saytzeff rule
(a) (iii) < (ii) < (i) (b) (ii) < (i) < (iii)
24. Which of the carbon atoms presents in the molecule given
below are asymmetric? (c) (iii) < (ii) < (i) (d) (i) < (iii) < (ii)
30. Which is the correct increasing order of boiling points of
OH H O
HO | | | || the following compounds?
1 2 4
1 - iodobutane, 1 - bromobutane, 1 - chlorobutane, Butane
|| C — C — C — C |
3

| | H (a) Butane < 1-chlorobutane < 1- bromobutane <


O H OH
1 - iodobutabe
(a) 1, 2, 3, 4 (b) 2, 3
(b) 1 - iodobutane < 1 - bromobutane < 1 - chloroubutane
(c) 1, 4 (d) 1, 2, 3
< Butane
EBD_7327
594 CHEMISTRY

(c) Butane < 1 - iodobutane < 1 - bromobutane < CH3 CH3


1 - chlorobutane
(d) Butane < 1 - chlorobutane < 1 - iodobutane < (d)
1 - bromobutane and
CH3 Br CH3
31. Which is the correct increasing order of boiling points of
the following compounds? Br
1 - bromoethane, 1 - bromopropane,1 - bromobutane, 33. In which of the following compounds, the C - Cl bond
Bromobenzene ionisation shall give most stable carbonium ion? [2015]
(a) Bromobenzene < 1 - bromobutane < 1 - bromopropane
< 1- bromoethane H
(b) Bromobenzene < 1 - bromobutane < 1 - bromopropane CH — Cl
< 1- bromobutane H3C
(c) 1 - bromopropane < 1 - bromorpropane < 1 - bromoethane C — Cl
(a) H3C (b)
< Bromobenzene CH3
(d) 1 - bromoethane < 1 - bromopropane < 1 - bromobutane
< Bromobenzene H H 3C H
(c) C — Cl (d) C — Cl
NEET/AIPMT (2013-2017) Questions O 2NH2C H H 3C

32. What products are formed when the following compounds 34. In an SN1 reaction on chiral centres there is : [2015 RS]
is treated with Br2 in the presence of FeBr3? [2014] (a) 100 % racemization
(b) inversion more than retention leading to partial
CH3 racemization
(c) 100 % retention
(d) 100 % inversion
CH3 35. For the following reactions : [2016]
(1) CH3CH2CH2Br + KOH ®
CH3 CH3 CH3CH=CH2+KBr + H2O
Br (2) H3C CH3 H3C CH3
(a) and + KOH + KBr
CH3 CH3 Br OH
Br Br
CH3 CH3 + Br2
(3)
Br Br Br
(b) and Which of the following statements is correct ?
CH3 CH3 (a) (1) and (2) are elimination reaction and (3) is addition
reaction
CH3 CH3
(b) (1) is elimination, (2) is substitution and (3) is addition
Br reaction
(c) and (c) (1) is elimination, (2) and (3) are substitution reactions
CH3 CH3 (d) (1) is substitution, (2) and (3) are addition reaction
Br
Haloalkanes and Haloarenes 595

Hints & Solutions


6. (a)
EXERCISE - 1 7. (d) Among the given structures, CH2 = CHCl is least
1. (b) In basic conditions, the reactivity by elimination increases reactive.
from 1° carbocation to 3° carbocation. So, (CH3)3CCl As reaction of substitution involves breakage of
suffers elimination the most among the given choices. carbon-halogen bond. Here the carbon is vinylic
Note: Elimination reactions are of two types E1 and carbon hence is sp2 hybridised. The bond length is
E2. Reactivity by E1 and E2 mechanism increases in shorter than single bond and has a very high bond
the same order but due to different reasons. The rate energy. This is because of resonance:
of E2 depends upon the concentration of base and rate H H H H
of E1 depends upon the nature of base. ¬¾®
H – C = C – Cl H – C – C = Cl
Here we can not decide whether the reaction is – +
proceed via E1 mechanism or E2 mechanism because Also vinylic carbocation is very less stable.
nothing is given about the basic conditions. Thus option (d) is correct choice.
2. (b) C4H7 Cl is a monochloro derivative of C4H8 which In structure CH2 = CHCH2Cl, allylic carbon is present.
itself exists in three acyclic isomeric forms. Allylic carbocation is infact stabilized by resonance.
CH3 1°
CH3 1°
CH3
| 1° 2° 1°
CH3CH 2CH = CH 2 CH3CH = CHCH3 CH3 - C = CH 2 8. (b) CH3 – CH – C – CH2 – CH3
I II III 3° 4°
(Its four C 's are (It has 2 types (It has 2 types Cl
different) of carbon) of carbon) 3-Chloro-2,3-dimethylpentane
Four monochloro derivatives of I 9. (a)
Cl Cl - - -
| | 10. (b) Halides Cl , Br , I React with AgNO3 to give
*
CH 2 CH 2 CH = CH 2 CH3 CHCH = CH 2 AgCl ¾¾ ® soluble in NH4OH
(optically active) AgBr ¾¾ ® spraringly soluble in NH4OH
d and l isomers
AgI ¾¾ ® Insoluble
Cl and the C – Cl bond is weakest in benzyl chloride [(sp3)
|
CH3CH 2 C = CH 2 CH 3 CH 2 CH = CHCl hybridised carbon is attached to Cl]
cis and trans isomers 11. (a) Peroxide effect is observed only in case of HBr.
Hence total isomers from I = 6 Therefore, addition of HCl to propene even in the
Two monochloro derivatives of II presence of benzoyl peroxide occurs according to
Cl Markonikov’s rule :
Cl
| | HCl
CH 2 CH = CHCH3 CH 3 C = CHCH3 CH3 - CH = CH 2 ¾¾¾¾¾¾¾
®
Benzoyl peroxide
cis and trans isomers cis and trans isomers
Hence total isomers from II = 4 CH 3 - CHCl - CH 3
Two monochloro derivatives from III
H Cl Cl
CH 2Cl CH3 Cl H |
| |
CH3 - C = CH 2 CH3 - C = CHCl H Cl
12. (c) + KOH ¾¾¾¾
-3HCl
®
Grand total of acyclic isomers = 6 + 4 + 2 = 12 Cl H
|

3. (a) Gem dihalides on treatment with alcoholic KOH gives Cl Cl


H
alkyne as follows : Cl H Cl
alc.KOH 13. (c) Potassium ethoxide is a strong base, and 2-
R , CH 2 CX 2 ¾¾ ¾¾↑ RC º CH bromopentane is a 2º bromide, so elimination reaction
,2HX
| predominates
H
OC H -
CH 3 CH (Br )CH 2 CH 2 CH 3 ¾¾ ¾¾®
2 5
4. (b) CHCl 2 CH 2 Cl
| | CH 3 CH = CHCH 2 CH 3 +CH 2 = CHCH 2 CH 2 CH 3
CH3 CH 2 Cl Pentene - 2(major) trans Pentene -1(min or ) cis

(gem-dihalide) (vic-dihalide) Since trans- alkene is more stable than cis.thus


trans-pentene -2 is the main product.
Cl NaNH
14. (b) CH 3 — CH 2 — CHCl 2 ¾¾ ¾¾®
2
5. (a) CCl3CH D

® CH 3 — C º CH
Cl NaNH2
CH 3 — CH = CHCl ¾¾ ¾¾
D Final Product
(DDT)
EBD_7327
596 CHEMISTRY

15. (d) Alcoholic KOH, reduces haloalkane into alkene by the 27. (c) The best method for the conversion of an alcohol into
process dehydrohalogenation an alkyl chloride is reaction of the alcohol with thionyl
CH 3 - CHBr - CH 2 - CH 3 + KOH (Alc) chloride (SOCl2) in the presence of pyridine.
Pyridine
¾¾¾® CH 3 - CH = CH - CH 3 R – OH + SOCl2 ¾¾¾¾¾ ® RCl + SO2­ + HCl­
- HBr SO2 and HCl being gases escape leaving behind pure
Br alkyl halide.
|
Alc. KOH
16. (c) CH3 - CH - CH 2 - CH3 ¾¾¾¾¾ ® 28. (a) (CH3 )3 C - MgCl + D 2 O ¾¾
®
CH3 - CH = CH - CH3 + HBr (CH3 )3 C - D + Mg(OD)Cl
Alc NaOH
The formation of 2-butene is in accordance to 29. (b) CH3–CH2–CH2Br 80° C
CH3–CH=CH2
Saytzeff’s rule. The more substituted alkene is formed (X) acetic acid
HBr, 20° C
in major quantity. (Y)
17. (a)
CH3–CH–CH3
18. (b) 3° halide on reaction with strong base (CH3OQ )
undergo elimination reaction and forms alkene as major
Br
product.
ethanolic KOH
CH3 CH 2 30. (b) CH3 - CH 2 - CH 2 Br ¾¾¾¾¾¾
® CH3 - CH = CH 2
| n - Propyl bromide - HBr Propene
Q ||
CH3 - C - Br + HBr
® CH3 - C
CH3O 31. (a) 1, 1-dichloroethane on heating with dil. NaOH gives
¾¾¾¾
| Strong base | acetaldehyde.
CH3 CH3 Cl OH
3° Halide | |
dil NaOH
alc. CH3 — C — H ¾¾¾¾¾® CH3 — C —H
19. (b) CH3 – CH2 – CH2 – Cl ¾¾¾
® CH3CH = CH 2 | |
KOH (B) Cl OH
¾® CH3CHO + H2O
Br CH3 CH3 32. (b) Addition of HBr to 2-pentyne gives two structural
| Na
HBr
¾¾¾ ® CH3 - CH - CH 3 ¾¾® | | isomers (I) and (II)
(C) ether CH 3 CH - CHCH 3 HBr
(D) CH3 - C º C - CH 2 CH 3 ¾¾¾
®
20. (a) CHCl 3 + 3HF ¾ ¾® CHF3 + 3HCl CH3C(Br) = CHCH 2 CH 3 + CH3CH = C(Br)CH 2CH3
wt. of CHF3 = (12 + 1 + 19 × 3) = 70 (I) (II)
Each one of these will exist as a pair of geometrical
21. (b) CHCl 2 CH 2 Cl isomers. Thus, there are two structural and four
| |
CH3 CH 2 Cl configurational isomers.
CH3 CH2Cl CH2OH
gem-dihalide vic–dihalide
22. (a) Friedel Craft reactions are examples of aromatic 33. (d) Cl2, h u aq. KOH
electrophilic substitution. In this, a Lewis acid (like
AlCl3 , FeBr3 etc.) is used as catalyst. Toluene Benzyl chloride Benzyl alcohol

C2 H5OH
AlCl
C 6 H 6 + CH 3Cl ¾¾¾3 ® C 6 H 5CH 3 + HCl 34. (b) (CH3)2 CHCH2MgBr ¾¾ ¾¾® (CH3)2 CHCH3
OC2H5
200°C
23. (a) C6 H5Cl+NH3 +Cu 2O ¾¾¾¾
pressure
®
+ Mg
C 6 H 5 NH 2 + 2CuCl + H 2O Br
AgCN Reduction 35. (d) Debromination is a trans-elimination reaction.
24. (d) C2H5Br ¾¾¾¾ ® C2H5NC ¾¾¾¾¾ ® meso- 2, 3-Dibromobutane on debromination gives
Ethyl bromide Ethyl isocyanide trans-2-butene.
C2H5NHCH3 36. (a) 3-methlhexane can yield seven different
Ethyl methyl amine monochlorinated products upon free radical
25. (b) Heavy metal ions, particularly Ag+, catalyse S N1 chlorination.
7
reaction because of presence of empty orbital. CH3
+ 6 5 4 3| 2 1
CH3CH 2Br + Ag H3C–H2C–H2C–CH–CH2–CH3
+
[CH3CH2 .... Br .... Ag] CH 3
– |
Anhyd .
slow + 37. (c) C 6 H 6 + CH 3CH 2 CH 2 CI ¾¾ ¾
¾® C6 H 5 - CH - CH 3
CH3CH 2 OH CH 3CH 2OH AlCl 3
(–AgBr) from H 2O
Isopropyl benzene
26. UV light
(a) C2H6 + Cl2 ¾¾¾¾ 38. (b) CH3Br + AgF—® CH3F + AgBr
® C2 H 5 Cl+HCl
Haloalkanes and Haloarenes 597

Mg 48. (d) The boiling points of alkyl chlorides increases with


39. (b) C6 H 5Br ¾¾¾ ® C6 H 5 MgBr increase in molecular weight. In case of isomeric alkyl
CH CH OH chloride, the order of boiling point is
¾¾¾¾¾¾
3 2 ® C6 H 6 + CH 3CH 2OMgBr primary > secondary > tertiary
40. (d) CH 3 OH + C 6 H 5 MgBr ¾ ¾® CH 3 O.MgBr + C 6 H 6 49. (a) Grignard reagents react with compounds containing
41. (d) SN1 (Unimolecular nucleophilic substitution) : a cti ve h ydr ogen t o for m h ydroca r bon s
Although it is a two step process, the rate of reaction corresponding to alkyl (or aryl) part of the Grignard
depends only upon the first (slow) step which involves reagent.
ionization of the alkyl halide to form carbocation. Hence C6H5MgBr + Me3COH ¾¾® C6H6 + Me3COMgBr
rate of reaction depends only upon the concentration of 50. (d) CH 2 < CH , Cl
the alkyl halides, r = k[RX] and is independent of the (Vinyl Chloride)
concentration of the nucleophile which adds on the The halogen atom in vinyl chloride is not reactive as
carbocation in the second (fast) step. in other alkyl halides. The non-reactivity of chlorine
atom is due to resonance stabilisation. The l.P. on Cl-
-X- :Z atom can participate in delocalisation (Resonance) to
R - X ¾¾® R + ¾® R - Z
(slow) (fast) give two canonical structure.
42. (c) –CH3 group is o, p–directing. Π ∗
43. (d) –Cl is o, p–directing. CH 2 < CH , Cl ¾¾
↑ CH 2 , CH < C l
44. (d) CH3 — CH — CH — CH2 — CH3 alc. KOH
½ ½ 51. (a) CH3 CH2 CH CH CH3 –HBr
CH3 Br
3-bromo-2-methyl Pentane H Br
2-bromopentane
dehydrohalogenation
CH 3 — CH 2 — CH == CH — CH 3
H3CC CH — CH2CH3 + H3C— CH — CH = CHCH3 2-pentene
½ ½
CH3 CH3 These reactions are known as b-elimination.
52. (d) Chloropicrin is nitrochloroform. It is obtained by the
(X) (Y)
nitration of chloroform with HNO3.
ozonolysis ozonolysis
(conc.) HNO
HCCl3 ¾¾¾¾¾¾
3
® O2 NCCl3
H3C
C O (CH3)2CHCHO Chloroform Chloropicrin
H3C 53. (c) 54. (d)
+
+ CH3CHO CH3 CH3
2° OH
CH3CH2 — CHO 55. (b) [OH ]

CCl2 C
Hunsdiecke r reaction CH3CH2 CH3CH2 OH
45. (d) C 6 H 5 COOAg + Br2 ¾¾ ¾¾ ¾¾ ¾¾®
2° Butylidene Chloride unstable
C 6 H 5 Br + CO 2 + AgBr
CH3
Cl N(CH 3 ) 2
[– H 2O]
C=O
CH3CH2
46. (a) + (CH3 ) 2 NLi ¾(¾
CH3 ) NH
¾¾ ¾® + LiCl
Butanone
distil
47. (c) 56. (a) CCl3·CH(OH)2 + NaOH ¾¾¾® CHCl3 + HCOONa + H2O
CH3 – CH2 – Mg – Br + CH3 – CH2 SbCl5
57. (d) 3CCl4 + 2 SbF5 ¾¾¾¾ ® 2SbCl3 + 3CCl 2 F2
C=O freon -12
CH3 – CH 2 C,FeCl3
CCl 4 + 2H ¾¾¾¾¾
® 2HCl + CCl 2 F2
1 Light
CH2 – CH 3 58. (b) CHCl3 + O 2 ¾¾¾ ® CCl(OH)3 ® COCl 2 + HCl
CH3 – CH 2 2 unstable

C – O Mg Br D
59. (a) HCCl3 + 6Ag + HCCl3 ¾¾® HC º CH + 6AgCl
CH3 – CH2 anhy. AlCl3
OH
60. (b) CS2 + 3Cl2 ¾¾¾¾¾ ® CCl 4 + S2 Cl2
or SbCl5
HOH
–Mg 2S2 Cl2 + CS2 ¾¾ ® CCl4 + 6S
Br 61. (a) This is carbylamine reaction
CH2– CH 3 ArNH 2 + CHCl3 + 3KOH
CH3 – C H2
1o aromatic amine
C – OH
CH3 – CH 2 ®
® Ar - N = C+ 3KCl + 3H 2 O
¾¾
3-ethylpentan-3-ol isocyanide
EBD_7327
598 CHEMISTRY

62. (b) CHI3 + 3KOH ¾¾® CH(OH)3 + 3KI Cl


iodoform ¯ 68. (c) Cl 3C–C=O + conc. H2 SO4
KOH
HCOOK ¬¾¾¾ HCOOH + H 2 O
H Cl
63. (d) CHCl3 on exposure to air forms phosgene which is
poisonous gas and removed by converting it into diethyl
carbonate (which is non-poisonous substance). Cl
Cl C–C=O conc. H SO
Cl2C – CH
O 2 / light
CHCl3 ¾¾¾¾ ¾ ® COCl2 + HCl H Cl
Cl
Phosgene (Poisonous)
DDT
COCl2 + 2C2H5OH ¾¾
® (C2H5)2CO3 + 2HCl 69. (c) SN1 reactions involve the formation of carbocations,
Diethyl carbonate (non-poisonous) order of stability of carbocation is 3° > 2° > 1° hence
higher the stability of carbocation, more will be the
Cl Cl reactivity of the parent alkyl halide. Moreover the
tertiary carbocation formed from (c) is stabilized by
COCH3 two phenyl groups.
Anhy AlCl
64. (c) + ClCOCH3 ¾ ¾ ¾ ¾ ¾3 ¾
® +
KCN H 3O
70. (d) CH3 - Br ¾¾¾ ® CH3 - CN ¾¾¾¾ ®
2-Chloro acetophenone (A)
(minor) LiAlH
4 ® CH - CH - OH
CH3 - COOH ¾¾¾¾ 3 2
Cl (B) (C)
Ethyl alcohol
71. (c) In SN2 mechanism transition state is pentavalent. Thus
+ + HCl bulky alkyl group will be sterically hindered and smaller
alkyl group will favour the SN2 mechanism. So the
COCH3 decreasing order of reactivity of alkyl halides is
RCH2X > R2CHX > R3CX
4-Chloro acetophenone (major) 72. (d) Iodoform test is given by methyl ketones,
acetaldehyde and methyl secondary alcohols.
[Note : Para product predominates over the ortho
CH3 CH CH2 OH
product]
65. (d) For such a reaction the rate of SN2 substitution reaction CH3
is maximum in case of CH3 CH2 Br because SN2 Isobutyl alcohol is a primary alcohol hence does'nt
mechanism is followed in case of primary and secondary give positive iodoform test.
halides i.e., SN2 reaction is favoured by small groups 73. (d) 4-Bromobut-l-ene is not an allylic halide
on the carbon atom attached to halogens so order of BrH 2C —CH 2 —CH== CH 2
4 - Bromobut -1-ene
SN2 substitution reaction will be 74. (d) SN2 reaction is favoured by small groups on the carbon
CH3 CH2 Br > CH3 CH2 CH2 Br > atom attached to halogen.
So, the order of reactivity is
CH3 CH 3Cl > (CH 3 ) 2 CHCl > (CH 3 ) 3 CCl
| > (C 2 H 5 ) 2 CHCl
CH3– CH – Br > CH3 – C – Br
| | SN2 reaction is shown to maximum extent by primary
CH3 CH3 halides. The only primary halides given is CH3Cl so
the correct answer is (d).
i.e. option (d) is correct.
75. (a) + –
Me Br ionisation +Br
66. (d) It is a nucleophilic substitution reaction as here stronger Me
nucleophile OH– is replacing weaker nucleophile X– (A)

Me Me
CH3 ionisation Me
+ + Br –
Br
Anhyd. (B)
67. (d) + CH3Cl +HCl3
AlCl3
Me
Toluene ionisation Me –
Me + Br
Friedel Craft’s reaction. Me +
Br
(C)
Haloalkanes and Haloarenes 599

Since SN1 reactions involve the formation of carbocation 7. (d) Fluoroalkanes are difficult to prepare directly because
as intermediate in the rate determining step, more is the flourination of hydrocarbons with pure F2 gas occurs
stability of carbocation higher will be the reactivity of explosively. Therefore these are prepared by treating
alkyl halides towards SN1 route. Now we know that alkyl chloride or bromide with salts such as Hg 2F2, AgF.
stability of carbocations follows the order : 3° > 2° > 1°, The reaction is called swarts reaction.
so SN1 reactivity should also follow the same order. CH3Br + AgF ® CH3F + AgBr
3° > 2° > 1° > Methyl (SN1 reactivity) CH2Cl CH2CN
NaCN
8. (b) ¾¾ ¾
¾®
EXERCISE - 2 DM F

1. (a) Cl in 2, 4, 6-trinitrochlorobenzene is activated by three I I


NO2 groups at o, and p-positions and hence undergoes Nuclear substitution will not take place.
hydrolysis most readily.
2. (d) This reaction occurs in two steps. In first step, a 9. (b) C 2 H 5 I ¾alc. KOH Br2
¾ ¾ ¾® CH 2 = CH 2 ¾¾¾®
carbocation is formed from alkyl halide molecule. First KCN
step is slow step so it is also rate determining step. In BrCH 2 - CH 2 Br ¾¾¾® CNCH 2 .CH 2 CN
second step, an attacking nucleophile attacks on this 10. (b)
carbocation and forms the final product. 11. (b)
Thus the stability of the carbocation influences the rate aq. KOH Al O
CH 3 CH 2 CH 2 Br ¾¾ ¾¾® CH 3CH 2 CH 2 OH ¾¾2¾
¾3®
of reaction. More stable the carbocation, higher is its heat
rate of formation. Thus those alkyl halides which form
stable, 3º carbocations undergo SN1 reaction readily. Cl / H O
Thus, t-butyl bromide is the favourable substrate. CH 3 CH = CH 2 ¾¾2¾ ¾ 2¾® CH .CHOH .CH Cl
3 2
CH3 CH3 12. (a) In the first case the reaction gives side chain
| | – substitution product which is easier in A. In the second
Slow step
(i) CH3 - C - X ¾¾¾¾® CH3 - CÅ + X case the reaction will proceed by benzyne mechanism
| |
CH3 CH3 for which ortho position w. r. to Cl must have H-atoms.
Ter. butyl carbocation Cl Nu Cl
Nu Cl
CH3
| CH3 Nu:
| 13. (a) ¾ ¾¾®
Å Fast
(ii) CH3 - C + O H ¾¾¾¾ step Slow
® CH3 - C - OH
| |
CH3 CH3
Nu
Rate of reaction µ [(CH3)3C – X] Nu Cl
Cl / hn Fast
3. (d) CH3 — CH2 — CH2 — CH3¾ ¾ ¾¾®
2
¾¾
¾® + Cl

CH3 CH3 Rate µ [C 6 H 5Cl] [ Nu :] =


H — C — Cl + Cl — C — H K [Aryl halide] [nucleophile]

C2H5 C2H5 Br NaNH / NH


d l 14. (a) ¾¾¾ ¾
2 ¾¾3®
+ HBr
Racemic mixture H
50% d form + 50% l form HBr
H 3C
Cl• may attack on either side and give a racemic mixture 15. (a) H 3C C= CH 2 ¾¾ ¾¾®
peroxide
CH– CH 2 Br
H 3C H 3C
of 2-chloro butane which contain 50% d form and 50%
l-form. KCN H 3C
4. (b) Aryl halides are less reactive towards nucleophilic ¾¾¾® CH – CH 2 – C º N
H3 C
substitution because of the partial double bond character
of carbon-halogen bonds. It is also partly due to H 3C
dil. H SO
repulsion between the electron cloud of the benzene ring ¾¾¾¾¾
2 4® CH– CH 2 – COOH
and the nucleophile. H3 C

5. NaCN
(a) CH3CH 2 Cl ¾¾¾¾ ® + ( NH 4 ) 2 SO 4 .
Br
Ni / H
CH3CH 2 CN + NaCl ¾¾¾¾

16. (a) NaNH / NH
¾¾ ¾ ¾
2 ¾¾3®
(X) – 33°C
CH 3CH 2 CH 2 NH 2 ¾¾ ¾ ¾ ¾
¾®
( CH 3CO ) 2 O
(Y ) NH 2
CH 3CH 2 CH 2 NHCOCH3 ∗ CH 3COOH NH 2
NH
(Z) ¾¾¾
2® +
6. (d)
EBD_7327
600 CHEMISTRY

O 23. (c) CHCl3 is stored in dark bottles to prevent oxidation of


CHCl3 in presence of sunlight.
17. (a) C + Mg Br ¾
¾® 24. (b)
25. (d) CCl4 is used as a fire extinguisher. The dense, non
Ph CH 3 combustible vapours cover the burning substance and
OH prevents the availability of oxygen around burning
O MgBr material.
1
2
Ph C CH 3 EXERCISE - 3

|
|
H2 O
C ¾¾¾
® 4
CH 3 6
Ph 3 Exemplar Questions
5
2 – phenyl -2-hexanol 1. (b) Alchols and h alogen acid react thr ough S N 1
18. (d) Smaller the R group reactivity will be higher towards mechanism.
SN2 reaction. For alkyl halides containing similar alkyl +
group better will be the leaving group, more facile is CH3 —CH 2 —CH 2 —OH ® CH3 —CH2 —CH+ OH -
the nucleophilic substitution reaction.
Amongst the halide ions, the order in which the leaving In this case, 1° carbocation is formed. It is least stable.
groups depart follows the sequence : So, here SN2 mechanism is followed. In this SN2
I– > Br– > Cl– > F– mechanism a transition state is observed in which
It is because of this reason that the order of reactivity a - carbon is linked with two nucleophilies.
of haloalkanes follows the sequence : + –
iodoalkanes > bromoalkanes CH3— CH2 — CH — OH ® CH3— CH2 — CH — OH
> chloroalkanes > fluoroalkanes | |
CH3 CH3
P + I2
19. (d) CH3CH 2 OH ¾¾¾ ® CH3CH 2 I 2º Carbocation (more stable than 1º carbocation)
A
CH3 CH3
Mg HCHO | |+
¾¾¾® CH3CH 2 MgI ¾¾¾¾
® –
Ether
(B)
CH3— CH2 — C — OH ® H3C — CH2 — C + OH
CH 2 CH3 | |
CH 2 CH 3
| | CH3 CH3
H2 O
H - C - OMgI ¾¾¾ ® H - C - OH 3° carbocation (most stable)
| |
H Greater the stability of carbocation, greater will be the
H
(D) possibilities of attack of X– ion on the carbocation.
n - propyl alcohol Order of stability of carbocation is : 3° > 2° > 1.
So order of reactivity will be C > B > A.
CH3 H CH3 2. (d) Reaction of alcohols with conc. HCl at room
C Br temperature follows SN1 mechanism.
20. (d) H
|| + Br2 The attack of nucleophile to the carbocation is possible
C H Br
(trans reagent) only on stable carbocation as at room temperature only
H CH3 CH3 3° carbocation is stable.
(Meso) Step I
(trans-2-butene)
21. (a) A strong nucleophile favours the SN2 reaction and a CH3 CH3
weak nucleophile favours the SN1 reaction. | | –

First reaction is SN1 reaction because C2H5OH is used CH3 CH 2 C OH ® CH3 CH2 CÅ + OH
| |
as solvent which is a weak nucleophile. CH3 CH3
Second reaction is SN2 reaction because C2H5O– is
strong nucleophile. Step II
CH2Br CH3 CH3
22. (d) Alcholic |r s |
AgNO3
AgBr¯ CH3 — CH2 — C + Cl ® H3C — CH2 — C—Cl
A CH3 | |
CH3 CH3
Oxidation
3. (a) Sand meyer’s Reaction
COOH CO +
D NH3 N2 Cl Cl
O Na NO + HCl
¾ ¾ ¾2 ¾ ¾® Cu 2Cl2
¾ ¾ ¾® +N2
COOH CO 273- 278 K

Acid (B) Phthalic Anhydride (Y)


Haloalkanes and Haloarenes 601

4. (b) benzene < chlorobenzene < dichlorobenzene <


FeCl3 r s r s bromochlorbenzene
Cl – Cl Cl – Cl – FeCl3 Cl + FeCl4 8. (c) Greater the surface area, greater will be the boiling point
CH3 of a compound. Surface area decreases with increases
CH3 CH3 in branching.
r s H
+ Cl – Cl – FeCl3 rds Increasing order of boiling point
Cl or Å
r CH3
H Cl H3C
H3C C CH3 < CH — CH2Br
H3C
Br
CH3 CH3 < CH3CH 2 CH 2CH 2 Br
H
+ Cl or + 9. (b) Carbon atom in which all four valencies are different is
known as Asymmetrical/chiral.
H Cl 10. (a) Enantiomers are the steroismers which are related to
each other as non – superimposable mirror images.
FeCls
4 FeCls
4
The enantiomer of molecule (A) is
Mirror
CH3 CH3
Cl H H
CH3 H3C
C C
H5C2 Br Br C2H5
Cl
As electrophile Cl+ attacks on electron rich benzene
ring and substitutes hydrogen. So, the reaction is 11. (b) Halides in which two halogen atoms are present on the
electrophilic substitution reaction. two adjacent carbon atoms are known as vic - dihalides.
5. (a) Halogen exchange reactions are those reactions in H
which one hailde replaces another (Finkelstein reaction). | CH 2 — CH 2
6. (a) It is a substitution reaction which involves the Cl — C — Cl | |
replacement of 1° and 2° hydrogen of alkanes by | Cl Cl
H
chlorine. It occurs in presence of ultraviolet light. 1, 2 - dichloroethane
Dichloromethane
·
UV
Step 1 : Cl — Cl ¾ ¾ ¾® 2C l H Cl
light | | Cl
· H — C— C — H |
C l + CH 3 — CH 2 — CH 2 — CH 3 ® CH 3CH 2 CH 2 | | C H 2 — CH = CH 3
· H Cl
— C H 2 + HCl Ethylidene chloride
Allyl chloride
Step 2 :
·
CH3 — CH 2 — CH 2 — C H 2 + Cl 2 ® 12. (a) Compounds in which the halogen atom is bonded to
· sp3 hybridised carbon atom next to carbon carbon-
CH3 — CH 2 — CH 2 — CH 2 Cl + C l double bond are known as allyl halides.
· CH3 CH = CHC ( Br ) ( CH3 ) 2
CH3 – CH2 – C H – CH3 + Cl2 ®
-
Cl 13. (b) ® [ AlCl 4 ] + Cl+
AlCl3 + Cl2 ¾¾
| ·
CH3 – CH2 – CH – CH3 – C l Cl
+
Step 3 : + Cl
· ·
CH3 — CH 2 — CH 2 — C H 2 + C l ® Chlorobenzene

CH3CH 2 CH 2 CH 2 Cl 14. (b) If halogen atoms present on the same carbon atom
then they are known as gem – dihalides or alkylidene
· ·
CH3 - CH 2 - C H - CH3 + Cl ® halides.
15. (c) Addition of HCl takes place in accordance with
Cl
| Markownikoff's rule.
CH3CH2 CH – CH3 CH2 — CH — CH2 CH2 — CH — CH3
7. (a) Density is directly related to molecular mass. More the
molecular mass, more will be the density of the Cl
compound. The order of molecular mass is + HCl
EBD_7327
602 CHEMISTRY

16. (b) It would prefer to undergo SN2 reaction. 25. (a) All those compounds which follow SN1 mechanism
SN2 reactions occur if there is less steric crowding during nucleophilic subsitution reaction will give
on a - carbon of alkyl halide. In case of primary aklyl racemic mixture.
halides, steric crowding is very less. So, it would prefer Order of reactivity of alkyl halides for SN1.
to undergo SN2 reaction. 3° > 2° > 1° CH3 X
17. (d) All compounds are tertiary alkyl but bond formed
Thus, CH3 — CH — Br contains a 2° carbon so
between carbon and iodine (C — I) is weakest bond |
due to higher difference in size of carbon and iodine. C 2 H5
18. (c) IUPAC name
gives a racemic product.
2 1
CH3 — CH — CH2 — Br 26. (c) The substitution is faster if the electron withdrawing
3
| group is at ortho and para position because electron
CH2 density is high at these positions as chlorine is
| electron donating group which increase electron
CH3
4 density at ortho and para position. Therefore, it has
partial double bond character, and is not easy to break.
1-bromo-2 methylbutane
In compound (ii) and (iii) both has one electron
Br withdrawing group but in compound (ii) electron
| withdrawing (— NO2 ) group is present at ortho
19. (b) H3 C— H 2 C— H C — C H 2 — C H3
1 2 3 4 5 position, so rate of reaction in compound (ii) is more
than that of (iii) while (i) has no electron withdrawing
IUPAC name is 3-bromopentane.
group.
20. (d) Toluene react with chlorine in presence of iron and
27. (d) If electron releasing group is present at ortho or para
in absence of light, by substitution on benzene ring.
position it decreases the rate of nucleophilic
CH3 CH3 CH3 substitution reaction. In compound (iii) electron
releasing group is present at meta position w.r.t.
Cl
Fe chlorine, so the impact is nothing but in compound
+ Cl2 ¾ ¾® + (ii) it is present at ortho position.
Toluene o-chlorotoluene 28. (d) If electron withdrawing group is present at ortho and
Cl para position then the nucleophilic substitution
p-chlorotoluene reaction rate increases.
29. (c) If electron releasing group is present at ortho and para
21. (c) CH3Cl + NH3 ® CH3 NH 2 + HCl position w.r.t. to chlorine it decreases the rate of
Excess Methanamine
nucleophilic substitution reaction.
22. (a) Carbon in which four bonds are different is known as 30. (a) Greater the surface area, greater will be the
Chiral carbon. intermolecular forces of attraction and
1 2 3 4 intermolecular forces of attraction is directly
H3C — CH — CH2— CH3
| proportional to boiling point. Surface area is larger for
Br larger size of halogen.
(2- bromobutane) 31. (d) Boiling point is directly proportional to size of the
Å molecule. All contains same halogen atom but different
23. (a) In C6H5CH2Br carbocation is C6 H5 C H 2 which is hydrocarbon part. Larger the different hydrocarbon part
stable due to resonance. larger the boiling point.

CH3 NEET/AIPMT (2013-2017) Questions


Å
CH 2 Å CH 2 CH2 CH2 32. (c) Methyl group is ortho para directing but due to steric
hind rance effect, generated by two CH3 groups
Å Å substitution will not take place on position (I). Hence
only two products are possible.
CH2
CH3
+
III I

24. (b) If carbon atom has all four valencies with four CH3
II
different groups then it is called as asymmetric/chiral
carbon.
Haloalkanes and Haloarenes 603

i.e. CH3 CH3 So attack from below or above the plane can take place.
Br2
If 50% attack below and above the plane of carbocation take
FeBr3
place than 100% racemization occurs but it may not be
CH3 CH3 highly probable.
Br i.e. if inversion occurs more than retention leading to partial
(A) racemization.
CH3 35. (b) (1) CH3CH2CH2–Br + KOH
Br
® CH3CH=CH2 + KBr + H2O
+ This is dehydrohalogenation reaction which is an
CH3 example of elimination reaction.

(2) H3C CH3 H3C CH3


33. (a) Tertiary butyl chloride will give the most stable tertiary C + KOH ® C + KBr
carbonium ion among the other given compounds
Br OH
H3C – H3C
C Cl
Cl +
C Replacement of Br– by OH– is substitution reaction
thus it is a nucleophilic substitution reaction.
H3C CH3 H3C CH3 Br
t-butyl carbonium ion
(3) + Br2 ®
34. (b) In SN 1 reaction, carbocation a planar species as Br
intermediate is formed. Above reaction involves addition of Br 2 across double
bond. Thus it is called addition reaction.
R1
Nu
Å
C R3
Nu
R2
EBD_7327
604 CHEMISTRY

Alcohols, Phenols
25 And Ethers
A. ALCOHOLS AND PHENOLS
CLASSIFICATION
Alcohols and phenols are classified as mono–, di–, tri– or polyhydric depending upon whether they contain one, two, three or many
hydroxyl groups in their structures.
Monohydric alcohols are further classified according to hybridisation of C-atom to which –OH is attached.

C sp3 – OH bond Csp 2 – OH bond


Here –OH group is attached
to one of the carbon atoms of
Alcohols can be classi ed Allylic
alcohols
Benzylic
alcohols
double bond.
as primary (1º), secondary
(2º) and tertiary (3º) The sp 3 hybridised The sp 3 hybridised
according as the –OH group carbon containing carbon containing – OH
is attached to a primary, – OH group is attached Vinylic alcohols Phenols
group is attached to an
secondary and tertiary to C = C. Example OH OH
aromatic ring. Example CH2 = CH – OH
carbon atoms H CH3
1° : CH2 – OH C = C C OH C OH
2° : CH – OH H H
3° : C – OH

Ethers are classified as This is because of


(i) simple or symmetrical, if alkyl or aryl groups attached to (i) partial double bond character of C–O bond
O-atom are same. Example: CH3OCH3. (ii) sp2 – hybridised state of C to which O is attached.
(ii) mixed or unsymmetrical if the two groups attached to
In alcohols and phenols due to high electronegativity
O–atom are different. Example : CH3OC2H5.
O–atom aquires partial negative charge and results in formation
Ethers can be further classified into aliphatic and aromatic ethers.
of dipole.
Aromatic ethers can be further classified into phenolic or
alkylarylethers (contain alkyl and aryl group) and aryl ethers d+ d– d+
R— O— H
(contain aryl group only).
Dipole moment of alcohols are higher than phenols. It is because
STRUCTURES OF FUNCTIONAL GROUPS
C–O bond in phenol is less polar due to electron-withdrawing
In alcohols, O of –OH bond is attached to sp3 hybridised carbon.
effect of the benzene ring whereas in alcohols C–O bond is more
O polar due to electron donating effect of – R group.
The bond angle is less than tetrahedral angle (109°28’) due
C H In ethers, 2 bond pairs and 2 lone pairs on O are arranged
to repulsion between unshared pair of e–1s on O. approximately tetrahedrally. The bond angle is slightly greater
In phenols, –OH group is attached to sp2 hybridised C of aromatic than tetrahedral angle due to repulsive interaction between bulky
ring. The C–O bond length in phenols is less than methanol. –R groups.The C–O bond length is almost same as that in alcohols.
Alcohols, Phenols And Ethers 605

PREPARATION OF ALCOHOLS
(i) From Alkenes:

Step 2:

H H H OH
+
Step 3: C C O H + H2O C C + H3O
+

(ii) Carbonyl Compounds


H2/Pd
(a) Reduction of aldehydes:RCHO RCH2OH
or LiAlH 4 or NaBH 4
and ketones 1° alcohol
NaBH4
RCOR’ RCHR’
or LiAlH4 |
OH
2° alcohol
Note: 3ºalcohol cannot be obtained by this method.
(i) LiAlH4 RCH OH
(b) Reduction of acids and : RCOOH 2
(ii) H2O
esters
(i) LiAlH4
RCOOR’ RCH2OH + R’OH
(ii) H2O
Commercially, acids are reduced to alcohols by first converting them into esters.
(iii) Grinard Reagents:

When formaldehyde is used primary alchols are formed.


H H OMgX H 2O OH
R – MgX + C = O ¾¾® C ¾¾¾ ® RCH2OH + Mg
H H R X
Alcohol
with aldehydes other than HCHO secondary alcohols are obtained. Secondary alchol can also be prepared by addition of esters.
H H H
H 2O
R Mg X + R' C ¾¾¾® R' C R ¾¾¾
® R' C R
O O Mg X OH
Aldehyde (Secondary alcohol)
EBD_7327
606 CHEMISTRY

If carbonyl compound is ketone, product will be tertiary (ii) From Diazonium Salts
alcohol.
+ –
NH2 N2Cl OH

273 – 278K Dil. H2SO4,H2O


+
NaNO2 + HCl warm
N2 + HCl
R OH
C + HOMgBr (iii) From Benzene Sulphonic Acid
R R
Tertiary alcohol

(iv) With Ethylene Oxide :


The alcohol obtained has two carbon atoms more than alkyl H2S2O7
group of the Grignard reagent.
NaOH
R Mg X + CH2 CH2 (Ethylene oxide)
SO3Na
O
¾¾® R–CH2–CH2–O–Mg–X
H O
¾¾¾
2 ® R–CH CH OH + Mg(X)OH
2 2
Alcohol (i) NaOH (ii) Dil. HCl
623 K
(v) From Alkyl Halides (Hydrolysis) :
OH
The hydrolysis occurs by nucleophilic substitution reaction.
R–CH2–X + AgOH(aq.) ¾¾® R–CH2–OH + AgX
1° Alcohol
+Na2SO3

R R Phenol
| |
R – CH –X + K–OH (aq.) ¾¾® R– CH –OH + KX
Alkyl halide 2° Alcohol

R R (iv) From Cumene


| |
R - C - X + H–OH ¾¾® R - C - OH + HX
| | CH3 CH3
R R
H3C CH H3C C O O H OH
3° Alcohol
O2 +
(vi) From Primary Amines : H
+ CH3COCH3
Primary alcohols are formed when primary amines are treated H2O
Phenol
with nitrous acid (HNO2)
HCl
R – NH2 + HNO2 ¾¾¾
® R–OH + N2 + H2O (v) By Distilling a Phenolic Acid with Soda Lime
(Decarboxylation).
PREPARATION OF PHENOLS
(i) From Haloarenes OH
OH
Cl ONa OH COOH
CaO
+ NaOH ¾¾¾
D
® + Na2CO3
623 K HCl
+ 2NaOH
320 atm –NaCl Phenol
Phenol –NaCl, –H2O
Alcohols, Phenols And Ethers 607

PHYSICAL PROPERTIES OF ALCOHOLS AND (ii) Solubility:


PHENOLS Their solubility in water is due to formation of H-bonds with
water. Thus more will be the capability to form H-bonds more
(i) Boiling points: will be solubility of given alcohol and phenol.
Boiling point increases with increase in number of C-atoms Solubility decreases with increase in size of alkyl / aryl groups
due to increase in van der Walls force. (i.e., hydrophobic group).
Among isomeric alcohols, the boiling points decreases with The solubility of phenols in water is much lower than that of
branching as surface area decreases. Thus b. pts. decrease alcohols because of the larger hydrocarbon part.
in the order.
(iii) Formation of Alcoholate
Primary > Secondary > Tertiary
B Pts. of alcohols and phenols are higher than hydrocarbons, Lower alcohols such as CH3OH and C2H5OH react with
alkyl halides and ethers. This is due to intermolecular anhydrous CaCl2 or MgCl2 to form solid derivatives called
H-bonding in them as a result of which they exist as alcoholates.
associated molecules. This intermolecular H-bonding is CaCl2 + 4 CH3OH ® CaCl2.4 CH3OH
missing in hydrocarbons and ethers. CaCl2 + 3 C2H5OH ® CaCl2.3 C2H5OH
MgCl2 + 6 C2H5OH ® MgCl2.6C2H5OH.
CHEMICAL PROPERTIES OF ALCOHOLS AND PHENOLS

Reactions involving Reactions involving cleavage of


cleavage of O–H bond or C–OH bond
alcohols/ phenols as nucleophiles or
+ protonated alcohols/phenols as electrophiles
R O H+ C R O C + +
RCH2OH + H RCH2 OH2
H
– +
Nu + CH2 OH2 Nu CH2 + H2O
+
R O C +H R R

I. Cleavage of O–H Bond Acidity of alcohols: They are acidic due to polar O–H bond.
The general order of reactivity of alcohols in this type of Alkyl groups increases the e– density on O atom of the O–H
reaction is :– bond. As a result, the electrons of the O–H bond are not
Primary > Secondary > Tertiary sufficiently attracted towards the oxygen atom thereby
decreasing the polarity of O–H bond. Thus tending to
(i) Acidity of Alcohols and Phenols:
decrease the acid strength. Acid strength order of alcohols
2R – O – H + 2Na ¾¾ ® 2 R – O – Na + H2 will be 1° > 2 ° > 3°. Due to same reason, alcohols are weaker
6 ( CH 3 ) COH + 2Al ¾¾
3 ( 3 3
)
® 2 ( CH 3 ) CO Al + 3H 2 acids than water.
Basic strength order of alkoxides.
OH R3CO– > R2CHO– > RCH2O–
ONa Acidity of phenols: Phenols are stronger acids than alcohols
and water. This is explained as follows:
+ NaOH + H2O (a) In phenols, –OH is directly attached to sp2 – hybridised
carbon. As a result, e– density on O decreases. This
The acidic order of alcohols is increases polarity of O–H bond resulting in increased
MeOH > 1º > 2º > 3º ionisation of phenols than alcohols.

Further, greater acidity of a phenol than an alcohol is due to possibility of resonance in phenol which leads to electron-
deficient oxygen atom. Presence of electron-deficient oxygen atom (see structures II, III and IV) in turn weakens the
+
- O ¾¬ H bond, and thus facilitates release of proton.
+ + +
: O—H
:

O—H O—H O—H

:– –:
¬¾® ¬¾® ¬¾®

:

I II III IV
Such structures are not possible in alcohols.
EBD_7327
608 CHEMISTRY

(b) Once hydrogen atom is removed from phenol, the ion (phenoxide) is very much stabilized due to delocalization of its
negative charge.
: O: – :O :O

:
O

:– –:
¬¾® ¬¾® ¬¾®

:
V VI VII VIII
Resonance in phenoxide ion (note that structures VI to VIII are equivalent)
Remember that phenoxide ion is very much more stable than the parent compound phenol because phenoxide ion does not
involve charge separation, while in phenol three equivalent resonanting structures (II to IV) involve charge separation.
(c) Electron-withdrawing substituents increase the acidity of phenols ; while electron-releasing substituents decrease acidity.

OH O
G withdraws electrons, thus disperses the –ve
+
charge of the ion, stabilises it and hence
H + increases ionization of the parent phenol.
¾® ¾®
G G
+
(where G = — NO2, —CN, —CHO, —COOH,— NR 3 , —X)

OH O
G releases electrons, thus intensifies the -ve charge
+ of the ion, destabilises it and hence decreases
H + ionization of the parent phenol.
¾® ¾®
G G
(ii) Esterification:
H +
ˆˆ†
Ar / R – OH + R '– COOH ‡ˆ ˆ Ar / ROCOR '+ H 2O
H+
Ar / R – H + ( R 'CO ) 2 O ‡ˆ
ˆˆ†
ˆ Ar / ROCOR '+ R ' COOH
Py
R / Ar – OH + R 'COCl ¾¾® R / ArOCOR '+ HCl
The order of ease of formation of ester
Primary alcohol > Secondary alcohol > Tertiary alcohol
HCOOH > CH 3COOH > (CH 3 ) 2 CH.COOH > (CH 3 )3 C.COOH
(iii) Reaction with Ethylene Oxide :
R–O–H+ ¾¾® CH 2 - CH 2 ¾¾¾
ROH
® CH 2 - CH 2
| | -H2 O
| |
OR OH OR OR
1, 2-dialkoxy ethane
(iv) Reaction with Grignard Reagent :
OR
ROH + R'MgX ¾¾® Mg + R'H
X
Alkane
II. Cleavage of C–OH Bond
The order of reactivity in this type of reaction is –
3º alcohol > 2º alcohol > 1º alcohol
Alcohols, Phenols And Ethers 609

Note:
(a) Reaction of R–OH with HX is used to distinguish between
1°, 2°, 3° alcohols (Lucas test) is discussed later. Reactivity
order of halogen acid is HI > HBr > HCl
(b) 1° alcohols on oxidation give aldehydes while 2° alcohols
give ketones. 3° alcohols do not undergo oxidation reactions.
Under strong conditions like strong oxidising agent (KMnO4)
and elevated temperatures, cleavage of various C–C bonds
takes place and a mixture of acids with lesser no. of C–atoms
is formed.
(c) With Cu, 3° alcohols undergo dehydration to form alkenes.
(where G = — R, —OR, —NR2)
Reactions Involving the Alcohol Molecule as a Reactions of Phenols
Whole. (i) Electrophilic aromatic substitution
(i) Dehydration: (a) Nitration

(ii) Reaction with H2SO4 :


110 C o
(a) C2H5OH + H2SO4 ¾¾¾® CH3CH2OSO3H
Ethyl hydrogen sulphate

(b) CH3–CH2–OH + H2SO4 ¾¾¾® 140ºC

(excess) (conc.)
··
CH3–CH2– O –CH2–CH3
··

Diethyl ether
160º
(c) CH3 – CH2 – OH + H2SO4 ¾¾ ¾ ® CH2 = CH2
(excess)
(iii) Oxidation :
CrO or
(a) RCH 2OH ¾¾¾¾
3
PCC
® RCHO
CrO or
(b) R 2 CHOH ¾¾ ¾
3 ¾
PCC
® R 2 CO
(iv) Reduction :
Re d P
R–O–H + 2HI ¾¾¾
150º C
® R–H + H2O + I2
(v) Dehydrogenation :
H H
| |
Cu / ZnO
R -C-O ¾¾¾¾® R - C = O + H2
| | 300º
H H aldehyde (b) Halogenation
primary alcohol
R
| Cu / ZnO
R -C-O ¾¾¾¾
300º
® R - C - R + H2
| | ||
H H O
secondary alcohol Ketone
R
|
R - C - OH
| Cu / ZnO
H 2 C– H ¾¾¾¾
300º
® R - C - R + H2O
||
tert.alcohol CH 2
alkene
EBD_7327
610 CHEMISTRY

(ii) Kolbe’s Reaction (v) Coupling Reaction


+ –
N º N – Cl + H OH

pH 9-10
¾¾¾¾
® N=N OH
(–HCl)
p-hydroxyazobenzene

Note: o – and p– nitrophenols can be separated by steam


distillation. o – isomer is steam volatile due to intramolecular
H-bonding while p-isomer is less volatile due to intermolecular
H-bonding which causes association of molecules.
SOME IMPORTANT ALCOHOLS
(i) Methanol (CH3OH)
(iii) Riemer-Tiemann Reaction It is also known as wood spirit
OH ZnO–Cr2O3
CO + 2H2 ¾¾¾¾¾¾ ® CH3OH
200 - 300atm
CHCl3 573 - 673K
+
340 K aqNaOH It is a colourless liquid and boils at 337 K. It is highly
ONa poisonous, even small quantities can cause blindness and
CHCl2 large amounts can cause death. It is used as a solvent in
+ paints, varnishes and for making formaldehyde
(ii) Ethanol (C2H5OH)
–2NaCl 2NaOH It is obtained by fermentation
ONa invertase
C12 H 22 O11 + H 2 O ¾¾¾¾® C 6 H12 O6 + C6 H12 O6
CH(OH)2 Glu cos e fructose
zymase
C6 H12 O6 ¾¾¾¾
® 2C 2 H5 OH + 2CO2
–H2O
It is a colourless liquid and boils at 351 K. It is used as a
ONa solvent in paint industry and in preparation of a number of
CHO
carbon compounds.
(iii) Denatured Alcolol
+ The commercial alcohol is made unfit for drinking by mixing
H
in it some CuSO4 (to give it a colour) and pyridine (a foul
OH smelling liquid) It is known as denaturation of alcohol.
CHO (iv) Power Alcohol
Alcohol used for generating power is called power alcohol. It
Salicylaldehyde consists of a mixture of absolute alcohol and petrol roughly
(iv) Oxidation in the ratio 20:80.
DISTINCTION BETWEEN 1°, 2° AND 3° - ALCOHOLS
(i) Oxidation
3 CrO or
1° alcohols ¾¾¾¾® aldehyde
PCC

3 CrO or
2° alcohol ¾¾¾¾® ketone
PCC

4 KMnO
3° alcohol ¾¾¾¾®
High temperature
Mixture of acids with lesser no. of C-atoms.
Alcohols, Phenols And Ethers 611

(ii) Action of hot copper (iv) Victor Meyer’s Test


Cu
1° alcohols ¾¾¾
® aldehyde + H
2
573K
Cu
2° alcohol ¾¾¾
® ketone + H
2
573K
Cu
3° alcohol ¾¾¾
® alkene + H O
2
573K
(iii) Lucas Test
Lucas reagent is an equimolar mixture of conc. HCl and
anhydrous ZnCl2.
Lucas
1° alcohols ¾¾¾¾ ®
reagent
No reaction occurs no turbidity appears
Lucas reagent
2° alcohol ¾¾¾¾¾¾®

Alkyl chloride (R2CHCl) is formed turbidity appears in


about 5 minutes
Lucas reagent DISTINGUISH BETWEEN A LCOHOLS AND
3° alcohol ¾¾¾¾¾¾® PHENOLS:
Alkyl chloride (R3CCl) is formed turbidity appears (i) Phenols react with FeCl 3 to give violet colouration whereas
immediately. alcohols do not react.
(ii) Phenols on shaking with bromine water gives a white ppt. of
2, 4, 6-tribromophenol but alcohols do not.

(B) ETHERS
PREPARATION OF ETHERS
(i) By Dehydration of Alcohols (iii) Williamson’s Synthesis
4

+ H
CH3CH2 – O
H
(ii) By reaction of ethyl halide with dry Ag 2O :
CH2 CH2 I + Ag O Ag + I CH2 CH3 ¾¾® CH3 –CH2–O–CH2–CH3 + 2AgI

CH3 – I + Ag – O – Ag + I – CH 2 – CH 3 CH3 – O – CH2 – CH3 + 2AgI


EBD_7327
612 CHEMISTRY

PHYSICAL PROPERTIES OF ETHERS (iv) Reaction with H2SO4 :


Ethers are polar but very weakly. Their B.Pts. are comparable to (a) With conc. H2SO4
those of alkanes and are much less than alcohols due to absence
of H-bonding. D
C2H5—O—C2H5 + H2SO4 ¾¾® C2H5OSO2OH + C2H5OH
Ethers are only slightly soluble in water. Solubility of lower ethers
in water is due to formation of H-bonds. As the molecular mass Ethyl hydrogen sulphate
increases, the solubility of ethers in water decreases due to increase (b) With dil. H2SO4 :
in hydrocarbon part.
dil.H SO
CHEMICAL PROPERTIES OF ETHERS 2 4®
C2H5 – O – C2H5 + H – OH ¾¾¾¾¾ 2C2H5OH
(i) Reaction with HX (v) Electrophilic Substitution
(a) Halogenation:

If HX is hot :
D
CH3–CH2–O–CH2–CH3+2 H–X ¾¾® 2CH3–CH2–X+H2O (b) Friedel-Crafts reaction:
(ii) Halogenation :
(a) In dark :
b' a' a b
CH3 - CH2 - O - CH2 - CH3 + 2Cl —Cl
Cl
|
dark
¾¾¾ ® CH3 - CH - O - CH - CH3 + 2HCl
|
Cl
a, a'-Dichlorodiethyl ether
(b) In light
CH3–CH2–O–CH2–CH3 + 10Cl2
hn
¾¾® CCl3 – CCl2 – O – CCl2 – CCl3 + 10HCl
Perchlorodiethyl ether

(iii) Oxidation :
CH3 CH2 CH3 CH2 (c) Nitration:
1
·O·
· · + O 2 ¾¾® O ®O
2
CH3 CH2 CH3 CH2
C2H5—O—O—C2H5
ether peroxide

H
1 |
C2H5–O–O–C2H5 + O 2 ¾® C2H 5 - O - C - CH3
2 |
O-O-H
Ethoxyethyl hydroperoxide
Alcohols, Phenols And Ethers 613

CONCEPT MAP
EBD_7327
614 CHEMISTRY

1. Lucas reagent is 11. Which of the following is dihydric alcohol ?


(a) conc. HCl and anhydrous ZnCl2 (a) Glycerol (b) Ethylene glycol
(b) conc. HNO3 and hydrous ZnCl2 (c) Catechol (d) Resorcinol
(c) conc. HCl and hydrous ZnCl2 CH 3
(d) conc. HNO3 and anhydrous ZnCl2 |
2. Glycerol on oxidation with bismuth nitrate produces 12. The IUPAC name of CH 3 - CH - CH 2 - C - CH 3 is :
(a) oxalic acid (b) glyceric acid | |
OH OH
(c) glyoxalic acid (d) meso oxalic acid
3. The alcohol manufactured from water gas is (a) 1, 1-dimethyl-1, 3-butanediol
(a) ethanol (b) methanol (b) 2-methyl-2, 4-pentanediol
(c) isobutanol (d) butanol (c) 4-methyl-2, 4-pentanediol
4. Which of the following are isomers ? (d) 1, 3, 3-trimethyl-1, 3-propanediol
(a) Methyl alcohol and dimethyl ether Br2
(b) Ethyl alcohol and dimethyl ether 13. OH SO3H X,
H2O
(c) Acetone and acetaldehyde
X is identified as
(d) Propionic acid and propanone
(a) 2, 4, 6-tribromophenol
5. Dehydration of alcohol is an example of (b) 2-bromo-4-hydroxylbenzene sulphonic acid
(a) Redox reaction (b) Elimination reaction (c) 3, 5-dibromo-4-hydroxybenzene sulphonic acid
(c) Substitution reaction (d) Addition reaction
(d) 2-bromophenol
6. Methylated spirit is 14. The C–O–H bond angle in ethanol is nearly
(a) methanol (b) methanol + ethanol (a) 90° (b) 104°
(c) methanoic acid (d) methanamide (c) 120° (d) 180°
7. The structural formula of cyclohexanol is
15. Methanol is industrially prepared by
CH 2 CH 2 (a) oxidation of CH4 by steam at 900°C
(b) reduction of HCHO using LiAlH4
H2 C CHOH H2 C CHOH (c) reaction HCHO with a solution of NaOH
(a) (b) (d) reduction of CO using H2 and ZnO – Cr 2O3.
H2 C CH 2 H2 C CH 2
16. When alcohol reacts with concentrated H2SO4, intermediate
CH 2 CH 2 compound formed is :
(a) carbonium ion
(b) alkoxy ion
CH 2 OH
(c) alkyl hydrogen sulphate
(c) (d) (d) none of the above
17. HBr reacts fastest with
(a) 2-Methylpropan-1-ol (b) 2-Methylpropan-2-ol
8. Molecular formula of amyl alcohol is (c) propan-2-ol (d) propan-1-ol.
(a) C7 H14 O (b) C6 H13 O (c) C5H12 O (d) C5H10 O 18. In the following reaction ‘A ’ is
9. Phenol is more readily soluble in H O
C2H5MgBr + H2C – CH2 2 A
(a) NaOH solution
(b) dil. HCI O
(c) both NaOH and HCl (a) C2H5CH2CHO (b) C2H5CH2CH2OH
(c) C2H5CH2OH (d) C2H5CHO
(d) NaHCO 3 19. n-Propyl alcohol and isopropyl alcohol can be chemically
10. Absolute alcohol is distinguished by which reagent?
(a) 100% pure ethanol (a) PCl5
(b) 95% alcohol + 5% H 2 O (b) Reduction
(c) ethanol + water + phenol (c) Oxidation with potassium dichromate
(d) 95% ethanol + 5% methanol (d) Ozonolysis
Alcohols, Phenols And Ethers 615

20. Ethylene oxide when treated with Grignard reagent yields 31. Which one of the following statements is correct?
(a) tertiary alcohol (b) cyclopropyl alcohol (a) Phenol is more acidic than carbonic acid
(c) primary alcohol (d) secondary alcohol (b) Phenol is less acidic than ethyl alcohol
21. Among the following the one that gives positive iodoform (c) Phenol is more acidic than ethanol
test upon reaction with I2 and NaOH is (d) Phenol is more acidic than acetic acid
32. Which one of the following alcohols is least soluble in water?
CH3
| (a) CH 3 OH (b) C 3 H 7 OH
(a) CH 3 - C HCH 2 OH
(c) C 4 H 9 OH (d) C10 H 21OH
(b) PhCHOHCH3 33. In a reaction :
(c) CH3CH2CH(OH)CH2CH3
(d) C6H5CH2CH2OH CH 2OH
hypochloro us acid R
CH 2 = CH 2 ¾¾ ¾ ¾ ¾ ¾¾® M ¾¾® |
22. The best method to prepare cyclohexene from cyclohexanol CH 2OH
is by using
(a) Conc. HCl + ZnCl2 (b) Conc. H3PO4 Here M = molecule and R = reagent. The
(c) HBr (d) Conc. HCl M and R are :
(a) CH2–CH2 and KOH
X
23. C6 H5 - CH = CHCHO ¾¾® C6 H5 CH = CHCH 2 OH . In (b) CH3CH2Cl and NaOH
the above sequence X can be (c) CH2OH–CH2Cl and aq. NaHCO3
(a) H 2 / Ni (b) NaBH 4 (d) CH3CH2OH and KOH
34. The dehydration of 2-methylbutanol with conc. H 2SO 4
(c) K 2 Cr2 O7 / H + (d) Both (a) and (b) gives :
24. Which of the following reactions will not result in the (a) 2-Methylbutene as major product
formation of anisole?
(b) Pentene
(a) Phenol + dimethyl sulphate in presence of a base
(b) Sodium phenoxide is treated with methyl iodide (c) 2-Methylbut-2-ene as major product
(c) Reaction of diazomethane with phenol (d) 2-Methylpent-2-ene
(d) Reaction of methylmagnesium iodide with phenol 35. The product formed by the reduction of glycolic acid with
25. Acid catalyzed hydration of alkenes except ethene leads to HI, is :
the formation of (a) formic acid (b) ethane
(a) primary alcohol (c) iodoacetic acid (d) glycol
(b) secondary or tertiary alcohol 36. Ethyl alcohol can be prepared from Grignard reagent by the
(c) mixture of primary and secondary alcohols reaction of :
(d) mixture of secondary and tertiary alcohols (a) HCHO (b) R2CO
26. Isopropyl alcohol is obtained by reacting which of the
(c) RCN (d) RCOCI
following alkenes with concentrated H2SO4 followed by
boiling with H2O? 37. Which one can differentiate between C2H5OH and CH3OH
(a) Ethylene (b) Propylene (a) H2O (b) Na2CO3 + I2
(c) 2-Methylpropene (d) Isoprene (c) NH3 (d) HCl
27. Which one of the following is not formed when glycerol 38. In the reaction :
reacts with HI ? K Cr O oxidation
A ¾¾¾¾®
2 2 7
acetone ¾¾¾¾® acetic acid
(a) CH3 — CHI — CH3 H 2SO4
(b) CH3 — CH = CH2 A is
(c) CH2OH — CHI — CH2OH
(a) 1- propanol (b) 2-butanol
(d) CH2 = CH — CH2I
28. Dehydration of 2-butanol yields (c) 2-propanol (d) ethanol
(a) 1-butene (b) 2-butene 39. In the reaction :
(c) 2-butyne (d) both (a) and (b) oxidation
CH 3 OH ¾¾¾¾® A ¾¾¾
NH 3
® B ; A and B respectively
29. Which of the following reagent is best to change glycerol to are
acrolein ? (a) HCHO, HCOONH4 (b) HCOOH, HCOONH4
(a) P2 O 5 (b) Conc. H 2 SO 4 (c) HCOOH, HCONH2 (d) HCHO, HCONH2
(c) Anhydrous CaCl2 (d) KHSO4 40. The end product of the reaction
30. The reaction of Lucas reagent is fastest with:
KCN H O+
C 2 H 5 OH ¾PCl
¾¾ 5 ® A ¾¾¾® B ¾¾ 3¾
¾® C is
(a) (CH 3 ) 2 CHOH (b) CH3 (CH 2 )2 OH
(a) propanol (b) propanoic acid
(c) CH 3 CH 2 OH (d) (CH 3 ) 3 COH (c) propanamide (d) none of these
EBD_7327
616 CHEMISTRY

41. Glycerol is more viscous than ethanol due to in the above reaction, Z is :
(a) high molecular weight (a) phenol (b) benzoic acid
(b) high boiling point (c) salicylaldehyde (d) carbolic acid
(c) many hydrogen bonds per molecule 47. When phenol is heated with CHCl 3 and NaOH then
(d) Fajan’s rule
salicylaldehyde is produced. This reaction is known as
42. The product obtained from the reaction is:
(a) Rosenmund’s reaction (b) Reimer-Tiemann reaction
OH
( i) NaOH
(c) Friedel-Crafts reaction (d) Sommelet reaction
¾¾¾ ¾
¾®
(ii) CO 2 48. The intermediate formed in aldol condensation is
(iii ) Hydrolysis (a) aldol (b) carbanion
(a) Benzene (b) Toluene (c) alcohol (d) a-hydrogen ester
(c) Salicylic acid (d) Benzoic acid
49. When phenol is treated with excess bromine water. It gives
43. Which one of the following compounds will be most readily
(a) m-Bromophenol (b) o-and p-Bromophenols
dehydrated?
OH (c) 2,4-Dibromophenol (d) 2,4, 6-Tribromophenol.
50. Which is formed when benzalamine react with nitrous acid
(a)
CH3 (a) C6 H5OH (b) C6 H5ON
O O
(c) C6 H5 N 2 OH (d) C6 H5CH 2OH

(b) CH 51. The reaction :


3
Pyridine
OH C 2 H 5 OH + SOCl 2 ¾¾ ¾¾® C 2 H 5Cl + SO 2 + HCl
O
is known as
(c) (a) Kharasch effect (b) Williamson's synthesis
H3 C
OH (c) Darzen's procedure (d) Hunsdiecker reaction
O OH 52. Benzyl alcohol is obtained from benzaldehyde by:
(a) Perkin’s reaction (b) Wurtz reaction
(d) (c) Cannizzaro’s reaction (d) Claisen’s reaction
CH3
53. On distillation with Zn dust, phenol gives :
44. Following compounds are given: (a) acetophenone (b) benzoic acid
(i) CH3CH2OH (ii) CH3COCH3 (c) benzene (d) benzophenone
54. The most suitable reagent for the conversion of
(iii) CH 3 - CHOH (iv) CH3OH
RCH2OH ¾ ¾® RCHO is :
CH 3 (a) KMnO4
(b) K2Cr2O7
Which of the above compound(s), on being warmed with (c) CrO3
iodine solution and NaOH, will give iodoform?
(d) PCC (pyridine chlorochromate)
(a) (i) and (ii) (b) (i), (iii) and (iv) 55. Phenol is more acidic than alcohol because.
(c) only (i) (d) (i), (ii) and (iii) (a) phenol is more stable than water
45. Ethanol and dimethyl ether form a pair of functional isomers. (b) phenol is aromatic and alcohol is aliphatic
The boiling point of ethanol is higher than that of dimethyl (c) phenoxide ion is resonance stabilised
ether, due to the presence of (d) None of these
(a) H-bonding in ethanol
56. To distinguish between salicylic acid and phenol, one can
(b) H-bonding in dimethyl ether
use :
(c) CH3 group in ethanol
(d) CH3 group in dimethyl ether (a) NaHCO3 solution
NaOH CHCl +KOH (b) 5% NaOH solution
46. X ¾¾ ¾
¾® Y ¾¾ ¾3¾ ¾¾®
Heat (c) neutral FeCl3
OH (d) bromine water
57. Which of the following has lowest boiling point ?
COOH
[ O] (a) p-Nitrophenol (b) m-Nitrophenol
Z ¾¾
¾®
(c) o-Nitrophenol (d) Phenol
Alcohols, Phenols And Ethers 617

58. The major product obtained on the monobromination (with 67. The major product of the reaction between tert-butyl chloride
Br2/FeBr3) of the following compound A is : and sodium ethoxide is
(a) 2-methylprop-1-ene
OCH3
(b) 1-butene
(c) 2-butene
CH 3 (d) ethene
A 68. Which of the following product is formed, when ether is
exposed to air ?
OCH3 OCH3
(a) Oxide (b) Alkanes
Br
(a) (b) (c) Alkenes (d) Peroxide of diethyl ether
CH 3 CH 3 69. Ether which is liquid at room temperature is
Br (a) C2H5OCH3 (b) CH3OCH3
(c) C2H5OC2H5 (d) None of these
OCH 3
OCH 3 70. What is the correct order of acidity from weakest to
(c) (d) Br strongest acid for these compounds?
Br CH 3 O OH OH CH2OH
CH 3
59. Which one is formed when sodium phenoxide is heated with
ethyl iodide ? CF3CH2OH
(a) Phenetole (b) Ethyl phenyl alcohol
(c) Phenol (d) None of these
60. CH3OC2H5 and (CH3)3 C – OCH3 are treated with hydriodic I II III IV
acid. The fragments obtained after reactions are (a) I < IV < III < II
(a) CH3I + HOC2H5; (CH3)3 CI + HOCH3
(b) III < IV < I < II
(b) CH3OH + C2H5I; (CH3)3 Cl + HOCH3
(c) CH3OH + C2H5I; (CH3)3COH + CH3I (c) IV < I < III < II
(d) CH3I + HOC2H5; CH3I + (CH3)3 COH. (d) II < III < I < IV
alkali 71. In the following reaction
61. In the reaction Ar - OH + RX ¾¾¾ ® A, A is
Red P + HI
C2H5OC2H5 ∗4[H] ¾¾¾
(a) an aldehyde (b) an aryl chloride ¾¾ ↑ 2X ∗ H 2O, X is
(c) an ether (d) a ketone
(a) ethane (b) ethylene
62. In Williamson’s synthesis, ethoxyethane is prepared by
(a) passing ethanol over heated alumina (c) butane (d) propane
(b) sodium ethoxide with ethyl bromide 72. The ether that undergoes electrophilic substitution reactions
(c) ethyl alcohol with sulphuric acid is
(d) ethyl iodide and dry silver oxide (a) CH3OC2H5 (b) C6H5OCH3
63. Maximum enol content is in (c) CH3OCH3 (d) C2H5OC2H5
O O O 73. Etherates are
(a) (b)
(a) ethers
O O O (b) solution in ether
(c) (d) (c) complexes of ethers with Lewis acid
H H (d) complexes of ethers with Lewis base
64. Formation of diethyl ether from ethanol is based on a 74. The compound that undergoes dehydration very easily is
(a) dehydration reaction (a) 2-methylpropan-2-ol
(b) dehydrogenation reaction
(b) ethyl alcohol
(c) hydrogenation reaction
(d) heterolytic fission reaction (c) 3-methyl-2-butanol
65. A fruity smell is produced by the reaction of C2H5OH with (d) propyl alcohol
(a) CH3COCH3 (b) CH3COOH 75. Diethyl ether on heating with conc. HI gives two moles of
(c) PCl5 (d) CH3CHO (a) ethanol (b) iodoform
66. The reaction of sodium ethoxide with ethyl iodide to form (c) ethyl iodide (d) methyl iodide
diethyl ether is termed 76. Methylphenyl ether can be obtained by reacting
(a) electrophilic substitution (a) phenolate ions and methyl iodide
(b) nucleophilic substitution (b) methoxide ions and bromobenzene
(c) electrophilic addition (c) methanol and phenol
(d) radical substitution
(d) bromo benzene and methyl bromide
EBD_7327
618 CHEMISTRY

77. Which of the following alcohols gives the best yield of dialkyl 83. Among the following four compounds
ether on being heated with a trace of sulphuric acid? (i) phenol (ii) methylphenol
(a) 2-Pentanol (iii) meta-nitrophenol (iv) para-nitrophenol
(b) Cyclopentanol the acidity order is :
(c) 2-Methyl-2-butanol (a) ii > i > iii > iv (b) iv > iii > i > ii
(d) 1-Pentanol (c) iii > iv > i > ii (d) i > iv > iii > ii
78. In the reaction:
84. When glycerol is treated with excess of HI, it produces:
CH3 (a) glycerol triiodide (b) 2–iodopropane
|
Heated (c) allyl iodide (d) propene
CH3 - CH - CH 2 - O - CH 2 - CH3 + HI ¾¾¾¾ ®
85. Which of the following compounds can be used as antifreeze
Which of the following compounds will be formed? in automobile radiators ?
(a) CH3 - CH - CH3 + CH 3CH 2 OH (a) Methyl alcohol (b) Glycol
| (c) Nitrophenol (d) Ethyl alcohol
CH3 86. In the following sequence of reactions,
P +I Mg HCHO
(b) CH3 - CH - CH 2 OH + CH3CH3 CH 3CH 2 OH ¾¾¾
2 ® A ¾¾¾® B ¾¾¾¾
®
ether
|
CH3 H O
C ¾¾¾
2 ®D
the compound D is
CH3
(a) propanal (b) butanal
|
(c) CH3 - CH - CH 2 OH + CH3 - CH 2 - I (c) n-butyl alcohol (d) n-propyl alcohol.
87. Phenol, when it first reacts with concentrated sulphuric acid
CH3 and then with concentrated nitric acid, gives
| (a) 2, 4, 6-trinitrobenzene (b) o-nitrophenol
(d) CH3 - CH - CH 2 - I + CH3CH 2OH
(c) p-nitrophenol (d) nitrobenzene
79. Consider the following reaction, 88. The major product obtained on interaction of phenol with
PBr3 alc.KOH sodium hydroxide and carbon dioxide is
ethanol ¾¾¾ ® X ¾¾¾¾® Y
(i) H 2SO4 room temperature (a) salicylaldehyde (b) salicylic acid
¾¾¾¾¾¾¾¾¾¾® Z;
(ii) H 2O, heat (c) phthalic acid (d) benzoic acid
the product Z is: 89. From amongst the following alcohols the one that would
(a) CH3CH2 – O – CH2 – CH3 react fastest with conc. HCl and anhydrous ZnCl 2, is
(b) CH3 – CH2 – O – SO3H (a) 2-Butanol (b) 2- Methylpropan-2-ol
(c) CH3CH2OH (c) 2-Methylpropanol (d) 1- Butanol
(d) CH2 = CH2 90. The correct order of acid strength of the following
80. H2COH · CH2OH on heating with periodic acid gives:
compounds :
CHO (A) Phenol (B) p–Cresol
(a) 2 HCOOH (b) |
CHO (C) m–Nitrophenol (D) p–Nitrophenol
H (a) D > C > A > B (b) B > D > A > C
(c) 2 C=O (d) 2 CO2 (c) A > B > D > C (d) C > B > A > D
H 91. Consider the following reaction :
81. Consider the following reaction: C2H5OH + H2SO4 ¾¾ ® Product
Zn dust CH Cl
Phenol ¾¾¾¾
® X ¾¾¾ ¾¾¾¾
3
®Y Among the following, which one cannot be formed as a
Anhydrous AlCl3
product under any conditions ?
Alkaline KMnO
¾¾¾¾¾¾¾ 4®Z
(a) Ethylene (b) Acetylene
The product Z is:
(a) benzaldehyde (b) benzoic acid (c) Diethyl ether (d) Ethyl-hydrogen sulphate
(c) benzene (d) toluene 92. Arrange the following compounds in order of decreasing
82. Which one of the following compounds has the most acidic acidity :
nature? OH OH OH OH

CH2OH OH ; ; ; ;
(a) (b)

OH Cl CH3 NO2 OCH3


(I) (II) (III) (IV)
OH CH (a) II > IV > I > III (b) I > II > III > IV
(c) (d) (c) III > I > II > IV (d) IV > III > I > II
Alcohols, Phenols And Ethers 619

93. Isopropylbenzene is oxidized in the presence of air to


compound 'A'. When compound 'A' is treated with dilute (b) Br and CH3Br
mineral acid, the aromatic product formed is
(a) phenol (b) benzene
(c) benzaldehyde (d) acetophenone (c) Br and CH3OH
94. The main product of the following reaction is
conc.H SO
C6 H 5CH 2CH(OH)CH(CH 3 )2 ¾¾¾¾¾
2 4¾
®?
(d) OH and CH3Br
(a) H5C6 H
C=C
H CH(CH3)2 98. The major product of the following reaction is
(b) C6H5CH2 CH3
C=C
H CH3 RCH OH
¾¾¾¾¾¾®
Å
2
(c) H5C6CH2CH2 H (anhydrous)
C = CH 2
H 3C
(d) C6H5 CH(CH3)2 (a) a hemiacetal (b) an acetal
C=C (c) an ether (d) an ester
H H
99. Which one of the following compounds will be most readily
95. Phenol is heated with a solution of mixture of KBr and KBrO3.
The major product obtained in the above reaction is : attacked by an electrophile ?
(a) 2-Bromophenol (b) 3-Bromophenol (a) Chlorobenzene (b) Benzene
(c) 4-Bromophenol (d) 2, 4, 6-Tribromophenol (c) Phenol (d) Toluene
96. Which of the following reagents may be used to distinguish 100. Propene, CH 3CH = CH 2 can be converted into 1-propanol
between phenol and benzoic acid? by oxidation. Indicate which set of reagents amongst the
(a) Aqueous NaOH (b) Tollen’s reagent following is ideal to effect the above conversion ?
(c) Molisch reagent (d) Neutral FeCl3 (a) KMnO4 (alkaline)
97. In the reaction
HBr
the products are (b) Osmium tetraoxide (OsO4/CH2Cl2)
OCH3
(c) B2H6 and alk. H2O2
(d) O3/Zn
(a) OCH3and H2

1. Which one of the following reagents is used to reduce an 4. Which one of the following reaction provides a primary
aldehyde to primary school? alcohol?
(a) N2H4/KOH (a) Hydration of alkyne
(b) Zn/Hg and conc. HCl (b) Oxymercuration of alkene
(c) LiAlH4 (c) Oxidation hydroboration of alkene
(d) Alkaline CuSO4 containing Rochelle salt (d) Reaction of Grignard reagent with acetaldehyde
2. Reaction of CH — CH with RMgX leads to formation of 5. Propan-1-ol may be prepared by the reaction of propene with
2 2
O (a) H3BO3 (b) H2SO4/H2O
(a) RCHOHR (b) RCHOHCH3 O
R ||
CHCH2OH (c) B2H6,NaOH–H2O2 (d) CH 3 – C - OH
(c) RCH2CH2OH (d)
R 6. During dehydration of alcohols to alkenes by heating with
3. The ionization constant of phenol is higher than that of
conc. H2SO4 the initiation step is
ethanol because :
(a) formation of carbocation
(a) phenoxide ion is bulkier than ethoxide
(b) elimination of water
(b) phenoxide ion is stronger base than ethoxide
(c) formation of an ester
(c) phenoxide ion is stabilized through delocalization
(d) protonation of alcohol molecule
(d) phenoxide ion is less stable than ethoxide
EBD_7327
620 CHEMISTRY

7. When 2-methoxypropane is heated with HI, in the mole ratio The electrophile involved in the above reaction is
1 : 1, the major products formed are
(a) methanol and 2-iodopropane (a) trichloromethyl anion (CCl3)
(b) methyl iodide and 2-propanol Å
(b) formyl cation (CHO)
(c) methyl iodide and 2-iodopropane
Å
(d) methanol and 2-propanol (c) dichloromethyl cation (CHCl2)
8. Among the following compounds which can be dehydrated
(d) dichlorocarbene (: CCl2)
very easily is
13. NaNO / H SO
Phenol ¾¾ ¾2¾ ¾
2
¾
4
® B ¾¾2 ¾
H O
® C ¾¾ ¾® D
NaOH
CH3
| Name of the above reaction is
(a) CH3CH 2CCH 2CH3 (a) Liebermann’s reaction
|
OH (b) Phthalein fusion test
OH (c) Reimer-Tiemann reaction
|
(b) CH3CH 2CH 2 C HCH3 (d) Schotten-Baumann reaction
(c) CH3CH2 CH2 CH2 CH2 OH 14. Phenol can be converted to o-hydroxy-benzaldehyde by
(d) CH3CH 2 CHCH 2CH 2OH (a) Kolbe’s reaction (b) Reimer-Tiemann reaction
| (c) Wurtz reaction (d) Cannizaro reaction
CH3
conc. HI
9. p-cresol reacts with chloroform in alkaline medium to give 15. Y (mix) ¬¾¾¾¾ (CH 3 )3 C - O - CH 3
the compound A which adds hydrogen cyanide to form, the 2

compound B. The latter on acidic hydrolysis gives chiral Anhydrous HI


¾¾¾¾¾¾
® X (mix)
carboxylic acid. The structure of the carboxylic acid is 1
CH 3 CH 3 (a) X and Y are identical mixture of CH3I and (CH3)3C –
OH
CH 2 COOH
(a) (b) (b) X and Y are identical mixture of CH3OH & (CH3)3C–I
CH 2 COOH (c) X is mixture of CH3I and (CH3)3C – OH
OH OH (d) Y is mixture of CH3OH & (CH3)3C–I
CH 3 CH 3 16. What are the starting materials to get 2-methylpropene as
CH(OH)COOH the major product ?
(c) (d) (a) Sodium methoxide and sec-butyl bromide.
CH(OH)COOH (b) Sodium ethoxide and sec-butyl bromide.
OH (c) Sodium tert-butoxide and ethyl bromide.
OH
10. A compound ‘A’ having the molecular formula C5H12O, on (d) Sodium methoxide and tert-butyl bromide.
oxidation gives a compound ‘B’ with molecular formula 17. ClCH2CH2OH is stronger acid than CH3CH2OH because of:
C5H10O. Compound ‘B’ gave a 2,4-dinitrophenylhydrazine (a) – I effect of Cl increases negative charge on O atom of
derivative but did not answer haloform test or silver mirror alcohol
test. The structure of compound ‘A’ is (b) – I effect of Cl disperses negative charge on O atom to
(a) CH3 – CH2 – CH2 – CH2 – CH2– OH produce more stable cation
(b) CH 3 - CH 2 - CH 2 CH - CH 3 (c) – I effect of Cl disperses negative charge on O atom to
| produce more stable anion
OH (d) None of these
(c) CH3 - CH 2 - CHCH 2 - CH3 (i) Hg(OAc) /H O
18. 2 2 ® X + Na
CH3 - CH = CH 2 ¾¾¾¾¾¾¾¾
|
(ii) NaBH 4
OH
0° C
(d) CH3 - CH 2 - CH - CH 2 - OH ¾¾ ® Y + CH 3Cl ¾¾® Z + HI ¾¾¾ ® A+B
| What are A and B ?
CH3 (a) CH3 – CH2 – CH2 – OH & CH3I
11. HBr reacts with CH2 = CH – OCH3 under anhydrous (b) CH3 - CH - OH & CH3I
conditions at room temperature to give |
(a) BrCH2 – CH2 – OCH3 (b) H3C – CHBr – OCH3 CH3
(c) CH3CHO and CH3Br (d) BrCH2CHO and CH3OH (c) CH3 - CH - I & CH3OH
– +
OH O Na |
+ CHCl3 + NaOH CH3
12.
CHO (d) CH3 – CH2CH2I & CH3OH
Alcohols, Phenols And Ethers 621

19. How many iodides are produced in more than 5% yield? H O+


3 ®Y+Z
Me 22. (X) ¾¾¾¾ (Y and Z both give the Iodoform
(C5H10O)
test). The compound X is –
HI
OH ¾¾® (a) CH3 – CH = CH – O – CH2 – CH3
H
Et |
Ph (b) CH 3 - C - O - CH 2 - CH 3
(a) 5 (b) 2 |
(c) 3 (d) 1 CH 3
20. CH3CH - CH = CH 2 ¾¾¾¾®
(i) B H
2 6 H SO
2 4® Y .
X ¾¾¾¾ (c) CH3 - C - O - CH 2 - CH3
| (ii) H 2 O 2 /OH - 140° C ||
CH3 CH 2
What is Y ? (d) Both (a) and (c)
(a) CH3 - CH - CH 2 - CH 2 - O - CH 2 - CH 2 - CH - CH 3 DIRECTIONS for Qs. 23 to 25 : These are Assertion-Reason
| | type questions. Each of these questions contains two statements:
CH 3 CH 3 Statement-1 (Assertion) and Statement-2 (Reason). Answer these
(b) CH 3 - CH - CH = CH 2 questions from the following four options.
|
CH 3 (a) Statement-1 is true, Statement-2 is true, Statement-2 is a
correct explanation for Statement -1
(c) CH3 - CH - CH - O - CH - CH - CH3
(b) Statement-1 is true, Statement-2 is true ; Statement-2 is NOT
| | | |
CH3 CH3 CH3 CH3 a correct explanation for Statement-1
(c) Statement-1 is true, Statement-2 is false
CH3 CH3
| | (d) Statement-1 is false, Statement-2 is true
(d) CH3 - C - O - C - CH3 23. Statement-1 : A triester of glycerol and palmitic acid on
| | boiling with aqueous NaOH gives a solid cake having soapy
C 2 H5 C2 H5 touch
21. Compound 'A' of molecular formula C4H10O on treatment Statement-2 : Free glycerol is liberated which is a greasy
with Lucas reagent at room temperature gives compound solid
‘B’. When compound ‘B’ is heated with alcoholic KOH, it 24. Statement-1 : Phenol undergo Kolbe reaction, ethanol does
gives isobutene. Compound ‘A’ and ‘B’ are respectively not.
(a) 2-methyl-2-propanol and 2-methyl-2-chloropropane Statement-2 : Phenoxide ion is more basic than ethoxide ion.
(b) 2-methyl-1 -propanol and 1-chloro-2-methylpropane 25. Statement-1 : Lucas reagent is a mixture of anhydrous ZnCl2
(c) 2-methyl-1 -propanol and 2-methyl-2-chloropropane and concentrate HCl
Statement-2 : Primary alcohol produce ppt. with Lucas
(d) butan-2-ol and 2-chlorobutane
reagents.

Exemplar Questions (a) catalytic hydrogenation


1. Monochlorination of toluene in sunlight followed by (b) treatment with LiAlH4
hydrolysis with aq. NaOH yields (c) treatment with pyridinium chlorochromate
(a) o – cresol (b) m – cresol (d) treatment with KMnO4
(c) 2, 4 – dihydroxytoluene (d) benzyl alcohol 5. The process of converting alkyl halides into alcohols
2. How many alcohols with molecular formula C4H10O are chiral involves ............ .
in nature? (a) addition reaction (b) substitution reaction
(a) 1 (b) 2 (c) dehydrohalogenation (d) rearrangement reaction
(c) 3 (d) 4 6. Which of the following compounds is aromatic alcohol?
3. What is the correct order of reactivity of alcohols in the OH CH2OH CH2OH OH
following reaction?
ZnCl
R — OH + HCl ¾¾¾¾ 2 ® R – Cl + H O
2
(a) 1° > 2° > 3° (b) 1° < 2° < 3° CH3 CH3
(c) 3° > 2° > 1° (d) 3° > 1° > 2° (A) (B) (C) (D)
4. CH3CH2OH can be converted into CH3CHO by ........ . (a) A, B, C, D (b) A, D
(c) B, C (d) A
EBD_7327
622 CHEMISTRY

7. Give IUPAC name of the compound given below. (a) I < II < III (b) II < I < III
CH3 — CH — CH 2 —C H 2 — CH3 (c) II < III < I (d) III < II < I
| | | 16. Arrange the following compounds in increasing order of
Cl OH OH boiling point.
(a) 2-chloro-5-hydroxyhexane Propan - 1- ol, butan - 1 - ol, butan - 2 - ol, pentan - 1 - ol
(b) 2-hydroxy-5-chlorohexane (a) Propan-1-ol, butan-2-ol, butan-1-ol, pentan-1-ol
(c) 5-chlorohexan-2-ol (b) Propan-1-ol, butan-1-ol, butan-2-ol, pentan-1-ol
(d) 2-chlorohexan-5-ol (c) Pentan-1-ol, butan-2-ol, butan-1-ol, propan-1-ol
8. IUPAC name of m-cresol is ........... . (d) Pentan-1-ol, butan-1-ol, butan-2-ol, propan-1-ol
(a) 3-methylphenol (b) 3-chlorophenol
NEET/AIPMT (2013-2017) Questions
(c) 3-methoxyphenol (d) benzene-1,3-diol
17. Among the following ethers, which one will produce methyl
9. IUPAC name of the compound CH3 —CH — OCH3 is
| alcohol on treatment with hot concentrated HI?
CH3 [2013]
............... . (a) CH3 - CH 2 - CH - O - CH 3
(a) 1-methoxy-1methylethane |
CH 3
(b) 2-methoxy-2-methylethane
(c) 2-methoxypropane CH3
(d) isopropylmethyl ether |
10. Which of the following species can act as the strongest (b) CH3 - C- O - CH3
|
base? CH3
(a) – OH (b) – OR

(c) CH3 - CH- CH2 - O - CH3
– |
(c) OC 6H 5 (d) O CH3
NO2 (d) CH3 – CH2 – CH2 – CH2 – O – CH3
11. Which of the following compounds will react with sodium 18. Phenol is distilled with Zn dust followed by Friedel Crafts
hydroxide solution in water? alkylation with propyl chloride in the presence of AlCl3 to
(a) C6H5OH (b) C6H5CH2OH give a compound (B). (B) is oxidised in the presence of air to
(c) (CH3)3COH (d) C2H5OH form the compound (C). The structural formula of (C) is
[NEET Kar. 2013]
12. Phenol is less acidic than ............. .
(a) ethanol (b) o - nitrophenol COOH
(c) o-methylphenol (d) o-methoxyphenol
13. Which of the following is most acidic?
(a)
(a) Benzyl alcohol (b) Cyclohexanol
(c) Phenol (d) m-chlorophenol
14. Mark the correct order of decreasing acid strength of the COOH
following compounds. OH
OH OH OH OH (b)
OH

H3C
OCH3 C O O H
NO2 H3C
(c)
NO2 OCH3
(i) (ii) (iii) (iv) (v)
(a) V > IV > II > I > III (b) II > IV > I > III > V
CH3
(c) IV > V > III > II > I (d) V > IV > III > II > I
15. Mark the correct increasing order of reactivity of the H3C C O O H
following compounds with HBr/HCl. (d)
OH
CH2OH CH2OH CH2OH

19. Number of isomeric alcohols of molecular formula C6H14O


which give positive iodoform test is [NEET Kar. 2013]
NO2 Cl (a) two (b) three
(I) (II) (III) (c) four (d) five
Alcohols, Phenols And Ethers 623

20. Among the following sets of reactants which one produces


anisole? [2014]
(a) CH3CHO; RMgX (c) (d)
(b) C6H5OH; NaOH ; CH3I 27. Which of the following reagents would distinguish cis-
(c) C6H5OH; neutral FeCl3 cyclopenta-1,2-diol from the trans-isomer? [2016]
(d) C6H5–CH3; CH3COCl; AlCl3 (a) Acetone
(b) Ozone
21. Which of the following will not be soluble in sodium
(c) MnO2
hydrogen carbonate? [2014] (d) Aluminium isopropxide
(a) 2, 4, 6-trinitrophenol 28. The reaction
(b) Benzoic acid
(c) o-Nitrophenol NaOH Å Me–1 Me
OH O– Na O
(d) Benzenesulphonic acid
22. Identify Z in the sequence of reactions: [2014] Can be classified as :- [2016]
HBr/ H 2 O 2
CH 3 CH 2 CH=CH 2 ¾ ¾ ¾ ¾ ¾
C 2 H 5 ONa
® Y ¾¾ ¾ ¾® Z (a) Williamson ether synthesis reaction
(b) Alcohol formation reaction
(a) CH3—(CH2)3 —O—CH2CH3
(c) Dehydration reaction
(b) (CH3)2 CH2—O—CH2CH3 (d) Williamson alcohol synthesis reaction
(c) CH3(CH2)4—O—CH3 29. Which one is the most acidic compound ? [2017]
(d) CH3CH2—CH(CH3)—O—CH2CH3 OH
OH
23. The reaction
CH3 CH3 (a) (b)

CH3–C–ONa + CH3CH2Cl–—®CH3–C–O–CH2–CH3 NO2


–NaCl
CH3 OH OH
CH3
is called :- [2015] (c) O2N NO2 (d)
(a) Williamson continuous etherification process
(b) Etard reaction
NO2 CH3
(c) Gatterman - Koch reaction
(d) Williamson Synthesis 30. The heating of phenyl-methyl ethers with HI produces
24. Reaction of phenol with chloroform in presence of dilute (a) Iodobenzene (b) Phenol [2017]
sodium hydroxide finally introduces which one of the (c) Benzene (d) Ethyl chlorides
following functional group ? [2015 RS] 31. Identify A and predict the type of reaction [2017]
(a) –CH2Cl (b) –COOH OCH3
(c) –CHCl2 (d) –CHO
25. Which of the following reaction (s) can be used for the NaNH2
preparation of alkyl halides ? [2015 RS] A

(I) CH3CH2OH + HCl ¾anh.ZnCl


¾ ¾ ¾2¾ ® Br
OCH3
(II) CH3CH2OH + HCl ¾¾® NH2
(III) (CH3)3COH + HCl ¾¾ ® (a) and elimination addition reaction
(IV) (CH3)2CHOH + HCl ¾anh.ZnCl
¾ ¾ ¾2¾ ® OCH3
(a) (I), (III) and (IV) only Br
(b) (I) and (II) only (b) and cine substitution reaction
(c) (IV) only
(d) (III) and (IV) only OCH3
26. Which of the following is not the product of dehydration of
(c) and cine substitution reaction
OH ? [2015 RS]
OCH3

(d) and substitution reaction


(a) (b)
NH2
EBD_7327
624 CHEMISTRY

Hints & Solutions


EXERCISE - 1 – OH group is highly activating. This is a type of
electrophillic substitution reaction at ortho and para
1. (a) Lucas reagent is conc. HCl + anhyd. ZnCl2. position.
14. (b) In C2H5OH,
CH2OH COOH
| | O
2. (d) C HOH
Bismuth nitrate
¾¾¾¾¾¾ ® CO
[O] CH3 CH2
| | 104° H
CH2OH COOH
Glycerol Meso -oxalic acid Due to presence of lone pair of electrons on oxygen,
there occurs a small decrease in bond angle from the
3. (b) Methanol
normal tetrahedral bond angle (109°28¢)
Cr O - ZnO 15. (d) Water gas is mixed with half its volume of hydrogen.The
CO + 2H2 ¾¾¾¾¾¾
2 3 ® CH3OH
673K
water gas High pressure Methanol mixture is compressed to approximately 200 – 300
atmospheres. It is then passed over a catalyst
4. (b) C2 H5OH and CH3 - O - CH3 are isomers. [ZnO + Cr2O3] at 300°C. Methyl alcohol vapours are
5. (b) Example formed which are condensed
Al O ZnO+ Cr O
R - CH 2 - CH 2 - OH ¾¾¾¾
2 3 ® R - CH = CH
2 + H2 O CO + 2H 2 ¾¾¾¾¾®
2 3
CH3OH
300° C
D Water gas Methyl alcohol
6. (b) 5-10 % methanol and remaing ethanol is called
methylated spirit. It is also known as denatured alcohol H+
16. (a) R - CH 2 - OH ¾¾¾¾¾ ®
because it is unfit for drinking. Protonation
+
CH 2 R - CH 2 + H 2 O
OH
H2 C CH - OH carbonium ion
7. (a) or 17. (b) Greater the stability of the intermediate carbocation,
H2 C CH 2 more reactive is the alcohol. Since 2-methylpropan-2-ol
CH 2 generates 3° carbocation, therefore, it reacts fastest
with HBr.
8. (c)
9. (a) Phenol is more soluble in NaOH and forms salts. 18. (b) C2H5 MgBr + H2C – CH2
ONa O
OH
+ NaOH ® + H2O
H 2O
Phenol Sodium phenoxide
C2H 5 CH2 – CH 2 C2H 5 CH2CH2OH
n-butanol
10. (a) OMgBr
11. (b) Glycols are dihydric alcohols (having two hydroxyl 19. (c) Primary alcohol on oxidation give aldehyde which on
groups). Ethylene glycol is the first member of this series. further oxidation give carboxylic acid whereas
CH 2OH secondary alcohols give ketone.
|
CH 2 OH [O]
(Ethylene glycol) CH 3CH 2CH 2OH ¾¾¾
®
n - propyl alcohol
CH3
| [O]
5 4 3 2 1 CH 3 CH 2 CHO ¾¾®
¾ CH 3 CH 2 COOH
12. (b) CH3 - CH- CH 2 - C- CH3
| |
OH OH H3C [O] H3C
CH – OH C=O
2-methyl- 2, 4-pentanediol.

Br isopropyl alcohol Ketone

Br2 20. (c) Ethylene oxide when treated with Grignard Reagent
13. (c) HO SO3H ¾¾¾ ® HO SO3H
H2 O gives primary alcohol.
Br
Alcohols, Phenols And Ethers 625

CH 2 CH 2 – OMgX CH3 CH = CHCH3 + CH3CH2 CH = CH2


| O + R–MgX | + H 2O
CH 2 CH 2 – R (Major product) (Minor product)

X 29. (d) Glycerol is dehydrated by using dehydrating agent like


R – CH2 – CH 2 – OH + Mg P2O5 or conc H2SO4 or KHSO4 but KHSO4 is best of
OH them.
H
| 30. (d) Lucas reagent is anhydrous ZnCl2 and conc. HCl. It is
21. (b) Th e compounds containing CH 3 – C – OH or used to distinguish between 1°, 2° and 3° alcohols.
| 3° alcohols ® immediate turbidity
R
O 2° alcohols ® turbidity after 5 minutes
|| 1° alcohols ® No turbidity at room temp.
CH 3 - C - R (R = H, alkyl or aryl) give CHI3 with I2
and NaOH. CH3
22. (b) Conc. HCl, HBr and conc. HCl + ZnCl 2 all are |
CH 3 - C - OH is a tertiary alcohol hence it will give
nucleophiles, thus convert alcohols to alkyl halides.
|
However, conc. H3PO4 is a good dehydrating agent CH3
which converts an alcohol to an alkene. fastest reaction with Lucas reagent.
23. (b) NaBH4 and LiAlH4 attacks only carbonyl group and 31. (b) Phenol is more acidic than ethanol. Phenols can turn
reduce it into alcohol group. blue litmus paper red but ethanol can not do so.
32. (d) Solubility of alcohol in water decreases with increase
NaBH in molecular mass due to increase in water repelling
C6 H5 - CH = CHCHO ¾¾¾¾

cinnamic aldehyde alkyl part in alcohol.


CH 2 CH 2OH
C6 H 5 - CH = CH.CH 2 OH 33. (c) || + HOCl ¾¾ ® |
cinnamic alcohol CH 2 CH 2Cl
24. (d) Phenol has active (acidic) hydrogen so it reacts with CH 2 OH
NaHCO3
CH3MgI to give CH4, and not anisole ¾¾¾¾ ¾® |
OH - CH 2 - OH
C 6 H 5 OH + CH 3 MgI ¾
¾® CH 4 + C 6 H 5 OMgI
25. (b) CH3
|
26. (b) Since the compound is formed by hydration of an 34. (c) CH 3 CH 2 CHCH 2 OH ¾Conc
¾¾®
.
alkene, to get the structure of alkene remove a molecule 2,Methylbu-tanol H 2SO 4
of water from the alcohol.
CH 3 CH 3
-H O | + |
CH3 C H CH 3 ¾ ¾ 2¾® CH 2 = CHCH3 Rearrangement
| Propylene
CH 3 CH 2 CHCH 2 ¾¾¾ ¾¾¾ ↑ CH 3 – CH 2 – C – CH 3
OH 1º carbocatio n Å
Isopropyl alcohol (3º carbocation)

CH 2OH CH 2 I CH 2 CH 3
| | || |
3HI - I2 H I (excess) ¾
¾® CH 3CH = C - CH 3
27. (c) CHOH ¾¾¾ ® CHI ¾¾¾ ® CH ¾¾¾¾¾® 2 - Methylbut - 2 -ene
| | |
CH 2OH CH 2 I CH 2 I CH2OH 2HI CH3
35. (b) | ¾¾¾
® | + H2 + I2
COOH CH3
CH3 CH3 H H
| | CH3 | |
- I2 HI | 36. (a)
CH3MgBr
H - C = O ¾¾¾ ¾¾® H - C - OMgBr
CHI ¾¾¾ ® CH ¾¾¾ ® CHI
| || |
| CH 3
CH 2 I CH 2 CH3
H
\ All the compounds except (C), |
H 3O +
¾¾¾ ¾® H - C - OH
CH 2 OH - CHI - CH 2 OH are formed during reaction of
|
glycerol with excess HI. CH 3
-H O 37. (b) Iodoform test can differentiate between ethyl alcohol
28. (d) CH3CH 2 CHOH ¾¾¾
2 ¾
®
| and methyl alcohol. Ethyl alcohol contains CH3CO
CH3 group so it gives Iodoform test whereas methyl alcohol
does not give this test.
EBD_7327
626 CHEMISTRY

K Cr O
38. (c) CH3 - CH - CH3 ¾¾¾¾¾
2 2 7®
CH3 - C - CH3 O Na ONa
| H 2SO4 || CH(OH)2 CHO
OH O 2NaOH
(A) 2-propanol acetone –H2O
[O] dil.HCl
¾¾¾
® CH3COOH
acetic acid OH
Oxidation NH3
CHO
39. ® HCOOH ¾¾¾® HCOONH 4
(b) CH3OH¾¾¾¾¾
(A) (B)
PCl KCN
40. (b) C 2 H 5OH ¾¾¾
5
® CH3 CH 2 Cl ¾¾¾® Reimer-Tiemann reaction.
H 3O + 48. (b) Aldol condensation is carried out on aldehydes or
CH3CH 2 CN ¾¾¾® CH 3CH 2 COOH ketones having a-hydrogen atom in presence of base
propanoic acid
which abstracts a- H atom from aldehyde/ ketone to
41. (c) Because of larger (three per molecule) number of form corresponding carbanion which is stabilised by
intermolecular hydrogen bonding in case of glycerol resonance.
(CH2OH – CHOH – CH2OH ) as compared to ethanol O
||
(CH3CH2OH), the attraction between molecules of ˆˆ†
OH - + H - CH 2 - C - H ‡ˆˆ
glycerol is more than that of molecules of ethanol. Due
to this glycerol is more viscous than ethanol. O O-
|| |
H2O + [CH 2- - C - H ¬¾® CH 2 = C - H]
OH OH
COONa 49. (d) With Br2 water, phenol gives 2, 4, 6- tribromophenol.
42. (c) NaOH
OH
CO 2

+ 3Br2 (excess)
OH
hydrolysis
COOH
OH
Br Br
H2O
Salicylic acid +3HBr

43. (d) The intermediate is carbocation which is destabilised Br


2, 4, 6-Tribromophenol
by C = O group (present on a-carbon to the –OH
group) in the first three cases. In (d), a–hydrogen is CH2NH2 CH2OH

more acidic which can be removed as water. Moreover,


50. (d) + HNO2 + N2+ H2O
the positive charge on the intermediate carbocation is
relatively away from the C = O group. 51. (c) Alkyl halides can be prepared by treating alcohol with
44. (d) Among the given compounds only CH3OH does not SOCl2.
give iodoform reaction. CH 3 CH 2 OH + SOCl 2 ¾¾
®

45. (a) Due to H–bonding boiling point of C2H5OH is much CH3CH 2 Cl + SO2 + HCl
This reaction is known as Darzen's procedure.
higher than isomeric (CH3)2O.
Note : SOBr2 and SOI2 are not used in this reaction
OH ONa because SOBr2 is less stable and SOI2 does not exist.
52. (c) Perkin reaction involves reaction of aromatic aldehyde
46. (c) NaOH
with aliphatic acid anhydride in presence of sodium
salt of the same acid as anhydride to form a, b-
unsaturated acid.
ONa ONa
CHO COOH
C6 H5 CHO + ( CH3 CO ) 2 O ¾¾¾¾¾®
CHCl3 + NaOH (O) 3 CH COONa
¾¾¾¾¾¾® ¾¾¾
®
C6 H5CH = CHCOOH + CH3COOH
Salicylaldehyde
OH ONa Wurtz reaction involves the reaction of an alkyl halide
CHCl2 with metallic sodium in dry ether to form higher
47. (b) NaOH hydrocarbon.
+ CHCl3
Alcohols, Phenols And Ethers 627

dry However, in case of tertiary alkyl ether following


RX + 2Na + XR ¾¾¾® R - R + 2NaX reaction occurs.
ether

Cannizzaro reaction is given by aldehydes having no 373K


CH3 - OC(CH3 )3 +HI ¾¾¾ ↑
a-hydrogen atom to form a mixture of alcohol and ter -butyl methyl ether
sodium salt of the acid. It is a disproportionation reaction
(CH3)3C – I + CH3OH
in which half of the aldehyde molecules are oxidised
The alkyl halide is formed from the tertiary alkyl group
and other half reduced in presence of NaOH.
and the cleavage of such ethers occurs by S N 1
2C6 H5 CHO ¾¾¾¾
®
NaOH mechanism as the product is controlled by the formation
of more stable intermediate tertiary carbocation from
C6 H5 CH 2 OH + C6 H 5COONa protonated ether.
Claisen reaction is the self condensation of esters OH O–R
having a-hydrogen atom in presence of strong base to
form b-keto ester. 61. (c) + RX
Alkali
+ HX
C2H5OH
2CH3COOC2 H5 ¾¾¾¾¾ ®
C2 H5ONa 62. (b) Williamson’s synthesis -
O CH3 - CH 2 - ONa + Cl - CH 2 - CH3 ®
||
CH3CCH 2COOC2 H5 CH3 - CH 2 - O - CH 2 - CH3
OH 63. (b) Maximum enol content is in structure (b) because here
the enol form is stabilised by H-bonding.
D
53. (c) + Zn ¾¾ ® + ZnO O O O–H------O
|| || | ||
Phenol Benzene ˆˆ† CH3 – C = CH – C– CH3
CH3 – C– CH 2 – C– CH3 ‡ˆˆ
Phenol Benzene keto form Acetylacetone enol form
54. (d) The most suitable reagent for converting alcohol to
acetaldehyde is PCC. Other reagent will convert alcohol 64. (a) Dehydration of alcohols gives ethers
to acid. 65. (b) C2H5OH + HOOCCH3 —® C 2H 5COOCH3
+ H2O
O– O O ethyl acetate (ester)
– fruity smell
..

55. (c) etc. 66. (b) Reaction of sodium ethoxide with ethyl iodide to
produce diethyl ether is known as Williamson synthesis.
. .–
Resonance stabilisation of phenoxide ion
It is a nucleophilic substitution reaction and proceeds
(conjugate base of phenol) via SN2 mechanism.
67. (a) In Williamson’s synthesis the reaction of alkyl halides with
Conjugate base of ethyl alcohol, i.e., C2H5O– does not sodium alkoxides give ethers. However, if the alkyl halide is
show resonance. 3°, it undergoes elimination to give an alkene.

OH OH CH3
56. (a) | +–
H3C – C – Cl + NaOC2H5 D
COOH |
sod. cthoxide
Salicyclic acid Phenol CH3
(evolves CO 2 with NaHCO3) (No reaction with NaHCO3 )
tert-butyl chloride
57. (c) o-Nitrophenol has intramolecular H-bonding.
58. (b) The reaction is an example of electrophilic aromatic CH2
||
substitution. Although both OCH3 and CH3 groups CH3 – C – CH3 + C2H5OH + NaBr
are o,p-directing, the OCH3 group dominates. Product 2-Methyl-1-propene
(b) is favoured because the new coming group (Br)
experiences least hindrance. hv
68. (d) C2 H5 , O , C2 H5 ∗ O2 ¾¾¾

25↓C
59. (a) D
C 6 H 5 ONa + C 2 H 5 I ¾¾® C6 H 5OC 2 H 5 + NaI
Phenetole CH3 ,CH(OOH) , O , C2H5
60. (a) In case of unsymmetrical ethers, the site of cleavage 69. (c) CH3OCH3 and C2H5OCH3 are gases while C2H5OC2H5
depends on the nature of alkyl group. (b. p. 308 K) is low boiling liquid.
373K 70. (c) Carboxylic acids are stronger acids than phenols which
e.g. CH3 – O – C2H5 + HI ¾¾ ¾↑ CH3I + C2H5OH
in turn are stronger than alcohols. Out of benzyl alcohol
The alkyl halide is formed from the smaller alkyl group. and trifluoroethanol, the latter is more acidic due to the
EBD_7327
628 CHEMISTRY

presence of highly electronegative fluorine atoms. OH


Thus, order of acidity is:
CH2OH OH COOH Zn CH3Cl
81. (b)
anhy
AlCl3
< CF3CH2OH < < Phenol X
CH3 COOH
(IV) (I) (III) (II)
Red P / HI alc.
71. (a) C2 H5OC2 H5 ¾¾¾¾¾
↑ 2C2 H5I KMnO4
Cleavage of
ethers Y Z
Red P / HI
82. (b) Phenol is most acidic because its conjugate base is
¾¾ ¾¾¾
↑ 2C2 H6 stabilised due to resonance, while the rest three
Reduction
Ethane compounds are alcohols, hence, their corrosponding
72. (b) Only alkyl aryl ethers e.g., C6H5OCH3 undergoes conjugate bases do not exhibit resonance
electrophilic substitution reactions. OH OH

73. (c) 3↑ R
R , O , R ¾¾¾
BF
O ↑ BF3
R 83. (b) >
Etherate
NO2
74. (a) The ease of dehydration of alcohols is 3° > 2° > 1°.
NO2
Tertiary alcohols undergo dehydration more easily than
primary and secondary alcohols. (iv) (iii)
(– I and – M effects, (only – I effect)
D
75. (c) C2H5OC2H5 + 2 HI ¾¾® 2 C2H5I + H2O both increase acidity)
OH OH
76. (a) C6 H 6 O - + CH3I ® C6 H5OCH3 + I - CH3
77. (d) 1-pentanol > >
Primary alcohols readily form ether when heated with
conc. H2SO4. (i) (ii)
(+ I effect of CH3 group decreases acidity)
78. (c) In the cleavage of mixed ethers having two different 84. (b) Glycerol when treated with excess HI produces
alkyl groups, the alcohol and alkyl iodide that form 2–iodopropane
depend on the nature of alkyl group. When primary or CH2 OH CH2 I CH 2
secondary alkyl groups are present, it is the lower alkyl
-I
group that forms alkyl iodide therefore CHOH + 3HI ¾¾
® CHI ¾¾¾
2 ® CH

CH3 - CH - CH 2 - O - CH 2 - CH 3 + HI ¾¾®
D CH2 OH CH2 I CH 2 I
|
CH3 CH3 CH3 CH3
| | |
+HI +I2 +HI
CH3 ¾¾¾
® CHI ¾¾¾
® CH ¾¾¾
® CHI
| | || |
CH3 - CH - CH 2 OH + CH3CH 2 I CH2I CH2 CH3
85. (b) Glycol is used as an antifreeze in automobiles.
PBr alc.KOH
79. (c) CH3 CH2 OH ¾ ¾ ¾
3 ® CH CH Br ¾ ¾ ¾ ¾® CH = CH
P + I2 Mg
3 2 2 2 86. (d) CH3CH 2 OH ¾¾¾ ® CH3CH 2 I ¾¾¾®
Ether
H2SO4 A
HO
HO
CH3 CH 2 OH ¬¾22¾¾ CH3 – CH2 – HSO4 CH 2 CH 3
heat |
80. (c) 1, 2 – Diols, when treated with an aqueous solution of HCHO
CH3CH 2 MgI ¾¾¾¾ ® H - C - OMgI
periodic acid give aldehyde (B) |
CH2OH HIO4 H
| CH2O +CH2O (C)
CH2OH
Note that a 1° alcohol gives CH2O. Since in glycol both CH 2 CH3
|
the OH groups, are primary hence give 2 molecules of H2 O
¾¾¾ ® H - C - OH
CH2O as by product. |
H
(D)
n - propyl alcohol
Alcohols, Phenols And Ethers 629

87. (b) Phenol on reaction with conc. H2SO4 gives a mixture Electron withdrawing substituents increases the acidity
of o- and p- products (i.e., –SO3H group, occupies o-, of phenols; while electron releasing substituents
p- position). At room temperature o-product is more decreases acidity. Further the particular effect (acidity
stable, which on treatment with conc. HNO3 will yield increasing or decreasing) is more when a substituent is
o-nitrophenol. present in o-orp position to phenolic group. Thus the
correct order will be D > C > A > B
OH OH OH
SO3H 91. (b) C2 H5 - OH + H2SO4 433K CH2 = CH2
Conc.H2SO4
+ ethylene
413 K
CH3 – CH2 – O – CH 2 – CH3
o-product di ethyl ether
SO3H 383 K
p-product CH3 CH 2 HSO4 + H2 O
ethyl hydrogen sulphate
At room temperature o- product is more stable
Acetylene is not formed under any conditions.
OH OH 92. (c) Electron withdrawing substituents like –NO2 , Cl
SO3H NO2
Conc. HNO 3 increase the acidity of phenol while electron releasing
substituents like – CH3, – OCH3 decreases acidity.
hence the correct order of acidity will be
o- nitrophenol
OH OH OH OH
OH OH
NaOH
88. (b) CO2 > > >
COOH
(salicylic acid)
89. (b) Tertiary alcohols react fastest with conc. HCl and NO2 Cl CH3 OCH3
anhydrous ZnCl2 (lucas reagent) as its mechanism
III I II IV
proceeds through the formation of stable tertiary
(–M, –I) (–I > +M) (+I, + HC) (+ M)
carbocation.
Mechanism CH3 CH3
93. (a) CH3—C—H CH3 —C—OOH
CH3 O2
|
Step 1 : CH3 — C — OH + H - Cl Aerial oxidation
|
Isopropylbenzene cumene
CH3 (cumene) hydroperoxide
2-Methyl Propan-2-ol (A)
OH
+ O
ˆˆ† (CH3 )3 C — OH 2 + Cl-
‡ˆˆ ½½
dil.H2S O4
+ CH3 —C—CH3
+
Step 2 : ˆˆ† (CH3 )3 C + + H 2 O
(CH3 )3 C - OH 2 ‡ˆˆ phenol
3° Carbocation It is a commercial method for the manufacture of phenol.
ˆˆ† (CH 3 )3 C - Cl 94. (a) Whenever dehydration can produce two different
Step 3 : (CH3 )3 C+ + Cl- ‡ˆˆ
t -Butyl chloride alkenes, major product is formed according to Saytzeff
rule i.e. more substituted alkene (alkene having lesser
90. (a) number of hydrogen atoms on the two doubly bonded
carbon atoms) is the major product.
Such reactions which can produce two or more
OH OH OH OH
structural isomers but one of them in greater amount
than the other are called regioselective ; in case a
reaction is 100% regioselective, it is termed as
regiospecific.
NO2
In addition to being regioselective, alcohol
CH3 NO 2
dehydrations are stereoselective (a reaction in which
+I –I – M, – I a single starting material can yield two or more
stereoisomeric products, but gives one of them in
(A) (B) (C) (D) greater amount than any other).
EBD_7327
630 CHEMISTRY

Conc. H SO
C6 H 5 , CH 2 , CH, CH , CH3 ¾¾¾¾¾¾
2 4↑
3. (c) ˆˆ† C 6 H 5 O - + H 3O +
C6 H 5 OH + H 2 O ‡ˆˆ
| |
Phenoxide ion
OH CH3
The phenoxide ion is stable due to resonance.
H H H CH(CH3)2 –
O O O
C=C + C=C

C6H5 CH(CH3)2 C6H5 H ••
cis trans
(minor) (major)
(less stable due to bulky
••

(more stable)
groups lying on same side)
95. (d) 5KBr + KBrO3 + 3H2O ® 3Br2 + 6KOH
OH O– O
OH OH
Br Br
d

d

••

+ Br2 ®
d

The negative charge is delocalized in the benzene ring


Br which is a stabilizing factor in the phenoxide ion and
because of this reason ionization constant of phenol is
96. (d) Phenol gives a violet colour with neutral ferric chloride
higher whereas no resonance is possible in alkoxide
solution whereas benzoic acid does not give this test.
ions (RO–)derived from alcohol. The negative charge
H+ + Br – is localized on oxygen atom in case of alcohols.
97. (d) OCH3 O–CH3 (i) B H
4. (c) R - CH = CH 2 ¾¾¾¾¾¾¾
2 6
- ® R - CH 2 - CH 2 OH
(ii) H 2O 2 , OH
H (oxidative hydroboration
of alkene)
B H
OH + CH3Br 5. (c) CH3– CH=CH2 ¾ ¾2 ¾6 ® CH3–CH2–CH2–OH
NaOH / H 2O 2 Propanol
+ 6. (d) The dehydration of alcohol to form alkene occurs in
H
98. (b) following three step. Step (1) is initiation step.
.. +
O O O Step (1) Formation of protonated alcohol.
.. +
+ + H
+
CH3–CH2 – O –H + H CH3CH2 –– O
RCH OH
–H H
¾¾¾¾¾
2 ® (Protonated
+ ethanol)
O O CH2R O OCH2R
Step (2) Formation of carbocation
H an acetal H Slow
+ +
99. (c) Due to strong electron-donating effect of the OH group, CH3–CH2–– O ––––––®
H Ethyl
the electron density in phenol is much higher than that
carbocation
in toluene, benzene and chlorobenzene and hence phenol Step (3)Elimination of a proton to form ethene
is readily attacked by the electrophile.
+ Fast +
100. (c) KMnO4 (alkaline) and OsO4 / CH2 Cl2 are used for H– CH 2––– CH2 CH2 = CH 2+ H
hydroxylation of double bond while O3 /Zn is used for ethene
ozonolysis. Therefore, the right option is (c), i.e., CH3 CH3

3CH 3CH = CH 2 ¾¾¾¾¾®


BH3 in THF
(CH 3CH 2CH 2 )3 B 7. (b) CHOCH3 + HI CH3I + CHOH
CH3 CH3
3H O 2-propanol
¾¾¾¾
2 2®
3CH 3CH 2CH 2OH + H3BO3 8. (a) 3-methyl pentanol-3 will be dehydrated most readily
NaOH
1-propanol since it produces tertiary carbonium ion as intermediate.
CH3
EXERCISE - 2 |
CH3 - CH 2 - C - CH 2 - CH3
LiAlH |
1. (c) C = O ¾ ¾ ¾ ¾®
4 - CH 2 OH
primary alcohol
OH
2. (c) We know that CH3
|
H+
H2C – CH2+RMgX CH2 – CH2 ¾¾¾
® CH3 - CH 2 - C - CH 2 - CH3
Å
O CH3 CH3
OMgX R
CH – CH CHCl3 + NaOH
H 2O CH2 – CH2 9. (c)
HCN
Reimer Tiemman reaction C=O
–Mg(OH)X
OH R
OH OH H
Alcohols, Phenols And Ethers 631

CH3 CH3 17. (c) ClCH 2 CH 2 OH is stronger acid than CH3CH 2 OH due
OH HOH to – I effect of Cl.
C CH(OH).COOH
CN ® Cl -¬ CH 2CH 2O - + H +
Cl -¬ CH 2CH 2OH ¾¾
OH H OH Stronger acid - ve charge on O
dispersed hence
Cyanohydrin conjugate base, stable
10. (c) According to question
[O] 2,4-DNP ® CH3 ®- CH 2O - + H +
CH3CH 2 OH ¾¾
(A) (B) Derivative Weaker acid - ve charge intensified,
C5H12O C5H10O formed hence conjugate
Haloform test Ag mirror test base unstable

–ve –ve 18. (b) (i) Hg(OAc) /H O


2 2 ®
CH3 - CH = CH 2 ¾¾¾¾¾¾¾¾
Since (B) on reaction with 2,4-DNP forms a derivative, (ii) NaBH4
it implies that (B) has carbonyl ( C = O ) group (B) CH3 - CH - CH3
Na
gives –ve Tollen's test, hence it is not an aldehyde, it is | ¾¾↑ CH3 - CH - CH3
a ketone. OH |
(B) gives –ve haloform test, thus it is not a methyl ketone. (X) ONa
(B) is formed from the oxidation of (A), thus (A) is a 2º (Y)
alcohol, and among the given options, CH3Cl
CH3 - CH - CH3 ¬¾¾¾ ¾
(A) is : CH3 — CH 2 — CH — CH 2 — CH 3 |
| OCH3
OH (Z)
and \ (B) is CH3 — CH 2 — C — CH2 — CH3
|| HI
O
11. (b) Methyl vinyl ether under anhydrous condition at room CH3 - CH - CH3 + CH3I
temperature undergoes addition reaction. | (B)
HBr
OH
CH 2 = CH - OCH 3 ¾¾¾
® CH 3- CH - O - CH 3 (A)
|
Br 19. (b) Me Me [Two]
12. (d) This is Reimer-Tiemann reaction and the electrophile is
dichlorocarbene.
•• I + I
HCCl3 + NaOH Cl – C – Cl + NaCl + H 2O
dichlorocarbene Et Et
Ph Ph
13. (a)
OH ONa 20. (a) CH3CH - CH = CH 2
14. (b) CHCl2
|
NaOH CH3
+ CHCl3
–NaCl, –H 2O (i) B H
¾¾¾¾® 2 6 CH3 - CH - CH 2 - CH 2
–2NaCl 2NaOH (ii) H2 O2 /OH - | |
CH3 OH
ONa ONa
H 2SO 4 / 140° C
CHO –H2O
CH(OH)2
CH3 - CH - CH 2 - CH 2 - O - CH 2 - CH 2 - CH - CH3
unstable | |
OH CH3 CH3
CHO 21. (a) Reaction involved is given as :
dil. HCl
–NaCl
CH3 CH 3
o-hydroxybenzaldehyde ½ HCl ½
This reaction is called Reimer-Tiemann reaction. CH 3 —C—OH ¾¾¾® CH 3 —C—Cl
½ ZnCl 2 ½
15. (b) By path 1 (CH3)3C–I and CH3OH are formed and path 2 CH3 CH 3
also form the same. (A) (B)
16. (d) tert-butylbromide undergoes elimination readily in CH3
presence of the base sodium methoxide to form alc. KOH ½
2-methylpropene ¾¾¾® CH3 —C
CH3 CH2 ½½
|
CH3ONa
|| CH 2
CH3 - C - Br ¾¾¾¾® CH 3 -C Hence (a) is the correct option.
| - HBr |
CH3 CH3
EBD_7327
632 CHEMISTRY

22. (c) CH3 – C – O – CH2 – CH3 The reaction follow nucleophilic substitution reaction
in which — OH group is replaced by — Cl. In this
CH2 reaction carbocation is formed as intermediate. Higher
the stability of intermediate carbocation higher will be
CH3 – C – OH + HO – CH2 – CH3 the reactivity of reactant molecule. Since 3° carbocation
(Z) is more stable than 2° carbocation as well as 1°
CH2
carbocation.
Hence, the order of reactivity of alcohols is 3° > 2° > 1°.
CH3 – C – CH3 4. (c) Less powerful oxidizing agent, Pyridinium
O
(Y)
( -
chlorochromate C5 H 5 N HCl C
+
rO 3 oxidises primary )
alcohols to aldehydes.
23. (c)
24. (c) It is correct that sodium phenoxide (sodium salt of PCC
CH3CH 2 OH ¾¾¾
® CH3CHO
phenol) and CO2 on heating form sodium salicylate.
Ethanol Ethanal
This is known as Kolbe’s reaction. Ethanol does not
respond to this reaction. Therefore, statement-1 is true. 5. (b) The process of conversion of alkyl halides into
But the statement-2 that phenoxide ion is more basic alcohols involves substitution reaction.
than ethoxide ion is not correct. OH -
R — X ¾¾¾® R — OH
25. (c) Lucas reagent is a mixture of anhydrous ZnCl2 and
Alkyl halide Alcohol
conc. HCl. It is used for the distinction of monohydric
alcohol. Tertiary alcohols on addition produce a 6. (c) Compound (A) i.e., phenol and compound (D) i.e., a
precipitate immediately while secondary alcohols derivative of phenol cannot be considered as aromatic
produce ppt. after 5 minutes. Primary alcohols do not alcohol.
produce any precipitate. Therefore, statement-1 is true On the other hand. In compounds (B) and (C), — OH
but statement-2 is false. group is bonded to sp3 hybridised carbon which inturn
is bonded to benzene ring. Hence, are considered as
EXERCISE - 3 aromatic.
Exemplar Questions 6 5 4 3 2 1
7. (c) CH3 CH CH2 CH2 CH CH3
1. (d) CH3 CH2Cl CH2OH
Cl OH
Cl2 aq NaOH 5-chlorohexan-2-ol
¾¾¾¾ ® ¾¾¾¾¾ ®
sunlight 8. (a) The structure of m – cresol is
Toluene Benzyl chloride Benzyl alcohol CH3
2. (a) Following are the three possible isomers of butanol.
3
2
(i) CH3 CH 2 — CH 2 — CH 2 OH
1
Butan - 1 - ol OH
m-cresol
no chiral carbon. Here – OH is the functional group and the methyl is
* substituents. Hence the IUPAC name is 3-
(ii) CH3 CH2 CH CH3 methylphenol.

OH 9. (c) 3 2
H3C — CH — OCH3
Butan –2 – ol |
1-Chiral Carbon 1
CH3
(iii) CH3
IUPAC name of the above compund is 2-
methoxypropane.
H3C C CH3
10. (b) Weakest acid has the strongest conjugate base.
OH Among all these acids, ROH is the weakest acid.
Therefore, the strongest base is RO– .
2 – methylpropan – 2 – ol
11. (a) Phenol being more acidic in nature reacts with sodium
No Chiral Carbon hydroxide solution and by the loss of one proton, it
3. (c) HCl + An. ZnCl2 is known as lucas reagent. It is used give phenoxides ion. This phenoxide ion is resonance
to determine degree of an alcohol. stabilised.
Alcohols, Phenols And Ethers 633

OH ONa
CH3 CH3
| | H
HI
CH3 - C- O - CH3 1
CH3 - C- O
| SN | + CH 3
+NaOH +H2O CH3 CH 3
CH3 CH3
|+ |
12. (b) Presence of electron withdrawing group at ortho I–
CH3 - C ® CH3 - C- I
position increase theacidic strength. In o-nitrophenol, | |
nitro group is present at ortho position. On the other CH3 CH3
(Highly stable)
hand, in o-methylphenol and in o-methoxyphenol, 18. (c) OH
electron releasing group (—CH3, —OCH3) are present.
Presence of these groups at ortho & Para positions of AlCl3
Zn
phenol decreases the acidic strength of phenols. So,
CH3CH2CH2Cl
phenol is less acidic than o-nitrohenol.
13. (d) Presence of electron withdrawing group increases the CH3
acidic strength. So, m-chlorophenol is most acidic C O O H CH3 CH CH3
among all the given compounds. CH3
14. (b) Electron withdrawing substituents increase the acidic O2
strength of phenols. so, p-nitrophenol (II) and
m-nitrophenol (IV) are stronger acid than Phenol (I). If
— NO2 group is present at p-position, then it exerts 19. (c) Compound containing CH3CH(OH) or CH3CO–group
both — I and — R effect but if it is present at meta give positive iodoform test.
position, then it exerts only–I effect. Therefore, OH
p-nitrophenol is stronger than m-nitrophenol. |
On the other hand, electron releasing substituents CH3 CH— CH 2 CH 2 CH 2 CH 3
decreases the acidic strength of phenol. If — OCH3 OH
group is present at meta position, it will exert – I effect |
only. CH3 — CH — CH CH 2CH 3 ,
|
But, if it is present at para position, it will exert + R CH3
effect. Thus, m - methoxy phenol is more acidic than
p- methoxy phenol. Hence, the correct order of OH
|
decreasing acidic strength will be : II > IV > I > III > V. CH3 — CH — CH 2 — CH — CH3 ,
15. (c) Nucleophilic substitution reactions depends upon the |
stability of carbocation. As, presence of electron CH3
withdrawing group decreases the stability of
carbocation in compound (II) and (III). Therefore, will OH CH3
| |
give less stable carbocation than (I). CH3 — CH — C — CH 3
Now NO2 group is a stronger EWG than — Cl. |
CH3
+
Thus, NO2 — C6H5 — C H 2 will be less stable than 20. (b) Phenols react with alkyl halides in alkaline medium to
+ form ethers. Therefore,
Cl — C6H5 — C H 2
OH OCH3
Hence, the order of reactivity of carbocatiors will be:
n + (i) NaOH
+ +
O2 N C H 2 < Cl C H2 < C H2 (ii) CH3I

16. (a) With increase in molecular mass boiling point increases. 21. (c) o-nitrophenol will not be soluble in NaHCO3. Due to
Thus the b.p. of pentan-1-ol will be more than other intramolecular hydrogen bonding hydrogen on OH is
given compounds. Now, among isomeric alcohols 1° strongly bound. So it can not be have as an acid and
alcohols will have higher boiling points than 2° alcohols can not react with sodium bicarbonate.
due of higher surface area. HBr/H2O2
CH 3 CH 2 CH CH 2
Hence, increasing order of b.pts. will be (Peroxide effect)
Propan-1-ol < butan-2-ol < butan-1-ol < pentan-1-ol. 22. (a) CH3 CH2 CH2 CH2
NEET/AIPMT (2013-2017) Questions (Y)
Br
17. (b) The reaction will proceed via SN 1 mechanism because
alkyl group attached is 3°. CH3 (CH2)3 O CH2 CH3
(Z)
EBD_7327
634 CHEMISTRY

23. (d) Williamson synthesis is one of the best methods for


the preparation of symmetrical and unsymmetrical 26. (b) OH + HÅ ® O—H
Å
ethers. In this method, an alkyl halide is allowed to
H
react with sodium alkoxide.
d
-
¯ H
24. (d) CHCl3 + NaOH ƒ : CCl3 + H 2 O
d
Å
- d
- ¬
: CCl3 ¾¾
® : CCl2 + Cl H

¬
– –
O O Cl ¯
Cl C
– Cl + H O
+ :C 2 –
Cl – OH

– – 27. (a)
O O
28. (a) This is an example of Williamson ether synthesis reaction
C=O – CCl2
OH in which sodium alkoxide reacts with alkyl halide and
H – 2HCl H gives ether.
29. (c) Electron withdrawing – NO2 group has very strong –I
Therefore functional group – CHO is introduced. and –R effects so, compound 3 will be most acidic.
30. (b) When Ar – O – R ethers are reacted with HI, they are
25. (a) ZnCl2 is a lewis acid and interact with alcohol. cleaved at weaker O – R bond to give phenol and alkyl
iodide.
Å
CH 3 — CH 2OH + ZnCl 2 ® R — O— ZnCl 2 O – CH3 OH
|
H HI
(R = CH3 — CH 2 —) (I) + CH3I

Å 31. (d) OCH3 OCH3


Å , ΢
O
R — O— ZnCl2 ® R + [HOZnCl 2 ]
| H
H NH 2
(I)
Carbocation is formed as intermediate in the SN1 Br Br
mechanism which these reaction undergoes.
In the absence of ZnCl 2 formation of primary OCH3
carbocation is difficult which is the case with (ii) while
(i) undergoes reaction.
(iii) Tertiary carbocation casily formed due to the stability.
OH
| Benzyne
CH3 — C — CH3 ¾¾
® OCH3
|
CH3 NH 2
OCH3
Å a
CH3 — C — CH3 + H 2 O (Less stable)
| NH 2
CH3 OCH3 OCH3

(iv) In the presence of ZnCl2, 2° carbocation is formed b


H — NH 2
from (CH3 ) 2 — C— OH
|
H NH 2 NH 2
(More stable)
Å More stable as –ve charge is close to electron
i.e., CH3 — C H— CH3 withdrawing group.
Also, incoming nucleophile gets attached on same 'C'
on which 'Br' (Leaving group) was present.
\ not a cine substitution reaction
26
Aldehydes, Ketones
and Carboxylic acids
(A) ALDEHYDES AND KETONES (iii) From Hydrocarbons
Structure of carbonyl group (a) By ozonolysis of alkenes
Hybridisation state of carbon in carbonyl group is sp2 and sp2
orbital overlaps with p–orbital of oxygen and forms C–O, sp2–p,
s-bond. To prevent further oxidation of carbonyl compound
The carbonyl group is polar in nature. The p-bond cloud is by H2O2, we add zinc in the reaction to destroy
unsymmetrical because of the greater electronegativity of oxygen H2O2
(3.5) than the carbon (2.5). Zn + H2O2 ® ZnO + H2O
(b) By hydration of alkynes
PREPARATION OF ALDEHYDES AND KETONES
I. For Both Aldehydes and Ketones 333K, H2O
C C C=C
Dil. H2SO4 Tauto-
(i) By Oxidation of Alcohols +
H OH merises
HgSO4
H C C

H O
This is called Kucherov reaction.
Aldehydes are quite susceptible to further oxidation to Note:
acids. (i) Formaldehyde can't be prepared by this method
(ii) Only ethyne gives acetaldehyde while other
[O] [O]
RCH2OH ¾¾
¾® R–CHO ¾¾
¾® R–COOH alkynes give ketones.
Thus oxidation of primary alcohols is made at the (c) By Wacker’s process
temperature much above the B.P. of aldehyde and thus Reagent : Acidified aqueous solution of PdCl2 and CuCl2
aldehydes are vapourised out and prevented from being H
oxidised. PdCl2 + H 2 O
|
CH2=CH2 ¾¾¾¾¾® CH3 - C = O
Note : Collins reagent (CrO3. 2C5H5N) and pyridinium CuCl2
chlorochromate (C5H5NH+CrO3Cl–) are mild oxidising PdCl2 + H 2 O
agents thereby preventing the further oxidation of CH3–CH=CH2 ¾¾¾¾¾® CH3 - C - CH3
CuCl2 ||
aldehydes to carboxylic acids. O
(ii) By Dehydrogenation of Alcohols (iv) Oxidation of Nitroalkane (NEF Reaction)
Conc. H SO
2 4 ® RCHO
RCH2NO2 + NaOH ¾¾¾¾¾¾
O
||
Conc. H2SO4
R – CH – NO2 + NaOH ¾¾¾¾¾¾ ® R–C–R
|
R
EBD_7327
636 CHEMISTRY

(v) By Hydrolysis of Oximes: (b) By side-chain chlorination followed by hydrolysis


CH3– CH= NOH + H2O ¾¾® CH3CHO + NH2OH CH3 CHCl 2
Acetaldoxime
Cl2, hv
CH3 CH3 383 K
| |
CH3 - C = N - OH + H2O ¾¾® CH3 - C = O + NH2– OH
373 K H2O
Acetone oxime
(vi) By Dry Distillation of Calcium Salts of Fatty Acids CHO

CH3COO OOCH CH3 CHO


Ca + Ca ¾¾
® + + 2CaCO3
CH3COO OOCH CH3 CHO
Cal. acetate Cal. formate Acetaldehyde (c) By Gatterman-Koch reaction
CO + HCl ® HCOCl
Formyl chloride (unstable)
CH3COO OOCCH3 CH3COCH3
Ca + Ca ¾¾
® + + 2CaCO3
CH3COO OOCCH3 CH3COCH3
Calcium acetate (2 moles) Acetone (2 moles)

II. For Aldehydes Only


(i) From Acyl/Acid Chloride

(d) By Gattermann reaction


CHO
Note: Formaldehyde can not be prepared by this method
because, the corresponding acid chloride (HCOCl) is + HCN + HCl ¾¾¾¾
3
®
AlCl
unstable.
In the above reaction BaSO4 acts as catalyst poison and
decreases the catalytic efficiency of Pd, due to which III. For Ketones Only
further reduction of aldehyde to alcohol can’t take place. (i) From Acyl Chloride
(ii) From Nitriles and Esters 2R¢ C Cl + R2Cd
SnCl2 / HCl
R – C º N + 2H ¾¾¾¾¾
® R – CH = NH.HCl O
H O
2 ® 2R¢ C R + CdCl2
¾¾¾ RCHO + NH4Cl
(i) AlH(iBu)2 or DIBAL – H O
RCN RCHO Dialkyl cadmium can be obtained by
(ii) H2O
2R– Mg – Cl + CdCl2 ¾¾® R – Cd – R + 2MgCl2
This method is very important for synthesis of Ketones.
(ii) From Nitriles

(iii) From Hydrocarbons


(a) By oxidation of methyl benezene
CH3 CHO

(i) CrO2Cl2 + CS2


+
(ii) H3O
(Etard reaction) Note: If instead of alkyl or aryl nitriles, HCN is used
aldehydes are formed.
CH3 CHO Dry
HCN + R – MgI ¾¾¾
ether
®[R – CH = NMgI]
(i) CrO3, (CH3CO)2O H O+
3 ® R – CHO + Mg(OH)I
+ ¾¾¾¾
– NH ,
(ii) H3O , D 3
Aldehydes, Ketones and Carboxylic Acids 637

(iii) By Friedal-Crafts Acylation Reaction Boiling point of aldehydes and ketones are higher than those
of hydrocarbons and ethers. This is due to dipole-dipole
interactions between the opposite ends of C = O dipoles.
Ketones have higher Boiling point than isomeric aldehydes.
This is due to presence of two electrons donating alkyl groups
around C = O group which makes them more polar.
Boiling point of alcohols and carboxylic acids are higher than
Note: Oppenauer oxidation: aldehydes and ketones. This is due to intermolecular
CH3 H-bonding in alcohols and acids which is much stronger
R
[(CH ) CO] Al than dipole-dipole interactions in aldehydes and ketones.
CHOH + C = O ¾¾¾3 ¾
3 ¾3¾®

R' CH3 (iv) Solubility :


Lower aldehydes and ketones are soluble in water due to
2° Alcohol Acetone
H-bonding between the polar carbonyl group and the water
R CH3 molecules. Solubility in H2O decreases with increase in size
C=O + CH – OH of alkyl group. Due to larger hydrocarbon part, aromatic
R' CH3 aldehydes and ketones are much less soluble than aliphatic
analogues. All aldehydes and ketones are fairly soluble in
Ketone 2° Alcohol organic solvents.
PHYSICAL PROPERTIES OF ALDEHYDES AND
KETONES
CHEMICAL PROPERTIES OF ALDEHYDES AND
(i) Physical State KETONES
Formaldehyde is a gas. All other aldehydes and ketones upto The reaction of aldehydes and ketones can be divided into
C11 are colourless volatile liquids. Higher members are solids the following categories :
at room temperature.
1. Nucleophilic addition reactions.
(ii) Odour
2. Reduction reactions.
Lower aldehydes have an unpleasant odour. Higher
3. Oxidation reactions
aldehydes and ketones have a pleasant odour.
4. Reactions with alkalies.
(iii) Boiling and melting points:
5. Miscellaneous reactions.
Boiling point and melting point µ Molecular weight µ
1
Branching

1. Nucleophilic Addition Reactions


Mechanism of nucleophilic addition to carbonyl group.


Nu
Nu Nu
a + – +
d d Slow H , Fast
C=O C Step II C
Step I –
b O OH
a a
b b

Aldehydes are more reactive than ketones due to:


(i) Inductive effect: Alkyl group has + I effect which decreases polarity of carbonyl group. Thus ketones containing two alkyl groups
are more reactive in comparison to one alkyl group containing aldehydes.
(ii) Steric effects: As the number and size of alkyl groups increases the attack of the nucleophile on carbonyl group C becomes more
and more difficult due to steric hindrance.
EBD_7327
638 CHEMISTRY

The various nucleophilic addition reactions are:


– + OH
HCN O H
C C
CN
CN
Cyanohydrin
– +
NaHSO3 OSO2H Proton transfer SO 3 Na
C – +
C
O Na OH
Bisulphite compound
C=O

RMgX OMgX + OH
H
C C
R H2O R
alcohol

R¢ OH OR¢ R¢OH
C C + H2 O
Dry HCl OH Dry. HCl OR¢
Hemiacetal acetal / ketal

H2N – Z OH +
H
C C = N – Z + H2 O
NHZ
(for Z ® see table)
Some ammonia-substituted derivatives of aldehydes and ketones

Note: (i) Benzaldehyde forms NaHSO3 adduct but acetophenone does not.
(ii)In addition of alcohols if dihydric alcohol is used a cyclic acetal or ketal is formed.
(iii)In addition of NH3 derivatives resulting compounds formed are crystalline solids with sharp melting points. Thus these
derivatives are used for identification and characterization of aldehydes and ketones.
Aldehydes, Ketones and Carboxylic Acids 639

2. Reduction Reactions
Reduction to

Alcohols Hydrocarbons

H2 / Pd or Ni or Pt
RCHO RCH2OH
or
LiAlH4 or NaBH4 (1°) Clemmensen Wolff-Kishner
reduction reduction
LiAlH4 or NaBH4 C=O
RCOR¢ R CH R¢ Zn – Hg C=O NH2NH2
or
HCl (–H2O)
H2 / Ni or Pt or Pd
OH
CH2 C = NNH2
(2°)
KOH
(ethylene glycol)
CH2

Note: Aldehydes and ketones can be also reduced to Note: Aldehydes and ketones having one methyl group linked
corresponding hydrocarbons by heating with HI in presence to carbonyl C-atom, i.e., methyl ketones, show haloform
of red P at 423K. reaction
3. Oxidation Reactions O O
Oxidation of Aldehydes NaOX
i.e., R C CH R C ONa + CHX3
3

Aldehydes get easily oxidised by strong oxidising agents R – COCH 3 ¾¾¾ NaOI
® RCOO – Na + + CHI3
like HNO3, K2Cr2O7, KMnO4 etc. as well as by weak This is Iodoform test and is used for characterising compounds
oxidising agents like Fehling’s and Tollen’s reagents. with CH3CO group or CH3CH (OH) group.
Oxidation of Ketones 4. Reactions with Alkalies
Strong oxidising agent and elevated temperatures needed. Due to strong electron withdrawing nature (–I effect) of the
In case of symmetrical ketones mixture of two acids is always carbonyl group and resonance stabilisation of the conjugate
obtained. base, a-H-atoms of carbonyl compounds are acidic in nature.
R – CH 2COCH 2 – R ¾¾® [O]
RCH 2COOH + RCOOH This acidic nature of a-H gives rise to a large number of
In case of unsymmetrical ketones (R ¹ R'), keto group stays reactions of carbonyl compounds.
with smaller alkyl group (Popoff’s rule). (i) Aldol Condensation
Two molecules of aldehyde or ketone containing atleast one
a-H atom condense in presence of a dilute alkali to form a b-
hydroxyaldehyde or a b-hydroxyketone respectively.

For example:
+
dil. NaOH D, H
2CH3CHO CH3 CH CH2 CHO CH3 CH = CH CHO
– H2O
But-2-enal (Crotonaldehyde)
OH
3-Hydroxybutanal
(Aldol)
CH3 CH3
+
Ba(OH)2 D, H
2CH3COCH3 CH3 C CH2COCH3 CH3 C = CHCOCH3
– H2O
(Mesityl oxide)
OH
EBD_7327
640 CHEMISTRY

(ii) Cross-aldol condensation: An aldol condensation between two different aldehydes or two ketones or between one aldehyde and
one ketone.
For example,
OH OH
| |
CH3 - CHCH 2 CH 2 CHO + CH 3CH 2 CH - CH 2 - CHO
From 2 molecules of From 2 molecules of
ethanol Propanal
dil
CH3CHO + CH3CH 2 CHO ¾¾¾®
NaOH
OH CH3 OH
| | |
CH3 - CH - CHCHO + CH3CH 2 - CH - CH 2 CHO
(Ethanal as a carbonyl group (Propanal as a carbonyl group and
and propanal as a nucleophile) ethanal as a nucleophile)
O

OH
CHO + C CH3 CH = CH C + H2O
293 K
O
Note: Cross aldol condensation between an aromatic aldehyde and an aliphatic aldehyde or a ketone is called Claisen-Schmidt
condensation.
(iii) Cannizaro reaction: Those aldehydes, which do not contain a–hydrogen atoms give this reaction with conc. alkali like conc.
NaOH or KOH. In this reaction two molecules of aldehyde react in such a way that one molecule gets oxidised and the other gets
reduced. This type of reaction is called disproportionation reaction.
When two molecules in a reaction are same it is called simple Cannizzaro reaction. In case of different molecules, it is a mixed cross
Cannizzaro reaction. In mixed cross Cannizzaro reaction
More reactive aldehyde ¾® Gets oxidised
Less reactive aldehyde ¾® Gets reduced
H
H O
H Conc.
For example, C=O+ C=O H C OH + H C
H H KOH
OK
H

Conc. NaOH
2 CHO CH 2OH + COONa

Conc. NaOH
HCHO + C6H5CHO ¾¾¾ ¾¾® HCOONa + C6H5CH2OH
(Oxidation) (Reduction)
Miscellaneous Reactions
(i) Electrophilic substitution reactions: Carbonyl group is a deactivating and meta-directing group.
(a) Nitration :
O2N
HNO3 / H2SO4
CHO CHO
273 – 283 K
(b) Sulphonation :
CHO CHO

+ HO – SO3H ¾¾
®
SO3H
(ii) Benzoin condensation :
H O H O
| || | ||
Alcoholic
C6H5 – C + C – C6H5 ¾¾¾¾® C6H5 – C – C – C6H5
|| | KCN |
O H OH
Benzoin
Aldehydes, Ketones and Carboxylic Acids 641

(iii) Cold dilute Schiff's reagent (colour less) + Aldehyde ¾¾® Pink coloured solution
(iv) With ammonia :
CH3 O CH 3 O
| || | ||
D
CH3 – C = O + H + H– CH2 – C – CH3 ¾¾ ® CH3 – C - CH2 – C – CH3
- H2O
| |
NH 2 NH 2
Diacetoneamine

H
C6H5 – C = O N–H
H H
H C6H5 – CH = N
+ + O = CH – C6H5 C – C6H5
C6H5 – CH = N
H
C6H5 – C = O N–H
H
H
Hydrobenzamide (amide without – CONH2 group)

DISTINCTION BETWEEN ALDEHYDES AND KETONES

Test Aldehydes Ketones

1. With Tollen’s reagent give silver mirror No action


+ –
RCHO + 2 [Ag(NH3)2] + 3OH


RCOO + 2Ag + 2H 2O + 4NH3

2. With Fehling’s solution give red ppt of Cu2O No action


(only aliphatic aldehydes)
3. Reduction with LiAlH4 1° alcohols are formed 2° alcohols are formed

4. Action with alcohols in Form acetals easily


presence of dry HCl gas Do not form ketales easily

5. Action with ammonia Give aldehyde- Form complex


ammonia adducts condensation products
6. With Schiff’s reagent Restore pink colour No action

(B) CARBOXYLIC ACIDS


PREPARATION OF CARBOXYLIC ACIDS
(i) From 1° Alcohols, Aldehydes and Ketones.

O O
|| Acidic K 2 Cr2O7 ||
R - C - CH 2 - R ' + [O] ¾¾¾¾¾¾¾ ® R - C - OH + R' – COOH
D

This gives poor yield of carboxylic acids.


EBD_7327
642 CHEMISTRY

(ii) From alkylbenzenes. This is due to extensive H-bonding resulting in association


of carboxylic acid molecules. The H-bonds formed in
carboxylic acids are even more stronger than alcohols.
Carboxylic acids exist as cyclic dimer in vapour phase or in
aprotic solvents.
(iii) Solubility: Simple aliphatic acids (upto 4 C-atoms) are soluble
in H2O due to H-bonding. Higher acids are insoluble in H2O
due to increased hydrophobic interaction of hydrocarbon
part. Aromatic acids are also insoluble due to their large
hydrocarbon part. Carboxylic acids are soluble in organic
solvents like C6H6, ether, CHCl3, etc.
Note : Presence of electron withdrawing groups in benzene ring
increases the reactivity of the benzylic hydrogens. Thus CHEMICAL PROPERTIES OF CARBOXYLIC ACIDS
p-nitrotoluene is much more easily oxidised than toluene. I. Reactionsd Due to Cleavage of O–H Bond:
(iii) From Nitriles and Amides. Acid strength of acids
(i) ® 2RCOO - Na + + H 2
2RCOOH + 2Na ¾¾
® RCOO - Na + + H 2O
(ii) RCOOH + NaOH ¾¾
(iii) RCOOH + NaHCO3 ¾¾ ® RCOO– Na + + H2O + CO2
Note: Acids liberate CO2 (gas) with NaHCO3. This reaction
(iv) From Grignard Reagent. is used to distinguish carboxylic acids from phenols which
Dry ether O do not react with NaHCO3.
RMgX + O = C = O R C
–Mg(OH)X OMgX (iv) 2RCOOH + Ca(OH) 2 ® (RCOO) 2 Ca + 2H 2O
+
H3O In water, carboxylic acids dissociate as :
RCOOH O
(v) From Hydrolysis of Acid Derivatives: +
R C + H2O H3O + RCOO–
O O
|| || OH
Ar/R – C – Z + H – OH ——® Ar/R – C – OH + HZ é H O + ù é RCOO - ù
ê 3 úû êë úû
K eq = ë
(a) [ H 2O ][ RCOOH ]
éH O + ù é RCOO - ù
êë 3 úû êë úû
Þ K a = K eq [ H 2 O] =
[ RCOOH ]
and pka = – log Ka
(b) \ smaller the pka, stronger the acid.
(c) The acidity of carboxylic acids, phenols and alcohols follows
the order:
(d) R – CONH 2 + HOH ¾¾¾¾
dil.HCl
® RCOOH + NH 4Cl Carboxylic acid > phenol > alcohol
Consider the resonating structures of carboxylate ion and
PHYSICAL PROPERTIES OF CARBOXYLIC ACIDS phenoxide ion.
.. –
(i) Melting point (M.P.) : Melting point of carboxylic acid do not O O
..
..

C ..– C
vary smoothly from one molecule to another O
.. O
..
..

..

(I) (II)
For first ten members .. .. .. .. ..
O O– O O O
M.P. of C2n > M.P. of C2n+1
..
..

..
..

..

..

..

(n = 1, 2, 3, 4, 5) (n = 0, 1, 2, 3, 4) – –
..

..

Reason : In acids of even number of carbon atoms, terminal


..–
– CH3 and – COOH. Groups lie on the opposite sides of the (III) (IV) (V) (VII)
(VI)
carbon chain which results in more effective packing of the
This is because the carboxylate ion is stabilised by two
molecules in the lattice. While in acids of odd number of
equivalent resonance structures with –ve charge at more
carbon atoms they lie on the same sides on the carbon chain. electronegative O-atom. On the other hand, phenoxide ion
(ii) Boiling points: Boiling point µ Molecular weight has non-equivalent structures with –ve charge on less
The boiling points of carboxylic acids are higher than electronegative C-atom. Thus, contribution of resonating
hydrocarbons, aldehydes, ketones and even alcohols of structures of phenoxide ion towards resonance is very small.
comparable molecular masses.
Aldehydes, Ketones and Carboxylic Acids 643

Therefore, carboxylate ion is more stable than phenoxide ion so carboxylic acids are more acidic than phenols.
EWG increase the acidity of acid by stabilising the carboxylate ion whereas EDG decrease the acidity by destabilising the ion.
Note:
(i) More will be the electron withdrawing nature of substituent greater will be acidic strength.
(ii) More will be the number of electron withdrawing substituents greater will be acidic strength.
(iii) As the distance between electron withdrawing substituent and – COOH group increases acidic strength decreases.
(iv) Direct attachment of phenyl or vinyl groups increase the acidity of acid due to resonance:

In general, acidity increases as the electronegativity of the carbon atom directly attached to –COOH group increases or the
hybridization of the carbon atom directly attached to –COOH changes from sp3 ® sp2 ® sp.
The order of acidity of various acids is as follows:
CF3COOH > CCl3COOH > CHCl 2COOH > NO2CH 2 COOH > NC - CH 2COOH > FCH 2COOH > ClCH 2 COOH
> BrCH 2COOH > HCOOH > ClCH 2CH 2COOH > C6 H5COOH > C6 H5CH 2COOH > CH3COOH > CH3CH 2COOH
Note: o–Substituted benzoic acids are usually stronger acids than benzoic acid regardless of the nature of the substituent.
II. Cleavage of C – OH Bond
(a) Reduction to alkanes:
(i) Anhydride Formation

O O
R C + C R¢
OH HO
Conc. H2SO4, D or P2O5
D NaBH4 does not reduce the carboxyl group B2H6 does
OO
R C C R¢ not reduce groups like ester, – NO2, halo, etc.
O (b) Reduction to alkanes:
Red P
R – COOH + 6HI ¾¾¾
473K
® R – CH 3 + 2H 2O + 3I 2
Acid anhydrides can also be obtained by treating acid
chlorides with carboxylic acids. (ii) Decarboxylation
(ii) Esterification
Dry HCl or
RCOOH + R ¢OH RCOOR ¢ + H 2O
Conc. H2SO4
Note: As the number and size of the substituents around the
–COOH or –OH group increases, the rate of esterification (iii) Kolbe’s electrolytic synthesis :
decreases. 2RCOOK Þ 2RCOO– + 2K+
(iii) Formation of Acid Chlorides. At Anode : 2RCOO– ¾®R – R + CO2 ­
Reaction with PCl5, PCl3 and SOCl2 At Cathode : 2K+ + HOH ¾® 2KOH + H2 ­
RCOOH + PCl5 RCOCl + POCl3 + HCl (iv) Hunsdiecker reaction :
RCOOH + PCl3 3RCOCl + H3PO3 O
P CCl 4
RCOOH + SOCl2 RCOCl + SO 2 + HCl R - C - OAg + X 2 ¾¾¾ ® R–X + CO2 ­ + AgX ¯
Note: In this reaction, SO2 being a gas escapes out and HCl (X = Cl or Br)
gets absorbed by basic pyridine and pure acyl chloride is (v) Formation of carbonyl compounds [Dry Distil-
left. So it is best method for preparing acyl chloride from acid. lation of calcium salt] :
(iv) Reaction with NH3 O
||
D ( - H 2O )
ˆˆ† RCOO - NH+4 ¾¾¾¾¾
RCOOH + NH3 ‡ˆˆ ® R–C–O
D R
RCONH2 Ca ¾¾ ® C = O + CaCO3
R–C–O R
III. Reaction Involving — COOH Group
||
(i) Reduction: O
EBD_7327
644 CHEMISTRY

IV. Substitution in Hydrocarbon Part (ii) Ring Substitution :


(i) Halogenation
R CH2COOH
(i) X2, Red P
(ii) H2O

R CH COOH

X
a - Halocarboxylic acid (X = Cl, Br)
Note: – COOH group is m-directing. Aromatic acids do not
This is Hell-Volhard-Zelinsky (HVZ) reaction undergo Friedal Crafts reaction.
Acids which do not have a–hydrogens do not show HVZ
reaction

DISTINCTION BETWEEN PHENOL AND CARBOXYLIC ACID

Test Carboxylic acid Phenol


1. Sodium bicarbonate R – COOH with NaHCO3, give brisk No reaction
test effervescence due to evolution of CO2 gas
2. Ferric chloride test R – COOH give coloured ppt with neutral C6H5OH give violet coloured ppt. with FeCl3
FeCl3 sol solution
Ex: 3C6H5OH + FeCl3 (C6H5O)3Fe + 3HCl
3CH3COOH + FeCl3 (CH3COO)3Fe + 3HCl ferric phenoxide
Buff coloured (violet coloured)
Aldehydes, Ketones and Carboxylic Acids 645

CONCEPT MAP
EBD_7327
646 CHEMISTRY

1. Benzaldehyde and alcohol KCN reacts to produce which of 9. An unknown compound 'D' first oxidised to aldehyde and
the following : then acetic acid by a dilute solution of K2Cr2O7 and H2SO4.
(a) ethyl benzoate (b) benzyl alcohol The compound 'D' is :
(c) benzoin (d) benzyl benzoate (a) CH3OH (b) C2H5OH
2. Reaction of phenylacetylene with dil. H2SO4 and HgSO4 (c) CH3CH2COOH (d) CH3CH2CHO
gives 10. Identify the product C in the series
(a) acetophenone (b) 2-phenylethanol Na / C H OH
2 5 HNO Cu / 573K
2 ® B ¾¾¾¾¾
CH 3CN ¾¾¾¾¾¾ ® A ¾¾¾¾ ®C
(c) phenylacetaldehyde (d) phenylacetic acid
3. A new C — C bond is formed in : (a) CH3COOH (b) CH3CH2NHOH
(c) CH3CONH2 (d) CH3CHO
(a) Cannizzaro’s reaction
11. Benzaldehyde can be prepared by oxidation of toluene by
(b) Reimer Tiemann reaction
(a) Acidic KMnO4 (b) K2Cr2O7 / H+
(c) Friedel Crafts reaction
(c) CrO2Cl2 (d) All of these
(d) both (b) and (c)
12. Formalin is an aqueous solution of
4. Two molecules of acetic acid on heating with phosphorous (a) fluorescein (b) formic acid
pentoxide produce : (c) formaldehyde (d) furfuraldehyde
(a) ethyl alcohol 13. Th e reagent (s) which can be used to distin guish
(b) acetic anhydride acetophenone from benzophenone is (are)
(c) acetyl pentaoxide (a) 2,4- dinitrophenylhydrazine
(d) pentaoxide of acetic acid (b) aqueous solution of NaHSO3
5. Which of the following gases does not form a ketone on (c) benedict reagent
treatment with dil H 2SO 4 and 1% HgSO4 ? (d) I2and Na2CO3.
14. The test used for the distinction of HCOOH and CH3COOH
(a) C4 H6 (b) C 2 H 2
is treatment with :
(c) C 3H 4 (d) All of these (a) sodium (b) sodium hydroxide
6. A compound X of formula C3 H 8 O yields a ketone C3 H 6 O (c) Tollen's reagent (d) alcohol
on oxidation. To which of the following class of compounds 15. Sodium formate on heating yields
would X belong? (a) oxalic acid and H2 (b) sodium oxalate and H2
(a) Aldehyde (b) Tertiary alcohol (c) CO2 and NaOH (d) sodium oxalate.
(c) Secondary alcohol (d) Alkene 16. (CH3)2 C = CHCOCH3 can be oxidized to
7. Clemmensen's reduction of ketones is carried out in : (CH3)2C = CHCOOH by
(a) LiAlH4 in H2O (b) glycol and KOH (a) Chromic acid (b) NaOI
(c) Zn-Hg and HCl (d) H2 and Pd catalyst (c) Cu at 300°C (d) KMnO4.
8. Suggest appropriate structures for the missing final 17. Pinacolone is
compound. (The number of carbon atom remains the same (a) 2, 3-dimethyl-2 3-butanediol
throughout the reaction) (b) 3, 3-dimethyl-2 butanone
(c) 1-phenyl-2propanone
CH3 (d) 1,1-diphenyl-2-ethandiol.
dil. KMnO HIO OH- 18. Urotropine is obtained when ammonia reacts with
¾¾¾¾¾
4 ® A ¾¾¾®
4 B ¾¾¾® C (a) HCHO (b) CH3COCH3
CH3 (c) CH3CHO (d) C6H5OH
19. Benzaldehyde reacts with ethanoic KCN to give
O (a) C6H5CHOHCN
CHO (b) C6H5CHOHCOC6H5
(c) C6H5CHOHCOOH
(a) (b)
(d) C6H5CHOHCHOHC6H5
CH3
CH3
20. Aldehydes and ketones will not form crystalline derivatives
with
CHO (a) sodium bisulphite
CHO
(c) (d) (b) phenylhydrazine
(c) semicarbazide hydrochloride
CHO (d) dihydrogen sodium phosphate.
Aldehydes, Ketones and Carboxylic Acids 647

21. When formaldehyde is polymerised in the presence of dilute


CHO
H2SO4, the compound obtained is
(a) paraldehyde (b) paraformaldehyde Dil. NaOH
(c) metaldehyde (d) trioxy formaldehyde 30. + CH3CHO ¾¾¾¾¾
® A . (A) will be –
(Major)
22. (CH3)3C–CHO does not undergo Aldol condensation due
to
(a) three electron donating methyl groups OH
|
(b) cleavage taking place between —C— CHO bond (a) C6 H5 CH - CH 2CHO (b) C6H5CH = CH – CHO
(c) absence of alpha hydrogen atom in the molecule
(c) C6H5CH2CH2CHO (d) Both (b) & (c)
(d) bulky (CH3)3 C—group
23. In the reaction CH3
HCl Boiling |
CH 3CN + 2 H ¾¾¾® X ¾¾ ¾ ¾® Y; the term Y is :
Ether H 2O
31. CH3 - C - CHO does not show aldol condensation
(a) acetone (b) ethyl amine
(c) acetaldehyde (d) dimethyl amine |
24. Phenylmethyl ketone can be converted into ethylbenzene in CH 3
one step by which of the following reagents?
(a) LiAlH4 (b) Zn-Hg/HCl because
(a) carbon is bounded by 3 heavy methyl group
(c) NaBH4 (d) CH3MgI
(b) due to absence of a-hydrogen
25. Which one of the following is reduced with zinc and
(c) it is a neutral molecule
hydrochloric acid to give the corresponding hydrocarbon?
(d) due to hindrance created by methyl for carbanion.
(a) Acetamide (b) Acetic acid 32. Which of the following is best method for reducing
(c) Ethyl acetate (d) Butan-2-one 3-bromopropanal to 1-bromopropane –
26. Acetone reacts with bleaching powder to give (a) Wolf-Kishner reduction
(a) CCl4 (b) Clemmenson reduction
(b) CHCl3 (c) Either (a) or (b)
(c) CCl3 - COCH3 (d) Stephen's reduction
33. Cannizzaro reaction occurs with
(d) none of these.
27. Appropriate reducing agent for the following conversion (a) CH 3 - CH 2 OH (b) C 6 H 5 CHO
is– (c) CH 3 CHO (d) CH 3 - CO - CH 3
O 34. In an alkaline medium, acetaldehyde undergoes:
|| (a) benzoin condensation (b) aldol condensation
CH2 = CH - CH 2 - C - H (c) polymerisation (d) Cannizzaro reaction
35. Which of the following compound will show positive silver
¾¾
® CH 3 - CH 2 - CH 2 - CH 2 OH mirror test ?
(a) LiAlH4/H2O (b) NaBH4/H2O (a) HCOOH (b) CH 3 (CHOH ) 3 CHO
(c) Na + C2H5OH (d) B2H6/H+
28. A compound called chloral whose formula is C2Cl3OH is a/ (c) CH 3CO(CHOH )CH 3 (d) Both (a) and (b)
an 36. Oxalic acid on heating with conc. H 2SO 4 produces
(a) aldehyde (b) alcohol
(a) CO + CO 2 (b) CO 2 + H 2
(c) ketone (d) alkanoyl chloride
(c) C + H 2 O (d) CO + H 2
O
37. Aldehydes and ketones are generally reduced by :
OH - (a) Clemmensen reduction (b) H2S
29. 2 ¾¾¾® ? Product is
D (c) H2/Ni (d) None of these
38. The product obtained by the reaction of an aldehyde and
O hydroxylamine is
(a) hydrazone (b) aldoxime
(a) (b)
(c) primary amine (d) alcohol
39. In which reaction, > C = O can be reduced to > CH2?
(a) Wolf-Kishner reaction
(b) Reimer-Tiemann reaction
(c) (d) O (c) Wurtz reaction
O (d) None of these
EBD_7327
648 CHEMISTRY

40. In benzilic acid rearrangement, 49. Which of the following is disproportionation reaction?
(a) benzoin is converted into benzilic acid NaOH
(b) benzaldehyde is converted into benzoin (a) 2HCHO ¾¾¾¾
® CH3OH + HCOONa
(c) benzil is converted into benzilic acid NaOH
(d) benzilic acid is converted into benzil (b) 2CH3CHO ¾¾¾¾
® CH3CHCH 2CHO
|
41. Which of the following does not show Cannizzaro reaction? OH
(a) CH3CHO (b) HCHO
(c) C6H5CHO (d) (CH3)3C.CHO NO2
+
42. A compound does not react with 2, 4-dinitrophenylhydrazine, (c) + NO 2 ¾
¾®
the compound is :
(a) Acetone (b) Acetaldehdye (d) Both (a) & (b)
(c) CH3OH (d) CH3CH2COCH3 50. Formaldehyde reacts with ammonia to give urotropine is
43. Which gives lactic acid on hydrolysis after reacting with (a) (CH2)6N4 (b) (CH2)4N3
HCN ? (c) (CH2)6N6 (d) (CH2)3N3
(a) HCHO (b) CH3CHO 51. The property which distinguishes formic acid from acetic
(c) C6H5CHO (d) CH3COCH3 acid is
44. Ketones react with Mg-Hg over water to give :
(a) only ammonium salt of formic acid on heating gives
(a) pinacolone (b) pinacols
amide
(c) alcohols (d) none of these
(b) when heated with alcohol/H2SO4 only acetic acid forms
45. Which of the following functional groups cannot be reduced
ester
to alcohol using NaBH4 in ethanolic solution ?
(a) R – O – R (b) RCOCl (c) only acetic acid forms salts with alkali
(c) R – COOH (d) R – CHO (d) only formic acid reduces Fehling’s solution
46. What is the final product of the following reaction ? 52. Which of the following is an example of aldol condensation?

O CH3
| dil NaOH
C – CH3 (a) 2CH3COCH3 ¾¾¾¾¾
® CH3 C(OH)CH 2 COCH3
CH OH
¾¾¾¾
3 ® ¾¾¾¾
®? dil NaOH
- H 2O
C – OH (b) 2HCHO ¾¾¾¾¾
® CH3OH
dil NaOH
O (c) C6H5CHO + HCHO ¾¾¾¾¾ ® C6H5CH2OH
(d) None of the above
O O 53. The reagent which does not give acid chloride on treating
with a carboxylic acid is
C C
(a) PCl5 (b) Cl2
(a) (b) O
(c) SOCl2 (d) PCl3
C CH2 54. The reactant (X) in the reaction
O CH COONa
(X) ¾¾¾¾¾
3 ¾® Cinnamic acid, is
(CH CO) O
3 2
O COOH CHO
CH3O CH3
C CH3 (a) (b)
(c) (d) O
C CH 3 CH2OH
O (c) (d)
O
47. 2-pentanone and 3-pentanone can be distinguished by : 55. Main product obtained from the reaction of ammonia and
(a) Cannizaro's reaction formaldehyde is
(b) Aldol condensation (a) formic acid (b) methylamine
(c) Iodoform reaction (c) methanol (d) urotropin
(d) Clemmensen's reduction 56. Which of the products is formed when acetone is reacted
48. When dihydroxyacetone reacts with HIO4, the product is/ with barium hydroxide solution?
are :
(a) HCHO O CH3
(b) HCOOH P |
(c) HCHO and HCOOH (a) CH3 - C - CH 2 - C - CH3
|
(d) HCHO and CO2 OH
Aldehydes, Ketones and Carboxylic Acids 649

66. In the Friedel Craft's acylation reaction, the effective


O electrophile is
P
(b) CH3 - C - CH - CH - CH3 (a) RCOCl Å (b) AlCl3
| | (c) RCOCl (d) RCO Å
CH3 OH 67. The compound which is not formed during the dry distillation
of a mixture of calcium formate and calcium acetate is
O (a) methanal (b) propanal
P
(c) CH3 - C - CH - CH - CH3 (c) propanone (d) ethanal
| | 68. Which one of the following esters cannot undergo Claisen
OH CH3 self-condensation?
(a) CH 3 - CH 2 - CH 2 - CH 2 - COOC 2 H 5
OH OH (b) C6H5COOC2H5
| |
(d) CH3 - C - C - CH 3 (c) C6H5CH2COOC2H5
| | (d) C6H11CH2COOC2H5
CH3 CH3 69. The cyanohydrin of a compound on hydrolysis gives an
57. Hydride ion transfer takes place in optically active a-hydroxy acid. The compound is
(a) Frankland method (b) Wurtz reaction (a) diethyl ketone (b) formaldehyde
(c) Cannizzaro's reaction (d) Wolf-Kishner reduction (c) acetaldehyde (d) acetone
58. Ketone upon treatment with Grignard Reagent gives 70. The product of following reaction is
(a) primary alcohol (b) secondary alcohol 2CH3MgBr + CH2O —®
(c) tertiary alcohol (d) aldehyde (a) CH3OH (b) C2H5OH
59. Which of the following compounds would be the main (c) CH4 (d) C2H6
product of an aldol condensation of acetaldehyde and Cl2
CH3CH2COOH ¾red
¾¾® A ¾alc. KOH
acetone? 71. ¾ ¾¾® B. What is B?
P
(a) CH3CH = CH.CHO (b) CH3CH = CHCOCH3
(c) (CH3)2C=CH.CHO (d) (CH3)2C = CHCOCH3 (a) CH3CH2COCl (b) CH3CH2CHO
60. What is formed, when acetonitrile is hydrolysed partially (c) CH2=CHCOOH (d) ClCH2CH2COOH.
with cold concentrated HCl? 72. When dihydroxy acetone reacts with HIO4, the product is/
(a) Acetamide (b) Acetic acid are
(c) Methyl cyanide (d) Acetic anhydride (a) HCHO
61. Among acetic acid, phenol and n-hexanol, which of the (b) HCOOH
following compounds will react with NaHCO3 solution to (c) HCHO and HCOOH
give sodium salt and carbon dioxide ? (d) HCHO and CO2
(a) acetic acid (b) n-Hexanol 73. When CH2 = CH — COOH is reduced with LiAlH4, the
(c) acetic acid and phenol (d) phenol. compound obtained will be
62. Formic acid is obtained when (a) CH2 = CH — CH2OH (b) CH3 — CH2 — CH2OH
(a) calcium acetate is heated with conc. H2SO4 (c) CH3 — CH2 — CHO (d) CH3 — CH2 — COOH
(b) calcium formate is heated with calcium acetate 74. On mixing ethyl acetate with aqueous sodium chloride, the
(c) glycerol is heated with oxalic acid at 373 K composition of the resultant solution is
(d) acetaldehyde is oxidised with K2Cr2O7 and H2SO4. (a) CH3COCl+ C2H5OH + NaOH
63. Schotten-Baumann reaction is a reaction of phenols with (b) CH3COONa + C2H5OH
(a) benzoyl chloride and sodium hydroxide (c) CH3COOC2H5 + NaCl
(b) acetyl chloride and sodium hydroxide (d) CH3Cl + C2H5COONa
(c) salicylic acid and conc. H2SO4 75. Acetyl bromide reacts with excess of CH3MgI followed by
(d) acetyl chloride and conc H2SO4 treatment with a saturated solution of NH4Cl gives
64. Which of the following products is formed when (a) 2-methyl-2-propanol (b) acetamide
benzaldehyde is treated with CH3MgBr and the addition (c) acetone (d) acetyl iodide
product so obtained is subjected to acid hydrolysis ? 76. Acetic anhydride reacts with diethyl ether in the presence
(a) A secondary alcohol of anhydrous AlCl3 to give :
(b) A primary alcohol
(a) CH3CH2COOH (b) CH3CH2COOC2H5
(c) Phenol
(c) CH3COOCH3 (d) CH3COOC2H5
(d) tert-Butyl alcohol
77. An important reaction of acetone is auto condensation in
65. An ester is boiled with KOH. The product is cooled and
presence of concentrated sulphuric acid to give the aromatic
acidified with concentrated HCl. A white crystalline acid
compound......
separates. The ester is
(a) methyl acetate (b) ethyl acetate (a) Mesitylene (b) Mesityl oxide
(c) Trioxan (d) Phorone
(c) ethyl formate (d) ethyl benzoate
EBD_7327
650 CHEMISTRY

78. The acid(s) which do not contain –COOH groups is/are : (a) Formic acid — Methanoic acid
(a) palmitic acid (b) Acetic acid — Ethanoic acid
(b) lactic acid (c) n-Butyric acid — Butanoic acid
(c) ethanoic acid (d) Iso-butyric acid — 2-methylbutanoic acid
(d) picric acid and p-toluene sulphonic acid 90. Silver benzoate will react with bromine in CCl4 to form :
79. The best combination of reagents for carrying out the
conversion RCH2CH2OH ® RCH2CH2COOH is COOBr
(a) PBr3, KCN, H3O+ (b) PBr3, KCN, H2/Pt COOAg
(c) KCN, H3O+ (d) PBr3, H3O+ (a) (b)
A
80. CH 3COOH ¾¾® CH 3 COCl . What is A ?
(a) PCl5 (b) Cl2 Br Br
(c) HCl (d) COCl2
COOH
81. Ethyl acetate on reaction with Grignard’s reagent, forms a/
an (c) (d)
(a) alcohol (b) ketone
(c) ether (d) hydrocarbon 91. Which of the following pairs can be distinguished by sodium
82. The compound not soluble in acetic acid is : hypoiodite?
(a) CaCO3 (b) CaO (a) CH3CHO and CH3COCH3
(c) CaC2O4 (d) Ca(OH)2 (b) CH3CH2CHO and CH3COCH3
83. Among the following, the most acidic is : (c) CH3CH2OH and CH3CH2CHOHCH3
(a) CH3COOH (b) ClCH2COOH (d) CH3OH and CH3CH2CHO
(c) Cl2CHCOOH (d) Cl2CHCH2COOH 92. Reduction of aldehydes and ketones into hydrocarbons
84. The compound that neither forms semicarbazone for oxime using zinc amalgam and conc. HCl is called
is (a) Cope reduction
(a) HCHO (b) CH3COCH2Cl (b) Dow reduction
(c) CH3CHO (d) CH3CONHCH3
(c) Wolf-Kishner reduction
85. Consider the following transformations :
(d) Clemmensen reduction.
CaCO heat I
CH3COOH ¾ ¾ ¾¾3 ® A ¾ ¾¾ ® B ¾¾®
2
C 93. Which of the following represents the correct order of the
NaOH
acidity in the given compounds?
The molecular formula of C is
(a) FCH2 COOH > CH 3 COOH > BrCH 2 COOH >
OH ClCH2COOH
|
(b) BrCH2 COOH > ClCH2 COOH > FCH2 COOH >
(a) CH 3 - C - CH 3 (b) ICH2 — COCH3
| CH3COOH
I (c) FCH2 COOH > ClCH2 COOH > BrCH2 COOH >
(c) CHI3 (d) CH3I CH3COOH
86. In the following reaction (d) CH3 COOH > BrCH2 COOH > ClCH2 COOH >
Br / P excess NH FCH2COOH
RCH 2 COOH ¾¾¾¾
2 ® X ¾¾¾¾¾
3® Y
94. Which one of the following on treatment with 50% aqueous
The major compounds X and Y are sodium hydroxide yields the corresponding alcohol and
(a) RCHBrCONH2 ; RCH(NH2)COOH acid?
(b) RCHBrCOOH ; RCH(NH2)COOH (a) C6H5CHO (b) CH3CH2CH2CHO
(c) RCH2COBr ; RCH2COONH4
O
(d) RCHBrCOOH ; RCH2CONH2 ||
87. An organic compound C3H6O neither gives precipitate with (c) CH 3 - C - CH 3 (d) C6H5CH2CHO
semicarbazide nor reacts with sodium. It could be 95. The product formed in Aldol condensation is
(a) CH3CH2CHO (b) CH3COCH3
(a) a beta-hydroxy aldehyde or a beta-hydroxy ketone
(c) CH2 = CH —CH2—OH (d) CH2 = CH —O—CH3
(b) an alpha-hydroxy aldehyde or ketone
88. Which one of the following compounds will not react with
CH3MgBr? (c) an alpha, beta unsaturated ester
(a) Ethyl acetate (b) Acetone (d) a beta-hydroxy acid
(c) Dimethyl ether (d) Ethanol 96. Propionic acid with Br2 /P yields a dibromo product. Its
structure would be:
89. Some carboxylic acids and their IUPAC names are given
below. Which of the following is not correctly matched?
Aldehydes, Ketones and Carboxylic Acids 651

Br (a) (i) and (ii) (b) (i), (iii) and (iv)


| (c) Only (ii) (d) (i), (ii) and (iii)
(a) H– C – CH COOH (b) CH2Br – CH2 – COBr
|
2 99. Clemmensen reduction of a ketone is carried out in the
Br presence of which of the following ?
Br (a) Glycol with KOH (b) Zn-Hg with HCl
| (c) Li Al H4 (d) H2 and Pt as catalyst
(c) CH3– C – COOH (d) CH2 Br – CHBr – COOH
| 100. Match the compounds given in List-I with List-II and select
Br the suitable option using the code given below :
97. Which of the following reactions will not result in the List I List-II
formation of carbon-carbon bonds? (A) Benzaldehyde (i) Phenolphthalein
(a) Reimer-Tieman reaction (b) Cannizaro reaction (B) Phthalic anhydride (ii) Benzoin condensation
(c) Wurtz reaction (d) Friedel-Crafts acylation (C) Phenyl benzoate (iii) Oil of wintergreen
98. Following compounds are given: (D) Methyl salicylate (iv) Fries rearrangement
(i) CH3CH2OH (ii) CH3COCH3 Code :
(iii) CH 3 - CHOH (iv) CH3OH (A) (B) (C) (D)
(a) (iv) (i) (iii) (ii)
CH 3 (b) (iv) (ii) (iii) (i)
Which of the above compound(s), on being warmed with (c) (ii) (iii) (iv) (i)
iodine solution and NaOH, will give iodoform? (d) (ii) (i) (iv) (iii)

1. The catalyst used in the Wacker process of oxidation of The above compound describes a condensation polymer
ethyne to ethanal is which can be obtained in two ways : either treating 3 molecules
(a) PdCl2 (b) V2O5 of acetone (CH3COCH3) with conc. H2SO4 or passing propyne
(c) Nickel complexes (d) TiCl4 and Al(CH3)3 (CH3 C º CH) through a red hot tube. The polymer is
2. Which one of the following reagents enables the purification (a) Phorone (b) Mesityl oxide
of benzaldehydes? (c) Deacetonyl alcohol (d) Mesitylene
(a) HCN (b) CH3MgBr
7. In which of the following, the number of carbon atoms does
(c) NH2OH (d) NaHSO3
not remain same when carboxylic acid is obtained by oxidation
3. Which of the following is the correct decreasing order of
acidic strength of (a) CH 3COCH 3 (b) CCl 3CH 2 CHO
(i) Methanoic acid (ii) Ethanoic acid
(c) CH 3CH 2CH 2OH (d) CH3CH 2CHO
(iii) Propanoic acid (iv) Butanoic acid
(a) (i) > (ii) > (iii) > (iv) (b) (ii) > (iii) > (iv) > (i) 8. Aldehydes that do not undergo aldol condensation are
(c) (i) > (iv) > (iii) > (ii) (d) (iv) > (i) > (iii) > (ii) 1. propanal 2. trichloroethanal
CH 2 O 3. methanal 4. ethanal
5. benzaldehyde
4. O CH 2
(a) 3 and 4 only (b) 3 and 5 only
CH 2 O (c) 1, 2 and 3 only (d) 2, 3 and 5 only
The above shown polymer is obtained when a carbonyl 9. Benzoic acid gives benzene on being heated with X and
compound is allowed to stand. It is a white solid. The polymer phenol gives benzene on being heated with Y. Therefore X
is and Y are respectively
(a) Trioxane (b) Formose (a) Soda-lime and copper (b) Zn dust and NaOH
(c) Paraformaldehyde (d) Metaldehyde
(c) Zn dust and soda-lime (d) Soda-lime and zinc dust
5. Which of the following gives reddish brown precipitate on
heating with a solution of CuSO4 and alkaline sodium 10. An ester (A) with molecular fomula, C9H10O2 was treated
potassium tartarate? with excess of CH3MgBr and the complex so formed was
treated with H2SO4 to give an olefin (B). Ozonolysis of (B)
(a) Ethanol (b) Ethanal
gave a ketone with molecular formula C8H8O which shows
(c) Propanone (d) Ethanoic acid
positive iodoform test. The structure of (A) is
6. CH 3 (a) C6H5COOC2H5
(b) C2H5COOC6H5
(c) H3COCH2COC6H5
CH 3 (d) p — H 3CO — C 6 H 4 — COCH 3
CH 3
EBD_7327
652 CHEMISTRY

11. Polarization of electrons in acrolein may be written as: In the above sequence of reaction X and Y are respectively
d- d+ (a) H2/Pt ; Br2 (b) KMnO4; H2/Pt
(a) C H 2 = CH — CH = O (c) KMnO4 (aq); HI/P (d) NH2 – NH2/KOH, HI/P
d+ d- O O
(b) C H 2 = C H — CH = O
d+ d- PhMgBr
3 2 CH MgBr H + /H O
20. ¾¾¾¾® ¾¾¾¾¾
® ¾¾¾¾® Product.
(c) C H 2 = CH — CH = O (1 eq.) (1 eq.) (1 eq.)

d- d+
O
(d) C H 2 = C H — C H = O
The product formed in the reaction is –
12. Which compound among the following gives positive
iodoform test ? OH OH
1. Ethanol 2. Ethanal O O
3. 1-Butanol 4. 2-Butanol || ||
5. Phenyl ethanol (a) Me Ph (b) Me - C - C - Ph
(a) 1, 2 and 5 (b) 1, 3 and 4
(c) 1, 2 and 3 (d) 1, 2 and 4 O
13. Reaction of cyclohexanone with dimethylamine in the
presence of catalytic amount of an acid forms a compound if OH O O
water during the reaction is continuously removed. The | || ||
(c) Me - C - COOH (d) Ph - C - C - O - Me
compound formed is generally known as |
(a) an amine (b) an imine Ph
(c) an enamine (d) a schiff’s base
14. The increasing order of the rate of HCN addition to compound 21. An aromatic compound 'X' with molecular formula C9H10O
A – D is gives the following chemical tests. It
(A) HCHO (B) CH3COCH3 (i) forms 2, 4-DNP derivative
(C) PhCOCH3 (D) PhCOPh (ii) reduces Tollen's reagent
(a) D < C < B < A (b) C < D < B < A (iii) undergoes Cannizaro reaction and
(c) A < B < C < D (d) D < B < C < A (iv) on vigorous oxidation 1, 2-benzenedicarboxylic acid is
15. Reaction of butanone with methylmagnesium bromide obtained.
X is
followed by hydrolysis gives
CHO CHO
(a) 2-methyl-2-butanol
(b) 2-butanol C2H5
(c) 3-methyl-2-butanol (a) (b)
(d) 2, 2-dimethyl-1-butanol C2H5
16. The correct order of increasing acid strength of the O
CHO
compounds
(A) CH3CO2H (B) MeOCH2CO2H C
(c) CH3 (d)
Me
(C) CF3CO2H (D) CO2H CH3 C2H5
Me O
is
OH
(a) D < A < B < C (b) A < D < B < C
(c) B < D < A < C (d) D < A < C < B 22. OH X + H2O
H2SO4
17. How many cross aldol products (without counting OH
stereoisomers) are produced in the following reaction? Product (X) of the reaction is–
O O
||
NaOH
CH3 - CH = O + CH3 - C - CH3 ¾¾¾¾ ® OH O
(a) 5 (b) 2 O
(c) 3 (d) 1 (a) (b) O
18. Monocarboxylic acids are functional isomers of : OH
(a) Ethers (b) Amines
(c) Esters (d) Alcohols O O
X O
19. CH3 COOH (c) (d)
Y HO
OH
Aldehydes, Ketones and Carboxylic Acids 653

23. In a set of reactions acetic acid yielded a product D. What is Z ?


SOCl
2 ® (A) ¾¾¾¾ Benzene
CH3COOH ¾¾¾¾ ® (B) OH
AlCl3
|
HCN HOH (a) CH3 - C - CH 2 - CH3
¾¾¾® (C) ¾¾¾® (D) |
The structure of (D) would be – CH3
COOH
(b) CH3 - C = CH - CH3
CH2 – C – CH3 |
(a) CH3
OH

CN C2 H5 C2 H5
| |
(c) CH3 - C - O - C - CH3
C – CH3 | |
(b) CH3 CH3
OH

OH (d) CH 2 = C - CH 2 - CH3
|
CH2 – C – CH3 CH3
(c)
CN O O

OH 27. CH3 – C – CH 2 – C
OEt
C – COOH
CH3 - C - Cl
(d) ||
CH3 Na (1 mole) dil. KOH O
¾¾¾¾¾® ¾¾¾¾¾® ¾¾ ¾ ¾®
dil. H Å
CH = O
(i) KOH/D The final product is –
24. [X] . What is [X]?
(ii) H O/H+
2 (a) CH3 - C - CH 2 - C - CH3
CH = O || ||
O O
CH2 – OH CH = O
(b) CH3 - C - OH
(a) (b) ||
O
COOH COOH

CH2 – OH
(c) CH3 - C - CH 2 - COOH
CH2 – OH
||
(c) (d) O

CH = O COO K+ (d) CH3 - C - CH 2 - CH 2 - C - CH3
25. CH3CO2C2H5 on reaction with sodium ethoxide in ethanol || ||
O O
gives A, which on heating in the presence of acid gives B.
Compound B is : 28. o-Toluic acid on reaction with Br2 + Fe gives
(a) CH3COCH2COOH (b) CH3COCH3 CH3
CH2Br
O OC2H5 CO2H CO2H
(c) CH2 O (d) CH2 =C (a) (b)
OC2H5
Br
O CH3
|| CH3
26. CH3 - C - CH 2 - CH3 + CH3MgBr CO2H
CO2H
(c) (d)
H O+
3 ® Y ¾¾¾¾
2 4® Z . H SO
¾¾
® X ¾¾¾¾
170°C Br Br
EBD_7327
654 CHEMISTRY

29. Which of the following compound will not give benzoic acid 34. In a set of reactions, ethylbenzene yield a product D.

on oxidation with KMnO4/ OH /D – CH2CH3 KMnO4 Br2
¾¾¾¾¾ ® B ¾¾¾¾ ®
CH3 KOH FeCl3
|
(a) C6H5 – CH3 (b) C6 H5 - C - CH3
C2H 5OH
| C ¾¾¾¾¾ ®D
CH3 H+
Identify D :
CH3
(c) C 6H 5 – CH (d) C 6H 5 – CH 2 – COOH CH2 – CH – COOC 2H5
(a)
CH3 Br
30. The final product (III) obtained in the reaction sequence –
3® I PCl Br
CH 3 - CH 2 - COOH ¾¾¾
C H /AlCl
6 6 3 ® II ¾¾¾¾¾
2 NH - NH
2 ® III
¾¾¾¾¾¾ (b)
base/heat
Br
CH 2 – CH 2 – CH 3
CH2 COOC2H5
(a)

COOH
(b) CH – CH2 – CH3
(c)
OH OCH2CH3
O
||
(c) C – O CH2 – CH3 COOC2H5

O (d)
|| Br
(d) C – CH2 – CH3 35. Which one of the following compounds will be most readily
dehydrated?
31. Calculate number of molecules of Grignard reagent consumed
OH
by 1 molecule of following compound.
H (a)
CH3
O
O O
O O
O (b) CH
3
OH
HO NO 2 O
(a) 5 (b) 2
(c) 3 (d) 1 (c)
H3 C
32. Cannizaro’s reaction is not given by : OH
CHO O OH
(a) CHO (b)
CH 3 (d)
CH3
(c) CH3CHO (d) HCHO 36. The order of reactivity of phenyl magnesium bromide
33. Which of the following reagent(s) used for the conversion? (PhMgBr) with the following compounds
O O CH3 CH3 Ph
OH C=O, C=O and C=O
CO2Et H CH 3 Ph
I II III
(a) glycol/LiAlH4/H3O+ (b) glycol/NaH/H3O+ (a) III > II > I (b) II > I > III
(c) LiA1H4 (d) NaBH4 (c) I > III > II (d) I > II > III
Aldehydes, Ketones and Carboxylic Acids 655

37. An organic compound ‘A’ on treatment with NH3 gives ‘B’


which on heating gives ‘C’, ‘C’ when treated with Br 2 in the (b) NH
presence of KOH produces ethylamine. Compound ‘A’ is: C CH3
(a) CH3COOH (b) CH3 CH2 CH2 COOH
O
(c) CH3 – CHCOOH (d) CH3CH2COOH
CH3 (c) H3C NH
C
38. In Cannizzaro reaction given below
O
e
-
: OH .. -
2PhCHO ¾¾¾ ® PhCH 2 OH + PhCOe2
O
(d) H3C C O
the slowest step is :
NH C
(a) the transfer of hydride to the carbonyl group
(b) the abstraction of proton from the carboxylic group
(c) the deprotonation of Ph CH2OH
41. The compounds P, Q and S
e
-
(d) the attack of : OH at the carboxyl group O
39. In the following reaction sequence, the correct structures of COOH OCH3 C
E, F and G are O

O O HO H3C
P Q S
Heat I
¾¾¾
®[E] ¾¾¾®
2
NaOH
[F] + [G] were separately subjected to nitration using HNO3/H2SO4
Ph * OH mixture. The major product formed in each case respectively,
13
[* implies C labelled carbon) is :
O
O O
COOH OCH3 C
O
(a) E= * F= * – + (a)
CH3 O Na G = CHI3
Ph Ph HO H3C O2N
NO2 NO2
O O
O
COOH OCH3
(b) E= F= * – +
C
* O Na G = CHI3
O
Ph CH3 Ph (b)
HO NO2 H3C NO2
O O NO2

(c) E= F= *
NO2
* – + O
CH3 O Na G = CHI3 COOH OCH3 C
Ph Ph
O
O O (c)
HO H3C NO2
(d) E= F=
– + NO2
* *
CH3 O Na G = CH3I
Ph Ph
O
40. In the reaction COOH OCH3 C
O
O (1) NaOH/Br2 NO2
(d)
H3C C T HO H3C NO2
O NO2
NH2 (2) C 42. The number of aldol reaction(s) that occurs in the given
Cl transformation is :
the structure of the product T is :
OH
OH
O conc. aq. NaOH
CH3CHO + 4HCHO ¾¾¾¾¾¾¾
®
(a) H3C C O
C HO
O OH
(a) 1 (b) 2
(c) 3 (d) 4
EBD_7327
656 CHEMISTRY

43. The major product H of the given reaction sequence is –


CH2OH COO
e
– (c) +
CN
95% H 2SO 4
CH3 — CH2 — CO — CH3 ¾¾¾¾
® G ¾¾¾¾¾® H
Heat
Cl Cl
(a) CH3 – CH = C – COOH


CH3
(d)
CH2OH COO–
+
(b) CH3 – CH = C – CN

OH OH
CH3
47. Acetone is treated with excess of ethanol in the presence of
OH
hydrochloric acid. The product obtained is :
— —

(c) CH3 – CH2 – C – COOH


O
CH3
(a) CH3CH2CH2 C CH3
(d) CH3 – CH = C – CO – NH2

O
CH3
44. The compound that undergoes decarboxylation most readily (b) CH3CH2CH2 C CH 2CH 2CH 3
under mild condition is
OH
COOH COOH (c) (CH3)2C
CH2COOH O OC2H5
(a) (b)
OC2H5
(d) (CH3)2C
COOH CH2COOH OC2H5
COOH O DIRECTIONS for Qs. 48 to 50 : These are Assertion-Reason
(c) (d) type questions. Each of these question contains two statements:
Statement-1 (Assertion) and Statement-2 (Reason). Answer these
45. An organic compound A upon reacting with NH3 gives B. questions from the following four options.
On heating B gives C. C in presence of KOH reacts with Br 2 (a) Statement-1 is true, Statement-2 is true, Statement-2 is a
to given CH3CH2NH2. A is : correct explanation for Statement -1
(a) CH3COOH (b) CH3CH2CH2COOH
(b) Statement-1 is True, Statement-2 is True ; Statement-2 is NOT
(c) CH3 - CH - COOH (d) CH3CH2COOH a correct explanation for Statement-1
| (c) Statement-1 is True, Statement-2 is False
CH3
(d) Statement-1 is False, Statement-2 is True
46. Predict the product in the given reaction.
48. Statement-1 : Acetic acid does not undergo haloform reaction.
CHO Statement-2 : Acetic acid has no alpha hydrogens.
50 % KOH 49. Statement-1 : Benzaldehyde is more reactive than ethanol
towards nucleophilic attack.
Cl
Statement-2 : The overall effect of –I and + R effect of phenyl
CH2OH CH2COO
– group decreases the electron density on the carbon atom of
(a) + > C = O group in benzaldehyde.

Cl 50. Statement-1 : Lower aldehyde and ketones are soluble in


Cl
water but the solubility decreases as molecular mass
increases.
CH2OH OH
(b) + Statement-2 : Aldehydes and ketones can be distinguished
by Tollen’s reagent.
OH OH
Aldehydes, Ketones and Carboxylic Acids 657

Exemplar Questions 7. Which product is formed when the compound


1. Addition of water to alkynes occurs in acidic medium and in
the presence of Hg 2+ ions as a catalyst. Which of the CHO is treated with concentrated aqueous KOH
following products will be formed on addition of water to solution?
but - 1-yne under these conditions?
+ –
O (a) KO CHO
||
(a) CH 3 — CH 2 — CH 2 — C— H O
|| - +
O (b) C– OK + CH2OH
||
(b ) CH3 — CH 2 — C — CH 3
O
+ – || - + + - – +
O (c) KO C O K+ K O OK
||
(c) CH3 — CH 2 — C — OH + CO 2
O
O O || - +
|| || –+
(d) C— O K + OK
(d) CH3 — C— OH + H — C— H
2. Which of the following compounds is most reactive towards 8. 40% H SO
CH3 — C º CH ¾¾¾¾¾®
2 4 A
nucleophilic addition reactions? 1% HgSO 4
O O Isomerisation
|| || ¾¾¾¾¾¾ ® CH3 — C — CH3
||
(a) CH 3 — C — H (b) CH 3 — C — CH 3 O
O O structure of 'A' and type of isomerism in the above reaction
|| || are respectively
(c) —C—H (d) — C — CH 3 (a) Prop-1-en-2-ol, metamerism
(b) Prop-1-en-1-ol, tautomerism
3. The correct order of increasing acidic strength is ........ .
(c) Prop-2-en-2-ol, geometrical isomerism
(a) phenol < ethanol < chloroacetic acid < acetic acid
(d) Prop -1-en-2-ol, tautomerism
(b) ethanol < phenol < chloroacetic acid < acetic acid
9. Compounds A and C in following reaction are ........... .
(c) ethanol < phenol < acetic acid < chloroacetic acid
( i ) CH MgBr H SO ,D
(d) chloroacetic acid < acetic acid < phenol < ethanol CH 3 CHO ¾¾¾¾¾¾
( )
3 ® ( A ) ¾¾¾¾¾
2 4 ® (B)
ii H 2O
O Hydroboration oxidation
|| ¾¾¾¾¾¾¾¾¾® ( C )
4. Compound Ph — O — C— Ph can be prepared by the (a) identical (b) positional isomers
reactions of ............ . (c) functional isomers (d) optical isomers
(a) phenol and benzoic acid in the presence of NaOH 10. Which is the most suitable reagent for the following
(b) phenol and benzoyl chloride in the presence of pyridine conversion?
(c) phenol and benzoyl chloridein the presence of ZnCl 2 O
(d) phenol and benzaldehyde in the presence of palladium ||
5. The reagent which does not react with both, acetone and CH3 — CH = CH — CH 2 — C— CH 3 ¾¾
®
benzaldehyde?
(a) Sodium hydrogen sulphite O
||
(b) Phenyl hydrazine
CH3 — CH = CH — CH 2 — C — OH
(c) Fehling's solution
(d) Grignard reagent (a) Tollen's reagent (b) Benzoyl peroxide
6. Cannizzaro's reaction is not given by ........ . (c) I2 and NaOH solution (d) Sn and NaOH solution
CHO 11. Which of the following compounds will give butanone on
(a) (b) CHO oxidation with alkaline KMnO4 solution?
CH3 (a) Butan-1-ol (b) Butan-2-ol
(c) HCHO (d) CH3CHO (c) Both (a) and (b) (d) None of these
EBD_7327
658 CHEMISTRY

12. In Clemmensen reduction, carbonyl compounds is treated 16. An organic compound 'X' having molecular formula C5H10O
with ......... . yields phenyl hydrazone and gives negative response to the
(a) zinc amalgam + HCl Iodoform test and Tollen's test. It produces n-pentane on
(b) sodium amalgam + HCl reduction. 'X' could be :- [2015]
(c) zinc amalgan + nitric acid (a) 2-pentanone (b) 3-pentanone
(d) sodium amalgam + HNO3
(c) n-amyl alcohol (d) pentanal
NEET/AIPMT (2013-2017) Questions
13. The order of stability of the following tautomeric compounds 17. Treatment of cyclopentanone =O with methyl lithium
is :
gives which of the following species? [2015]
OH O (a) Cyclopentanonyl cation
| ||
CH 2 = C - CH 2 - C - CH 3 (b) Cyclopentanonyl radical
(c) Cyclopentanonyl biradical
O O
|| || (d) Cyclopentanonyl anion
CH3 - C- CH 2 - C- CH 3
18. The enolic form of ethyl acetoacetate as below has: [2015]
II
H H2
OH O H3C C O H3C C O
| || C C C C
CH 3 - C = CH - C - CH 3 [2013]
OH OC2H5 O OC2H5
(a) III > II > I (b) II > I > III
(c) II > III > I (d) I > II > III (a) 16 sigma bonds and 1 pi - bond
14. Reaction by which Benzaldehyde cannot be prepared : (b) 9 sigma bonds and 2 pi - bonds
[2013] (c) 9 sigma bonds and 1 pi - bond
COCI (d) 18 sigma bonds and 2 pi - bonds
19. Which one of the following esters gets hydrolysed most
(a) + H2 in presence of
easily under alkaline conditions? [2015 RS]

Pd - BaSO4 OCOCH3
(a)

(b) + CO + HCl in presence of anhydrous AlCl 3 ON


2

OCOCH3
COOH
(c) + Zn/Hg and conc. HCl (b)
H3CO

CH3 OCOCH3
(d) + CrO2Cl2 in CS2 followed by H3O+ (c)

15. Which one is most reactive towards Nucleophilic addition OCOCH3


reaction? [2014]
(d)
CHO COCH3 Cl
(a) (b)
20. Reaction of a carbonyl compound with one of the following
reagents involves nucleophilic addition followed by
CHO CHO elimination of water. The reagent is : [2015 RS]
(a) a Grignard reagent
(c) (d) (b) hydrazine in presence of feebly acidic solution
(c) hydrocyanic acid
CH3 NO2
(d) sodium hydrogen sulphite
Aldehydes, Ketones and Carboxylic Acids 659

21. The correct statement regarding a carbonyl compound with (b) A - Ethanal, X-Ethanol, Y - But - 2-enal,
a hydrogen atom on its alphacarbon, is : [2016] Z-Semicarbazone
(a) a carbonyl compound with a hydrogen atom on its alpha- (c) A-Ethanol, X-Acetaldehyde, Y - Butan one,
carbon never equilibrates with its corresponding enol. Z-Hydrazone
(b) a carbonyl compound with a hydrgen atom on its alpha- (d) A-Methoxymethane, X-Ethanoic acid, Y-Acetate ion,
carbon rapidly equilibrates with its corresponding enol Z-hydrazine.
and this process is known as aldehyde-ketone 24. Of the following, which is the product formed when
cyclohexanone undergoes aldol condensation followed by
equilibration.
heating ? [2017]
(c) a carbonyl compound with a hydrogen atom on its alpha-
carbon rapidly equilibrates with its corresponding enol
(a)
and this process is known as carbonylation.
(d) a carbonyl compound with a hydrogen atom on its alpha-
carbon rapidly equilibrates with its corresponding enol O
and this process is known as keto-enol tautomerism.
22. The product formed by the reaction of an aldehyde with a (b)
primary amine is [2016]
(a) Schiff base (b) Ketone OH
(c) Carboxylic acid (d) Aromatic acid
23. Consider the reactions :- [2017] (c)
+
Cu [Ag(NH3)2]
X A Silver mirror O O
(C2H6O) 573K –OHD observed
–OHD O
O Y

NH2–NH–C –NH2 (d)


Z
Identify A, X, Y and Z
(a) A-Methoxymethane, X-Ethanol, Y-Ethanoic acid, Z- OH
Semicarbazide.
EBD_7327
660 CHEMISTRY

Hints & Solutions


EXERCISE - 1 6. [O]
(c) C3H8O ¾¾¾ ® C3H6 O (Ketone)
1. (c) Benzaldehyde and alcoholic KCN react to form Benzoin. ( x)
This is known as Bezoin condensation. OH O
| [O ] P
OH CH3 - CH - CH3 ¾¾¾ ® CH3 - C – CH3
2° Alcohol Ketone
C CH HO C CH 3
Ketones are oxidation products of 2° alcohols.
H 2SO 4 /HgSO 4 7. (c) Clemensen’s Reduction of ketones is carried out in
2. (a) Zn–Hg and HCl.
O CH3 CH3
C–CH3 dil. KMnO OH HIO4
8. (a) ¾¾¾¾¾

CH3 ¾¾¾®
–H2O
CH3 OH
Acetophenone CH3
O
3. (d) (a) Cannizzaro reaction C=O
OH -
NaOH - ¾¾¾¾¾¾¾®
2HCHO ¾¾¾¾ ® CH 3 OH + HCOO aldol condensation
No new C—C bond is formed
C=O CH3
(b) Reimer - Tiemann reaction : CH3
OH
| 9. (b) The compound is C2H5OH.
D / HCl
+ CHCl3 + 3KOH ¾¾¾¾
® C2H5OH ¾¾¾
[O]
® CH3 CHO
Phenol [O]
¾¾¾
® CH3 COOH
OH Na / C2H 5OH HNO2
10. (d) CH3CN ¾¾¾¾¾¾ ® CH3CH 2 NH 2 ¾¾¾¾ ®
CHO
[A]
+ 3KCl + 2H2O
Cu /573K
CH3CH2OH ¾¾¾¾¾
® CH3CHO
Salicylaldehyde [B] [C]
New C—C bond is formed 11. (c)
(c) Fridel-Craft reaction : COOH CH3 CHO
CH3
acidic K Cr O
2 2 7¾ CrO Cl
2 2
Anh. AlCl3
¬¾¾¾¾¾¾ ¾¾¾¾®
+ CHCl3 ¾¾¾¾¾ or KMnO4
®
Benzene Toluene (Etard reaction)
New C—C bond is formed Acidic KMnO4 and K2Cr2O7 oxidise toluene to benzoic
(d) Clemmensen reduction acid but CrO2Cl2 oxidises it to Benzaldehyde.
12. (c) Formalin is an aqueous solution (40%) of formaldehyde.
Zn / Hg
C = O ¾¾¾¾
® > CH 2 13. (d) I2 and Na2CO3 react with acetophenone (C6H5COCH3)
HCl
to give yellow ppt. of CHI 3 but benzophenone
4. (b) Acetic Anhydride is produced due to dehydration of
(C6H5 COC6H5 ) does not and hence can be used to
two molecules of acetic acid by P2O5
distinguish between them.
CH3CO OH H COOCH 3 ¾¾
® 14. (c) The test used for the distinction of HCOOH and
CH 3COOH is treatment with Tollen’s reagent because
(CH3CH)2 O + H 2O
formic acid can reduce Tollen’s reagent but acetic acid
O can not.
||
D COONa
5. (b) (CH3COO)2 Ca ¾¾ ® CH3 - C - CH3 + CaCO3 15. (b) 2HCOONa ¾¾ D
® | + H2
Calcium acetate Acetone 360° COONa
Aldehydes, ketones and carboxylic acids 661

16. (b) Chromic acid and KMnO4 will cleave the molecule at the 25. (d) It is Clemmensen’s reduction
site of double bond while Cu at 300°C cannot oxidise
O
COCH3 ¾ ¾® COOH. The only reagent suitable for this ||
Zn - Hg
conversion is NaOI or NaOH + I2 (iodoform test): CH3 - C - CH 2 - CH3 ¾¾¾¾® CH3CH 2 - CH 2CH3
Conc.HCl
NaOI Butane-2-one (Butane)
(CH 3 ) 2 C = CHCOCH 3 ¾¾¾®
O
(CH 3 ) 2 C = CHCOOH + CHI 3 . ||
NaOH
17. (b) Pinacolone is 3,3-dimethyl-2 butanone. 26. (b) CH 3 - C - CH 3 + 3Cl 2 ¾¾¾¾ ®
CH 3 CCl3COCH3 + 3HCl
|
CH 3 - C - C - CH 3 CCl3 COCH3 + NaOH ¾¾
®
| ||
CH 3 O CHCl3 + CH3COONa
18. (a) Formaldehyde reacts with ammonia to form hexamethylene O
tetramine which is also known as urotropine. It is used as ||
27. (d) LiAlH4/H2O will reduce only - C - H
urinary tract antiseptic
O
6HCHO + 4 NH 3 ¾
¾®( CH 2 ) 6 N 4 + 6 H 2 O ||
NaBH4/H2O will reduce only -C - H
19. (b) When benzaldehyde is refluxed with aqueous alcoholic
potassium cyanide, two molecules of benzaldehyde O
condense together to form benzoin ||
Na + C2H5OH will reduce only - C - H
H O 28. (a) Chloral is CCl3CHO so, it is an aldehyde.
KCN (alc)
—C +C— 29. (c) In the presence of base, cyclohexanone show aldol
D
O H condensation.
H O
O
—C –C— –
OH OH

Benzoin O O+
20. (d) Dihydrogen sodium phosphate (NaH2PO4) does not have
O O–
a lone pair of electrons on the P atom. As such it can not
act as a nucleophile and hence does not react with
aldehydes and ketones.
21. (d) The compound formed is trioxy formaldehyde.
22. (c) Aldol condensation is given by the compounds which HOH
O O
contain a hydrogen atom. As the given compound does
not contain a hydrogen atom. Hence it does not undergo D
aldol condensation. – H2O
23. (c) Acetaldehyde is formed
H OH
HCl
CH3CN + 2H ¾¾¾® CH 3CH = NH.HCl
Ether
30. (b) Aldol formed in aromatic aldehydes itself loses water
Boiling molecule without heating because double bond formed
¾¾¾¾ ® CH3 CHO + NH 4Cl
H 2O
is more stable due to conjugation with benzene ring.
NaOH C 6 H 5 CH=CHCHO is commonly known as
2CH 3COCH 3 + 3I 2 ¾¾¾
¾® 2CHI 3
cinnamaldehyde.
+ 2CH 3COONa 31. (b) (CH3)3 C – CHO does not show aldol condensation
because it does not containa-hydrogen atom.
O 32. (b) In Wolf-Kishner reduction NH2NH2/ OH is used. Br–
C–CH3 CH2–CH3 can be replaced by OH–.
33. (b) Cannizzaro reaction is given by aldehydes and ketones
Zn-Hg/HCl which do not have a -hydrogen atom. Benzaldehyde
24. (b) ¾ ¾ ¾ ¾¾® (C6H5CHO) does not have a -H atom and hence gives
Cannizzaro reaction.
Phenyl methyl Ethyl 34. (b) Acetaldehyde has a - H atoms. In alkaline medium it
ketone benzene
will undergo aldol condensation to produce aldol
This reaction is known as Clemmensen's reduction. (b-hydroxyaldehyde).
EBD_7327
662 CHEMISTRY

35. (d) Compounds having – CHO group reduce Tollen’s reagent 44. (b) Example :
to silver mirror. It is called silver mirror test.
O CH3 CH3
O O ||
Mg - Hg
| |
|| || CH 3 - C - CH 3 ¾¾¾¾ ® CH3 - C -- C - CH 3
H 2O
H - C - OH CH3 - (CHOH) 3 - C - H | |
OH OH
(a) (b)
O Pinacol
(Pinacols have adjacent hydroxyl groups)
Both (a) and (b) have – C – H group so both of them
O
give positive silver mirror test. ||
36. (a) COOH + H2SO4(conc.) ® CO + CO2 + H 2O 45. (a) Compounds having - C - moiety in their structure are
| reduced to alcohols using NaBH4 in ethanolic solution.
COOH O O
O
37. (a) Aldehydes and ketones are reduced to alkanes by || || ||
Clemmensen reduction. Thus R - C - Cl, R - C - OH and R - C - H are
reduced while, R – O – R (ethers) are inert and can’t be
Zn–Hg/HCl H
C=O C + H2O reduced by NaBH 4 .
H
38. (b) O CH3O CH3
39. (a) Wolf- Kishner reduction
C – CH3 C–O H
(i) NH2 – NH2 H CH OH
C=O C 46. (d) ¾¾¾¾
3 ®
(ii) KOH H
C – OH C—– OH
OH
| O O
40. (c) OH -
C6 H5COCOC6 H5 ¾¾¾® (C6 H5 ) 2 C - COO -
Benzil CH3O CH3
OH C
| ¾¾¾¾
® O
H+ - H2 O
¾¾¾ ®(C6 H5 ) 2 C - COOH

Benzilic acid. O
41. (a) Aldehydes having a -hydrogen atom do not show 47. (c) Iodoform test is given by compounds which have CH3CO
Cannizzaro reaction. group.
a O
CH 3CHO HCHO ||
CH 3 - CH 2 - CH 2 - C - CH 3
(a) (b)
2-pentanone
O CH3 O
| || O
a
aC H CH3 - C - C - H ||
| CH 3 - CH 2 - C - CH 2 - CH 3
CH3 3-pentanone
(c) (d) Q 2-pentanone has CH3CO group, so it gives iodoform
Thus only CH3CHO has a -hydrogen atom and hence it test, while 3-pentanone does not have CH3CO group, so
will not show Cannizzaro reaction. it does not give iodoform test.
42. (c) Only aldehydes and ketones react with
2, 4-dinitrophenylhydrazine. CH 2 OH
|
O OH HIO4
48. (d) CO ¾¾¾ ® 2CH 2 O + CO 2
|| HCN
| |
43. (b) CH3 - C - H ¾¾¾® CH3 - C - H
acetaldehyde | CH 2 OH
CN
49. (a) In this reaction, one molecule is oxidised and other is
OH reduced simultaneously.
Hydrolysis
|
¾¾¾¾¾® CH3 - C - H 50. (a) 6HCHO + 4NH 3 ¾¾
® (CH 2 )6 N 4 + 6H 2 O
| urotropine
COOH
Lactic acid ( hexamethylene
tetramine )
Aldehydes, ketones and carboxylic acids 663

51. (d) We can distinguish between formic acid and acetic acid Mechanism : First of all base OH– acts as a nucleophile
by their action on Fehling’s solution. Formic acid gives a and attack other one of carbonyl compound to generate a
red ppt of cuprous oxide but acetic acid does not give red hydroxy alkoxide ion which acts as a hydride ion donor
ppt. to the other molecule of carbonyl compounds. In the final
52. (a) Aldol condensation involves an aldehyde or ketone step there is a exchange of proton from acid to alkoxide
having an a–hydrogen atom. This type of condensation ion to get stable product.
occurs in presence of dilute base (i.e., dil NaOH).
Only CH3COCH3 will give aldol condensation (Both H –
H
HCHO and C6 H5CHO lack a-hydrogen). OH –
H–C=O H–C–O H–C=O
53. (b) Cl2 does not give acid chloride on treating it with a
Formaldehyde O–H H
carboxylic acid.
(i) Formaldehyde
While PCl 3 , PCl 5 and SOCl 2 gives nucleophilic
substitution reaction with carboxylic acid (Cl– replaces –
OH– group of – COOH) H – C = O + CH 3O H – C = O + CH 3OH
54. (b) Benzaldehyde forms cinnamic acid as follows. OH
CHO
R R OMgX
CH 3COONa 58. (c) C = O + R'MgX C
+ (CH3CO)2O R R R'
HOH

b R
CH = CH.COOH R C – OH + Mg(OH)X
R'
3° alcohol
+ CH3COOH
59. (b) When aldehyde condensed with ketone, a-hydrogen of
Cinnamic the ketone involved in condensation.
Cinnamic acid
55. (d) Formaldehyde on reaction with ammonia forms a H O
crystalline compound, hexamethylene tetramine. | P
OH -
CH3 - C = O + H.H 2C - C - CH3 ¾¾¾®
6HCHO + 4NH3 ¾¾
® (CH 2 )6 N 4 + 6H 2O Acetaldehyde Acetone
Formaldehyde Urotropine
(hexamethylene
tetramine)
H O
56. (a) Completing the given reaction. | P
CH3 - C - CH 2 - C - CH3
|
O O
P P OH
OH - 4-hydroxypentan-2-one
CH3 - C + CH3 - C ¾¾¾®
| |
CH3 CH3
H O
| ||
Heat
OH O CH3 - C - C H - C - CH3 ¾¾¾¾
®
- H2 O
| P | |
CH3 - C - CH 2 - C OH H
| |
CH3 CH3
Diacetonyl alcohol CH3CH = CHCOCH3
[Note: Carbonyl compounds having a – H atom 60. (a) When acetonitrile is hydrolysed partially with cold
undergoes aldol condensation in presence of dilute concentrated HCl it forms acetamide.
base {e.g., NaOH, Ba(OH)2 etc.} to form a class of HCl
compounds known as aldol]. CH3 – C º N ¾¾¾
® CH3CO NH 2
Acetonitrile Acetamide
57. (c) In the Cannizzaro reaction, two moles of carbonyl
compounds having no a-hydrogen atom when treated On further hydrolysis acetamide may give either
with strong alkali undergo, redox or disproportionation CH3COOH (acetic acid) or its salt.
reaction. 61. (a) among acetic acid, phenol and n-hexanol only CH 3 COOH
H O reacts with NaHCO3 to evolve CO2 gas.
| P
NaOH CH3COOH + NaHCO3 ¾¾ ® CH3COONa + CO2 + H2O
2H - C = O ¾¾¾¾
® H - C - ONa + CH 3OH
EBD_7327
664 CHEMISTRY

62. (c) When glycerol is heated with oxalic acid following reaction 70. (b) 2CH3MgBr + HCHO —® CH3CH2OH + Mg (OH) Br
occurs. Cl2
71. (c) CH3CH2COOH ¾¾¾® CH3CHClCOOH
CH 2OH HOOC CH 2OOC.COOH red P
| | |
100 -110°C alc.KOH
CHOH + HOOC ¾¾¾¾¾
® CHOH ¾¾ ¾¾® CH 2 = CHCOOH
| oxalic - H 2O | - HCl Acrylic acid
CH 2OH acid CH 2OH
72. (d)
CH2COOH CH 2OH 73. (a) LiAlH4 can reduce COOH group and not the double bond.
| | LiAlH
-CO H O CH 2 = CH - COOH ¾¾¾¾
4
® CH 2 = CH - CH 2 OH
¾¾¾2¾
® CHOH ¾¾¾
2 ® CHOH + HCOOH
| | Formic acid 74. (c) There is no reaction hence the resultant mixture contains
CH2OH CH 2OH CH3 COOC2H5 + NaCl.
63. (a) OH O.COC6H5 O CH 3
|| |
(i)CH
75. (a) CH - C - Br ¾¾¾¾¾¾¾¾ MgI CH - C - OH
+ C6H5COCl
aq. NaOH
3
3
® 3
(ii)Saturated NH4Cl |
CH 3
phenyl benzoate 2 - methyl - 2 - propanol
The function of NaOH is
AlCl
(i) To convert phenol to morestronger nucleophilePhO– 76. (d) (CH3CO)2 O + C2H5 OC2 H5 ¾¾¾®
3
2CH3COOC2H5
(ii) To neutralize the acid formed acetic anhydride diethyl ether
64. (a) Aldehydes, other than formaldehyde, when treated with 77. (a) When distilled with conc. H2SO4, three molecules of
RMgX give 2º alcohols. acetone condense to form mesitylene, i.e. 1, 3, 5-
65. (d) Methyl acetate and ethyl acetate on hydrolysis give trimethylbenzene.
CH3COOH which is a liquid. Similarly ethyl formate on
hydrolysis will give formic acid which is also a liquid. CH3
Only ethyl benzoate on hydrolysis will give benzoic acid
which is a solid.
Conc. H2SO4
O 3CH3COCH3 H3C CH3
66. (d) R COCl + AlCl3 R CÅ + AlCl4
– Mesitylene
Electrophile
67. (b) Propanal is not formed during the dry distillation of a OH CH3
mixture of calcium formate and calcium acetate. While 78. (d) O2N NO2
methanal, propanone and ethanal are formed as follows:
O
HCO — O

Ca D
H — C — H + CaCO3 NO2 SO3H
H — COO methanal picric acid p-toluene sulphonic acid
CH3 — COO OOCH O
Ca + Ca D 2CH3 — C

— Other three acids contain – COOH group.
+ 2CaCO3
CH3 — COO OOCH ethanal H Palmitic acid CH3(CH2)14COOH
Lactic acid CH3CH(OH) COOH
CH3CO — O CH3
D Ethanoic acid CH3COOH
Ca C = O + CaCO3
PBr3 KCN
CH3 — COO CH3 Propanone 79. (a) RCH 2CH 2OH ¾¾¾® RCH 2 CH 2 Br ¾¾¾¾ ®
68. (b) The ester havi ng a hydrogen atom show Claisen H O+
condensation reaction. RCH 2CH 2CN ¾¾¾¾
3 ® RCH 2CH 2COOH + NH 3
We know that ethyl benzoate (C 6 H 5COOC 2 H 5 ) does not PCl
80. (a) CH 3 COOH ¾¾¾ 5
® CH3COCl + HCl
contain a-hydrogen. Therefore C 6 H 5 COOC 2 H 5 does not \ A is PCl5. It can also be SOCl2.
undergo Claisen self condensation.
O R''
O OH H2O
81. (a) R – C – OR' + 2R''MgX R – C – OH + R'OH
69. (c) CH3 C H + HCN CH3 C H ester
R''
3°alcohol
CN
Hydrolysis 82. (c) CaC2O4 is the only compound of Ca which is not soluble
CH3 CH COOH ¬¾ ¾¾
¾¾ in acetic acid. This property of Ca is used in inorganic
OH salt analysis.
2-Hydroxy propanoic acid 83. (c) Cl 2CHCOOH is most acidic because it has two chlorine
(As it has a chiral C-atom thus it is optically active) at a-position.
Aldehydes, ketones and carboxylic acids 665

84. (d) charge, destabilise the carboxylate ion and thus


85. (c) CH 3COOH + CaCO 3 ® (CH 3COO) 2 Ca decrease acidity of the parent acid.
Electronegativity decreases in order
CH3
Heat
¾¾¾
®
CH3
> CO 2 I + NaOH
¾¾¾¾¾
® CHI3 F > Cl > Br
and hence –I effect also decreases in the same order,
therefore the correct option is
Br2 / P
86. (b) RCH 2 COOH ¾¾¾¾¾¾ ® R - CH - COOH [FCH2COOH > ClCH2COOH > BrCH2COOH >
HVZ reaction | CH3COOH]
Br
94. (a) Aldehydes containing no a-hydrogen atom on
'X'
warming with 50% NaOH or KOH undergo
NH disproportionation i.e., self oxidation - reduction known
¾¾¾¾ 3 ® R - CH COOH
(Excess) | as cannizzaro’s reaction.
NH 2 2C 6 H 5CHO + NaOH ¾¾ ®
'Y' C6 H 5 COONa + C6 H 5 CH 2 OH
87. (d) Ethers neither react with sodium nor with semicarbazide. 95. (a) Aldehydes and ketones having at least one a-hydrogen
O O atom in presence of dilute alkali give b-hydroxy
P
CH3MgBr
P aldehyde or b-hydroxy ketone
88. (c) (a) C2 H5O - C - CH3 ¾¾¾¾¾ ® CH3 - C - CH3
Acetone O
||
CH3 CH3 CH3 - C + HCH 2 CHO
CH3MgBr |
(b) C=O ¾¾¾¾¾ ® CH3 OH H
H 2O
CH3 Acetaldehyde
CH3
3° Alcohol
OH
CH MgBr |
(c) CH 3 OCH3 ¾¾¾¾¾
3
® No reaction dil.NaOH
¾¾¾¾¾
® CH3 - C - CH 2 - CHO
CH MgBr |
(d) C 2 H5 OH ¾¾¾¾¾
3 ® CH 4 H
methane Aldol
CH3 D
| ¾¾
® CH3 - CH = CH.CHO
89. (d) CH3–CH–COOH - H 2O Crotonaldehyde
General : Iso-butyric acid
96. (c) This reaction is an example of Hell - Volhard Zelinsky
IUPAC: 2-methylpropanoic acid.
reaction. In this reaction acids containing a – H on
90. (c) This is Hunsdiecker’s reaction.
treatment with X2 /P give di-halo substituted acid.
COOAg Br Br /P
| | CH 3 – CH 2 COOH ¾¾¾
2 ® CH - CBr - COOH
3 2

+ Br2 ¾¾¾
4® CCl
+ AgBr + OH OH

silver benzoate CHCl CHO


bromobenzene 97. (b) (a) ¾¾¾¾

NaOH
CO2
91. (b) Oxidation of acetal aldehydes and methyl ketones with
sodium hypoiodite gives this test. So, here in option O
||
(b) ketoneishaving (CH3CO —) group and the other is (b)
NaOH
H - C - H ¾¾¾® CH3OH + HCOONa
having (CH3CH2CO—) group which do not give
hypoiodite test. So thus they can be distinguished. (c) 2CH3Cl ¾¾¾¾® CH3 – CH3
Na
Zn-Hg / conc. HCl dry ether
92. (d) C=O CH2 + H2O
Clemmensen reduction
e.g. CH Cl
(d) ¾¾¾¾¾
3 ®
CH3 Zn-Hg / CH3 anh.AlCl3
conc. HCl
C=O CH2 + H2O
CH3 CH3 Note that new C–C bond is formed in a, c and d.
98. (d) Among the given compounds only CH3OH does not
93. (c) Electron withdrawing substituent (like halogen, —NO2,
give iodoform reaction.
C6H5 etc.) would disperse the negative charge and
99. (b) Clemmensen reduction is
hence stabilise the carboxylate ion and thus increase
acidity of the parent acid. On the other hand, electron- Zn–Hg/HCl
C=O CH 2
releasing substituents would intensify the negative
EBD_7327
666 CHEMISTRY

100. (d)
CH2
On keeping O O
OH
KCN
KCN 4. (a) 3 HCHO CH2 CH2
(a) 2C6H5CHO C6H5–CH–C–C6H5
HH , CC2HH5OH
2OO OH aq. solution O
O Trioxane
(Benzoin) (meta formaldehyde)

O O 5. (b) Only aldehydes give reddish brown ppt. with Fehling’s


solution.
(b) H+
O O 6. (d) Acetone or Propyne form mesitylene (1,3,5-trimethyl
benzene) on distillation with conc. H2SO4.
H O H
7. (a) Ketones on oxidation give carboxylic acids with lesser
number of carbon atoms, i.e.,
[O]
OH OH OH OH CH 3 COCH 3 ¾¾¾® CH 3 COOH +CO 2 + H 2O
Phenolphthalein 8. (d) Structures of given aldehydes
CH 3CH 2 CHO
propanal

(c) Cl3CCHO
trichloroethanal

HCHO
methanal

CH 3CHO
K RT ethanal
70
Fries rearrangement >3

CHO

benzaldehyde
Trichloroethanal, methanal and benzaldehyde do not
undergo aldol condensation. Aldol condensation is not
given by aldehydes and ketones which do not contain
COOCH3 a-hydrogen atom(s).
OH 9. Soda -lim e(X)
(d) C 6 H 5COOH ¾¾¾¾¾¾ ® C6 H 6 + Na 2CO3 + H 2 O
(d) (Oil of wintergreen)
Zn dust (Y)
C 6 H 5 OH ¾¾¾¾¾® C6 H 6 + ZnO

OMgBr
EXERCISE - 2 |
CH3MgBr
10. (a) C6 H5 COOC2 H5 ¾¾¾¾¾
1. (a) Ethyne get converted to aldehydes and ketones when ® C6 H5 - C - OC2 H5
|
treated with an acidified aqueous solution of palladium CH3
chloride (PdCl2) and cupric chloride (CuCl2) in presence
of air or oxygen. This method is known as Wacker’s O
process. ||
- Mg(OC H )Br Excess
¾¾¾¾¾¾¾
2 5 ® C6 H5 - C - CH3 ¾¾¾¾
®
CuCl2
CH º CH + PdCl2 + H2O ¾¾¾¾¾ ® CH3MgBr
Ethyne air or O 2

OMgBr CH3
CH3CHO+ Pd + 2HCl |
ethanal H2 O
C6 H5 C CH3 C6 H5 — C — CH3
2. (d) Carbonyl compound form crystalline addition product | |
with NaHSO3 which on treatment with dilute acid or CH3 OH
alkali regenerates the carbonyl compound.
3. (a) An electron releasing substituent (+I) intensify the CH3
|
negative charge on the anion resulting in the decrease of Conc. H SO Ozonolysis
¾¾¾¾¾¾
2 4 ® C H — C = CH ¾¾¾¾¾
6 5 2 ®
stability and thus decreases the acidity of the acid. Hence D
acid character decreases as the + I-effect of the alkyl group
'B'
increases as
CH3– < CH3CH2– < CH3CH2CH2– < CH3CH2CH2CH2– C 6 H 5 COCH 3 + HCHO
Hence the order becomes : (i) > (ii) > (iii) > (iv)
Aldehydes, ketones and carboxylic acids 667

3I + 4 NaOH CF3 . COOH > MeOCH2COOH > CH3COOH


C 6 H 5 COCH 3 ¾¾2 ¾ ¾ ¾
¾® CHI 3 > (Me)2CH.COOH
11. (c) In CH 2 = CH — CHO due to — M effect of — CHO Electron withdrawing groups increase the acid strength
and electron donating groups decrease the acid strength.
group polarization of electron takes place as follows
+ – OH O OH
CH 2 = CH — C = O « CH 2 — CH = C — O | || |
| | 17. (b) CH3 - CH - CH 2 - C - CH 3 + CH 3 - C - CH 2 - CHO
H H |
Hence partial polarization is represented as CH 3
d+ d- Two products are formed.
C H 2 = CH — CH = O
18. (c) Mono-carboxylic acids are functional isomers of esters.
12. (d) Structures of given compounds
e.g., CH 3COOH HCOOCH 3
CH3CH 2OH Acetic acid Methyl formate
Ethanol
19. (c) KMnO4 converts – CH3 group of toluene into – COOH
CH3CHO while HI reduces – COOH group into – CH3 group.
Ethanal
CH3CH 2CH 2CH 2OH
Butanol
20. (c) O O
CH3CH 2CH(OH)CH3 O O
2-Butanol PhMgBr
Ph – C – C – OMgBr
CH2CH2 OH MeMgBr
O
Phenyl ethanol
OMgBr
O
P Ph – C – C – OMgBr
Compounds containing CH3 - C - or
CH3 – CHOH – group give positive iodoform test. Me O
OH +
H /H2O
13. (c) O + HN(CH3)2 N (CH3)2
OH
–H2O
N (CH3)2 Ph – C – COOH
enamine
Me
14. (a) Addition of HCN to carbonyl compounds is nucleophilic
addition reaction. The order of reactivity of carbonyl
compounds is 21. (a)
Aldehydes (smaller to higher) Ketones (smaller to higher),
Then
H
HCHO > CH3COCH3 > Ph.COCH3 > PhCOPh
The lower reactivity of Ketones is due to presence of two 22. (b)
alkyl group which shows +I effect. The reactivity of
Ketones decreases as the size of alkyl group increases. OH
O H
15. (a) CH3 CH2 C CH3 + CH3MgBr
OMgBr
hydrolysis
CH3 CH2 C CH3 OH

CH3
OH O
Br
Mg + CH3 CH2 C CH3 SOCl2
|| Benzene
OH
23. (d) CH 3 COOH ¾¾¾¾ ® CH3 - C - Cl ¾¾¾¾®
AlCl3
CH3
2-methyl-2-butanol

16. (a) The correct order of increasing acid strength


EBD_7327
668 CHEMISTRY

O OH O O
|| Na (1 mole)
C – CH3 C – CH3 27. (a) CH2 – C – CH 2 – C
HCN OEt
CN O
Å CH3– C
Na
– Cl
CH3 – C – CH – COOEt CH3 – C – CH – COOEt
OH
O O C=O
HOH C – CH3
CH3
COOH Ketonic
hydrolysis
CH3 - C - CH 2 - C - CH3
24. (a) It is an example of intramolecular cannizaro reaction. || ||
O O
CHO KOH CH2 OH 28. (c) In o - toluic acid, –CH3 group is ortho-para directing
CHO COOK CH 3 CH 3
COOH COOH
Fe
H+ CH2 OH ¾¾®
Br2
H2O COOH
Br
O and –COOH group is meta-directing. So, the resulting
|| product will be (c) in which Br is attached at para to –
25. (b) CH3COOC 2 H5 + CH 3 - C - OC 2 H 5 CH3 and meta to –COOH group.
29. (b) Generally benzene ring is very resistant to oxidation thus
O side chain is always oxidised to – COOH group. But if the
C2 H5ONa
|| side chain is tert-alkyl group, oxidation is difficult, but on
¾¾¾¾¾ ® CH 3 - C - CH 2 - COOC2 H 5 vigrous oxidation benzene ring is oxidised.
(A) CH3
CH3
+ | |
® CH3 - COCH 2 COOH
H KMnO 4
¾¾¾ C6 H5 - C - CH3 ¾¾¾¾ ® HOOC - C - CH3
b- keto acid | D |
CH3 CH 3
O
heat || PCl3
¾¾¾ ® CH3 - C - CH3 + CO2 30. (a) CH 3 - CH 2 - COOH ¾¾¾ ® CH 3 - CH 2 - CO - Cl
(I)
(B)
b-Keto acids, on heating, undergo decarboxylation. C6H6 /AlCl3
O
|| CO – CH2 – CH 3
26. (b) CH3 – C – CH 2 – CH 3 + CH3MgBr

OMgBr
|
CH3 - C - CH 2 - CH3 (II)
|
CH3 Wolff Kishner
¯ H3 O+ reduction

CH2 – CH2– CH 3
OH
H 2SO 4 /170° C
|
CH3 - C = CH - CH 3¬¾¾¾¾¾¾CH3 - C - CH 2 - CH 3
| - H 2O |
CH3 CH3
(Major)
(III)
Aldehydes, ketones and carboxylic acids 669

H NH
CH3 - CH2 - COOH ¾¾¾
3
®
(A)
[1] C - + D
||| CH3 - CH 2 - CO O N H 4 ¾¾
®
C (B)

O KOH + Br
O CH3 - CH 2 - CONH 2 ¾¾¾¾¾ 2
® CH 3 - CH 2 - NH2
31. (a) O Hoffmann
bromamide (Ethylamine)
O reaction
[3]
HO NO2
O
||
fast
[1] 38. (a) ˆˆˆ†
Ph – C –H + OH – ‡ˆˆˆ
32. (c) Only those aldehydes which do not have a-H atom – O
undergo Cannizaro’s reaction. Hence CH3CHO will not O
Ph – C – H
undergo Cannizaro’s reaction as it has 3 a H atoms. Ph – C – H slow
33. (a) OH
O O
O O–
H3O+ || |
OC2H5 OH
Ph – C + Ph – C –H
O O | |
CH2OH
½ OH H
O
CH2OH
(i) LiAlH4 O OH
OH || |
OH (ii) H3O
+
H + exchange
O
¾¾¾¾¾¾ ® Ph – C + Ph – C –H
fast | |
34. (d) O– H
CH2 – CH3 COOH O O
O
Br
¾¾¾®
2 C Heat C
(i) [O] 39. (c) Ph ¾¾¾® Ph *
¾¾¾¾¾¾¾¾¾ ® FeCl3
* OH - CO CH3
(ii) KMnO4 , KOH 2
(b - keto acid) (E)
COOH COOC2H5
C H OH O
¾¾¾¾
2 5
+
®
H I2 /NaOH
Br Br ¾¾¾¾
® Ph (–) (+)
*
ONa + CHI3
35. (d) The intermediate is carbocation which is destabilised (F) (G)
by C = O group in the first three cases. In (d),
a–hydrogen is more acidic which can be removed as 40. (c)
water. Moreover, the positive charge on th e O
(1) NaOH/Br2
intermediate carbocation is relatively away from the H3C C Hofmann
Me NH2
C = O group. NH2 Bromamide
36. (d) The reactivity of the carbonyl group toward the reaction
nucleophilic addition reactions depend upon the O
magnitude of the positive charge on the carbonyl C O
carbon atom (electronic factor) and also on the Cl
Me NH — C —
crowding around the carbonyl carbon atom in the – HCl
transition state (steric factor). Both these factors predict
the following order CO2H CO2H
CH3 CH3 Ph
C=O> C=O> C=O HNO3/H2SO4
41. (c)
H CH3 Ph
(due to steric crowding). NO2
37. (d) Since, C when heated with Br2 in presence of KOH OH OH
produces ethylamine, hence it must be propanamide (P)
and hence the organic compound (A) will be propanoic
acid. The reactions follows. (OH group is activing)
EBD_7327
670 CHEMISTRY

Reaction (III) is a Hofmann bromamide reaction. Now


OCH3 OCH3 formation of CH3CH2NH2 is possible only from a
NO2 compound CH3CH2CONH2(C) which can be obtained
HNO3/H2SO4 from the compound CH3CH2COO– NH+4 (B).
Thus (A) should be CH3CH2COOH
O
CH3 CH3 P
(Q) 3 ↑ CH CH COO, NH∗
CH3CH 2 , C, OH ¾¾¾
NH
3 2 4
(OCH3 group is more activating) (A) (B)
Χ
O ¾¾↑ CH 3CH 2 CONH 2
HNO 3/H2SO4 (C)
O
KOH Br2
CH 3CH 2 NH 2
O 46. (c) Cannizzaro reaction - when an aldehyde containing no
a – H undergo reaction in presence of 50% KOH. It
O NO2
disproportionates to form a molecule of carboxylic acid
and a molecule of alcohol.
(Benzene ring having –O– is activated) O
a – CH=O CH2–OH
OH C–O
42. (c) CH3 – CHO + HCHO ¾¾¾¾ ®
1st aldol
condensation 2 50% KOH +
a CH2 OH Cl
CH2 CHO –
Cl Cl
OH / HCHO a
CH CHO 47. (d) Anhydrous alcohols add to the carbonyl group of
2nd aldol
CH2 OH condensation aldehydes in the presence of anhydrous hydrogen
CH2 OH
chloride to form acetals via hemiacetals.
CH2OH

OH / HCHO
OC2H5
3rd aldol
HOCH2 C CHO
C 2H 5OH
condensation CH3 C CH3 CH3 C CH3
CH2OH
O OH
CH2OH
– Hemiacetal
OH / HCHO
HOCH2 C CH2OH
Cannizzaro OC2H5
reaction
CH2OH C2H5OH
CH3 C CH3
O
– OC2H5
CN
43. (a) CH3 CH2 C CH3 Acetal

O OH 48. (c)
95% 49. (d) Benzaldehyde is less reactive than ethanol towards
CH3CH2 C CH3 ¾¾¾¾
H SO
® CH3CH2 C CH3 nucleophilic attack. The combined effect of –I and +R
2 4
effect of phenyl group is electron donating which
CN COOH increases the electron density on the carbon atom of the
[G] > C = O in benzaldehyde.
D 50. (b) It is true that lower aldehyde and ketones are soluble in
¾¾
® CH3CH C CH 3 water but as the molecular mass increases their solubility
decreases. On adding Tollen’s reagent to a solution of
COOH
[H] Carbonyl compound if silver mirror is obtained than
44. (b) b-Ketoacids undergo decarboxylation easily. it is aldehyde. Therefore Tollen’s reagent is used for
the identification of aldehydes and ketones. Here,
NH
3 ® B ¾¾ D
2 ® CH CH NH Br
45. (d) A ¾¾¾ ® C ¾¾¾¾¾ 3 2 2 Statement-1 and Statement-2 both are true but the
(I) II KOH,(III)
Statement-2 is not the correct explanation of Statement-1.
Aldehydes, ketones and carboxylic acids 671

EXERCISE - 3 –+
CHO CH2OH COOK
Exemplar Questions
1. (b) But-1-yne on reaction with water in presence of Hg2+ KOH (aq )
¾¾¾¾¾ ® +
ions as a catalyst produces butan-2-one.
benzaldehyde Benzyl Potassium
CH2 CH Hg2+
CH2 CH2 alcohol benzoate
¾¾® C 8. (d) Chemical reaction can be shown as
H2 O CH3
CH3 C OH H
But–1–yne OH | |
40% H 2SO4
CH3 — C º CH ¾¾¾¾¾® CH3 — C = CH
CH2 CH3 1% HgSO 4
Tautomerism Propyne Prop-1-en-2-ol
¾¾¾¾¾¾
®
C O
CH3 ||
Keto-enol tautomerism
¾¾¾¾¾¾¾¾¾ ® CH3 — C— CH 3
O Acetone
Butan-2-one 9. (b) Chemical reaction can be shown as
2. (a) The carbonyl group in ketones being influenced by CH3CHO ¾ ¾CH
(i)MgBr H 2SO 4 ,D
¾3¾ ¾¾
( ii ) H 2O
® CH3 — CH— OH ¾ ¾ ¾ ¾ ¾®
two alkyl group is less reactive than in aldehydes where Ethanal | Dehydration
the carbonyl group is under the influence of one alkyl CH3
group only. As the number of alkyl group increases Propan-2-ol
both the +I effect and the steric hinderance get increases (A)
preventing the attack of nucleophile. Hydroboration
CH3 — C = CH2 CH3 — CH2 — CH2OH
Now among benzaldehyde and acetaldehyde former is | propan -1-ol
less electrophilic than carbon atom of carbonyl group H (C)
Propene
present in ethanal. The polarity of carbonyl group is
reduced in benzaldehyde due to resonance hence it is (B)
less reactive than ethanal. 10. (c)
3. (c) Due to the electron withdrawing characteristic of –Cl, O
||
chloroacetic acid is more acidic than acetic acid further, I2 / NaOH solution
CH3 — CH = CH — CH 2 — C— CH 3 ¾¾¾¾¾¾¾ ¾®
in phenol phenoxide ion obtained on ionisation of ( Iodoform reaction )
phenol is stabilised due to resonance where as no such Hex - 4 - en - 2 - one
stabilisation occurs in case of ethanol. Hence phenol
O
is stronger acid than ethanol. Thus acidic strength ||
increases in the order. CH3 — CH = CH — CH 2 — C— OH
ethanol < phenol < acetic acid < chloro acetic acid Pent -3-en -1-oic acid

4. (b) Compound Ph — COO — Ph can be prepared by the 11. (b) 2° alcohols on oxidation with alkaline KMnO4 solution
reaction of phenol & benzoyl chloride produce ketones.

[O ] CH3
O ¾¾¾¾®
|| KMnO 4

O C O
|| OH Butan-2-one
OH O Butan-2-ol
Cl N (2° alcohol)
+
12. (a) Clemmensen reduction is used to convert carbonyl
Benzoyl
Phenol
chloride group as follows
Zn (Hg )+ HCl
This an example of Schotten-Baumann reaction. C = O ¾¾¾¾¾¾ ® CH2
5. (c) Acetone and benzaldehyde both do not react with Zinc amalgam and HCl act as reagent in this reaction.
Fehling's solution.
NEET/AIPMT (2013-2017) Questions
6. (d) Cannizzaro reaction is given by those aldehydes which
have absence of a-hydrogen atom. So, CH3CHO will 13. (a) Enolic form predominates in compounds containing two
not give Cannizzaro reaction. carbonyl groups separated by a – CH2 group. This is
due to following two factors.
7. (b) Benzaldehyde having no a-hydrogen on reaction with (i) Presence of conjugation which increases stability.
aqueous KOH solution undergo Cannizzaro reaction (ii) Formation of intramolecular hydrogen bond
and produces benzyl alcohol and potassium benzoate. between enolic hydroxyl group and second
EBD_7327
672 CHEMISTRY

carbonyl group which leads to stablisation of the


O
molecule. Hence the correct answer is III > II > I. ||

14. (c) Zn/Hg and conc. HCl reduce carboxyl group to 20. (b) R — C— R1 + NH 2 — NH 2 ¾¾¾
®
methylene group (Clemmensen reduction).
15. (d) Any substituent in the carbonyl compound that
increases the positive charge on the carbonyl carbon NH2
will increase reactivity towards nucleophilic addition. OH N
–NO2 shows –M effect hence R—C—R
1
¾¾¾¾¾
2 H O
® R—C—R¢
Elimination
CHO
NH—NH2
is most reactive towards nucleophilic addition (Addition)
While in all other case no elimination take place.
21. (d) Keto-enol tautomerism is possible only in those
NO 2 aldehydes and ketones which have at least one a-
reaction.
hydrogen atom, which can convert the ketonic group
16. (b) H3C—CH2— C—CH2—CH3
to the enolic group. e.g.
O O O
does not give iodoform test due to absence of || ||
O CH3 — C — CH2 — C CH3
Ketonic form
(CH3—C—) group. It also does, not give Tollen’s test.
On reduction it gives n-pentane OH O
reduction
| ||
H3C—CH2—C—CH2—CH3 CH 3 — C CH — C — CH 3

||
Zn-Hg/HCl

O or NH2-NH2/OH enolic form
CH 3—CH 2—CH2—CH 2—CH 3 22. (a) Schiff base is formed when 1° amine reacts with
n-pentane
aldehydes.
17. (d) –
O O R R
CH3 H+
C = O + R' — NH 2 ¾¾¾ ® C = N – R'
– Å H H
+ CH3 Li ® Aldehyde + primary amine Schiff base
23. (b) Since 'A' gives positive silver mirror test therefore, it
Cylopentanoyl anion
must be an aldehyde of a-Hydroxyketone. Also, reaction
18. (d) Enolic form of ethyl acetoacetate has 18 sigma and 2 pi-
with OH– i.e., aldol condensation (by assuming alkali to
bonds as shown below:
be dilute) indicates that A is aldehyde as aldol reaction
H H
s s
of ketones is reversible and carried out in special
s s s s apparatus. It indicates that A is an aldehyde
H C C p C s
C p
O
s s H H Cu
s
s s CH 3 - CH 2 OH ¾ ¾ ¾® CH3 - CHO
H Os H s
573K
Os C s C H
(X)
+
s s é Ag(NH3 ) ù OH
H H ¾ ë¾ ¾ ¾ ¾ 2û
¾ ¾® silver mirror observed
D
19. (a) Among the substituent attached to the benzene ring, – (A)
NO2 group is the most electron withdrawing, thus ethanal
withdraws electron density from carbonyl carbon thus
– O
facilitate the attack of OH ion. OH
H 2 N - NH - C - NH 2 OH ,D

|
O O CH3 - CH - CH 2 - CHO
. . || || 3-Hydroxybutanal
O – C – CH3 O – C – CH3
..

O D
CH3 - CH = N - NH - C - NH 2 CH3 - CH = CH - CHO
(Z) (Y)
Semicarbazone But-2-enal
O O
N N
– – – H s
O O O O (i) OH
+H
||

(–R effect of –NO2 group) 24. (a) (ii) D


O
Amines 673

27 Amines

(A) AMINES (ii) By the Reduction of Nitriles: (Mendius


They are derivatives of ammonia and are obtained by reduction)
replacement of one, two or all three H-atoms of NH3 by alkyl Raney Ni / H or LiAlH
R / Ar – CN ¾¾¾¾¾¾¾¾® 2 4 R / Ar – CH NH
or aryl groups. or Na ,C H OH 2 5 2 2

For Ex: CH3NH2, CH3 – NH – CH3, etc.


Raney Ni / H 2
R – N º C ¾¾¾¾¾® R – NH – CH3

STRUCTURE OF AMINES (iii) Ammonolysis of Alkyl Halides (Hofmann’s


N in amines is sp3 - hybridised and the geometry of amines is Method) :
pyramidal. D , 100ºC
D
R–X + H - NH 2 ¾¾¾¾® R–NH2HX ¾¾ ® R–NH2
The fourth orbital of N contains an unshared pair of e–s, due (Excess) Pressure Alkali
to which the bond angle decreases from the tetraheral angle On taking the alkyl halides in excess, the primary amines gets
of 109.28¢ to 107° in 1° and 2° amines. However in 3° amines further alkylated to form 2º & 3º amines and finally quaternary
bond angle increased upto 108° due to steric hindrance. ammonium salt.

RX RX RX Å s
CLASSIFICATION OF AMINES R–NH2 ¾¾¾ ® R2NH ¾¾¾ ® R3N ¾¾¾ ® R4 N X
D D
Amines are classified as 1°, 2° and 3° depending upon the Note:
number of H-atoms replaced by alkyl or aryl groups in NH3.
(a) This reaction is an example of a nucleophilic substitution
reaction in which ammonia molecule (NH3) acts as a
nucleophile.
(b) The order of reactivity of haloalkanes in ammonolysis
reaction is RI > RBr > RCl.
(c) This method cannot be used for preparing aryl amines
Amines are said to be simple if all the alkyl or aryl groups are because simple aryl halides do not undergo nucleophilic
same and mixed if they are different. substitution easily.
(iv) Reduction of amides
PREPARATION OF AMINES (i)LiA1H / ether
RCONH 2 ¾¾¾ ¾¾
4 ¾® RCH 2 NH 2
(i) Reduction of Nitro Compounds (ii)H O 2

(v) Hoffmann Bromamide Degradation


RCONH2 + Br2 + 4NaOH ®
R — NH2 + Na2CO3 + 2NaBr + 2H2O
(vi) By Oximes:
Note: R LiAlH R
C = N – OH ¾¾¾¾

CH – NH2 + H2O
The yield can be increased to about 90% by using more R 4H R
powerful reductant like LiAlH4 or NaBH4.
(Ketoxime) (2° amine)
EBD_7327
674 CHEMISTRY

(vii) By Reductive Amination of carbonyl compounds :


H2
R – CH = O + H2 NH ¾¾¾®
-H O
R – CH = NH ¾¾
Ni
® R – CH2 – NH2
2
An aldimine
H2
R2C = O + H2NH ¾¾¾®
-H O
R2C = NH ¾¾
Ni
® R2CH2 – NH2
2
A ketimeine

(viii) Gabriel Phthalimide Synthesis

O O O
+ COOH
C KOH C RX C HOH/H
NH NK NR – +RNH2
C alc C (– KX) C or OH
(–H2O) COOH
O O O
Phthalimide Pot. phthalimide N-alkyl phthalimide Phthalic acid 1º amine

(ix) Ammonolysis of Alkyl Halides (Hofmann’s Method) :


D , 100ºC
D
R–X + H - NH 2 ¾¾¾¾® R–NH2HX ¾¾ ® R–NH2
Pressure Alkali
(Excess)
On taking the alkyl halides in excess, the primary amines gets further alkylated to form 2º & 3º amines and finally quaternary
ammonium salt.

RX RX RX Å Q
R–NH2 ¾¾¾ ® R2NH ¾¾¾
® R3N ¾¾¾ ® R4 N X
D D
This reaction is an example of a nucleophilic substitution reaction in which ammonia molecule (NH3) acts as a nucleophile.The
order of reactivity of haloalkanes in ammonolysis reaction is RI > RBr > RCl. This method cannot be used for preparing aryl amines
because simple aryl halides do not undergo nucleophilic substitution easily.
Note:
(a) In reduction of nitro compounds, Fe + HCl is preferred because FeCl 2 formed gets hydrolysed to release HCl during the reaction.
Thus, only a small amount of HCl is required to initiate the reaction.
(b) Ammonolysis yields a mixture of 1°, 2°, 3° amines and quaternary ammonium salt. 1° amine is obtained as a major product by taking
large excess of ammonia.
(c) Reduction of nitriles to amines results in ascent of amine series i.e. for preparation of amine with one carbon atom more than the
starting amine.
(d) The amines formed by Hofmann bromamide degradation method contain one carbon atom less than the parent amide.
PHYSICAL PROPERTIES OF AMINES
(i) Pure amines are almost colourless but develop colour on keeping in air due to atmospheric oxidation.
(ii) Lower aliphatic amines are gases with fishy odour. 1° amines with 3 or more C-atoms are liquid and higher ones are solid.
(iii) Lower aliphatic amines are soluble in H2O due to H–bonding. Solubility decreases with increase is size of hydrophobic alkyl part
Alcohols are more soluble than amines in H2O.
(iv) Boiling point : b.p. of alcohols & carboxylic acids > b.p. of amines > b.p. of alkanes and, b.p. of 1° amine > 2° amine > 3° amine
Explaination :
(a) Primary and secondary amines form inter molecular H-bonds, while tertiary amines do not form inter-molecular H-bonds. As
a result, 1° & 2° amines show high B.P. than 3° amines.
(b) H-bonding in amines is through nitrogen atoms while in alcohols and carboxylic acids, it is through oxygen atoms and
nitrogen is less electronegative than oxygen. So H-bonding in amines is weaker than that in carboxylic acids and alcohols.

CHEMICAL PROPERTIES OF AMINES


Amines are reactive due to (i) difference in electronegativity between N and H, (ii) presence of lone pair of e–1 s of N.
Amines 675

Chemical reactions are as follows:

Basic Character
(i)

(ii)

(iii)
These amine salts are ionic compounds.
(iv)

Reactions With Electrophiles


(i)

+ – + –
[R4N ]X + AgOH [R4N ]OH + AgX

(ii)

¢
¢ ¢
¢

(iii)

(iv) Schotten- Baumann reaction


chloride
(v)
(alc.)
EBD_7327
676 CHEMISTRY

Miscellaneous Reactions
(i)

(ii)
(iii)
Amines 677

BASIC CHARACTER OF AMINES


(i) Amines have an unshared pair of e–1 s on nitrogen due to -C(CH 3 )3 NH3 >tert.Bu.NH 2 > tert. Bu 2 NH> tert. Bu 3 N
which they behave as Lewis base.
(vii) An electron releasing group such as –CH3, – OH, etc., present
+ in the benzene ring at ortho and para positions increases the
(ii) RNH 2 + H 2O ƒ RNH3 + OH - basic strength of aromatic amines. This is due to the
stabilization of the ammonium cation formed after protonated
é + ù é + ù of amine.
- -
ê RNH3 ú éê OH ùú ê RNH 3 ú éê OH ùú (viii) An electron withdrawing group such as –NO2, –CN, –SO3
ê ë û ë û
K eq = ë ûú ê
Þ Keq [H 2O] = ë ûú
H, –X, etc. present in the benzene ring at ortho and para
[ RNH 2 ] [ H 2O] [ RNH 2 ] positions decreases the basic strength of aromatic amines.
This due to the destabilization of the ammonium cation
é + ù - formed after protonation of amine.
ê RNH3 ú éêOH ùú
ê úû ë û Note:
Þ Kb = ë (Keq[H2O] = Kb)
[ RNH 2 ] (a) Carbylamine reaction is used to distinguish between 1°
(aliphatic and aromatic) and 2° / 3° amines.
also, pKb = – log Kb. (b) Reaction with HNO2 is used to distinguish between 1°
Larger the value of Kb or smaller the value of pKb, stronger is aliphatic and 1° aromatic amines.
the base. (c) In aniline, – NH2 is a strong activating group. Thus, it gives
(iii) Aliphatic amines are stronger bases than NH3 due to trisubstituted bromine derivative. To avoid this, the –NH2
+ I - effect of alkyl groups leading to high e– – density on group is protected by acetylation and then carrying out the
N – atom. desired substitution. Electron attracting group (>C=O) present
(iv) Aromatic amines are weaker bases than NH3 due to e– with - in acetanilide decreases availability of lone pair on nitrogen
drawing nature of aryl group resulting is decreased e– density atom and hence decreases electron density on benzene ring
on N. of aniline.
(v) The order of basicity of amines in gaseous phase is: (d) Aniline does not undergo Friedel - Crafts reaction due to salt
formation with AlCl3 (a Lewis acid) which is used as a
3° amine > 2° amine > 1° amine
catalyst.
This is because with increase in the number of alkyl groups
+ I - effect increases and thus e– – density on N - atom also H H
increases. | |Å –
C6 H5 - N : + AlCl3 ¾¾® C6H5 – N – AlCl3
(vi) In aqueous solution, besides I-effect, solvation and steric | |
effects also play a role. Thus in aqueous medium, basic H H
strength increases. Order for different alkyl groups is:
Alkyl group Relative strength
-CH 3 Me 2 NH > MeNH 2 > Me 3 N > NH 3

-C 2 H 5 Et 2 NH > Et NH 2 > NH 3 > Et 3 N

-CH(CH 3 ) 2 Iso Pr .NH 2 > NH 3 > Iso Pr2 NH > Iso Pr3 N
EBD_7327
678 CHEMISTRY

DISTINCTION BETWEEN 1°, 2° AND 3° AMINES


(i) Hinsberg’s Test:
Hinsberg’s reagent is benzene sulphonyl chloride (C6H5SO2Cl).

(ii) Carbylamine or Isocyanide Test


Heat
R — NH2 + CHCl3 + 3KOH ¾¾¾ ® R — NC + 3KCl + 3H2O
Shown only by 1°, 2° and 3° amines do not show this reaction.

(iii) Nitrous Acid Test:

1° amine 2° amine 3° amine


R2NH or Ar2NH
Aliphatic HONO Aliphatic aromatic
aromatic
They react with They react with R2/Ar2 – N – N = O R3N + HNO2 Ar – N – R2+ HNO 2
HNO2 at HNO2 at (yellow oil)
273 – 278 K 273 – 278 K to
to form D, C6H5OH + –
form diazonium Conc. H2SO4 [R3N H ]NO2 Green
alcohols salt which couple
Green solution coloured
with brisk with alk. b -naphthol Water
p-nitrosoamines
effervescence to form orange soluble
due to evolution or red coloured aq NaOH nitrite
of N2 gas dye dilute salts
Deep blue red
Note: This is Liebermann’s
nitroso test and distinguishes 2°
amines from 1° and 3° amines
Amines 679

(B) DIAZONIUM SALTS

They have the general formula RN +2 X - where R ® aryl group X - ® Cl- , Br - , HSO -4 , BF4-
The stability of arenediazonium ion is explained on the basis of resonance:

PREPARATION OF DIAZONIUM SALT


273 - 278 K
C6 H5 NH 2 + NaNO 2 + 2HCl ¾¾¾¾¾® C6 H 5 N +2 Cl – + NaCl + 2 H 2O

The process is called diazotisation. Due to its instability, diazonium salt is not stored and is used immediately after its preparation.

PHYSICAL PROPERTIES OF DIAZONIUM SALTS


(i) It is a colourless crystalline solid.
(ii) Soluble in water, stable in cold reacts with warm H2O.
(iii) Decomposes in dry state.
(iv) Benzediazonium fluoroborate is water insoluble and stable at room temperature.
EBD_7327
680 CHEMISTRY

CHEMICAL PROPERTIES OF DIAZONIUM SALTS

(a)

(b)

273-278K

>283K

Note: Conversion of diazonium salt to haloarenes by suing CuX/HX is called sandmeyer reaction whereas if Cu/HX is used reaction is
called Gattermann reaction.
CONCEPT MAP
Amines
681
EBD_7327
682 CHEMISTRY

1. Acetaldoxime reacts with P2O5 to produce OH



R
(a) ethyl cyanide (b) propyl cyanide 10. C=N [X] , the product and name of
(c) methyl cyanide (d) none of these R'
2. Acetamide and ethylamine can be distinguished by reacting this reaction is –
with
O
(a) aqueous HCl and heat ||
(b) aqueous NaOH and heat (a) R - C - NHR ' ; Pinnacole rearrangement
(c) acidified KMnO4
(d) bromine water. O
||
3. The compound obtained by heating a mixture of a primary (b) R ¢ - C - NHR ; Fries rearrangement
amine and chloroform with ethanolic potassium hydroxide
(KOH) is O
||
(a) an alkyl cyanide (c) R - C - NHR ¢ ; Beckmann's rearrangement
(b) a nitro compound
(c) an alkyl isocyanide O
||
(d) an amide (d) R - C - NHR ' , Allylic rearrangement
4. Which of the following reagents will convert 11. Which of the following reactions can produce aniline as
p-methylbenzenediazonium chloride into p-cresol? main product?
(a) Cu powder (b) H2O (a) C6H5NO2 + Zn/KOH
(c) H3PO2 (d) C6H5OH (b) C6H5NO2 + Zn/NH4Cl
5. C3H9N can not represent the following (c) C6H5NO2 + LiAlH4
(a) primary amine (d) C6H5NO2 + Zn/HCl
(b) secondary amine
(c) tertiary amine 12. R - NH 2 + CH3COCl ¾¾ ®A.
(excess)
(d) quaternary ammonium salt
6. Which one of the following methods is neither meant for the The product (A) will be –
synthesis nor for separation of amines? (a) RNHCOCH3 (b) RN(COCH3)2
(a) Curtius reaction (b) Wurtz reaction +
(c) RN(COCH3 )3 Cl- (d) R – CONH2
(c) Hofmann method (d) Hinsberg method
7. Benzamide on reaction with POCl3 gives NH2
(a) aniline (b) chlorobenzene
(c) benzylamine (d) benzonitrile NaNO /HCl
2 CuBr
13. ¾¾¾¾¾¾
® (P) ¾¾¾® (Q)
8. Which of the following will produce isopropyl amine – 0° C HBr

NH 2OH LiAlH4
(I) (CH 3 ) 2 CO ¾¾¾¾ ® X ¾¾¾¾ ® The compound Q is –
NH3 LiAlH 4 (a) bromo benzene (b) chlorobenzene
(II) CH 3 - CH 2 - CHO ¾¾¾® X ¾¾¾¾
®
(c) benzyl bromide (d) benzyl chloride
heat
NH3 14. Identify Z in the following sequence of reactions –
(III) (CH3 ) 2 CH - OH + PCl5 ¾¾
® X ¾¾¾ ®
D H 2O/H P O Å
CH 3 - COONH 4 ¾¾ 2 5® Y
® X ¾¾¾
heat
(IV) CH 3 - CH 2 - CH 2 - NH 2 ¾¾¾ ¾¾¾¾¾ ®Z
®
(a) CH3 – CH2 – CO – NH2 (b) CH3 – CN
(a) I, II (b) II, III (c) (CH3CO)2O (d) CH3 – COOH
(c) I, III (d) IV only 15. Among the following which one does not act as an
O
|| intermediate in Hofmann rearrangement ?
9. Reaction of R - C - NH 2 with a mixture of Br2 and KOH gg
(a) RNCO (b) RCON
produces RNH2. During the reaction, which of the intermediate
gg
product is formed : (c) RCONHBr (d) RNC
(a) R – NH – Br (b) H – CO – NBr 2 16. Propionamide on Hofmann degradation gives –
(c) R – N = C = O (d) all of these (a) methyl amine (b) ethyl amine
(c) propyl amine (d) ethyl cyanide
Amines 683

17. The basicity of aniline is less than that of cyclohexylamine. 29. In the following sequence of reactions, the compound C
This is due to formed would be
(a) + R effect of —NH2 group HNO [O]
(b) – I effect of —NH2 group CH3 - CH - CH3 ¾¾¾¾ 2 ® A ¾¾¾
® B ¾¾¾¾
CH 3MgI
¾®C
H 2O
(c) – R effect of —NH2 group H + / H 2O
|
(d) hyperconjugation effect NH 2
18. When phenol and benzene diazonium chloride are coupled, (a) 1,1-dimethyl ethanol
the main product is : (b) butanol-1
(a) aniline (b) p-hydroxyazobenzene (c) butanol-2
(c) azobenzene (d) chlorobenzene (d) 2-methyl-propanol-1
19. Ethylamine can be prepared by the action of bromine, caustic
NH PO PO
potash and : 30. CH 3COOH ¾¾¾3 ® A ¾¾2¾5¾® B ¾¾2¾5¾® C
Heat ( - H 2O ) ( - H 2 O)
(a) acetamide (b) propionamide
The compound 'C' is :
(c) formamide (d) methyl cyanide
(a) acetamide (b) ammonium acetate
20. The number of primary amines of formula C4H11N is : (c) methyl cyanide (d) aniline
(a) 1 (b) 3 (c) 4 (d) 2 31. Azo dye is prepared by the coupling of phenol and
21. In the diazotization of arylamines with sodium nitrite and (a) diazonium chloride
hydrochloric acid, an excess of hydrochloric acid is used (b) o-nitroaniline
primarily to (c) benzoic acid
(a) Supress the concentration of free aniline available for (d) chlorobenzene
coupling 32. Nitrosoamines (R2N – N = O) are soluble in water. On heating
(b) Supress hydrolysis of phenol them with concentrated H2SO4, they give secondary amines.
(c) Ensure a stoichiometric amount of nitrous acid This reaction is called
(d) Neutralise the base liberated (a) Perkin reaction
22. p-Chloroaniline and anilinium hydrogen chloride can be (b) Sandmeyer's reaction
distinguished by : (c) Fitting reaction
(a) Sandmeyer reaction (b) Carbylamine reaction (d) Liebermann nitroso reaction
(c) Hinsberg’s reaction (d) AgNO3 33. Primary amines react with benzaldehyde to form:
23. The conversion of acetophenone to acetanilide is best (a) diazonium salts (b) azo dyes
(c) anilides (d) schiff’s base
accomplished by using :
34. In the reaction sequence
(a) Beckmann rearrangement
NH2
(b) Curtius rearrangement
(c) Lossen rearrangement NaNO ,HCl CuCN
(d) Hofmann rearrangement ¾¾¾¾¾
2 ® A ¾¾¾¾
¾ ®B
0° C
24. High basicity of Me 2 NH relative to Me3N is attributed to : LiAlH
¾¾¾¾ 4 ® C, the product ‘C’ is:
(a) effect of solvent (b) inductive effect of Me
(c) shape of Me2NH (d) shape of Me3N (a) benzonitrile (b) benzaldehyde
25. In the reaction, (c) benzoic acid (d) benzylamine
35. The end product (Y) in the reaction sequence
HNO
RNH 2 ¾¾¾¾
2
® A + B + C ­ ; C is PO Sn / HCl
CH 3 CONH 2 ¾¾¾
2 5 ® X ¾¾¾¾® Y , is :
(a) NH3 (b) N2 (c) O2 (d) CO2 D
(a) ethane nitrile (b) acetic acid

N2+ Cl Cl
(c) ethanamine (d) chloroethane
+ N2
36. The structural formula of methyl aminomethane is
26. Conc. HCl (a) (CH3)2CHNH2 (b) (CH3)3N
+ Cu2Cl2 ¾¾¾¾
¾®
(c) (CH3)2NH (d) CH3NH2
Chlorobenzene
37. The end product of the reactions is
Above reaction is known as:
HNO PCl H.NH
(a) Strecker's reaction (b) Sandmeyer's reaction C2 H5 NH 2 ¾¾¾ 2
® A ¾¾¾ 5
® B ¾¾¾2¾ ®C
(c) Wohl-Ziegler reaction (d) Stephen's reaction (a) ethyl cyanide (b) ethyl amine
27. Nitrobenzene and hydrogen in presence of zinc combines to (c) methyl amine (d) acetamide
form : 38. Which of the following is strongest base :
(a) azobenzene (b) benzene
(c) azoxybenzene (d) aniline (a) (b)
28. An organic amino compound reacts with aqueous nitrous N N
acid at low temperature to produce an oily nitrosoamine. The NH3
compound is
(a) CH3NH2 (b) CH3CH2NH2 (c) (d)
(c) CH3CH2NHCH2CH3 (d) (CH3CH2)3 N N
EBD_7327
684 CHEMISTRY

39. Aniline when treated with conc. HNO3 gives 48. Ethyl isocyanide on hydrolysis in acidic medium generates
NH2 NH2 (a) propanoic acid and ammonium salt
NO2 NH2 (b) ethanoic acid and ammonium salt
NO2 (c) methylamine salt and ethanoic acid
(a) (b) (d) ethylamine salt and methanoic acid
49. What will be the final product in the following reaction
NH2 sequence –
O
NH2 2 H+ /H O
3 ® B ¾¾¾¾ NH NaOBr
CH3CH2CN ¾¾¾¾® A ¾¾¾ ®C
D
(c) (d) (a) CH3CH2CONH2 (b) CH3CH2COBr
(c) CH3CH2NH2 (d) CH3CH2CH2NH2
O 50. In the reaction :
40. If methyl is alkyl group, then which order of basicity is correct
(a) R2NH > RNH2 > R3N > NH3 C6 H 5 CHO + C6 H 5 NH 2 ¾¾
®
(b) R2NH > R3N > RNH2 > NH3
(c) RNH2 > NH3 > R2NH > R3N C6 H5 N = CHC6H5 + H2O
(d) NH3 > RNH2 > R3NH > R3N The compound, C6H5N = CHC6H5 is known as
41. Among the following, the strongest base is (a) aldol (b) Schiff’s base
(a) C6H5NH2 (b) p – NO2C6H4NH2 (c) Schiff’s reagent (d) Benedict’s reagent
(c) m – NO2 C6H4NH2 (d) C6H5CH2NH2 51. Methyl cyanide is less basic than methylamine because:
42. A primary amine is formed an amide by the treatment of (a) there is a triple bond between carbon and nitrogen atoms
bromine and alkali. The primary amine has : (b) molecular weight is higher than methylamine
(a) 1 carbon atom less than amide
(c) the lone pair of electrons in nitriles belongs to sp-orbital
(b) 1 carbon atom more than amide
and lone pair of electrons in amines belongs to sp3 -orbital
(c) 1 hydrogen atom less than amide
(d) 1 hydrogen atom more than amide (d) None of these
43. Benzene diazonium chloride reacts with hypophosphorous X
52. In the reaction : R - C º N + 4(H) ¾¾® RCH 2 NH 2
acid to produce:
(a) phenol X can be :
(b) benzene (a) LiAlH 4 (b) H 2SO 4 (c) Ni (d) 2KBr
(c) p-hydroxyazobenzene 53. Aromatic nitriles (ArCN) are not prepared by reaction :
(d) benzonitrile
44. Nitration of aniline also gives m-nitro aniline, in strong acidic (a) ArX + KCN (b) ArN+2 + CuCN
medium because (c) ArCONH2 + P2O5 (d) ArCONH2 + SOCl2
(a) in electrophilic substitution reaction amino group is meta
directing O
|| POCl 3
(b) inspite of substituents nitro group always goes to 54. R – NH – C – H ¾¾¾¾ ® Product
Pyridine
m-position
(c) in strong acidic medium, nitration of aniline is a In the given reaction what will be the product
nucleophilic substitution reaction +
(d) in strong acidic medium aniline present as anilinium ion (a) R – N = C = O (b) R – N º C-
45. Which is formed, when acetonitrile is hydrolysed partially (c) R – C º N (d) None of these.
with cold concentrated HCl? 55. When primary amine is heated with CS2 in presence of excess
(a) Acetic acid (b) Acetamide mercuric chloride, it gives isothiocyanate. This reaction is
(c) Methyl cyanide (d) Acetic anhydrides called
46. HgCl
C 6 H 5 NH 2 + CS 2 ¾¾¾¾2 ® C H NCS + HgS + 2HCl (a) Hoffmann bromide reaction
6 5
(b) Hoffmann mustured oil reaction
the above reaction is:
(a) Libermann’s nitroso reaction (c) Carbylamine reaction
(b) Carbylamine reaction (d) Perkin reaction
(c) Hinsberg’s reaction 56. Hydrolysis of phenyl isocyanide forms
(d) Hoffmann mustard oil reaction (a) Benzoic acid (b) Formic acid
47. Consider the following sequence of reactions : (c) Acetic acid (d) None of these
Reduction HNO 57. Nitrobenzene on treatment with zinc dust and aqueous
Compound[A] ¾¾¾¾¾
® [B] ¾¾¾¾
2 ® CH CH OH
3 2
ammonium chloride gives:
The compound [A] is
(a) CH3CH2CN (b) CH3NO2 (a) C6 H5 N = NC6 H5 (b) C 6 H 5 NH 2
(c) CH3NC (d) CH3CN (c) C 6 H 5 NO (d) C 6 H 5 NHOH
Amines 685

58. n-Propylamine yields a volatile compound X on warming with 67. Nitrobenzene, on reaction with fuming nitric acid at 90°C, gives
alc. alkali and chloroform. X has an offensive odour. NO2 NO2
The structure of X is
(a) CH3CH2CH2CN (b) (CH3)2 CHCN NO2
(a) (b)
(c) CH3CH2CH2NC (d) (CH3)2 CHNC NO2
X
59. CH 3 - CH 2 C º N ¾¾® CH 3CH 2CHO. Th e compound NO2 NO2
X is NO2
(a) SnCl2 / HCl / H2O, boil (b) H2 / Pd – BaSO4 (c) (d)
(c) LiAIH4 / ether (d) NaBH4/ ether / H3O+
60. Benzamide and benzyl amine can be distinguished by NO2 NO2
(a) cold dil. NaOH 68. In the series of reaction
(b) cold dil. HCl NaNO / HCl HNO
(c) both a & b C6 H 5 NH 2 ¾¾¾¾¾
2
0–5°C
® X ¾¾¾
H O
2
® Y + N 2 + HCl X and
2
(d) NaNO2, HCl, 0ºC, then b-naphthol Y are respectively
61. Acetanilide on nitration followed by alkaline hydrolysis mainly
gives – (a) C 6H 5 N = N C 6 H 5 ,C 6 H 5 N 2Å Cl –
(a) o-Nitroaniline (b) p-Nitroaniline (b) C6 H5 N 2Å Cl,– C6 H5 – N = N – C6 H 5
(c) m-Nitroaniline (d) 2, 4, 6-Trinitroaniline
(c) C6 H 5 N Å –
2 Cl , C6 H 5 NO 2
62. In the reaction :
NO 2 NH 2 (d) C6H5NO2,C6H6

X 69. A primary amine that can be obtained both by the reduction


; X is of cyanides and amides is
NO 2 NO 2 (a) methyl amine (b) benzyl amine
(a) SiC (b) H 2SO 4 (c) aniline (d) iso-propyl amine
(c) KMnO4 (d) Fe + HCl NO2
63. Amine that cannot be prepared by Gabriel phthalimide Conc. H SO
70. ¾ ¾ ¾ ¾2 ¾ 4¾® A Product ‘A’ in above reaction is
synthesis is D
(a) aniline (b) benzylamine
(c) methylamine (d) iso-butylamine NO2 NO2
64. Which of the following will be easily nitrated? SO3H
CH3 (a) (b)
SO3H
(a) (b) NO2

(c) CH3NO2 (d) C6H5NO2 (c) (d) None of these


65. Nitrobenzene is reduced to azoxybenzene using
(a) Sn/HCl (b) Zn/NH4Cl SO3H
(c) As2O3/NaOH (d) Zn/NaOH 71. Anilinium hydrogensulphate on heating with sulphuric acid
66. Product of the reaction at 453-473 K produces
NH2 (a) benzene sulphonic acid (b) anthranilic acid
(c) aniline (d) sulphanilic acid
72. Which one of the following on reduction with lithium
KNH 2 / liq. NH3
aluminium hydride yields a secondary amine?
NH2 ¾¾¾¾¾¾¾ ® ? is (a) Methyl isocyanide (b) Acetamide
(c) Methyl cyanide (d) Nitroethane
73. Nitrobenzene can be prepared from benzene by using a mixture
of conc. HNO3 and conc. H2SO4 in the mixture, nitric acid
(a) (b) acts as a/an :
(a) acid (b) base
(c) catalyst (d) reducing agent
74. Acetamide is treated with the following reagents separately.
NO NH2 Which one of these would yield methylamine?
NO NH2 (a) NaOH – Br2 (b) Sodalime
(c) (d)
(c) Hot conc. H 2SO4 (d) PCl5
EBD_7327
686 CHEMISTRY

75. In the chemical reactions : the compounds A and B respectively are :


(a) benzene diazonium chloride and benzonitrile
NH2
(b) nitrobenzene and chlorobenzene
(c) phenol and bromobenzene
NaNO CuCN
¾¾¾¾¾ 2 ® A ¾¾¾¾® B, (d) fluorobenzene and phenol
HCl, 278K D

1. Aniline is treated with bromine water to give an organic (c) p-bromoaniline


compound ‘X’ which when treated with NaNO2 and HCl at (d) 2,4, 6-tribromofluorobenzene
0° C gives a water soluble compound ‘Y’. Compound ‘Y’ on 6. Choose the amide which on reduction with LiAlH4 yields a
treatment with Cu2Cl 2 and HCl gives compound ‘Z’. secondary amine
Compound ‘Z’ is (a) Ethanamide (b) N-Methylethanamide
(a) o-bromochlorobenzene (c) N, N-dimethylethanamide (d) Phenylmethanamide
(b) p-bromochlorobenzene
reduction CHCl + KOH
(c) 2, 4, 6-tribromophenol 7. [A ] ¾¾ ¾ ¾®[B] ¾¾ ¾3¾ ¾¾®
(d) 2, 4, 6-tribromochlorobenzene reduction
[C] ¾¾ ¾ ¾® N - Methylanil ine , A is
2. Which one of the following statements is not correct?
(a) pKb of aniline is greater than that of methyl amine (a) Formaldehyde (b) Trichloromethane
(b) Aniline does not undergo Friedel-Crafts reaction (c) Nitrobenzene (d) Toluene
(c) Dimethyl amine is capable of forming intermolecular 8. Intermediates formed during reaction of R C NH 2 with Br2
hydrogen bonding ||
and KOH are O
(d) NH3 is a stronger base than ethanamine (a) RNHBr and RCONHBr
3. When an iline reacts with oil of bitter almonds
(b) RNHCOBr and RNCO
(C 6 H 5CHO ) condensation takes place and benzal derivative
(c) RCONHBr and RNCO
is formed. This is known as
(a) Million's base (b) Schiff's reagent (d) RCONBr2
(c) Schiff's base (d) Benedict's reagent
4. Aniline is an activated system for electrophilic substitution. C N
The compound formed on heating aniline with acetic
anhydride is H O+
9. + CH3MgBr Q ¾¾3 ¾® P
NH2 NH2
OCH3

(a) (b) Product 'P' in the above reaction is


COCH3
OH O
COCH3
CH – CH3 C – CH3
NH2 NHCOCH3

(c) (d) (a) (b)


COCH3 OCH3 OCH3
5. Aniline is reacted with bromine water and the resulting
product is treated with an aqueous solution of sodium nitrite CHO COOH
in presence of dilute hydrochloric acid. The compound so
formed is converted into a tetrafluoroborate which is
subsequently heated dry. The final product is (c) (d)
(a) 1,3, 5-tribromobenzene OCH3 OCH3
(b) p-bromofluorobenzene
Amines 687

10. The final product C, obtained in this reaction O – NHCH3


NH2 (a)

Ac2O Br2 H2O


A B + C
CH3COOH H
(b) NCH3
CH3 H
would be
H
NHCOCH3 NH2 NCH3
(c)
Br COCH3
(a) (b) OH

H
CH3 CH3 NCH3
(d)
OH
COCH3 NH2
Br 15. Among the following amines, which one has the highest
Br
pKb value in aqueous solution?
(c) (d)
(a) Methanamine (b) N,N-Dimethylaniline
CH3 CH3 (c) Ethanamine (d) Benzenamine
16. The major product (70% to 80%) of the reaction between
11. Choose the incorrect statement m-dinitrobenzene with NH4HS is
(a) Primary amines show intermolecular hydrogen bonds
(b) Tert-butylamine is a primary amine. NO2 NO2
(c) Teritary amines do not show intermolecular hydrogen
bonds. (a) (b)
(d) Isopropylamine is a secondary amine. NO2 HS NO2
12. Aniline in a set of reactions yielded a product D.
NH2 NH2 NH2
NaNO
¾¾¾¾ 2
® A ¾¾¾
CuCN
¾® B
HCl
(c) (d)
H HNO NO2 HS NH2
¾¾®
2
Ni
C ¾¾¾¾
2
®D

The structure of the product D would be: NO2


(a) C6H5NHOH (b) C6H5NHCH2CH3 CH3
(c) C6H5CH2NH2 (d) C6H5CH2OH 17.
(i) NaOH
+ CH3 CHO ¾¾¾¾® [ X ]
13. Ethanoic acid on heating with ammonia forms compound A (ii) D
which on treatment with bromine and sodium hydroxide gives
compound B. Compound B on treatment with NaNO2/dil.
HCl gives compound C. The compounds A, B and C In this reaction [X] will be –
respectively are
NO2 NH2
(a) ethanamide, methanamine, methanol
CH = CH – CH3 CH3
(b) propanamide, ethanamine, ethanol
(c) N-ethylpropanamide, methaneisonitrile, methanamine (a) (b)
(d) ethanamine, bromoethane, ethanedia-zonium chloride
(e) methanamine, ethanamide, methanol
14. The major organic product formed from the following reaction: NO2 NH2
CH2 – CH2 – CH2OH CH2 – CH2 – CHO
O (c) (d)
(i) CH3 NH2
...... is
(ii) LiAlH4, (iii) H2O
EBD_7327
688 CHEMISTRY

NH2 O
NH2 – C – NH – NH2
(CH CO) O
3 2 ® ( X ) ¾¾¾¾ HNO
3 ® (Y ) ¾¾¾ H+
18. ¾¾¾¾¾¾ ® (Z ) 21. O
H 2SO 4 H 2O Major product

Product Z of the reaction


N — C – NH – NH2 N — NH – C – NH2
NH2 NH2
O O
(a) (b)
(a) (b)
NO2 (c) Both of these (d) None of these
NO2
gg
NO2 NO2 (CH ) N H
3 2
22. F NO2 ¾¾¾¾¾ ® (A) ¾¾¾¾
Zn / HCl
® (B)
DMF, D

(c) (d)
(B) is –
NO2
NO2 .. CH 3
(a) H 2N N
19. In which of the following pairs of reactants is most effective CH 3
in forming an enamine
O
|| NH2
(a) CH3 - CH 2 - CH + [(CH3 ) 2 CH]2 NH (b) H2N

O
CH3
(b) + (CH)3CNH2 (c) O2N N
H2N

O
||
(c) (CH3 )3 C CH + (CH3 ) 2 NH (d) O2N NH2

(d) None of these forms an enamine


NH2
CºN
+
H3 O 23. 3 CH COCl
2 Br - H O
2 ® B ¾¾¾¾
3 ®C H O+
20. + CH3MgBr Q P ¾¾¾¾¾
® A ¾¾¾¾¾
OCH3
C (major product) is –
The product ‘P’ in the above reaction is
NH2 NH2
OH CO CH3
CH Br
CH3 (a) (b)
(a) (b) Br
OCH3
NH2

CHO COOH
(c) (d) None of these

(c) (d) Br
OCH3 OCH3
Amines 689

(CH CO) O,Pyridine


Cl
24. 3 2
NH2 ¾¾¾¾¾¾¾¾® (a) Br Br (b)

(I)
Br Br
(i) LiAlH
4 ® III
(II) ¾¾¾¾¾ NO 2
(ii) H2O
Cl
The basicity order of I, II and III is –
(c) Br Br (d) None of these
(a) III > I > II (b) I > II > III
(c) III > II > I (d) II > III > I
25. Which of the following compounds is an enamine ? 29. The most unlikely representation of resonance structures of
p-nitrophenoxide ion is
(a) –N=C=N–
– O O O– O–
(b) – NH 2 + +
N N

(c) N
(a) (b)
(d) =N–
O– O
26. Which one of the following compound is most basic? –
O O O O
H + +
| N N
N
N
|
H (c) (d)
(A) (B) –

H O– O
| 30. The correct order of basicities of the following compounds is
N
NH
O 1. CH3 - C 2. CH3 - CH2 - NH2
(C) NH2 O
(a) (A) (b) (B) 3. (CH3)2NH 4. CH3 - C - NH2
(c) (C) (d) All are equally basic (a) 2 > 1 > 3 > 4 (b) 1 > 3 > 2 > 4
27. The correct stability order of the following resonance
structures is (c) 3 > 1 > 2 > 4 (d) 1 > 2 > 3 > 4

+ – + – 31. An organic compound A containing nitrogen, on acid


H2C = N = N H2C – N = N catalysed hydrolysis produces a water soluble organic
(I) (II) compound B and a gaseous compound C. When methyl
magnesium bromide is slowly added to A in 1 : 1 ratio and
– + – + hydrolysed, it produces a compound which can be obtained
H 2C – N = N H2C – N = N
(III) (IV) by dry distillation of the calcium salt of B. The compound A is
(a) N-methylmethanamide
(a) (I) > (II) > (IV) > (III) (b) (I) > (III) > (II) > (IV)
(c) (II) > (I) > (III) > (IV) (d) (III) > (I) > (IV) > (II) (b) N-ethylmethanamide
28. The product – (C) obtained in the following sequence of (c) acetonitrile
reactions is (d) N, N-dimethylmethanamide
NH2 32. Match the compounds given in List - I with their characteristic
Br reactions given in List - II. Select the correct option.
Br ¾(¾
1) HONO
¾¾ ¾® A ¾Sn / HCl (1) HONO
¾ ¾¾® B ¾¾¾¾ ¾® C List - I List - II
(2 ) CuCl ( 2) H3PO 2
Compounds Reactions
NO2 (1) CH3CH2CH2CH2NH2 (i) alkaline hydrolysis
EBD_7327
690 CHEMISTRY

(2) CH3C º CH (ii) with KOH (alcohol) and (b) O2N


O
CHCl 3 produces bad smell
N
(3) CH3CH2COOCH3 (iii) gives white ppt. with H
ammonical AgNO3
(4) CH3CH(OH)CH3 (iv) with Lucas reagent (c)
cloudiness appears after O
5 minutes N
Options : H
NO2
(1) (2) (3) (4)
(a) (iv) (ii) (iii) (i) (d) O
(b) (ii) (i) (iv) (iii)
O2N N
(c) (iii) (ii) (i) (iv) H
(d) (ii) (iii) (i) (iv)
33. In a set of reactions m-bromobenzoic acid gave a product D. 36. The major product of the following reaction is
Identify the product D.
Me Br
COOH F Å
PhSNa
SOCl2 NH3 NaOH ¾ ¾ ¾ ¾ ¾ ¾ ¾¾
®
B C D dimethylformamide
Br2
Br NO2
SO2NH2 COOH
(a) (b) Me SPh Me SPh
F F
Br NH2
(a) (b)
NH2 CONH2
(c) (d) NO2 NO2

Br Br Me Br Me SPh
34. An organic compound (C3H9N) (A), when treated with nitrous SPh SPh
acid, gave an alcohol and N2 gas was evolved. (A) on warming
(c) (d)
with CHCl3 and caustic potash gave (C) which on reduction
gave isopropylmethylamine. Predict the structure of (A).
NO2 NO2
CH3
(a) CH NH2 (b) CH3CH2 NH CH3 37. Amongst the compounds given, the one that would form a
CH3
brilliant colored dye on treatment with NaNO2 in dil. HCl
(c) CH3 N CH3 (d) CH3CH2 CH2 NH2 followed by addition to an alkaline solution of b-naphthol is

CH3 N(CH3)2 NHCH3

35. In the following reaction (a) (b)

O
NH2 CH2NH2
N
H
(c) (d)
conc. HNO
3 ® X,
H3C
¾¾¾¾¾¾
conc. H 2SO 4
38. The major product of the following reaction is
the structure of the major product ‘X’ is

(a) O (i) KOH


NO2
N
H
2
Amines 691

41. Predict the product:


NHCH3+NaNO 2 + HCl Product
(a)

CH3

(a) N NO2

(b)
NHCH3 NHCH3

NO
(b) +

(c)
NO

OH

N CH3
(c)

(d)
CH3

N N=O
(d)
39. A compound with molecular mass 180 is acylated with
CH3COCl to get a compound with molecular mass 390. The 42. What is the product obtained in the following reaction :
number of amino groups present per molecule of the former
compound is : NO2 Zn
........... ?
NH4Cl
(a) 2 (b) 5
(c) 4 (d) 6 NHOH
40. In a reaction of aniline a coloured product C was obtained. (a)

CH3
N
NH2 –N
(b) N
NaNO2 CH3
B C
HCl Cold

A O
The structure of C would be : (c) N=N
+
(a) – N = N – CH2– N –
NH 2
CH3 (d)
CH3 CH3
43. In the chemical reactions,
(b) –N=N– NH2
NaNO2 HBF 4
A B
CH3 HCl, 278 K
– NH – NH – –N
(c) the compounds ‘A’ and ‘B’ respectively are
CH3
(a) nitrobenzene and fluorobenzene
CH3 (b) phenol and benzene
(d) –N=N– –N (c) benzene diazonium chloride and fluorobenzene
CH3 (d) nitrobenzene and chlorobenzene
EBD_7327
692 CHEMISTRY

44. Which one of the following is the strongest base in aqueous NH2 NH2
solution ?
(a) Methylamine (b) Trimethylamine (c) (d)
(c) Aniline (d) Dimethylamine HS NO2 HS NH2
45. Which of the following statements about primary amines is DIRECTIONS for Qs. 48 to 50 : These are Assertion-Reason
‘False’ ? type questions. Each of these question contains two statements:
(a) Alkyl amines are stronger bases than aryl amines Statement-1 (Assertion) and Statement-2 (Reason). Answer these
questions from the following four options.
(b) Alkyl amines react with nitrous acid to produce alcohols
(a) Statement-1 is true, Statement-2 is true, Statement-2 is a correct
(c) Aryl amines react with nitrous acid to produce phenols explanation for Statement -1
(d) Alkyl amines are stronger bases than ammonia (b) Statement -1 is true, Statement-2 is true ; Statement-2 is NOT
46. Ortho-Nitrophenol is less soluble in water than p- and m- a correct explanation for Statement - 1
Nitrophenols because : (c) Statement - 1 is true, Statement-2 is false
(a) o-Nitrophenol is more volatile than those of m- and p- (d) Statement -1 is false, Statement-2 is true
isomers.
48. Statement-1 : Amines are basic in nature.
(b) o-Nitrophenol shows intramolecular H-bonding Statement-2 : Presence of lone pair of electron on nitrogen
(c) o-Nitrophenol shows intermolecular H-bonding atom.
(d) Melting point of o-Nitrophenol is lower than those of m- 49. Statement-1 : Nitrating mixture used for carrying out nitration
and p-isomers. of benzene consists of conc. HNO3 + conc. H2SO4.
47. The major product (70% to 80%) of the reaction between m- Statement-2 : In presence of H2SO4, HNO3 acts as a base and
dinitrobenzene with NH4HS is produces NO2+ ions.
50. Statement-1 : CN- ion is an ambident nucleophile.
NO2 NH2
Statement-2 : Nucleophiles are electron rich species.

(a) (b)
NO2 NO2

Exemplar Questions 4. Which of the following is the weakest Bronsted base?


1. Which of the following is a 3° amine? NH2
(a) 1-methylcyclohexylamine (a) (b) N—H
(b) Triethylamine
(c) tert-butylamine NH2
(d) N-methylaniline
(c) (d) CH3NH2
2. The correct IUPAC name for CH2 = CHCH2NHCH3 is
(a) allyl methylamine
5. Benzylamine may by alkylated as shown in the following
(b) 2-amino-4-pentene
equation?
(c) 4-aminopent-1ene
C6H5CH2NH2 + R — X —® C6H5CH2NHR
(d) N-methylprop-2-en-1-amine
Which of the following alkyl halides is best suited for this
3. Amongst the following, the strongest base in aqueous reaction through SN1 mechanism?
medium is ............... .
(a) CH3 Br (b) C6 H5 Br
(a) CH3NH2 (b) NCCH2NH2
(c) C6 H5CH2Br (d) C2 H5 Br
(c) (CH3)2NH (d) C6H5NHCH3
Amines 693

6. Which of the following reagents would not be a good choice 14. Methylamine reacts with HNO2 to form ........... .
for reducing an aryl nitro compound to an amine? (a) CH3 – O – N = O (b) CH3 – O – CH3
(a) H2(excess) / Pt (b) LiAlH4 in ether (c) CH3OH (d) CH3CHO
(c) Fe and HCl (d) Sn and HCl
15. The gas evolved when methylamine reacts with nitrous acid
7. In order to prepare a 1° amine from an alkyl halide with is ................... .
simultaneous addition of one CH2 group in the carbon chain,
the reagent used as a source of nitrogen is .......... . (a) NH3 (b) N2
(a) sodium amide, NaNH2 (c) H2 (d) C2H6
(b) sodium azide, NaN3 16. In the nitration of benzene using a mixture of conc. H2SO4
(c) potassium cyanide, KCN and conc. HNO3, the species which initiates the reaction
is ............ .
(d) potassium phthalimide, C6H4(CO)2N–K+
8. The source of nitrogen in Gabriel synthesis of amines (a) NO2 (b) NO+
is ............ .
(c) NO+2 (d) NO -2
(a) sodium azide, NaN3
(b) sodium nitrite, NaNO2 17. Reduction of aromatic nitro compounds using Fe and HCl
gives ............. .
(c) potassium cyanide, KCN
(d) potassium phthalimide, C6H4(CO)2N–K+ (a) aromatic oxime (b) aromatic hydrocarbon
9. Amongst the given set of reactants, the most appropriate (c) aromatic primary amine (d) aromatic amide
for preparing 2° amine is ............ . 18. The most reactive amine towards dilute hydrochloric acid is
(a) 2° R – Br + NH3 ............. .
(b) 2° R – Br + NaCN followed by H2/Pt CH3
(c) 1° R – NH2 + RCHO followed by H2/Pt (a) CH3 — NH2 (b) NH
CH3
(d) 1° R – Br(2 mol) + potassium phthalimide followed by
H3O+/heat
NH2
10. The best reagent for converting 2-phenylpropanamide into
2-phenylpropanamine is ............ . CH 3
(c) N – CH3 (d)
(a) excess H2 CH 3
(b) Br2 in aqueous NaOH
(c) iodine in the presence of red phosphorus 19. Acid anh ydrides on reaction with primary amines
(d) LiAlH4 in ether give ................. .
11. The best reagent for converting 2-phenylpropanamide into (a) amide (b) imide
1-phenylethanamine is ............... . (c) secondary amine (d) imine
(a) excess H2 / Pt (b) NaOH / Br 2 +
Cu/HCl
(c) NaBH4 / methanol (d) LiAlH4 / ether 20. The reaction Ar N 2Cl - ¾¾¾¾
® ArCl + N 2 + CuCl is
12. Hoffmann bromamide degradation reaction is shown named as ........... .
by ............. . (a) Sandmeyer reaction (b) Gattermann reaction
(a) ArNH2 (b) ArCONH2 (c) Claisen reaction (d) Carbylamine reaction
(c) ArNO2 (d) ArCH2NH2
21. Best method for preparing primary amines from alkyl halides
13. The correct increasing order of basic strength for the without changing the number of carbon atoms in the chain
following compounds is .............. . is
NH2 NH2 NH2 (a) Hoffmann bromamide reaction
(b) Gabriel phthalimide synthesis
(c) Sandmeyer reaction
(d) reaction with NH3
NO2 CH 3
(I) (II) (III) 22. Which of the following compound will not undergo azo
coupling reaction with benzene diazonium chloride?
(a) II < III < I (b) III < I < II
(a) Aniline (b) Phenol
(c) III < II < I (d) II < I < III
(c) Anisole (d) Nitrobenzene
EBD_7327
694 CHEMISTRY

23. Which of the following compounds is the weakest Bronsted 28. Nitrobenzene on reaction with conc. HNO 3/H2SO4 at
base? 80 -100°C forms which one of the following products ?
NH2 NH2 (a) 1, 3 - Dinitrobenzene [2013]
(b) 1, 4 - Dinitrobenzene
(a) (b)
(c) 1, 2, 4 - Trinitrobenzene
(d) 1, 2 - Dinitrobenzene
OH OH 29. On hydrolysis of a “compound”, two compounds are
obtained. One of which on treatment with sodium nitrite and
(c) (d) hydrochloric acid gives a product which does not respond
to iodoform test. The second one reduces Tollen’s reagent
and Fehling’s solution. The “compound” is
24. Among the following amines, the strongest Bronsted base
is ............. . (a) CH3 CH2 CH2 CON(CH3)2 [NEET Kar. 2013]
NH2 (b) CH3 CH2 CH2 NC
(c) CH3 CH2 CH2 CN
(a) (b) NH3 (d) CH3 CH2 CH2 ON = O
30. Some reactions of amines are given. Which one is not
correct ? [NEET Kar. 2013]
H H
(a) (CH3)2NH + NaNO2 + HCl ® (CH3)2 N – N = O
N N
(c) (d)
(b) (CH3)2N – + NaNO2 + HCl ®

25. The correct decreasing order of basic strength of the (CH3)2N – – N = NCl
following species is ........... . H2O, NH3, OH–, NH -2
(c) CH3CH2NH2 + HNO2 ® CH3CH2OH + N2
(a) NH 2– > OH – > NH3 > H 2 O
(d) CH3NH2 + C6H5SO2Cl ® CH3NHSO2C6H5.
-
(b) OH > NH -2 > H 2O > NH3 31. In the following reaction, the product (A) [2014]
+
(c) NH3 > H 2 O > NH 2- > OH - NºNCl

NH2

(d) H 2 O > NH3 > OH - > NH -2 is :


+ H+ ( A)
¾¾®
26. Which of the following should be most volatile? Yellow dye
I. CH3CH2CH2NH2 II. (CH3)3N
III. CH3CH2 IV. CH3CH2CH3 (a) N=N–NH
NH
CH3 NH2
(a) II (b) IV
(b) N=N
(c) I (d) III
NEET/AIPMT (2013-2017) Questions
NH2
27. In the reaction
NO2 NO2 (c) N=N

A
(d) N=N NH2
Br Br

Å N2Cl 32. Which of the following will be most stable diazonium salt
A is : [2013] RN2+X– ? [2014]
(a) Cu2Cl2 (b) H3PO2 and H2O (a) CH3 N2+X– (b) C6H5N2+X–
(c) H+/H2O (d) HgSO4/H2SO4 (c) CH3CH2N2+X– (d) C6H5CH2N2+X–
Amines 695

33. The electrolytic reduction of nitrobenzene in strongly acidic 37. The correct statement regarding the basicity of arylamines
medium produces : [2015] is [2016]
(a) Azoxybenzene (b) Azobenzene (a) Arylamines are generally less basic than alkylamines
(c) Aniline (d) p-Aminophenol because the nitrogen lone-pair electrons are delocalized
by interaction with the aromatic ring p electron system.
34. The following reaction
(b) Arylamines are generally more basic than alkylamines
NH2 because the nitrogen lone-pair electrons are not
+ Cl delocalized by interaction with the aromatic ring p
electron system.
O
(c) Arylamines are generally more basic than alkylamines
H because of aryl group.
N
NaOH (d) Arylamines are generally more basic than alkylamines,
O because the nitrongen atom in arylamines is
O sp-hybridized.
is known by the name : [2015 RS] 38. The correct increasing order of basic strength for the
(a) Friedel-Craft's reaction following compounds is: [2017]
(b) Perkin's reaction NH2
NH2
(c) Acetylation reaction
(d) Schotten-Baumen reaction (I) (II)
35. The number of structural isomers possible from the molecular NO2
formula C3H9N is: [2015 RS]
NH2
(a) 4 (b) 5
(c) 2 (d) 3
36. Method by which Aniline cannot be prepared is: (III)
[2015 RS] CH3
(a) hydrolysis of phenylisocyanide with acidic solution (a) III < I < II (b) III < II < I
(b) degradation of benzamide with bromine in alkaline (c) II < I < III (d) II < III < I
solution
39. Which of the following reactions is appropriate for
(c) reduction of nitrobenzene with H2/Pd in ethanol converting acetamide to methanamine ? [2017]
(d) potassium salt of phthalimide treated with (a) Hoffmann hypobromamide reaction
chlorobenzene followed by hydrolysis with aqueous
(b) Stephens reaction
NaOH solution.
(c) Gabriel phthalimide synthesis
(d) Carbylamine reaction
EBD_7327
696 CHEMISTRY

Hints & Solutions


EXERCISE - 1 O
|| NaOBr (NaOH + Br2 )
1. (c) CH3CHO + H2NOH ® CH3CH = N - OH 9. (c) R - C - NH 2 ¾¾¾¾¾¾¾¾ ¾ ®
Acetaldehyde Hydroxylamine Acetaldoxime - NaBr

P2O5 (dehydrating agent) O H O


- H2 O || | - || -
CH3 - CN OH Br -
R - C - N - Br R - C - N - Br¾¾¾ ®
Methyl cyanide gg

2. (b) Acetamide and ethylamine can be distinguished by O


heating with NaOH solution.Acetamide evolves NH3 but ||

:
H O
R - N = C = O ¾¾¾
2 ®
ethylamine does not. R - C - N:
D isocyanate
CH 3CONH 2 + NaOH ¾¾® CH 3COONa + NH 3 intermediate

¾® No reaction.
CH 3CH 2 NH 2 + NaOH ¾ RNH2 + CO2
3. (c) We know that CO 2 + 2NaOH ® Na 2CO3 + H 2 O
CH 3CH 2 NH 2 + CHCl 3 + 3KOH ¾
¾® Overall reaction
CH 3CH 2 NC + 3KCl + 3H 2O
RCONH 2 + Br2 + 4NaOH ®
In this reaction, bad smelling compound ethyl isocyanide
(CH 3CH 2 NC) is produced. This equation is known as RNH 2 + 2NaBr + Na 2 CO3 + H 2 O
carbylamine reaction. 10. (c) Backmann rearrangement is used to form N-alkyl amide.
11. (d) Various products are formed when nitroarenes are
– reduced. These are given below for C6H5NO2.
N2+ Cl OH
Medium Main product
In acidic medium Aniline (C6H5NH2) (metal/HCl)
H O
¾¾2¾® + N2+HCl In neutral medium Phenyl hydroxylamine, (Zn/NH4Cl)
4. (b)
NHOH
CH3 CH3
p- cresol

5. (d) 1° amine RNH2


2° amine RNHR NH NH
In alkaline medium
3° amine N Hydrazobenzene
|
R | R
|

R Thus, Aniline will be main product in case of (d).


12. (a) Acylation occurs in one step only because lone pair of
quaternary ammonium salt is R 4 N + . nitrogen is delocalized with acyl group.
6. (b) Wurtz reaction is for the preparation of hydrocarbons O O–
..
from alkyl halide R – NH – C – CH 3 R – NH – C – CH 3
RX + 2 Na + XR ¾
¾® R – R + 2 NaX NH2 Å .. –
N º NCl
7. (d) POCl3 is a dehydrating agent. Hence

C 6 H 5 CONH 2 ¾¾ ¾
¾
POCl
3 ® C H CN + H O 13. (a) 2NaNO /HCl
6 5 2 ¾¾¾¾¾¾
®
0°C
NH 2OH Diazotisation
8. (c) (CH3 )2 CO ¾¾¾¾® (CH3 ) 2 CH = N - OH
¯ LiAlH 4 Br
(CH3 ) 2 CHNH 2
CuBr/HBr
PCl
5 ® (CH ) CHCl ¾¾¾¾¾
®
(CH 3 ) 2 CHOH ¾¾¾ 3 2 Sandmeyer reaction
NH3
¾¾¾® (CH 3 ) 2 CHNH 2
Amines 697

D 21. (a) Excess of HCl is used to convert free aniline to aniline


14. (d) CH3 - COONH 4 ¾¾ ® CH3 - CONH 2 hydrochloride otherwise free aniline would undergo
(X)
coupling reaction with benzenediazonium chloride.
D/P O H OÅ 22. (d) p-Chloroaniline and anilinium hydrogen chloride can be
2 5 ® CH - CN ¾¾¾¾
¾¾¾¾ 3 ® CH3 - COOH
3 distinguished by AgNO3. Anilinium hydrogen chloride
(Y) (Z) will give white ppt of AgCl on reaction with AgNO3 but
p-chloronoaniline will not react with it because Cl is
O directly attached to benzene nucleus.
||
15. (d) (i) RCNH 2 + Br2 + KOH ¾¾
® C6 H 5 C6 H 5
H NOH
23. (a) C = O ¾¾2 ¾¾® C = NOH
RCONHBr + KBr + H 2 O CH 3 CH 3
(ii) RCONHBr + KOH ® RNCO + KBr + H2O Oxime
(iii) RNCO + 2KOH ® RNH2 + K2CO3 PCl5
RCONH2 + Br 2 + 4KOH ® ¾¾¾¾¾® C6 H5 NHCOCH3
( Beckmann Acetanilide
rearrangement )
RNH 2 + 2KBr + K 2 CO3 + 2H 2O
24. (a) Secondary amines are more basic than tertiary amines
O due to stabilisation of 2° amine by hydrogen bonding
|| with solvent molecule.
16. (b) CH 3 - CH 2 - C - NH 2 + Br2 + KOH HNO
25. (b) RNH2 ¾¾¾¾

ROH + H2O + N2 ­
gg (A) (B) (C)
D
¾¾
® CH 3 - CH 2 - NH 2 26. (b) The given reaction is known as Sandmeyer’s reaction.
27. (d) Aniline is formed
17. (b) +
N NCl + OH NO 2 NH 2

Benzene diazonium chloride Phenol ¾+HCl


+ 6[H] ¾Zn ¾¾® + 2H 2 O

Aniline
¾¾
® N= N OH This is reduction in acidic medium.
28. (c) Since the organic amino compound on reaction with
p-Hydroxyazobenzene nitrous acid at low temperature produces an oily
(orange dye)
nitrosoamine so the organic amino compound is a
18. (a) The reason for the lesser basicity of aniline as compared secondary aliphatic amines.
to that of cyclohexylamine is +R effect of –NH2 group HNO
29. (a) CH3 - CH - CH3 ¾¾¾¾

(i.e., electron releasing resonance effect of – NH2 group). H 2O
19. (b) CH3CH2CONH2 + Br2 + 4KOH ¾ ¾® |
NH 2
Propanamide
CH3CH2NH2 + 2KBr + K2CO3 + 2H2O CH3 - CH - CH3 + N 2 + H 2 O
Ethylamine |
This is Hofmann bromamide reaction in which amide is OH
(A)
converted into amine having one carbon atom less than
amide. [O] CH 3Mgl
(A) ¾¾¾ ® CH3 - C - CH3 ¾¾¾¾¾ ®
20. (c) 1° amines have –NH2 group in their structure. 4 primary || H + / H2 O
amines are possible by C4H11N. O
(B)
CH 3 - CH 2 - CH 2 - CH 2 NH 2
(i) CH3
|
CH 3 - CH 2 - CH - CH 3 CH3 - C - CH3
| |
NH 2 OH
(C)
(ii) 1,1-dimethyl
ethanol
CH3 CH 3
| | 30. (c) CH 3COOH ¾¾¾
NH
3 ® CH COONH
3 4
CH3 - C - CH 2 - NH 2 CH 3 - C - CH 3 Heat
| | PO PO
H NH 2 ¾¾¾
2 5¾
® CH 3CONH 2 ¾¾¾¾
2 5 ® CH CN
3
- H 2O - H 2O
(iii) (iv) (Methyl cynide)
EBD_7327
698 CHEMISTRY

31. (a) Azo dye is prepared by diazo coupling reaction of phenol 38. (a) In (b), (c) and (d) lone pair of nitrogen is linked with sp2
with diazonium salt. hybridised carbon which is acidic in nature therefore it
N+2Cl
– attracts the electron pair towards itself. In (a) lone pair of
OH nitrogen is free because it is attached with carbon which
is sp3 hybridised. So it is most basic.
+
NH2 O
Benzene
diazonium
chloride 39. (c) HNO3

N=N
O
p-Benzoquinone
p–hydroxyazobenzene
(– N = N –) group is called azo – group. 40. (a) R 2 NH > RNH 2 > R 3 N > NH3 .
32. (d) The given reaction is known as Liebermann Nitroso
reaction. 41. (d)
33. (d) Primary amines react with benzaldehyde in presence of a 42. (a) The reaction is Hoffmann bromamide reaction
trace of an acid as catalyst to form Schiff’s base or Anil O
||
H+ R - C - NH 2 + Br2 + 4NaOH ¾¾
® R - NH2
R - NH 2 + O = CHC 6 H 5 ¾¾¾
®
1°amine benzaldehyde
+2NaBr + Na 2CO 3 + 2H 2 O

O
RN < CHC6 H 5 ∗ H 3 O
R – NH2 contains one carbon less than ||
Schiff 'sbase
or R - C - NH 2
Anil
43. (b) C 6 H 5 N 2 Cl + H 3 PO 2 + H 2 O ¾¾
®
C 6 H 6 + N 2 + HCl + H 3 PO3
34. (d) 44. (d)

NH2 NH2 NH2 NH2


NO2
HNO3 + +
H2SO4
NO2
Aniline NO2
p-nitro aniline m-nitro aniline o-nitro aniline
(51%) (47%) (2%)

The reason for this is that, in acidic condition protonation


+
of –NH2 group gives anilinium ion ( N H 3), which is of
deactivating natureand of m-directing nature.
45. (b) Methyl cyanide on treatment with conc. HCl give
acetamide.
35. (c) Completing the given reaction we get
Conc .HCl
P O CH 3C º N ¾¾ ¾ ¾
¾® CH 3CONH 2.
CH 3 CONH 2 ¾¾¾
2 5®
D Acetonitri le Acetamide
46. (d) The reaction is known as Hoffmann’s mustard oil
Sn / HCl
CH 3 C º N ¾¾¾¾
® CH 3CH 2 NH 2 reaction.
Ethanamine Reduction
47. (d) CH3CN CH3 – CH2 – NH2
36. (c)
HNO2 PCl5
HONO
37. (b) C2 H 5 NH 2 ¾¾¾ ® C2 H 5OH ¾¾¾ ® C2 H 5Cl CH3 – CH2 – OH + N2+ HO2
Ethyl amine Ethyl alcohol
48. (d) Ethyl isocyanide on hydrolysis form primary amines.
NH 3
¾¾¾
® C2 H 5 NH 2 = C + H 2 O ¾¾¾ H+
CH3 CH 2 N ® ® CH CH NH + HCOOH
Ethyl amine 3 2 2
Amines 699

+ CO alc CO
H /H 2O NH3
49. (c) CH3CH2CN ¾¾¾¾® CH3CH 2 COOH ¾¾¾ ® NH
KOH
N+K–
D CO CO
Hoffman bromide reaction
CH3 CH 2 CONH 2 ¾¾¾¾¾¾¾¾¾¾
® CH3CH 2 NH 2 RX
CO
NaOBr N–R
D
50. (b) Primary amines react with aldehydes or ketones to form CO
compound known as Schiff’s base D H+/H2O

COOH
–N H2 + O HC–
+ RNH2
COOH

N = CH 64. (a) Of the given compounds toluene which contains an


electron donating group in the ring will be nitrated easily.
Schiff’s base 65. (c) When nitrobenzene is reduced in alkaline medium like
Na3AsO3 / NaOH or glucose / NaOH, azoxybenzene is
:

51. (c) C H3 — C º N ; CH 3 — NH 2 formed as the reduction product.


sp sp3 2H 2H
C6 H5 NO 2 ¾¾¾¾
® C6 H5 NO ¾¾¾
® C6 H5 NHOH
- H 2O
52. (a) Among the given reagents, only LiAlH4 is the reducing
agent. C 6H 5 NO + C 6H 5 NHOH ¾¾®
Nitroso Phenyl
53. (a) Aryl halide (ArX) does not undergo nucleophilic benzene hydroxyl
substitution because they have strong C—X bond due amine

to resonance. O
54. (b) N-alkyl formamides when dehydrated with POCl3 in ­
presence of pyridine give isocyanides. C6 H5 N = NC6 H5 + H 2O
Azoxybenzene
55. (b)
NH2
56. (b) Hydrolysis of phenyl isocyanide forms formic acid.
KNH 2 /liq.NH3
NO 2 NHOH 66. (d)
NH2
Zn / NH 4Cl NH2 NH2
57. (d) ¾¾ ¾ ¾¾®
NH2
58. (c) +
59. (a) It is Stephen’s reaction.
SnCl 2 /HCl NH2
CH3CH2C º N ¾¾¾¾¾ ® CH 3CH 2 CH = NH.HCl
It is an example of nucleophilic aromatic substitution. It
¾¾¾
H O
2 ® CH CH CHO + NH Cl procceds via the formation of an intermediate benzyne.
3 2 4
NO2 NO2
60. (b) Cold dil. NaOH does not attack to either of the compound,
while cold dil. HCl reacts only with benzyl amine fuming HNO
C6H5CH2NH2. 67. (a) ¾¾¾¾¾¾

90° C NO2
m-dinitrobenzene
O O nitrobenzene
® C6 H5 N 2+ Cl- ¾¾¾®
NaNO2 / HCl HNO 2
.. .. .. 68. (c) C6 H5 NH 2 ¾¾¾¾¾
0° C–5° C H O
NH – C – CH3 NH – C – CH3 NH2 (X) 2

+ C6 H5 NO2 + N 2 + HCl
NO2 HOH (Y)
61. (b)
69. (b) Out of the given options, only benzyl amine can be
obtained by both the methods.
NO2 NO2 CN CH2NH2 CONH2

62. (d) The reaction involves the conversion of –NO2to –NH2 H2/Ni LiA1H4
group (reduction) which occurs in presence of Fe/HCl.
63. (a) Aniline cannot be prepared by this method because aryl Benzene nitrile Benzyl amine Benzamide

halides do not undergo nucleophilic substitution with NO2 NO2


potassium phthalimide under ordinary conditions to give
70. (b) Conc. H SO
N-phenyl phthalimide (i.e., cleavage of C–X bond in ¾ ¾ ¾ ¾2 ¾4¾
®
D SO3H
haloarenes is quite difficult).
EBD_7327
700 CHEMISTRY

71. (d) NH3+ HSO4– NH3+ NH2 3. (c) Benzaldehyde reacts with primary aromatic amine to
form schiff's base
180°-200°C Rearrangement

C6 H 5CH = O + C 6 H 5 NH 2 ¾
¾®
Benzaldehyde Aniline
SO3– SO3H
Zwitter ion Sulphanilic acid
C 6 H 5 C H = NC 6 H 5 + H 2 O
72. (a) Reduction of alkyl isocyanides in presence of LiAlH4 Benzyliden e aniline
yields secondary amines containing methyl as one of
the alkyl group. This is know as Schiff’s base reaction.
r LiAlH4 4. (d) Aniline when treated with acetic anhydride forms
R - N = C + 4[H] ¾¾¾¾ ® R - NH - CH3 acetanilide.
2°amine

e.g., r LiAlH 4 O
CH3 - N = C + 4[H] ¾¾¾¾ ® CH 3 - NH - CH 3
dimethyl amine NH2 NH–C–CH3
O O
whereas, alkyl cyanides give 1° amine on reduction. O
CH 3–C–O –C –CH 3
+ CH3–C–OH
® NO +2 + H 2 O + HSO -4
73. (b) HONO2 + H2SO4 ¾¾

Nitric acid acts as a base by accepting a proton.


NH2 NH2
NaOH Br Br
74. (a) CH3CONH2 ¾¾¾¾ ® CH3 NH 2 NaNO
Br2 ¾¾ ¾¾

5. (d) +3Br2
& dil HCl
(Hofmann bromamide reaction)
+ – Br
NH 2 N 2 Cl
(2, 4, 6 tribromoaniline)

NaNO2 CuCN + –
75. (a) N2Cl N2BF4
HCl, 278K
Br Br Br Br
Diazotization Benzene diazonium NaBF4
¾¾ ¾
¾®
chloride ( - NaCl )
(A)
Br Br
C ºN
Diazonium Diazonium
salt tetrafluoroborate

F
Br Br
Benzonitrile D
(B) -N 2
Sandmeyer reaction
- BF3
Br
EXERCISE - 2 2,4,6 tribromofluorobenzene
1. (d) NH2 NH2 6. (b) Reduction reactions of given amines are following
Br Br
Br2 /H 2O NaNO 2/HCl LiAlH
CH3CONH2 ¾¾¾¾
4 ® CH CH NH
3 2 2
0°C Ethanamide ethanamine (1° amine)
aniline
LiAlH 4
Br CH3CON(CH 3 ) 2 ¾¾¾¾ ® CH 3CH 2 N(CH 3 ) 2
(X)
N,N - dimethyl ethanamide N,N - dimethylethanamine
N2Cl Cl (3ºamine)
Br Br Br Br
LiAlH
Cu 2Cl2 C6H5CONH2 ¾¾¾¾
4® C6H5CH2 NH2
HCl Phenylmethanamide phenylmethanamine (1° amine)

Br Br LiAlH
(Y) (Z) CH3CONH -CH3 ¾¾¾4® CH3CH2 NHCH3
2,4,6-Tribromo N-methylethanamide N-methylethanamine (2° amine)
chlorobenzene
2. (d) NH3 is a weaker base than ethanamine. Thus N-methyl ethanamide gives 2º amine on reduction.
Amines 701

NO2 Å –
NH2 N2Cl
NaNO
7. (c) Sn + HCl
¾ ¾ ¾¾® 12. (d) ¾¾¾
¾2®
reduction HCl

Nitrobenzene (B) N C
(A) Arene diazonium salt
CHCl + K OH (follows SN1 path) CuCN
¾ ¾ ¾3¾ ¾¾®
NH–CH3
(C)
Re duction
¬¾¾¾
¾¾
Na/C2H5OH
CH2NH 2 CN
N-methylaniline H / Ni
¬¾2¾¾
8. (c) The mecahnism of Hoffmann bromide reaction is
(i) RCONH 2 + Br2 ® RCONHBr + HBr HNO2
- –
(ii) RCONHBr + HBr ¾¾¾
OH ··
® R — CO N Br + H 2 O

+
K
(iii) R — C — N —Br R — C — N + KBr
O O Benzyl alcohol

R NH
13. (a) CH3COOH ¾¾¾
3 ® CH CONH
3 2
D
Ethanoic acid (A)
(iv) O = C N•• R—N=C=O Ethanamide
••

(rearrangement) 2Br / NaOH


¾¾¾¾¾
® CH3 NH 2 ¾¾¾¾¾¾¾
2 ® CH 3OH
NaNO /dil.HCl

· · (B) (C)
(v) R - N = C = O + 2KOH ® RNH 2 + K 2 CO 3 Methanamine Methanol

CºN
14. (b)
9. (b) + CH3MgBr ¾¾
®
OCH3

H3C – C = NMgBr COCH3


+
H3O
– Mg(OH)Br2 – NH3
OCH3 OCH3
15. (d) Weak base has high pkb value. Benzenamine (aniline)
is the weakest base among the given amines. Therefore
10. (d) NH2 NHCOCH3 it has highest pkb value.
(CH 3CO)2O Br2 NO2 NH2
CH3COOH
16. (c) + NH 4 HS ¾¾
®
CH3 CH3
(A) NO2 NO2
NHCOCH3 NH2
+ NO2 NO2
Br H /H2O Br CH2
CH3
NaOH
17. (a) ¾¾¾¾
®
CH3 CH3 - H 2O
(B) (C)
OH
11. (d) Isopropylamine is a primary amine. NO2
CH3 CH2 – CH – CH3
CH NH2
CH - CHO
3
CH3 ¾¾¾¾¾®
1°amine H+
EBD_7327
702 CHEMISTRY

NO2 O O
CH = CH – CH3
NH – C – CH3 NH – C – CH3
D
¾¾
®
Br - H O
2 2 ®
¾¾¾¾¾
NH2 NH – CO – CH3

(CH CO) O
3 2 ®
¾¾¾¾¾¾ Br
18. (b) (A) (B)
NH2
NH – CO – CH3
3 ® H O+
¾¾¾¾
HNO

¾¾¾¾
H 2SO 4
Br
(C)
NO2
24. (a)
H3O+ NH2

(I)
NH2
3(CH CO) O,Pyridine
2
¾¾¾¾¾¾¾¾® NH – C – CH3

(II) O

NO2 (i) LiAlH


4® NH – CH2 – CH3
¾¾¾¾¾
19. (a) CH – CH – CH + HN [CH(CH ) ] (ii) H 2 O (III)
3 2 3 2 2

O Product (II) is NH – C – CH3 and (III) is


CH3 – CH – CH – N [CH(CH3)2]2
O
(II)
H OH
NH – CH2 – CH3
CH3 – CH = CH – N [CH(CH3)2 ]2
(enamine) (III)
20. (b) \ III > I > II, As + I effect increases the basic strength
and – R, – I effect shown by – COCH3 reduces the basic
21. (b) strength.
O + H2 N – NH – C – NH2 25. (c) Enamine (ene for carbon-carbon double bond and amine
for amine group). >C = C – N<
O
26. (b) In (b) the lone pair of electrons on N is not in conjugation
pH = 4.5 with benzene ring, hence it is not involved in resonance
¾¾¾¾
® N – NH – C – NH2 causing the compound to be most basic.
- H 2O
O 27. (b) H 2 C = N + = N - > H 2 C - - N + º N
gg I III
(CH3 )2 N H Octet complete, Octet complete,
22. (a) F NO2 ¾¾¾¾¾® 6 covalent bond, 6 covalent bond,
DMF, D –ve charge on N –ve charge on C
CH3 > H 2 C+ - N = N - > H 2 C - - N = N +
N NO2
II IV
CH3 Octet incomplete, Octet incomplete,
.. 5 covalent bond, 5 covalent bond,
CH3 –ve charge on N –ve charge on C
Reduction
¾¾¾¾¾
® N NH2
CH3 NH 2 Cl
Br Br Br Br
NH2
HONO Sn / HCl
28. (c) ¾¾¾¾
® ¾¾¾¾®
CuCl
23. (c) CH COCl
3 Sandmeyer
¾¾¾¾¾
®
NO 2 NO 2
Amines 703

Cl Cl
Br Br reduction
Br Br
HONO CH3 CH NH CH3
¾¾¾¾
®
H 3PO 2
elimination of NH 2 CH3
NH 2
Isopropyl methyl amine
29. (c) N can't have more than 8 electrons in its valence shell as
it does not have any d orbital. In (c), N has 10 electrons. 35. (b) The benzene ring which is directly attached with
NH nitrogen atom of acidamide group is more activated than
30. (d) CH3 – C is most basic the other, so nitration will take place at para position of
NH2 this activated ring.
(CH3)2NH is less basic than CH3CH2NH2 due to steric
O O2N
hindrance because of two bulky –CH3 groups on nitrogen
atom. N conc. HNO3
O
H3O+ H ¾¾¾¾¾¾ ®
conc. H 2SO 4 N
31. (c) CH 3 —C ºº N ¾® CH 3 —COOH + NH 3
Acetonitrile (B) (C) H
(A)
O Me Br Me SPh
½½
(i) CH3MgBr
CH3 —C ºº N ¾¾¾¾® CH3 —C—CH3 F Å F
PhSNa
(A) (ii) H3O + ¾ ¾ ¾ ¾ ¾ ¾ ¾¾
®
36. (a) dimethylformamide
O
dry distillation ½½
(CH3COO)2Ca¾¾¾¾® CH3 —C—CH3 NO2 NO2
From above reaction sequence, Acetonitrile is the given organic SN2 Mechanism \ Inversion takes place
compounds.
32. (d) (1) CH3 CH2 – CH2 – CH2 – NH2 37. (c) Only primary aromatic amines undergo diazotisation
(ii) with KOH (alcohol) and CHCl3 produces bad followed by coupling.
smell 38. (a)
(2) CH3C º CH
(ii) gives white ppt with ammonical AgNO3 O O
(3) CH3 CH2 COOCH3 Br CH2 Cl
(i) alkaline hydrolysis (1) KOH – +
NH N K
(4) CH3 CHOH – CH3 Acid base SN2 reaction
(iv) with Lucas reagent cloudiness appears after reaction
5 minutes O O
O
O
COOH C – Cl

33. (c) SOCl2 NH3 N CH2 Br

Br Br O
O NH2 Due to resonance C – Br bond shows partial double
C NH2 bond character. Therefore nucleophile does not replace
Br in above reaction.
NaOH
O
+Br2 P
Br Br 39. (b) R , NH 2 ∗ CH3 , C, Cl ¾¾¾↑
HNO2 { ,HCl
34. (a) CH3 CH NH2 Mol.Mass<16
O
CH3 (A) P
isopropyl amine R , NH , C, CH3
144444424444443
CH3 CH OH + N2 ­ Mol.mass<58

CH3 Now since the molecular mass increases by 42 unit as a


isopropyl alcohol result of the reaction of one mole of CH3COCl with one-NH2
CHCl3 / KOH group and the given increase in mass is 210. Hence the num-
(A) CH3 CH N C ber of –NH2 groups is = 210/42 = 5.

CH3
isopropyl isocyanide
EBD_7327
704 CHEMISTRY

40. (d) The reaction can be completed as follows: 46. (b) o-nitrophenol shows intramolecular H-bonding and
NH2 forms chelate. Compounds involved in chelation
NaNO2 /HCl CH3 become non-polar. Consequently such compounds are
N2Cl + N
(diazotisation) CH3 soluble in non-polar solvents like ether, benzene etc.
‘A’ ‘B’ NN- dimethylaniline and are only sparingly soluble in water whereas meta
(Aniline) Benzene
diazonium chloride and para isomers are more soluble in water & less
cold
soluble in non-polar solvents.
CH3 H
N=N N O O
CH3
Å
‘C’ N
p-dimethylaminazobenzene O intra-molecular H-bonding
41. (d) Secondary amine with (NaNO2 + HCl) gives a nitroso
product
NO2 NH2
CH3
| 47. (b) + NH4HS ®
NHCH3 N–N=O
NaNO 2 /HCl
¾
¾¾¾® NO2 NO2
48. (a) Amines are basic due to the presence of a lone pair of
electrons on nitrogen atom. The lone pair can be easily
N– Nitroso – N– methylaniline
donated.
42. (a) When nitro compound is reduced with a neutral reducing
agent (Zn dust + NH4Cl) the corresponding hydroxyl 49. (a) HNO3 + 2H 2SO4 2HSO -4 + NO2+ + H3O+
amine is formed 50. (b) Nucleophilic species which have more than one site of
reaction are called ambident nucleophiles.
Zn dust + NH Cl
C6 H5 NO 2 + 4[H] ¾¾¾¾¾¾®
4
C6 H5 NHOH : C = N :«: C = N :
Phenyl hydroxylamine
43. (c) Primary aromatic amines react with nitrous acid to yield EXERCISE - 3
arene diazonium salts.
Exemplar Questions
ArNH2 cold
+ NaNO2 + 2HX ¾¾¾ ® 1. (b) 3° amine is the amine in which nitrogen is attached to
1° Aromatic amine
three alkyl groups whether they are same or different.
Ar—N = N+X– + NaX + 2H2O
:

Arene diazonium salt


The diazonium group can be replaced by fluorine by N CH2CH3
CH3CH2
treating the diazonium salt with fluoroboric acid (HBF4).
The precipitated diazonium fluoroborate is isolated, CH2CH3
Triethylamine
dried and heated until decomposition occurs to yield (3° amine)
the aryl fluoride. This reaction is known as Balz- 3 2 1
Schiemann reaction. 2. (d) IUPAC name of
CH 2 = C HC H 2 NHCH3 is
Ar—N2+X– ¾¾¾®
HBF4 Ar—N2+BF4–¯ ¾¾¾
heat N-methylprop-2-en-1-amine.
®
3. (c) More the hydrogen bonding more will be the basicity in
Ar—F + BF3 + N2 gaseous phase but in aqueous solution solvation effect
44. (d) Aromatic amines are less basic than aliphatic amines. also play the role. By combining both the effects order
Among aliphatic amines the order of basicity is of basicity in aqueous medium is 2° > 3° > 1° > NH3. The
2° > 1° > 3°. The electron density is decreased in 3° inductive effect comes to play when order of amines is
amine due to crowding of alkyl group over N atom which same as in case of (c) & (d) where both the amines are
makes the approach and bonding by a proton relatively secondary. +I group increase the basicity and –I group
difficult. Therefore the basicity decreases. Further Phenyl
will decrease the basicity of amines.
group show – I effect, thus decreases the electron density
4. (a) In aniline, lone pair of nitrogen is not available for
on nitrogen atom and hence the basicity.
donation to the acid as it is involved in the resonance.
\ dimethylamine (2° aliphatic amine) is strongest base Thus, it will act as the weakest Bronsted base amongst
among given choices. the given compounds.
\ The correct order of basic strength is
+ + +
Dimethylamine > Methyl amine > Trimethyl amine > NH2 NH2 NH2 NH2 NH2
Aniline. – –
45. (c) Aryl amines do not produce phenol on treatment with

nitrous acid
I II III IV V
Amines 705

5. (c) SN1 reaction involves the formation of carbocation, so, 11. (b) Hoffmann bromamide reaction.
more stable carbocation will have higher reactivity
towards SN1 mechanism. O
H3C ||
r NH2 CH3
C6 H5CH 2 Br ¾¾ ® C6 H5 – CH 2 + Br s
Å NaOH / Br2 CH — NH2
As C6H5 C H 2 is the most stabilised carbocation due to
resonance, hence it will be most reactive towards SN1 2-phenylpropanamide 1-phenylethanamine
mechanism.
12. (b) Hoffmann bromamide degradation reaction:
H (excess)/Pt NH2
2
O NH2
NO2 ||
NH2
6. (b) Fe/HCl NH2 Br / NaOH
¾ ¾2¾ ¾ ¾® + Na2CO3 + NaBr + 2H2O
Sn/HCl NH2 Benzamide Aniline

13. (d)
LiAlH4/ether NH2 NH2 NH2
No reaction
7. (c) 1° amine with increased –CH2 group can be achieved
by following the given steps: > >
KCN Na/C2O5H
X C ºN CH2NH2
alkyl –KX
s (1° amine)
halide (Nucleophilic (reduction of CN ) CH3 NO2
substitution of III I II
s
alkyl halide by CN )
8. (d) Potassium phthalimide is the source of nitrogen in Gabriel Higher the electron density towards ring, higher will be
phthalimide synthesis. its basic strength. Electron donating group increases
O the basic strength while electron withdrawing group
|| decreases the basic strength.
C CO – +
- H 2O R–X + –
NH + KOH (alc.) NK ¾¾ ¾® 14. (c) CH3— NH2 + HNO2 CH3 — N2 Cl
C CO methylamine nitrous acid
||
O H2O
CO H + / H 2 O, D COOH
N–R or + RNH 2
HO - / H 2 O, D CH3OH
CO COOH 1° amine
Methanol
Phthalic acid
9. (c) Reaction can be represented as HNO 2 + –
R — NH2 + RCHO [R — N = C — R] 15. (b) CH3— NH2 CH3 — N º NCl
| methylamine
1° amine
H /Pt H D H2 O
2

H
| CH3OH + N2­
R—N—C—R Methanol
| |
H H
2° amine 16. (c) For the nitration of benzene, NOÅ2 acts as an electrophile
10. (d) LiAlH4 in ether is the best reagent for converting (electrophilic substitution reaction).
2-phenylpropanamide into 2-phenylpropanamine Step 1: H2SO4 ¾® H+ + HSO4–
because LiAlH4 will reduce carbonyl group but does + +

not reduce benzene ring. Step 2: H + H – O – NO2 H 2 O + NO2


nitronium ion
O
|| NO2
NH2 NO2+
NH2

LiAlH in ether
¾ ¾ ¾ 4¾ ¾ ¾® 17. (c) Reduction of aromatic nitro compounds gives aromatic
1° amine.
2-phenylpropanamide 2-phenylpropanamine
EBD_7327
706 CHEMISTRY

NO2 24. (d) Pyrrolidine will be the strongest base as the lone pair on
NH2
Fe / HCl nitrogen are not involved in resonance and are easily
¾ ¾ ¾ ¾® available for the donation. So, basic strength becomes
Aromatic Aromatic high.
nitro compound primary amine s
18. (b) Reactivity of a base towards dilute HCl is directly H N
25. (a) H
proportional to the strength of the base. Most basic
Thus, as (CH3)2NH has the highest basic strength, so it Amide ion is the most basic among the given species
will have highest reactivity. due to the presence of negative charge and two lone
H 3C H 3C Å H pairs of electron on nitrogen atom. The species which
N — H ¾ HCl¾¾ ® N + Cl

are having lone pair of electrons are more basic than
H 3C H 3C H neutral species. So, NH3 and H2O are less basic than
R NH2– and OH–. Also, nitrogen is less electronegative
Å than oxygen hence it can easily donate e–s.
O O O NH2 O –
··
|| || R – N H2 || \ The correct order of basic strength will be:
19. (a) R O R ¾ ¾ ¾ ¾® R O NH2– > OH– > NH3 > H2O
Acid R 26. (b) On comparison to amines, hydrocarbons have lower
anhydride boiling point, hence they are more volatile whereas
O amines are having higher boiling point due to H-bonding.
||
Å – NEET/AIPMT (2013-2017) Questions
R—N R + R — COO
NO2 NO2
H H

27. (b) ¾¾
®
O Br + H 3 PO 2 + H 2O Br
.. ||
R—N R + COOH – +H3 PO 3 + HCl
ÅN2C l
HAmide R NO 2
NO 2
20. (b) Gattermann reaction:
Cl 28. (a) HNO / H SO
¾¾¾¾¾¾®
3 2 4
N +2 Cl- 80 -100°C
Cu / HCl NO 2
¾ ¾ ¾ ¾® + N2 + CuCl Nitrobenzene 1,3 - Dinitrobenzene
Benzene diazonium Chlorobenzene 29. (b) Hydrolysis of propyl isocyanide (CH3CH2CH2NH2) gives
chloride CH3CH2CH2NH2 + HCOOH.
21. (b) Gabriel phthalimide synthesis is the best method of On treatment with NaNO 2 and HCl I gives
preparing primary amines from alkyl halides without CH3CH2CH2OH which does not give iodoform test. II
changing the number of carbon atoms in the chain. (HCOOH) reduces Tollen’s reagent and Fehling’s
O O O solution.
|| || ||
KOH alc.
NH ¾¾
( ) s+ R X + -
CH3CH 2CH 2 NC ¾¾¾
2
®
H O
¾¾¾ ® NK ¾¾ ¾¾¾® N— R
|| || || Propyl isocyanide
O O O CH3CH 2 CH 2 NH 2 + HCOOH
NaOH/H2O II
O 30. (b) Secondary amine react with nitrous acid to give N-
|| s+ Nitrosoamines.
ONa
R – NH2 + s+
primary amine O Na (CH3)2 N + NaNO2 + HCl
||
O CH3
22. (d) Diazonium cation is a weak E+ so it will not react with
compound containing electron withdrawing group. Also, N — N = O + NaCl + CH3OH
NO2 group is an electron withdrawing group thus,
NH 2
nitrobenzene will not undergo azo coupling reaction with
benzene diazonium chloride. 31. (d) N 2 Cl +
23. (c) Phenol has the least tendency to accept a proton and H
also exhibit acidic character due to polar O – H bond
hence, it is a weak Bronsted base. N=N NH 2
Amines 707

32. (b) Arene diazonium salts are most stable among the given NH2
options because of the dispersal of +ve charge on the Cl
+
benzene ring due to resonance. NaOH
O
33. (d) Schotten-Baumen Reaction

NO2 HN—OH NH2 NH – C –


Electrolytic O
rearrangement
reduction
(Strongly acidic 35. (a) C3H9N
medium) CH3—CH2—CH2—NH2
nitrobenzene Phenyl
hydroxylamine OH CH3 – NH – CH2 – CH3.
CH3 — CH — CH 3 , CH 3 —N — CH 3
CH3 | |
NH2 CH3
AlCl3
34. (d) 36. (d) Because aryl halides do not undergo nucleophilic
CH3 Cl
substitution reaction with potassium phthalimide easily.
(Friedel Craft’s 37. (a) Arylamines are generally less basic than alkylamines due
Reaction) to following factors
H (a) Due to resonance in aromatic amines.
O O
C=O (b) Lower stability of anilinium ion
O Base
+ 38. (c) – NO2 group has strong – R effect and –CH3 shows +R
R R effect.
Perkin
O Reaction O \ Order of basic strength is
NH2 NH2 NH2
Ar OH + HO
< <
R R
O
O NO2 CH3
OH O – C – CH3
+ CH3 – C – Cl O
39. (a) CH3 - C - NH 2 + Br2 + 4NaOH ¾ D¾®
Acetylation Reaction
acetamide
CH3 – NH2 + 2NaBr + Na2CO3 + 3H2O
methanamine
It is called Hoffmann Bromamide reaction.
EBD_7327
708 CHEMISTRY

28 Biomolecules
Biomolecules may be defined as complex lifeless chemical Monosaccharides
substances which form the basis of life i.e. they not only build up Monosaccharides are further classified as : aldose and ketose.
living system but are also responsible for their growth, maintenance If a monosaccharide contains an aldehyde group, it is called an
and their ability to reproduce.
aldose and if it contains a keto group, it is called a ketose.
CARBOHYDRATES
Glucose
Carbohydrates are a major source of energy to our body.
Glucose is an aldohexose. It is a monomer of many of the larger
They are defined as optically active polyhydroxy aldehydes or
polyhydroxy ketones or substances which give these on carbohydrates such as starch, cellulose.
hydrolysis. Preparation
They are also called saccharides (meaning sugar) (i) By hydrolysis of sucrose:
Classification of Carbohydrates H+
They are classified in number of ways C12 H 22O11+ H 2O ¾¾¾
® C6H12O6 + C6H12O6
Sucrose Glu cos e Fructose
(I) Based on molecular size :
(i) Monosaccharides: It cannot be hydrolysed further to (b) By hydrolysis of starch:
give simpler unit of polyhydroxy aldehyde or ketone.
H+
For ex: glucose, fructose, ribose, etc. ( C6H10O5 )n + nH 2O ¾¾¾
® n C6 H12O6
(ii) Oligosaccharides: They yield 2-10 monosaccharide units 393K,2 - 3 atm glucose
starch or
on hydrolysis. They are further classified as di-, tri, cellulose
tetra-, saccharides etc. depending upon the number of
monosaccharides formed by them on hydrolysis ex: Structure of glucose:
sucrose on hydrolysis gives one molecule each of
CHO
glucose and fructose whereas maltose gives two
molecules of glucose only. (CHOH)4
(iii) Polysaccharides: They yield a large number of The structure of glucose assigned was:
monosaccharide units (more than ten) upon hydrolysis. CH2OH
For ex: starch,, cellulose, glycogen, gums etc.
(II) Based on nature : It was on the basis of following evidences :
Carbohydrates are also classified as reducing and non- (i) Its molecular formula was found to be C 6H12O6
reducing sugars depending on whether they reduce Fehling’s (ii) On prolonged heating with HI, it forms n-hexane, suggesting
and Tollen’s reagent or not. that all six C-atoms are linked in a straight chain.
(i) All monosaccharides whether aldose or ketose are
reducing sugars. CHO

(CHOH)4 HI, D CH3 – CH2 – CH2 – CH2 – CH2 – CH3


(ii) In disaccharides, if reducing groups of monosaccharides,
i.e., aldehydic or ketone groups are bonded, these are
n-hexane
non-reducing sugars, ex:- sucrose. If, however these CH2OH
groups are free in the sugar, then it is reducing sugar ex:
maltose and lactose (iii) Relation dueo to –CHO group
(III) Based on taste : (a) It reacts with hydroxylamine to form an oxime and adds a
Carbohydrates with sweet taste are called sugars while those molecule of HCN to give cyanohydrin. These reactions
without a sweet taste are called non-sugars. Mono and
oligosaccharides are sugars while polysaccacharides are non- confirm the presence of carbonyl group C = O in
sugars. glucose
Biomolecules 709

Fischer projection for glucose is :


CHO CH = N – OH
CHO
NH2OH
(CHOH)4 (CHOH)4 + H2O ;
H OH
CH2OH CH2OH
HO H
CN H OH
CHO CH
OH
HCN H OH
(CHOH)4 (CHOH)4

CH2OH CH2OH
CH2OH
D – (+) – glucose
(b) On reaction with bromine water, it gives gluconic acid. This Here ‘D’ represents the configuration while (+) implies rotation of
shows that carbonyl group is present as aldehydic group. the path of plane polarised to the right.
Limitation of open chain structure of D (+) glucose :
CHO COOH The following reactions of glucose could not be explained on the
Br2 water basis of its open chain structure.
(CHOH)4 (CHOH)4
(a) Glucose does not give, 2, 4-DNP test, Schiff’s test and does
CH2OH CH2OH not form addition product with NaHSO3.
(b) The penta acetate of glucose does not react with
gluconic acid
hydroxylamine indicating the absence of free –CHO group.
(iv) Reactions due to –OH group (c) It is found to exist in 2 forms: a and b.
(a) Formation of glucose pentaacetate with acetic anhydride Cyclic structure of D (+) glucose :
confirms the presence of five –OH groups. In this structure, the aldehyde (–CHO) group is involved in the
form of a ring with the –OH group attached to C5 carbon. It is a six
CHO CHO membered ring, often called d-oxide ring. The ring accounts for the
acetic two isomeric forms a and b as shown below :
(CHOH)4 (CH – OCOCH3)4 O
anhydride
CH2OH 1
CH2OCOCH3 1 H–C
H – C – OH
(b) On oxidation with nitric acid, glucose and gluconic acid both 2 2
H OH H OH
yield a dicarboxylic acid, saccharic acid. This indicates the 3
HO H O 3
presence of primary alcoholic group in glucose. HO H
4
CHO COOH COOH H OH
5 4
HNO3 HNO3 H OH
(CHOH)4 (CHOH)4 (CHOH)4 H
6 5
H OH
CH2OH COOH CH2OH CH2OH 6
Saccharic acid a-D–(+)– Glucose
CH2OH
(v) Formation of osazone
Like the normal aldehydes, glucose reacts with phenylhydrazine 1
in equimolecular proportion to form phenylhydrazone but unlike HO – C – H
the normal aldehydes, glucose reacts with excess of 2
H OH
phenylhydrazine (three molecular proportions) to form 3 O
HO H
glucosazone. 4
H OH
CHO 5
CH = N.NHC6H5 H
6
CHOH C = N.NHC6H5
C 6 H 5 NHNH 2 CH2OH
( excess )
(CHOH)3 ¾¾ ¾ ¾ ¾® (CHOH)
- H 2O 3
b-D–(+)– Glucose
The two cyclic hemi acetal forms of glucose differ only in
CH2OH CH2OH configuration of –OH at C1. Such isomers (a and b) are called
Glucosazone anomers.
EBD_7327
710 CHEMISTRY

Note : Both the forms are optically active, a -D- glucose has (v) It exists in 2 cyclic forms obtained by addition of –OH at
specific rotation + 111.5º and b -D-glucose has specific rotation + C-5 to C = O group. The ring thus formed is a five
19.5º. membered ring and is named as furanose.
Mutarotation - When either of the two forms of glucose is dissolved
in water there is change in rotation till the equilibrium value of +
1
52.5º. This is known as mutarotation. 2 2 1
HOH2C – C – OH HO C CH2OH
a–D(+)Glucose Equilibrium Mixture
3
+111.5º +52.5º HO H HO
3
H O
b–D–(+)Glucose 4 4
+111.5º +19.5º H OH H OH
O
Haworth structure of glucose 5 5
The two cyclic structures for D (+) glucose shown in the pyranose H H
form as : 6
6 CH2OH
CH2OH
6 a-D–(–)– Fructofuranose b-D-(–)-Fructofuranose
CH2OH
O
(vi) The Haworth structures of the two anomers are :
H H5 H
4 1 6 1
HOH2 C O CH2OH
OH OH H OH
5 2
3 2 H HO OH
H OH H

4 3
a – D – (+) – Glucopyranose
OH H
6 a-D–(–)– Fructofuranose
CH2OH
O
6
H H5
OH HOH2 C O OH
4 1
5 2
HO OH H H
H H HO CH2OH
3 2 1
H OH 4 3
OH H
b – D– (+) – Glucopyranose b-D–(–)– Fructofuranose
Fructose (Laevulose)
(i) Its molecular formula is C6H12O6.
Epimer
(ii) It is isomeric with glucose. However it is a ketohexose
Carbohydrates which differ in configuration at any
(iii) It consists of a keto group at C-2 and 6 C-atoms in a
carbon other than glycosidic carbon are called epimers. For
straight chain
example glucose and mannose are called epimers since they
(iv) Its open chain structure is:
differ in configuration at C2 (glycosidic carbon)
CH2OH
Disaccharides
C=O Sucrose
(i) On hydrolysis, it gives an equimolar mixture of D–(+)– glucose
H and D-(–) fructose, i.e.,
HO
C12 H 22 O11 + H 2 O ¾¾
® C6 H12 O6 + C6 H12 O6
H OH sucrose D -( + ) -Glu cose D -( -) -Fructose

H OH (ii) The two monosaccharides are held together by glycosidic


linkage between Cl of a-glucose and C2 of b-fructose.
(iii) It is a non-reducing sugar.
CH2OH (iv) It is an invert sugar because an hydrolysis, it brings a change
D– (–) – Fructose in the sign of rotation, from dextro (+) to laevo (–).
Biomolecules 711

Maltose (ii) Depending upon the relative position of –NH2 group with
(i) It is composed of two a–D–glucose units in which Cl of one respect to the carboxyl group, amino acids are classified as
glucose unit is linked to C4 of another glucose unit. a, b, g, d and so on.
(ii) It is a reducing sugar. (iii) Only a-amino acids are obtained on hydrolysis of proteins.
Lactose R – CH – COOH
(i) Also known as milk sugar as it is found in milk. a-amino acid
(ii) It is composed of b–D–galactose and b–D–glucose with a NH2
linkage between Cl of galactose and C4 of glucose.
Classification of Amino Acids
(iii) It is a reducing sugar.
Acidic, basic and neutral amino acid
Polysaccharides
They are classified as acidic, basic or neutral depending upon the
Starch relative number of amino and carboxyl groups in the molecule.
It is a polymer of a-glucose and consists of two components - Equal number of amino and carboxyl groups make it neutral; more
amylose and amylopectin. amino than carboxyl make it basic while more carboxyl groups
Amylose is water soluble component with 200-1000 a–D–(+) than amino make it acidic.
glucose units held by C1–C4 glycosidic linkage. Essential and non essential amino acids - Human body can
Amylopectin is insoluble in water and is a branched chain polymer synthesise ten amino acids called non essential amino acids. The
of a–D–glucose. remaining ten amino acids required for protein synthesis are called
Cellulose essential amino acids. They are
It is a straight chain polysaccharide composed only of b–D– (i) Phenylalanine (ii) Histidine
glucose units joined by C1-C4 glycosidic linkages. (iii) Tryptophane (iv) Valine
Glycogen (i) Methionine (vi) Theonine
It is also known as animal starch because its structure is similar to (vii) Arginine (viii) Leucine
amylopectin. It is present in liver, muscles and brain. (ix) Isoleucine (x) Lysine
Importance of Carbohydrates Properties of Amino Acids
(i) Used as storage molecules as starch in plants and glycogen (i) They are usually colourless, crystalline solids.
in animals. (ii) They are water-soluble, high melting solids and behave like
(ii) Cell wall of bacteria and plants is made up of cellulose. solids rather than simple amines or carboxylic acids.
(iii) Provide raw materials for many important industries like (iii) This behaviour is due to the presence of both acidic (–COOH)
textiles, paper, lacquers and breveries. and basic (–NH2) groups in the same molecule. In aqueous
solution, the carboxyl group can lose a proton and amino
LIPIDS
group can accept a proton, giving rise to a dipolar ion known
Lipids are organic compounds soluble in non-polar fat solvents as zwitter ion.
such as acetone, ether, chloroform, benzene, etc. and insoluble (iv) The zwitter ion is neutral but contains both positive and
in water. The most important role of lipids is that of biological
negative charges. In zwitter ionic form, amino acids show
fuel. Lipids supply more energy than carbohydrates, excess of
amphoteric behaviour.
lipids is stored in the body and used at the time of starvation.
Lipids are of three types. O O
(i) Simple lipids: They yield fatty acids and an alcohol on

hydrolysis. Fats, oils and waxes are examples of this type. R – CH – C – O – H R – CH – C – O
(ii) Compound lipids: These yield fatty acids, alcohol and
other compound like phosphoric acid, sugar, etc. on NH2 + NH3
hydrolysis. Phospholipids are the most common examples. (Zwitter ion)
(iii) Derived lipids: These lipids do not contain any ester
linkage. These are those hydrolysis products of simple or (v) Except glycine, all naturally occuring a-amino acids are
compound lipids which still retain the properties of lipids. optically active.
Fatty acids, alcohols (e.g., sterols), vitamins D, E and K (vi) Amino acids have been assigned notations D or L depending
are examples of this type. upon the configuration about the chiral carbon atom most of
PROTEINS the naturally occuring amino acids have L configuration.
All proteins are polymers of a-amino acids. Peptide Bond
Chief sources of proteins are milk, cheese, pulses, peanuts, fish, (i) Two or more than two amino acids link to form a peptide.
meat etc.They are required for growth and maintenance of body. (ii) The bond present between peptides is called peptide bond.
Amino acids (iii) Peptide linkage (–CO–NH–) is formed by combination of
(i) They contain amino (–NH2) and carboxyl (–COOH) functional –NH2 of one group and –COOH of the other with the
groups. elimination of H2O.
EBD_7327
712 CHEMISTRY

backbone of the polypeptide chain (due to H-bonding


R R
| | O
H - N - C - C - O - H + H - N - C - C - OH
| | || | | || between C and –NH– groups of peptide bond) gives
H H O H H O rise to two structures : a helix and b-pleated sheet.
Amino acid Amino Acid
(a) a-helix
Polypeptide chains twist into a right handed screw (helix)
R R with – NH of each amino acid residue H-bonded to
| |
-H O C = O of an adjacent turn of the helix.
¾¾ 2¾¾® H - N - C - C - N - C - C - OH
| | || | | ||
H H O H H O Example : Myosin, Keratin, etc.
Peptide (b) b-pleated
Bond All peptide chains stretch out to the maximum extension
and then lay side by side and are held together by
(iv) When a large number of amino acid(>10) combine, polypeptide
intermolecular H-bonds.
is formed. A polypeptide with more than 100 amino acid
Example : Silk fibres.
residues, having molecular mass higher than 10,000 m is called
(iii) Tertiary Structure: It represents the overall folding of the
a protein.
polypeptide chains. It gives rise to two major molecular
Types of Proteins shapes: fibrous and globular.
Classification of proteins is based upon three general The main forces which stabilise 3° structure of protein are H–
properties:shape, solubility and chemical composition. bonds, disulphide linkages, ionic bond and hydrophobic
Classification on the basis of hydrolysis products interactions. Which can be explained as :
(i) Simple proteins: The simple proteins are those which (a) Hydrogen bond :
yield only amino acids on hydrolysis.
(ii) Conjugated proteins: Conjugated proteins on hydrolysis |
C = O........H — N —
yield a-amino acids and other non-proteinous substances | |
in addition to a-amino acids. The non-proteinous moiety H
is usually referred to prosthetic group.
Hydrogen bond
(iii) Derived proteins: Derived proteins are the products formed
by the action of physical (such as heat), chemical or They are formed between oxygen of acidic amino acid
enzymatic agents on natural proteins. The derived proteins and –H of basic amino acid.
are thus intermediate hydrolysis products of proteins. (b) Hydrophobic bond -
On the basis of their composition, proteins are classified as : (1) Non - polar side chains of neutral amino acid
(i) Fibrous proteins: These have thread like molecules which tends to be closely associated with one another
lie side by side to form fibres. The various molecules are in proteins.
held together by hydrogen bonds. These are insoluble in (2) Present in between the amino acids.
water but soluble in concentrated acids and alkalies. (3) These are not true bonds.
Examples are keratin of hair, nails, wood, feathers and (c) lonic bond :
horn, myosin of muscles, and fibroin of silk. – COO–.....H3+N–
(ii) Globular proteins: This type of protein has molecule folded Ionic bond
into compact units which often acquire spheroidal shape. These are salt bonds formed between oppositely
Such proteins are soluble in water, dilute acids and alkalis. charged groups in side chains of Amino acids
Examples are insulin, haemoglobin, albumin, etc. Eg. Aspartic acid
Structure and shape of proteins are studied at four different Glutamic acid
levels:
(d) Disulphide bonds :
(i) Primary structure: Each polypeptide chain in a protein has
amino acids linked to each other in a specific sequence and | ------ S - S ---- |
this sequence is called primary structure of protein. Any 1. Relatively stable bond and thus is not broken readily
change in primary structure results in a different protein. under usual conditions of denaturation.
In primary structure all amino acids are attached in a 2. Formed between the -SH group of Amino acid Ex.
straight chain by peptide bond. Cystine and Methionine .
(iv) Quaternary Structure: Some proteins are composed of 2 or
It is important as the replacement of just one amino acid
more polypeptide chains referred to as sub-units. The spatial
in the sequence of a protein destroys its biological activity. arrangement of these sub-units wrt each other is known as
(ii) Secondary structure: It refers to the shape in which a long quarternary structure.
polypeptide chain can exist. The regular folding of the
Biomolecules 713

Denaturation of Proteins Classification of Vitamins


When a protein in its native form is subjected to a change in Vitamins are classified into two groups depending upon their
temperature or pH, H-bonds are disturbed. Due to this, globules solubility in water or fat.
unfold, helix get uncoiled and protein loses its biological activity. (i) Fat soluble vitamins:
This is called denaturation of protein. During denaturation, 2° and Vitamins which are soluble in fat and oils but insoluble in
3° structures are destroyed but 1° structure remains intact. water are kept in this group. These are vitamins A, D, E and K.
ex: coagulation of egg white on boiling; curdling of milk They are stored in liver and adipose (fat storing) tissues.
Tests for Proteins (ii) Water soluble vitamins:
B-complex and Vitamin vitamin C are soluble in water so they
(i) Biuret - Protein solution + NaOH + dil. CuSO4 ® Pink or are grouped together. Water soluble vitamins must be supplied
violet colour regularly in diet because they are readily excreted in urine
(ii) Ninhydrin - Protein solution + Ninhydrin ® blue colour and can not be stored (except vitamin B12) in our body.
(iii) Hopkin’s cole - Protein solution + glyoxalic acid + conc. H2SO4 Some Important Vitamins
® Blue - violet
(iv) Million’s - Protein solution + Millon’s reagent ® Pink colour.. Name of Vitamin Sources Deficiency Diseases
Millon’s reagent - Solution of mercuric nitrate and nitrite in 1. vit. A fish, liver oil, Xerophthalmia, night
nitric acid containing traces of nitrous acid. (Retinal) carrots, butter, milk blindness
(v) Iodine reaction - Protein solution + iodine in potassium iodide 2. vit B1 (Thiamine) yeast, milk, green Beri beri
solution ® yellow colour vegetables, cereals
(vi) Xanthoproteic test - Protein solution + conc HNO3 ® yellow 3. vit B2 (Riboflavin) milk, eggwhite, chielosis, digestive
NaOH liver, kidney disorders, burning
colour ¾¾ ¾
¾® orange colour
sensation of skin
ENZYMES
4. vit B6 (pyridoxine) yeast, milk, egg convulsions
Enzymes are biological catalysts which catalyse biological
yolk, cereals, grams
reactions by providing alternate lower activation energy pathways
thereby increasing rate of reaction. 5. vit B12 meat, fish, egg, pernicious anaemia
Chemically, all enzymes are globular proteins. Some enzymes are (Cyanocobalamine) curd
associated with some non-protein component called cofactor for
6. vit. C (Ascorbic citrus fruits, amla, scurvy
their activity. These cofactors can be inorganic ions or organic
molecules. acid) green leafy
vegetables.
Characteristics of Enzymes
7. vit. D sunlight, fish, egg Rickets, osteomalacia
(i) Specificity, i.e., each enzyme catalysis only one chemical
reaction. (Calciferol) yolk
(ii) Efficiency, i.e., they can speed up the rate of a reaction by a 8. vit. E vegetable oils increased fragility of
factor of 1020. (Tocoferol) RBC's and muscular
(iii) Small quantity i.e., only small quantity of enzymes can be weakness.
highly efficient. 9. vit. K green leafy increased blood
(in) Enzymes function best at an optimum temperature (Phylloquinone) vegetables clotting time
(25°C – 40°C) and show maximum efficiency at an optimum
pH(6 – 7.7) NUCLEIC ACIDS
Mechanism of Enzyme Catalysis Nucleic acids are long chain polymers of nucleotides and are hence
It is known as ‘lock and key’ Mechanism also called polynucleotides.
Step 1: Formation of enzyme-substrate complex Nucleic acids are made up of three units : bases, sugar and
phosphoric acid. They are required for the storage and expression
E + S ES (fast and reversible)
Enzyme Substrate of genetic information.
Step 2: Dissociation of the complex to form products Nucleic acids are of two types:
(i) Deoxyribonucleic acid (DNA)
ES [EP] E+P (ii) Ribonucleic acid (RNA)
Enzyme- Product
Chemical Composition of Nucleic Acids
product
Complete hydrolysis of DNA (or RNA) yields a pentose sugar,
association
(slow and rate determining) phosphoric acid and nitrogen containing heterocyclic compounds
(called bases).
VITAMINS Sugars: Two sugars are found to be present in nucleic acids,
They are regarded as organic compounds required in the diet in
namely D-ribose in R.N.A. and 2-deoxyribose in D.N.A.
small amounts to perform specific biological functions for normal
maintenance of optimum growth and health of the organism. Bases: Two types of bases have been isolated from the
EBD_7327
714 CHEMISTRY

hydrolysis products of nucleosides; viz. purines and forms two H-bonds with ‘T’ while ‘C’ form thr ee
pyrimidines. Important purine bases are adenine and guanine; H-bonds with ‘C’.
while pyrimidine bases are uracil, thymine and cytosine. Structure of RNA
Adenine, guanine and cytosine are present in RNA as well as In RNA, only single stranded helices are present. RNA molecules
in DNA, while thymine is present only in DNA and uracil only are of 3 types and perform different functions. They are m-RNA, r-
RNA and t-RNA.
in RNA.
A simplified version of nucleic acid chain is
Biological Functions of Nucleic Acids
(i) DNA is responsible for maintaining identity of different
Base Base Base species of organisms. A DNA molecule is capable of self
duplication during cell division and identical DNA strands
Sugar Phosphate ( Sugar Phosphate )n Sugar are transferred to daughter cells.
(ii) They help in protein synthesis in the cell.
Nucleosides and Nucleotides Gene
Nucleosides: These are condensation products of a base with The portion of DNA carrying information about a specific protein
sugar. The five important nucleosides are named as adenosine, is called gene.
guanosine, cytidine, uridine and thymidine. Genetic Code
Nucleotides: These are condensation products of nucleosides The relation between the amino acid and the nucleotide triplet is
with phosphoric acid. The five important nucleotides are named called genetic code.
as adenylic acid, guanylic acid, cytidylic acid, uridylic acid and Codons
thymidylic acid. The nucleotide bases in RNA function in groups of three (triplet)
Structure of DNA in coding amino acids. These base triplets are called codons.
Primary structure tells about the sequence of nucleotides in DNA Mutation
The two polynucleotide chains of DNA molecule are twisted A chemical change in DNA molecule that could lead to the synthesis
around a common axis but run in opposite directions to form a of proteins with an altered sequence of amino acids.
right bonded helix. Mutation leads to the formation of defective genes which causes
In secondary structure of DNA, the nucleic acid chains wound abnormalities or disease.
about each other and are held by H-bonds between specific pairs
of bases. The two strands are complementary to each other. ‘A’
Biomolecules 715

CONCEPT MAP
EBD_7327
716 CHEMISTRY

1. Which one of the following metals is required as co-factor 16. Anaemia is caused by the deficiency of vitamin
by all enzymes utilizing ATP in phosphate transfer? (a) B6 (b) B1 (c) B2 (d) B12
(a) K (b) Ca (c) Na (d) Mg 17. Which of the following monosaccharide is a pentose ?
2. Which of the following is the sweetest sugar?
(a) Sucrose (b) Glucose (c) Fructose (d) Maltose (a) Galactose (b) Glucose
3. In cells the net production of ATP molecules generated from (c) Fructose (d) Arabinose
one glucose molecule is 18. Glucose gives silver mirror with Tollen’s reagent. It shows
(a) 46 (b) 32 (c) 36 (d) 40 the presence of
4. In aqueous solution, an amino acid exists as (a) an acidic group (b) an alcoholic group
(a) cation (b) anion
(c) a ketonic group (d) an aldehydic group
(c) dianion (d) zwitter ion
5. Which is correct statement? 19. Hydrolysis of sucrose is called
(a) Starch is a polymer of a-glucose (a) esterification (b) saponification
(b) In cyclic structure of fructose, there are four carbons (c) inversion (d) hydration
and one oxygen atom 20. Glucose on oxidation with bromine water gives
(c) Amylose is a component of cellulose
(a) gluconic acid (b) tartaric acid
(d) Proteins are composed of only one type of amino acids
6. Glycolysis is (c) saccharic acid (d) mesoxalic acid
(a) conversion of glucose to haem 21. The highest calorific value is found in
(b) oxidation of glucose to glutamate (a) proteins (b) fats
(c) conversion of pyruvate to citrate (c) vitamins (d) carbohydrates
(d) oxidation of glucose to pyruvate
22. The number of essential amino acids in man is
7. Phospholipids are esters of glycerol with
(a) three phosphate groups (a) 8 (b) 10 (c) 18 (d) 20
(b) three carboxylic acid residues 23. Number of chiral carbon atoms in glucose and fructose are
(c) two carboxylic acid residues and one phosphate group (a) 4 in each
(d) one carboxylic acid residue and two phosphate groups
(b) 3 in each
8. Glucose does not react with
(a) Br2/H2O (b) H2NOH (c) HI (d) NaHSO3 (c) 4 in glucose and 3 in fructose
9. a-D- Glucose and b- D-glucose differ from each other due (d) 3 in glucose and 4 in fructose
to difference in one carbon with respect to its O
|| · ·
(a) size of hemiacetal ring (b) number of OH groups 24. For - C - N H - (peptide bond )
(c) configuration (d) conformation
10. The glycosidic linkage involved in linking the glucose units (a) C — N bond length in proteins is longer than usual
in amylose part of starch is bond length of the C — N bond
(a) C1 – C4 b linkage (b) C1 – C6 a linkage (b) Spectroscopic analysis shows planar structure of the
(c) C1 – C5 a linkage (d) C1 – C4 a linkage — C — NH — group
11. In fructose, the possible optical isomers are ||
O
(a) 12 (b) 8 (c) 16 (d) 4 (c) C — N bond length in proteins is smaller than usual
12. Which of the following does not reduce Benedict’s
bond length of the C—N bond
solution?
(a) Glucose (b) Fructose (d) None of the above
(c) Sucrose (d) Aldehyde 25. Which is not a true statement?
13. Sucrose on hydrolysis gives (a) a-Carbon of a-amino acid is asymmetric
(a) fructose+ribose (b) glucose + fructose
(b) All proteins are found in L-form
(c) glucose+glucose (d) fructose + fructose
14. Pick out the one which does not belong to the same family: (c) Human body can synthesize all proteins they need
(a) pepsin (b) cellulose (d) At pH = 7 both amino and carboxylic groups exist in
(c) ptyalin (d) lipase ionised form
15. The change in optical rotation, with time, of freshly prepared 26. The metal present in vitamin B12 is :
solution of sugar is known as
(a) cobalt (b) iron
(a) rotatory motion (b) inversion
(c) specific rotation (d) mutarotation (c) manganese (d) magnesium
Biomolecules 717

27. Which one of the following structures represents the peptide 38. The process by which synthesis of protein takes place based
chain? on the genetic information present in m-RNA is called
(a) Translation (b) Transcription
H O
| | || (c) Replication (d) Messenger hypothesis
(a) - N - C - N - C - NH - C - NH - 39. The bond that determines the secondary structure of proteins
|| | | is
O H (a) coordinate bond (b) covalent bond
(c) hydrogen bond (d) peptide bond
H H
| | | | | | | | 40. DNA has deoxyribose, a base and the third component which is
(b) - N - C - C- C - C- N - C - C- C - (a) phosphoric acid (b) ribose
|| | | | | | | (c) adenine (d) thymine
O 41. Enzymes take part in a reaction and
(a) decrease the rate of a chemical reaction
H H H O (b) increase the rate of a chemical reaction
| | | | | | || (c) Both (a) and (b)
(c) - N - C- C - N - C - C - N - C - C - (d) None of these
| || | || |
O O 42. Lysine is least soluble in water in the pH range:
(a) 3 to 4 (b) 5 to 6 (c) 6 to 7 (d) 8 to 9
H O H 43. Which one of the following chemical units is certainly to be
| | | || | | | | |
(d) - N - C- C- C - N - C- C- N- C - C- C- found in an enzyme?
| | | | | || | |
H O OH H
O
28. A sequence of how many nucleotides in messenger RNA O N—C
(a) (b)
makes a codon for an amino acid? HO HO O O
(a) Three (b) Four (c) One (d) Two
29. Glucose molecule reacts with X number of molecules of
O
phenylhydrazine to yield osazone. The value of X is
(a) three (b) two (c) one (d) four O O R
30. The helical structure of protein is stabilized by N
(a) dipeptide bonds (b) hydrogen bonds (c) (d) O R
(c) ether bonds (d) peptide bonds N O R
31. A triglyceride can have how many different acyl groups?
(a) 3 (b) 2 (c) 1 (d) 4 O
32. Which functional group participates in disulphide bond 44. When adenine is attached to ribose sugar, it is called
formation in proteins? adenosine. To make a nucleotide from it, it would require
(a) Thioester (b) Thioether (a) oxygenation (b) addition of a base
(c) Thiol (d) Thiolactone (c) addition of phosphate (d) hydrogenation
33. Which of the following is not an amino acid? 45. Fructose on reduction gives a mixture of two alcohols which
(a) Glycine (b) Adenine are related as
(c) Histidine (d) Benzidine (a) diastereomers (b) epimers
34. Which of the following is not a function of proteins? (c) both (a) and (b) (d) anomers.
(a) Nail formation 46. Which of the following is correct about H-bonding in
(b) Skin formation nucleotide?
(c) Muscle formation (a) A --- A and T --- T (b) G --- T and A --- C
(d) Providing energy for metabolism (c) A --- G and T --- C (d) A --- T and G --- C
35. The structural feature which distinguishes proline from 47. Chargaff's rule states that in an organism
natural a-amino acids? (a) Amounts of all bases are equal
(a) Proline is optically inactive (b) Amount of adenine (A) is equal to that of thymine (T)
(b) Proline contains aromatic group and the amount of guanine (G) is equal to that of cytosine
(c) Proline is a dicarboxylic acid (C)
(d) Proline is a secondary amine (c) Amount of adenine (A) is equal to that of guanine (G)
36. Pepsin enzyme hydrolyses and the amount of thymine (T) is equal to that of cytosine
(a) proteins to amino acids (C)
(b) fats to fatty acids (d) Amount of adenine (A) is equal to that of cytosine (C)
(c) glucose to ethyl alcohol and the amount of thymine (T) is equal to that of guanine
(d) polysaccharides to monosaccharides (G)
37. Which of the following tests is not used for testing proteins? 48. The enzyme which hydrolyses triglycerides to fatty acids
(a) Millon’s test (b) Molisch’s test and glycerol is called
(c) Biuret test (d) Ninhydrin test (a) maltase (b) lipase (c) zymase (d) pepsin
EBD_7327
718 CHEMISTRY

49. Of the following statements about enzymes which ones (b) three -dimensional structure, especially the bond
are true? between amino acid residues that are distinct from each
(i) Enzymes lack in nucleophilic groups other in the polypeptide chain
(ii) Enzymes are highly specific both in binding chiral (c) linear sequence of amino acid residues in the
substrates and in catalysing their reactions polypetide chain
(iii) Enzymes catalyse chemical reactions by lowering the (d) regular folding patterns of continous portions of the
energy of activation polypeptide chain
(iv) Pepsin is a proteolytic enzyme 64. RNA and DNA are chiral molecules, their chirality is due to
(a) (i) and (iv) (b) (i) and (iii) (a) chiral bases
(c) (ii), (iii) and (iv) (d) (i) (b) chiral phosphate ester units
50. Which of the following is not present in a nucleotide? (c) D-sugar component
(a) Guanine (b) Cytosine (d) L-sugar component
(c) Adenine (d) Tyrosine 65. Which one of the following is an amine hormone ?
51. If one strand of DNA has the sequence ATGCTTGA, the (a) Thyroxine (b) Oxypurin
sequence in the complimentary strand would be (c) Insulin (d) Progesterone
(a) TCCGAACT (b) TACGTAGT 66. In DNA, the complimentary bases are:
(c) TACGAACT (d) TAGCTAGT (a) adenine and thymine; guanine and cytosine
52. Vitamin K is associated with the disease: (b) adenine and thymine ; guanine and uracil
(a) Scurvy (b) Beri-beri (c) adenine and guanine; thymine and cytosine
(c) Blood coagulation (d) Sore throat (d) uracil and adenine; cytosine and guanine
53. Which of the following is a steroid hormone? 67. The segment of DNA which acts as the instrumental manual
(a) Cholesterol (b) Adrenaline for the synthesis of the protein is:
(c) Thyroxine (d) Progesterone (a) ribose (b) gene
(c) nucleoside (d) nucleotide
54. Which one is a female sex hormone?
68. Fructose reduces Tollen’s reagent due to:
(a) Estrogen (b) Estradiol
(a) enolisation of fructose followed by conversion to
(c) Progesterone (d) All of these
glucose (having aldehydic group) by the base present
55. For osazone formation, the effective structural unit necessary is
in Tollen’s reagent
(a) CH2OCH3 (b) CH 2OH (b) asymmetric carbons
| | (c) primary alcoholic group
CO CO (d) secondary alcoholic group
| | 69. Which one of the following statements is incorrect about
(c) CH 2OH (d) CHO enzyme catalysis?
| | (a) Enzymes are mostly proteinous in nature.
CHOCH 3 CHOCH 3 (b) Enzyme action is specific.
| | (c) Enzymes are denaturated by ultraviolet rays and at high
56. Vitamin B12 contains temperature.
(a) Ca(II) (b) Fe(II) (c) Co(III) (d) Zn(II) (d) Enzymes are least reactive at optimum temperature.
57. The human body does not produce 70. Deficiency of vitamin B1 causes the disease
(a) Convulsions (b) Beri-Beri
(a) vitamins (b) hormones (c) enzymes (d) DNA
(c) Cheilosis (d) Sterility
58. Biotin is an organic compound present in yeast. Its
71. Which of the following acids does not exhibit optical
deficiency in diet causes dermatitis and paralysis. It is also
isomerism ?
known as
(a) Maleic acid (b) a -amino acids
(a) Vitamin H (b) Vitamin B3 (c) Lactic acid (d) Tartaric acid
(c) Vitamin B12 (d) Vitamin D 72. a - D-(+)-glucose and b-D-(+)-glucose are
59. The chemical change in DNA molecule that could lead to (a) conformers (b) epimers
synthesis of protein with an altered amino acid sequence is (c) anomers (d) enatiomers
called 73. The two functional groups present in a typical carbohydrate
(a) replication (b) lipid formation are:
(c) cellular membrane (d) mutation (a) – CHO and – COOH (b) > C = O and – OH
60. Which one of the following is a non-steroidal hormone? (c) – OH and – CHO (d) – OH and – COOH
(a) Estradiol (b) Prostaglandin 74. Biuret test is not given by
(c) Progesterone (d) Estrone (a) carbohydrates (b) polypeptides
61. Vitamin B6 is known as (c) urea (d) proteins
(a) pyridoxine (b) thiamine 75. The presence or absence of hydroxyl group on which carbon
(c) tocopherol (d) riboflavin atom of sugar differentiates RNA and DNA?
62. Which one of the following is a peptide hormone ? (a) 1st (b) 2nd
(a) Testosterone (b) Thyroxin (c) 3 rd (d) 4 th
(c) Adrenaline (d) Glucagon
63. Secondary structure of protein refers to
(a) mainly denatured proteins and structure of prosthetic
groups
Biomolecules 719

1. Match the vitamin of column I with deficiency disease given 13. Vitamin A is present in
in column II (a) cod liver oil (b) carrot
Column I Column II (c) milk (d) in all of these
1. Vitamin A a. Scurvy 14. The base present in DNA, but not in RNA is
2. Vitamin B12 b. Hemorrhagic (a) guanine (b) adenine
condition (c) uracil (d) thymine
3. Vitamin C c. Sterility 15. Energy is stored in our body in the form of
4. Vitamin E d. Xerophthalmia
(a) ATP (b) ADP
5. Vitamin K e. Pernicious anaemia
(c) Fats (d) carbohydrates
(a) 1-e, 2-d, 3-a, 4-b, 5-c (b) 1-d, 2-e, 3-a, 4-b, 5-c
16. Which of the following is a non-reducing sugar ?
(c) 1-d, 2-e, 3-a, 4-c, 5-b (d) 1-e, 2-d, 3-a, 4-c, 5-b
(a) Sucrose (b) Maltose
2. The base adenine occurs in
(c) Lactose (d) Fructose
(a) DNA only (b) RNA only
17. Glycosidic linkage is actually an :
(c) DNA and RNA both (d) Protein (a) Carbonyl bond (b) Ether bond
3. Glucose is converted into alcohol by the action of the enzyme (c) Ester bond (d) Amide bond
(a) invertase (b) maltase 18. When a-D-glucose and b-D-glucose are dissolved in water
(c) zymase (d) diastase in two separate beakers I and II respectively and allowed to
4. Which one of the following forms the constituent of cell stand, then –
wall of plant cells ? (a) specific rotation in beaker I will decrease while in II will
(a) Starch (b) Glycogen increase upto a constant value
(c) Cellulose (d) Amylose (b) the specific rotation of equilibrium mixture in two
5. Starch is converted into maltose by the beakers will be different
(a) maltase (b) invertase (c) the equilibrium mixture in both beakers will be
(c) zymase (d) diastase leavorotatory
6. The disaccharide present in milk is (d) the equilibrium mixture in both beakers will contain only
(a) maltose (b) lactose cyclic form of glucose
(c) sucrose (d) cellobiose 19. Nucleotide in DNA are linked by –
7. How many amino acids are present in insulin? (a) hydrogen bond
(a) 25 (b) 51 (c) 20 (d) 22 (b) 3'-5' phosphodiester bond
8. On heating glucose with Fehling’s solution we get a (c) glycosidic bond
precipitate whose colour is (d) peptide bond
(a) yellow (b) red 20. Natural glucose is termed D-glucose because :
(c) black (d) white (a) – OH on the second carbon is on the right side in Fischer
projection
9. Which of the following gives maximum energy in metabolic (b) – OH on the sixth carbon is on the right side in Fischer
processes? projection.
(a) Proteins (b) Carbohydrates (c) – OH on the fifth carbon is on the right side in Fischer
(c) Lipids (d) Vitamins projection.
10. A distinctive and characteristic functional group of fats is (d) It is dextrorotatory.
(a) an ester group (b) a peptide group 21. The dipeptide, Gly. Ala has structure –
(c) a ketonic group (d) an alcoholic group O
O
11. Which one of the following is NOT correct? Å || – Å || –
(a) D(–) Fructose exists in furanose structure (a) H 3 NCH 2 CNHCHCO 2 (b) H 3 NCH 2 CNHCHCO 2
(b) D (+) Glucose exists in pyranose structure | |
CH 3 CH 3
(c) In sucrose the two monosaccharides are held together
by peptide linkage
O O
(d) Maltose is a reducing sugar – || Å – || Å
12. Hydrolytic reaction of fats, with caustic soda, is known as (c) O 2 CCHNH - CCH2 NH3 (d) O2 CCH2 NH - CCHNH3
(a) acetylation (b) carboxylation | |
CH 3 CH3
(c) saponification (d) esterification
EBD_7327
720 CHEMISTRY

22. The pair of compounds in which both the compounds give 31. Which one of the following statements is correct?
positive test with Tollen’s reagent is (a) All amino acids except lysine are optically active
(a) glucose and sucrose (b) All amino acids are optically active
(b) fructose and sucrose (c) All amino acids except glycine are optically active
(c) acetophenone and hexanal (d) All amino acids except glutamic acids are optically
(d) glucose and fructose active
23. The two forms of D-glucopyranose obtained from the 32. Cellulose upon acetylation with excess acetic anhydride/
solution of D-glucose are called H2SO4 (catalytic) gives cellulose triacetate whose structure
(a) isomers (b) anomers is
(a)
(c) epimers (d) enantiomers
AcO
24. Double stranded DNA virus with 20,000 base pairs has O
nucleotides AcO
H H O

(a) 20,000 (b) 10,000 O


AcO H OAc H H
(c) 666 (d) 40,000 H O
O
25. Which of the following statement is not correct ? H
H OAc H H OAc
O H
(a) Amylopectin is a branched polymer of a - glucose.
(b) Cellulose is a linear polymer of b-glucose O OAc H
H
H OAc

(c) Glycogen is the food reserve of plants H OAc


(d) All proteins are polymers of a - amino acids. (b)
26. Which of the following hormones contains iodine?
AcO
(a) Testosterone (b) Adrenaline
O
(c) Thyroxine (d) Insulin AcO H O
H
27. Which one of the following does not exhibit the O
OH H
phenomenon of mutarotation ? AcO
H H O H

(a) (+) – Sucrose (b) (+) – Lactose H O


OH H H OH
H O H
(c) (+) – Maltose (d) (–) – Fructose
28. Which one of the following statements is not true regarding O OH H
H
H OH

(+) Lactose ?
H OH
(a) On hydrolysis (+) Lactose gives equal amount of D(+) (c)
glucose and D(+) galactose. AcO
AcO
(b) (+) Lactose is a b-glycoside formed by the union of a AcO
O H O
H H H O
O
molecule of D(+) glucose and a molecule of D(+) H
H
H H
galactose. H O OAc H H
O OAc
(c) (+) Lactose is a reducing sugar and does not exhibit O OAc H
H OAc
mutarotation. H OAc
H OAc
(d) (+) Lactose, C12H22O11 contains 8-OH groups. (d)
29. Which of the statements about "Denaturation" given below AcO
AcO
AcO
are correct ? H O H O
O
O H H
(A) Denaturation of proteins causes loss of secondary and H
H
H H

tertiary structures of the protein. O H H O H H H


H H
(B) Denturation leads to the conversion of double strand O
OAc OAc
of DNA into single strand OAc
OAc OAc
OAc
(C) Denaturation affects primary structure which gets 33. The correct statement about the following disaccharide is
distorted CH2OH HOH2C O H
Options : H O H
(a) (B) and (C) (b) (A) and (C) H (A) H (B)
OH H HO
(c) (A) and (B) (d) (A), (B) and (C) OCH2CH2O CH2OH
HO
30. The secondary structure of a protein refers to OH H
H OH
(a) fixed configuration of the polypeptide backbone (a) Ring (A) is pyranose with a - glycosidic link
(b) a– helical backbone (b) Ring (A) is furanose with a - glycosidic link
(c) hydrophobic interactions (c) Ring (B) is furanose with a - glycosidic link
(d) sequence of a– amino acids. (d) Ring (B) is pyranose with b - glycosidic link
Biomolecules 721

34. The following carbohydrate is 43. Which one of the following metals is required as co-factor
by all enzymes utilizing ATP in phosphate transfer?
(a) K (b) Ca
(c) Na (d) Mg
44. Secondary structure of protein is mainly governed by
(a) hydrogen bonds (b) covalent bonds
(a) a ketohexose (b) an aldohexose (c) ionic bonds (d) disulphide bonds
(c) an a-furanose (d) an a-pyranose 45. The linkage between the two monosaccharide units in
35. Synthesis of each molecule of glucose in photosynthesis lactose is
involves : (a) C1 of b-D-glucose and C4 of b-D-galactose
(a) 18 molecules of ATP (b) 10 molecules of ATP (b) C1 of b-D-galactose and C4 of b-D-glucose
(c) 8 molecules of ATP (d) 6 molecules of ATP (c) C1 of a-D-galactose and C4 of b-D-glucose
36. Which of the following compounds can be detected by (d) C1 of b-D-galactose and C4 of a-D-glucose
Molisch's Test ? DIRECTIONS for Qs. 46 to 50 : These are Assertion-Reason
(a) Nitro compounds (b) Sugars type questions. Each of these question contains two statements:
Statement-1 (Assertion) and Statement-2 (Reason). Answer these
(c) Amines (d) Primary alcohols
questions from the following four options.
37. The glycosidic linkage involved in linking the glucose units
(a) Statement-1 is True, Statement-2 is True, Statement-2 is a
in amylose part of starch is
correct explanation for Statement-1
(a) C1 – C4 b linkage (b) C1 – C6 a linkage
(b) Statement-1 is True, Statement-2 is True ; Statement-2 is
(c) C1 – C5 a linkage (d) C1 – C4 a linkage
NOT a correct explanation for Statement-1
38. Which of the following are neutral amino acids?
(c) Statement-1 is True, Statement-2 is False
(a) Glycine (b) Valine
(d) Statement-1 is False, Statement-2 is True
(c) Proline (d) All of these
46. Statement-1 : Glycine is amphoteric in nature.
39. Glucose on oxidation with bromine water gives
Statement-2 : Glycine contains both acid and basic groups.
(a) gluconic acid (b) tartaric acid
47. Statement-1 : Hydrolysis of sucrose is known as inversion
(c) saccharic acid (d) mesoxalic acid of cane sugar.
40. Number of chiral carbon atoms in glucose and fructose are Statement-2 : Sucrose is a disaccharide.
(a) 4 in each 48. Statement-1 : Proteins on hydrolysis produce amino acids.
(b) 3 in each Statement-2 : Amino acids contain –NH2 and –COOH
(c) 4 in glucose and 3 in fructose groups.
(d) 3 in glucose and 4 in fructose 49. Statement-1 : Sequence of bases in DNA is TGAACCCTT
41. Glucose does not react with and sequence of bases in m-RNA is CATTAAACC.
(a) Br2/H2O (b) H2NOH Statement-2 : In DNA, nitrogenous bases have hydrogen
(c) HI (d) NaHSO3 bonds.
42. The vitamin that is not soluble in water is 50. Statement-1 : Vitamin B5 is also called as pyridoxin.
(a) Vitamin B1 (b) Vitamin B2 Statement-2 : Deficiency of vitamin B5 causes dermatitis
(c) Vitamin B6 (d) Vitamin D and dementia.

Exemplar Questions (a) amylose (b) amylopectin


1. Glycogen is a branched chain polymer of a-D glucose units (c) cellulose (d) glucose
in which chain is formed by C1–C4 glycosidic linkage where 2. Which of the following polymer is strored in the liver of
as branching occurs by the formation of C1–C6 glycosidic animals ?
linkage. Structure of glycogen is similar to .......... . (a) Amylose (b) Cellulose
(c) Amylopectin (d) Glycogen
EBD_7327
722 CHEMISTRY

3. Sucrose (Cane sugar) is a disaccharide. One molecule of CH2OH CH2OH


sucrose on hydrolysis gives .......... . OH O OH O H
H H
(a) 2 molecules of glucose
(c) OH H OH H
(b) 2 molecules of glucose + 1 molecule of fructose H O OH
H
(c) 1 molecule of glucose + 1 molecule of fructose H OH H OH
(d) 2 molecules of fructose CH2 OH CH2OH
4. Which of the following pairs represents anomers? OH O H O OH
CHO CHO H H
HO (d) OH H OH H
H OH H H O H
H
HO H HO H H OH H OH
(a) H OH H OH 7. Which of the following acids is a vitamin?
H OH H OH (a) Aspartic acid (b) Ascorbic acid
CH2OH CH2OH (c) Adipic acid (d) Saccharic acid
CHO CHO 8. Dinucleotide is obtained by joining two nucleotides together
H OH HO H by phosphodiester linkage. Between which carbon atoms of
HO H H OH pentose sugars of nucleotides are these linkages present?
(b) OH HO H (a) 5' and 3' (b) 1' and 5'
H
(c) 5' and 5' (d) 3' and 3'
H OH HO H
9. Nucleic acids are the polymers of......
CH2OH CH2OH (a) nucleosides (b) nucleotides
H OH HO H (c) bases (d) sugars
H 10. Which of the following statements is not true about glucose?
H OH OH
(a) It is an aldohexose
(c) HO H O HO H O
(b) On heating with HI it forms n-hexane
H OH H OH (c) It is present in furanose form
H H (d) It does not give 2, 4-DNP test
CH2OH CH2OH 11. Each polypeptide in a protein has amino acids linked with
each other in a specific sequence. This sequence of amino
H OH HO H
acids is said to be ......... .
H OH HO H
(a) primary structure of proteins
(d) HO H O H OH O (b) secondary structure of proteins
H OH HO H (c) tertiary structure of proteins
H H (d) quateranary structure of proteins
CH2OH CH2OH 12. DNA and RNA contain four bases each. Which of the
5. Proteins are found to have two different types of secondary following bases is not present in RNA?
structures viz. a-helix and b-pleated sheet structure. a-helix (a) Adenine (b) Uracil
structure of protein is stabilised by: (c) Thymine (d) Cytosine
(a) peptide bonds 13. Which of the following B group vitamins can be stored in
(b) van der Waal’s forces our body?
(c) hydrogen bonds (a) Vitamin B1 (b) Vitamin B2
(d) dipole-dipole interactions (c) Vitamin B6 (d) Vitamin B12
6. In disaccharides, if the reducing groups of monosaccharides, 14. Which of the following bases is not present in DNA?
i.e., aldehydic or ketonic groups are bonded, these are non- (a) Adenine (b) Thymine
reducing sugars. Which of the following disaccharide is a (c) Cytosine (d) Uracil
non-reducing sugar? 15. Three cyclic structures of monosaccharides are given below
CH2OH
which of these are anomers.
CH2OH
H O H H O H HO H
H H
OH H OH H H OH HO H HO H
(a) O OH H O
OH H OH H OH HO
H OH H OH HO H O HO H O H OH
H OH H OH HO H
CH2OH H H H
O H O H CH2OH CH2OH CH2OH
H HOH2C
H
H OH (i) (ii) (iii)
(b) OH H
OH O CH2OH (a) I and II (b) II and III
H OH OH H (c) I and III (d) III is anomer of I and II
Biomolecules 723

16. Which of the following reactions of glucose can be explained (b) (A) and (B) are between C1 and C4, (C) is between C1
only by its cyclic structure? and C6
(a) Glucose forms pentaacetate (c) (A) and (C) are between C1 and C4, (B) is between C1
(b) Glucose reacts with hydroxylamine to form an oxime and C6
(c) Pentaacetate of glucose does not react with hydroxyl amine (d) (A) and (C) are between C1 and C6, (B) is between C1
(d) Glucose is oxidised by nitric acid to gluconic acid and C4
17. Optical rotations of some compounds alongwith their
NEET/AIPMT (2013-2017) Questions
structures are given below which of them have D configuration.
CH2OH 20. In DNA the linkages between different nitrogenous bases
CHO are: [NEET Kar. 2013]
CHO
H OH C=O (a) peptide linkage (b) phosphate linkage
H OH HO H HO H (c) H-bonding (d) glycosidic linkage
H OH H OH 21. D (+) glucose reacts with hydroxylamine and yields an oxime.
CH2OH H OH H OH The structure of the oxime would be : [2014]
(a) CH = NOH (b) CH = NOH
CH2OH CH2OH
(+) rotation (+) rotation (–) rotation H – C – OH HO – C – H
(i) (ii) (iii) HO – C – H HO – C – H
(a) I, II, III (b) II, III
HO – C – H H – C – OH
(c) I, II (d) III
18. Structure of disaccharide formed by glucose and fructose is H – C – OH H – C – OH
given below. Identify anomeric carbon atoms in
CH2OH CH2OH
monosaccharide units. CH = NOH CH = NOH
f (c) (d)
CH2OH HO – C – H H – C – OH
e a
O H O H
H HOH2C
H a b
e H – C – OH HO – C – H
d H
OH H
OH
O CH2OH HO – C – H H – C – OH
c b c df
OH
H OH OH H H – C – OH H – C – OH
(a) 'a' carbon of glucose and 'a' carbon of fructose
(b) 'a' carbon of glucose and 'e' carbon of fructose CH2OH CH2OH
(c) 'a' carbon of glucose and 'b' carbon of fructose 22. Which of the following hormones is produced under the
(d) 'f ' carbon of glucose and 'f ' carbon of fructose condition of stress which stimulates glycogenolysis in the
19. Three structures are given below in which two glucose units liver of human beings?
are linked. Which of these linkages between glucose units (a) Thyroxin (b) Insulin [2014]
are between C1 and C4 and which linkages are between C1 (c) Adrenaline (d) Estradiol
and C6? 23. In a protein molecule various amino acids are linked together
CH2OH CH2OH
by [2016]
O O OH
H (a) a-glycosidic bond (b) b-glycosidic bond
H
H (A) H
OH H OH H (c) peptide bond (d) dative bond
OH O H
H 24. The correct statement regarding RNA and DNA, respectively
H OH H OH is [2016]
(I) (a) The sugar component in RNA is arabinose and the
CH2OH sugar component in DNA is 2'-deoxyribose.
CH2OH
O H (b) The sugar component in RNA is ribose and the sugar
O OH
H
H H component in DNA is 2'-deoxyribose.
OH H H
(C) OH H (c) The sugar component in RNA is arabinose
HO H (d) The sugar component in RNA is 2'-deoxyribose and the
H OH CH2OH O H
OH sugar component in DNA is arabinose
O (B) H O
H 25. Which one given below is a non-reducing sugar?
CH2 OH H (a) Maltose (b) Lactose [2016]
H O H HO H (c) Glucose (d) Sucrose
H H OH 26. Which of the following statements is not correct : [2017]
OH H
OH OH (III) (a) Ovalbumin is a simple food reserve in egg-white
H OH
(b) Blood proteins thrombin and fibrinogen are involved in
(II) blood clotting
(a) (A) is between C1 and C4, (B) and (C) are between C1 (c) Denaturation makes the proteins more active
and C6 (d) Insulin maintains sugar level in the blood of a human body
EBD_7327
724 CHEMISTRY

Hints & Solutions


EXERCISE - 1 12. (c) Sucrose, being a non-reducing sugar, does not reduce
Benedict’s solution. Remember that fructose has an a-
1. (d) Mg acts as a cofactor to the enzyme as it forms complex hydroxy ketonic group, which is also reducing group
with phosphate group of ATP that is utilized in the (difference from ordinary ketonic group)
transfer of ATP phosphate.
13. (b) Sucrose is a disaccharide which on hydrolysis gives
2. (c) Fructose is the sweetest sugar.
one molecule of glucose (monosaccharide) and fructose
3. (c) C 6 H12 O 6 + 6O 2 + 2 ATP ¾
¾® (monosaccharide).

6CO 2 + 6H 2O + 38 ATP Molecules H+


C12 H 22 O11 + H 2 O ¾¾¾
®
Net total number of ATP molecules evolved = 36 sucrose
molecules.
4. (d) In neutral solution, amino acids exists as dipolar ion C6 H12 O 6 + C6 H12 O 6
glucose fructose
(also known as zwitter ions or inner salts) where the
proton of –COOH group is transferred to the – NH2 14. (b) Cellulose is a type of carbohydrate while pepsin, ptyalin
group to form inner salt, known as dipolar ion. and lipase are the enzymes which helps in digestion.
R R 15. (d) a – D – Glu cos e ƒ Equilibrium mixture ƒ
.. | H 2O .. | [a ]=+112° [a ]D =+52°
H2N–CH–COOH –
H2N–CHOO +H
+
( 36%) ( 0.02%)
a-Amino acid
R
|
+
H3N–CH–COO– b – D – Glu cos e
Zwitter ion [a ]D =+19°
( 64%)
5. (a) Starch is also known as amylum which occurs in all green
plants. A molecule of starch (C 6 H10 O5 ) n is built of a Glucose has two forms a and b . When either of these
large number of a-glucose ring joined through oxygen- two is dissolved in water and allowed to stand, it gets
atom. converted to an equilibrium mixture of a and b forms.
6. (d) It is a common pathway for both the aerobic & anaerobic 16. (d) Vitamin Disease caused by deficiency
respiration in which 1 glucose molecule is converted to
B6 Dermatitis
2 molecules of pyruvate.
B1 Beri-beri
7. (c) Phospholipids are derivatives of glycerol in which two
B2 Photophobia, glossitis
of the hydroxyl groups are esterified with fatty acids
B12 Pernicious anaemia
while the third is esterified with some derivative of
phosphoric acid with some alcohol such as choline, C Scurvy
ethanolamine, serine or inositol. 17. (d) Arabinose is an aldopentose HOCH2–(CHOH)3–CHO
R¢¢ 18. (d) Glucose + Tollen’s reagent ® Gluconic acid + Ag-mirror
O 19. (c)
O = P – O – CH2 CH 2OH ( CHOH )4 CHO + [O] ¾¾¾¾¾
Br / H O
O 20. (a) 2 2 ®

Glucose
OH CH – O – C –R¢
CH2 OH ( CHOH )4 COOH
CH2O – C – R Gluconic acid
O 21. (b) 1 g fat provide 37 kJ of energy on oxidation while 1 g
8. (d) Weak reagent like NaHSO3 is unable to open the chain carbohydrate on oxidation gives 17 kJ of energy. Hence,
and can’t react with glucose. This explains the inability fat has highest calorific value.
of glucose to form aldehyde bisulphite compound. 22. (b) There are 20 amino acid in man out of which 10 amino
9. (c) a and b-D-glucoses differ in the arrangement of groups acids are essential amino acids. These essential amino
around one carbon atom (C 1) i.e., they differ in acids are supplied to our bodies by food which we take
configuration at C1, hence these are also known as because they cannot be synthesised in the body. These
anomers. are (1) Valine (2) Leucine (3) Isoleucine (4) Phenylalanine
10. (d) (5) Threonine (6) Methionine (7) Lysine (8) Tryptophan
11. (b) Fructose has 3 chiral centres and hence number of optical (9) Arginine (10) Histidine.
isomers are 23 = 8
Biomolecules 725

23. (c) 35. (d) 36. (a) 37. (b)


H–C=O H2C – OH 38. (a) Synthesis of polypeptide is known as translation. For
| |
H – C – OH C=O this process three type of RNA are essential.
| |
H – C – OH HC – OH
39. (c)
| | 40. (a)
H – C – OH HC – OH
| | (Fructose) 41. (b) Enzymes can increase the rate of a reaction upto 10 million
No. of H – C – OH HC – OH No. of chiral times. Even very small amount can accelerate a reaction.
| | C=3
chiral C = 4 42. (c) Lysine
H2C – OH (glucose) H2C – OH
24. (c) Due to resonance C — N bond in protein acquires double NH2
bond character and is smaller than usual double bond. |
H 2 NCH 2 CH 2 CH 2 CH 2 C HCOOH , is least soluble in

O O water in the pH range 6–7.
C NH C NH 43. (b) Peptide bonds are present in enzyme.
Å H
25. (b) All proteins are not found in L-form but they may be
present in form of D or L N C
26. (a) Metal present in vitamin B12 is Co (cobalt). O
27. (c) The bond formed between two amino acids by the 44. (c)
elimination of a water molecule is called a peptide linkage 45. (c) Ketoses on reduction produce a new chiral carbon leading
or bond. The peptide bond is simply another name for to the formation of two isomeric alcohols which are
amide bond. diastereomeric as well as C–2 epimers.
- C OH + H — N— ¾¾
® — C– N— + H 2 O 46. (d) O Deoxyribose-Adenine ... Thymine -Deoxyribose O
| | || |
OH P P OH
O H O H O Deoxyribose-Guanine ... Cytosine -Deoxyribose
O
Carboxyl group Amine group of Peptide bond P P
of one amino acid other amino acid OH Deoxyribose-Guanine ... Cytosine -Deoxyribose
OH
O O
The product formed by linking amino acid molecules
OH P Deoxyribose-Adenine ... Thymine -Deoxyribose
P OH
through peptide linkages. —CO—NH—, is called a
peptide. The hydrogen bonds are formed between the base
(shown by dotted lines). Because of size and geometries
28. (a) The sequence of bases in mRNA are read in a serial
of the bases, the only possible pairing in DNA is between
order in groups of three at a time. Each triplet of
G(Guanine) and C(Cytosine) through three H-bonds and
nucleotides (having a specific sequence of bases) is
between A (Adenosine) and T (Thymine) through two
known as codon. Each codon specifies one amino acid.
H-bonds.
Further since, there are four bases. therefore, 43 = 64
triplets or codons are possible. A G
47. (b) = =1
T C
29. (a) CHO CH=N.NHPh
Amount of A = T and that of G = C.
3PhNHNH2 48. (b) Triglycerides are lipids, hence these are hydrolysed by
CHOH CH=N.NHPh + PhNH2+ NH 3
lipases to glycerol and fatty acids.
(CHOH)3 (CHOH)3 49. (c)
50. (d) Tyrosine is an a-amino acid, and not a purine
CH2OH CH2OH
51. (c) In a DNA molecule, A = T (Two H–bond)
30. (b) The a-helix structure is formed when the chain of a- C º G (Three H–bond)
amino acids coils as a right handed screw (called a- Purine ® Adenine (A), Guanine (G)
helix) because of the formation of hydrogen bonds
Pyrimidine ® Cytosine (C), Thymine (T)
between amide groups of the same peptide chain, i.e.,
So the complimentary sequence of ATGCTTGA is
NH group in one unit is linked to carbonyl oxygen of
TACGAACT.
the third unit by hydrogen bonding. This hydrogen
bonding between different units is responsible for 52. (c)
holding helix in a position. 53. (d) Progesterone (Gestogens) is a steroid hormone, which
controls the development and maintainance of pregnancy.
31. (a) Since glycerol has three –OH groups, it can have three
Thryoxine and Adrenaline are Amine hormones.
acyl (similar or different) groups
54. (d) 55. (b)
32. (c) 2R - S - H R -S-S- R 56. (c) Vitamin B12 also called cyanocobaltamine, is anti-
Thiol Disulphide pernicious anaemia vitamin. It is C63H84O14N14PCo.
33. (d) Benzidine is 4,4'-diaminodiphenyl 57. (a) The human body does not produce vitamins.
34. (d) Proteins do not provide energy for metabolism. 58. (a) 59. (d)
EBD_7327
726 CHEMISTRY

60. (b) Prostaglandin is a non-steroidal hormone. From the structures it is clear that 2nd carbon in DNA
61. (a) Vitamin B6 is called pyridoxine. It is found in fruits, green- do not have OH group.
vegetables, milk, etc.
62. (d) Testosterone and Adrenaline are steroid harmone, EXERCISE - 2
Thyroxin is non-steroided harmone glucagon is peptide
1. (c) Vitamin A - Xerophthalmia
harmone.
63. (d) Vitamin B12 - Percicious anaemia
64. (c) Each nucleic acid consists of a pentose sugar a Vitamin C - Scurvy
heterocyclic base, and phosphoric acid. The sugar Vitamin E - Sterility
present in DNA is 2-deoxy -D (–) ribose and the sugar Vitamin K - Haemorrhage
present in RNA is D (–)- ribose. The chirality of DNA 2. (c) Adenine is a purine base which is present in RNA and
and RNA molecules are due to the presence of sugar DNA both.
components. 3. (c) Glucose is converted into alcohol by the action of
O O zymase .
HOCH2 OH HOCH 2 OH zymase
C6 H12 O6 ¾¾¾¾ ® C 2 H5OH
H H H H 4. (c) Cellulose is the constituent of cell wall of plant cells.
5. (d) Diastase enzyme converts starch into maltose.
H H H H 6. (b) Lactose is present in milk (Glucose + Galactose).
OH OH OH H 7. (b) Insulin contains 51 amino acids.
D(–)–ribose 2-deoxy–D(–)–ribose 8. (b) Glu cos e + Fehling solution ® Gluconic acid + Cu 2O
65. (a) Thyroxine is an amine hormone. ( Red ppt )
66. (a) In DNA the complimentary base are Adenine and 9. (c) 10. (a)
thymine. 11. (c) In sucrose the two monosaccharides are held together
Guanine and cytosine by glycosidic linkage and not by peptide linkage.
The genetic information for cell is contained in the Saponification
sequence of bases A, T, G and C in DNA molecule. 12. (c) Oil / fat + Alkali ¾¾¾¾¾® Soap + Glycerol.
67. (b) The DNA sequence that codes for a specific protein is 13. (d)
called a Gene and thus every protein in a cell has a
14. (d) Thymine is present in DNA while in RNA there is Uracil.
corresponding gene.
68. (a) 15. (a) Energy is stored in our body in the form of A.T.P
69. (d) Enzymes are most reactive at optimum temperature. The 16. (a) Sucrose is formed by 1-2 linking of glucose and fructose.
optimum temperature for enzyme activity lies between Thus both reducing groups are involved in glycosidic
40°C to 60°C. linkage.
70. (b) Beri-Beri. 17. (b) Glycosidic linkage is actually an ether bond as the
H H HOOC H linkage forming the rings in an oligosaccharide or
71. (a) C— —C C——C polysaccharide is not just one bond, but the two bonds
HOOC COOH H COOH
(cis) (Trans) sharing an oxygen atom e.g. sucrose
maleic acid
It shows geometrical isomerism but does not show
optical isomerism.
72. (c) Since a - D - (+) - glucose and b – D – (+) glucose
differ in configuration at C – 1 atom so they are anomers.
Anomers are those diastereomers that differ in
configuration at C – 1 atom.
73. (c) Glucose is considered as a typical carbohydrate which
contains –CHO and –OH group.
74. (a) Biuret test produces violet colour on addition of dilute
CaSO4 to alkaline solution of a compound containing
peptide linkage.
75. (b) RNA has D (–) – Ribose and the DNA has 2–Deoxy
D (–) – ribose as the carbohydrate unit.
O 5 O
HOCH2 OH HOCH2 OH
H H 4 H H 1
H H H H
3 2
OH OH OH H
ribose 2-deoxy ribose
Biomolecules 727

18. (a) a-D-glucose or b-D-glucose when dissolved in water 27. (a) Sucrose does not have free — CHO or CO group, hence
and allowed to stand, following equilibrium is it does not undergo mutarotation.
stablished, which is called mutarotation. H OH
CH2OH
a-D-glucose ƒ Open chain form ƒ b-D-glucose 28. (c) O
HO H
(+111°) (+19°) H O
OH H
Specific rotation of a-form falls until a constant value H
OH H H
OH
H H O
of +52.5° is reached. On the other hand, specific rotation H OH CH2OH
of b form increases. Specific rotation of equilibrium
mixture is 52.5°.
(Lactose)
19. (b) Phosphate is linked to 3 rd & 5 th carbon of
corresponding sugar All reducing sugar shows mutarotation.
20. (c) Fischer gave the prefix “D” to compounds whose 29. (c) When the proteins are subjected to the action of heat,
bottom chiral has its OH to the right. So natural glucose mineral acids or alkali, the water soluble form of globular
is called D-glucose or dextrose. protein changes to water insoluble fibrous protein. This
Structure of D-Glucose : is called denaturation of proteins. During denaturation
1CHO secondary and tertiary structures of protein destroyed
H 2 OH but primary structures remains intact.
HO 3 H 30. (b) The secondary structure of a protein refers to the shape
4
H OH in which a long peptide chain can exist. There are two
5
H OH different conformations of the peptide linkage present
6CH OH
2 in protein, these are a-helix and b-conformation. The
D-Glucose a-helix always has a right handed arrangement. In
21. (a) By convention, the amino acid with the free amino b-conformation all peptide chains are stretched out to
group (N-terminal) is written at the left end and the one nearly maximum extension and then laid side by side
with the unreacted carboxyl group (C-terminal) at the and held together by intermolecular hydrogen bonds.
right end. Thus, the structure of Gly. Ala is The structure resembles the pleated folds of drapery
O and therefore is known as b-pleated sheet.
Å || –
H 3 NCH 2 CNHCHCO 2 31. (c) With the exception of glycine all the 19 other common
| amino acids have a uniquely different functional group
CH 3 on the central tetrahedral alpha carbon.
22. (d) Glucose being an aldose responds to Tollen’s test while H
fructose, although a ketose, undergoes rearrangement |
in presence of basic medium (provided by Tollen’s H — C — COOH
reagent) to form glucose, which then responds to |
NH 2
Tollen’s test.
23. (b) The two isomeric forms (a – and b –) of D- glycine
glucopyronose differ in configuration only at C–1; 32. (a) Cellulose is a polysaccharide composed of only D-
hence these are called anomers. glucose units. Every adjacent glucose units are joined
by b-glycosidic linkage between C1 of one glucose and
24. (d) C4 of the next.
25. (c) Glycogen is called animal starch and is found in all Thus in every glucose units only three –OH groups
animal cells. It constitutes the reserve food material. are free to form triacetate.
26. (c) Thyroxine is the only hormone among the given HO 6
choices that contains iodine. Its structure is as follows: 5
HO 6 H O O
H 1
HO 5
O O 4 OH H
I I 6 H H 1 3 2 H
H 5 4 OH H H OH
O O 3 2 H
H 1
I 4
OH H
O H H OH
3 2

H OH

I Cellulose triacetate (CH3CO)2O, H2SO4

(–OH = –OCOCH3 )
EBD_7327
728 CHEMISTRY

33. (a) 47. (b) Hydrolysis of sucrose is known as inversion of cane


OH sugar because sucrose produce equimolecular mixture
H H
O of glucose and fructose. Sucrose is dextrorotatory while
OH
glucose and fructose mixture is laevorotatory. Sucrose
34. (b) HO is disaccharide.
HO OH 48. (b) Proteins on hydrolysis gives a-amino acid because
H H
H amino acids are the building blocks of proteins. It is
It is a b-pyranose hence it is an aldohexose. also fact that amino acids contain both –NH2 and
35. (a) 6CO2 + 12NADPH + 18ATP ® C6H12O6 + 12NADP –COOH group.
+ 18ADP Here statement-1 and statement-2 both are correct but
36. (b) Molisch's Test : This is a general test for carbohydrates.
statement-2 is not a correct explanation of statement-1.
One or two drops of alcoholic solution of a-naphthol is
added to 2 ml glucose solution. 1 ml of conc. H2SO4 49. (d) Sequence of bases in DNA is TGAACCCTT, since
solution is added carefully along the sides of the test- according to base-pairing principle, T in DNA faces A
tube. The formation of a violet ring at the junction of two in m-RNA, while G faces C and A faces U. Therefore,
liquids confirms the presence of a carbohydrate or sugar. sequence of bases in m-RNA is ACUUGGGAA.
37. (d) 50. (d) Vitamin B5 is also called as nicotinic acid. Nicotinic acid
38. (d) All the given options are example of neutral amino acids.
in the form of nicotinamide is found usually in all living
39. (a) CH 2OH ( CHOH )4 CHO + [O] ¾¾¾¾¾
Br2 / H 2O
®
cells in small amounts.
Glucose
CH 2OH ( CHOH ) 4 COOH
Gluconic acid
EXERCISE - 3
40. (c) Exemplar Questions
H–C=O H2C – OH
| |
H – C – OH C=O
1. (b) Structure of glycogen is similar to the structure of
| | amylopectin.
H – C – OH HC – OH CH2 OH CH2OH
| | O H O H
H – C – OH HC – OH H H
H H
| | (Fructose) 4 OH 1 4 OH H 1
H – C – OH HC – OH O O a-link
No. of No. of chiral O
| | C=3
chiral C = 4 H OH H OH
H2C – OH (glucose) H2C – OH
41. (d) Weak reagent like NaHSO3 is unable to open the chain
and can’t react with glucose. This explains the inability CH2OH 6 CH2 CH2OH
O H
of glucose to form aldehyde bisulphite compound. H O H H5 O H H
H H H
42. (d) Vitamin D is a fat soluble vitamin. 1 4 OH H 1 4 OH H 1
OH
43. (d) Mg acts as a cofactor to the enzyme as it forms complex O4 O O O
a-link a-link
with phosphate group of ATP that is utilized in the H OH H OH H OH
transfer of ATP phosphate.
Structure of amylopectine
44. (a) The arrangement of polypeptide chains formed as a 2. (d) Glycogen is stored in the liver, brain and muscles of
result of hydrogen bonding is called secondary
animals.
structure of proteins.
H O
45. (b) Lactose is obtained by the condensation of one 3. (c) C12 H 22 O11 ¾¾¾
2 ®C H O + C H O
+ 6 12 6 6 12 6
molecule of b-D-glactopyranose (C1—b) unit and one H
Cane sugar D( +) glucose D ( -) fructose
molecule ofb-D-glucopyranose (C4—b) unit. 4. (c) Structures having different configuration at C-1 are
known as anomers
5. (c) In a-helix structure, — NH group of one amino acid is
hydrogen bonded to C = O group of adjacent amino
acid, forming a helix.
6. (b) Reducing groups of glucose and fructose are involved
in glycosidic bond formation 40 H2C.
6
CH2OH 1
46. (a) Glycine is an amino acid, it contains both NH2 as well H O H OH CH2 O H
as –COOH groups and therefore, its aqueous solution 5 2 5
form Zwitter ion which is amphoteric in nature. 4
OH H 1 H HO
HO O 3 4 CH2OH
NH 2 CH 2 COOH NH 3+ CH 2 COO - 3 2
6

Glycine Zwitter ion H OH OH H


a-D-glucose b-D-fructose
Biomolecules 729

7. (b) Ascorbic acid is the chemical name of vitamin C. and when OH is written on left hand side, it is
8. (a) Dinucleotides are formed by phosphodiester linkage represented as L configuration.
between 5' and 3' carbon atom of pentose sugar. 18. (c) Carbon adjacent to oxygen atom in the cyclic structure
of glucose or fructose is known as anomeric carbon.
O 5' end of chain
|| 6 6
s CH2 Base 19. (c) CH2 OH CH2 OH
O P O 5 O 5
O
|s O H 1 (A) H H OH
O 1¢ H
4' 4 O 4 1
Sugar OH H H OH H
OH H
3' 2¢ 3 2 3 2
H OH OH
O C1 – C4 linkage
|
s
O P=O (I)
| 6 6
CH2 OH
O CH2 OH
Base O 5 O
5'CH2 O H 5 H H OH
H H
4 1 4 1
4' 1¢ OH H OH H
Sugar OH (C) O H
6 3 2
3' 3 2 CH2 OH
2¢ H OH 5 H OH
O
OH C 1 to C2 ® O(B) H C 1 to C2 linkage
H 1
linkage 4 OH H
3' end of chain 6
CH2 HO H
9. (b) The polymer of nucleotides in which nucleic acids are O 3 2
H 5 H H OH
linked together by phosphodiester linkage are known 4
H 1
OH H
as nucleic acid. OH OH
10. (c) Glucose is present in pyranose form, 3 2
H OH
CH2OH (II) (III)
H O H
H NEET/AIPMT (2013-2017) Questions
OH H 20. (c) The base pairs of the two strands of DNA are linked
HO OH together through H-bonds.
H OH 21. (d) Glucose reacts with hydroxyl amine to form an oxime.
Pyranose means six membered ring containing oxygen.
11. (a) Primary structure of proteins. When each polypeptide
in a protein has amino acids linked with each other in a
specific sequence.
12. (c) RNA does not contain thymine.
13. (d) Vitamin B12 can be stored in our body because it is not
water soluable.
14. (d) DNA does not contain uracil.
15. (a) Cyclic structures of monosaccharides which differ in 22. (c) Adrenaline is a hormone produced by adrenal glands
structure at carbon -1 are known as anomers. during high stress or exciting situations. This powerful
Here, I and II are anomer because they differ from each hormone is part of the human body’s acute stress
other at carbon-1 only. response system, also called the fight or flight response.
16. (c) "Pentaacetate of glucose does not react with
hydroxylamine" showing absence of free CHO group. 23. (c) Peptide bond
This can not be explained by open structure of glucose
while all other properties can be easily explained. — C — NH —
||
CH2OH
17. (a) CHO O
CHO
H OH C=O
HO H HO H 24. (b) Sugar in DNA is 2-deoxyribose whereas sugar in RNA
H OH
OH OH is ribose.
H H
CH2OH H OH H OH 25. (d) Sucrose is non-reducing disaccharide as the two
CH2OH CH2OH monosaccharide units are linked through their respective
(+) (+) (–) carbonyl groups.
(i) (ii) (iii)
26. (c) Due to denaturation of proteins, helix get uncoiled and
When OH on lowest asymmetric carbon is written at
right hand side, it is represented as D configuration protein loses its biological activity.
EBD_7327
730 CHEMISTRY

29 Polymers

Polymer is defined as very large molecule having high molecular Based on Mode of Polymerisation
mass (103 – 107 m). They are also referred to as macromolecules,
Addition polymers
which are formed by joining repeating structural units on a large
scale. The repeating structural units are derived from some simple (i) These polymers are formed by repeated addition of monomer
and reactive molecules known as monomers and are linked to molecules possessing double or triple bond.
each other by covalent bonds. This process of formation of For examples : polyethene, polypropene, Buna–S, Buna–N,
polymers from respective monomers is called polymerisation. etc.
CLASSIFICATION OF POLYMERS (ii) If only one type of monomeric species are present, the
addition polymer is called homopolymer.
Based on Source
ex: polythene, polyvinylchloride, etc.
Natural polymers :
Found in nature and generally obtained from plants and animals. n CH 2 = C H 2 – C H 2 – C H 2– n
For examples : proteins, cellulose, starch, resins, rubber, etc. ethene Polyethene
Semi-synthetic polymers (iii) If the monomers are different, the addition polymer is called
These are mostly derived from naturally occuring polymers by copolymer.
chemical modifications. ex: Buna–S, Buna – N etc.
For examples : cellulose acetate (rayon) and cellulose nitrate
Synthetic polymers n CH2 = CH – CH = CH2 + nC6H5CH = CH2
Polymers prepared by synthesis (man made), are known as 1, 3-Butadiene Styrene
synthetic polymers. C6H5
For examples : plastics like polyethene, synthetic fibres like nylon
6, 6 and synthetic rubbers like Buna–S – CH2 – CH = CH – CH2 – CH2 – CH –
n
Based on Structure of Polymers Buna-S
Linear polymers Note: In addition polymerisation a small amount of an organic
These polymers consist of long and straight chains. As a result of peroxide is normally used as a free radical initiator.
close packing of polymer chains, linear polymers have high melting Condensation polymers
points densities and tensile strength.
These polymers are formed by repeated condensation reaction
For examples : HDPE, PVC, etc. between two different bi- functional or tri-functional monomeric
Branched chain polymers units.
These polymers contain linear chains having some branches As a In this, elimination of small molecules such as water, alcohol, HCl,
result of branching these polymers do not pack well thus have etc., takes place.
low melting points, densities and tensile strength. Ex: Nylon 6, 6, terylene, nylon-6, etc.
For examples : LPDE, etc.
n H2N (CH2)6 NH2 + n HOOC(CH2)4COOH
Cross-linked or Network polymers
These polymers are formed from bi-functional and tri-functional
monomers and contain strong covalent bonds between various – NH(CH2)6NHCO(CH2)4CO –n + n H2O
linear polymer chains. Because of presence of crosslinks these
Nylon 6, 6
polymers are hard, rigid and brittle.
For examples : bakelite, melamine, etc. Condensation is also known as step growth polymerisation.
Polymers 731

Based on Inter Molecular Forces Condensation or step growth polymerisation


Elastomers It involves repetitive condensation reaction between two bi-
These polymers are rubber-like solids with elastic properties. functional monomers. These reactions result in the loss of some
Polymers having weakest inter-molecular forces between polymer simple molecules like water, alcohol, HCl, NH3, etc.
chains. For example : nylon, polyesters, bakelite, etc.
These polymer can be stretched ten times their normal length and Mechanism for addition polymerization
they return to original position when force is withdrawn. Addition polymerisation proceeds via free radical addition
For examples : Buna – S, Buna – N, neoprene etc. polymerization. Free radical mechanism of addition polymerization
Fibres is given as follows :
(i) Chain initiation step:
Polymers in which the intermolecular force of attraction are the
strongest are called fibres. These forces are either due to H- O O
bonding or dipole-dipole interaction.
They are thread forming solids possessing high tensile strength, C6H5 C – O – O – C – C6H5
high modulus and least elasticity. In these polymers strong Benzoyl peroxide
bonding results in close packing of chains and crystalline structure O
Example : nylon 6, 6; terylene etc.
Thermoplastics 2 C6H5 – C – O 2 C6H5 + CO2
In these polymers intermolecular forces of attraction are
intermediate between elastomers and fibers. They are linear or Benzyl free radical Phenyl radical
slightly branched chain molecules.
They can repeatedly soften on heating and gets harden on cooling. C6H 5 + CH 2 = CH2 C 6H 5 – CH 2 – CH 2
Some important examples of this class of polymers are polyvinyl (ii) Chain propagating step:
chloride, polythene, polypropylene, polystyrene etc.
Thermosetting C6H5CH2CH2 + CH2 = CH2
They are semi-fluid substances with low molecular masses which
when heated become hard and infusible, because of sufficiently C6H5 – CH2 – CH2 – CH2 – CH2
large number of cross links. They acquire a shape of three-
dimensional network. Important examples of this class of polymers
are bakelite. urea formaldehyde resin, etc.
TYPES OF POLYMERISATION REACTIONS C6H5 – CH2 – CH2 – CH2 – CH2
n
Addition or Chain Growth Polymerisation 3. Chain terminating step:
In this, monomers of same or different type add together on a
large scale to form polymer. Monomers used are unsaturated C6 H5 – CH2 – CH2 – CH2 – CH2 + C6 H5 – CH2– CH 2 – CH2 – CH2
n n
compounds, like alkenes, alkadienes, and their derivatives.
C6 H5 – CH2– CH 2 –n CH 2 – CH 2 – CH 2 – CH2– CH2–– CH 2 n C6H5
For example : polythene, HDPE, Teflon, PAN, etc
Polyethene

SOME IMPORTANT COMMERCIAL POLYMERS

Structure of
Name of polymer Monomer Preparation Reaction Uses
repeating unit

I. Addition polymers

1. (a) LDPE (Low ethene –(CH2 – CH 2 )– Polymerisation of ethene It is used in


n
Density Polyethene) under high pressure of 1000 – insulation of
2000 atm at a temperature of electricity
350 – 570 K in presence of carrying wires
traces of O 2 or peroxide and
initiator. It has highly manufacture of
branched structure. squeeze bottles,
toys, flexible pipes
(it is chemically
inert, tough,
flexible and
poor conductor
of electricity)
( )
EBD_7327
732 CHEMISTRY

1. (b) HDPE (High ethene –(CH2 – CH 2 )– Polymerisation of ethene in It is used for


n
density polyethene) hydrocarbon solvent in making buckets,
presence of zeigler-Natta dustbins,
catalyst at 333– 343 K and bottles, pipes,
6 – 7 atm. It consists of linear etc. ( it is compara-
chains which undergoes to tively more
close packing. Thus it chemically inert,
possesses high density. tougher and
harder)
2. Polytetrafluoro Tetrafluoro –(CF2 – CF2 )–
nCF2 = CF2 It is used in
n catalyst making oil seals
ethene (Teflon) ethene high pressure
and gaskets and
–( CF2 – CF2 –) n also used for
non-stick
surface coated
utensils as it is
chemically
inert and
resistant to
attack by
corrosive
reagents.
It is used
3. Polyacrylonitrile Acrylonitrile nCH 2 = CHCN as a substitute
(PAN) CN Peroxide
catalyst
for wool in
(
– CF2 – CF – )n making
–( CH 2 – CH )– n commercial
fibres as orlon or
CN
acrilan.

4. Polypropene propene CH3 CH3 It is used in


| manufacture
–( CH2 – CH–) n nCH = CH2 of ropes, toys,
Propylene pipes, fibres, etc.
Ziegler Natta Catalyst
Al(C2 H5 ) 3 + TiCl3

é CH3 ù
ê | ú
êë - CH - CH2 - úû n
Polypropylene

5. Polystyrene Styrene 6 5 It is used as


C6H5 | an insulator,
–( CH2 – CH)–n nCH = CH2 wrapping material
Styrene
Benzoyl Peroxide in manufacture of
® toys, radio and
é C6 H 5 ù television
ê | ú cabinets.
ëê - CH - CH2 - ûú n
Polystyrene

6. Polyvinyl Vinyl nCH2 = CH ® It is used in


Cl | manufacture of
chloride (PVC) chloride
–( CH2 – CH –) n Cl rain coats, hand
Vinyl chloride
[– CH2 – CH –]n bags, vinyl
| flooring, water
Cl pipes, etc.
Polyvinyl chloride
Polymers 733

II. Condensation polymers


1. Polyamides hexamethylene It is used
(a) Nylon 6, 6 diamine and –(NH(CH2)6NHCO(CH 2)4CO)–n nHOOC(CH2)4COOH +
in making
adipic acid nH2N(CH2)6NH2 sheets, bristles
553 K
O for brushes and
pressure in textile
–(NH(CH2)6NH – C – (CH2)4 – C)–n industry
O
(b) Nylon 6 caprolactum O H It is used
H in manufacture
–( C – (CH2)5 – N)–n N of tyre cords,
fabrics and
H2C C=O ropes

H2C CH2

H2C CH2

(i) H2O, D (ii) D, Polymerisation

O H
–( C – (CH2)5 – N)–n

2. Polyesters ethylene O O It is used in


nHO – CH2 – CH2 – OH manufacture of
Terylene glycol –(O – CH2CH2 – O – C C)–n O
+ wear fabrics,
or Dacron and terephthalic
acid nHO – CO C – OH tyre cords and
420-460 K Zn(OCOCH3)2 seat belts it is
+ Sb2O3 used in blending
cotton and
O O
wool fibres and
–(O–CH2 –CH2 –O–C C –)–n also used as glass
reinforcing
materials in
safety helmets
3. Phenol formaldehyde
polymer
(a) Novalac o -Hydroxy-
methylphenol OH OH OH
CH2OH
CH2 CH2 n

n polymerisation

OH OH
CH 2 CH 2

n
Novalac (linear polymer) It is used
(b) Bakelite Novalac + in making
formaldehyde OH OH OH OH OH combs,
(Phenol and CH2
– CH2 CH2 CH2 CH–2 phonograph
CH2
formaldehyde) CH2 CH2 CH2
records,
electrical
n —CH2 CH2 CH2 CH–2 switches and
OH OH OH n handles of
Bakelite various utensils,
etc.
EBD_7327
734 CHEMISTRY
etc.
4. Melamine Melamine + N H2N N NH2 It is used
formaldehyde formaldehyde -NH NH-CH-2
+ HCHO in manufacture
polymer N N of unbreakable
N N
crockery
NH- n NH2
Melmac
H2 N N NHCH2OH

N N

NH2
intermediate
polymerisation

–( HN N NH – CH2–)n

N N

NH
Melamine polymer
It is used
5. Urea Urea + formal- for making
formaldehyde dehyde –( NH – CO – NH – CH2)–n unbreakable
resin — cups and
laminated
sheets
6. Glyptal Ethylene It is used in
glycol + –( OCH2 – CH2OOC CO)–n manufacture of
phthalic acid COOH + n HO – CH2 – CH2– OH paints and
n HOOC
Phthalic acid
(1, 2-Benzenedicarboxylic acid)
Ethylene glycol lacquers.

[– OC CO – O – CH2– CH2 – O–] n


Alkyd resin

COPOLYMERISATION properties. For example : Buna–S is tough and a good substitute


for natural rubber.
It is a polymerisation reaction in which a mixture of more than one
monomeric species is allowed to polymerise and form a copolymer. RUBBER
It can be made by chain as well as step growth polymerisation. Natural rubber
Ex: Buna–S It is manufactured from rubber latex which is a colloidal dispersion
CH = CH2 of rubber in water.
It is a linear polymer of isoprene (2-methyl –1, 3-butadiene) and is
also called cis –1, 4 – polyisoprene. Cis-polyisoprene molecule
n CH2 = CH – CH = CH2 + consists of various chains held together by weak Van der waals
interactions and has a coiled structure. Thus, it can be stretched
1, 3-Butadiene like a spring and exhibits elastic properties.
Styrene
Vulcanization of rubber:
Natural rubber becomes soft at high temperature and brittle at low
temperatures and shows high water absorption capacity. It is
soluble in non-polar solvents and is non-resistant to attack by
– CH2 – CH = CH – CH2 – CH – CH2 –
n oxidising agents. To improve upon these physical properties,
Butadiene-Styrene vulcanization is carried out. It consists of heating a mixture of raw
copolymer rubber with sulphur and an appropriate additive at a temperature
range between 373K to 415K. On vulcanization, sulphur forms
Properties of copolymers are different from homopolymers. cross-links at reactive sites of double bonds and thus rubber gets
Actually, copolymers have better physical and mechanical stiffened.
Polymers 735

Note : (i) About 5% sulphur is used for making tyre rubber, 20- Weight Average Molecular Weight ( M n )
25% sulphur for making ebonite and 30% sulphur for making battery
case rubber. w1M1 + w 2 M 2 + w 3 M3 + - - - wM
(ii) Natural rubber is cis-polyisoprene whereas gutta parcha is Mw = =å i i
w1 + w 2 + w 3 + - - - å wi
trans-polyisoprene.
Synthetic rubbers N1M12 + N 2 M 22 + N3 M32 + ........ SNi M i2
or Mw = =
They are either homopolymers of 1, 3-butadiene derivatives or N1M1 + N 2 M 2 + N3 M3 + ........ SN i M i
copolymers of 1, 3-butadiene or its derivatives with another
[weight (w) = no. of molecules (N) × molecular weight (M)]
unsaturated monomer.
Preparation: M w is generally determined by the light scattering method.
(i) Neoprene (or polychloroprene) PDI (POLY DISPERSITY INDEX)
It is prepared by free radical polymerisation of chloroprene.
The ratio of the M w and M n is called PDI
Cl
Mw
polymerisation PDI = M
n CH2 = C – CH = CH2 n
Chloroprene In natural polymers, which are generally mono dispersed, the PDI
Cl
| is unity ( M w = M n )
– CH2 – C = CH – CH2 – In synthetic polymers which are poly dispersed, PDI is greater
n
Neoprene than unity because M w is always higher than M n .
It is resistant to vegetable and mineral oils. It is used for BIODEGRADABLE POLYMERS
manufacturing conveyor belts, gaskets and hoses. A large number of polymers are resistant to environmental
(ii) Buna-N degradation processes and are thus responsible for accumulation
It is obtained by copolymerisation of 1, 3-butadiene and of polymeric solid waste materials. To overcome this problem,
acrylonitrile in presence of peroxide catalyst. new biodegradable synthetic polymers have been designed and
developed.
n CH2 = CH – CH = CH2 + n CH2 = CH Aliphatic polyesters are one of the important classes of
biodegradable polymers. Some important examples are:
CN
(i) Poly b-hydroxy buty rate - co-b-hydroxy valerate (PHBV)
It is obtained by following reaction:
copolymeri- – CH – CH = CH – CH – CH – CH –
2 2 2
sation n OH OH
CN
CH3 – CH – CH2 – COOH + CH3CH2CHCH2COOH
It is resistant to action of petrol, lubricating oil and organic
3-Hydroxy butanoic acid 3-Hydroxypentanoic acid
solvents. It is used in making oil seals, tank lining, etc.
MOLECULAR MASS OF POLYMERS
Polymer sample contains chains of varying lengths and hence its
molecular mass is always expressed as an average and can be – O – CH – CH2 – C – O – CH – CH2 – C –
n
determined by chemical and physical methods.
CH3 O CH2CH3 O
Number Average Molecular Weight ( M n )
PHBV
Total weight of the molecules
Mn = Total number of molecules It undergoes bacterial decomposition in environment.
It is used in speciality packaging, orthopaedic devices and
N1M1 + N 2 M 2 + N3 M3 + - - - in controlled release of drugs.
Mn =
N1 + N 2 + N3 + - - - (ii) Nylon-2-nylon-6
It is an alternating copolymer of glycine (H2N – CH2 – COOH)
SN i M i and amino caproic acid (H 2 N(CH 2 ) 5 COOH) It is
Mn = SN i biodegradable
M n is generally determined by osmotic pressure method.
736

CONCEPT MAP
CHEMISTRY

EBD_7327
Polymers 737

1. Which of the following is a polymer containing nitrogen? 20. Which of the following is not correctly matched?
(a) Polyvinyl chloride (b) Bakelite O O
(c) Nylon (d) Terylene
(a) Terylene –OCH2–CH2– C– –C –
2. In elastomer, intermolecular forces are
n
(a) strong (b) weak (c) zero (d) None of these
3. Natural rubber is a polymer of (b) Neoprene — CH 2 — C = CH — CH 2 —
(a) butadiene (b) isoprene
(c) 2-methylbutadiene (d) hexa-1, 3-diene Cl
n
4. ~~~[NH(CH 2 )6 NHCO(CH 2 ) 4 CO~]n~~ is a
O O
(a) addition polymer (b) thermosetting polymer
(c) homopolymer (d) copolymer (c) Nylon-66 –NH–(CH2)6–NH–C–(CH2)4–C–O–n
5. Which of the following is an example of thermosetting CH3
polymer?
(a) Polythene (b) PVC (c) Neoprene(d) Bakelite (d) PMMA – CH2–C
6. A condensation polymer among the following is COOCH3 n
(a) dacron (b) PVC (c) polystyrene (d) teflon
7. Which of the following is not polyamide ? 21. The monomer(s) used to prepare polyvinyl polythene is
(a) Nylon 6 (b) Glyptal structure (a) vinyl chloride and ethene
(c) Protein (d) Nylon-66 (b) ethene
8. Ebonite is (c) isoprene
(a) natural rubber (b) synthetic rubber (d) 1, 3-butadiene
(c) highly vulcanized rubber (d) polypropene CH3
9. Which of the following is not an example of addition
22. Monomer of — C — CH2 — is
polymer ?
(a) Polystyrene (b) Nylon CH3 n
(c) PVC (d) Polypropylene (a) 2-methylpropene (b) styrene
10. Dacron is obtained by the condensation polymerisation of: (c) propylene (d) ethene
(a) Dimethyl terephthalate and ethylene glycol 23. Acrilan is a hard, horny and a high melting material. Which
(b) Terephthalic acid and formaldehyde of the following represents its structure ?
(c) Phenol and phthalic acid æ ö
(d) Phenol and formaldehyde (a) ç —CH 2 — CH — ÷
11. Which of the following is a polyamide? ç | ÷
(a) Bakelite (b) Terylene (c) Nylon-66 (d) Teflon ç Cl ÷
è øn
12. PVC is :
(a) thermoplastic polymer (b) compound polymer (b) æ —CH 2 — CH — ö
ç ÷ |
(c) thermosetting polymer (d) simple polymer ç ÷
çè CN ÷ø
13. The synthetic polymer which resembles natural rubber is n
(a) neoprene (b) chloroprene
(c) glyptal (d) nylon CH3
æ | ö
14. The condensation of hexamethylenediamine with sebacoyl (c) ç —CH 2 — C— ÷
chloride at 525 K gives ç | ÷
(a) nylon-6,20 (b) nylon-6,01 ç COOCH ÷
çè 3 ÷ø
(c) nylon-6,10 (d) none of these n
15. Which of the following is not a biopolymer ?
æ —CH 2 — CH — ö
(a) Proteins (b) Rubber (c) Cellulose (d) RNA |
(d) ç ÷
16. Which is not an example of copolymer ? çè COOC2 H5÷ø
(a) SAN (b) ABS (c) Saran (d) PVC
17. For natural polymers PDI is generally 24. Match List-I (Monomer) with List II (Polymer) and select
(a) 0 (b) 1 (c) 100 (d) 1000 the correct answer using the codes given below the lists:
18. Which is used as medicine? List I List II
(a) PVC (b) Terylene (c) Glyptal (d) Urotropine I. Hexamethylenediamine A. Bakelite
19. Nylon 66 is a polyamide obtained by the reaction of II. Phenol B. Dacron
(a) COOH(CH2)4 COOH + NH2C6H4NH2–(p) III. Phthalic acid C. Glyptal
(b) COOH(CH2)4 COOH + NH2 (CH2)6 NH2 IV. Terephthalic acid D. Melamine
(c) COOH (CH2)6 COOH + NH2 (CH2)4 NH2 E. Nylon
(d) COOHC6H4 COOH–(p) + NH2 (CH2)6 NH2
EBD_7327
738 CHEMISTRY

Codes: 36. Head-to-tail addition takes place in chain-growth


(a) I-E, II-A, III-B, IV-C (b) I-E, II-A, III-C, IV-B polymerisation when monomer is
(c) I-D, II-C, III-A, IV-B (d) I-D, II-C, III-A, IV-B
25. The monomer of the polymer; (a) CH2 = CH (b) CH 2 = CH - CH = CH 2

CH3 CH3
|
ÚÚÚÚÚCH 2 - C - CH 2- C Å (c) CH 2 < C , COCH3 (d) CH 2 < CH , C º N
| is | P
CH3 CH3 CH3 O

CH3 37. Polymer used in bullet proof glass is


(a) H 2C = C (b) CH3CH=CHCH3 (a) lexan (b) PMMA (c) nomex (d) kevlar
38. The monomer(s) used in the preparation of Orlon, a substitute
CH3
for wool is/are
(c) CH3CH = CH2 (d) (CH3)2C = C(CH3)2 (a) caprolactam
26. Orlon is a polymer of (b) tetrafluoroethene
(a) styrene (b) tetrafluoroethylene (c) styrene and 1, 3-butadiene
(c) vinyl chloride (d) acrylonitrile (d) acrylonitrile
27. P.V.C. is formed by polymerisation of 39. Glyptals are chiefly employed in
(a) ethene (b) 1-chloropropene
(a) toy making (b) surface coating
(c) propene (d) 1-chloroethene
(c) photofilm making (d) electrical insulators
28. Caprolactam polymerises to give
40. The polymer used for making contact lenses for eyes is
(a) terylene (b) teflon (c) glyptal (d) nylon-6
(a) polymethylmethacrylate(b) polyethylene
29. The process involving heating of rubber with sulphur is
called (c) polyethylacrylate (d) nylon-6
(a) galvanisation (b) vulcanization 41. Which polymer is used for making magnetic recording tapes?
(c) bessemerisaion (d) sulphonation (a) Dacron (b) Acrilan (c) Glyptal (d) Bakelite
30. Which of the following is used in paints? 42. The polymer used in making synthetic hair wigs is made up of
(a) Terylene (b) Nylon (a) CH2 = CHCl (b) CH2 = CHCOOCH3
(c) Glyptal (d) Chloroprene (c) C6H5CH = CH2 (d) CH2 = CH – CH=CH2
31. Which of the following polymers do not involve cross 43. The monomer of polystyrene is
linkages? (a) C2H5 — CH — — CH
2
(a) Melmac (b) Bakelite —
(b) CH2 — CHCl
(c) Polythene (d) Vulcanised rubber (c) C6H5 — CH — — CH
2
(d) CH2 — — CHCHO
32. Number average molecular mass, M n and weight average
44. Which of the following has been used in the manufacture of
molecular mass ( M w ) of synthetic polymers are related as
non-inflammable photographic films?
(a) M n = (M w )1/ 2 (b) M n = M w (a) Cellulose nitrate (b) Cellulose xanthate
(c) Cellulose perchlorate (d) Cellulose acetate
(c) M w > M n (d) M w < M n
45. Which of the following is currently used as a tyre cord ?
33. Select the correct statement. (a) Terylene (b) Polyethylene
(a) Vinyon is a copolymer of vinyl chloride and vinyl (c) Polypropylene (d) Nylon - 6
acetate 46. The plastic household crockery is prepared by using
(b) Saran is a copolymer of vinyl chloride and vinylidine (a) melamine and tetrafluoroethane
chloride
(b) malonic acid and hexamethyleneamine
(c) Butyl rubber is a copolymer of isobutylene and isoprene
(c) melamine and vinyl acetate
(d) All of the above are correct
(d) melamine and formaldehyde
34. The polymer used in the manufacture of lacquers is
47. The polymer used in orthopaedic devices and in controlled
(a) bakelite (b) glyptal (c) PVC (d) PHBV
drug release is
35. Low density polythene is prepared by
(a) Free radical polymerisation (a) Orlon (b) PTFE (c) SBR (d) PHBV
(b) cationic polymerisation 48. Which one of the following polymers is prepared by
(c) anionic polymerisation condensation polymerisation?
(d) Ziegler-Natta polymerisation (a) Teflon (b) Natural rubber
(c) Styrene (d) Nylon-66
Polymers 739

49. Which one of the following statement is not true? 59. The synthetic polymer which resembles natural rubber is
(a) In vulcanization the formation of sulphur bridges (a) neoprene (b) chloroprene
between different chains make rubber harder and (c) glyptal (d) nylon
stronger. 60. Characteristic property of Teflon is
(b) Natural rubber has the trans -configuration at every (a) 2000 poise viscosity
double bond (b) high surface tension
(c) Buna-S is a copolymer of butadiene and styrene (c) non-inflammable and resistant to heat
(d) Natural rubber is a 1, 4 - polymer of isoprene (d) highly reactive
50. Which of the following structures represents neoprene
61. Which of the following is used in vulcanization of rubber ?
polymer?
(a) SF6 (b) CF4 (c) Cl2F2 (d) C2F2
CN 62. What is the percentage of sulphur used in vulcanization of
| rubber
(a) –( CH 2 – C = CH – CH 2 –) n (b) –(CH 2 – CH –) n (a) 05% to 30% (b) 03% to 25%
|
Cl (c) 10% to 20% (d) 05% to 25%
63. Perlon is
Cl (a) Rubber (b) Nylon-6 (c) Terylene (d) Orlon
| –( CH – CH 2 –)n
(c) –( CH 2 – CH –)n (d) | 64. Plexiglas (PMMA) is a polymer of
C6 H5 (a) acrylic acid (b) methyl acrylate
51. Of the following which one is classified as polyester (c) methyl methacrylate (d) None of these
polymer ? 65. Which one of the following sets forms the biodegradable
(a) Terylene (b) Bakelite (c) Melamine (d) Nylon-66 polymer?
52. Which one of the following is not a condensation polymer ? (a) CH2 = CH – CN and CH2 = CH – CH = CH2
(a) Melamine (b) Glyptal (b) H2N – CH2 – COOH and H2N–(CH2)5 – COOH
(c) Dacron (d) Neoprene (c) HO – CH2 – CH2 – OH and
53. Which of the following statements is false?
(a) Artificial silk is derived from cellulose.
HOOC COOH
(b) Nylon-66 is an example of elastomer.
(c) The repeat unit in natural rubber is isoprene.
(d) Both starch and cellulose are polymers of glucose.
54. Bakelite is obtained from phenol by reacting with (d) CH = CH2 and CH2 = CH – CH = CH2
(a) (CH2OH)2 (b) CH3CHO
(c) CH3 COCH3 (d) HCHO 66. Buna-N synthetic rubber is a copolymer of :
55. Vinyl chloride can be converted into PVC. In this reaction, (a) H2C = CH – CH = CH2 and H5C6 – CH = CH2
the catalyst used is (b) H2C = CH – CN and H2C = CH – CH = CH2
(a) peroxides (b) cuprous chloride
(c) H2C = CH – CN and H 2 C = CH – C = CH 2
(c) anhydrous zinc chloride (d) anhydrous AlCl3 |
56. The mass average molecular mass & number average CH3
molecular mass of a polymer are 40,000 and 30,000
respectively. The polydispersity index of polymer will be Cl
|
(a) < 1 (b) > 1 (c) 1 (d) 0 (d) H 2 C = CH – C = CH 2 and H 2C = CH – CH = CH 2
57. Ethylene-propylene rubber (EPR) is 67. The polymer containing strong intermolecular forces e.g.
(a) unsaturated and stereoregular
hydrogen bonding, is
(b) saturated and stereoregular
(a) teflon (b) nylon 6, 6
(c) atactic and unsaturated
(c) polystyrene (d) natural rubber
(d) syndiotactic and unsaturated
58. The monomeric units of terylene are glycol and which of the 68. Among cellulose, polyvinyl chloride, nylon and natural
following rubber, the polymer in which the intermolecular force of
attraction is weakest is
OH (a) nylon (b) polyvinyl chloride
OH
(c) cellulose (d) natural Rubber
(a) (b) 69. The monomer(s) used to prepare polyvinyl polythene is
OH OH (a) vinyl chloride and ethene
OH (b) ethene
(c) COOH (d) (c) isoprene
COOH OH
(d) 1, 3-butadiene
OH
EBD_7327
740 CHEMISTRY

70. The polymer used in orthopaedic devices and in controlled 73. The monomer(s) used in the preparation of Orlon, a substitute
drug release is for wool is/are
(a) Orlon (b) PTFE (a) caprolactam
(c) SBR (d) PHBV (b) tetrafluoroethene
71. The monomer of polystyrene is (c) styrene and 1, 3-butadiene
(a) C2H5 — CH — — CH (d) acrylonitrile
2
(b) CH2 —— CHCl 74. The polymer used in the manufacture of lacquers is
(c) C6H5 — CH — — CH (a) bakelite (b) glyptal
2
(d) CH2 —— CHCHO (c) PVC (d) PHBV
72. A network polymer among the following is 75. Teflon, styron and neoprene are all
(a) teflon (b) polythene (a) Copolymers (b) Condensation polymers
(c) bakelite (d) PVC (c) Homopolymers (d) Monomers

1. The monomers used for the preparation of nylon- 2-nylon-6 8. Among the following, the wrong statement is
is/are (a) PMMA is plexiglass
(a) caprolactam (b) SBR is natural rubber
(b) alanine and amino caproic acid (c) PTFE is teflon
(c) glycine and amino caproic acid (d) LDPE is low density polythene
(d) hexamethylenediamine and adipic acid 9. If a polythene sample contains two monodisperse fractions
2. Polymer formation from monomers starts by in the ratio 2 : 3 with degree of polymerization 100 and 200,
(a) condensation or addition reaction between monomers respectively, then its weight average molecular weight will
(b) coordinate reaction between monomers be :
(c) conversion of monomer to monomer ions (a) 4900 (b) 4600 (c) 4300 (d) 5200
(d) hydrolysis of monomers. 10. When condensation product of hexamethylenediamine and
3. Nylon threads are made of adipic acid is heated to 525K in an atmosphere of nitrogen
(a) polyester polymer (b) polyamide polymer for about 4-5 hours, the product obtained is
(c) polyethylene polymer (d) polyvinyl polymer (a) solid polymer of nylon 66
4. The polymer containing strong intermolecular forces e.g. (b) liquid polymer of nylon 66
hydrogen bonding, is (c) gaseous polymer of nylon 66
(a) teflon (b) nylon 6, 6 (d) liquid polymer of nylon 6
(c) polystyrene (d) natural rubber 11. The polymer used for optical lenses is :
n(CF2 =
Benzoyl peroxide or
CF2 ) ¾¾¾¾¾¾¾¾ ®X (a) polypropylene
5. (NH 4 )2 S2O8 (b) polyvinyl chloride
Here, X is : (c) polythene
(a) RMMA (b) PVC (d) polymethyl methacrylate
(c) PAN (d) None of these 12. The compound which cannot be used as a plasticizer, is
6. Which pair of polymers have similar properties ? (a) di-n-butylphthalate (b) tricresyl phosphate
(a) Nylon, PVC (b) PAN, PTFE (c) di-n-octyphthalate (d) diethyl phthalate
(c) PCTFE, PTFE (d) Bakelite, alkyl resin 13. In which of the following polymers, empirical formula
7. Which compound/set of compounds is used in the resembles with monomer ?
manufacture of nylon 6? (a) Bakelite (b) Teflon
(c) Nylon-6, 6 (d) Dacron
(a) CH = CH2 14. Which is a polymer of three different monomers ?
(a) ABS (b) SBR
(b) HOOC(CH2)4COOH + NH2(CH2)6 NH2 (c) NBR (d) Nylon-2-Nylon-6
O 15. Which of the following fibres is made of polyamide ?
CH3 (a) Dacron (b) Orlon (c) Nylon (d) Rayon
| 16. The polymer which has conducting power is
(c) CH 2 = CH - C = CH 2 (d) (a) polyethylene (b) polybutadiene
(c) polystyrene (d) polyacetylene
Polymers 741

17. Orlon is a – (c) Nylon 66; [ –NH(CH 2 ) 6 NH CO(CH 2 ) 4 CO - ]n


(a) homopolymer and addition polymer
(b) copolymer and addition polymer (d) Teflon; ( -CF2 - CF2 -)n
(c) homopolymer and condensation polymer DIRECTIONS for Qs. 22 to 25 : These are Assertion-Reason
(d) copolymer and condensation polymer type questions. Each of these question contains two statements:
18. Dacron is a – Statement-1 (Assertion) and Statement-2 (Reason). Answer these
(a) crease resistant questions from the following four options.
(b) polyamide (a) Statement-1 is true, Statement-2 is true, Statement-2 is a
(c) addition polymer correct explanation for Statement -1
(d) polymer of ethylene glycol and phthalic acid (b) Statement-1 is true, Statement-2 is true ; Statement-2 is NOT
19. Which of the following is a branched polymer ? a correct explanation for Statement-1
(a) Low density polymer (b) Polyester (c) Statement-1 is true, Statement-2 is false
(c) High density polymer (d) Nylon (d) Statement-1 is false, Statement-2 is true
20. Given the polymers (i) Nylon-66; (ii) Buna-S; (iii) Polythene, 22. Statement-1 : In vulcanisation of rubber, sulphur cross links
arrange these in increasing order of their inter-molecular are introduced.
forces (lower to higher) Statement-2 : Vulcanisation is a free radical initiated chain
(a) (i) > (ii) > (iii) (b) (ii) > (iii) > (i) reaction.
(c) (ii) < (iii) < (i) (d) (iii) < (i) < (ii) 23. Statement-1 : Bakelite is a thermosetting polymer.
21. Structures of some common polymers are given. Which one Statement-2 : Bakelite can be melted again and again without
is not correctly presented? any change.
æ ö 24. Statement-1 : The time of vulcanisation and temperature is
ç ÷ increased by adding accelerators.
(a) Neoprene; ç - CH 2 - C = CH - CH 2 - CH 2 - ÷ Statement-2 : By vulcanising, a material of high tensile
ç | ÷
è Cl øn strength can be obtained.
(b) Terylene; 25. Statement-1 : Teflon has high thermal stability and chemical
inertness.
(– OC COOCH2 – CH2 – O–)n Statement-2 : Teflon is a thermoplastic.

Exemplar Questions (c) –( O CH CH2 C O CH CH2 C –)n


1. Which of the following polymers of glucose is stored by
CH3 O CH2CH3 O
animals?
(a) Cellulose (b) Amylose H H O O
(c) Amylopectin (d) Glycogen
(d)
2. Which of the following is not a semisynthetic polymer? –( N (CH2)6 N C (CH2)4 C –)n
(a) cis-polyisoprene (b) Cellulose nitrate
5. In which of the following polymers ethylene glycol is one of
(c) Cellulose acetate (d) Vulcanised rubber the monomer units?
3. The commercial name of polyacrylonitrile is ....... .
(a) (OCH CH OOC CO )n
(a) dacron (b) orlon (acrilan) 2 2
(c) PVC (d) bakelite
4. Which of the following polymer is biodegradable?
(a) –( CH2 C = CH CH2 n
–(
(b) ( CH 2 — CH 2 )n

Cl
CN
(c) (CH 2 CH= CH CH 2 CH CH 2 )n
(b) –( CH2 CH = CH CH2 CH2 CH –)n
(d) (O CH CH 2 C O CH CH 2 C )n
| || | ||
CH3 O CH2CH 3 O
EBD_7327
742 CHEMISTRY

6. Which of the following statements is not true about low 10. Nylon is an example of : [2013]
density polyethene? (a) Polysaccharide (b) Polyamide
(a) Tough (c) Polythene (d) Polyester
(b) Hard 11. Which one of the following is an example of a thermosetting
(c) Poor conductor of electricity polymer? [2014]
(d) Highly branched structure
(a) ( CH 2 - C = CH - CH 2 ) n
CH3 CH3 |
| | Cl
7. (O C CH2 C )n is a polymer having monomer
| | (b) ( CH 2 - CH ) n
CH3 CH3 |
Cl
units ......... .
H H O O
(a) (b) | | || ||
(c) ( N - (CH 2 )6 - N - C - (CH 2 )4 - C )n
H
OH OH
(c) (d)
H CH2 CH2
(d)
8. Which of the following polymer can be formed by using the
following monomer unit? n
12. Which of the following organic compounds polymerizes to
H
| form the polyester Dacron? [2014]
N O (a) Propylene and para HO—(C6H4)— OH
C
||

H2C (b) Benzoic acid and ethanol


(c) Terephthalic acid and ethylene glycol
H2C CH2 (d) Benzoic acid and para HO–(C6H4)—OH
13. Biodegradable polymer which can be produced from glycine
H2C CH2 and aminocaproic acid is : [2015]
(a) PHBV (b) Buna - N
(a) Nylon-6, 6 (b) Nylon-2-nylon-6
(c) Melamine polymer (d) Nylon-6 (c) Nylon 6, 6 (d) Nylon 2- nylon 6
14. Caprolactum is used for the manufacture of : [2015 RS]
NEET/AIPMT (2013-2017) Questions (a) Nylon - 6 (b) Teflon
9. Which is the monomer of Neoprene in the following ? (c) Terylene (d) Nylon - 6,6
CH 2 = C ¾ CH = CH 2 15. Natural rubber has [2016]
(a) [2013]
½ (a) all cis-configuration
CH 3 (b) all trans-configuration
(b) CH 2 = C ¾ CH = CH 2 (c) alternate cis-and trans-configuration
½ (d) random cis-and trans-configuration
Cl
(c) CH2 = CH ¾ C º CH
(d) CH2 = CH ¾ CH = CH2
Polymers 743

Hints & Solutions


EXERCISE - 1 O O

1. (c) Nylon is a polymer of adipic acid and OCH2CH2 – O C C + nH2O


n
hexamethylenediamine thus it contains nitrogen.
Polyvinyl chloride is a polymer of vinyl chloride. It does Terylene or Dacron
not contain nitrogen. It is a condensation polymer.
Bakelite is a resin of formaldehyde and phenol. It does [Note: Now a days dimethyl terepthalate is used instead
not contain nitrogen. of terephthalic acid. This is due to the fact that terephthalic
Terylene is a polymer of ethylene glycol and terephthalic acid is non-volatile and very slightly soluble in most of
acid. It does not contain nitrogen. It is also called dacron. the solvents. Dimethyl terepthalate on the other hand,
2. (b) Elastomers are the polymers having very weak can be purified by distillation very easily]. Thus, option
intermolecular forces of attraction between the polymeric (a) is correct.
chains. The weak forces permit the polymer to be 11. (c) Nylon is a general name for all synthetic fibres forming
streched. polyamides.
12. (a) PVC is an example of thermoplastic polymer.
13. (a) The synthetic polymer neoprene (polychloroprene) is
3. (b) n CH2=CH–C=CH2 Polymerisation prepared by polymerisation of chloroprene.
CH3 2 O or peroxides
Isoprene nCH 2 = C - CH = CH 2 ¾¾¾¾¾¾¾
®
–CH2–CH=C–CH2 – |
n Cl
CH3 Chloroprene
Polyisoprene
(Natural rubber) é
—CH 2 - C = CH-CH 2 —
ù
4. (d) The given compound is a copolymer of hexamethylene ê | ú
êë Cl úûn
diamine and adipic acid. It is actually Nylon-66.
Neoprene
5. (d) Bakelite is a thermosetting polymer. On heating it becomes
hard and infusible thus cannot be remoulded. This is inferior to natural rubber but is more resistant to
6. (a) Polymers which form from condensation polymerisation aerial oxidation and to other solvents.
are called condensation polymers dacron is a O O
|| ||
condensation polymer of terepthalic acid and ethylene
14. (c) H 2 N - (CH 2 - )6 NH 2 + Cl - C - (CH 2 )8 - C - Cl
glycol. Hexamethylene diamine Sebacoyl chloride
7. (b) Glyptal is not a polyamide
O O
O || ||
D
|| ¾¾
® - HN - (CH 2 )6 - (NH - C - (CH 2 )8 - C - ) n
A polyamide has amide linkage -C - N - Nylon 6, 10
| 15. (b) Since proteins, cellulose and RNA control various
H activities of plants and animals, they are called
Example biopolymers.
16. (d) 17. (b)
æ H H O ö 18. (d) Urotropine is used as antibiotic for urinary tract infection.
ç | | || ÷
ç - N - (CH 2 )6 - N - C - (CH 2 ) 4 - C - ÷ 19. (b) nHOOC(CH 2 )4 COOH + nH 2 N(CH 2 )6 NH 2
ç || ÷ adipic acid Hexamethylene
ç O ÷ diamine
è ø
Nylon 6,6 O O
525K
8. (c) Ebonite is a hard highly vulcanized rubber, containing ¾¾¾® [– C – (CH 2)4 – C –NH–(CH2) 6 – NH –]n
20-25%, sulphur. Polymerisation Nylon 6, 6
9. (b) Nylon is a condensation polymer 20. (d) Terylene is prepared by condensing terephthalic acid
10. (a) The polymer of ethylene glycol and tere-phthalic acid is and ethylene glycol
known as terylene or dacron.
n(HO - CH 2 - CH 2 - OH) +
nHOOC COOH + nHOCH2CH2OH
Ethylene glycol Terephthalic acid Ethylene glycol

O O
COOH ¾¾ || ||
n HOOC ® C C – OCH2 – CH2 – O
Terephthalic acid
Terylene
n
EBD_7327
744 CHEMISTRY

21. (d) 43. (c) Polystyrene is manufactured by polymerization of styrene


in presence of benzoyl peroxide.
é CH3 ù
ê | ú CH=CH2 CH–CH2
22. (a) Monomer of —
ê C — CH 2 ú polymer is

ê | ú (C6H5CO)2O2
ë CH3 ûn
2-methylpropene. n
44. (d) Cellulose acetate has been used in the manufacture of
23. (b) Acrilan is a polyacrylonitrile ( PAN).
non inflammable photographic films.
24. (b)
45. (d) Nylon tyre cord is made from high tenacity continuous
25. (a) Addition of monomers follows isoprene rule filament yarn by twisting and plying.
CH3 CH3 46. (d) The unbreakable plastic household crockery is made from
C = CH2 C = CH2 copolymer of formaldehyde (HCHO) and melamine.
CH3 CH3
H2N N NH2
H T H T Polymerisation
+ HCHO ¾ ¾ ¾ ¾ ¾ ¾® Melamine
¯ N N formaldehyde polymer

CH 3 CH 3 NH2
| |
Melamine
Å
C — CH 2 - C — CH 2 47 (d) Polyhydroxybutyrate-co-b-hydroxyvalerate or PHBV
| | is a biodegradable polymer and thus it can be used as
CH 3 CH 3 biomaterials in orthopaedic devices and in controlled
drug release.
26. (d) Orlon is a trade name of polyacrylonitrile
CH3 - CHCH 2COOH + CH3CH 2 - CH - CH 2 COOH
27. (d) P.V.C. (Polyvinyl chloride) is formed by polymerisation of |
vinyl chloride, CH2 =CHCl, whose IUPAC name is |
OH
1-chloroethene OH

28. (d) 29. (b) 30. (c) é O ù


31. (c) Polythene is a linear polymer ê || ú
¾¾® -ê O - CH - CH 2 - C - O ú-
ê | ú
Mw ê ú
32. (c) PDI = ë R ûn
Mn (PHBV)
For synthetic polymer, PDI > 1 R =CH3 , C2 H5

48. (d) Copolymer of adipic acid (6C) and hexamethylene


\ Mw >Mn diamine (6C).
33. (d)
n HOOC(CH 2 ) 4 COOH + nH 2 N (CH 2 ) 6 NH 2 ®
34. (b) Glyptal is used in preparation of lacquers. Adipic acid Hexamethyl ene diamine

35. (a) Ethene on free radical polymerisation gives low density


O O
polythene. || ||
36. (a) -( C- (CH 2 ) 4 - C- NH - (CH 2 ) 6 - NH -) n
Nylon 66
37. (b) PMMA and polycarbonate is used in bullet proof glass
38. (d) Acrylonitrile is the monomer used in the preparation of It has high tenacity and elasticity. It is resistant to
orlon. abrasion and not affected by sea water. It is used for
reinforcement of rubber tyres, manufacture of
39. (b) Glyptal is an alkyd resin obtained from polyhydric alcohols parachute, safety belts, carpets and fabrics.
and polybasic organic acids or their anhydrides. They
are used for surface coatings. 49. (b)
40. (a) Also known as PMMA. It is a transparent, excellent light CH3 H CH2 CH2 CH3 H
C=C C=C C=C
transmitter and its optical clarity is better than glass so it – CH2 CH2 CH3 H CH2
is used in the preparation of lenses for eyes. CH2–
Natural rubber
Natural rubber
41. (d)
(All cis configuration)
42. (a) SARAN, a polymer of vinyl chloride (CH2=CHCl) and
vinylidene chloride, is used for making synthetic hair wigs. All statements except (b) are correct
Polymers 745

50. (a) Neoprene is a polymer of chloroprene 56. (b) Average number molecular weight M n = 30,000
(2 – chloro – 1, 3 – butadiene).
Average mass molecular weight M w = 40,000
51. (a) Polyesters are condensation polymers of a dibasic acid
and a diol. e.g., Terylene M w 40,000
Polydispersity index (PDI) = = = 1.33
M n 30,000
57. (b)
Ethylene glycol Terephthalic acid 58. (c) Terylene is made from glycol and Terephthalic acid

HO - CH2 - CH2 - OH and HOOC COOH


(Glycol)
(Terephthalic acid)

Terylene 59. (a) Th e synthetic polymer neoprene is prepared by


52. (d) Neoprene is an addition polymer of isoprene. polymerisation of chloroprene.

Cl n CH 2 - C - CH = CH 2 + CH 2 - C - CH = CH 2
| | |
O 2 or peroxides Cl
nCH 2 = CH - C = CH 2 ¾¾¾¾¾¾ ¾ ® Cl
Chloroprene

æ Cl ö
¯ O2 or peroxide
ç | ÷ ( -CH2 - C = CH - CH 2 - CH 2 - C = CH - CH 2 - )n
—çè CH 2 - C = CH - CH2 —
÷
ø | |
Neoprene Cl Cl
53. (b) Nylon (polyamides) are fibres. Polychloroprene of Neoprene
54. (d) Phenol and formaldehyde undergo condensation This is inferior to natural rubber but is more resistant to
polymerisation under two different conditions to give aerial oxidation and to other solvents.
a cross linked polymer called bakelite. 60. (c) Teflon is non-inflammable and resistant to heat so it is
OH OH OH used in coating, particularly in non-sticking frying pans.

CH2OH 61. (a) SF6 is used in the vulcanisation of rubber. Sulphur is
OH heated with polymer to introduce cross-linking and thus,
+ HCHO +
form tough polymer.
CH2OH 62. (a) Vulcanization is a process in which natural rubber is
o & p-hydroxymethylphenol treated with sulphur and certain organic compounds which
(intermediate) accelerate the reaction between the rubber and sulphur.
OH Thus about 5% S is used for making tyre rubber, 20-25%
OH OH S for making ebonite and 30% S for making battery case
CH2OH
n Polymerisation CH2 rubber.
– (n – 1) H2 O
63. (b) Perlon is Nylon-6. It is prepared from a single monomer
Linear Polymer (Novalac) having a potential amino group at one end and a potential
OH OH carbonyl group at other end.
CH2OH 64. (c)
n Polymerisation 65. (b) Biodegradable polymer is Nylon-2- Nylon-6 which is
+ n
– H2O copolymer of glycine (H2N – CH2– COOH) and amino
caproic acid (H2N–(CH2)5 – COOH).
CH2OH
nH2N – CH2 – COOH + nH2N – (CH2)5– COOH
Glycine Amino caproic acid

O O

–( HN – CH2 – C – HN – (CH2)5 – C –) n
nylon – 2 – nylon – 6
66. (b) Buna – N is a copolymer of butadiene
55. (a) (CH2=CH–CH=CH2) and acrylonitrile (CH2 = CHCN).
EBD_7327
746 CHEMISTRY

67. (b) Nylon 6, 6 has amide linkage capable of forming hydrogen 7. (d) Nylon–6 can be manufactured from
bonding. O
68. (d) Nylon and cellulose, both have intermolecular hydrogen
bonding, polyvinyl ch lor ide has dipole-dipole NH2OH
interactions, while natural rubber has van der Waal forces
which are weakest.
O
69. (d)
NOH C
70. (d) Polyhydroxybutyrate-co-b-hydroxyvalerate or PHBV is conc H2SO4
a biodegradable polymer and thus it can be used as NH
(Beckmann
biomaterials in orthopaedic devices and in controlled rearrangement) C
drug release. H
H
CH3 - CHCH 2COOH + CH3CH 2 - CH - CH2 COOH caprolactum
| | H2O, D
OH OH
é Où
é O ù ê || úú ,(n,1)H 2O
ê polymerization
ê || ú –êë NH , (CH 2 )5 , C –úû ↔¾¾¾ ¾¾¾
¾¾® -ê O - CH - CH 2 - C - O ú- Nylon,6
ê | ú é ∗ ù
ê ú ê H 3 N, (CH 2 )5 , COO, ú
ë R ûn ë û
(PHBV)
R =CH3 , C2 H5 8. (b) SBR (styrene-butadiene) is a synthetic rubber.

71. (c) Polystyrene is manufactured by polymerization of styrene 9. (b)


in presence of benzoyl peroxide. 10. (b) The condensation polymerisation of hexamethylene
CH=CH2 CH–CH2 diamine and adipic acid is done in solution form by
interface technique. In this liquid nylon polymer is
(C6H5CO)2O2 obtained.

n n.H 2 N - (CH 2 )6 - NH 2 +
72. (c) Bakelite is a cross linked polymer. Polymerisation
+ nHOOC – (CH 2 ) 4 – COOH ¾¾¾¾¾¾
- nH O
®
73. (d) Acrylonitrile is the monomer used in the preparation of 2

orlon.
[ - HN - ( CH 2 ) 6 - NHCO - ( CH 2 ) 4 - CO -]n
74. (b) Glyptal is used in preparation of lacquers. Nylon

75. (c) 11. (d) Polymethyl methacrylate is hard, fairly rigid. It is used for
optical lenses.
EXERCISE - 2
12. (d) Generally high boiling esters or haloalkanes act as
1. (c) Glycine and amino caproic acid are the monomers used
for the preparation of nylon-2-nylon-6. plasticizer.

2. (a) Polymerisation starts either by condensation or addition 13. (b) In addition homopolymers such as teflon, empirical
reactions between monomers formula resembles with monomer.
3. (b) Nylon is a polyamide polymer 14. (a) ABS is acrylonitrile-butadiene-styrene rubber which is
obtained by copolymerisation of acrylonitrile,
4. (b) Nylon 6, 6 has amide linkage capable of forming hydrogen
bonding. 1, 3-butadiene and styrene.

Benzoyl peroxide or
5. (d) n(CF2 = CF2 ) ¾¾¾¾¾¾¾¾ ® ( CF2 - CF2 ) CH 2 = CH + CH 2 = CH + CH 2 = CHC6 H5 ¾¾
®
(NH 4 )2 S2O8
PTFE | | styrene
(X) CN CH = CH 2
acrylonitrile 1,3- butadiene
Here X is polytetrafluoroethylene. So, none of these i.e.,
option (d) is correct choice.
6. (c) PCTFE and PTFE both have same carbon backbone. é CH 2 - CH - CH 2CH = CHCH 2 - CH 2 - CH —
— ù
ê | | ú
-[ CIFC - CF2 –] n ëê CN C6 H5ûú n
PCTFE (polymonochloro trifluoroethylene)
-[ F C - CF –]
2 2 n ABS rubber
PTFE (poly tetrafluoro ethylene)
Polymers 747

15. (c) Nylon fibres are made up of polyamides. Dacron and orlon
n CN
are made up of polyester. Rayon is artificial silk made up |
of polysaccharides. CN ¾ ¾ ¾ ¾ ¾ ¾ ¾
® ( CH 2 CH )n
Peroxide catalyst
PAN
16. (d) Polyacetylene, due to presence of double bonds, is a or orlon
or acrilan
conducting polymer.
4. (c) PHBV is an example of biodergradable polymer.
5. (a)
æ CH 2 = CHö
17. (a) Orlon is polymer of acrylonitrile ç | ÷ HOOC COOH
è CN ø
n HOCH2 — CH2 OH + n
Glycol
nCH 2 < CH ¾¾
↑—æ CH 2 , C H —
ö
çç | ÷÷÷
Phthalic acid
| ççè
CN CN ø÷ n O
Zn (OCOCH3) 2 + Sb 2O3 ||
420-460 K C )n
18. (a) Dacron is a polyester and is the condensation polymer of (O CH 2 CH 2OOC
–nH2O
ethylene glycol and terephthalic acid. It is crease resistant
19. (a)
Terylene or dacron
20. (c) Nylon 6, 6 is a fibre with hydrogen bonds thus have
strongest intermolecular forces. Buna-S is a elastomer 6. (d) Low density polythene has slightly branched structure.
whereas polythene is thermoplastic. the intermolecular
CH3
forces of attraction in thermoplastics is intermediate 7. (a) n CH2 = C
between elastomer and fibres. Therefore forces in Monomer
CH3
elastomers are weakest.
CH3 CH3
21. (a) Neoprene is a polymer of chloroprene. Hence, correct | |
Polymerisation
representation is ( CH2 C CH2 C )n
| |
Cl CH3 CH3
Polymer
|
O 2 or peroxides
nCH 2 = CH — C = CH 2 ¾¾¾¾¾¾® H
Chloroprene | O O
N || D, Polymerisation ||
æ ö 8. (d) –[ NH — (CH2 )5 — C]n
– (n – 1)H 2O
ç ÷ Caprolactum
Nylon-6
ç -CH 2 - C = CH - CH 2 -÷
ç | ÷ NEET/AIPMT (2013-2017) Questions
è Cl øn
Neoprene
Cl
22. (b) Vulcanisation is a process of treating natural rubber with |
sulphur or some compounds of sulphur under heat so as 9. (b) CH 2 = C- CH = CH 2 (chloroprene) is the monomer
to modify its properties. This cross-linking give of neoprene.
mechanical strength to the rubber.
10. (b) Nylon is a synthetic polyamide polymer.
23. (c) Bakelite can be heated only once.
11. (d) Thermosetting polymers undergo chemical changes
24. (d) The time of vulcanisation is reduced by adding when heated and set to hard mass when cooled e.g.
accelerators and activators. Bakelite.
25. (b) Due to the presence of strong C–F bonds, teflon has
high thermal stability and chemical inertness.

EXERCISE -3
Exemplar Questions
1. (d) Glycogen is a polymer of glucose found in liver, brain
and muscles of animals.
2. (a) cis-polyisoprene is a synthetic polymer.
3. (b) The commercial name of polyacrylonitrile is orlon
(acrilan).
EBD_7327
748 CHEMISTRY

12. (c) O O
H
NH N
14. (a) Nylon–6 n
(Caprolactum)

Ethylene glycol Terephthalic acid


15. (a) Natural rubber is found to be a polymer of cis-isoprene
O O i.e. it is cis-polyisoprene
D || ||
-( OCH2CH2–O– C C- )n H2C = C — CH = CH 2 ¾ polymerisation
¾ ¾ ¾ ¾ ¾®
|
cis-polyisoprene
Terylene
CH3
CH2 CH2
It is resistant to mineral and organic acids. It is used for
blending with wool to provide better crease, in safety
H3C H
helmets and aircraft battery boxes.
Isoprene
13. (d) H2 N—CH2—COOH + H2 N — (CH2)5—COOH ®
(HN—CH2—CO—NH— (CH2)5—CO) —
Glycine Aminocapric acid
Nylon-2-nylon-6
30
Chemistry in
Everyday Life
Drugs are chemicals of low molecular masses (~ 100 – 500u). They Neurologically Active Drugs
interact with macromolecular targets and produce a biological (i) Tranquilizers:
response. When the biological response is therapeutic and useful, They are used for treatment of stress, and mild or even severe
these chemicals are called medicines and are used in diagnosis, mental diseases. They relieve anxiety, stress, irritability or
prevention and treatment of diseases. If taken in doses higher excitement by inducing a sense of well-being.
than those recommended, most of the drugs used as medicines They also form an essential component of sleeping pills.
are potential poisons. Use of chemicals for therapeutic effect is Example :
called chemotherapy. (a) Noradrenaline plays a role in mood changes.
Note : Medicine is a chemical substance which cures the disease (b) Chlordiazepoxide and meprobamate are suitable for
and is safe to use it does not cause any addiction whereas drug is relieving tension.
a chemical substance which cures disease but it causes addiction (c) Equanil is used in controlling depression and
and has serious side effects. hypertension.
(d) Barbiturates, i.e., derivatives of barbituric acid like
CLASSIFICATION OF DRUGS veronal, amytal, nembutal, luminal and seconal. They
(i) On the basis of pharmacological effect. are hypnotic i.e., sleep producing agents.
For example : analgesics have pain killing effect, antiseptics (ii) Analgesics : They reduce or abolish pain without causing
kill or arrest the growth of microorganisms. impairment of consciousness, mental confusion,
incoordination or paralysis or some other disturbances of
(ii) On the basis of drug action i.e., action of drug on a particular nervous system.
biochemical process. For example : all antihistamines inhibit
the action of the compound, histamine.
(iii) On the basis of chemical structure of the drug. Non-narcotic Narcotic
analgesics analgesics
(iv) On the basis of molecular targets. Drugs usually interact
with biomolecules such as carbohydrates, lipids, proteins (a) They relieve skeletal pain, (a) Relieve pain and
and nucleic acids. These are called target molecules or drug reduce fever (antipyretic). produce sleep.
Ex: aspirin, paracetamol
targets. Drugs possessing some common structural features (b) Aspirin is used in (b) Chiefly used for relief
may have same mechanism of action on targets. prevention of heart of postoperative pain,
TYPES OF DRUGS attacks due to cardiac pain and pains
anti blood clotting of terminal cancer and
Antacids action. in child birth.
They are used for the treatment of acidity (due to over production Ex: morphine, heroin,
of acid in stomach). codeine
For example : NaHCO3, a mixture of Al(OH)3 and Mg(OH)2. Antimicrobials
Metal hydroxides are better alternatives because they are insoluble They tend to destroy / prevent development or inhibit the
and do not increase the pH above neutrality. pathogenic action of microbes such as bacteria, fungi, virus or
other parasites selectively. Antibiotics, antiseptics and
Antihistamines disinfectants are antimicrobial drugs.
The drugs which have been used to fight allergy are called Antibiotics
antihistamines. These are so called because they check the They are used to treat infections due to their low toxicity for humans
production of histamines. Thus, antihistamines are widely used and animals. An antibiotic refers to a substance produced wholly
for treatment to hay fever, conjunctivitis, nasal discharges, or partly by chemical synthesis, which in low concentrations
irradiation sickness, nausea in pregnancy and post operative inhibits the growth or destroys micro-organisms by intervening
vomiting. For example : cimetidine, brompheniramine, terfenadine. in their metabolic processes.
EBD_7327
750 CHEMISTRY

For example : sulphanilanide, sulphapyridine, etc. Note : Sodium benzoate is used in limited quantities and is
Types of antibiotics metabolised in the body.
Antibiotics which kill or inhibit a wide range of Gram-positive and CLEANSING AGENTS
Gram-negative bacteria are said to be broad spectrum antibiotics.
Two types of detergents are used as cleansing agents: soaps and
For example : chloramphenicol, vancomycin, Ampicilin, Amoxycilin,
synthetic detergents. They improve cleansing properties of water
etc.
by helping in removal of fats which bind other materials to the
Antibiotics effective few Gram-positive or Gram-negative bacteria
fabric or skin.
are narrow spectrum antibiotics. For example : penicillin.
If effective against a single organism or disease, they are referred Soaps
as limited spectrum antibiotics. They are sodium or potassium salts of long chain fatty acids, ex:
Antiseptic and disinfectants stearic, oleic and palmitic acids. Only Na and K-soaps are soluble
Antiseptics are applied to living tissues such as wounds, cuts, in water and are used for cleaning purposes. K-soaps are
ulcers and diseased skin surfaces. comparatively soft to the skin than Na-soaps. Alkaline hydrolysis
Examples : of oils or fats by NaOH or KOH gives glycerol and sodium or
(i) furacine, soframicine, dettol (mixture of chloroxylenol and potassium salt of the fatty acid. This reaction is known as
saponification.
terpeniol)
(ii) Bithional is added to soaps to impart antiseptic properties. CH2OCOC17H35
(iii) Tincture of iodine (2-3% solution of iodine in alcohol-water CHOCOC17H35 + 3NaOH
mixture) is applied on wounds.
(iv) Iodoform is also used on wounds. CH2OCOC17H35
(v) Boric acid in dilute aqueous solution is weak antiseptic for
eyes.
Disinfectants are applied to inanimate objects such as floors, CH2OH
drainage system, instruments etc. For example : Chlorine in CHOH + 3C17 H35 COONa (sodium stearate)
concentration of 0.2 to 0.4 ppm in aqueous solution and SO2 in
CH2OH
very low concentrations, are disinfectants. Same substances can
act as an antiseptic as well as disinfectant by varying the Soaps do not work in hard water. This is because hard water
contains calcium and magnesium ions. These ions form insoluble
concentration. For ex: 0.2% solution of phenol is an antiseptic
Ca– and Mg soaps when Na or K soaps are dissolved in hard
while its 1% solution is disinfectant.
water. These insoluble soaps separate as scum in water and are
Antifertility Drugs useless as cleansing agents.
They are also called birth control pills. They essentially contain a Note : Hair washed with hard water looks dull due to sticky ppt.
mixture of synthetic estrogen and progesterone. Both of these Dye does not absorb evenly or cloth washed with soap using
compounds are hormones. hard water because of gummy mass.
Example : Norethindrone, ethynylestradiol (novestrol) Synthetic Detergents
Antimalarials They can be used in both soft and hard water as they give foam
Chemical substances which are used to bring down the body even in hard water.
temperature during malaria fever are called antimalarials. These They are classified into three categories:
are Chloroquine, Paraquine, Primaquine etc. (i) Anionic detergents:
These are sodium salts of sulphonated long chain alcohols
CHEMICALS IN FOOD
or hydrocarbons. The anionic part of the molecule is
Chemicals are added to food for involved in cleansing action. They are used for household
(i) their preservation work and also in toothpastes.
(ii) enhancing their appeal For example : sodium laurylsulphate, sodium salts of
(iii) adding nutritive value in them. alkylbenzene-sulphonates.
Artificial Sweetening Agents (ii) Cationic detergents : These are quarternary ammonium salts
These are of use to diabetic patients and people who need to of amines with acetates, chlorides or bromides as anions.
control intake of calories. Cationic part possess a long hydrocarbon chain and a
For example : positive charge on N-atom. Hence, the name. They have
(i) saccharin (ortho-sulphobenzimide) germicidal properties and are expensive.
(ii) asparatame (added to cold foods and soft drinks as it is For example : cetyltrimethyl ammonium bromide, used in hair
unstable at cooking temperature) conditioners.
(iii) alitame (iii) Non-ionic detergents
(iv) sucralose (trichloro derivative of sucrose) They do not contain any ion in their composition. Liquid
dishwashing detergents are non-ionic type.
Food Preservatives ex: CH3(CH2)16COO(CH2CH2O)n CH2CH2OH
They prevent spoilage of food due to microbial growth. Note : As the hydrocarbon chain of the detergent becomes
For example : table salt, sugar, vegetable oils, salts of sorbic acid branched, degradation becomes difficult which leads to their
and propanoic acid and sodium benzoate (C6H5COONa) accumulation and hence pollution.
Chemistry in Everyday Life 751

CONCEPT MAP
EBD_7327
752 CHEMISTRY

1. Synthetic detergents are more effective in hard water than 10. The general structure of penicillin is
soaps because
(a) they are non-ionic O
S CH3
(b) their Ca++ and Mg++ salts are insoluble in water R C NH
(c) their Ca++ and Mg++ salts are water soluble CH3
(d) they are highly soluble in water N COOH
H
2. Aspirin is
(a) antibiotic (b) antipyretic In ampicillin R =
(c) sedative (d) psychedelic (a) CH3(CH2)6–
3. An antipyretic is (b) –CH2–CH=CH–CH2–CH3
(a) quinine (b) paracetamol
(c) luminal (d) piperazine (c) CH2
4. Salol can be used as
(a) antiseptic (b) antipyretic
(d) CH
(c) analgesic (d) None of these
5. Which of the following is used as a ‘morning after pill’ NH2
(a) Norethindrone (b) Ethynylestradiol 11. Amoxillin is semi-synthetic modification of
(c) Mifepristone (d) Bithional (a) penicillin (b) streptomycin
6. Various phenol derivatives, tincture of iodine (2 – 3%) I2 in (c) tetracycline (d) chloroampheniol
(water / alcohol) and some dyes like methylene blue are 12. Which of these is a hypnotic
(a) antiseptics (b) disinfectants (a) metaldehyde (b) acetaldehyde
(c) analgesics (d) antipyretics
(c) paraldehyde (d) None of these
7. The insecticide containing 99% g – isomer of benzene
13. Barbituric acid and its derivatives are well known
hexachloride is known as (a) antipyretics (b) analgesics
(a) lindane (b) TNT (c) antiseptics (d) traquillizers
(c) malathion (d) methoxychlor 14. The drug used for prevention of heart attacks is
8. The following compound is used as
(a) aspirin (b) valium
O (c) chloramphenicol (d) cephalsoprin
O – C – CH3 15. Serotonin is usually used as :
(a) analgesic (b) tranquilizer
(c) antiseptic (d) antioxidant
COOH 16. Omeoprazole and lansoprazole are used as –
(a) an anti-inflammatory compound (a) antifertility (b) antiallergic
(b) analgesic (c) antibiotic (d) antacid
(c) hypnotic 17. Sulpha drugs are used for
(d) antiseptic (a) precipitating bacteria
9. The correct structure of the drug paracetamol is (b) removing bacteria
OCH3 OH (c) decreasing the size of bacteria
(d) stopping the growth of bacteria
18. Which one of the following is an antihistamine?
(a) (b) (a) Iproniazid (b) Salvarsan
(c) Zantac (d) Chloramphenicol
19. Chloramine-T is a/an
CONH2 NHCOCH3 (a) disinfectant (b) antiseptic
OH OH (c) analgesic (d) antipyretic
20. Which of the following is a hypnotic drug?
(a) luminal (b) salol
(c) (d) (c) catechol (d) chemisol
21. Streptomycin is effective in the treatment of
(a) tuberculosis (b) malaria
CONH2 (c) typhoid (d) cholera
COCH3
Chemistry in Everyday Life 753

22. An antibiotic with a broad spectrum (b) It is 600 times sweeter than sugar
(a) kills the antibodies (c) It is used as sweetening agent
(b) acts on a specific antigen (d) All of these
(c) acts on different antigens 37. Which of the following acts as an antioxidant is edible oils
(d) acts on both the antigens and antibodies (a) Vitamin B (b) Vitamin C
23. Chloramine-T is used as an :
(c) Vitamin D (d) Vitamin E
(a) analgesic (b) antiseptic
38. Which of the following is used as an antioxidant in food?
(c) antipyretic (d) anti-inflamatory
24. Which of the following is not an antiseptic drug? (a) BTX (b) BHT
(a) Iodoform (b) Dettol (c) BHC (d) All the three
(c) Gammexane (d) Genation violet 39. End of detergents have
25. A medicine which promotes the secretion of urine is called (a) ester group (b) sodium sulphate
(a) uretic (b) monouretic (c) aldehyde (d) amine group
(c) diuretic (d) triuretic 40. Which one of the following is not a tranquilizer?
26. Veronal, a barbiturate drug is used as (a) Equanil (b) Veronal
(a) anaesthetic (b) sedative (c) Salvarsan (d) Serotonin
(c) antiseptic (d) None of these 41. Which of the following represents a synthetic detergent?
27. A drug effective in the treatment of pneumonia, bronchitis, (a) C15H31COOK
etc, is
(b) CH3[CH2]16COONa
(a) streptomycin (b) chloramphenicol
(c) penicillin (d) sulphaguanidine
(c) C12H25 SO3Na
28. The use of chemicals for treatment of diseases is called as
(a) isothermotherapy (b) angiotherapy (d) None of these
(c) physiotherapy (d) chemotherapy
42. Which of the following represents soap
29. Select the incorrect statement.
(a) Equanil is used to control depression and hypertension. (a) C17H35COOK (b) C17H35COOH
(b) Mifepristone is a synthetic steroid used as “morning (c) C15H31COOH (d) (C17H35COO)2Ca
after pill”. 43. Benzalkonium chloride is a
(c) 0.2 per cent solution of phenol is an antiseptic while its (a) cationic surfactant and antiseptic
1.0 per cent solution is a disinfectant. (b) anionic surfactant and soluble in most of organic
(d) A drug which kills the organism in the body is called solvents
bacteriostatic. (c) cationic surfactant and insoluble in most of organic
30. Terfenadine is commonly used as a/an solvents
(a) tranquilizer (b) antihistamine (d) cationic surfactant and antimalarial
(c) antimicrobial (d) antibiotic 44. Cetyltrimethyl ammonium bromide is a popular
31. Structuraly biodegradable detergent should contain (a) anionic detergent (b) cationic detergent
(a) normal alkyl chain (b) branched alkyl chain
(c) non-ionic detergent (d) sweetener
(c) phenyl side chain (d) cyclohexyl side chain
32. Detergents are prepared by the action of H2SO4 on which 45. Commonly used antiseptic 'Dettol' is a mixture of
of the following? (a) o-chlorophenozylenol + terpeneol
(a) Cholesterol (b) Lauryl alcohol (b) o-cresol + terpeneol
(c) Cyclohexanol (d) p-Nitrophenol (c) phenol + terpeneol
33. Arsenic containing medicine used for the treatment of (d) chloroxylenol + terpeneol
syphilis, is 46. Which one of the following is employed as a tranquilizer drug?
(a) erythromycin (b) ofloxacin (a) Promethazine (b) Valium
(c) tetracycline (d) salvarsan (c) Naproxen (d) Mifepristone
34. Sodium alkyl benzene sulphonate is used as 47. Which one of the following is employed as Antihistamine ?
(a) soap (b) fertilizers (a) Chloramphenicol (b) Diphenylhydramine
(c) pesticides (d) detergents
(c) Norothindrone (d) Omeprazole
35. Which is correct about vanillin?
48. Aspirin is known as :
(a) A flavouring agent having vanila flavour
(b) p-hydroxy-m-methoxy benzaldehyde (a) acetyl salicylic acid (b) phenyl salicylate
(c) A food additive (c) acetyl salicylate (d) methyl salicylic acid
(d) All of these 49. Which one among the following is not an analgesic?
36. Which is correct about saccharin? (a) Ibuprofen (b) Naproxen
O (c) Aspirin (d) Valium
50. Further growth of cancerous cells in the body is arrested by
C (a) physiotherapy (b) chemotherapy
(a) It is NH
SO2 (c) electrotherapy (d) psychotherapy
EBD_7327
754 CHEMISTRY

1. Which one of the following is not used as a filler in laundry 12. An antibiotic contains nitro group attached to aromatic
soaps? nucleus. It is
(a) Sodium silicate (b) Glycerol (a) penicillin
(c) Sodium rosinate (d) Borax (b) streptomycin
2. The drug which is effective in curing malaria is (c) tetracycline
(a) quinine (b) aspirin (d) chloramphenicol
(c) analgin (d) equanil 13. The cationic detergent that is used in hair conditioners is
3. Which of the following is a hallucinogenic drug? (a) sodium dodecylbenzene sulphonate
(a) Methedrine (b) Calmpose
(b) sodium lauryl sulphate
(c) LSD (d) Seconal
(c) tetramethyl ammonium chloride
4. Which one of the following is not a neurologically active
drug? (d) cetyltrimethyl ammonium bromide
(a) Veronal (b) Bithionol 14. Placebo is often given to patients. It is
(c) Equanil (d) Nardil (a) an antidepressant
5. Interferon is connected with (b) a broad spectrum antibiotic
(a) tonic (b) virus (c) a sugar pill
(c) carbohydrate (d) ore of iron (d) a tonic
6. Which of the following statements about aspirin is not true? 15. Sulphaguadine is used for
(a) It is effective in relieving pain. (a) aysentery (b) urinary infections
(b) It is a neurologically active drug. (c) antiseptic (d) antipyretic
(c) It has antiblood clotting action. 16. Salts of sorbic acid and propionic acid are used as
(d) It belongs to narcotic analgesics.
(a) antioxidants
7. Which of the following is an alkaloid?
(b) flavouring agents
(a) Nicotine (b) Piperine
(c) food preservatives
(c) Cocaine (d) All of these
(d) nutritional supplements
8. Phenacetin is used as
(a) antipyretic (b) antiseptic 17. The structure given below is known as

(c) antimalarial (d) analgesic O


H H
P S CH3
9. The artificial sweetener containing chlorine that has the CH 2 - C - NH
appearance and taste as that of sugar and is stable at cooking CH3
temperature is N COOH
O H
(a) Aspartame (b) Saccharin
(a) Penicillin F (b) Penicillin G
(c) Sucrolose (d) Alitame
(c) Penicillin K (d) Ampicillin
10. Morphine is
(a) an alkaloid (b) an enzyme 18. Consider the following antibiotics.
(c) a carbohydrate (d) a protein (i) Erythromycin (ii) Ofloxacin
11. A large number of antibiotics have been isolated from (iii) Chloramphenicol (iv) Penicillin
(a) Bacteria actinomycetes (v) Tetracycline
(b) Acids The pair of bactericidal antibiotics is
(c) Alkanals (a) i-iii (b) ii-iv
(d) Bacteria rhizobium (c) iii-v (d) i-iv
Chemistry in Everyday Life 755

19. Match the chemicals in Column I with their uses in (b) Statement-1 is true, Statement-2 is true ; Statement-2 is NOT
Column II. a correct explanation for Statement-1
(c) Statement-1 is true, Statement-2 is false
Column I Column II
(A ) Sodium Perborate (I) Disinfectant
(d) Statement-1 is false, Statement-2 is true
(B) Chlorine (II) A ntis eptic 21. Statement-1 : The drugs which act on the central nervous
(C) Bithional (III) M ilk bleaching agent system and help in reducing anxiety are called antibiotics.
(D) Potas sium s tearate (IV) Soap Statement-2 : Pencillin is an antibiotic.
22. Statement-1 : Equanil is a tranquilizer.
(a) A - I, B - II, C - III, D - IV
Statement-2 : Equanil is used to cure depression and
(b) A - II, B - III, C - IV, D - I
hypertension.
(c) A - III, B - I, C - II, D - IV
23. Statement-1 : Tetracycline is a broad spectrum antibiotic.
(d) A - IV, B - I, C - II, D - III
Statement-2 : Tetracyclin is effective against a number of
20. Bithional is added to soap as an additive to function as a/an types of bacteria, large viruses and typhus fever.
(a) softener (b) hardener 24. Statement-1 : Antiseptics are applied to living tissues.
(c) dryer (d) antiseptic Statement-2 : Iodine is a powerful antiseptic.
DIRECTIONS for Qs. 21 to 25 : These are Assertion-Reason
25. Statement-1 : Sedatives are given to patients who are
type questions. Each of these question contains two statements:
mentally agitated and violent.
Statement-1 (Assertion) and Statement-2 (Reason). Answer these
questions from the following four options. Statement-2 : Sedatives are used to suppress the activities
(a) Statement-1 is true, Statement-2 is true, Statement-2 is a of central nervous system.
correct explanation for Statement-1

Exemplar Questions (c) on the basis of molecular targets


(d) on the basis of pharmacological effect
1. Which of the following statements is not correct?
5. Which of the following statements is correct?
(a) Some antiseptics can be added to soaps
(a) Some tranquilizers function by inhibiting the enzymes
(b) Dilute solutions of some disinfectants can be used as
which catalyse the degradation of noradrenaline
antiseptic
(b ) Tranquilizers are narcotic drugs
(c) Disinfectants are antimicrobial drugs
(c) Tranquilizers are chemical compounds that do not affect
(d) Antiseptic medicines can be ingested
the message transfer from nerve to receptor
2. Which is the correct statement about birth control pills?
(d) Tranquilizers are chemical compounds that can relieve
(a) Contain estrogen only
pain and fever
(b) Contain progesterone only
6. Salvarsan is arsenic containing drug which was first used
(c) Contain a mixture of estrogen and progesterone for the treatment of ............... .
derivatives
(a) syphilis (b) typhoid
(d) Progesterone enhances ovulation
(c) meningitis (d) dysentery
3. Which statement about aspirin is not true?
7. A narrow spectrum antibiotic is active against .......... .
(a) Aspirin belong to narcotic analgesics
(a) gram positive or gram negative bacteria
(b) It is effective in relieving pain
(b) gram negative bacteria only
(c) It has antiblood clotting action
(c) single organism or one disease
(d) It is a neurologically active drug
(d) both gram positive and gram negative bacteria
4. The most useful classfication of drugs for medicinal chemists
8. The compound that causes general antidepressant action
is .......... .
on the central nervous system belong to the class of ......
(a) on the basis of chemical structure
(a) analgesics (b) tranquilizers
(b) on the basis of drug action
(c) narcotic analgesics (d) antihistamines
EBD_7327
756 CHEMISTRY

9. Compound which is added to soap to impart antiseptic (b) Prevent the binding of substrate
properties is .......... (c) Generally a strong covalent bond is formed between
(a) sodium lauryl sulphate an inhibitor and an enzyme
(b) sodium dodecylbenzenesulphonate (d) Inhibitors can be competitive or non – competitive
(c) rosin 17. Which of the following chemicals can be added for
(d) bithional sweetening of food items at cooking temperature and does
10. Equanil is ........... not provides calories?
(a) artificial sweetener (b) tranquilizer (a) Sucrose (b ) Glucose
(c) antihistamine (d) antifertility drug (c) Aspartame (d) Sucralose
11. Which of the following enhances leathering property of 18. Which of the following will not enhance nutritional value of
soap? food?
(a) Sodium carbonate (b) Sodium rosinate (a) Minerals (b) Artificial sweetners
(c) Sodium strearate (d) Trisodium phosphate (c) Vitamins (d) Amino acids
12. Glycerol is added to soap. It functions ..............
(a) as a filler (b ) to increase leathering NEET/AIPMT (2013-2017) Questions
(c) to prevent rapid drying (d) to make soap granules
19. Antiseptics and disinfectants either kill or prevent growth of
13. Which of the following is an example of liquid dishwashing
microorganisms. Identify which of the following statements
detergent?
is not true: [2013]
(a) CH3 ( CH 2 )10 — CH 2 OSO3- Na + (a) Chlorine and iodine are used as strong disinfectants.
(b) Dilute solutions of boric acid and hydrogen peroxide
are strong antiseptics.
(b) C 9H 19 O ( CH2 – CH2 – O –) 5 – CH 2CH2OH
(c) Disinfectants harm the living tissues.
– +
(d) A 0.2% solution of phenol is an antiseptic while 1%
(c) CH 3 SO3 Na solution acts as a disinfectant.
20. Dettol is the mixture of [1996, NEET Kar. 2013]
+ (a) Terpineol and Bithionol
CH3
| (b) Chloroxylenol and Bithionol
(d) CH3 ( CH)15 — N – CH3 Br - (c) Chloroxylenol and Terpineol
|
(d) Phenol and Iodine
CH3
21. Artificial sweetner which is stable under cold conditions only
14. Polyethyleneglycols are used in the preparation of which is :- [2014]
type of detergents? (a) Saccharine (b) Sucralose
(a) Cationic detergents (c) Aspartame (d) Alitame
(b) Anionic detergents 22. Bithional is generally added to the soaps as an additive to
(c) Non - ionic detergents function as a/an : [2015]
(d) Soaps (a) Dryer (b) Buffering agent
15. Which of the following is not a target molecule for drug (c) Antiseptic (d) Softner
function in body? 23. Which of the following is an analgesic? [2016]
(a) Carbohydrates (b) Lipids (a) Novalgin (b) Penicillin
(c) Vitamins (d) Proteins (c) Streptomycin (d) Chloromycetin
16. Which of the following statements is not true about enzyme 24. Mixture of chloroxylenol and terpineol acts as: [2017]
inhibitors? (a) antiseptic (b) antipyretic
(a) Inhibit the catalytic activity of the enzyme (c) antibiotic (d) analgesic
Chemistry in Everyday Life 757

Hints & Solutions


EXERCISE - 1 14. (a) Due to anti-blood clotting action of aspirin, it is used
to prevent heart attack.
1. (c) Structural features of soaps and detergents are almost
same except that the polar end in detergents is 15. (a) Serotonin is 5- hydroxytrptamine (or 5-HT) is a
- monoamine neurotransmitter synthesized in serotonergic
-OSO3 Na + while in soaps polar end is - COO - Na + .
neurons in the central nervous system (CNS) and
Detergents have an advantage over soaps that its polar enterochromaffin cells in the gastrointestinal tract of
end sulphate and sulphonate retain their efficiency in animals including humans . Cerebral serotonin has anti-
hard water, since the corresponding Ca and Mg salts depressant and analgesic effects and there have been
are soluble. Being salts of strong acids, they yield reports that cerebral serotonin can be released by the
neutral solution, in contrast, to the soaps, which being stimulation of certain acupuncture points
salts of weak acids yield slightly alkaline solutions.
2. (b) Aspirin is antipyretic i.e., a drug which is responsible HN
for lowering the temperature of feverish organism to
normal, other antipyretic drugs are Paracetamol,
Phenacetin.
3. (b) Paracetamol is an antipyretic NH2
HO
4. (a) Salol is Phenyl Salicylate used as antiseptic.
5. (c) It is correct answer. C10H12N2O
6. (a) Antiseptic drugs cause destruction of micro-organism 16. (d) Antacid decreases acidity in stomach.
that produce septic disease e.g. Dettol, Savlon, Boric
17. (d) Sulpha drugs (antibacterial and antibiotic) are group of
acid, Phenol, Iodoform, KMnO4 and some dye such as
drugs which are derivative of sulphanilamide.
methylene blue, genation violet.
18. (c) Iproniazid ® Tranquilizer
7. (a) Lindane or gammexane is g isomer of BHC.
Salvarsan ® Antimicrobial
8. (b) It is acetyl salicylic acid i.e., aspirin analgesic and
antipyretic. Zantac (ranitidine) ® Antihistamine
9. (b) Paracetamol act as analgesic and antipyretic. Chloramphenicol ® Antibiotic
10. (d) Ampicillin is a semi-synthetic modification of pencillin. 19. (b) Antiseptic drugs causes destruction of micro-organism
that produce septic disease e.g. Dettol, Savlon,
Chemical structure of ampicillin is
acriflavin, Boric acid, Phenol, Iodoform, KMnO4 and
some dyes such as Chloramine T, methylene blue.
O CH3
|| H H S 20. (a) These drugs induce sleep and are habit forming common
C6 H5 - CH - C - NH CH3 example of hypnotic drugs are Luminal and Saconal.
|
NH COOH 21. (a) It is the very effective antibiotic for tuberculosis.
N
O 22. (c) Broad spectrum antibiotics act on different antigens.
H
23. (b) Chloramin - ‘T’ is used as antiseptic.
So, an comparing it with general structure of pencillin
24. (c) It is an insecticide.
CH 25. (c) 26. (b)
we found that R is
NH2

Вам также может понравиться